Download as pdf or txt
Download as pdf or txt
You are on page 1of 880

UNION PUBLIC SERVICE COMMISSION

Civil Services Preliminary Examination

GENERAL STUDIES
&

CSAT
Solved Papers 1993-2023
Yearwise & Topicwise
Chief Editor
A.K. Mahajan

Compiled and Edited by


IAS Exam Expert Team
Computer Graphics by
Balkrishna, Charan Singh

Editorial Office
12, Church Lane Prayagraj-211002
Mob. : 9415650134
Email : yctap12@gmail.com
website : www.yctbooks.com/www.yctfastbooks.com
© All rights reserved with Publisher

Publisher Declaration
Edited and Published by A.K. Mahajan for YCT Publications Pvt. Ltd.
and printed by R.A. Security Printers, Prayagraj. In order to publish the book,
full care has been taken by the Editor and the Publisher,
still your suggestions and queries are welcomed. Rs. : 1595/-
In the event of any dispute, the Judicial area will be Prayagraj.
CONTENT
Union Public Service Commission Civil Services Preliminary Examination
GENERAL STUDIES
IAS (Pre) Examination 2023, General Studies : PAPER-I Solved Paper with Detail Explanations ---------- 3A-18A
IAS (Pre) Examination 2022, General Studies : PAPER-I Solved Paper with Detail Explanations ---------19A-34A
IAS (Pre) Examination 2021, General Studies : PAPER-I Solved Paper with Detail Explanations ---------------3-20
IAS (Pre) Examination 2020, General Studies : PAPER-I Solved Paper with Detail Explanations ------------- 21-45
IAS (Pre) Examination 2019, General Studies : PAPER-I Solved Paper with Detail Explanations ------------- 46-65
IAS (Pre) Examination 2018, General Studies : PAPER-I Solved Paper with Detail Explanations ------------- 66-85
IAS (Pre) Examination 2017, General Studies : PAPER-I Solved Paper with Detail Explanations ----------- 86-104
IAS (Pre) Examination 2016, General Studies : PAPER-I Solved Paper with Detail Explanations ---------- 105-125
IAS (Pre) Examination 2015, General Studies : PAPER-I Solved Paper with Detail Explanations ---------- 126-140
IAS (Pre) Examination 2014, General Studies : PAPER-I Solved Paper with Detail Explanations ---------- 141-154
IAS (Pre) Examination 2013, General Studies : PAPER-I Solved Paper with Detail Explanations ---------- 155-169
IAS (Pre) Examination 2012, General Studies : PAPER-I Solved Paper with Detail Explanations ---------- 170-186
IAS (Pre) Examination 2011, General Studies : PAPER-I Solved Paper with Detail Explanations ---------- 187-204
IAS (Pre) Examination 2010, General Studies : PAPER-I Solved Paper with Detail Explanations ---------- 205-227
IAS (Pre) Examination 2009, General Studies : PAPER-I Solved Paper with Detail Explanations ---------- 228-246
IAS (Pre) Examination 2008, General Studies : PAPER-I Solved Paper with Detail Explanations ---------- 247-266
IAS (Pre) Examination 2007, General Studies : PAPER-I Solved Paper with Detail Explanations ---------- 267-287
IAS (Pre) Examination 2006, General Studies : PAPER-I Solved Paper with Detail Explanations ---------- 288-309
IAS (Pre) Examination 2005, General Studies : PAPER-I Solved Paper with Detail Explanations ---------- 310-331
IAS (Pre) Examination 2004, General Studies : PAPER-I Solved Paper with Detail Explanations ---------- 332-355
IAS (Pre) Examination 2003, General Studies : PAPER-I Solved Paper with Detail Explanations ---------- 356-376
IAS (Pre) Examination 2002, General Studies : PAPER-I Solved Paper with Detail Explanations ---------- 377-401
IAS (Pre) Examination 2001, General Studies : PAPER-I Solved Paper with Detail Explanations ---------- 402-425
IAS (Pre) Examination 2000, General Studies : PAPER-I Solved Paper with Detail Explanations ---------- 426-451
IAS (Pre) Examination 1999, General Studies : PAPER-I Solved Paper with Detail Explanations ---------- 452-479
IAS (Pre) Examination 1998, General Studies : PAPER-I Solved Paper with Detail Explanations ---------- 480-503
IAS (Pre) Examination 1997, General Studies : PAPER-I Solved Paper with Detail Explanations ---------- 504-528
IAS (Pre) Examination 1996, General Studies : PAPER-I Solved Paper with Detail Explanations ---------- 529-550
IAS (Pre) Examination 1995, General Studies : PAPER-I Solved Paper with Detail Explanations ---------- 551-574
IAS (Pre) Examination 1994, General Studies : PAPER-I Solved Paper with Detail Explanations ---------- 575-600
IAS (Pre) Examination 1993, General Studies : PAPER-I Solved Paper with Detail Explanations ---------- 601-624
CSAT
IAS (Pre) Examination 2011, CSAT : PAPER-II Solved Paper with Detail Explanations --------------------------1-18
IAS (Pre) Examination 2012, CSAT : PAPER-II Solved Paper with Detail Explanations ------------------------ 19-35
IAS (Pre) Examination 2013, CSAT : PAPER-II Solved Paper with Detail Explanations ------------------------ 36-52
IAS (Pre) Examination 2014, CSAT : PAPER-II Solved Paper with Detail Explanations ------------------------ 53-68
IAS (Pre) Examination 2015, CSAT : PAPER-II Solved Paper with Detail Explanations ------------------------ 69-83
IAS (Pre) Examination 2016, CSAT : PAPER-II Solved Paper with Detail Explanations ------------------------ 84-99
IAS (Pre) Examination 2017, CSAT : PAPER-II Solved Paper with Detail Explanations --------------------- 100-115
IAS (Pre) Examination 2018, CSAT : PAPER-II Solved Paper with Detail Explanations --------------------- 116-133
IAS (Pre) Examination 2019, CSAT : PAPER-II Solved Paper with Detail Explanations --------------------- 134-152
IAS (Pre) Examination 2020, CSAT : PAPER-II Solved Paper with Detail Explanations --------------------- 153-172
IAS (Pre) Examination 2021, CSAT : PAPER-II Solved Paper with Detail Explanations --------------------- 173-192
IAS (Pre) Examination 2022, CSAT : PAPER-II Solved Paper with Detail Explanations --------------------- 193-208
IAS (Pre) Examination 2023, CSAT : PAPER-II Solved Paper with Detail Explanations --------------------- 209-224
2
UNION PUBLIC SERVICE COMMISSION
Civil Services (Preliminary Exam) - 2023
GENERAL STUDIES : PAPER-I
Time : 2 hours (Exam date : 26.06.2023) Maximum Number : 200
1. In which one of the following regions was 5. Consider the following dynasties :
Dhanyakataka, which flourished as a prominent 1. Hoysala 2. Gahadavala 3. Kakatiyat 4. Yadava
Buddhist centre under the Mahasanghikas, How many of the above dynasties established
located? their kingdoms in early eighth century AD?
(a) Andhra (b) Gandhara (c) Kalinga (d) Magadha (a) Only one (b) Only two (c) All three (d) None
Ans. (a) : Dhanyakataka also known as Dharaniketa is a Ans. (d) :
town near Amaravati in the Guntur district of Andhra
Pradesh was the capital city of the Satvahana Kingdom Dynasty Establishment
and it was also flourished as a prominent Buddhist Hoysala – Between 10th to 14th centuries.
centre. Ghadavala – Between 11th to 13th centuries.
2. With reference to ancient India, consider the Kakatiya – Between 12th to 14th centuries.
following statements : Yodava – Between 12th to 14th centuries.
1. The concept of Stupa is Buddhist in origin.
2. Stupa was generally a repository of relics. 6. With reference to ancient Indian History,
3. Stupa was a votive and commemorative consider the following pairs :
structure in Buddhist tradition. Literary work Author
How many of the statements given above are 1. Devichandragupta : Bilhana
correct? 2. Hammira- : Nayachandra
(a) Only one (b) Only two (c) All three (d) None Mahakavya Suri
Ans. (b) : The term stupa comes from the Sanskrit 3. Milinda-panha : Nagarjune
Wora stupa, which means heap or pile. Originally 4. Nitivakyamrita : Somadeva Suri
stupas were simple mounds of earth or stones that
served as commemorative markers for important events How many of the above pairs are correctly
or burial sites. The concept of stupa is not Buddhist in matched?
origin. Hence statement-1 is incorrect. One of the (a) Only one (b) Only two (c) Only three (d) All four
primary functions of stups is a serve as a repository of Ans. (c) :
relies. Relies are objects associated with Buddha such
as his physical remains personal belonging or items Literary work Author
used by him. Hence statement-II is correct. Devichandragupta – Vishakha Deva
Stupas are also associated with votive and Hammira-Mahakavya – Nayachandra Suri
commemorative purposes. Stupas are often built as acts Milinda-panha – Nagarjune
of devotion and as offerings to the Buddha or others
enlightened beings. Nitivakyamrita – Somadeva Suri
Hence statement III is also correct. 7. "Souls are not only the property of animal and
3. With reference to ancient South India, Korkai, plant life, but also of rocks, running water and
Poompuhar and Muchiri were well known as. many other natural objects not looked on as
(a) capital cities living by other religious sects."
(b) ports The above statement reflects one of the core
(c) centres of iron-and-steel making beliefs of which one of the following religious
sects of ancient India?
(d) shrines of Jain Tirthankaras (a) Buddhism (b) Jainism
Ans. (b) : Korkai, poompuhar (also known as Ka (c) Shaivism (d) Vaishnavism
Veripattinam) and Muchiri are ancient part cities in the
southern region of present day Tamil Nadu India. Ans. (b) : Jain religion believe in reincarnation which
means the cycle of birth, and rebirth is determined by
4. Which one of the following explains the one's Karma. They also believe that plants animals and
practice of 'Vattakirutal' as mentioned in even some non-living things like air and water have
Sangam poems? souls, just as human do.
(a) Kings employing women bodyguards
(b) Learned persons assembling in royal courts to 8. Who among the following rulers of
discuss religious and philosophical matters Vijayanagara Empire constructed a large dam
(c) Young girls keeping watch over agricultural across Tungabhadra River and a canal-cum-
fields and driving away birds and animals aqueduct several kilometres long from the river
(d) A king defeated in a battle committing ritual to the capital city?
suicide by starving himself to death (a) Devaraya 1 (b) Mallikarjuna
Ans. (d) : Vatakkiruttas, also known as Vadakiruttal (c) Vira Vijaya (d) Virupaksha
was a Tamil ritual of fasting till death, during, sangam Ans. (a) : Devaraya-1, the ruler of Vijaynagar empire
age widespreadly. The Tamil kings, in order to save credited for constructed a large dam across
their honour and prestige were prepared to meet thieve Tungabbadra river and commissioned a 24 km long
death after defeated in a battle. aqueduct from the Tungabhadra river to the capital.

IAS (Pre) GS 2023 Paper I 3A YCT


9. Who among the following rulers of medieval 13. Which one of the following statements best
Gujarat surrendered Diu to Portuguese? reflects the Chief purpose of the 'Constitution'
(a) Ahmad Shah (b) Mahmud Begarha of a country?
(c) Bahadur Shah (d) Muhammad Shah (a) In determines the objective for the making of
necessary laws.
Ans. (c) : Bahadur Shah of Gujarat surrendered Diu to
Portuguese in 1534. Bahadur Shah during his conflict (b) It enables the creation of political offices and
with the Mughal emperor Humayun, secured help from a government.
the Portuguese, and he signed the treaty of Bassein with (c) It defines and limits the powers of
the Portuguese by this, he ceded Diu to the Portuguese. government.
(d) It secures social justice, social equality and
10. By which one of the following Acts was the social security.
Governor General of Bengal designated as the
Governor General of India? Ans. (c) : Function of a constitution is to set some limits
on the functions of the government and what a
(a) The Regulating Act government can impose on it's Citizens in terms of
(b) The Pitt's India Act rights. These limits are fundamental in the sense that the
(c) The Charter Act of 1793 government may never trespass them.
(d) The Charter Act of 1833 14. In India, which one of the following
Ans. (d) : The Charter Act of 1833 renamed the Constitutional Amendments was widely
position of the Governor-general of Bengal as the believed to be enacted to overcome the judicial
Governor-General of India. Lord William Bentinck interpretations of the Fundamental Rights?
became the first governor-general of India. (a) 1st Amendment (b) 42nd Amendment
11. In essence, what docs True Process of Law' mean? (c) 44th Amendment (d) 86th Amendment
(a) The principle of natural justice Ans. (a) : When the Constitution of India enacted in
(b) The procedure established by law 1950, then Supreme Court and high court were given a
(c) Fair application of law series of judgements that had struck down the
provisions of public safety, to overcome with a series of
(d) Equality before law judicial interpretation the government passed first
Ans. (c) : "Due process of law" refers to just, rational Constitution Amendment Bill.
fair and fair treatment under the regular judicial 15. Consider the following organizations/bodies in
proceedings, that it means the fair application of law. It India :
states that a person cannot be denied their life, liberty or 1. The National Commission for Backward
property without adhering to required legal procedures Classes
and receiving the appropriate protections. 2. The National Human Rights Commission
12. Consider the following statements : 3. The National Law Commission
Statement-I : In India, prisons are managed by 4. The National Consumer Disputes Redressal
State Governments with their own rules and Commission
regulations for the day-to-day administration of How many of the above are constitutional bodies?
prisons. (a) Only one (b) Only two (c) All three (d) All four
Statement II : In India, prisons are governed by Ans. (a) : In the given options, the National
the Prisons Act, 1894 which expressly kept the Commission for backward classes (NCBC) is a
subject of prisons in the control of Provincial constitutional body under Article 338(B) While the
Governments. National Humans rights commission and the National
Which one of the following is correct in respect Consumer Disputes Redressal Commission are statutory
of the above statements : body under protection of Human Rights Act, 1993, and
(a) Both Statement-I and Statement-II are correct Consumer Protection Act, 1986 respectively. The
and Statement-II is the correct explanation for National Law Commission is neither Constitutional nor
Statement-I. statutory body. Hence only one pair is correctly
(b) Both Statement-I and Statement-II are correct matched.
and Statement-II is not the correct 16. Consider the following statements :
explanation for Statement-I. 1. If the election of the President of India is
(c) Statement-I is correct but Statement-II is declared void by the Supreme Court of
incorrect. India, all acts done by him/her in the
(d) Statement-I is incorrect but Statement-II is performance of duties of his/her office of
President before the date of decision
correct. become invalid.
Ans. (a) : The Prison Act 1894 is one of the oldest part 2. Election for the post of the President of
of legislation in India under Person Act, 1894 prisons India can be postponed on the ground that
are managed by State Governments with their own rules some Legislative Assemblies have been
and regulation for the day-to-day administration of dissolved and elections are yet to take place.
prisons. The Section-IV of prisons act 1894 clearly 3. When a Bill is presented to the President of
mentions. Prison mean any jail or place used India, the Constitution prescribes time
permanently or temporarily under the general or special limits within which he/she has to declare
orders of a state governments for detention of prisoners. his/her assent.
Hence both (A) and (R) is correct and (R) is the correct How many of the above statements are correct?
explanation of (A) (a) Only one (b) Only two (c) All three (d) None
IAS (Pre) GS 2023 Paper I 4A YCT
Ans. (d) : If the election of a person as a president of or wildlife Department under whose jurisdiction the
India in declared void by the Supreme Court acts done community reserve is located. Hence Statement-1 is
by him before the date of such declaration of Supreme incorrect.
Court are not invalidated and continue to remain i9n As per Section 36(c)(2) of the wildlife protection
force. Hence statement-I is not correct. (Amendment) Act, 2002, restrictions and regulation
The Supreme Court in 1974 held that the dissolution of applicable to the wild life sanctuaries are applicable to
state legislature will not be a ground for preventing the community reserve too. Hence statements 2, 3 and 4 are
holding of the election on the expiry of the term of the correct.
president. Hence statement-2 is also incorrect. 19. With reference to 'Schedule Areas' in India,
The Constitution of India does not prescribe any time consider the following statements :
limit within which the president has to take decision 1. Within a State, the notification of an area
with respect to bill presented to him/her for assent. as Scheduled Area takes place through an
Hence statement-3 is also incorrect. Order of the President.
17. With reference to Finance Bill and Money Bill 2. The largest administrative unit forming the
in the Indian Parliament, consider the Scheduled Area is the District and the
following statements : lowest is the cluster of villages in the Block.
1. When the Lok Sabha transmits Finance 3. The Chief Ministers of the concerned
Bill to the Rajya Sabha, it can amend or States are required to submit annual
reject the Bill. reports to the Union Home Ministry on the
2. When the Lok Sabha transmits Money Bill administration of Scheduled Areas in the
to the Rajya Sabha, it cannot amend or States.
reject the Bill, it can only make How many of the above statements are correct?
recommendations. (a) Only one (b) Only two (c) All three (d) None
3. In the case of disagreement between the Ans. (b) : As per Article-244 'scheduled areas' in India
Lok Sabha and the Rajya Sabha, there is shall be declared by the President. The largest
no joint sitting for Money Bill, but a joint administrative unit forming the scheduled area is the
sitting becomes necessary for Finance Bill. District and the lowest is the cluster of villages in the
How many of the above statements are correct? Block. The Governor of the concerned states are
(a) Only one (b) Only two (c) All three (d) None required to submit annual reports to the President on the
Ans. (a) : Finance bill is an example o money bill. It is administration of scheduled areas in the states. Hence
an example of money bill. It is passed during the budget statements 1, 2 are correct while statement-3 is not
for enacting tax proposals. The Rajya Sabha cannot correct.
amend or reject the finance bill but only can 20. Consider the following statements :
recommend changes which the Lok Sabha can accept or Statement-I : The Supreme Court of India has
not. Hence statement-1 is incorrect when money bill is held in some judgements that the reservation
passed by the Lok-Sabha. It is transmitted to the Rajya policies made under Article 16(4) of the
Sabha for it's consideration. The Rajya Sabha has Constitution of India would be limited by Article
restricted powers with regard to money bill. It cannot 335 for maintenance of efficiency of administration.
reject or amend a money bill. It can only make Statement II : Article 335 of the Constitution of
recommendations. Hence statement-2 is correct. India defines the term 'efficiency of administration'.
Because of finance bill is a money bill and there is no Which one of the following is correct in respect
provision for holding joint sitting for a money bill. of the above statements :
Hence statement-3 is also incorrect. (a) Both Statement-I and Statement-II are correct
[Finance bill and Financial bill both are different.] and Statement-II is the correct explanation for
18. Consider the following statements : Statement-I.
Once the Central Government notifies an area (b) Both Statement-I and Statement-II are correct
as a 'Community Reserve' and Statement-II is not the correct
1. the Chief Wildlife Warden of the State explanation for Statement-I.
becomes the governing authority of such (c) Statement-I is correct but Statement-II is
forest incorrect.
2. hunting is not allowed in such area (d) Statement-I is incorrect but Statement-II is
3. people of such area are allowed to collect correct.
non-timber forest produce Ans. (c) : Supreme Court of India has held in some
4. people of such area are allowed traditional judgements that the reservation policies made under
agricultural practices Article 16(4) of the Constitution of India would be
How many of the above statements are correct? limited by Article 335 for maintenance of efficiency of
(a) Only one (b) Only two administration while the Article 335 of the Constitution
(c) All three (d) All four of India does not defines the term efficiency of
administration. Hence statement-1 is correct whole
Ans. (c) : As per the Section 36-D of Wildlife statement-II is not correct.
(Protection) Amendment Act, 2002. The State
government shall Constitute a community reserve 21. Consider the following statements :
management committee which shall be the authority Statement-I : India, despite having uranium
responsible for conserving, maintaining and managing deposits, depends on coal for most of its
the community reserve. The committee shall consist of electricity production.
the representatives nominated by the village Panchayat Statement II : Uranium, enriched to the extent of
or where such panchayat does not exist by the members at least 60%, is required for the production of
of gram sabha and one representative of the state forests electricity.
IAS (Pre) GS 2023 Paper I 5A YCT
Which one of the following is correct in respect 24. Consider the following fauna :
of the above statements : 1. Lion-tailed Macaque
(a) Both Statement-I and Statement-II are correct 2. Malabar Civet
and Statement-II is the correct explanation for 3. Sambar Deer
Statement-I. How many of the above are generally necturnal
(b) Both Statement-I and Statement-II are correct or most active after sunset?
and Statement-II is not the correct (a) Only one (b) Only two
explanation for Statement-I. (c) All three (d) None
(c) Statement-I is correct but Statement-II is Ans. (c) :
incorrect.
(d) Statement-I is incorrect but Statement-II is 25. Which of the following organisms perform
correct. waggle dance for others of their kin to indicate
the direction and the distance to a source of
Ans. (c) : India, despite having Uranium deposits, their food?
depends on coal for most of it's electricity production. (a) Butterflies (b) Dragonflies
Natural Uranium contains only 0.7% of the fossile U- (c) Honeybees (d) Wasps
235 and 99.3%, non fossile U-238. This require
enrichment to the level of at least 3% in case of Ans. (c) : Waggle dance is a form of communication by
electricity production and at least 80%. In case of which the bees tell their nest mates where to go to get
bombs. Thus, the second statement is incorrect while the best source of food to bring back to the hive.
statement-1 is correct. 26. Consider the following statements :
22. Consider the following statements : 1. Some mushrooms have medicinal properties.
Statement-I : Marsupials are not naturally found 2. Some mushrooms have psychoactive
in India. properties.
Statement II : Marsupials can thrive only in 3. Some mushrooms have insecticidal properties.
montane grasslands with no predators. 4. Some mushrooms have bioluminescent
properties.
Which one of the following is correct in respect How many of the above statements are correct?
of the above statements :
(a) Both Statement-I and Statement-II are correct (a) Only one (b) Only two (c) All three (d) All four
and Statement-II is the correct explanation for Ans. (d) : Medicinal mushrooms such as shiltake, reishi
Statement-I. have been found to have antitumor and
(b) Both Statement-I and Statement-II are correct immunustimulant properties.
and Statement-II is not the correct Psychoactive properties mean a drug of other substance
explanation for Statement-I. that affects how the brain works and causes changes in
(c) Statement-I is correct but Statement-II is mood, awareness, thoughts, feelings or behavior.
incorrect. Psilocybin is a hallucinogenic chemical in certain
mushrooms known as magic mushrooms.
(d) Statement-I is incorrect but Statement-II is
correct. Recently on increasing number of mushrooms have
been found to contain insecticidal compounds.
Ans. (c) : Marsupials are not naturally found in India Panellus stiptices is one of the brightest glowing
marsupials are any members of the mammalian bioluminescent mushrooms on earth.
infraclass Marsupialia. All extant marsupials are Thus all the statements are correct.
endemic to Australiasia, Wallacea and the Americas.
Most Australian Marsupials, lives in dry scrup or desert 27. Consider the following statements regarding
habitat. Hence statement-1 is correct while statement-2 the Indian squirrels :
is incorrect. 1. They build nests by making burrows in the
23. 'Invasive Species Specialist Group' (that ground.
develops Global Invasive Species Database) 2. They store their food materials like nuts
belongs to which one of the following and seeds in the ground.
organizations? 3. They are omnivorous.
(a) The International Union for Conservation of How many of the above statements are correct?
Nature (a) Only one (b) Only two (c) All three (d) None
(b) The United Nations Environment Programme Ans. (c) : Indian squirrels construct their nests in tree
(c) The United Nations World Commission for branches and hollow trunks.
Environment and Development Nuts are their main food source throughout year. They
(d) The World Wide Fund for Nature store trains by burying them underground so that they
can retrieve them in winter squirrels are omnivores as
Ans. (b) : Lion-tailed Macaque is an old world alongwith fruits and nuts they also eat on eggs, Small
monkey endemic to the Western Ghats of South India, it insects, caterpillars, small animals and even young
is a diurnal creature. Hence it is not a nocturnal animal snakes. Thus all the statements are correct.
animal. 28. Consider the following statements :
Malabar Civet is a nocturnal animal, endemic to 1. Some microorganisms can grove in
Western Ghats and listed as critically endangered in environments with temperature above the
IUCN Red data book. boiling point of water.
2. Some microorganisms can grow in
Sambar deer is a large deer native to the Indian environments with temperature below the
subcontinent and southeast Asia. These species are quite freezing point of water.
ensure elusive and are mot active at dusk and night 3. Some microorganisms can grow in highly
hence it is consider as nocturnal animal. acidic environment with a pH below 3.
IAS (Pre) GS 2023 Paper I 6A YCT
How many of the above statements are correct? 2. Mundra Port : Largest privately
(a) Only one (b) Only two (c) All three (d) None owned port in India
Ans. (c) : A thermophile is an organism, a type of 3. Visakhapatnam : Largest container
extremophile, that thrives at relatively high Port port in India
temperatures between 41 and 122oC. One thermophile,
Pyrdobus Fumarii, has been found at temperatures at How many of the above pairs are correctly
113oC which is hotter than boiling point of water. matched?
Psychrophiles are organisms capable of growth and (a) Only one pair (b) Only two pairs
reproduction in cold temperatures and are often (c) All three pairs (d) None of the pairs
xerophilic and halophilic as well. The reported Ans. (?) : Kamrajar Port, located on the coromandel
minimum growth of microorganism varying from 10oC coast about 24 km. north of Chennai Port. It is the First
to –10oC. But for practical cases lower limit ofo growth Port in India which is public company. The Kamrajar
for bacteria, yeast or moulds can be taken as –7 C. Port is the only corporatized major port is registered as
Acidophiles are microorganisms that show optimal a company. Mundra port is the larged port in India. It is
growth in highly acidic environments. Extreme operated by Ports and SE2 limited Jawaharlal Nehru Port
Citophiles dwell in environments with a pH value less
than 3. Thus all the statements are Trust Nhaya Sheva is the largest container port in India.
29. Which one of the following makes a tool with a Thus pair 1 & Part 2 are correctly matched.
stick to scrape insects from a hole in a tree or a 33. Consider the following trees :
log of wood? 1. Jackfruit (Artocarpus heterophyllus)
(a) Fishing cat (b) Orangutan 2. Mahua (Madhuca indica)
(c) Otter (d) Sloth bear 3. Teak (Tectona grandia)
Ans. (b) : In a recent study of five captive Sumatran How many of the above are deciduous trees?
Orangutans it was found that they frequently stripped (a) Only one (b) Only two (c) All three (d) None
leaves and twigs from a branch and then inserted the Ans. (b) : Jackfruit is a tropical evergreen free white
tool into a hole to obtain foodstuffs. Thus orangutans
can make fools with a stick to scrape insects from a hole Mahua and Teak are tropical deciduous free. Thus (b) is
in a free or log of wood. correct.
30. Consider the following : 34. Consider the following statements :
1. Aerosols 2. Foam agents 1. India has more arable area than China.
3. Fire retardants 4. Lubricants 2. The proportion of irrigated area is more in
In the making of how many of the above are India as compared to China.
hydrofluorocarbons used? 3. The average productivity per hectare in
(a) Only one (b) Only two (c) All three (d) All four Indian agriculture is higher than that in
Ans. (d) : HFC's are entirely man-made. They are China.
primarily produced for use in refrigeration, air- How many of the above statements are correct?
conditioning, insulating foams and aerosol propellants, (a) Only one (b) Only two (c) All three (d) None
with minor uses as solvents and for fire protection. Ans. (b) : India's arable land area is 156 million hectare
The HFS-type hydraulic fluids are water-glycol while China's arable land area is 11 & million hectares.
hydraulic fluids. Lubricants also contain aerosols. India's proportion of irrigated area is 48% while China's
31. Consider the following statements : proportion of irrigated area is 41%. Thus proportion of
1. Jhelum River passes through Wular Lake. irrigated area in India is more as compared to China.
2. Krishna River directly feeds Kolleru Lake. Agricultural productivity in China is almost 50 to 100%
3. Meandering of Gandak River formed more than that of China. Thus Statements (1 & 2) are
Kanwar Lake. correct.
How many of the statements given above are 35. Which one of the following is the best example
correct?
(a) Only one (b) Only two (c) All three (d) None of repeated falls in sea level, giving rise to
present-day extensive marshland?
Ans. (b) : The Jhelum river rises from a spring at (a) Bhitarkanika Mangroves
verinag situated at the foot of Dir Panjal in the south-
eastern part of valley of Kashmir. It flows through (b) Marakkanam Salt Pans
Srinagar and the Vulular lake before entering Pakistan (c) Naupada Swamp
through a deep harrow gorge. (d) Rann of Kutch
The Kolleru lake is fed directly by seasonal Budameru Ans. (d) : The Rann of Kutch was once shallow part of
and Tammilmer Streams. It is connected to the Krishna Arabian Sea, Until a geological shift closed off the
and Gotavari systems by over 68 inflowing drains and connection with sea. The region became a seasonal
channels but not directly Fed. marshy salt desert over the years. During Monsoons the
Kanwar lake in Bihar is residual oxbow take in Bihar region filis up with water and forms a wetland. In the
formed due in meandering of Gantak river. summer the water dries to create a bed of white salty
Thus statements 1 & 3 are correct and statement is land. The Great Rann is a bowl shaped depression, its
incorrect. eastern edge merging into the drainage area of the Luni
32. Consider the following pairs : river.
Port Well known as 36. Ilmenite and rutile, abundantly available in
1. Kamarajar Port : First major port in certain coastal tracts of India, are rich sources
India registered as a of which one of the following?
company (a) Aluminium (b) Copper (c) Iron (d) Titanium
IAS (Pre) GS 2023 Paper I 7A YCT
Ans. (d) : Ilmenite and rutile are both minerals that Ans. (d) : The East-West corridor is a part of National
contain high concentration of titanium. Highways Development Project which aims to connect
In India certain coastal trails are known for significant Silchar in Assam to Porbandar in Gujarat.
ilnenite and rutile deposits. These are found in beach The Trilateral Highway is a connectivity project that
deposits and are extracted through beach sand mining aims to connect Moreh is Manipur, India with Maesot
operations. in Thailand via Myanmar.
The extracted ilnenite and rutile are processed to obtain The Bangladesh-China-India-Myanmar (BCIM) is a
titanium dioxide which is then further refined to proposed economic corridor that aims to connect
produce metallic titanium. Kunning in China with Kolkata in India via Myanmar
and Bangladesh.
37. About three-fourths of world's cobalt, a metal
required for, the manufacture of batteries for Thus all the statements are incorrect.
electric motor vehicles, is produced by 41. Consider the following statements :
(a) Argentina Statement-I : Interest income from the deposits in
(b) Botswana Infrastructure Investment Trusts (InvITs)
(c) the Democratic Republic of the Congo distributed to their investors is exempted from tax,
but the dividend is taxable.
(d) Kazakhstan Statement II : InvITs are recognized as
Ans. (c) : The Democratic Republic of Congo is the borrowers under the Securitisation and
world's largest producer of cobalt accounting for about Reconstruction of Financial Assets and
three-fourths of the global protection. The country Enforcement of Security Interest Act, 2002'.
reported output of 1,30,000 MT in 2022. Cobalt is Which one of the following is correct in respect
crucial for manufacture of some electronic products of the above statements :
(Smartphones & Laptops) and batteries of Electric (a) Both Statement-I and Statement-II are correct
Vehicles. and Statement-II is the correct explanation for
38. Which one of the following is a part of the Statement-I.
Congo Basin? (b) Both Statement-I and Statement-II are correct
(a) Cameroon (b) Nigeria (c) South Sudan (d) Uganda and Statement-II is not the correct
Ans. (a) : The Congo Basin in region is Central Africa explanation for Statement-I.
including Multiple countries which are as follows : (c) Statement-I is correct but Statement-II is
Cameroon incorrect.
Central African Republic (d) Statement-I is incorrect but Statement-II is
correct.
Democratic Republic of Congo
Republic of Congo Ans. (d) : InvITs are investment vehicle that pool Funds
From investors to invest in infrastructure projects.
Equatorial Guinea Latest budget 2023-24 made the provision that all
Gabbon income distributed by InvITs, including interest income,
Thus 'Cameroon' is the correct answer. dividend income and rental income will be taxable.
39. Consider the following statements : InvITs are classified as borrowers under the securitization
1. Amarkantak Hills are at the confluence of and Reconstruction of Financial Assets and
Vindhya and Sahyadri Ranges. Enforcement of Securities Interest Act, 2002.
2. Biligirirangan Hills constitute the Thus statement (1) is incorrect and statement 2 is correct.
easternmost part of Satpura Range. 42. Consider the following statements :
3. Seshachalam Hills constitute the Statement-I : In the post-pandemic recent past,
southernmost part of Western Ghats. many Central Banks worldwide had carried out
How many of the statements given above the interest rate hikes.
correct? Statement II : Central Banks generally assume
(a) Only one (b) Only two (c) All three (d) None that they have the ability to counteract the rising
Ans. (d) : Amarkantak is at confluence of Vindhya and consumer prices via monetary policy means.
Satpura Range. Which one of the following is correct in respect
Ananalai hills and cartmon hills are the southernmost of the above statements :
point of Western Ghats. (a) Both Statement-I and Statement-II are correct
and Statement-II is the correct explanation for
Sheshalhalam Hills are part of eastern Ghats. Statement-I.
Thus none the statements are correct. (b) Both Statement-I and Statement-II are correct
40. With reference to India's projects on and Statement-II is not the correct
connectivity, consider the following statements: explanation for Statement-I.
1. East-West Corridor under Golden (c) Statement-I is correct but Statement-II is
Quadrilateral Project connects Dibrugarh incorrect.
and Surat. (d) Statement-I is incorrect but Statement-II is
2. Trilateral Highway connects Moreh in correct.
Manipur and Chiang Mai in Thailand via Ans. (a) : In the post-pandemic recent past, many
Myanmar. central banks worldwide have indeed carried out
3. Bangladesh-China-India-Myanmar interest rate hikes central banks use interest rate
Economic Corridor connects Varanasi in adjustments as one of their monetary policy tools to
Uttar Pradesh with Kunming in China. manage the economy.
How many of the above statements are correct? Central banks believe that by adjusting interest rates,
(a) Only one (b) Only two (c) All three (d) None they can influence borrowing costs, spending patterns,
IAS (Pre) GS 2023 Paper I 8A YCT
and overall economic activity. When inflation rises, 46. Which one of the following best describes the
central banks may raise interest rates to make borrowing concept of 'Small Farmer Large Field?
more expensive thereby discouraging excessive (a) Resettlement of a large number of people,
spending and slowing down economic growth. uprooted from their countries due to war, by
Thus Assertion and Reason both are true and reason giving them a large cultivable land which
explains the assertion. they cultivate collectively and share the
43. Consider the following statements : produce.
Statement-I : Carbon markets are likely to be one (b) Many marginal farmers in an area organize
of the most widespread tools in the fight against themselves into groups and synchronize and
climate change. harmonize selected agricultural operations
Statement II : Carbon markets transfer resources (c) Many marginal farmers in an area together
from the private sector to the State. make a contract with a corporate body and
Which one of the following is correct in respect surrender their land to the corporate body for
of the above statements : a fixed term for which the corporate body
(a) Both Statement-I and Statement-II are correct makes a payment of agreed amount to the
and Statement-II is the correct explanation for farmers
Statement-I. (d) A company extends loans, technical
(b) Both Statement-I and Statement-II are correct knowledge and material inputs to a number of
and Statement-II is not the correct explanation small farmers in an area so that they produce
for Statement-I. the agricultural commodity required by the
(c) Statement-I is correct but Statement-II is company for its manufacturing process and
incorrect. commercial production.
(d) Statement-I is incorrect but Statement-II is Ans. (b) : The concept of "Small Former Large Field"
correct. refers to the idea where many marginal farmers in a
Ans. (b) particular area come together and organize themselves
into groups. In this approach, instead of individually
44. Which one of the following activities of the working on their small plots of land, these farmers pool
Reserve Bank of India is considered to be part their resources, knowledge and efforts to cultivate larger
of 'sterilization'? fields collectively. By doing so they can achieve
(a) Conducting 'Open Market Operations' economics of scale improve efficiency and enhance
(b) Oversight of settlement and payment systems productivity.
(c) Debt and cash management for the Central Thus state option (b) is the correctly answer.
and State Governments 47. Consider the following statements :
(d) Regulating the functions of Non-banking 1. The Government of India provides
Financial Institutions Minimum Support Price for niger
Ans. (a) : Sterlization refers to actions taken by a (Guizotia abyssinica) seeds.
central bank to offset the impact of its foreign exchange 2. Niger is cultivated as a Kharif crop.
operations on the domestic money supply. 3. Some tribal people in India use niger seed
The RBI carries out sterlisation operations such as open oil for cooking.
Market Operations to control the total amount of money How many of the above statements are correct?
supply in the economy. As part of OMO'S RBI can
perform two actions– (a) Only one (b) Only two (c) All three (d) None
Buy G-secs to inject money during slowdown. Ans. (c) : The Government of India provides Minimum
Sell G-secs to such out excess during inflation. Support Price of Rs. 6830 for niger seeds for MSP
2021-22.
45. Consider the following markets : Niger, a minor oilseed crop, is primarily cultivated in
1. Government Bond Market rainfed areas as Kharif crop.
2. Call Money Market The tribial communities incorporate Niger seed oil into
3. Treasury Bill Market their cooking practices and utilize the remaining press
4. Stock Market cake remaining after oil extraction as livestock feed.
How many of the above are included in capital Thus all the 3 statements are correct.
markets?
(a) Only one (b) Only two (c) All three (d) All four 48. Consider the investments in the following
assets :
Ans. (b) : Capital Markets are Financial Markets where 1. Brand recognition 2. Inventory
long term securities such as stocks and bonds are traded. 3. Intellectual property 4. Mailing list of clients
They provided platform for raising capital for business
and governments. How many of the above are considered
Government bonds are longterm debt securities issued intangible investments?
by governments to finance their activities. Call money (a) Only one (b) Only two (c) All three (d) All four
market is a short term market where funds ar borrowed Ans. (c) : Brand equity is classified as an intangible
and lent for very short durations, usually overnight asset since its worth does not derive from a physical
treasury bills are short term debt instruments issued by entity and is primarily influenced by how consumers
the governments to finance their short term cash flow perceive the brand. Inventory refers to the goods or
requirements. products that a company holds for sale. It represents the
The stock market, is where shares or stocks of publicly tangible assets, as inventory consists of physical items
listed companies are brought and sold. It is part of that can be seen, touched and measured.
capital market. Thus statements 1 & 4 are correct while Intellectual property (IP) included intangible assets. Such
statements 2 & 3 are incorrect. as patents, trademarks, copyrights, and trade secrets.
IAS (Pre) GS 2023 Paper I 9A YCT
The mailing list of client is among the intangible assets 52. With reference to India, consider the following
of company encompassing a compilation of past pairs :
customers. Action The Act under
Thus statements 1, 3 and 4 are correct. which it is
49. Consider the following : covered
1. Demographic performance 1. Unauthorized wearing : The Official
2. Forest and ecology of police or military Secrets Act,
3. Governance reforms uniforms 1923
4. Stable government 2. Knowingly :
5. Tax and fiscal efforts misleading or
For the horizontal tax devolution, the Fifteenth otherwise interfering
Finance Commission used how many of the with a police officer
or military officer
above as criteria other than population area when engaged in their
and income distance? duties
(a) Only one (b) Only two (c) All three (d) All five
3. Celebratory gunfire : The Arms
Ans. (c) : For horizontal devolution the 15th Finance which can endanger (Amendment)
Commission used the following Parameters– the personal safety of Act, 2019
Parameters 15th Finance Commission others
Population (2011) 15% How many of the above pairs are correctly
Area 15% matched?
Forests Ecology 10% (a) Only one (b) Only two (c) All three (d) None
Income distance 45% Ans. (b) : The Section 7 of the official Secrets Act,
Tax & Fiscal Efforts 2.5% 1923 provides for prohibition knowingly misleading or
Demographic performance 12.5% interfering with a police officers or military officers
Thus among the given options only 3 criteria were used. when engaged in their duties. Hence pair-2 is in
50. Consider the following infrastructure sectors : correctly matched while the rest pairs are correctly
matched.
1. Affordable housing 2. Mass rapid transport
3. Health care 4. Renewable energy 53. Consider the following pairs :
On how many of the above does UNOPS Regions often Reason for being in
Sustainable Investments in Infrastructure and mentioned in news news
Innovation (S3i) initiative focus for its 1. North Kivu and : War between
investments? Ituri Armenia and
(a) Only one (b) Only two (c) All three (d) All four Azerbaijan
Ans. (c) : The sustainable investments in infrastructure 2. Nagarno- : Insurgency in
and innovation initiative was established by the United Karabakh Mozambique
Nations Office for Project Services 10 fill the SPG 3. Kherson and : Dispute between
Financing gap in developing countries. It seeks to Zaporizhzhia Israel and Lebanon
channelise public and private Financing into large scale How many of the above pairs are correctly
infrastructure projects in the following sectors. matched?
1. Renewable Energy (a) Only one (b) Only two (c) All three (d) None
2. Affordable Housing Ans. (d) :
3. Health infrastructure North Kivu and Ituri – Conflict in Democrate
51. With reference to Home Guards, consider the Republic of congo
following statements : Nogorno-Karabakh – War between Armenia
1. Home Guards are raised under the Home and Azerbaiyan
Guards Act and Rules of the Central Kherson and Zaporizhzhia – Allocks sites in Ukraine
Government. Hence none of the above pairs are correctly matched.
2. The rule of the Home Guards is to serve as 54. Consider the following statements :
an auxiliary force to internal security. Statement-I : Israel has established diplomatic
3. To government infiltration on the relations with some Arab States.
international border/coastal areas, the Statement II : The 'Arab Peace Initiative'
Border Wing Home Guards Battalions mediated by Saudi Arabia was signed by Israel
have been raised in, some States. and Arab League.
How many of the above statements are correct? Which one of the following is correct in respect
(a) Only one (b) Only two (c) All three (d) None of the above statements :
Ans. (b) : Home Guards are raised under the Home (a) Both Statement-I and Statement-II are correct
Guards Act and Rules of states/Union Territories. and Statement-II is the correct explanation for
Hence statement-I is incorrect. The Role of home Statement-I.
guards is to serve as an auxiliary force to the police in (b) Both Statement-I and Statement-II are correct
maintenance of internal security situations, help the and Statement-II is not the correct explanation
community in any kind of emergency such as an air for Statement-I.
raid, fire, cyclone earthquake etc. Hence statement-2 is (c) Statement-I is correct but Statement-II is
correct some states (Rajasthan) have raised battalions incorrect.
with the name of border wing home guard to prevent (d) Statement-I is incorrect but Statement-II is
infiltration on the international border. correct.
IAS (Pre) GS 2023 Paper I 10A YCT
Ans. (c) : Israel has established diplomatic relations Ans. (b) : 44th FIDE Chess Olympiad 2022 was the
with some Arab states. Since Israil signed Abraham first time the Chess Olympiad is being hosted in India.
Accords which is the main focus of Arab-Israil peace The official Mascot was named thambi (younger
making. The Arab peace initiative (API) introduces by brother) a chess knight dressed in the traditional Tamil
the late Saudi King Abdullah 20 year ago but it is not Nadu male attire vetti with a white shirt. Hamilton
signed by Israel yet. Russell cup, is the trophy for the winning team in the
55. Consider the following pairs with regard to open section while the Menchik cup is the trophy for the
sports awards : winning team in the women's section. Hence only pair 1
1. Mayor Dhyan : For the most & 2 are correctly matched.
Chand Khel spectacular and 57. Consider the following pairs :
Ratna Award outstanding Area of conflict Country where it is
performance by a mentioned in news located
sportsperson over
period of last four 1. Donbas : Syria
years 2. Kachin : Ethiopia
2. Arjuna Award : For the lifetime 3. Tigray : North Yemen
achievement by a How many of the above pairs are correctly
sportsperson matched?
3. Dronacharya : To honour eminent (a) Only one (b) Only two (c) All three (d) None
Award coaches who have Ans. (d) :
successfully
trained Area of conflict Country
sportspersons or Donbas – Ukraine
teams Kochin – Myanmar
4. Rashtriya Khel : To recognize the Tigray – Ethiopia
Protsahan contribution made Hence none of the above given pairs are correctly
Puraskar by sportspersons matched.
even after their
retirement 58. In the recent years Chad, Guinea, Mali and
Sudan caught the international attention for
How many of the above pairs are correctly which one of the following reasons common to
matched? all of them?
(a) Only one (b) Only two (c) All three (d) All four (a) Discovery of rich deposits of rare earth
Ans. (b) : elements
Majors Dhyan Chand Khel Ratna Award (b) Establishment of Chinese military bases
For most spectacular and outstanding performance by a (c) Southward expansion of Sahara Desert
sports person over a period of last 4 years. (d) Successful coups
Ans. (a) : In recent years Chad, Geinea, Mali and Sudan
Ajuna Award It is officially known as Arjuna hence a experienced coup attempts or successful coups.
Awards for outstanding performance, in sports and 59. Consider the following heavy industries :
games and is the second highest sporting honour of 1. Fertilizer plants 2. Oil refineries 3. Steel plants
India good performance in the field of sports over a
period of 4 years. Green hydrogen is expected to play a
significant role in decarbonizing how many of
Dronacharya Award : To honour eminent coaches the above industries?
who have successfully trained sports persons or teams. (a) Only one (b) Only two (c) All three (d) None
Rashtriya Khel Protsahan Puruskar Ans. (c) : Given hydrogen in expected to play a
significant role in decarbonizing in all given heavy
The award recognised the involvement of corporates, industries.
voluntary organisations and sports Control Boards, in
the promotion and development of sports in the country 60. Consider the following statements about G-20 :
over the previous three years. 1. The G-20 group was originally established
Hence only pair 1 & 3 are correctly matched. as a platform for the Finance Ministers
56. Consider the following statements in respect of and Central Bank Governors to discuss the
the 44th Chess Olympiad, 2022 : international economic and financial issues.
1. It was the first time that Chess Olympiad 2. Digital public infrastructure is one of
was held in India. India's G-20 priorities.
2. The official mascot was named 'Thambi'. Which of the statements given above is/are
3. The trophy for the winning team in the correct?
open section is the Vera Menchik Cup. (a) 1 only (b) 2 only (c) Both 1 and 2 (d) Neither 1 nor 2
4. The trophy for the winning team in the Ans. (c) : The G-20 group was founded in 1949 after
women's section is the Hamilton-Russell the Asian financial crisis as a forum for the Finance
Cup. Minister's and Central Bank governors to discuss global
How many of the statements given above are economic and financial issues. Digital public
correct? infrastructure is one of the India's G-20 priorities.
(a) Only one (b) Only two (c) All three (d) All four Hence both the statements are correct.
IAS (Pre) GS 2023 Paper I 11A YCT
61. With reference to the Indian History, launched on August 7 in 1905 in Calcutta town Hall to
Alexander Rea. A.H. Longhurst, Robert Sewell, protest against the Partition of Bengal by the British
James Burgess and Walter Elliot were government. Hence both the statement are true and
associated with : statement 2 is the correct explanation of statement 1.
(a) archaeological excavations 64. Consider the following statements in respect of
(b) establishment of English Press in Colonial the National Flag of India according to the Flag
India Code of India, 2002 :
(c) establishment of Churches in Princely States Statement-I : One of the standard sizes of the
(d) construction of railways in Colonial India National Flag of India is 600 mm × 400 mm.
Ans. (a) : Alexander Rea, All Longhurst, Robert Statement II : The ratio of the length to the
Sewell, James Burgess and Walter Elliot all were height (width) of the Flag shall be 3 : 2.
associated with archaeological excavations. Which one of the following is correct in respect
of the above statements :
Alexander Rea is known for unearthing a (a) Both Statement-I and Statement-II are correct
sacrophagus from the hillocks of Pallavarnam in Tamil and Statement-II is the correct explanation for
Nadu. Statement-I.
A.H. Longhurst was in charge of the systematic (b) Both Statement-I and Statement-II are correct
and Statement-II is not the correct
digging of Nagarjunakonda. explanation for Statement-I.
Robert Sewell & Water Elliot was associated with (c) Statement-I is correct but Statement-II is
their archacological work at Buddhist Stupa at incorrect.
Amarvati. (d) Statement-I is incorrect but Statement-II is
correct.
James Burgess was the Director General
Ans. (d) : The use, display and hoisting of National
Archaeological Survey of India. Flag in the country is guided by an overarching set of
62. Consider the following pairs : instructions called the Flag code of India 2002. As per
Site Well known for Flag Code of India 2002 the National Flag Code cannot
be in the dimension of 600 mm × 400 mm. The code
A. Besnagar : Shaivite cave shrine states that the National Flag can be of none standard
B. Bhaja : Buddhist cave shrine dimensions as follows–
C. Sittanavasal : Jain cave shrine Flag Size No. Dimension in mm
How many of the above pairs are correctly 1 – 6300 × 4200
matched?
(a) Only one (b) Only two (c) All three (d) None 2 – 3600 × 2400
3 – 2700 × 1800
Ans. (b) : Besnagar (Vidisha) site is well known for 4 – 1800 × 1200
Vishnavite sect not Shaivite sect. Hence pair 1 is 5 – 1350 × 900
incorrectly matched. 6 – 900 × 600
7 – 450 × 300
Bhaya Caves is a group of 22 rockcut causes build 8 – 225 × 150
during 2nd century B.C. The caves belong to the 9 – 150 × 100
Hinayana Buddhism sect in Maharashtra. Hence statement-1 is incorrect.
Sithan Viavasal Caves located in Pudukottai district of The National Flat shad be rectangular in shape and the
modern Tamil Nadu belong to Jainism. Hence pairs 2 & ratio of length to the height of the flat shall be 3 : 2.
3 are correctly matched. Hence statement (2) is correct.
63. Consider the following statements : 65. Consider the following statements in respect of
Statement-I : 7th August is declared as the the Constitution Day :
National Handloom Day. Statement-I : The Constitution Day is celebrated
Statement II : It was in 1905 that the Swadeshi on 26th November every year to promote
Movement was launched on the same day. constitutional values among citizens.
Which one of the following is correct in respect Statement II : On 26th November, 1949, the
of the above statements : Constituent Assembly of India set up a Drafting
(a) Both Statement-I and Statement-II are correct Committee under the Chairmanship of Dr. B.R.
and Statement-II is the correct explanation for Ambedkar to prepare a Draft Constitution of
Statement-I. India.
(b) Both Statement-I and Statement-II are correct Which one of the following is correct in respect
and Statement-II is not the correct of the above statements :
explanation for Statement-I. (a) Both Statement-I and Statement-II are correct
(c) Statement-I is correct but Statement-II is and Statement-II is the correct explanation for
incorrect. Statement-I.
(d) Statement-I is incorrect but Statement-II is (b) Both Statement-I and Statement-II are correct
correct. and Statement-II is not the correct
Ans. (a) : The first national handloom day was held explanation for Statement-I.
on7th August, 2015, from then on 7th August observed (c) Statement-I is correct but Statement-II is
as National Handloom day annually. August 7 was incorrect.
choosen as the National Handloom day to (d) Statement-I is incorrect but Statement-II is
commemorate the Swadeshi movement, which was correct.

IAS (Pre) GS 2023 Paper I 12A YCT


Ans. (c) : The constitution day is celebrated in India on (a) Both Statement-I and Statement-II are correct
26 November every year to commemorate the adoption and Statement-II is the correct explanation for
of the Constitution of India and also to promote the Statement-I.
constitutional values among the citizen of India. (b) Both Statement-I and Statement-II are correct
66. Consider the following statements : and Statement-II is not the correct
Statement-I : Switzerland is one of the leading explanation for Statement-I.
exporters of gold in terms of value. (c) Statement-I is correct but Statement-II is
Statement II : Switzerland has the second largest incorrect.
gold reserves in the world. (d) Statement-I is incorrect but Statement-II is
Which one of the following is correct in respect correct.
of the above statements : Ans. (a) : As per latest data available (Economic
(a) Both Statement-I and Statement-II are correct Survey 2022-23) India accounts for 1.8% of Global
and Statement-II is the correct explanation for export of goods and 4% of global exports of services.
Statement-I. Thus statement-I is incorrect Production Linked
(b) Both Statement-I and Statement-II are correct Incentive (PLI) scheme is a form of performance linked
and Statement-II is not the correct incentive given to companies based on their incremental
explanation for Statement-I. sales from product5s manufactured in domestic units.
(c) Statement-I is correct but Statement-II is It's aimed at boosting the manufacturing sector and
incorrect. reduce imports of India.
(d) Statement-I is incorrect but Statement-II is PLI schemes can be availed by both domestic as well as
correct. foreign companies. Many domestic and foreign
companies operating in India have taken advantage of
Ans. (c) : It is true Switzerland is the leading exporter PLI scheme. Hence statement II is correct.
of gold in the world.
69. Consider the following statements :
In 2021, Switzerland became 1st largest exporter of
Gold in the world. While the Switzerland has 7th largest The 'Stability and Growth Pact' of the
gold deposit in the world. Home statement-1 is right European Union is a treaty that
while statement-2 is wrong. 1. limits the levels of the budgetary deficit of
the countries of the European Union
67. Consider the following statements : 2. makes the countries of the European Union
Statement-I : Recently, the United States of to share their infrastructure facilities
America (USA) and the European Union (EU) 3. enables the countries of the European
have launched the 'Trade and Technology Union to share their technologies
Council'.
Statement II : The USA and the EU claim that How many of the above statements are correct?
through this they are trying to bring technological (a) Only one (b) Only two (c) All three (d) None
progress and physical productivity under their Ans. (a) : The Stability and Growth Pact (SGP) is an
control. agreement, among all the 27 member states of European
Which one of the following is correct in respect Union. It's aim to level the budget deficits of European
of the above statements : countries the corrective arm of the stability and Growth
(a) Both Statement-I and Statement-II are correct pact ensure that member states adopt appropriate policy
and Statement-II is the correct explanation for responses to correct excessive deficits (and/or debts) by
Statement-I. implementing the excessive deficit procedure (CDP).
(b) Both Statement-I and Statement-II are correct The SGP treaty does not require its members to share
and Statement-II is not the correct their infrastructure facilities nor their technologies with
explanation for Statement-I. other countries. The purpose of SGP was to ensure that
fiscal discipline would be maintained and enforced in
(c) Statement-I is correct but Statement-II is the European Union. hence statement-1 is correct while
incorrect. statements 2 & 3 are incorrect.
(d) Statement-I is incorrect but Statement-II is
correct. 70. Consider the following statements :
1. Recently, all the countries of the United
Ans. (c) : Trade and Technology Council was launched Nations have adopted the first-ever
by the United States of America and Europian Union compact for international migration, the
during the EU-US Summit on 15 June, 2021 in Brussels 'Global Compact for Safe, Orderly and
with the aim to promote pooling of digital resources Regular Migration (GCM)'.
such as AI models and computing power and make 2. The objectives and commitments stated in
them available to partner countries to address the GCM are binding on the UN member
challenges in area such as climate change and extreme countries.
weather, health care or smart agriculture. 3. The GCM addresses internal migration or
Hence statement-I is correct while statement-II is internally displaced people also in its
incorrect. objectives and commitments.
68. Consider the following statements : How many of the above statements are correct?
Statement-I : India accounts for 32% of global (a) Only one (b) Only two (c) All three (d) None
export of goods. Ans. (a) : The Global compact for safe, orderly and
Statement II : Many local companies and some regular migration is an intergovernmental agreement
foreign companies operating in India have taken prepared under the overall aegis of United Nations. It is
advantage of India's 'Production-linked Incentive' first intergovernmental agreement. Covering
scheme. international migration in a holistic and comprehensive
Which one of the following is correct in respect manner. Till now only 163 countries have adopted the
of the above statements : Global Compact for safe, orderly and regular migration,
IAS (Pre) GS 2023 Paper I 13A YCT
the United States. Hungary, Israel, Czech Republic and (b) Both Statement-I and Statement-II are correct
Poland are some of the countries that voted against the and Statement-II is not the correct
compact. explanation for Statement-I.
It is a non-legally binding agreement, that compresses (c) Statement-I is correct but Statement-II is
23 objectives for better managing migration at local, incorrect.
national regional and global levels. Hence statements 1 (d) Statement-I is incorrect but Statement-II is
& 2 are incorrect while statement 3 is correct. correct.
71. Consider the following countries : Ans. (a) : Tropical rain forest soil is very poor in
1. Bulgaria 2. Czech Republic 3. Hungary nutrient which are required by plants to grow. This is
4. Latvia 5. Lithuania 6. Ramania due to acidic nature of soil and incessant rains
How many of the above-mentioned countries Potassium, Calcium, Magnesium and Phosphorous are
share a land border with Ukraine? lest away due to leaching, thus regeneration is very
(a) Only one (b) Only three (c) All four (d) Only five slow. Hence statement-1 is incorrect.
Ans. (a) : Ukraine is bordered by Belarus, Russia Tropical rainforests typically experience warm and
Moldova, Romania, Hungary Slovakia and Poland. consistent temperatures throughout the year. These
elevated temperatures enhance the activity of soil
72. With reference to the Earth's atmosphere, micro-organism rate which is responsible for faster
which one of the following statements is decompositions of organic matters. Hence statement-2
correct? is correct.
(a) The total amount of insolation received at the
equator is roughly about 10 times of that 74. Consider the following statements :
received at the poles. Statement-I : The temperature contrast between
(b) Infrared rays constitute roughly two-thirds of continents and oceans is greater during summer
insolation. than in winter.
(c) Infrared waves are largely absorbed by water Statement II : The specific heat of water is more
vapour that is concentrated in the lower than that of land surface.
atmosphere. Which one of the following is correct in respect
(d) Infrared waves are a part of visible spectrum of the above statements :
of electromagnetic waves of solar radiation. (a) Both Statement-I and Statement-II are correct
and Statement-II is the correct explanation for
Ans. (c) : Equator receives more direct sunlight Statement-I.
compared to the poles, the actual difference in
insolation is not as large as 10 times. Hence option (a) is (b) Both Statement-I and Statement-II are correct
incorrect. and Statement-II is not the correct
explanation for Statement-I.
Insolation refers to the total solar radiation received at
Earth (c) Statement-I is correct but Statement-II is
incorrect.
s surface, which includes a broad spectrum of
electromagnetic radiation. While infrared radiation is a (d) Statement-I is incorrect but Statement-II is
significant component of solar radiation it does not correct.
accounts for 2/3rd of insolation includes visible lights, Ans. (d) : The temperature contrast between the
ultraviolet (UV) radiation and other wavelengths as well continents and oceans is more during winter rather than
hence option (b) is also incorrect water vapour is a summer. Hence statement-1 is incorrect the specific
patent absorber of infrared radiation particularly in heat is the amount of heat energy required to raise the
certain wavelength bands. In the lower atmosphere temperature of a substance by a certain amount. Water
where the concentration of water vapour a relatively has a relatively high specific heat compare to land
higher, absorbs a significant amount of infrared surfaces. Hence statement 2 is correct.
radiation emitted by the Earth's surface. This absorption 75. Consider the following statements :
contributes to the greenhouse effect and plays a crucial 1. In a seismograph, P waves are recorded
role in regulating the earth's temperature. Hence option earlier than 5 waves.
(c) is correct. 2. In P waves, the individual, particles vibrate
Infrared waves and visible light waves are distinct parts to and fro in the direction of wave
of electromagnetic spectrum. propagation whereas in S waves, the
The visible spectrum ranges from approximately 400 to particles vibrate up and down at right
700 Nano meters and includes the colours are perceive, angles to the direction of wave
such as red, orange yellow green blue. Hence option (d) propagation.
is incorrect. Which of the statements given above is/are
73. Consider the following statements : correct?
Statement-I : The soil in tropical rain forests in (a) 1 Only (b) 2 Only (c) Both 1 and 2 (d) Neither 1 nor 2
rich in nutrients. Ans. (c) : P-waves, also known as primary waves are
Statement II : The high temperature and the first waves to arrive at seismograph, P. waves are
moisture of tropical rain forests cause dead the fastest seismic waves and can move through solid,
organic matter in the soil to decompose quickly. liquid and gas. P-waves are longitudinal waves, which
Which one of the following is correct in respect means that the particle motion occur parallel to the
of the above statements : direction of wave propagation which means the
(a) Both Statement-I and Statement-II are correct particles oscillates back and forth in the same direction
and Statement-II is the correct explanation for that the wave is travelling. Hence both statements are
Statement-I. correct.
IAS (Pre) GS 2023 Paper I 14A YCT
76. With reference to coal-based thermal power 79. 'Aerial metagenomics' best refers to which one
plants in India, consider the following of the following situations?
statements : (a) Collecting DNA samples from air in a habitat
1. None of them uses seawater. at one go
2. None of them is set up in water stressed (b) Understanding the genetic makeup of avian
district. species of a habitat
3. None of them is privately owned. (c) Using air-home devices to collect blood
How many of the above statements are correct? samples from moving animals
(d) Sending drones to inaccessible areas o collect
(a) Only one (b) Only two (c) All three (d) None plant and animal samples from land surfaces
Ans. (d) : The Mundra Thermal Power Plant setlizes and water bodies
seawater drawn from Gulf Kutch. Thus statement (1) is Ans. (a : 'Aerial met genomics' refers to the collection
incorrect. and analysis of DNA samples from the air or aerosol
According to recent study of World Resources Institute particles present in a specific habitat. It involves
40 percent India's thermal power plants are situated in sampling the air in a particular environment to capture
regions experiencing significant water stress. Thus the genetic material (PDNA) in micro-organisms,
statement (2) is incorrect. including bacteria, viruses, Fungi and other microscopic
India has a total 269 thermal power plants of which 138 organisms.
owned by public sector and remaining 131 by private. 80. 'Microsatellite DNA' is used in the case of
Thus statement (3) is incorrect. which one of the following?
77. 'Wolbachia method' is sometimes talked about (a) Studying the evolutionary relationships
with reference to which one of the following? among various species of fauna
(a) Controlling the viral diseases spread by (b) Stimulating 'stem cells' to transform into
mosquitoes diverse functional tissues
(b) Converting crop residues into packing (c) Promoting clonal propagation of horticultural
material plants
(d) Assessing the efficacy of drugs by conducting
(c) Producing biodegradable plastics series of drug trials in a population
(d) Producing biochar from thermo-chemical
conversion of biomass Ans. (a) : Microsatellite DNA, also known as short
tandem repeats, is a type of genetic marker that consist
Ans. (a) : The 'Walbachia method' is a technique used of repeating sequences of DNA. It is commonly used in
to control the spread of viral disease transmitted by genetic studies, including population genetics and
mosquitoes, such as dengu Zika and Chikangunya. phylogentic analysis, to study relationship among
Wolbachia is a type of bacterium that can be introduced different species of Fauna.
into mosquitoes to reduce their ability to transmit these 81. Consider the following statements in relation to
diseases. When mosquitoes carrying Wolbachia mate Janani Suraksha Yojana :
with mosquitoes without Wolbachia the resulting eggs 1. It is a safe motherhood intervention of the
do not hatch or produce oppspring, thus reducing the State Health Departments.
population of disease carrying mosquitoes. 2. Its objective is to reduce maternal and
78. Consider the following activities : neonatal mortality among poor pregnant
1. Spreading finely ground basalt pick on women.
farmlands extensively. 3. It aims to promote institutional delivery
2. Increasing the alkalinity of oceans by among poor pregnant women.
adding lime. 4. Its objective includes providing public
3. Capturing carbon dioxide released by health facilities to sick infants up to one
various industries and pumping it into year of age.
abandoned subterranean mines in the form How many of the statements given above are
of carbonated waters correct.
(a) Only one (b) Only two (c) All three (d) All four
How many of the above activities are often
considered and discussed for carbon capture Ans. (c) : Janani Suraksha Yojna is a safe motherhood
and sequestration? intervention under the National Health Mission. It is not
(a) Only one (b) Only two (c) All three (d) None the intervention of State Health Departments.
The main objectives JSX are to reduce maternal and
Ans. (c) : Spreading finely ground basalt rock on neonatal mortality by promoting institutional delivery
Farmlands accelerates natural process of carbon dioxide among pregnant women especially with weak socio-
capture from the atmosphere. The basalt reacts with economic status.
carbon dioxide to form stable carbonates, effectively In 2014 the programme was extended to all antenatal &
storing carbon. post-natal complications of all sick newborns and
Increasing alkalinity of the ocean by addition of lime infants (upto one year of age) accessing public health
enhances fire capacity of oceans to absorb and store institutions for treatment.
carbon dioxide from the atmosphere thereby aiding in 82. Consider the following statements in the
carbon sequestration. context of interventions being undertaken
Capturing carbon dioxide released by industries and under Anaemia Mukt Bharat Strategy :
pumping it into abandoned subterranean mines is 1. It provides prophylactic calcium
carbon capture and storage method. Abandoned supplementation for pre-school children,
subterranean sites, with the captured carbon dioxide adolescents and pregnant women.
pumped into these locations in the form of carbonated 2. It runs a campaign for delayed cord
waters. clamping at the time of childbirth.
IAS (Pre) GS 2023 Paper I 15A YCT
3. It provides for periodic deworming to 2. Biofilters convert ammonia present in fish
children and adolescents. waste to nitrate.
4. It addresses non-nutritional causes of 3. Biofilters increase phosphorus as nutrient
anaemia in endemic pockets with special for fish in water.
focus on malaria, hemoglobinopathies and How many of the statements given above are
fluorosis. correct?
How many of the statements given above are (a) Only one (b) Only two (c) All three (d) None
correct? Ans. (b) : Biofilters are important components of
(a) Only one (b) Only two (c) All three (d) All four Recirculating Aquococulture systems to remove
Ans. (c) : Anaemia Mukt Bharat strategy provides nitrogenous wastes excretes by aquatic organisms. Here
prophylactic Iron folic acid supplement (not nitrifying bacteria are used as biofilters to oxidise
prophylactic calcium) for pre-school children, ad ammonia and convert it into nitrites which are also
descents and pregnant women. It also ruins a campaign toxic. Some bacteria also convert these nitrites into final
for delayed cord clamping at the time of child birth. It products which are non-toxic nitrates.
provides for periodic dewarming to children and Biofilters are designed to filter pollutants and not to add
adolescents. It also addresses non nutritional causes of nutrients. Thus statements 1 & 2 are correct and 3 is
anemia endemic pockets with special focus on malaria incorrect.
hemoglobin pathies and flurosis.
86. Consider the following pairs :
83. Consider the following statements :
1. Carbon fibres are used in the manufacture Objects in space Description
of components used in automobiles and 1. Cepheids : Grant clouds of dust
aircrafts. and gas in space
2. Carbon fibres once used cannot be correct. 2. Nebulae : Stars which brighten
Which of the statements given above is/are and dim periodically
correct? 3. Pulsars : Neutron stars that
(a) 1 only (b) 2 only (c) Both 1 and 2 (d) Neither 1 nor 2 are formed when
Ans. (a) : Carbon Fibre is increasingly celebrated as a massive stars run out
wonder material for the clean economy. Its unique of fuel and collapse
combination of high strength and low weight has helped How many of the above pairs are correctly
drive the wind power revolution and makes planes more matched?
fuel efficient Auto makers are also waking up to the (a) Only one (b) Only two (c) All three (d) None
materials potential to make lighter and more efficient Ans. (a) : Cepheids are stars which brighted and dim
vehicles. periodically. Thus Pair-1 is incorrect.
Though difficult to recycle, carbon Fiber can be A nebula is giant cloud of dust and gas in space. Thus
recycled. pair 2 is incorrect.
Thus statement 1 is correct and statement 2 is incorrect. Pulsars are neutron stars. Thus pair 3 is correct.
84. Consider the following actions : 87. Which one of the following countries has its
1. Detection of car crash/collision which own Satellite Navigation System?
results in the deployment of airbags almost (a) Australia (b) Canada (c) Israel (d) Japan
instantaneously
2. Detection of accidental free fall of a laptop Ans. (d) : Quasi-Zenith satellite (8255) system is a
towards the ground which results in the regional satellite navigation system owned by
immediate turning off of the hard drive Government of Japan.
3. Detection of the tilt of the smart-phone 88. Consider the following statements :
which results in the rotation of display 1. Ballistic missiles are jet-propelled at
between portrait and landscape mode subsonic speeds throughout their flights,
In how many of the above actions is the while cruise missiles are rocket-powered
function of accelerometer required? only in the initial phase of flight.
(a) Only one (b) Only two (c) All three (d) None 2. Agni-V is a medium-range supersonic
cruise missile, while BrahMos is a solid-
Ans. (c) : Accelerometer are devices that measure the fuelled intercontinental ballistic missile.
vibration or acceleration of motion of a structure.
Accelerometer is used in section of car collision/crash Which of the statements given above is/are
which results in the deployment of airbags almost correct?
instantaneously. If laptop were to suddenly drop while (a) 1 only (b) 2 only (c) Both 1 and 2 (d) Neither 1 nor 2
in use, the accelerometer would defect sudden free fall Ans. (d) : Cruise Missiles are jet propelled at subsonic
and immediately turn off the hard drive to avoid hitting speeds throughout their flights, while ballistic missiles
the reading heads into hard drive platter. are rocket-powered only in the initial phase of flight
An accelerometer detect tilt of the Smartphone which after which they follow an arching trajectory to the
results in rotation of display between portrait and target.
landscape mode Agni-V is a ballistic missile while the BRAHMOS is a
Thus all three statements are correct. medium range ramjet Supersonic cruise missile.
85. With reference to the role of biofilters in Thus both the statements are incorrect.
Recirculating Aquaculture System, consider 89. Consider the following statements regarding
the following statements : mercury pollution :
1. Biofilters provide waste treatment by 1. Gold mining activity is a source of
removing uneaten fish food. mercury, pollution in the world.
IAS (Pre) GS 2023 Paper I 16A YCT
2. Coal-based thermal power plants cause Ans. (c) : Statement 1 is correct. It is possible to make
mercury pollution. payments in digital currencies without using US dollar
3. There is no known safe level of exposure to or SWIFT system statement 2 is correct. The digital
mercury. Yuan is programmable to the extent that the currency
How many of the above statements are correct? can be designed to expire, forcing users to spend it all
(a) Only one (b) Only two (c) All three (d) None by a specific date. It means that the digital currencies
Ans. (c) : Mercury is used in mining of Gold thus a can be programmed to be spent within a predetermined
source of pollution. Statement 1 is correct. time range.
Coal based Thermal power plants are major source of 93. In the context of finance, the term 'beta' refers to
mercury. Thus statement 2 is correct. (a) the process of simultaneous buying and
Mercury is a highly toxic element, there is no sate level selling of an asset from different platforms
of exposure. Thus statement 3 is also correct. (b) an investment strategy of a portfolio manager
90. With reference to green hydrogen, consider the to balance risk versus reward
following statements : (c) a type of systemic risk that arises where
1. It can be used directly as a fuel for internal perfect hedging is not possible
combustion. (d) a numeric value that measures that
2. It can be blended with natural gas and fluctuations of a stock to changes in the
used as fuel for heat or power generation. overall stock market.
3. It can be used in the hydrogen fuel cell to Ans. (d) : Beta is a concept by which a stock's expected
run vehicles. movement in relation to overall market movements is
How many of the above statements are correct? measured. A stock with a beta greater than is considered
(a) Only one (b) Only two (c) All three (d) None to be more volatile than the broader market while the
stock with Beta less than 1 is considered to be less
Ans. (c) : Both hydrogen internal combustion engines volatile.
and hydrogen fuel cell can power vehicles using
hydrogen. Hydrogen engines burn hydrogen in an 94. Consider the following statements :
internal combustion engine. Thus statement 1 is correct. 1. The Self-Help Group (SHG) programme
Hydrogen can be blended with natural gas and can be was originally initiated by the State Bank
used as fuel with lower emission. Thus statement 2 is of India by providing microcredit to the
correct. financially deprived.
Fuel cell electric vehicles are powered by hydrogen. 2. In an SHG, all members of a group take
They are more efficient than internal combustion responsibility for a loan that an individual
engine. Thus statement is correct. member takes.
3. The Regional Rural Banks and Scheduled
91. Consider the following statements with Commercial Banks support SHGs.
reference to India : How many of the above statements are correct?
1. According to the Micro, Small and Medium (a) Only one (b) Only two (c) All three (d) None
Enterprises Development (MSMED) Act,
2006', the 'medium enterprises' are those Ans. (b) : Statement 1 is incorrect : NABARD began
with investments in plant and machinery promoting SHG's in 1981-1982 and it served as the
between r 15 crore and r 25 crore. launch pat for SHG movement.
2. All bank loans to the Micro, Small and Statement 2 is correct. The banks provide the loan to
Medium Enterprises qualify under the SHG as a whole and it is the collective responsibility of
priority sector. the SHG to repay that loan. Hence all members of a
Which of the statements given above is/are group take responsibility for a loan that an individual
correct? member takes.
(a) 1 only (b) 2 only Statement 3 is correct. Both SCB's and RRB's support
(c) Both 1 and 2 (d) Neither 1 nor 2 the SHG is avoiding credit as per their needs.
Ans. (b) : As per the MSMED Act 2006, the medium Thus option (b) is correct.
enterprises are those for which the investment in plant 95. Consider the following statements :
and machinery is not more than Rs. 50 crore and Statement-I : India's public sector health care
turnovers not more than Rs. 250 crore. Thus statement 1 system largely focuses on curative care with
is incorrect. limited preventive, promotive and rehabilitative care.
According to RBI's Master circular on Priority sector Statement II : Under India's decentralized
lending, all the Back loans to the MSME's qualify and approach to health care delivery, the States are
priority sector lending. Thus statement 2 is correct. primarily responsible for organizing health services.
92. With reference to Central Bank digital Which one of the following is correct in respect
currencies, consider the following statements : of the above statements :
1. It is possible to make payments in a digital (a) Both Statement-I and Statement-II are correct
currency without using US dollar or and Statement-II is the correct explanation for
SWIFT systems. Statement-I.
2. A digital currency can be distributed with (b) Both Statement-I and Statement-II are correct
a condition programmed into it such as a and Statement-II is not the correct
time-frame for spending it. explanation for Statement-I.
Which of the statements given above is/are (c) Statement-I is correct but Statement-II is
correct? incorrect.
(a) 1 only (b) 2 only (d) Statement-I is incorrect but Statement-II is
(c) Both 1 and 2 (d) Neither 1 nor 2 correct.
IAS (Pre) GS 2023 Paper I 17A YCT
Ans. (b) : As per the National Health Policy 2017. 98. Which one of the following countries has been
India's Public Sector health core system focuses on suffering from decades of civil strife and food
improving health status through coordinated police shortages and was in news in the recent past
action in all sectors. It also focuses on expanding for its very severe famine?
preventive promotive, curative, palliative and (a) Angola (b) Costa Rica (c) Ecuador (d) Somalia
rehabilitative service through public health sector Ans. (d) : In Somalia 6.5 million people are currently
which focuses on quality of these services. facing Severe Food insecurity amid the driest condition
The states are primarily incharge of organising health in 40 years after is consecutive failed rainy seasons.
services under initials decentralised system at providing Also there was a civil strike of three decades in
health core. 'Health and Sanitation' is in states list under Somalia.
schedule 7 of the constitution. 99. Consider the following statements :
Thus Statement I and Statement II both are correct 1. In India, the Biodiversity Management
however as statement II does not explain statement I. Committees are key to the realization of
96. Consider the following statements : the objectives of the Nagoya Protocol.
Statement-I : According to the United Nations 2. The Biodiversity Management Committees
World Water Development Report, 2022, India have important functions in determining
extracts more than a quarter of the world's access and benefit sharing, including the
groundwater withdrawal each year. power to levy collection fees on the access
Statement II : India needs to extract more than a of biological resources within its jurisdiction.
quarter of the world's groundwater each year to Which of the statements given above is/are
satisfy the drinking water and sanitation needs of correct?
almost 18% of world's population living in its (a) 1 only (b) 2 only (c) Both 1 and 2 (d) Neither 1 nor 2
territory.
Ans. (c) : NAGOYA protocol aims to equitable share
Which one of the following is correct in respect the benefits resulting from the use of genetic resources.
of the above statements : According to Biological Diversity Act 2002, every local
(a) Both Statement-I and Statement-II are correct body in the state shall establish a Biodiversity
and Statement-II is the correct explanation for Management Committee within its area of jurisdiction
Statement-I. for the purpose of promoting biological diversity
(b) Both Statement-I and Statement-II are correct conservation, sustainable use, and documentation. In
and Statement-II is not the correct India, the BHC's functions as the agency for the
explanation for Statement-I. implementations of the objectives of Nagoya Protocol.
(c) Statement-I is correct but Statement-II is The Biodiversity Act gives BHC's the independent
incorrect. authority to charge those who collect biological
(d) Statement-I is incorrect but Statement-II is resources within their territorial jurisdiction by way of
correct. collection of fees.
Ans. (c) : According to United Nations World Water 100. Consider the following statements in respect of
Development Report 2022, India is world's largest election to the President of India :
groundwater user. This accounts for more than a quarter 1. The members nominated to either House of
of the total ground water extracted internationally. the Parliament or the Legislative
India uses 89% of extracted ground water for irrigation. Assemblies of States are also eligible to the
Thus major share of extracted groundwater is used in included in the Electoral College.
irrigation and not for drinking and sanitation purpose. 2. Higher the number of elective Assembly
Thus Statement I is correct and Statement II is incorrect. seats, higher is the virtue of vote of each
97. Consider the following statements : MLA of that State.
1. According to the Constitution of India, the 3. The value of vote of each MLA of Madhya
Central Government has a duty to protect Pradesh is greater than that of Kerala.
States from internal disturbances. 4. The value of vote of each MLA of
2. The Constitution of India exempts the Puducherry is higher than that of
States from providing legal counsel to a Arunachal Pradesh because the ratio of
person being held for preventive detention. total population to total number of elective
3. According to the Prevention of Terrorism seats in Puducherry is greater as compared
Act, 2002, confession of the accused before to Arunachal Pradesh.
the police cannot be used as evidence. How many of the above statements are correct?
How many of the above statements are correct? (a) Only one (b) Only two (c) All three (d) All four
(a) Only one (b) Only two (c) All three (d) None Ans. (a) : The electoral college for election of President
includes elected members of Parliament and state
Ans. (b) : Article 355 of the constitution provides for legislative assemblies. Thus statement I is incorrect.
the responsibility of Union to protect states from The value of vote of each MCA depends on the ratio of
external aggression and internal disturbance. the state is population and the number of MLAs in its
Article 22 of constitution of India exempts the states legislative assembly and not just on number of elective
from providing legal counsel to a person being held for seats. Thus statement is incorrect.
preventive detention. The value of vote of each MLA of Madhya Pradesh is
As per Section 32(1) of POTA, 2002, Confusion of the less than that of vote value of each MLA of Kerala.
accused before a police officer not lower in rank than a Thus statement is incorrect.
Superintendent of police can be used as evidence. The vote value of each MLA of Puducherry is higher
Thus statements 1 and 2 are correct and statement 3 is than that of Arunachal Pradesh. Thus statement is
incorrect. correct.
IAS (Pre) GS 2023 Paper I 18A YCT
UNION PUBLIC SERVICE COMMISSION
Civil Services (Preliminary Exam) - 2022
GENERAL STUDIES : PAPER-I
Time : 2 hours (Exam date : 05.06.2022) Maximum Number : 200
1. “Rapid Financing Instrument” and “Rapid 2. If the rupee is rapidly depreciating, RBI is likely
Credit Facility” are related to the provisions of to sell dollars in the market.
lending by which one of the following? 3. If interest rates in the USA or European Union
(a) Asian Development Bank were to fall, that is likely to induce RBI to buy
(b) International Monetary Fund dollars.
(c) United Nations Environment Programme Which of the statements given above are correct?
Finance Initiative (a) 1 and 2 only (b) 2 and 3 only
(d) World Bank (c) 1 and 3 only (d) 1, 2 and 3
Ans. (b) : The Rapid Financing Instrument (RFI) is Ans. (b): If the inflation is too high, the Reserve Bank
the lending facility of the International Monetary Fund of India (RBI) is likely to reduce the money supply in
(IMF) that provides rapid financial assistance, which is the economy to control inflation. Thus, RBI sells the
available to all member countries, who are facing an government securities so as to suck the excess of the
urgent balance of payments need. The Rapid Financing money supply from the economy to control the
Instrument was created as part of a broader reform to inflation.
make the IMF’s financial support more flexible to The Reserve Bank of India intervenes in the currency
address the diverse needs of member countries. market to support the rupee as a weak domestic unit can
Whereas, the Rapid Credit Facility (RCF) facility of increase country’s import bill. If RBI wishes to prop up
IMF provides rapid concessional financial assistance to the rupee value, then it can sell dollar and when it needs
low-income countries facing an urgent balance of to bring down rupee value, it can buy dollars.
payments (BoP) need with no ex post conditionality, When the United States raises its domestic interest
where a full-fledged economic program is neither rates, this tends to make India less attractive for the
necessary nor feasible. currency trade. As a result, some of the money may be
2. With reference to the Indian economy, consider expected to move out of the Indian markets and flow
the following statements: back to the United States, therefore decreasing the
1. An increase in Nominal Effective Exchange value of India’s currency against the US dollar. Thus, if
Rate (NEER) indicates the appreciation of interest rates in the USA or European Union were to
rupee. fall, the value of rupee against the dollar increases and
2. An increase in the Real Effective Exchange that is likely to induce RBI to buy dollars.
Rate (REER) indicates an improvement in Hence only 2nd and 3rd statements are correct.
trade competitiveness. 4. With reference to the “G20 Common
3. An increasing trend in domestic inflation Framework”, consider the following statements:
relative to inflation in other countries is 1. It is an initiative endorsed by the G20
likely to cause an increasing divergence together with the Paris Club.
between NEER and REER. 2. It is an initiative to support Low Income
Which of the above statements are correct? Countries with unsustainable debt.
(a) 1 and 2 only (b) 2 and 3 only Which of the statements given above is/are correct?
(c) 1 and 3 only (d) 1, 2 and 3 (a) 1 only (b) 2 only
Ans. (c) : Statement 1 is correct: NEER (Nominal (c) Both 1 and 2 (d) Neither 1 nor 2
Effective Exchange Rate) is a measure of the value of a Ans. (c) : The Common Framework for debt treatment
currency against a weighted average of several foreign beyond the DSSI (Common Framework) is an initiative
currencies. An increase in NEER indicates an endorsed by the G20, together with the Paris Club to
appreciation of the local currency against the weighted support in a structural manner for Low-Income
basket of currencies of its trading partners. Countries with unsustainable debt. The Common
Statement 2 is incorrect: REER (Real Effective Framework is intended to deal with insolvency and
Exchange Rate) is the measure of the value of a protracted liquidity problems, along with the
currency against a weighted average of several foreign implementation of an IMF-supported reform program.
currencies divided by a price deflator or index of costs. 5. With reference to the Indian Economy, what are the
An increase in REER implies that exports become more advantages of “Inflation-Indexed Bonds (IIBs)”?
expensive and imports become cheaper; therefore, an 1. Government can reduce the coupon rates on its
increase indicates a loss in trade competitiveness. borrowing by way of IIBs.
Statement 3 is correct: An increasing trend in 2. IIBs provide protection to the investors from
domestic inflation relative to inflation in other countries uncertainty regarding inflation.
is likely to cause an increasing divergence
between NEER and REER. 3. The interest received as well as capital gains
on IIBs are not taxable.
3. With the reference to the Indian economy Which of the statements given above are correct?
consider the following statements : (a) 1 and 2 only (b) 2 and 3 only
1. If the inflation is too high, Reserve Bank of
India (RBI) is likely to buy government securities. (c) 1 and 3 only (d) 1, 2 and 3
IAS (Pre) GS 2022 Paper I 19A YCT
Ans. (a) : Inflation-indexed bonds (IIBs) provide Ans. (d) : Indirect transfers refer to situations when
insurance to investors from inflation and cost savings foreign entities own shares or assets in India. The
for the Government on account of reduction in coupon shares of such foreign entities are transferred instead of
payments with lowering inflation rate, elimination of a direct transfer of the underlying assets in India.
uncertainty risk premium and containing inflationary 9. With reference to the expenditure made by an
expectations. Hence statements 1 and 2 are correct. organization or a company, which of the
However, extant tax provisions will be applicable on following statements is /are correct?
interest payment and capital gains on IIBs. There will 1. Acquiring new technology is capital expenditure.
be no special tax treatment for these bonds. Therefore 2. Debt financing is considered capital expenditure,
3rd statement is incorrect. while equity financing is considered revenue
6. With reference to foreign-owned e- commerce expenditure.
firms operating in India, which of the following Select the correct answer using the code given
statements is/are correct? below :
1. They can sell their own goods in addition to (a) 1 only (b) 2 only
offering their platforms as market-place. (c) Both 1 and 2 (d) Neither 1 nor 2
2. The degree to which they can own big Ans. (a) : Capital expenditures are funds used by a
sellers on their platforms is limited. company to acquire, upgrade, and maintain physical
Which of the statement given above are correct? assets such as property, plants, buildings, technology,
(a) 1 only (b) 2 only or equipment.
(c) Both 1 and 2 (d) Neither 1 nor 2 When a company borrows money to be paid back at a
Ans. (b) : Statement 1 is incorrect: Foreign firms are future date with interest it is known as debt
prevented from holding inventory or selling their own financing. Repayment of loan is an example of capital
goods. They can offer their platforms only as expenditure. Equity financing is the process of raising
“marketplaces” for other buyers and sellers. capital through the sale of shares. It is an example of
non-debt capital receipts. Capital receipts are receipts
Statement 2 is correct: The degree to which they can that create liabilities or reduce financial assets. They
own big sellers on their platforms is limited, to prevent also refer to incoming cash flows. Therefore only
those sellers from acting covertly on their behalf. statement1 is correct.
7. Which of the following activities constitute real 10. With reference to the Indian economy, consider
sector in the economy? the following statements:
1. Farmers harvesting their crops 1. A share of the household financial savings goes
2. Textile mills converting raw cotton into fabrics towards government borrowings.
3. A commercial bank lending money to a trading 2. Dated securities issued at market-related rates in
company auctions form a large component of internal
4. A corporate body issuing Rupee Denominated debt.
Bonds overseas Which of the above statements is/are correct?
Select the correct answer using the code given (a) 1 only (b) 2 only
below : (c) Both 1 and 2 (d) Neither 1 nor 2
(a) 1 and 2 only (b) 2, 3 and 4 only Ans. (c) : Household Savings is often deposited in
(c) 1, 3 and 4 only (d) 1, 2, 3 and 4 financial institutions, which are used by them to buy
Ans. (a) : The real sector of the economy consists of government securities. Hence, Statement-1 is correct.
enterprises (non-financial corporations), households Internal debt consists of marketable debt and non-
and non-profit institutions serving households. Non- marketable debt. Marketable debt comprises of
financial organizations comprise all resident units Government dated securities and Treasury Bills. Dated
involved in the production of market goods and non- Securities form a large component of internal debt.
financial services. For example, farmers harvesting their 11. Consider the following statements:
crops and Textile mills converting raw cotton into fabrics. 1. Pursuant to the report of H.N. Sanyal
Corporations principally engaged in financial Committee, the Contempt of Courts Act, 1971
intermediation or in auxiliary financial activities that was passed.
contribute to financial intermediation are considered as 2. The Constitution of India empowers the
the financial sector. Hence only options 1 and 2 are correct. Supreme Court and the High Courts to punish
8. Which one of the following situations best reflects for contempt of the mselves.
“Indirect Transfers” often talked about in media 3. The Constitution of India defines Civil
recently with reference to India? Contempt and Criminal Contempt.
(a) An Indian company investing in a foreign 4. In India, the Parliament is vested with the
enterprise and paying taxes to the foreign Powers to make laws on Contempt of Court.
country on the profits arising out of its investment Which of the statements given above is/are correct?
(b) A foreign company investing in India and (a) 1 and 2 only (b) 1, 2 and 4
paying taxes to the country of its base on the (c) 3 and 4 only (d) 3 only
profits arising out of its investment Ans. (b) : The Contempt of Courts Act, 1971 was
(c) An Indian company purchases tangible assets in passed after the Report of the H. N. Sanyal Committee
a foreign country and sells such assets after their on Contempt of Courts, 1963. Hence 1st statement is
value increases and transfers the proceeds to correct.
India The Supreme Court and the High Courts of India have
(d) A foreign company transfers share and such been empowered with the power to penalize for
shares derive their substantial value from assets Contempt of Court under Articles 129 and 215 of the
located in India. Constitution of India. Hence 2nd statement is correct.
IAS (Pre) GS 2022 Paper I 20A YCT
The Constitution of India does not define Civil Which of the statements given above is/are correct?
Contempt and/or Criminal Contempt. Hence Statement (a) 1 only (b) 2 only (c) Both 1 and 2 (d) Neither 1 nor 2
3 is incorrect. Ans. (b): There is no classification in the Constitution
Parliament framed the Contempt of Courts Act, 1971, of India for the Council of Ministers. Article 74
laying down procedure and punishment for contempt of provides that there shall be a Council of Ministers with
court. Parliament has power to amend certain clauses of the Prime Minister at the head to aid and advise the
the Contempt of Courts Act, 1971 but according to President who shall, in the exercise of his stfunctions, act
latest Supreme Court ruling, no law can take away in accordance with such advice. Hence 1 statement is
Court’s power to punish for contempt. Hence statement incorrect.
4 is correct. According to the Constitution (91st Amendment) Act,
12. With reference to India, consider the following 2003, that amended article 75, the total number of
statements: ministers, including the Prime Minister, in the cabinet
1. Government law officers and legal firms are of ministers shall not exceed 15% of the total strength
recognised as advocates, but corporate lawyers of the Lok Sabha. Hence 2nd statement is correct.
and patent attorneys are excluded from 15. Which of the following is/are the exclusive
recognition as advocates. power(s) of Lok Sabha?
2. Bar Councils have the power to lay down the 1. To ratify the declaration of Emergency
rules relating to legal education and recognition
of law colleges. 2. To pass a motion of no-confidence against the
Council of Ministers.
Which of the statements given above is/are correct?
(a) 1 only (b) 2 only (c) Both 1 and 2 (d) Neither 1 nor 2 3. To impeach the President of India
Select the correct answer using the code given below:
Ans. (b): Corporate Lawyers, as well as patent (a) 1 and 2 (b) 2 only (c) 1 and 3 (d) 3 only
attorneys, are too recognized as lawyers and there’s no
prohibition on their recognition as lawyers. Hence 1st Ans. (b) : The Proclamation of Emergency must be
statement is incorrect. The Bar Council of India visits approved by both the Houses of Parliament within one
and inspects Universities/Law colleges in the country month from the date of its issue. If approved by both
as part of its statutory function of promoting legal the Houses of Parliament, the emergency continues for
education and laying down standards in consultation six months, and can be extended to an indefinite period
with the Universities in India and the State Bar with an approval of the Parliament for every six
Councils. The Bar Council of India is a statutory body months. As per Article 61, when a President is to be
established under section 4 of the Advocates Act 1961 impeached for violation of the Constitution, the charges
that regulates the legal practice and legal education in can be initiated by either House of Parliament. The
India. Hence 2nd statement is correct. impeachment resolution needs to be passed by a
13. Consider the following statements: majority of two-thirds of the total membership in each
1. A bill amending the Constitution requires a house. Thus, it is not an exclusive power of the Lok
prior recommendation of the President of India. Sabha.
2. When a Constitution Amendment Bill is However Article 75 (3) provides that the Council of
presented to the President of India, it is Ministers shall be collectively responsible to the House
obligatory for the President of India to give of the People. So, only LokSabha can pass a no-
his/her assent. confidence motion against the Council of Ministers.
Hence it is an exclusive power of the Lok Sabha.
3. A Constitution Amendment Bill must be passed
by both the Lok Sabha and the Rajya Sabha by a 16. With reference to anti-defection law in India,
special majority and there is no consider the following statements:
provision for joint sitting. 1. The law specifies that a nominated legislator
Which of the statements given above is/are cannot join any political party within six months
correct? of being appointed to the House.
(a) 1 and 2 only (b) 2 and 3 only 2. The law does not provide any time-frame within
(c) 1 and 3 only (d) 1, 2 and 3 which the presiding officer has to decide a
defection case.
Ans. (b): The Bill amending the constitution does not Which of the statements given above is/are correct?
require a priorst recommendation from the President of
India. Hence 1 statement is incorrect. (a) 1 only (b) 2 only (c) Both 1 and 2 (d) Neither 1 nor 2
When a Constitution Amendment Bill is presented to Ans. (b): A nominated member of a House shall be
the President of India, it is obligatory for the President disqualified for being a member of the House if he joins
of India to give his/her assent. The President cannot any political party after the expiry of six months from
exercise his veto powers with reference to the the date on which he takes his seat after complying with
Constitution amendment bill. Also according to Article the requirements of article 99 or, as the case may be.
368, the Constitution can be amended only by a 2/3rd Hence 1st statement is incorrect. The law does not
majority in both Houses. There is no provision for a specify a time period for the Presiding Officer to decide
joint sitting in case of a disagreement between both on a disqualification plea under the Tenth Schedule.
Houses. Hence 2nd and 3rd statements are correct. Hence 2nd statement is correct.
14. Consider the following statement: 17. Consider the following statements:
1. The Constitution of India classifies the 1. Attorney General of India and Solicitor General
ministers into four ranks viz. Cabinet Minister, of India are the only officers of the Government
Minister of State with Independent Charge, who are allowed to participate in the meetings
Minister of State and Deputy Minister. of the Parliament of India.
2. The total number of ministers in the Union 2. According to the Constitution of India, the
Government, including the Prime Minister, shall Attorney General of India submits his
not exceed 15 percent of the total number of resignation when the Government which
members in the Lok Sabha. appointed him resigns.
IAS (Pre) GS 2022 Paper I 21A YCT
Which of the statement given above is/are correct? 2. There is a mandatory provision that election of a
(a) 1 only (b) 2 only (c) Both 1 and 2 (d) Neither 1 nor 2 candidate as Deputy Speaker of Lok Sabha shall
Ans. (d) : However, the Attorney General of India has be from either the principal opposition party or
the right to audience in all Courts in India as well as the the ruling party.
right to take part in the proceedings of Parliament 3. The Deputy Speaker has the same power as of
without the right to vote. The Solicitor General of India the Speaker when presiding over the sitting of
has no such privilege. the House and no appeal lies against his rulings.
As per Article 76, the Attorney General shall hold 4. The well established parliamentary practice
office during the pleasure of the President. There is no regarding the appointment of Deputy Speaker is
mention in the Indian Constitution of the need for him that the motion is moved by the Speaker and
to submit his resignation when the Government which duly seconded by the Prime Minister.
appointed him resigns. Which of the statements given above are correct?
Hence both the statements are incorrect. (a) 1 and 3 only (b) 1, 2 and 3
(c) 3 and 4 only (d) 2 and 4 only
18. With the reference to the writs issued by the
Courts in India, consider the following Ans. (a): As per Rule 8 of Procedure and Conduct of
statements: Business in Lok Sabha, the election of a Deputy
1. Mandamus will not lie against a private Speaker shall be held on such date as the Speaker may
oragnisation unless it is entrusted with a public fix, and the Secretary-General shall send to every
duty. member notice of this date. There is no mandatory
provision that the election of a candidate, as Deputy
2. Mandamus will not lie against Company even Speaker of Lok Sabha, shall be from either the principal
though it may be a Government Company. opposition party or the ruling party. It is only by
3. Any public minded person can be a petitioner to convention that the position of Deputy Speaker is
move the Court to obtain the writ of Quo offered to the opposition party in India. In case of the
Warranto. absence of the Speaker, the Deputy Speaker presides
Which of the statements given above are correct? over the sessions of the Lok Sabha and conducts the
(a) 1 and 2 only (b) 2 and 3 only business in the house. The Deputy Speaker has the
(c) 1 and 3 only (d) 1, 2 and 3 same power as the Speaker when presiding over the
Ans. (c): Mandamus is a command issued by the court sitting of the House. The Deputy Speaker is also elected
to a public official asking him to perform his official by the Lok Sabha from amongst its members right after
duties that he has failed or refused to perform. It can the election of the Speaker has taken place. There is no
also be issued against any public body, a corporation, provision and or established practice of moving the
an inferior court, a tribunal, or a government for the motion for his election by the speaker and it is
same purpose. It is usually not usable against a private seconded by the prime minister.
entity unless it is entrusted with a public duty. 21. Among the following crops, which one is the most
Quo Warranto is issued by the court to enquire into the important anthropogenic source of both methane
legality of the claim of a person to a public office. and nitrous oxide?
Hence, it prevents illegal usurpation of public office by (a) Cotton (b) Rice (c) Sugarcane (d) Wheat
a person. Unlike the other writs, this can be sought by Ans. (b): Rice (Paddy) is considered as one of the most
any interested person and not necessarily by the important sources of methane and nitrous oxide
aggrieved person. emissions, which have attracted considerable attention
Hence only 1st and 3rd statements are correct. due to their contribution to global warming.
19. With reference to Ayushman Bharat Digital 22. “System of Rice Intensification” of cultivation, in
Mission, consider the following statements: which alternate wetting and drying of rice fields
1. Private and public hospitals must adopt it. is practised, results in:
2. As it aims to achieve universal health coverage, 1. Reduced seed requirement
every citizen of India should be part of it 2. Reduced methane production
ultimately. 3. Reduced electricity consumption
3. It has seamless portability across the country. Select the correct answer using the code given below:
Which of the statements given above is/are correct? (a) 1 and 2 only (b) 2 and 3 only
(a) 1 and 2 only (b) 3 only (c) 1 and 3 only (d) 1, 2 and 3 (c) 1 and 3 only (d) 1, 2 and 3
Ans. (b): Only the 3rd statement is correct. Ayushman Ans. (d): The System of Rice Intensification involves
Bharat Digital Mission will have a national footprint cultivating rice with as much organic manure as
and will enable seamless portability across the country possible. It includes a method called Alternate Wetting
through a Health ID – Personal Health Identifier. The and Drying (AWD) which is a form of controlled or
Mission (ABDM) has not made it mandatory for intermittent irrigation of the rice crops. It has the
adoption by private and public hospitals. Its adoption so following benefits and impacts, compared to
far would be voluntary. The aim of the Ayushman conventional methods of paddy cultivation:
Bharat Digital Mission is to facilitate the ease of access It increases paddy yields usually by 20-50% and
to medical records and it is not been mandated to have sometimes up to 100%,
every citizen as its part though the facility is available It reduces required seeds for transplanting by 60-80%,
to every citizen on a consent basis. The electricity consumption will be less as it enables
reduce use of chemical fertilizers and agrichemicals and
20. With reference to Deputy Speaker of Lok Sabha, reduced use of irrigation water by 25-50%,
consider the following statements: By stopping flooding and relying mostly on organic
1. As per the Rules of Procedure and Conduct of fertilization, methane emissions are greatly reduced
Business in Lok Sabha, the election of Deputy without offsetting increases in nitrous oxide.
Speaker shall be held on such date as the Hence all the given statements are correct.
Speaker may fix.
IAS (Pre) GS 2022 Paper I 22A YCT
23. Which one of the following lakes of West Africa Which of the statements given above are correct?
has become dry and turned into a desert? (a) 1, 2 and 3 only (b) 1, 2 and 4 only
(a) Lake Victoria (b) Lake Faguibine (c) 3 and 4 only (d) 1, 2, 3 and 4
(c) Lake Oguta (d) Lake Volta Ans. (b): Though monazite is found in most coastal
Ans. (b) : Lake Faguibine in northern Mali started to areas of India. It is questionable for monazite to be
disappear after catastrophic droughts in the 1970s. Over found along entire coast. The main mines are found
several years, droughts in the 1970s dried up the lake. along the coasts of southern India in Kerala, Tamil
24. Gandikota canyon of South India was created by Nadu and in Orissa. Rests of the statements are correct.
which one of the following rivers?
29. In the northern hemisphere, the longest day of
(a) Cauvery (b) Manjira the year normally occurs in the:
(c) Pennar (d) Tungabhadra
(a) First half of the month of June
Ans. (c): Gandikota canyon of South India was created (b) Second half of the month of June
by The river Pennar. The swift-flowing waters of River
Pennar cut through the granite rocks near the village of (c) First half of the month of July
Gandikota, naturally eroding the rock and forming (d) Second half of the month of July
Gandikota Canyon. Ans. (b): The longest day of the year for those living
25. Consider the following pairs: north of the Equator is June 21. This day is referred to
Peak Mountains as the summer solstice. It occurs when the sun is
1. Namcha Barwa - Garhwal Himalaya directly over the Tropic of Cancer. Hence the correct
2. Nanda Devi - Kumaon Himalaya answer to the question would be 2nd half of the month
3. Nokrek - Sikkim Himalaya of June.
Which of the pairs given above is/are correctly 30. Consider the following pairs:
matched? Wetland/Lake Locations
(a) 1 and 2 (b) 2 only (c) 1 and 3 (d) 3 only 1. Hokera Wetland - Punjab
Ans. (b): Namcha Barwa is a mountain in Arunachal 2. Renuka Wetland - Himachal Pradesh
Pradesh and not in Garhwal Himalaya. It is considered 3. Rudrasagar Lake - Tripura
as the Eastern point of the Himalayas. Nanda Devi peak 4. Sasthamkotta Lake - Tamil Nadu
is in the state of Uttarakhand. It is a part of the Garhwal
Himalayas. The Nokrek is located in the northeast of How many pairs given above are correctly
India on the Tura Range, which forms part of the matched?
Meghalaya Plateau. Nokrek is the highest peak of the (a) Only one pair (b) Only two pairs
Garo hills. (c) Only three pairs (d) All four pairs
26. The term “Levant” often heard in the news Ans. (b) : The Hokera wetland is a Ramsar site at
roughly corresponds to which of the following Zainakote near Srinagar in Jammu and Kashmir.
regions? Renuka wetland is in the Sirmaur district of Himachal
(a) Region along the eastern Mediterranean shores Pradesh. Rudrasagar Lake, also known as Rudijala is a
(b) Region along North African shores stretching famous lake situated in the Melaghar Block under
from Egypt to Morocco Sonamura Sub-Division in the West Tripura District in
(c) Region along Persian Gulf and Horn of Africa Tripura. Sasthamkotta lake also known as the Queen of
(d) The entire coastal areas of Mediterranean Sea Lakes is the largest freshwater lake in Kerala. Hence
Ans. (a) : The Levant is an approximate historical only two pairs are correctly matched.
geographical term referring to a large area in the 31. Consider the following :
Eastern Mediterranean region of Western Asia, roughly 1. Aarogya Setu 2. CoWIN
corresponding to modern-day Israel, Jordan, Lebanon, 3. DigiLocker 4. DIKSHA
Syria, and certain adjacent areas.
Which of the above are built on top of open-
27. Consider the following countries: source digital platforms?
1. Azerbaijan 2. Kyrgyzstan (a) 1 and 2 only (b) 2, 3 and 4 only
3. Tajikistan 4. Turkmenistan (c) 1, 3 and 4 only (d) 1, 2, 3 and 4
5. Uzbekistan
Which of the above have borders with Afghanistan? Ans. (d): Open source software is software with source
code that anyone can inspect, modify, and enhance.
(a) 1, 2 and 5 only (b) 1, 2, 3 and 4 only
Digilocker, Diksha, Aarogya Setu, the Covid-19
(c) 3, 4 and 5 only (d) 1, 2, 3, 4 and 5 vaccination platform CoWIN have been built on top of
Ans. (c): Afghanistan is bounded to the east and south open-source digital platforms.
by Pakistan (including those areas of Pak occupied
Kashmir), to the west by Iran, and to the north by the 32. With reference to Web 3.0, consider the following
Central Asian states of Turkmenistan, Uzbekistan, and statements:
Tajikistan. 1. Web 3.0 technology enables people to control
28. With reference to India, consider the following their own data.
statements: 2. In Web 3.0 world, there can be blockchain
1. Monazite is a source of rare earths. based social networks.
2. Monazite contains thorium. 3. Web 3.0 is operated by users collectively rather
3. Monazite occurs naturally in the entire Indian than a corporation.
coastal sands in India. Which of the statements given above are correct?
4. In India, Government bodies only can process (a) 1 and 2 only (b) 2 and 3 only
or export monazite. (c) 1 and 3 only (d) 1, 2 and 3
IAS (Pre) GS 2022 Paper I 23A YCT
Ans. (d) : Web 3.0 is the third generation web 36. Consider the following communication
technology. Web 3.0 will have a strong emphasis on technologies:
decentralized applications and make extensive use of 1. Closed-circuit Television
blockchain-based technologies. An emerging 2. Radio Frequency Identification
governance ideal within the blockchain and Web 3.0 3. Wireless Local Area Network
community is the concept of a Decentralized Which of the above are considered Short-Range
Autonomous Organization (DAO). Instead of having a devices/ Technologies?
central authority that governs the operations of a (a) 1 and 2 only (b) 2 and 3 only
platform, with a DAO, Web 3.0 technologies and
communities provide a form of self-governance in an (c) 1 and 3 only (d) 1,2 and 3
attempted decentralized approach. Presently, only Ans. (d) : Short Range Devices (SRD) are radio
centralized repositories are the ones that own user data devices that offer a low risk of interference with other
and profit from it. In Web 3.0, users can own and be radio services, usually because their transmitting
properly compensated for their time and data. In Web power, and their low range. The definition 'Short Range
3.0 people will control their own data. Device' may be applied to many different types of
33. With reference to “Software as a Service (SaaS),” wireless equipment, including various forms of:
consider the following statements: - Access control (including door and gate openers)
1. SasS buyers can customise the user interface - Alarms and movement detectors
and can change data fields. - Closed-circuit television (CCTV)
2. SaaS users can access their data through their - Cordless audio devices, including wireless
mobile devices. microphones
3. Outlook, Hotmail and Yahoo! Mail are forms of - Wireless Local Area Networks
SaaS. - Remote control
Which of the statements given above are correct? - Radio frequency identification (RFID)
(a) 1 and 2 only (b) 2 and 3 only - Telemetry etc.
(c) 1 and 3 only (d) 1, 2 and 3 37. Consider the following statements:
Ans. (d): Software as a service (SaaS) is a way of 1. Biofilms can form on medical implants within
delivering applications over the Internet as a service. human tissues.
Instead of installing and maintaining software, user can 2. Biofilms can form on food and food processing
simply access it via the Internet, freeing themselves surfaces.
from complex software and hardware management. 3. Biofilms can exhibit antibiotic resistance.
Examples of popular SaaS providers include: Which of the statements given above are correct?
BigCommerce, Google Workspace, Salesforce,
Dropbox, Outlook, Hotmail and Yahoo! Mail etc. Also (a) 1 and 2 only (b) 2 and 3 only
SasS buyers can customize the user interface and can (c) 1 and 3 only (d) 1, 2 and 3
change data fields and can access their data through Ans. (d) : A biofilm is an assemblage of surface-
their mobile devices. Therefore all the given statements associated microbial cells that is enclosed in an
are correct. extracellular polymeric substance matrix. Biofilms can
34. Which one of the following statements best form on medical implants within human tissues. It can
reflects the idea behind the “Fractional Orbital form on food and food processing surfaces and can
Bombardment System” often talked about in exhibit antibiotic resistance. The biofilm provides a
media? favorable environment for the microorganisms.
(a) A hypersonic missile is launched into space 38. Consider the following statements in respect of
to counter the asteroid approaching the Earth probiotics:
and explode it in space. 1. Probiotics are made of both bacteria and yeast.
(b) A spacecraft lands on another planet after 2. The organisms in probiotics are found in foods
making several orbital motions. we ingest but they do not naturally occur in our gut.
(c) A missile is put into stable orbit around the 3. Probiotics help in the digestion of milk sugars.
Earth and deorbits over a target on the Earth. Which of the statements given above is/are correct?
(d) A spacecraft moves along a comet with the (a) 1 only (b) 2 only (c) 1 and 3 (d) 2 and 3
same speed and places a probe on its Ans. (c) : Probiotics are defined as live micro-
surface. organisms which confer a health benefit on the host.
Ans. (c): A Fractional Orbital Bombardment System Although most probiotics are bacteria, one strain of
(FOBS) is a warhead delivery system that uses a low yeast, Saccharomyces boulardii, has been found to be
earth orbit towards its target destination. Just before an effective probiotic in double-blind clinical studies.
reaching the target, it de-orbits through a retrograde Probiotics live naturally in our body. L. acidophilus is a
engine burn. It was developed in 1960s by Soviet probiotic bacteria that naturally occurs in the human
Union. gut. Yogurt is a well-known food source of probiotics.
35. Which one of the following is the context in which Certain strains of bacteria in yogurt have ß-D-
the term “qubit” is mentioned? galactosidase, which is an enzyme that helps to break
(a) Cloud Services down lactose in dairy products into the sugars glucose
(b) Quantum Computing and galactose. Thus Probiotics help in the digestion of
(c) Visible Light Communication Technologies milk sugars.
(d) Wireless Communication Technologies 39. In the context of vaccines manufactured to
Ans. (b): A “Qubit” is a quantum bit, the counterpart in prevent COVID-19 pandemic, consider the
quantum computing to the binary digit or bit of following statements:
classical computing. Just as a bit is the basic unit of 1. The Serum Institute of India produced COVID-
information in a classical computer, a qubit is the basic 19 vaccine named Covisheild using mRNA
unit of information in a quantum computer. platform.

IAS (Pre) GS 2022 Paper I 24A YCT


2. Sputnik V vaccine is manufactured using vector 3. EP100 brings together leading companies
based platform. committed to driving innovation in energy
3. COVAXIN is an inactivated pathogen based efficiency and increasing competitiveness while
vaccine. delivering on emission reduction goals.
Which of the statements given above are correct? 4. Some Indian companies are members of EP100.
(a) 1 and 2 only (b) 2 and 3 only 5. The International Energy Agency is the
(c) 1 and 3 only (d) 1, 2 and 3 Secretariat to the “Under2 Coalition.”
Ans. (b) : Serum Institute’s Covishield used the viral Which of the statements given above are correct?
vector platform. It used a chimpanzee adenovirus, (a) 1, 2, 4 and 5 (b) 1, 3 and 4 only
ChAdOx1, a type of common cold virus, which (c) 2, 3 and 5 only (d) 1, 2, 3, 4 and 5
attaches itself to cells and injects DNA that tells thest Ans. (b): EP100 is a global initiative led by The
cells to make corona virus spike protein. Hence 1 Climate Group and the Alliance to Save ndEnergy, not by
statement is incorrect. Rests of the statements are International Energy Agency. Hence 2 statement is
correct. incorrect. Also, 5th Statement is incorrect as Climate
40. If a major solar storm (solar-flare) reaches the Group is the Secretariat to the Under 2 Coalition and
Earth, which of the following are the possible not the International Energy Agency. Rests of the
effects on the Earth? statements are correct.
1. GPS and navigation systems could fail. 43. “If rainforests and tropical forests are the lungs
2. Tsunamis could occur at equatorial regions. of the Earth, then surely wetlands function as its
3. Power grids could be damaged. kidneys.” Which one of the following functions of
4. Intense auroras could occur over much of the wetlands best reflects the above statement?
Earth. (a) The water cycle in wetlands involves surface
5. Forest fires could take place over much of the runoff, subsoil percolation and evaporation.
planet. (b) Algae form the nutrient base upon which
6. Orbits of the satellites could be disturbed. fish, crustaceans, molluscs, birds, reptiles
7. Shortwave radio communication of the aircraft and mammals thrive.
flying over Polar Regions could be interrupted. (c) Wetlands play a vital role in maintaining
Select the correct answer using the code given sedimentation balance and soil
below: stabilization.
(a) 1, 2, 4 and 5 only (b) 2, 3, 5, 6 and 7 only (d) Aquatic plants absorb heavy metals and
excess nutrients
(c) 1, 3, 4, 6 and 7 only (d) 1, 2, 3, 4, 5, 6 and 7
Ans. (d) : According to The Ramsar Convention,
Ans. (c) : Statements 2 and 5 are incorrect. Solar flares “wetlands are areas of marsh, fen, peatland or water,
can temporarily alter the upper atmosphere creating whether natural or artificial, permanent or temporary,
disruptions. But it does not cause Tsunamis. It only with water that is static or flowing, fresh, brackish or
affects the upper atmosphere. Also there is no causal salt, including areas of marine water, the depth of
link between the coronal mass ejections (CMEs) and which at low tide does not exceed six metres.”
the deadly forest fires. There is a common
misconception that solar storms are associated with Wetlands are vital for humans, for other ecosystems
extreme geological and atmospheric effects on Earth. and for our climate, providing essential ecosystem
services such as water regulation, including flood
Rests of the statements are correct. control and water purification. Wetlands also absorb
41. “Climate Action Tracker” which monitors the carbon dioxide to help slow global heating and reduce
emission reduction pledges of different countries pollution, hence have often been referred to as the
is a: “Kidneys of the Earth”.
(a) Database created by coalition of research Therefore, option (d) is the correct answer.
organizations 44. In the context of WHO Air Quality Guidelines,
(b) Wing of “International Panel of Climate Change” consider the following statements:
(c) Committee under “United Nations Framework 1. The 24 hour mean of PM2.5 should not exceed
Convention on Climate Change” 15 µg/m3 and 3annual mean of PM2.5 should not
(d) Agency promoted and financed by United exceed 5 µg/m .
Nations Environment Programme and World 2. In a year, the highest levels of ozone pollution
Bank occur during the periods of inclement weather.
Ans. (a) : The Climate Action Tracker is an independent 3. PM10 can penetrate the lung barrier and enter the
scientific analysis that tracks government climate action bloodstream.
and measures it against the globally agreed Paris 4. Excessive ozone in the air can trigger asthma.
Agreement aim of "holding global warming well below Which of the statements given above are correct?
2°C, and pursuing efforts to limit warming to 1.5°C."
A collaboration of two organisations, Climate Analytics (a) 1, 3 and 4 (b) 1 and 4 only
and New Climate Institute, the Climate Action Tracker (c) 2, 3 and 4 (d) 1 and 2 only
has been providing this independent analysis to Ans. (b) : Ozone at ground level is formed by the
policymakers since 2009. reaction with sunlight (photochemical reaction) of
42. Consider the following statements: pollutants such as nitrogen oxides (NOx) from vehicle
1. “The Climate Group” is an international non- and industry emissions and volatile organic compounds
profit organization that drives climate action by (VOCs) emitted by vehicles, solvents and industry. As
building large networks and runs them. a result, the highest levels of ozone pollution occur
during periods of sunny weather and not during
2. The International Energy Agency in partnership inclement weather. Hence 2nd statement is incorrect.
with the Climate Group launched a global Again, while particles with a diameter of 10 microns or
initiative “EP100”.
IAS (Pre) GS 2022 Paper I 25A YCT
less, (≤ PM10) can penetrate and lodge deep inside the Select the correct answer using the code given below:
lungs, the even more health-damaging particles are (a) 1, 3 and 4 only (b) 1, 3, 5 and 6 only
those with a diameter of 2.5 microns or less, (≤ PM2.5). (c) 2, 4, 5 and 6 only (d) 1, 2, 4, 5 and 6
PM2.5 can penetrate the lung barrier and enter the Ans. (a): Out of the given options only Alfalfa,
blood system. Hence 3rd statement is also incorrect. Chickpea and Clovers are nitrogen- fixing plants.
Rests of the statements are correct. Nitrogen-Fixing Plants are capable of fixing Nitrogen
45. With reference to “Gucchi” sometimes mentioned in a natural way to provide plant-adjusted Nitrogen
in the news, consider the following statements: without any industrial harm to nature.
1. It is a fungus. 49. “Biorock technology” is talked about in which
2. It grows in some Himalayan forest areas. one of the following situations?
3. It is commercially cultivated in the Himalayan (a) Restoration of damaged coral reefs
foothills of north-eastern India. (b) Development of building materials using
Which of the statements given above is/are correct? plant residues
(a) 1 only (b) 3 only (c) 1 and 2 (d) 2 and 3 (c) Identification of areas for exploration/
Ans. (c) : Guchhi mushroom is a species of fungus in extraction of shale gas
the family Morchellaceae of the Ascomycota. They are (d) Providing salt licks for wild animals in
pale yellow in colour with large pits and ridges on the forest/protected areas
surface of the cap, raised on a large white stem. guchhi Ans. (a) : The Zoological Survey of India (ZSI), with
faces lots of challenges in its cultivation. The help from Gujarat’s forest department, is attempting a
mushrooms cannot be cultivated commercially and process for the first time to restore coral reefs using bio
grow in conifer forests across temperature regions, and rock or mineral accretion technology. A bio rock
the foothills in Himachal Pradesh, Uttarakhand, and structure was installed one nautical mile off the
Jammu and Kashmir. And it takes months for villagers Mithapur coast in the Gulf of Kachchh.
to collect enough of these mushrooms, dry them and 50. The “Miyawaki method’ is well known for the:
bring them to the market. (a) Promotion of commercial farming in arid
46. With reference to polyethylene terephthalate, the and semi-arid areas
use of which is so widespread in our daily lives, (b) Development of gardens using genetically
consider the following statements: modified flora
1. Its fibres can be blended with wool and cotton (c) Creation of mini forests in urban areas
fibres to reinforce their properties. (d) Harvesting wind energy on coastal area and
2. Containers made of it can be used to store any on sea surfaces.
alcoholic beverage. Ans. (c) : Miyawaki is a method of afforestation that
3. Bottles made of it can be recycled into other requires quite a small space, at least 20 square feet. One
products. must seed plants very close to save space and dense
4. Articles made of it can be easily disposed of by plant growth. It was developed by Akira Miyawaki of
incineration without causing greenhouse gas Japan. Bombay Municipal Corporation (BMC) has
emissions. been using the Miyawaki method to create tiny urban
Which of the statement given above are correct? forests in the Metropolitan areas of Mumbai.
(a) 1 and 3 (b) 2 and 4 (c) 1 and 4 (d) 2 and 3 51. In the Government of India Act 1919, the
Ans. (a) : Plastic packaging is dangerous to human functions of Provincial Government were divided
health and the country liquor and country made foreign into "Reserved" and "Transferred" subjects.
liquor cannot be sold in such bottles. That’s why Which of the following were treated as
Maharashtra government has announced the ban on sale "Reserved" subjects ?
of alcohol in PET bottles from April 1. Therefore 2nd 1. Administration of Justice
statement is incorrect. Again, Polyethylene, like other 2. Local Self-Government
plastics, is not inert and is known to release additives 3. Land Revenue
and other degradation products into the environment 4. Police
throughout its lifetime. Hence 4th statement is also Select the correct answer using the code given below
incorrect.
(a) 1, 2 and 3 (b) 2, 3 and 4 (c) 1, 3 and 4 (d) 1, 2 and 4
PET fibres are often used in durable-press blends with
other fibres such as rayon, wool, and cotton, reinforcing Ans. (c): Subjects were divided into two lists reserved
the inherent properties of those fibres while and transferred in the Government of India act 1919.
contributing to the ability of the fabric to recover from Reserved subjects: Law and order, finance, land
wrinkling. Also Polyethylene terephthalate is the most revenue, irrigation, police etc. to be administered by the
widely recycled plastic. governor through his executive council.
47. Which of the following is not a bird? Transferred subjects: Education, health, local
government, industry, agriculture, excise etc. to be
(a) Golden Mahseer (b) Indian Nightjar administered by ministers nominated from among the
(c) Spoonbill (d) White Ibis elected members of legislative council.
Ans. (a): Golden Mahseer is a fish that lives in fast- 52. In medieval India, the term "Fanam" referred to:
moving waters, inhabiting hill streams with a rocky and (a) Clothing (b) Coins (c) Ornaments (d) Weapons
stony substrate. Rests are type of birds.
Ans. (b): The Fanam coins were the regular unit of
48. Which of the following are nitrogen-fixing currency in medieval Travancore and appear to have
plants? been extensively used for trading in the region of South
1. Alfalfa 2. Amaranth India. The words Fanam and Panam literally mean
3. Chickpea 4. Clover money and are still used as a synonym for wealth in
5. Purslane (Kulfa) 6. Spinach Kerala in the native language of Malayalam.
IAS (Pre) GS 2022 Paper I 26A YCT
53. Consider the following freedom fighters: mahakavya by Hemachandra which details the
1. Barindra Kumar Ghosh histories of the earliest Jain teachers.
2. Jogesh Chandra Chatterjee 3. The Avadanasataka (A Hundred Glorious Deeds) are
3. Rash Behari Bose Jataka Stories (Buddhist tales) are thematically
Which of the statement given above are correct? organized into ten books that portray the truth of the
(a) 1 and 2 (b) 2 only (c) 1 and 3 (d) 3 only doctrine of karma (action) and the power of religious
dana (giving), faith, and devotion.
Ans. (d): The Ghadar Party was a political
revolutionary organization founded in the United States 4. Trishashthilkshana Mahapurana is a major Jain text
of America by migrated Indians. The formation of the composed largely by Acharya Jinasena during the
Ghadar Party was primarily the work of Sikhs. Sohan rule of Rashtrakutas.
Singh, Kartar Singh, Abdul Mohamed Barakatullah, 56. With reference to Indian history, consider the
and Rashbehari Bose were among the prominent following pairs:
leaders who laid the groundwork for the establishment Historical person Known as
of an Indian political organization in the United States 1. Aryadeva - Jaina scholar
and Canada. 2. Dignaga - Buddhist scholar
54. With reference to the proposals of Cripps 3. Nathamuni - Vaishnava scholar
Mission, consider the following statements: How many pairs given above are correctly
1. The Constituent Assembly would have matched?
members nominated by the Provincial (a) None of the pairs (b) Only one pair
Assemblies as well as the Princely States. (c) Only two pairs (d) All three pairs
2. Any Province, which is not prepared to
accept the new Constitution would have the Ans. (c) : Correct match is-
right to sign a separate agreement with 1. Aryadeva - Buddhist scholar
Britain regarding its future status. 2. Dignaga - Buddhist scholar
Which of the statements given above is/are correct? 3. Nathamuni - Vaishnava scholar
(a) 1 only (b) 2 only 57. With reference to Indian history, consider the
(c) Both 1 and 2 (d) Neither 1 nor 2 following statements:
Ans. (b): In March 1942, a mission headed by Stafford 1. The first Mongol invasion of India happened
Cripps was sent to India with constitutional proposals during the reign of Jalal-ud-din Khalji.
to seek Indian support for the war. 2. During the reign of Ala-ud-din Khalji, one
The main proposals of the Cripps mission were as Mongol assault marched up to Delhi and
follows: An Indian Union with a dominion status would besieged the city.
be set up; it would be free to decide its relations with 3. Muhammad-bin-Tughlaq temporarily lost portions
the Commonwealth and free to participate in the United of north-west of his kingdom to Mongols.
Nations and other international bodies. Which of the statements given above is/are correct?
After the end of the war, a constituent assembly would (a) 1 and 2 (b) 2 only (c) 1 and 3 (d) 3 only
be convened to frame a new constitution. Members of
this assembly would be partly elected by the provincial Ans. (b): Statement:1 The first Mongol invasion of
assemblies through proportional representation and India happened during the reign of Behram Shah in
partly nominated by the princes. (Hence, Statement 1 is 1241 AD. Hence statement 1 is not correct.
incorrect). Statement: 2 During the reign of Alauddin Khilji,
The British government would accept the new Mongols invaded India and put a siege around Delhi in
constitution subject to two conditions: (i) any province 1303. So, it is correct.
not willing to join the Union could have a separate Statement: 3 After 1307 AD, the Mongol threat
constitution and form a separate Union, and (ii) the new suddenly receded; and only on one occasion
constitution- making body and the British government subsequently does it appear to have attained to the same
would negotiate a treaty to effect the transfer of power proportions. Early in the reign of Muhammad Tughluq,
and to safeguard racial and religious minorities. (Hence, the Chaghatai Khan Tarmashirin led an incursion in
Statement 2 is correct). which the Mongol forces once again advanced to the
In addition to that, defence of India would remain in Jumna but retired without actually entering the capital
British hands and the governor-general’s powers would when he was paid a ransom. Thereafter, India was free
remain intact. of major inroads from this quarter for another sixty
55. With reference to Indian history, consider the years or more, until the invasion of Delhi by Timur in
following texts: 1398. So, Statement 3 is not correct.
1. Nettipakarana 58. With reference to Indian history, who of the
2. Parishishtaparvan following were known as "Kulah-Daran"?
3. Avadanashataka (a) Arab merchants (b) Qalandars
4. Trishashtilakshana Mahapurana (c) Persian calligraphists (d) Sayyids
Which of the above are Jaina texts? Ans. (d): The Sayyids put on a pointed cap (kulah) and
(a) 1,2 and 3 (b) 2 and 4 only they were known as ‘Kulah Daran’ during Delhi
(c) 1,3 and 4 (d) 2, 3 and 4 sultanate.
Ans. (b) : 1. Nettipakarana is a guide to help those who The Sayyids were claimed to descent from the Prophet
already understand the teaching and present it to others. through his daughter Fatima. They Commanded special
The Netti methods were taught by the Buddha's disciple respect in Muslim society.
Kaccana (also Katyayana or Kaccayana). Even the Timur protected the life of Sayyids during his
2. The Parishishtaparvan also known as the invasion in India, although his policy was of general
Sthaviravalicharitra is a 12th-century Sanskrit slaughter.
IAS (Pre) GS 2022 Paper I 27A YCT
59. With reference to Indian history, consider the Ans. (a): Tight monetary policy is an action undertaken
following statements: by a central bank such as the Federal Reserve to slow
1. The Dutch established their factories/ down overheated economic growth. Central banks
warehouses on the east coast on lands granted to engage in tight monetary policy when an economy is
them by Gajapati rulers. accelerating too quickly or inflation is rising too fast.
2. Alfonso de Albuquerque captured Goa from Capital flight destabilizes interest rates and exchange
the Bijapur Sultanate. rates and also reduces monetary control. It drives up the
3. The English East India Company established marginal costs of foreign borrowing. Thus, capital
a factory at Madras on a plot of land leased flight may increase the interest cost of firms with
from a representative of the Vijayanagara existing external commercial borrowings. So, statement
empire. 2 is correct.
Which of the statements given above are correct? Statement 3 is incorrect: Devaluation of domestic
(a) 1 and 2 only (b) 2 and 3 only currency does not affect the External Commercial
(c) 1 and 3 only (d) 1, 2 and 3 Borrowings as it is denominated in the foreign currency
Ans. (b): Gajapati rulers in Odisha lasted from and not in the domestic currency.
1434AD to 1541AD. Dutch factories established on the 62. Consider the following States:
eastern coast are: Masulipattanam (1605), Pulicat 1. Andhra Pradesh 2. Kerala
(1610), Bimilipatam / Bhimunipatnam (1641), Chinsura 3. Himachal Pradesh 4. Tripura
(1653). So, statement 1 is not correct. How many of the above are generally known as
Afonso de Albuquerque captured Goa for Portuguese in tea-producing States?
1510 from Adil Shahi rulers of Bijapur. So, statement 2 (a) Only one State (b) Only two States
is correct. (c) Only three States (d) All four States
In 1639, the English East India Company received Ans. (c): Major tea producing states are Assam, hills of
permission from local ruler Darmala Venkatadri Darjeeling and Jalpaiguri districts, West Bengal, Tamil
Nayaka to establish a factory at Madras. Fort Saint Nadu and Kerala. Apart from these, Himachal Pradesh,
George was built in 1640. Madras was the first piece of Uttarakhand, Meghalaya, Andhra Pradesh and Tripura
land leased by the English East India Company, and are also tea-producing states, but in a small quantity.
significantly without any war but through friendship
and negotiation. Raja Darmala Venkatadri was a 63. Consider the following statements
representative of the Vijayanagara Empire. So, 1. In India, credit rating agencies are regulated
statement 3 is correct. by Reserve Bank of India.
60. According to Kautilya's Arthashastra, which of 2. The rating agency popularly known as ICRA
the following are correct? is a public limited company.
1. A person could be a slave as a result of a 3. Brickwork Ratings is an Indian credit rating
judicial punishment. agency.
2. If a female slave bore her master a son, she Which of the statements given above are correct?
was legally free. (a) 1 and 2 only (b) 2 and 3 only
3. If a son born to a female slave was fathered (c) 1 and 3 only (d) 1, 2 and 3.
by her master, the son was entitled to the Ans. (b): Statement 1 is incorrect: Credit Rating
legal status of the master's son. Agencies form an essential part of the financial
Which of the statements given above are correct? markets. They are regulated by SEBI and not by RBI
(a) 1 and 2 only (b) 2 and 3 only under the powers derived from the Securities and
(c) 1 and 3 only (d) 1, 2 and 3 Exchange Board of India (Credit Rating Agencies)
Regulations, 1999.
Ans. (d): The Arthashastra is an Indian treatise on ICRA Limited (formerly Investment Information and
politics, economics, military strategy, the function of Credit Rating Agency of India Limited) was set up in
the state, and social organization attributed to Kautilya 1991 by leading financial/investment institutions,
(also known as Chanakya). He was instrumental in commercial banks and financial services companies as
establishing the reign of the great king Chandragupta an independent and professional investment
Maurya (321-297 BCE). Information and Credit Rating Agency. ICRA is a
There were variety of protection offered to the slaves, Public Limited Company, with its shares listed on the
especially to the women slaves. As per the Bombay Stock Exchange and the National Stock
Arthashastra, when a child is begotten on a female Exchange. So, statement 2 is correct.
slave by her master, both the child and its mother shall Statement 3 is correct: Brickwork Ratings (BWR) is
at once be recognised as free. a SEBI registered Indian Credit Rating Agency. It has
If a son born to a female slave was fathered by her also been accredited by RBI. It offers rating services on
master, the son was entitled to the legal status of the Bank Loans, NCD, Commercial Paper, Fixed deposits,
master's son. So, statement 2 and statement 3 are correct. Securitized paper, Security receipts etc.
61. Consider the following statements: 64. With reference to the Banks Board Bureau (BBB),
1. Tight monetary policy of US Federal which of the following statements are correct?
Reserve could lead to capital flight. 1. The Governor of RBI is the Chairman of BBB.
2. Capital flight may increase the interest cost 2. BBB recommends for the selection of heads
of firms with existing External Commercial for Public Sector Banks.
Borrowings (ECBs). 3. BBB helps the Public Sector Banks in
3. Devaluation of domestic currency decreases developing strategies and capital raising plans.
the currency risk associated with ECBs. Select the correct answer using the code given
Which of the statements given above are correct ? below :
(a) 1 and 2 only (b) 2 and 3 only (a) 1 and 2 only (b) 2 and 3 only
(c) 1 and 3 only (d) 1, 2 and 3 (c) 1 and 3 only (d) 1, 2 and 3
IAS (Pre) GS 2022 Paper I 28A YCT
Ans. (b): Statement 1 is incorrect: 67. Consider the following statements:
Banks Board Bureau comprises the Chairman, three ex- 1. Vietnam has been one of the fastest growing
officio members i.e Secretary, Department of Public economies in the world, in the recent years.
Enterprises, Secretary of the Department of Financial 2. Vietnam is led by a multi-party political
Services and Deputy Governor of the Reserve Bank of system
India, and five expert members, two of which are from 3. Vietnam's economic growth is linked to its
the private sector. The Chairman is selected by the integration with global supply chains and
central government and RBI governor does not head it. focus on exports.
The BBB advises the Government on matters relating 4. For a long time Vietnam's low labour costs
to appointments, confirmation or extension of tenure and stable exchange rates have attracted
and termination of services of the Board of Directors in global manufacturers.
Public Sector Banks (PSBs) and financial institutions 5. Vietnam has the most productive e-service
(FIs). It is responsible for the selection and appointment sector in the Indo-Pacific region.
of Board of Directors of PSBs and FIs. So, statement 2 Which of the statements given above are correct?
is correct. (a) 2 and 4 (b) 3 and 5 (c) 1, 3 and 4 (d) 1 and 2
One of the functions of the BBB is to help Public
Sector Banks in terms of developing business strategies Ans. (c) : According to a forecast by Pricewaterhouse
and capital raising plan etc. So, statement 3 is correct. Coopers in February 2017, Vietnam may be the fastest-
growing economy of the world, with a potential annual
65. With reference to Convertible Bonds, consider GDP growth rate of about 5.1%, which would make its
the following statements: economy the 10th-largest in the world by 2050. Hence,
1. As there is an option to exchange the bond Statement 1 is correct.
for equity, Convertible Bonds pay a lower Vietnam is a Marxist–Leninist one-party state based on
rate of interest. democratic centralism. Hence, Statement 2 is incorrect.
2. The option to convert to equity affords the Vietnam’s economic growth is linked to its integration
bondholder a degree of indexation to rising with global supply chains and focus on exports. Hence,
consumer prices. Statement 3 is correct.
Which of the statements given above is/are Vietnam has low labour costs and stable exchange rates
correct? which attract global investments. Hence, Statement 4 is
(a) 1 only (b) 2 only correct.
(c) Both 1 and 2 (d) Neither 1 nor 2 68. In India, which one of the following is responsible
Ans. (c): Statement 1 is correct: Convertible bonds for maintaining price stability by controlling
tend to offer a lower coupon rate or rate of return in inflation?
exchange for the value of the option to convert the bond (a) Department of Consumer Affairs
into common stock. Investors will generally accept a (b) Expenditure Management Commission
lower coupon rate on a convertible bond, compared (c) Financial Stability and Development Council
with the coupon rate on another identical regular bond, (d) Reserve Bank of India
because of its conversion feature. This enables the
issuer to save on interest expenses, which can be Ans. (d): Reserve Bank of India is India’s central bank,
substantial in the case of a large bond issue. which has key function to keep check on the inflation
One of the advantages of convertible bonds is that the by use of monetary policy in forms of qualitative and
option to convert to equity affords the bondholder a quantitative measures.
degree of indexation to rising consumer prices. So, 69. With reference to Non-Fungible Tokens (NFTs),
statement 2 is correct. consider the following statements:
66. Consider the following: 1. They enable the digital representation of
1. Asian Infrastructure Investment Bank physical assets.
2. Missile Technology Control Regime 2. They are unique cryptographic tokens that
exist on a blockchain.
3. Shanghai Cooperation Organisation 3. They can be traded or exchanged at
India is a member of which of the above? equivalency and therefore can be used as a
(a) 1 and 2 only (b) 3 only medium of commercial transactions.
(c) 2 and 3 only (d) 1, 2 and 3 Which of the statements given above are correct?
Ans. (d): The Asian Infrastructure Investment Bank (a) 1 and 2 only (b) 2 and 3 only
(AIIB) is a multilateral development bank that aims to (c) 1 and 3 only (d) 1, 2 and 3
improve economic and social outcomes in Asia. India Ans. (a): Non-fungible tokens, often referred to as
joined Asian Infrastructure Investment Bank on 11th NFTs, are blockchain-based tokens that each represent
January, 2016, Option 1 is correct. a unique asset like a piece of art, digital content, or
Option 2 is correct. The Missile Technology Control media. An NFT can be thought of as an irrevocable
Regime (MTCR) is a multilateral export control regime digital certificate of ownership and authenticity for a
whose members have an informal political given asset, whether digital or physical. They cannot be
understanding to limit the proliferation of missiles and traded or exchanged at equivalency and therefore,
missile technology. India became a member of the cannot be used as a medium of commercial
MTCR in 2016. transactions.
The Shanghai Cooperation Organization (SCO) is an 70. Consider the following pairs:
intergovernmental organization founded in Shanghai on Reservoirs States
15 June 2001. The SCO currently comprises eight 1. Ghataprabha - Telangana
Member States (China, India, Kazakhstan, Kyrgyzstan,
Russia, Pakistan, Tajikistan and Uzbekistan). India 2. Gandhi Sagar - Madhya Pradesh
became permanent member of the SCO in 2017. So, 3. Indira Sagar - Andhra Pradesh
statement 3 is correct. 4. Maithon - Chhattisgarh

IAS (Pre) GS 2022 Paper I 29A YCT


How many pairs given above are not correctly (a) This would prevent the transfer of land of
matched? tribal people to non-tribal people.
(a) Only one pair (b) Only two pairs (b) This would create a local self-governing
(c) Only three pairs (d) All four pairs body in that area.
Ans. (c): (c) This would convert that area into a Union
Reservoirs States Territory.
Ghataprabha - Karnataka (d) The State having such areas would be
Gandhi Sagar – Madhya Pradesh declared a Special Category State.
Indira Sagar – Madhya Pradesh Ans. (a): The Fifth Schedule of the Constitution deals
Maithon - Jharkhand with the administration and control of Scheduled Areas
as well as of Scheduled Tribes residing in any State
71. In India, which one of the following compiles other than the States of Assam, Meghalaya, Tripura and
information on industrial disputes, closures, Mizoram.
retrenchments and lay-offs in factories employing Under the provisions of 5th scheduled of the
workers? constitution of India the Governor can make regulations
(a) Central Statistics Office which would prevent the transfer of land of tribal
(b) Department for Promotion of Industry and people to non-tribal people.
Internal Trade 74. Consider the following statements:
(c) Labour Bureau 1. The India Sanitation Coalition is a platform
(d) National Technical Manpower Information to promote sustainable sanitation and is
System funded by the Government of India and the
Ans. (c): Labour Bureau is an attached office under World Health Organization.
Ministry of Labour and Employment. It was set up on 2. The National Institute of Urban Affairs is an
1st October 1946. It is entrusted with the work of apex body of the Ministry of Housing and
compilation, collection, analysis and dissemination of Urban Affairs in Government of India and
statistics on different aspects of labour. provides innovative solutions to address the
72. In India, what is the role of the Coal Controller's challenges of Urban India.
Organization (CCO)? Which of the statements given above is/are correct?
1. CCO is the major source of Coal Statistics in (a) 1 only (b) 2 only
Government of India. (c) Both 1 and 2 (d) Neither 1 nor 2
2. It monitors progress of development of Ans. (b): The India Sanitation Coalition was launched
Captive Coal/Lignite blocks. on June 25, 2015, at FICCI, New Delhi. ISC is a multi-
3. It hears any objection to the Government's stakeholder platform that brings together the private
notification relating to acquisition of coal- sector, government, financial institutions, civil society
bearing areas. groups, media, donors/bi-lateral/multilateral, experts
4. It ensures that coal mining companies etc. to work in the sanitation space to drive sustainable
deliver the coal to end users in the sanitation through a partnership model. It is not funded
prescribed time. by WHO. Hence 1st statement is incorrect.
Select the correct answer using the code given The National Institute of Urban Affairs (NIUA) is
below: India’s leading national think tank on urban planning
(a) 1, 2 and 3 (b) 3 and 4 only and development. As a hub for the generation and
(c) 1 and 2 only (d) 1, 2 and 4 dissemination of cutting-edge research in the urban
Ans. (a): The Coal Controller’s Organisation (CCO) is sector, NIUA seeks to provide innovative solutions to
a subordinate office of the ministry of coal, having its address the challenges of a fast urbanizing India, and
headquarters at Kolkata. It collects and maintains coal pave the way for more inclusive and sustainable cities
production data of all private and public sector coal of the future. Hence 2nd statement is correct.
mines in the country. Under Collection of Statistics 75. Which one of the following has been constituted
Act, 2008, Coal Controller has been made the statistical under the Environment (Protection) Act, 1986?
authority with respect to coal and lignite statistics, (a) Central Water Commission
entrusted with the responsibility of carrying out Annual (b) Central Ground Water Board
Coal & Lignite survey and publishing of Provisional (c) Central Ground Water Authority
Coal Statistics and Coal Directory of India. It is also (d) National Water Development Agency
entrusted with the task of monitoring captive mines.
Work such as permission for opening and reopening of Ans. (c): Central Ground Water Authority (CGWA)
coal mine has been entrusted to CCO. has been constituted under Section 3 (3) of the
Environment (Protection) Act, 1986 to regulate and
Under Coal Bearing Area (Acquisition and control development and management of ground water
Development) Act, 1957- Coal Controller is the resources in the country.
competent authority to hear any objection to the Central
Government’s Notification relating to acquisition of 76. With reference to the “United Nations
coal bearing land and to furnish his reports to Central Credentials Committee", consider the following
Government. statements:
Ensuring that coal mining companies deliver the coal to 1. It is a committee set up by the UN Security
end users in the prescribed time is not the function of Council and works under its supervision.
Coal Controller’s Organization (CCO). 2. It traditionally meets in March, June and
73. If a particular area is brought under the Fifth September every year.
Schedule of the Constitution of India, which one 3. It assesses the credentials of all UN
of the following statements best reflects the members before submitting a report to the
consequence of it? General Assembly for approval.
IAS (Pre) GS 2022 Paper I 30A YCT
Which of the statements given above is/are correct? Which of the statements given above are correct ?
(a) 3 only (b) 1 and 3 (c) 2 and 3 (d) 1 and 2 (a) 1 and 3 (b) 2 and 4 (c) 3 and 4 (d) 1 and 4
Ans. (a) : The United Nations Credentials Committee Ans. (d): The Tea Board of India is a statutory body set
is a committee stof the United Nations General up under section 4 of the Tea Act, 1953 and constituted
Assembly. Hence 1 statement is incorrect. on 1st April 1954. Hence 1st statement is correct.
A Credentials Committee is appointed at the beginning It is functions under the Ministry of Commerce and its
of each regular session (intensively from September to Head Office is situated in Kolkata and not in
December) of the General Assembly. Hence 2nd Bengaluru. (Hence 2nd and 3rd statements are incorrect).
statement is also incorrect. The Tea Board of India has overseas offices in
The Committee reports to the Assembly on the Moscow, Dubai, Hamburg, London and New York.
credentials of representatives. The Committee is Hence 4th statement is correct.
mandated to examine the credentials of representatives 80. Which one of the following best describes the term
of Member States and to report to the General "greenwashing"?
Assembly thereon (Rule 28 of the Rulesrd of Procedure (a) Conveying a false impression that a
of the General Assembly). Therefore 3 statement is Company’s products are eco-friendly and
correct. environmentally sound
77. Which one of the following statements best (b) Non-inclusion of ecological/ environmental
describes the ‘Polar Code'? costs in the Annual Financial Statements of
(a) It is the international code of safety for ships a country
operating in polar waters. (c) Ignoring the disastrous ecological consequences
(b) It is the agreement of the countries around while undertaking infrastructure development
the North Pole regarding the demarcation of (d) Making mandatory provisions for environmental
their territories in the polar region. costs in a government project/ programme
(c) It is a set of norms to be followed by the Ans. (a): Greenwashing is the process of conveying a
countries whose scientists undertake research false impression or providing misleading information
studies in the North Pole and South Pole. about how a company's products are more
(d) It is a trade and security agreement of the environmentally sound. Greenwashing is considered an
member countries of the Arctic Council. unsubstantiated claim to deceive consumers into
Ans. (a) : Polar Code is the International code of safety believing that a company's products are environmentally
for ships operating in polar waters that covers the full friendly.
range of design, construction, equipment, operational, 81. Consider the following statements:
training, search and rescue and environmental 1. High clouds primarily reflect solar radiation
protection matters relevant to ships operating in the and cool the surface of the Earth.
inhospitable waters surrounding the two poles.
2. Low clouds have a high absorption of
78. With reference to the United Nations General infrared radiation emanating from the Earth's
Assembly, consider the following statements: surface and thus cause warming effect.
1. The UN General Assembly can grant Which of the statements given above is/are correct?
observer status to the non-member States. (a) 1 only (b) 2 only
2. Inter-governmental organisations can seek (c) Both 1 and 2 (d) Neither 1 nor 2
observer status in the UN General
Assembly. Ans. (d) : High clouds are often thin and do not reflect
3. Permanent Observers in the UN General very much. They let lots of the Sun's heat into it. They
Assembly can maintain missions at the UN radiate less energy into space than the lower warmer
headquarters. clouds. Therefore, high clouds work to "trap" more
energy than the low clouds. Statement (1) is incorrect.
Which of the statements given above are correct ?
(a) 1 and 2 only (b) 2 and 3 only Low clouds are often quite thick and reflect lots of
sunlight back to space. Low clouds are excellent
(c) 1 and 3 only (d) 1, 2 and 3 reflectors. But, they don't stop the longwave energy
Ans. (d): All the given statements are correct regarding from escaping to space. Therefore, low clouds help to
the United Nations General Assembly i.e. The United cool the Earth. Statement (2) is incorrect.
Nations General Assembly may grant non-member 82. Consider the following statements:
states, international organizations and other entities 1. Bidibidi is a large refugee settlement in
Permanent Observer Status. However, The UN Charter north-western Kenya.
and the General Assembly Rules of Procedure have no
provisions related to granting permanent observer status 2. Some people who fled from South Sudan
and is based on practice only. Permanent Observers civil war live in Bidibidi.
may participate in the sessions and workings of the 3. Some people who fled from civil war in
General Assembly and maintain missions at the UN Somalia live in Dadaab refugee complex in
Headquarters. Kenya.
79. With reference to the "Tea Board" in India, Which of the statements given above is/are correct?
consider the following statements: (a) 1 and 2 (b) 2 only (c) 2 and 3 (d) 3 only
1. The Tea Board is a statutory body. Ans. (c) : The Bidibidi Refugee Settlement is located in
2. It is a regulatory body attached to the Ministry the West Nile Area of Uganda, and is home to over
of Agriculture and Farmers Welfare. 270,000 South Sudanese refugees, the second largest
3. The Tea Board's Head Office is situated in refugee settlement in the world. Hence, statement 1 is
Bengaluru. not correct and statement 2 is correct.
4. The Board has overseas offices at Dubai and The Dadaab refugee complex has a population of over 2
Moscow. lakh registered refugees and asylum seekers. Dadaab

IAS (Pre) GS 2022 Paper I 31A YCT


refugee complex consists of three camps. The first The 1982 United Nations Convention on the Law of the
camp was established in 1991, when refugees fleeing Sea (UNCLOS) defines an Exclusive Economic Zone
the civil war in Somalia started to cross the border into (EEZ). It is an area that shall not extend beyond 200
Kenya. So, statement 3 is correct. nautical miles from the baselines. The EEZ is measured
Therefore, option (c) is the correct answer. from the breadth of the territorial sea. Hence, Statement
83. Consider the following countries: 3 is correct.
1. Armenia 2. Azerbaijan 3. Croatia Hence, option (d) is the correct answer.
4. Romania 5. Uzbekistan 86. Which one of the following statements best
Which of the above are members of the reflects the issue with Senkaku Islands,
Organization of Turkic States? sometimes mentioned in the news?
(a) 1, 2 and 4 (b) 1 and 3 (c) 2 and 5 (d) 3, 4 and 5 (a) It is generally believed that they are artificial
Ans. (c) : The Organization of Turkic States, formerly islands made by a country around South China
called the Turkic Council or the Cooperation Council of Sea.
Turkic Speaking States, is an international organization (b) China and Japan engage in maritime
comprising prominent independent Turkic countries: disputes over these islands in East China Sea.
Azerbaijan, Kazakhstan, Kyrgyzstan, Turkey and (c) A permanent American military base has
Uzbekistan. Therefore the correct answer is option (c). been set up there to help Taiwan to increase
84. Consider the following statements: its defence capabilities.
1. Gujarat has the largest solar park in India. (d) Though International Court of Justice
2. Kerala has a fully solar powered International declared them as no man's land, some
Airport. South-East Asian countries claim them.
3. Goa has the largest floating solar photovoltaic Ans. (b) : The Senkaku Islands are a group of
project in India. uninhabited islands in the East China Sea. They are
Which of the statements given above is/are known in China as the Diaoyu Islands. The islands are
correct? the focus of a territorial dispute between Japan and
(a) 1 and 2 (b) 2 only (c) 1 and 3 (d) 3 only China. China claims the discovery and ownership of the
Ans. (b) : India's Bhadla Solar Park in Rajasthan is the islands from the 14th century, while Japan maintained
largest solar power park in the world. Hence, Statement ownership of the islands from 1895 until its surrender
(1) is incorrect. at the end of World War II. Hence option (b) is the
Kerala's Cochin International Airport Ltd (CIAL) is the correct answer.
first airport in the world that would be running fully on 87. Consider the following pairs:
solar power. Hence, Statement (2) is correct. Country Important reason for
The 600 MW capacity floating solar project on Being in the news recently
Omkareshwar reservoir (in Madhya Pradesh) is not
only India's but also the world's largest floating solar 1. Chad - Setting up of permanent
project so far. Statement 3 is incorrect. military base by China
85. With reference to the United Nations Convention 2. Guinea - Suspension of Constitution
on the Law of Sea, consider the following and Government by military
statements: 3. Lebanon - Severe and prolonged
1. A coastal state has the right to establish the economic depression
breadth of its territorial sea up to a limit not 4. Tunisia - Suspension of Parliament by
exceeding 12 nautical miles, measured from President
baseline determined in accordance with the How many pairs given above are correctly matched?
convention. (a) Only one pair (b) Only two pairs
2. Ships of all states, whether coastal or (c) Only three pairs (d) All four pairs
land-locked, enjoy the right of innocent Ans. (c) : In 2020, President Idris Deby Itno was killed
passage through the territorial sea. during fighting against rebels in northern Chad, he was
3. The Exclusive Economic Zone shall not succeeded by his son, Mahamat Idriss Deby Itno, as the
extend beyond 200 nautical miles from the head of a military junta. It was not in the news for
baseline from which the breadth of the setting up of permanent military base by China. So, pair
territorial sea is measured.
(1) is not correctly matched.
Which of the statements given above are correct?
(a) 1 and 2 only (b) 2 and 3 only Some US officials are reported to have warned that
Beijing plans to establish a permanent military
(c) 1 and 3 only (d) 1,2 and 3 installation in Guinea. Recently, Guinea's military
Ans. (d) : According to Article 3 under section 2 of leaders have suspended the constitution and closed the
United nations conventions (UNCLOS) regarding the country's borders. So, pair (2) is correctly matched.
territorial limits of the sea, every state has the right to Lebanon is enduring a severe and prolonged economic
establish the breadth of its territorial sea up to a limit depression. According to the latest World Bank
not exceeding 12 nautical miles, measured from Lebanon Economic Monitor (LEM), the economic and
baselines determined in accordance with this financial crisis is likely to rank in the top 10, possibly
Convention. Hence, Statement 1 is correct. top 3, most severe crises episodes globally since the
According to Article 17 under section 3 of United mid-nineteenth century. So, pair (3) is correctly
nations conventions (UNCLOS), ships of all States, matched.
whether coastal or land-locked, enjoy the right of
innocent passage through the territorial sea. Hence, Tunisia’s President Kais Saied has dissolved the
Statement 2 is correct. country’s parliament. So, pair (4) is correctly matched.

IAS (Pre) GS 2022 Paper I 32A YCT


88. Consider the following pairs: Ans. (b) :
Region often mentioned in the news Country Site of Ashoka's Location in the
1. Anatolia - Turkey major rock edicts State of
2. Amhara - Ethiopia 1. Dhauli — Odisha
3. Cabo Delgado - Spain 2. Erragudi — Andhra Pradesh
4. Catalonia - Italy 3. Jaugada — Odisha
How many pairs given above are correctly matched? 4. Kalsi — Uttarakhand
(a) Only one pair (b) Only two pairs 92. Consider the following pairs:
(c) Only three pairs (d) All four pairs King Dynasty
Ans. (b) : Anatolia – Turkey, Amhara – Ethiopia 1. Nannuka — Chandela
Cabo delgado – Mozambique, Catalonia – Spain 2. Jayashakti — Paramara
3. Nagabhata II — Gurjara-Pratihara
89. With reference to Indian laws about wildlife
protection, consider the following statements: 4. Bhoja — Rashtrakuta
1. Wild animals are the sole property of the How many pairs given above are correctly matched?
government. (a) Only one pair (b) Only two pairs
2. When a wild animal is declared protected, (c) Only three pairs (d) All four pairs
such animal is entitled for equal protection Ans. (b) : Nannuka was the founder of the Chandela
whether it is found in protected areas or dynasty of India. He ruled in the Jejakabhukti region.
outside. Jayashakti was a 9th-century ruler from the Chandela
dynasty of Central India.
3. Apprehension of a protected wild animal
becoming a danger to human life is sufficient Nagabhata II was an Indian Emperor from Gurjara-
ground for its capture or killing. Pratihara dynasty. He ascended the throne of
GurjaraPratihara dynasty after his father Vatsraja.
Which of the statements given above is/are correct? Bhoja popularly known as Raja Bhoj Parmar (reigned
(a) 1 and 2 (b) 2 only (c) 1 and 3 (d) 3 only c. 1010–1055 CE) was an Indian King from the
Ans. (a) : In a significant verdict, the Bombay High Paramara dynasty.
Court has ruled that wild animals including tigers 93. Which one of the following statements about Sangam
should be treated as "government property for all literature in ancient South India is correct?
purposes" and any damage caused by them should be (a) Sangam poems are devoid of any reference
compensated by the Government. Hence statement 1 is to material culture.
correct. (b) The social classification of Varna was
When an animal is declared as a protected animal under known to Sangam poets.
Wildlife (Protection) Act (WPA), 1972 then it will (c) Sangam poems have no reference to warrior ethic.
enjoy the same protection either if it is present in a (d) Sangam literature refers to magical forces as
protected area or outside of a protected area. Hence, irrational.
statement 2 is correct. Ans. (b) : The Tamil society during the Sangam period
Only if the wild animal becomes a danger to human life was broadly divided into several groups. In the
or is diseased or disabled beyond recovery can it be beginning of the Sangam Age, The Tamil society was
allowed to be captured or killed by the competent not organized on the basis of the Vedic caste system,
authority, the Chief Wildlife Warden of the State. This namely Brahmanas, Kshatriyas, Sudras. However, the
provision is applicable to wild animals listed in earliest of the Sangam literature, Tolkappiyam refers to
Schedule I of the Wildlife (Protection) Act, 1972, the four divisions, which were prevalent in the Sangam
which includes leopards. Mere apprehension or fear society namely, Anthanar, Arasar, Vaislyar and
that a wild animal could endanger human life is not a Vellalar. It may be said that this classification roughly
ground for capture or killing. Hence statement 3 is corresponds to the Vedic Social division.
incorrect. 94. "Yogavasistha"was translated into Persian by
Nizamuddin Panipati during the reign of :
90. Certain species of which one of the following (a) Akbar (b) Humayun (c) Shahjahan (d) Aurangzeb
organisms are well known as cultivators of fungi?
(a) Ant (b) Cockroach (c) Crab (d) Spider Ans. (a) : YogaVasistha was translated multiple times
into Persian language throughout the Mughal Dynasty,
Ans. (a) : Species of Ant is well known as the as commanded by Akbar, Jahangir, and Darah Shikuh.
cultivator of the fungi. Few other insects such as Nizam al-Din Panipati completed one of these
termites, beetles, and marsh periwinkles are also able to translations, known as the Jug-Basisht, in the late
cultivate fungi. sixteenth century AD.
Hence, option (a) is correct. 95. The world's second tallest statue in sitting pose of
91. Consider the following pairs: Ramanuja was inaugurated by the Prime
Site of Ashoka's Location in the Minister of India at Hyderabad recently. Which
major rock edicts State of one of the following statements correctly represents
1. Dhauli — Odisha the teachings of Ramanuja?
(a) The best means of salvation was devotion.
2. Erragudi — Andhra Pradesh
(b) Vedas are eternal, self-existent and wholly
3. Jaugada — Madhya Pradesh authoritative.
4. Kalsi — Karnataka (c) Logical arguments were essential means for
How many pairs given above are correctly matched? the highest bliss.
(a) Only one pair (b) Only two pairs (d) Salvation was to be obtained through
(c) Only three pairs (d) All four pairs meditation
IAS (Pre) GS 2022 Paper I 33A YCT
Ans. (a) : Ramanuja was deeply influenced by the Ans. (d) : Nanoparticles are incredibly tiny particles of
Alvars. He firmly believed that intense devotion to a given substance. Nanoparticles are less than 100
Vishnu was the best means to attain salvation. He nanometers thick. To give some perspective, a
propounded the doctrine of Vishishtadvaita. nanometer is 1000 times smaller than the thickness of
The outdoor 216-ft Statue of Equality is the world’s one strand of hair.
second tallest statue featuring a sitting posture. It is Nanoparticles can be naturally created, for example,
composed of ‘panchaloha’, a combination of five minuscule droplets of sea spray. However, most
metals comprising gold, silver, copper, brass, and zinc. nanoparticles are created in the lab. For sunscreen, the
Born in 1017 in Sri Perumbudur, Tamil Nadu, Sri nanoparticles are zinc oxide and titanium dioxide.
Ramanujacharya liberated millions from social, These ingredients are broken down into ultrafine
cultural, gender, educational and economic particles before being added to sunscreen. So, statement
discrimination with the foundational conviction that 1 is not correct and statement 2 is correct.
every human is equal regardless of nationality, gender, Inhaled particulate matter can be deposited throughout
race, caste, or creed. the human respiratory tract, and an important fraction
96. The Prime Minister recently inaugurated the new of inhaled nanoparticles deposit in the lungs.
Circuit House near Somnath Temple at Veraval. Nanoparticles can potentially move from the lungs to
Which of the following statements are correct other organs such as the brain, the liver, the spleen and
regarding Somnath Temple? possibly the foetus in pregnant women. So, statement 3
1. Somnath Temple is one of the Jyotirlinga is correct.
shrines. Therefore, option (d) is the correct answer.
2. A description of Somnath Temple was given
by Al-Biruni 99. Consider the following statements:
3. Pran Pratishtha of Somnath Temple DNA Barcoding can be a tool to:
(Installation of the present day temple) was 1. assess the age of a plant or animal.
done by President S. Radhakrishnan. 2. distinguish among species that look alike.
Select the correct answer using the code given below: 3. identify undesirable animal or plant
(a) 1 and 2 only (b) 2 and 3 only materials in processed foods.
(c) 1 and 3 only (d) 1, 2 and 3 Which of the statements given above is/are correct?
Ans. (a) : Statement 3 is incorrect: Pran Pratishtha of (a) 1 only (b) 3 only (c) 1 and 2 (d) 2 and 3
Somnath Temple (installation of the present day Ans. (d) : DNA barcoding deals primarily with species
temple) was done by Dr. Rajendra Prasad and not by S. identification and has limited utility for lower
Radhakrishnan. Statement 1 and 2 are correct. Hence taxonomic levels (e.g. subspecies, animal breeds, plant
the correct answer is option (a). varieties).
97. Which one of the following statements best DNA barcoding is a novel system designed to provide
describes the role of B cells and T cells in the rapid, accurate, and automatable species identifications
human body? by using short, standardized gene regions as internal
(a) They protect the body from environmental species tags. Hence, statement 2 is correct.
allergens. There is an increasing demand for the improvement of
(b) They alleviate the body's pain and quality controls, hence addressing scientific research
inflammation towards the development of reliable molecular tools for
(c) They act as immunosuppressants in the body. food analysis. DNA barcoding is a widely used
(d) They protect the body from the diseases molecularbased system, which can identify biological
caused by pathogens. specimens, and is used for the identification of both raw
Ans. (d) : The overall ability of the body to fight the materials and processed food. Hence, statement 3 is
disease causing organisms (pathogens), conferred by correct.
the immune system is called immunity. The primary 100. Consider the following:
and secondary immune responses are carried out with 1. Carbon monoxide 2. Nitrogen oxide
the help of two special types of lymphocytes present in 3. Ozone 4. Sulphur dioxide
our blood, i.e., B-lymphocytes (B-cells) and T- Excess of which of the above in the environment
lymphocytes (T-cells). Thus, both B-cells and T-cells is/are cause(s) of acid rain?
are responsible for acquired immunity. B-cells are (a) 1, 2 and 3 (b) 2 and 4 only (c) 4 only (d) 1, 3 and 4
responsible for antibody-mediated immunity response.
T-cells are responsible for cell-mediated immunity Ans. (b) : Acid rain refers to the ways in which acid
response. Bcells produce antibodies to fight with the from the atmosphere is deposited on the earth’s surface.
pathogens into our blood. The T-cells themselves do Oxides of nitrogen and sulphur which are acidic in
not secrete antibodies but help B cells produce them. nature can be blown by wind along with solid particles
Hence option (d) is the correct answer. in the atmosphere and finally settle down either on the
98. Consider the following statements: ground as dry deposition or in water, fog and snow as
1. Other than those made by humans, wet deposition.
nanoparticles do not exist in nature. Burning of fossil fuels (which contain sulphur and
2. Nanoparticles of some metallic oxides are nitrogenous matter) such as coal and oil in power
used in the manufacture of some cosmetics. stations and furnaces or petrol and diesel in motor
3. Nanoparticles of some commercial products engines produce sulphur dioxide and nitrogen oxides.
which enter the environment are unsafe for SO2 and NO2 after oxidation and reaction with water
humans. are major contributors to acid rain, because polluted air
usually contains particulate matter that catalyze the
Which of the statements given above is/are correct? oxidation. Therefore, option (b) is the correct answer.
(a) 1 only (b) 3 only (c) 1 and 2 (d) 2 and 3
IAS (Pre) GS 2022 Paper I 34A YCT
UNION PUBLIC SERVICE COMMISSION
Civil Services (Preliminary Exam) - 2021
GENERAL STUDIES : PAPER-I
Time : 2 hours (Exam date : 04.11.2021) Maximum Number : 200
Select the correct answer using the code given
ANCIENT HISTORY below.
(a) 1, 2 and 5 (b) 1, 3, 4 and 6
Pre-Historic Period (c) 2, 3 and 4 (d) 5 and 6
1. Which one of the following ancient towns is Ans. (b): The Guptas of Magadha, The Pushyabhutis of
well-known for its elaborate system of water Thanesar, the Maukharis of Kannauj, and the Maitrakas
harvesting and management by building a of Valabhi, were holding power from the decline of
series of dams and channelizing water into Guptas unitl the rise of Harshavardhana in the early
connected reservoirs? seventh century. Hence, option (b) is the correct answer.
(a) Dholavira (b) Kalibangan
(c) Rakhigarhi (d) Ropar Buddhism & Jainism Religion
Ans. (a): Dholavira is the ancient town, is well known
for its elaborate system of water harvesting and 4. With reference to the history of ancient India,
management by building a series of dams and Bhavabhuti, Hastimalla and Kshemeshvara
channelizing water into connected reservoirs. were famous
Dholavira is located on Khadirbet Island in the Rann of (a) Jain monks (b) Playwrights
Kachchh in Gujarat. (c) Temple (d) Philosophers
Hence, option (a) is the correct answer. Ans. (b): Bhavabhuti was a famous Sanskrit playwright
2. Consider the following pairs: who wrote famous plays like Mahaviracharita,
(Historical place) (Well-known for) Malatimadhava etc. He is believed to be the court poet
1. Burzahom : Rock-cut shrines of King Yashovarman of Kannauj. Other two were also
playwrights in the ancient period.
2. Chandra-ketugarh : Terracotta art
Hence, option (b) is the correct answer.
3. Ganeshwar : Copper artefacts
Which of the pairs given above is/are correctly
matched? MEDIEVAL HISTORY
(a) 1 only (b) 1 and 2
5.
According to Portuguese writer Nuniz, the
(c) 3 only (d) 2 and 3
women in Vijayanagara Empire were expert in
Ans. (d): Chandraketugarh: Major findings of this which of the following areas?
site include Buddhist stupas and images, decorative 1. Wrestling 2. Astrology
designs belonging to the Pala period, coins of the Gupta 3. Accounting 4. Soothsaying
and Kushana period, different kinds of beads, terracotta
seals, bricks and plaques depicting scenes from Jataka Select the correct answer using the code given
tales and figurines of Sunga, Maurya, Kushana and below.
Gupta periods. Hence, pair 2 is correctly matched. (a) 1, 2 and 3 only (b) 1, 3 and 4 only
Ganeshwar: Copper artefacts, objects, microliths and (c) 2 and 4 only (d) 1, 2, 3 and 4
Ans. (d): Fernao Nuniz, Portuguese traveler visited the
pottery were found throughout the deposits. Copper
empire during the reign of Achyuta Deva Raya.
objects included arrowheads, spearheads, fish hooks,
According to Nuniz a large number of women were
bangles and chisels. Hence, pair 3 is correctly matched.
Therefore, option (d) is the correct answer. employed in royal places as dancers, domestic servants
and palanquin bearers. There were also wrestlers,
Gupta & Post Gupta Period astrologers and soothsayers among them. Some women
3. From the decline of Guptas unitl the rise of were also appointed as accountants, Judges, bailiffs, and
Harshavardhana in the early seventh century, watch women.
which of the following kingdoms were holding Hence, option (d) is the correct answer.
power in Northern India? 6. Consider the following statements:
1. The Guptas of Magadha 1. It was during the reign of Iltutmish that
2. The Paramaras of Malwa Chengiz Khan reached the Indus in pursuit of
3. The Pushyabhutis of Thanesar the fugitive Khwarezm prince.
4. The Maukharis of Kannauj 2. It was during the reign of Muhammad bin
5. The Yadavas of Devagiri Tughluq that Taimur occupied Multan and
6. The Maitrakas of Valabhi crossed the Indus.
IAS (Pre) GS 2021 Paper I 3 YCT
3. It was during the reign of Deva Raya II of
Vijayanagara Empire that Vasco da Gama MODERN HISTORY
reached the coast of Kerala. 9. In the first quarter of seventeenth century, in
Which of the statements given above is/are which of the following was/were the
correct? factory/factories of the English East India
(a) 1 only (b) 1 and 2 Company located?
(c) 3 only (d) 2 and 3 1. Broach
Ans. (a): Statement – 1 (Correct) – Khwarizm ruler 2. Chicacole
Jalaluddin who was being pursued by Chengiz Khan 3. Trichinopoly
fled to Delhi. Sultanate Iltutmish, who was ruling Delhi Select the correct answer using the code given
then, refused the request of Jalaluddin for asylum. below.
Statement – 2 (Not correct) – Muhammad Tughluq (a) 1 only (b) 1 and 2
died in 1351 and Timur had started his career of (c) 3 only (d) 2 and 3
conquest in 1370. He crossed Indus during the reign of Ans. (a) : By 1st quarter of 17th century , English East
Firoz Shah Tughluq. India Company had established factories at Surat,
Statement – 3 (not correct) – Deva Raya – II died in Broach, Ahmedabad, Agra, and Masulipatam and
1446 and Vasco Da Gama arrived in 1498 at Calicut. not at Chicacole and Tichonopoly.
Hence, Option (a) is the correct answer. Hence, Option (a) is the correct answer.
7. With reference to medieval India, which one of 10. With reference to Madanapalle of Andhra
the following is the correct sequence in Pradesh, which one of the following statements
ascending order in terms of size? is correct?
(a) Paragana–––Sarkar–––Suba (a) Pingali Venkayya designed the tricolour
(b) Sarkar––Paragana––Suba Indian National Flag here.
(c) Suba––Sarkar––Paragana (b) Pattabhi Sitaramaiah led the Quit India
Movement of Andhra region from here.
(d) Paragana––Suba––Sarkar
(c) Rabindranath Tagore translated the National
Ans. (a): The entire Suba was divided into Sarkars Anthem from Bengali to English here.
(districts), each being looked after by a military officer (d) Madame Blavatsky and Colonel Olcott set up
called the chief Shiqdar. Each Sarkar, was divided into headquarters of Theosophical Society first
smaller units called Parganas. There was a Shiqdar here.
(military officer), Amin or Munsif (Civilian Judge), one Ans. (c): The original song 'Jana Gana Mana' (National
Fotahdar (treasurer) and two Karkuns (clerks or writers) Anthem) was written in Bengali, but in a Sanskritized
in each Pargana. dialect known as Sadhu Bhasha. The idea of translating
Hence option (a) is the correct answer. the song from Bengali to English came to Rabindranath
8. With reference to Indian history, which of the Tagore while he was visiting the Besant Theosophical
following statements is/are correct? College on the invitation of Irish poet James H.
1. The Nizamat of Arcot emerged out of Cousins. He penned down the English translation during
Hyderabad State. his stay at Madanapalle, a small town in the Chittoor
2. The Mysore Kingdom emerged out of district of Andhra Pradesh.
Vijayanagara Empire. Hence, option (c) is the correct answer.
3. Rohilkhand Kingdom was formed out of the 11. Consider the following statements:
territories occupied by Ahmad Shah Durrani. 1. The Montagu-Chelmsford Reforms of 1919
Select the correct answer using the code given recommended granting voting rights to all the
below. women above the age of 21.
(a) 1 and 2 (b) 2 only 2. The Government of India Act of 1935 gave
(c) 2 and 3 (d) 3 only women reserved seats in legislature.
Which of the statements given above is/are
Ans. (a): The Nizamat of Arcot was a dependency of correct?
Hyderabad and was under the legal purview of the (a) 1 only (b) 2 only
Nizam of Hyderabad, until its demise. Later it emerged
(c) Both 1 and 2 (d) Neither 1 nor 2
out of Hyderabad. Hence statement 1 is correct.
Mysore kingdom emerged as a powerful state under the Ans. (b): In line with the government policy contained
in Montagu’s statement of August 1917, the
Wodeyar dynasty, which initially served as a vassal of
government announced further constitutional reforms in
Vijaynagar empire. Statement 2 is also correct.
July 1918, known as Montagu-Chelmsford reforms;
Rohilkhand state arose in the modern districts of Based on these, the Government of India Act, 1919 was
Rampur, Bareilly etc. under the declining Mughal enacted. Under the Act, Women were given the right to
empire in 1721. Ahmad Shah Durrani started invading vote. But there was no universal adult suffrage. Hence
India from 1748. Hence option C is wrong. statement 1 is not correct.
IAS (Pre) GS 2021 Paper I 4 YCT
The Government of India Act, 1935 extended the 15. In the context of Colonial India, Shah Nawaz
principle of communal representation by providing Khan, Prem Kumar Sehgal and Gurbaksh
separate electorates for depressed classes (scheduled Singh Dhillon are remembered as
castes), women and labourers (workers). A separate (a) leaders of Swadeshi and Boycott Movement
electorate meant that not only were the seats reserved (b) members of the Interim Government in 1946
for a specific group, but voting for the reserved (c) members of the Drafting Committee in the
constituency was allowed for only members of that Constituent Assembly
specific community. Hence statement 2 is correct. (d) officers of the Indian National Army
Hence option (b) is the correct answer. Ans. (d): In the context of colonial India, Shah Nawaz
12. With reference to 8th August, 1942 in Indian Khan, Prem Kumar Sehgal and Gurbaksh Singh Dhillon
history, which one of the following statements were the officers of the Indian National Army.
in correct? Hence, Option (d) is the correct answer.
(a) The Quit India Resolution was adopted by the
AICC.
Art and Culture
(b) The Viceroy's Executive Council was 16. With reference to Chausath Yogini Temple
expanded to include more Indians. situated near Morena, consider the following
(c) The Congress ministries resigned in seven statements :
provinces. 1. It is a circular temple built during the reign of
(d) Cripps proposed an Indian Union with full Kachchhapaghata Dynasty.
Dominion Status once the Second World War 2. It is the only circular temple built in India.
was over. 3. It was meant to promote the Vaishnava cult in
Ans. (a): On 8th August, 1942 the quit India Movement the region.
resolution was adopted by the All-India Congress 4. Its design has given rise to a popular belief that
Committee. it was the inspiration behind the Indian
Hence, option (a) is the correct answer. Parliament building.
13. Who among the following is associated with Which of the statements given above are correct?
'Songs from Prison', a translation of ancient (a) 1 and 2 (b) 2 and 3 only
Indian religious lyrics in English? (c) 1 and 4 (d) 2, 3 and 4
(a) Bal Gangadhar Tilak Ans. (c): The Chausath Yogini temple is in Morena
(b) Jawaharlal Nehru district in M.P. the temple was built by the
(c) Mohandas Karamchand Gandhi Kachchhapaghata king Devapala (1055-1075).
1. It is not the only circular temple built in India.
(d) Sarojini Naidu
2. It was to promote the Shiva temple.
Ans. (c): ‘Songs from Prison' is a book of translation 3. The uniqueness of this yogini temple is its circular
from Sanskrit hymns and lyrics, from the Upanishads shape that is popularly believed to have inspired the
and other scriptures. Mahatma Gandhi made these design of the Indian parliament.
translations during his incarceration in Yerwada Prison Hence, option (c) is the correct answer.
in 1930. Translations of works by Manu, Tulsidas,
Kabir, Nanak, Mirabai, Ramdas, Tukaram and many 17. Which one of the following statements is
other poets. Gandhi prepared these translations mainly correct?
for his English friends and especially for Mirabehn. (a) Ajanta Caves lie in the gorge of Waghora
river.
Hence, option (c) is the correct Answer.
(b) Sanchi Stupa lies in the gorge of Chambal
14. Who among the following was associated as river.
Secretary with Hindu Female School which (c) Pandu-lena Cave Shrines lie in the gorge of
later came to be known as Bethune Female Narmada river.
School? (d) Amaravati Stupa lies in the gorge of Godavari
(a) Annie Besant river.
(b) Debendranath Tagore Ans. (a): The most prominent examples of the rock-cut
(c) Ishwar Chandra Vidyasagar architecture of this period are found at Ajanta and Bagh
(d) Sarojini Naidu (Huntington, 1985: 239–74). The spectacular Buddhist
Ans. (c): In 1856, the Government took charge of the site of Ajanta consists of several caves nestled in a
Hindu Female School, later renamed as Bethune curving section of the Sahyadri hills, overlooking the
School. The Managing Committee of the school was Waghora river. There are 28 caves at Ajanta. Hence
then formed and Pandit Ishwar Chandra Vidyasagar, the option (a) is the correct answer.
celebrated social reformer responsible for the Hence, option (a) is the correct answer.
eradication of the custom of Sati and a relentless 18. With reference to India, the terms 'Halbi, Ho
supporter of women's emancipation was made the and Kui' pertain to
Secretary. (a) dance forms of Northwest India
Hence, Option (c) is the correct answer. (b) musical instruments
IAS (Pre) GS 2021 Paper I 5 YCT
(c) pre-historic cave paintings Monsoon/ forest/soil/ Irrigation project/ and
(d) tribal languages Agricultural
Ans. (d): Some of the popular languages spoken by the 22. The black cotton soil of India has been formed
tribes of India are as follows: Bhatri, Halbi, Ho, Kui etc.
due to the weathering of
Halabi is spoken by over 5.25 Lakh people in the states
(a) brown forest soil
of Maharashtra and Madhya Pradesh. It belongs to the
group of Indo Aryan Tribal languages. Important (b) fissure volcanic rock
Dravidian Tribal languages are Kolami, Kui, Konda, (c) granite and schist
Koya, Gondi, Oraon/Kurukh, Parji. (d) shale and limestone
Hence, option (d) is the correct answer. Ans. (b) : The Black soil are so called because of their
black colouration and derived from the Basalt rock
INDIAN GEOGRAPHY under semi-arid conditions. It is also known as ‘Regur’
(from the Telugu word Reguda) or black cotton soil as it
Physical Structure & Human Geography/ is best suited for cotton cultivation. In India black soil
Drainage System are largely found over Deccan trap region of the states
19. With reference to the Indus river system, of the of Maharshtra, Madhya Pradesh, parts of Andhra
following four rivers, three of them pour into Pradesh, Northern part of Karnataka, Gujarat, parts of
one of them which joins the Indus direct. Tamil Nadu and Rajasthan.
Among the following, which one is such river Several theories have been put forward regarding the
that joins the Indus direct? origin of this group of soils but most pedologists believe
(a) Chenab (b) Jhelum that these soils have been formed due to the solidification
(c) Ravi (d) Sutlej of lava spread over large areas during volcanic activity in
the Deccan Plateau, in the Triassic Period.
Ans. (a): The Indus originates in the northern slopes of
the Kailash range in Tibet near Lake Manasarovar. It Hence, option (b) is the correct answer.
follows a northwesterly course through Tibet. It enters
Indian territory in Jammu and Kashmir. The main WORLD GEOGRAPHY
tributaries of the Indus in India are Jhelum, Chenab, Ravi,
Beas and Sutlej. Jhelum and Ravi join Chenab, Beas joins
Tropical Zone & World Forest Area.
Sutlej, and then Sutlej and Chenab join to form Panjnad, 23. Consider the following statements :
10 miles north of Uch Sharif in Muzaffar Garh district. 1. In the tropical zone, the western sections of the
The combined stream runs southwest for approximately oceans are warmer than the eastern sections
44 miles and joins the Indus River at Mithankot. owing to the influence of trade winds.
Hence, option (a) is the correct answer. 2. In the temperate zone, westerlies make the
20. With reference to India, Didwana, Kuchaman, eastern sections
Sargol and Khatu are the names of Which of the statements given above is/are
(a) glaciers (b) mangrove areas correct?
(c) Ramsar sites (d) saline lakes (a) 1 only (b) 2 only
Ans. (d): Didwana, Kuchaman, Sargal and Khatu are (c) Both 1 and 2 (d) Neither 1 nor 2
the name of saline lakes which is situated in Rajasthan.
Ans. (c): Statement 1 is correct. Warmer water is
Hence, option (d) is the correct answer. transported westward in the ocean by the Northeast
21. Consider the following rivers : trade winds in the Northern hemisphere of the tropical
1. Brahmani 2. Nagavali zone (as the figure given below shows). So, in tropical
3. Subarnarekha 4. Vamsadhara zones, the western section of ocean is warmer than
Which of the above rise from the Eastern eastern sections due to trade winds.
Ghats? Statement 2 is correct. Similarly, the Westerlies play
(a) 1 and 2 (b) 2 and 4 an important role in carrying the warm, equatorial
(c) 3 and 4 (d) 1 and 3 waters and winds to the western coasts of continents
Ans. (b) : (that is eastern section of the Oceans in temperate zone).
Brahmani and Subarnarekha originate from central Thus, in temperate zones, westerlies make the eastern
Indian Plateau regions. section of the ocean warmer than the western sections.
Vamsadhara River is an east-flowing river which Hence, option (c) is the correct answer.
originates in Kalahandi district of Odisha ie. Eastern 24. "Leaf litter decomposes faster than in any
Ghats. The River Nagavali also known as Langulya is other biome and as a result the soil surface is
one of the main rivers of Southern Odisha and Northern often almost bare. Apart from trees, the
Andhra Pradesh States in India, between Rushikulya vegetation is largely composed of plant forms
and Godavari basins. that reach up into the canopy vicariously, by
Statement- 1 and 3 are incorrect, while 2 and 4 are climbing the trees or growing as epiphytes,
correct. rooted on the upper branches of trees." This is
Hence, option (b) is the correct answer. the most likely description of
IAS (Pre) GS 2021 Paper I 6 YCT
(a) coniferous forest 2. In India, matters related to the constitutionality
(b) dry deciduous forest of the Amendment of an Act of the Parliament
(c) mangrove forest are referred to the Constitution Bench by the
(d) tropical rain forest Supreme Court.
Ans. (d): The hot and humid conditions make tropical Select the correct answer using the code given
below.
rainforests an ideal environment for bacteria and other
microorganisms. Because these organisms remain active (a) 1 only (b) 2 only
throughout the year, they quickly decompose matter on (c) Both 1 and 2 (d) Neither 1 nor 2
the forest floor. Ans. (c): Parliamentary sovereignty is a principle of the
Hence, option (d) is the correct answer. UK constitution. It makes Parliament the supreme legal
authority in the UK, which can create or end any law.
Hydrosphere Parliament of India is a creation of the Constitution and
hence for that reason itself, it has its limitations too.
25. With reference to the water on the planet Statement 1 is correct.
Earth, consider the following statements :
The Constitution (Forty-second Amendment) Act, 1976,
1. The amount of water in the rivers and lakes in inserted various articles in the Constitution to curtail,
more than the amount of groundwater. both directly and indirectly, the jurisdiction of the
2. The amount of water in polar ice caps and Supreme Court and the High Courts to review the
glaciers is more than the amount of constitutionality of laws. Statement 2 is incorrect.
groundwater. Hence, option (a) is the correct answer.
Which of the statements given above is/are
28. With reference to the Union Government,
correct? consider the following statements :
(a) 1 only (b) 2 only 1. N. Gopalaswamy Iyengar Committee
(c) Both 1 and 2 (d) Neither 1 nor 2 suggested that a minister and a secretary be
Ans. (b): Water in groundwater is 0.62% while in lakes designated solely for pursuing the subject of
and rivers it is 0.008 (Salt lakes)+0.009(freshwater administrative reform and promoting it.
lakes) + 0.0001 (rivers) = 0.0171. So, the statement 1 is 2. In 1970, the Department of Personnel was
not correct. constituted on the recommendation of the
Water in polar ice caps and glaciers is 2% while in Administrative Reforms Commission, 1966,
groundwater it is 0.62%. So, the statement 2 is correct. and this was placed under the Prime Minister's
Hence, option (b) is the correct answer. charge.
Which of the statements given above is/are
Agriculture correct?
26. Among the following which one is the least (a) 1 only (b) 2 only
water-efficient crop? (c) Both 1 and 2 (d) Neither 1 nor 2
(a) Sugarcane (b) Sunflower Ans. (d): 1st Administrative reforms committee
(c) Pearl millet (d) Red gram suggested that a minister and a secretary be designated
Ans. (a): It takes about 210 litres of water to produce 1 solely for pursing the subject of administrative reform
kg of sugar cane. Water footprint of sunflower is 7– 9 and promoting it. Hence statement 1 is not correct.
litre per kg. Millets are sown in dry areas with minimal In 1970, the Department of Personnel was constituted
irrigation. Red gram requires 35 to 40 cm water, during on the recommendation of the Administrative Reforms
its entire growth period. Optimum moisture is required Commission, 1966, and this was placed under the
during budding, flowering, and pod formation stages. charge of Cabinet Secretariat. In 1985, it was placed
As Red gram is a rain-fed crop grown in assured rainfall under the overall charge of the Prime Minister assisted
areas, usually it does not need any irrigation. by a Minister of State. Hence statement 2 is not correct
Hence option (a) is the correct answer. the prime minister assisted by a minister of State.
temperature. Hence statement 3 is incorrect. Hence, option (d) is the correct answer.
29. Constitutional government means
INDIAN CONSTITUTION & POLITY (a) a representative government of a nation with
federal structure
Central/ State & Union Territory (b) a government whose Head enjoys nominal
Parliamentary System powers
(c) a government whose Head enjoys real powers
27. We adopted parliamentary democracy based (d) a government limited by the terms of the
on the British model, but how does on the Constitution
British model, but how does our model differ
from that model? Ans. (d): Constitutional government in essence is about
constitutionalism which is about limited government. It is
1. As regards legislation, the British Parliament is defined by the existence of a constitution—which may be
supreme or sovereign but in India, the power a legal instrument or merely a set of fixed norms or
of the Parliament to legislate is limited. principles generally accepted as the fundamental law of
IAS (Pre) GS 2021 Paper I 7 YCT
the polity— that effectively controls the exercise of 34. What is the position of the Right to Property in
political power. In many cases, constitutional government India?
is used interchangeably with “constitutionally limited (a) Legal right available to citizens only
government” or “limited government”. (b) Legal right available to any person
Hence, option (d) is the correct answer. (c) Fundamental Right available to citizens only
30. What was the exact constitutional status of (d) Neither Fundamental Right nor legal right
India on 26th January, 1950?
Ans. (b): The Forty Fourth Constitutional Amendment,
(a) A Democratic Republic
1978, deleted Articles 19(1)
(b) A Sovereign Democratic Republic
(c) A Sovereign Secular Democratic Republic (f) and 31 from Part III, the chapter on Fundamental
(d) A Sovereign Socialist Secular Democratic Rights in the Constitution. Instead, it inserted Article
Republic 300A in a new chapter IV of Part XII of the
Constitution, thereby depriving the ‘right to property’ of
Ans. (b): In original preamble, during 26th January –
its ‘fundamental right’ status. Article 300A directs that -
1950, constitutional status of India was a, "Sovereign
Democratic Republic". Persons not to be deprived of property save by authority
Later, with 42nd Constitutional Amendment two more of law.—No person shall be deprived of his property
words were added to constitutional status of India are save by authority of law. Hence, option (b) is the
“socialist and secular”. correct answer.
Hence, option (b) is the correct answer. 35. Which one of the following best defines the
term 'State'?
Fundamental Right/ Fundamental Duties & (a) A community of persons permanently
Directive Principle of State Policy occupying a definite territory independent of
31. 'Right to Privacy' is protected under which external control and possessing an organized
Article of the Constitution of India? government
(a) Article 15 (b) Article 19 (b) A politically organized people of a definite
(c) Article 21 (d) Article 29 territory and possessing an authority to
Ans. (c) : The supreme court of India has held that right govern them, maintain law and order, protect
to privacy is a fundamental Right and it is protected their natural rights and safeguard their means
under Article 21 of the Constitution of India. of sustenance
Hence, option (c) is the correct answer. (c) A number of persons who have been living in
32. A legislation which confers on the executive or a definite territory for a very long time with
administrative authority an unguided and their own culture, tradition and government
uncontrolled discretionary power in the matter (d) A society permanently living in a definite
of application of law violates which one of the territory with a central authority, an executive
following Articles of the Constitution of India? responsible to the central authority and an
(a) Article 14 (b) Article 28 independent judiciary
(c) Article 32 (d) Article 44
Ans. (a): In political science, the term “State” has a
Ans. (a): Article 14 ensures that all citizens are equal and
that no person shall be discriminated on the basis of sex, more specific and definite meaning- “word State means
religion, race or place of birth. It ensures that there is a a community or society politically organized under one
separation of power between the three wings of the independent government within a definite territory. It
government and the executive. By these methods, the alone has the prerogative of making laws. The
constitution fulfils all the requirements of Dicey’s theory lawmaking power derives from sovereignty, which is
to be recognized as a country following the Rule of Law. the most distinctive characteristic of the State.
Thus, a legislation which confers on the executive or Hence option (a) is the correct answer.
administrative authority, an unguided and uncontrolled
36. Consider the following statements :
discretionary power in the matter of application of law,
will violate Rule of law as prescribed under Article 14. 1. In India, there is no law restricting the
So, option (a) is correct answer. candidates from contesting in one Lok Sabha
Hence option (a) is the correct answer. election from three constituencies.
33. Under the Indian Constitution concentration of 2. In 1991 Lok Sabha Election, Shri Devi Lal
wealth violates : contested from three Lok Sabha constituencies.
(a) the Rights to Equality 3. As per the existing rules, if a candidate
(b) the Directive Principles of State Policy contests in one Lok Sabha election from many
(c) the Right to Freedom constituencies, his/her party should bear the
(d) the Concept of Welfare cost of by-elections to the constituencies
Ans. (b): In Indian constitution, part IV (Directive vacated by him/her in the event of him/her
Principles of State Policy) Article – 39(c) says that "The winning in all the constituencies.
operation of the economic system does not result in the Which of the statements given above is/are
concentration of wealth and means of production to the correct?
common detriment". (a) 1 only (b) 2 only
Hence, option (b) is the correct answer. (c) 1 and 3 (d) 2 and 3
IAS (Pre) GS 2021 Paper I 8 YCT
Ans. (b) : Statement – 1 (Not – Correct) – Because Ans. (b): Judicial Custody is the custody of court. It is
According to Section 33 of Representation of People mentioned in section 167 of the Code of Criminal
Act 1951, "A person shall not be nominated as a Procedure (CrPC). Remand order is issued by the
candidate for election in the case of a general election to Judicial Magistrate and on the basis of remand order the
the house of the people, from more than two accused is lodged to Judicial Custody. To release the
parliamentary constituencies". accused from custody, a warrant of release is necessary.
Statement – 2 (correct) – In 1991 Lok Sabha Election, Police Custody is different from Judicial custody, where
Shri Devi Lal contested from three Lok Sabha the Police can seek for 15 Day, for the purpose of
Constituencies. interrogation and investigation, where the accused is in
Statement – 3 (Not – correct) – There is no such police custody/lockup.
provision that candidate contests in one Lok Sabha Therefore, the correct answer is (b).
election from many constituencies, his/her party should 39. With reference to Indian judiciary, consider
bear the cost of by-elections to the constituencies the following statements :
vacated by him/her in the event of him/her winning in 1. Any retired judge of the Supreme Court of
all the constituencies. India can be called back to sit and act as a
37. Consider the following statements : Supreme Court judge by the Chief Justice of
1. 'Right to the City' is an agreed human right and India with prior permission of the President of
the UN-Habitat monitors the commitments India.
made by each country in this regard. 2. A High Court in India has the power to review
2. 'Right to the City' gives every occupant of the its own judgment as the Supreme Court does.
city the right to reclaim public spaces and Which of the statements given above is/are
public participation in the city. correct?
3. 'Right to the City' means that the State cannot (a) 1 only (b) 2 only
deny any public service or facility to the (c) Both 1 and 2 (d) Neither 1 nor 2
unauthorized colonies in the city.
Ans. (c): Statement 1 is correct: Under Article 128 of
Which of the statements given above is/are the Constitution, the Chief Justice of India may, at any
correct? time, with the previous consent of the President, request
(a) 1 only (b) 3 only any person who has held the office of a Judge of the
(c) 1 and 2 (d) 2 and 3 Supreme Court to sit and act as a Judge of the Supreme
Ans. (d) : On 10 December, we celebrate Human Court.
Rights Day and take the opportunity to denounce Statement 2 is correct: Being a Court Of Record, High
Human Rights violations and strengthen the connection Court can review its own judgments under Article 226
between Habitat Rights, the Right to the City and Of the Constitution. The Kerala High Court has iterated
Human Rights Cities. the legal proposition that High Courts as Courts of
Statement – 2 (Correct) – Right to the city means right Record could review their own orders. A Division
to reclaim public spaces and public participation in the Bench of Chief Justice S Mani kumar and Shaji P Chaly
city. were confronted with an appeal against a review
Statement – 3 (correct) – Because, 'Right to the city' petition.
means that, the State cannot deny any public service or Hence, option (c) is the correct answer.
facility to the unauthorized colonies in the city. A 40. With reference to India, consider the following
dominant "privatization ethic" increasingly subjects statements:
access to adequate housing and land to income
competition, leading to new forms of discrimination. 1. When a prisoner makes out a sufficient case,
This is unacceptable, particularly given the obligations parole cannot be denied to such prisoner
of states and cities to respect protect and fulfill the because it becomes a matter of his/her right.
human right to adequate housing and land. 2. State Governments have their own Prisoners
Hence option (d) is the correct answer. Release on Parole Rules.
Which of the statements given above is/are
Judiciary System correct?
38. With reference to India, consider the following (a) 1 only (b) 2 only
statements : (c) Both 1 and 2 (d) Neither 1 nor 2
1. Judicial custody means an accused is in the Ans. (b): Parole is a system of releasing a prisoner with
custody of the concerned magistrate and such suspension of the sentence. The release is conditional,
accused is locked up in police station, not in usually subject to behaviour, and requires periodic
jail. reporting to the authorities for a set period of time.
2. During judicial custody, the police officer in Parole is considered a reformative process. The
charge of the case is not allowed to interrogate provision (along with furlough) was introduced with a
the suspect without the approval of the court. view to humanising the prison system. It is not a matter
Which of the statements given above is/are of right. Hence statement 1 is not correct.
correct? In India, parole (as well as furlough) are covered under
(a) 1 only (b) 2 only The Prisons Act of 1894. Prisoners convicted of multiple
(c) Both 1 and 2 (d) Neither 1 nor 2 murders or under the anti-terror Unlawful Activities
IAS (Pre) GS 2021 Paper I 9 YCT
Prevention Act (UAPA) are not eligible for parole. Since A foreigner once granted the citizenship can be
prisons is a State subject in the Constitution, the Prisons deprived of the citizenship under circumstances like
Act of each state government defines the rules under registration is done by fraud, person is disloyal towards
which parole is granted in that state constitution, voluntarily acquires citizenship of other
Hence option (b) is the correct answer. country, etc. So, statement 3 is not correct.
Hence, option (a) is the correct answer.
41. Which one of the following in Indian polity is
an essential feature that indicates that it is 44. Which one of the following factors constitutes
federal in character? the best safeguard of liberty in a liberal
(a) The independence of judiciary is safeguarded. democracy?
(a) A committed judiciary
(b) The Union Legislature has elected
representatives from constituent units. (b) Centralization of powers
(c) The Union Cabinet can have elected (c) Elected government
representatives from regional parties. (d) Separation of powers
(d) The Fundamental Rights are enforceable by Ans. (d): Separation of power (executive, legislature
Courts of Law. and judiciary) checks the abuse of power by other
organs, this helps in checking the authoritarian
Ans. (a): The federal character of the Indian Polity is
tendencies, this is the best safeguard of liberty in
indicated by following features: Written Constitution,
democracy.
Supremacy of the Constitution, Rigid Constitution,
Division of Powers, Independent Judiciary, Bicameral Hence, option (d) is the correct answer.
Legislature, Dual Government Polity, etc.
Hence, option (a) is the correct answer.
ECONOMICS
Fifth & Sixth Schedule / Tribal Region Banking System & Monetary Policy/
Inflation
42. At the national level, which ministry is the
nodal agency to ensure effective 45. Consider the following statements:
implementation of the Scheduled Tribes and 1. The Governor of the Reserve Bank of India
Other Traditional Forest Dwellers (Recognition (RBI) is appointed by the Central Government.
of Forest Rights) Act, 2006? 2. Certain provisions in the Constitution of India
(a) Ministry of Environments, Forest and Climate give the Central Government the right to issue
Change directions to the RBI in public interest.
(b) Ministry of Panchayati Raj 3. The Governor of the RBI draws his power the
(c) Ministry of Rural Development RBI Act.
(d) Ministry of Tribal Affairs Which of the above statements are correct?
(a) 1 and 2 only (b) 2 and 3 only
Ans. (d): Ministry of Tribal Affairs is the nodal agency
at the national level to ensure effective implementation (c) 1 and 3 only (d) 1, 2 and 3
of the schedule tribes and other traditional forest Ans. (c) : Statement- 1 (correct) – According to RBI
dwellers under Recognition of Forest Rights Act, 2006. Act 1934, Section 8(1)(a), a Governor (not more than
Hence, option (d) is the correct answer. four) and Deputy Governors to be appointed by the
central Government.
CITIZENSHIP Statement– 2 (in-correct) – RBI Act, 1934 - The
Central Government may from time to time give such
43. With reference to India, consider the following directions to the Bank as it may, after consultation with
statements the Governor of the Bank, consider necessary in the
1. There is only one citizenship and one domicile. public interest.
2. A citizen by birth only can become the Head of Statement– 3 (correct) – The Governor and in his
State. absence the Deputy Governor nominated by him in this
3. A foreigner once granted the citizenship behalf, shall also have powers of general
cannot be deprived of it under any superintendence and direction of the affairs and the
circumstances. business of the Bank, and may exercise all powers and
Which of the statements given above is/are do all acts and things which may be exercised or done
correct? by the Bank.
(a) 1 only (b) 2 only As per the given statements, statement 1 and 3 are
(c) 1 and 3 (d) 2 and 3 correct and statement 2 are incorrect. Hence option (c)
Ans. (a): In India, we have single citizenship, i.e., a is the correct answer of this question.
person can have only one citizenship. Also, at a 46. Which among the following steps is most likely
particular time, a person can have only one domicile. to be taken at the time of an economic
Thus, statement 1 is correct. recession?
Any citizen (Citizen by birth or Naturalised Citizen) can (a) Cut in tax rates accompanied by increase in
become the Head of State, i.e., the President, in India. expenditure on public projects
So, statement 2 is not correct. (b) Increase in expenditure on public projects
IAS (Pre) GS 2021 Paper I 10 YCT
(c) Increase in tax rates accompanied by Statement 2 is correct. A cooperative bank can, with
reduction of interest rate prior approval of the RBI, issue equity shares,
(d) Reduction of expenditure on public projects preference shares, or special shares to its members or to
Ans. (b) Economic Recession is characterized by any other person residing within its area of operation,
negative growth rate of GDP in two successive quarters, by way of public issue or private placements.
fall down in the economy and fall in investments are the Statement 3 is correct. Large cooperative banks with
indicators of recessions. paid-up share capital and reserves of Rs.1 lakh were
Cut in tax rates accompanied by increases in interest brought under the purview of the Banking Regulation
rate resulted in credit crunch in the economy which are Act 1949 through an amendment in 1966.
not desirable in the time of Economic Recession. So The correct answer is option (b).
option (a) is in-correct.
49. Indian Government Bond Yields are influenced
Increase in expenditure on public projects is the main
tool to stimulate the economy in the time of recession, by which of the following?
which triggers the virtuous cycle of investment that lead 1. Actions of the United States Federal Reserve
to increase in GDP and increase in economy. So this is 2. Actions of the Reserve Bank of India
the most desirable step to boost the economy. Hence 3. Inflation and short-term interest rates
option (b) is correct. Select the correct answer using the code given
47. Consider the following statements : below.
Other things remaining unchanged, market (a) 1 and 2 only (b) 2 only
demand for a good might increase if (c) 3 only (d) 1, 2 and 3
1. price of its substitute increases Ans. (d) : Bonds yields in India are affected by –
2. price of its complement increases • There are a number of economic factors that impact
3. the good is an inferior good and income of the Treasury yields, such as interest rates, inflation, and
consumers increases economic growth.
4. its price falls
• The major factors affecting the yield are the
Which of the above statements are correct? monetary policy of the Reserve Bank of India,
(a) 1 and 4 only (b) 2, 3 and 4 especially the course of interest rates, fiscal position
(c) 1, 3 and 4 (d) 1, 2 and 3 of the government, global markets, economy and the
Ans. (a): Statement -1 (correct) – Because the demand inflation.
for a good usually moves in the direction of the price of • US Bond Yields.
its substitutes, when it increases or decreases.
Statement – 2 (in-correct) – Goods which are 50. Consider the following :
consumed together are called complementary goods. 1. Foreign currency convertible bonds
An increase in price of complementary goods leads to 2. Foreign institutional investment with certain
reduce in demand. conditions
Statement – 3 (in-correct) – In Inferior Goods, as the 3. Global depository receipts
income of the consumer increase, the demand for an 4. Non-resident external deposits
inferior good falls, and when the income decreases, the Which of the above can be included in Foreign
demand for an inferior good increases. Direct Investments ?
Statement -4 (correct) – Because, when price of the (a) 1, 2 and 3 (b) 3 only
commodity increases, demand for it falls and when
price of the commodity decreases, demand for it rise, (c) 2 and 4 (d) 1 and 4
other factors remaining the same. Ans. (a): In Capital Account of Balance of Payment, we
Hence option (a) is correct answer. can classify into Investment, Borrowings and External
48. With reference to 'Urban Cooperative Banks' Assistance. Investment includes Equity flow in the
in India, consider the following statements: economy. Foreign Currency Convertible Bonds
1. They are supervised and regulated by local (FCCB), Foreign Institutional Investment with certain
boards set up by the State Governments. conditions (subject to the overall limit of 24%), and
2. They can issue equity shares and preference Global Depository Receipts (GDR) are the instruments
shares. for the foreign investment in India. Hence options 1, 2
3. They were brought under the purview of the and 3 are correct. Non-Resident external deposits are a
Banking Regulation Act, 1949 through an ‘debt creating' flow in balance of payments accounts
Amendment in 1966. and therefore, not part of Foreign Direct investments.
Which of the statements given above is/are Hence options 4 is not correct.
correct? 51. Consider the following statements :
(a) 1 only (b) 2 and 3 only The effect of devaluation of a currency is that it
(c) 1 and 3 only (d) 1, 2 and 3 necessarily.
Ans. (b): Statement 1 is incorrect. The Urban Banks 1. improves the competitiveness of the domestic
Department of the Reserve Bank of India is vested with exports in the foreign markets
the responsibility of regulating and supervising primary 2. increases the foreign value of domestic
(urban) cooperative banks, which are popularly known currency
as Urban Cooperative Banks (UCBs). 3. improves the trade balance
IAS (Pre) GS 2021 Paper I 11 YCT
Which of the above statements is/are correct ? Banks create money by making loans. A bank loans or
(a) 1 only (b) 1 and 2 invests its excess reserves to earn more interest. A one-
(c) 3 only (d) 2 and 3 dollar increase in the monetary base causes the money
Ans. (a): A devaluation means there is a fall in the supply to increase by more than one dollar.
value of a currency. The main effects are: The increase in the money supply is the money
multiplier.
Statement 1 is correct.
Exports are cheaper to foreign customers: If the value of 55. With reference to Indian economy, demand-
the rupee decreases against the dollar, the price of the pull inflation can be caused/increased by which
cars sold by Indian manufacturers in America, in of the following?
dollars, will be effectively less expensive than they 1. Expansionary policies
were before. On the other hand, a more valuable 2. Fiscal stimulus
currency makes exports relatively more expensive for 3. Inflation-indexing wages
purchase in foreign markets. 4. Higher purchasing power
Statement 2 and 3 are incorrect. 5. Rising interest rates
Devaluation of a currency decreases the foreign value of Select the correct answer using the code given
domestic currency. Devaluation also increases the debt below.
burden of foreign-denominated loans when priced in the (a) 1, 2 and 4 only (b) 3, 4 and 5 only
home currency. Thus, devaluation may not improve (c) 1, 2, 3 and 5 only (d) 1, 2, 3, 4 and 5
the trade balance in the long run. Ans. (a): With reference to Indian economy demand-
52. Which one of the following effects of creation of pull inflation can be caused/increased by expansionary
black money in India has been the main cause policies, fiscal stimulus and higher purchasing power.
of worry to the Government of India? Hence, option (a) is the correct answer.
(a) Diversion of resources to the purchase of real 56. With reference to India, consider the following
estate and investment in luxury housing statements:
(b) Investment in unproductive activities and 1. Retail investors through demat account can
purchase of precious stones, jewellery, gold, invest in 'Treasury Bills' and 'Government of
etc. India Debt Bonds' in primary market.
(c) Large donations to political parties and 2. The 'Negotiated Dealing System-Order
growth of regionalism Matching' is a government securities trading
(d) Loss of revenue to the State Exchequer due to platform of the Reserve Bank of India.
tax evasion 3. The 'Central Depository Services Ltd.' is
Ans. (d): Loss of revenue to the state Exchequer due to jointly promoted by the Reserve Bank of India
tax evasion are the main cause of worry to the and the Bombay Stock Exchange.
Government of India effects the creation of Black- Which of the statements given above is/are
money in India. correct?
Hence, option (d) is correct. (a) 1 only (b) 1 and 2
53. Which one of the following is likely to be the (c) 3 only (d) 2 and 3
most inflationary in its effects? Ans. (b): Statement -1 (Correct) – RBI allowed the
(a) Repayment of public debt retail investors to directly purchase government bond by
(b) Borrowing from the public to finance a opening gilt accounts in RBI.
budget deficit Statement – 2 (Correct) – Negotiated Dealing System
(c) Borrowing from the banks to finance a budget Order matching is a screen based electronic anonymous
deficit order matching system for secondary market trading in
(d) Creation of new money to finance a budget Government Securities owned by RBI.
deficit Statement – 3 (Incorrect) – Central Depositary
Ans. (d): Deficit financing always brings inflationary Services Ltd. was promoted by BSE Ltd. jointly with
pressure whatever be the means but 'creation' (printing) leading Banks Such as, SBI, BOB, HDFC, and Union
of new money has the highest inflationary impact. So, Bank not by R.B.I.
the correct answer is option (d). Hence, option (b) is the correct answer.
54. The money multiplier in an economy increases
with which one of the following? World's Organization/ Agriculture &
(a) Increase in the Cash Reserve Ratio in the Industry and Foreign Trade.
banks 57. With reference to 'Water Credit', consider the
(b) Increase in the Statutory Liquidity Ratio in following statements :
the banks 1. It puts microfinance tools to work in the water
(c) Increase in the banking habit of the people and sanitation sector.
(d) Increase in the population of the country 2. It is a global initiative launched under the aegis
Ans. (c): The money created by the Federal Reserve is of the World Health Organization and the
the monetary base, also known as high-powered money. World Bank.
IAS (Pre) GS 2021 Paper I 12 YCT
3. It aims to enable the poor people to meet their (a) it is dipolar in nature
water needs without depending on subsidies. (b) it is a good conductor of heat
Which of the statements given above are correct ? (c) it has high value of specific heat
(a) 1 and 2 only (b) 2 and 3 only (d) it is an oxide of hydrogen
(c) 1 and 3 only (d) 1, 2 and 3 Ans. (a): Water is called the Universal Solvent it is
Ans. (c): Statement 1 is correct. Water Credit is a capable of dissolving a variety of different substances
powerful solution and the first to put micro finance more than any other liquid. Water can dissolve more
tools to work in the water and sanitation sector. Water substances than any other compound because of its
Credit helps bring small loans to those who need access polar nature.
to affordable financing and expert resources to make Hence, option (a) is the correct answer.
household water and toilet solutions a reality. Chemistry
Statement 2 is incorrect. Water credit is an initiative of 61. Bisphenol A (BPA), a cause of concern, is a
Water.org. Water.org is a global non-profit organization structural/key component in the manufacture
working to bring water and sanitation to the world. of which of the following kinds of plastics?
Statement 3 is correct. It helps people get access to (a) Low-density polyethylene
safe water and sanitation through affordable financing, (b) Polycarbonate
such as small loans. (c) Polyethylene terephthalate
58. In India, the central bank's function as the (d) Polyvinyl chloride
'lender of last resort' usually refers to which of Ans. (b): Bisphenol A (BPA) is an industrial chemical
the following? used to make polycarbonate, a hard, clear plastic, which
1. Lending to trade and industry bodies when is used in many consumer products. BPA is also found
they fail to borrow from other sources in epoxy resins, which act as a protective lining on the
2. Providing liquidity to the banks having a inside of some metal-based food and beverage cans.
temporary crisis Bisphenol A can leach into food from the protective
3. Lending to governments to finance budgetary internal epoxy resin coatings of canned foods and from
deficits consumer products such as polycarbonate tableware,
Select the correct answer using the code given food storage containers, water bottles, and baby bottles.
The degree to which BPA leaches from polycarbonate
below.
bottles into liquid may depend more on the temperature
(a) 1 and 2 (b) 2 only of the liquid or bottle, than the age of the container.
(c) 2 and 3 (d) 3 only BPA can also be found in breast milk. One reason
Ans. (b): In India, the central Bank's function as the people may be concerned about BPA is because human
'lender of last resort' usually refers to providing liquidity exposure to BPA is widespread.
to the Banks having a temporary crisis. Hence, option (b) is the correct answer.
Hence, option (b) is the correct answer. Zoology
SCIENCE 62.
Consider the following statements :
1. Adenoviruses have single-stranded DNA
Physics genomes whereas retroviruses have double-
59. In a pressure cooker, the temperature at which stranded DNA genomes.
the food is cooked depends mainly upon which 2. Common cold is sometime caused by an
of the following? adenovirus whereas AIDS is caused by a
1. Area of the hole in the lid retrovirus.
2. Temperature of the flame Which of the statements given above is/are
3. Weight of the lid correct?
(a) 1 only (b) 2 only
Which of the statements given above is/are
correct? (c) Both 1 and 2 (d) Neither 1 nor 2
(a) 1 and 2 only (b) 2 and 3 only Ans. (b): It is actually the opposite the statement given.
(c) 1 and 3 only (d) 1, 2 and 3 Adenoviruses have double stranded DNA genomes
while Retroviruses have single stranded DNA genomes.
Ans. (c): Inside a pressure cooker the boiling point is So, statement 1 is not correct.
increased by increasing the pressure. Hence cooking is Common cold can sometimes be caused by adenoviruses
faster. Therefore, the temperature at which food is while AIDS is caused by retrovirus. So, statement 2 is
cooked is affected by the pressure and not temperature correct. Therefore, the correct answer is (b).
of the flame. This can also be understood by
understanding why cooking is difficult on hills. At high Botany
altitudes, atmospheric pressure is lower, reducing the 63. Consider the following :
boiling point of water as compared to that at sea level. 1. Bacteria
60. Water can dissolve more substances than any 2. Fungi
other liquid because 3. Virus
IAS (Pre) GS 2021 Paper I 13 YCT
Which of the above can be cultured in In Pronuclear transfer technique MRT is done after In
artificial/synthetic medium? vitro fertilisation (IVF). Hence Statement 1 is true.
(a) 1 and 2 only (b) 2 and 3 only A child inherits mitochondrial diseases only from
(c) 1 and 3 only (d) 1, 2 and 3 Mother. Hence, Statement 2 is true.
Ans. (a): Options 1 and 2 are correct: So, option (c) is the correct answer.
Microbes like bacteria and many fungi can be grown on 66. Bollgard I and Bollgard II technologies are
nutritive media to form colonies that can be seen with mentioned in the context of :
the naked eyes. Such cultures are useful in studies on (a) clonal propagation of crop plants
micro -organisms. Option 3 is not correct: Unlike (b) developing genetically modified crop plants
bacteria, many of which can be grown on an artificial (c) production of plant growth substances
nutrient medium, viruses require a living host cell for
(d) production of biofertilizers
replication. Infected host cells (eukaryotic or
prokaryotic) can be cultured and grown, and then the Ans. (b) : Bollgard I, the predominantly cultivated pest
growth medium can be harvested as a source of virus. tolerant cotton, contains only one gene, the Cry 1Ac,
Hence, option (a) is the correct answer. from Bacillus thuringiensis. Bollgard II contains the Cry
2 Ab gene, in addition to Cry 1 Ac.
Science & Bio- technology Hence, option (b) is the correct answer.
64. With reference to recent developments 67. With reference to street-lighting, how do
regarding 'Recombinant Vector Vaccines', sodium lamps differ from LED lamps?
consider the following statements : 1. Sodium lamps produce light in 360 degrees but
1. Genetic engineering is applied in the it is not so in the case of LED lamps.
development of these vaccines. 2. As street-lights, sodium lamps have longer life
2. Bacteria and viruses are used as vectors. span that LED lamps.
Which of the statements given above is/are 3. The spectrum of visible light from sodium
correct? lamps is almost monochromatic while LED
(a) 1 only (b) 2 only lamps offer significant colour advantages in
(c) Both 1 and 2 (d) Neither 1 nor 2 street-lighting.
Ans. (c): As per the given statements in reference to Select the correct answer using the code given
recent developments regarding 'Recombinant Vector below.
Vaccines' both the given statements are correct. (a) 3 only (b) 2 only
The recombinant virus vectors vaccine is one of the (c) 1 and 3 only (d) 1, 2 and 3
important emerging technology are live replicating Ans. (c): Sodium lamps are multi-directional that they
viruses that are engineered to carry extra genes derived emit light in 360 degrees, while led lamps emit light in
from a pathogen and these extra genes produce proteins
180 degrees. Hence this statement 1 is correct.
against which we want to generate immunity.
Lifespan of LED is more than Sodium lamps. Hence,
It exposes the body to proteins made by a virus as
this statement is incorrect.
bacteria, are often made by a using weakened or
inactive versions of that virus or bacteria. Sodium lamps are monochromatic and LEDs can be
Hence, option (c) is the correct answer. designed to generate the entire spectrum of visible light
colors without having to use the traditional color filters
65. In the context of hereditary diseases, consider required by traditional lighting solutions. This is correct
the following statements: statement.
1. Passing on mitochondrial diseases from parent
to child can be prevented by mitochondrial 68. 'Triclosan', considered harmful when exposed
replacement therapy either before or after in to high levels for a long time, is most likely
vitro fertilization of egg. present in which of the following?
2. A child inherits mitochondrial diseases entirely (a) Food preservatives
from mother and not from father. (b) Fruit-ripening substances
Which of the statements given above is/are (c) Reused plastic containers
correct? (d) Toiletries
(a) 1 only (b) 2 only Ans. (d): Triclosan is a lipophilic compound with
(c) Both 1 and 2 (d) Neither 1 nor 2 antimicrobial properties. It is an ingredient added to
Ans. (c): Mitochondrial Replacement Therapy(MRT), many consumer products intended to reduce or prevent
sometimes called mitochondrial donation, is the bacterial contamination. It is added to some
replacement of mitochondria in one or more cells to antibacterial soaps and body washes, toothpastes, and
prevent or ameliorate disease. some cosmetics. It also can be found in clothing,
MRT originated as a special form of In Vitro kitchenware, furniture, and toys.
Fertilisation in which some or all of the future baby’s Hence, option (d) is the correct answer.
mitochondrial DNA (mtDNA) comes from a third party. 69. Which one of the following is a reason why
This technique is used in cases when mothers carry astronomical distances are measured in light-
genes for mitochondrial diseases. years?
IAS (Pre) GS 2021 Paper I 14 YCT
(a) Distances among stellar bodies do not change. Ans. (d): Statement 1 is correct: Furnace oil or fuel oil
(b) Gravity of stellar bodies does not change. is a dark viscous residual fuel obtained by blending
(c) Light always travels in straight line. mainly heavier components from crude distillation unit,
(d) Speed of light is always same. short residue and clarified oil from fluidized catalytic
Ans. (d): Year is the distance a beam of light travels in cracker unit.
a vacuum in one year. The reasons for measuring Statement 2 is correct: It is used primarily for steam
astronomical distances in light years are familiar units boilers in power plants, aboard ships, and in
like kilometers and miles are absurdly small for industrial plants.
describing the vastness of the cosmos and the speed of Statement 3 is correct: Sulphur contained in fuel causes
light is both finite and constant with a value of emissions of sulphur dioxide (SO2) and also contributes
approximately 2.998 x 108 m/s. to the formation of secondary particulate matter (PM).
Hence option (d) is the correct answer. 72. In the nature, which of the following is/are
most likely to be found surviving on a surface
ENVIRONMENT AND ECOLOGY without soil?
1. Fern 2. Lichen
70. Why is there a concern about copper smelting 3. Moss 4. Mushroom
plants: Select the correct answer using the code given
1. They may release lethal quantities of carbon below
monoxide into environment. (a) 1 and 4 only (b) 2 only
2. The copper slag can cause the leaching of (c) 2 and 3 (d) 1, 3 and 4
some heavy metals into environment.
Ans. (c): Ferns require indirect sunlight, moist soil, and
3. They may release sulphur dioxide as a
pollutant. a humid atmosphere. Lichens occur from sea level to
high alpine elevations, in many environmental
Select the correct answer using the code given
conditions, and can grow on almost on any surface.
below.
Mosses are non-flowering plants which produce spores
(a) 1 and 2 only (b) 2 and 3 only
and have stems and leaves, but don’t have true roots.
(c) 1 and 3 only (d) 1, 2 and 3 The individual plants are usually composed of simple
Ans. (b): Every smelting plant has its own oxygen leaves that are generally only one cell thick, attached to
production unit, to feed in the smelting process as well a stem that may be branched or un-branched and has
as reduce the exhaust of gases. So, carbon monoxide only a limited role in conducting water and nutrients. A
although produced in the smelting process would get mushroom or toadstool is the fleshy, spore-bearing
converted to carbon dioxide if the plant is functioning fruiting body of a fungus, typically produced above
properly and if there are any issues with the oxygen- ground, on soil, or on its food source.
producing unit, the smelting process will slow down. So
the release of a lethal amount of carbon mono oxide. So, 73. Which one of the following is used in preparing
statement 1 is correct. a natural mosquito repellent?
The other byproduct of smelting is slag, the waste (a) Congress grass (b) Elephant grass
matter separated from metals. This slag may leach (c) Lemongrass (d) Nut grass
heavy metals (arsenic, cadmium, lead or mercury Ans. (c): Lemongrass used in preparing a natural
depending on the composition of the ore) into ground mosquito repellent found in candles, sprays and lotions.
water reservoirs. Water which has a high heavy metal Hence, option (c) is the correct answer.
content is very toxic to humans. Slag may also increase 74. Consider the following kinds of organisms :
the concentration of other, less harmful salts in water
1. Copepods 2. Cyanobacteria
resources, which may change the taste of water. So,
statement 2 is correct. 3. Diatoms 4. Foraminifera
Most copper ores are sulphur-based and smelting Which of the above are primary producers in
releases sulphur dioxide, an air pollutant known to the food chains of oceans?
have many harmful effects. So, statement 3 is correct. (a) 1 and 2 (b) 2 and 3
Therefore, the correct answer is (b). (c) 3 and 4 (d) 1 and 4
71. With reference to furnace oil, consider the Ans. (b) : Cyanobacteria and Diatoms are the primary
following statements : producers in the food chains of oceans and copepods
1. It is a product of oil refineries. and foraminifera are not the primary producers in the
2. Some industries use it to generate power. food chains of oceans.
3. Its use causes sulphur emission into Hence, option (b) is the correct answer.
environment. 75. Consider the following animals :
Which of the statements given above are correct? 1. Hedgehog
(a) 1 and 2 only (b) 2 and 3 only 2. Marmot
(c) 1 and 3 only (d) 1, 2 and 3 3. Pangolin
IAS (Pre) GS 2021 Paper I 15 YCT
To reduce the chance of being captured by barrier. These tiny particles, each less than a thousandth
predators, which of the above organisms rolls the width of a human hair, are generated by a wide
up/roll up and protects/protect its/their variety of processes, both natural and fabricated,
vulnerable parts? including forest fires and wood -burning stoves, road
(a) 1 and 2 (b) 2 only traffic pollution, and many high -temperature industrial
(c) 3 only (d) 1 and 3 processes.
Ans. (d): Hedgehogs have spikes on their back, which In addition to occupational settings (including, for
help to deter any threat when they roll up & protect example, exposure to printer toner powders), higher
their vulnerable parts. concentrations of magnetite pollution nano particles
Marmots build elaborate burrows for dinning and may arise in the indoor environment from open fires or
protection from predators. They have no rolling up poorly-sealed stoves used for cooking and/or heating,
mechanism like other 2. and in the outdoor environment from vehicle (tailpipe
Pangolins have hard scales on their back which help to and brake wear) and/or industrial PM sources.
protect its vulnerable parts when it rolls up. So option (d) is the correct answer.
Therefore, the correct answer is (d). 78. Which one of the following is a filter feeder?
76. With reference to the 'New York Declaration (a) Catfish (b) Octopus
on Forests', which of the following statements (c) Oyster (d) Pelican
are correct? Ans. (c): Filter feeder is an animal that obtains its good
1. It was first endorsed at the United Nations by filtering organic matter or minute organisms from a
Climate Summit in 2014. current of water that passes through some part of its
2. It endorses a global timeline to end the loss of system, Oysters are natural filter feeders.
forests. Hence, option (c) is the correct answer.
3. It is a legally binding international declaration. 79. In case of which one of the following
4. It is endorsed by governments, big companies biogeochemical cycles, the weathering of rocks
and indigenous communities. is the main source of release of nutrient to
5. India was one of the signatories at its enter the cycle?
inception. (a) Carbon cycle (b) Nitrogen cycle
Select the correct answer using the code given (c) Phosphorus cycle (d) Sulphur cycle
below.
Ans. (c): Phosphorus is tied up in rock and sedimentary
(a) 1, 2 and 4 (b) 1, 3 and 5
deposits from which it is released by weathering,
(c) 3 and 4 (d) 2 and 5
leaching and mining.
Ans. (a): As per the given statements, statements 1, 2 Hence, option (c) is the correct answer.
and 4 are correct in reference to the 'New York
Declaration on Forests'. The New York Declaration on 80. Which of the following are detritivores?
forests (NYDF) is a political declaration calling for 1. Earthworms 2. Jellyfish
global action to protect and restore forests. It offers a 3. Millipedes 4. Seahorses
common, multi-stakeholder framework for forest action, 5. Woodlice
consolidating various initiatives and objectives that Select the correct answer using the code given
drive forest protection, restoration and sustainable use. below.
Hence option (a) is the correct answer. (a) 1, 2 and 4 only (b) 2, 3, 4 and 5 only
77. Magnetite particles, suspected to cause (c) 1, 3 and 5 only (d) 1, 2, 3, 4 and 5
neurodegenerative problems, are generated as Ans. (c): An animal that feeds on detritus. Examples of
environmental pollutants from which of the detritivores are earthworms, blowflies, maggots,
following?
millipedes, dung beetles, fiddler crabs, sea cucumbers
1. Brakes of motor vehicles and woodlice.
2. Engines of motor vehicles
Detritivores play an important role in the breakdown of
3. Microwave stoves within homes organic matter from decomposing animals and
4. Power plants plants
5. Telephone lines
A seahorse gains energy by eating other live organisms,
Select the correct answer using the code given meaning it is a carnivore. A
below.
seahorse is not a decomposer because decomposers gain
(a) 1, 2, 3 and 5 only (b) 1, 2 and 4 only
energy from breaking down dead
(c) 3, 4 and 5 only (d) 1, 2, 3, 4 and 5
organisms.
Ans. (d): Magnetite can have potentially large impacts Jellyfish are carnivores and excellent predators. They
on the brain due to its unique combination of redox
activity, surface charge and strongly magnetic behavior. sting with tentacles to subdue small
Airborne magnetite pollution particles < ~200 nm in aquatic fish and eat the eggs and invertebrates that stick
size can access the brain directly via the olfactory to their tentacles
and/or trigeminal nerves, bypassing the blood -brain Therefore, the correct answer is (c)..
IAS (Pre) GS 2021 Paper I 16 YCT
81. The 'Common Carbon Metric', supported by Permaculture cultivation discourages monoculture
UNEP, has been developed for? practices, improves soil fertility and encourages
(a) assessing the carbon footprint of building mulching. Hence, Statements 1, 2 and 4 are true.
operations around the world Hence option (b) is the correct answer.
(b) enabling commercial farming entities around 84. With reference to 'palm oil', consider the
the world to enter carbon emission trading following statements:
(c) enabling governments to assess the overall 1. The palm oil tree is native to Southeast Asia.
carbon footprint caused by their countries
2. The palm oil is a raw material for some
(d) assessing the overall carbon footprint caused industries producing lipstick and perfumes.
by the use of fossil fuels by the world in a
unit time 3. The palm oil can be used to produce biodiesel.
Which of the statements given above are correct?
Ans. (a): The 'common carbon Metric', supported by
UNEP has been developed for assessing the carbon foot (a) 1 and 2 only (b) 2 and 3 only
print of building operations around the world. (c) 1 and 3 only (d) 1, 2 and 3
Hence, option (a) is the correct answer. Ans. (b): Palm oil is native to Africa and brought to
82. Which of the following have species that can southeast Asia during colonial times, palm oil
establish symbiotic relationship with other plantations now cover huge areas of Malaysia and
organisms? Indonesia.
1. Cnidarians Palm oil is used in lipstick as it holds color well, doesn’t
2. Fungi melt at high temperatures, and has a smooth application
3. Protozoas and virtually no taste.
Select the correct answer using the code given Palm can be used for making biodiesel also.
below. Hence, Statement-1 is not correct while Statement-2 and
(a) 1 and 2 only (b) 2 and 3 only 3 are correct.
(c) 1 and 3 only (d) 1, 2 and 3 85. Consider the following statements :
Ans. (d): Symbiosis is a close ecological relationship 1. The Global Ocean Commission grants licenses
between the individuals of two (or more) different for seabed exploration and mining in
species. The relationship between cnidarians and dinofl international waters.
agellate algae is termed as “symbiotic”. Two common 2. India has received licenses for seabed mineral
mutualistic relationships involving fungi are mycorrhiza exploration in international waters.
and lichen. Symbiosis in protozoa mostly represents a 3. 'Rare earth minerals' are present on seafloor in
close mutualistic association between a protozoan and international waters.
unicellular symbionts (bacteria, cyanobacteria or/and Which of the statements given above is/are
unicellular algae) or protozoans and a multicellular correct?
organism (ruminants, lower termites, wood-eating
(a) 1 and 2 only (b) 2 and 3 only
cockroaches, plants).
(c) 1 and 3 only (d) 1, 2 and 3
83. How is permaculture farming different from
conventional chemical farming? Ans. (b) : Statement – 1 (Not – Correct) – The Global
1. Permaculture farming discourages Ocean Commission was international initiative between
monocultural practices but in conventional 2013 and 2016 to raise awareness and promote action to
chemical farming, monoculture practices are address, the degradation of the ocean and help restore it
predominant. to full health and productivity. Licence for Seabed
2. Conventional chemical farming can cause exploration and mining in international waters are
increase in soil salinity but the occurrence of granted by International Seabed Authority.
such phenomenon is not observed in Statement – 2 (Correct) – India has been exploring
permaculture farming. deep sea for minerals since 1981. In 2017, India's
3. Conventional chemical farming is easily exclusive rights to explore polymetallic nodules from
possible in semi-arid regions but permaculture seabed in central Indian Ocean Basin have been
farming is not so easily possible in such rgions. extended by five years.
4. Practice of mulching is very important in Statement – 3 (Correct) – The deep seabed contains
permaculture farming but not necessarily so in two potential sources for rare earth elements,
conventional chemical farming. polymetallic nodules which typically contain,
Select the correct answer using the code given manganese, nickel, cobalt, copper and rare earth
below. minerals.
(a) 1 and 3 (b) 1, 2 and 4 Hence option (b) is the correct answer.
(c) 4 only (d) 2 and 3 86. In the context of India's preparation for
Ans. (b) : Since the principles of permaculture Climate-Smart Agriculture, consider the
discourage monoculture, it opens up the opportunity for following statements :
growing a wide variety of grains, fruits and vegetables, 1. The 'Climate-Smart Village' approach in India
and widens one’s food basket. It can be possible in all is a part of a project led by the Climate change,
kinds of climate including semi-arid regions. Hence, Agriculture and Food Security (CCAFS), an
Statement-3 is not correct. international research programme.
IAS (Pre) GS 2021 Paper I 17 YCT
2. The project of CCAFS is carried out under 3. In India, most of the tamarind is collected as
Consultative Group on International minor forest produce.
Agriculatural Research (CGIAR) headquartered 4. India exports tamarind and seeds of moringa.
in France. 5. Seeds of moringa and tamarind can be used in
3. The International Crops Research Institute for the production of biofuels.
the Semi-Arid Tropics (ICRISAT) in India is Which of the statements given above are
one of the CGIAR's research centres. correct?
Which of the statements given above are (a) 1, 2, 4 and 5 (b) 3, 4 and 5
correct? (c) 1, 3 and 4 (d) 1, 2, 3 and 5
(a) 1 and 2 only (b) 2 and 3 only Ans. (b) : Statement 1 is incorrect: Moringa Oleifera
(c) 1 and 3 only (d) 1, 2 and 3 is a medium-sized evergreen tree that is native to Africa
Ans. (d): Statement 1 is correct. The Climate-Smart and Asia. Also known as the Moringa Tree, the
Village project in India is a program of CGIAR Drumstick Tree or the Miracle Tree. This is non-
Research Program on Climate Change, Agriculture and leguminous tree.
Food Security (CCAFS). The CCAFS started piloting Statement 2 is incorrect: Tamarind, (Tamarindus
the Climate-Smart Village in 2012 in Africa (Burkina indica), evergreen tree of the pea family (Fabaceae),
Faso, Ghana, Mali, Niger, Senegal, Kenya, Ethiopia, native to tropical Africa.
Tanzania, and Uganda) and South Asia (Bangladesh, Hence, option (b) is the correct answer.
India, and Nepal).
Statement 2 is correct. Climate Change, Agriculture and CURRENTAFFAIRS
Food Security (CCFAS) is carried out under CGIAR
(formerly the Consultative Group for International 89. With reference to casual workers employed in
Agricultural Research). Headquarter of CGIAR is in India, consider the following statements :
Montpellier, France. CGIAR is a global partnership that 1. All casual workers are entitled for Employees
unites international organizations engaged in research Provident Fund coverage.
about food security. 2. All casual workers are entitled for regular
Statement 3 is correct. International Crops Research working hours and overtime payment.
Institute for the Semi-Arid Tropics (ICRISAT) is a 3. The government can by a notification specify
CGIAR Research Center. ICRISAT is a non-profit, non- that an establishment or industry shall pay
political public international research organization that wages only through its bank account.
conducts agricultural research for development in Asia Which of the above statements are correct?
and sub-Saharan Africa with a wide array of partners (a) 1 and 2 only (b) 2 and 3 only
throughout the world. (c) 1 and 3 only (d) 1, 2 and 3
Hence, option (d) is the correct answer.
Ans. (d) : Statement – 1 (Correct) – According to
87. The vegetation of savannah consists of Employees Provident Funds and Miscellaneous
grassland with scattered small trees, but Provisions Act, An employee cannot differentiate
extensive areas have no trees. The forest between contractual and permanent employees and all
development in such areas is generally kept in causal workers are also entitled to social security
check by one or more or a combination of some benefits under the EPFO Act.
conditions. Which of the following are such Statement – 2 (correct) – The Employees Provident
conditions? Funds Act, 1952, provide Social Security to All Casual
1. Burrowing animals and termites Workers are entitled for regular working hours and over
2. Fire time payment.
3. Grazing herbivores Statement – 3 (Correct) – According to the Payment
4. Seasonal rainfall of Wages Act, 1936, Government to specify the
5. Soil properties industrial or other establishment, by notification in the
Select the correct answer using the code given official Gazette, which enable the employers to pay
below. wages to their employees by crediting to their bank
(a) 1 and 2 (b) 4 and 5 account.
(c) 2, 3 and 4 (d) 1, 3 and 5 Hence, option (d) is the correct answer.
Ans. (c) : Savannas are defined based on vegetation 90. 'R2 Code of Practices' constitutes a tool
structure, the central concept being a discontinuous tree available for promoting the adoption of
cover in a continuous grass under storey. Fire, grazing (a) environmentally responsible practices in
herbivore and seasonal rainfall are behind the scarce electronics recycling industry
forest development in the region. (b) ecological management of 'Wetlands of
Hence, option (c) is the correct answer. International Importance' under the Ramsar
Convention
88. Consider the following statements: (c) sustainable practices in the cultivation of
1. Moringa (drumstick tree) is a leguminous agricultural crops in degraded lands
evergreen tree. (d) 'Environmental Impact Assessment' in the
2. Tamarind tree is endemic to South Asia. exploitation of natural resources
IAS (Pre) GS 2021 Paper I 18 YCT
Ans. (a) : R2 Code of Practices constitutes a tool 93. Consider the following statements :
available for promoting the adoption of environmentally Statement I :
responsible practices in electronics recycling industry. The United Nations Capital Development Fund
Hence, option (a) is the correct answer. (UNCDF) and the Arbor Day Foundation have
91. Consider the following statements : recently recognized Hyderabad as 2020 Tree
st City of the World.
1. 21 February is declared to be the International
Mother Language Day by UNICEF. Statement 2:
2. The demand that Bangla has to be one of the Hyderabad was selected for the recognition for
a year following its commitment to grow and
national languages was raised in the
maintain the urban forests.
Constituent Assembly of Pakistan.
Which one of the following is correct in respect
Which of the above statements is/are correct? of the above statements?
(a) 1 only (b) 2 only (a) Both Statement I and Statement 2 are correct
(c) Both 1 and 2 (d) Neither 1 nor 2 and Statement 2 is the correct explanation for
Ans. (b) 21st February is declared to be the Statement 1
International Mother Language Day by UNESCO not (b) Both Statement I and Statement 2 are correct
UNICEF. So A is wrong. but Statement 2 is not correct explanation for
Member of Constituent Assembly of Pakistan, Statement I
Dhirendranath Datta proposed legislation in the (c) Statement 1 is correct but Statement 2 is not
Constituent Assembly of Pakistan to allow members to correct
speak in Bengali and authorise its use for official (d) Statement 1 is not correct but Statement 2 is
purposes. So B is correct. correct
Therefore, the correct answer is (b). Ans. (d) : Hyderabad is the only city from India to have
been recognised as a 2020 Tree City of the World by
AWARDS the Arbor Day Foundation and the Food and Agriculture
Organization (FAO) for its commitment to growing and
92. Consider the following statements in respect of maintaining urban forests. Statement 1 is not correct
Bharat Ratna and Padma Awards : and statement 2 is correct.
1. Bharat Ratna and Padma Awards are titles Hence, option (d) is the correct answer.
under the Article 18(1) of the Constitution of
India. SPORTS
2. Padma Awards, which were instituted in the
year 1954, were suspended only once. 94. Consider the following statements in respect of
3. The number of Bharat Ratna Awards is the Laureus World Sports Award which was
restricted to a maximum of five in a particular instituted in the year 2000 :
year. 1. American golfer Tiger Woods was the first
winner of this awards.
Which of the above statements are not correct?
2. The award was received mostly by 'Formula
(a) 1 and 2 only (b) 2 and 3 only
One' players so far.
(c) 1 and 3 only (d) 1, 2 and 3
3. Roger Federer received this award maximum
Ans. (d): According to above question instruction number of times compared to others.
Statement (1) is correct - In 1996, the Supreme Court Which of the above statements are correct ?
upheld the constitutional validity of the National (a) 1 and 2 only (b) 2 and 3 only
Awards–Bharat Ratna, Padma Vibhushan, Padma (c) 1 and 3 only (d) 1, 2 and 3
Bhushan and Padma Sri. It ruled that these awards do
not amount to ‘titles’ within the meaning of Article 18 Ans. (c): Statement (1) is correct: Laureus World
that prohibits only hereditary titles of nobility. Sportsman of the year 2000 was given to Tiger Woods.
Therefore, they are not violative of Article 18 as the Statement (2) is incorrect: The award was received
theory of equality does not mandate that merit should mostly by tennis players
not be recognised. Statement (3) is correct: Most awards: Roger Federer
Statement (2) is incorrect: Padma Awards, which were (5)
instituted in the year 1954, is announced every year on Hence, option (c) is the correct answer.
the occasion of Republic Day except for brief 95. Consider the following statements in respect of
interruption(s) during the years 1978 and 1979 and 1993 the 32nd Summer Olympics :
to 1997. 1. The official motto for this Olympics is: 'A
Statement (3) is incorrect: Padma Awards, which were New World'.
instituted in the year 1954, is announced every year on 2. Sport climbing, surfing, skate boarding, Karate
the occasion of Republic Day except for brief and Baseball are included in this Olympic.
interruption(s) during the years 1978 and 1979 and 1993 Which of the above statements is/are correct?
to 1997. (a) 1 only (b) 2 only
Hence, option (d) is the correct answer. (c) Both 1 and 2 (d) Neither 1 or 2
IAS (Pre) GS 2021 Paper I 19 YCT
Ans. (b): Statement (1) is incorrect: The International Ans. (c) : Statement – 1 (correct) – St. Francis Xavier
Olympic Committee decided upon the change with the of the society of Jesus.
new motto being, "Faster, Higher, Stronger - Together." Statement – 2 (Not correct) – St. Francis Xavier died
The specific motto for the Tokyo games remains of a fever on a Chinese island in 1552 at just 46 years
unchanged, "United by Emotion." and was brought back to Goa a few year later.
Statement (2) is correct : Surfing, Skateboarding, Sport Statement -3 (correct) – The feast of St. Francis
Climbing, Karate, Baseball and Softball are included in Xavier is celebrated in Goa every year 3rd December.
this Olympics. Hence, option (c) is the correct answer.
96. Consider the following statements is respect of 99. With reference to the history of ancient India,
the ICC World Test Championship which of the following statements is/are
1. The finalists were decided by the number of correct?
matches they won. 1. Mitakshara was the civil law for upper castes
2. New Zealand was ranked ahead of England and Dayabhaga was the civil law for lower
because it won more matches than England. castes.
Which of the above statements is/are correct ? 2. In the Mitakshara system, the sons can claim
(a) 1 only (b) 2 only rights to the property during the lifetime of the
(c) Both 1 and 2 (d) Neither 1 nor 2 father, whereas in the Dayabhaga system, it is
only after the death of the father that the sons
Ans. (d): Statement 1 is not correct: The ICC
can claim rights to the property.
considered percentage of points earned from the
contested matches in deciding the finalists of the 3. The Mitakshara system deals with the matters
inaugural World Test Championship (WTC), which had related to the property held by male members
been affected by the COVID-19 pandemic. As per the only of a family, whereas the Dayabhaga
current regulations WTC league standings would be system deals with the matters related to the
determined by the percentage of points (PCT) earned by property held by both male and female
teams. PCT is the percentage of points won out of the members of a family.
total number of points contested by each team. Select the correct answer using the code given
Statement 2 is not correct: New Zealand finished in the below.
second spot with 70 percentages of points, including (a) 1 and 2 (b) 2 only
seven wins and four loses, while England was at 4th (c) 1 and 3 (d) 3 only
with 61. Percentage of points, including 11 wins and 7 Ans. (b): Statement – 1- (Nor correct) – Dayabhaga
losses. and Mitakshara is the system of family/civil law,
Hence, option (d) is the correct answer. Dayabhaga and Mitakshara are the basic to the upper
caste alone. Dayabhaga system prevailed in eastern
MISCELLANEOUS India and Mitakshara in remaining parts of the country.
Statement – 2 (Correct) – According to the Dayabhaga
97. What is blue carbon? system, only on the death of the father could the sons
(a) Carbon captured by oceans and coastal claim rights to property and partition the property. By
ecosystems Mitakshara system, the sons could claim this right even
(b) Carbon sequestered in forest biomass and during the life time of the father.
agricultural soils Statement – 3 (Not correct) – Because both the
(c) Carbon contained in petroleum and natural systems deal with the property rights of both of men and
gas women.
(d) Carbon present in atmosphere Hence, option (b) is the correct answer.
Ans. (a) : Blue carbon is the term for carbon captured 100. The term 'ACE2' is talked about in the context
by the world's ocean and coastal ecosystems. of /'ACE2'
Hence, option (a) is correct answer. (a) genes introduced in the genetically modified
98. Consider the following statements : plants
1. St. Francis Xavier was one of the founding (b) development of India's own satellite
members of the Jesuit Order. navigation system
2. St. Francis Xavier died in Goa and a church is (c) radio collars for wildlife tracking
dedicated to him there. (d) spread of viral diseases
3. The Feast of St. Francis Xavier is celebrated in Ans. (d) : The Term 'ACE2' is talked about in the
Goa each year. context of – spread of viral diseases.
Which of the statements given above are On the Surface of human cells is an enzyme called
correct? ACE2, which acts as the receptor that enables SARS-
(a) 1 and 2 only (b) 2 and 3 only CoV2 to launch its attack.
(c) 1 and 3 only (d) 1, 2 and 3 Hence, option (d) is the correct answer.
IAS (Pre) GS 2021 Paper I 20 YCT
UNION PUBLIC SERVICE COMMISSION
Civil Services (Preliminary Exam) - 2020
GENERAL STUDIES : PAPER-I
Time : 2 hours (Exam date : 04.10.2020) Maximum Number : 200

ANCIENT HISTORY The Ten Paramis are:


1. Generosity, 2. Morality, 3. Wisdom, 4. Energy, 5.
• Pre-Historic Period Truthfulness, 6. Determination, 7. Loving-Kindness, 8.
1. With reference to the history of India, consider Renunciation, 9. Patience, 10. Equanimity.
the following pairs : The ten Paramitas of the Eastern and Northern tradition
Famous Place Present State are :
1. Bhilsa – Madhya Pradesh 1. Generosity, 2. Morality, 3. Patience, 4. Energy, 5.
2. Dwarasamudra – Maharashtra Meditation, 6. Wisdom, 7. Skilful means, 8. Resolution,
3. Girinagar – Gujarat 9. Power, 10. Knowledge.
4. Sthanesvara – Uttar Pradesh Despite slight differences in the wording, the two sets of
Which of the pairs given above are correctly qualities are similar.
matched?
(a) 1 and 3 only (b) 1 and 4 only • Buddhism, Jainism, Bhagavata,
(c) 2 and 3 only (d) 2 and 4only Shaiva and other Religion
Ans. (a) : The correct sequence is as follows–
Famous place Present State 3. With reference to the cultural history of India,
1. Bhilsa – Madhya Pradesh consider the following pairs-
2. Dwarasamudra – Karnataka 1. Parivrajaka – Renunciant and Wanderer
3. Girinagar – Gujarat 2. Shramana – Priest with a high status
4. Sthanesvara – Haryana 3. Upasaka – Lay follower of Buddhism
Therefore, of the above option Bhilsa and Girinagar are Which of the pairs given above are correctly
correctly matched. matched?
(a) 1 and 2 only (b) 1 and 3 only
• Vedic Civilization (Buddhism)
(c) 2 and 3 only (d) 1, 2 and 3
2. With reference to the cultural history of India,
which one of the following is the correct Ans. (b): The correct answer is as follows :
description of the term ‘paramitas’? (1) Parivrajaka – Renunciant and Wanderer (A
(a) The earliest Dharmashastra texts written in wandering religious mendicant)
aphoristic (sutra) style
(b) Philosophical schools that did not accept the
(2) Shramana – The Buddhist and Jain monks were
authority of Vedas called Shramans.
(c) Perfections whose attainment led to the (3) Upasaka – Lay followers of Buddhism.
Bodhisattva path Therefore, it is clear from the given option that
(d) Powerful merchant guilds of early medieval
Shramana was not correctly matched.
South India
Ans. (c) : Paramita is commonly known as 'Perfections' 4. With reference to the religious history of India,
(Pararmis) that a pilgrim to achieve in course of consider the following statements:
'Nirvana'. The Fourth Noble Truth, the Noble Eightfold 1. Sthaviravadins belong to Mahayana Buddhism
Path gives what the Buddhist pilgrim (practitioner) has to 2. Lokottaravadin sect was an offshoot of
practice and the path which he has to follow, to achieve Mahasanghika sect of Buddhism
enlightenment and realize Nirvana. A parallel path consists
3. The deification of Buddha by Mahasanghikas
of perfecting certain qualities, which leads the pilgrim to
becoming a Samma Sambuddha, a self enlightened fostered the Mahayana Buddhism
Universal Buddha. The qualities are called the Paramis Which of the statements given above is/are
(perfections) in the Southern traditions and the Paramitas correct?
in the Eastern and Northern traditions. The elements of the (a) 1 and 2 only (b) 2 and 3 only
Noble Path and the Paramis (Qualites) are similar. (c) 3 only (d) 1, 2 and 3

IAS (Pre) GS 2020 Paper I 21 YCT


Ans. (b) : The division of the Buddhist Community in Ashoka mentions his religious tolerance, it talks about
India in the first three centuries following the death of respect for all sects. The given line "whosoever praises
the Buddha in 483 B·C· . The first division occurred as a his religious sect or blames other sects with a view of
result of the Second Buddhist Council, held 100 years glorifying his own sect, he rather injuries his own sect
after the Buddha's death, at Vaishali. This council was very severely", is mentioned on XII Edicts (Ashokan
called to condemn certain practices of some monks Inscription).
which were contrary to the Vinaya or Monk' code of
conduct. When the Acariyavadins (followers of the
• South India
6. Consider the following events in the history of
traditional teaching) split away from the Sthaviravadins
India:
(followers of the way of the elders) and formed their
own school, known as the Mahasanghikas. 1. Rise of Pratiharas under King Bhoja
These sthaviravadins followed a realist line, stating that 2. Establishment of Pallava power under
all phenomena exist and are unstable compounds of Mahendravarman-I
elements. They taught that it is necessary for all humans 3. Establishment of Chola power by Parantaka-I
to strive for Arahantship or release from the constant 4. Pala dynasty founded by Gopala
round of rebirth (Samsara). They taught that Buddhas What is the correct chronological order of
are men-pure and simple, rejecting any notion of their the above events, starting from the earliest
being transcendental. The other group, which were in time?
the majority were known as the Mahasanghikas. Like (a) 2 – 1 – 4 – 3 (b) 3 – 1 – 4 – 2
the Sthaviravadins, they accepted the fundamental (c) 2 – 4 – 1 – 3 (d) 3 – 4 – 1 – 2
doctrines as taught by the Buddha. Such as the four Ans. (c): The correct chronology–
noble truths, but they differed in believing that Buddhas Mahendravarman-I – 600-630 A.D.
are supermundane and transcendental. They also
Gopala – 750-770 A.D.
believed that the original nature of the mind is pure and
that is contaminated when it is stained by passions and Mihir Bhoja – 836-886 A.D.
defilements. It was from the Mahasanghikas that the Parantaka I – 907-955 A.D.
Mahayana was to evolve. Hence statement 1 is not 7. Which of the following phrases defines the
correct. They continue to differ significantly from the nature of the ‘Hundi’ generally referred to in
Sthaviravada in how they understood the nature of the the sources of the Post-Harsha period?
Buddha and they are the first to attribute divinity to him (a) An advisory issued by the king to his
and represent him in the anthropomorphic form in subordinates
statuary, setting a precedent that has continued to the
(b) A diary to be maintained for daily accounts
present day. Hence statement 3 is correct. Further
(c) A bill of exchange
subdivisions of the Mahasanghikas over the next seven
(d) An order from the feudal lord to his
centuries included sect of Lokottarvadins whose
subordinates
philosophy was based on Lokottara Buddha or
supernatural Buddha’s concept Ekavyavaharikas and Ans. (c) : In India, instruments of credit have been in
the Kaukkutikas. Hence statement 2 is correct. use since time immemorial and are popularly known as
‘Hundi’. The movement of goods during the medieval
• Mauryan and Post Mauryan Period period was facilitated by the growth of a financial
5. Who among the following rulers advised his system that permitted easy transfer of money from one
subjects through this inscription? part of the country to another. This was done through
“Whosoever praises his religious sect or blames the use of 'Hundis'. The 'Hundis' was a letter of credit
other sects out of excessive devotion to his own payable after a period of time at a discount.
sect, with the view of glorifying his own sect, he
rather injures his own sect very severely”
• Gupta and Post-Gupta Period
(a) Ashoka (b) Samudragupta 8. With reference to the period of Gupta dynasty
(c) Harshavardhana (d) Krishnadeva Raya in ancient India, the towns Ghantasala, Kadura
and Chaul were well known as–
Ans. (a) : In India, rock inscription was popularised by
Ashoka. The total number of Ashoka’s inscriptions is (a) Ports handling foreign trade
14. James Princep had deciphered an inscription in 1837 (b) Capitals of powerful kingdoms
written in Brahrmi script which referred to a king called (c) Places of exquisite stone art and architecture
'Devanampiya Piyadasi'. The XII Major Rock Edicts of (d) Important Buddhist pilgrimage centres

IAS (Pre) GS 2020 Paper I 22 YCT


Ans. (a) : International trade was in vogue during the Kalidasa was a famous Sanskrit writer and poet in the
Gupta period. Foreign trade was possible through the court of Chandragupta II (Vikramaditya). He was the
ports. The major ports of the Gupta period were author of three famous plays–
Ghantashala, Kadura, Chaul, Male, Tamralipti Sopara 1. Abhijnanasakuntalam, 2. Malavikagnimitram,
Arikamedu Kalyan, Bhrigukachchha etc. Through these 3. Kumarasambhava.
port, the national and international trade were regulated. Amarasimha was one of the nine gems in the court of
9. With reference to the scholars/litterateurs of Vikramaditya of Gupta era. Notably known for his
ancient India, consider the following famous Sanskrit thesaurus Amarakosha. It is also
statements: known as Namalinganushasanam.
1. Panini is associated with Pushyamitra Shunga 10. With reference to the history of India, the
2. Amarasimha is associated with terms “kulyavapa” and “dronavapa” denote–
Harshavardhana (a) measurement of land
(b) coins of different monetary value
3. Kalidasa is associated with Chandra Gupta-II
(c) classifications of urban land
Which of the statements given above is/are
(d) religious rituals
correct?
Ans. (a) : Gupta Economy: The agricultural crops
(a) 1 and 2 only (b) 2 and 3 only
constituted the main resources that the society produced
(c) 3 only (d) 1, 2 and 3
and the major part of the state’s revenues came from
Ans. (c) : Panini (4th century BCE or 6th to 5 th century agriculture. Various types of land are mentioned in the
BCE) was an ancient Sanskrit grammarian and a inscriptions, land under cultivation was usually called
revered scholar in ancient India. Panini likely lived in Kshetra, Khila was the uncultivate land, Aprahata was
the northwest Indian subcontinent during the the jungle or forest land, Gopata Sarah was the pasture
Mahajanapada era. Hence statement 1 is not correct. and Vasti was the habitable land. Different land
He is said to have been born in shalatula of ancient measures were known in different regions such as
Gandhara, a small town at the junction of the Indus and Nivartana, Kulyavapa and Dronavapa are used. In the
Kabul rivers, Pakistan. Panini is known for his text inscription of Bengal, terms like Kulyavapa and
Ashtadhyayi, a Sutra-style treatise in sanskrit grammar, Dronavapa are used.
3959 "verses" or rules on linguistics syntax and 11. With reference to the history of India, consider
semantics with "eight chapters", which is the the following pairs:
foundational text of the Vyakarana branch of the 1. Aurang – In-charge of treasury of
Vedanga. Ashtadhyayi is admired for its simplicity and the state
its rigorous and consistent use of meta language. Sutras 2. Banian – Indian agent of the East
are like mathematical formulas. So a lot of information India Company
can be given using a few words. 3. Mirasidar – Designated revenue payer
Pushyamitra Sunga (185 BC to 151 BC) was the to the State
founder and first ruler of the Shunga Empire in East Which of the pairs given above is/are correctly
India. He was follower of Hinduism. Pushyamitra was matched?
originally a Senapati "General" of the Maurya Empire. (a) 1 and 2 only (b) 2 and 3 only
In 185 BCE he assassinated the last Mauryan Emperor, (c) 3 only (d) 1, 2 and 3
Brihadratha Maurya and proclaimed himself emperor. Ans. (b) : Aurang was a Persian term for a warehouse
Chandragupta II also called Vikramaditya, powerful where goods are collected before being sold, also refers
Emperor (reigned 380 CE to 415 CE) of Northern to a workshop for finised good. Thus, pair (1) is not
correct.
India. During his regin art, architecture and sculpture
Banians (banias) were the agents of the company
flourished and the cultural development of ancient India
merchants by whom the English gentlemen in general
reached its Golden period. He was known for his deep
interest in art and culture and nine gems or Navratna conduct all their business. The banian brought his skill and
adorned his court. his capital to the partnership; and the European contributed
The group (Team of Navratna) comprised of– his privileges. Thus, pair (2) is correct. mirasidar in Urdu
1. Kalidasa, 2. Vetala Bhatta, 3. Varahamihira, means hereditary landowner, co-proprietor. In Persian
4. Varruchi, 5. Amarasimha, 6. Dhanvantari, Miras is an inheritance estate. He was also a revenue payer
7. Kshanpanak, 8. Shanku, 9. Ghatakarpura to the state. Thus pair (3) is correct.

IAS (Pre) GS 2020 Paper I 23 YCT


MEDIEVAL HISTORY limitation in British law and found no previous cases of
such nature in Hindu law. Hence his judgement on the
12. Wellesley established the Fort William College case stated that Rukhmabai had been wed as an
at Calcutta because– innocent child, had no say in the matter and now
(a) he was asked by the Board of Directors at couldn't be forced. Subsequently, after numerous
London to do so hearings, the marriage was affirmed, Rukhmabai wrote
(b) he wanted to revive interest in oriental to Queen Victoria. The Queen overruled the Court's
learning in India verdict and dissolved the marriage. The ripples that the
(c) he wanted to provide William Carey and his case created led to the influence on the passage of the
associates with employment Age of Consent Act, 1891, child marriage illegal across
(d) he wanted to train British civilians for the British Empire.
administrative purpose in India
Ans. (d) : Fort William College was founded by Lord • Social, Cultural Awakening, Lower
Richard Wellesley in Calcutta on the Eastern bank of the Caste, Trade Unions and Peasant
Hooghly River on 10th July, 1800. The British territories
Movement
in India had expanded greatly and there was an urgent
need for trained men who could communicate in the local 14. With reference to the history of India,
languages and understand the country and the people. “Ulgulan” or the Great Tumult is the
The objective to train these officials, the Fort William description of which of the following events?
College was established which aim was to create Civil (a) The Revolt of 1857
Servants who were familiar with Indian languages, (b) The Mappila Rebellion of 1921
history, culture and local laws. Eventually which could (c) The Indigo Revolt of 1859-60
bring in organised administration throughly. (d) Birsa Munda’s Revolt of 1899-1900
Ans. (d) : Munda Rebellion is one of the prominent 19th
MODERN HISTORY
century tribal rebellions in the subcontinent. Birsa
• Disintegration of Mughal Empire & Munda led this movement in the region South of Ranchi
in 1899-1900. The 'Ulgulan' meaning 'Great Tumult'
Advent of European Countries sought to establish Munda Raj and Independence.
13. In the context of Indian history, the
Rakhmabai case of 1884 revolved around– 15. Indigo cultivation in India declined by the
1. women’s right to gain education beginning of the 20th century because of–
2. age of consent (a) peasant resistance to the oppressive conduct
of planters
3. restitution of conjugal rights
(b) its unprofitability in the world market because
Select the correct answer using the code given
of new inventions
below:
(c) national leader’s opposition to the cultivation
(a) 1 and 2 only (b) 2 and 3 only of indigo
(c) 1 and 3 only (d) 1, 2 and 3 (d) Government control over the planters
Ans. (b) : Rukhmabai (1864-1955) was the first Ans. (b) : Because of its high value as a trading
practising female doctor in Colonial India. Rukhmabai commodity, indigo was often referred to as blue gold. It
also made her mark in history due to the legal case she has been the foundation of textile export from India for
was involved in, which contributed to the enactment of centuries. But German chemical dyes invention in the
the age of Consent Act, 1891. She was married off at late 19th century is the cause of decline in Indigo
the young age of eleven, while her husband, Dadaji cultivation. The cultivation of Indigo was at its peak in
Bhikaji, was nineteen years old. He later got in touch the 19th century. It is estimated that in the year 1856,
with her. She refused to go, continued to live with her India produced about 19000 tons of Indigo, while after
stepfather and pursued her education, going against the the Industrial Revolution in 1914, it was reduced to
norms of society. In 1885, after 12 years of marriage, 1100 tons, due to which European Indigo planters of
Bhikaji Sought restitution of conjugal rights. The Champaran were forced to close their factories. Farmers
British precedent could not be implied in this case, as also wanted to get rid of the thinkathia system (3/20)
British laws were meant to be applied in the case of and indigo cultivation. Over time in this background,
consenting adult. Hence, Justice Pinhey found this Gandhiji started the Champaran Satyagraha.
IAS (Pre) GS 2020 Paper I 24 YCT
• Major Institutions, Treaties, Ans. (a) : During the late 18th century and early 19th
century, there was a big change in the technical, social,
Commissions, Acts, Pacts economic and cultural status of Western countries,
16. The Gandhi-Irwin Pact included which of the which is known as Industrial Revolution. The beginning
following? of the industrial revolution began with the
1. Invitation to Congress to participate in the mechanization of the textile industry. From the point of
view of employment, the capital invested and the value
Round Table Conference
of goods produced, the textile industry itself was the
2. Withdrawal of Ordinances promulgated in most important industry in the industrial revolution. The
connection with the Civil Disobedience textile industry itself first used modern methods of
Movement production; due to this technological development, the
3. Acceptance of Gandhiji’s suggestion for market in India was flooded with inexpensive machine-
enquiry into police excesses made goods, which not only adversely affected the
Indian handicrafts industry but they reached the verge
4. Release of only those prisoners who were not
of closure.
charged with violence
18. With reference to the book “Desher Katha”
Select the correct answer using the code given written by Sakharam Ganesh Deuskar during
below: the freedom struggle, consider the following
(a) 1 only (b) 1, 2 and 4 only statements:
(c) 3 only (d) 2, 3 and 4 only 1. It warned against the Colonial State’s
hypnotic conquest of the mind
Ans. (*) : On 5th March, 1931, the Gandhi-Irwin Pact
2. It inspired the performance of swadeshi street
was signed by Gandhiji on behalf of Congress and by
plays and folk songs.
Lord Irwin on behalf of the Government. The pact
3. The use of ‘desh’ by Deuskar was in the
placed the Congress on an equal footing with the specific context of the region of Bengal.
government. The terms of the agreement included Which of the statements given above are correct?
immediate release of all political prisoners not (a) 1 and 2 only (b) 2 and 3 only
convicted in violence, remission of all fines not yet (c) 1 and 3 only (d) 1, 2 and 3
collected, return of all lands not yet sold to third parties; Ans. (a) : Sakharam Ganesh Deuskar made the
lenient treatment to those government servants who has country’s independence the goal of his life and kept
resigned; right to make salt in coastal villages for contact with the revolutionary movement and
personal consumption; right to peaceful and non contributed to the public awakening by composing
aggressive picketing; withdrawal of emergency literature in Bengali and Hindi. In 1904, he wrote a
ordinances. The viceroy, however turned down two of book called "Desher Katha" to address the indigenous
idea of Ranade and Dada Bhai Naoroji. In this book, the
Gandhi's demands,
British state was warmed for their compelling victory
• public inquiry into police excesses through indigenous dramas and folk songs. For him, this
• commutation of Bhagat Singh and his comrades’ book "Desherkatha" played an important role in the
death sentence to life sentence. Indian freedom struggle. Arvind Ghosh wrote that the
Gandhi agreed to on behalf of Congress, first use of the word "Swarajaya" was done by
Sakharam Ganesh Deuskar in "Desher Katha". This
• To suspend the civil disobedience movement. book was translated into the "Desh ki Baat" by Babu
• To participate in Second Round table conference. Rao Vishnu Paradkar.
19. The Vital-Vidhvansak, the first monthly
• Miscellaneous journal to have the untouchable people as its
17. Which of the following statements correctly target audience was published by–
explains the impact of Industrial Revolution on (a) Gopal Baba Walangkar
India during the first half of the nineteenth (b) Jyotiba Phule
century? (c) Mohandas Karamchand Gandhi
(a) Indian handicrafts were ruined (d) Bhimrao Ramji Ambedkar
(b) Machines were introduced in the Indian Ans. (a) : Gopal Baba Walangkar was a social reformer
textile industry in large numbers who did many things for the improvement of
(c) Railway lines were laid in many parts of the untouchables. He worked to release the people from
country socio-economic exploitation. In 1888, he published a
(d) Heavy duties were imposed on the imports of monthly gournal called Vital-Vidhvansak with the aim
British manufactures of making the untouchable society aware.
IAS (Pre) GS 2020 Paper I 25 YCT
INDIAN GEOGRAPHY rules for grant of concessions in respect of extraction of
minor minerals and levy and collection of royalty on
• Physical Structure/Drainage System/ minor minerals. Thus, minor minerals are building
stones, gravel, ordinary clay, ordinary sand other than
Human Geography
sand used for prescribed purposes etc. Therefore,
20. Siachen Glacier is situated to the
chromite, kyanite and sillimanite are major minerals,
(a) East of Aksai Chin (b) East of Leh whereas Bentonite is a minor mineral.
(c) North of Gilgit (d) North of Nubra Valley
23. With reference to chemical fertilizers in India,
Ans. (d) : Siachen Glacier is the largest glacier in the consider the following statements:
Himalayas. The Siachen Glacier is the world's second 1. At present, the retail price of chemical
largest glacier outside the polar region. The length of fertilizers is market-driven and not
this glacier is about 72 km. This glacier lies to the north
administered by the Government.
of the Nubra valley.
2. Ammonia, which is an input of urea, is
• Monsoon, Forest, Soil, Irrigation produced from natural gas.
3. Sulphur, which is a raw material for
Projects, Agriculture
phosphoric acid fertilizer, is a by-product of
21. "The crop is subtropical in nature. A hard
oil refineries.
frost is injurious to it. It requires at least 210
Which of the statement given above is/are
frost-free days and 50 to 100 centimeters of
correct?
rainfall for its growth. A light well-drained soil
(a) 1 only (b) 2 and 3 only
capable of retaining moisture is ideally suited
for the cultivation of the crop." Which one of (c) 2 only (d) 1, 2 and 3
the following is that crop? Ans. (b) : The Government of India subsidizes
(a) Cotton (b) Jute fertilizers to ensure that fertilizers are easily available to
(c) Sugarcane (d) Tea farmers and the country remains self-sufficient in
agriculture. Some has been achieved largely by
Ans. (a) : Cotton is a cash crop. It is a member of
controlling the price of fertilizer and the amount of
Malvaceae group. Cotton crop is of subtropical nature,
production. The urea is being provided to the farmers at
Harsh frost is harmful to this crop. It requires at least
210 frost free-days and 50 to 100 cm of rainfall for its a statutorily notified maximum retail price (MRP).
growth. A light well-drained soil capable of retaining Under the NBS policy, the government announces a
moisture (black soil) is suitable for the cultivation of the fixed rate of subsidy (in Rs. per kg basis), on each
crop. For cotton crops, the temperature should not nutrient of subsidized fertilizers, namely Nitrogen (N),
exceed 30ºC. In the commercial world, cotton is called Phosphate (P), Potash (K) and Sulphur (S) on an annual
white gold. basis. Hence statement 1 is not correct. Synthetic
ammonia (NH3) refers to ammonia that has been
• Minerals, Power Resources and synthesized for Natural gas. Natural gas molecules are
Industry & Trade reduced to carbon and hydrogen. The hydrogen is then
22. Consider the following minerals: purified and reacted with nitrogen to produce ammonia.
1. Bentonite 2. Chromite Approximately 75 percent of the ammonia produced is
3. Kyanite 4. Sillimanite
used as fertilizer, either directly as ammonia or
In India, which of the above is/are officially
designated as major minerals? indirectly after synthesis as urea, ammonium nitrate and
(a) 1 and 2 only (b) 4 only mono-ammonium or diammonium phosphates. Hence
(c) 1 and 3 only (d) 2,3 and 4 only statement 2 is correct.
Ans. (d) : Major minerals are those specified in the first Sulphur is a major by-product of oil refining and gas
schedule appended in the MMDR Act. There is no processing. Most crude oil grades contain some sulphur,
official definition for major minerals in the MMD Act. most of which must be removed during the refining
Hence, whatever is not declared as a "minor mineral" process to meet strict sulphur content limits in refined
may be treated as the major mineral. The Central products. Most of the sulphur produced by refineries is
Government has the power to notify 'minor mineral' sold into the petrochemicals market where it is used to
under Section 3(e) of the MMDR Act, 1957. On the make sulfuric acid. Commercial phosphoric acid is
other hand, as per Section 15 of the MMDR Act, 1957 produced by applying an excess of sulphuric acid to
State Government, have complete powers for making ground rock phosphate. Hence statement 3 is correct.

IAS (Pre) GS 2020 Paper I 26 YCT


WORLD GEOGRAPHY (3) Volga – Caspian Sea
(4) Zambezi – Indian Ocean
• Hydrosphere So, based on the above description, it is clear that the
24. With reference to Ocean Mean Temperature mouth of the Volga and Zambezi rivers are correctly
(OMT), which of the following statements matched.
is/are correct?
1. OMT is measured up to depth of 26ºC • Atmosphere
isotherm which is 129 meters in the south- 26. Consider the following statements:
western Indian Ocean during January-March. 1. Jet streams occur in the Northern Hemisphere
2. OMT collected during January - March can only
be used in assessing whether the amount of 2. Only some cyclones develop an eye.
rainfall in monsoon will be less or more than 3. The temperature inside the eye of a cyclone is
a certain long-term mean. nearly 10ºC lesser than that of the surroundings.
Select the correct answer using the code given Which of the statement given above is/are
below : correct?
(a) 1 only (b) 2 only (a) 1 only (b) 2 and 3 only
(c) Both 1 and 2 (d) Neither 1 nor 2 (c) 2 only (d) 1 and 3 only
Ans. (b) : (i) Sea surface temperature is routinely used Ans. (c) : (i) Jet streams flow in both the hemispheres.
for predicting whether the total amount of rainfall that Hence statement 1 is not correct.
India receives during the monsoon season will be less or (ii) The eye is a region of mostly calm weather at the
more than the long-term mean of 887.5 mm. center of the strong tropical cyclones. It is not
(ii) Now, scientists from Pune's Indian Institute of associated with temperate cyclones. Hence
Tropical Mettorology (IITM), finds that Ocean Mean statement 2 is correct.
Temperature (OMT) has a better ability to predict this (iii) Eye is an area of low pressure and high
than sea surface temperature (SST). Compared to SST temperature. Hence statement 3 is incorrect.
which has a 60% success rate of predicting the Indian
Summer Monsoon, OMT has 80%. INDIAN CONSTITUTION & POLITY
(iii) OMT is analysed by measuring the ocean thermal
energy during the period from January to March. Using • Preamble, Union Territories,
OMT data, scientists are able to predict with 80% Citizenship, Fundamental Rights,
probability the monsoon of any year. Hence statement 2 Fundamental Duties, Directive
is correct.
(iv) OMT is measured upto a depth of 26ºC isotherm
Principles of State Policies
which is seen at a depth varying from 50-100 meters. 27. Which part of the Constitution of India
declares the ideal of Welfare state?
During January-March, the mean 26ºC isotherm depth
in the South Western Indian Ocean is 59 meters. Hence (a) Directive Principles of State Policy
statement 1 is not correct. (b) Fundamental Rights
(c) Preamble
25. Consider the following pairs:
(d) Seventh Schedule
River Flows into
Ans. (a): The welfare state is that concept of governance
1. Mekong – Andaman Sea
so that the citizen of the state can have all-round
2. Thames – Irish Sea development. The welfare state provides social, economic,
3. Volga – Caspian Sea political security to all its citizens which is dedicated to the
4. Zambezi – Indian Ocean public interest, public safety and public upliftment. In the
Which of the pairs given above is/are correctly Indian Constitution, it is described in "Directive principle
matched? of state policy", mentioned in part IV.
(a) 1 and 2 only (b) 3 only 28. Consider the following statements:
(c) 3 and 4 only (d) 1,2 and 4 only 1. The Constitution of India defines its ‘basic
Ans. (c) : The correct matches are structure’ in terms of federalism, secularism,
fundamental rights and democracy.
River - Flows into 2. The Constitution of India provides for
(1) Mekong – South China Sea ‘Judicial review’ to safeguard the citizens’
(2) Thames – North Sea liberties and to preserve the ideals on which
the Constitution is based.
IAS (Pre) GS 2020 Paper I 27 YCT
Which of the statements given above is/are 31. Which one of the following categories of
correct? Fundamental Rights incorporates protection
(a) 1 only (b) 2 only against untouchability as a form of
(c) Both 1 and 2 (d) Neither 1 nor 2 discrimination?
Ans. (d) : The concept of 'Basic Structure' came into (a) Right against Exploitation
existence in the landmark judgment in the Keshwanand (b) Right to Freedom
Bharati Vs state of Kerala case in 24 April, 1973. The (c) Right to Constitutional Remedies
supreme Court has defined the basic structures. Hence, (d) Right to Equality
statement (1) is incorrect. Ans. (d) : Untouchability as a form of discrimination
• The concept of judicial Review is the (under article goes against the notion of equality amongst citizens
13 it is given) basic principle of the constitution of irrespective of caste. Therefore, it has been incorporated
India, Although there is no explicit provision in the under Article 17 as one of the five rights under the
Indian constitution for Judicial Review. It is an Spectrum of right to equality (Articles 14-18).
integral part of our constitution. Hence, statement (2) 32. In India, separation of judiciary from the
is also incorrect.
executive is enjoined by–
29. The Preamble to the Constitution of India is-
(a) the Preamble of the Constitution
(a) a part of the Constitution but has no legal
effect (b) a Directive Principle of State Policy
(b) not a part of the Constitution and has no legal (c) the Seventh Schedule
effect either (d) the conventional practice
(c) a part of the Constitution and has the same Ans. (b) : Article 50 of the Indian Constitution
legal effect as any other part recommends the state to take efforts to separate the
(d) a part of the Constitution but has no legal judiciary from the executive in the public services of the
effect independently of other parts state. The Criminal Procedure Code (1973) has affected
Ans. (d) : In 1960 Berubari's case, the Supreme Court the separation of the Judiciary from the Executive in
held that the Preamble was not a part of the Constitution pursuance of Article 50 under the Directive Principles
and therefore it could never be regarded as a source of
of state policy.
any substantive powers. But in 24 April, 1973.
Kesavananda Bharati's case, the Supreme Court 33. Other than the Fundamental Rights, which of
rejected the above view and held that– the following parts of the Constitution of India
1. The Preamble is a part of the Constitution and can be reflect/reflects the principles and provisions of
amended. the Universal Declaration of Human Rights
2. The Preamble is not a source of power nor a source (1948)?
of limitations or prohibitions. 1. Preamble
3. Preamble has a significant role to play in the 2. Directive Principles of State Policy
interpretation of rule or law, also in the interpretation 3. Fundamental Duties
of provisions of the Constitution. Select the correct answer using the code given
30. With reference to the provisions contained in below:
Part IV of the Constitution of India, which of (a) 1 and 2 only (b) 2 only
the following statements is/are correct? (c) 1 and 3 only (d) 1, 2 and 3
1. They shall be enforceable by courts Ans. (d) : The preamble to the Universal Declaration of
2. They shall not be enforceable by any court Human Rights mentions about the dignity of an
3. The principles laid down in this part are to individual. The preamble of the Indian Constitution
influence the making of laws by the State. speaks about "Equality of status and of opportunity,
Select the correct answer using the code given assuring the dignity of the individual and the unity and
below: integrity". Hence statement 1 is correct.
(a) 1 only (b) 2 only • Article 23 of the Universal Declaration of Human
(c) 1 and 3 only (d) 2 and 3 only Rights mentions the Right to work. A similar concept
Ans. (d) : DPSPs, contained in Part IV (Article 36-51) is in Article 41 of the Indian Constitution under the
of the Constitution of India are not enforceable by any heads of DPSP. Hence statement 2 is correct.
Court. But the principles laid down therein are • Article 29 of the Universal Declaration of Human
considered in the governance and deemed to be basic Rights mentions duties. A similar concept was
guidelines of the country, making it the duty of the state inserted in the Indian Constitution by the 42nd
to follow these principles while making laws to Constitution Amendment Act, 1976 under Part IV-A
establish a just society. Hence statements 2 and 3 are of the Constitution (Article 51A). Hence statement 3
correct and statement 1 is incorrect. is correct.
IAS (Pre) GS 2020 Paper I 28 YCT
• Parliamentary System 36. Rajya Sabha has equal powers with Lok Sabha
in–
34. A Parliamentary System of Government is one
(a) The matter of creating new All India Services
in which–
(b) Amending the Constitution
(a) All political parties in the Parliament are
(c) The removal of the government
represented in the Government
(d) Making cut motions
(b) The Government is responsible to the
Ans. (b) : Rajya Sabha has equal powers with
Parliament and can be removed by it
Loksabha in the amendment of the Constitution bill.
(c) The Government is elected by the people and
Unlike the money bill, the constitutional amendment
can be removed by them bill has to be approved by both the houses of
(d) The Government is chosen by the Parliament parliament with a special majority. There is no
but cannot be removed by it before provision of a Joint Session in case of disagreement.
completion of a fixed term So, Lok Sabha cannot override the decision of the
Ans. (b) : Democratic governance is the system in Rajya Sabha. However, in the matters of formation
which the executive derives its democratic legitimacy and removal of the government, the Rajya Sabha has
through the legislature and is responsible to the unequal status with respect to Lok Sabha. Similarly,
legislature. Parliamentary government is a system in Rajya Sabha has fewer power in matters of finance
which the executive (cabinet) is responsible to the like making cut motions, passing money bills etc., vis-
legislature (parliament) for its functions; hence it is also a-vis Lok Sabha. The Rajya Sabha has been given
called responsible government. In this, the executive some special powers which it enjoys exclusively. This
can remain in his post as long as he has the majority includes the power to authorize the parliament to
(confidence) of the legislature (Parliament). Article 74 create new All India Services common to both the
(council of ministers) and 75 (PM) are related to the center and states (Article 312) by passing a resolution.
parliament system in the center and Article 163 (State Hence the correct answer is (b).
council of ministers) and 164 (CM) are about the 37. Along with the Budget, the Finance Minister
parliamentary system in the state.
also places other documents before the
35. Consider the following statements: parliament which include ‘The Macro
1. The President of India can summon a session Economic Framework statement’. The
of the Parliament at such place as he/she aforesaid document is presented because this is
thinks fit. mandated by
2. The Constitution of India provides for three (a) Long standing parliamentary convention
sessions of the Parliament in a year, but it is (b) Article 112 and Article 110(1) of the
not mandatory to conduct all three sessions Constitution of India
3. There is no minimum number of days that the (c) Article 113 of the Constitution of India
Parliament is required to meet in a year (d) Provisions of the Fiscal Responsibility and
Which of the statements given above is/are Budget Management Act, 2003
correct? Ans. (d) : Fiscal Responsibility and Budget Management
(a) 1 only (b) 2 only (FRBM) became an act in 2003. The objective of the
(c) 1 and 3 only (d) 2 and 3 only Act is to ensure intergenerational equity in fiscal
Ans. (c) : Article 85(1) of the Constitution empowers management, long-run macroeconomic stability, better
co-ordination between fiscal and monetary policy and
the President to summon each House of parliament to
transparency in the fiscal operation of the Government.
meet at such time and place as he thinks fit, but six
FRBM Act provides a legal, institutional framework for
months shall not intervene between its last sitting in one
fiscal consolidation. The Act also requires the
session and the date appointed for its first sitting in the
government to lay before the parliament three policy
next session. In other words, the Parliament should meet
statements each financial year namely.
at least twice a year. Hence statement 1 is correct and
(i) Medium term fiscal policy statement.
statement 2 is not correct.
(ii) Fiscal policy strategy statement and
There is no minimum number of days that parliament is (iii) Macroeconomic framework policy statement.
required to meet in a year. Hence statement 3 is correct. Hence (d) is the correct answer.

IAS (Pre) GS 2020 Paper I 29 YCT


MISCELLANEOUS Ans. (d) : The Constitution lays down only the
following four qualifications for a person to be chosen a
38. One common agreement between Gandhism member of the state legislature.
and Marxism is– 1. He must be a citizen of India.
(a) the final goal of a stateless society 2. He must make and subscribe to an oath or
(b) class struggle affirmation before the person authorised by the
(c) abolition of private property Election Commission for this purpose. In his oath
(d) economic determinism or affirmation, he swears,
Ans. (a) : Mahatma Gandhi and Marx Ideology were (a) To be true faith and allegiance to the Constitution
two opposite streams of thought, where Gandhiji of India.
considered democracy favourable for India and the (b) To uphold the sovereignty and integrity of India.
world and was a supporter of truth and non-violence. 3. He must not be less than 30 years of age in the case
Marxism connects democracy with capitalism; Marx of the legislative council and not less than 25 years
ideology sowed violent seeds in many countries. But of age in the case of the legislative assembly.
both have conceptualized a stateless society as an end Whereas voting right is only 18 years.
goal. Therefore, Gandhism and Marxism are considered 4. He must possess other qualifications prescribed by
anarchists. parliament.
39. In the context of India, which one of the Therefore, statement 1 is not correct.
following is the characteristic appropriate for According to the Representation of People Act, 1951 a
bureaucracy? person convicted of any offence and sentenced to
(a) An agency for widening the scope of imprisonment for not less than 2 years shall be
parliamentary Democracy disqualified from the date of such conviction and shall
(b) An agency for strengthening the structure of continue to be disqualified for a further period of six
federalism years since his release. Hence, statement 2 is not
(c) An agency for facilitating political stability correct.
and economic growth 41. Consider the following statements:
(d) An agency for the implementation of public 1. Aadhar metadata cannot be stored for more
policy than three months
Ans. (d) : The bureaucracy is responsible for carrying 2. State cannot enter into any contract with
out and implementing the policies of the government. private corporations for sharing of Aadhar
They are responsible for the day to day administration data
and are also called the permanent executives. While the 3. Aadhar is mandatory for obtaining insurance
heads of government and their ministers, saddled with products
government policy's overall responsibility, are together 4. Aadhar is mandatory for getting benefits
known as the political executives having a shorter term funded out of the Consolidated Fund of India.
of office (In India it is generally for 5 years). Which of the statements given above is/are
40. Consider the following statements: correct?
1. According to the Constitution of India, a (a) 1 and 4 only (b) 2 and 4 only
person who is eligible to vote can be made a (c) 3 only (d) 1, 2 and 3 only
minister in a State for six months even if Ans. (b) : Aadhaar metadata cannot be stored beyond 6
he/she is not a member of the Legislature of months. According to the Insurance Regulatory and
that State. Development Authority (IRDA), Aadhaar is not
2. According to the Representation of People mandatory for purchasing insurance products. The
Act, 1951, a person convicted of a criminal Supreme Court has clearly said, it is necessary to file
offence and sentenced to imprisonment for income tax and link PAN to Aadhar. According to
five years is permanently disqualified from Section 57 of the Aadhar Act, the state and the
contesting an election even after his release corporate or any person have no right to ask for an
from prison. Aadhar card for identification.
Which of the statements given above is/are 42. With reference to the funds under Members of
correct? Parliament Local Area Development Scheme
(a) 1 only (b) 2 only (MPLADS), which of the following statements
(c) Both 1 and 2 (d) Neither 1 nor 2 are correct?
IAS (Pre) GS 2020 Paper I 30 YCT
1. MPLADS funds must be used to create Ans. (d) : Constitutional government is defined by the
durable assets like physical infrastructure for existence of a Constitution, which may be a legal
health, education, etc. instrument or merely a set of fixed norms generally
2. A specified portion of each MP’s fund must accepted as the fundamental law of the polity, that
benefit SC/ST populations. effectively controls the exercise of political power. The
3. MPLADS funds are sanctioned on yearly essence of constitutionalism is the control of power by
basis and the unused funds cannot be carried its distribution among several state organs or offices in
forward to the next year. such a way that they are each subjected to reciprocal
4. The district authority must inspect at least controls and forced to cooperate, in formulating the will
10% of all works under implementation every of the state. Constitutional government, in essence, is
year. about constitutionalism which is about limited
Select the correct answer using the code government. In many cases, the constitutional
given below: government uses "constitutionally limited government"
(a) 1 and 2 only (b) 3 and 4 only or "limited government".
(c) 1, 2 and 3 only (d) 1, 2 and 4 only
ECONOMICS
Ans. (d) : Statement 1 is correct; members of
parliament local area development schemes is for • Economic Planning and National
development works and the creation of durable
community assets. The emphasis is the creation of
Income/ Budget
durable community assets based on locally felt needs in 44. With reference to the Indian economy after the
the areas of roads, electricity, drinking water, health and 1991 economic liberalization, consider the
education etc. following statements:
Development of areas inhabited by scheduled caste and 1. Worker productivity (` per worker at 2004-05
scheduled tribes, MPs are to recommend every year prices) increased in urban areas while it
work costing at least 15 percent of the MPLADS decreased in rural areas.
entitlement for the year for areas inhabited by schedule 2. The percentage share of rural areas in the
caste population and 7.5 percent for areas inhabited by workforce steadily increased.
S.T. population. In other words, Rs. 75 lakhs for the 3. In rural areas, the growth in non-farm
S.C. population. In case there is insufficient tribal economy increased.
population in the area of Lok Sabha members, they may 4. The growth rate in rural employment
recommend this amount for the creation of community decreased.
assets in tribal areas outside of their constituency but Which of the statements given above is/are
within their state of election. In case a state does not correct?
have S.T. inhabited area this amount may be utilised in (a) 1 and 2 only (b) 3 and 4 only
S.C. Therefore, statement 2 is correct. (c) 3 only (d) 1, 2 and 4 only
MPLADS funds are non-lapsable; funds released to the Ans. (b): Statement 1 is incorrect: Productivity is a
district authority by the Government of India are non- measure of the efficiency with which resources, both
lapsable. Funds left in the district can be carried forward human and material, are converted into goods and
for utilization in the subsequent years. Therefore, services. There is a wide disparity in worker
statement 3 is not correct. productivity between rural and urban areas. India's
Role of the district authority, the district authority's role labour productivity grew by over 14 percent every year.
has been outlined in different paragraphs of the But between the financial years of 2011 and 2015, this
MPLADS guidelines. The district authority would be rate fell to just half of that (7.4%) and continued its
responsible for overall coordination and supervision of declaration to just 3.7 percent between financial years
the works under the scheme at the district level and of 2016 and 2018.
inspect at least 10% of the works under implementation Statement 2 is incorrect: The rate of growth in
every year. Therefore, statement 4 is correct. population and workforce during the last four decades
43. A constitutional government by definition is a– in rural areas have always been lower than of the urban
(a) government by legislature areas.
(b) popular government Statement 3 is correct: Rural non-farm economy in
(c) multi-party government recent times, is considered as an effectual strategy for
(d) limited government decentralisation of economic activities to rural India.
IAS (Pre) GS 2020 Paper I 31 YCT
The economic census of India estimates that around stalks act as mulch. It prevents harmful gases that are
41.89 million rural people are employed in the non- released when stubbles are burnt, provides enhanced
agricultural establishments which registered a growth nutrients and improves moisture holding capacity of the
rate of 4.56% during 1998-2005. soil. The same is also true for rice when it is directly
Statement 4 is correct : As per the census 2011, 68.8 sown without transplantation. As the soil is not tilled,
percent of countries population and 72.4 percent of the remains of previous crops hold carbon and prevent it
workforce resided in rural areas. However, the steady from entering the atmosphere as carbon dioxide. All
transition to urbanisation over the years is leading to the statements are correct.
decline in the rural share in population, workforce and 47. With reference to pulse production in India,
GDP of the country. consider the following statements:
1. Black gram can be cultivated as both kharif
• Agriculture, Industry and Trade
and rabi crop.
45. What are the advantages of fertigation in
2. Green-gram alone accounts for nearly half of
agriculture?
pulse production.
1. Controlling the alkalinity of irrigation water
3. In the last three decades, while the production
is possible.
of kharif pulses has increased, the production
2. Efficient application of Rock Phosphate and
of rabi pulses has decreased.
all other phosphatic fertilizer is possible.
Which of the statement given above is/are
3. Increased availability of nutrients to plants is
correct?
possible.
(a) 1 only (b) 2 and 3 only
4. Reduction in the leaching of chemical
(c) 2 only (d) 1, 2 and 3
nutrients is possible.
Select the correct answer using the code given Ans. (a) : During Kharif, Black Gram is cultivated
below: throughout the country. It is best suited to rice fallows
during rabi in southern and south-eastern parts of India.
(a) 1, 2 and 3 only (b) 1, 2 and 4 only
Blackgram needs relatively heavier soils than green
(c) 1, 3 and 4 only (d) 2, 3 and 4 only
gram. So, statement 1 is correct.
Ans. (c) : Statement 1 is correct: Fertigation can control Gram is the most dominant pulse having a share of
the alkalinity of irrigation water. around 40 percent in the total production, followed by
Statement 2 is incorrect: Efficient application of all Tur/Arhar at 15 to 20 percent and Urad/Black Matpe
phosphatic fertilizers is not possible. and Moong (Green gram) at around 8-10 percent each.
Statement 3 is correct: Increased availability of Madhya Pradesh, Maharashtra, Rajasthan, Uttar Pradesh
nutrients to plants is possible. and Karnataka are the top five pulses producing states.
Statement 4 is correct: Reduction in leaching is The Productivity of pulses is 764 kg/ha. So, statement 2
possible. is not correct.
46. What is/are the advantage/advantages of zero In the last three decades, the production of kharif pulses
tillage in agriculture? first decreased then increased and the production of
1. Sowing of wheat is possible without burning rabi, pulses production has also increased. So, statement
3 is not correct.
the residue of previous crop.
2. Without the need for nursery of rice saplings, 48. With reference to the current trends in the
cultivation of sugarcane in India, consider the
direct planting of paddy seeds in the wet soil
following statements:
is possible. 1. A substantial saving in seed material is made
3. Carbon sequestration in the soil is possible. when 'bud chip settlings' are raised in a
Select the correct answer using the code nursery and transplanted in the main field.
given below: 2. When direct planting of setts is done, the
(a) 1 and 2 only (b) 2 and 3 only germination percentage is better with single-
budded setts as compared to setts with many
(c) 3 only (d) 1, 2 and 3
buds.
Ans. (d) : Zero tillage is the process of agriculture that 3. If bad weather conditions prevail when setts
promotes conservation sustainable agriculture. Here the are directly planted, single-budded setts have
soil is not tilled and left with at least 30% of cover over better survival as compared to large setts.
the soil, which may be a previous crop. When wheat 4. Sugarcane can be cultivated using settling
seeds are sown in soil using a happy seeder, the rice prepared from tissue culture.
IAS (Pre) GS 2020 Paper I 32 YCT
Which of the statements given above is/are 2. When biochar is a part of the growing
correct? medium, it promotes the growth of nitrogen-
(a) 1 and 2 only (b) 3 only fixing microorganisms.
(c) 1 and 4 only (d) 2, 3 and 4 only 3. When biochar is a part of the growing
medium, it enables the growing medium to
Ans. (c) : Sugarcane is one of the important commercial retain water for longer time.
crops of India. It is a tropical and subtropical crop. Which of the statements given above is/are
Sugarcane crop is grown under 21ºC to 27ºC correct?
temperature and annual rainfall of 75 cm to 100 cm with (a) 1 and 2 only (b) 2 only
humid climate is all it required. In the nursery method (c) 1 and 3 only (d) 1, 2 and 3
of sowing sugarcane, the bud chip method can produce
Ans. (d) : Biochar has the potential to produce farm -
on an average 20 plants from a sowed sugarcane ; by
based renewable energy in an eco-friendly way.
this method, about 80 percent of the seed is saved.
Specifically, the quality of biochar depends on several
Sugarcane cultivation can be done with a setting
factors, such as the type of soil, metal and the raw
prepared with tissue culture.
material used for carbonization, the pyrolysis conditions
49. In the context of India, which of the following and the amount of biochar applied to the soil.
is/are considered to be practice(s) of eco- In addition, the biochar amendment to the soil proved to
friendly agriculture? be beneficial to improve enhancing plant growth, since
1. Crop diversification biochar contains organic matter and nutrients, its
2. Legume intensification addition increased soil pH, electric conductivity (EC),
3. Tensiometer use organic carbon (C), total nitrogen (N), available
4. Vertical farming phosphorous (P) and the cation exchange capacity
Select the correct answer using the code given (CEC). Earlier, it reported that the biochar application
below: affected the toxicity, transport of various heavy metals
(a) 1, 2 and 3 only (b) 3 only in the soil due to improved soil absorption capacity. The
(c) 4 only (d) 1, 2, 3 and 4 presence of plant nutrients and ash in the biochar and its
large surface area, porous nature and the ability to act as
Ans. (a) : Crop diversification refers to the addition of
a medium for microorganisms have been identified as
new crops or cropping systems to agricultural
the main reasons for the improvement in soil properties
production on a particular farm, taking into account the
and increase in the absorption of nutrients by plants in
different returns from value-added crops with
soils treated with biochar.
complimentary marketing opportunities. Introducing a
Biochar application decreased the tensile strength of
greater range of vanishes in a particular agro-ecosystem
leads to diversification of agricultural production, which soil cores, indicating that the use of biochar can reduce
can also increase natural biodiversity, strengthening the the risk of soil compaction. A lot has already been
ability of the agro-ecosystem to respond to these discussed on the benefits of inoculation of rhizobacteria
stresses. So statement 1 is correct. in soil, but the addition of biochar can also provide
The discourse on legume intensification is currently more nutrients to the soil, thus benefitting the
framed as sustainable intensification (SI) and commonly agricultural crops. The mixing of the plant growth-
encompasses three dimensions: (1) Increased promoting microorganisms with biochar was referred to
productivity, (2) Maintenance of ecosystem services as the best combination for the growth and yield of
and (3) Increased resilience to shocks. So 2 is correct. french beans. Therefore, all statements are correct.
When the pressure in the tensiometer is determined to 51. In India, which of the following can be
be in equilibrium with the water pressure in the soil, the considered as public investment in agriculture?
tension meter gauge reading represents the matric 1. Fixing Minimum Support Price for
potential of the soil. Such tension meters are used in agricultural produce of all crops.
irrigation scheduling to help farmers and other irrigation 2. Computerization of Primary Agricultural
managers to determine when to water. It is also Credit Societies.
considered an eco-friendly agriculture as it helps control 3. Social Capital development.
the excessive use of water, distorting the quality of the 4. Free electricity-support to farmers.
soil. So, statement 3 is correct. 5. Waiver of agricultural loans by the banking
50. What is the use of biochar in farming? system.
1. Biochar can be used as a part of the growing 6. Setting up of cold storage facilities by the
medium in vertical farming. governments.
IAS (Pre) GS 2020 Paper I 33 YCT
Select the correct answer using the code given 53. Under the Kisan Credit Card scheme, short-
below: term credit support is given to farmers for
(a) 1, 2 and 5 only (b) 1, 3 4 and 5 only which of the following purposes?
1. Working capital for maintenance of farm
(c) 2, 3 and 6 only (d) 1, 2, 3, 4, 5 and 6
assets.
Ans. (c) : Public investment in agriculture constitutes 2. Purchase of combine harvesters, tractors and
investment made by the government to create capital or mini trucks.
social assets to improve agricultural productivity. 3. Consumption requirements of farm
Fixing MSP gives farmers the remunerative price households.
without creating any durables infrastructure. So, 4. Post-harvest expenses.
statement 1 is not correct. 5. Construction of family house and setting up
Computerization Agri Credit Societies improve their of village cold storage facility.
productivity by digitising their functioning. Social Select the correct answer using the code given
below:
capital development like Agri education improves the
(a) 1, 2 and 5 only (b) 1, 3 and 4 only
knowledge base of farmers, paving the way for
(c) 2, 3, 4 and 5 only (d) 1, 2, 3, 4 and 5
improved productivity. So, statement 2 and 3 are
Ans. (b) : The schemes aim at providing adequate and
correct.
timely credit for the comprehensive credit requirements
While free electricity and waiver of Agri loans create of farmers under the single window for their cultivation
government liabilities without creating any durable and other needs as indicated below:
assets on the ground. So, statements 4 and 5 are not • To meet the short term credit requirements for
correct. cultivation of crops.
Cold chain infra also helps reduce post- harvest losses • Post- harvest expenses.
creating durable assets. So, statement 6 is correct. • Produce marketing loan.
52. Which of the following factors/policies were • Consumption requirement of farmer household.
affecting the price of rice in India in the recent • Working capital for maintenance of farm assets,
past? activities allied to agriculture, like dairy animals
inland fishery and also working capital required for
1. Minimum Support Price
floriculture, horticulture etc.
2. Government's trading • Investment credit requirement for agriculture and
3. Government's stockpiling allied activities like pump sets, sprayers, dairy
4. Consumer subsidies animals, floriculture, horticulture etc.
Select the correct answer using the code given • Short term credit requirements of rearing of animals,
below: birds, fish, shrimp, other aquatic organism, capture of
(a) 1, 2 and 4 only (b) 1, 3 and 4 only fish.
(c) 2 and 3 only (d) 1, 2, 3 and 4 54. Consider the following statements:
1. In the case of all cereals, pulses and oil-seeds,
Ans. (d) : By the following ways, the price of rice in the procurement at Minimum Support Price
India gets affected: (MSP) is unlimited in any State/UT of India.
• MSP increases the overall price of rice in the market 2. In the case of cereals and pulses, the MSP is
as the traders have to buy near MSP value otherwise, fixed in any State/UT at a level to which the
farmers have the option to sell to an appointed market price will never rise.
government agency at MSP. Which of the statements given above is/are
correct?
• Government also carries out auctions of the excess
(a) 1 only (b) 2 only
stockpile or intervenes when market prices soar to
(c) Both 1 and 2 (d) Neither 1 nor 2
stabilize them.
Ans. (d) : The procurement of oilseeds, pulses and
• Similarly, stockpiling is done to offload the stock cereals is not unlimited. Under PM-AASHA guidelines,
when market price soar providing cheap rice to limit the procurement to 25% of pulses and oil seeds
consumers and stocking when prices fall ensuring, production at MSP. So, statement 1 is not correct.
remunerative prices to farmers. Market price, by definition, is determined through
• The government also provides subsidised food to the demand and supply forces. So it is difficult to predict
poor at fair price shops. This reduces the overall whether the market price for any crop must be lower or
demand in the open market and prices are affected. higher as compared to MSP. So, statement 2 is also not
So, all the statements are correct. correct. Hence, the correct answer is (d).
IAS (Pre) GS 2020 Paper I 34 YCT
• Money/Banking, Tax System, 57. With reference to the Indian economy, consider
the following statements:
Center- State Relation 1. 'Commercial Paper' is a short-term unsecured
55. What is the importance of the term "Interest
promissory note.
coverage Ratio" of a firm in India?
1. It helps in understanding the present risk of a 2. 'Certificate of Deposit' is a long-term
firm that a bank is going to give loan to. instrument issued by the Reserve Bank of
2. It helps in evaluating the emerging risk of a India to a corporation.
firm that a bank is going to give loan to. 3. 'Call Money' is a short-term finance used for
3. The higher a borrowing firm's level of Interest interbank transactions.
Coverage Ratio, the worse is its ability to 4. 'Zero-Coupon Bonds' are the interest bearing
service its debt. short-term bonds issued by the Scheduled
Select the correct answer using the code given Commercial Banks to corporations.
below:
Which of the statements given above is/are
(a) 1 and 2 only (b) 2 only
(c) 1 and 3 only (d) 1, 2 and 3 correct?
(a) 1 and 2 only (b) 4 only
Ans. (a): One of the significant and most crucial
liquidity ratios is the Interest Coverage Ratio, which (c) 1 and 3 only (d) 2, 3 and 4 only
indicates the level of a company's ability to afford the Ans. (c) : • Commercial paper, in the global finance is
interest that is to be paid by the company for raising an unsecured promissory note with a fixed maturity.
debt. It does not measure the ability to make principal Hence, statement 1 is correct.
payments on the debt; instead it depicts how much the
company can promptly pay the debt’s interest. Regulated by the Reserve Bank of India, a certificate of
deposit is a type of money market instrument issued
• The interest coverage ratio is used to see how well a
firm can pay the interest on outstanding debt. So, against the funds deposited by an investor with a bank
statement 1 is correct. in a dematerialized form for a specific period of time.
• Also called as' interest earned ratio, creditors and CDs can only be issued to individuals, companies, fund
prospective lenders use this ratio to assess the risk of houses and such. Hence, statement 2 is incorrect.
lending capital to a firm. So, statement 2 is correct. • An interbank call money market is a short term money
• A higher coverage ratio is better, although the ideal market that allows for large financial institutions to
ratio may vary by industry. So, statement 3 is not borrow and lend money at interbank rates. Hence,
correct. statement 3 is correct.
56. Consider the following statements: • A zero- coupon bond is a debt security that does not
1. The weightage of food in Consumer Price pay interest but instead trades at a deep discount,
Index (CPI) is higher than that in Wholesale rendering a profit at maturity when the bond is
Price Index (WPI) redeemed for its full face value.
2. The WPI does not capture changes in the 58. In the context of the Indian economy, non-
prices of services, which CPI does. financial debt includes which of the following?
3. Reserve Bank of India has now adopted WPI 1. Housing loans owed by households
as its key measure of inflation and to decide 2. Amounts outstanding on credit cards
on changing the key policy rates. 3. Treasury bills
Which of the statements given above is/are Select the correct answer using the code given
correct? below:
(a) 1 and 2 only (b) 2 only (a) 1 only (b) 1 and 2 only
(c) 3 only (d) 1, 2 and 3 (c) 3 only (d) 1, 2 and 3
Ans. (a) : Statement 1 is correct; the weightage of food Ans. (d) : Non-financial debt consists of credit
in the CPI is close to 50% weightage of food group in instruments issued by governmental entities, households
WPI is 24.38%. and businesses that are not included in the financial
The statement 2 is correct, the WPI inflation does not sector. (The financial sector comprises commercial
capture price change of services but the CPI does. banks, insurance companies, non-banking financial
The statement 3 is incorrect, the Reserve Bank of India companies, co-operatives, pension funds, mutual funds
on 1st April, 2014 adopted the Consumer Price Index and other smaller financial entities). Non-financial debt
(CPI) as the key measure of inflation. Till now, the includes household or commercial loans, treasury bills
Reserve Bank of India (RBI) was using the Wholesale and credit card balances. They share most of the same
Price Index (WPI) to gauge and measure indicative characteristics with financial debt, except the issuers are
inflation projections. non-financial. Hence, all the statements are correct.

IAS (Pre) GS 2020 Paper I 35 YCT


59. If the RBI decides to adopt an expansionist Ans. (b) : District Central Co-operative Banks
monetary policy, which the of the following (DCCBs) are a type of rural co-operatives, along with
would not do? State Co-operative Banks (StCBs) and Primary
1. Cut and optimize the Statutory Liquidity Agricultural Credit Societies (PACs). Short term co-
Ratio operatives are arranged in a three tier structure in most
of the states, with StCBs at the apex level, DCCBs at
2. Increase the Marginal Standing Facility Rate
the intermediate level and PACs at the grassroots level.
3. Cut the Bank Rate and Repo Rate 1. Although the focus of rural cooperative lending in
Select the correct answer using the code given agriculture with the share in credit flow to agriculture
below: of rural cooperative is only 12.1% as compared to
(a) 1 and 2 only (b) 2 only 76% of scheduled commercial banks, and 11.9 of
(c) 1 and 3 only (d) 1, 2 and 3 Regional Rural Banks. Hence statement 1 is
incorrect.
Ans. (b) : Expansionary monetary policy involves
2. DCCBs mobilize deposits from the public and
cutting interest rates or increasing the money supply to
provide credit to the public and PACs. Hence,
boost economic activity. An increase in the SLR
statement 2 is correct.
constricts the ability of the bank to inject money into the
economy, which affects growth. Thus, statement 1 is 61. If you withdraw ` 1,00,000 in cash from your
not correct. Demand Deposit Account at your bank, the
immediate effect on aggregate money supply in
The MSF or Marginal Standing Facility Rate is the rate
the economy will be
at which RBI lends funds overnight to scheduled banks
(a) to reduce it by ` 1,00,000
against government securities. RBI has introduced this
(b) to increase it by ` 1,00,000
borrowing scheme to regulate short term asset- liability
mismatch in a more effective manner. MSF basically (c) to increase it by more than ` 1,00,000
provides a greater liquidity cushion. The higher the (d) to leave it unchanged
MSF rate, the more expensive is borrowing for banks Ans. (d) : If you withdraw an amount of Rs. 1,00,000 in
and corporate borrowers and individuals. It is used by cash from your bank's demand deposit account then the
RBI to control the money supply in the country's overall supply of currency in the economy will remain
financial system. Thus, statement 2 is correct. unchanged.
The Repo Rate is the rate at which the Reserve Bank of Since, M3 = Currency in circulation + Demand and
India lends money to commercial banks in order to help Time deposits in bank.
these lenders meet their short-term liquidity loans. This 62. “Gold Tranche” (Reserve Tranche) refers to
impacts the loans taken by customers, who can also (a) a loan system of the World Bank
avail loans at lower interest rates. This boosts economic (b) one of the operations of a Central Bank
activity. The rate of interest charged by the central bank (c) a credit system granted by WTO to its
on the loans they have extended to commercial banks members
and other financial institutions is called Bank Rate. (d) a credit system granted by IMF to its
When Bank Rate is increased by RBI, bank's borrowing members
costs increased which in return, reduces the supply of Ans. (d) : Gold Tranche refers to the amount of gold
money in the market. Thus, statement 3 is not correct. that each member country of the International Monetary
60. Consider the following statements: Fund (IMF) contributes as part of its membership
1. In terms of short-term credit delivery to the obligations to the fund and can readily borrow when
facing economic difficulties. Hence statement 4 is
agriculture sector, District Central
correct.
Cooperative Banks (DCCBs) deliver more
credit in comparison to Scheduled • Foreign Trade of India, Economic
Commercial Banks and Regional Rural Organizations & Share Market
Banks. 63. Consider the following statements:
2. One of the most important functions of 1. The value of Indo-Sri Lanka trade has
DCCBs is to provide funds to the Primary consistently increased in the last decade.
Agricultural Credit Societies. 2. "Textile and textile articles" constitute an
Which of the statements given above is/are important item of trade between India and
correct? Bangladesh.
(a) 1 only (b) 2 only 3. In the last five years, Nepal has been the
(c) Both 1 and 2 (d) Neither 1 nor 2 largest trading partner of India in South Asia
IAS (Pre) GS 2020 Paper I 36 YCT
Which of the statements given above is/are Select the correct answer using the code given
correct? below:
(a) 1 and 2 only (b) 2 only (a) 1 and 2 only (b) 2 and 4 only
(c) 3 only (d) 1, 2 and 3 (c) 3 only (d) 1, 3 and 4 only
Ans. (b) : The trade between India and Sri Lanka was
Ans. (*) : At present, the following statement is true in
valued at USD 4.5 billion during 2016-17 as compared
to USD 6.05 billion during 2015-16, registering a sharp the context of India's International Trade.
decline of 25.25%. So it is clear that statement 1 is not 1. India's goods export are lower compared to its
correct. imports of goods.
Textile contributed 31.3% of Indian exports to 2. India's imports of iron and steel, chemicals, fertilisers
Bangladesh in 2016-17. The other prominent products and machinery have decreased in recent years.
of exports were engineering products (25% of exports),
3. India's export of services are more than imports of
chemical products (9% of exports) and vegetable
products (Fresh vegetables 8% of exports). All these services.
products accounted for 73% of total export to 4. India is incurring an overall trade/current account
Bangladesh. Textiles and textiles products formed the deficit.
largest component of imports from Bangladesh and Note : This question UPSC has been dropped out
contributed to 60% of total imports. Hence statement 2 because given option would be not appropriate.
is correct.
66. With reference to Trade-Related Investment
As per 2016-17 data, Bangladesh is the largest trading
partner in South Asia, followed by Nepal, Sri Lanka, Measures (TRIMS), which of the following
Pakistan, Bhutan, Afghanistan and Maldives. The level statements is/are correct?
of Indian Exports also follows the same order. Hence 1. Quantitative restrictions on imports by
statement 3 is not correct. foreign investors are prohibited.
64. With reference to Foreign Direct Investment in 2. They apply to investment measures related to
India, which one of the following is considered trade in both goods and services.
its major characteristic? 3. They are not concerned with the regulation of
(a) It is the investment through capital foreign investment.
instruments essentially in a listed company. Select the correct answer using the code given
(b) It is a largely non-debt creating capital flow. below:
(c) It is the investment which involves debt- (a) 1 and 2 only (b) 2 only
servicing.
(c) 1 and 3 only (d) 1, 2 and 3
(d) It is the investment made by foreign
institutional investors in the Government Ans. (c) : The agreement on trade-related investment
securities. (belongs to Uruguay round) measures (TRIMs) recognizes
that certain investment measures can restrict and distort
Ans. (b) : Foreign Direct Investment (FDI) means when
trade. It states that WTO members may not apply any
an investor establishes foreign business operations or
measure that discriminates against foreign products or
acquires foreign business assets, including ownership or
leads to quantitative restrictions, both of which violate
controlling interest in a foreign company. Thus, foreign
direct investment (FDI), being a non-debt capital flow, basic WTO principles. Hence statement 1 is correct.
is a leading source of external finance. Hence statement TRIMs applies only to measure that affect trade in
2 is the correct answer. goods. Hence, statement 2 is incorrect.
65. With reference to the international trade of The agreement is not concerned with the regulation of
India at present, which of the following foreign investment. Hence, statement 3 is correct.
statements is/are correct? 67. If another global financial crisis happens in the
1. India's merchandise exports are less than its near future, which of the following
merchandise imports. actions/policies are most likely to give some
2. India's imports of iron and steel, chemicals, immunity to India?
fertilisers and machinery have decreased in 1. Not depending on short-term foreign
recent years. borrowings
3. India's exports of services are more than its 2. Opening up to more foreign banks
imports of services. 3. Maintaining full capital account convertibility
4. India suffers from an overall trade/current Select the correct answer using the code given
account deficit. below:
IAS (Pre) GS 2020 Paper I 37 YCT
(a) 1 only (b) 1 and 2 only (a) 1 only (b) 2 and 3 only
(c) 3 only (d) 1,2 and 3 (c) 2 only (d) 1, 2, and 3
Ans. (a) :Not depending on short term foreign Ans. (d) : Germline gene therapy targets the
borrowings and focusing on long term plans will give reproductive cells meaning any changes made to the
immunity to the economy. So, statement 1 is correct. DNA will be passed to the next generation.
Consequently, the practice has dramatically divided
• Foreign banks will further increase India's exposure to opinion. Germline gene therapy is when DNA is
foreign money and may, in fact, aggravate the transferred into the cells that produce reproductive cells,
financial crisis. So, statement 2 is incorrect. eggs or sperm in the body. Hence statement 1 is correct.
• India's rupee is a partially convertible currency rupees Although germline gene therapy is illegal, embryonic
can be exchanged at market rates in certain cases, but genetic editing is different from that and it is possible to
approval is required for larger amounts making the make genome changes to eliminate the vertical transfer
(through inheritance) of certain diseases and medical
rupee a fully convertible currency would mean conditions. Hence statement 2 is correct.
increased liquidity in financial markets, improved Human-animal chimaeras are animals bearing human
employment and business opportunities and easy cells and organs. They offer insight into early human
access to capital. Some of the disadvantages include development and disease onset and provide a realistic
higher volatility and increased burden of foreign debt drug testing platform. Researchers have experimented
by injecting several different forms of humans stem
and an effect on the balance of trade and exports. cells into pig embryos to see which cell type would
Hence, statement 3 is incorrect. survive best. The cells that survived longest and showed
the most potential to continue to develop were
SCIENCE intermediate human pluripotent stem cells. Hence
statement 3 is correct.
• Chemistry Zoology
68. Which of the following are the reasons/factors 70. Which of the following statement are correct
regarding the general difference between plant
for exposure to benzene pollution?
and animal cells?
1. Automobile exhaust
1. Plant cells have cellulose cell walls whilst
2. Tobacco smoke animal cells do not.
3. Wood burning 2. Plant cells to do not have plasma membrane
4. Using varnished wooden furniture unlike animal cells which do
5. Using products made of polyurethane 3. Mature plant cell has one large vacuole whilst
Select the correct answer using the code given animal cell has many small vacuoles.
below: Select the correct answer using the code given
(a) 1, 2 and 3 only (b) 2 and 4 only below:
(a) 1 and 2 only (b) 2 and 3 only
(c) 1, 3 and 4 only (d) 1, 2, 3, 4 and 5 (c) 1 and 3 only (d) 1, 2 and 3
Ans. (a) : Benzene is a colourless or pale yellow colour Ans. (c) :
chemical produced from crude oil, gasoline and cigarette Characteristics Plant Cell Animal Cell
smoke. It evaporates and dissolves in the air. In some Cell wall Present Absent
industries, benzene is also used to produce other Shape of cell Distinct edges Round and
chemicals, vehicular pollution, gas stations, industries and shape is either irregular shape
tobacco, wood burning have high exposure to benzene rectangular or
pollution. square shaped
Hence statement 1, 2 and 3 are correct. Nucleus Present, lies Present, lies in
69. Consider the following statements: on one side of the center of
the cell the cell
1. Genetic change can be introduced in the cells
Lysosomes Rarely present Always present
that produce eggs or sperms of a prospective
Plastids Present Absent
parent.
Structure of Single or a Presence of
2. A person's genome can be edited before birth
vacuoles few large numerous and
at the early embryonic stage. vacuole that is smaller
3. Human induced pluripotent stem cells can be centrally vacuoles
injected into the embryo of a pig. located
Which of the statement given above is/are Both(Plant cell & Animal cell) also have plasma
correct? membrane. Therefore, statement 2 is not correct.
IAS (Pre) GS 2020 Paper I 38 YCT
foamed bitumen. Unlike hot mix asphalt (HMA), Cold
• Science & Technology asphalt mix does not require any heating of aggregate
71. Steel slag can be the material for which of the making, it is economical and relatively pollution free
following? (no objectionable fumes or odours). So, statement 2 is
1. Construction of base road correct and 4 is not correct.
2. Improvement of agricultural soil • Geotextile reinforces the soil by adding tensile
3. Production of cement strength to it. It is used as a rapid dewatering layer in
the roadbed. So, statement 3 is correct.
Select the correct answer using the code given
below: • Portland cement production needs energy and can
impact the environment. So, statement 5 is not correct.
(a) 1 and 2 only (b) 2 and 3 only
73. Consider the following statements:
(c) 1 and 3 only (d) 1, 2 and 3
1. Coal ash contains arsenic, lead and mercury.
Ans. (d) : Steel slag or by- product of steel-making 2. Coal-fired power plants release sulphur
industries is produced during the separation of the dioxide and oxides of nitrogen into the
molten steel from impurities in steel making furnaces. environment.
The slag occurs as a molten liquid melt and is a 3. High ash content is observed in Indian coal.
complex solution of silicates and oxides that solidifies Which of the statements given above is/are
upon cooling. It is used in various activities such as in road correct?
construction, these are the preferred aggregates as they (a) 1 only (b) 2 and 3 only
harden well, enabling a thinner layer of pavement than (c) 3 only (d) 1, 2 and 3
what can be achieved with crushed stone aggregates. Slag Ans. (d) : Statement 1 is correct. Coal itself isn't a
is also very hardy, resistant to both weather and wear. So, particularly toxic material. But after it burned, what
remains in the ash includes lead, mercury, cadmium,
statement 1 is correct.
chromium, arsenic and selenium, that may threaten
Benefits of using steel slag for crops: Slag application human health.
favours the increase of pH and the availability of Statement 2 is correct. Thermal power plants produce a
nutrients. Such as Ca, Mg and Si in the soil, which leads large amount of nitrogen oxides and sulphur dioxide.
to the increase in the absorption of these elements by • The pollutants that cause acid rain (due to sulfuric and
the plant, favouring the growth and yield of the crops. nitric acid) are burning fossil fuels, especially coal to
So statement 2 is correct. produce energy.
Steel slag can be used to produce energy- saving cement Statement 3 is correct. India's domestic coal reserves
by co-grinding with OPC clinker and blast furnace slag. have a high ash content - up to 40 to 45 percent.
It can replace Portland cement in various applications 74. In India, why are some nuclear reactors kept
and is especially suitable for projects where a low heat under "IAEA Safeguards" while others are
of hydration is required. So, statement 3 is correct. not?
(a) Some use uranium and others use thorium
72. In rural road construction, the use of which of (b) Some use imported uranium and others use
the following is preferred for ensuring domestic supplies
environmental sustainability or to reduce (c) Some are operated by foreign enterprises and
carbon footprint? others are operated by domestic enterprises
1. Copper slag (d) Some are State-owned and others are
2. Cold mix asphalt technology privately-owned
3. Geotextiles Ans. (b) : The nuclear reactors in India are placed under
4. Hot mix asphalt technology IAEA safeguards only if fuelled by uranium procured
from abroad.
5. Portland cement
At present, 22 operational reactors, of which 14 are
Select the correct answer using the code given
under the International Atomic Energy Agency (IAEA)
below: safeguards as these use imported fuel. India currently
(a) 1, 2 and 3 only (b) 2, 3 and 4 only imports uranium from Russia, Kazakhstan and Canada.
(c) 4 and 5 only (d) 1 and 5 only Plans are also afoot to procure the fuel from Uzbekistan
and Australia. By placing the reactors under the IAEA
Ans. (a) : • The usage of copper slag reduces the usage
safeguards, India gives the international nuclear energy
of primary materials as well as reduces the construction watchdog access to them. This step was taken by the
depth, which in turn reduces energy demand in building. country in 2014 to demonstrate that its nuclear energy
So, statement 1 is correct. programme was for peaceful purposes. It was a
• Cold asphalt mix is produced by mixing unheated necessary step under the Indo-US nuclear deal. Hence
mineral aggregate with either emulsified bitumen or option (b) is the correct answer.
IAS (Pre) GS 2020 Paper I 39 YCT
75. In India, under cyber insurance for individuals, (CNTs) can be functionalised with by active peptides,
which of the following benefits are generally proteins, nucleic acids and drugs. They are also used to
covered, in addition to payment for the loss of deliver their cargos to cells and organs. Therefore,
funds and other benefits? statement 1 is correct.
1. Cost of restoration of the computer system in The other applications of CNTs have been extensively
case of malware disrupting access to one's performed not only for drug and gene therapies but also
computer. for tissue regeneration, biosensor diagnosis, enantiomer
2. Cost of a new computer if some miscreant separation of chiral drugs, extraction and analysis of
willfully damages it, if proved so drugs and pollutants. Hence, statement 3 are true.
3. Cost of hiring a specialized consultant to CNTs are biodegradable. Several types of microbes
minimize the loss in case of cyber extortion including bacteria and fungi can degrade CNTs,
4. Cost of defence in the Court of Law if any graphene and their derivatives. Therefore, statement 4 is
third party files a suit also correct.
Select the correct answer using the code given Hence Statement 1,3 and 4 are correct.
below: 77. Consider the following activities:
(a) 1,2 and 4 only (b) 1,3 and 4 only 1. Spraying pesticides on a crop field
(c) 2 and 3 only (d) 1, 2, 3 and 4 2. Inspecting the craters of active volcanoes
Ans. (b) : Cyber insurance is designed to guard 3. Collecting breath samples from spouting
businesses from the potential effects of cyber attacks. It whales for DNA analysis
helps an organisation mitigate risk exposure by of At the present level of technology, which of the
setting costs, after a cyber attack/breach has happened. above activities can be successfully carried out
To simplify, cyber insurance is designed to cover the by using drones?
fees, expenses and legal costs associated with cyber (a) 1 and 2 only (b) 2 and 3 only
breaches that occur after an organisation has been (c) 1 and 3 only (d) 1, 2 and 3
hacked or from theft or loss of client/employee Ans. (d) : Drones are already well known to spread
information. In India, cyber insurance covers pesticides and to inspect volcanoes remotely. Hence
(generally) the following: statements 1 and 2 are true.
Identify theft, Cyber-bullying and cyber stalking, cyber Since 2015 researchers at the woods hole oceanographic
extortion. Malware Intrusion: Financial loss due to institute in cape cod have begun using a helicopter-
unauthorised and fraudulent use of bank account, credit style drone to monitor humpback whales off the coast,
card and mobile wallets. Legal expenses arising out of collecting breathe samples from their blowholes and
any covered risk, Social media cover, Phishing cover, taking aerial pictures. Getting a breath sample from a
E-mail spoofing, Media liability claims cover, Cyber whale isn't the easiest task in the world. But with the
Extortion cover, Privacy breach and data breach by help of a hexacopter, scientists are getting the job done.
third party. Therefore, statement 3 is also correct.
Hence only option 1, 3 and 4 are correct.
78. "The experiment will employ a trio of
76. With reference to carbon nanotubes, consider spacecraft flying in formation in the shape of
the following statements: an equilateral triangle that has sides one
1. They can be used as carriers of drugs and million kilometers long, with lasers shining
antigens in the human body. between the craft.” The experiment in
2. They can be made into artificial blood questions refers to -
capillaries for an injured part of human body. (a) Voyager-2 (b) New Horizons
3. They can be used in biochemical sensors. (c) LISA Pathfinder (d) Evolved LISA
4. Carbon nanotubes are biodegradable. Ans. (d) : The evolved Laser Interferometer Space
Which of the statements given above are Antenna (eLISA) a mission planned that uses three
correct? separate, but not independent, interferometers installed
(a) 1 and 2 only (b) 2, 3 and 4 only in three spacecraft located at the corners of a triangle
(c) 1, 3 and 4 only (d) 1, 2, 3 and 4 with sides of same 5 million km (3 million miles).
Ans. (c) : Carbon nanotubes (CNTs) are cylindrical This mission is to be carried out jointly by NASA and
molecules that consist of rolled-up sheets of single- ESA. A mission to test the technology of eLISA with
layer carbon atoms (graphene). Carbon nanotubes LISA pathfinder, was launched by ESA in 2015. This is

IAS (Pre) GS 2020 Paper I 40 YCT


stationed around the L1 point between the earth and the 81. In the correct of recent advances in human
sun, which is nearly 1.5 million km away from the reproductive technology, “Pronuclear
earth. The aim of the mission is to demonstrate the Transfer” is used for
technology needed to establish a space-based (a) fertilization of egg in vitro by the donor
observatory to study gravitational waves. Therefore, the sperm
correct answer is (d). (b) genetic modification of sperm producing cells
79. What is the importance of using Pneumococcal (c) development of stem cells into functional
embryos
Conjugate Vaccines in India?
(d) prevention of mitochondrial diseases in
1. These vaccines are effective against
offspring
pneumonia as well as meningitis and sepsis.
Ans. (d) : The mother's egg is first fertilized with the
2. Dependence on antibiotics that are not
father's sperm, producing a zygote in pronuclear transfer.
effective against drug-resistant bacteria can
The pronuclei of the egg and sperm are then removed
be reduced.
from the zygote and inserted into a donor egg that has
3. These vaccines have no side effects and cause been fertilized and has had its own nucleus removed (a
no allergic reactions. pronucleus is the nucleus of the egg or sperm at the stage
Select the correct answer using the code given of fertilization prior to nucleus fusion). The zygote
below: derived from the donor egg is then implanted into the
(a) 1 only (b) 1 and 2 only mother's uterus. It is not fully in vitro.
(c) 3 only (d) 1, 2 and 3 Option (d) is correct. This is a new strategy to prevent
Ans. (b) : Pneumococcus is a leading cause of bacterial the transmission of mitochondrial diseases in offspring.
pneumonia, meningitis and sepsis in children. The 82. With the present state of development,
burden of pneumococcal meningitis is such that it Artificial Intelligence can effectively do which
constitutes about half of all childhood meningitis cases of the following?
in most settings and a greater proportion of meningitis 1. Bring down electricity consumption in
deaths. The vaccine will protect children against severe industrial units
forms of pneumococcal diseases, such as pneumonia, 2. Create meaningful short stories and songs
3. Disease diagnosis
meningitis and sepsis. The vaccine also prevents
4. Text-to-Speech Conversion
antibiotic-resistant pneumococcal infections caused by
5. Wireless transmission of electrical energy
vaccine serotypes. Hence statements 1 and 2 are correct.
Select the correct answer using the code given
However, anyone who has ever had a life threatening below:
allergic reaction to a dose of this vaccine, to an earlier (a) 1, 2, 3 and 5 only (b) 1, 3 and 4 only
pneumococcal vaccine should not get it. Anyone with a (c) 2, 4 and 5 only (d) 1, 2, 3, 4 and 5
severe allergy to any component of it should not get the Ans. (b) : Artificial Intelligence (AI) refers to the
vaccine. With any medicine including vaccines, there is simulation of human intelligence in machines that are
a chance of side effects. These are usually mild and go programmed to think like humans and mimic their
away on their own, but serious reactions are also actions. Artificial intelligence has various applications
possible. Hence, statement 3 is incorrect. in today's society. In multiple industries, such as
healthcare, entertainment, finance, education etc. AI has
80. In India, the term "Public Key Infrastructure"
is used in the context of- been used in the text-to-speech conversion. Hence
(a) Digital security infrastructure statement 1, 3, and 4 are correct.
(b) Food security infrastructure 83. With reference to Visible Light
(c) Health care and education infrastructure Communication (VLC) technology, which of
(d) Telecommunication and transportation the following statements are correct?
infrastructure 1. VLC uses electromagnetic spectrum
Ans. (a): Public key infrastructure (PKI) is a wavelengths 375 to 780 nm
technology for authenticating users and devices in the 2. VLC is known as long-range optical wireless
digital world. The basic idea is to have one or more communication
trusted parties digitally sign documents certifying that a 3. VLC can transmit large amounts of data faster
particular cryptographic key belongs to a particular user than Bluetooth
or device. Hence option (a) is correct. 4. VLC has no electromagnetic interference

IAS (Pre) GS 2020 Paper I 41 YCT


Select the correct answer using the code given The structure of blockchain to eliminate intermediary
below: and enhance authentication such that it supports a wide
(a) 1, 2 and 3 only (b) 1, 2 and 4 only range of applications in health, education, governance,
(c) 1, 3 and 4 only (d) 2, 3 and 4 only diamond industry, financial sector etc. Hence, statement
Ans. (c) : Visible Light Communication (VLC) systems 2 is not correct.
employ visible light for communication that occupies Most of the applications using blockchain technology
the spectrum from 380 nm to 750 nm. The technology do not fulfill the criteria of patent filing in blockchain
uses fluorescent lamps (ordinary lamps, not special technology. Distributed ledger in peer to peer network
communications devices) to transmit signals at 10 kbit/s is used for recording the transaction which does not
or LEDs for upto 500 Mbit/s over short distances. generate much confidence in getting the patent as it is
Visible light communication is an emerging technology, like getting a patent for an algorithm/computer program
which provides distinct facilities such as fast data and as per Indian Patent Act, 1970 Section 3(k) which
communication, secure data communication; high data states that “a mathematical or business method or a
rate wireless communication, instead of radio computer program per se or algorithms are not
frequency, VLC uses light to transfer data. The visible patentable”. As per the ruling of Delhi High Court in
light spectrum is unlicensed and 10,000 times larger the 2014 case of Ericson Vs. Intex, the court stated that
than the range of radio frequencies. It can be used to any invention which has a technical contribution or has
alternate to the existing ratio based wireless a technical effect and is not merely a computer program
communication technologies or in hybrid. Moreover, per se as alleged by the defendant and the same is
integrated VLC resolve the issue of load balancing in patentable. Hence statement 3 is correct.
existing wireless networks. Light fidelity is a recent
technology under VLC, which can be used for wireless ENVIRONMENT AND ECOLOGY
communication as it has many advantages. Such as no
85. With reference to India's Desert National Park,
electromagnetic interference, low cost and high data
which of the following statements are correct?
transfer rate, much higher than traditional bluetooth.
1. It is spread over two districts.
Hence, the correct option is (c).
2. There is no human habitation inside the Park.
84. With reference to “Blockchain Technology”,
3. It is one of the natural habitats of Great
consider the following statements:
Indian Bustard.
1. It is a public ledger that everyone can inspect,
Select the correct answer using the code
but with single user controls
given below:
2. The structure and design of blockchain is
such that all the data in it are about (a) 1 and 2 only (b) 2 and 3 only
cryptocurrency only (c) 1 and 3 only (d) 1,2 and 3
3. Applications that depend on basic features of Ans. (c) : Statement 1 is correct: The Desert National
blockchain can be developed without Park is situated in the Jaisalmer and Barmer districts of
anybody’s permission Indian state of Rajasthan stretched in 3162 km2.
Which of the statements given above is/are Statement 2 is correct: The park's vegetation is revered
correct? and protected by the local communities specially the
(a) 1 only (b) 1 and 2 only Bishnois.
(c) 2 only (d) 1 and 3 only
Statement 3 is correct: The endangered great Indian
Ans. (d) : Blockchain can be defined as a distributed bustard is a magnificent bird found in relatively fair
ledger technology that can record transactions between numbers.
parties securely and permanently. By 'sharing' databases
86. Consider the following statements:
between multiple parties, blockchain essentially removes
1. 36% of India's districts are classified as
the need for intermediaries who were previously required "overexploited" or "critical" by the
to act as trusted third parties to verify records and Central Ground Water Authority
coordinate transactions. It is a foundational technology or a (CGWA).
platform that allows designing a secure way to record 2. CGWA was formed under the
transactions and circulate it among signatories, or any kind Environment (Protection) Act.
of target group with an internet connection. At its core, it is 3. India has the largest area under
an extremely democratic ledger that cannot be arbitrarily groundwater irrigation in the world.
manipulated and is easily shareable. Hence, statement 1 is Which of the statement given above is/are
correct. correct?
IAS (Pre) GS 2020 Paper I 42 YCT
(a) 1 only (b) 2 and 3 only 3. Groundnut seeds
(c) 2 only (d) 1 and 3 only 4. Horse gram
Ans. (b) : Statement 1: Out of the 5723 assessment 5. Rotten potatoes
units assessed jointly by the state groundwater 6. Sugar beet
department and CGWA in the country, as per report of Select the correct answer using the code
Niti Aayog the following is the Data- safe- 71%, semi given below:
critical- 10%, critical- 4% & our exibited- 15% Hence, (a) 1,2, 5 and 6 only (b) 1, 3 4 and 6 only
the statement 1 is incorrect. (c) 2, 3 4 and 5 only (d) 1, 2, 3, 4, 5 and 6
Statement 2: Central Ground Water Authority (GWA) Ans. (a) : The National Policy on Biofuels expands the
constituted under Section 3(3) of 'The Environment scope of raw materials for ethanol production by
(Protection) Act, 1986' regulates extraction of allowing the use of sugarcane juice, sugar containing
groundwater through guidelines that are updated materials like sugar beet, sweet sorghum, starch -
regularly. Hence the statement is correct. containing materials like corn, cassava, damaged food
Statement 3: At 39 million hectares (60% of its total grains like wheat, broken rice, rotten potatoes. Unfit for
irrigation) India has the world's largest groundwater human consumption for ethanol production. Hence, the
well-equipped irrigation system (China with 19 mha is correct is (a).
second, USA with 17 mha is third).
90. Which one of the following statements best
87. If a particular plant species is placed under describes the term 'Social Cost of Carbon'?
Schedule VI of The Wildlife Protection Act, It is a measure, in monetary value, of the
1972, what is the implication? (a) long-term damage done by a tonne of CO2
(a) A licence is required to cultivate that plant. emissions in a given years.
(b) Such a plant cannot be cultivated under any (b) requirement of fossil fuels for a country to
circumstances. provide goods and services to its citizens,
(c) It is a Genetically Modified crop plant. based on the burning of those fuels.
(d) Such a plant is invasive and harmful to the (c) efforts put in by a climate refugee to adapt to
ecosystem live in a new place.
Ans. (a) : The specified endemic plants in Schedule VI (d) contribution of an individual person to the
of the wild life protection act, 1972 are prohibited from carbon footprint on the planet Earth.
cultivation and planting. As such a licence is required to
Ans. (a) : The Social Cost of Carbon (SCC) is the
cultivate that plant. Hence, the correct statement is (a).
marginal cost of the impacts caused by emitting one
88. Among the following Tiger Reserves, which one extra tonne of greenhouse gas (carbon dioxide
has the largest area under "Critical Tiger equivalent) at any point in time, inclusive of non market
Habitat"? impacts on the environment and human health. The
(a) Corbett purpose of putting a price on a ton of emitted CO2 is to
(b) Ranthambore aid policy makers or other legislative in evaluating
(c) Nagarjunsagar-Srisailam whether a policy designed to curb climate change is
(d) Sunderbans justified. Hence, option (a) is correct answer.
Ans. (c) : 91. With reference to Indian elephants, consider
Tiger Reserve Area of the the following statements:
core/Critical Tiger 1. The leader of an elephant group is a female
Habitat (in sq.km.) 2. The maximum gestation period can be 22
Corbett 821.99 months.
Ranthambore 1113.364 3. an elephant can normally go on calving till
Nagarjunsagar Srisailam 2595.72 the age of 40 years only.
Sunderbans 1699.62 4. Among the States in India, the highest
Therefore, the correct answer is (c). elephant population is in Kerala.
89. According to India's National Policy on Which of the statements given above is/are
Biofuels, which of the following can be used as
correct?
raw materials for the production of biofuels?
1. Cassava (a) 1 and 2 only (b) 2 and 4 only
2. Damaged wheat grains (c) 3 only (d) 1,3 and 4 only

IAS (Pre) GS 2020 Paper I 43 YCT


Ans. (a) : Statement 1: The leader of a group is the Ans. (a) : The revival of Barasingha, (It listed in
oldest female. She guides the herd in their search of appendix I of CITES and schedule I of wildlife
food and water sources. These matriarchal units can protection Act of 1972) an endangered and endemic
occasionally divide into smaller, temporary groups. cervidae, belonging to the cervidae family, in the Kanha
Hence, it is correct. Tiger Reserve is hailed as one of the most successful
Statement 2: A breeding pair remains together for about and inspiring conservation projects in the world. While
3 weeks. The gestation period lasts for 22 months, Kanha's name may be synonymous with the tiger, it is
yielding a single baby. During the birth, females of the actually the hard ground barasingha that has
herd surround the mother in order to protect her. Hence, consistently challenged the professionalism, including
it is correct. foresightedness and managerial adaptability of the
Statement 3: Adulthood starts at about 18 years of age Kanha management. Hence, the correct is (a).
and the elephant has an average life expectancy of 70 95. Which of the following are the most likely
years. Although sexually mature in their early teens, places to find the musk deer in its natural
elephants generally only start to mate at about 20 years habitat?
and stop bearing calves at about 50. Hence, it is not 1. Askot Wildlife Sanctuary
correct. 2. Gangotri National Park
Statement 4: South India had the highest number of 3. Kishanpur Wildlife Sanctuary
wild elephants–14,612. Among the South Indian states, 4. Manas National Park
Karnataka leads the table with 6049 elephants, followed Select the correct answer using the code given
by Assam (5719) and Kerala (3054). below:
92. Which of the following Protected Areas are (a) 1 and 2 only (b) 2 and 3 only
located in Cauvery basin? (c) 3 and 4 only (d) 1 and 4 only
1. Nagarhole National Park Ans. (a) : Musk deer is a small compact deer belonging
2. Papikonda National Park to the family Cervidae. A solitary shy animal, the musk
3. Sathyamangalam Tiger Reserve deer, lives in mountainous regions from Siberia to the
4. Wayanad Wildlife Sanctuary Himalayas.
Select the correct answer using the code given Askot Musk Deer Sanctuary is located 54 km from
Pithoragarh near the town of Askot in Uttarakhand. As
below:
the name suggests, the sanctuary has been set up
(a) 1 and 2 only (b) 3 and 4 only
primarily for the conservation of musk deer and its
(c) 1, 3 and 4 only (d) 1, 2, 3 and 4 natural habitat.
Ans. (c) : All except (2) are part of the Cauvery basin. Gangotri National park is a national park in the
Papikonda remains entirely inside the East and West Uttarkashi district of Uttarakhand in India. Various rare
Godavari districts near the Godavari basin after 2014 and endangered species like Bharal or blue sheep, black
and after the construction of Polavaram Dam. bear, brown bear, Himalayan manal, Himalayan
93. With reference to India's biodiversity, Ceylon snowcock, Himalayan Thar, musk deer and snow
frogmouth, Coppersmith barbet, Gray-chinned leopard are found in the park.
minivet and white-throated redstart are The Kishanpur Wildlife Sanctuary is a part of the
(a) Birds (b) Primates Dudhwa Tiger Reserve near Mailani in Uttar Pradesh,
(c) Reptiles (d) Amphibians India. It covers an area of 227 km2 (88 sq.m.) and was
founded in 1972. The tiger, chiral, hog deer, wild boars,
Ans. (a) : The Srilankan frogmouth or Ceylon others and many more animals find themselves a home
frogmouth (Batrachostomus maniliger) is a small here.
frogmouth found in the western ghats of South India Manas National Park or Manas Wildlife Sanctuary is a
and Sri Lanka. It is a bird that reaches 23 m (9.1 in) in national park, UNESCO Natural World Heritage site, a
length. The Coppersmith barbet, is a resident bird in the project tiger reserve, an elephant reserve and a
Indian Sub-continent and parts of South East Asia. The biosphere reserve in Assam, India, located in the
grey chinned minivet is found from the Himalayas to Himalayan foothills; it is contiguous with the Royal
China, Taiwan and Southeast Asia. The white throated Manas National Park in Bhutan.
redstart is found in Nepal, Bhutan, Central China and 96. Consider the following pairs:
far northern areas of Myanmar and North east India. Its International Subject
Agreement/set-up
natural habitat is temperate forest. Hence, option (a) is 1. Alma-Ata – Healthcare of
correct. Declaration the people
94. Which one of the following protected areas is 2. Hague – Biological and
well-known for the conservation of a sub- Convention Chemical
species of the Indian swamp deer (Barasingha) 3. Talanoa – Global climate
that thrives well on hard ground and is Dialogue change
exclusively graminivorous? 4. Under 2 Coalition – Child rights
(a) Kanha National Park Which of the pairs given above is/are correctly
(b) Manas National Park matched?
(c) Mudumalai Wildlife Sanctuary (a) 1 and 2 only (b) 4 only
(d) Tal Chhapar Wildlife Sanctuary (c) 1 and 3 only (d) 2, 3 and 4 only
IAS (Pre) GS 2020 Paper I 44 YCT
Ans. (c) : Statement 1: Alma-Ata Declaration expressed Ans. (a) : The group of twenty (G20) is the premier
the need for urgent action by all governments, all health international forum for global economic cooperation. Its
and development workers and the world community to members are,
protect and promote the health of all people. It was the Argentina, Australia, Brazil, Canada, China, France,
first international declaration underlining the Germany, India, Indonesia, Italy, Japan, Republic of
importance of primary health care. Korea, Mexico, Russia, Saudi Arabia, South Africa,
Statement 2: The Hague convention protects children Turkey, United Kingdom, Units States and the
and their families against the risks of illegal, irregular, European Union. Therefore, the correct answer is (a).
premature or ill-prepared adoptions abroad. 99. The term 'West Texas Intermediate' sometimes
Statement 3: The Talanoa dialogue is a process designed to found in news, refers to a grade of
help countries implement and enhance their nationally (a) Crude oil (b) Bullion
determined contributions by 2020. The parties mandated (c) Rare earth elements (d) Uranium
the dialogue to the United Nations Framework convention Ans. (a) : West Texas Intermediate (WTI) crude oil a
for climate change to take stock of the collective global specific grade of crude oil and one of the main three
efforts to reduce the emissions of greenhouse gases, in line benchmarks in oil pricing, along with Brent and Dubai
with the goals of the Paris Agreement. Crude.
Statement 4: Under 2 coalitions, a global community of WTI is a light sweet crude oil because it contains 0.24%
state and regional governments committed to ambitious sulphur, making it sweet and low density, making it
climate actions in line with the Paris agreement. light. It is the underlying commodity of the New York
Signatories commit to keeping global temperature rises Mercantile Exchanges (NIMEX) oil futures contract and
to will be below 2ºC with effort to reach 1.5ºC is considered a high-quality oil that is easily refined.
Therefore, the correct answer is (a).
MISCELLANEOUS 100. In India, Legal Service Authorities provide free
97. With reference to solar water pumps, consider legal services to which of the following type of
the following statements: citizens?
1. Solar power can be used for running surface 1. Person with an annual income of less than
pumps and not for submersible pumps. `1,00,000.
2. Solar power can be used for running 2. Transgender with and annual income of less
centrifugal pumps and not the ones with than ` 2,00,000.
piston. 3. Member of Other Backward Classes (OBC)
Which of the statements given above is/are with an annual income of less than `
correct? 3,00,000.
(a) 1 only (b) 2 only 4. All Senior Citizens.
(c) Both 1 and 2 (d) Neither 1 nor 2 Select the correct answer using the code given
Ans. (d) : Solar water pumps are a relatively new below:
concept in mechanics. It is commonly seen in (a) 1 and 2 only (b) 3 and 4 only
residential and commercial uses, as well as for irrigation (c) 2 and 3 only (d) 1 and 4 only
of agricultural land. Through solar panels, the pump can Ans. (a) : The persons eligible for getting free legal
eliminate the cost of energy and provide a more feasible services to include:
option that uses energy from the sun (and not fuel • Women and children
burning mechanism) for pumping water. • Member of SC/ST
Classification and types of solar pumps • Industrial workmen
When it comes to standing alone solar pumping system, • Victims of mass disasters, violence, flood, drought,
the main types include rotating and positive earthquake industrial disasters.
displacement pumps. Centrifugal pumps are the • Disable persons
common choice for rotation and are designed for fixed • Persons in custody
head applications. Their output increases in proportion • Persons whose annual income does not exceed Rs. 1
to their speed of rotation. A positive displacement (PD) lakh.
pump moves a fluid by repeatedly enclosing a fixed
• Victims of trafficking in human beings or beggars.
volume and moving it mechanically through the system.
Therefore statement 1 is correct.
The pumping action is cyclic and can be driven by
Senior citizens eligibility for free legal aid depends on
pistons, screws, gears, rollers, diaphragms or vanes.
the rules framed by the respective state governments in
Hence, station 2 is not correct.
this regard. In Delhi, for example, senior citizens are
Additionally, pumps are also classified as submersible
eligible for free legal aid subject to the prescribed
and surface pumps based on their placement
ceiling of annual income. Therefore, all senior citizens
(underwater and above the waterline). Hence, statement
are not eligible for free aid. Therefore, statement 4 is
1 is not correct.
not correct.
98. In which one of the following groups are all the There is free aid to members of SC and ST communities
four countries members of G20? but not OBCs. Hence, statement 3 is not correct.
(a) Argentina, Mexico, South Africa and Turkey There is a provision for free legal aid to transgender
(b) Australia, Canada, Malaysia and New Zealand upto 2 lakhs in Delhi. (But it is yet to be implemented at
(c) Brazil, Iran, Saudi Arabia and Vietnam an all India level). Hence, (a) seems to be the most
(d) Indonesia, Japan Singapore and South Korea appropriate option among the given list.
IAS (Pre) GS 2020 Paper I 45 YCT
UNION PUBLIC SERVICE COMMISSION
Civil Services (Preliminary Exam) - 2019
GENERAL STUDIES : PAPER-I
Time: 2 hours (Exam date : 02.06.2019) Maximum Number: 200
ANCIENT HISTORY Ans: (a) Kanganahalli is located in the Gulbarga district
of Karnataka. An inscription of Emperor Ashoka has
• Indus Valley Civilization been extracted from a maha stupa here on which the
word 'Ranyo Ashoka' (King Ashoka) is engraved in the
1. Which one of the following is not a Harappan site? Brahmi script, as well as a picture of Ashoka engraved
(a) Chanhudaro (b) Kot Diji with his queen and many other women.
(c) Sohgaura (d) Desalpur
Ans : (c) Sohgaura is not a Harappan site. Sohgaura is • Gupta period & Post–Gupta Period
a Mauryan site located on the bank of the Rapti river 4. With reference to forced labor (Vishti) in India
in Gorakhpur, Uttar Pradesh. From here, the first during the Gupta period, which one of the
copper plate inscription attesting to the antiquity of following statements is correct?
Indian writing art is found. The script of this copper (a) It was considered as a source of income for
plate inscription is Brahmi and the language is Prakrit. the State, a sort of tax paid by the people.
It accounts for the construction of three storehouses (b) It was totally absent in the Madhya Pradesh
for the grains for the use of the public during times of and Kathiawar regions of the Gupta Empire.
famine. (c) The forced labourer was entitled to weekly
wages.
• Buddhism, Jainism, Bhagavata and (d) The eldest son of the labourer was sent as the
Shaiva Religion forced labourer.
Ans : (a) The correct statement in the context of forced
2. Consider the following statements : labour (vishti) in India during the Gupta period is that it
1. Deification of the Buddha was considered as a source of income for the state, a
2. Treading the path of Bodhisattvas sort of tax paid by the people. The Gupta period is
3. Image worship and rituals designated as the golden period in the history of India.
Which of the above is/are the feature/features In this period, the major source of income of the state
of Mahayana Buddhism? was various types of taxes. The major taxes prevalent in
(a) 1 only (b) 1 and 2 only
this period were Bhag, Bhoga, Udang, Uparikar,
(c) 2 and 3 only (d) 1, 2 and 3
Shulka, Vishti etc. The Vishti tax was imposed on the
Ans : (d) Mahayana, a Largest Sect. of Buddhism, is merchants and craftsmen as royalty. Under this tax, they
believed to have originated around the first century AD. have to do labour without any pay i.e., began. However,
In Mahayana, devotion and worship is its Soul. The the evidence of forced labour is evident in the Mauryan
effect of this belief led to the inculcation of divinity in era itself. But this practice became very prominent
the Buddha. The construction of statues of Buddha and during the Gupta period.
worship and rituals were incorporated. The preachers of
the Mahayana religion built Buddha idols and stupas for MEDIEVAL HISTORY
prayer all over the world. This devotional spirit
embodied the spirit of Bodhisattva in Buddhism. This • Delhi Sultanate
feeling was endowed with virtue, benevolence,
generosity etc. According to this view, the attainment of 5. Consider the following statements :
enlightenment is the paramount goal. Amoghvarsh, 1. In the revenue administration of Delhi
Nagarjuna and Asanga are prominent Mahayana Sultanate, administration of revenue
thinkers. collection was know as 'Amil'.
2. The lqta system of Sultanate of Delhi was an
• Maurya & Post –Mauryan period ancient indigenous institution.
3. The office of 'Mir Bakshi' came into
3. In Which of the following relief sculpture
existence during the reign of Khalji Sultans
inscriptions is 'Ranyo Ashoka' (King Ashoka) of Delhi.
mentioned along with the stone portrait of Which of the statements given above is/are
Ashoka? correct?
(a) Kanganahalli (b) Sanchi (a) 1 only (b) 1 and 2 only
(c) Shahbazgarhi (d) Sohgaura (c) 3 only (d) 1, 2 and 3
IAS (Pre) GS 2019 Paper I 46 YCT
Ans : (a) In the revenue administration of the Delhi in South India. Akbar was a prominent Mughal ruler
sultanate, the administration of revenue collection was who was born on 15 October, 1542 in Amarkot,
known as "Amil". The Iqta system was a salient feature Pakistan. Kabirdasji was born in 1440 AD in Varanasi.
of the Delhi Sultanate. Its main objective was to He was an Indian mystic poet and saint. He was the
eliminate the feudal system prevalent in India and to founder of the Gyanashri Nirguna branch in the Bhakti
connect remote regions of the empire with the center. It movement era in Hindi literature. Sheikh Ahmad
began during the reign of Iltutmish. The Delhi Sultanate Sirhindi was born in 1564 AD. He was a noted Indian
was divided into several provinces, which were called Sufi scholar of the 16th and 17th centuries. Hence, both
Iqtas, which was governed by Nayak, Wali and Mukti. statements are not correct.
The post of Mir Bakshi was very important in the • Mughal Empire
Mansabdari system of the Mughals. It was only after
8. With reference to Mian Tansen, which one of
signing the letter of Sarkhat by him the army used to get
the following statements is not correct?
every month’s salary. Waqianavis appointed in the (a) Tansen was the title given to him by Emperor
provinces sent messages directly to Mir Bakshi. Akbar.
(b) Tansen composed Dhrupads on Hindu gods
• Bahmani & Vijayanagara Empire and goddesses.
6. Building 'Kalyaana Mandapas' was a notable (c) Tansen composed songs on his patrons.
(d) Tansen invented many Ragas.
feature in the temple construction in the
Ans : (a) The title of Tansen was not conferred by
kingdom of–
Akbar. Historians have two opinions in this regard.
(a) Chalukya (b) Chandela According to the first opinion, the title of Tansen was
(c) Rashtrakuta (d) Vijayanagara conferred by Raja Vikramjit Singh Tomar of Gwalior.
Ans : (d) The creation of the Kalyan Mandap in the According to the second opinion, this title was
Vijayanagara kingdom was a characteristic feature of conferred by Raja Ramchandra Singh of Bandhavgarh.
temple construction. The Vijayanagar Empire has been Tansen was one of the prominent Navratnas in Akbar's
court. He was a great musician. He had studied music
the patron of every genre of art. Significant from Swami Haridas ji. His real name was Ramtanu
development took place during this regime in the field Pandey. He was born in 1506 AD in Gwalior, Madhya
of music, literature and architecture. Despite being Pradesh. Before Akbar's court, he was in the court of
covered under Dravidian art, it can be said that King of Rewa. Pleased with Tansen's talent, Akbar gave
Vijayanagara added many new elements to the him the title of 'Mia'. The famous Dhrupad style of
architecture. A series of subsidiary temples were also singing was started by Tansen and his mentor Swami
Haridas. Tansen was the creator of many ragas such as
linked with the main temples, known as Ammanshirin. Darbari Kanada, Mian ki Todi, Mian ki Malhar and
The pillars of the Vitthal Swamy temple in Mian ki Sarang. He composed two important works of
Vijayanagara had musical qualities; hence they were music called Sangeeta Sara and Ragamala.
also called musical pillars. The construction of one 9. Who among the following Mughal Emperors
thousand pillared pavilions was the ideal form of shifted emphasis from illustrated manuscripts
Vijayanagara temple architecture. to album and individual portrait?
(a) Humayun (b) Akbar
• Bhakti Movement (c) Jahangir (d) Shah Jahan
Ans : (c) The Mughal emperor Jahangir focused more
7. Consider the following statements : on pictorial manuscripts and individual portraits than
1. Saint Nimbarka was a contemporary of focusing on illustrated manuscripts. During Jahangir's
Akbar. reign, there was a lot of uproar in the field of painting.
2. Saint Kabir was greatly influenced by He was a lover of self-portraiture. Under Jahangir,
Shaikh Ahmad Sirhindi. painting acquired greater charm, refinement and dignity.
Which of the statements given above is/are He had a great fascination for nature and took delight in
correct? the portraiture of birds, animals and flowers.
(a) 1 only (b) 2 only 10. With reference to Mughal India, what is/are
(c) Both 1 and 2 (d) Neither 1 nor 2 the difference /differences between Jagirdar
Ans : (d) Saint Nimbark was not a contemporary of and Zamindar?
Akbar. Nimbarkacharya was the promoter of the 1. Jagirdars were holders of land
assignments in lieu of judicial and police
Nimbark Sect. Other names of this sect are the Sanak
duties, whereas Zamindars were holders of
sect, Hans sect, Devarshi sect. He promoted 'Dwaita
revenue rights without obligation to
Dwait'. He was born in 1250 AD at Arunashar near perform any duty other than revenue
Vaidyauryapatan, situated on the coast of the Godavari collection.
IAS (Pre) GS 2019 Paper I 47 YCT
2. Land assignments to Jagirdars were 12. With reference to land reforms in independent
hereditary and revenue rights of India, which one of the following statements is
Zamindars were not hereditary. correct?
Select the correct answer using the code given (a) The ceiling laws were aimed at family
below. holdings and not individual holdings.
(a) 1 only (b) 2 only
(c) Both 1 and 2 (d) Neither 1 nor 2 (b) The major aim of land reforms was
Ans : (d) The jagirdars were mansabdars who were providing agricultural land to all the landless.
given certain revenue areas on the basis of Zamindari (c) It resulted in cultivation of cash crops as a
(estimated income) in exchange for some revenue areas. predominant form of cultivation.
Those were called jagirs. The foundation of Jagirdari (d) Land reforms permitted no exemptions to the
system arose during the reign of Akbar. Jagirs were not ceiling limits.
hereditory. The Jagirdar had only the right to determine Ans : (b) At the time of the independence of the
and collect revenue, while the administrative rights country, ownership of land was concentrated in the
were vested in the Faujdar. In the Mughal period, the hands of only a few people. This increased exploitation
zamindars were landlords who had the hereditary right
of farmers and at the same time, remained a major
to collect land revenue, from villages. For the collection
of land revenue a part of the revenue was given to the obstacle in the direction of socio-economic
landlords as commission. Hence, both statements 1 and development of the rural population. India's land reform
2 are incorrect. programs included the elimination of intermediaries,
tenancy reforms, consolidation of land and fixation of
MODERN HISTORY land per family and main distribution of surplus land
among landless people. Laws of land ownership were
• Social, Cultural Awakening, Low enacted in various states in the 1950 to 60s, which were
Caste, Trade Union & Peasant amended in 1972 on the directions of the Central
Movement Government.
11. With reference to the British colonial rule in
India, consider the following statements : • Major Institutions, Treaties
1. Mahatma Gandhi was instrumental in the Commissions, Acts
abolition of the system of 'indentured 13. With reference to Indian National Movement,
labour'.
Consider the following pairs :
2. In Lord Chelmsford's 'War conference',
Mahatma Gandhi did not support the Person Position held
resolution on recruiting Indians for World 1. Sir Tej bahadur – President, All India
War. Sapru Liberal Federation
3. Consequent upon the breaking of Salt Law 2. K. C. Neogy – Member, The
by Indian people, the Indian National Constituent Assembly
Congress was declared illegal by the 3. P.C. Joshi – General Secretary,
colonial rulers.
Which of the statements given above are Communist Party of
correct? India
(a) 1 and 2 only (b) 1 and 3 only Which of the pairs given above is/are correctly
(c) 2 and 3 only (d) 1, 2 and 3 matched?
Ans : (b) The British took Indians to work in all of their (a) 1 only (b) 1 and 2 only
colonial countries. These people were called labourers (c) 3 only (d) 1, 2 and 3
brought on the agreement. The word agreement later Ans : (d) Sir Tej Bahadur Sapru - President All India
changed to the word 'Girmit' and then Girmitiya. The Liberal Federation.
Girmitiya system was introduced by the British in 1834 K.C. Neogy - Member, The Constituent Assembly
AD and was declared prohibited in 1917 AD. Mahatma P.C. Joshi- General Secretary, Communist Party of India
Gandhi started a campaign in South Africa against this 14. Consider the following statements about 'the
inhuman practice. Charter Act of 1813' :
Lord Chelmsford convened a war conference on 27th 1. It ended the trade monopoly of the East
April, 1918, in which Gandhiji was also present. In this
India Company in India except for trade
conference, the Viceroy called upon Indians to join the
army, which was supported by Mahatma Gandhi. in tea and trade with China.
Hence, Gandhi was also called a recruiting agent. In 2. It asserted the sovereignty of the British
1930 Gandhiji launched the countrywide salt Crown over the Indian territories held by
satyagraha. The political atmosphere in the country the Company.
became unstable, so the British government banned the 3. The revenues of India were now controlled
Congress by declaring it illegal. by the British Parliament.
IAS (Pre) GS 2019 Paper I 48 YCT
Which of the statements given above are correct? Ans : (c) The Swadeshi Movement received a lot of
(a) 1 and 2 only (b) 2 and 3 only strength from the Bang-Bhang movement in the year
(c) 1 and 3 only (d) 1, 2 and 3 1905. It lasted until 1911 and was one of the most
Ans : (a) The East India Company's monopoly of trade successful movements before Gandhiji made his debut
in India was abolished by the Charter Act of 1813, in India. Under this movement, not only was the boycott
although it was granted a waiver of monopoly on trade of foreign clothes but it was decided to boycott all
with China and the trade of tea. The constitutional foreign goods. The major achievement of this
position of the British in India was clarified for the first movement was self-reliance and self-power. This
movement played an important role in reviving the
time by this act. This strengthened the sovereignty of
skills and industries of the indigenous craftsmen and in
the British crown over the Indian territories occupied by
upgrading and promoting them. The National Council
the company. The superintendence and controlling of Education was established on 15 August, 1906 as an
power of the Board of Control were not only defined element of the Swadeshi Movement.
but also expanded by the charter of 1813. The East
India Company was given the right to control the Indian GEOGRAPHY OF INDIA
territories and the revenue for the next 20 years. Whereas
the revenue of India was brought under the control of the • Physical Structure / Drainage
British parliament by the Act of 1853 and not by the System / Human Geography / Tribes
Charter Act of 1813. Hence, statement 3 is wrong. 17. Consider the following statements about
15. Consider the following pairs : Particularly Vulnerable Tribal Groups
Movement/organization Leader (PVTGs) in India :
(a) All India Anti Untouchability league: 1. PVTGs reside in 18 States and one Union
Territory.
Mahatma Gandhi 2. A stagnant or declining population is one
(b) All India Kisan Sabha: of the criteria for determining PVTG
Swami Sahajanand Saraswati status.
(c) Self-Respect Movement : E. V. 3. There are 95 PVTGs officially notified in
Ramaswami Naicker the country so far.
Which of the Pairs given above is/are correctly 4. Irular and Konda Reddi tribes are
matched? included in the list of PVTGs.
(a) 1 only (b) 1 and 2 only Which of the statements given above are
(c) 2 and 3 only (d) 1, 2 and 3 correct?
(a) 1, 2 and 3 (b) 2, 3 and 4
Ans : (d)
(c) 1, 2 and 4 (d) 1, 3 and 4
Movements/ Leader Establishment
Ans : (c) The 75 tribal groups have been classified as
organization Year
Particularly Vulnerable Tribal Groups (PVTGs) by the
All India Anti Mahatma Gandhi 1932
Ministry of Home Affairs, Government of India. PVTGs
Untouchability
are inhabited in the 17 states and one union territory of
League Andaman and Nicobar Islands (census 2011). The
All India Kisan Swami Sahajanand 1936 Ministry of Tribal Affairs implements the plan for the
Sabha Saraswati development of vulnerable Tribal Groups (PVTGs).
Self-respect E.V. Ramaswami 1920 The Irular (Tamil Nadu) and Konda Reddi tribes,
Movement Naicker which inhabit both on the banks of the Godavari river in
Andhra Pradesh and Tamil Nadu and the hilly areas and
• Freedom Struggle & National forests of the East and West Godavari and Khamman
Movement districts, are included in the list of PVTGs.
16. With reference to Swadeshi Movement, 18. Consider the following pairs :
consider the following statements : Glacier River
1. It contributed to the revival of the 1. Bandarpunch : Yamuna
indigenous artisan crafts and industries. 2. Bara Shigri : Chenab
2. The National Council of Education was 3. Milam : Mandakini
established as a part of Swadeshi 4. Siachen : Nubra
Movement. 5. Zemu : Manas
Which of the statements given above is/are Which of the pairs given above are correctly
correct? matched?
(a) 1 only (b) 2 only (a) 1,2 and 4 (b) 1, 3 and 4
(c) Both 1 and 2 (d) Neither 1 nor 2 (c) 2 and 5 (d) 3 and 5
IAS (Pre) GS 2019 Paper I 49 YCT
Ans : (a) In the given pairs 1, 2 and 4 are correctly Ans : (c) According to Indian state of Forest Report
matched while pairs 3 and 5 are incorrectly matched, 2019 published by the Forest Survey of India (FSI), area
because the origin of the Mandakini river is in the wise Madhya Pradesh has the largest forest cover in the
Chorabari Glacier near Kedarnath (Uttarakhand) and country followed by Arunachal Pradesh, Chhattisgarh,
Zemu glacier is the origin of river Teesta. Odisha and Maharashtra. So the correct order is-
So, the correct matches are– 3 < 4 < 1 < 2.
Glacier River 22. Which one of the following groups of plants
Bandarpunch Yamuna was domesticated in the 'New world' and
Bara Shigri Chenab introduced into the 'Old world'?
Milam Gori Ganga (a) Tobacco, cocoa and rubber
Siachen Nubra (b) Tobacco, cotton and rubber
Zemu Teesta (c) Cotton, Coffee and sugarcane
19. Consider the following pairs : (d) Rubber, coffee and wheat
Famous place River Ans : (a) The New World (Americas Region) and the
(1) Pandharpur Chandrabhaga old world refer to Africa, Asia and Europe. Plants that
(2) Tiruchirapalli Cauvery were practiced in the ancient world like sunflower,
(3) Hampi Malaprabha pineapple, papaya, potato, tomato, tobacco, rubber,
cocoa, cotton, etc. In these plants mainly tobacco,
which of the pairs given above are correctly
cocoa, rubber were made arable in the new world.
matched? However, apart from these cotton, maize, etc were also
(a) 1 and 2 only (b) 2 and 3 only practised from the ancient world, but they could not be
(c) 1 and 3 only (d) 1, 2 and 3 made arable on a large scale in the new world.
Ans : (a) 23. Consider the following statements :
Famous places River (1) As per law, the Compensatory Afforestation
1. Pandharpur Chandrabhaga (Maharashtra) Fund Management and Planning Authority
2. Tiruchirappalli Cauvery (Tamil Nadu) exists at both National and State Levels.
3. Hampi Tungabhadra (Karnataka) (2) People's participation is mandatory in the
So, the correct matches from the following pairs are 1 compensatory afforestation programmes
and 2 only. carried out under the Compensatory
Afforestation Fund Act. 2016
• Monsoon, Forest, Soil, Irrigation Which of the statements given above is/are
Projects and Agriculture correct?
(a) 1 only (b) 2 only
20. What is common to the places known as Alyar,
(c) Both 1 and 2 (d) Neither 1 nor 2
Isapur and Kangsabati?
Ans : (a) The compensatory Afforestation Fund Act,
(a) Recently discovered uranium deposits
2016 came into force in the county in the year 2016.
(b) Tropical rain forests The act provides for the setting up of compensatory
(c) Underground cave systems Afforestation fund Management and planning authority
(d) Water reservoirs (CAMPA) a both the central and state levels to ensure
Ans : (d) Aliyar dam is located in Tamil Nadu. This prompt and transparent use of the amounts received in
dam was built during 1959 to 1969. This dam was lieu of the given land for non-forest purposes.
constructed under the Aliyar project. Parambikulam- There is no provision for participation of the people in
Aliyar project is a joint project of Kerala and Tamil the compensatory Afforestation programs run under the
compensatory fund act, 2016.
Nadu.
Isapur Dam is one of the major dams in Maharashtra, • Minerals, Power Resources,
which is built on the Penganga river. The Penganga Industries & Trade
river divides Maharashtra in two regions of Vidarbha
and Marathwada. 24. With reference to the management of minor
minerals in India, consider the following
The Kangsabati project was started in 1956 in West statements :
Bengal as part of India's second Five-year plan to 1. Sand is a 'minor mineral' according to
provide 3,484,77 square kilometers of groundwater in the prevailing law in the country.
Midnapore, Bankura and Hooghly districts. 2. State Governments have the power to
21. Consider the following States : grant mining leases of minor minerals, but
1. Chhattisgarh 2. Madhya Pradesh the powers regarding the formation of
3. Maharashtra 4. Odisha rules related to the grant of minor
With reference to the States mentioned above, minerals lie with the Central Government.
in terms of percentage of forest cover to the 3. State Governments have the power to
total area of State, which one of the following is frame rules to prevent illegal mining of
the correct ascending order? minor minerals.
(a) 2–3–1–4 (b) 2–3–4–1 Which of the statements given above is/are
(c) 3–2–4–1 (d) 3–2–1–4 correct?
IAS (Pre) GS 2019 Paper I 50 YCT
(a) 1 and 3 only (b) 2 and 3 only WORLD GEOGRAPHY
(c) 3 only (d) 1, 2 and 3
Ans : (a) Statement (1) is correct. Sand is a minor • Universe & Solar System
mineral as defined under section 3(e) of the mines and
minerals (development and regulation) Act, 1957 27. On 21st June, the Sun
(MMDR Act.) (a) does not set below the horizon at the Arctic
Statement (2) is not Correct; section 15 of the MMDR Circle
Act empowers the state government to make rules for (b) does not set below the horizon at Antarctic
regulating the grant of mineral concessions in respect of Circle
minor minerals and for purposes connected therewith. (c) shines vertically overhead at noon on the
The regulation of grant of mineral concessions for Equator
minor minerals is therefore within the legislative and
administrative domain of the state governments. (d) shines vertically overhead at the Tropic of
Statement (3) Correct as per section 15 of the MMDR Capricorn
st
Act, 1957, the state Government have been empowered Ans : (a) On 21 June, the sun is directly over tropic of
to frame rules regulating the grant of quarry leases, cancer. Geometrically, it means the sun is normal (90º)
mining leases or other mineral concessions in respect of to the tropic of cancer and makes an angle of 47º to the
minor minerals and for purposes connected therewith. arctic circle. As such, the sun is above the horizon for
Both as a consequence of this and the fact that the states all areas north of Arctic circle. The entire area in the
have been granted powers to frame rules regarding
Arctic circle experiences day. The north pole is inclined
illegal mining in terms of section 23C of MMDR Act,
1957, this is a subject that is entirely under the towards the sun and the places beyond the Arctic circle
legislative and administrative jurisdiction of the states. experience continuous daylight for about six months.
Since a large portion of the Northern Hemisphere is
25. Consider the following satatements : getting light from the sun, it is summer in the regions
1. Coal sector was nationalized by the north of the equator. The longest day and the shortest
Government of India under Indira night of these places occur on 21st June.
Gandhi.
2. Now, coal blocks are allocated on lottery At this time in the Southern Hemisphere all these
basis. conditions are reversed. It is winter season there. The
3. Till recently, India imported coal to meet nights are longer than days. This position of the earth is
the shortages of domestic supply, but now called the summer solstice.
India is self-sufficient in coal production.
Which of the statements given above is/are • World Map
correct? 28. Consider the following pairs :
(a) 1 only (b) 2 and 3 only Sea Bordering country
(c) 3 only (d) 1, 2 and 3 1. Adriatic Sea Albania
Ans : (a) Indira Gandhi nationalised all coal mining
2. Black Sea Croatia
blocks in the early 1970s. Right to extract coal from
demarcated blocks is offered on the basis of an auction, 3. Caspian Sea Kazakhstan
not the lottery, despite having large reserves of coal. 4. Mediterranean Sea Morocco
Indian coal quality is not very good. Also, the supply 5. Red Sea Syria
lines are not very efficient and thus, many power plants Which of the pairs given above are correctly
in India import coal. E.g. Indonesia Supplies thermal matched?
coal to power plants run by Tata's and Adanis. Thus (a) 1, 2 and 4 only (b) 1, 3 and 4 only
India is not self -reliant in coal production. (c) 2 and 5 only (d) 1, 2, 3, 4 and 5
26. Recently, there was a growing awareness in our Ans : (b) Croatia is not bordering the Black Sea.
country about the importance of Himalayan The Countries bordering the Adriatic sea –Italy,
nettle (Girardinia diversifolia) because it is Slovenia, Croatia, Bosnia, Montenegro, Albania,
found to be a sustainable source of Greece.
(a) anti-malarial drug (b) biodiesel Countries bordering the Black Sea- From west
(c) pulp for paper industry (d) textile fibre Bulgaria, Romania, Russia to the north-east and
Ans : (d) In many parts of the world, natural resources Ukraine and Turkey to the south.
are the only sources of livelihood opportunities Countries bordering the Caspian Sea- Russia, Iran,
available to people. Recently, Girardinia diversifolia Azerbaijan, Kazakhstan and Turkmenistan.
(Himalayan nettle), a fibre yielding plant, has become
Countries bordering the Red Sea- Saudi Arabia,
an important livelihood option for people living in the
remote mountainous village of the Hindu Khush Yemen, Egypt, Sudan, Eritrea, Djibouti
Himalaya. There is a community in khar, a hamlet in Countries bordering the Mediterranean Sea-
Darchula district in far-western Nepal, which produces Albania, Algeria, Bosnia, and Herzegnovina, Croatia,
Fabrics from Himalayan nettle. The Fabric and the Cyprus, Egypt, France, Greece, Israel, Italy, Lebanon,
things made from it are sold in local as well as national Libya, Malta, Monaco, Montenegro, Morocco,
and international markets as high- end products. Slovenia, Spain, Syria, Tunisia and Turkey.
IAS (Pre) GS 2019 Paper I 51 YCT
• Atmosphere • Executive of Centre and State,
29. In the context of which of the following do Emergency Provision, Committees/
some scientists suggest the use of cirrus cloud Commissions
thinning technique and the injection of
sulphate aerosol into stratosphere? 32. In India, which of the following review the
(a) Creating the artificial rains in some regions Independent regulators in sectors like
(b) Reducing the frequency and intensity of telecommunications, insurance, electricity, etc?
tropical cyclones 1. Ad Hoc Committees set up by the
(c) Reducing the adverse effects of solar wind on Parliament.
the Earth 2. Parliamentary Department Related
(d) Reducing the global warming Standing Committees
Ans : (d) Thinning cirrus cloud would be achieved by 3. Financial commission
injecting ice nuclei (such as dust) into regions where 4. Financial commission Sector Legislative
cirrus clouds form, making the ice crystals bigger and Reforms Commission
reducing the cirrus optical depth thinning the clouds 5. NITI Aayog
allow more heat to escape into space and thereby cool Select the correct answer using the code
the planet. given below.
Stratospheric sulfate aerosol injection has been (a) 1 and 2 (b) 1, 3 and 4
proposed to counteract anthropogenic greenhouse gas (c) 3, 4 and 5 (d) 2 and 5
warming and prevent regional climate emergencies. It Ans : (a) The parliamentary committees are established
increases the reflectivity of the lower stratosphere to to study and deal with various matter that cannot be
cast a small proportion of the inbound sunlight back directly handled by a legislature due to their volume.
into space and cool the plan. They also monitor the functioning of the executive
branch.
INDIAN CONSTITUTION & POLITY The Parliamentary committees are of two kinds-
standing or permanent committees and Ad-hoc
• Citizenship, Fundamental Rights committees. The former are elected or appointed
Fundamental Duties, Directive periodically and they work on a continuous basis. The
Principles of State Policies latter are created on an ad hoc basis as the need arises
30. In the context of polity, which one of the and they are dissolved after they complete the task
following would you accept as the most assigned to them. They are responsible for reviewing
appropriate definition of liberty? the independent regulators in sectors like
(a) Protection against the tyranny of political rulers telecommunications, insurance electricity, etc.
(b) Absence of restraint 33. Consider the following statements :
(c) Opportunity to do whatever one likes 1. The Parliament (Prevention of
(d) Opportunity to develop oneself fully Disqualification) Act, 1959 exempts several
Ans : (d) The term liberty means the absence of posts from disqualification on the
restraints on the activities of individuals and at the same grounds of 'Office of Profit'
time, providing opportunities for the development of 2. The above-mentioned Act was amended
individual personalities. five times
Liberty, as elaborated in the preamble is very essential 3. The term 'Office of Profit' is well defined
for the successful functioning of the Indian democratic in the Constitution of India
system. However, liberty does not mean 'license' to do Which of the statements given above is/are
what one lies and has to be enjoyed within the correct
limitations mentioned in the constitution itself. In brief,
the liberty conceived by the preamble or fundamental (a) 1 and 2 only (b) 3 only
rights is not absolute but qualified. Hence in the above (c) 2 and 3 only (d) 1, 2 and 3
question, the most appropriate definition of liberty is to Ans : (a) Temporary suspension of disqualification in
provide the opportunity to develop oneself fully. certain cases under the Parliament (Prevention of
31. Which Article of the Constitution of India disqualification) Act, 1959 if a person being a member
safeguards one's right to marry the person of of parliament who immediately before the
one's choice? commencement of this Acts held an office of profit
(a) Article 19 (b) Article 21 declared by any law repealed by this Act not to
(c) Article 25 (d) Article 29 disqualify the holder thereof for being such member,
becomes so disqualified by reason of any of the
Ans : (b) The right to marry a person of one's choice is
provision contained in this Act, such office shall not if
integral to Article 21 (right to life and liberty) of the held by such person for any period not exceeding
constitution. Right to marriage is also stated under beyond a period of six months from the commencement
Human Rights Charter within the meaning of right to of this Act disqualify him for being a member of
marry is a universal right and it is available to everyone Parliament. It has been amended 5 times till date i.e. in
irrespective of their gender. Various courts across the 1993, 1999, 2000, 2006 and 2013.
country have also interpreted right to marry as an integral The office of profit is not defined in the constitution.
post of right to life under Article 21. The SC has Rather its meaning is ascertained through Supreme
reaffirmed this in the recent ruling in the Hadiya case. Court Judgments and Election commission's guidelines.
IAS (Pre) GS 2019 Paper I 52 YCT
34. Which one of the following suggested that the • Judiciary (Central & State)
Governor should be an eminent person from
outside the State and should be a detached figure 36. Consider the following statements :
without intense political links or should not have 1. The motion to impeach a judge of the
taken part in politics in the recent past? Supreme Court of India cannot be rejected
(a) First Administrative Reforms Commission by the Speaker of the lok Sabha as per
(1966) the Judges (Inquiry) Act. 1968
(b) Rajamannar Committee (1969) 2. The Constitution of India defines and
(c) Sarkaria Commission (1983) gives details of what constitutes 'incapacity
(d) National Commission to Review the Working and proved misbehaviour' of the Judges of
of the Constitution (2000) the Supreme Court of India.
Ans : (c) The Sarkaria Commission was formed by the 3. The details of the process of impeachment
Government of India in June 1983 to make its
of the Judges of the Supreme Court of
recommendations on the balance of power related to
center-state relations. It was headed by Justice Rajinder India are given in the Judges (Inquiry) Act.
Singh Sarkaria, retired Judge of the Supreme Court. 1968
This commission made several recommendations with 4. If the motion for the impeachment of a
reference to the Governor, namely Governor should be Judge is taken up for voting, the law
an eminent person and not belong to the state where he requires the motion to be backed by each
is to be posted. House of the Parliament and supported by
I. State chief minister should have a say in the a majority of total membership of that
appointment of governor. House and by not less than two-thirds of
II. Governor should be a detached figure without total members of that House present and
intense political links or should not have taken post voting.
in politics in recent past. Which of the statements given above is/are
III. Governor should not be a member of the ruling correct
party. (a) 1 and 2 (b) 3 only
(c) 3 and 4 only (d) 1, 3 and 4
• Legislature : Council of Ministers Ans : (c) The motion to impeach a judge of the
35. With reference to the Legislative Assembly of Supreme Court of India can be rejected by the speaker
a State in India, consider the following of the Lok Sabha as per the Judges (Inquiry) Act 1968.
statements : The constitution of India does not define and gives
1. The Governor makes a customary address details of the term incapacity and proved misbehaviour
to Members of the House at the of the judges of the Supreme Court of India which are
commencement of the first seasion of the the criteria for the impeachment of the Judges.
year. The details of the process of impeachment of the Judges
2. When a State Legislature does not have a of the Supreme Court of India are given in the Judges
rule on a particular matter it follows the (Inquiry) Act, 1968.
Lok Sabha rule on that matter. The motion for the impeachment of a Judge is taken up
Which of the statements given above is/are for voting the law requires the motion to be backed each
correct? House of the parliament and supported by a majority of
(a) 1 only (b) 2 only total membership of that house and by not less than
(c) Both 1 and 2 (d) Neither 1 nor 2 two-thirds of total members of that house present and
Ans : (c) Under Article 176 (1) of the constitution, At voting.
the commencement of the first session after each 37. With reference to the Constitution of India,
general election to the legislative Assembly and at the consider the following statements :
commencement of the first session of each year, the 1. No High Court shall have the jurisdiction
Governor shall address the Legislative Assembly under to declare any central law to be
Article 208 (1) A House of the Legislature as state may
constitutionally invalid.
make rules for regulating subject to the provisions of
this constitutions its procedure and the conduct of its 2. An amendment to the Constitution of
business. Article 208 also provision that when a state India cannot be called into question by the
legislature does not have a rule on a particular matter it Supreme Court of India.
follows the Lok Sabha rule on that matter. Which of the statements given above is/are
Under Article 208 (2) until rules are made under clause correct?
(1) the rules of procedure and standing orders in force (a) 1 only (b) 2 only
immediately before the commencement of this (c) Both 1 and 2 (d) Neither 1 nor 2
constitution with respect to the legislature for the
corresponding province shall have effect in relation to Ans : (d) High Courts are constitutional courts. They
the legislature of the state subject to such modifications are the principal civil courts of original jurisdiction in
and adaptations as may be made therein by the speaker each state and union territory. 43rd amendment
of the Legislative Assembly, or the chairman of the provision that high courts have the jurisdiction to
legislative council, as the case may be. declare any central law to be constitutionally invalid.
IAS (Pre) GS 2019 Paper I 53 YCT
The Supreme Court is the guardian of the constitution. (a) 1 only (b) 2 only
In this capacity, anything that ultravires the constitution (c) Both 1 and 2 (d) Niether 1 nor 2
is cut down by the highest judiciary, including the
constitutional amendment. The basic structure of the Ans : (b) The 44th amendment of the Indian
constitution derived in the Kesava Nanda Bharti case constitution was significant as it removed partially the
(24 April, 1973) plays an extremely important role in distortions that were introduced into the constitution by
this function. 42nd amendment. But it had no proposal of an Article
38. With reference to the Constitution of India,
Prohibitions or limitations or provisions placing the election of the Prime Minister beyond
contained in ordinary laws cannot act as judicial review.
prohibitions or limitations on the constitutional A five-judge Constitutional bench in the NJAS case, in
powers under Article 142 it could mean which a majority of 4 : 1 rejected the NJAC Act, brought in by
one of the following?
the 99th constitutional amendment by stating it as "
(a) The decisions taken by the Election
Commission of India while discharging its unconstitutional and void". Justice Kehar stated that the
duties cannot be challenged in any court of expectation from the judiciary to safeguard the rights of
law the citizens of this country can only be ensured, by
(b) The Supreme Court of India is not keeping it absolutely insulated and independent from
constrained in the exercise of its powers by
laws made by the Parliament. the other organs of governance and the proposed NJAC
(c) In the event of grave financial crisis in the violated the independence of the Judiciary.
country, the President of India can declare 40. The Ninth Schedule was introduced in the
Financial Emergency without the counsel
from the Cabinet. Constitution of India during the prime
(d) State Legislatures cannot make laws on ministership of
certain matters without the concurrence of (a) Jawaharlal Nehru
Union Legislature. (b) Lal Bahadur Shastri
Ans : (b) Article 142 states that the Supreme Court, in (c) Indira Gandhi
the exercise of its jurisdiction may pass such decree or (d) Morarji Desai
make such order as is necessary for doing complete
justice in any case or matter pending before it. Ans : (a) The first amendment to the Indian constitution
Any decree so passed or orders so made shall be added the Ninth Schedule to it . It was introduced by the
enforceable throughout the territory of India in such Nehru government on 10 May, 1951 to address judicial
manner as may be prescribed by the parliament. decisions and pronouncements, especially about the
Until a provision in that behalf is made, then it would be
enforced in such a manner as the president may by order chapter of fundamental rights. Amendment was made so
prescribe. One of the important instances of application that the land reforms to be carried out by it could not be
of the Supreme Court of Article 142 was in the union challenged in the court. Laws previously included in
carbide case- relating to the victims of the Bhopal gas this schedule could not be judicially reviewed, but on 11
tragedy. In this judgment, the Supreme Court went to
the extent of saying that to do complete justice, it could January, 2007 the Supreme Court ruled that any law
even override the laws made by Parliament by holding included in the 9th schedule could be challenged on the
that, "prohibitions or limitations or provisions contained grounds that it would violate fundamental rights.
in ordinary laws cannot ipso facto, act as prohibitions or
41. Under which Schedule of the Constitution of
limitations on the constitutional powers under Article
142." By this statement the Supreme Court of India India can the transfer of tribal land to private
placed itself above the laws made by parliament or the parties for mining be declared null and void?
legislature of the states. (a) Third Schedule (b) Fifth Schedule
(c) Ninth Schedule (d) Twelfth Schedule
• Panchayati Raj System, Ans : (b) The Indian constitution provides autonomy to
Constitutional Amendments and tribal areas in matters of governance under the fifth and
Schedules sixth schedules, which is further fortified by the
39. Consider the following statements : Samatha V. State of Andhra Pradesh & ors (1997)
1. The 44th Amendment to the Constitution Judgment where the Supreme Court declared that the
of India introduced an Article placing the
transfer of tribal land to private parties for mining was
election of the Prime Minister beyond
null void under the Fifth schedule. The framework for
judicial review.
2. The Supreme Court of India struck down the protection of the rights of tribal and indigenous
the 99th Amendment to the Constitution of people is further strengthened by the Recognition of
India as being violative of the Forest Rights Act, 2006, which protects the individual
independence of judiciary. and community rights of tribal people in forest areas
Which of the statements given above is/are and their right to free and prior informed consent in the
correct? event of their displacement and resettlement.
IAS (Pre) GS 2019 Paper I 54 YCT
ECONOMY Select the correct answer using the code given
below
• Developing & Developed Economy (a) 1 and 2 only (b) 2 only
42. Consider the following statements : (c) 3 only (d) 1, 2 and 3
1. Purchasing Power Parity (PPP) exchange Ans : (a) In the second Five Year Plan (1956-61) an
rates are calculated by comparing the attempt was made to promote such a framework of
prices of the same basket of goods and economic development that the concept of socialist
services in different countries. economy and welfare state in the country could be
2. In terms of PPP dollars. India is the sixth realized. Special emphasis was laid on the development
largest economy in the world. of basic and heavy industrie on the replacement of basic
Which of the statements given above is/are and capital goods industries.
correct?
The Fourth Five-year plan (1969-1974) is also known as
(a) 1 only (b) 2 only
the Gadgil Scheme. The philosophical basis of the
(c) Both 1 and 2 (d) Neither 1 nor 2
fourth plan was the development of stability and
Ans : (a) Purchasing power parity refers to the
attainment of self-sufficiency. In this Five Year Plan,
difference or parity in the purchasing power of currency
between the two countries. Under this, exchange rates the objective of improving the earlier tendency of
are calculated by comparing prices of similar goods and increasing concentration of property and economic
services in different countries. India is the largest power was adopted. So that the goal of equality and
economy in the world, surpassing Japan in terms of social justice can be achieved .
PPP-Dollars. In this context, America and China are in The main objective of the Fifth Five Year Plan (1974-
first and second place. Whereas the Indian economy 79) was to achieve self- sufficiency, eradicate poverty,
ranks fifth in terms of GDP. Hence it is clear that and unemployment, fulfill minimum basic
statement 2 is incorrect. requirements. Whereas, the financial sector was
included first time in the Ninth five- year plan as an
• Poverty & Unemployment integral part.
43. In a given year in India, official poverty lines are
higher in some States than in others because. • Agriculture, Industry & Trade
(a) poverty rates vary from State to State
45. With reference to the cultivation of Kharif
(b) price levels vary from State to State
crops in India in the last five years, consider
(c) Gross State Product varies from State to
State the following statements :
(d) quality of public distribution varies from 1. Area under the cultivation is the highest.
State to State 2. Area under the cultivation of sorghum is
Ans : (b) The official poverty lines in some states in more than that of oilseeds.
India are higher than in other states because the price 3. Area of cotton cultivation is more than
level varies from state to state. The reasons for poverty that of sugarcane.
in India are the following widespread unemployment 4. Area under sugarcane cultivation has
and underemployment, uneven distribution of income steadily decreased.
and wealth, low level of capital formation in India, Which of the statements given above are
rapidly increasing population, etc. correct?
(a) 1 and 3 only (b) 2, 3 and 4 only
• Economic Planning & National (c) 2 and 4 only (d) 1, 2 3 and 4
Income/Budget Ans : (a) According to the Economic Survey 2017-18,
44. With reference to India's Five-year Plans, which the area under paddy cultivation is 43.2 million hectares
of the following statements is/are correct? at the All-India level ,which is the highest in
1. From the Second Five-Year Plan, there agriculture. 26.2 million hectares of area is used under
was a determined thrust towards oilseeds, while 5.1 million hectares area under sorghum
substitution of basic and capital good cultivation. The area under cotton cultivation is 108
industries. million hectares and the sugarcane cultivation area is
2. the Fourth Five-Year Plan adopted the 4.4 million hectares. There has been a decrease of
objective of correcting the earlier trend of 10.9% in the cultivation of sugarcane in 2017-18.
increased concentration of wealth and 46. Among the following, which one is the largest
economic power. exporter of rice in the world in the last five
3. In the Fifth Five-year Plan, for the first years?
time, the financial sector was included as (a) China (b) India
an integral part of the Plan. (c) Myanmar (a) Vietnam
IAS (Pre) GS 2019 Paper I 55 YCT
Ans : (b) India reached the first position in 2011-12, messages or payment instructions. By the end of the
surpassing Thailand in the export of rice; since then calendar year, it is mandatory for all providers to submit
India has remained on the top among the rice exporting system audit reports which have been audited by CERT-
IN experts. By 15 October, 2018, 6 months’ time were
countries. In recent years (2014-18), India alone
given to follow the guidelines.
accounted for 25-26 percent of the world's rice exports.
49. The money multiplier in an economy increases
Whereas in Thailand, it has been 22 to 25 percent. with which one of the following?
The highest dollar value of rice exports during 2018 is (a) Increase in the cash reserve ratio
in the following table- (b) Increase in the banking habit of the
population
Country Price in USD Total Rice Export in% (c) Increase in the statutory liquidity ratio
India 7.4 billion 30.1% (d) Increase in the population of the country
Thailand 5.6 billion 22.7% Ans : (b) Money multiplier is the ratio of the stock of
money to the stock of high powered money. It is the
Vietnam 2.2 billion 9% relationship between the monetary base and the money
supply of an economy. It explains the increase in the
• Money, Banking & Tax System amount of cash in circulation generated by the bank’s
ability to lend money out of their depositor’s funds.
Central-State Financial Relations Therefore, it refers to how an initial deposit can lead to
47. Which one of the following is not the most a bigger final increase in the total money supply.
likely measure the Government RBI takes to Therefore, an increase in the banking habits of the
stop the slide of Indian rupee? population would lead to more deposits and hence an
increase in the money multiplier.
(a) Curbing imports of non-essential goods and
promoting exports 50. The Service Area Approach was implemented
(b) Encouraging Indian borrowers to issue rupee under the purview of
(a) Integrated Rural Development Programme
denominated Masala Bonds
(b) Lead Bank Scheme
(c) Easing conditions relating to external (c) Mahatama Gandhi National Rural
commercial borrowing Employment Guarantee Scheme
(d) Following an expansionary monetary policy (d) National Skill Development Mission
Ans : (d) To curb the depreciation of the Indian rupee, Ans : (b) The Service Area Approach (SAA)
measures such as control of import of non-essential introduced in April 1989, in order to bring about an
commodities and promotion of exports, incentives for orderly and planned development of rural and semi–
Indian borrowers to issue Masala bonds of rupee urban areas of the country, was extended to all Indian
denomination. Easing conditions related to foreign Scheduled Commercial Banks including Regional Rural
commercial borrowing may prove effective. The
expansionary monetary policy is not followed by the Banks (RRBs).
Reserve Bank as it may cause the rupee to depreciate Service Area Approach is an Alternative and improved
rather than rise. method of lead bank scheme for development of bank
48. Consider the following statements : credit for rural development under SAA, each bank
The Reserve Bank of India's recent directives branch in rural and semi – Urban area was designated
relating to 'Storage of Payment System Data', to serve an area of 15 to 25 villages and the branch was
popularly know as data diktat, command the responsible for meeting the needs of bank credit of its
payment system providers that- service Area.
1. they shall ensure that entire data relating
51. Consider the following statements :
to payment systems operated by them are
stored in a system only in India. 1. Most of India's external debt is owed by
2. they shall ensure that the systems are governmental entities.
owned and operated by public sector 2. All of India's external debt is
enterprises. denominated in US dollars.
3. they shall submit the consolidated system Which of the statements given above is/are
audit report to the Comptroller and
Auditor General of India by the end of the correct?
calendar year. (a) 1 only (b) 2 only
Which of the statements given above is/are (c) Both 1 and 2 (d) Neither 1 nor 2
correct? Ans : (d) India's external debt was 543 billion dollars
(a) 1 only (b) 1 and 2 only by the end of 2019 of which government debt is 103.8
(c) 3 only (d) 1, 2 and 3 billion dollar and non-government debts are 392 billion
Ans : (a) Guidelines regarding the storage of data of dollar. Hence, most of the foreign debt is due to non-
payment system have been issued by the Reserve Bank.
According to this guidelines, it has been asked to ensure governmental entities being indebted. Apart from this,
that the data related to the payment system is stored India's foreign debt is in US Dollar, SDR, Indian Rupee.
only in India under one system. The data includes all Japanese Yen and Euro. So, the correct answer is
information related to end-to-end transactions, details, option (d).
IAS (Pre) GS 2019 Paper I 56 YCT
52. Which of the following is not included in the • Foreign Trade & Economic
assets of a commercial bank in India?
(a) Advances Organizations of India, Stock
(b) Deposits Market /Regulatory Board
(c) Investments 56. Among the agricultural commodities imported
(d) Money at call and short notice by India, which one to the following accounts
Ans : (b) A bank places its funds in assets to earn for the highest imports in terms of value in the
profits. The assets include investments, loans and last five years?
advances, money at call purchased. It also includes the (a) Spices (b) Fresh fruits
cash in hand with the banks and also the cash held with (c) Pulses (d) Vegetable oils
the RBI. The liabilities include deposits (both time and Ans : (d) India has the highest proportion of edible oil
demands) and borrowings. So, the correct answer is among agriculturally imported commodities, which
option (b) makes up about two-thirds of agricultural imports. India
53. In the context of India, which of the following is a self-sufficient and apex producer in spices, fresh
factors is/are contributor fruits and pulses production. The above mentioned
1. The foreign currency earnings of India's agricultural commodities are also exported by India.
IT sector Vegetable oil has been imported maximum as the basis
of price in the last 5 years; its details are as follows-
2. Increasing the government expenditure
Year Price (in USD)
3. Remittances from Indians abroad
Select the correct answer using the code given 2016-17 10892
below. 2017-18 11637
2018-19 9890
(a) 1 only (b) 1 and 3 only
(c) 2 only (d) 1, 2 and 3 57. Which of the Following is issued by registered
foreign portfolio investors to overseas
Ans : (b) To reduce the risk of currency crisis, factors Investors who want to be a part of Indian
like remittance of foreign exchange earnings by citizens Stock Market without registering themselves
living abroad are helpful. In the context of India, the directly?
risk of a currency crisis can be reduced by the (a) Certificate of Deposit
acquisition of foreign exchange by the IT sector, money (b) Commercial paper
sent by foreign Indians. The Policy of austerity (c) Promissory Note
measures helps to reduce the risk of the currency crisis (d) Participatory Note
and not the excess of government expenditure. Ans : (d) Participatory notes, also referred to as P-
54. The Chairmen of public sector banks are Notes or PNs, are financial instruments required by
selected by the investors or hedge funds to invest in Indian securities
(a) Banks Board Bureau without having to register with the securities and
(b) Reserve Bank of India exchange board of India (SEBI). Investing through P-
(c) Union Ministry of Finance Notes is very simple. Hence, this method is very
(d) Management of concerned bank popular among foreign institutional investors. It was
started in 1992.
Ans : (a) The presidents of public sector banks are
selected by the Bank Boards Bureau (BBB). The BBB 58. Consider the following statements :
has been formed as an autonomous advisory body with 1. Petroleum and Natural Gas Regulatory
effect from 1 April, 2016. Its main function is to take Board (PNGRB) is the first regulatory
body set up by the Government of India
administrative decisions related to public banks and
appointment of heads of the bank. 2. One of the tasks of PNGRB is to ensure
competitive markets for gas.
55. The economic cost of food grains to the Food
3. Appeals against the decisions of PNGRB
Corporation of India is Minimum Support Price
go before the Appellate Tribunals for
and bonus (if any) paid to the farmers plus
Electricity.
(a) transportation cost only
(b) interest cost only Which of the statements given above are
(c) procurement incidentals and distribution cost correct?
(d) procurement incidentals and charges for (a) 1 and 2 only (b) 2 and 3 only
(c) 1 and 3 only (d) 1, 2 and 3
godowns
Ans : (b) The Petroleum and Natural Gas Regulatory
Ans : (c) The economic cost of food grains for food Board (PNGRB) is not the first regulatory body set up
corporations of India includes the relevant procurement by the government of India, before which a number of
charges and distribution cost along with the MSP and regulatory bodies have been formed. PNGRB has been
bonus. In order to encourage agricultural production, formed in the year 2006 under the Petroleum and
the government announces a minimum support price for Natural Gas Regulatory Board Act, 2006. Whereas
23 important agricultural produce. MSP has been earlier Khadi and Village Industries Commission was
considered as the base price. established in 1956, SEBI in 1992, TRAI in 1997 and
Economic cost = MSP + Ancillary Expenses related to IRDA in 1999. One of the key function markets for gas
purchases (bonus) + Relevant procurement charges and appeals against the decisions of PNGRB is made before
distribution costs. the Appellate Tribunals for Electricity.
IAS (Pre) GS 2019 Paper I 57 YCT
59. With reference to Asian Infrastructure (a) 'Higgs boson particles' were detected.
Investment Bank (AIIB), consider the following (b) 'Gravitational waves' were detected.
statements : (c) Possibility of inter-galactic space travel
1. AIIB has more than 80 member nations. through 'wormhole' was confirmed.
2. India is the largest shareholder in AIIB.
3. AIIB does not have any members from (d) It enabled the scientists to understand
outside Asia. 'singularity'.
Which of the statements given above is/are Ans : (b) Scientists expressed the possibility of a
correct collision of a possible black hole neutron star in the
(a) 1 only (b) 2 and 3 only laser. Interferometer Gravitational- wave observatory
(c) 1 and 3 only (d) 1, 2 and 3 and Virgo Interferometer on 26th April, 2019. the
Ans : (a) The Asian Infrastructure Investment Bank incident was detected with the help of the LIGO and
(AIIB) is a multilateral development bank established
on 16 January, 2016, headquartered in Beijing. It European Gravity observatory located in the United
currently has 102 members. The authorized capital of States. Thus the observation of the merger of giant
this bank is 100 billion dollars. The maximum blackholes billion of light-years away was a Unique
shareholding is that of China, India and Russia phenomenon. The significance of this observation is
respectively. Members of this bank are also countries that it detected Gravitational waves.
outside of Asia, such as Canada, Egypt, United
Kingdom, France, Germany etc. Hence, statements 2 • Chemistry
and 3 are wrong, so the correct answer is option (a). 62. Which of the following statements are correct
about the deposits of 'methane hydrate'?
SCIENCE 1. Global warming might trigger the release
• Physics of methane gas from these deposits.
60. In the context of digital technologies for 2. Large deposits of 'methane hydrate' are
entertainment, consider the following statements. found in Arctic Tundra and under the
(1) In Augmented Reality (AR), a simulated seafloor.
environment is created and the physical 3. Methane in atmosphere oxidizes to carbon
world is completely shut out. dioxide after a decade or two.
(2) In Virtual Reality (VR), images generated Select the correct answer using the code given
from a computer are projected onto real-life below.
objects or surroundings. (a) 1 and 2 only (b) 2 and 3 only
(3) AR allows individuals to be present in the (c) 1 and 3 only (d) 1, 2 and 3
world and improves the experience using the
camera of smart-phone or PC. Ans : (d) Methane hydrates can only form under very
(4) VR closes the world, and transposes an specific physical, chemical and geological conditions.
individual, providing complete immersion High water pressures and low temperatures provide the
experience. best conditions. Methane Hydrate deposits may be
Which of the statements given above is/are several hundred meters thick and generally occur in two
correct? types of settings. Under Arctic permafrost and beneath
(a) 1 and 2 only (b) 3 and 4 the ocean floor.
(c) 1, 2 and 3 (d) 4 only
Global warming led to the increase in the temperature
Ans : (b) Virtual Reality (VR) is an artificial,
computer- generated simulation or recreation of a real- which consequently destabilize the methane hydrates and
life environment or situation. It immerses the user by thus release of methane. Methane is relatively short-lived
making them feel like they are experiencing the in the atmosphere; a molecule carbon dioxide within a
stimulating reality first-head, primarily by stimulating decade or so, mainly by reaction with another trace gas,
their vision and hearing. Hence, statement 2 is not the hydroxyl radical (OH).
correct but 4 is correct. 63. In India, the use of Carbofuran, Methyl
Augmented Reality (AR) is a technology that layers Parathion, Phorate and Triazophos is viewed
computer-generated enhancements atop an existing with apprehension. These chemicals are used
reality in order to make it more meaningful through the as
ability to interact with it. AR is developed into apps and (a) pesticides in agriculture
used on mobile devices to blend digital components into
the real world in such a way that they enhance one (b) preservatives in processed foods
another but can also be told apart easily. Hence, (c) fruit-ripening agents
statement 1 is not correct but 3 is correct. (d) moisturizing agents in cosmetics
61. Recently, scientists observed the merger of Ans : (a) Chemicals such as phonsphamidon, lindane,
giant 'blackholes' billions of light-years away fluoropyriphos, heptachlor, malathion, carbofuran,
from the Earth. What is the significance of this methyl parathion, phorates and triazophos are known as
observation? pesticides in agriculture.
IAS (Pre) GS 2019 Paper I 58 YCT
• Medical Science, Anthropology, 1. It is used in developing gene silencing
therapies.
Zoology 2. It can be used in developing therapies for
64. With reference to the recent developments in the treatment of cancer.
science, which one of the following statements 3. It can be used to develop hormone
is not correct? replacement therapies
(a) Functional chromosomes can be created by 4. It can be used to produce crop plants that
joining segments of DNA taken from cells of are resistant to viral pathogens.
different species. Select the correct answer using the code given
(b) Pieces of artificial functional DNA can be below :
created in laboratories. (a) 1, 2 and 4 (b) 2 and 3
(c) A piece of DNA taken out from an animal (c) 1 and 3 (d) 1 and 4 only
cell can be made to replicate outside a living Ans : (a) RNA is a gene- silencing technology that
cell in a laboratory. inhibits protein synthesis in target cells using double-
(d) Cells taken out from plants and animals can stranded RNA.
be made to undergo cell division in RNA has huge significance within the Indian context,
laboratory petri dishes. considering the deep-seated resistance over the years to
Ans : (a) Biotechnology is the development of various BT cotton and other genetically modified seeds. RNA-
stuff or services through the actions of material by reliant solutions could be a viable alternative. RNA is
adopting scientific and engineering principles. Cloning used in functional genomics (systematic analysis of loss
DNA fingerprinting, biochips etc are studied under of function phenotypes induced by RNA triggers) and
biotechnology. Functional DNA, rather than functional developing therapies for the treatment of viral infection
chromosomes, are produced by combining DNA dominant disorders, neurological disorders, and many
segments taken from cells of different races. Hence types of cancers (in vivo inactivation of gene products
statement (a) is false. linked to human disease progression and pathology).
The application of biotechnology can also be seen as 67. Which of the following are the reasons for the
follows- occurrence of multi-drug resistance in-
(1) Artificial Functional DNA parts can be created in microbial pathogens in India?
laboratories using this technology. (1) Genetic predisposition of some people.
(2) Under this any parts of the DNA extracted from an (2) Taking incorrect doses of antibiotics to cure
animal cell can be replicated outside the living cell diseases.
in the laboratory. (3) Using antibioties in livestock farming.
(3) Cell extracted from plants and animals may undergo (4) Multiple chronic diseases in some people.
cell division in laboratory petri dish. Select the correct answer using the codes given
Based on the above interpretation option B,C and D below :
are correct. (a) 1 and 2 (b) 2 and 3 only
65. In the context of wearable technology, which of (c) 1, 3 and 4 (d) 2, 3 and 4
the following tasks is/are accomplished by Ans : (b) The following are the major reasons for multi-
wearable devices? drug resistance in Indian microbial diseases-improper
1. Location identification of a person use of antibiotics for the treatment of diseases, improper
2. Sleep monitoring of a person use of antibiotics in livestock farming and apart from
3. Assisting the hearing impaired person these in the treatment of any type of disease, not taking
Select the correct answer using the code given entire course of antibiotics as well as the use of the
below. wrong medicines.
(a) 1 only (b) 2 and 3 only 68. What is Cas9 protein that is often mentioned in
(c) 3 only (d) 1, 2 and 3 news?
Ans : (d) Wearable technology is a technology that has (a) A molecular scissors used in targeted gene
editing
the ability to connect to the internet. Wearable devices (b) A biosensor used in the accurate detection of
are implanted into clothing through fashion technology. pathogens in patients
Information about the users’ location, sleep monitoring, (c) A gene that makes plants pest-resistant
heart rate, blood pressure, time spent in exercise, (d) A herbicidal substance synthesized in
biochemical emissions, hearing defects etc, can be genetically modified crops
obtained through their device. Presently, this smart Ans : (a) The key step in editing an organism’s genome
technology, mobile application, computing has become is selective targeting of a specific sequence of DNA.
very popular due to digital watch and broadband Two biological macromolecules, the Cas9 protein and
connectivity. Devices used for hearing impaired people guide RNA; interact to form a complex that can identify
are good examples of this. target sequences with high selectivity. The Cas-9
66. 'RNA interference (RNA)' technology has protein is responsible for locating and cleaving target
gained popularity in the last few years. DNA both in natural and in artificial CRISPR/Cas9
Why? systems.
IAS (Pre) GS 2019 Paper I 59 YCT
69. Which one of the following statements is not (a) 1 and 3 only (b) 2 and 3 only
correct? (c) 3 only (d) 1, 2 and 3
(a) Hepatitis B virus is transmitted much like Ans : (c) Article 3 (j) of India's patent act. excludes
HIV. from patentability of "plants and animals in whole or in
(b) Hepatitis B, unlike Hepatitis C, does not any part, thereof other than microorganisms but
have a vaccine. including seeds, varieties and species, and essentially
(c) Globally, the number of people infected with biological processes for production or propagation of
Hepatitis B and C viruses are several times
plants and animals" Hence statement 1 is not correct.
more than those infected with HIV.
(d) Some of those infected with Hepatitis B and The Intellectual Property Appellate Board (IPAB) was
C viruses do not show the symptoms for constituted on 15 September, 2003 by the Indian
many years. Government to hear and resolve the appeals against the
Ans : (b) Hepatitis B is a common infectious disease in decisions of the registrar under the Indian Government
the liver. The disease is caused by the 'Hepatitis B' to hear and resolve the appeals against the decisions of
virus. This disease is spread by blood, unprotected sex, the registrar under the Indian Trademarks Act, 1999 and
infected needles, etc. This infection happens unknowingly; the Geographical Indication of Goods (Registration and
that is why it is also called a silent infection. There is a Protection Act, 1999.) Hence, statement 2 is not correct.
high possibility of having liver cirrhosis due to this 72. Why are dewdrops not formed on a cloudy
infection. According to the assessment of the World Health night?
Organization, about 240 million people have Hepatitis-B (a) Clouds absorb the radiation released from the
globally and about 71 million people with the hepatitis-C Earth's surface.
virus. Several vaccines for the prevention of Hepatitis B (b) Clouds reflect back the Earth's radiation.
virus infection were developed by Maurice Hilleman. (c) The Earth's surface would have low
Currently, vaccines are being prepared with the help of temperature on cloudy nights.
recombinant DNA technology for the prevention of this (d) Clouds deflect the blowing wind to ground
virus. level.
• Agriculture, Animal Husbandry & Ans : (b) Earth's temperature is high during the day;
consequently water evaporates and at night there is
Dairy radiation of heat from the earth and if the sky is clean
70. Consider the following statements: then it is deposited in the radiated heat environment and
1. Agricultural soils release nitrogen oxides the earth becomes cold. Due to the low temperature of
into environment. the earth, condensation of water vapour in the
2. Cattle release ammonia into environment. atmosphere occurs, and as a result it is produced in the
3. Poultry industry releases reactive nitrogen form of dew drops. When there are clouds in the sky,
compounds into environment. they again reflect the heat radiated by the earth back to
Which of the statements given above is/are the earth. As a result, the temperature of the earth is not
correct? reduced enough to condense the atmosphere. As a result
(a) 1 and 3 only (b) 2 and 3 only dew droplets are not formed.
(c) 2 only (d) 1, 2 and 3
Ans : (d) Agricultural soils release nitrogen oxides into 73. For the measurement/estimation of which of
the following are satellite images/remote
the environment. According to an estimate since 2010
sensing data used?
agricultural soils in India contribute to the emission of
1. Chlorophyll content in the vegetation of
more than 70 percent nitrogen oxides in the
a specific location.
environment. Large quantities of ammonia are released
from the cattle in the form of ammonia gas. It is 2. Greenhouse gas emissions from rice
estimated that 80 percent of the ammonia emission in paddies of a specific location.
the environment are from cattle, which is increasing at 3. Land surface temperatures of a specific
the rate of 1 percent per year. The poultry industry location.
releases large quantities of reactive nitrogen Select the correct answer using the code given
compounds, which was estimated to be 0.415 tonnes in below.
2016 and by 2030 is estimated to be 1.089 tonnes. (a) 1 only (b) 2 and 3 only
(c) 3 only (d) 1, 2 and 3
• Science & Technology
71. Consider the following statements : Ans : (d) Through different vegetation index (NDVI)
1. According to the Indian Patents Act, a and Near-infrared radiation, remote sensing satellites
biological Process to create a seed can be can detect healthy vegetation in agriculture. Healthy
patented in India. vegetation reflects green light and absorbs red and blue
2. In India, there is no Intellectual Property light. The green light that our eyes see in chlorophyll
Appellate Board. created by plants during photosynthesis. Chlorophyll
3. Plant varieties are not eligible to be will reflect more light in the green and near- infrared
patented in India. spectrum compared to other wavelengths.
Which of the statements given above is/are Remote sensing satellite data offers the possibility for
correct? measuring land surface temperature over the entire earth
IAS (Pre) GS 2019 Paper I 60 YCT
with sufficiently high temporal resolution and with Ans : (d) LTE stands for long-term Evolution. It
complete spatially averaged rather than point values. provides 4G network service and not the 3G network
Remote sensing from satellites can deliver information on service. The launch of LTE service in India was started
GHG soil emissions by estimating tropospherical, near- by the service provider company Airtel. It works on
surface CO2 and CH4 concentrations IR Infrared portion of voice-only technology and not on data only technology.
the spectrum. Therefore all the statements are correct. VoLTE stands for voice over long-term Evolution. It
74. In the context of which one of the following are also works solely on 4G technology. The VoLTE
the terms 'pyrolysis and plasma gasification' service was launched by Reliance company. VoLTE
mentioned? works on voice-over technology and not voice- only
technology, so statement (2) is incorrect.
(a) Extraction of rare earth elements. Note- LTE is commonly marketed as 4G while VoLTE
(b) Natural gas extraction technologies. is commonly marketed as advanced 4G.
(c) Hydrogen fuel-based automobiles.
(d) Waste-to-energy technologies. ENVIRONMENT & ECOLOGY
Ans : (d) Pyrolysis is a process by which chemical 77. In India, 'extended producer responsibility'
substances are decomposed in the absence of oxygen. was introduced as an important feature in
Plasma gasification is an extreme thermal process by which of the following?
which plasma is converted into organic gas (hydrogen, (a) The Bio-medical Waste (Management and
carbon monoxide), by using plasma. Handing) Rules, 1998.
Electricity/heat is generated through combustion by (b) The Recycled Plastic (Manufacturing and
waste-to-energy technology. It can refer to methane, Usage) Rules, 1999.
methanol, ethanol or synthetic fuel plasma gasification (c) The e-Waste(Management and Handing) Rules,
in the context of waste-to-energy technology. Clear 2011.
energy is produced by recycling waste through (d) The Food Safety and Standard Regulations,
pyrolysis and plasma gasification. 2011.
75. Consider the following statements : Ans : (c) The e-waste (Management and Handling)
A digital signature is Rules, 2011 recognizes producers liability for recycling
1. an electronic record that identifies the and reducing e-waste in the country.
certifying authority issuing it. Extended producer responsibility (EPR) is a policy
approach under which producers are given a significant
2. used to-serve as a proof of identity of an responsibility-financial and/ or physical for the
individual to access information server treatment or disposal of post-consumer products.
on Internet. The central pollution control Board (CPCB) has been
3. an electronic method of signing an given the Extended producer responsibility (EPR)
electronic document and ensuring that authorization under the new e-waste rules.
the original content is unchanged. 78. In the context of proposals to the use of
Which of the statements given above is/are hydrogen-enriched CNG (H-CNG) as fuel for
correct? buses in public transport, consider the
(a) 1 only (b) 2 and 3 only following statements :
(c) 3 only (d) 1, 2 and 3 1. The main advantage of the use of H-CNG
is the elimination of carbon monoxide
Ans : (c) Digital signature is an electronic method of emissions.
signing an electronic document and ensuring that the 2. H-CNG as fuel reduces carbon dioxide and
original content is unchanged. The digital signatures are hydrocarbon emissions.
often used to implement electronic signatures. A valid 3. Hydrogen up to one fifth by volume can be
digital signature assumes the recipient that the message has blended with CNG as fuel for buses.
been generated by a known sender one has not been altered 4. H-CNG makes the fuel less expensive than
in transit. The above statement (3) is correct. CNG.
Which of the statements given above is/are
76. With reference to communication technologies, correct?
what is/are the difference/differences between (a) 1 only (b) 2 and 3 only
LTE (Long-Term Evolution) and VoLTE (c) 3 only (d) 1, 2 and 3
(Voice over Long-Term Evolution)? Ans : (b) The use of H-CNG as a fuel for buses by
1. LTE is commonly marketed as 3G and combining hydrogen with CNG is at an experimental
VoLTE is commonly marketed as level. Its use can reduce carbon dioxide & Hydro carbon
advanced 3G. emission but not eliminate carbon monoxide emissions
2. LTE is data-only technology and VoLTE is by upto 70 percent. Simultaneously, this fuel reduces
carbon dioxide and Hydrocarbon emissions Hydrogen
Voice-only Technology. can be mixed with CNG upto 18 percent, which is
Select the correct answer using the code given equivalent to one-fifth of this fuel. However, H-CNG is
below. more expensive than CNG because both its price and
(a) 1 only (b) 2 only storage are expensive. Hence both (2) and (3) are
(c) Both 1 and 2 (d) Neither 1 nor 2 correct in the above statements.
IAS (Pre) GS 2019 Paper I 61 YCT
79. Consider the following statements : Tusser, cocoon, honey waxes, Lac, tendu/Kendu, leaves,
1. Under Ramsar Convention, it is medicinal plants and herbs, roots tuber, etc. Therefore,
mandatory on the part of the Government statement (2) is correct.
of India to protect and conserve all the The Forest Rights Acts, 2006 defines forest, rights as
wetlands in the territory of India. inclusive of right of ownership, access to collect, use
2. The Wetlands (Conservation and and dispose of minor forest produce which have
Management) Rules, 2010 were framed by traditionally been collected within or outside village
the government of India based on the boundaries: Individuals, communities and gram shabhas
recommendations of Ramsar Convention. having rights under this particular section of the Act will
3. The Wetlands (Conservation and not only have the rights to use but also rights of ownership
over MFPs. Therefore, statement (3) is correct.
Management) Rules. 2010 also encompass
the drainage area or catchment regions of 81. Which one of the following National Parks lies
completely in the temperate alpine zone?
the wetlands as determined by the
(a) Manas National Park
authority. (b) Namdapha National Park
Which of the statements given above is/are (c) Neora Valley National Park
correct (d) Valley of Flowers National Park
(a) 1 and 2 only (b) 2 and 3 only Ans : (d) The alpine forests are found all along the
(c) 3 only (d) 1, 2 and 3 Himalayas at an altitude ranging between 2500 m to
Ans : (c) The Ramsar convention is an 3500 m.
intergovernmental treaty for the conservation of Valley of flowers National Park- At 3352 to 3658
wetlands. When a country joins the Ramsar convention, meters above level.
it gets listed in the list of the countries that signed the Namdapha National Park has a very wide altitude
treaty. Once a country joins the treaty, it has some variation from 200 to 4500 m in the snow-capped
commitments that it has to fulfill as obligations mountain. Thus, it does not lie completely in the
statement, (1) of the above statements is incorrect, temperate alpine zone hence the correct option is (d).
because according to the Ramsar Convention, it is not 82. Consider the following statements :
binding on the Government of India to protect and 1. Asiatic lion is naturally found in India only.
preserve all the wetlands in the territory of India. 2. Double-humped camel is naturally found in
India only.
80. Consider the following statements :
(c) One-horned rhinoceros is naturally found in
1. As per recent amendment to the Indian
India only.
Forest Act, 1927, forest dwellers have the Which of the statements given above is/are
right to fell the bamboos grown on forest correct?
areas. (a) 1 only (b) 2 only
2. As per the Scheduled Tribes and Other (c) 1 and 3 only (d) 1, 2 and 3
Traditional Forest Dwellers (Recognition Ans : (a) Double- humped camel, which is also called
of Forest Rights) Act, 2006, bamboo is a Bactrian camel, is found in the Ladakh region of India
minor forest produce. & also in the Gobi desert, Mongolia, Kazakhstan and
3. The Scheduled Tribes and Other some parts of China. One-Horned rhinoceros is
Traditional Forest Dwellers (Recognition commonly found in Nepal, Bhutan, Pakistan and Assam
of Forest Rights) Act, 2006 allows in India, while the Asiatic lion is naturally found only in
ownership of minor forest produce to the Gir National Park of India. The Asiatic lion is
forest dwellers. included in the Red Data book of the IUCN, it is noted
Which of the statements given above is/are that, the Red Data List, include threatened animals. The
Asiatic lion is protected under the wildlife protection
correct?
Act, 1972 so the correct answer is option (a).
(a) 1 and 2 only (b) 2 and 3 only
83. Which of the following are in Agasthyamala
(c) 3 only (d) 1, 2 and 3
Biosphere Reserve?
Ans : (b) The Amendment Act removes bamboo as a (a) Neyyar, Peppara and Shendurney Wildlife
tree from section 2 (7) of India forest Act, 1927 which Sanctuaries: and Kalakad Mundanthura
would ease transit and feeling restrictions on bamboo in Tiger Reserve
private lands; moreover the Act still has legal (b) Mudumalai , Sathyamangalam and Wayanad
provisions unauthorized extraction of bamboo from the Wildife Sanctuaries; and Silent Valley
forest areas and ampoules forest departments to National Park
prosecute any person or agency found to be doing so (c) Kaundinya, Gundla Brahmeswaram and
therefore, statement (1) is incorrect. Papikonda Wildlife Sanctuaries: and
Minor forest produce is defined under the scheduled tribes Mukurthi National Park
and other traditional forest Dwellers (Recognition of forest (d) Kawal and Sri Venkateswara Wildlife
rights) Act, 2006 as all non-timber forest produce of plant Sancturaries: and Nagarjunasagar-Srisailam
origin and includes bamboo, brushwood, stumps, cares , Tiger Reserve
IAS (Pre) GS 2019 Paper I 62 YCT
Ans : (a) Neyyar, Peppara and Shendurney Wildlife 86. Consider the following statements :
Sanctuaries: and Kalakad Mundanthura Tiger Reserve The Environment Protection Act, 1986
are in Agasthyamala Biosphere reserve. It is located in empowers the Government of India to
the western Ghats in south India. Consisting mostly of 1. state the requirement of public
tropical forest, the site is home to 2, 254 species of participation in the process of
environmental protection, and the
higher plants including about 400 that are endemic. It is procedure and manner in which it is
also a unique genetic reservoir of cultivated plants, in sought
particular cardamom, jamun, nutmeg, pepper and 2. lay down the standards for emission or
plantain. discharge of environmental pollutants
84. Consider the following statements : from various sources
1. Some species of turtles are herbivores Which of the statements given above is/are
2. Some species of fish are herbivores. correct?
3. Some species of marine mammals (a) 1 only (b) 2 only
4. Some species of snakes are viviparous. (c) Both 1 and 2 (d) Neither 1 nor 2
Which of the statements given above are Ans : (c) The Environment impact assessment
correct? notification was made by the Government of India in
(a) 1 and 3 only (b) 2, 3 and 4 only pursuance of the powers conferred on it by the
(c) 2 and 4 only (d) 1, 2, 3 and 4 Environment Protection Act [sub -section (1) and clause
(v) of sub-section (2) of section (3) of the Environment
Ans : (d) Green sea turtles eat, sea grasses and algae, (protection) Act -1986] one of the most significant
though juveniles snack on crabs, sponges, and jellyfish. determinants of EIA is the procedure of public Hearing
In the wild, they can live up to 80 years and grow up to and Public participation on any developmental project.
5 feet long. Once mature, it is the only sea turtle that is therefore statement (1) is correct.
strictly herbivorous. As per section 3 of the EPA the Government of India is
Statement (1) is correct empowered to make rules in laying down standards for
Parrotfish are algae eaters. They obtain the algae by emission or discharge of environmental pollutants from
ripping small chunks of coral from a reef. Many other various sources. Therefore, Statement (2) is correct.
herbivores thrive among the fish population. Other 87. Consider the following :
herbivores include the Japanese angelfish, yellow blotch 1. Carbon monoxide 2. Methane
rabbit fish and tilapia. Statement (2) in correct. 3. Ozone 4. Sulphur dioxide
The diet of Manatees consists of water, grasses, weeds Which of the above are released into
and algae. Dugongs are related to manatees, which are atmosphere due to the burning of crop/biomass
both endangered and protected animals. Their slow- residue?
(a) 1 and 2 only (b) 2, 3 and 4 only
moving herbivores graze on underwater grasses, rooting (c) 1 and 4 only (d) 1, 2, 3 and 4
them out with bristled, sensitive snouts and chomping
Ans : (d) Combustion of Crop/biomass result in the
them with rough lips statement (3) is correct. release of pollutants such as carbon monoxide, methane,
Snakes that are viviparous no wish their developing carbon dioxide, nitrous oxide, sulphur dioxide, ozone,
young through a placenta and yolk sac, something that lead, mercury and particulate matter into the
is highly unusual among reptiles. Boa constrictors and atmosphere. According to a report, only Haryana and
green anacondas are two examples of viviparous snakes, Punjab contribute 48% of the pollutants generated due
meaning they give birth to live young with no eggs to burning of crops.
involved at any stage of development. Statement (4) is At present, the problem of air pollution is increasing
correct. due to the burning of crop residue (parali)
85. Consider the following pairs 88. Why is there a great concern about the
Wildife Naturally found in 'microbeads' that are released into
1. Blue-finned Mahseer : Cauvery River environment?
2. Irrawaddy Dolphin : Chambal River (a) They are considered harmful to marine
ecosystems.
3. Rusty-spotted Cat : Eastern Chats (b) They are considered to cause skin cancer in
Which of the pairs given above are correctly
matched? children.
(a) 1 and 2 only (b) 2 and 3 only (c) They are small enough to be absorbed by
(c) 1 and 3 only (d) 1, 2 and 3 crop plants in irrigated fields.
Ans : (c) Blue- finned Mahseer is found naturally in (d) They are often found to be used as food
the Cauvery River. adulterants.
The Irrawaddy dolphin is a critically endangered Ans : (a) Microbeads are manufactured solid plastic
species. It is found near sea coasts and in estuaries and particles of less than one millimeter in their largest
rivers in parts of the Bay of Bengal and southeast Asia. dimension. They are most frequently made of
In India, it is found in lake Chilika and not the Chambal polyethylene but can be made of other petrochemical
river. The genetic dolphin is found in the Chambal plastics such as polypropylene and polystyrene. They
river. are used in exfoliating personnel care products
Rusty spotted cat is one of the world's smallest feline. toothpaste and in biomedical science research.
The rusty spotted cat, one of the few wild cats that Microbeads can cause plastic particle water pollution
inhabit the forests of Andhra Pradesh, is among the and pose an environmental hazard for aquatic animals in
animals in the Eastern Ghats. freshwater and ocean water.
IAS (Pre) GS 2019 Paper I 63 YCT
CURRENT AFFAIRS Which of the statements given above are
correct
(a) 1 and 3 only (b) 2, 3 and 4 only
89. Atal Innovation Mission is set up under the (c) 2 and 4 only (d) 1, 2, 3 and 4
(a) Department of Science and Technology
(b) Ministry of Labour and Employment Ans : (c) UNCAC is the first legally binding universal
(c) NITI Aayog anti-corruption instrument. The United Nations Office
(d) Ministry of Skill Development and on Drugs and Crime (UNODC) is mandated by UN
Entrepreneurship member states to cooperate in the implementation of
both UNCAC and UNTOC.
Ans : (c) The Atal Innovation Mission (AIM) is a flag
ship initiative initiated by the NITI AYOG. Atal 93. As per the Solid Waste Management Rules,
Innovation mission (AIM) including self-Employment and 2016 in India, Which one of the following
Talent Utilization (SETU) is Government of India's statements in correct?
endeavor to promote a culture of innovation and (a) Waste generator has to segregate waste into
entrepreneurship. Its objective is to serve as a platform for five categories.
promotion of world-class Innovation Hubs, Grand (b) The Rules are applicable to notified urban
Challenges, start-up Businesses and other self-employment local bodies, notified towns and all industrial
activities. particularity in technology driven areas. townships only.
(c) The Rules provide for exact and elaborate
90. The Global Competitiveness Report is criteria for the identification of sites for
published by the landfills and waste processing facilities.
(a) International Monetary Fund (d) It is mandatory on the part of waste
(b) United Nations Conference on Trade and generator that the waste generated in one
Development district cannot be moved to another district.
(c) World Economic Forum
(d) World Bank Ans : (c) The solid waste management rules 2016
provide for detailed criteria for setting-up solid waste
Ans : (c) The Global Competitiveness Report is a processing and treatment facility, solid waste
yearly report published by the World Economic Forum. management in hilly areas for waste to energy process,
Since, 2004, the Global Competitiveness Index for sanitary, landfills, for site selection, development of
developed by Xavier salt-i-Martin and Elsa V Artadi. facilities for the sanitary landfills, specifications for
Its headquarter is located in Geneva. The report has land filling operation and closure on completion of
twelveth pillars of competitiveness. India has been ranked landfilling, pollution prevention, closure and
at 68 position while Singapore topped the Index in Rehabilitation of old dumps etc.
2019. The SWM rules will also apply outside the municipal
91. Recently, India signed a deal known as 'Action areas. The rules will now also include city-related
Plan for Prioritization and Implementation of groups census towns, notified industrial townships, area
Cooperation Areas in the Nuclear Field' with controlled by Indian Railways, airport, ports, defence
which of the following countries? establishments, special economic zones, originations of
(a) Japan centre and states pilgrimage sites and places of religious
(b) Russia and historical importance are also included.
(c) The United Kingdom 94. Consider the following statements :
(d) The United States of America As per the Industrial Employment (Standing
Ans : (b) India and Russia on 5 October, inked an Orders) Central (Amendment) Rules, 2018
action plan for expanding civil nuclear partnership 1. if rules for fixed-term employment are
comprising second site for Russian nuclear reactors in implemented, it becomes easier for the
India. The two countries intend to develop a project of firms/companies to lay off workers.
six nuclear power unit of Russian design at a new site 2. no notice of termination of employment shall
in India, further enhance cooperation in third countries be necessary in the case of temporary
and bring in new perspective nuclear technologies workman.
together with joint construction of nuclear power plants. Which of the statements given above is/are
For the new nuclear project in India Russia will offer correct
the evolutionary VVER generation "3+" technical (a) 1 only (b) 2 only
solutions and will increase the level of Indian industry's (c) Both 1 and 2 (d) Neither 1 nor 2
involvement and localization. Ans : (c) The Government has notified fixed term
92. Consider the following statements : employment for all sections through an amendment to
1. The United Nations Convention against the Industrial Employment (standing orders) central
Corruption (UNCAC) has a 'Protocol rules, 1946, fixed them employment for all sectors will
against the Smuggling of Migrants by make it easier for companies to hire-and-fire work along
Land, Sea and Air'. with reducing the role of middlemen statement (1) is
2. The UNCAC is the ever-first legally correct
binding global anti-corruption instrument. No notice of termination of employment shall be
3. A highlight of the United Nations necessary in the case of temporary workman whether
Convention against Transnational monthly rated weekly rated or piece rated are
Organized Crime (UNTOC) is the probationers or badli workmen. Statement (2) is correct.
inclusion of a specific chapter aimed at 95. What was the purpose of Inter-Creditor
returning assets to their rightful owners Agreement signed by Indian banks and
from whom they had been taken illicitly. financial institutions recently?
4. The United Nations Office on Drugs and (a) To lessen the Government of India's
Crime (UNODC) is mandated by its perennial burden of fiscal deficit and current
member States to assist in the account deficit.
implementation of both UNCAC and (b) To support the infrastructure projects of
UNTOC. Central and State Governments.
IAS (Pre) GS 2019 Paper I 64 YCT
(c) To act as independent regulator in case of 98. Which of the following adopted a law on data
applications for loans of ` 50 crore or more. protection and privacy for its citizens known as
(d) To aim at faster resolution of stressed assets 'General Data protection Regulation' in April
of ` 50 crore or more which are under 2016 and started implementation of it from 25
consortium lending. May, 2018?
(a) Australia
Ans : (d) Project sashakt was proposed by a panel led (b) Canada
by the PNB chairman Sunil Mehta. Bad loans upto Rs (c) The European Union
50 crore will be managed at the bank level, within a (d) The United States of America
deadline of 90 days. Ans : (c) The General Data Protection Regulation
For bad loan of Rs 50-500 crore, banks will enter an (GDPR), agreed upon by the European Union Parliament
inter-creditor agreement, authorizing the lead bank to and Council in April 2016, will replace the Data
implement a resolution plan in 180 days, or refer the Protection Directive 95/46/ec in spring 2018 as the
asset to NCLT. primary law regulating how companies protect EU
96. Which of the following statements is/are citizens personal data. Companies that are already in
correct regarding the Maternity Benefit compliance with the Directive must ensure that they are
(Amendment) Act, 2017? also compliant with the new requirements of the GDPR
1. Pregnant women are entitled for three months before it becomes effective on 25 May, 2018. Companies
that fail to achieve GDPR compliance before the deadline
pre-delivery and three months post-delivery will be subject to stiff penalties and fines.
paid leave.
2. Enterprises with crèches must allow the
mother minimum six crèche visits daily. MISCELLANEOUS
3. Women with two children get reduced
entitlements. 99. In the context of any country, which one of the
following would be considered as part of its
Select the correct answer using the code given social capital?
below. (a) The proportion of literates in the population
(a) 1 and 2 only (b) 2 only (b) The stock of its buildings, other infrastructure
(c) 3 only (d) 1, 2 and 3 and machines
Ans : (c) The Maternity Benefit (Amendment) Act, (c) The size of population in the working age
2017 provides for 26 weeks paid maternity leave for group
women employees. The maternity leave can be availed (d) The level of mutual trust and harmony in the
8 weeks before the expected date of delivery so society
statement (1) is wrong. Ans : (d) Social Capital is defined by the OECD as
• Every establishment with more than 80 employees to "networks together with shared norms, values and
provide crèche facilities for working mothers and understanding that facilitate co-operation within or
such mothers will be permitted to make four visits among groups". In this definition, we can think of the
network as real-world links between groups or
during working hours to look after and feed the child individuals. Think of networks of friends, family
in the crèche. So statement (2) is wrong. networks, networks of former colleagues and so on. Out
• Women, who are expecting after having 2 children, shared norms, values and understandings are less
get a reduced 12 weeks paid maternity leave. So concrete than our social networks.
statement (3) is correct. 100. The word 'Denisovan' is sometimes mentioned
97. Which one of the following is not a sub-index of in media in reference to
the World Bank's 'Ease of Doing Bussiness Index? (a) fossils of a kind of dinosaurs
(a) Maintenance of law and order (b) an early human species
(b) Paying taxes (c) a cave system found in North-East India
(c) Registering property (d) a geological period in the history of Indian
(d) Dealing with construction permits subcontinent
Ans : (a) Doing Business covers 12 areas of business Ans : (b) The first evidence for Denisovans or
regulation. Ten of these areas- Denisova hominins was first discovered in 2008 in a
cave in the Altai mountain in Siberia. The Denisovans
(i) Starting a business or Denisova hominins are an extinct species or sub
(ii) Dealing with construction permits species of archaic humans in the genus Homo.
(iii) Getting electricity Scientists have uncovered the complete remains yet
(iv) Registering property from the mysterious ancient- hominin group known as
(v) Getting credit the Denisovans. The jawbone discoursed high on the
(vi) Protecting minority investors Tibetan plateau and dated to more than 1,60,000 years
(vii) Paying taxes ago, is also the first Denisovan specimen found outside
(viii)Trading across borders the Siberian cave in which the hominin was uncovered a
(ix) Enforcing contracts decade ago-confirming suspicions that Denisovan were
(x) Resolving Insolvency more widespread than the fossil record currently
Doing Business also measures regulation on employing suggests.
workers and contracting with the government, which are The research marks the first time an ancient human has
not, included in the ease of doing business score and been identified solely through the analysis of proteins.
ranking. With no usable DNA, scientists examined proteins in
Note- It is published by World Bank India was ranked the specimen's teeth, raising hopes that more fossils
63rd out of 190 countries. could be identified even when DNA is not preserved.
IAS (Pre) GS 2019 Paper I 65 YCT
UNION PUBLIC SERVICE COMMISSION
Civil Services (Preliminary Exam) - 2018
GENERAL STUDIES : PAPER-I
Time: 2 hours (Exam date : 03.06.2018) Maximum Number: 200
3. Annamacharya and Tyagaraja are
ANCIENT HISTORY contemporaries.
4. Annamacharya kirtanas are devotional songs
• Buddhism, Jainism, Bhagavata, in praise of Lord Venkateshwara.
Shaiva and other Religion Which of the statements given above are
1. With reference to the religious practices in correct?
(a) 1 and 3 only (b) 2 and 4 only
India, the "Sthanakvasi" sect belongs to (c) 1, 2 and 3 (d) 2, 3 and 4
(a) Buddhism (b) Jainism
(c) Vaishnavism (d) Shaivism Ans. (b): Tyagaraja was born on 4th May 1767, in
Thanjavur district of Tamilnadu. He was an ardent
Ans. (b): Sthanakavasi is a sect of Svetambara Jainism devotee of Lord Rama. He composed many new janya
founded by a merchant named Lavaji in 1653 AD. It ragas as well as Melakarta raga.
believes that idol worship is not essential in the path of Annamacharya (Annamayya) was born in May 1408
soul purification and attainment of Nirvana/Moksha. AD; he was a 15th Century Hindu saint and is the
The sect is essentially a reformation of the one founded earliest known Indian musician to compose songs called
on the teachings of Lonka, a fifteenth-century Jain Saknkirtanas in praise of God Ventakateshwara, a form
reformer. Stanakavasins accept thirty-two of the jain of Vishnu.
,Agamas, the Svetamabara canon. Hence, the correct answer is option (b).
2. With reference to Indian history, who among
the following is a future Buddha, yet to come to MODERN HISTORY
save the world ?
(a) Avalokiteshvara (b) Lokesvara • Disintegration of Mughal Empire &
(c) Maitreya (d) Padmapani Advent of European Countries
Ans. (c): Buddhism is divided into two sects Hinayana 5. Economically, one of the results of the British
and Mahayana, in which Mahayana has the idol as rule in India in the 19th century was the
Bodhisattva. Bodhisattvas are individuals who have (a) increase in the export of Indian handicrafts
attained nirvana but they come to help others in nirvana. (b) growth in the number of Indian owned
According to Buddhist beliefs, Maitreya is the future factories
Buddha. According to Buddhist tradition, Maitreya is a (c) commercialization of Indian agriculture
Bodhisattva who will appear on earth to achieve (d) rapid increase in the urban population
complete enlightenment and teach the pure dharma. Ans. (c) : A major impact of the British policies in India
was the introduction of the large number of commercial
MEDIEVAL HISTORY crops such as tea, coffee, indigo, opium, cotton, jute,
sugarcane and oilseed. Exports of handicrafts suffered
• Bhakti Movement during British rule. Thus, option A is correct. Further,
there is no conclusive evidence of growth in number of
3. Which one of the following foreign travellers
Indian owned factories or, rapid increase in the urban
elaborately discussed about diamonds and population.
diamond mines of India ? Thus option (c) is the most appropriate answer.
(a) Francois Bernier
(b) Jean-Baptiste Tavernier 6. The staple commodities of export by the
English East India Company from Bengal in
(c) Jean de Thevenot the middle of the 18th century were
(d) Abbe Barthelemy Carre (a) Raw cotton, oil-seeds and opium
Ans. (b): Jeans-Baptiste Tavernier (1605-1689) was (b) Sugar, salt, zinc and lead
best known for discovering or purchasing the 116-carat (c) Copper, silver, gold, spices and tea
Tavernier Blue Diamond in 1666. The diamond was (d) Cotton, silk, saltpetre and opium
certainly Indian in origin and likely sourced by Ans. (d) : During the period 1780-1860, India changed
Tavernier in 1666 at the Kollur Mine in the Guntur from exports of processed goods paid for in bullion to
district of Andhra Pradesh. export of raw materials and a buyer of manufactured
4. With reference to cultural history of India, goods. In the 1750's fine cotton and silk was exported
consider the following statements : from India to markets in Europe. Asia and Africa, while
th
1. Most of the Tyagaraja Kritis are devotional by the second quarter of the 19 century, raw materials,
songs in praise of Lord Krishna. which chiefly consisted of raw cotton, opium and
2. Tyagaraja created several new ragas. indigo, accounted for most of India's exports

IAS (Pre) GS 2018 Paper I 66 YCT


• Social, Cultural Awakening, Lower Maniben Kara, Shibnath Banerjee, R.A. Khedgikhar,
T.S. Ramanujam, V.S. Mathur, G.G. Mehta. R.S.
Caste, Trade Unions and Peasant Ruikar was elected president and Ashok Mehta general
Movement secretary.
7. After the Santhal Uprising subsided, what HMS absorbed the Royist Indian Federation of Labour
was/were the measure/measures taken by the and the Hind Mazdoor Panchayat, which was formed in
colonial government? 1948 by socialists leaving the increasingly communist
1. The territories called 'Santhal Paraganas' dominated AITUC.
were created. 10. Which among the following events happened
2. It became illegal for a Santhal to transfer earliest ?
land to a non-Santhal. (a) Swami Dayanand established Arya Samaj.
Select the correct answer using the code given (b) Dinabandhu Mitra wrote Neeldarpan.
below: (c) Bankim Chandra Chattopadhyay wrote
(a) 1 only (b) 2 only Anandmath.
(c) Both 1 and 2 (d) Neither 1 nor 2 (d) Satyendranath Tagore became the first Indian
Ans. (c): The area of concentration of the Santhals was to succeed in the Indian Civil Services
called Damn-i-Koh. It extends from Bhagalpur in Bihar Examination.
to Orissa in the south. By the 1850s, the Santhals felt Ans. (b) : The correct order of the following events are-
that the time had come to rebel against zamindars, Swami Daynand Saraswati – 1875 AD
moneylenders and the colonial state, to create an ideal established Arya Samaj
world for themselves where they would rule. After the Dinabandhu Mitra wrote – 1857-59 AD
Santhal Revolt (1855-56) Santhal Pargana was created Neeldarpan
carving out 5,500 square miles from the district of Bankim Chandra Chatt- – 1882 AD
Bhagalpur and Birbhum.
opadhyay wrote Anandmath
8. Which one of the following is a very significant Satyendranath Tagore – 1863 AD
aspect of the Champaran Satyagraha ?
become the first Indian
(a) Active all-India participation of lawyers,
to succeed in the Indian Civil
students and women in the National
Services Examination
Movement
So, the correct answer is option (b).
(b) Active involvement of Dalit and Tribal
communities of India in the National 11. With reference to educational institutions
Movement during colonial rule in India, consider the
(c) Joining of peasant unrest to India's National following pairs :
Movements Institution Founder
(d) Drastic decrease in the cultivation of 1. Sanskrit College William Jones
plantation crops and commercial crops at Banaras
2. Calcutta Madarsa Warren Hastings
Ans. (c) : The centenary of Mahatma Gandhi's first
3. Fort William College Arthur Wellesley
Satyagraha in India was marked in April 2017. It was
undertaken in the erstwhile undivided Champaran Which of the pairs given above is/are correct ?
(a) 1 and 2 (b) 2 only
district in Northern Bihar. Champaran Satyagraha of
(c) 1 and 3 (d) 3 only
1971 opened a new phase in the national movement by
joining it to the great struggle of the Indian peasantryAns. (b): The correct match is as follows:
for bread and land. It was the first peasant movement to Institution Founder
gave garnered nationwide attention. It evoked the Sanskrit College Jonathan Duncan
peasant unrest to the freedom struggle. Culcutta Madarsa Warren Hastings
Fort William College Richard Wellesly
9. Who among the following were the founders of
So, the correct answer is option (b)
the "Hind Mazdoor Sabha" established in 1948?
(a) B. Krishna Pillai, E.M.S. Namboodiripad and • Important Organizations, Treaties,
K.C. George
(b) Jayaprakash Narayan, Deen Dayal Upadhyay
Commissions, Acts
and M.N. Roy 12. Which one of the following statements does not
(c) C.P. Ramaswamy Iyer. K. Ramanujam and apply to the system of Subsidiary Alliance
Veeresalingam Pantulu introduced by Lord Wellesley ?
(d) Ashok Mehta, T.S. Ramanujam and G.G. (a) To maintain a large standing army at other's
Mehta expense
Ans. (d) : The Hind Mazdoor Sabha (HMS) was (b) To keep India safe from Napoleonic danger
founded in Howarh in 1948 by socialists forward Bloc (c) To secure a fixed income for the Company
flowers and independent, unionists which included (d) To establish British paramountcy over the
Basawon Singh (Sinha), Ashok Mehta, R.S. Ruikar, Indian States
IAS (Pre) GS 2018 Paper I 67 YCT
Ans. (c) : The Subsidiary Alliances system was Ans. (c): Lala Lajpat Rai regularly contributed to
introduced by Lord Wellesley in 1798 AD. The British several major Hindi, Punjabi, English and Urdu
under the subsidiary alliance system agreed to protect
newspapers and magazines. He also authored many
the Indian rulers against external threats and internal
books- "Unhappy India", "Young India : An
disorder, but in return, the Indian rulers who accepted
Interpretation", "History of Arya Samaj", "England's
the Subsidiary Alliance system were to agree to the Debt of India" and a series of popular biographies on
stationing of British contingent for whose maintenance
Mazzini, Garibaldi and Swami Dayanand. Also, Lala
they would pay a subsidy to the British. Napoleon Lajpat Rai went to the USA after the Swadeshi
Bonaparte was advancing towards East; hence, the movement petered out.
subsidiary alliance would defend against Napoleonic 15. In 1920, which of the following changed its
danger. name to "Swarajya Sabha" ?
There is no mention of fixed income for the company. (a) All India Home Rule League
Therefore, the correct answer is option (c) (b) Hindu Mahasabha
13. Which of the following led to the introduction (c) South Indian Liberal Federation
of English Education in India ? (d) The Servants of India Societyr
1. Charter Act of 1813 Ans. (a): After the Montagu Reform Declaration in
2. General Committee of Public Instruction, 1917, the congress among the Indian Political parties
1823 considered it unsatisfactory and disappointing. Tilak
3. Orientalist and Anglicist Controversy termed it as sunless summer. But Annie Besant
Select the correct answer using the code givenannounced the abolition of her Home rule when the
below :
declaration provided" accountable governance". In
(a) 1 and 2 only (b) 2 only
1920, Mrs., Annie Besant changed the name of her
(c) 1 and 3 only (d) 1, 2 and 3
organisation "All India Home Rule" to "Swarajya
Ans. (d): The first humble beginning with respect to Sabha". Gandhi joined this league and accepted the
education in India was made by the Charter Act of presidentship of the renamed organisation.
1813, which directed the East India Company to
16. Regarding Wood's Dispatch, which of the
sanction `1 Lakh annually to encourage learned Indians
following statements are true?
and for promotion of modern sciences. But no concrete
1. Grants-in-Aid system was introduced.
steps were taken in this regard before 1823.
2. Establishment of universities was
At last in 1823 an official agency–General Committee
recommended
of public instruction was created to deal with
3. English as a medium of instruction at all
educational matters particularly the expenditure of the
levels of education was recommended.
sum of one lakh of rupees. " Thus a state system of
Select the correct answer using the code given
education was begun almost simultaneously in all the
below :
three presidencies by about 1823 and continued to
(a) 1 and 2 only (b) 2 and 3 only
expand till 1833. The educational grant of Indian was
(c) 1 and 3 only (d) 1, 2 and 3
also increased form one lakh to ten lakh of rupees per
annum". Ans. (a): Wood’s Dispatch is called Magna Carta of
Thus the activities of the GCPI for the decade from English Education in India. Major Recommendations
of Wood's Dispatch:
1823 clearly indicate-its inclination towards orientation.
• An education department was to be established in
The enlightened Indians severely attacked this policy of
Orientalism under the leadership of Raja Ram Mohan every province.
Roy. He submitted a memorandum to the Governor- • Universities on the model of the London's
General on 11th December, 1823 and urged the university are established in big cities such as
Government to abandon the proposal for establishing a Bombay, Calcutta and Madras.
Sanskrit College in Calcutta. • At least one government school is opened in every
But no heed was paid to this memorial and the district.
committee went against the trend of history and the • Affiliated private schools should be given grant-in-
desire for English education among Indians. aid.
Hence, the correct answer is option (d). • The Indian native should be given training in their
mother tongue also.
• Freedom Struggle and National • In accordance with the Wood's Dispatch, Education
Movement Departments were established in every province and
14. He wrote biographies of Mazzini, Garibaldi, universities were opened at Calcutta, Bombay and
Shivaji and Shrikrishna; stayed in America for Madras in 1857 AD and in Punjab in 1882 AD and
some time; and was also elected to the Central at Allahabad in 1887 AD.
Assembly. He was: • The medium of instruction of the primary level was
(a) Aurobindo Ghosh (b) Bipin Chandra Pal to be vernacular while at the higher levels, it would
(c) Lala Lajpat Rai (d) Motilal Nehru be English. So, the answer is option is (a)
IAS (Pre) GS 2018 Paper I 68 YCT
ART &CULTURE (c) Nainital (Uttarakhand)
(d) Renuka (Himachal Pradesh)
17. The well-known painting "Bani Thani" Ans. (a): Kodaikanal Lake, also known as Kodi Lake
belongs to the is a manmade lake located in the Kodaikanal city in
(a) Bundi school (b) Jaipur school Dindigual district in Tamil Nadu, India. Sir Vere Henry
(c) Kangra school (d) Kishangarh school Levinge the then collector of Madurai, was instrumental
Ans.(d) Bani-Thani is an Indian miniature painting in creating the lake in 1863, amidst the Kodikonal town
painted by Nihal Chand from the Marwar school of which was developed by the British and early
Kishangarh. The painting's subject, Bani Thani was a missionaries from USA.
singer and poet in Kishangrh in time of the king Sawant 21. Consider the following statements :
Singh. (1748-1764). She has been compared with the 1. The Barren Island volcano is an active
Mona Lisa. This painting was featured in an Indian volcano located in the Indian territory.
stamp issued in 1973. 2. Barren Island lies about 140 km east of
Great Nicobar.
18. With reference to the cultural history of India,
3. The last time the Barren Island volcano
consider the following statements : erupted was in 1991 and it has remained
1. White marble was used in making Buland inactive since then.
Darwaza and Khankah at Fatehpur Sikri. Which of the statements given above is/are
2. Red sandstone and marble were used in correct ?
making Bara Imambara and Rumi (a) 1 only (b) 2 and 3
Darwaza at Lucknow. (c) 3 only (d) 1 and 3
Which of the statements given above is/are Ans. (c): Barren Island volcano is not active and it is
correct ? located 140 kms to the east of Port Blair (lies in South
(a) 1 only (b) 2 only Andaman), while great Nicobar is the southernmost
(c) Both 1 and 2 (d) Neither 1 nor 2 region of Andaman & Nicobar and lies further South of
Ans. (d): The Buland Darwaza at Fatehpur Sikri was Port Blair.
built in 1602 AD by the great Mughal Emperor Akabar The Barren Island volcano had been lying dormant for
to commemorate victory over Gujarat. It is made of red more than 150 years until it saw a major eruption in
and buff sandstone, decorated by white and black 1991. Since, then it has shown intermittent activity,
marble and higher than the mosque's courtyard. including eruptions in 1995, 2005 and 2017.
Therefore statement (1) is incorrect. Asaf-ud Daula rule • Monsoon, Forest, Soil, Irrigation,
saw a devastating famine, which created an economic
crisis, a food-for-work programme was started. The
Projects, Agriculture
famous Asafi Imambara or Bara Imambara of Lucknow, 22.As per the NSSO 70th Round "Situation
was built to give employment and revenue to the public. Assessment Survey of Agricultural Households",
Instead of stones and marble, brick and lime were used. consider the following statements:
Therefore, statement (2) is also incorrect. 1. Rajasthan has the highest percentage
share of agricultural households among its
19. Consider the following pairs : rural households.
Tradition State 2. Out of the total agricultural households in
1. Chapchar Kut festival – Mizoram the country, a little over 60 percent belong
2. Khongjom Parba ballad – Mainpur to OBCs.
3. Thang-Ta dance – Sikkim 3. In Kerala, a little over 60 percent of
Which of the pairs given above is/are correct? agricultural households reported to have
(a) 1 only (b) 1 and 2 received maximum income from sources
(c) 3 only (d) 2 and 3 other than agricultural activities.
Ans. (b): The statement in question (1) and (2) are Which of the statements given above is/are
correctly matched while statement (3) is incorrect as correct?
Thang-Ta dance is an ancient martial art form of (a) 2 and 3 only (b) 2 only
Manipur. Chapchar Kut is an agricultural festival (c) 1 and 3 only (d) 1, 2 and 3
celebrated in February- March in Mizroam while Ans. (c): Rajasthan has the highest agricultural
Khongjom Parba is a singing style of the ballad song of households (78.4%) among rural households, in terms
Manipur. of proportion, in absolute numbers, it is Uttar Pradesh.
The share of OBC households among rural households
INDIAN GEOGRAPHY is about 45 percent.
In Kerala, about 61 percent of agricultural families have
• Physical Structure, Drainage more income from agricultural sources.
Therefore, the correct answer is option ( c).
System, Human Geography
23. Which of the following is/are the possible
20. Which one of the following is an artificial lake? consequence/s of heavy sand mining in
(a) Kodaikanal riverbeds ?
(b) Kolleru (Andhra Pradesh) 1. Decreased salinity in the river
IAS (Pre) GS 2018 Paper I 69 YCT
2. Pollution of groundwater 2. Andhra Pradesh and Jharkhand do not
3. Lowering of the water-table have gold mines.
Select the correct answer using the code given 3. Rajasthan has iron ore mines.
below : Which of the statements given above is/are
(a) 1 only (b) 2 and 3 only correct?
(c) 1 and 3 only (d) 1, 2 and 3 (a) 1 and 2 (b) 2 only
Ans. (b): The removal of sand from the river bed (c) 1 and 3 (d) 3 only
increases the flowing water's velocity, with the distorted Ans. (d): As per the new mining law Mines and
flow regime eventually eroding the river banks. Minerals (Development and Regulation) Amendment
The sand acts like a sponge, which helps in recharging Act, 2015- which came into effect in January 2015, the
the water table; its progressive depletion in the river is non-coal mines have to be auctioned by the respective
accompanied by declining water tables in the nearby state governments.
areas. Depletion of sand in the stream bed causes the • Jharkhand has both alluvial and native gold.
deepening of rivers and estuaries and the enlargement of Alluvial gold is obtained from the sands of the
river mouths and coastal inlets. It leads to saline water Subarnarekha river, Sona Nadi in Singhbhum district
incursion. and the streams draining the Sonapat valley. Native
The sand acts as an efficient filter for various pollutants gold is found near Lawa in the Singhbhum district.
and thus maintains the quality of water in rivers and • In Andhra Pradesh, gold deposits are found in
other aquatic ecosystems. Ranagiri in Anantapur district.
Hence, the correct answer is option (b). • In Rajasthan, the iron-ore deposits are located in the
24. With reference to agricultural soils, consider districts of Jaipur, Udaipur, Jhunjhuna, Sikar,
the following statements : Bhilwara, Alwar, Bharatpur, Dausa and Banswara.
1. A high content of organic matter in soil So the correct answer is option (d).
drastically reduces its water holding
capacity. WORLD GEOGRAPHY
2. Soil does not play any role in the sulphur
cycle. • Universe and Solar System
3. Irrigation over a period of time can 26. Consider the following statements:
1. The Earth's magnetic field has reversed
contribute to the salinization of some every few hundred thousand years.
agricultural lands. 2. When the Earth was created more than
Which of the statements given above is/are 4000 million years ago, there was 54%
correct ? oxygen and no carbon dioxide.
(a) 1 and 2 only (b) 3 only 3. When living organisms originated, they
(c) 1 and 3 only (d) 1, 2 and 3 modified the early atmosphere of the
Ans. (b): Statement 1 is wrong because it increases Earth.
water holding capacity. A higher organic matter content Which of the statements given above is/are
in the soil will result in more humus and an increase in correct ?
its water holding capacity. (a) 1 only (b) 2 and 3 only
Statement 2 is incorrect because water plays a role in (c) 1 and 3 only (d) 1, 2 and 3
the Sulphur cycle. Sulphur, like nitrogen and carbon is Ans. (c): Statement 1- Reversal in Earth's magnetic
an essential part of living matter because sulphur - field is called geomagnetic reversal. It occurs every few
containing amino acids are always present in almost all million years. It is a change in a planet's magnetic field
kinds of proteins. Plants can absorb directly the sulphur such that the positions of magnetic north and magnetic
containing amino acids but these amino acids fulfill south are interchanged, while geographic north and
only a small proportion or requirements of plants. geographic south remain the same. Thus, the statement
Sulphur passes through the cycle of transformation 1 is correct.
mediated by microorganisms. It accumulates in the soil Statement 2- Early atmosphere has an abundance of
mainly as a constituent of organic compounds and has hydrogen sulphide and carbon dioxide. There were
to be converted to sulphates to become readily available smaller proportions of water vapour, ammonia and
to plants. methane. The present composition got finalized very
Irrigation causes organic matter to leach and making late around, 600 million years ago (Cambrian period).
Volcanism and the very evolution of life played an
land saline over-irrigation can lead to salinity in soils important role in changing the atmospheric
because of over-use of ground-water and/or the rise of composition. Thus statement (2) is incorrect.
the water table. Statement 3- Respiration and microbial action by
So, correct answer is option (b). organisms modified the early atmosphere. Thus, this
statement is correct.
• Minerals, Energy Resources, So correct answer is option (c)
Industry and Trade • Hydrosphere
25. Consider the following statements: 27. Which of the following has/have shrunk
1. In India, State Governments do not have immensely/dried up in the recent past due to
the power to auction non-coal mines. human activites?
IAS (Pre) GS 2018 Paper I 70 YCT
1. Aral Sea • Citizenship, Fundamental Rights,
2. Black Sea Fundamental Duties, Directive
3. Lake Baikal
Principles of State Policies
Select the correct answer using the code given
30. Consider the following statements:
below: 1. As per the Right to Education (RTE) Act,
(a) 1 only (b) 2 and 3 to be eligible for appointment as a teacher
(c) 2 only (d) 1 and 3 in a State, a person would be required to
Ans. (a): The Aral Sea is a saline lake located in central possess the minimum qualification laid
Asia, once the world's fourth-largest salt lake. In the down by the concerned State Council of
Teacher Education.
1960's the Aral sea, which was the drainage basin for 2. As per the RTE Act, for teaching primary
Kyrgyzstan, Turkmenistan, Tajikistan, Uzbekistan, classes, a candidate is required to pass a
Pakistan, Afghanistan and Kazakhstan, began to shrink as Teacher Eligibility Test conducted in
the Soviet Union began to divert water for agricultural accordance with the National Council of
Teacher Education guidelines.
purposes. The Aral sea, which was once 26,300 square
3. In India, more than 90% of teacher
miles in size, decreased in size by 2007 to only 10% of its education institutions are directly under
original area. Lake Baikal, the water level has gone below the State Governments.
the critical mark of 456m eleven times since 1962. Lake Which of the statements given above is/are
Baikal shows more of a fluctuating regime and shrinkage correct?
(a) 1 and 2 (b) 2 only
is not comparable to that of Aral Sea. So the correct (c) 1 and 3 (d) 3 only
answer is option (a) Aral Sea. Ans. (b): In accordance with the provisions of sub-
section (1) of section 23 of the Right of Children to Free
POPULATION AND URBANIZATION and Compulsory Education (RTE) Act, 2009, the
National Council for Teacher Education (NCTE) has
28. Among the following cities, which one lies on a laid down the minimum qualifications for a person to be
longitude closest to that of Delhi? eligible for appointment as a teacher in class 1 to 8 vide
(a) Bengaluru (b) Hyderabad its Notification dated 23 August, 2010. So Statement (1)
(c) Nagpur (d) Pune is not correct.
• One of the essential qualifications for a person to be
Ans. (a): The longitude of Delhi is 77º East and eligible for appointment as a teacher in any of the
Bengaluru lies close to Delhi in terms of longitude. primary schools is that he/she should pass the
Hyderabad and Nagpur are further on the East. Teacher Eligibility Test (TET) which will be conducted
by the appropriate government. So, statement (2) is
Cities North Latitude East Latitude correct.
Delhi 28.7041º N 77.1025º E • From the mid-1960s to 1993, the number of TETs
Bengaluru 12.9716º N 77.5946º E in India went up from about 1200 to about 1500.
After the NCTE was set up, the number of TETs
Hyderabad 17.3850º N 78.4867º E exploded, to about 16,000 (over 90% private) by
Nagpur 21.1458º N 79.0882º E 2011. So statement (3) is not correct.
31. Which one of the following reflects the most
Pune 18.5204º N 73.8567º E appropriate relationship between law and
So the correct answer is option (a). liberty?
(a) If there are more laws, there is less liberty.
(b) If there are no laws, there is no liberty.
Indian Constitution and Polity (c) If there is liberty, laws have to be made by
the people.
• Constitutional Development of India (d) If laws are changed too often, liberty is in
danger.
29. In the Federation established by The
Ans. (b): The term 'Liberty' means the absence of
Government of India Act of 1935, residuary restraints on the activities of individuals and at the same
powers were given to the ? time, providing opportunities for the development of
(a) Federal Legislature individuals. However, liberty does not mean 'license' to
(b) Governor General do what one likes and has to be enjoyed within the
(c) Provincial Legislature limitations mentioned in the constitution and various
laws. Therefore, the concept of liberty is not absolute.
(d) Provincial Governors The absence of laws does not ensure liberty.
Ans. (b) : The residuary power was not allocated either Liberty entails the responsible use of freedom under the
to the Federation or to the provinces but was under rule of law without depriving anyone else of their
section 105 of the Government of India. Act of 1935 freedom. Freedom is broader in that it represents a total
lack of restraint or the unrestrained ability to fulfill
reserved to be allocated by the Governor -General in his one's desires.
discretion to the federation or to the provinces. The correct answer is option (b)
IAS (Pre) GS 2018 Paper I 71 YCT
32. Which of the following are regarded as the • Executive of Center and States,
main features of the "Rule of Law" ? Emergency Provisions
1. Limitation of powers
34. With reference to the election of the President
2. Equality before law of India, consider the following statements :
3. People's responsibility to the Government 1. The value of the vote of each MLA varies
4. Liberty and civil rights from State to State.
2. The value of the vote of MPs of the Lok
Which of the statements given above are Sabha is more than the value of the vote of
correct? MPs of the Rajya Sabha.
(a) 1 and 3 only (b) 2 and 4 only Which of the statements given above is/are
correct ?
(c) 1, 2 and 4 only (d) 1, 2, 3 and 4 (a) 1 only (b) 2 only
Ans. (c) : The concept of the rule of the law is a very (c) Both 1 and 2 (d) Neither 1 nor 2
dynamic concept capable of interpretations to enable the Ans. (a) : Value of a vote and securing parity.
(a) Article 55 provides for uniformity in the scale of
successful working of a democracy. It is the restriction representation of different states at the election of the
on the arbitrary exercise of power by subordinating it to president.
well-defined and established laws. Features of the (b) Besides there should also be a parity between the
modern meaning of the rule of law– states as a whole and the union.
(c) For this purpose the value of votes of an MLA and
1. Certainty of the law. an MP is counted under the following formula.
2. Supremacy of the law. (1) Value of an MLA Vote
3. Equality before the law. TotalPopulation of theState / UT 1
= ×
Total Number of elected membersof thestate LegistativeAssembly 1000
4. Right to personal freedom
5. Laws must not take retrospective effect (2) Value of an MP Vote =
TheSum of VoteValue of elected members of all thestates
6. Independent judiciary.
7. Protection of human rights. TheSumof elected membersof both thehousesof Parliament
The value of the vote of each MP of Lok Sabha and
Therefore the correct answer is option (c) Rajya Sabha is equal, i.e, 708 total value of the current
33. Right to Privacy is protected as an intrinsic strength of Lok Sabha MPs is 3,84,444 and Rajya Sabha
part of Right to Life and Personal Liberty. (233) total value is 1, 64,964.
The value of the vote of each MLA of each state is
Which of the following in the Constitution of different. It is determined by the average constituency
India correctly and appropriately imply the size and 1971 census. The value of MLA of Uttar
above statement ? Pradesh is the highest, i.e, 208.
So, the correct answer is option (a)
(a) Article 14 and the provisions under the 42nd
35. If the President of India exercises his power as
Amendment to the Constitution provided under Article 356 of the Constitution
(b) Article 17 and the Directive Principles of in respect of a particular State, then
(a) the Assembly of the State is automatically
State Policy in Part IV dissolved.
(c) Article 21 and the freedoms guaranteed in (b) the powers of the Legislature of that State
Part III shall be exercisable by or under the authority
of the Parliament.
(d) Article 24 and the provisions under the 44th (c) Article 19 is suspended in that State.
Amendment to the Constitution (d) the President can make laws relating to that
Ans. (c): Article 21 guarantees the right to life and State.
Ans. (b) : Article 356- Provisions in case of failure of
personal liberty under part III of the constitution. The constitutional machinery in state-
historic fallout of the nine-judge Bench judgment (i) If the President on receipt of the report from the
declaring privacy an inherent right protected by part III Governor of the state or otherwise is satisfied that a
situation has arisen in which the government of the
of the constitution, is that an ordinary man can now state cannot be carried on in accordance with the
directly approach the Supreme Court and the High provisions of this constitution, the President may be
Court to violate his fundamental right under the Proclamation.
(a) Assume to himself all or any of the functions of the
constitution. Government of the state and all or any of the powers
By making privacy an intrinsic part of life and liberty vested in or exercisable by the Governor or anybody
under Article 21, it is not just a citizen but anyone, or authority in the state other than the Legislature of
the state;
whether an Indian national or not, can move the
(b) Declare that the powers of the Legislature of the
constitutional courts of the land under Article 32 and state shall be exercisable by or under the authority
226, respectively to get justice. or Parliament;
So, the correct answer is option (c) So the correct answer is option (b).
IAS (Pre) GS 2018 Paper I 72 YCT
36. Consider the following statements : function is to examine the rules and regulations enacted
1. No criminal proceedings shall be instituted by the executive to fill the gaps in the laws enacted by
against the Governor of a State in any the parliament and report how these rules are within
court during his term of office. limits prescribed in the main law. This committee has
2. The emoluments and allowances of the 15 members each in Rajya Sabha and Lok Sabha.
Governor of a State shall not be diminished 39. Consider the following statements :
during his term of office. 1. In the first Lok Sabha, the single largest
Which of the statements given above is/are party in the opposition was the Swatantra
correct ? Party.
(a) 1 only (b) 2 only 2. In the Lok Sabha, a "Leader of the
(c) Both 1 and 2 (d) Neither 1 nor 2 Opposition" was recognised for the first
Ans. (c) : Article 361(2) - No criminal proceeding time in 1969.
whatsoever shall be instituted or continued against the 3. In the Lok Sabha, if a party does not have
president, or the governor of a state in any court during a minimum of 75 members, its leader
his term of office. cannot be recognised as the Leader of the
Article 158(4)- The emoluments and allowances of the Opposition.
governor shall not be diminished during his term of Which of the statements given above is/are
office. So, the correct answer is option (c) correct?
37. Consider the following events : (a) 1 and 3 only (b) 2 only
1. The first democratically elected communist (c) 2 and 3 only (d) 1, 2 and 3
party government formed in a State in Ans. (b): In the 1952 election, the Congress has won
India. 364 out of 489 Lok Sabha seats in the first-ever general
2. India's then largest bank, 'Imperial Bank election held in India. Its vote share was 45%. The CPI
of India', was renamed 'State Bank of was the second party with just 16 seats.
India'. • The Swatantra Party was an Indian liberal-
3. Air India was nationalised and became the conservative political party that existed from 1959 to
national carrier. 1974.
4. Goa became a part of independent India.
Which of the following is the correct • In 1969, as the official leader of the opposition was
chronological sequence of the above events ? recognized for the first time. However, it was given
(a) 4 – 1 – 2 – 3 (b) 3 – 2 – 1 – 4 statutory recognition in 1977.
(c) 4 – 2 – 1 – 3 (d) 3 – 1 – 2 – 4 • The leader of the largest opposition party, having not
Ans. (b): In Kerala Legislative Assembly election of less than one-tenth seats of the total strength of the
1957 was the first assembly election in the Indian state House is recognized as the leader of opposition in the
of Kerala. The Communist Party of India won the House. The 10% of the total strength of the house in
election with 60 seats. The election led to the formation 55 seats in the Lok Sabha.
of the first democratically elected communist Therefore, statements (1) and (3) are incorrect and only
government in India. (2) is correct. So the right answer is option (b).
• The Government of India took control of the imperial 40. Regarding Money Bill, which of the following
Bank of India in 1955 with the Reserve Bank of India statements is not correct ?
(India's Central Bank) taking a 60% stake, renaming it (a) A bill shall be deemed to be a Money Bill if
the State Bank of India. it contains only provisions relating to
• In 1953, Air India was nationalized. imposition, abolition, remission, alteration or
• On 19th December, 1961,Goa officially became part regulation of any tax.
of India. (b) A Money Bill has provisions for the custody
So, the correct answer is option (b). of the Consolidated Fund of India or the
Contingency Fund of India.
• Parliament (c) A Money Bill is concerned with the
38. With reference to the Parliament of India, appropriation of moneys out of the
which of the following Parliamentary Contingency Fund of India.
Committees sccrutinizes and reports to the (d) A Money Bill deals with the regulation of
House whether the powers to make regulations, borrowing of money or giving of any
rules, sub-rules, by-laws, etc. conferred by the guarantee by the Government of India.
Constitution or delegated by the Parliament Ans. (c): Article 110- Definition of the money bill :
are being properly exercised by the Executive (1) For the purpose of the Chapter, a bill shall be
within the scope of such delegation? deemed to be a Money Bill if it contains only
(a) Committee on Government Assurances provisions dealing with all or any of the following
(b) Committee on Subordinate Legislation matters, namely.
(c) Rules Committee (a) The imposition, abolition, remission, alteration or
(d) Business Advisory Committee regulation of any tax.
Ans. (b) : Every house of the parliament has a (b) The regulation of the borrowing of money or the
committee on subordinate legislation whose main giving of any guarantee by the government of India
IAS (Pre) GS 2018 Paper I 73 YCT
or the amendment of the law with respect to any
financial obligations undertaken or to be undertaken ECONOMICS
by the Government of India.
(c) The custody of the consolidated fund or the • Poverty and Unemployment
Contingency Fund of India, the payment of money 43. Consider the following statements:
into or the withdrawal of money from any such 1. The Food Safety and Standards Act, 2006
fund. replaced the Prevention of Food
(d) The appropriation of money out of the consolidated Adulteration Act, 1954.
fund of India (not contingency fund of India). 2. The Food Safety and Standards Authority
So the incorrect option is (c) of India (FSSAI) is under the charge of
41. Consider the following statements : Director General of Health Services in the
1. The Speaker of the Legislative Assembly Union Ministry of Health and Family
shall vacate his/her office if he/she ceases to Welfare.
be member of the Assembly. Which of the statements given above is/are
2. Whenever the Legislative Assembly is correct?
dissolved, the Speaker shall vacate his/her (a) 1 only (b) 2 only
office immediately. (c) Both 1 and 2 (d) Neither 1 nor 2
Which of the statements given above is/are
correct ? Ans. (a): After commencement of the Food Safety and
(a) 1 only (b) 2 only Standards Act, 2006, various central Acts like
(c) Both 1 and 2 (d) Neither 1 nor 2 Prevention of Food Adulteration Act, 1954, Fruit,
Ans. (a) : Speaker of Assembly vacates his/her office Products Order, 1955, Meat Food Products Order, 1973,
earlier in any of the following there cases- Vegetable Oil Products (control) Order, 1947, Edible
(1) If he/she ceases to be a member of the assembly. oils Packaging (Regulation) Order 1988 were replaced.
(2) If he/she resigns by writing to the deputy speaker If function under the administrative control of Union
(3) If he/she is removed by a resolution passed by a Ministry of Health and Family Welfare.
majority of all the then members of the assembly. Statement (1) is correct.
Such a resolution passed by a majority of all the ten Chairperson and Chief Executive Officer of FSSAI are
members of the assembly. Such a resolution can be appointed by The Government of India. The
moved only after giving 14 days advance notice. Chairperson is in the rank of secretary to The
Whenever the Assembly is dissolved, the speaker does Government of India. Statement (2) is incorrect.
not vacate his office and continues till the newly elected
member meets. 44. With reference to the provisions made under
the National Food Security Act, 2013, consider
• Panchayati Raj System and the following statements:
Constitution Amendments, Schedules 1. The families coming under the category of
42. Consider the following statements: 'below poverty line (BPL)' only are eligible
1. The Parliament of India can place a to receive subsidised food grains.
particular law in the Ninth Schedule of the 2. The eldest woman in a household, of age 18
Constitution of India. years or above, shall be the head of the
2. The validity of a law placed in the Ninth household for the purpose of issuance of a
Schedule cannot be examined by any court ration card.
and no judgment can be made on it. 3. Pregnant women and lactating mothers are
Which of the statements given above is/are entitled to a 'take-home ration' of 1600
correct? calories per day during pregnancy and for six
(a) 1 only (b) 2 only months thereafter.
(c) Both 1 and 2 (d) Neither 1 nor 2 Which of the statements given above is/are
Ans. (a): The Ninth Schedule was added to the correct?
Constitution by the first constitutional amendment in (a) 1 and 2 (b) 2 only
1951 along with Article 31-B with a view to providing a
" protective umbrella" to land reforms laws to save (c) 1 and 3 (d) 3 only
them from being challenged in courts on the ground of Ans. (b): Some of the salient features of the National
violation of fundamental rights. The SC upheld the Food Security Act, (NFSA) 2013 are-
validity of Article 31-B and the Parliaments power to • Coverage and Entitlement under Targeted Public
place a particular law in the Ninth Schedule. Distribution System (TPDS) up to 75% of the rural
Hence statement (1) is correct. population and 50% of the urban population will be
The Supreme Court has said that laws placed in the covered under TPDS, with uniform entitlement of 5
Ninth Schedule are open to judicial scrutiny and that
such laws do not enjoy a blanket protection laws placed kg per person per month. Not restricted to only BPL
the Ninth schedule after the Kesavananda Bharti families.
Judgment on 24th April, 1973, when it propounded the • Maternity benefit to pregnant women and lactating
"basic structure" doctrine were open to challenge. mothers will also be entitled to receive maternity
Hence statement (2) is not correct. benefit of not less than `6000.
IAS (Pre) GS 2018 Paper I 74 YCT
• Eldest women of the household of age 18 years or candidates of Indian nationality who are either
above to be the head of the household for the purpose school/ college dropouts or unemployed.
of issuance of ration cards. • Apart from providing training according to the
• Food grains under TPDS will be made available at National Skills Qualification Framework (NSQF)
TCs shall also impart training in soft skills,
subsidized prices of `3/2/1 per kg of rice, wheat and
Entrepreneurship, Financial and Digital Literacy.
coarse grains for a period of three years from the date Individuals with prior learning experience or skills
of commencement of the Act. will also be assessed and certified under-
So the correct answer is option (b). Recognition of Prior Learning (RPL).
45. International Labour Organization's Conventions So, statement (2) and (3) are correct.
138 and 182 are related to :
(a) Child labour • Economic Planning and National
(b) Adaptation of agricultural practices to global Income/Budget
climate change
(c) Regulation of food prices and food security 48. Consider the following statements :
(d) Gender parity at the workplace Human capital formation as a concept is better
Ans. (a): The Union Cabinet has given its approval for explained in terms of a process which enables-
ratification of the two fundamental conventions of the 1. individuals of a country to accumulate
International Labour Organization (ILO), namely the more capital.
Minimum Age Convention (No. 138) concerning 2. increasing the knowledge, skill levels and
minimum age for admission to employment and the capacities of the people of the country.
worst form of child labour convention (No. 182) 3. accumulation of tangible wealth.
concerning the prohibition and immediate action for the 4. accumulation of intangible wealth.
elimination of the worst forms of child labour. Which of the statements given above is/are
correct ?
46. Increase in absolute and per capita real GNP
(a) 1 and 2 (b) 2 only
do not connote a higher level of economic
(c) 2 and 4 (d) 1, 3 and 4
development, if
(a) industrial output fails to keep pace with Ans. (c): Statement (1) Human capital formation is the
agricultural output. outcome of investment is education health, on, the job
(b) agricultural output fails to keep pace with training migration and information refer to the
industrial output. aggregated of gross additions to fixed assets (that is
(c) poverty and unemployment increase. fixed capital formation) plus change in stock during the
(d) imports grow faster than exports. counter period. As per RBI Gross Capital Formation
Ans. (c): Development means qualitative change which (GCF) representing physical capital. Hence statement
are always value positive. This means that development first is incorrect.
cannot take place unless there is an increment or Statement (2) According to the OECD, human capital,
addition to the existing conditions. Development occurs is defined as " the knowledge, skills competencies and
when positive growth takes place. Yes, positive growth other attributes embodied in individuals or groups of
does not always lead to development. This happens individuals acquired during their life and used to
when poverty and unemployment increase. produce goods, services or ideas in market
circumstances".
47. With reference to Pradhan Mantri Kaushal
Statement (3) Tangible capital is also GCF as it
Vikas Yojana, consider the following
involves largely infrastructural components.
statements :
1. It is the flagship scheme of the Ministry of Statement (4) Intangible wealth of a nation is
Labour and Employment. comprised of the skilled population, human resource
2. It, among other things, will also impart base, culture, arts, etc.
training in soft skills, entrepreneurship, So, the correct answer is option (c).
financial and digital literacy. 49. Despite being a high saving economy, capital
3. It aims to align the competencies of the formation may not result in significant increase
unregulated workforce of the country to in output due to
the National Skill Qualification Framework. (a) weak administrative machinery
Which of the statements given above is/are (b) illiteracy
correct ? (c) high population density
(a) 1 and 3 only (b) 2 only (d) high capital-output ration
(c) 2 and 3 only (d) 1, 2 and 3 Ans. (d): Without the availability of adequate, capital
Ans. (c): Pradhan Mantri Kaushal Vikas Yojana either in the form of physical capital or in the form of
(PMVKY) is the flagship scheme of the Ministry of human capital development of the nation is not possible.
The higher is rate of capital formation, the faster is the
Skill Development and Entrepreneurship (MSDE). pace of economic growth. Saving and investment are
Statement (1) is incorrect. essential for capital formation, but savings are different
• The short term training imparted at PMKVY from hoardings. For savings to be utilized for investment
training centre (TCs) is expected to benefit purposes, they must be mobilised in banks and financial
IAS (Pre) GS 2018 Paper I 75 YCT
institutions and the businessmen, the entrepreneurs and the 52. With reference to India's decision to levy an
farmers invest these community savings or capital goods equalization tax of 6% on online advertisement
by taking loans from these banks and financial services offered by non-resident entities, which
institutions. This is capital formation. of the following statements is/are correct?
The process of capital formation involves three steps : 1. It is introduced as a part of the Income Tax
increase in the volume of real savings, mobilization of Act.
saving through financial and credit institutions and 2. Non-resident entities that offer
investment of savings. So, if a county has high savings advertisement services in India can claim a
but poor technology, low efficiency, then economic tax credit in their home country under the
growth will not be possible. "Double Taxation Avoidance Agreements".
Select the correct answer using the code given
• Agriculture, Industry and Trade below:
50. Consider the following : (a) 1 only (b) 2 only
1. Areca nut 2. Barley (c) Both 1 and 2 (d) Neither 1 nor 2
3. Coffee 4. Finger millet Ans. (d): Equalization tax was introduced as part of the
5. Groundnut 6. Sesamum Finance Act, 2016. Taking a cue from OECD's BEPS
7. Turmeric Action Plan 1, India proposed an 'equalisation levy' on
The Cabinet Committee on Economic Affairs 12 digital services. The government has put a 6%
has announced the Minimum Support Price for equalization levy on the income accrued to a foreign e-
which of the above ? commerce company that is not a resident of India. This
(a) 1, 2, 3 and 7 only will affect Google, Amazon, Facebook, etc. It is also
(b) 2, 4, 5 and 6 only called 'Google Tax'. Since it was not a part of the
(c) 1, 3, 4, 5 and 6 only Income Tax Act, equalization levy was independent of
(d) 1, 2, 3, 4, 5, 6 and 7 taxability of the income embedded in the said services.
Ans. (b): The MSP is announced by the Government of Statement (1) is incorrect.
India for 25 crops currently at the beginning of each • Tax credit can't be claimed. Statement (2) is incorrect.
season viz, Rabi and Kharif. Kharif crops 14 crops 53. Consider the following statements:
covered in MSP these are Paddy, Jowar, Bajra, Maize, 1. The Fiscal Responsibility and Budget
Ragi, Arhar (Tur), Moong, Urad, Cotton, Groundnut, Management (FRBM) Review Committee
Sunflower seed, Soyabean, Black Sesamum and Report has recommended a debt to GDP
Nigerseed. Crops covered under Rabi crops are 7 they ratio of 60% for the general (combined)
are- Wheat, Barley, Gram, Masur (Lentil), government by 2023, comprising 40% for
Rapeseed/Mustard, Sunflower, Toria. Other crops are- the Central Government and 20% for the
copra, De-Husked Coconut, Jute and Sugarcane. State Government.
• Money, Banking and Tax System, 2. The Central Government has domestic
liabilities of 21% of GDP as compared to
Financial Relations of Centre & that of 49% of GDP of the State
State Governments.
51. Which one of the following best describes the 3. As per the Constitution of India, it is
term "Merchant Discount Rate" sometimes mandatory for a State to take the Central
seen in news? Government's consent for raising any loan
(a) The incentive given by a bank to a if the former owes any outstanding
merchant for accepting payments through liabilities to the latter.
debit cards pertaining to that bank. Which of the statements given above is/are
(b) The amount paid back by banks to their correct?
customers when they use debit cards for (a) 1 only (b) 2 and 3 only
financial transactions for purchasing goods or (c) 1 and 3 only (d) 1, 2 and 3
services. Ans. (c): According to the FRBM committee review
(c) The charge to a merchant by a bank for report headed by NK Singh, the combined debt to GDP
accepting payments from his customers ratio of the centre and states should be brought down to
through the bank's debit cards. 60 percent by 2023 (comprising 40% for the centre and
(d) The incentive given by the Government to 20 percent for states) against the existing 49.4% and
merchants for promoting digital payments by 21% respectively. Statement (1) is correct.
their customers through Point of Sale (PoS) • The Union Government has larger domestic liabilities
machines and debit cards. of 49.23% of GDP as compared to that of the state
Ans. (c): MDR is a fee charged from a merchant by a (21% of GDP). Statement (2) is incorrect.
bank for accepting payments from customers through • Under Article 293, a state may not without the consent
credit and debit cards in their establishments. MDR of the Government of India (GOI) raise any loan if
compensates the card-issuing bank, the leader who puts
the PoS terminal and payment gateways such as master there is still outstanding any part of a loan which has
card or visa for the services. been made to the state by the Government of India or
MDR charges are usually shared in pre-agreed by its predecessor government, or in respect of which
proportion between the bank and a merchant and are a guarantee has been given by the GOI or by its
expressed in percentage of the transaction amount. predecessor government. Statement (3) is correct.

IAS (Pre) GS 2018 Paper I 76 YCT


54. Consider the following statements: 57. Which one of the following statements
1. Capital Adequacy Ratio (CAR) is the correctly describes the meaning of legal tender
amount that banks have to maintain in the money ?
form of their own funds to offset any loss (a) The money which is tendered in courts of law
that banks incur if the account-holders fail to defray the fee of legal cases
to repay dues. (b) The money which a creditor is under
2. CAR is decided by each individual bank. compulsion to accept in settlement of his
Which of the statements given above is/are claims
correct? (c) The bank money in the form of cheques,
(a) 1 only (b) 2 only
drafts, bills of exchange, etc
(c) Both 1 and 2 (d) Neither 1 nor 2
(d) The metallic money in circulation in a
Ans. (a) : Capital Adequacy Ratio (CAR) is the ratio of
country
a banks capital in relation to its risk weighted assets and
current liabilities. It is decided by central banks and Ans. (b) : Legal tender is any official medium of
bank regulations to prevent commercial banks from payment recognized by law that can be used to
taking excess leverage and becoming insolvent in the extinguish a public or private debt or meet a financial
process. obligation. The national currency is legal tender in
55. Which one of the following links all the ATMs practically every country. A creditor is obligated to
in India? accept legal tender toward repayment of a debt.
(a) Indian Banks Association 58. Consider the following statements :
(b) National Securities Depository Limited 1. The Reserve Bank of India manages and
(c) National Payments Corporation of India services Government of India Securities
(d) Reserve Bank of India but not any State Government Securities.
Ans. (c): National Financial Switch (NFS) is the largest 2. Treasury bills are issued by the
network of shared Automated Teller Machines (ATMs) Government of India and there are no
in India. It was designed, developed and deployed by treasury bills issued by the State
the Institute for Development and Research in Banking Governments.
Technology (IDRBT) in 2004, with the goal of inter- 3. Treasury bills offer are issued at a discount
connecting the ATMs in the counting and facilitating from the par value.
convenience banking. Which of the statements given above is/are
It is run by the National payments corporation of India correct ?
(NCPI). NCPI is an umbrella organisation for all retail (a) 1 and 2 only (b) 3 only
payments in India. It was set up with the guidance and (c) 2 and 3 only (d) 1, 2 and 3
support of the Reserve Bank of India (RBI) and Indian Ans. (c): There are two types of bills viz. Treasury Bills
Banks Association (IBA). The correct option is (c). and commercial bills. Treasury Bills are issued by the
56. With reference to digital payments, consider central Government, commercial bill are issued by
the following statements: financial institutions.
1. BHIM app allows the user to transfer Reserve Bank of India maintains the Principal Accounts
money to anyone with a UPI-enabled bank of Central as well as State Government at its Central
account. Account Section, Nagpur. Treasury Bills are issued only
2. While a chip-pin debit card has four by the central government in India. The state
factors of authentication, BHIM app has governments do not issue any treasury bills. Interest on
treasury bills is determined by market forces. T-Bills
only two factors of authentication. are issued on discount to face value, while the holder
Which of the statements given above is/are gets the face value on maturity. The return on T-Bills is
correct? the difference between the issue price and face value.
(a) 1 only (b) 2 only When referring to the value of financial instruments,
(c) Both 1 and 2 (d) Neither 1 nor 2 There's no difference between par value and face value.
Ans. (a): Bharat interface for money (BHIM) is a 59. With reference to the governance of public
payment app that allows simple, easy and quick sector banking in India, consider the following
transactions using a unified payments interface (UPI). statements :
The money can be sent through direct bank payment
1. Capital infusion into public sector banks
to anyone on UPI using their UPI ID or scanning their
QR with the BHIM app statement (1) is correct. by the Government of India has steadily
• There are three levels of authentication that are increased in the last decade.
required in this app. One is the device ID and 2. To put the public sector banks in order, the
mobile number, then the bank account which you merger of associate banks with the parent
are linking to this app and finally the UPI pin, State Bank of India has been affected.
which is needed to complete the transaction, Which of the statements given above is/are
whereas chip-pin debit cards only has two factors of correct ?
authentication that is card and pin. Statement (2) is (a) 1 only (b) 2 only
incorrect. (c) Both 1 and 2 (d) Neither 1 nor 2
IAS (Pre) GS 2018 Paper I 77 YCT
Ans. (b) : Statement (1) is not correct as a capital (a) 1 and 2 only (b) 3 only
infusion into public sector banks by the government of (c) 1 and 3 only (d) 1, 2 and 3
India has not steadily increased in the last decade. Ans. (d): General relativity predicts radical things, such
Statement (2) is correct. The merger of SBI associated that light is deflected by gravity. It also predicts black
banks under section 35 of the State Bank of India Act, holes. It showed that space and time are malleable as
1955 will result in the creation of a stronger merged flexible, and ever- expanding, which basically predicted
entity. This will minimize vulnerability to any that our universe was ever expanding.
geographic concentration risks faced by subsidiary
banks. It will create improved operational efficiency • Biology
and economies of scale. it will also result in improved 63. Which of the following leaf modifications occur(s)
risk management and unified treasury operations. in the desert areas to inhibit water loss?
60. Consider the following items : 1. Hard and waxy leaves
1. Cereal grains hulled 2. Tiny leaves
2. Chicken eggs cooked 3. Thorns instead of leaves
3. Fish processed and canned Select the correct answer using the code given
4. Newspapers containing advertising material below:
Which of the above items is/are exempted (a) 2 & 3 (b) 2 only
under GST (Goods and Services Tax) ? (c) 3 only (d) 1, 2 & 3
(a) 1 only (b) 2 and 3 only Ans. (d): Leaf modifications of desert plant place to
(c) 1, 2 and 4 only (d) 1, 2, 3 and 4 minimize the water loss for adaptations and survival of
Ans. (c): Goods and Services Tax (GST) council has desert plant.
exempted cereal grains hulled and chicken eggs cooked. • Wax coating on leaves prevents water loss through
Newspapers containing advertising material : 0% GST evaporation, which in the hot desert can cause loss of
rate. However fish processes canned 5% GST. Hence water from both the surface and the inside of leaves.
statement 1,2,4 are correct. • Many cacti have spines in place of leaves, which
• Foreign Trade and Economic conduct photosynthesis and catch dew when the
climate is suitable. These small structures also reflect
Organization of India, Stock Exchange tiny light leaves, further reducing water loss. During
61. Consider the following statements: heavy rains, cacti will grow temporary root systems
1. The quantity of imported edible oils is and absorb water.
more than the domestic production of 64. Consider the following :
edible oils in the last five years. 1. Birds 2. Dust blowing
2. The Government does not impose any 3. Rain 4. Wind blowing
customs duty on all the imported edible oils Which of the above spread plant diseases?
as a special case. (a) 1 and 3 only (b) 3 and 4 only
Which of the statements given above is/are (c) 1, 2 and 4 only (d) 1, 2, 3 and 4
correct?
Ans. (d): Nematodes, snails, birds and wild and
(a) 1 only (b) 2 only
(c) Both 1 and 2 (d) Neither 1 nor 2 domestic animals after help dissemination of plant
diseases. The spores of many parasitic fungi are
Ans. (a): India occupies a prominent position in the disseminated by air currents from diseased to disease -
world oilseeds industry with the contribution of around free host pathogens like bacteria are often disseminated
10% in worldwide production. But the demand of edible by the splashing of raindrops, as in the case of citrus
oils (extracted from oilseeds in addition to palm oil) is canker disease.
significantly higher than the domestic production, Soil and field operations also disseminate the diseases
leading to dependence on imports (60% of
as they result in dust blowing.
requirement). Statement (1) is correct.
• Government imposes customs duty on edible oils to • Agriculture, Animal Husbandry
safeguard the interests of domestic oil crushing and Dairy
industry. The duty on two major edible oils, namely 65. With reference to the circumstances in Indian
crude sunflower seed oil and crude canola/ rapeseed/ agriculture, the concept of "Conservation
mustard is 25 percent. While crude soyabean oil Agriculture" assumes significance. Which of
attracts 30 percent duty. Statement (2) is not correct. the following fall under the Conservation
Agriculture?
SCIENCE 1. Avoiding the monoculture practices
• Physics 2. Adopting minimum tillage
3. Avoiding the cultivation of plantation
62. Consider the following phenomena :
1. Light is affected by gravity. crops
2. The Universe is constantly expanding. 4. Using crop residues to cover soil surface
3. Matter warps its surrounding space-time. 5. Adopting spatial and temporal crop
Which of the above is/are the prediction/ sequencing/crop rotations
predictions of Albert Einstein's General Select the correct answer using the code given
Theory of Relativity, often discussed in media ? below:
IAS (Pre) GS 2018 Paper I 78 YCT
(a) 1, 3 and 4 (b) 2, 3, 4 and 5 • Science & Technology
(c) 2, 4 and 5 (d) 1, 2, 3 and 5
Ans. (c) : Conservation Agriculture (CA) can be 68. What is/are the consequence/consequences of a
defined by a statement given by the Food and country becoming the member of the 'Nuclear
Agricultural Organization of the United Nation as "a Suppliers Group'?
concept for resource-saving agricultural crop production 1. It will have access to the latest and most
that strives to achieve acceptable profits together with efficient nuclear technologies.
high and sustained production levels while concurrently 2. It automatically becomes a member of
conserving the environment". The 3 principle of CA "The Treaty on the Non-Proliferation of
are: Nuclear Weapons (NPT)".
(i) Minimum tillage and soil disturbance. Which of the statements given above is/are
(ii) Permanent soil cover with crop residues and live correct?
mulches. (a) 1 only (b) 2 only
(iii) Crop rotation and intercropping. (c) Both 1 and 2 (d) Neither 1 nor 2
66. With reference to the Genetically Modified Ans. (a) : The Nuclear Supplier Group (NSG) is a group
mustard (GM mustard) developed in India, of nuclear supplier countries that seeks to contribute to
consider the following statements: the non-proliferation of nuclear weapons through the
implementation of two sets of guidelines for nuclear
1. GM mustard has the genes of a soil exports and nuclear- related exports. But becoming a
bacterium that give the plant the property member of the NSG will not automatically make a
of pest-resistance to a wide variety of pests. country a member of the treaty on the Non-proliferation
2. GM mustard has the genes that allow the of Nuclear Weapons (NPT).
plant cross-pollination and hybridization. The NSG is a multilateral export control regime and a
3. GM mustard has been developed jointly by group of nuclear supplier countries that seek to prevent
the IARI and Punjab Agricultural nuclear proliferation by controlling the export of
materials, equipment and technology that can be used to
University. manufacture nuclear weapons.
Which of the statements given above is/are Membership of the NSG enables access to technology
correct? for a range of uses, from medicine to building nuclear
(a) 1 and 3 only (b) 2 only power plants for India from the NSG, which is
(c) 2 and 3 only (d) 1, 2 and 3 essentially a traders cartel.
So, the correct answer is option (a).
Ans. (b): DMH -11 is a Genetically Modified (GM)
69. Consider the following statements:
mustard hybrid. Hybrids are normally obtained by 1. Aadhaar card can be used as a proof of
crossing two genetically diverse plants from the same citizenship or domicile.
species. The 1st generation offspring resulting from it 2. Once issued, Aadhaar number cannot be
has higher yields than what either of the parents is deactivated or omitted by the Issuing
individually capable of giving. But there is no natural Authority.
hybridization system in mustard, unlike in rice, cotton, Which of the statements given above is/are
maize or tomato. This is because its flowers contain correct?
both the female (pistil) and male (stamen) reproductive (a) 1 only (b) 2 only
organs, making the plant naturally self-pollinating. So (c) Both 1 and 2 (d) Neither 1 nor 2
the correct answer is option (b). Ans. (d): The Aadhaar (Targeted Delivery of Financial
and Other Subsidies, Benefits and Services) Bill 2016
COMPUTERS says that the number can't be used as proof of
citizenship or domicile.
67. The terms 'WannaCry, Petya and EternalBlue' Statement (1) is not correct.
sometimes mentioned in the news recently are • The Aadhaar Act regulations state that an individual
related to Aadhaar number may be "omitted" permanently or
(a) Exo-planets (b) Crypto-currency deactivated temporarily by the Unique Identification
(c) Cyber attacks (d) Mini satellites Authority of India, the agency responsible for issuing
the numbers and managing the database. The
Ans. (c): The Petya and Wanna Cry cyber-attacks in regulations give the authority the power to deactivate
May and June 2017; are two of the biggest in history Aadhaar numbers even in the absence of an effective
and impacted the finances of companies throughout the grievance redressed procedure for those whose
globe. Wanna Cry, which affected large numbers of numbers have been suspended.
organization, including the NHS, spread to 150 Statement (2) is not correct.
countries and is estimated to have cost the global 70. The identity platform 'Aadhaar' provides open
economy 6 billion euro. "Application Programming Interfaces (APIs)".
Eternal Blue an exploit developed by the U.S. National What does it imply?
Security Agency (NSA) in an older windows systems 1. It can be integrated into any electronic
released by the Shadow Brokers and was used as part of device.
the worldwide Wannacry Ransomware attack. 2. Online authentication using iris is possible.
IAS (Pre) GS 2018 Paper I 79 YCT
Which of the statements given above is/are Ans. (b): In order to transform the entire ecosystem of
correct? public services through the use of information
(a) 1 only (b) 2 only technology, the Government of India has launched the
(c) Both 1 and 2 (d) Neither 1 nor 2 Digital India programme with the vision to transform
Ans. (c) : The Aadhaar Act regulations, which are India into a digitally empowered society and
currently in parliament, state that an individual's knowledge economy. Under the Digital India Plan,
Aadhaar number may be "omitted" permanently or NII would integrate the network and cloud
deactivated temporarily by the Unique Identification infrastructure in the country to provide high-speed
Authority of India, the agency responsible for issuing connectivity and platform to various government
the numbers and managing the database. The regulations departments up to the panchayat level. Cities with a
gave the Authority the power to deactivate Aadhaar
numbers even in the absence of an effective grievance population of over 1 million and tourist centres could
redressal procedure for those whose numbers have been be provided with public wi-fi hotspots to promote
suspended. Calcutta High Court on 26th December, digital cities. The scheme would be implemented by
2016 ruled that Aadhaar card is not proof of citizenship DoT and the Ministry of Urban Development
and stated that "Aadhaar Card by itself should not (MoUD). Hence only statement (3) is correct.
confer any right of or be proof of citizenship or 73. What is "Terminal High Altitude Area Defence
domicile in respect of the holder there to" while (THAAD)", sometimes seen in the news ?
rejecting a claim of citizenship of the accused who was (a) An Israeli radar system
issued the card in view of his long residence in the (b) India's indigenous anti-missile programme
country. (c) An American anti-missile system
Hence, the correct answer is option (d) (d) A defence collaboration between Japan and
71. In the Indian context, what is the implication of South Korea
ratifying the 'Additional Protocol' with the Ans. (c) : Terminal High Altitude Area Defense
'International Atomic Energy Agency (IAEA)'? (THAAD), formerly Theater High Altitude Area
(a) The civilian nuclear reactors come under Defence, is an American anti-ballistic missile defense
IAEA safeguards. system designed to shoot down short, medium and
(b) The military nuclear installations come under intermediate-range ballistic missiles in their terminal
the inspection of IAEA. phase (descent or recently) by intercepting with a hit-
(c) The country will have the privilege to buy to-kill approach.
uranium from the Nuclear Suppliers Group 74. In which of the following areas can GPS
(NSG). technology be used ?
(d) The country automatically becomes a 1. Mobile phone operations
member of the NSG. 2. Banking operations
Ans. (a) : India has ratified the Additional Protocol, a 3. Controlling the power grids
commitment given under India-US nuclear deal by the Select the correct answer using the code given
previous dispensation, to grant greater ease to below:
International Atomic Energy Agency to monitor (a) 1 only (b) 2 and 3 only
India's civilian atomic programme. The IAEA had in (c) 1 and 3 only (d) 1, 2 and 3
March 2009 approved an additional protocol to India's Ans. (d): Global Positioning System (GPS) applications
safeguards agreement consequent to a pact reached generally fall into 5 major categories-
with the agency the previous year to place its civilian (i) Location - determining a position
nuclear facilities under IAEA safeguards. That (ii) Navigation - getting from one location to another
agreement had paved the way for the 45-number (iii) Tracking- monitoring object or personal movement.
Nuclear Suppliers Group to grant India-specific (iv) Mapping - creating maps of the world.
waiver for it to have commercial relations with other (v) Timing - bringing precise timing to the world.
countries in the civilian atomic field. GPS's accurate time facilities in everyday activities such
72. Which of the following is/are the aim/aims of as banking, mobile phone operations and even the
"Digital India" Plan of the Government of control power grids by allowing well synchronized
India ? hand-off switching.
1. Formation of India's own Internet companies 75. With reference to the Indian Regional
like China did. Navigation Satellite System (IRNSS), consider
2. Establish a policy framework to encourage the following statements :
overseas multinational corporations that 1. IRNSS has three satellites in geostationary
collect Big Data to build their large data and four satellites in geosynchronous orbits.
centres within our national geographical 2. IRNSS covers entire India and about 5500
boundaries. sq. km area beyond its borders.
3. Connect many of our villages to the Internet 3. India will have its own satellite navigation
and bring Wi-Fi to many of our schools, system with full global coverage by the
public places and major tourist centres. middle of 2019.
Select the correct answer using the code given Which of the statements given above is/are
below : correct ?
(a) 1 and 2 only (b) 3 only (a) 1 only (b) 1 and 2 only
(c) 2 and 3 only (d) 1, 2 and 3 (c) 2 and 3 only (d) None
IAS (Pre) GS 2018 Paper I 80 YCT
Ans. (a) : IRNSS is an independent regional navigation 78. When the alarm of your smart phone rings in
satellite system being developed by India. It is designed the morning, you wake up and tap it to stop the
to provide accurate position information service to the alarm which causes your geyser to be switched
user in India as well as the region extending upto 1500 on automatically. The smart mirror in your
km from its boundary, which is its primary service area. bathroom shows the day's weather and also
Statement (2) is incorrect. indicates the level of water in your overhead
• Out of 8 satellites, 3 satellites are located in suitable tank. After you take some groceries from your
orbital slots in the geostationary orbit and the refrigerator for making breakfast, it recognises
remaining 4 are located in geosynchronous orbit, with the shortage of stock in it and places an order
the required inclination and equatorial crossings in for the supply of fresh grocery items. When
two different planes- So, statement (1) is correct. you step out of your house and lock the door,
• There are plans to add more satellites to the IRNSS all lights, fans, geysers and AC machines get
and expand it to the SAARC region by 2019. So switched off automatically. On your way to
statement (3) is incorrect. office, your car warns you about traffic
76. Consider the following pairs : congestion ahead and suggests an alternative
Terms sometimes Context/Topic route, and if you are late for a meeting, it sends
seen in news a message to your office accordingly.
1. Belle II – Artificial Intelligence In the context of emerging communication
experiment technologies, which one of the following terms
2. Blockchain – Digital/Cryptocurrency best applies to the above scenario?
(a) Border Gateway Protocol
technology
(b) Internet of Things
3. CRISPR-Cas9 – Particle Physics (c) Internet Protocol
Which of the pairs given above is/are correctly (d) Virtual Private Network
matched ?
(a) 1 and 3 only (b) 2 only Ans. (b): The Internet of Things (IOT) is the network of
physical devices, vehicles, home appliances and other
(c) 2 and 3 only (d) 1, 2 and 3
items embedded with electronics, software, sensors and
Ans. (b): The Belle II experiment is a particle physics connectivity. They enable these devices to connect and
experiment designed to study the properties of B exchange data creating opportunities for more direct
mesons (heavy particles containing a bottom quark). It integration of the physical world into computer-based
is currently being commissioned at the Super KEKB systems resulting in efficiency improvements, economic
accelerator complex at KEK in Tsukuba, Ibaraki, benefits and reduced human intervention.
Prefecture, Japan. Often, these devices use Internet Protocol (IP) the same
Blockchain technology can be integrated into protocol that identifies computers over the world wide
multiple areas. The primary use of blockchains today web and allows them to communicate with one another.
is as a distributed ledger for cryptocurrencies, most 79. With reference to solar power production in
notably bitcoins. India, consider the following statements :
• CRISPR- Cas 9 is a genome- editing tool. It is short 1. India is the third largest in the world in the
for Clustered regularly interspaced short palindromic manufacture of silicon wafers used in
repeats and CRISPR-associated protein. photovoltaic units.
77. Which of the following statements best 2. The solar power tariffs are determined by
describes "carbon fertilization? the Solar Energy Corporation of India.
(a) Increased plant growth due to increased Which of the statements given above is/are
concentration of carbon dioxide in the correct ?
atmosphere (a) 1 only (b) 2 only
(b) Increased temperature of Earth due to (c) Both 1 and 2 (d) Neither 1 nor 2
increased concentration of carbon dioxide in Ans. (d): Not a single silicon chip for solar energy is
the atmosphere manufactured in India. Every solar panel which is made
(c) Increased acidity of oceans as a result of in India is assembled, while all the material comes from
increased concentration of carbon dioxide in China, Europe and some other countries. India is yet to
the atmosphere develop semiconductor clusters. Statement (1) is
(d) Adaptation of all living beings on Earth to incorrect.
the climate change brought about by the • CERC determines solar power tariffs and regulates the
increased concentration of carbon dioxide in tariff of generating companies owned or controlled by
the central Government.
the atmosphere
• SECI has a power trading license, but it does not set
Ans. (a): The carbon fertilization effects suggests that solar power tariffs.
the increase of carbon dioxide in the atmosphere 80. "3D printing" has applications in which of the
increases the rate of photosynthesis in plants. The effect following?
varies depending on the plant species, the temperature 1. Preparation of confectionery items
and the availability of water and nutrients. 2. Manufacture of bionic ears
A quarter to half of earth's vegetated lands has shown 3. Automotive industry
significant greening over the last 35 years, largely due 4. Reconstructive surgeries
to the rising level of atmospheric carbon dioxide. 5. Data processing technologies
IAS (Pre) GS 2018 Paper I 81 YCT
Select the correct answer using the code given ENVIRONMENT AND ECOLOGY
below :
(a) 1, 3 and 4 only (b) 2, 3 and 5 only 82. How is the National Green Tribunal (NGT)
(c) 1 and 4 only (d) 1, 2, 3, 4 and 5 different from the Central Pollution Control
Ans. (d) : 3D printing is being used in a wide range of Board (CPCB)?
industries, from construction to medical technology. 1. The NGT has been established by an Act
Some of the applications of 3D printing: where as the CPCB has been created by an
• Through 3D printing, we can create ornamental executive order of the Government.
prototypes, starting from construction drawings, as 2. The NGT provides environmental justice
well as mechanical plants. and helps reduce the burden of litigation in
• 3D printing of confectionery has focused primarily
the higher courts whereas the CPCB
on chocolate and hard sugar candy, and recent
innovations have led to the printing of gummy candy. promotes cleanliness of streams and wells,
• The 3D printed bionic ear that can "hear" radio and aims to improve the quality of air in
waves. the country.
• 3 D printing is Facial Reconstructive Surgery, people Which of the statements given above is/are
can now receive custom 3D-printed facial implants to correct?
help after illness or injuries. (a) 1 only (b) 2 only
• Multilateral voxel-printing method enabling the (c) Both 1 and 2 (d) Neither 1 nor 2
physical visualization of data sets commonly Ans. (b): The National Green Tribunal (NGT) has been
associated with scientific imaging. established on 18.10.2010 under the National Green
• It is also used in Maritime Industry, Electronics and Tribunal Act 2010 for effective and expeditious
Entertainment and Broadcasting etc. disposal of cases relating to environmental protection
81. With reference to India's satellite launch and conservation of forests and other natural resources,
vehicles, consider the following statements : including enforcement of any legal right relating to the
environment and giving relief and compensation for
1. PSLVs launch the satellites useful for damages to persons and property and for matters
Earth resources monitoring whereas connected therewith or incidental thereto.
GSLVs are designed mainly to launch The tribunal's dedicated jurisdiction in environmental
communication satellites. matters shall provide speedy environmental justice and
2. Satellites launched by PSLV appear to help reduce the burden of litigation in the high courts.
remain permanently fixed in the same The tribunal is mandated to make and endeavour for
position in the sky, as viewed from a disposal or applications or appeals finally within 6
particular location on Earth. months of filing of the same.
3. GSLV Mk III is a four-staged launch The CPCB statutory organisation was constituted in
vehicle with the first and third stages using September, 1974 under the Water (Prevention and
solid rocket motors; and the second and Control of Pollution) Act, 1974. Further, CPCB was
entrusted with the powers and functions under the Air
fourth stages using liquid rocket engines.
(Prevention and Control of Pollution) Act, 1981.
Which of the statements given above is/are
Functions of the CPCB, as spelt out in the water
correct ?
(Prevention and Control of Pollution) Act,1974, and the
(a) 1 only (b) 2 and 3
(c) 1 and 2 (d) 3 only Air (Prevention and Control of Pollution) Act, 1981 (i)
to promote cleanliness of streams and wells in different
Ans. (a) : PSLV is designed mainly to deliver the
"earth-observation" or "remote-sensing" satellites with a areas of the states by prevention, control and abatement
lift-off mass of up to about 1750 kg to Sun- of water pollution and (ii) to improve the quality of air
Synchronous circular polar orbits of 600-900 km and to prevent, control as abate air pollution in the
altitude. The GSLV is designed mainly to deliver the country. Hence, statement (1) is incorrect and statement
communication-satellites to the highly elliptical (2) is correct.
(typically 250×3,6000 km) Geosynchronous Transfer 83. Consider the following statements:
Orbit (GTO). The satellite in GTO is further raised to its 1. Most of the world's coral reefs are in
final destination viz, Geo-synchronous Earth Orbit tropical waters.
(GEO) of about 3,6000 km altitude (and zero degrees 2. More than one-third of the world's coral
inclination on the equatorial plane) by firing its in-built
reefs are located in the territories of
on-board engines. Due to their geo-synchronous nature
the satellites in these orbits appear to remain Australia, Indonesia and Philippines.
permanently fixed in the same position in the sky, as 3. Coral reefs host far more number of
viewed from a particular location on Earth, thus animal phyla than those hosted by tropical
avoiding the need for a tracking ground antenna and rainforests.
hence are useful for communication applications. Which of the statements given above is/are
GSLV MK-III is a three-stage heavy- lift launch vehicle correct?
developed by ISRO. The vehicle has two solid strap- (a) 1 and 2 only (b) 3 only
one, a core liquid booster and a cryogenic upper stage. (c) 1 and 3 only (d) 1, 2 and 3
IAS (Pre) GS 2018 Paper I 82 YCT
Ans. (d): The vast majority of large reefs created by in many parts of the world that may result in
corals in shallow waters (< 50 m water depth) and the loss of good native ecosystems.
located within a tropical zone located between 30ºN and (b) Fears of a possible collision of a meteorite
30ºS latitude with a preferred temperature range of with the Earth in the near future in the
approximately 22º to 29º C. Coral reefs are often called manner it happened 65 million years ago that
the "tropical rainforests of the sea" for their astounding caused the mass extinction of many species
richness of life. So, statement (1) and (2) are correct. including those of dinosaurs.
• Coral reefs are the most diverse communities on the (c) Large scale cultivation of genetically
planet. These tropical marine communities occupy modified crops in many parts of the world
less than 1% of the ocean floor but are inhabited by at and promoting their cultivation in other parts
least 25% of all marine species. Scientists estimate of the world which may cause the
that more than 25,000 described species from thirty- disappearance of good native crop plants and
two of the world's thirty- three animal phyla live in the loss of food biodiversity
reef habitats-four times the number of animal phyla (d) Mankind's over-exploitation/misuse of
found in tropical rain forests. So statement (3) is also
natural resources, fragmentation/loss of
correct.
natural habitats, destruction of ecosystems,
84. With reference to the 'Global Alliance for pollution and global climate change.
Climate-Smart Agriculture (GACSA)', which
Ans. (d) : The term "sixth mass extinction/ sixth
of the following statements is/are correct?
1. GACSA is outcome of the Climate Summit extinction in often referred to as the Holocene
held in Paris in 2015. Extinction, which is the ongoing extinction event of
2. Membership of GACSA does not create species during the present Holocene epoch, mainly as a
any binding obligations. result of human activity.
3. India was instrumental in the creation of • The five mass extinction that took place in the last
GACSA. 450 million years have led to the destruction of 70-95
Select the correct answer using the code given percent of the species of plants, animals and
below: microorganisms that existed earlier.
(a) 1 and 3 only (b) 2 only • These extinctions were caused by 'catastrophic'
(c) 2 and 3 only (d) 1, 2 and 3 alterations to the environment, such as massive
Ans. (b): The Global Alliance for Climate-Smart volcanic eruptions, depletion of oceanic oxygen or
Agriculture (GACSA) was launched on 23 September collision with an asteroid.
2014 at the UN Climate Summit. GACSA is an • A large number of extinctions spans numerous
independent alliance governed by its members through families of plants and animals, including mammals,
a strategic committee and its Co-chairs. Since the 1 birds, amphibians, reptiles and arthropods.
January, FAO is hosting the Facilitation unit of • The current extinction rate is approximately 100
GACSA which is supported through a multi-donor trust extinctions per million per year, or 1000 times higher
fund. than natural background rates.
GACSA is a voluntary alliance of partners dedicated to 86. The Partnership for Action on Green Economy
addressing the challenges facing food security and (PAGE), a UN mechanism to assist countries
agriculture under a changing climate. In particular, the transition towards greener and more inclusive
alliance has the objective of upscaling the climate- economies, emerged at:
smart agriculture approach a concept which were (a) The Earth Summit on Sustainable
originally developed by FAO. Development 2002, Johannesburg
GACSA's ambitious future is based upon supporting (b) The United Nations Conference on
and inspiring action. Farmers, fisherman, foresters, and Sustainable Development 2012, Rio de
ranches are at the centre of this action and therefore, Janeiro
GACSA devoting its Annual Forum to showcasing (c) The United Nations Framework Convention
climate-smart agriculture in action. on Climate Change 2015, Paris
The forum will be a dynamic gathering, where the (d) The World Sustainable Development Summit
participants share solutions, discuss challenges, and 2016, New Delhi
build partnerships on climate- smart agriculture. Ans. (b): The partnership for Action on Green
Seeking out action-oriented state holders to come to Economy (PAGE) was launched in 2013 as a response
both shares and learn at the GACSA annual forum. to the call at Rio+20 in 2012 in Rio de Janeiro to
85. The term "sixth mass extinction/sixth support those countries wishing to embark on greener
extinction" is often mentioned in the news in and more inclusive growth trajectories. The PAGE
seeks to put sustainability at the heart of economic
the context of the discussion of:
policymaking. The partnership supports nations and
(a) Widespread monoculture practices in regions in reframing economic policies and practices
agriculture and large-scale commercial around sustainability to foster economic growth, create
farming with indiscriminate use of chemicals income and jobs, reduce poverty and inequality and
IAS (Pre) GS 2018 Paper I 83 YCT
strengthen the ecological foundations of their 90. Consider the following statements:
economies. PAGE is a direct response to the Rio+20 1. The definition of "Critical Wildlife
Declaration. The Future We want which called upon the Habitat" is incorporated in the Forest
United Nations System and the international community Rights Act, 2006.
to provide assistance to interested countries in
developing adopting and implementing green economy 2. For the first time in India, Baigas have
policies and strategies. been given Habitat Rights.
87. Why is a plant called Prosopis juliflora often 3. Union Ministry of Environment, Forest
mentioned in news? and Climate Change officially decides and
(a) Its extract is widely used in cosmetics. declares Habitat Rights for Primitive and
(b) It tends to reduce the biodiversity in the area Vulnerable Tribal Groups in any part of
in which it grows. India.
(c) Its extract is used in the synthesis of Which of the statements given above is/are
pesticides. correct?
(d) None of the above (a) 1 and 2 only (b) 2 and 3 only
Ans. (b) : Prosopis juliflora is a shrub or small tree in (c) 3 only (d) 1, 2 and 3
the family Fabaceae a kind of mesquite. It is native to
Mexico, South America and the Caribbean. It is a Ans. (a): The phrase 'critical wildlife habitat' is defined
contributing factor to the continuing transmission of only in the Scheduled Tribes and Other Traditional
malaria, especially during dry periods when sugar Forest Dwellers (Recognition of Forest Rights) Act,
sources from native plants are largely unavailable to 2006. The statement is correct.
mosquitoes. • In a bid to undo historical injustice meted out to
This is not a big threat to the country's biodiversity but primitive tribal communities living in central India,
also has become a burden on the environment. This tree the government of Madhya Pradesh, for the first
is scattered in around 12 states of the country. It has time, recognized the habitat rights of seven villages
destroyed about 500 species of native plants. If it has in Dindori district mostly inhabited by the Baigas.
not been eliminated on time, it will destroy the Statement (2) is correct.
remaining bio-diversity of the country. Its roots are too • Under the act the District level committee shall
deep which absorbs a lot of water also. It was brought to answer that all particularly vulnerable Tribal Groups
India in 1870. receive habitat rights, in consultation with the
• It was included forage, wood and environmental concerned traditional intuitions of particularly
management. vulnerable Tribal Groups and their claims for habitat
88. "Momentum for Change : Climate Neutral rights are filed before the concerned Gram Sabha.
Now" is an initiative launched by. Statement (3) is not correct.
(a) The Intergovernmental Panel on Climate 91. With reference to organic farming in India,
Change consider the following statements:
(b) The UNEP Secretariat 1. 'The National Programme for Organic
(c) The UNFCCC Secretariat Production' (NPOP) is operated under the
(d) The World Meteorological Organization guidelines and directions of the Union
Ans. (c): The UNFCCC secretariat launched its Climate Ministry of Rural Development.
Neutral Now initiative in 2015. The following year, the 2. 'The Agricultural and Processed Food
secretariat launched a new pillar under its Momentum Products Export Development Authority
for change initiative focused on Climate Neutral Now as (APEDA) functions as the Secretariat for
part of larger efforts to showcase successful climate the implemention of NPOP.
action around the world. 3. Sikkim has become India's first fully
89. In which one of the following States is Pakhui organic State.
Wildlife Sanctuary located? Which of the statements given above is/are
(a) Arunachal Pradesh (b) Manipur correct?
(c) Meghalaya (d) Nagaland (a) 1 and 2 only (b) 2 and 3 only
Ans. (a): Pakhui is a wildlife sanctuary and a dedicated (c) 3 only (d) 1, 2 and 3
Tiger Reserve (also known as the Pakke Tiger Reserve) Ans. (b): Ministry of Commerce has implemented the
in the district of East Kameng in Arunachal Pradesh. It National Programme for Organic Production (NPOP)
has an area of over 860 square kilometers and is since 2001. Agricultural and Processed Food Products
bordered by the kameng River and Pakke River, which Export Development Authority (APEDA) functions as
are important sources of water for the animal and the the secretariat for the implementation of the NPOP.
irrigation of the vegetation. The Pakhui reserve is Sikkim has become India's first fully organic state, in
neighbour to the Nameri Tiger Reserve in Assam- 2016 by implementing organic practices on around
making this area a true treasure trove in terms of these 75000 hectares of agricultural land.
spectacular animals. So, correct answer is option (b).

IAS (Pre) GS 2018 Paper I 84 YCT


CURRENT AFFAIRS Ans. (d) : The Geographical Indications of Goods
(Registration and Protection) Act, 1999, coupled with
92. Consider the following pairs : the Geographical Indications of Goods (Registration
Regions sometimes Country and Protection) Rules, 2002 which became operational
mentioned in news with effect from 15 September 2003, was drafted as part
of the exercise in the country to set in place national
1. Catalonia – Spain IPR laves as much in compliance with India's
2. Crimea – Hungary obligations under the Agreement on Trade Related
3. Mindanao – Philippines Aspects of International Property Rights (TRIPS) of the
4. Oromia – Nigeria 'Minimum' standards of GI protection that the TRIPS
Which of the pairs given above are correctly sets out for the WTO members to comply with in their
mateched ? respective national legislations.
(a) 1, 2 and 3 (b) 3 and 4 only 97. Consider the following countries :
(c) 1 and 3 only (d) 2 and 4 only 1. Australia 2. Canada
Ans. (c) : Correct match is- 3. China 4. India
5. Japan 6. USA
(1) Catalonia – Spain Which of the above are among the 'free-trade
(2) Crimea – Soviet Union (Ukraine) partners' of ASEAN ?
(3) Mindanao – Philippines (a) 1, 2, 4 and 5 (b) 3, 4, 5 and 6
(c) 1, 3, 4 and 5 (d) 2, 3, 4 and 6
(4) Oromia – Ethiopia Ans. (c): The Association of Southeast Asian Nations,
So, the correct answer is option (c) or ASEAN, was established on 8 August, 1967 in
93. Very recently, in which of the following Bangkok, Thailand, with the signing of the ASEAN
countries have lakhs of people either suffered Declaration (Bangkok Declaration) by the Founding
from severe famine/acute malnutrion or died fathers of ASEAN, namely Indonesia, Malaysia,
due to starvation caused by war/ethnic conflicts? Philippines, Singapore and Thailand.
(a) Angola and Zambia ASEAN has existing free trade agreements with
(b) Morocco and Tunisia Australia, China, India, Japan, South Korea and New
Zealand.
(c) Venezuela and Colombia 98. Consider the following pairs :
(d) Yemen and South Sudan Towns sometimes Country
Ans. (d) : The world faces one of the largest food crises mentioned in news
in 70 years with 20 million people in four countries 1. Aleppo Syria
namely: Yemen, Nigeria, South Sudan and Somalia, 2. Kirkuk Yemen
according to FAO. 3. Mosul Palestine
94. The term "two-state solution" is sometimes 4. Mazar-i-sharif Afghanistan
mentioned in the news in the context of the Which of the pairs given above are correctly
matched ?
affairs of: (a) 1 and 2 (b) 1 and 4
(a) China (b) Israel (c) 2 and 3 (d) 3 and 4
(c) Iraq (d) Yemen Ans. (b) : Correct match is-
Ans. (b): Two-state Solution of Israel Palestine Issue: Aleppo – Syria
It envisages an independent state of Palestine Kirkuk – Iraq
alongside the state of Israel west of the Jordan River. Mosul – Iraq
The UNSC Resolution 1937 agreed in 2000 with Mazar-i-sharif – Afghanistan
support from the USA and becoming the first UNSC 99. If a commodity is provided free to the public
resolution to agree on two- state solution. by the Government, then
(a) the opportunity cost is zero.
(b) the opportunity cost is ignored.
MISCELLANEOUS (c) the opportunity cost is transferred from the
consumers of the product to the tax-paying
95. "Rule of Law Index" is released by which of public
the following? (d) the opportunity cost is transferred from the
(a) Amnesty International consumers of the product to the Government.
(b) International Court of Justice Ans. (c) : Opportunity cost represents the benefits an
(c) The Office of UN Commissioner for Human
Rights individual investor or business misses out on when
(d) World Justice Project choosing one alternative over another.
Ans. (d): The World Justice Project Rule Index When any have the opportunity to access public
measures rule of law adherence in 113 countries and services for free, this would always come at the cost of
somebody paying for it. In this case, the taxpayers bear
jurisdictions based on more than 1,10,000 households the opportunity cost.
and 3,000 expert surveys. Furthering primary data, the 100. Consider the following pairs :
WJP Rule of Law Index measures countries rule of law Craft Heritage of
performance across eight factors : constraints on 1. Puthukkuli shawls – Tamil Nadu
Government Powers, Absence of Corruption, Open 2. Sujni embroidery – Maharashtra
Government, Fundamental Rights Order and Security, 3. Uppada Jamdani saris – Karnataka
Regulatory Enforcement, Civil Justice and Criminal Which of the pairs given above is/are correct ?
Justice. (a) 1 only (b) 1 and 2
96. India enacted The Geographical Indications of (c) 3 only (d) 2 and 3
Goods (Registration and Protection) Act, 1999 Ans. (a) : Correct matches are–
in order to comply with the obligations to: Puthukkuli shawls – Tamil Nadu
(a) ILO (b) IMF Sujni embroidery – Bihar
(c) UNCTAD (d) WTO Uppada Jamdani saris – Andhra Pradesh
IAS (Pre) GS 2018 Paper I 85 YCT
UNION PUBLIC SERVICE COMMISSION
Civil Services (Preliminary Exam) - 2017
GENERAL STUDIES : PAPER-I
Time : 2 hours (Exam date : 18.06.2017) Maximum Number : 200

ANCIENT HISTORY • Buddhism, Jainism, Bhagavata and


• Vedic Civilization Shaiva Religion
2. With reference to the religious history of India,
1. With reference to the difference between the consider the following statements:
culture of Rigvedic Aryans and Indus Valley 1. Sautrantika and Sammitiya were the sects of
Jainism.
people, which of the following statements is/are 2. Sarvastivadin held that the constituents of
correct? phenomena were not wholly momentary, but
existed forever in a latent form.
1. Rigvedic Aryans used the coat of mail and
Which of the statements given above is/are
helmet in warfare whereas the people of Indus correct?
Valley Civilization did not leave any evidence (a) 1 only (b) 2 only
(c) Both 1 and 2 (d) Neither 1 nor 2
of using them.
Ans : (b) Statement 1 is not correct:
2. Rigvedic Aryans knew gold, silver and copper Sautrantika and Sammitya were the sects of Buddhism.
whereas Indus Valley people knew only Statement 2 is correct:
Sarvastivadins (they who say "All is real") had the
copper and iron. view that the constituents of phenomena (dharma)
3. Rigvedic Aryans had domesticated the horse were not wholly momentary but existed forever in a
latent form.
whereas there is no evidence of Indus Valley 3. The painting of Bodhisattva Padmapani is one
people having been aware of this animal. of the most famous and oft–illustrated
paintings at
Select the correct answer using the code given (a) Ajanta (b) Badami
below: (c) Bagh (d) Ellora
(a) 1 only Ans : (a) The painting is on the back wall of the
interior hall before the shrine antechamber in cave
(b) 2 and 3 only no.1 at Ajanta, dating back to the late fifth century
(c) 1 and 3 only CE. The Boddhisattva is holding a Padma (lotus), has
large shoulders and has their bents in the body,
(d) 1, 2 and 3 creating a movement in the picture space. The
modelling is soft. The figure of the Boddhisattva is
Ans : (c) Statement (1) is correct- Rig-Vedic Aryans wearing a big crown in which detailed rendering is
used the coat of mail and helmet in warfare, whereas visible.
the people of Indus Valley Civilization did not leave 4. Which of the following is/are famous for Sun
temples?
any evidence of using them.
l. Arasavalli
Statement (2) is not correct- Various materials were 2. Amarakantak
used in Harappan Civilization to make beads, stones 3. Omkareshwer
Select the correct answer using the code given
like carnelian jasper crystal, quartz and steatite, metals below:
like copper, bronze and gold and shell artefacts and (a) 1 only (b) 2 and 3 only
terracotta or burnt clay. Some beads were made of two (c) 1 and 3 only (d) 1, 2 and 3
Ans : (a) The famous Sun God Temple Situated in
or more stones cemented together, some of stone with Arasavalli village, which is at a distance of about 1 km
gold caps. East of Srikakulam town district headquarters of the
North coastal Andhra Pradesh. It is one of the ancient
Statement (3) is correct : Rigvedic Aryans had and all among two sun- god temples in our country.
domesticated the horse whereas there is no evidence of According to Padmapuranam, Sage Kasyapa installed
indus valley people have been aware of this annimal. the idol of Surya at Arasavalli for the welfare of
mankind.
IAS (Pre) GS 2017 Paper I 86 YCT
MEDIEVAL HISTORY Veeraraghava chariar, G. Subramania Iyer and P.
Anandacharlu established the Madras Mahajana
5. Which one of the following was a very Sabha, Gazula Lakshminarasu Chetty was an Indian
important seaport in the Kakatiya kingdom? merchant Indian independence activist who founded
(a) Kakinada the Madras Native Association. Indian Association
(b) Motupalli was the first declared Nationalist Organization found
(c) Machlipatnam (Masulipatnam) in British India by Surendranath Banerjee and Anand
(d) Nelluru Mohan Bose in 1876.
Ans : (b) Motupalli was the chief port of the Kakatiyas • Major Institutions, Treaties
and this port was visited by the Venetian traveller,
Marco Polo. Some texts suggest that Motupalli, now in Commissions, Acts
the Krishna district, was an important seaport in the 8. The Trade Disputes Act of 1929 provided for
Kingdom of Ganpathi. (a) the participation of workers in the
management of industries.
MODERN HISTORY (b) arbitrary powers to the management to quell
industrial disputes.
• Social, Cultural Awakening, Low (c) an intervention by the British Court in the
Caste, Trade Union & Peasants event of a trade dispute.
(d) a system of tribunals and a ban on strikes.
Movement
Ans : (d) Trade Disputes Act (TDA), 1929 made
6. Who among the following was/were associated compulsory the appointment of courts of inquires and
with the introduction of Ryotwari Settlement in consultation boards for setting industrial disputes,
India during the British rule? Made illegal the strikes in public utility services like
1. Lord Cornwallis posts, railways, water and electricity unless each
2. Alexander Read individual worker planning to go on strike gave an
3. Thomas Munro advance notice of one month to the administration.
Select the correct answer using the code given Forbade trade union activity of coercive or purely
below: political nature and even sympathetic strikes.
(a) 1 only (b) 1 and 3 only 9. Consider the following statements :
(c) 2 and 3 only (d) 1, 2 and 3 1. The Factories Act, 1881 was passed with a
Ans : (c) In the British territories in the south, there view to fix the wages of industrial workers
was a similar move away from the idea of Permanent and to allow the workers to form trade unions.
settlement. The new system that was devised came to be 2. N.M. Lokhande was a pioneer in organizing
known as the Ryotwar (or Ryotwari). It was tried on a the labour movement in British India.
small scale by Captain Alexander Read in the some of Which of the above statements is/are correct?
the areas that were taken over by the company after the (a) 1 only (b) 2 only
wars with the Tipu sultan. Subsequently developed by (c) Both 1 and 2 (d) Neither 1 nor 2
Thomas Munro, this system was gradually extended all Ans : (b) Statement 1 : It is wrong.
over south India. Lord Cornwallis was associated with The 1881 Act aimed to improve the working
Zamindari/Permanent statement. conditions of labour. It prohibited the employment of
7. Consider the following pairs: children under the age of seven, limited the number of
1. Radhakanta Deb working hours for children below the age of twelve
First President of the British Indian and required that dangerous machinery should be
Association fenced properly.
2. Guzulu Lakshminarasu Chetty Statement 2- Lokhande was a pioneer of the labour
Founder of the Madras Mahajana Sabha movement in India. He is remembered not only for
3. Surendranath Banerjee ameliorating the working conditions of textile mill
hands in the 19th century but also for his courageous
Founder of the Indian Association
initiative on caste and communal issues.
Which of the above pairs is/are correctly matched?
10. The object of the Butler Committee of 1927
(a) 1 only (b) 1 and 3 only
was to
(c) 2 and 3 only (d) 1, 2 and 3 (a) Define the jurisdiction of the Central and
Ans: (b) The British Indian Association was Provincial Governments.
established on 31 October 1851. The first committee (b) Define the powers of the Secretary of State
of the association was composed of: for India.
Raja Radhakanta Deb- President Raja Kalikrishan (c) Impose censorship on national press.
Dev-Vice President, Debendranath Tagore- Secretary, (d) Improve the relationship between the
Digambar Mitra- Asst. Secretary. In May 1884, M. Government of India and the Indian States.
IAS (Pre) GS 2017 Paper I 87 YCT
Ans : (d) A three member Committee headed by (a) 1 and 2 only (b) 3 only
Harcourt Butler appointed on 16th December 1927, to (c) 2 and 3 only (d) 1 and 3 only
examine the relationship between the Indian states and Ans : (b) Statement 1 is not correct : In India, the
British India. Himalayas are spread over almost all the Northern and
It gave the following recommendations: northeastern Indian states.
• Paramountcy must remain supreme. Statement 2 is not correct : Western Ghats are spread
• States should not be handed over to an Indian over six states namely- Gujarat, Maharashtra, Goa,
government in British India, responsible to an Karnataka, Kerala & Tamil Nadu.
Indian legislature. Statement 3 is correct- Pulicat, the second largest
• But it could be done with the consent of states. brackish water lake in India, lies on the border of
• It must fulfill its allegation, adopting and defining Andhra Pradesh and Tamil Nadu. Over 40% of it lies in
itself according to the shifting necessities of time Andhra Pradesh and 60% in Tamil Nadu.
and progressive development of states. 14. At one of the places in India, if you stand on
The viceroy was made the crown's agent in dealing the seashore and watch the sea, you will find
with states.
that the sea water recedes from the shore line a
• Freedom Struggle & National few kilometres and comes back to the shore,
twice a day, and you can actually walk on the
Movement sea floor when the water recedes. This unique
11. With reference to Indian freedom struggle, phenomenon is seen at–
consider the following events: (a) Bhavnagar (b) Bheemunipatnam
1. Mutiny in Royal Indian Navy (c) Chandipur (d) Nagapttinam
2. Quit India Movement launched Ans : (c) Chandipur is located in the Balasore district
3. Second Round Table Conference (Odisha). This beach is unique in the whole world, no
What is the correct chronological sequence of where on earth you can find a beach where the sea
the above events? water recedes inside the sea from 1 km to 5 km every
(a) 1–2–3 (b) 2–1–3
day and it again comes back to the shore slowly during
(c) 3–2–1 (d) 3–1–2
high tide. This happens twice every day. It is also
Ans : (c) Second Round Table conference took place on known as Odisha's hide and seek beach. Bhavnagar has
7th September 1931. highest tidal range in India.
The Quit India Movement is also known as the August
movement, was a civil disobedience movement 15. With reference to river Teesta, consider the
launched by Gandhi Ji on 8th August 1942. Royal Indian following statements:
Navy (RIN) mutiny was a rebellion launched on 1. The source of river Teesta is the same as that
February 18, 1946 by Seamen on the HMIS Talwar. of Brahmaputra but it flows through Sikkim.
Hence, the correct chronological sequence of the above 2. River Rangeet originates in Sikkim and it is a
event is 3-2-1. tributary of river Teesta.
3. River Teesta flows into Bay of Bengal on the
GEOGRAPHY OF INDIA border of India and Bangladesh.
Which of the statements given above is/are
• Physical Structure, Drainage correct?
System and Human Geography (a) 1 and 3 only (b) 2 only
(c) 2 and 3 only (d) 1, 2 and 3
12. Which of the following is geographically closest
to Great Nicobar? Ans : (b) Statement 1 and 3: Teesta river originates
(a) Sumatra (b) Borneo from the Pahunri glacier. The Brahmaputra originates in
(c) Java (d) Sri Lanka the Angsi glacier. Teesta is a tributary of Brahmaputra.
So, statement 1 is wrong. But Teesta flows through west
Ans : (a) Great Nicobar is the Southernmost and Bengal and Sikkim before going to Bay of Bengal
largest of the Nicobar Islands of India, north of through Bangladesh. Hence statement 3 is incorrect.
Sumatra. The Island of Sumatra is located 1192 kms to
Statement 2 : Rangeet is a tributary of the Teesta river,
the south of Great Nicobar.
which is the largest river in Sikkim. It also originates in
13. Consider the following statements: Sikkim.
1. In India, the Himalayas are spread over five
States only. • Monsoon, Forest, Soil, Irrigation
2. Western Ghats are spread over five States Projects and Agriculture
only. 16. With reference to 'Indian Ocean Dipole (IOD)'
3. Pulicat Lake is spread over two States only. sometimes mentioned in the news while
Which of the statements given above is/are forecasting Indian monsoon, which of the
correct? following statements is/are correct?
IAS (Pre) GS 2017 Paper I 88 YCT
1. IOD phenomenon is characterised by a 2. IONS is a voluntary initiative that seeks to
difference in sea surface temperature between increase maritime co–operation among navies
tropical Western Indian Ocean and tropical of the littoral states of the Indian Ocean
Eastern Pacific Ocean. Region.
2. An IOD phenomenon can influence an El Which of the above statements is/are correct?
Nino's impact on the monsoon. (a) 1 only (b) 2 only
Select the correct answer using the code given (c) Both 1 and 2 (d) Neither 1 nor 2
below: Ans : (b) The Indian Ocean Naval Symposium (IONS)
(a) 1 only (b) 2 only is a voluntary initiative that seeks to increase maritime
(c) Both 1 and 2 (d) Neither 1 nor 2 co-operation among navies of the littoral states of the
Ans : (b) Statement 1 is not correct: The Indian Ocean Indian Ocean Region by providing an open & inclusive
Dipole is an irregular oscillation of sea-surface forum for discussion of regionally relevant maritime
temperatures in which the western Indian ocean issues.
becomes alternately warmer and then colder than the There are 35 members- navies of the IONS, which have
eastern part of the Ocean. been geographically grouped into the following four sub
Statement 2 is correct : An IOD can either aggravate or regions : South Asian Literates, West Asian Littorals,
weaken the impact of EL NINO on Indian Monsoon. If East African Littorals, South East Asian and Australian
there is a positive IOD. It can bring good rains to India Littorals.
despite an EL Nino year. The inaugural IONS-2008 was held in New Delhi, India
on 14 February 2008. CNS, Indian Navy was
• Transport designated the chairman of IONS for the period 2008-
17. If you travel by road from Kohima to 10. The theme of the IONS-2008 was contemporary
Kottayam, what is the minimum number of Trans- national challenges-International maritime
States within India through which you can connectivity. Hence statement 2 is correct and statement
travel, including the origin and the 1 is not correct.
destination?
(a) 6 (b) 7 INDIAN POLITY & CONSTITUTION
(c) 8 (d) 9
Ans : (b) The shortest route from Kohima (Nagaland)
• Constitutional Development of India
to Kottayam (Kerala) passes through the following 20. Which one of the following is not a feature of
states : Nagaland- Assam-West Bengal- Odisha- Indian federalism?
Andhra Pradesh- Tamil Nadu-Kerala. (a) There is an independent judiciary in India.
(b) Powers have been clearly divided between
WORLD GEOGRAPHY the Centre and the States.
(c) The federating units have been given unequal
• Hydrosphere representation in the Rajya Sabha.
18. Mediterranean Sea is a border of which of the (d) It is the result of an agreement among the
following countries? federating units.
1. Jordan 2.Iraq Ans : (d) The phrase 'Union of states' has been
3. Lebanon 4.Syria preferred to 'federation of states' for two reasons: one,
Select the correct answer using the code given the Indian federation is not the result of an agreement
below: among the states like the American Federation, and
(a) 1, 2 and 3 only (b) 2 and 3 only two, the states have no right to secede from the
(c) 3 and 4 only (d) 1, 3 and 4 only federation.
The federation is a union because it is indestructible.
Ans : (c) The countries with coastlines on the
The country is an integral whole and divided into
Mediterranean sea are Albania, Algeria, Bosnia &
different states only for the convenience of
Herzegovina, Croatia, Cyprus, Egypt, France, Greece,
Israel, Italy, Lebanon, Libya, Malta, Morocco, Monaco, administration.
Montenegro, Slovenia, Spain, Syria, Tunisia and 21. In the context of Indian history, the principle
Turkey. In addition, the Gaza strip and the British of 'Dyarchy (diarchy)' refers to
overseas Territories of Gibraltar and Akrotiri and (a) Division of the central legislature into two
Dhekelia have coastlines on the sea. On the Eastern houses.
shore, Turkey, Syria, Lebanon and Israel have their (b) Introduction of double government i.e.,
coastline on the Mediterranean Sea. Central and State governments.
19. Consider the following in respect of Indian (c) Having two sets of rulers; one in London and
Ocean Naval Symposium (IONS): another in Delhi.
1. Inaugural IONS was held in India in 2015 (d) Division of the subjects delegated to the
under the chairmanship of the Indian Navy. provinces into two categories.
IAS (Pre) GS 2017 Paper I 89 YCT
Ans : (d) Lord Chelmsford the viceroy and Edwin • Citizenship, Fundamental Rights
Montagu, the Secretary of state, in 1918, introduced
constitutional reforms which led to the enactment of Fundamental Duties, Directive
the Government of India Act of 1919. Principles of State Policies
• The provincial legislative councils were 25. Right to vote and to be elected in India is a
enlarged and the majority of their members were
to be elected. (a) Fundamental Right (b) Natural Right
• Under the system of Diarchy the provincial (c) Constitutional Right (d) Legal Right
governments were given more powers. Ans : (d) According to Supreme Court judgment Jyoti
• Under Diarchy some subjects, such as finance Basu Vs Devi Ghosal- A right to elect fundamental
and law and order remained under the direct though it is to democracy, is, anomalously neither a
control of the governor and were called reserved fundamental right nor a common law Right. It is a
subjects. statutory right. So is the right to be elected and the right
• Other subjects such as education public health, to dispute and election.
by ministers responsible to the legislatures and Note: As per NCERT one of the important decisions of
were called 'transferred' subject. the framers of the Indian Constitution was to guarantee
22. Which one of the following objectives is not every adult citizen in India, the right to vote i.e.
embodied in the Preamble to the Constitution universal adult franchise. Article 326 of the Constitution
of India? provides for the right to vote and to be elected in India.
(a) Liberty of thought (b) Economic liberty What is true of the right to vote is also true of the right
(c) Liberty of expression (d) Liberty of belief to contest the election. All citizens have the right to
Ans : (b) The preamble secures to all citizens of India contest for election and become the representative of the
liberty of thought, expression, belief, faith and worship people.
through their fundamental rights, enforceable in a court 26. Which of the following are envisaged by the
of law, in case of violation. The ideals of liberty, Right against Exploitation in the Constitution
equality and fraternity in our preamble have been taken of India?
from the French Revolution (1789-1799).
1. Prohibition of trafficking in human beings
23. The mind of the makers of the Constitution of and forced labour
India is reflected in which of the following?
2. Abolition of untouchability
(a) The Preamble
3. Protection of the interests of minorities
(b) The Fundamental Rights
4. Prohibition of employment of children in
(c) The Directive Principles of State Policy
factories and mines
(d) The Fundamental Duties
Select the correct answer using the code given
Ans : (a) In the Berubari Union case (1960) the below:
Supreme Court Said that the preamble shows the
(a) 1, 2 and 4 only (b) 2, 3 and 4 only
general purposes behind the several provisions in the
constitution and is thus a key to the minds of the (c) 1 and 4 only (d) 1, 2, 3 and 4
makers of the constitution. Ans : (c) Article 23 prohibits trafficking in human
24. Democracy's superior virtue lies in the fact beings beggar (forced labour) and other similar forms
that it calls into activity of forced labour. Any contravention of this provision
(a) the intelligence and character of ordinary men shall be an offence punishable in accordance with the
and women. law.
(b) the methods for strengthening executive Article 24 prohibits the employment of children below
leadership. the age of 14 years in any factory mines or other
(c) a superior individual with dynamism and hazardous activities like construction of works or
vision. railways. But is does not prohibit their employment in
(d) a band of dedicated party workers. any harmless or innocent work.
Ans : (a) According to John Stuart Mill, the objective 27. One of the implications of equality in society is
of government as per his views, being the attainment the absence of
of values and virtue in men, their self development. It (a) Privileges (b) Restraints
is essential that the government should be of those
(c) Competition (d) Ideology
whose self development is being sought. The voting
right in democracy allows the citizen to exercise Ans : (a) The term equality means the absence of
his/her choice and ensures adherence of moral values special privileges to any section of the society and the
or character. Such as justice and equality. So, option provision of adequate opportunities for all individuals
(a) is the correct answer. without any discrimination.

IAS (Pre) GS 2017 Paper I 90 YCT


28. Which one of the following statements is Ans : (d) It is the fundamental rights that constitute
correct? limitations upon state action (whether legislative or
(a) Rights are claims of the State against the executive). The directive principle is in the nature of
citizens instruments of instruction to the government of the day
(b) Rights are privileges which are incorporated to achieve certain ends by their actions. It guides them,
in the Constitution of a State. not restrict them.
(c) Rights are claims of the citizens against the Emergency
State. 32. Which of the following are not necessarily the
(d) Rights are privileges of a few citizens against consequences of the proclamation of the
the many. President's rule in a State?
Ans : (c) Rights are legal and moral entitlements or 1. Dissolution of the State Legislative Assembly
claims of a person over other fellow beings over 2. Removal of the Council of Ministers in the
society and over the government. State
3. Dissolution of the local bodies
29. Which of the following statements is/are true of
Select the correct answer using the code given
the Fundamental Duties of an Indian citizen? below:
1. A legislative process has been provided to (a) 1 and 2 only (b) 1 and 3 only
enforce these duties. (c) 2 and 3 only (d) 1, 2 and 3
2. They are correlative to legal duties.
Ans : (b) When the President rule is imposed in a
Select the correct answer using the code given state, the president dismisses the state council of
below: ministers headed by the chief minister. The state
(a) 1 only (b) 2 only governor, on behalf of the president carries on the state
(c) Both 1 and 2 (d) Neither 1 nor 2 administration with the help of the chief secretary of
state or the advisors appointed by the president. This is
Ans : (d) There is no legislative process provided to
the reason why a proclamation under Article 356 is
enforce the fundamental duties and they are not popularly known as the imposition of 'President' Rule'
correlative to legal duties. Like the directive principles, in a state. Further the president either suspends or
the fundamental duties are also non-justiciable. The dissolves the state legislative assembly. The parliament
constitution does not provide for their direct passes the state legislative bills and the state budget.
enforcement by the courts. Moreover, there is no legal The president does not lead to the dissolution of local
sanction against their violation. bodies. Hence, the answer is 1 & 3 only.
30. In the context of India, which one of the Parliament
following is the correct relationship between
33. The main advantage of the parliamentary form
Right and Duties? of government is that
(a) Rights are correlative with Duties. (a) the executive and legislature work
(b) Rights are personal and hence independent of independently.
society and Duties. (b) it provides continuity of policy and is more
(c) Rights, not Duties, are important for the efficient.
advancement of the personality of the citizen. (c) the executive remains responsible to the
(d) Duties, not Rights, are important for the legislature.
stability of the State. (d) the head of the government cannot be
changed without election.
Ans : (a) Rights are what we want others to do for us
whereas the duties are those acts which we should Ans : (c) The parliamentary system is also known as
the Westminister model of government a responsible
perform for others. Thus a right comes with an obligation
government and cabinet government. The constitution
to show respect for the rights of others. The obligations establishes the parliamentary system not only at the
that accompany rights are in the form of duties. centre but also in the states. The features of
31. Consider the following statements: parliamentary government in India are :-
With reference to the Constitution of India, the (a) Presence of nominal and real executives;
Directive Principles of State Policy constitute (b) Majority party rule,
limitations upon (c) Collective responsibility of the executive to the
1. legislative function. legislature.
(d) Membership of the ministers in the legislature,
2. executive function.
(e) Leadership of the Prime minister or the chief
Which of the above statements is/are correct? minister.
(a) 1 only (b) 2 only (f) Dissolution of the lower house (Loksabha or
(c) Both 1 and 2 (d) Neither 1 nor 2 Assembly)
IAS (Pre) GS 2017 Paper I 91 YCT
• Legislature : Council of Ministers Ans : (c) Qualification for election to Loksabha:
• Must be a citizen of India.
34. Consider the following statements:
• Not less than 25 years of age.
1. In the election for Lok Sabha or State
• Must be registered as an elector for a
Assembly, the winning candidate must get at parliamentary constituency
least 50 percent of the votes polled, to be
declared elected. 37. With reference to the Parliament of India,
consider the following statements::
2. According to the provisions laid down in the
Constitution of India, in Lok Sabha, the 1. A private member's bill is a bill presented by
Speaker's post goes to the majority party and a Member of Parliament who is not elected
but only nominated by the President of India.
the Deputy Speaker's to the Opposition.
2. Recently, a private member's bill has been
Which of the statements given above is/are passed in the Parliament of India for the first
correct? time in its history.
(a) 1 only (b) 2 only Which of the statements given above is/are
(c) Both 1 and 2 (d) Neither 1 nor 2 correct?
Ans : (d) Statement 1 is not correct. In the election for (a) 1 only (b) 2 only
Loksabha and state assembly, the first past the post (c) Both 1 and 2 (d) Neither 1 nor 2
system is followed where a candidate who polls more Ans : (d) A private member's bill is introduced by any
votes than any other candidates is elected. member of parliament other than a minister. Its
Statement 2 is not correct. As per the Constitution the introduction in the house required one month's notice.
speaker and deputy speaker in Loksabha are elected Its drafting is the responsibility of the member
among its members. concerned. The last private member's bill passed by
35. Out of the following statements, choose the one parliament was the Supreme Court (Enlargement of
Criminal Appellate Jurisdiction) Bill, 1968, which
that bring out the principle underlying the
became an Act on August 9, 1970. The Rights of
Cabinet form of Government: Transgender Persons Bill, 2014, passed by the Rajya
(a) An arrangement for minimizing the criticism Sabha on Friday, is the first private members' bill to
against the Government whose responsibilities get the upper house approval in the past 45 years.
are complex and hard to carry out to the
38. The Parliament of India exercises control over
satisfaction of all. the functions of the Council of Ministers
(b) A mechanism for speeding up the activities through
of the Government whose responsibilities are 1. Adjournment motion
increasing day by day. 2. Question hour
(c) A mechanism of parliamentary democracy 3. Supplementary questions
for ensuring collective responsibility of the Select the correct answer using the code given
Government to the people. below:
(d) A device for strengthening the hands of the (a) 1 only (b) 2 and 3 only
head of the Government whose hold over the (c) 1 and 3 only (d) 1, 2 and 3
people is in a state of decline. Ans : (d) The parliament Exercises Control over the
Ans : (c) Under Cabinet form of government, the ministers through various devices like question hour,
cabinet is responsible to the parliament. The system is discussions, adjournment motion no-confidence
also known as 'Westminster' model of government and motion, etc.
collective responsibility implies that ministers are
bound by the decision of the cabinet even when they • Judiciary (Central & State)
had no part in their discussion or decision. Secondly, 39. In India, Judicial Review implies:
all members of the government can speak in the (a) the power of the Judiciary to pronounce upon
parliament. Unless the prime minister relieves them of the constitutionality of laws and executive
that duty. orders.
36. For election to the Lok Sabha, a nomination (b) the power of the Judiciary to question the
wisdom of the laws enacted by the
paper can be filed by
Legislatures.
(a) anyone residing in India.
(c) the power of the Judiciary to review all the
(b) a resident of the constituency from which the legislative enactments before they are
election is to be contested. assented to by the President.
(c) any citizen of India whose name appears in (d) the power of the Judiciary to review its own
the electoral roll of a constituency. judgments given earlier in similar or different
(d) any citizen of India. cases.
IAS (Pre) GS 2017 Paper I 92 YCT
Ans : (a) Judicial review is a process under which Ans : (b) The 42nd Amendment Act of 1976 added
executive and legislative action are subject to review by four new Directive Principles to the original list. They
the judiciary. The power of courts to assess whether a require the state–
law is in compliance with the constitution. A court with (i) To secure opportunities for healthy development
judicial review power may invalidate laws and of children (Article 39A)
decisions that are incompatible with a higher authority, (ii) To promote equal justice and to provide free legal
an executive decision may be invalidated for being aid to the poor (Article 39A)
unlawful or a statue may be invalidated for violating the (iii) To take steps to secure the participation of workers
terms of a written constitution. Judicial review is one of in the management of Industries (Article 43A).
the checks and balances in the separation of powers, the (iv) To protect & improve the environment and to
power of the judiciary to supervise the legislative and safeguard forests and wildlife (Article 48 A).
executive branches when the latter exceed their 42. Local self–government can be best explained as
authority. The doctrine varies between jurisdictions, so an exercise in
the procedure and scope of judicial review may differ (a) Federalism
between and within countries. (b) Democratic decentralization
(c) Administrative delegation
• Election Commission, Official (d) Direct democracy
Language Ans :(b) The main purpose of democratic
40. Consider the following statements: decentralization however is to bring fundamental
changes in the traditional outlook about the power
1. The Election Commission of India is a five– structure of the government. Thus democratic
member body. decentralization means decentralization of powers. The
2. Union Ministry of Home Affairs decides the source from which this power is decentralized in based
election schedule for the conduct of both on the democratic structure and hence, such
general elections and bye–elections. decentralization is called the democratic
3. Election Commission resolves the disputes decentralization.
relating to splits/mergers of recognised
political parties. ECONOMY
Which of the statement given above is/are
correct? • Economic Planning, National
(a) 1 and 2 only (b) 2 only Income/ Budget
(c) 2 and 3 only (d) 3 only 43. Consider the following statements:
Ans : (d) Election Commission of India is a 3 member 1. Tax revenue as a percent of GDP of India has
body, comprising one chief Election commissioner and steadily increased in the last decade.
2 other Election commissioners. It is the Election 2. Fiscal deficit as a percent of GDP of India
commission which decides the election schedule for has steadily increased in the last decade.
both general elections bye-elections. It is also the Which of the statements given above is/are
function of election commission to resolve the disputes correct?
relating to splits/mergers of recognised political (a) 1 only (b) 2 only
parties. (c) Both 1 and 2 (d) Neither 1 nor 2
Ans : (d) Service tax, personal income tax and
• Panchayati Raj System, corporation tax have been reduced in the recent past to
Constitutional Amendments and boost aggregate demand. So as to protect the economy
from the global recession. Excise duty rates have been
Schedules reduced in the recent past to boost aggregate demand.
41. Which principle among the following was So as to protect past from boosting the economy from
added to the Directive Principles of State global recession hence tax revenue as a percent of GDP
Policy by the 42nd Amendment to the of India has steadily not increased in the last decade.
Constitution? Hence statement 1 is not correct. The Fiscal Deficit as a
(a) Equal pay for equal work for both men and percent of GDP of India has not steadily increased in
women the last decade. Hence statement 2 is not correct.
(b) Participation of workers in the management 44. Which of the following has/have occurred in
of industries India after its liberalization of economic
(c) Right to work, education and public policies in 1991?
1. Share of agriculture in GDP increased
assistance
enormously.
(d) Securing living wage and human conditions 2. Share of India's exports in world trade
of work to workers increased.
IAS (Pre) GS 2017 Paper I 93 YCT
3. FDI inflows increased. Which of the statements given above is/are
4. India's Foreign exchange reserves increased correct?
enormously. (a) 1 only (b) 2 only
Select the correct answer using the codes given (c) Both 1 and 2 (d) Neither 1 nor 2
below : Ans : (c) Statement 1 : NPCI is an umbrella
(a) 1 and 4 only organisation for all retail payments system in India.
(b) 2, 3 and 4 only
(c) 2 and 3 only Thus an important objective of NPCI is to facilitate an
(d) 1, 2, 3 and 4
affordable payment mechanism to benefit the common
Ans : (b) Statement 2, 3 and 4 in question are true after
man across the country and help financial inclusion.
liberalization of economic policies 1991 in India share
Statement 2 : Rupay is a domestic card payment
of India's exports in world trade increased and FDI
network that provides an alternative to international
inflows as well as foreign exchange reserves increased
card schemes. It was introduced by NPCI.
whereas statement 1 is false because the share of 47. What is/are the most likely advantages of
agriculture in GDP is decreased and not increased. implementing 'Goods and Services Tax
While the share of agriculture in GDP was 23% in the (GST)'?
year 2000, the share of agriculture in GDP has come 1. It will replace multiple taxes collected by
down to 17% in the year 2015. At present (2016-17), multiple authorities and will thus create a
the share of agriculture and allied sector in various single market in India.
sectors in GVA is 17.77% at constant prices and 2. It will drastically reduce the 'Current Account
19.41% at prevailing prices. The share of exports in Deficit' of India and will enable it to increase
foreign trade in the year 2016-17 and 2017-2018 is its foreign exchange reserves.
3. It will enormously increase the growth and
276280.3 (+5.33%) and 302840 (9.78%) respectively.
size of economy of India and will enable it to
Foreign direct investment (FDI) equity inflows are as overtake China in the near future.
follows: Select the correct answer using the code given
2017–18 (April-December) – 35.941 billion dollar below:
2016–17 – 43.478 billion dollar (a) 1 only (b) 2 and 3 only
2015–16 – 46.403 billion dollar (c) 1 and 3 only (d) 1, 2 and 3
2014–15 – 39.328 billion dollar
Ans : (a) GST is one indirect tax for the whole nation
2013–14 which will make India one unified common market.
– 28.75 billion dollar
Hence statement 1 is correct.
As of April 13, 2018, the total foreign currency assets
are 400.978 billion dollar. There is no information which says GST will drastically
reduce the CAD of India. Neither it is true that it will
45. With reference to 'National Investment and enable India to take over china. Hence, statement 2 and
Infrastructure Fund', which of the following
3 are not correct.
statements is/are correct? 48. Which of the following statements is/are
1. It is an organ of NITI Aayog. correct regarding the Monetary Policy
2. It has a corpus of `4,00,000 crore at present. Committee (MPC)?
Select the correct answer using the code given 1. It decides the RBI's benchmark interest rates.
below: 2. It is a 12–member body including the
(a) 1 only (b) 2 only Governor of RBI and is reconstituted every
(c) Both 1 and 2 (d) Neither 1 nor 2 year.
Ans : (d) National Investment and Infrastructure Fund 3. It functions under the chairmanship of the
(NIIF) is the fund created by the government of India Union Finance Minister.
Select the correct answer using the code given
for enhancing infrastructure financing in the country. Its
creation was announced in the union budget 2015-16. below :
Governing Council of NIIF is an advisory body chaired (a) 1 only (b) 1 and 2 only
by the finance minister. It is not an organ of NITI (c) 3 only (d) 2 and 3 only
Ans : (a) The Reserve Bank of India Act, 1934 (RBI
Aayog. Hence statement 1 is not correct. The proposed
corpus of NIIF is Rs. 40,000 crores (around USD 6 Act) has been amended by the Finance Act, 2016 to
provide for a statutory and institutionalized frame work
billion). Hence, statement 2 is also not correct. for a monetary policy committee it would be entrusted
• Money/ Banking & Tax System, with the task of fixing the benchmark policy rate (repo
rate) required to contain inflation within the specified
Financial Relation of Center- State target level. Hence statement 1 is correct. As per the
46. Consider the following statements: provisions of the RBI Act, out of the six members of the
1. National Payments Corporation of India Monetary Policy Committee, three members will be
from the RBI and the other three members of MPC will
(NPCI) helps in promoting the financial be appointed by the central government. Hence
inclusion in the country. statement 2 is not correct. The Governor of the Bank-ex
2. NPCI has launched RuPay, a card payment office chairperson of MPC. Hence statement 3 is not
scheme. correct.
IAS (Pre) GS 2017 Paper I 94 YCT
49. What is the purpose of setting up of Small 52. The Global Infrastructure Facility is a/an
Finance Banks (SFBs) in India? (a) ASEAN initiative to upgrade infrastructure in
1. To supply credit to small business units Asia and financed by credit from the Asian
2. To supply credit to small marginal farmers Development Bank.
3. To encourage young entrepreneurs to set up (b) World Bank collaboration that facilitates the
business particularly in rural areas. preparation and structuring of complex
Select the correct answer using the code given infrastructure Public–Private Partnerships
below:
(PPPs) to enable mobilization of private
(a) 1 and 2 only (b) 2 and 3 only
sector and institutional investor capital.
(c) 1 and 3 only (d) 1, 2 and 3
(c) Collaboration among the major banks of the
Ans : (a) Small Finance banks are a type of niche bank world working with the OECD and focused
in India. Banks with a small finance bank license can
provide basic banking services of acceptance of deposit on expanding the set of infrastructure
and lending. The objective of setting up small finance projects that have the potential to mobilize
banks is to further financial inclusion by the provision private investment.
of savings vehicles and supply credits to small business (d) UNCTAD funded initiative that seeks to
units, small and marginal farmers, micro and small finance and facilitate infrastructure
industries and other unorganized sector entities through development in the world.
high technology-low cost operations, However apart
from the provision of credit there is no explicit Ans : (b) The Global Infrastructure Facility (GIF) was
provision that SFBs have been setup to encourage launched by the World Bank to cater to the
setting of business in rural areas. Hence, statement 3 is Infrastructure needs of countries. It is a global open
incorrect. platform that facilitates the preparation and structuring
of complex infrastructure public- private partnerships
• Foreign Trade & Economic (PPPs) to enable mobilization of the private sector and
Organization of India, Stock Market instructional investor capital. The GIF platform
50. 'Broad-Based Trade and Investment Agreement coordinates and integrates the efforts of Multilateral
(BTIA)' is sometimes seen in the news in the Development Banks (MDBs), private sector investors
context of negotiations held between India and and financiers and governments interested in
(a) European Union infrastructure investment in Emerging Markets and
(b) Gulf Cooperation Council Developing Economies (EMDEs).
(c) Organization for Economic Cooperation and
Development SCIENCE
(d) Shanghai Cooperation Organization
Ans : (a) Negotiations for a free trade pact, formally • Physics
called a broad- based trade and investment agreement 53. What is the purpose of 'evolved Laser
(BTIA), between the EU and India, started in 2007, but Interferometer Space Antenna (eLISA)' project?
were unofficially suspended in 2013 when both sides (a) To detect neutrinos
realized that no progress was happening on contentious
matters. (b) To detect gravitational waves
EU has raised concern over what is called India's (c) To detect the effectiveness of missile defence
"unilateral termination" of existing Bilateral Investment system
Treaties (BITs) with a significant number of "EU (d) To study the effect of solar flares on our
member countries." communication systems
51. Consider the following statements: Ans : (b) The Laser Interferometer Space Antenna
1. India has ratified Trade Facilitation (Spectacular plan of setting into space three Spacecraft)
Agreement (TFA) of WTO. is a European Space Agency mission designed to detect
2. TFA is a part of WTO's Bali Ministerial and accurately measure gravitational waves tiny ripples
Package of 2013. in the fabric of space time-from astronomical sources.
3. TFA came into force in January 2016. LISA would be the first dedicated space-based
Which of the statement given above is/are gravitational wave detector.
correct?
(a) 1 and 2 only (b) 1 and 3 only • Chemistry
(c) 2 and 3 only (d) 1, 2 and 3 54. Consider the following pairs:
Ans : (a) WTO members concluded negotiations at the Commonly used/consumed materials
2013 Bali Ministerial conference on the landmark Trade Unwanted or controversial chemicals likely to
Facilitation Agreement (TFA), which entered into force be found in them
on February 22, 2017 following its ratification by two- 1. Lipstick – Lead
thirds of the WTO membership. India has ratified the 2. Soft drinks – Brominated
Trade Facilitation Agreement (TFA) of the World Trade vegetable oils
Agreement (WTO) and the instrument of acceptance for
Trade Facilitation Agreement was handed over to WTO 3. Chinese – Monosodium
Director- General by India on April 26, 2016. fast food glutamate
IAS (Pre) GS 2017 Paper I 95 YCT
Which of the pairs given above is/are correctly Ans : (c) In genetics and developmental biology,
matched? Somatic Cell Nuclear Transfer (SCNT) is a laboratory
(a) 1 only (b) 2 and 3 only strategy for creating a viable embryo from a body cell
(c) 1 and 3 only (d) 1, 2 and 3 and an egg cell. The technique consists of taking an
enucleated oocyte (egg cell) and implanting a donor
Ans : (d) Lead is a common impurity found in cosmetic
nucleus from a somatic (body) cell. It is used in both
lip products, Such as lipsticks and other cosmetics, such therapeutic and reproductive cloning. Dolly the sheep
as eye shadows, blushes, compact powders, shampoos became famous for being the first successful case of the
and body lotions. reproductive cloning of a mammal.
Brominated Vegetable oil is a food additive sometimes 57. Consider the following statements:
used to keep citrus flavouring from separating out in the 1. In tropical regions, Zika virus disease is
some sodas and soft drinks. It is banned in Europe transmitted by the same mosquito that
against use as a food additive in Europe and Japan. transmits dengue.
Monosodium Glutamate (MSG) is a Glutamate or salt 2. Sexual transmission of Zika virus disease is
of Glutamic acid, a "non-essential" amino acid. It is a possible.
neurotransmitter- transporting messages from one nerve Which of the statements given above is/are
cell to another. Glutamic acid is found naturally in correct?
tomatoes, grapes, cheese, mushrooms and other foods. (a) 1 only (b) 2 only
It is commonly used in Chinese fast food. (c) Both 1 and 2 (d) Neither 1 nor 2
55. Organic Light Emitting Diodes (OLEDs) are Ans : (c) Statement 1 : Zika Virus is transmitted to
used to create digital display in many devices. people primarily through the bite of an infected Aedes
What are the advantages of OLED displays species mosquito (Ae. aegypti and Ae. albopictus ).
over Liquid Crystal displays? These are the same mosquitoes that spread dengue and
1. OLED displays can be fabricated on flexible chikunguniya viruses. Hence, it is correct.
plastic substrates. Statement 2: Zika can be passed through sex from a
2. Roll–up displays embedded in clothing can be person who has Zika to his or her partners. Zika can be
made using OLEDs. passed through sex even if the infected person does not
have symptoms at the time. Hence, it is correct.
3. Transparent displays are possible using
OLEDs • Agriculture, Animal Husbandry &
Select the correct answer using the code given Dairy
below:
58.
With reference to agriculture in India, how can
(a) 1 and 3 only
the technique of 'genome sequencing', often
(b) 2 only seen in the news, be used in the immediate
(c) 1, 2 and 3 future?
(d) None of the above statements is correct 1. Genome sequencing can be used to identify
Ans : (c) A major advantage of OLED over LCD is that genetic markers for disease resistance and
it is lightweight and has flexible plastic substrates. drought tolerance in various crop plants.
OLED displays can be fabricated on flexible plastic 2. This technique helps in reducing the time
Substrates leading to the possible fabrication of flexible required to develop new varieties of crop
organic light-emitting diodes for other new plants.
applications, such as roll-up displays embedded in 3. It can be used to decipher the host–pathogen
fabrics or clothing. If a substrate like polyethylene relationships in crops.
terephthalate (PET) can be used, the displays may be Select the correct answer using the code given
produced inexpensively; furthermore, plastic substrates below:
are shatter- resistant unlike the glass displays used in (a) 1 only (b) 2 and 3 only
LCD devices. (c) 1 and 3 only (d) 1, 2 and 3
Ans : (d) All the statements are correct. Genome
• Medical Science, Zoology, Sequencing is figuring out the orders of DNA
Anthropology nucleotides or bases in a genome-the order of As, Cs,
Gs, and Ts that make up an organism's DNA. Genome
56. What is the application of Somatic Cell Nuclear Sequencing of wide varities of plants can be used to
Transfer Technology? identify disease resistance and drought tolerance genes
(a) Production of biolarvicides in various plants and develop new varieties of crop
(b) Manufacture of biodegradable plastics plants in lesser time. Genome sequencing of crop plants
(c) Reproductive cloning of animals can be helpful in deciphering and understanding the
(d) Production of organisms free of diseases host- pathogen relationship in crops.

IAS (Pre) GS 2017 Paper I 96 YCT


ENVIRONMENT & ECOLOGY 2. It provides technical and financial support to
targeted developing countries to integrate
59. The term 'M-STrIPES' is sometimes seen in climate change into their development
the news in the context of policies and budgets.
(a) Captive breeding of Wild Fauna 3. It is coordinated by World Resources Institute
(b) Maintenance of Tiger Reserves (WRI) and World Business Council for
(c) Indigenous Satellite Navigation System Sustainable Development (WBCSD).
(d) Security of National Highways Select the correct answer using the code given
below:
Ans : (b) Monitoring system for tiger's Intensive
(a) 1 and 2 only (b) 3 only
protection and Ecological status is a software
(c) 2 and 3 only (d) 1, 2 and 3
monitoring system launched by the Indian Government
in 2010 in some tiger reserves to reduce the Ans : (a) The GCCA was established by the European
vulnerability of Tigers. The system would enable field Union (EU) in 2007 to strengthen dialogue and
managers to assist intensity and Spatial Coverage of cooperation with developing countries, in particular,
patrols in a geographic information system (GIS) least developed countries (LDCs) and small island
domain. developing states (SIDS). Statement 1 is correct.
The GCCA also provides technical & financial support
60. In India, if a species of tortoise is declared to targeted developments countries to integrated climate
protected under Schedule I of the Wildlife change into their development policies and budgets
(Protection) Act, 1972, what does it imply? Statement 2 is correct.
(a) It enjoys the same level of protection as the Under the second pillar, the GCCA+ acts as a source of
tiger. technical and financial support for the worlds’ climate -
(b) It no longer exists in the wild, a few vulnerable countries whose populations need climate
individuals are under captive protection and finance the most. International organizations, notably
now it is impossible to prevent its extinction. the centre for International Forestry Research.
(c) It is endemic to a particular region of India. World Resources Institute (WRI) and the World
(d) Both (b) and (c) stated above are correct in Business Council for Sustainable Development
this context. (WBCSD) are not partners of GCCA; other institutions
Ans : (a) A number of wild animal and bird species like FAO, UNDP etc. are part of it. Therefore Statement
have been listed in various schedules of the Indian 3 is incorrect.
wildlife (protection) Act, 1972. The Act provides for the 63. Biological Oxygen Demand (BOD) is a
protection of wild animals, birds and plants and for standard criterion for
matters connected there with or ancillary there to. It has (a) Measuring oxygen levels in blood
six schedules which give varying degrees of protection. (b) Computing oxygen levels in forest
Animals under schedule I of the Wildlife Act enjoy the ecosystems
highest protection in India since the tiger is also covered (c) Pollution assay in aquatic ecosystems
in schedule I; therefore, option (a) is correct. (d) Assessing oxygen levels in high altitude
61. Recently there was a proposal to translocate regions
some of the lions from their natural habitat in Ans : (c) Biochemical Oxygen Demand (BOD), also
Gujarat to which one of the following sites? called Biological oxygen demand, is the amount of
(a) Corbett National Park dissolved oxygen needed (i.e. demanded) by the aerobic
(b) Kuno Palpur Wildlife Sanctuary biological organism to break down organic material
(c) Mudumalai Wildlife Sanctuary present in a given water sample at a certain temperature
(d) Sariska National Park over a specific time period. The BOD value is most
commonly expressed in milligrams of oxygen
Ans : (b) An environmental ministry expert consumed per litre of sample during 5 days of
committee has approved Kuno Palpur in Madhya incubation at 20ºC and is often used as a gauge of the
Pradesh as the second home for Asiatic lions found effectiveness of waste water treatment plants. It is
only in Gir National Park. hyped as a conventional pollutant in the US clean water
In 2013 Supreme Court ruled in favours of Act.
translocation of the Asiatic lions to Kuno in the 64. With reference to the role of UN–Habitat in the
interest of the genetic stability of the species. United Nations programme working towards a
However, the Gujarat Government will not share lions better urban future, which of the statements
unless 33 studies as mandated by international wildlife is/are correct?
watchdog IUCN is completed. l. UN–Habitat has been mandated by the
62. With reference to 'Global Climate Change United Nations General Assembly to promote
Alliance', which of the following statements socially and environmentally sustainable
is/are correct? towns and cities to provide adequate shelter
1. It is an initiative of the European Union. for all.
IAS (Pre) GS 2017 Paper I 97 YCT
2. Its partners are either governments or local (a) Sathyamangalam Tiger Reserve
urban authorities only. (b) Nallamala Forest
3. UN–Habitat contributes to the overall (c) Nagarhole National Park
objective of the United Nations system to (d) Seshachalam Biosphere Reserve
reduce poverty and to promote access to safe
Ans : (a) Sathyamangalam Tiger Reserve:- It is
drinking water and basic sanitation.
located in the Erode district of Tamil Nadu and is part
Select the correct answer using the code given
of Western Ghats and Eastern Ghats. Hence the option 1
below:
is correct.
(a) 1, 2 and 3 (b) 1 and 3 only
Sathyamangalam forest range is an important wildlife
(c) 2 and 3 only (d) 1 only
corridor in Niligiri Biosphere Reserve.
Ans : (b) UN-Habitat is the United Nations programme It is a genetic link between the four other protected
working towards a better urban future. It is mandated by areas which it adjoins including the Billigiriranga
the UN General Assembly in 1978 and aims to promote
Swamy Temple wildlife sanctuary Sigur plateau,
socially and environmentally sustainable human
Mudumalai National Park, and Bandipur National Park.
settlements development and the achievement of
adequate shelter for all. Through drawing together Nallamala Forest: It is located in Kodagu district and
cooperation with committed partners, relevant Mysore district of Karnataka and is also called Rajiv
stakeholders, and urban actors including at all levels of Gandhi National park.
government as well as the private sector, UN-Habitat is Seshachalam Biosphere Reserve:
applying its technical expertise to the New Urban It is located in the Southern region of Andhra Pradesh
Agenda and sustainable Development. So, statement 2 in the Eastern Ghats. This reserve is a habitat of famous
is incorrect. red sanders and slender lories species.
65. If you want to see gharials in their natural 68. Consider the following statements in respect of
habitat, which one of the following is the best Trade Related Analysis of Fauna and Flora in
place to visit? Commerce (TRAFFIC):
(a) Bhitarkanika Mangroves 1. TRAFFIC is a bureau under United Nations
(b) Chambal River Environment Programme (UNEP).
(c) Pulicat Lake 2. The mission of TRAFFIC is to ensure that
(d) Deepor Beel trade in wild plants and animals is not a threat
Ans : (b) National Chambal Sanctuary, also called the to the conservation of nature.
National Chambal Gharial wildlife sanctuary, is a tri Which of the above statements is/are correct?
state- protected area in northern India for the critically (a) 1 only (b) 2 only
endangered Gharial (small crocodiles), the red crowned (c) Both 1 and 2 (d) Neither 1 nor 2
roof turtle and the endangered Ganges river dolphin. Ans : (b) TRAFFIC was established in 1976 by IUCN
66. Which of the following practices can help in and WWF, it is a wildlife trade monitoring network, and
water conservation in agriculture? the leading non-governmental organisation working
1. Reduced or zero tillage of the land globally on trade in wild animals and plants in the
2. Applying gypsum before irrigating the field context of biodiversity conservation and sustainable
3. Allowing crop residue to remain in the field development. TRAFFIC is governed by the TRAFFIC
Select the correct answer using the code given committee, a steering group composed of members of
below: TRAFFIC’S partner organizations WWF and IUCN. It
(a) 1 and 2 only (b) 3 only is not a bureau under UNEP.
(c) 1 and 3 only (d) 1, 2 and 3 69. In the context of solving pollution problems,
Ans : (c) 1. No till farming (also called zero tillage or what is/are the advantage/advantages of
direct drilling) is a way of growing crops or pasture bioremediation technique?
from year to year without disturbing the soil through 1. It is a technique for cleaning up pollution by
tillage. enhancing the same biodegradation process
2. Major benefits of gypsum are: Source of calcium and that occurs in nature.
sulfur for plant nutrition, Improves Saline and alkaline 2. Any contaminant with heavy metals such as
soils and treats aluminum toxicity, improves soil cadmium and lead can be readily and
structure. Hence statement 2 is incorrect. completely treated by bioremediation using
3. Residue from the crops lie on the surface of the field microorganisms.
cooling it and increasing the moisture. 3. Genetic engineering can be used to create
67. From the ecological point of view, which one of microorganisms specifically designed for
the following assumes importance in being a bioremediation.
good link between the Eastern Ghats and the Select the correct answer using the code given
Western Ghats? below :
IAS (Pre) GS 2017 Paper I 98 YCT
(a) 1 only (b) 2 and 3 only Ans : (b) Statement 1 is incorrect. Productions of algal
(c) 1 and 3 only (d) 1, 2 and 3/1, 2 and 3 biofuel are possible both in seas and on continents.
Ans : (c) Bioremediation uses naturally occurring They can grow marginal or non-crop land also on
organisms to break down hazardous substances into less brackish or polluted water. Land based systems are
toxic substances. It uses microorganisms to degrade more developed than sea- based systems.
organic contaminations in soil, groundwater, sludge and Statement 2 and 3 are correct. Developing and
solids. The microorganisms break down contaminate by engineering ABB technology requires a high level of
using them as an energy source or co-metabolizing them expertise until constriction is finished. Innovation for
with an energy source. higher productivity also requires the same knowledge
Not all contaminates are easily treated by and/or experience. All algal based biofuel concepts
bioremediation using microorganisms. For example require significant capital investment. Access to this
heavy metals such as cadmium and lead are not readily technology by the poor may be largely difficult. Scale
absorbed or captured by microorganisms. facilities are more economically viable but are also
Genetic engineering has been used to create organisms more likely to have higher social and ecological
designed for specific purposes. For e.g. bacterium impacts.
Deinococcus radiodurans (the most radiation resistant 72. In the context of mitigating the impending
organism known) has been modified to consume highly global warming due to anthropogenic
radioactive nuclear waste. emissions of carbon dioxide, which of the
following can be the potential sites for carbon
70. Due to some reasons, if there is a huge fall in sequestration?
the population of species of butterflies, what 1. Abandoned and uneconomic coal seams
could be its likely consequence/consequences? 2. Depleted oil and gas reservoirs
1. Pollination of some plants could be adversely 3. Subterranean deep saline formations
affected. Select the correct answer using the code given
2. There could be a drastic increase in the fungal below:
infections of some cultivated plants. (a) 1 and 2 only (b) 3 only
3. It could lead to a fall in the population of (c) 1 and 3 only (d) 1, 2 and 3
some species of wasps, spiders and birds. Ans : (d) Carbon sequestration is the process involved
Select the correct answer using the code given in carbon capture and the long-term storage of
below: atmospheric carbon dioxide. Geological sequestration
(a) 1 only (b) 2 and 3 only involves the storage of CO2 underground in depleted oil
(c) 1 and 3 only (d) 1, 2 and 3 and gas reservoirs, saline formations or deep, unmixable
Ans : (c) Statement 1 and 3 are correct. Butterflies are coal beds.
pollinating insects. They help in pollination of many 73. The term 'Domestic Content Requirement' is
flowering plants. Butterfly also act as a lower member sometimes seen in the news with reference to
of the food chain. A number of animal, including birds (a) Developing solar power production in our
and mice feed on butterfly. (As population of butterfly country
diminish, so will populations of birds. This is “butterfly (b) Granting licenses to foreign T.V. channels in
effect”. Statement 2 is not correct as it does no lead to our country
increase in the fungal infections of some cultivated (c) Exporting our food products to other
plants. countries
71. It is possible to produce algae based biofuels, but (d) Permitting foreign educational institutions to
what is/are the likely limitation(s) of developing set up their campuses in our country
countries in promoting this industry? Ans : (a) The term 'Domestic Content Requirement' is
1. Production of algae based biofuels is possible sometimes seen in the news with reference to
in seas only and not on continents. developing solar power production in our country.
2. Setting up and engineering the algae based • It was instituted in the Jawaharalal Nehru National
biofuel production requires high level of solar mission
expertise/technology until the construction is
National Solar Mission-
completed.
• The National Solar Mission is also known as the
3. Economically viable production necessitates Jawharlal Nehru National Solar Mission.
the setting up of large scale facilities which • It was adopted by India in 2010.
may raise ecological and social concerns.
• It aims to target the generation of 100,000 MW of
Select the correct answer using the code given grid connected Solar power capacity by 2022.
below: • A mandatory domestic content requirement was
(a) 1 and 2 only (b) 2 and 3 only imposed on solar power developers participating in
(c) 3 only (d) 1, 2 and 3 phase I and phases II.
IAS (Pre) GS 2017 Paper I 99 YCT
74. According to the Wildlife (Protection) Act, schools. The volunteer’s responsibility is towards the
1972, which of the following animals cannot be overall development of the child, not academics. The
hunted by any person except under some volunteer service will be used in developing skills like
provisions provided by law? public speaking, creative writing counselling, music and
1. Gharial dance.
2. Indian wild ass 77. What is the aim of the programme 'Unnat
3. Wild buffalo Bharat Abhiyan'?
Select the correct answer using the code given (a) Achieving 100% literacy by promoting
below: collaboration between voluntary organizations
(a) 1 only (b) 2 and 3 only and government's education system and local
(c) 1 and 3 only (d) 1, 2 and 3 communities.
Ans : (d) Gharial (Gavialis Gangeticus), Indian wild ass (b) Connecting institutions of higher education
(Equus hemionus khur) and wild Buffalo (Bubalus with local communities to address development
bubalis) are all maintained under schedule 1 for the challenges through appropriate technologies.
wildlife (Protection) Act, 1972. (c) Strengthening India's scientific research
75. Consider the following statements: institutions in order to make India a scientific
1. Climate and Clean Air Coalition (CCAC) to and technological power.
Reduce Short Lived Climate Pollutants is a (d) Developing human capital by allocating
unique initiative of G20 group of countries. special funds for health care and education of
2. The CCAC focuses on methane, black carbon rural and urban poor, and organizing skill
and hydro fluorocarbons. development programmes and vocational
Which of the statements given above is/are training for them.
correct? Ans : (b) With a view to uplift rural India, the Unnat
(a) 1 only (b) 2 only Bharat Abhiyan programme is launched in collaboration
(c) Both 1 and 2 (d) Neither 1 nor 2 with the Indian Institutes of Technology (IIT) and the
Ans : (b) The Climate and Clean Air Coalition to National Institutes of Technology (NITs) across the
reduce short lived climate pollutants (CCAC) was country.
launched in 2012. The government of Bangladesh, The programme aims to connect institutions of higher
Canada, Ghana Mexico, Sweden and the United states, education, including Indian Institutes of Technology
along with the United Nation & Environment (IITs), National Institute of Technology (NITs), and
Programme (UNEP), came together to initiate the first Indian Institutes of Science Education & Research
effort to treat these pollutants as a collective challenge.
(IISERs) etc, with local communities to address the
Statement 1 is incorrect.
development challenges through appropriate
The coalitions initial focus is on methane, black carbon
technologies.
and HFCs. Statement 2 is correct.
78. The term 'Digital Single Market Strategy' seen
CURRENT AFFAIRS in the news refers to
(a) ASEAN (b) BRICS
76. What is the purpose of 'Vidyanjali Yojana'? (c) EU (d) G 20
1. To enable the famous foreign educational
institutions to open their campuses in India. Ans : (c) The Digital Single Market Strategy aims to
open up digital opportunities for people and businesses
2. To increase the quality of education provided
and enhance Europe's position as a world leader in the
in government schools by taking help from
digital economy. The Digital Single Market is part of
the private sector and the community.
the Digital Agenda for Europe 2020 program of the EU
3. To encourage voluntary monetary contributions an initiative of Europe 2020 proposed strategy. It is
from private individuals and organizations so as defined by a Digital Single Market Strategy for Europe
to improve the infrastructure facilities for by European Commission.
primary and secondary schools. 79. In India, it is legally mandatory for which of
Select the correct answer using the code given the following to report on cyber security
below: incidents?
(a) 2 only (b) 3 only 1. Service providers
(c) 1 and 2 only (d) 2 and 3 only 2. Data centres
Ans : (a) The ministry of HRD recently launched the 3. Body corporate
'Vidyanjali Scheme aimed at boosting the education Select the correct answer using the code given
system by delivering volunteer teachers to government below:
schools. It will not replace the regular and (a) 1 only (b) 1 and 2 only
professionally qualified teachers in the government (c) 3 only (d) 1, 2 and 3
IAS (Pre) GS 2017 Paper I 100 YCT
Ans : (d) Reporting on Cyber Security incidents in 2. National Career Service has been launched in
India is legally mandatory for the service providers, a Mission Mode to improve the employment
data centers and corporate bodies. It is noteworthy that opportunities to uneducated youth of the
extensive provisions have been made for this in the country.
Information Technology Rule, 2013 released in India. Which of the above statements is/are correct?
80. What is the importance of developing (a) 1 only (b) 2 only
Chabahar Port by India? (c) Both 1 and 2 (d) Neither 1 nor 2
(a) India's Trade with African countries will Ans : (d) Statement 1 : It is an initiative launched by
enormously increase. the Ministry of Labour and Employment. Statement 2 :
(b) India's relations with oil-producing Arab It is a National ICT based portal developed primarily to
countries will be strengthened. connect the opportunities with the aspirations of youth.
(c) India will not depend on Pakistan for access This portal facilitates the registration of job seekers, Job
to Afghanistan and Central Asia. providers, skill providers, career counselors etc.
(d) Pakistan will facilitate and protect the 83. Which of the following statements best
installation of a gas pipeline between Iraq describes the term 'Scheme for Sustainable
and India. Structuring of Stressed Assets (S4A)', recently
Ans : (c) The port of Chabahar is located in South seen in the news?
Eastern Iran in the Gulf of Oman. It is the only Iranian (a) It is a procedure for considering ecological
port with direct access to the Ocean. India will develop costs of developmental schemes formulated
and operate the Chabahar port, No other international by the Government.
port has seen the level of involvement and enthusiasm (b) It is a scheme of RBI for reworking the
from Chabahar as India. The port will make way for financial structure of big corporate entities
India to bypass Pakistan in transporting goods to facing genuine difficulties.
Afghanistan using a sea-land route. At present Pakistan (c) It is a disinvestment plan of the Government
does not allow India to transport through its territory to regarding Central Public Sector Undertakings.
Afghanistan. It has however recently allowed some (d) It is an important provision in 'The
Afghan shipments to come to India. Insolvency and Bankruptcy Code' recently
81. With reference to 'National Skills Qualification implemented by the Government.
Framework (NSQF)'. which of the statements Ans : (b) The RBI's Scheme for Sustainable Structuring
given below is/are correct? of Stressed Assets (S4A) allows bank to separate the
l. Under NSQF, a learner can acquire the total debt of a stressed company into two. While the
certification for competency only through company will service the sustainable part of debt, the
formal learning. rest is converted into equity.
2. An outcome expected from the implementation The lenders are required to make this classification.
of NSQF is the mobility between vocational Sustainable level of debt is one which the banks think
and general education. the stressed borrower can service with its current cash
Select the correct answer using the code given flows.
below: 84. Consider the following statements:
(a) 1 only (b) 2 only The nation–wide 'Soil Health Card Scheme'
(c) Both 1 and 2 (d) Neither 1 nor 2 aims at
Ans : (b) National Skills Qualification Framework 1. expanding the cultivable area under
(NSQF), a quality assurance framework which irrigation.
organizes qualifications according to services of levels 2. enabling the banks to assess the quantum of
of knowledge, skills and aptitude. These levels are loans to be granted to farmers on the basis
defined in terms of learning outcomes which the learner soil quality.
must possess regardless of whether they were acquired 3. checking the overuse of fertilizers in
through formal, non-formal or informal learning. So farmlands.
Statement 1 is incorrect. Which of the above statements is/are correct?
NSQF will also facilitate Recognition of Prior Learning (a) 1 and 2 only (b) 3 only
(RPL) that is largely lacking in the present education (c) 2 and 3 only (d) 1, 2 and 3
and training scenario. Additionally, it would help Ans : (b) Launched by the central government in
alignment of Indian qualifications to international February 2015, the schemes is tailor- made to issue 'Soil
qualifications. It also provides mobility between Card' to farmers which will carry crop wise
vocational and general education by alignment of recommendations of nutrients and fertilizers required
degrees with NSQF, statement 2 is correct. for the individual farms. This is aimed to help farmers
82. Consider the following in respect of 'National to improve productivity through judicious use of inputs.
Career Service': Thus only statement 3 is correct.
1. National Career Service in an initiative of the 85. 'Recognition of Prior Learning Scheme' is
Department of Personnel and Training, sometimes mentioned in the news with
Government of India. reference to
IAS (Pre) GS 2017 Paper I 101 YCT
(a) Certifying the skills acquired by construction Ans : (a) National Nutrition Mission has two
workers through traditional channels. components as follows:
(b) Enrolling the persons in Universities for (i) Information, Education and Communication (IEC)
distance learning programmes. campaign against malnutrition.
(c) Reserving some skilled jobs to rural and (ii). Multi-Sectoral Nutrition Programme.
urban poor in some public sector The key objective of the Programme- To create
undertakings. awareness relating to malnutrition amongst pregnant
(d) Certifying 'the skills acquired by trainees women, lactating mothers, promote healthy lactating
under the National Skill Development practices and importance of balanced Nutrition.
Programme. • To improve maternal and child under nutrition in
200 high burdened districts and to prevent and
Ans : (a) The Recognition of Prior Learning (RPL) is
reduce the under nutrition prevalent among children
an assessment of the skills acquired by the individual below 3 years.
through experience, observation and self learning. The • To reduce incidence of anaemia among young
assessment which is followed by a certification gives an children, adolescent girls and women.
edge to an otherwise informal worker. It provides the Statement 3 and 4 are wrong because there is no
employee with the confidence of social recognition and mention of eggs or unpolished rice or millets in the
empowerment that are necessary to negotiate his future programme.
employment. Following successful assessment, a 88. Consider the following statements:
candidate is given a financial reward in addition to 1. The Nuclear Security Summits are periodically
certification. held under the aegis of the United Nations.
86. Which of the following statements is/are 2. The International Panel on Fissile Materials is
correct regarding Smart India Hackathon an organ of International Atomic Energy
2017? Agency.
1. It is a centrally sponsored scheme for Which of the statements given above is/are
developing every city of our country into correct?
Smart Cities in a decade. (a) 1 only (b) 2 only
2. It is an initiative to identity new digital (c) Both 1 and 2 (d) Neither 1 nor 2
technology innovations for solving the many Ans : (d) The first Summit was held in Washington DC,
problems faced by our country. United states, on April 12-13,2010. US President
Barack Obama hosted the first Nuclear Security Summit
3. It is a programme aimed at making all the
(NSS) in order to draw attention at the highest possible
financial transactions in our country completely level to the need to secure nuclear material and thus
digital in a decade. prevent nuclear terrorism. The International Panel on
Select the correct answer using the code given Fissile Materials (IPFM), established in 2006, is a group
below: of independent nuclear experts from 18 countries (not
(a) 1 and 3 only (b) 2 only by IAEA). It aims to advance international initiatives to
(c) 3 only (d) 2 and 3 only Secure and to sharply reduce all stocks of highly
Ans : (b) Smart India Hackathan 2017 was 36 hours enriched Uranium and Separated Plutonium, the key
non-stop digital product development competition materials in nuclear weapons and to limit any further
during which teams of thousands of technology students production.
had built innovative digital platforms posted by 29 89. Who among the following can join the National
different central governments e.g. Ministry of Railways, Pension System (NPS)?
Ministry of External Affairs, Ministry of Defense, (a) Resident Indian citizens only
ISRO, Ministry of Tourism, Dept. of Atomic Energy
(b) Persons of age from 21 to 55 only
etc.
(c) All State Government employees joining the
87. Which of the following are the objectives of services after the date of notification by the
'National Nutrition Mission'? respective State Governments
1. To create awareness relating to malnutrition (d) All Central Government employees including
among pregnant women and lactating those of Armed Forces joining the services
mothers. on or after 1st April, 2004
2. To reduce the incidence of anaemia among
Ans : (c) NPS (National Pension System) launched by
young children, adolescent girls and women.
Government of India is a defined contribution based
3. To promote the consumption of millets, Pension Scheme.
coarse, cereals and unpolished rice. Eligibility: NPS is applicable to all new employees of
4. To promote the consumption of poultry eggs. Central Government Service (except armed Forces') and
Select the correct answer using the code given Central Autonomous Bodies joining Government
below: Service on or after 1st January 2004.
(a) 1 and 2 only (b) 1, 2 and 3 only NPS is applicable to all the employees of State
(c) 1, 2 and 4 only (d) 3 and 4 only Governments State Autonomous Bodies jointing
IAS (Pre) GS 2017 Paper I 102 YCT
services after the date of notification by the respective Ans : (d)
State Governments. 1. The 1st APMCHUD was held in New Delhi India
All Citizens of India between the age of 18 & 60 years from 13th - 16th December 2006 on the theme of ' A
as on the date of Submission of his/her application to vision for sustainable Urbanisation in the Asia
point of presence (Pop)/ Point of Presence-Service Pacific by 2020'. The theme for the 6th Asia Pacific
Provider (POP-SP) can join NPS. Ministerial conference on Housing & Urban
NRIs were also allowed to open National Pension Development (APMCHUD), New Delhi, India 2016
Scheme (NPS) accounts online. is "Emerging Urban Farms- Policy Responses and
90. What is/are the advantage/advantages of Governance structure".
implementing the 'National Agriculture 2. The second Conference was held in Tehran, Iran.
Market' scheme?
1. It is a pan–India electronic trading portal for MISCELLANEOUS
agricultural commodities.
93. With reference to the 'Prohibition of Benami
2. It provides the farmers access to nationwide
Property Transactions Act, 1988 (PBPT Act)',
market, with prices commensurate with
consider the following statements:
quality of their produce. 1. A property transaction is not treated as a
Select the answer using the code given below: benami transaction if the owner of the
(a) 1 only (b) 2 only property is not aware of the transaction.
(c) Both 1 and 2 (d) Neither 1 nor 2 2. Properties held benami are liable for
Ans : (c) Both the statements in the question are confiscation by the Government.
correct. Prime Minister launched the National 3. The Act provides for three authorities for
Agricultural Market scheme on 14th April 2016. It is the investigations but does not provide for any
all India electronic trade portal for agricultural appellate mechanism.
commodities that provides a nationwide market for Which of the statements given above is/are
farmers to ensure a consistent price for the quality of correct?
their produce. (a) 1 only (b) 2 only
91. With reference to the 'National Intellectual (c) 1 and 3 only (d) 2 and 3 only
Property Rights Policy', consider the following Ans : (b) The Prohibition of Benami Property
statements: Transactions Act, 1988 (PBPT Act), defines Benami
1. It reiterates India's commitment to the Doha Transactions' prohibits them and further provides that
Development Agenda and the TRIPS violation of the PBPT Act is punishable with
Agreement. imprisonment and fine.
2. Department of Industrial Policy and The definition of a benami transaction has been
Promotion is the nodal agency for regulating widened to include a transaction made in a fictitious
name, where the owner is not aware or denies
intellectual property rights in India.
knowledge of the ownership of the property or the
Which of the above statements is/are correct? person providing the consideration for the property is
(a) 1 only (b) 2 only not traceable. Properties held benami are liable for
(c) Both 1 and 2 (d) Neither 1 nor 2 confiscation.
Ans : (c) National Intellectual Property Rights Policy Properties held benami are liable for confiscation by the
approved by the Government of India on May 12, 2016 government without payment of compensation. An
reflects India's commitment to the DOHA development appellate mechanism has been provided under the PBPT
Agenda and TRIPS Agreement. It is noteworthy that the Act in the form of Adjudicating Authority and
Department of Industrial policy and promotion is a Appellate Tribunal.
nodal agency for regulation of intellectual property 94. Consider the following pairs:
rights in India. Traditions Communities
92. With reference to 'Asia Pacific Ministerial 1. Chaliha Sahib Festival – Sindhis
2. Nanda Raj Jaat Yatra – Gonds
Conference on Housing and Urban
3. Wari–Warkari – Santhals
Development (APMCHUD)', consider the Which of the pairs given above is/are correctly
following statements: matched?
1. The first APMCHUD was held in India in (a) 1 only (b) 2 and 3 only
2006 on the theme 'Emerging Urban Forms– (c) 1 and 3 only (d) None of the above
Policy Responses and Governance Structure'. Ans : (a) Statement 1: Chalisa /Chaliha also called
2. India hosts all the Annual Ministerial Chaliha Sahib is a forty day long festival celebrated by
Conferences in partnership with ADB, APEC Hindu Sindhi community.
and ASEAN. Statement 2 : Nanda Devi Raj Jaat Yatra is a festival of
Which of the statements given above is/are Gharwali and Kumaoni people in Uttarkhand.
correct? Statement 3 : Wari-Warkari or Varkari (Pandanpur
(a) 1 only (b) 2 only wari) is a Maharashtrian festival celebrated by
(c) Both 1 and 2 (d) Neither 1 nor 2 Marathas.
IAS (Pre) GS 2017 Paper I 103 YCT
95. Which of the following gives 'Global Gender 98. With reference to 'Quality Council of India
Gap Index' ranking to the countries of the (QCI)', consider the following statement:
world? 1. QCI was set up jointly by the Government of
(a) World Economic Forum India and the Indian Industry.
(b) UN Human Rights Council 2. Chairman of QCI is appointed by the Prime
(c) UN Women Minister to the recommendations of the
(d) World Health Organization industry to the Government.
Ans : (a) Since 2006, the World Economic Forum has Which of the above statements is/are correct?
released the Global Gender Gap Index for the countries (a) 1 only (b) 2 only
of the world. It is primarily an Index to measure gender
(c) Both 1 and 2 (d) Neither 1 nor 2
equality. Through which gender inequality has to be
eliminated, under which education, health and safety Ans : (c) The 'Quality' council of India' (QCI)
have been given priority. established in the year 1997, was formed jointly by the
Government of India and the Indian Industry. Its
96. Which reference to Manipuri Sankirtana,
consider the following statements: chairman is appointed by the Prime Minister on the
1. It is a song and dance performance. recommendations made by the industry to the
2. Cymbals are the only musical instruments government. Thus, both the statements in question are
used in the performance. true.
3. It is performed to narrate the life and deeds of 99. Which of the following is a most likely
Lord of Lord Krishna. consequence of implementing the 'Unified
Which of the statements given above is/are Payments Interface (UPI)'?
correct? (a) Mobile wallets will not be necessary for
(a) 1, 2 and 3 (b) 1 and 3 only online payments.
(c) 2 and 3 only (d) 1 only (b) Digital currency will totally replace the
Ans : (b) Sankirtana is one of the few important Indian physical currency in about two decades.
heritage inscribed on the UNESCO Representative List (c) FDI inflows will drastically increase.
of the Intangible Cultural Heritage of Humanity.
(d) Direct transfer of subsidies to poor people
Statement 1 and 3 : Sankirtana performers narrate the
will become very effective.
lives and deeds of Krishna through ritual Singing
drumming and dancing of Manipur. So both 1 and 3 are Ans : (a) UPI is a payment system that allows money
transfer between any two bank accounts by using a
correct.
Smartphone UPI allows a customer to pay directly from
Statement 2 : Since drums are also used, statement 2 is
a bank account to different merchants, both online and
incorrect. offline, without the hassle of typing Credit Cards
97. Consider the following statements: details, IFSC code or net banking/Wallet passwords.
1. The Standard Mark of Bureau of Indian 100. The terms 'Event Horizon', 'Singularity',
Standards (BIS) is mandatory for automotive 'String Theory' and 'Standard Model' are
tyres and tubes. sometimes seen in the news in the context of
2. AGMARK is a quality Certification Mark (a) Observation and understanding of the
issued by the Food and Agriculture Universe
Organisation (FAO). (b) Study of the solar and the lunar eclipses
Which of the statements given above is/are (c) Placing satellites in the orbit of the Earth
(d) Origin and evolution of living organisms on
correct?
the Earth
(a) 1 only (b) 2 only
Ans : (a) The terms 'Event Horizon' explained as the
(c) Both 1 and 2 (d) Neither 1 nor 2
boundary in space-time, 'Singularity' is defined as a
Ans : (a) In 2010, a certification from the Bureau of
location in space-time where the gravitational field of a
Indian Statements (BIS) has been made mandatory for
celestial body becomes infinite. Both are related to
all types of automotive tyres and tubes.
AGMARK is a certification mark employed on Black Holes.
agricultural products in India, assuring that they 'String Theory' links together all physical aspects and
conform to a set of standards approved by the 'Standard Model'- unifying the four fundamental forces,
Directorate of Marketing and Inspection, an agency of are related to the observation and understanding of the
the Government of India. universe.

IAS (Pre) GS 2017 Paper I 104 YCT


UNION PUBLIC SERVICE COMMISSION
Civil Services (Preliminary Exam) - 2016
GENERAL STUDIES : PAPER-I
Time: 2 hours (Exam date : 07.08.2016) Maximum Number: 200
3. With reference to the cultural history of India the
ANCIENT HISTORY memorizing of chronicles, dynastic histories and
1. In the context of the history of India, consider the
epic tales was the profession of who of following?
following pairs :
(a) Shramana (b) Parivraajaka
Term Description
1. Eripatti : Land, revenue from which was set (c) Agrahaarika (d) Magadha
apart for the maintenance of the Ans. (d): Memorizing of chronicles, dynastic hysterics
village or epic tales was the work of a different groups of
2. Taniyurs : Villages donated to a single people, the Sutas and Magadhas, who were the
Brahmin or a group of Brahmins descendants of the priestly families of the Vedic period.
3. Ghatikas : Collages generally attached to the 4. Consider the following pairs :
temples
Famous place Region
Which of the pairs given above is / are correctly
matched ? 1. Bodhgaya Baghelkhand
(a) 1 and 2 (b) 3 only 2. Khajuraho Bundelkhand
(c) 2 and 3 (d) 1 and 3 3. Shirdi Vidarbha
Ans. (d): Taniyur is a large village under Chola 4. Nasik (Nashik) Malwa
administrative division. 5. Tirupati Rayalaseema
Eripatti was a special category of land known in South Which of the pairs given above are correctly
India. This land was donated by individuals, revenue matched?
from which was set apart for the maintenance of the
(a) 1, 2 and 4 (b) 2, 3, 4, and 5
village tank. Land grants and generous donations were
(c) 2 and 5 only (d) 1, 3, 4 and 5
given to educational intuitions like Ghatikas and
Mathas. Mathas were residential schools for early Ans. (c) : Bodhgaya is an important Buddhist site
education in Pallava Kingdoms. located in Bihar and Baghelkhand is a mountain range
2. Which one of the following books of ancient India in central India that covers the northeastern regions of
has the love story of the son of the founder of Madhya Pradesh and a small area of western Uttar
Sunga dynasty ?
Pradesh.
(a) Swapnavasavadatta (b) Malavikagnimitra
(c) Meghadoota (d) Ratnavali • The Khajuraho Group of Monuments is a group of
Ans. (b): Malavikagnimitram is a Sanskrit play written Hindu temples which are located in the Bundelkhand
by Kalidas in the 5th century CE. It narrates the love region of the Madhya Pradesh, southeast of Jhansi. It
story of King Agnimitra, the Sunga emperor of Vidisha.
Agnimitra was the son of Pushyamitra Shunga, founder is one of the UNESCO World Heritage sites in India.
of the Sunga dynasty. • Shirdi, famous town for Sai Baba temple, is a town
• Kalidas is the most distinguished dramatist and he and falls under the Ahmednagar district in
treated the rasa of love in all its possible
Maharashtra and it is not in the Vidarbha region.
manifestations.
• Swapnavasavadatta is a Sanskrit play of six acts • Malwa Plateau generally refers to the volcanic
written by Bhasa. upland north of the Vindhya Range covering the
• Meghadoota is a Sanskrit poem by Kalidasa. region in Madhya Pradesh and Rajasthan, Nasik is
• Ratnavali is a Sanskrit drama about a princess for away from the region.
Ratnavali and a king Udayana written by • Tirupati, famous for Sri Venkateswara temple, lies in
Harshavardhan. the Rayalaseema region of Andhra Pradesh.

IAS (Pre) GS 2016 Paper I 105 YCT


• Buddha, Jaina, Bhagavata, Shiva and examples of Indian art, particularly painting", which are
masterpieces of Buddhists religious art with figures of
Other Religion the Buddha and depictions of the Jataka tales.
5. With reference to the religious history of India Mahabalipuram has a group of sanctuaries which was
consider the following statements : carved out of rock along the coromandel coast in the 7th
1. The concept of Bodhisattva is central to and 8th centuries rathas (temple is the from of chariots),
Hinayana sect of Buddhism. mandapas (cave sanctuaries), giant open-air rock reliefs
2. Bodhisattva is a compassionate one on his way such as the famous Descent of Ganges and the shore
to enlightenment. temple with thousands of sculptures to the glory of
3. Bodhisattva delays achieving his own salvation Shiva. The group of Monuments at Mahabalipuram has
been classified as a UNESCO World Heritage site.
to help all sentient beings on their path to it
Which of the statements given above is / are • Political-Social System between
correct ? 800 AD to 1200 AD
(a) 1 only (b) 2 and 3 only 8. Banjaras during the medieval period of Indian
(c) 2 only (d) 1, 2 and 3 history were generally–
Ans. (b): The Bodhisattva ideal is central to the (a) Agriculturists (b) Warriors
Mahayana Buddhist tradition as the individual who (c) Weavers (d) Traders
seeks enlightenment both for himself and few others. In Ans. (d): The Banjaras were the most important trader-
nomads. Their canvas was called tanda. Sultan
non-Mahayana Buddhism, it usually refers either to
Alauddin Khilji used the Banjaras to transport grain to
Maitreya, the Buddha of the Future or to the historical the city market. Emperor Jahangir wrote in his memoirs
Buddha, Gautama, prior to his enlightenment. that the Banjaras carried grain on their bullocks from
Statement (2) and (3): A bodhisattva is literally a living different areas and sold it in towns.
(sattva) who aspires to enlightenment (bodhi) and
carries out altruistic practices. Clearly, option (2) is MEDIEVAL HISTORY
correct. Bodhisattva delays achieving his own salvation
to help all sentient beings on their path to it. • Vijayanagara Empire and Bahmanids
• Mauryan and Post Mauryan Period 9. Regarding the taxation system of Krishna Deva,
the ruler of Vijayanagar, consider the following
6. Who of the following had first deciphered the Statements.
edicts of Emperor Ashoka ? 1. The tax rate on land was fixed depending on the
(a) Georg Buhler (b) James Prinsep quality of the land.
(c) Max Muller (d) William Jones 2. Private owners of workshops paid an industries
tax.
Ans. (b): It was James Prinsep in 1837 who succeeded
Which of the statements given above is/are
in deciphering an ancient inscription on a large stone correct?
pillar in Delhi, which was the edict of Emperor Ashoka. (a) 1 only (b) 2 only
7. What is/are common to the two historical places (c) both 1 and 2 (d) Neither 1 nor 2
known as Ajanta and Mahbalipuram ? Ans. (c): Land revenue was the chief source of income.
1. Both were built in the same period. The land was divided into four categories for the purpose
2. Both belong to the same religious of assessment, wetland, dry land, orchards and woods.
denomination. Usually the share was one-sixth of the produce. Land
3. Both have rock-cut monuments. revenue could be paid in cash or kind. The rates varied
Select the correct answer using the code given according to the types of the crops, soil method of
below. irrigation, etc. so option (1) is correct.
(a) 1and 2 only
• Besides land tax, many professional taxes were also
(b) 3 only
imposed. They were imposed on shopkeeper, servants,
(c) 1 and 3 only
(d) None of the statements given above is correct. workmen, posters, shoemakers, musicians, etc. There
Ans. (b) : The Ajanta caves in Aurangabad district of was also a tax on property. Grazing and house taxes
Maharashtra, are about 30 rock- cut Buddhist cave were also imposed; commercial taxes consisted of
monuments that date from the 2nd century BCE to about levied, duties and customs on manufactured articles of
480 as 650 CE. The caves include paintings and trade were also levied. Private owners of workshops
sculptures described by the Governments. paid an industries tax, therefore option (2) is also
Archaeological Survey of India as "the finest surviving correct.
IAS (Pre) GS 2016 Paper I 106 YCT
• Mughal Period Ans. (c) : Satyashodhak Samaj was an association
started by Jyotiba Phule in Poona in 1873 for asserting
10. With reference to the economic history of
the worth of man irrespective of caste. He demanded
medieval India. the term'Araghatta' refers to
representation of all classes of the Hindus in all the
(a) bonded labour
local bodies, in services and institutions and also
(b) land grants made to military officers
established a primary school for the so-called
(c) waterwheel used in the irrigation of land untouchables in Poona.
(d) waterland converted to cultivated land 13. Consider the following :
Ans. (c) : In Sanskrit, the word Araghatta has been 1. Calcutta Unitarian Committee
used in ancient texts to describe the Persian wheel. The (Calcutta Unitarian Committee)
Ara-ghatta comes from the combination of the words 2. Tabernacle of New Dispensation
'ara' meaning spoke and 'ghatta' meaning pot. There is (Tabernacle of New Dispensation)
evidence to argue that this system of lifting water from 3. Indian Reform Association
open wells was probably invented in India of the past. (Indian Reform Association)
With its use in Iran, then Persia and perhaps its Keshab Chandra Sen is associated with the
discovery there is coming to be called the Persian estalishment of which of the above ?
wheel. (a) 1 and 3 only (b) 2 and 3 only
(c) 3 only (d) 1, 2 and 3
Modern History Ans. (b): Calcutta Unitarian Committee was established
in 1823 by Rammohan Roy, Dwarkanath Tagore and
William Adam. So, (1) is wrong.
• Social, Cultural, Awakening, Lower,
• In 1868, Keshab laid in the foundation stone of his
Caste Trade Unions and Peasant new church, the Tabernacle of New Dispensation. So,
Movements (2) is correct.
11. With reference to the cultural history of medieval • Indian reform association was founded in 1870 with
Keshab Chandra Sen as president. In represented the
India, Consider the following statements.
secular side of the Brahmo Samaj and included many
1. Siddha (Sittars) of Tamil region were
shows did not belong to the Brahmo Samaj. So, (3) is
monotheistic and condemned idolatry.
correct.
2. Lingayats of Kannada region questioned the
theory of rebirth and rejected the caste hierarchy. • Freedom Struggle and National
Which of the statements given above is/ are Movement
correct
14. What was the main reason for the split in the
(a) 1 only (b) 2 only
Indian National Congress at Surat in 1907 ?
(c) Both 1 and 2 (d) Neither 1 nor 2 (a) Introduction of communalism into Indian
Ans. (c) : Siddhas (Sittars) Saiva school in Tamil Nadu politics by Lord Minto.
held a monotheistic puritan creed and vehemently (b) Extremists lack of faith in the capacity of the
condemned idolatry, their history are not known. They moderates to negotiate with the British
seem to be known as most active during 16th and 17th Government.
century. (c) Foundation of Muslim League
The Lingayats worshiper of Saiva challenged the idea of (d) Aurobindo Ghosh's inability to be elected as the
caste and the "pollution" attributed to certain groups by President of the Indian National Congress .
Brahmanas. They also questioned the theory of rebirth. Ans. (b) : Surat split refers to the splitting of the
These won them, followers, amongst those who were Congress party into 'Moderates' and 'Extremists' after a
marginalized within the Brahmanical social order. clash of the session. The extremists were led by
Lokmanya Tilak, Lala Lajpat Rai and Sri Aurobindo
12. Satya Shodhak samaj organized
(a) a movement for upliftment of tribals in Bihar and the Moderates were led by Gopal Krishna Gokhale,
(b) a temple-entry movement in Gujarat Pherozeshah Mehta and Surendranath Banerjee. The
(c) an anti -caste movement in Maharashtra divided Congress re-united in the crucial Lucknow
(d) a peasant movement in Punjab session of Congress in 1916.
IAS (Pre) GS 2016 Paper I 107 YCT
15. The plan of Sir Stafford Cripps envisaged that
after the Second World War.
INDIAN GEOGRAPHY
(a) India should be granted complete Independence
(b) India Should be partitioned into two before
• Physical Geology, Drainage System,
granting independence Human Geography
(c) India should be made a republic with condition 18. Recently, linking of which of the following rivers
that she will join the Commonwealth was undertaken ?
(d) India should be given Dominion status (a) Cauvery and Tungabhadra
(b) Godavari and Krishna
Ans. (d) : Cripps Mission was sent to India under the
(c) Mahanadi and Sone
presidentship of Stafford Cripps in March 1942. The (d) Narmada and Tapti
intention was the get Indian support for Britishers in the Ans. (b) : The plan is to eventually divert water from
ongoing World War II. the Polavaram dam since the dam is still under
• It proposed to establish an Indian Dominion that will construction and will take at least 4 to 5 years to be
be free to decide its relations with the commonwealth ready. The Chandrababu Naidu government decided to
divert Godavari water from the Pattiseema Lift
and United Nations constituent assembly to be
Irrigation Scheme. The link plan kicked off on 16
convened to frame a new constitution. Any province September, 2015.
not willing to join the Union could have a separate 19. Which of the following is/are tributary/
constitution and form a separate state. tributaries of Brahmaputra?
Acceptance of Indian constitution by British 1. Dibang
government would be subject to condition that any 2. Kameng
province would have the right to have a separate union 3. Lohit
Select the corrrect answer usin the code given
and a treaty to be negotiated to affect the power transfer
below :
to safeguard racial and religious minorities. (a) 1 only (b) 2 and 3 only
It does not have any condition regarding partition of (c) 1 and 3 only (d) 1, 2 and 3
India for making India republic or joining the common- Ans. (d) : The Brahmaputra enters India in the state of
wealth of Nation. Arunachal Pradesh from its original source Tibet, and is
16. The 'Swadeshi' and 'Boycott' adopted as methods joined by the Dibang river and the Lohit River at the
head of the Assam Valley. It is joined in Sonitpur by the
of struggle for the first time during the
Kameng River (or Jia Bhoreli). Brahmaputra's main left
(a) agitation against the Partition of Bengal bank tributaries are viz Dibang or Sikang and Lohit.
(b) Home Rule Movement The important right bank tributaries are the Subansiri
(c) Non-Cooperation Movement Kameng, Manas and Sankosh. Therefore (d) is correct
(d) visit of the Simon Commission to India option.
Ans. (a) : 'Swadeshi and boycott' were first used as a 20. With reference to 'Red Sanders', sometimes seen
method of struggle and protest during the movement in the news, consider the following statements :
1. It is a tree species found in a part of South
against the partition of Bengal. It is noteworthy that the India.
Swadeshi movement started on 7 August, 1905 in 2. It is one of the most important trees in the
protest against the declaration of partition of Bengal in tropical rain forest areas of South India.
July 1905 by Lord Curzon. Which of the statements given above is/are
17. The Montague-Chelmsford Proposals were correct?
related to (a) 1 only (b) 2 only
(a) Social reforms (c) Both 1 and 2 (d) Neither 1 nor 2
(b) educational reforms Ans. (a) : Pterocarpus Santaldinus, with the common
(c) reforms in police administration names red sanders, red sandalwood and saunderswood,
(d) constitutional reforms is found in southern Eastern Ghats mountain. They are
Ans. (d): The Government of India Act, 1919 was placed under the 'near threatened' category by the
passed to expand the participation of Indians in the International Union for Conservation of Nature (IUCN).
It occurs in the forest formation, which is classified as
government of India. The Act embodied the reforms
"Southern Tropical Dry Deciduous Forests".It is
recommended by Edwin Montague and the Viceroy
generally formed at an altitude of 150-900 m. It grows
Lord Chelmsford (Montague Chelmsford Reforms). The on dry, hilly, often rocky ground.
Act was convened in ten years from 1919 to 1929. So clearly (b) is incorrect.

IAS (Pre) GS 2016 Paper I 108 YCT


• Monsoon, Forest, Soil, Irrigation 23. In which of the following regions of India are
shale gas resources found ?
Projects, Agriculture 1. Cambay Basin
21. Why does the Government of India promote the 2. Cauvery Basin
use of 'Neem -coated Urea' in agriculture ? 3. Krishna-Godavari Basin
(a) Release of Neem oil in the soil increases Select the correct answer using the code given
nitrogen fixation by the soil microorganisms. below:
(b) Neem coating slows down the rate of (a) 1 and 2 only (b) 3 only
dissolution of urea in the soil (c) 2 and 3 only (d) 1, 2 and 3
(c) Nitrous oxide, which is a greenhouse gas, is not Ans. (d): Shale gas is natural gas that is found trapped
at all released into the atmosphere by crop within non-porous rocks.
fields. • India has identified 6 basins as areas for shale gas
(d) It is a combination of a weedicide and a exploration : Cambay (Gujrat), Assam-Arakan (North-
fertilizer for particular crops East), Gondwana (Central India), Krishna Godavari
Ans. (b): Urea, which is necessary for the development onshore and Indo-Gangetic basins.
of plants, is an important supplier of nitrogen. But only
30-40 percent of nitrogen present in the area is utilized • Transport
by crops and the rest gets degraded in various stages. 24. Recently, which of the following States has
• When ordinary urea is applied, it gets converted to explored the possibility of constructing an
ammonium carbonate. Some of this gets converted to artificial inland port to be connected to sea by a
ammonia gas in what is called ammonia volatilisation. long navigational channel ?
•About 8-10 percent of nitrogen is lost during (a) Andhra Pradesh (b) Chhattisgarh
volatilisation. (c) Karnataka (d) Rajasthan
• Neem has properties that check nitrogen loss at each Ans. (d): The government of Rajasthan is planning to
stage. It slows down the process of nitrate formation develop an Inland shipping port at Jalore, which would
and hence excess nitrate is not available for be connected to the Arabian Sea by developing a
desertification. Hence, Neem coating slows down the channel along the Kutch Creek.
rate of dissolution of urea in the soil.
World Geography
• Minerals Energy Resources, Industry
and Trade • Atmosphere
22. What is /are the purpose /purposes of 'District 25. In the cities of our country, which among the
Mineral Foundations' in India ? following atmospheric gases are normally
1. Promoting mineral exploration activates in considered in calculating the value of Air Quality
mineral-rich districts Index ?
2. Protecting the interests of the persons affected by 1. Carbon dioxide
mining operations 2. Carbon monoxide
3. Authorizing State Governments to issue 3. Nitrogen dioxide
4. Sulfur dioxide
licenses for mineral exploration .
5. Methane
Select the correct answer using the code given Select the correct answer using the code given
below : below :
(a) 1 and 2 only (b) 2 only (a) 1, 2 and 3 only (b) 2, 3 and 4 only
(c) 1 and 3 only (d) 1, 2 and 3 (c) 1, 4 and 5 only (d) 1, 2, 3, 4 and 5
Ans. (b): District Mineral Foundation (DMF) is a trust Ans. (b): National Air Quality Index : There are 6
set up as a non-profit body in those districts affected by AQI categories, namely Good, Satisfactory, Moderately
the mining works to work for the interest and benefit of polluted, Poor, Very, Poor and Severe. The AQI
persons and areas affected by mining -related considers eight pollutants (PM 10 , PM 2.5, NO2, SO2,
operations. It is founded through the contributions from CO, O3, NH3 and Pb) for which short-term (upto 24-
miners. Its manner of operation comes under the hourly averaging period). National Ambient Air Quality
jurisdiction of the relevant State Government. Standards are prescribed.

IAS (Pre) GS 2016 Paper I 109 YCT


• Human Geography and Mapping Select the correct answer using the code given
below
26. Consider the following pairs :
(a) 1 only
Community sometimes In the affairs of
(b) 2 only
mentioned in the news
(c) both 1 and 2
1. Kurd – Bangladesh
(d) Neither 1 nor 2
2. Madhesi – Nepal
Ans. (b): When the Lok Sabha is dissolved, all
3. Rohingya – Myanmar
business, including bills, motions, resolutions, notices,
Which of the pairs given above is/are correctly
petitions and so on pending before it or its committees
matched ?
lapse. They need to be reintroduced in the newly
(a) 1 and 2 (b) 2 only
constituted Lok Sabha.
(c) 2 and 3 (d) 3 only
A bill pending in the Rajya Sabha but not passed by the
Ans. (c): Kurds are an ethnic group in the Middle East,
Lok Sabha does not lapse. A bill passed by the Lok
so option (1) is incorrect.
Sabha but pending in the Rajya Sabha lapses. A bill
Madhesis were in the news due to Nepal's constitutional
passed by both Houses but pending assent of the
debates about their representation in the polity. So (2) is
president does not lapse.
correct. Rohingyas Muslims and Buddhist rakhines
have been indulging into fights and riots in Myanmar, • Centre-State Relations, Administration,
which has frequently been in news. Jammu and Kashmir
So, (3) is correct.
29. Consider the following statements :
Indian Constitution and Governance 1. The Chief Secretary in a State is appointed by
the Governor of that State.
• Legislature : Council of Ministers 2. The Chief Secretary in a State has a fixed tenure
Which of the statements given above is/are correct?
27. The Parliament of India acquires the power to
(a) 1 only (b) 2 only
legislate on any item in the State List in the
(c) Both 1 and 2 (d) Neither 1 nor 2
national interest if a resolution to that effect is
Ans. (d) : The Incumbent to the post of chief secretary
passed by the –
is chosen by and appointed by the chief minister and
(a) Lok Sabha by a simple majority of its total
ministers of the state, the trend is that the CM consults
membership
the union government regarding the appointments of the
(b) Lok Sabha by a majority of not less then two-
chief secretary but the consultation is not obligatory.
thirds of its total membership
(c) Rajya Sabha by a simple majority of its total • Judicial System (Centre and State)
membership
(d) Rajya Sabha by a majority of not less than two - 30. With reference to the 'Gram Nyayalaya Act'
thirds of its members present and voting which of the following statements is/are correct ?
Ans. (d) : If Rajya Sabha passes a resolution by a 1. As per the Act, Gram Nyayalayas can hear only
majority of not less than two-thirds of numbers present civil cases and not criminal cases
and voting saying it is "necessary or expedient in the 2. The Act allows local social activists as
"national interest" that Parliament should make a law on
mediators/ reconciliators.
a matter enumerated in the State List, Parliament
becomes empowered to make a law on the subject Select the correct answer using the code given
specified in the resolution, for the whole as any parts of below–
the territory of India. Such as resolution remains in (a) 1 only (b) 2 only
force for a maximum period of one year but this period (c) Both 1 and 2 (d) Neither 1 nor 2
can be extended by one year at a time by passing a Ans. (b): The Gram Nyayalayas are mobile village
similar resolution further. courts in India established under Gram Nyayalayas Act
28. Which of the following statements is/are correct -2008 for speedy and easy access to the justice system
1. A Bill pending in the Lok Sabha lapses on its in the rural area. The Yojana mainly addresses the
prorogation.
longevity risks among the workers in unorganised
2. A Bill pending in the Rajya Sabha, which has
not been passed by the Lok Sabha, shall not sector and encourages them to voluntarily save for their
lapse on dissolution of the Lok Sabha. retirement. So, (1) is correct.

IAS (Pre) GS 2016 Paper I 110 YCT


• Panchayati Raj System, Constitution age of 60 years, depending on their contributions, which
itself would vary on the age of joining the APY. The
Amendments and Schedules minimum age of joining APY is 18 years and the
31. Consider the following statements : maximum age is 40 years. Therefore, the minimum
1. The minimum age prescribed for any person to be period of fixed pension would be guaranteed by the
a member of Panchayat is 25 years. government.
2. A Panchayat reconstituted after premature • Atal Pension Yojana (APY) is open to all bank
dissolution continues only for the remainder account holders who are not members of any
period. statutory social security scheme.
Which of the statements given above is/are correct So, (2) is wrong.
(a) 1 only (b) 2 only • As per the Ministry of Finance, " The spouse of the
(c) Both 1 and 2 (d) Neither 1 nor 2 subscriber shall be entitled to receive the same
Ans. (b): A candidate for the seat of Member or pension amount as that of the subscriber until the
Sarpanch of Gram Panchayat must be a registered voter death of the spouse. After the death of both the
in the electoral roll of the Gram Panchayat. A candidate subscriber and the spouse, the nominee of the
can not be disqualified if he is less than 25 years but subscriber shall be entitled to receive the pension
more than 21 years. wealth, as accumulated till age of 60 years of the
Every Panchayat, unless, sooner dissolved under any subscribe". So, clearly (3) incorrect.
law for the time being in force, shall continue for five 33. There has been a persistent deficit budget year
years from the date of appointment. A panchayat after year. Which action/actions of the following
constituted upon the dissolution of a Panchayat before can be taken by the Government to reduce the
the expiration of its duration shall continue only for the deficit ?
remainder of the period for which the dissolved 1. Reducing revenue expenditure
panchayat would have continued under clause (1) had it 2. Introducing new welfare schemes
not been dissolved. 3. Rationalizing subsidies
4. Reducing import duty
ECONOMY Select the correct answer using the code given
below.
• Economic Planning and National (a) 1 only (b) 2 and 3 only
Income/Budget (c) 1 and 3 only (d) 1, 2, 3 and 4
Ans. (c): Unnecessary revenue expenditure bloats the
32. Regarding 'Atal Pension Yojana', which of the
fiscal deficit, and since it forms the majority of
following statement is/are correct ?
government spending, its reduction has a very large
1. It is a minimum guaranteed pension scheme
effect on the fiscal deficit. So, (1) is correct.
mainly targeted at unorganized sector workers
• It will further increase the fiscal deficit. So, (2) is
2. Only one member of a family can join the
incorrect.
scheme. • Subsidies are a major component of government
3. Same amount of pension is guaranteed for the spending and their reduction will cut down the fiscal
spouse for life after subscriber's death. deficit. So, (3) is correct
Select the corect answer using the code given • It reduces tax revenues and thus increases fiscal
below: deficit. So, (4) is incorrect.
(a) 1 only (b) 2 and 3 only 34. Which one of the following is a purpose of
'UDAY', a scheme of the Government ?
(c) 1 and 3 only (d) 1, 2 and 3 (a) Providing technical and financial assistance to
Ans. (c) : Atal Pension Yojana (APY) is a government- start-up entrepreneurs in the field of renewable
backed scheme in India targeted at the unorganized source of energy.
sector. It was originally mentioned in the 2015 Budget (b) Providing electricity to every household in the
speech by Finance Minister Arun Jaitley in February country by 2018
2015. It was formally launched by Prime Minister (c) Replacing the coal-based power plants with
natural gas, nuclear, solar, wind and tidal power
Narendra Modi on 9th May, 2015 in Kolkata. plants over a period of time.
Under the APY, the subscriber would receive the fixed (d) Providing for financial turnaround and revival
pension of 1000/2000/3000/4000/5000 per month at the of power distribution companies.
IAS (Pre) GS 2016 Paper I 111 YCT
Ans. (d) : It is the Ujwal DISCOM Assurance Yojana, 37. With reference to 'Pradhan Mantri Fasal Bima
where DISCOMs stands for Distribution companies Yojana', consider the following statements :
(electricity) that are state-owned and suffering huge 1. Under this scheme, farmers will have to pay a
financial losses, mis-governance, low tariff rates etc. uniform premium of two percent for any crop
The scheme aims at their financial turnaround and they cultivate in any season of the year.
revival. 2. This scheme covers post-harvest losses arising
35. Which of the following is/are included in the out of cyclones and unseasonal rains.
capital budget of the Government of India ? Which of the statements given above is/are correct?
1 Expenditure on acquisition of assets like roads, (a) 1 only (b) 2 only
buildings, machinery etc. (c) Both 1 and 2 (d) Neither 1 nor 2
2. Loans received from foreign governments. Ans. (b): Farmers premium has been kept at a
3. Loans and advances granted to the States and maximum of 2 percent for food grains and oilseeds and
Union Territories. upto 5 percent for horticulture and cotton crops. So, it is
Select the correct answer using the code given different for different crops. So statement (1) is
below : incorrect.
(a) 1 only (b) 2 and 3 only • Post- Harvest Losses coverage is available only upto a
(c) 1 and 3 only (d) 1, 2 and 3 maximum period of two weeks from harvesting for
Ans. (d): The main items of the capital budget are those crops which are allowed to dry in cut and spread
receipts and expenditure for capital (financial) gains. It condition in the field after harvesting against specific
also includes loans raised by Government from the perils of cyclones and cyclonic rains and unseasonal
public, Reserve Bank and other parties and from foreign rains. So, statement (2) is correct.
Government bodies. It also includes capital expenditure 38. With reference to 'Stand Up India Scheme',
on acquisition of assets like land, buildings, machinery, which of the following statements is/are correct?
equipment etc. and loans and advances granted by 1. Its purpose is to promote entrepreneurship
central Government to state and union territory among SC/ST and woman entrepreneurs.
government, government companies, corporations and 2. It provides for refinance through SIDBI.
other parties. Select the correct answer using the code given
36. 'Rashtriya Garima Abhiyaan' is a national below :
campaign to– (a) 1 only (b) 2 only
(a) rehabilitate the homeless and destitute persons (c) both 1 and 2 (d) Neither 1 nor 2
and provide then with suitable sources of Ans. (c) : Stand up India scheme facilitates bank loans
livelihood. between 10 Lakh and 1 core to at least one Scheduled
(b) release the sex workers from their practice and Caste (SC) or Scheduled Tribe (ST), borrower and at
least one woman per bank branch for setting up a green
provide them with alternative sources of
field enterprise. This enterprise may be in
livelihood. manufacturing, services, as the trading sector. In the
(c) eradicate the practice of manual scavenging and case of non-individual enterprises, at least 51% of the
rehabilitate the manual scavengers shareholding and controlling stake should be held by
(d) release the bonded labourers from their bondage either and SC/ST or woman entrepreneur. The scheme
and rehabilitate them. is anchored by the department of Financial Services
Ans. (c): Campaign was started in 2001 known as (DFS), Ministry of Financial Services, Ministry of
Finance. So, option (1) is correct.
'Rashtriya Garima Abhiyaan' (National Campaign for • It provides for refinance window through Small
Dignity and Elimination of Manual Scavenging) has Industries Development Bank of India (SIDBI) with
proven to be a very innovative and effective program to an initial amount of Rs. 10,000 crore .
eliminate manual scavenging practice. The campaign So option (2) is correct.
39. 'Mission Indradhanush' launched by the
has liberated 11,000 women manual scavengers in
Government of India pertains to–
various parts of India and urged them to give up this (a) Immunization of children and pregnant women
caste-based, unconstitutional practice. In its efforts (b) Construction of smart cities across the country
campaign attain social, economic, political and cultural (c) India's own search for the Earth-like planets in
outer space
rehabilitation of manual Scavenger. (d) New Educational policy
IAS (Pre) GS 2016 Paper I 112 YCT
Ans. (a) : Mission Indradhanush was launched by the • Agriculture, Industry and Trade
Ministry of Health and Family welfare on 25th December,
41. With reference to pre-Packaged items in India, it
2014. The objective is to ensure that all children under is mandatory to the manufacturer to put which
the age of two years, as well as pregnant women, are of the following information on the main label, as
fully immunized with seven vaccine- preventable per the Food Safety and standards. (Packaging
diseases. By 2020 provide vaccination to all children who and labeling Regulations, 2011 ?
are either unvaccinated or partially vaccinated against: 1. List of ingredients including additives
Diptheria, Pertussis (whooping cough), Tetanus, 2. Nutrition information
Tuberculosis, Polio, Measles and Hepatitis B. The aim is 3. Recommendations, if any, made by the medical
to reach at least 90% vaccination. At present, the profession about the possibility of any allergic
immunisation is around 65%. reactions.
4. Vegetarian/non -vegetarian
40. With reference to 'Initiative for Nutritional
Select the correct answer using the code given
Security through intensive Millets Promotion'
below
which of the following statements is/are correct ? (a) 1, 2 and 3 (b) 2, 3 and 4
(Initiative for Nutritional Security through (c) 2, 4 (d) 1 and 4 only
Intensive `Millets Promotion)' Ans. (c): For every pre-packaged item in India, the
1. This initiative aims to demonstrate the improved following information on the label are given:
production and post-harvest technologies, and to • Name of the food,
demonstrate value addition techniques, in an • List of ingredients
integrated manner, with cluster approach. • Declaration of food additives
• Net quantity or net weight
2. Poor, small, marginal and tribal farmers have
• Batch identification
larger stake in this scheme.
• Name and address of the manufacturer
3. An important objective of the scheme is to • Date marking
encourage farmers of commercial crops to shift
• Veg/Non-Veg Declaration
to millet cultivation by offering them free kits of
critical inputs of nutrients and micro-irrigation 42. With reference to 'Financial Stability and
equipment. Development Council' consider the following
statements :
Select the correct answer using the code given
1. It is an organ of NITI Aayog.
below
2. It is headed by the Union Finance Minister
(a) 1 only (b) 2 and 3 only 3. It monitors macro prudential supervision of the
(c) only 1 and 2 (d) 1, 2 and 3 economy.
Ans. (c) : The Initiative for Nutritional Security through Which of the statements given above is/are
Intensive Millets Promotion aims to demonstrate the correct?
(a) 1 and 2 only (b) 3 only
improved production and post-harvest technologies in
(c) 2 and 3 only (d) 1, 2 and 3
an integrated manner with a cluster approach. So,
Ans. (c) : The Central Government is at a consensus to
statement (1) is correct.
establish the Financial Stability and Development
• Millets are mainly grown in the region of low annual Council (FSDC) to strengthen and institutionalise the
rainfall and poor, arid soil. It is poor, small, marginal mechanism for maintaining financial stability and
and tribal farmers, who cannot afford costly irrigation development. FSDC will prefer the following roles to
practices, depend on the cultivation of millets. Hence, engage in marcoprudential supervision of the economy,
these farmers benefit from the schemes and have a including the functioning of large financial coordination
larger stake in this scheme. So, statement (2) is also issues. To focus on financial literacy and financial
correct. inclusion. To periodically look into issue relating to
financial development. The Council would have one
• There is no such provision of encouraging farmers of
sub-committee which would be headed by RBI
commercial crops to shift to millet cultivation. So, (3)
Governor. The secretariat of the council would be in the
is not correct. department of Economic Affairs, Ministry of Finance.
IAS (Pre) GS 2016 Paper I 113 YCT
43. The term 'Base Erosion and Profit Shifting' is 1. Shishu : Covering loans upto Rs. 50,000.
sometimes seen in the news in the context of 2. Kishore : Covering loans above Rs. 50,000 and upto
(a) mining operation by multinational companies in Rs. 5,00,000.
resource-rich but backward areas 3. Tarun : Covering loans above Rs. 5,00,00 and upto
(b) curbing of the tax evasion by multinational Rs. 10,00,000.
companies • Money, Banking and Tax System,
(c) exploitation of genetic resources of a country by Centre State Financial Relations
multinational companies
46. The establishment of ''Payment Banks' is being
(d) Lack of consideration of environmental costs in
allowed in India to promote Financial inclusion .
the planning and implementation of
Which of the following statements is/are correct
developmental projects
in this context ?
Ans. (b) : Base erosion and profit shifting (BEPS) is 1. Mobile telephone companies and supermarket
related to tax avoidance strategies adopted by
chains that are owned and controlled by residents
companies that exploit gaps and mismatches in tax rules
are eligible to be promoters of Payment Banks.
to artificially move profits to low or no-tax locations.
2. Payment Banks can issue both credit cards and
The inclusive framework collaborates over 135
debit cards.
countries and jurisdictions to implement the BEPS
3. Payment Banks cannot undertake lending
measures and to curb BEPS.
activities.
• Double- tax avoidance treaties and tax-information Select the correct answer using the code given
exchange between member nations are made to curb below
this practice. (a) 1 only (b) 2 and 3 only
So, correct answer is option (b). (c) 1,2 and 3 (d) 1 and 3 only
44. Recently, India's first 'National Investment and Ans. (d): Payments banks is an Indian new model of
Manufacturing Zone' was proposed to be set up banks conceptualised by the Reserve Bank of India
in (National Investment and Manufacturing (RBI) without issuing credit. These banks can accept a
Zone)' restricted deposit, which is currently limited to
(a) Andhra Pradesh (b) Gujarat `200,000 per customer and may be increased further.
(c) Maharashtra (d) Uttar Pradesh These banks cannot issue loans and credit cards. which
Ans. (a): In order to boost the 'Make in India' is currently limited to Rs 2,00,000 per customer and
campaign, the first national investment and may be increased further. These banks cannot issue
manufacturing zone come up in Andhra Pradesh in loans and credit cards. Hence statement 2 is not
2015. The state assured the centre of availability of correct.
10sq. km of land in one place in Prakasham district. 47. The term 'Core Banking Solution' is sometimes
45. Pradhan Mantri MUDRA Yojana is aimed at seen in the news. Which of the following
(a) bringing the small entrepreneurs into formal statements best describe/describes this term ?
financial system 1. It is a networking of a bank's branches which
(b) Providing loans to poor farmers for cultivating enables customers to operate their accounts
particular crops
from any branch of the bank on its network
(c) Providing pensions to old and destitute persons
regardless of where they open their accounts.
(d) funding the voluntary organizations involved in
the promotion of skill development and 2. It is an effort to increase RBI's control over
employment generation commercial banks through computerization.
Ans. (a) : Pradhan Mantri Mudra Yojana is aimed at 3. It is a detailed procedure by which a bank with
bringing sole-proprietors or entrepreneurs of small and huge non-performing assets is taken over by
medium enterprises into the formal financial system another bank.
'funding the unfounded'. Select the correct answer using the code given
Under PMMY there are three products i.e., as per the below
state of growth and funding needs of the beneficiary (a) 1 only (b) 2 and 3 only
micro unit- (c) 1 and 3 only (d) 1, 2 and 3

IAS (Pre) GS 2016 Paper I 114 YCT


Ans. (a) : Statement (1) : Core Banking Solution (CBS) (c) It is the ratio between the value of exports and
is the networking of branches, which enables customers that of imports between two countries
to operate their accounts and avail banking services (d) It is the number of months of imports that
from any branch of the Bank of CBS network, could be paid for by a country's international
regardless of where customer maintains their account. reserves.
So, (1) is correct. Ans. (d): Import cover is an important indicator of the
Statement (2) and (3) : Unlike a regular bank, a stability of the currency. It measures the number of
payment bank can't loan money to people or issue credit months of money available in the national bank to cover
cards. While the payment bank can't issue credit cards, the cost of imports or the stock of foreign exchange
they can issue ATM and debit cards. reserves in terms of months of retained imports of
48. What is/are the purpose/purposes of the goods as at the end of the year. During the currency
'Marginal Cost of Funds based Lending Rate crisis of 2013, when foreign exchange reserves fell to
(MCLR)' announced by RBI ? around $ 275 billion, import cover dipped to around
[Marginal Cost of Funds based Lending Rate seven months.
(MCLR)]' 50. India's ranking in the 'Ease of Doing Business
1. These guidelines help improve the transparency Index' is sometimes seen in the news. Which of
in the methodology followed by banks for the following has declared that ranking ? (Ease of
determining the interest rates on advances. Doing Business Index)'
2. These guidelines help ensure availability of bank (a) Organization for Economic Cooperation and
credit at interest rates which are fair to the Development ( OECD)
borrowers as well as the banks. (b) World Economic Forum
Select the correct answer using the code given (c) World Bank
below (d) World Trade Organization (WTO)
(a) 1 only (b) 2 only Ans. (c): In World Banks’ Ease of Doing Business
(c) Both 1 and 2 (d) Neither 1 nor 2 Report, India’s position has improved from 142 to 130.
Ans. (c): The Reserve Bank of India has brought a new This is on account of the ease of starting a business
dealing with construction permits and getting electricity
methodology of setting lending rates by commercial
permits. Now, companies can get connected to the grid
banks under the name Marginal Cost of Funds based and get on with their business 14 days sooner than
Lending Rate (MCLR). It has modified the existing before. The number of days taken to start a new
base rate system from April 2016 onwards. As per the business has gone up marginally from last year, from
new guidelines by the RBI, banks have to prepare the 28.4 to 29. Credit is one area where India performs
Marginal Cost of Funds based Lending Rate (MCLR), poorly and needs to improve. Access to credit and ease
which will be the internal benchmark lending rates. of paying taxes has worsened, according to the World
Bank's Doing Business Report 2016.
Based upon this MCLR interest rate for different types
51. What is /are the purpose/ purposes of
of customers should be fixed in accordance with their Government's ' Sovereign Gold Bond Scheme'
riskiness. The base rate will be now determined on the and 'Gold Monetization Scheme' ?
basis of the MCLR calculation. The MCLR should be 1. To bring the idle gold lying with Indian
revised monthly by considering a few factors, including households into the economy.
the repo rate and other borrowing rates. Specifically the 2. To promote FDI in the gold and Jewellery sector.
repo rate and other borrowing rates that were not 3. To reduce India's dependence on gold imports.
explicitly considered under the base rate system. Select the correct answer using the code give
below :
• Foreign Trade and Economic (a) 1 only (b) 2 and 3 only
Organization of India, Stock Exchange (c) 1 and 3 only (d) 1, 2 and 3
Ans. (c): The sovereign gold bond scheme was
49. Which of the following best described the term
launched in November 2015 to reduce the demand for
'import cover', sometimes seen in the news ?
(a) It is the ratio of value of imports to the Gross physical gold and shift a part of the domestic saving
Domestic product of a country used for the purchase of gold into financial savings.
(b) It is the total value of imports of a country in a Gold and crude oil have a significant role in India's
year widening current account deficit.

IAS (Pre) GS 2016 Paper I 115 YCT


SCIENCE (a) 1 and 2 only (b) 3 only
(c) 2 and 3 only (d) 1, 2 and 3
• Medical Science, Biology Ans. (a): The Kharai camels are a unique breed of the
camel found in Kutch of Gujarat and feed on mangroves
52. Which of the following statements is/are correct ?
on the island offshore. Hence, statement (2) is correct.
Viruses can infect–
• The Kharai camels can swim up to three kilometres
1. Bacteria
into the sea in search of mangroves, their primary
2. Fungi
food. Hence, statement (1) is correct.
3. Plants
• People consume their milk, while male calves are sold
Select the correct answer using the code given
for economic returns. Hence it can be domesticated.
below :
Hence, statement (3) is incorrect.
(a) 1 and 2 only (b) 3 only
(c) 1 and 3 only (d) 1, 2 and 3 • Given the breed's ability to survive both on land and
sea, the Kharai camel is one of the most preferred
Ans. (d): Viruses are known to infect almost any kind
choices of graziers in the arid coastal region of Kutch.
of host that has living cells. Animals, plants, fungi and
bacteria are all subject to viral infection. • Science & Technology
A bacteriophage is a virus that infects and replicates
55. On which of the following can you find the
within a bacterium. Mycoviruses are viruses that infect
Bureau of Energy Efficiency Star Label ?
fungi. A host of other viruses affect plants.
l. Ceiling fans
• Agriculture, Animal Husbandry and 2. Electric geysers
Dairy 3. Tubular fluorescent lamps
Select the correct answer using the code given
53. Which of the following is /are the advantage/
below :
advantages of practicing drip irrigation?
1. Reduction in weed (a) 1 and 2 only (b) 3 only
(c) 2 and 3 only (d) 1, 2 and 3
2. Reduction in soil salinity
3. Reduction in soil erosion Ans. (d): The Bureau initiated the Standards for
Select the correct answer using the code given Labeling programme for equipment and appliances in
below : 2006 to provide the consumer as an informed choice
(a) 1 and 2 only about the energy-saving and thereby the cost-saving
(b) 3 only potential of the relevant marketed product scheme is
(c) 1 and 3 only invoked for 21 equipment/ appliances, i.e., electric
(d) None of the above is an advantage of practicing storage type geyser, ceiling fans Tubular Fluorescent
drip irrigation Tube Lights, Frost Free Refrigerators, Distribution
Ans. (c): Drip irrigation is the application of small Transformers, Induction Motors, Direct, Cool
quantities of water directly into above and below the Refrigerator, Color TVs, Agricultural, pumps sets, LPG
soil surface, usually as discrete drops as tiny streams stoves, washing machine, Laptops, Room air
with the help of emitters placed along a delivery line. It Conditioners, ballast, floor standing ACs, office
is also known as Micro-irrigation. The advantage/ automation products, Diesel Generating sets and Diesel
advantages of practicing drip irrigation include pumpsets.
reduction in weed, reduction in soil erosion, reduction
56. India is an important member of the
in soil salinity is not the advantage of drip irrigation
‘International Thermonuclear Experimental
practices.
Reactor’. If this experiment succeeds, what is the
54. What is/are unique about ‘Kharai camel’ a breed
found in India? immediate advantages for India ?
1. It is capable of swimming up to three kilometers (a) It can use thorium in place of uranium for power
in seawater generation
2. It survives by grazing on mangroves. (b) It can attain a global role in satellite navigation
3. It lives in the wild and cannot be domesticated. (c) It can drastically improve the efficiency of its
Select the correct answer using the code given fission reactors in power generation
below : (d) It can build fusion reactors for power generation
IAS (Pre) GS 2016 Paper I 116 YCT
Ans. (d) : The member countries of International India's observatory will be the fourth in space, after the
thermonuclear Experimental Reactor (ITER) are- China, Hubble (USA), Russia's Spektr R and Suzaku of Japan.
the European Union, India, Japan, Korea, Russia and ASTROSAT with a lift-off mass of about 1513 kg, was
the United States are now engaged in a 35-year launched into a 650 km orbit inclined at an angle of 6
collaboration to build and operate the ITER degrees to the equator by PSLV-C 30.
experimental device and together bring fusion to the 59. Consider the following statements :
point where a demonstration fusion reactor can be The Mangalyaan launched by ISRO
designed will be the first fusion device to test the 1 is also called the Mars Orbiter Mission
integrated technologies, materials, and physics regimes 2. made India the second country to have a
necessary for the commercial production of fusion- spacecraft orbit the Mars after USA.
based electricity. 3. made India the only country to be successful in
• If this experiment succeeds, the immediate advantage making its spacecraft orbit the mars in its very
of India is that it can build fusion reactors for power first attempt.
generation. So, the correct answer is option (d). Which of the statements given above is/are correct ?
57. Regarding ‘DigiLocker’, sometimes seen in the (a) 1 only (b) 2 and 3 only
news, which of the following statements is/ are (c) 1 and 3 only (d) 1, 2 and 3
correct ? Ans. (c) : The Mars orbiter Mission, also called
1. It is a digital locker system offered by the Mangalyaan, is a space probe orbiting Mars since 24
Government under Digital India Programme. September 2014. It was launched on 5 November, 2013
2. It allows you to access your e-documents by the Indian Space Research Organization. India is the
irrespective of your physical location. first Asian country and ISRO is the fourth space agency
Select the correct answer using the code given to send a satellite to Mars.
below : It is the first interplanetary mission realized by India
(a) 1 only (b) 2 only and the first Indian spacecraft to incorporate full -scale
(c) Both 1 and 2 (d) Neither 1 nor 2 onboard autonomy to overcome the long distances and
Ans. (c): Digilocker is one of the key initiatives under the communication gaps due to non-visibility periods.
the Digital India Programme. This was released by the It is also the first Mars mission in the world to succeed
Department of Electronics and Information Technology mars orbit insertion in the first attempt.
(DeitY), Government of India. 60. In the context of the developments in
The space can be utilized for storing personal bioinformatics, the term ‘transcriptome’,
documents like university certificates, permanent sometimes seen in the news, refere to-
account number (PAN) cards, voter ID cards, and the (a) a range of enzymes used in genome editing
URIs of the e-documents issued by various issuer (b) the full range of m RNA molecules expressed
departments. by an organism
58. With reference to ‘Astrosat’, the astronomical (c) the description of the mechanism of gene
observatory launched by India, which of the expression
following statements is/are correct ? (d) a mechanism of genetic mutation taking place in
1. Other than USA and Russia, India is the only cells
country to have launched a similar observatory Ans. (b): Bioinformatics refers to the application of
into space. computer technology to the management of biological
2. Astrosat is a 2000 kg satellite placed in an orbit at information.
1650 km above the surface of the Earth. Transcriptome refers to the full range of messenger
Select the correct answer using the code given RNA (mRNA) molecules expressed from the genes of
below :
an organism.
(a) 1 only (b) 2 only
(c) Both 1 and 2 (d) Neither 1 nor 2 61. ‘Project Loon’, sometimes seen in the news, is
Ans. (d): ASTROSAT is India's first dedicated multi- related to
wavelength space observatory. ASTROSAT will (a) Waste management technology
observe the universe in the optical, ultraviolet, low and (b) Wireless communication technology
high energy X-Ray regions of the electromagnetic (c) Solar power production technology
spectrum. (d) Water conservation technology

IAS (Pre) GS 2016 Paper I 117 YCT


Ans. (b): Project loon is a network of stratospheric 64. With reference to the Agreement at the UNFCCC
balloons that help to connect rural and remote regions Meeting in Paris in 2015, which of the following
of the world to internet connectivity via wireless statements is /are correct.
communication technology. 1. The Agreement was signed by all the member
• Project loon is a research and development project
countries of the UN and it will go into effect in
being developed by Google X.
• It helps to improve communication during natural 2017.
disasters to affected regions. 2. The Agreement aims to limit the greenhouse gas
62. With reference to ‘Bitcoins’ sometimes seen in the emissions so that the rise in average global
news, which of the following statements is/are temperature by the end of this century does not
correct ? exceed 2oC or even 1.5oC above pre-industrial
1. Bitcoins are tracked by the Central Banks of the levels.
countries.
3. Developed countries acknowledged their
2. Anyone with a Bitcoin address can send and
receive Bitcoins from anyone else with a Bitcoin historical responsibility in global warming and
address . committed to donate $ 1000 billion a year from
3. Online payments can be sent without either side 2020 to help developing countries to cope with
knowing the identity of the other. climate change.
Select the correct answer using the code given Select the correct answer using the code given
below :
below:
(a) 1 and 2 only (b) 2 and 3 only
(c) 3 only (d) 1, 2 and 3 (a) 1 and 3 only (b) 2 only
Ans. (b) : Bitcoins can be sent directly to anyone, (c) 2 and 3 only (d) 1, 2 and 3
st
anywhere in the world. The only requirement is that you Ans. (b): 21 Conference of the Parties (COP 21) is
should have the Bitcoin address of the recipient. The also known as Paris Agreements, successfully
process doesn't require you to reveal your identity. So, concluded in Paris under the United Nations Framework
(2) and (3) are correct. Convention on Climate Change (UNFCCC). The
The system is a peer-to peer and transactions take place
agreement is scheduled to go into effect from 2020.
between users directly, without an intermediary. So this
Statement (1) is not correct.
is not tracked by the Central bank of any intermediary.
One of the main focuses of the agreement is to hold the
So (1) is incorrect.
increase in the global average temperature to well below
ENVIRONMENT AND ECOLOGY 2ºC above pre-industrial level and on driving efforts to
limit it even further to 1.5ºC. It covers all the crucial
63. Consider the following statements : areas identified as essential for a comprehensive and
1. The international Solar Alliance was launched at balanced agreement, including migration adaption, loss
the United Nations Climate Change Conferencre and damage finance, technology development and
in 2015. transfer, capacity building and transparency of action
2. The Alliance includes all the member countries and support. A marked departure from the past is the
of the United Nations. Agreements bottom-up approach, allowing each nation
Which of the statements given above is /are correct: to submit its own national plan for reducing greenhouse
(a) 1 only (b) 2 only gas emissions, rather than trying to repeat a top-down
(c) Both 1 and 2 (d) Neither 1 nor 2 approach advocated by the Kyoto protocol, giving each
Ans. (a) : The COP-21 Climate Conference was held in country an emission reduction target.
Paris, France, from 30th November to 12th December Statement (3) is not correct.
2015. During this conference, India and France have 65. Consider the following statements :
launched the International Solar Alliance (ISA). ISA 1. The Sustainable Developments Goals were first
includes an alliance of 121 countries located between proposed in 1972 by a global think tank called the
Tropic of Cancer and Tropic of Capricorn. While ISA ‘Club of Rome’.
was launched on the sidelines of the Paris summit, 2. The Sustainable Development Goals have to be
therefore, statement (1) is right, but it includes only 121 achieved by 2030.
countries between the tropic of Capricorn and the tropic Which of the statements given above is/are correct?
of Cancer receiving sunlight for 300 days as more. (a) 1 only (b) 2 only
Hence statement (2) is wrong. (c) Both 1 and 2 (d) Neither 1 nor 2
IAS (Pre) GS 2016 Paper I 118 YCT
Ans. (b): At the United Nations Sustainable Summit on Select the correct answer using, the code given
25 September 2015, world leaders adopted the 2030 below:
Agenda for Sustainable Development which includes a (a) 1 only (b) 2 and 3 only
set of 17 sustainable development goals (SDGS) to end (c) 1 and 3 only (d) 1, 2 and 3
poverty, fight inequality and injustice and tackle climate Ans. (c) : United Nations Convention to Combat
change by 2030. The history of the SDGS can be traced Desertification (UNCCD) established in 1994 is the sole
to 1972 when governments met under the auspices of legally binding international agreement linking
the United Nations Human and Environment environment and development to sustainable land
Conference. The 'Club of Rome' is a global think tank management.
that deals with a range of international political issues. Though desertification affects Africa the most, where
66. The term ‘Intended Nationally Determined two-thirds of the continent is desert or drylands in
Contribution’ is sometimes seen in the news in Africa, United States, Latin America and the Caribbean
the context of : have a high percentage of degraded lands. The UNCCD
(a) Pledges made by the European countries to is particularly committed to a bottom-up approach
rehabilitate refugees from the war-affected encouraging the participation of local people in
Middle East. combating desertification and land degradation. The
(b) Plan of action outlined by the countries of the UNCCD secretariat facilitates cooperation between
world to combat climate changes. developed and developing countries, particularly around
(c) Capital contributed by the member countries in knowledge and technology transfer for sustainable land
the establishment of Asian Infrastructure management.
Investment Bank . 68. In which of the following regions of India are you
(d) Plan of action outlined by the countries of the most likely to come across the ‘Great Indian
world regarding Sustainable Development Hornbill’ in its natural habitat ?
Goals. (a) Sand deserts of northwest India
Ans. (b) : Countries across the globe adopted the (b) Higher Himalayas of Jammu and Kashmir
International Climate Agreement at the UN Framework (c) Salt marshes of western Gujarat
Convention on Climate Change (UNFCCC), Conference (d) Western Ghats
of the Parties (COP-21) in Paris 2015.
Ans. (d): Great Hornbills are found in three separate
Under the agreement, countries publicly outlined what
areas in south Asia, in the Western Ghats, the
post-2020 climate actions they intended to take under
Himalayans foothills in Uttarakhand to south Nepal and
the new international agreement, known as their
Bhutan, and north-east India.
Intended Nationally Determined Contributions
They occur in primary evergreen and moist deciduous
(INDSCs), largely determine whether the world
forest mainly in lowland forests, but they can be found
achieves the long-term goals of the Paris Agreement to
upto 2000 meters in some areas.
hold the increase in global average temperature to
All other options except from (d) can thus be easily
below 2ºC, to peruse efforts to limit the increase to
eliminated.
1.5ºC and to achieve net- zero emissions in the second
69. Which of the following are the key features of
half of this century.
‘National Ganga River Basin Authority
67. What is/are the importance /importances of the
(NGRBA)’ ?
‘United Nations Convention to Combat
Desertification’ ? 1. River basin is the unit of planning and
1. It aims to promote effective action through management.
innovative national programmes and supportive 2. It spearheads the river conservation efforts at
international programmes and supportive inter- the national level.
national partnerships. 3. One of the Chief Ministers of the State through
2. It has a special / particular focus on South Asia which the Ganga flows becomes the Chairman
and North Africa regions, and its secretariat
facilitates the allocation of major portion of of NGRBA on rotation basis .
financial resources to these regions. Select the correct answer using the code given
3. It is committed to bottom-up approach, below:
encouraging the participation of local people in (a) 1 and 2 only (b) 2 and 2 only
combating the desertification. (c) 1 and 3 only (d) 1, 2 and 3
IAS (Pre) GS 2016 Paper I 119 YCT
Ans. (a) : The Central Government, by a notification Ans. (b): Statement (1) In order to safeguard and
dated 20.02.2009, has set up the National Ganga River support the world's agricultural heritage systems, in
Basin Authority (NGRBA) as an empowered planning 2002, FAO started an initiative for the dynamic
financing, monitoring and coordinating authority for the conservation of the Globally Important Agricultural
Ganga River, in the exercise of the powers conferred Heritage System (GIAHS). So, clearly, modern
under the Environment (Protection) Act, 1986. The practices wouldn't be relevant to the initiative (1) is
prime minister is the ex-officio chairperson of the incorrect.
Authority, and it has as its members the union ministers Statement (2) : The initative intends to safeguard the
concerned and the chief ministers of states through social, cultural, economic and environmental goods and
which Ganga flows, viz, Uttarakhand, Uttar Pradesh, services of these heritage systems. So, (2) is correct.
Bihar, Jharkhand and West Bengal, among others. The Statement (3) : There is no such provision. GI is
objective of the Authority is to ensure effective accorded to products from a certain origin (e.g.
abatement of pollution and conservation of the river Hyderabadi Biryani) and not to traditional agricultural
Ganga by adopting a holistic approach with the river systems.
basin as the unit of planning. The functions of the 71. Consider the following pairs :
Authority include all measures necessary for planning Terms sometimes seen Their origin
and execution of programmes for abatement of In the news
pollution in the Ganga in keeping with sustainable 1. Annex-I Countries Cartagena Protocol
development needs. 2. Certified Emissions Nagoya Protocol
• National Ganga River Basin Authority (NGRBA) Reductions
established in 2009 and mandated to take up 3. Clean Development Kyoto Protocol
regulatory and developmental functions with Mechanism
sustainability needs for effective abatement of Which of the pairs given above is /are correctly
pollution and conservation of the river Ganga by matched?
adopting a river basin approach for comprehensive (a) 1 and 2 only (b) 2 and 3 only
planning and management. (c) 3 only (d) 1, 2 and 3
Ans. (c) : The Clean Development Mechanism (CDM),
• River basin is the unit of planning and management.
created multilaterally under the UNFCCC is one of the
This is an internationally accepted strategy for
mitigation instruments under the Kyoto Protocol. The
integrated management of rivers. NGRBA will
CDM, defined in Article 12 of the protocol, allows a
spearhead river conservation efforts at the national
country with an emission-reduction as an emission-
level and implementation at the state level will be by
limitation commitment under the Kyoto Protocol
the state Agencies and Urban Local Bodies.
(Annex B party) to implement on the emission-
70. The FAO accords the status of ‘Globally
reduction project in developing countries. Such projects
Important Agricultural Heritage Systems
can earn saleable certified emission reduction (CER
(GIAHS)’ to traditional agricultural systems.
credits), each equivalent to one time of CO2, which can
What is the overall goal of this initiative-
be counted towards meeting Kyoto targets.
1. To provide modern technology, training in
Annex -I Countries are parties to the Kyoto Protocol.
modern farming methods and financial support
of local communities of identified GIAHS so as 72. Which of the following best described/describe
to greatly enhance their agricultural productivity. the aim of ‘Green India Mission’ of the
2. To identify and safeguard eco-friendly Government of India ?
traditional farm practices and their associated 1. Incorporating environmental benefits and costs
landscapes, agricultural biodiversity and
into the Union and State Budgets thereby
knowledge systems of the local communities.
3. To provide Geographical Indication status to all implementing the ‘green accounting’.
the varieties of agricultural produce in such 2. Launching the second green revolution to
identified GIAHS. enhance agricultural output so as to ensure food
Select the correct answer using the code given security to one and all in the future.
below: 3. Restoring and enhancing forest cover and
(a) 1 and 3 only (b) 2 only responding to climate change by a combination
(c) 2 and 3 only (d) 1, 2 and 3 of adaptation and mitigation measures.
IAS (Pre) GS 2016 Paper I 120 YCT
Select the correct answer using the code given Select the correct answer using the code given
below: below
(a) 1 only (b) 2 and 3 only (a) 1 and 2 only (b) 3 only
(c) 3 only (d) 1, 2 and 3 (c) 2 and 3 only (d) 1,2 and 3
Ans. (c): Green India Mission was launched in 2014. It Ans. (d): Reducing emissions from deforestation and
is one of the eight missions under the National Action forest degradation (REDD) is a mechanism that has
Plan on Climate change (NAPCC). Its objective is to been under negotiation by the United Nations
respond to climate change by a combination of Framework Convention on Climate Change (UNFCCC)
adaptation and mitigation measures. It is aimed at since 2005, with the objective of mitigating climate
protecting, restoring and enhancing India's diminishing change through reducing net emissions of greenhouse
forest cover. Hence, (3) is correct. gases through enhanced forest management in
It is one of the main objectives to enhance annual CO2 developing countries.
sequestration by 50-60 million tons by 2020. Hence, (1) • Under REDD + Developing countries will have to
and (2) are irrelevant to the mission. prove the 'result' that they have fought deforestation
73. With reference to an initiative called The without harming local communities or biological
Economics of Ecosystems and Biodiversity diversity, only then they will get the money.
(TEEB), Which of the following statements is/ are Therefore, (1) and (2) are correct. Although REDD+
correct?/ has galvanized significant support internationally
1. It is an initiative hosted by UNEP,IMF and among both developed and developing countries, its
World Economic Forum. implications for poverty alleviation at the local level
2. It is a global initiative that focuses on drawing remain unclear. Hence, statement (3) is incorrect.
attention to the economic benefits of Therefore, the answer is (a).
biodiversity. 75. What is ‘Greenhouse Gas Protocol’ ?
3. It presents an approach that can help decision- (a) It is an international accounting tool for
makers recognize, demonstrate and capture the government and business leaders to understand,
value of ecosystems and biodiversity. quantify and manage greenhouse gas emissions
Select the correct answer using the code given (b) It is an initiative of the United Nations to offer
below: financial incentives to developing countries to
(a) 1 and 2 only (b) 3 only reduce green house gas emissions and to adopt
(c) 2 and 3 only (d) 1, 2 and 3 eco-friendly technologies.
Ans. (c) : The Economics of Ecosystems and (c) It is an inter-governmental agreement ratified by
Biodiversity (TEEB) is a global initiative whose all the member countries of the ratified by all of
objective is to mainstream the values of biodiversity and the United Nations to reduce greenhouse gas
ecosystem services into the decision- making at all emission to specified level by the year 2022.
levels. It aims to achieve this good by following a (d) It is one of the multilateral REDD+initiatives
structured approach to valuation that helps decision- hosted by the World Bank
makers recognize the wide range of benefits provided Ans. (a) : The Greenhouse Gas (GHG) protocol
by ecosystems and biodiversity, demonstrate. Their developed by World Resources Institute (WRI) and
values in economic terms and, appropriate, capture World Business Council of Sustainable Development
those values in decision-making. Therefore, statement (WBCSD), sets the global standard for how to measure,
(2) and (3) are correct. It was started by G8+5 manage, and report greenhouse gas emissions. Many
companies and organization around the world are using
Environment Ministers in 2007.
GHG protocol standards and tools to manage their
74. Which of following statements is/are correct ?
emissions and become more efficient, resilient and
Proper design and effective implementation of
prosperous organizations.
UN-REDD+ Programme can significantly
76. Which reference to ‘Agenda 21’ sometimes seen
contribute to in the news, consider the following statements :
1. Protection of biodiversity 1. It is a global action plan for sustainable development.
2. Resilience of forest ecosystems 2. It originated in the World Summit on Sustainable
3. Poverty reduction Development held in Johannesburg in 2002.
IAS (Pre) GS 2016 Paper I 121 YCT
Which of the statements given above is /are 79. ‘European Stability Mechanism’, sometimes seen
correct? in the news, is an
(a) 1 only (b) 2 only (a) agency created by EU to deal with the impact of
(c) Both 1 and 2 (d) Neither 1 nor 2 millions of refugees arriving from Middle East.
Ans. (a): Agenda 21 is a non-binding action plan of the (b) agency of EU that provides financial assistance
United Nations with regard to sustainable development. to eurozone countries.
It is the product of the Earth Summit (UN Conference (c) agency of EU to deal with all the bilateral and
on Environment and Development) held in Rio de
multilateral agreements on trade.
Janeiro, Brazil, in 1992.
(d) agency of EU to deal with the conflicts arising
The implementation of the Agenda is primarily the
responsibility of governments, through national among the member countries.
strategies, plans, policies and procedures. International Ans. (b): The European Stability Mechanism is a
and regional organizations are also called upon to European Union Agency that provides financial
contribute to this effort. The broadest public assistance, in the form of loans, to eurozone countries as
participation and the active involvement of non- new capital to banks in difficulty.
governmental organizations and other groups are
encouraged. 80. Recently, for the first time in our country, which
of the following State has declared a particular
77. ‘Gadgil Committee Report’ and ‘Kasturirangan
Committee Report’ sometimes seen in the news, butterfly as ‘State Butterfly’?
are related to (a) Arunachal Pradesh (b) Himachal Pradesh
(a) Constitutional reforms (c) Karnataka (d) Maharashtra
(b) Ganga Action Plan Ans. (d): Maharashtra has become the first state in the
country to have a state as the Butterfly State. The
(c) Linking of rivers
Government has declared the Blue Mormon (Papilio
(d) Protection of Western Ghats
polymnestor) as the state butterfly.
Ans. (d): The Kasturirangan panel was set up to study
81. A recent movie titled The Man Who Knew
the Gadgil committee report on the Western Ghats. The
Infinity is based on the biography of
Kasturirangan report seeks to bring just 37% of the
(a) S. Ramanujan (b) S. Chandrasekhar
Western Ghats under the Ecologically Sensitive Area
(c) S. N. Bose (d) C.V. Raman
(ESA) zones-down from the 64% suggested by the
Ans. (a): 'The man who knew Infinity' movie is based
Gadgil report.
on a book by the same name written by Robert Kanigel
It recommended prohibition on development and on the life and times of Ramanujan. The film stars Dev
commercial activities like mining, thermal power plants, Patel as the real -life Srinavasa Ramanujan.
polluting industries and large housing plans in Distinguished mathematicians Manjul Bhargava and
Ecologically Sensitive Areas (ESA) zones. Ken Ono are Associate producers of the film.
82. With reference to ‘LiFi’ recently in the news.
CURRENT AFFAIRS Which of the following statements is/ are correct?
78. The term ‘Regional Comprehensive Economic 1. It uses light as the medium for high-speed data
Partnership' often appears in the news in the transmission.
context of the affairs of a group of countries
2. It is a wireless technology and is several time
known as
(a) G-20 (b) ASEAN faster than ‘WiFi’.
(c) SCO (d) SAARC Select the correct answer using the code given
Ans. (b): Regional Comprehensive Economic below:
Partnership (RCEP) is a proposed free trade agreement (a) 1 only (b) 2 only
(FTA) between the ten member states of the Association (c) 1 and 2 only (d) Neither 1 nor 2
Ans. (c): Statement 1: Li-Fi is loosely expanded to
of Southeast Asian Nations (ASEAN)+(RCEP) (Brunei,
Myanmar, Cambodia, Indonesia, Laos, Malaysia, Light- Fidelity using precisely controlled light
Philippines, Singapore, Thailand and Vietnam) and the modulation data that can be transmitted using regular,
six states with which ASEAN has existing FTAs visible light.
(Australia, China, Japan, South Korea and New Statement 2 : It will be 10-100 times faster than current
Zealand). generation, WiFi, and also very secure.

IAS (Pre) GS 2016 Paper I 122 YCT


83. With reference to ‘IFC Masala Bonds’, sometimes (b) use of piped natural gas in the kitchens of
seen in the news, which of the statements given households.
below is /are correct ? (c) installation of CNG kits in motor-cars.
1. The International Financed Corporation, which (d) installation of water meters in urban household.
offers these bonds, is an arm of the World Bank. Ans. (a) : A net metering is a billing mechanism that
2. They are the rupee-denominated bonds and are credits solar energy system owners for the electricity
source of debt financing for the public and they add to the grid. Net metering allows residential and
private sector. commercial customers who generate their own
Select the correct answer using the code given electricity from solar power to feed electricity they do
below. not use back into the grid.
(a) 1 only (b) 2 only Under, net metering system, customers install a solar or
(c) Both 1 and 2 (d) Neither 1 nor 2 a wind power plant on his premise, gets its grid -
Ans. (c): The World Bank Group (WBG) consists of connected with the electricity distribution company
five organizations-IBRD, IDA, IFC, MIGA and ICSID. (DISCOM).
International Finance Corporation (IFC) is a member of 87. Recently, our scientists have discovered a new
the World Bank Group (WBG) and at the same time, it's and distinct species of banana plant which attains
called as the World Bank's private- sector lending arm a height of about 11 metres and has orange-
with a stated objective to ensure that any financing does coloured fruit pulp. In which part of India has it
not result in harm to communities and the environment. been discovered ?
IFC Masala Bond : The first Masala bond was issued by (a) Andaman Islands
IFC in 2014 for infrastructure projects in India. They (b) Anaimalai forests
are the rupee-denominated bonds issued to offshore (c) Maikala Hills
investors and are a source of debt financing for the (d) Tropical rain forest of northeast
public and private sectors. Ans. (a): Scientists at the Botanical Survey of India
84. In the context of which of the following do you (BSI) discovered the new species of banana from a
sometimes find the terms ‘amber box, blue box remote tropical rain forest on the Little Andaman
and green box in the news? islands. The new species has Orange-Coloured fruit
(a) WTO affairs pulp and is about 11 meters high, whereas the usual
(b) SAARC affairs banana species is about three to four meters high.
(c) UNFCCC affairs 88. Which one of the following is the best description
(d) India –EU negotiations of FTA of ‘INS Astradharini, that was in the news
Ans. (a): In WTO terminology, subsidies, in general, recently?/`INS
are identified by "Boxes" which are given the colours of (a) Amphibious warfare ship
traffic lights. Green (permitted), Amber (slowed down, (b) Nuclear-powered submarine
i.e, reduced) and Red (forbidden). (c) Torpedo launch and recovery vessel
85. Recently, which one of the following currencies (d) Nuclear – powered aircraft carrier
has been proposed to be added to the basket of Ans. (c): INS Astradharini is an indigenously designed
IMF’s SDR? built torpedo launch and recovery vessel built by a shoft
(a) Ruble (b) Rand shipyard for the Indian Navy. It was commissioned to
the Indian Navy on 6th October 2015 at Naval Base in
(c) Indian Rupee (d) Renminbi
Visakhapatnam.
Ans. (d): In the most recently concluded review 89. What is ‘Greased Lightning-10 (GL-10)’, recently
(November 2015), the Executive Board of IMF decided in the news ?
that effective 10 October, 2016, the Chinese Renminbi (a) Electric plane tested by NASA
(RMB) will be included in the SDR basket as a fifth (b) Solar–powered two-seater aircraft designed by
currency, along with the US Dollar, Euro, Japanese, Japan
Yen and Pound Sterling. (c) Space observatory launched by China
(d) Reusable rocket designed by ISRO
86. ‘Net metering’ is sometimes seen in the news in
Ans. (a) : The Greased Lightning or GL-10 is the
the context of promoting the electric plane with 10 engines tested by NASA that can
(a) production and use of solar energy by the take off and land like a helicopter and fly efficiently like
households/consumers. an aircraft.
IAS (Pre) GS 2016 Paper I 123 YCT
90. ‘Doctors Without Borders (Medecins Sans 93. With reference to the International Monetary and
Frontieres)’, often in the news, is Financial Committee (IMFC), consider the
(a) A division of World Health Organization following statements :
(b) a non-governmental international organization 1. IMFC discusses matters of concern affecting the
(c) an inter-governmental agency sponsored by global economy , and advises the International
European Union Monetary Fund (IMF) on the direction of its work.
(d) a specialized agency of the United Nations 2. The World Bank participates as observer in
Ans. (b) : Doctors Without Borders or Medecins Sans IMFC’s meetings.
Frontiers is a non-profitable international medical Which of the statements given above is/are correct?
(a) 1 only
humanitarian organisation created by doctors and
(b) 2 only
journalists in France in 1971. MSF gives emergency aid (c) Both 1 and 2
to people affected by wars, epidemics, famine, natural (d) Neither 1 nor 2
disasters, man-made disasters, or areas where there is no Ans. (c): Statement 1 : The IMFC advises and reports
healthcare available. It provides this help to all people, to the IMF Board of Governors on the supervision and
regardless of their race, religion or political beliefs. In management of the international monetary and financial
1999 Doctors Without Borders was awarded the Noble system, including on response to unfolding events that
Peace Prize. may disrupt the system.
91. ‘Belt and Road Initiative’ is sometimes mentioned Statement 2 : A number of international institutions,
including the World Bank, participate as observers in
in the news in the context of the affairs of the IMFC's meetings.
(a) African Union (b) Brazil
94. With reference to ‘Organization for the
(c) European union (d) China prohibition of Chemical Weapons (OPCW)’
Ans. (d): The 'One Belt One Road' imitative has also consider the following statements :
been referred to as the Belt and Road Initiative (BRI), 1. It is an organization of European Union in
which was announced in 2013 by China's President Xi working relation with NATO and WHO.
2. It monitors chemical industry to prevent new
Jinping.
weapons from emerging.
• The 'Belt' also refers to the 'Silk Road Economic 3. It provides assistance and protection to Stated
Belt'. The objective of the imitative is to improve Parties against chemical weapons threats.
sub-regional and regional cooperation. Which of the statements given above is /are
correct?
• It aims to connect the East Asian economic regions
(a) 1 only
with the European economic circle and runs across (b) 2 and 3 only
the continents of Asia, Europe and Africa. (c) 1 and 3 only
(d) 1, 2 and 3
Miscellaneous Ans. (c) : The Organization for the Prohibition of
Chemical Weapons is the Hague-based autonomous
92. Which of the following is/are the indicator/ body, which was established in 1977 by the Chemical
indicators used by IFPRI to compute the Global weapons Convention, not EU.
Hunger Index Report ? The OPCW Member states share the collective goal of
preventing chemistry from ever again being used for
1. Undernourishment
warfare thereby strengthening international security.
2. Child stunting
95. With reference to the ‘Trans-Pacific Partnership’,
3. Child mortality consider the following statements :
(a) 1 only 1. It is an agreement among all the Pacific Rim
(b) 2 and 3 only countries except China and Russia.
(c) 1, 2 and 3 2. It is a strategic alliance for the purpose of
(d) 1 and 3 only maritime security only.
Ans. (c) : Four indicators are undernourished Which of the statements given above is /are
population, wastage in children under 5; waste in form correct ?
(a) 1 only
of very low height, i.e, very small child; stunting in (b) 2 only
children under 5; stunting is very low height for age, i.e. (c) Both 1 and 2
very short child, mortality rate under 5. (d) Neither 1 nor 2

IAS (Pre) GS 2016 Paper I 124 YCT


Ans. (d): Statement 1 : It involves 12 pacific-rim 2. The headquaters of New Development Bank is in
countries: the US, Japan, Malaysia, Vietnam, Singapore, Shanghai.
Brunei, Australia, New Zealand, Canada, Mexico, Chile Which of the statements given above is/are correct?
and Peru. It's not an agreement between all the pacific (a) 1only
rim countries except Russia and China. Only 12 out of
(b) 2 only
nearly 50+ Rim countries are part of it.
(c) Both 1 and 2
Statement 2 : It is a trade bloc. So (2) is incorrect.
(d) Neither 1 nor 2
96. Consider the following statements :
Ans. (b): The proposal for setting up the New
The India-Africa Summit Development Bank (NDB) was given by India at the 4th
1. held in 2015 was the third such Summit BRICS summit in 2012 held in Delhi.
2. was actually initiated by Jawaharlal Nehru in • The NDB was created by BRICS in 2014. Hence (1)
1951 is incorrect.
Which of the statements given above is/are • It is headquartered in Shanghai, China. Hence (2) is
correct? correct.
(a) 1 only • The first regional office of the NDB is in
(b) 2 only Johannesburg, South Africa. The major role of the
(c) Both 1 and 2 bank is to support public or private projects through
(d) Neither 1 nor 2 loans, guarantees, equity participation and other
Ans. (a): India- Africa Summit stated in 2008 (first- financial instruments.
summit) is one of the most spectacular diplomatic 99. Which of the following is NOT a member of ‘Gulf
exercises hosted by India since the 1893 Non-Aligned Cooperation Council’ ?
Movement (NAM) summit. (a) Iran
The second and third summits were held in 2011 and (b) Saudi Arabia
2015. So (1) is correct and (2) is incorrect. (c) Oman
97. `SWAYAM, an initiative of the Government of (d) Kuwait
India, aims at Ans. (a) : It is a political and economic alliance of six
(a) Promoting the Self Help Groups in rural areas Middle East countries like Saudi Arabia, Kuwait, the
(b) Providing financial and technical assistance to United Arab Emirates, Qatar, Bahrain and Oman.
young start-up entrepreneurs 100. ‘Global Financial Stability Report’ is prepared
by the
(c) Promoting the education and health of
adolescent girls (a) European Central Bank

(d) Providing affordable and quality education to (b) International Monetary Fund
the citizens for free (c) International Bank for Reconstruction and
Development
Ans. (d): SWAYAM Study Webs of Active Learning
for Young Aspiring Minds programme of ministry of (d) Organization for Economic Cooperation and
Development
Human Resource Development, GoI. Professors of
Ans. (b) : Global Financial Stability Report is
centrally funded institutions like IITs, IIMs, central
published by International Monetary Fund. It assesses
universities will offer online courses to the citizen of
the stability of global financial markets and emerging
India. All courses would be offered free of cost under
market financing. Topics covered in the GFSR usually
this programme. include system risk assessments in world side financial
98. Consider the following statements markets, worldwide debt management, emerging
1. New Development Bank has been set up by economic markets and current economic crises that
APEC. ~ could affect finances worldwide.

IAS (Pre) GS 2016 Paper I 125 YCT


UNION PUBLIC SERVICE COMMISSION
Civil Services (Preliminary Exam) - 2015
GENERAL STUDIES : PAPER-I
Time: 2 hours (Exam date : 23.08.2015) Maximum Number: 200
Select the correct answer using the code given
ANCIENT HISTORY
below:
• Prehistoric Period (a) 1 and 2 only (b) 2 and 3 only
1. With reference to the art and archaeological (c) 3 only (d) 1, 2 and 3
history of India, which one among the following Ans : (b) Feudalism in its various forms usually
was made earliest? emerged as a result of the decentralization of an
(a) Lingaraja Temple at Bhubaneswar empire authority of the centre which diminished as
(b) Rock-cut Elephant at Dhauli local/ feudal power emerged. The administrative and
(c) Rock-cut Monuments at Mahabalipuram
(d) Varaha Image at Udayagiri social structuring of society is around relationships
Ans : (b) The Lingaraja a temple at Bhubaneswar was derived from the holding of land in exchange for
built by Yayati Keshari in the 11th Century ( 1025- service or labour landed nobility held lands from the
1040AD) the Rock – cut Elephant at Dhauli (Odisha) Crown in exchange for military service and vassals
was built during the period of the Mauryan rules. were in turn into tenants of the noble, while the
Ashoka in 250BC happened around the Rock – cut peasants were obliged to live on their lord's land and
Monuments at Mahabalipuram were build during the
give him homage, labour, and a share of the produce,
Pallava reign in the 7th - 8th Centuries. The Varaha
statue at Udayagiri (Vidhisha Madhya Pradesh) was nationally in exchange for military protection.
built by Chandragupta –II during the Gupta Period.
MEDIEVAL HISTORY
• Buddhism, Jainism, Bhagavata and
Shaiva Religion • Bahmani and Vijayanagara Empire
2. Which of the following kingdoms were associated
with the life of the Buddha? 4. Who of the following founded a new city on the
1. Avanti 2. Gandhara south bank of a tributary to river Krishna and
3. Kosala 4. Magadha undertook to rule his new kingdom as the agent
Select the correct answer using the code given of a deity to whom all the land south of the river
below: Krishna was supposed to belong?
(a) 1, 2 and 3 (b) 2 and 3 only
(a) Amoghavarsha I (b) Ballala II
(c) 1, 3 and 4 (d) 3 and 4 only
(c) Harihara I (d) Prataparudra II
Ans : (d) Magadha, Kosala, Vaishali, Vajji etc. were
the kingdoms and regions that the wandering Buddha Ans : (c) Vijayanagara or ‘City of Victory’ was the
covered relentlessly. Jetavana was perhaps the best name of both a city and an empire. The empire was
known amongst the beautiful gardens provided to founded in the 14th century. In its heyday, it stretched
Buddha's entourage and it was close to Sravasti, capital from the river Krishna in the north to the extreme
of the Kosala Kingdom. Perhaps the second-most
important monastic resort of Buddha was the "bamboo south of the peninsula, in the Krishna-Tungabhadra
grove" in Rajgriha, capital of Magadha, provided by doab.
king Bimbisara.
• Mughal Empire
• Gupta Period and Post-Gupta Period
5. Consider the following pairs :
3. With reference to Indian history, which of the
Medieval Indian Present Region
following is/are the essential element/elements of
the feudal system? State
1. A very strong centralized political authority and 1. Champaka : Central India
a very weak provincial or local political authority 2. Durgara : Jammu
2. Emergence of administrative structure based on 3. Kuluta : Malabar
control and possession of land Which of the above pairs is/are correctly matched?
3. Creation of lord-vassal relationship between the (a) 1 and 2 (b) 2 only
feudal lord and his overlord (c) 1 and 3 (d) 3 only
IAS (Pre) GS 2015 Paper I 126 YCT
Ans : (b) Ans : (d) Important features were consistent and
Medieval period Current Area militant anti-imperialism, anti-landlordism, the organisation
Indian states of workers and peasants in trade unions and Kisan Sabha
Champaka : Chamba, Himachal the acceptance of a socialist vision of independent
Pradesh India and the socialist programme of the economic and
Durgara : Jammu social transformation of society. They were to give the
Congress and the national movement a socialist
Kuluta : Kullu, Himachal
direction, and that to achieve this objective they must
Pradesh organize the workers and peasants in their class
6. Consider the following : organizations, were struggled for their economic
The arrival of Babur into India led to the demands and make them the social base of the national
1. introduction of gunpowder in the subcontinent struggle. They preferred methods adopted by Congress.
2. introduction of the arch and dome in the region's
architecture • Freedom Struggle And National
3. establishment of Timurid dynasty in the region Movements
Select the correct answer using the code given 9. With reference to Rowlatt Satyagraha, which of
below. the following statements is/are correct?
(a) 1 and 2 only (b) 3 only
1. The Rowlatt Act was based on the
(c) 1 and 3 only (d) 1, 2 and 3 recommendations of the ‘Sedition Committee’.
Ans : (b) Chinese Gunpowder technology is believed 2. In Rowlatt Satyagraha, Gandhiji tried to utilize
to have arrived in India by the mid -14th Century but the Home Rule League.
could have been introduced much earlier by the 3. Demonstrations against the arrival of Simon
Mongols who had conquered both china and some Commission coincided with Rowlatt Satyagraha.
borderlands of India. Perhaps as early as the mid 13th Select the correct answer using the code given
Century the Unification of a large single Mongol below:
empire resulted in the free transmission of previously (a) 1 only (b) 1 and 2 only
top – Secret china’s Technology into Mongol (c) 2 and 3 only (d) 1, 2 and 3
conquered parts of India. Ans : (b) The Rowlatt committee was based on the
Sedition committee appointed in 1918 by the British
MODERN HISTORY Indian Government with Mr. Justice Rowlatt, an
7. Who of the following was/were economic English Judge, as its president. The purpose of the
critic/critics of colonialism in India? committee was to evaluate political terrorism in India,
1. Dadabhai Naoroji especially Bengal and Punjab. In organizing his,
2. G. Subramania Iyer Satyagraha, Gandhi tried to utilize three types of
3. R.C. Dutt political networks- The Home rule Leagues, certain
Select the correct answer using the code given below: PAN Islamist groups and a Satyagraha Sabha, which he
(a) 1 only (b) 1 and 2 only himself started in Bombay on 24th February.
(c) 2 and 3 only (d) 1, 2 and 3 10. Who of the following organized a march on the
Ans : (d) Dadabhai Naoroji, R.C. Dutt, Ranade, Tanjore coast to break the Salt Law in April
Gokhale, G. Subramania Iyer were among those who 1930?
grounded Indian nationalism firmly on the foundation (a) V.O. Chidambaram Pillai (b) C. Rajagopalachari
of anti- imperialism by highlighting economic critique (c) K. Kamaraj (d) Annie Besant
of colonialism. The drain theory was established by Ans : (b) To protest against the Salt Tax imposed by
Dadabhai Naoroji. According to increased poverty and
the British Rule in India, the Vedaranyam March (also
lower wages, the indirect products of colonial rule.
called as Vedaranyarm Satyagraha) was organized by
• Social, Culture Awakening, Caste C. Rajagopalachari in April, 1930 along with a group
of volunteers from the Indian National Congress,
Trade Union and Movement which started form Tiruchirappalli and ended in
8. With reference to Congress Socialist Party, Vedaranyam, which is a small coastal town of Tanjore
consider the following statements : District in Tamilnadu.
1. It advocated the boycott of British goods and
evasion of taxes. 11. Consider the following statements :
2. It wanted to establish the dictatorship of 1. The first woman President of the Indian National
proletariat. Congress was Sarojini Naidu.
3. It advocated separate electorate for minorities 2. The first Muslim President of the Indian
and oppressed classes. National Congress was Badruddin Tyabji.
Which of the statements given above is/are correct? Which of the statements given above is/are correct?
(a) 1 and 2 only (b) 3 only (a) 1 only (b) 2 only
(c) 1, 2 and 3 (d) None (c) Both 1 and 2 (d) Neither 1 nor 2
IAS (Pre) GS 2015 Paper I 127 YCT
Ans : (b) Annie Besant became the first women Ans : (b)
president of the Indian National Congress in 1917. • For cyclone formation a sea temperature of at least
Sarojini Naidu became the first Indian woman 26ºC is needed. In the South Eastern Pacific and
president of the Congress in 1925, the 40th annual South Atlantic region, cold currents are found. This
session of the Congress in Kanpur. In the third session leads to lower sea temperature, Hence cyclones do
of the Congress held in Madras on 27-28 December not originate there.
1887, Badruddin Tyabji was made the first Muslim
president. • Coriolis force is weak only in the region around 0-5
degrees North or South latitudes.
12. With reference to the Cabinet Mission, which of • Absence of land boosts cyclones. The presence of
the following statements is/are correct? land cutts off moisture from the cyclone.
1. It recommended a Federal Government.
2. It enlarged the powers of the Indian Courts. 15. Which one of the following pairs of States of
3. It provided for more Indians in the ICS. India indicates the easternmost and westernmost
Select the correct answer using the code given below: State?
(a) 1 only (b) 2 and 3 (a) Assam and Rajasthan
(c) 1 and 3 (d) None (b) Arunachal Pradesh and Rajasthan
Ans : (a) The Cabinet Mission plan of 1946 proposed (c) Assam and Gujarat
that there shall be a Union of India which was to be (d) Arunachal Pradesh and Gujarat
empowered to deal with the defence, foreign affairs Ans : (d) The capital of Arunachal Pradesh is Itanagar
and communications. The Union Government and its and was formed on 20 February, 1987. It is the
legislature were to have limited powers dealing with easternmost state in India. The capital of Gujarat is
finances, Foreign Affairs and Communications. The
Gandhinagar and was formed on 1 May, 1960. It is the
union would have the power necessary to raise the
finances to manage the subjects. westernmost state of India .
• All subjects other than the Union subjects and all the 16. Consider the following pairs :
residuary powers would be vested in the provinces. Place of Pilgrimage Location
• Thus, the Cabinet Mission plan proposed a weak 1. Srisailam : Nallamala Hills
centre with provincial autonomy-essentially 2. Omkareshwar : Satmala Hills
proposing a federal structure of government. 3. Pushkar : Mahadeo Hills
13. Which one of the following movements has Which of the above pairs is/are correctly
contributed to a split in the Indian National matched?
Congress resulting in the emergence of (a) 1 only (b) 2 and 3 only
`moderates' and `extremists'? (c) 1 and 3 only (d) 1, 2 and 3
(a) Swadeshi Movement Ans : (a) Srisailam is reputed for the shrine of Lord
(b) Quit India Movement Mallikarjuna on the flat top of Nallamala Hills
(c) Non-Cooperation Movement (Andhra Pradesh) Omkareshwar is situated on the
(d) Civil Disobedience Movement Mandhata hills in Vindhya Mountains of Madhya
Ans : (a) Surat split is an important event in the Pradesh. The Pushkar Valley is formed between the
Modern History of India. It took place in 1907 when Aravali Hills (Rajasthan).
the moderates paired company with the Extremists.
The split in the Congress was due to many reasons. 17. Consider the following rivers :
The moderates had controlled the congress from its 1. Vamsadhara 2. Indravati
very beginning and had their own ways of thinking and 3. Pranahita 4. Pennar
doing, which were not acceptable to the younger Which of the above are tributaries of Godavari?
generations who were impatient with the speed at (a) 1, 2 and 3 (b) 2, 3 and 4
which the moderates were moving and leading the (c) 1, 2 and 4 (d) 2 and 3 only
nation. This was the time of the Swadeshi Movement. Ans : (d) The Godavari river originates from
`

Trimabkeshwar in Nashik (Maharastra) and crosses


GEOGRAPHY OF INDIA south-central India and flows southeast into the Bay of
Bengal. Its length is about 1465 km. Its left bank
• Physical Structure, Drainage System, tributaries are Poorna, Pranahita, Indravati, Sabri,
Human Geography Taliperu, Wainganga, Penganga, Wardha and Dudhna.
The right bank tributaries are pravara, Manjira,
14. In the South Atlantic and South-Eastern Pacific Nasardi, Sindphana, Manair and Kinnerasani. Kumbh
regions in tropical latitudes, cyclone does not Mela also takes place on the banks of the Godavari
originate. What is the reason? river in Nasik.
(a) Sea surface temperatures are low
(b) Inter-Tropical Convergence Zone seldom occurs 18. Which one of the following regions of India has a
(c) Coriolis force is too weak combination of mangrove forest, evergreen forest
(d) Absence of land in those regions and deciduous forest?
IAS (Pre) GS 2015 Paper I 128 YCT
(a) North Coastal Andhra Pradesh WORLD GEOGRAPHY
(b) South-West Bengal
(c) Southern Saurashtra
(d) Andaman and Nicobar Islands • World Map/Climate
Ans : (d) Andaman and Nicobar island of India has a 23. Which one of the following countries of South-
combination of Mangrove forest, evergreen forest and West Asia does not open out to the
deciduous forest. Mediterranean Sea?
19. Consider the following States : (a) Syria (b) Jordan
1. Arunachal Pradesh (c) Lebanon (d) Israel
2. Himachal Pradesh
3. Mizoram Ans : (b) Jordan does not open out in the
In which of the above States do `Tropical Wet Mediterranean sea, as it is bounded by Syria and
Evergreen Forests' occur? Israel toward the side of the Mediterranean sea.
(a) 1 only (b) 2 and 3 only Jordan opens out in the Red Sea.
(c) 1 and 3 only (d) 1, 2 and 3
24. What explains the eastward flow of the
Ans : (c) Tropical wet evergreen forests are areas with equatorial counter-current?
an average temperature of 25º-27 and 250 cm of (a) The Earth's rotation on its axis
rainfall. These forests are found in India in Assam,
(b) Convergence of the two equatorial currents
Arunachal Pradesh, Mizoram, Manipur, Nagaland,
Tripura, and Andaman and Nicobar island. (c) Difference in salinity of water
(d) Occurrence of the belt of calm near the equator
• Monsoon, Forest, Soil, Irrigation Ans : (b) The equatorial counter currents are driven by
Projects and Agriculture distinct surface wind patterns in the tropics. Strong
20. Which one of the following National Parks has a westward trade winds result in westward surface flow
climate that varies from tropical to subtropical, in most of the tropical Atlantic and Pacific oceans.
temperate and arctic? However, several hundred miles (km) north of the
(a) Khangchendzonga National Park equator, the winds are much weaker in comparison.
(b) Nanda Devi National Park The stronger winds to the south pile up water where
(c) Neora Valley National Park
the winds are weak. As a result, the surface of the
(d) Namdapha National Park
ocean can be upto 6 inches (15 cm) higher and the
Ans : (d) Namdapha National Park is located in thermocline (region of strongest decrease of
Arunachal Pradesh. The climate of this area varies
temperature with increasing depth) as much as 328 ft.
from tropical to subtropical, temperate and arctic. It is
tropical and subtropical in southern regions and arctic (100m) deeper than it is directly to the north. The
type found in the northern part of the park. excess water flows eastward under the influence of the
Earth's rotation, giving rise to the equatorial counter
21. In India, in which one of the following types of
currents.
forests is teak a dominant tree species?
(a) Tropical moist deciduous forest 25. ''Each day is more or less the same, the morning
(b) Tropical rain forest is clear and bright with a sea breeze; as the Sun
(c) Tropical thorn scrub forest climbs high in the sky, heat mounts up, dark
(d) Temperate forest with grasslands clouds form, then rain comes with thunder and
Ans : (a) Tropical moist deciduous forests are found in lightning. But rain is soon over."
areas of moderate rain fall of 100 to 200 cm per year Which of the following regions is described in the
mean annual temperature of about 27º C. Teak trees above passage?
are the most dominant species of trees found in these (a) Savannah (b) Equatorial
forests. Bamboos, Sal, Shisham, Sandalwood, Khair, (c) Monsoon (d) Mediterranean
Kusum, Arjun, mulberry are some of the other
commercially important species found here. Ans : (b) Equatorial climate is characterized by
consistently high temperatures (around 30ºC), with
• Minerals, Power Resources, Industry plentiful precipitation (150-1,000 cm), heavy cloud
and Trade cover, and high humidity with very little annual
22. In India, the steel production industry requires temperature variation. Wet equatorial regions lie
the import of within about 10º latitude of the Equator.
(a) Saltpetre (b) Rock Phosphate
(c) Coking Coal (d) All of the above • Hydrosphere
Ans : (c) Steel companies in India require a large 26. Tides occur in the oceans and seas due to which
amount of coking coal, which has to be imported among the following?
because good quality coking coal is limited in India. 1. Gravitational force of the Sun
0.8 tons of Coking Coal is required to manufacture one 2. Gravitational force of the Moon
ton of steel. 3. Centrifugal force of the Earth
IAS (Pre) GS 2015 Paper I 129 YCT
Select the correct answer using the code given 2. The provisions contained in these Principles are
below : not enforceable by any court.
(a) 1 only (b) 2 and 3 only Which of the statements given above is/are
(c) 1 and 3 only (d) 1, 2 and 3 correct?
(a) 1 only (b) 2 only
Ans : (d) The periodical rise and fall of the sea level, (c) Both 1 and 2 (d) Neither 1 nor 2
once or twice a day, mainly due to attraction of sun Ans : (c) The inspiration of the Directive principles
and moon. Another factor is centrifugal force, which is included in the Indian constitution is derived from the
the force that acts to counter balance gravity. 1937 constitution of Ireland. These elements explain the
socio-economic democracy of the country. Under
• Human Geography and Cartography Article 37, the provisions contained in these elements
are not enforceable by any court.
27. Consider the following statements :
1. The winds which blow between 30o N and 60o S 30. ''To uphold and protect the Sovereignty, Unity
latitudes throughout the year are known as and Integrity of India" is a provision made in the
westerlies. (a) Preamble of the Constitution
2. The moist air masses that cause winter rains in (b) Directive Principles of State Policy
(c) Fundamental Rights
North-Western region of India are part of (d) Fundamental Duties
westerlies.
Ans : (d) Article 51 A (c) of the constitution of India
Which of the statements given above is/are says that it shall be the duty of every citizen of India to
correct? uphold and protect the sovereignty, unity and integrity
(a) 1 only (b) 2 only of India. This is a part of 4A of the constitution
(c) Both 1 and 2 (d) Neither 1 nor 2 consisting of the Fundamental Duties.
Ans : (b) Westerlies flow between latitudes 30-60º 31. The ideal of `Welfare State' in the Indian
North and 30-60º South and not 30º N and 60º S. Constitution is enshrined in its–
Western Disturbance occurs in India, Pakistan, (a) Preamble
Bangladesh and Nepal to describe on extra tropical (b) Directive Principles of State Policy
storm originating in the Mediterranean that brings (c) Fundamental Rights
sudden winter rain and snow to the northwestern parts (d) Seventh Schedule
of the Indian subcontinent. This is a non-monsoonal Ans : (b) The concept of a welfare state is
precipitation pattern driven by the westerlies. incorporated in the Indian constitution in directive
principles of state policy. It is described in Articles 36
INDIAN CONSTITUTION AND to 51 under part 4 of the constitution.
POLITICAL SYSTEM • Parliament & State Assemblies
32. Consider the following statements :
• Constitutional Development of India 1. The Rajya Sabha has no power either to reject or
28. The Government of India Act of 1919 clearly to amend a Money Bill.
defined – 2. The Rajya Sabha cannot vote on the Demands
for Grants.
(a) the separation of power between the judiciary 3. The Rajya Sabha cannot discuss the Annual
and the legislature Financial Statement.
(b) the jurisdiction of the central and provincial Which of the statements given above is/are
governments correct?
(c) the powers of the Secretary of State for India and (a) 1 only (b) 1 and 2 only
the Viceroy (c) 2 and 3 only (d) 1, 2 and 3
(d) None of the above Ans : (b) Rajya Sabha can only keep a money bill for
Ans : (b) It relaxed the central control over the not more than 14 days before which the bill should be
provinces by demarcating and separating the central either returned to Lok sabha with recommendations or
passed by the Rajya Sabha. In case of a disagreement
and provincial legislatures were authorized to make between both the houses, the bill is deemed passed in
laws on their respective list of subjects. However, the the form that was passed by Lok Sabha.
structure of government continued to be centralized Only the Lok Sabha can vote on the demand for grants.
and unitary. It further divided the provincial subjects, The budget (annual financial statement as mentioned
which, is what we know as diarchy. in the constitution) is discussed by Rajya Sabha.
• Citizenship Fundamental Duties 33. Consider the following statements :
Directive Principle of State Policy 1. The Executive Power of the Union of India is
vested in the Prime Minister.
29. Consider the following statements regarding the 2. The Prime Minister is the ex officio Chairman of
Directive Principles of State Policy : the Civil Services Board.
1. The Principles spell out the socio-economic Which of the statements given above is/are
democracy in the country. correct?
IAS (Pre) GS 2015 Paper I 130 YCT
(a) 1 only (b) 2 only • Panchayati Raj System Constitution
(c) Both 1 and 2 (d) Neither 1 nor 2
Ans : (d) Executive power of the Union of India is
and Amendments and Schedules
vested in the President of India. Cabinet Secretary is the 38. The provision in Fifth Schedule and Sixth
ex-officio head of the Civil Services Board, the Cabinet Schedule in the Constitution of India are made in
Secretariat, the Indian Administrative Service (IAS) and order to –
head of all civil services under the rules of business of the
Government of India. (a) protect the interests of Scheduled Tribes
(b) determine the boundaries between States
• Legislature: Council of Ministers (c) determine the powers, authority and responsibilities
of Panchayats
34. When a bill is referred to a joint sitting of both (d) protect the interests of all the border States
the Houses of the Parliament, it has to be passed
by Ans : (a) The fifth schedule of the constitution of India
deals with the administration and control of scheduled
(a) a simple majority of members present and voting
(b) three-fourths majority of members present and areas and scheduled tribes in these areas. The sixth
voting schedule to the constitution of India contains provisions
(c) two-thirds majority of the Houses concerning the administration of tribal areas in the states
(d) absolute majority of the Houses of Assam, Meghalaya, Tripura and Mizoram.
Ans : (a) Such a sitting is summoned by the president 39. The fundamental object of Panchayati Raj
to settle a deadlock between the two houses on a bill. system is to ensure which among the following?
The bill is passed by a simple majority i.e. a majority 1. People's participation in development
of the number of members present and voting. 2. Political accountability
35. Consider the following statements : 3. Democratic decentralization
1. The Legislative Council of a State in India can 4. Financial mobilization
be larger in size than half of the Legislative Select the correct answer using the code given
Assembly of that particular State. below:
2. The Governor of a State nominates the Chairman (a) 1, 2 and 3 only (b) 2 and 4 only
of Legislative Council of that particular State. (c) 1 and 3 only (d) 1, 2, 3 and 4
Which of the statements given above is/are Ans : (c) Article 243-B provides for a three-tier
correct?
(a) 1 only (b) 2 only Panchayati Raj System in India. It is a system of self-
(c) Both 1 and 2 (d) Neither 1 nor 2 government at the local level. The basic objective of
Ans : (d) Under Article 171 (1) and Article 172 of the this system is to enable the general public to
constitution, the Legislative council must not have participate in developmental administration and ensure
more than a third of the total membership of the democratic decentralization of power/governance.
Assembly of the state, and in no case fewer than 40
members.
36. There is a Parliamentary System of Government
ECONOMICS
in India because the • Economic Planning and National
(a) Lok Sabha is elected directly by the people
(b) Parliament can amend the Constitution Income / Budget
(c) Rajya Sabha cannot be dissolved
(d) Council of Ministers is responsible to the Lok 40. A decrease in tax to GDP ratio of a country
Sabha indicates which of the following?
Ans : (d) This is the bedrock principle of parliamentary 1. Slowing economic growth rate
government. The ministers are collectively responsible 2. Less equitable distribution of national income
to the parliament in general and to the Lok Sabha in Select the correct answer using the code given
particular (Article 75). below:
• Judiciary (Union and State) (a) 1 only (b) 2 only
(c) Both 1 and 2 (d) Neither 1 nor 2
37. Who/Which of the following is the custodian of
Ans : (a) The tax-to-GDP is an economic
the Constitution of India? measurement that compares the amount of taxes
(a) The President of India collected by a government to the amount of income
(b) The Prime Minister of India that country receives for its products. That income is
(c) The Lok Sabha Secretariat measured in terms of the gross domestic product or
(d) The Supreme Court of India
GDP, which is the sum of all products and goods sold,
Ans : (d) The Supreme Court of India is the apex court personal and government investment, and net exports.
in India. As stated by the Indian constitution, the
function of the supreme court of India is that of the By comparing this amount to the amount that is
custodian of the constitution, a court established by the collected in tax revenue, economists can get a rough
authority of a federal government and the uppermost idea of how much the economy of a specific
court of appeal. government is fueled by its tax collection.
IAS (Pre) GS 2015 Paper I 131 YCT
41. With reference to the Union Government, Ans : (a) Budget deficit is used to define a status of
consider the following statements :
financial health in which expenditures exceed revenue.
1. The Department of Revenue is responsible for
the preparation of Union Budget that is presented Introducing a new welfare scheme and expanding
to the Parliament. industries by budgetary support will expand the budget
2. No amount can be withdrawn from the expenditure, which will widen the deficit.
Consolidated Fund of India without the Rationalizing subsidies and reducing revenue
authorization from the Parliament of India. expenditure are two direct ways of reducing the fiscal
3. All the disbursements made from Public Account burden of the government of India.
also need the authorization from the Parliament
of India.
Which of the statements given above is/are correct?
• Agriculture, Industry and Trade
(a) 1 and 2 only (b) 2 and 3 only 45. In the `Index of Eight Core Industries', which
(c) 2 only (d) 1, 2 and 3 one of the following is given the highest weight?
Ans : (c) Department of Economic affairs prepares the (a) Coal production
Union Budget. Public Accounts includes provident
fund deposits, judicial deposits, saving bank deposits, (b) Electricity generation
departmental deposits, remittances and so on. The (c) Fertilizer production
payments from this account can be made without (d) Steel production
parliamentary appropriation. Such payments are Ans : (b) Core industries account for 38% in IIP.
mostly in the nature of banking transactions. So, only
Electricity with 10.3% has the highest weight among
statement (2) is correct.
Core industries viz. Coal, Fertilizer, Electricity, Crude
42. With reference to Indian economy, consider the Oil, Natural Gas, Refinery Product, Steel and Cement.
following statements :
1. The rate of growth of Real Gross Domestic 46. In India, markets in agricultural products are
Product has steadily increased in the last decade. regulated under the–
2. The Gross Domestic Product at market prices (in (a) Essential Commodities Act, 1955
rupees) has steadily increased in the last decade.
(b) Agricultural Produce Market Committee Act
Which of the statements given above is/are correct?
(a) 1 only (b) 2 only enacted by States
(c) Both 1 and 2 (d) Neither 1 nor 2 (c) Agricultural Produce (Grading and Marking)
Ans : (b) Due to the recession in 2008 the growth rate Act, 1937
of the Indian economy had declined for the next few (d) Food Products Order, 1956 and Meat and Food
years from 8-9% to 5-6%. So statement (1) is wrong. Products Order, 1973
Even though the growth rate declined, it never became
negative. So the GDP at market prices has always Ans : (b) In India, markets in agricultural products are
increased year on year for the last decade. regulated under the agricultural produce market
committee act enacted by states.
43. With reference to inflation in India, which of the
following statements is correct? 47. Which of the following brings out the `Consumer
(a) Controlling the inflation in India is the Price Index Number for Industrial Workers'?
responsibility of the Government of India only (a) The Reserve Bank of India
(b) The Reserve Bank of India has no role in
controlling the inflation (b) The Department of Economic Affairs
(c) Decreased money circulation helps in controlling (c) The Labour Bureau
the inflation (d) The Department of Personnel and Training
(d) Increased money circulation helps in controlling Ans : (c) The Labour Bureau brings out the Consumer
the inflation Price Index Number for Industrial Workers.
Ans : (c) Decreasing the money circulation decreases
the demands of goods and services, which helps in 48. The substitution of steel for wooden ploughs in
controlling inflation. It is mainly effective for demand- agricultural production is an example of–
pull inflation. (a) labour-augmenting technological progress
44. There has been a persistent deficit budget year (b) capital-augmenting technological progress
after year. Which of the following actions can be (c) capital-reducing technological progress
taken by the government to reduce the deficit? (d) None of the above
1. Reducing revenue expenditure Ans : (b) Labour augmenting is the technology that
2. Introducing new welfare schemes increases skills and productivity of the existing
3. Rationalizing subsidies labour force (example-teaching people how to use the
4. Expanding industries computer). Capital augmenting technology increase
Select the correct answer using the code given
the productivity of existing capital goods. In this
below:
(a) 1 and 3 only (b) 2 and 3 only case, replacement of wood with steel increases the
(c) 1 only (d) 1, 2, 3 and 4 productivity of the plough.

IAS (Pre) GS 2015 Paper I 132 YCT


• Money, Banking, Tax System, 52. With reference to Indian economy, consider the
following :
Financial Relation of Centre and State 1. Bank rate
49. With reference to the Fourteenth Finance 2. Open market operations
Commission, which of the following statements 3. Public debt
is/are correct? 4. Public revenue
1. It has increased the share of States in the central Which of the above is/are component/components
divisible pool from 32 percent to 42 percent. of Monetary Policy?
(a) 1 only
2. It has made recommendations concerning sector-
(b) 2, 3 and 4
specific grants.
(c) 1 and 2
Select the correct answer using the code given (d) 1, 3 and 4
below :
Ans : (c) Monetary policy refers to the policy of the
(a) 1 only (b) 2 only Central bank with regard to the use of monetary
(c) Both 1 and 2 (d) Neither 1 nor 2 instruments under its control to achieve the goals
Ans : (a) The Fourteenth Finance Commission (FFC) specified in the act.
has radically enhanced the share of the states in the The following are the main tools of RBI-
central divisible pool from the current 32 percent to 42 (i) Bank Rate
percent, which is the biggest ever increase in vertical (ii) Cash Reserve Ratio
tax deduction. The FFC has not made any (iii) Statutory liquidity Ratio
recommendation concerning sector-specific grants, (iv) Repo rate
(v) Reverse Repo Rate
unlike the Thirteenth Finance Commission.
(vi) Open market operations (OMO).
50. When the Reserve Bank of India reduces the Public debt and public revenue are matters falling
Statutory Liquidity Ratio by 50 basis points, under the fiscal policy.
which of the following is likely to happen?
(a) India's GDP growth rate increases drastically • Foreign Trade of India, Economic
(b) Foreign Institutional Investors may bring more Organization and Stock Market
capital into our country 53. The Government of India has established NITI
(c) Scheduled Commercial Banks may cut their Aayog to replace the–
lending rates (a) Human Rights Commission
(d) It may drastically reduce the liquidity to the (b) Finance Commission
banking system (c) Law Commission
Ans : (c) The SLR cut by the RBI in likely to give (d) Planning Commission
more elbow room for banks to cut rates. When SLR is Ans : (d) The central government established the NITI
reduced, banks have to park lesser money with RBI in Aayog on 1st January, 2015, replacing the 65 years old
the form of Securities, case etc so they have more Planning commission. The full form of NITI Aayog-
National Institution for Transforming India.
money to lend and the return on lending is also greater
than what they get in SLR; hence the scheduled 54. The problem of international liquidity is related
Commercial bank may cut their lending rates. to the non-availability of
(a) goods and services
51. Convertibility of rupee implies (b) gold and silver
(a) being able to convert rupee notes into gold (c) dollars and other hard currencies
(b) allowing the value of rupee to be fixed by market (d) exportable surplus
forces Ans : (c) The concept of international liquidity is
(c) freely permitting the conversion of rupee to other associated with international payment. These payments
currencies and vice versa arise out of international trade in goods and services
(d) developing an international market for currencies in and also in connection with capital movements
India between one country and another. International
liquidity is a foreign currency like dollars sterling or
Ans: (c) Convertibility of rupee implies freely
Gold in the reserve of any country. It is very useful to
permitting the conversion of rupee to other currencies
pay the amount of imported goods and reduce the
and vice versa. Currency convertibility is the ease with balance of payment deficit. Every country should
which a country's currency can be converted into gold increase exports for reducing the international liquidity
or another currency. shortage.

IAS (Pre) GS 2015 Paper I 133 YCT


SCIENCE 58. The Fair and Remunerative Price (FRP) of
sugarcane is approved by the
• Chemistry (a) Cabinet Committee on Economic Affairs
(b) Commission for Agricultural Costs and Prices
55. With reference to 'fuel cells' in which hydrogen- (c) Directorate of Marketing and Inspection,
rich fuel and oxygen are used to generate Ministry of Agriculture
electricity, consider the following statements : (d) Agricultural Produce Market Committee
1. If pure hydrogen is used as a fuel, the fuel cell Ans : (a) With the amendment of the Sugarcane
emits heat and water as by-products. (control) Order 1966 on 22nd October 2009, the concept
2. Fuel cells can be used for powering buildings of Statutory Minimum Price (SMP) of Sugarcane was
and not for small devices like laptop computers. replaced with the Fair and Remunerative Price (FRP) of
3. Fuel cells produce electricity in the form of sugarcane for 2009-10 and subsequent sugar seasons.
Alternating Current (AC). The cane price announced by the central government is
Which of the statements given above is/are decided on the basis of the recommendation of the
correct? Commission for Agricultural Costs and Prices (CACP)
(a) 1 only (b) 2 and 3 only after consulting the state governments and associations
(c) 1 and 3 only (d) 1, 2 and 3 of the sugar industry and approved by the cabinet
committee on Economic Affairs.
Ans : (a) We know that alternating current is produced
with the help of magnetic fields, so fuel cells cannot 59. Consider the following statements :
produce that. This eliminates all the options except 1. The Accelerated Irrigation Benefits Programme
which has to be the answer. If pure hydrogen is used as was launched during 1996-97 to provide loan
fuel, the fuel cell emits heat and water as by- products. assistance to poor farmers.
2. The Command Area Development Programme
Fuel cells can be used for powering small devices like
was launched in 1974-75 for the development of
laptops computers. Hence option (a) is correct. water-use efficiency.
• Medical Science Which of the statements given above is/are
correct?
56. H1N1 virus is sometimes mentioned in the news (a) 1 only (b) 2 only
with reference to which one of the following (c) Both 1 and 2 (d) Neither 1 nor 2
diseases? Ans : (b) Accelerated Irrigation Benefits Programme
(a) AIDS (b) Bird flu (AIBP) was launched by the government of India
(c) Dengue (d) Swine flu during 1996-97 to assist states for early and time
Ans : (d) Influenza A (H1N1) virus is the most bound completion of major and medium irrigation
projects and extension renovation and modernisation
common cause of human influenza (flu). In June 2009,
of completed major medium projects. Under the
the World Health Organisation (WHO) declared the programme. Central assistance to the states was in the
new strain of swine origin H1N1 as a pandemic. This form of a loan. The Command Area Development
strain is often Called Swine Flu by the public media. Programme (CADP) started in 1974-75 as a centrally
sponsored scheme, envisaged execution of on- farm
• Agriculture, Animal Husbandry and development works like field channels, land levelling,
Drainage field drains and conjunctive use of ground and surface
water. Hence option (b) is correct.
57. What can be the impact of excessive/
inappropriate use of nitrogenous fertilizers in 60. Which one of the following best describes the
agriculture? main objective of `Seed Village Concept'?
1. Proliferation of nitrogen-fixing microorganisms (a) Encouraging the farmers to use their own farm
in soil can occur. seeds and discouraging them to buy the seeds
from others
2. Increase in the acidity of soil can take place.
(b) Involving the farmers for training in quality seed
3. Leaching of nitrate to the ground-water can production and thereby to make available quality
occur. seeds to others at appropriate time and affordable
Select the correct answer using the code given cost
below: (c) Earmarking some villages exclusively for the
(a) 1 and 3 only (b) 2 only production of certified seeds
(c) 2 and 3 only (d) 1, 2 and 3 (d) Indentifying the entrepreneurs in villages and
Ans : (c) Increasing the amount of nitrogen- based providing them technology and finance to set up
fertilizer does not lead to more micro-organism in the seed companies
field. Excessive use of nitrogen fertilizers results in Ans : (b) A village, wherein a trained group of farmers
soil acidity. Nitrogen fertilizers break down into are involved in the production of seeds of various
nitrates and travel easily through the soil because it is crops and cater to the needs of themselves, fellow
water -soluble and can remain in groundwater decades. farmer of the village, and farmers of neighbouring
The addition of more nitrogen over the years has an villages in the appropriate time and at affordable cost
accumulative effect. is called 'a seed village'.

IAS (Pre) GS 2015 Paper I 134 YCT


• Science & Technology 64. The Genetic Engineering Appraisal Committee is
constituted under the
61. With reference to the use of nano-technology in (a) Food Safety and Standards Act, 2006
health sector, which of the following statements (b) Geographical Indications of Goods (Registration
is/are correct? and Protection) Act, 1999
1. Targeted drug delivery is made possible by (c) Environment (Protection) Act, 1986
(d) Wildlife (Protection) Act, 1972
nanotechnology.
2. Nanotechnology can largely contribute to gene Ans : (c) The genetic engineering appraisal committee
therapy. (CEAC) is the Apex body Constituted in the
Ministry of Environment and Forests. The Genetic
Select the correct answer using the code given Engineering Appraisal Committee has been constituted
below: under the Environment (Protection) Act, 1986 and the
(a) 1 only (b) 2 only ministry of Environment and forest.
(c) Both 1 and 2 (d) Neither 1 nor 2
Ans : (c) Nanoparticle can be used in targeted drug 65. In which of the following activities are Indian
Remote Sensing (IRS) satellites used?
delivery at the site of disease to improve the uptake of
poorly soluble drugs. 1. Assessment of crop productivity
2. Locating groundwater resources
In gene therapy using the non-viral system, the main 3. Mineral exploration
issues are relatively transient gene expression and 4. Telecommunications
lower efficiency than viral vectors. Research efforts 5. Traffic studies
have focused on understating the barriers in gene Select the correct answer using the code given
delivery so that non-viral systems can be developed below:
that are as effective as viral systems in gene (a) 1, 2 and 3 only (b) 4 and 5 only
transfixion. (c) 1 and 2 only (d) 1, 2, 3, 4 and 5
62. With reference to `Near Field Communication Ans : (a) The Indian Space Programme has two major
(NFC) Technology', which of the following operating systems- the Indian National Satellite (INSAT),
statements is/are correct? which is a geostationary satellite is used for
1. It is a contactless communication technology that telecommunication, television broadcasting and
uses electromagnetic radio fields. meteorological services and the Indian Remote sensing
satellite (IRS) for monitoring and management of natural
2. NFC is designed for use by devices which can resources (e.g. locating groundwater resources, mineral
be at a distance of even a metre from each other. exploration etc.) and disaster management support.
3. NFC can use encryption when sending sensitive
information. ENVIRONMENT & ECOLOGY
Select the correct answer using the code given
below: 66. With reference to 'dugong', a mammal found in
(a) 1 and 2 only (b) 3 only India, which of the following statements is/are
(c) 1 and 3 only (d) 1, 2 and 3 correct?
1. It is herbivorous marine animal.
Ans : (c) Near Field Communication (NFC) is the set 2. It is found along the entire coast of India.
of protocols that enable the electronic devices to 3. It is given legal protection under Schedule I of
establish radio communication with each other by the Wildlife (Protection) Act, 1972
touching the devices together or bringing them into Select the correct answer using the code given
proximity to a distance of typically 10 cm or less. below–
There can be secure communications by applying an (a) 1 and 2 (b) 2 only
encryption algorithm as it is done for credit cards and (c) 1 and 3 (d) 3 only
if it fits the criteria for being considered a personal Ans : (c) Feeding on Seagrass, dugongs are found in
area network. Hence option (c) is correct. Seagrass beds, sheltered waters, lagoons and bays. In
India, it is found off the Gujarat coast (Gulf of Kutch).
63. The term `Goldilocks Zone' is often seen in the Tamil Nadu Coast (Gulf of Mannar), Palk Bay and
news in the context of– Andaman and Nicobar Islands. Dugongs had been
(a) the limits of habitable zone above the surface of brought under schedule I of the Wildlife Protection Act.
the Earth 67. Which one of the following is the national aquatic
(b) regions inside the Earth where shale gas is animal of India?
available (a) Saltwater crocodile (b) Olive ridley turtle
(c) search for the Earth-like planets in outer space (c) Gangetic dolphin (d) Gharial
(d) search for meteorites containing precious metals Ans : (c) On 5th October, 2009, the Ganga dolphin has
Ans : (c) Goldilocks Zone is an area in space where a been declared a national aquatic animal by the central
planet is so far away from its stars that the surface of government. They are locally known as Sush, because
of the noise it makes while breathing. Ganga Dolphin
that plant is neither too hot nor too cold. In the news, is a critically endangered species in India and
we often find references to the discovery of Earth- like therefore, has been included in schedule I for the
planets in outer space. wildlife (protection) Act 1972.
IAS (Pre) GS 2015 Paper I 135 YCT
68. Which of the following National Parks is unique (a) 1 only (b) 2 only
in being a swamp with floating vegetation that (c) Both 1 and 2 (d) Neither 1 nor 2
supports a rich biodiversity? Ans : (a) National Innovation Foundation India is an
(a) Bhitarkanika National Park autonomous body of the Department of Science and
(b) Keibul Lamjao National Park Technology, Government of India, based on the Honey
(c) Keoladeo Ghana National Park Bee Network philosophy. Which provides institutional
(d) Sultanpur National Park
support to grassroots innovators and outstanding
Ans : (b) Keibul Lamjao National Park is the world's traditional knowledge holders from the unorganized
only floating national park and the last natural habitat sector of the society. So, statement 1 is correct and
of Manipur Brow antlered deer-Sangai (Cervus eldi statement 2 is incorrect.
eldi). The park was established in the year 1977. It is
spread over 40 sq.km of area in the Vishnupur district 72. Which one of the following issues the `Global
of Manipur. Economic Prospects' report periodically?
69. With reference to an organization known as (a) The Asian Development Bank
'BirdLife International', which of the following (b) The European Bank for Reconstruction and
statements is/are correct? Development
1. It is a Global Partnership of Conservation (c) The US Federal Reserve Bank
Organizations. (d) The World Bank
2. The concept of `biodiversity hotspots' originated Ans : (d) World Bank publishes the ‘Global Economic
from this organization. Prospects’ report periodically.
3. It identifies the sites known/referred to as
`Important Bird and Biodiversity Areas'. 73. With reference to the International Union for
Select the correct answer using the code given Conservation of Nature and Natural Resources
below: (IUCN) and the Convention on International
(a) 1 only (b) 2 and 3 only Trade in Endangered Species of Wild Fauna and
(c) 1 and 3 only (d) 1, 2 and 3 Flora (CITES), which of the following statements
Ans : (c) Important Bird and Biodiversity Area (IBA) is/are correct?
programme aims to identify, monitor & protect a 1. IUCN is an organ of the United Nations and
global network of IBAs for the conservation of the CITES is an international agreement between
worlds birds and other wildlife. The concept of governments.
'Biodiversity hotspots' did not originate from this 2. IUCN runs thousands of field projects around the
organisation. It was Coined by Norman Meyers in world to better manage natural environments.
1988. Each Biodiversity hotspot represents a 3. CITES is legally binding on the States that have
remarkable universe of extraordinary floral and joined it, but this Convention does not take the
faunal endemicity struggling to service in the rapidly place of national laws.
Shrinking ecosystem. Hence option (c) is correct. Select the correct answer using the code given
70. 'BioCarbon Fund Initiative for Sustainable below:
Forest Landscapes' is managed by the– (a) 1 only (b) 2 and 3 only
(a) Asian Development Bank (c) 1 and 3 only (d) 1, 2 and 3
(b) International Monetary Fund Ans : (b) IUCN is a voluntary organisation, not an
(c) United Nations Environment Programme agency of the United Nations. This eliminates the other
(d) World Bank 3 options.
Ans : (d) The BioCarbon Fund Initiative for sustainable It was founded in 1948 as the world's first global
forest landscapes (ISFL) is a multilateral fund, environmental organisation. IUCN supports scientific
Supported by donor governments and managed by the research, manages field projects all over the world and
World Bank. It seeks to promote reduced Green House brings governments, NGOs the UN and companies
Gas emissions from the land sector from deforestation together to develop policy laws and best practise.
and forest degradation in developing countries
(REDD+) and from sustainable agriculture, as well as 74. With reference to `fly ash' produced by the
smarter land-use planning, policies and practices. power plants using coal as fuel, which of the
71. Which of the following statements is/are correct following statements is/are correct?
regarding National Innovation Foundation-India 1. Fly ash can be used in the production of bricks
(NIF)? for building construction.
1. NIF is an autonomous body of the Department of 2. Fly ash can be used as a replacement for some of
Science and Technology under the Central the Portland cement contents of concrete.
Government. 3. Fly ash is made up of silicon dioxide and
2. NIF is an initiative to strengthen the highly calcium oxide only, and does not contain any
advanced scientific research in India's premier toxic elements.
scientific institutions in collaboration with highly Select the correct answer using the code given
advanced foreign scientific institutions. below :
Select the correct answer using the code given (a) 1 and 2 (b) 2 only
below : (c) 1 and 3 (d) 3 only
IAS (Pre) GS 2015 Paper I 136 YCT
Ans : (a) Fly ash is used in the manufacture of Ans : (d) The deployment of IndARC, the country's
building materials and in construction activity like the first underwater moored observatory in the
replacement of cement in concrete. It contains trace Kongsfjorden Fjord, halfway between Norway and the
levels of toxic elements (e.g. arsenic barium beryllium,
boron, cadmium, Chromium, Thallium, Selenium, North pole, represents a major milestone in India's
Molybdenum and mercury). scientific endeavours in the Arctic region. The
observatory is anchored at a depth of 192 m and has an
75. Which one of the following is associated with the array of 10 states of the art oceanographic sensors
issue of control and phasing out of the use of
ozone-depleting substances? strategically positioned at various depths in the water.
(a) Bretton Woods Conference 79. In a particular region in India, the local people
(b) Montreal Protocol train the roots of living trees into robust bridges
(c) Kyoto Protocol across the streams. As the time passes, these
(d) Nagoya Protocol
bridges become stronger. These unique `living root
Ans : (b) Montreal Protocol on substances that bridges' are found in–
Deplete the Ozone layer (a protocol to the Vienna
Convention for the protection of the Ozone Layer) is (a) Meghalaya (b) Himachal Pradesh
an international treaty designed to protect the Ozone (c) Jharkhand (d) Tamil Nadu
layer by phasing out the production of numerous Ans : (a) Meghalaya's double- decker and single-
substances that are responsible for Ozone depletion. decker root bridges are unique in the world and are a
76. What is Rio+20 Conference, often mentioned in sight to behold. The bridges are tangles of massive
the news? thick roots, which have been intermingled to form a
(a) It is the United Nations Conference on bridge that can hold several people at a time. Khasi
Sustainable Development people have been trained to grow their bridges across
(b) It is a Ministerial Meeting of the World Trade the raised banks of streams to form a solid bridge made
Organization
(c) It is a Conference of the Inter-governmental from roots. The living bridges are made from the roots
Panel on Climate Change of the Ficus Elastica tree, which produces a sense of
(d) It is a Conference of the Member Countries of secondary roots that are perched atop huge boulders
the Convention on Biological Diversity along the streams or the river banks to form bridges.
Ans : (a) The United Nation Conference on Sustainable 80. Which one of the following is the best description
Development or Rio+20 took place in Rio-de-Janeiro,
of the term ecosystem'?
Brazil, on 20-22 June 2012. It resulted in a focused
political outcome document which contains clear and (a) A community of organisms interacting with one
practical measure for implementing sustainable another
development. (b) That part of the Earth which is inhabited by
77. Which of the following statements regarding living organisms
'Green Climate Fund' is/are correct? (c) A community of organisms together with the
1. It is intended to assist the developing countries in environment in which they live
adaptation and mitigation practices to counter (d) The flora and fauna of a geographical area
climate change. Ans : (c) Ecosystem is a community of living
2. It is founded under the aegis of UNEP, OECD, organisms in conjunction with the non-living
Asian Development Bank and World Bank. components of their environment interacting as a
Select the correct answer using the code given system. The ecosystem varies greatly in size from a
below: small pond to a large forest or a Sea. Many ecologists
(a) 1 only (b) 2 only regard the entire biosphere as a global ecosystem as a
(c) Both 1 and 2 (d) Neither 1 nor 2 composite of all local ecosystems on Earth. It can be
Ans : (a) The fund is governed by the GCF Board. GCF divided into two basic categories, namely the terrestrial
is currently structured into two themes- mitigation and and the aquatic. Forest, grassland and desert are some
adaptation. examples of terrestrial ecosystems, while pond, lake,
GCF is a fund within the framework of UNFCCC. wetland, river and estuary are some examples of the
78. The term 'IndARC', sometimes seen in the news, aquatic ecosystems.
is the name of 81. Which of the following has/have been accorded
(a) an indigenously developed radar system inducted 'Geographical Indication' status?
into Indian Defence 1. Banaras Brocades and Sarees
(b) India's satellite to provide services to the 2. Rajasthani Daal-Bati-Churma
countries of Indian Ocean Rim 3. Tirupathi Laddu
(c) a scientific establishment set up by India in Select the correct answer using the code given
Antarctic region below:
(d) India's underwater observatory to scientifically (a) 1 only (b) 2 and 3 only
study the Arctic region (c) 1 and 3 only (d) 1, 2 and 3
IAS (Pre) GS 2015 Paper I 137 YCT
Ans : (c) Banaras Brocades Sarees and Tirupati laddu It is buried beneath the surface, extending to a depth of
has been accorded Geographical Indication status. about 2500 meters; burying it deep in ice is important
• Banaras Brocades Sarees are among the first to enhance the precision of the detector.
Sarees in India and are known for their gold and 85. The terms 'Agreement on Agriculture',
silver brocade on zari fine silk and opulent 'Agreement on the Application of Sanitary and
embroidery. Phytosanitary Measures' and `Peace Clause'
• Rajasthani Daal- Bati- churma is a popular dish of appear in the news frequently in the context of
Rajasthan, also popular in Uttar Pradesh and the affairs of the
Madhya Pradesh. (a) Food and Agriculture Organization
(b) United Nations Framework Conference on
• Tirupati Laddu or Srivari Laddu is the laddu sweet Climate Change
offered as Navivedhyam to Venkateswara at (c) World Trade Organization
Tirumala Venkateswara temple in Tirupati, (d) United Nations Environment Programme
Chitoor district, Andhra Pradesh.
Ans : (c) The terms "Agreement on Agriculture", "
Agreement on the application of Sanitary and
CURRENT AFFAIRS Phytosanitary Measures" and "peace clause", we hear
in the context of the affairs of the World Trade
82. The 'Fortaleza Declaration', recently in the news,
Organisation.
is related to the affairs of–
(a) ASEAN (b) BRICS 86. The area known as 'Golan Heights' sometimes
(c) OECD (d) WTO appears in the news in the context of the events
related to
Ans : (b) The 6th BRICS Summit was held in (a) Central Asia (b) Middle East
Fortaleza (Brazil). The Theme of the Summit was (c) South-East Asia (d) Central Africa
Inclusive Growth-sustainable solutions. The Ans : (b) The Golan heights are a rocky plateau in
participating nations Brazil, Russia, India, China and western Asia that was captured by Israel from Syria
South Africa jointly issued the Fortaleza Declaration. in the 1967 six-day war. The international community
The 13th BRICS summit was held under India's recognizes the Golan heights to be official Syrian
chairship on 09 september 2021. It was the third time territory and widely rejects the Israel military
that India hosted the BRICS summit after 2012 & 2016. occupation
83. Among the following, which were frequently 'Golan Heights' is a disputed region between Israel
mentioned in the news for the outbreak of Ebola and Syria, which was captured by Israel in 1981 from
virus recently? Syria.
(a) Syria and Jordan 87. Indira Gandhi Prize for Peace, Disarmament
(b) Guinea, Sierra Leone and Liberia and Development for 2014 was given to which
(c) Philippines and Papua New Guinea one of the following?
(d) Jamaica, Haiti and Surinam (a) Bhabha Atomic Research Centre
(b) Indian Institute of Science
Ans : (b) Ebola virus causing hemorrhagic fever was (c) Indian Space Research Organization
detected on 23rd March, 2014 in Guinea. More than 80 (d) Tata Institute of Fundamental Research
persons have been detected to have been infected with
the Ebola virus. Out of this 59 people from Sierra Ans : (c) A jury chaired by Vice President Hamid
Leone and Liberia have died. This marks the first time Ansari chose ISRO for the award, given to individuals
or organisations who promote international
an outbreak among humans has been detected. development, a new international economic order or
84. In the context of modern scientific research, make scientific discoveries for the public good.
consider the following statements about 88. India is a member of which among the following?
'IceCube', a particle detector located at South 1. Asia-Pacific Economic Cooperation
Pole, which was recently in the news : 2. Association of South-East Asian Nations
1. It is the world's largest neutrino detector, 3. East Asia Summit
encompassing a cubic kilometre of ice. Select the correct answer using the code given
2. It is a powerful telescope to search for dark
below:
matter. (a) 1 and 2 only
3. It is buried deep in the ice. (b) 3 only
Which of the statements given above is/are (c) 1, 2 and 3
correct?
(a) 1 only (b) 2 and 3 only (d) India is a member of none of them
(c) 1 and 3 only (d) 1, 2 and 3 Ans : (b) India is a member of the East Asia Summit.
Ans : (d) The Ice Cube Neutrino Observatory is a India is a also a member of ASEAN.
neutrino telescope built at the South Pole of The East Asia Summit (EAS) is a regional forum
Antarctica. The project is being developed, operated held annually by leaders of initially countries in the
and managed by the University of Wisconsin, East Asian, Southeast Asian, South Asian and
Madison. According to the announcement made in Oceanian regions. The 18 EAS member countries
November 2013, 28 neutrinos were discovered by this represent collectively 55% of the world population
telescope. and account for around 55% of global GDP.

IAS (Pre) GS 2015 Paper I 138 YCT


89. In the Mekong-Ganga Cooperation, an initiative 92. With reference to `Forest Carbon Partnership
of six countries, which of the following is/are not Facility', which of the following statements is/are
a participant/participants? correct?
1. Bangladesh 2. Cambodia
3. China 4. Myanmar 1. It is a global partnership of governments,
5. Thailand businesses, civil society and indigenous peoples.
Select the correct answer using the code given 2. It provides financial aid to universities,
below: individual scientists and institutions involved in
(a) 1 only scientific forestry research to develop eco-
(b) 2, 3 and 4 friendly and climate adaptation technologies for
(c) 1 and 3 sustainable forest management.
(d) 1,2 and 5 3. It assists the countries in their `REDD+
Ans : (c) The Mekong-Ganga Cooperation (MGC) (Reducing Emissions from Deforestation and
comprises six member countries, namely India,
Forest Degradation+)' efforts by providing them
Thailand, Myanmar, Cambodia, Laos and Vietnam.
They emphasised four areas of cooperation, which are with financial and technical assistance.
tourism, culture, education, and transportation linkage. Select the correct answer using the code given
The organisation takes its name from the Ganga and below:
the Mekong, two large rivers in the region. Hence (a) 1 only (b) 2 and 3 only
option (C) is correct. (c) 1 and 3 only (d) 1, 2 and 3
90. 'Basel III Accord' or simply 'Basel III', often seen Ans : (c) The first Carbon Partnership facility is a
in the news, seeks to– global partnership of governments, businesses, civil
(a) develop national strategies for the conservation society and Indigenous peoples focused on reducing
and sustainable use of biological diversity emissions from deforestation and forest degradation,
(b) improve banking sector's ability to deal with forest carbon stock conversation, the sustainable
financial and economic stress and improve risk management of forests, and the enhancement of forest
management carbon stocks in developing countries (activities
(c) reduce the greenhouse gas emissions but places a commonly referred to as REDD+). It assists countries
heavier burden on developed countries in their REDD+ efforts by providing them with
(d) transfer technology from developed countries to financial and technical assistance in building their
poor countries to enable them to replace the use capacity to benefit from possible future systems of
of chlorofluorocarbons in refrigeration with positive incentives for REDD+. It does not provide
harmless chemicals direct financial aid to universities, individual scientists
Ans : (b) According to the Basel Committee on and institutions.
Banking Supervision, ‘Basel III is a comprehensive set 93. Which one of the following was given classical
of reform measures, developed by the Basel
Committee on Banking Supervision, to strengthen the language status recently?
regulation Supervision and risk management of the (a) Odia (b) Konkani
banking sector’. Basel III or Basel 3 released in (c) Bhojpuri (d) Assamese
December, 2010, is the third in the Series of Basel Ans : (a) Odia on 24th February, 2014, became the
Accords. These accords deal with risk management sixth language of the country to get 'classical language'
aspects for the banking sector. status after the Union cabinet conceded a long-
91. With reference to 'Indian Ocean Rim Association pending demand for putting it in the same league as
for Regional Cooperation (IOR-ARC)', consider Sanskrit, Tamil, Telugu, Kannada & Malayalam.
the following statements : Current classical languages in India are Tamil (2004),
1. It was established very recently in response to Sanskrit (2005), Telugu (2008), Kannada (2008),
incidents of piracy and accidents of oil spills. Malayalam (2013), Odia (2014).
2. It is an alliance meant for maritime security only. 94. `Beijing Declaration and Platform for Action',
Which of the statements given above is/are often seen in the news, is–
correct? (a) a strategy to tackle the regional terrorism, an
(a) 1 only (b) 2 only
outcome of a meeting of the Shanghai
(c) Both 1 and 2 (d) Neither 1 nor 2
Cooperation Organization
Ans : (d) Indian Ocean Rim Association for Regional (b) a plan of action for sustainable economic growth
Cooperation (IOR-ARC) is an international organisation in the Asia-Pacific Region, an outcome of the
consisting of coastal states bordering the Indian Ocean.
The organisation was first established as the Indian deliberations of the Asia-Pacific Economic
Ocean Rim Initiative in Mauritius in March 1995 and Forum
formally launched on 6-7 March 1997. It aims to (c) an agenda for women's empowerment, an
promote the sustained growth and balanced outcome of a World Conference convened by the
development of the region and of the member states United Nations
and to create common ground for regional economic (d) a strategy to combat wildlife trafficking, a
co-operation. declaration of the East Asia Summit
IAS (Pre) GS 2015 Paper I 139 YCT
Ans : (c) The 'Beijing Declaration and Platform for Ans : (c) Pradhan Mantri Jan-Dhan Yojana (PMJDY)
Action (BPFA) is an international declaration of is a National Mission for Financial Inclusion to ensure
women's rights set up at the UN & landmark Fourth access to Financial Services, namely, banking/Saving
World Conference on Women, held in Beijing (China) & Deposits Accounts, Remittance, Credit, Insurance,
in 1955 . The BPFA covers 12 key critical matters of
concern and areas for action including women and Pension in an affordable manner. The account can be
poverty, violence against women and access to power opened in any bank branch or Business Correspondent
and decision making. It completed its 20 years in 2015 (Bank Mitra) outlet. This financial inclusion campaign
and hence was in the news. was launched by Prime Minister Narendra Modi on
28th August, 2014.
95. Consider the following countries :
1. China 2. France
3. India 4. Israel
MISCELLANEOUS
5. Pakistan 98. Amnesty International is–
Which among the above are Nuclear Weapons (a) an agency of the United Nations to help refugees
States as recognized by the Treaty on the Non- of civil wars
Proliferation of Nuclear Weapons, commonly (b) a global Human Rights Movement
(c) a non-governmental voluntary organization to
known as Nuclear Non-Proliferation Treaty help very poor people
(NPT)? (d) an inter-governmental agency to cater to medical
(a) 1 and 2 only (b) 1, 3, 4 and 5 only emergencies in war-ravaged regions
(c) 2, 4 and 5 only (d) 1, 2, 3, 4 and 5 Ans: (b) Amnesty International is a non-governmental
Ans : (a) The United States, United Kingdom, France, organisation. It was founded in London in July 1961
Russia, and China are nuclear- weapon states by Peter Benenson. In the field of Human Rights
according to the Nuclear Non-Proliferation Treaty. It is organisation. Amnesty has the longest history and
noteworthy that the above five countries are permanent broadest name recognition and is believed by many to
members of the United Nations Security Council. Four set standards for the movement as a whole. It is 7
other states are known or believed to possess nuclear million members and supporters around the world.
weapons: India, North Korea, Pakistan and Isreal. 99. Kalamkari painting refers to–
They declared that they possess nuclear weapons. (a) a hand-painted cotton textile in South India
(b) a handmade drawing on bamboo handicrafts in
96. With reference to bio-toilets used by the Indian North-East India
Railways, consider the following statements : (c) a block-painted woollen cloth in Western
1. The decomposition of human waste in the bio- Himalayan region of India
toilets is initiated by a fungal inoculum. (d) a hand-painted decorative silk cloth in North-
Western India
2. Ammonia and water vapour are the only end
products in this decomposition which are Ans : (a) Kalamkari (Kalam=Pen, Kari= work)
literally means artwork done using a Pen. Vegetable
released into the atmosphere.
dyes are used to colour the design applied on cloth.
Which of the statements given above is/are The art of painting using organic dyes on cloth was
correct? popular in several parts of India, but this style of
(a) 1 only (b) 2 only Kalamkari flourished at Kalahasti and at
(c) Both 1 and 2 (d) Neither 1 nor 2 Masulipatnam. The Kalamkari tradition chiefly
Ans : (d) Under the bio toilet concept of the DRDO. consists of Scenes from Hindu mythology. Figures of
The decomposition of human waste in the bio-toilets is deities with rich border embellishments were created
for the temples. Here option (a) is correct.
initiated by a bacterial inoculums (and not fungal).
Carbon dioxide and Methane are the end products 100. With reference to the Indian Renewable Energy
released into the atmosphere. When human excreta Development Agency Limited (IREDA), which of
comes in contact with bacteria, it gets converted into the following statements is/are correct?
methane and water through a series of steps of anaerobic 1. It is a Public Limited Government Company.
2. It is a Non-Banking Financial Company.
digestion-hydrolysis, acidogenesis, acetogenesis and
Select the correct answer using the code given
methanogenesis. Hence option (d) is correct.
below:
97. `Pradhan Mantri Jan-Dhan Yojana' has been (a) 1 only (b) 2 only
launched for– (c) Both 1 and 2 (d) Neither 1 nor 2
(a) providing housing loan to poor people at cheaper Ans : (c) IREDA is a Public Limited Government
interest rates Company established as a Non-Banking Financial
(b) promoting women's Self-Help Groups in Institution in 1987 under the administrative control of
backward areas MNRE to promote, develop and extend financial
(c) promoting financial inclusion in the country assistance for renewable energy and energy
(d) providing financial help to the marginalized efficiency/conservation projects. Both the statements
communities are correct.
IAS (Pre) GS 2015 Paper I 140 YCT
UNION PUBLIC SERVICE COMMISSION
Civil Services (Preliminary Exam) - 2014
GENERAL STUDIES : PAPER-I
Time: 2 hours (Exam date : 24.08.2014) Maximum Number: 200
4. Which one of the following pairs does not form
ANCIENT HISTORY part of the six systems of Indian Philosophy?
• Vedic Age (a) Mimamsa and Vedanta
(b) Nyaya and Vaisheshika
1. The national motto of India, ‘Satyameva Jayate’
inscribed below the Emblem of India is taken (c) Lokayata and Kapalika
from - (d) Sankhya and Yoga
(a) Katha Upanishad Ans. (c): Mimansa, Vedanta, Nyaya, Vaisheshika,
(b) Chandogya Upanishad Sankhya and Yoga are part of Indian Philosophies.
(c) Aitareya Upanishad These six systems of philosophy are said to have been
(d) Mundaka Upanishad founded by sages Konada, Gotama, Kapila, Patanjali,
Ans. (d): Satyameva Jayate is the national motto of Jaimini and Vyasa, respectively. Lokayata and
India. It is inscribed in the Devanagari script at the Kapalika are not a part of Indian Philosophy.
base of the national emblem, an adaptation of the Lion 5. With reference to the Indian history of art and
Capital of Ashoka at Sarnath, near Varanasi, the north
culture, consider the following pairs :
Indian state of Uttar Pradesh. The origin of the motto
is mantra 3.1.6 from the Mundaka Upanishad. Famous work of sculpture Site
1. A grand image of Buddha’s - Ajanta
• Buddhism, Jainism, Bhagavata, Mahaparinirvana with
Shaiva and Other Religion numerous celestial musicians
2. Which of the following Kingdoms were associated above and the sorrowful
with the life of the Buddha ? figures of his followers
1. Avanti 2. Gandhara below.
3. Kosala 4. Magadha 2. A huge image of Varaha - Mount Abu
Select the correct answer using the code given Avatar (boar incarnation)
below: of Vishnu, as he rescues
(a) 1, 2 and 3 (b) 2 and 4 Goddess Earth from the
(c) 3 and 4 only (d) 1,3 and 4 deep and chaotic waters,
Ans. (c): Both the kingdoms of Magadha and Kosala sculpted on rock.
were related to Gautama Buddha's life. His mother was 3.‘‘Arjuna’s Penance’’/ Descent - Mamallapuram
a princess from the Kosala dynasty. Both the kingdoms of Ganga’’ sculpted on the
of Avanti and Gandhara were not directly related to the Surface of huge boulders.
life of Buddha. Which of the pairs given above is /are correctly
3. With reference to Buddhist history, tradition and matched?
culture in India, consider the following pairs : (a) 1 and 2 only (b) 3 only
Famous shrine Location (c) 1 and 3 only (d) 1, 2 and 3
1. Tabo monastery and Spiti Valley Ans. (c): The Mahaparinirvana of the Buddha in cave
temple complex 17 of Ajanta, with numerous celestial musicians above
2. Lhotsava Lhakhang Zanskar Valley and the sorrowful figures of his followers below, is one
Temple, Nako of the grandest and yet most delicately expressive
3. Alchi temple complex Ladakh scenes ever made in stone.
Which of the pairs given above is/are correctly The Udayagiri caves are 20 rock-cut caves near
matched ? Vidisha, Madhya Pradesh from the early years of the
(a) 1 only (b) 2 and 3 only 5th century CE. They contain a huge image of Varaha
(c) 1 and 3 only (d) 1, 2 and 3 Avatar. Arjuna's Penance is one of the magnificent
Ans. (c): Tabo monastery and temple complex is monuments of Mahabalipuram.
located in the Tago village of Spiti valley, Himachal
6. With reference to the cultural history of India,
Pradesh. Lhotsava Lhakhang temple is situated in Nako
the term ‘Panchayatan’ refer to-
village in the Kinnaur district of Himachal Pradesh. (a) An assembly of village elders
Aichi temple complex is situated in the Aichi village of (b) A religious sect
Ladakh, lies about 65 km from Leh on the banks of the (c) A style of temple construction
Indus River. (d) An administrative functionary
IAS (Pre) GS 2014 Paper I 141 YCT
Ans. (c) Panchayatana is a temple construction style Ans. (c) On 17th August 1947, the Radcliffe Line was
with a central shrine surrounded by four other shrines. declared as the boundary between India and Pakistan,
It has the main shrine which is surrounded by four following the Partition of India. In June 1947, Britain
subsidiary shrines. The origin of the name is the commissioned Sir Cyril Radcliffe to head the two
Sanskrit words Pancha(five) et ayatana (containing). Boundary (one for Punjab and the other for Bengal), to
Some of the examples of this style of temples are determine which territories will be assigned to which
Lakshmana temple in Khajuraho, Lingaraja temple in nation. The boundary commission was asked to
Bhubaneswar, etc. demarcate areas in Punjab based on religious majority.
MEDIEVAL HISTORY 11. What was/were the object/ objects of Queen
Victoria’s Proclamation (1858)?
7. In medieval India, the designations ‘Mahattara’ 1. To disclaim any intention to annex Indian State.
and ‘Pattakila’ were used for- 2. To place the Indian administration under the
(a) Military officers British Crown.
(b) Village headman 3. To regulate East India Company’s trade with
(c) Specialist in Vedic rituals India.
(d) Chiefs of craft guilds Select the correct answer using the code given
Ans. (b): In medieval India, the designations below:
'Mahattara' and 'Pattakila' were used for village (a) 1 and 2 only (b) 2 only
headmen. The Headmen is the leader of the (c) 1 and 3 only (d) 1,2 and 3
community in that village, who owned most of the
Ans. (a): The object of Queen Victoria’s Proclamation
land.
(1858) were to disclaim any intention to annex Indian
• Bhakti Movement states as the announcement reversed Lord Dalhousie's
pre-war policy of political unification through princely
8. Consider the following statements : state annexation. It was also to place the Indian
1. ‘Bijak’ is a composition of the teachings of administration under the British Crown. Therefore
Saint Dadu Dayal. statements (1) and (2) are correct. However, it was not
2. The Philosophy of Pushti Marg was to regulate East India Company's trade with India. So,
propounded by Madhavacharya. statement (3) is wrong.
Which of the statements given above is/are
correct? • Freedom Struggle and National
(a) 1 only (b) 2 only Movement
(c) Both 1 and 2 (d) Neither 1 nor 2
Ans. (d): Vallabhacharya's philosophy came to be 12. The Partition of Bengal made by Lord Curzon in
known as Pushtimarga (the path of grace). Kabir's 1905 lasted until –
ideas were later collected and preserved in Bijak. (a) The First World War when Indian troops were
needed by the British and the partition was ended
• Mughal Empire (b) King George V abrogated Curzon’s Act at the
9. Ibadat Khana at Fatehpur Sikri was- Royal Durbar in Delhi in 1911
(a) the mosque for the use of Royal Family. (c) Gandhiji launched his Civil Disobedience
(b) Akbar’s private prayer chamber. Movement
(c) the hall in which Akbar held discussions with (d) The Partition of India in 1947 when East Bengal
scholars of various religions. became East Pakistan
(d) the room in which the nobles belonging to Ans. (b) : Lord Curzon, the Viceroy of India,
different religions gathered to discuss religion. announced the decision to partition Bengal on 20th
Ans. (c): The Ibadat Khana (House of Worship) was a July 1905 and the partition came into effect from 16th
meeting house built in 1575 by the Mughal Emperor October 1905. A durbar was held in Delhi to welcome
Akbar at Fatehpur Sikri to gather spiritual leaders of British emperor George V and Queen Mary on their
different religious grounds so as to conduct a arrival in India in December 2011. In the same Delhi
discussion on the teachings of the respective religious Durbar on 12th December 1911, the British emperor
leaders. announced the cancellation of the partition of Bengal
and made Delhi the new capital instead of Calcutta.
MODERN HISTORY 13. The 1929 Session of Indian National Congress is
of significance in the history of the Freedom
• Major Institutions, Treaties, Movement because the-
Commissions, Acts, Pacts (a) Attainment of Self- Government was declared as
the objective of the Congress.
10. The Radcliffe Committee was appointed to-
(a) Solve the problem of minorities in India (b) Attainment of Poorna Swaraj was adopted as a
(b) Give effect to the Independence Bill goal of the Congress.
(c) Delimit the boundaries between India and (c) Non-Cooperation Movement was launched.
Pakistan (d) Decision to participate in the Round Table
(d) Enquire into the riots in East Bengal Conference in London was taken.
IAS (Pre) GS 2014 Paper I 142 YCT
Ans. (b) : The historic Lahore session of Congress was Ans. (a): Coral reefs are a ridge of rock in the sea
held in December 1929 under the chairmanship of Pt. formed by the growth and deposit by coral. Major reef
Jawaharlal Nehru. In this session, the attainment of formations in India mainly occur in the Gulf of
complete Swaraj was approved as the goal. Mannar, Palk Strait, Gulf of Kutch, the Andaman and
Nicobar Islands and the Lakshadweep.
14. The Ghadr (Ghadar) was a-
(a) revolutionary association of Indians with 18. Consider the following pairs :
headquarters at San Francisco. Hills Region
1. Cardamom Hills Coromandel Coast
(b) nationalist organization operating from 2. Kaimur Hills Konkan Coast
Singapore. 3. Mahadeo Hills Central India
(c) militant organization with headquarters at Berlin. 4. Mikir Hills North-East India
(d) communist movement for India’s freedom with Which of the above pairs are correctly matched ?
head-quarters at Tashkent. (a) 1 and 2 (b) 2 and 3
Ans. (a) Sohan Singh Bhakna founded the association (c) 3 and 4 (d) 2 and 4
called the 'Hindustan Association of the Pacific Coast' Ans. (c): The Cardamom Hills are southern hills of
in San Francisco in 1913. This association took out a India and part of the Southwestern Ghats located in
newspaper called 'Gadar' which also lended the name south east Kerala and southwest Tamil Nadu. They are
of this association as Gadar Party. not on the Coromandel coast.
Kaimur Range is the eastern portion of the Vindya
Range extending from Madhya Pradesh to Bihar. They
INDIAN GEOGRAPHY are not in Konkan Coast. The Mahadeo Hills are in
Madhya Pradesh, Mikir Hills are in Assam, i.e., in
• Physical Structure/Drainage System/ North East India.
Human Geography • Monsoon, Forest, Soils, Irrigation
15. Which one of the following pairs of islands is Projects, Agriculture
separated from each other by the ‘Ten Degree 19. With reference to Neem tree, Consider the
Channel’ ? following statements :
(a) Andaman and Nicobar. 1. Neem oil can be used as a pesticide to control the
(b) Nicobar and Sumatra. proliferation of some species of insects and mites.
(c) Maldives and Lakshadweep. 2. Neem seeds are used in the manufacture of bio
(d) Sumatra and Java. fuels and hospital detergents.
3. Neem oil has applications in pharmaceutical
Ans. (a): The entire group of Islands is divided into industries.
two broad categories - the Andaman in the north and Which of the statements given above is /are
the Nicobar in the South. They are separated by a correct ?
water body which is called the Ten Degree Channel. (a) 1 and 2 only (b) 3 only
16. If you travel through the Himalayas, you are (c) 1 and 3 (d) 1, 2 and 3
likely to see which of the following plants Ans. (c): Neem oil extracted by cold-pressing the seed
naturally growing there ? kernels of Neem is highly effective against soft-bodied
1. Oak 2. Rhododendron insects and mites. The presence of disulphide in Neem
3. Sandalwood oil is a major contributor to its bioactivity.
The seeds of neem contain 30-40% oil. Biodiesel
Select the correct answer using the code given obtained from Neem oil which are mono alkyl esters
below: produced using the 'Transesterification' process. The
(a) 1 and 2 only (b) 3 only Neem seeds are also used as hospital detergents.
(c) 1 and 3 only (d) 1,2 and 3 Neem oil used in the pharmaceutical industry contains
Ans. (a): Sandalwood trees are found in the foothills various compounds that have medicinal, cosmetic and
of the Himalayas. The moist deciduous forests are insecticidal properties.
more pronounced in the regions which record rainfall
between 100-200 cm. These forests are found in the • Minerals, Power Resources &
northeastern states along the foothills of Himalayas, Industry and Trade
eastern slopes of the Western Ghats and Odisha. Teak, 20. With reference to technologies for solar power
sal, Shisham, Hurra, Mahua, Amla, Semul, Kusum and production, consider the following statements :
sandalwood etc. are the main species of these forests. 1. ‘Photovoltaics’ is a technology that generates
17. Which of the following have coral reefs ? electricity by direct conversion of light into
electricity, while, ‘Solar Thermal’ is a
1. Andaman and Nicobar Islands
technology that utilizes the Sun’s rays to
2. Gulf of Kachchh generate heat which is further used in electricity
3. Gulf of Mannar generation process.
4. Sunderbans 2. ‘Photovoltaics’ generates Alternating Current
(a) 1, 2 and 3 only (b) 2 and 4 only (AC), while Solar Thermal generates Direct
(c) 1 and 3 only (d) 1, 2, 3 and 4 Current (DC).
IAS (Pre) GS 2014 Paper I 143 YCT
3. India has manufacturing base for Solar Thermal • Hydrosphere
technology, but not for ‘Photovoltaics’.
Which of the statements given above is/ are 24. Consider the following rivers :
correct ? 1. Barak 2. Lohit 3. Subansiri
(a) 1 only (b) 2 and 3 only Which of the above flows /flow through Arunachal
(c) 1, 2 and 3 (d) None Pradesh?
Ans. (a): 'Photovoltaic's is a technology that generates (a) 1 only (b) 2 and 3 only
electricity by direct conversion of light into electricity, (c) 1 and 3 only (d) 1, 2 and 3
while 'solar Thermal' is a technology that utilizes the Ans. (b) :The Barak is an important river in Manipur
Sun's rays to generate heat which is further used in the and Mizoram. The Barak river, the head- stream of the
electricity generation process. Both Photovoltaic cells Meghna river in the hills in Manipur. Lohit river in
and solar thermal generate Direct Current (DC). India Arunachal Pradesh in India. It is a tributary to the
has a manufacturing base for both. Brahmaputra river. The Subansiri River is a tributary
of the Brahmaputra river in the states of Assam and
21. Which of the following are some important Arunachal Pradesh.
pollutants released by steel industry India ?
1. Oxides of sulphur 2. Oxides of nitrogen • Climate Change
3. Carbon monoxide 4. Carbon dioxide
25. The seasonal reversal of winds is the typical
Select the correct answer using the code given characteristic of -
below : (a) Equatorial climate
(a) 1, 3 and 4 only (b) 2 and 3 only (b) Mediterranean climate
(c) 1 and 4 only (d) 1,2,3 and 4 (c) Monsoon climate
Ans. (d): In a steel furnace, coke reacts with iron ore, (d) All of the above climates
releasing iron and generating CO and CO2 gases. Due Ans. (c): A monsoon is a seasonal reversal in wind
to the use of coal, pollutants such as SOx and NOx are patterns over a region. A dramatic change in
released; thus all the options are correct. precipitation usually accompanies the seasonal wind
shift. The best-known example of a monsoon occurs
• Transport over the Indian subcontinent.
22. Consider the following pairs. Monsoon refers to the seasonal reversal in the wind
direction during a year.
National Highway Cities connected
1. NH 4 : Chennai and Hyderabad 26. The scientific view is that the increase in global
2. NH 6 : Mumbai and Kolkata temperature should not exceed 2oC above pre-
3. NH 15 : Ahemdabad and Jodhpur industrial level, If the global temperature
Which of the above pairs is /are correctly increases beyond 3oC above the pre-industrial
matched ? level, what can be its possible impact/impacts on
(a) 1 and 2 only (b) 3 only the world ?
(c) 1,2 and 3 (d) None 1. Terrestrial biosphere tends toward a net carbon
source.
Ans. (d): National Highway No. 4 connects Chennai to 2. Widespread coral mortality will occur.
Thane (Mumbai). National Highway No. 6 connects 3. All the global wetlands will permanently
Hazira (Gujrat) to Kolkata and National Highway No. disappear.
15 starts from Pathankot (Punjab) to Samakhiali 4. Cultivation of cereals will not be possible
(Gujrat). anywhere in the world.
Select the correct answer using the code given
WORLD GEOGRAPHY below:
(a) 1 only (b) 1 and 2 only
• Evolution of Organism (c) 2,3 and 4 (d) 1,2,3 and 4
Ans. (b): According to IPCC report, if there is an
23. Which of the following phenomena might have increase of 3ºC or more in temperature then the
influenced the evolution of organisms? terrestrial biosphere will tend towards a net carbon
1. Continental drift 2. Glacial cycles source and if the world temperature rises more than
Select the correct answer using the code given 2ºC at pre-industrial level, then there will be
below : widespread coral mortality around the world.
(a) 1 only (b) 2 only
(c) Both 1 and 2 (d) Neither 1 nor 2 • Human Geography
Ans. (c): Both continental drift and Glacial cycles 27. Every year a month long ecologically important
influenced the evolution of organisms. One of the most campaign/festival is held during which certain
prominent examples of the effect of continental drift communities/ tribes plant saplings of fruit-
on the evolution of organisms is the unique primates bearing trees. Which of the following are such
found in Australia and South America. The final stages communities/tribes ?
of the revolution of Genus Homo occur in the last there (a) Bhutia and Lepcha
glacial cycles. (b) Gond and Korku
IAS (Pre) GS 2014 Paper I 144 YCT
(c) Irula and Toda (a) 1, 2 and 3 (b) 2, 3 and 4
(d) Sahariya amd Agariya (c) 1, 4 and 5 (d) 1, 3 and 5
Ans. (b): Every monsoon, the Gond and Korku tribes Ans. (d): The Arctic council is a high-level inter-
of Madhya Pradesh's Betul and Harda districts government forum that addresses issues faced by the
celebrate Hari Jiroti. It is a month- long festival of Arctic government and the indigenous people of the
greenery, during which the tribals plant saplings of Arctic. It has eight member countries Canada,
fruit-bearing trees.
Denmark, Finland, Iceland, Norway, Russia, Sweden
28. With reference to ‘Changpa’ community of India, and the United States.
consider the following statements :
1. They live mainly in the State of Uttarakhand. Indian Constitution and Polity
2. They rear the Pashmina goats that yield a fine
wool.
3. They are kept in the category of Scheduled • Constitutional Development of India
Tribes. 32. Which of the following is / are the function/
Which of the statements given above is/are functions of the Cabinet Secretariat?
correct ? 1. Preparation of agenda for cabinet Meetings.
(a) 1 only (b) 2 and 3 only
2. Secretarial assistance to Cabinet Committees.
(c) 3 only (d) 1, 2 and 3 only
3. Allocation of financial resources to the Ministries.
Ans. (b): The Changpas are a semi-nomadic Tibetan Select the correct answer using the code given
people found mainly in the Changtang in Ladakh and
below:
in Jammu and Kashmir. So, statement (1) is incorrect.
(a) 1 only (b) 2 and 3 only
• The Changpas rear the highly pedigreed and prized
Changra goats (Capra Hircus) that yield the rare (c) 1 and 2 only (d) 1, 2 and 3
Pashmina (Cashmere) fiber. So, statement (2) is Ans. (c): The functions of the cabinet secretariat are
correct. preparation of agenda for cabinet meetings &
The Pashmina fibre is the finest fibre of all goat hair. secretariat assistance to cabinet committees. However,
• In 1989, Changpas were declared as the scheduled the allocation of financial resources to the ministries as
Tribes. So, statement (3) is correct. per the provision in the budget is prepared by the
29. What is the correct sequence of occurrence of the finance ministry.
following cities in South-Asia as on proceeds 33. Which one of the following is the largest
from South and North ? Committee of the Parliament ?
1. Bangkok 2. Hanoi
(a) The Committee on Public Accounts
3. Jakarta 4. Singapore
Select the correct answer using the code given (b) The Committee on Estimates
below: (c) The Committee on Undertaking
(a) 4-2-1-3 (b) 3-2-4-1 (d) The Committee on Petitions
(c) 3-4-1-2 (d) 4-3-2-1 Ans. (b): The Estimates Committee is the largest
Ans. (c): The correct sequence of occurrence of the committee of the parliament. It consists of 30
following cities in South-East Asia as one proceeds members, all are appointed from the Lok Sabha. The
from south to north is Jakarta-Singapore-Bangkok- function of this committee is to report on expenditures
Hanoi. and the economy and suggest measures to boost the
economy and related policies. It works continuously
• Countries & their Borders throughout the year.
30. Turkey is located between- 34.Consider the following statements:
(a) Black Sea and Caspian Sea
(b) Black Sea and Mediterranea Sea A Constitutional Government is one which
(c) Gulf of Suez and Mediterranea Sea 1. Places effective restriction on individual liberty
(d) Gulf of Aqaba and Dead Sea in the interest of State Authority.
Ans. (b): Turkey's smaller part is in southeastern 2. Places effective restriction on the Authority of
Europe and its larger part in Western Asia, located the State in the interest of individual liberty
between the Black Sea and Mediterranean sea. Which of the statements given above is/are correct?
31. Consider the following countries : (a) 1 only (b) 2 only
1. Denmark (c) Both 1 and 2 (d) Neither 1 nor 2
2. Japan Ans. (b): A constitutional government is one that
3. Russian Federation imposes effective restrictions on the power of the state
4. United Kindom in the interest of the liberty of the individual. The
5. United States of America
Which of the above are the members of the concept of government is basically the basis for
‘Arctic Council’ ? protecting individual liberty.

IAS (Pre) GS 2014 Paper I 145 YCT


35. Which of the following are associated with business of the Government of India, and for the
‘Planning’ in India ? allocation among Ministers of the said business.
1. The Finance Commission The President of India is the nominal head of the
2. The National Development Council executive and all the executive actions of the
3. The Union Ministry of Rural Development government of India are taken in the name of the
4. The Union Ministry of Urban Development President.
5. The Parliament 38. Which of the following are the discretionary
Select the correct answer using the code given powers given to the Governor of a State ?
below:
1. Sending a report to the President of India for
(a) 1, 2 and 5 only (b) 1, 3 and 4 only
imposing the President’s rule.
(c) 2 and 5 only (d) 1, 2, 3, 4 and 5
2. Appointing the Ministers.
Ans. (c): The draft five-year plan, prepared by the 3. Reserving certain bills passed by the State
planning commission is first submitted to the union Legislature for consideration of the President of
cabinet. After its approval, it is placed before the India.
National Development Council, for its acceptance.
4. Making the rules to conduct the business of the
Then the plan is presented to Parliament, with its
State Government.
approval it emerges as the official plan and published
Select the correct answer using, the code given
in the official Gazette. The Finance Commission and
below:
Union ministry of Rural Development are not involved (a) 1 and 2 only (b) 1 and 3 only
in planning. (c) 2, 3 and 4 only (d) 1, 2, 3 only
• Directive Principle of State Policies Ans. (b): Article 166 (3)- The Governor shall make
rules for the more convenient transaction of the
36. In the Constitution of India, promotion of business of the Government of the state, and for the
International peace and security is included in allocation among Ministers of the said business in so
the – far as it is not business with respect to which the
(a) Preamble to the Constitution Governor is by discretion. Statement (4) does not come
(b) Directive Principles of State Policy under the discretionary powers of the Governor.
(c) Fundamental Duties Ministers are appointed by the Governor on the advice
(d) Ninth Schedule of the CM and hence not a discretionary power.
Ans. (b): Article 51 of the constitution, which is • Legislature : Council of Ministers
Directive Principle of State Policy directs the state to 39. Consider the following statements regarding a
promote international peace and security, maintain just No-Confidence Motion in India :
and honourable relations between nations, foster 1. There is no mention of a No-Confidence Motion
respect for international law and treaty obligations in the in the Constitution of India.
dealings of organized peoples with one another; and 2. A Motion of No-Confidence can be introduced in
the Lok Sabha only.
encourage settlement of International disputes by
Which of the Statements given above is /are
arbitration.
correct?
• Centre-State Legislature, Emergency (a) 1 only (b) 2 only
(c) Both 1 and 2 (d) Neither 1 nor 2
Provisions Ans. (c):Article 75(3) says that the Council of Minister
37. Consider the following statements : shall be collectively responsible to Lok Sabha. In other
1. The President shall make rules for the more words, Lok Sabha can remove them by passing a no-
convenient transaction of the business of the confidence motion. But the word "no-confidence
Government of India, and for the allocation motion" itself is not given in the constitution; it comes
among Ministers of the said business. from Rule 198 of Lok Sabha Rules. Rajya Sabha
2. All executive actions of the Government of India cannot pass or remove the council of ministers passing
shall be expressed to be taken in the name of the a no-confidence motion. No confidence motion can be
Prime Minister. introduced only in Lok Sabha.
Which of the statements given above is / are • Judiciary (Centre & State)
correct : 40. The power of the Supreme Court of India to
(a) 1 only (b) 2 only decide disputes between the Centre and the
(c) Both 1 and 2 (d) Neither 1 nor 2 States falls under its-
Ans. (a): Clause (23) of Article 77 ("Conduct of (a) advisory jurisdiction
Business of the Government of India") of the (b) appellate jurisdiction
constitution of India states that the President shall (c) original jurisdiction
make rules for the more convenient transaction of the (d) writ jurisdiction
IAS (Pre) GS 2014 Paper I 146 YCT
Ans. (c): The following jurisdictions fall under the 3. Rainwater harvesting and recharge of
original jurisdiction of the Supreme Court under groundwater table
Article 131. 4. Regeneration of natural vegetation
• Dispute between the Government of India and one (a) 1 and 2 only (b) 2, 3 and 4
or more states. (c) 1, 3 and 4 only (d) 1,2,3 and 4
• Dispute between two or more states. Ans. (c): Under the Integrated Watershed
• Disputes between the Government of India and Development Programme' several goals such as
any State or States on one side and one or more prevention of overflow of soil, rainwater harvesting
other States on the other. and recharge of groundwater have been prescribed.
41. The power to increase the number of judges in The Integrated Watershed Development Board was
the Supreme Court of India is vested in- established in the year 1985. It works under the
(a) The President of India ministry of Environment, Forest and climate change.
(b) The Parliament 45. With reference to Union Budget, which of the
(c) The Chief Justice of India following is/are covered under Non Plan
(d) The Law Commission
Expenditure?
Ans. (b): According to Article 124 (1) - There shall be 1. Defence expenditure 2. Interest payments
a Supreme Court of India constituting of a Chief 3. Salaries and pensions 4. Subsidies
Justice of India and, until Parliament by law prescribes Select the correct answer using the code given
a larger number, of not more than seven other Judges. below:
Parliament increased the number of judges from the (a) 1 only (b) 2 and 3 only
original 8 in 1950 to 11 in 1956, 14 in 1960, 18 in (c) 1, 2, 3 and 4 (d) None
1978, 26 in 1986, 31 in 2008 and to 34 in 2019. Ans. (c): All the above dependence expenditure,
interest payments, salaries and pensions and Subsidies
• Panchayati Raj System and form the part of Non-plan expenditure.
Constitution Amendments and
Schedules • Agriculture, Industry and Trade
42. Which one of the following Schedules of the 46. What are the significances of a practical
Constitution of India contains provisions approach to sugarcane production known as
regarding anti-defection? ‘Sustainable Sugarcane Initiative’ ?
(a) Second Schedule (b) Fifth Schedule 1. Seed cost is very low in this compared to the
(c) Eight Schedule (d) Tenth Schedule conventional method of cultivation
2. Drip irrigation can be practiced very effectively in
Ans. (d): The Anti-Defection Act was passed on 30th this.
March, 1985. The provision of disqualification of 3. There is no application of chemical/inorganic
membership on the grounds of defection has been fertilizers at all in this.
implemented under the 10th schedule by the 52nd 4. The scope for intercropping is more in this
Constitution Amendment in the Indian constitution in
compared to the conventional method of
1985.
cultivation.
ECONOMIC Select the correct answer using the codes given
below:
(a) 1 and 3 only (b) 1, 2 and 4
• Economic Planning and National (c) 2, 3 and 4 only (d) 1, 2, 3 and 4
Income/Budget Ans. (b) The SSI method encourages the application
43. The main objective of the 12th five year Plan was- of organic manure as it enhances the macro and
(a) Inclusive growth and poverty reduction. micronutrient content in the soil in an eco- friendly
(b) Inclusive and sustainable growth. way and helps in optimum utilization of some of the
(c) Sustainable and inclusive growth to reduce chemical fertilizers.
unemployment. 47. In India, cluster bean (Guar) is traditionally used
(d) Faster, sustainable and more inclusive growth. as a vegetable or animal feed, but recently the
Ans. (d): The objective of the 12th Five Year Plan cultivation of this has assumed significance.
(2012-17) was faster, sustainable and more inclusive Which one of the following statements is correct
growth. Five-Year Plans (FYPs) were centralised in this context ?
economic and social growth programs. (a) The oil extracted from seeds is used in the
44. What are the benefits of implementing the manufacture of biodegradable plastics.
‘Integrated Watershed Development (b) The gum made from its seed is used in the
Programme’? extraction of shale gas.
1. Prevention of soil runoff (c) The leaf extract of this plant has the properties of
2. Linking the country’ perennial rivers with anti-histamines.
seasonal rivers (d) It is a source of high quality biodiesel.
IAS (Pre) GS 2014 Paper I 147 YCT
Ans. (b): Cluster bean (Guar) is native to India and is Ans. (b): The Government of India had announced the
widely grown in dry, warm and arid regions. It is a National Seed Policy in the year 2000 to achieve the
source of Guar gum. It is derived from guar seeds, a future food targets, under which the government in
legume crop that grows in semi-arid regions of collaboration with the private sector, will try to bridge
Rajasthan Gujarat and Haryana. the gap in demand and supply of quality seeds in
It is traditionally used as a vegetable or animal feed. relation to a low value and high volume crops.
The gum made from its seeds is used in the extraction
of shale gas. Guar gum is used as a stabilizing • Money/Banking, Tax System, Centre-
thickening and suspending agent in drilling fluids.
India is the world's largest producer of guar. State Relations
48. Consider the following pairs : 51. The sales tax you pay while purchasing a
Region Well-known for the toothpaste is a -
production of (a) Tax imposed by the Central Government.
Kinnaur Areca nut (b) Tax imposed by the Central Government but
Mewat Mango collected by the State Government.
Coromandel Soya bean (c) Tax imposed by the State Government but
Which of the above pairs is/are correctly collected by the Central Government.
matched? (d) Tax imposed and collected by the state
(a) 1 and 2 only (b) 3 only Government.
(c) 1, 2 and 3 (d) None Ans. (d): Taxes on toothpaste comes under CST Act,
Ans. (d): Kinnaur (Himachal Pradesh) is famous for which is administered by the state Government. Sales
apples. Areca nut is mostly confined to Karnataka, Tax is paid to the Sales Tax Authority in the state form
Kerala and Assam . Mewat in Haryana is not famous where the goods are moved, i.e., the state from where
for mango. U.P. is famous for mango. Similarily, the the movement of goods begins.
Coromandel coast is not famous for Soybeans. The top 52. What does venture capital mean ?
two producers of Soybeans are Madhya Pradesh and (a) A short-term capital provided to industries.
Maharashtra, which are outside the Coromandel coast. (b) A long-term start-up capital provided to new
Therefore (d) is the correct option. entrepreneurs.
(c) Funds provided to industries at times of incurring
49. Consider the following statements : losses.
1. Maize can be used for the production of starch. (d) Funds provided for replacement a renovation of
2. Oil extracted from maize can be a feedstock for industries.
biodiesel.
Ans. (b): Venture Capital (VC) is a long term financial
3. Alcoholic beverages can be produced by using
capital provided to early-stage high potential growth
maize. startup companies on new companies.
Which of the statements given above is/are
correct? 53. With reference to Balance of Payments, which of
(a) 1 only (b) 1 and 2 only the following constitutes/constitute the Current
(c) 2 and 3 only (d) 1, 2 and 3 Account ?
Ans. (d): All three statements are correct as Maize can 1. Balance of trade
be used for the production of starch. Oil extracted from 2. Foreign assets
maize can be used as a feedstock for biodiesel. 3. Balance of invisibles
4. Special Drawing Rights
Alcoholic beverages can be produced by using maize.
Select the correct answer using the code given
50. In the context of food and nutritional security of below
India, enhancing the ‘Seed Replacement Rates’ of (a) 1 only (b) 2 and 3
various crops helps in achieving the food (c) 1 and 3 (d) 1, 2 and 4
production targets of the future. But what is/ are Ans. (c): Current account includes the balance of trade
the constraints /constraints in its wider/greater (exports-imports), net remittances, trade-in services.
implementation? 54. The terms ‘ Marginal Standing Facility Rate’ and
1. There is no National Seed Policy in place. ‘Net Demand and Time Liabilities’, sometimes
2. There is no participation of private sector seed appearing in news, are used in relation to
companies in the supply of quality seeds of (a) Banking operations
vegetables and planting materials of horticultural
(b) Communication networking
crops.
(c) Military strategies
3. There is a demand-supply gap regarding quality
(d) Supply and demand of agricultural products.
seeds in case of low value and high volume
Ans. (a): MSF came into effect from 9th May 2011.
crops. MSF scheme is provided by RBI by which the banks
Select the correct answer using the code given can borrow overnight upto 1 percent of their Net
below Demand and Time Liabilities (NDTL) i.e, 1 percent of
(a) 1 and 2 (b) 3 only the aggregate deposits and other liabilities of the
(c) 2 and 3 (d) None banks.
IAS (Pre) GS 2014 Paper I 148 YCT
55. What is/are the facility /facilities the SCIENCE
beneficiaries can get from the services of Business
Correspondent (Bank Saathi) in branchless
areas ? • Biology
1. It enables the beneficiaries to draw their subsidies 59. Lichens, which are capable of initiating ecological
and social security benefits in their villages. succession even on a bare rock, are actually a
2. It enables the beneficiaries in the rural areas to symbiotic association of -
make deposits and withdrawals. (a) Algae and bacteria (b) Algae and fungi
Select the correct answer using the code below: (c) Bacteria and fungi (d) Fungi and mosses
(a) 1 only (b) 2 only
(c) Both 1 and 2 (d) Neither 1 nor 2 Ans. (b): Lichen grows mainly on rock, ground, old
walls and leaves and bark of trees. In fact, lichen, algae
Ans. (c): Business correspondents (BCs) are retail
and fungi are symbiotic associations.
agents engaged by banks for providing banking
services at locations other than a bank branch/ATM. 60. Which one of the following is the process involved
BCs enable a bank to provide its limited range of in photosynthesis ?
banking services at a low cost. Hence they are (a) Potential energy is released to form free energy
instrumental in promoting financial inclusion. (b) Free energy is converted into potential energy
56. In the context of Indian economy, which of the and stored.
following is/are the purpose/purposes of (c) Food is oxidized to release carbon dioxide and
water.
‘Statutory Reserve Requirements’? (d) Oxygen is taken, and carbon dioxide and water
1. To enable the Central Bank to control the amount
vapour are given out.
to advances the banks can create.
2. To make the people’s deposits with banks safe Ans. (b): Sunlight (free energy) is converted into
and liquid. carbohydrates (potential energy) using water and
3. To prevent the commercial banks from making carbon dioxide, oxygen is released in the process.
excessive profits. 61. Which of the following is/are correct regarding
4. To force the banks to have sufficient vault cash vegetative propagation of plants ?
to meet their day to day requirements. 1. Vegetative propagation produces clonal
Select the correct answer using the code below: population.
(a) 1 only (b) 1 and 2 only 2. Vegetative propagation helps in eliminating the
(c) 2 and 3 only (d) 1, 2, 3 and 4 virus.
Ans. (a): The statutory reserve requirements is a 3. Vegetative propagation can be practiced most of
banking sector arrangement under which all banks the year.
have to keep a certain percentage of their total deposits Select the correct answer using the code given
in the form of liquidity. Its purpose is basically to curbbelow.
the lending tendency of banks. (a) 1 only (b) 2 and 3 only
(c) 1 and 3 only (d) 1, 2 and 3
57. If the interest rate is decreased in an economy it,
Ans. (c) : The Second option is wrong because if the
will -
(a) decrease the consumption expenditure in the parent plant has a virus, then it will not be eliminated
economy but further propagated.
(b) increase the tax collection of the Government62. Consider the following techniques/phenomena :
(c) increase the investment expenditure in the 1. Budding and grafting in fruit plants
economy 2. Cytoplasmic male sterility
(d) increase the total savings in the economy 3. Gene silencing
Ans. (c) : Decreased interest rates would ensure the Which of the above is/are used to create
availability of capital for investment expenditure. transgenic crop?
There is no relation between the interest rate and the (a) 1 only (b) 2 and 3
Tax collection or Fiscal deficit of the government. (c) 1 and 3 (d) None
Ans. (b) : All three methods are related to transgenic
• Foreign Trade of India, Economic crops directly or indirectly. The Cytoplasmic male
Organizations & Share Market sterility and Gene silencing are new technologies
specifically developed for this purpose. So option (b)
58. Which of the following organizations brings out looks more appropriate.
the publication known as ‘World Economic
Outlook’ ? • Medical Science , Zoology
(a) The International Monetary Fund
(b) The United Nations Development Programme 63. Consider the following diseases :
(c) The World Economic Forum 1. Diphtheria 2. Chickenpox
(d) The World Bank 3. Smallpox
Ans. (a): The World Economic Outlook (WEO) Which of the above diseases has/have been
database contains selected macroeconomic data series eradicated in India ?
from the statistical appendix of the World Economic (a) 1 and 2 only (b) 3 only
Outlook report prepared by IMF. (c) 1, 2 and 3 (d) None
IAS (Pre) GS 2014 Paper I 149 YCT
Ans. (b): India was declared smallpox free in April • Science and Technology
1977. In February 2000, India was declared a disease-
free country by Guinea worm by the World Health 68. With reference to two non-conventional energy
Organization and on 27th March 2014 was granted the sources called ‘coal bed methane ’ and ‘shale
status of a polio-free country. gas’, consider the following statements :
64. Consider the following : 1. Coal bed methane is the pure methane gas
1. Bats extracted from coal seams, while shale gas is a
mixture of propane and butane only that can be
2. Bears
extracted from fine-grained sedimentary rocks.
3. Rodents
The phenomenon of hibernation can be observed 2. In India, abundant coal bed methane sources
in which of the above kinds of animals? exist, but so far, no shale gas sources have been
(a) 1 and 2 only found.
(b) 2 only Which of the statements given above is/are
(c) 1,2 and 3 correct?
(d) Hibernation cannot be obvserved in any of the (a) 1 only (b) 2 only
(c) Both 1 and 2 (d) Neither 1 nor 2
above
Ans. (d): Coal bed methane is a form of natural gas
Ans. (c) : Hibernation is a state of inactivity and extracted from coal beds and contains very little heavier
metabolic depression in endotherms. There are several hydrocarbons propane or butane, while statement (1) is
animals that hibernate-skunks, bees, snakes, and incorrect as coal bed methane is a form of natural gas
groundhogs , but bears and bats are the most well- extracted from coal beds and has butane and propane on
known. Rodents also undergo hibernation. hydrocarbons while shale gas carries methane. In India,
65. In addition to fingerprint scanning, which of the shale reserves are three in Cambay, Assam-Arakan,
following can be used in the biometric Gondwana, Krishana- Godavari, Kaveri and Indo
identification of a person? Gangetic plain, while coalbed methane is only available
at Asansol in West Bengal, so statement (2) is also
1. Iris scanning 2. Retinal scanning incorrect. Since both statements (1) and (2) are
3. Voice recognition incorrect, so option (d) is right.
State the correct answer using the code given
below: 69. Which of the following is/are the example/
(a) 1 only (b) 2 and 3 only examples of chemical change ?
(c) 1 and 3 only (d) 1, 2 and 3 1. Crystallization of sodium chloride
2. Melting of ice
Ans. (d) : Biometric verification is any means by 3. Souring of milk
which a person can be uniquely identified by Select the correct answer using the code given
evaluating one or more destining wishing biological below:
traits. Unique identifiers, hand geometry, earlobe (a) 1 and 2 only (b) 3 only
geometry, retina and iris patterns, voice waves, DNA (c) 1,2 and 3 (d) None
and signatures. Ans. (b): Chemical changes occur when a substance
66. Among the following organisms, which one does combines with another to form a new substance, called
not belong to the class of other three? Chemical synthesis into two are more different
(a) Crab (b) Mite substances. Crystallization of sodium chloride and
(c) Scorpion (d) Spider melting of ice are examples of physical changes, while
Ans. (a) : Mite, scorpion and spider belong to the class souring of milk is a chemical change.
Arachnida while crab belongs to class crustaceans. 70. Which of the following pairs is/are correctly
67. Consider the following pairs matched ?
Vitamin Deficiency disease Spacecraft Purpose
1. Vitamin C : Scurvy 1. Cassini-Huygens Orbiting the Venus and
2. Vitamin D : Rickets transmitting data to the Earth
3. Vitamin E : Night Blindness 2. Messenger Mapping and investigating the
Which of the pairs given above is / are correctly Mercury
matched ? 3. Voyager 1 and 2 Exploring the outer solar
(a) 1 and 2 only (b) 3 only system
(c) 1, 2 and 3 (d) None Select the correct answer using the code given
below:
Ans. (a): Scurvy is the name of disease for vitamin C (a) 1 only (b) 2 and 3 only
deficiency. Rickets is a skeletal disorder that’s caused (c) 1 and 3 only (d) 1, 2 and 3
by a lack of vitamin D, calcium, or phosphate. Night Ans. (b): Cassini Huygens is a spacecraft of NASA
blindness (in which it is difficult or impossible to see launched to probe into the mysteries of the planet
in relatively low light) is one of the clinical signs Saturn. Messenger is NASA mission to mercury and
of vitamin A deficiency. Voyager 1 and 2 are meant for exploring outer space.
IAS (Pre) GS 2014 Paper I 150 YCT
ECOLOGY on Climate Change (UNFCCC), to meet these
complex challenges with an integrated approach and
the best possible use of natural resources.
71. Which one of the following is the correct sequence
The rest of the two options are also connected with
of a food chain ?
biodiversity.
(a) Diatoms-Crustaceans-Herrings
(b) Crustaceans –Diatoms-Herrings 75. If a wetland of international importance is
(c) Diatoms-Herrings-Crustaceans brought under the ‘Montreux Record’, what does
(d) Crustaceans –Herrings-Diatoms it imply?
Ans. (a): Diatoms (autotrophic) → Crustaceans (a) Changes in ecological character have occurred
(herbivorous) → Herrings (carnivorous). This is the are occurring or are likely to occur in the wetland
correct sequence of the food chain. Omnivores in the as a result of human interference
food chain receive high nutrition. (b) The country in which the wetland is located
should enact a law to prohibit any human activity
72. There is some concern regarding the within five kilometers from the edge of the
nanoparticles of some chemical elements that are wetland
used by the industry in the manufacture of (c) The survival of the wetland depends on the
various products. Why? cultural practices and traditions of certain
1. They can accumulate in the environment and communities living in its vicinity and therefore
contaminate water and soil. the cultural diversity therein should not be
2. They can enter the food chains. destroyed.
3. They can trigger the production of free radicals. (d) It is given the status of ‘World Heritage Site’.
Select the correct answer using he code given Ans. (a): The Montreux Record is a register of wetland
below : sites on the list of wetlands of International Importance
(a) 1 and 2 only (b) 3 only where changes in ecological character have occurred, or
(c) 1 and 3 (d) 1, 2 and 3 are occurring, or likely to occur as a result of
Ans. (d): Many of the nano particles are very stable technological developments, pollution as other human
and hence have a tendency for accumulation. They can interference. It is maintained as part of the Ramsar List.
easily enter food chains because of their very small Currently 48 sites where listed in the Montreux Record.
size and stay in them because of their stable nature. Indian sites in the like Keoladeo National Park-
They have the ability to generate free radicals and Rajasthan, Loktak Lake - Manipur are included in
cause oxidative stress in cells. Montreux Record.
73. In India, the problem of soil erosion is associated 76. Consider the following pairs :
with which of the following? Programme/Project Ministry
1. Terrace cultivation 2. Deforestation 1. Drought –prone Area : Ministry of
3. Tropical climate Programme Agriculture
Select the correct answer using the codes given 2. Desert Development/ : Ministry of
below : Programme Environment and
(a) 1 and 2 only (b) 2 only
(c) 1 and 3 only (d) 1, 2 and 3 Forests
3. National Watershed / : Ministry of Rural
Ans. (b): Soil erosion is the result of human activities Development Project Development
against nature. The roots of plants keep the soil tight,
due to which soil weathering is not possible easily, but Which of the above pairs is/are correctly
where deforestation has taken place, soil erosion is matched?
occurring as a result. This problem has taken a serious (a) 1 and 2 only (b) 3 only
form in India. (c) 1,2 and 3 (d) None
Ans. (d): Drought prone Area Programme and Desert
74. Consider the following international agreements Development Programme are being run by the
1. The International Treaty on Plant Genetic Department of Land Resources, Ministry of Rural
Resource for Food and Agriculture. Development. The National Watershed Development
2. The United Nations Convention to Combat project for rainfed areas is being run by the
Desertification. Department of Agriculture with the cooperation of its
3. The World Heritage Convention. various departments.
Which of the above has/have a bearing on the 77. With reference to Bombay Natural History
biodiversity ? Society (BNHS), consider the following
(a) 1 and 2 only (b) 3 only statement-
(c) 1 and 3 only (d) 1, 2 and 3 1. It is an autonomous organization under the
Ans. (d): As per United Nations Convention to Ministry of Environment and Forests.
Combat Desertification (UNCCD)- "dynamic of land, 2. It strives to conserve nature through action –
climate and biodiversity are intimately connected, the based research, education and public awareness.
3. It organizes and conducts nature trails and camps
UNCCD collaborates closely with the other two Rio for the general public.
conventions: the Convention on Biological Diversity Which of the statements given above is/are
(CBD) and the United Nations Framework Convention correct?
IAS (Pre) GS 2014 Paper I 151 YCT
(a) 1 and 3 only (b) 2 only 1. They are highly resistant to degradation in the
(c) 2 and 3 only (d) 1,2 and 3 environment.
Ans. (c): The Bombay Natural History Society is one 2. They are able to accumulate in humans and
of the largest non-governmental organizations in India, animals.
established on 15 September, 1883 engaged in Select the correct answer using the code given
conservation and biodiversity research. In strives to below:
conserve nature through action-based research (a) 1 only (b) 2 only
education and public awareness. (c) Both 1 and 2 (d) Neither 1 nor 2
Ans. (c): Brominated flame retardants (BFRs) have
78. With reference to ‘Global Environment Facility’, routinely been added to customer products to make it
which of the following statements is/ are correct ? more difficult for a material to ignite or reduce the
(a) It serves as financial mechanism for ‘Convention spread of fire. But due to their chemical structure, they
on Biological Diversity’ and ‘United Nations are highly resistant to degradation in the environment,
Framework Convention on Climate Change’. due to which they can accumulate in humans and
(b) It undertakes scientific research on environmental animals if entered into the digestive system.
issues at global level. 82. Other than poaching, what are the possible
(c) It is an agency under OECD to facilitate the reasons for the decline in the population of
transfer of technology and funds to Ganges River Dolphins?
underdeveloped countries with specific aim to 1. Construction of dams and barrages on rivers.
protect their environment. 2. Increase in the population of crocodiles in rivers.
(d) Both (a) and (b) 3. Getting trapped in fishing nets accidentally.
Ans. (a): GEF is an independently operating financial 4. Use of synthetic fertilizers and other agricultural
organization, the GEF provides grants for projects chemicals in crop-fields in the vicinity of rivers.
related to biodiversity climate change, international Select the correct answer using the code given
waters land degradation, the Ozone layer and persistent below:
organic pollutants. (a) 1 and 2 only (b) 2 and 3 only
79. Consider the following pairs: (c) 1, 3 and 4 (d) 1, 2, 3 and 4
1. Dampa Tiger Reserve : Mizoram Ans. (c): The Ganga River Dolphin is found in the
Ganges-Brahmaputra-Meghna (GBM) and Karnaphuli-
2. Gumti Wildlife Sanctuary : Sikkim Sangu rivers of India and Bangladesh. The population
3. Saramati Peak : Nagaland of the Ganges Rivers Dolphin has declined to less than
Which of the above pairs is/are correct matched? 1800 from 6000 Since 1982 due to construction of
(a) 1 only (b) 2 and 3 only dams and water pollution caused by pesticides,
(c) 1 and 3 only (d) 1, 2 and 3 fertilizers and industrial effluents.
Ans. (c): Dampa Tiger Reserve is located in Mizoram. The crocodile population has nothing to do with the
The Gumti Wildlife Sanctuary is in Tripura, while the decline in population of dolphins.
Saramati peak extends from the state of Nagaland in 83. Which of the following add/adds carbon dioxide
India to the Sagaing region of Burma. to the carbon cycle on the planet Earth ?
80. With reference to conservation organization 1. Volcanic action
called’ Wetlands International’, Which of the 2. Respiration
3. Photosynthesis
following statements is/ are correct?
4. Decay of organic matter
1. It is an inter-governmental organization formed Select the correct answer using the code given
by the countries which are signatories of Ramsar below
Convention. (a) 1 and 3 only (b) 2 only
2. It works at the field level to develop and (c) 1, 2 and 4 (d) 1, 2,3 and 4
mobilize knowledge, and use the practical
experience to advocate for better policies. Ans. (c): Activities such as volcanic activity,
respiration and decay of organic matter etc, contribute
Select the correct answer using the code given
carbon dioxide to the carbon cycle on the Earth's
below; surface. In the process of photosynthesis, oxygen is
(a) 1 only (b) 2 only released by plants by consuming carbon dioxide.
(c) Both 1 and 2 (d) Neither 1 nor 2
84. If you walk through countryside, you are likely to
Ans. (b): Wetlands International is a non-profit and
see some birds stalking alongside the cattle to
non-governmental organisation working to protect and
seize the insects disturbed by their movement
enhance wetlands and its resources. Its headquarter is through grasses. Which of the following is/are
located in the Netherlands and the organization such bird/ birds?
operates in over 100 countries. 1. Painted Stork
81. Brominated flame retardants are used in many 2. Common Myna
household products like mattresses and 3. Black –necked Crane
upholstery. Why is there some concern about Select the correct answer using the code given
their use? below :
IAS (Pre) GS 2014 Paper I 152 YCT
(a) 1 and 2 (b) 2 only 88. With reference to ‘Eco–Sensitive Zones’, Which
(c) 2 and 3 (d) 3 only of the following statements is /are correct ?
Ans. (b): Common Myna are the birds stalking 1. Eco-Sensitive Zones are the areas that are
declared under the Wildlife (Protection) Act
alongside the cattle to seize the insects disturbed by their 1972.
movement through grasses. The common Myna is 2. The purpose of the declaration of Eco–Sensitive
readily indentified by the brown body, black hooded Zones is to prohibit all kinds of human activities
head and the bare yellow patch behind the eye. in these zones except agriculture.
85. Consider the following pairs : Select the correct answer using the code given
Wetlands Confluence of rivers below :
(a) 1 only (b) 2 only
1. Harike : Confluence of Beas and (c) Both 1 and 2 (d) Neither 1 nor 2
Wetlands Satluj/ Sutlej Ans. (d): The Eco-sensitive zone is declared under
2. Keoladeo Ghana : Confluence of Banas (Section 3) Environment (Protection) Act, 1986. Eco-
National Park and Chambal sensitive zones are areas in India around Protected
3. Kolleru Lake/ : Confluence of Musi and Areas, National Park and sanctuaries, which are
considered fragile eco zones. There are some activities
Krishna allowed in these areas are cottage industries, erection
Which of the above pairs is /are correctly of telecom and electricity wires, tourism, hot-air
matched? balloons, tourism etc.
(a) 1 only (b) 2 and 3 only 89. Consider the following statements :
(c) 1 and 3 only (d) 1, 2 and 3 1. Animal welfare Board of India is established
Ans. (a): Keoladeo Ghana National Park is located at under the Environment (Protection)Act, 1986 .
the confluence of the Banganga and Gambhira rivers in 2. National Tiger Conservation Authority is a
the Bharatpur district of Rajasthan. Kolleru Lake is statutory body.
3. National Ganga River Basin Authority is Chaired
located in the Godavari and Krishna River deltas in by the Prime Minister.
Andhra Pradesh. Which of the statements given above is/are
86. The most important strategy for the conservation correct?
of biodiversity together with traditional human (a) 1 only (b) 2 and 3 only
(c) 2 only (d) 1, 2 and 3
life is the establishment of
(a) Biosphere reserves (b) Botanical gardens Ans. (b): The Animal Welfare Board of India was
(c) National parks (d) Wildlife sanctuaries established in 1962 under section 4 of the prevention of
cruelty to Animals Act 1960. The National Tiger
Ans. (a): Botanical gardens and National Parks are the Conservation Authority is set up under the chairmanship
wrong options as in botanical gardens people don't live of the Minister for Environment and Forests and is a
and in national parks, human activity is totally statutory body. National Ganga River Basin Authority
prohibited. was established by the Central Government of India on
In wildlife sanctuaries, human activity is partially 20th February 2009. The Prime Minister is the
allowed. Therefore the biosphere reserves are the best chairperson of the authority.
option.
Biosphere Reserve is an area of active cooperation CURRENT AFFAIRS
between the management and the local people, where
activities like settlements, cropping, forestry, 90. Consider the following languages.
recreation and other economic activities are carried in 1. Gujarati 2. Kannada
harmony with the conservation goals. 3. Telugu
Which of the above has/have been declared as
87. Consider the following statements regarding ‘Classical Language/ Languages’ by the
‘Earth Hour’ : Government ?
1. It is an initiative of UNEP and UNESCO. (a) 1 and 2 only (b) 3 only
2. It is a movement in which the participants switch (c) 2 and 3 only (d) 1, 2 and 3
off the lights for one hour on a certain day every
year. Ans. (c) : Tamil language has been conferred classical
3. It is a movement to raise to awareness about the language status by the Government of India in 2004,
Sanskrit language in 2005, Telugu, Kannada and
climate change and the need to save the planet. Malayalam language in 2008. In February 2014, the
Which of the statements given above is /are Government of India also gave the status to Odia
correct? language. Now, all 6 languages have got the status of
(a) 1 and 3 (b) 2 only classical language.
(c) 2 and 3 only (d) 1, 2 and 3
91. With reference to a grouping of countries
Ans. (c): Earth Hour is a worldwide movement for the
planet organized by the World Wide Fund for Nature known as BRICS, consider the following
WWF. The event is held worldwide annually, statements :
encouraging individuals communities, households and 1. The first Summit of BRICS was held in Rio de
businesses to turn off 8:30 to 9:30 P.M. to raise Janeiro in 2009.
awareness about climate change and the need to save 2. South Africa was the last to join the BRICS
the planet. grouping.
IAS (Pre) GS 2014 Paper I 153 YCT
Which of the statements given above is/are Ans. (b): Sattriya form uses a variety of hand gestures
correct? and foot positions. It also has its own style of music
(a) 1 only (b) 2 only based on classical ragas and the talas (rhythm) of
(c) Both 1 and 2 (d) Neither 1 nor 2 Borgeet (development songs composed by Sankardeva
and Madhaveva).
Ans. (b): BRICS is the acronym for an association of
the five major emerging national economies : Brazil, 96. With reference to India’s culture and tradition.
What is ‘Kalaripayattu’?
Russia, India, China and South Africa. The grouping (a) It is an ancient Bhakri cult of Shaivism still
was originally known as "BRIC" before the inclusion prevealent in some parts of South India.
of South Africa in 2010. The BRIC first formal summit (b) It is an ancient style bronze and brass work still
th
held in Yekaterinburg (Russia) commenced on 16 found in southern part of Coromandel area.
June 2009. (c) It is an ancient form of dance-drama and a living
92.Recently a series of uprisings of people referred tradition in the northern part of Malabar.
(d) It is an ancient martial art and an living .
to as ‘Arab Spring’ originally started from? tradition in some parts of South India.
(a) Egypt (b) Lebanon Ans. (d): Kalaripayattu is an ancient martial art and a
(c) Syria (d) Tunisia living tradition in Kerala and Tamil Nadu.
Ans. (d): Arab Spring was a series of anti-government 97. Consider the following pairs
protests, uprisings, and armed rebellions that spread 1. Garba : Gujarat
across much of the Arab world in the early 2010s. 2. Mohiniattam : Odisha
Arab Spring began with the overthrow of ruler of 3. Yakshagana : Karnataka
Tunisia, Ben Ali who fled to Saudi Arabia. Which of the pairs given above is /are correctly
93. Consider the following pairs : matched?
Region often in news Country (a) only 1 (b) 2 and 3 only
(c) 1 and 3 only (d) 1,2 and 3
1. Chechnya : Russian Ans. (c) : Garba is a major folk dance of Gujarat.
2. Darfur : Federation Mali Mohiniattam is a classical dance of Kerala. The
3. Swat Valley : Iraq Yakshagana practised in Karnataka is an amalgam of
Which of the above pairs is/are correctly matched? music, dance and dialogue.
(a) 1 only (b) 2 and 3 only 98. A community of people called Manganiyars is
(c) 1 and 3 only (d) 1, 2 and 3 well known for their .
(a) Martial arts in North – East India
Ans. (a): Chechnya is a republic of Russia, which was (b) Musical tradition in North- West India
formed on 10th January, 1993. It is 100 km from the (c) Classical vocal music in South India
Caspian Sea in the southern part of Eastern Europe in (d) Pietra dura tradition in Central India
the North Caucasus. Darfur is located in western Ans. (b) : Manganiyars is a tribal community(Muslim)
Sudan, while Swat valley is located in the Khyber found in the desert of Rajasthan (mostly in Barmer and
Pakhutunkhwa province of Pakistan. Jaisalmer. This community is known for following a
particular type of music tradition.
ART & CULTURE
MISCELLANEOUS
94. Consider the following towns of India
1. Bhadrachalam 2. Chanderi 99. With reference to Agni-IV Missile, which of the
3. Kancheepuram 4. Karnal following statements is/are correct ?
1. It is a surface-to-surface missile.
Which of the above are famous for the 2. It is fuelled by liquid propellant only.
production of traditional sarees/ fabric ? 3. It can deliver one tonne nuclear warheads about
(a) 1 and 2 only (b) 2 and 3 only 7500km away.
(c) 1,2 and 3 (d) 1, 3 and 4 Select the correct answer using the code given
below:
Ans. (b): Kancheepuram, located in Tamil Nadu, is (a) 1 only (b) 2 and 3 only
famous worldwide for the production of traditional silk (c) 1 and 3 only (d) 1, 2 and 3
sarees. Chanderi is the township in the Ashok Nagar Ans. (a): Not only Agni-IV but all the missiles in this
district of Madhya Pradesh, which is also famous for series are surface to-surface missile. It uses solid
manufacturing special types of silk sarees. propellant as fuel. It can deliver one tonne nuclear
95. With reference to the Famous Sattriya dance warheads upto a maximum distance of 4000 km.
consider the following statements. 100. Chaitra 1 of the National calendar based on the
1. Sattriya is a combination of music, dance, and Saka Era corresponds to which one of the
following dates of the Gregorian calendar in a
drama normal year of 365 days.
2. It is a centuries-old living tradition of (a) 22nd
th
March (or 21th
st
March)
Vaishnavites of Assam. (b) 15 May (or 16 May)
3. It is based on classical Ragas and Talas of (c) 31stst March (or 30 th
th
March)
devotional songs composed by Tulsidas, Kabir (d) 21 April (or 20 April)
Ans. (a): Chaitra 1 of the National nd calendar based on
and Mirabai. the Saka Era corresponds to 22 March of the
Which of the statements given above is /are Gregorian calendar in a normal year of 355 days and
correct? on 21st March in a leap year.
The Saka era marked the beginning of the Saka
(a) 1 only (b) 1 and 2 only Samvat; a historic Hindu calendar which was later
(c) 2 and 3 only (d) 1, 2 and 3 introduced as 'Indian National Calendar' in 1957.
IAS (Pre) GS 2014 Paper I 154 YCT
UNION PUBLIC SERVICE COMMISSION
Civil Services (Preliminary Exam) - 2013
GENERAL STUDIES : PAPER-I
Time: 2 hours (Exam date : 19.05.2013) Maximum Number: 200

ANCIENT HISTORY 3. Some Buddhist rock-cut caves are called


Chaityas, while the others are called Viharas.
• Indus Valley Civilization What is the difference between the two ?
(a) Vihara is a place of worship, while Chaitya is the
1. Which of the following characterizes/characterize dwelling place of the monks.
the people of Indus Civilization? (b) Chaitya is a place of worship, while vihara is the
1. They possessed great palaces and temples. dwelling place of the monks.
2. They worshipped both male and female deities. (c) Chaitya is the stupa at the far end of the cave,
3. They employed horse-drawn chariots in warfare. while Vihara is the hall axial to it.
(d) There is no material difference between the two.
Select the correct statement/ statements using the
codes given below : Ans : (b) In Buddhist architecture, Chaitya is an
(a) 1 and 2 only assembly or prayer hall, which houses a Stupa. Vihara,
on the other hand, is a dwelling or refuge for monks.
(b) 2 only
(c) 1, 2 and 3 4. Which of the following statements is/ are
(d) None of the statements given above is correct applicable to jain doctrine ?
1. The surest way of annihilating Karma is to
Ans : (b) Excavation of Indus valley civilization has not practice penance.
found evidence of a place of worship or temple from 2. Every object, even the smallest particle has a
any site. Evidence of worship of Matri Devi and Adya soul.
Shiva is found on the basis of female Terracotta 3. Karma is the bane of the soul and must be
figurines found from places like Harppa, Mohenjodaro. ended.
There is no clear evidence of horse from any site other (a) 1only (b) 2 and 3 only
than horse bone found at Surkotada in Indus (c) 1 and 3 only (d) 1, 2 and 3
civilization. For this reason, any evidences of a chariot Ans: (d) Mahavir Swami was the 24th Tirthankara of
drawn by horses are inaccessible. Jainism and he is believed to be the real founder of
Jainism. The Jain text 'Acharang Sutra' states that the
• Buddhism, Jainism, Bhagavata and surest way to annihilate Karma is austerity. According
Shaiva Religion to Jain philosophy, the soul resides in everything in the
2. With reference to the history of philosophical universe, even if it is the smallest particle. In Jain
thought in India, consider the following philosophy, the flow of the molecules of Karma towards
statements regarding Samkhya school. the life 'Asrav', if the flow of the molecules of Karma
1. Samkhya does not accept the theory of rebirth or stops towards the organism, the burning of residual
transmigration of soul. Karma or the erosion of the already existing molecules
is called 'Nirjara'. According to these principles, karma
2. Samkhya holds that it is the self-knowledge that is the bane of the soul and it must be ended.
leads to liberation and not any exterior influence
or agent. 5. Which one of the following describes best the
Which of the statements given above is/are concept of Nirvana in Buddhism?
correct? (a) The extinction of the flame of desire
(b) The complete annihilation of self
(a) 1only (b) 2 only
(c) A state of bliss and rest
(c) Both 1 and 2 (d) Neither 1 nor 2 (d) A mental stage beyond all comprehension
Ans : (b) The Samkhya school is part of the six Ans : (a) According to Buddhism, the ultimate goal of
orthodox schools of Indian philosophy. The Sage human life is to attain Nirvana. Nirvana means the
Kapila was the founder of Samkhya School. This extinguishment of the flame of personal desires i.e.
school denies the existence of God. However, rebirth getting rid of the cycle of life and death. This Nirvana
and transmigration of the soul are inherent in the can be attained only when all the present desires cease
Samkhya School. In Samkhya School, liberation and the desires arising in the future are destroyed from
results from the disappearance of the limitations of the root. In Nirvana, cravings and malice are ceased
self-awareness. It is not from any external influence or from the root, due to which one attains freedom from all
agent. kinds of suffering and the rebirth cycle of the world.

IAS (Pre) GS 2013 Paper I 155 YCT


• Maurya & Post–Mauryan Period According to him, the tax was paid to the state by the
merchants on the trading ghats, routes and items of sale.
6. With reference to the history of Indian rock-cut Hiuen Tsang biography 'Si-Yu-Ki' provides many
architecture, consider the following statements: important information in the context of political, social,
1. The caves at Badami are the oldest surviving rock- religious and economic subjects of erstwhile India.
cut caves in India.
2. The Barabar rock-cut caves were originally made
for Ajivikas by Emperor Chandragupta Maurya. MODERN HISTORY
3. At Ellora, caves were made for different faiths.
Which of the statements given above is/are • Social-Cultural Awakening, Caste,
correct ? Trade Union and Peasant Movement
(a) 1 only (b) 2 and 3 only
(c) only 3 (d) 1, 2 and 3 9. Consider the following Bhakti Saints :
Ans : (c) The period of construction of rock- cut caves 1. Dadu Dayal
of Badami, Barabar, and Ellora was between the sixth 2. Guru Nanak
century, to the first century B.C. and the fifth to 3. Tyagaraja
seventh century B.C., respectively. The rock cut caves Who among the above was/ were preaching
of Badami are not the oldest as the caves of Barabar, When the Lodi dynasty fell and Babur took over?
Ajanta and Ellora were built earlier. The Barabar (a) 1 and 3 (b) 2 only
rock-cut caves were built by Emperor Ashoka and the (c) 2 and 3 (d) 1 and 2
Ellora caves belonged to Jain, Buddhist and Hindu Ans : (b) The period of the Lodi dynasty was from 1451
religions.
to 1526. At this time, Guru Nanak (1469-1539) gave his
• Gupta Period & Post Gupta Period Sermons. He conceived the formless God and called this
God as a man of Famine. However, as per the question,
7. Consider the following historical places : other devotee Saint, Dadu-Dayal, was born in 1544 and
1. Ajanta Caves died in 1603 A.D. and Tyagaraja was born in 1767 and
2. Lepakshi Temple died in 1847.
3. Sanchi Stupa
Which of the above places is/are also known for 10. The demand for the Tebhaga Peasant Movement
mural paintings ? in Bengal was for
(a) 1 only (b) 1 and 2 only (a) the reduction of the share of the landlords from
(c) 1, 2 and 3 (d) None one-half of the crop to one-third.
Ans : (b) It is well known that the Ajanta caves have (b) the grant of ownership of land to peasants as
mural paintings. These paintings have Buddhist they were the actual cultivators of the land.
religious themes and depict the Jataka tales. The (c) The uprooting of Zamindari system and the end
Lepakshi temple also has mural paintings of the of serfdom.
Vijaynagara kings. The Sanchi Stupa, on the other (d) Writing off all peasant debts.
hand, does not have mural paintings.
Ans : (a) The Tebhaga Peasant Movement was initiated
8. The Chinese traveller Yuan Chwang (Hiuen in Bengal in 1946 by the Bengal Provincial Kisan Sabha
Tsang) who visited India recorded the general to implement the Flood Commission recommendation,
conditions and culture of India at that time . In which asked for Share-croppers to give one-third of
this context, which of the following statements their harvest to the landowner instead of the one-half,
is/are correct . which they were paying at that time. In its response, the
1. The roads and river-routes were completely Bargadari Act was passed, which provided that share
immune from robbery. croppers need to pay only one-third of their harvest to
2. As regards punishment for offences, ordeals by the landlords.
fire, water and poison were the instruments for
determining the innocence or guilt of a person . • Major Institutions, Treaties,
3. The tradesmen had to pay duties at ferries and Commissions & Acts
barrier stations.
Select the correct answer using the codes given 11. Annie Besant was
(a) 1 only 1 (b) 2 and 3 only 1. Responsible for starting the Home Rule
(c) 1 and 3 only (d) 1, 2 and 3 Movement
2. The founder of the Theosophical Society
Ans : (b) Chinese traveller Hiuen Tsang came to India 3. Once the President of the Indian National
during Harshavardhana (606-647 AD) time. According Congress
to him, public on the roads was not always completely Select the correct statement/statements using the
safe. Hence statement (1) is wrong. According to him, codes given below :
for the crime or guilt/ innocent accomplishment, fire (a) 1 only (b) 2 and 3 only
water and poison were subjected to a truth test. (c) 1 and 3 (d) 1, 2 and 3
IAS (Pre) GS 2013 Paper I 156 YCT
Ans : (c) The Establishment of Home Rule League was GEOGRAPHY OF INDIA
originally planned by Annie Besant. She founded it in
September 1916 and through her newspaper • Physical Structure/Drainage System/
'Commonweal' and 'New India', she publicized the Human Geography
programs of the league. She was elected President at
the 1917 INC session. The Theosophical Society was 15. Consider the following pairs:
founded in Adyar (Chennai) in the year 1882 by Tribe State
Colonel H.S. Olcott and Madame Blavatsky. 1. Limboo (Limbu) – Sikkim
2. Karbi – Himachal Pradesh
12. The Ilbert Bill controversy was related to the
3. Dongaria Kondh – Odisha
(a) imposition of certain restrictions to carry arms
4. Bonda – Tamilnadu
by the Indians
Which of the above pairs are correctly matched?
(b) imposition of restrictions on newspapers and
(a) 1 and 3 only (b) 2 and 4 only
magazines published in Indian languages
(c) 1,3 and 4 (d) 1, 2, 3 and 4
(c) removal of disqualificaions imposed on the
Indian magistrates with regard to the trial of the Ans : (a) Correct matches are –
Europeans Tribe State
(d) removal of a duty on imported cotton cloth Limboo Sikkim
Karbi Assam
Ans : (c) The Ilbert Bill was introduced in 1883 by Dongaria Kondh Odisha
Viceroy Ripon to amend the existing laws to allow the Bonda Odisha
Indian judges to try British offenders in criminal cases
at the district level. 16. The narmada river flows to the west, while most
other large peninsular rivers flow to the east.
• Freedom Struggle & National Why?
Movement 1. It occupies a linear rift valley.
2. It flows between theVindhyas and the Satpuras.
13. The people of India agitated against the arrival of 3. The land slopes to the west from Central India
Simon Commission because :
Select the correct answer using the codes given
(a) Indians never wanted the review of the working below
of the Act of 1919 (a) 1 only (b) 2 and 3
(b) Simon Commission recommended the abolition (c) 1 and 3 (d) None
of Dyarchy (Diarchy) in the Provinces
(c) there was no Indian member in the Simon Ans : (a) The Narmada and the Tapi flow through the
Commission rift valley. Statement 3 is wrong, as land slopes to the
(d) the Simon Commission suggested the partition of east in this region. Statement 2 is factually correct
the country however it is not the reason for the rivers to flow to the
west.
Ans : (c) All the members of the 7 members of the
Simon Commission Constituted under the • Monsoon, Forest, Soil, Irrigation
chairmanship of Simon were English; hence
Congressmen called it white commission. Then an Projects and Agriculture
annual session of the congress held in Madras on 27 17. Which of the following leaf modifications
December 1927, presided over by M.A. Ansari, it was occurs/occur in desert areas to inhibit water loss
decided to complete boycott the Simon Commission. 1. Hard and waxy leaves
14. Quit India Movement was launched in response 2. Tiny leaves or no leaves
to 3. Thorns instead of leaves
(a) Cabinet Mission Plan Select the correct answer using the codes given
(b) Cripps Proposals below :
(c) Simon Commission Report (a) 1 and 2 only (b) 2 only
(d) Wavell Plan (c) 1 and 3 only (d) 1,2 and 3
Ans : (b) Quit India Movement was launched in August Ans : (d) To prevent water loss in the desert, there are
1942. It came in response to the Cripps’ Proposal. The three factors which are as follows- Hard and Waxy
Cripps Mission came to India in March 1942 and was leaf, small leaf or foliage, Thorn in place of the leaf.
headed by Sir Stafford Cripps. It had come to negotiate Since there is a shortage of water in the desert land, the
a proposal for self-government for India in return for plants do not get enough water. In order to maintain
cooperation in the World War II effort. Commenting on water compatibility, the emission of water is minimal,
Cripps proposals Gandhi said, it is a post-dated cheque for this the bark of desert plants turn into hard foliage
of a failing bank. and leaf into a thorn.
IAS (Pre) GS 2013 Paper I 157 YCT
18. Which of the following statements regarding Ans : (d) On its path around the Sun the earths axis
laterite soils of India are correct. always remains inclined to one side. Because of this
1. They are generally red in colour. constant inclination in one direction. The Northern
2. They are rich in nitrogen and Potash Hemisphere remains inclined towards the sun or faces
3. They are well-developed in Rajasthan and UP. the sun during one half of the year. Therefore, a larger
4. Tapioca and cashew nuts grow well on these soils. part of this hemisphere receives sunlight. Every point
Select the correct answer using the codes given in this hemisphere takes a longer time to go out of the
below : sunlight, with the result that the days are longer. In
(a) 1,2 and 3 (b) 2, 3 and 4 contrast, the Southern Hemisphere is away from the
(c) 1 and 4 (d) 2 and 3 only sun. It, therefore, has shorter days and longer nights.
Ans : (c) Statement 2 is wrong as these soil are poor in During the other half of the year, the Southern
nitrogen and phosphate. Statement 3 is wrong as Hemisphere is inclined towards the Sun. Hence, it has
laterite is not found in U.P. and Rajasthan. Red longer days and shorter nights.
Laterite soils found in Tamil Nadu, Andhra Pradesh
and Kerala are more suitable for tree crops like • Lithosphere
cashewnut and Tapioca. 22. The annual range of temperature in the interior
of the continents is high as compared to coastal
• Minerals, Power Resources, Industry areas. What is / are the reason/ reasons ?
and Trade 1. Thermal difference between land and water
19. Which of the following is/are the 2. Variation in altitude between continents and
characteristic/characteristics of Indian coal ? oceans
3. Presence of strong winds in the interior
1. High ash content
4. Heavy rains in the interior as compared to coasts
2. Low sulphur content
3. Low ash fusion temperature Select the correct answer using the codes given
below :
Which of the statements given above is/are
(a) 1 only (b) 1 and 2 only
correct ?
(c) 2 and 3 only (d) 1, 2, 3 and 4
(a) 1 and 2 only (b) 2 only
(c) 1 and 3 only (d) 1, 2 and 3 Ans : (a) Lands heats up and cool down very fast as
Ans : (a) Indian coal has a very high ash content, which compared to the water. Areas in the interior of India are
varies from 35 to 45% as compared to 15% in other far away from the moderating influence of the Sea. Such
parts of the world. Indian coal also has a high ash fusion areas have extremes of climate. Statement 2 is wrong as
temperature of 1500ºC. However, Indian coal has a low there may or may not be much difference between the
sulphur content at about 0.5%. altitudes of land and ocean. Statement 3 is wrong as
winds do not contribute enough to the variations in
20. Consider the following statements : temperature. Statement 4 is wrong as coastal regions
1. Natural gas occurs in the Gondwana beds receive more rainfall.
2. Mica occurs in abundance in Kodarma.
3. Dharwars are famous for petroleum. 23. Which one the of the following pairs is correctly
Which of the statements given above is/are matched ?
correct? Geographical Feature Region
(a) 1 and 2 (b) 2 only (a) Abyssinian Plateau : Arabia
(c) 2 and 3 (d) None (b) Atlas Mountains North-Western
Ans : (b) Natural gas is obtained along with oil in all Africa
the oil fields but exclusive reserves have been located (c) Guiana Highlands South-Western
along the eastern coast (Tamil Nadu, Odisha and Africa
Andhra Pradesh) as well as Tripura, Rajashtan and off- (d) Okavango Basin Patagonia
shore wells in Gujarat and Maharashtra. Crude Ans : (b) The correct Match are–
petroleum occurs in sedimentary rocks of the tertiary Geographical Feature Region
period. Koderma (Jharkhand) is a popular site of Abyssinian Plateau – Ethiopia
mining for Mica. Atlas Mountains – North-Western
Africa
Guiana Highlands – North-East South
WORLD GEOGRAPHY America
Okavango Basin – South-west Africa
• Universe & Solar System
21. Variations in the length of daytime and night time 24. On the planet earth, most of the freshwater exists
from season to season are due to as ice caps and glaciers. Out of the remaining
(a) the earth’s rotation on its axis freshwater, the largest proportion
(b) the earth’s revolution round the sun in an (a) is found in atmosphere as moisture and clouds
elliptical manner (b) is found in freshwater lakes and rivers
(c) latitudinal position of the place (c) exists as ground water
(d) revolution of the earth on a tilted axis (d) exists as soil moisture
IAS (Pre) GS 2013 Paper I 158 YCT
Ans : (c) Of the total water on Earth, the percentage of • Human Geography and Cartography
water in the freshwater resources are as follows –
28. To obtain full benefits of demographic dividend,
Ice caps and glaciers – 1.74%
what should India do ?
Fresh groundwater – 0.76%
(a) Promoting skill development
Freshwater Lakes – 0.007% (b) Introducing more social security schemes
Rivers – 0.002% (c) Reducing infant mortality rate
Soil moisture – 0.001% (d) Privatization of higher education
Atmosphere – 0.001% Ans: (a) Option (b) will not provide any benefit of
25. Consider the following pairs : demographic dividend. Social security schemes can be
1. Nokrek Biosphere Reserve : Garo Hills used for providing security to non-working populations
2. Logtak (Loktak) Lake : Barail Range like old-age pension etc. Option (c) is one of the
3. Namdapha National Park : Dafla Hills Millennium Development Goals, but it does not
provide any benefit of demographic dividend. Option
Which of the above pairs is /are correctly
(d), in fact, have a negative effect as it will make
matched ? education costlier, thus making it out of reach of a
(a) 1 only (b) 2 and 3 only large section of the eligible population.
(c) 1,2 and 3. (d) None
29. ‘’Climate is extreme, rainfall is scanty as the
Ans : (a) Pair 1 is correctly matched since the Nokrek
people used to be nomadic herders’’
Biosphere Reserve is located in the Garo Hills. Pair 2
The above statement best describes which of the
is incorrectly matched because Loktak Lake is located following regions ?
in the Laimatol Range (Manipur), not in Barail Range. (a) African Savannah
Pair 3 is also incorrect since the Namdapha National (b) Central Asian Steppe
Park is located far away from the Dafla Hills. While (c) North American Prairie
Dafla Hills is located in the western part of Arunachal (d) Siberian Tundra
Pradesh. Namdapha is located in the Extreme East. Ans : (b) Steppe's climate is continental with extremes
of temperature. Nomadic tribes like Kirghiz are found.
• Economic Geography Rainfall is expected to be light. African Savannah gets
26. The most important fishing grounds of the world moderate rainfall.
are found in the regions where
(a) warm and cold atmospheric currents meet INDIAN CONSTITUTION & POLITY
(b) rivers drain out large amounts of freshwater into
the sea • Constitutional Development of India
(c) warm and cold oceanic currents meet 30. With reference to Indian History, the Members
(d) continental shelf is undulating of the Constituent Assembly from the Provinces
Ans : (c) Planktons are abundant where warm and cold were
oceanic currents meet. That makes these areas (a) directly elected by the people of those provinces
important fishing grounds of the world oceanic. (b) nominated by the Indian National Congress and
the Muslim League
27. Which of the following is/are unique (c) elected by the Provincial Legislative Assemblies
characteristic/characteristics of equatorial forests (d) selected by the Government for their expertise in
1. Presence of tall, closely set trees with crowns constitutional matters
forming a continuos canopy Ans : (c) The Assembly members were elected to it
2. Coexistence of a large number of species indirectly by the members of the individual provincial
3. Presence of numerous varieties of epiphytes legislative assemblies.
Select the correct answer using the codes give • Citizenship, Fundamental Rights,
below
(a) 1 only (b) 2 and 3 only
Fundamental Duties, Directive
(c) 1 and 3 only (d) 1, 2 and 3 Principle of State Policy
Ans : (d) Equatorial forests are found between 0º to 5º 31. ‘Economic Justice’ as one of the objectives of the
northern and southern latitudes. The main features of Indian Costitution has been provided in
these forests are as follows– (a) The Preamble and the Fundamental Rights
• Existence of Tall, dense trees. (b) The Preamble and the Directive Principles of
State Policy
• Coexistence of the large number of various species.
(c) The Fundamental Rights and the Directive
• The existence of innumerable varieties of Principles of State Policy
epiphytes. (d) None of the above
IAS (Pre) GS 2013 Paper I 159 YCT
35. Consider the following statements :
Ans : (b) Directive Principle of state policy strives for
economic democracy justice. While fundamental rights 1. The Council of Ministers in the Centre shall be
strive for 'Political democracy justice.' collectively responsible to the Parliament.
2. The Union Ministers shall hold the office during
32. According to the Constitution of India, which of the pleasure of the President of India.
the following are fundamental for the governance 3. The Prime Minister shall communicate to the
of the country ? President about the proposals for legislation.
(a) Fundamental Rights Which of the statements given above is/are
(b) Fundamental Duties correct?
(c) Directive Principles of State Policy (a) 1 only (b) 2 and 3 only
(d) Fundamental Rights and Fundamental Duties (c) 1 and 3 only (d) 1, 2 and 3
Ans : (c) Directive Principles of state policy are Ans : (b) Statement 2 is from Article 75. Statement 3
fundamental for the governance of country . is from Article 78. Statement 1 is from Article 75,
where it is collectively responsible to the House of
• Executives of Center and States, People (Lok Sabha). Parliament means both Rajya
Emergency Provisions/Parliament Sabha and Lok Sabha. So the answer should be (b).
33. Consider the following statements : 36. Consider the following statements :
The Parliamentary Committee on Public 1. The Chairman and the Deputy Chairman of the
Accounts Rajya Sabha are not the members of that House.
1. consists of not more than 25 Members of the 2. While the nominated members of the two Houses
Lok Sabha of the Parliament have no voting right in the
2. scrutinizes appropriation and finance accounts presidential election, they have the right to vote in
the election of the Vice President
of the Government
Which of the statements given above is/are
3. examines the report of the Comptroller and
correct ?
Auditor General of India
(a) 1 only (b) 2 only
Which of the statements given above is /are
(c) Both 1 and 2 (d) Neither 1 Nor 2
correct?
(a) 1 only (b) 2 and 3 Ans : (b) Statement 1 is incorrect. The chairman of
(c) only 3 (d) 1, 2 and 3 Rajya Sabha is the Vice-President. However, the Deputy
Chairman is a member of the Rajya Sabha and is elected
Ans : (b) Statement 1 is incorrect because the PAC among them. Statement 2 is correct; the electoral college
consists of only 22 member, of which 15 members are in the presidential election consists of only the elected
from Lok Sabha and 7 from the Rajya Sabha. The PAC members of the Parliament and state Legislative
examines the report of the Comptroller and Auditor Assemblies. The Vice-Presidential election, however
General, which also includes the examination of the has a part of its electoral college of all the members of
appropriation and finance accounts of the Government the Parliament, whether elected or nominated.
of India.
37. What will follow if a Money Bill is substantially
34. In the context of India, which of the following amended by the Rajya Sabha ?
principles is /are implied institutionally in the (a) The Lok Sabha may still proceed with the Bill,
parliamentary government ? accepting or not accepting the recommendations
1. Members of the Cabinet are Members of the of the Rajya Sabha.
Parliament. (b) The Lok Sabha cannot consider the Bill further.
2. Ministers hold of the office till they enjoy (c) The Lok Sabha may send the Bill to the Rajya
confidence in the Parliament. Sabha for reconsideration.
3. Cabinet is headed by the Head of the State. (d) The President may call a joint sitting for passing
Select the correct answer using the codes given the Bill.
below
Ans : (a) The procedure to pass money Bill is
(a) 1and 2 only (b) 3 only
mentioned in Article 109 of the Constitution. Such a
(c) 2 and 3 (d) 1, 2 and 3
bill can be introduced only in the Lok Sabha and on
Ans : (a) In the parliamentary system as existing in the recommendations of the President. After being
India, members of the cabinet have to be members of passed in the Lok Sabha, Rajya Sabha then has three
the parliament. If they are not members of the options. It can either pass the bill; in that case it goes
parliament at the time of appointment as Cabinet to the President for his assent. It can keep the bill with
Ministers, then they have to do so in 6 months. Article itself, in which case, after 14 days, the bill is deemed
75 (3) ministers shall be collectively responsible to the to be passed by both houses in the form in which it was
house of people i.e. Lok Sabha. Statement 3 is wrong passed by the Lok Sabha. The third is, Rajya Sabha
as the cabinet is headed by the Head of Government can make recommendations that Lok Sabha may or
(i.e. P.M.) and not by the head of State (i.e. President). may not accept. So, Statement 1 is correct.
IAS (Pre) GS 2013 Paper I 160 YCT
38. Which one of the following statements is correct ? 41. Which of the following bodies does not / do not
(a) In India, the same person cannot be appointed as find mention in the Constitution ?
Governor for two or more States at the same
time. 1. National Development Council
(b) The Judges of the High Court of the States in 2. Planning Commission
India are appointed by the Governor of the State 3. Zonal Councils
just as the Judges of the Supreme Court are Select the correct answer using the code given
appointed by the President. below :
(c) No procedure has been laid down in the
Constitution of India for the removal of a (a) 1 and 2only (b) 2 only
Governor from his/ her post. (c) 1 and 3 only (d) 1,2 and 3
(d) In the case of a Union Territory having a Ans : (d) None of the bodies mentioned above are
legislative setup, the Chief Minister is appointed found in the Constitution. The Planning Commission
by the Lt. Governor on the basis of majority
and the National Development Council are not even
support.
statuary bodies and have been set by executive orders
Ans : (c) Statement 1– The same person can be
appointed as Governor of two or more states at the same in 1950 and 1952, respectively. The zonal councils are
time. statutory bodies and were set up under the States
Statement 2– Judges of HC are also appointed by the Reorganization Act 1956.
President.
Statement 4– In the case of UTs, CM is appointed by the 42. The Parliament can make any law for whole or
President. any part of India for implementing international
39. Consider the following statements. treaties –
Attorney General of India can – (a) with the consent of all the states
1. take part in the proceedings of the Lok Sabha (b) with the consent of majority of states
2. be a member of a committee of the Lok Sabha (c) with the consent of the states concerned
3. speak in the Lok Sabha (d) without the consent of any state
4. vote in the Lok Sabha
Which of the statements given above is/are Ans : (d) Article 253 of the constitution of India
correct? provides that the Union Parliament can make any law
(a) 1 only (b) 2 and 4 to implement international treaties and obligations. In
(c) 1, 2 and 3 (d) 1 and 3 only such a case, the Parliament is not bounded by the usual
Ans : (c) Article 88 of the constitution provides that divisions of legislative subjects between the Union and
the Attorney General has the right to speak in and take the states and does not need the consent of any of the
part in the proceedings of either house of parliament or
states concerned.
a joint sitting of the two houses or any of their
committees. However, he does not have the power to
vote in any such proceedings, according to the same • Judicial System (Central &State)
article. 43. With reference to National Legal Services
Authority, consider the following –
• Center-State Relations, Administra-
1. Its objective is to provide free and competent
tion, Jammu & Kashmir legal services to the weaker sections of the society
40. Who among the following constitute the National on the basis of equal opportunity.
Development Council ? 2. It issues guidelines for the State Legal Services
1. The Prime Minister Authorities to implement the legal programmes
2. The Chairman Finance Commission and schemes throughout the country.
3. Ministers of the Union Cabinet Which of the statements given above is /are
4. Chief Ministers of the States correct ?
Select the correct answer using the codes given (a) 1 only (b) 2 only
below : (c) Both 1 and 2 (d) Neither 1 nor 2
(a) 1, 2 and 3 only (b) 1, 3 and 4 only
(c) 2 and 4 only (d) 1, 2, 3, and 4 Ans: (c) The National Legal Services Authority has
Ans : (b) On August 6, 1952, the National Development been constituted to provide free legal service to the
Council was formed as a non-constitutional body by a weaker sections under the "Legal Services Authorities
resolution of the government. It is headed by the Prime Act, 1987." The National Legal Services Authority
Minister. Initially, the chief minister of the states used to issues directives to the state legal services authorities
be its member. However, since 1967 all members of the to provide free legal services to the weaker sections in
Union Council of Ministers and Planning Commission that state and to organize Lok Adalats for amicable
and Administrations of Union Territories are also its
members. settlement of disputes.

IAS (Pre) GS 2013 Paper I 161 YCT


• Panchyati Raj System, Constitution (c) To create autonomous regions in tribal areas
(d) To free tribal people form exploitation
Amendments and Schedules Ans : (c) The Panchayat Extension Act 1996 was
44. Consider the following statements : brought for Scheduled Areas for Tribal Self
1. National Development Council is an organ of the Governance to protect the traditional rights of tribes
Planning Commission. and ensure their freedom from exploitation. The Act
2. The Economic and Social Planning is kept in the does not include the purpose of creating autonomous
Concurrent List in the Constitution of India. areas in tribal areas.
3. The Constitution of India prescribes that
Panchayats should be assigned the task of ECONOMY
preparation of plans for economic development
and social justice. • Poverty & Unemployment
Which of the statements given above is/are
correct? 48. Disguised unemployment generally means –
(a) 1 only (b) 2 and 3 only (a) large number of people remain unemployed
(c) 1 and 3 only (d) 1, 2 and 3 (b) alternative employment is not available
(c) marginal productivity of labour is zero
Ans : (b) NDC is listed as an advisory body to the
(d) productivity of workers is low
planning commission. Moreover, the plans made by the
planning commission are placed before NDC for its Ans : (c) Disguised unemployment means that more
acceptance. So statement 1 is wrong. people are engaged in a job, which can be performed
with less number of people. In disguised
45. Under the Scheduled Tribes and Other unemployment, the output remains the same even after
Traditional Forest Dwellers (Recognition of the removal of some of the labour. Thus, the marginal
Forest Rights) Act, 2006, who shall be the productivity of labour is zero.
authority to initiate the process for determining
the nature and extent of individual or community • Economic Planning & National
forest rights or both?
(a) State Forest Department
Income/Expenditure
(b) District Collector / Deputy Commissioner 49. A rise in general level of prices may be caused by-
(c) Tahsildar/Block Development Officer/ Mandal 1. an increase in the money supply
Revenue Officer 2. a decrease in the aggregate level of output
(d) Gram Sabha 3. an increase in the effective demand
Ans : (d) Under section 6 of the Forest Right Act, Select the correct answer using the codes given
2006, the Gram Sabha has been given the authority to below :
initiate the process for determining the nature and (a) 1 only (b) 1 and 2 only
extent of these rights. (c) 2 and 3 only (d) 1, 2 and 3
46. Consider the following statements : Ans : (d) Rise or fall in prices is caused due to
changes in supply and demand. An increase in the
1. An amendment to the Constitution of India can
money supply increases the demand and thus,
be initiated by an introduction of a bill in the Lok
increases prices. A decrease in the aggregate level of
Sabha only.
2. If such an amendment seeks to make changes in output will reduce supply and thus, prices increase
the federal character of the Constitution, the again. An increase in effective demand would again
amendment also requires to be ratified by the raise up the prices. Thus, all three events will raise the
legislature of all the States of India. prices.
Which of the statements given above is/are 50. Which one of the following is likely to be the
correct? most inflationary in its effect ?
(a) 1 only (b) 2 only (a) Repayment of public debt
(c) Both 1 and 2 (d) Neither 1 nor 2 (b) Borrowing from the public to finance a budget
Ans : (d) Statement 1 is incorrect. A constitutional deficit
amendment can be initiated in any House of (c) Borrowing from banks to finance a budget
Parliament. Statement 2 is also incorrect. If certain deficit
federal features of the constitution are to be amended, (d) Creating new money to finance a budget deficit
then apart from Parliamentary approval, they require to Ans : (d) Deficit financing always brings inflationary
be ratified by the legislature of one half of the states. pressure whatever be the means but ‘creation’
47. The Government enacted the Panchayat (printing) of new money has the highest inflationary
(Extension to Scheduled Areas) (PESA) Act in impact.
1996. Which one of the following is not identified 51. Supply of money remaining the same when there
as its objective ? is an increase in demand for money, there will be–
(a) To provide self-governance (a) a fall in the level of prices
(b) To recognize traditional rights (b) an increase in the rate of interest
IAS (Pre) GS 2013 Paper I 162 YCT
(c) a decrease in the rate of interest • Agriculture, Industry & Trade
(d) an increase in the level of income and
employment 55. Which one among the following industries is the
Ans : (b) This is a simple supply and demand maximum consumer of water in India ?
question. Since the supply of money is constant and its (a) Engineering (b) Paper and pulp
demand is increasing, there will be an increase in (c) Textiles (d) Thermal power
interest rate. It does not have much to do with the level
Ans : (d) Of the total water used by the industry,
of prices and the increase in the level of income and
thermal power plants are the biggest users of water and
employment.
account for 88% of the total industrial water use. They
Interest Rate
are followed by engineering (5.05%), pulp & paper
(2.26%) and textiles (2.07%) industries.
56. With reference to the usefulness of the by-
products of sugar industry, which of the
following statements is/are correct ?
1. Bagasse can be used as biomass fuel for the
generation of energy.
Money Demand
2. Molasses can be used as one of the feedstocks
52. The national income of a country for a given for the production of synthetic chemical
period is equal to the – fertilizers.
(a) total value of goods and services produced by the 3. Molasses can be used for the production of
nationals. ethanol.
(b) Sum of total consumption and investment Select the correct answer using the codes given
expenditure. below :
(c) sum of personal income of all individuals. (a) 1 only (b) 2 and 3 only
(d) money value of final goods and services (c) 1 and 3 only (d) 1, 2 and 3
produced. Ans : (c) Option 1 : When burned by a Sugar mill,
bagasse, the fibrous material left over after the juice is
Ans: (d) National income means the sum of the
extracted from sugarcane, usually produces enough
monetary value of the final goods and services
electricity to power all of the mills operations.
produced as a result of the economic activities of a
nation in a year. This includes the prices of all final Option 2 : Molasses fertilizer is an organic fertilizer.
goods and services that have been produced by Option 3 : Sugarcane ethanol is an alcohol based fuel
ordinary residents of the country living within or produced by the fermentation of Sugarcane juice and
outside the domestic boundary. It also includes the molasses. Sugarcane ethanol has emerged as a leading
means of income earned from abroad. renewable fuel for the transportation sector.
53. Consider the following Statements: • Money & Banking, Tax System,
1. Inflation benefits the debtors.
2. Inflation benefits the bond-holders. Financial Relations of Center and States
Which of the statements given above is/are 57. Priority Sector Lending by banks in India
correct? constitutes the Lending to –
(a) 1 only (b) 2 only
(a) agriculture
(c) 1 and 2 both (d) Neither 1 nor 2
(b) micro and small enterprises
Ans : (a) For debtors,the "real" interest rate goes down (c) weaker sections
with inflation. Thus it benefits them. On the other (d) All of the above
hand, with inflation, the yield of the bond goes down, Ans : (d) According to the priority sector, lending
thus a negative effect for bondholders. norms of the RBI banks have to extend at least 40% of
54. In India, deficit financing is used for raising, their credit to the priority sector. Some of the priority
resources for – sectors mentioned by the RBI are agriculture, micro
(a) economic development and small enterprises, education, housing, exports,
(b) redemption of public debt credit & loans to weaker sections.
(c) adjusting the balance of payments 58. Economic growth in country X will necessarily
(d) reducing the foreign debt have to occur if –
(a) there is technical progress in the world economy.
Ans : (a) India is a developing county. For the (b) there is population growth in X.
development of infrastructure here, the government (c) there is capital formation in X.
takes the help of deficit financing so that economic (d) The volume of trade grows in the world
development can be made possible. economy.
IAS (Pre) GS 2013 Paper I 163 YCT
Ans : (c) a & d statement does not say anything about 62. In the context of Indian economy, 'Open Market
country X, only about the general world economy. In b, Operations' refers to
if there is population growth without any avenues for the (a) borrowing by scheduled banks from the RBI
economy to absorb them, it will have a deteriorating (b) lending by commercial banks to industry and
effect. C is the most appropriate option as whenever trade
there is capital formation in country, it will lead to (c) purchase and sales of government securities by
growth. the RBI
(d) None of the above
59. Which of the following grants / grant direct
credit assistance to rural households ? Ans : (c) The repo and reverse repo rates are used
1. Regional Rural Banks when RBI purchases or sells government securities.
2. National Bank for Agriculture and Rural The open market operations of RBI refer to the sale
and purchase of government securities by the RBI in
Development
the open market. Most central banks use this as their
3. Land Development Banks
primary tool of monetary policy. The sale of
Select the correct answer using the codes given
government securities by the RBI leads to an increase
below :
in interest rates and contraction in liquidity, while the
(a) 1 and 2 only (b) 2 only
purchase of government securities by the RBI leads to
(c) 1 and 3 only (d) 1, 2 and 3
a fall in interest rates and an expansion of liquidity.
Ans : (c) NABARD do not provide direct assistance. It
"refinance" the assistance by other institutions. 63. The Reserve Bank of India regulates the
commercial banks in matters of
60. Which of the following constitute Capital 1. liquidity of assets 2. branch expansion
Account? 3. merger of banks 4. winding-up of banks
1. Foreign Loans Select the correct answer using the codes given
2. Foreign Direct Investment below :
3. Private Remittances (a) 1 and 4 only (b) 2, 3 and 4 only
4. Portfolio Investment (c) 1, 2 and 3 only (d) 1, 2, 3 and 4
Select the correct answer using the codes given Ans : (d) The Reserve Bank of India Controls
below: Commercial banks as follows–
(a) 1,2 and 3 (b) 1,2 and 4 • By Liquidity of assets
(c) 2,3 and 4 (d) 1, 3 and 4
• By Merger of Banks
Ans : (b) Capital account is related to financial • By Branch expansion
transactions. It includes the following facts– • By Winding up of banks
• Short and long term international capital transfers.
• Exchange of gold. 64. An increase in the Bank Rate generally indicates
• Private payments. that the –
• Payments and receipts related to national (a) market rate of interest is likely to fall
institutions. (b) Central Bank is no longer making loans to
commercial banks
• Government Loans, interest, grants etc., are
(c) Central Bank is following an easy money policy
included.
(d) Central Bank is following a tight money policy
• All the transactions in the Capital Accounts are
Ans : (d) Increase in Bank Rate means that interest
only related to financial transfers, so it has no direct
rates will rise. This means that money becomes dearer
effect on the production, income and employment
and borrowing becomes more expensive. This
of the country. squeezes out liquidity from the economy and so, it can
61. Consider the following liquid assets : be said that RBI is following a tight money policy.
1. Demand deposits with the banks.
2. Time deposits with the banks. • Foreign Trade of India, Economic
3. Savings deposits with the bank. Organizations & Stock Market
4. Currency 65. Which one of the following groups of items is
The correct sequence of these assets in the included in India’s foreign-exchange reserves ?
decreasing order of liquidity is (a) Foreign-currency assets, Special Drawing Rights
(a) 1-4-3-2 (b) 4-3-2-1 (SDRs) and loans from foreign countries.
(c) 2-3-1-4 (d) 4-1-3-2 (b) Foreign-currency assets, gold holdings of the
Ans : (d) Liquidity of currency is highest. Next will RBI and SDRs.
come demand liabilities as they have to be cleared (c) Foreign-currency assets, loans from the World
when demanded.The next is savings and least liquidity Bank and SDRs.
will be of time deposits. [Currency > Demand (d) Foreign-currency assets, gold holdings of the
Deposits > Saving deposits > Time deposits] RBI and loans from the World Bank
IAS (Pre) GS 2013 Paper I 164 YCT
Ans : (b) India's foreign exchange reserves consist of 3. Moon being visible at dawn
the foreign-currency assets, gold, special drawing 4. Twinkle of stars in the sky
rights and the reserve tranche position in the 5. Polestar being visible in the sky
Which of the above are optical illusions ?
International Monetary Fund. It does not consist of the (a) 1, 2 and 3 (b) 3, 4 and 5
loans from World Bank or from foreign countries. (c) 1, 2 and 4 (d) 2, 3 and 5
66. The balance of payments of a country is a Ans : (c) The following are the optical illusions-
systematic record of – • Shimmer of the stars.
(a) all import and export transactions of a country • The red appearance of the sun at sunrise and sunet, it
during a given period of time, normally a year happen due to the refraction of light.
(b) goods exported from a country during a year • While, polestar and moon are visible at dawn in the
(c) economic transaction between the government of actual (real) scenario.
one country to another
(d) capital movements from one country to another 70. Rainbow is produced when sunlight falls on
Ans : (a) The Balance of payment (BoP) record the drops of rain. Which of the following physical
transactions in goods, services and assets between phenomena are responsible for this ?
residents of a country with the rest of the world for a 1. Dispersion
specified time period, typically a year. So the best 2. Refraction
option is A. 3. Internal reflection
Select the correct answer using the codes given
SCIENCE below
(a) 1 and 2 only (b) 2 and 3 only
• Physics (c) 1 and 3 (d) 1,2 and 3
67. Ball bearings are used in bicycles, cars, etc. Ans : (d) Option–1, Each individual droplet of water
because – acts as a tiny prism that both disperses the light and
(a) The actual area of contact between the wheel and reflects it back to our eye.
axle is increased Option–2, Since water is more denser than air, lights is
(b) The effective area of contact between the wheel refracted as it enters the drop.
and axle is increased Option–3, Some of the light will reflect off the back of
(c) The effective area of contact between the wheel the drop if the angle is larger than the critical angle
and axle is reduced (48º for water).
(d) None of the above statements is correct 71. The known forces of nature can be divided into
Ans : (c) The roundness of the bearing creates less four classes, viz., gravity, electromagnetism, weak
contact between the bearing that is trying to move nuclear force and strong nuclear force. With
objects and the objects being moved. A reduction in reference to them, which one of the following
contact area means less friction. statements is not correct ?
68. Consider the following : (a) Gravity is the strongest of the four
1. Electromagnetic radiation (b) Electromagnetism acts only on particles with an
2. Geothermal energy electric charge
3. Gravitational forces (c) Weak nuclear force causes radioactivity
4. Plate movements (d) Strong nuclear force holds protons and neutrons
inside the nucleus of an atom
5. Rotation of the earth
6. Revolution of the earth Ans : (a) The known forces of nature are divided into
four classes. These forces are gravity force,
Which of the above are responsible for bringing electromagnetism force, weak nuclear force and strong
dynamic changes on the surface of the earth ? nuclear force. The decreasing order of these forces is
(a) 1, 2,3, and 4 (b) 1,3,5 and 6 only as follows-strong nuclear force > electromagnetism
(c) 2,4,5 and 6 only (d) 1, 2, 3, 4, 5 and 6 force > weak nuclear force > Gravitational force.
Ans : (d) Endogenous forces have a role in bringing 72. The efforts to detect the existence of Higgs boson
dynamic changes to the surface of the earth. particle have become frequent news in the recent
Endogenous forces are generated due to thermal past. What is / are the importance/ importances
asymmetry in the interior of the Earth and contraction of discovering this particle ?
and expansion of rocks. Electromagnetic radiation, the 1. It will enable us to understand as to why
gravitational force also has a role. The rotation, elementary particles have mass.
revolution and geothermal energy of the Earth is also a 2. It will enable us in the near future to develop the
reason to bring in dynamic changes on the Earth's technology of transferring matter from one point
surface. to another without traversing the physical space
69. Consider the following phenomena : between them.
1. Size of the sun at dusk 3. It will enable us to create better fuels for nuclear
2. Colour of the sun at dawn fission.
IAS (Pre) GS 2013 Paper I 165 YCT
Select the correct answer using the codes given Select the correct answer using the codes given
below. below :
(a) 1 only (b) 2 and 3 only (a) 1 only (b) 2 and 3 only
(c) 1 and 3 only (d) 1, 2 and 3 (c) 1 and 3 only (d) 1, 2 and 3
Ans : (a) Higgs Boson is a particle postulated by the Ans : (b) AIDS and hepatitis B are diseases that are
standard model of physics to provide mass to all transmitted from person to person through tattooing or
particles in the universe. Discovery of the Higgs Boson infection. On the other hand, chikungunya is a viral
will help us to further understand why elementary disease and is spread by mosquito bites. Its carrier is
particles have mass. mosquito.
73. During a thunderstorm, the thunder in the skies 76. Consider the following minerals :
is produced by the 1. Calcium
1. meeting of cumulonimbus clouds in the sky
2. Iron
2. lightning that separates the nimbus clouds
3. Sodium
3.violent upward movement of air and water
particles. Which of the minerals given above is /are
Select the correct answer using the codes given required by human body for the contraction of
below : muscles ?
(a) 1 only (a) 1 only (b) 2 and 3 only
(b) 2 and 3 (c) 1 and 3 only (d) 1, 2 and 3
(c) 1 and 3 Ans : (d) Calcium : Calcium is essential for activating
(d) None of the above produces the thunder enzymes that cause muscle contraction.
Ans : (d) Thunderstorm is an intense local Iron : Iron helps support continued muscle function by
thunderstorm, with wide and dense rainy cumulus supporting energy production in your muscle cells so
clouds and a strong wind moving from the bottom to that muscles fibres always have the energy they need
the top. A thunderstorm often arises due to to contract properly. Iron also helps muscles store
atmospheric instability, i.e. low pressure and intense oxygen to fuel muscle contractions & promotes
convective airflow. In fact, thunderstorms are local healthy circulation so that muscles can get additional
storms in which the winds blow upward at high speeds, oxygen from the bloodstream.
accompanied by lightning and thunder clouds and also
Sodium : One need sodium for muscle contraction
heavy rain.
because it balances potassium to maintain membrane
• Medical Science (Zoology) potential.
74. Which of the following statements is/are correct ? 77. Recombinant DNA technology (Genetic
1. Viruses lack enzymes necessary for the generation of Engineering) allows genes to be transferred
energy 1. across different species of plants
2. Viruses can be cultured in any synthetic medium 2. from animals to plants
3 Viruses are transmitted from one organism to 3. from microorganisms to higher organisms
another by biological vectors only. Select the correct answer using the codes given
Select the correct answer using the codes given below
below :
(a) 1 only (b) 2 and 3 only
(a) 1 only (b) 2 and 3 only
(c) 1 and 3 only (d) 1, 2 and 3 (c) 1 and 3 only (d) 1, 2 and 3
Ans : (a) Statement 1 is correct and viruses do not have Ans : (d) Recombinant DNA are DNA sequences,
the capacity to survive on their own and they need a which result from bringing genetic material from
living host for survival. Statement 2 is incorrect; viruses different sources. Here the genes can be transferred
cannot grow in a synthetic medium. Statement 3 is between any species, that is, across different species of
incorrect because viruses enter the body from the plants from animals to plants and for microorganisms
environment or other individuals from soil to water or to higher organisms. Thus, all three options are correct.
air via nose, mouth, or any cracks in the skin and seek a 78. Consider the following animals :
cell to infect.
1. Sea cow
75. Which of the following diseases can be
2. Sea horse
transmitted from one person to another through
tattooing? 3. Sea lion
1. Chikungunya Which of the above is/are mammal/ mammals ?
2. Hepatitis B (a) 1 only (b) 1 and 3 only
3. HIV-AIDS (c) 2 and 3 only (d) 1, 2 and 3

IAS (Pre) GS 2013 Paper I 166 YCT


Ans : (b) Mammals are a class of animals, which are (a) bacteria (b) Protozoa
differentiated from other classes of animals such a (c) moulds (d) viruses
reptile, birds, fish, etc. by the presence of hair, three Ans: (c) Aflatoxins are naturally occurring
middle ear bones, mammary glands in females, among mycotoxins, which are produced by many species of
Aspergillus, a fungus. The fungus can be considered a
others. While Sea cow (also known as Dugong) and type of mould. Aflatoxins are toxic and are among the
Sea lion are examples of mammals, Sea horse is a most carcinogenic substances known.
vertebrate fish. 83. Which of the following adds/ add nitrogen to the
• Agriculture, Animal Husbandry, Dairy soil ?
1. Excretion of urea by animals
79. Fruits stored in a cold chamber exhibit longer 2. Burning of coal by man
storage life because – 3. Death of vegetation
(a) exposure to sunlight is prevented Select the correct answer using the codes given
below :
(b) concentration of carbon dioxide in the (a) 1 only (b) 2 and 3 only
environment is increased (c) 1 and 3 only (d) 1, 2 and 3
(c) rate of respiration is decreased Ans : (c) Waste products of animals, as well as dead
(d) there is an increase in humidity remains of organism, are converted into inorganic
Ans : (c) The stored fruits in the cold cell last longer ammonia by bacteria in the process of ammonification.
because the rate of respiration decreases as This ammonia is then converted into nitrate then
temperature drops, which reduces the decaying nitrites by Nitrosomonas and Nitrococcus bacteria,
respectively. Another soil bacteria Nitrobacter can
capacity and therefore the, food in the refrigerator does convert nitrate into nitrite. In this way, the process of
not spoil quickly. nitrogen cycle continues and the amount of the
80. Consider the following crops : nitrogen in the soil is replenished. Statement 2 is
wrong as the burning coal does not produce nitrogen.
1. Cotton 2. Groundnut
3. Rice 4. Wheat • Science & Technology
Which of these are Kharif crops ? 84. Mycorrhizal biotechnology has been used in
(a) 1 and 4 (b) 2 and 3 only rehabilitating degraded sites because mycorrhiza
(c) 1, 2 and 3 (d) 2, 3 and 4 enables the plants to –
1. resist drought and increase absorptive area
Ans : (c) In India, the cultivation seasons are divided 2. tolerate extremes of pH
into Kharif and Rabi. Wheat is a Rabi crop since it 3. resist disease infestation
requires a lower temperature. All the other three are Select the correct answer using the codes given
Kharif crops. Kharif Crops: Paddy (Rice) Maize, below :
Groundnut, Sugarcane, Horse grain, Jute, Flax, Cotton (a) 1 only (b) 2 and 3 only
(c) 1 and 3 only (d) 1, 2 and 3
etc.
Rabi Crops: -Wheat , Barley, Oats, tauria (lahi) and Ans : (d) Mycorrhizal biotechnology is used in the
rehabilitation of degraded sites. This gives plants the
mustard, Sun Flower etc. ability to resists drought and increases absorptive area.
81. Many transplanted seedlings do not grow because Also, the ability to tolerate pH extremes and resistance
to disease increases.
(a) the new soil does not contain favourable
minerals 85. Consider the following organisms :
1. Agaricus
(b) most of the root hairs grip the new soil too 2. Nostoc
hard 3. Spirogyra
(c) most of the root hairs are lost during Which of the above is /are used as bio
transplantation fertilizer/bio –fertilizers ?.
(d) leaves get damaged during transplantation (a) 1 and 2 (b) 2 only
(c) 2 and 3 (d) 3 only
Ans : (c) Newly transplanted trees or shrubs
experience some degree of transplant shock. This is Ans : (b) Bioinoculants are the microbial preparation
that enhances the uptake of nutrients by plants from
simply the plant reacting to having been moved. It is the soil, increase the availability of the nutrient and
generally caused by damage to the roots of the plants stimulate plant growth.
during transplant. While the thickest roots are nearest There are two types of organisms that are used as
to the root ball, the most important roots, those that inoculants:
actually "do the work", are located the farthest away (i) Symbolic organisms (such as Rhizobium,
from the plant. These tiny roots are covered by even Synorhizobium, Bradyrhizobium, Nostoc, Anabaena
tinier hairs which absorb most of the water that is etc..) and
eventually carried to the top growth of the plant. (ii) Non-Symbiotic organism (such as Azoto-bacter,
Azospirillum, Beijerinckia, etc.) Agaricus is a genus
82. Improper handling and storage of cereal grains of mushrooms, both edible and poisonous species.
and oilseeds result in the production of toxins Spirogyra is a genus of filamentous algae of the order
known as aflatoxins which are not generally of zygnematales, named for the helical or spiral
destroyed by normal cooking process. Aflatoxins arrangement of the chloroplasts that is diagnostic of
are produced by the genus.
IAS (Pre) GS 2013 Paper I 167 YCT
ENVIRONMENT & ECOLOGY Ans : (b) Beryllium is used in Circuits board, Hard
disk drive, Motherboard, wiring board and connectors.
86. With reference to the food chains in ecosystems. So option (c) is wrong.
Which of the following kinds. of organism is/ are Mercury is a key part of some modern technologies,
known as decomposer organism/organisms ? including LCD Screens, Laptops and Computers, so
1. Virus 2. Fungi 3. Bacteria option (A) is also wrong.
Select the correct answer using the codes given Heptachlor is an organochlorine compound that was
below : used as an insecticide. So option (d) is also wrong. So,
(a) 1 only (b) 2 and 3 only the correct answer is (b).
(c) 1 and 3 only (d) 1, 2 and 3 90. Acid rain is caused by the pollution of
Ans : (b) Viruses are not decomposers. They are like environment by
parasites and live off on a living organism. Fungi and (a) carbon dioxide and nitrogen
Bacteria, on the other hand, are decomposers and (b) carbon monoxide and carbon dioxide
(c) ozone and carbon dioxide
perform the important role of breaking down the dead
(d) nitrous oxide and sulphur dioxide
remains of plants and animals into nutrients.
Ans : (d) Nitrous acid and sulphur dioxide are mainly
87. In the grasslands, trees do not replace the grasses responsible for acid rain. Nitrogen and sulphur oxides
as a part of an ecological succession because of react with water to make ammonium nitrate and sulfuric
(a) insects and fungi acid, respectively and they cause damage in the form of
(b) limited sunlight and paucity of nutrients rain.
(c) water limits and fire 91. With reference to food chain in ecosystems,
(d) None of the above consider the following statements :
Ans : (c) In a grassland, trees have already been 1. A food chain illustrates the order in which a
cleared away due to fires in the past, which also chain of organisms feed upon each other.
changes the ground composition. Without the trees to 2. Food chain are found within the populations of a
hold them, water washes away most of the soil and the species.
underlying ground becomes rocky and barren. After 3. A food chain illustrates the numbers of each
this, lichens become the pioneer population, which is organism which are eaten by others.
then replaced by grasses. The grasses, however absorb Which of the statements given above is/are
most of the water before it reaches the roots of the correct?
shrubs. Thus, the ecological succession does not occur (a) 1 only (b) 1 and 2 only
beyond the grasses. (c) 1, 2 and 3 (d) None
Ans : (a) A food chain is a sequence of who eats whom
88. Consider the following fauna of India : in a biological community (an ecosystem) to obtain
1. Gharial 2. Leatherback turtle nutrition.
3. Swamp deer
92. Consider the following pairs :
Which of the above is/are endangered ?
National Park River flowing
(a) 1 and 2 only (b) 3 only through the Park
(c) 1, 2 and 3 (d) None 1. Corbett National Park – Ganga
Ans : (c) In Indian animals, Gharial have been declared 2. Kaziranga National Park – Manas
as endangered species by the IUCN. The world's most 3. Silent Valley National Park – Kaveri
endangered Sea turtle is a leatherback turtle found near Which of the above pairs is/are correctly
Nicobar Island and Swamp deer found in Central and matched ?
Northern India is also an endangered species. Hence the (a) 1 and 2 (b) 3 only
correct answer will be (c). (c) 1 and 3 (d) None
89. Due to improper/ indiscriminate disposal of old Ans : (d) Ramganga, a tributary of Ganga, passes
and used computers or their parts, which of the through Corbett National Park. While silent valley
following are released into the environment as e- National Park is located in the state of Kerala, the
Kunthipuzha river flows through this park and the
waste ?
Brahmaputra flows through Kaziranga National Park
1. Beryllium 2. Cadmium (Assam).
3. Chromium 4. Heptachlor
5. Mercury 6. Lead 93. In which of the following States is lion-tailed
7. Plutonium macaque found in its natural habitat ?
Select the correct answer using the codes given 1. Tamil Nadu 2. Kerala
below 3. Karnataka 4. Andhra Pradesh
(a) 1, 3, 4, 6 and 7 only Select the correct answer using the codes given
(b) 1, 2, 3, 5 and 6 only below :
(c) 2, 4, 5 and 7 only (a) 1, 2 and 3 (b) 2 only
(d) 1, 2, 3, 4, 5, 6 and 7 (c) 1, 3 and 4 (d) 1, 2, 3 and 4

IAS (Pre) GS 2013 Paper I 168 YCT


Ans : (a) Lion-tailed macaque is found in the Western (c) Mangroves, grasslands, lakes, oceans
Ghats of India. It is an endangered species, which is (d) Oceans, mangroves, lakes, grasslands
found mostly in Kerala, Karnataka and the border Ans : (c) In order of decreasing productivity of an
regions of Tamil Nadu. It is not found in Andhra ecosystem, their correct sequence is – Mangrove >
Pradesh. grassland > lakes > oceans.
94. Which one of the following terms describes not 98. Which of the following can be found as pollutants
only the physical space occupied by an organism, in the drinking water in some parts of India ?
but also its functional role in the community of 1. Arsenic 2. Sorbitol
organisms ? 3. Fluoride 4. Formaldehyde
(a) Ecotone (b) Ecological niche 5. Uranium
(c) Habitat (d) Home range Select the correct answer using the codes given
Ans : (b) In nature, many species occupy the same below:
habitat, but they perform different functions. The (a) 1 and 3 only (b) 2,4 and 5 only
functional characteristics of a species in its habitat are (c) 1,3 and 5 (d) 1, 2, 3, 4 and 5
referred to as 'niche' in that common habitat. Ans : (c) Uranium has been found in the water of
Ecological niche refers to both the ecological role and Punjab and Fluoride in the water of Andhra Pradesh,
the space that an organism occupies in the community Gujarat, Haryana, Odisha, Punjab etc. Arsenic has been
of organisms. found in Haryana and West Bengal, among others.
Sorbitol, on the other hand, is a bulk sweetener found in
95. Photochemical smog is a resultant of the reaction food products. Formaldehyde is an organic compound,
among – which is toxic to human beings. It is a gas at room
(a) NO2, O3 and peroxyacetyl nitrate in the presence temperature. It is not a water pollutant in India.
of sunlight
99. Contour bunding is a method of soil conservation
(b) CO, O2 and peroxyacetyl nitrate in the presence used in –
of sunlight (a) desert margins, liable to strong wind action
(c) CO, CO2 and NO2 at low temperature (b) low flat plains, close to stream courses, liable to
(d) high concentration of NO2 O3 and CO in the flooding
evening (c) scrublands, liable to spread of weed growth
(d) None of the above
Ans : (a) Major chemical pollutant in photochemical
Ans : (d) Contour bunding (or contour bundling),
smog :
involves the placement of lines of stones along with
(i) Nitrogen oxides (NO and NO2) the natural rises of a landscape. It helps to capture and
(ii) Volatile Organic Compounds (VOC5) hold rainfall before it can become runoff which
(iii) Ozone (O3) reduces soil erosion and retards overall water loss.
(iv) Peroxyacetyl Nitrates (PAN)
MISCELLANEOUS
96. Consider the following :
1. Star tortoise 2. Monitor lizard 100. In the context of cultural history of India, a pose
in dance and dramatics called Tribhanga’ has
3. Pygmy hog 4. Spider monkey been a favorite of Indian artists from ancient
Which of the above are naturally found in India? times till today. Which one of the following
(a) 1, 2 and 3 only (b) 2 and 3 only statements best describes this pose ?
(c) 1 and 4 only (d) 1, 2, 3 and 4 (a) One leg is bent and the body is slightly but
oppositely curved at waist and neck
Ans : (a) Spider monkeys live in tropical climates, (b) Facial expressions, hand gestures and make up
specifically the evergreen forests of Central and South are combined to symbolize certain epic or
America. They are not found in India. Star tortoise is historic characters
found in India in the dry and scrab forests. Pygmy Hog (c) Movements of body, face and hands are used to
is an endangered species found in Assam. Monitor express oneself or to tell a story
(d) A little smile, slightly curved waist and certain
Lizard is found in India, Sri Lanka and Pakistan.
hand gestures are emphasized to express the
97. Which one of the following is the correct feelings of love or eroticism
sequence of ecosystems in the order of decreasing Ans : (a) Tribhanga literally meaning three parts
productivity ? break, consists of three bends in the body, at the neck,
(a) Oceans, lakes, grasslands mangroves waist and knee; hence the body is oppositely curved at
(b) Mangroves, oceans, grasslands, lakes waist and neck, which gives it a gentle 'S' shape.
IAS (Pre) GS 2013 Paper I 169 YCT
UNION PUBLIC SERVICE COMMISSION
Civil Services (Preliminary Exam) - 2012
GENERAL STUDIES : PAPER-I
Time : 2 hours (Exam date : 20.05.2012) Maximum Number : 200

ANCIENT HISTORY • Buddhism, Jainism, Bhagavata, Shaiva


and Other Religion
• Pre-Historic Age 3. Lord Buddha’s image is sometimes shown with
1. With reference to the scientific progress of the hand gesture called ‘Bhumisparsha Mudra’ It
symbolizes.
ancient India, which of the statements given
(a) Buddha’s calling of the Earth to watch over
below are correct? Mara and to prevent Mara from disturbing his
1. Different kinds of specialized surgical instruments meditation.
were in common use by 1st century AD. (b) Buddha’s calling of the Earth to witness his
2. Transplant of Internal organs in the human body purity and chastity despite the temptation of
had begun by the beginning of 3rd century AD. Mara.
3. The concept of sine of angle was known in 5th (c) Buddha’s reminder to his followers that they all
century AD. arise from the Earth and finally dissolve into the
4. The concept of cyclic quadrilaterals was known in Earth, and thus this life is transitory.
7th century AD. (d) Both the statements (a) and (b) are correct in this
context.
Select the correct answer using the codes given
below : Ans. (b): The 'Bhumisparsha Mudra' is one of the most
(a) 1 and 2 only (b) 3 and 4 only common iconic images of Buddhism. It depicts the
Buddha sitting in meditation with his left hand, palm
(c) 1, 3 and 4 only (d) 1, 2, 3 and 4 upright, in his lap, and his right hand touching the
Ans. (c): There was significant progress in India in earth. This represents the moment of the Buddha's
ancient times. Various sources suggest that by the first enlightenment.
century AD, a variety of surgical instruments were in 4. With reference to the history of ancient India,
use, but organ transplantation was not possible until which of the following was/were common to both
the third century AD. By the 5th century AD, in the Buddhism and Jainism?
area of mathematics in the Gupta period, the principles 1. Avoidance of extremities of penance and
of sine, cosine and inverse sine were known in India. enjoyment.
They are mentioned in 'Surya Siddhanta' and 2. Indifference to the authority of the Vedas.
'Aryabhatiya'. In the seventh century CE, the theory of 3. Denial of efficacy of rituals.
the cyclic quadrilateral was known by Brahmagupta. Select the correct answer using the codes given
below:
• Vedic Civilization (a) 1 only (b) 2 and 3 only
(c) 1and 3 only (d) 1, 2 and 3
2. The religion of early Vedic Aryans was primarily
Ans. (b): The avoidance of excess asceticism and
of : indulgence and following the middle path is found
(a) Bhakti. only in Buddhism. While ideologies such as
(b) Image worship and Yajnas. disobedience to the authenticity of the Vedas,
(c) Worship of nature and Yajnas. prohibitions of the virtue of rituals and prohibitions of
(d) Worship of nature and Bhakti. violence of beings, etc., are found in both Buddhism
and Jaina religions alike.
Ans. (c): Vedic deities were predominantly Indra,
Agni (fire), Soma, Mitra, Varuna, Ushas, Aditi & MEDIEVAL HISTORY
Rudra. Aryans used to worship nature deities such as
Surya (Sun), Vayu (Wind) and Prithvi (Earth). Rivers, • Vijayanagara Empire and Bahmanids
especially Saraswati, were worshipped as the goddess. 5. The Nagara, the Dravida and the Vesara are the
Yajna was the mode of worship, which was the (a) three main racial group of the Indian
performance of sacrifices. It included vedic chants, subcontinent.
samans singing and sacrificial mantra i.e, yajus. Yajna (b) three main linguistic division into which the
involved sacrificing havan samagri in the fire while languages of India can be classified.
(c) three main styles of Indian temple architecture.
chanting the vedic mantra. (d) three main musical Gharanas prevalent in India.
IAS (Pre) GS 2012 Paper I 170 YCT
Ans. (c) : Ancient Indian temples are classified into Ans. (b) : Rowlatt Act passed on the recommendations
three broad types. This classification is based on of the Rowlatt Committee and named after its
different architectural styles employed in the president, British Judge Sir Sidney Rowlatt. This Act
construction of these temples. The three types are effectively authorized the government to imprison any
Nagara, Vesara and Dravida. Nagara style temples are person suspected of terrorism living in British India for
found in Northern India, Vesara style temples in the upto two years without a trial and gave the colonial
region between Vindhyas and the river Krishna and authorities power to deal with all revolutionary
Dravida style temples are found between Godavari and activities.
Krishna rivers.
• Social, Cultural Awakening, Lower
• Bhakti Movement
Caste, Trade Unions and Peasant
6. With reference to the religious history of Movements
medieval India, the Sufi mystics were known to
pursue which of the following practices? 9. The Lahore Session of the Indian National
1. Meditation and control of breath. Congress (1929) is very important in history,
2. Severe ascetic exercises in a lonely place. because-
3. Recitation of holy songs to arouse a state of 1. the Congress passed a resolution demanding
ecstasy in their audience. complete independence.
Select the correct answer using the codes given 2. the rift between the extremists and moderates was
below : resolved in that Session.
(a) 1 and 2 only (b) 2 and 3 only 3. a resolution was passed rejecting the two-nation
(c) 3 only (d) 1, 2 and 3
theory in that Session.
Ans. (d): The original Sufis were mystic people who Which of the statements given above is/are correct?
followed a pious form of Islam believed that God’s (a) Only 1 (b) 2 and 3
direct personal experience could be achieved through (c) 1 and 3 (d) None of the above
meditation and self-discipline. Sufi mysticism
endeavored to produce a personal experience of the Ans. (a): The 1929 Lahore Session under the
divine through mystic and ascetic discipline. presidency of Jawaharlal Nehru holds special
significance as in this session, "Purna Swaraj"
MODERN HISTORY (complete independence) was declared as the goal of
the Indian National Congress. 26th January 1930 was
• Land Revenue Systems in British India declared as "Purna Swaraj Diwas".
10. Which of the following statements is/are correct
7. With reference of Ryotwari Settlement, consider
regarding Brahma Samaj.
the following statements :
1. It opposed idolatry.
1. The rent was paid directly by the peasants to the
2. It denied the need for a priestly class for
Government.
interpreting the religious texts.
2. The Government gave Pattas to the Ryots.
3. It popularized the doctrine that the Vedas are
3. The lands were surveyed and assessed before
infallible.
being taxed.
Select the correct answer using the codes given
Which of the statements given above is/are correct?
below:
(a) 1 only (b) 1 and 2 only
(a) 1 only (b) 1 and 2 only
(c) 1, 2 and 3 (d) None
(c) 3 only (d) 1, 2 and 3
Ans. (c) : Under the Ryotwari system, every registered
Ans. (b): Brahmo Samaj and the majority of
holder of land was recognized as its proprietor and
contemporary social reform organisations were
paid directly to the government. The registered
Vedantists in their philosophy. They were opposed to
agreement called Pattas were given to the Ryots to
idolatry and priestly class and Brahmo samaj spread
recognize their ownership rights. The lands were
the religion treasure and Kachings of Vedas but never
surveyed and assessed before being taxed.
claimed that Vedas are infallible.
• Revolt of 1857 and Mass Movement of 11. The Congress ministries resigned in the seven
19th Century provinces in 1939, because-
(a) The congress could not form ministries in the
8. The Rowlatt Act aimed at - other four provinces.
(a) compulsory economic support to war efforts. (b) Emergence of a ‘left wing’ in the Congress
(b) imprisonment without trial and summary made the working of the ministries impossible.
procedures for trial. (c) There were widespread communal disturbances
(c) suppression of the Khilafat Movement . in their provinces.
(d) imposition of restrictions on freedom of the (d) None of the statements (a), (b) and (c) given
press. above is correct.
IAS (Pre) GS 2012 Paper I 171 YCT
Ans. (d) : Option (d) is correct in the given Which of the statements given above is /are correct?
alternatives. The Congress ministers resigned in (a) 1 only (b) 2 and 3 only
October 1939, in protest against Viceroy Lord (c) 1 and 3 only (d) 1, 2 and 3
Linlithgow’s action of declaring India to be a Ans. (a) : Dadabhai Naoroji's most effective
belligerent in the Second World War (started in contribution to the Indian National Movement was that
September 1939) without consulting the Indian people. he exposed the fact that Britain was exploiting India
economically. Dadabhai Naoroji's unveiled the
12. Which of the following parties were established exploitative policies of the British and said that these
by Dr. B.R. Ambedkar? policies engaged in plundering India day by day, due
1. The Peasants and Workers Party of India. to which India is becoming a poor country. In his book
2. All India Scheduled Castes Federation. “Poverty and Un-British Rule in India” (1901), he
3. The Independent Labour Party. proved his articles and points with facts.
Select the correct answer using the codes given 15. During Indian freedom struggle, the National
below: Social Conference was formed. What was the
(a) 1 and 2 only (b) 2 and 3 only reason for its formation?
(c) 1 and 3 only (d) 1, 2 and 3 (a) Different social reform groups or organizations
Ans. (b): B.R. Ambedkar formed the Scheduled Caste of Bengal region united to form a single body to
Federation in 1942 as an All-India Party. The discuss the issues of larger interest and to
Independent Labour Party was also founded by B.R. prepare appropriate petitions/representations to
Ambedkar in 1936. The peasants and workers party of the government.
India (Bharatiya Kisan Mazdoor Dal) was founded in (b) Indian National Congress did not want to
Maharastra in 1948 by members Nana Patil, Datta include social reforms in its deliberations and
Deshmukh, Keshavrao Jedhe, Madhavrao Bagal, etc.. decided to form a separate body for such a
purpose.
• Major Institutions, Treaties, (c) Behramji Malabar and M.G. Ranade decided to
bring together all the social reform groups of the
Commissions, Acts, Pacts country under one organization.
13. Which of the following is/are the principal (d) None of the statements (a), (b) and (c) given
feature (s) of the Government of India Act, 1919? above is correct in this context.
1. Introduction of diarchy in the executive Ans. (b) : The National Social Conference was formed
government of India provinces. in the year 1887. The main reason for its formation
2. Introduction of separate communal electorates for was that the Indian National Congress did not want to
Muslims. put social programs in its programs in the initial phase,
3. Devolution of legislative authority by the centre so it suggested to form a separate organization for the
to the provinces. purpose presented.
Select the correct answer using the codes given 16. Mahatma Gandhi undertook fast unto death in
below: 1932, mainly because-
(a) 1 only (b) 2 and 3 only (a) Round Table Conference failed to satisfy Indian
(c) 1 and 3 only (d) 1, 2 and 3 political aspirations.
Ans. (c) The Act provided a dual form of government (b) Congress and Muslim League had differences of
(dyarchy) for the major provinces. In each such opinion.
province, control of some areas of government, (the (c) Ramsay Macdonald announced the Communal
“transferred list”), were given to a Government of Award.
ministers answerable to the Provincial Council. The (d) None of the statements (a), (b) and (c) given
transferred list included agriculture, supervision of the above is correct in this context.
local government, health and education. Ans. (c): On 17th August 1932, Ramsay MacDonald
All other areas of government (reserved list) remained had announced his communal award, reserving seats
under the control of the Viceroy. The ‘Reserved List’ for the "Depressed Classes" as well as Muslims and
included defence, foreign affairs and communications. Sikhs. On the very next day, Gandhi wrote to
MacDonald- "I have to resist your decision with my
• Freedom Struggle and National life". He declared fast unto death against the
Communal Award.
Movement
14. Consider the following statements: INDIAN GEOGRAPHY
The most effective contribution made by
Dadabhai Naoroji to the cause of Indian National • Physical Structure/Drainage System/
Movement was that he- Human Geography/Climate
1. Exposed the economic exploitation of India by
the British. 17. Consider the following statements:
2. Interpreted the ancient Indian texts and restored 1. The duration of the monsoon decreases from
the self-confidence of Indians. southern India to northern India.
3. Stressed the need for eradication of all the social 2. The amount of annual rainfall in the northern
evils before anything else. plains of India decreases from east to west.
IAS (Pre) GS 2012 Paper I 172 YCT
Which of the statements given above is/are Ans. (d): Young fold mountains are rising under the
correct? influence of the earth's tectonic forces. They have a
(a) 1 only (b) 2 only variety of rock structures, deep gorges and high
(c) Both 1 and 2 (d) Neither 1 nor 2 pyramidal peaks. In the high Himalayas, the rivers
Ans. (c): Both given statements are correct. The have steep gradients, which result from the differential
duration of the monsoon decreases from southern India uplift of the high Himalayas. It suggested a long and
to northern India. The amount of annual rainfall in the narrow area of High Himalayas being uplifted during
quaternary. Then, the Himalayas are called young fold
northern plains of India decreases from east to west.
mountains because parallel ranges are found here.
Since the rainfall in the Indian sub-continent is caused Finally, all statements are correct. The correct answer
by moisture- bearing south-west monsoon winds, the is option (d).
duration of the monsoon is highest in nearby coastal
regions and decreases onwards once the wind starts • Monsoon, Forest, Soils, Irrigation
moving towards the inland regions. This happens due Projects, Agriculture
to the fast loosening of moisture in these winds. The
21. Consider the following crops of India:
same could be associated with the annual rainfall 1. Cowpea
distribution in northern plains from east to west. 2. Green Gram
18. A particular State in India has the following 3. Pigeon pea
characteristics: Which of the above is /are used as pulse, fodder
1. It is located on the same latitude which passes and green manure?
through northern Rajasthan. (a) 1 and 2 only (b) 2 only
2. It has over 80% of its area under forest cover. (c) 1 and 3 only (d) 1, 2 and 3
3. Over 12% of forest cover constitutes Protected Ans. (d): Green Manure Crops or legumes like
Area Network in this State. cowpea, horse gram and green gram improves
Which one among the following States has all the precipitation- use efficiency during off- season. Cow -
pea is Lobia. Lobia as green manure puts in nitrogen,
above characteristics?
builds organic matter, smothers weeds and controls
(a) Arunachal Pradesh (b) Assam
nematodes. Pigeon pea (Tur/Arhar) is also extensively
(c) Himachal Pradesh (d) Uttarakhand used in green manure. Pulses and Fodder are all of
Ans. (a): Arunachal Pradesh is located on the same them. So the correct answer is option (d).
latitude which passes through northern Rajasthan.
22. Consider the following crops of India :
More than 12% of the forested area is in the form of a
1. Groundnut
protected area network of this state and more than 80% 2. Sesamum
area of this state comes under forest cover. 3. Pearl millet
19. With reference to the wetlands of India, consider Which of the above is /are predominantly rain-
the following statements: fed crop/crops ?
1. The country’s total geographical area under the (a) 1 and 2 only (b) 2 and 3
category of wetlands is recorded more in Gujarat (c) 3 only (d) 1, 2 and 3
as compared to other States. Ans. (d) : Groundnut, sesamum and pearl millet are
2. In India, the total geographical area of coastal cultivated in dry agricultural areas. These crops are
wetlands is larger than that of inland wetlands. mainly rainfed crops.
Which of the statements given above is /are
correct? • Minerals, Power Resources &
(a) 1only (b) 2 only Industry and Trade
(c) Both 1 and 2 (d) Neither 1 nor 2
Ans. (c): The country’s total geographical area under 23. To meet its rapidly growing energy demand,
the wetlands in recorded highest in Gujarat as some opine that India should pursue research
compared to other states. In India, the total and development on thorium as the future fuel of
geographical area of coastal wetlands in lesser than nuclear energy. In this context, what advantage
that of inland wetlands. does thorium hold over uranium ?
1. Thorium is far more abundant in nature than
20. When you travel in Himalayas, you will see the
following ? uranium.
1. Deep gorges 2. On the basis of per unit mass of mined mineral,
2. U-turn river courses thorium can generate more energy compared to
3. Parallel mountain ranges natural uranium.
4. Steep gradients causing land-sliding 3. Thorium produces less harmful waste compared
Which of the above can be said to be the to uranium.
evidences for Himalayas being young fold Which of the statements given above is/are
mountains? correct?
(a) 1 and 2 only (b) 1, 2 and 4 only (a) 1 only (b) 2 and 3 only
(c) 3 and 4 only (d) 1, 2, 3 and 4 (c) 1 and 3 only (d) 1, 2 and 3
IAS (Pre) GS 2012 Paper I 173 YCT
Ans. (d): Thorium could be used to fuel nuclear 27. What is the difference between the antelopes
reactors, just like Uranium. Thorium is three times Oryx and Chiru?
more abundant in nature than uranium, is not fissile on (a) Oryx is adapted to live in hot and arid areas
its own (which means reactions can be stopped when whereas Chiru is adapted to live in steppes and
necessary), produces waste products that are less semi-desert areas of cold height mountains.
radioactive, and generates more energy per ton. All (b) Oryx is poached for its antlers whereas Chiru is
statement are correct. poached for its musk.
24. Despite having large reserves of coal, why does (c) Oryx exists in western India only whereas Chiru
India import millions of tonnes of coal ? exists in north-east India only.
1. It is the policy of India to save its own coal (d) None of the statements (a), (b) and (c) given
reserves for future, and import it from other above is correct.
countries for the present use.
2. Most of the power plants in India are coal –based Ans. (a): The Oryx is well-known for living in hot and
and they are not able to get sufficient supplies of dry areas, while Chiru is adapted to live in cold, high
coal from within the country. mountain meadows and semi-desert areas.
3. Steel companies need large quantity of coking 28. Consider the following factors :
coal which had to be imported.
Which of the statements given above is /are 1. Rotation of the Earth
correct 2. Air pressure and wind
(a) 1 only (b) 2 and 3 only 3. Density of ocean water
(c) 1 and 3 only (d) 1, 2 and 3 4. Revolution of the Earth
Ans. (b) : Indian imports several million tonnes of Which of the above factors influence the ocean
coal. Despite being a large reserve of coal, as steel currents?
companies require large amounts of coke coal and (a) 1 and 2 only (b) 1, 2 and 3
most of India's power plants are based on coal and (c) 1 and 4 (d) 2, 3 and 4
have a substantial amount of coal that could not be Ans. (b): Earth's rotation, air pressure, wind and the
supplied internally. density of ocean water affect ocean currents. It is an
important physical property of ocean water. It is
WORLD GEOGRAPHY determined by the amount of sunset, heat balance,
density of ocean water, salinity, evaporation and
• Universe and Solar System condensation, local seasonal conditions, etc.
25. Normally, the temperature decreases with the
increase in height from the Earth’s surface, • Hydrosphere
because.
1. The atmosphere can be heated upwards only from 29. What would happen if phytoplankton of an ocean
the Earth’s surface. is completely destroyed for some reason?
2. There is more moisture in the upper atmosphere. 1. The ocean as a carbon sink would be adversely
3. The air is less dense in the upper atmosphere. affected.
Select the correct answer using the codes given 2. The food chains in the ocean would be adversely
below: affected.
(a) 1 only (b) 2 and 3 only
(c) 1 and 3 only (d) 1, 2 and 3 3. The density of ocean water would drastically
decrease.
Ans. (c): Generally, the temperature decreases as the
altitude increases from the earth's surface because the Select the correct answer using the codes given
atmosphere can only warm upward from the Earth's below :
surface and the air in the upper atmosphere is less (a) 1 and 2 only (b) 2 only
dense. The atmosphere is mainly composed of gases, (c) 3 only (d) 1, 2 and 3
water vapour and dust. The amount of water vapour in Ans. (a) : If the phytoplankton of an ocean is
the atmosphere is between 0 to 5%. destroyed due to some reason, it will have the effect
that the ocean will be adversely affected in the form of
• Grasslands/Lithosphere carbon sinks and the food chain of the ocean will be
26. Which one of the following is the characteristic adversely affected.
climate of the Tropical Savannah Region ?
(a) Rainfall throughout the year 30. The acidification of oceans is increasing . Why is
(b) Rainfall in winter only this phenomenon a cause of concern ?
(c) An extremely short dry season 1. The growth and survival of calcareous
(d) A definite dry and wet season phytoplankton will be adversely affected.
Ans. (d): Tropical Savannah has a definite dry and wet 2. The growth and survival of coral reefs will be
season. The wet summer season lasts 6-8 months and adversely affected.
during these days, there is plenty of rainfall. Winter 3. The survival of some animals that have
lasts for 4-6 months and there might be no rain in phytoplankton larvae will be adversely affected.
winter. This winter is the dry season in which there are 4. The cloud seeding and formation of clouds will
frequent forest fires. be adversely affected.
IAS (Pre) GS 2012 Paper I 174 YCT
Which of the statements given above is/are Select the correct answer using the codes given
correct? below:
(a) 1, 2 and 3 only (b) 2 only (a) 1 and 2 only (b) 3, 4 and 5 only
(c) 1 and 3 only (d) 1, 2, 3, and 4 (c) 1, 2 and 5 (d) 1, 2, 3, 4 and 5
Ans. (a): The acidification of the oceans is increasing. Ans. (d): Given all statement is correct Hence option
This phenomenon is a matter of concern as the growth (d) is correct answer.
and survival of coral reefs is adversely affected and the ⇒ Directive Principles of State Policy
growth and survival of calcareous phytoplankton will ⇒ Rural and Urban Local Bodies
be adversely affected. In addition, the survival of some
animals whose ovaries are phytoplankton is adversely
⇒ Fifth Schedule
affected. ⇒ Sixth Schedule
⇒ Seventh Schedule
• Agriculture According to the Directive Principles of State Policy,
31. Which of the following is the chief characteristic Article 41 provides right to education. The 86th
of ‘mixed farming’? Constitutional Amendment 2002 of article 45 states
(a) Cultivation of both cash crops and food crops. that the state shall endeavour to provide early
(b) Cultivation of two or more crops in the same childhood care and education for all children until they
complete the age of six years. In addition, rural and
field.
urban local bodies and the seventh schedule
(c) Rearing of animals and cultivation of crops
(concurrent) have an impact on education.
together.
(d) None of the above. • Citizenship, Fundamental Rights,
Ans. (c): The major feature of 'mixed farming' is that Fundamental Duties, Directive
animal husbandry and cultivation of crops are carried
out simultaneously. Principles of State Policies
34. Consider the following provisions under the
• Population and Urbanization Directive Principles of State Policy as enshrined
in the Constitution of India :
32. Consider the following specific stages of
1. Securing for citizens of India a uniform civil
demographic transition associated with economic
code.
development : 2. Organizing village Panchayats.
1. Low birthrate with low death rate. 3. Promoting cottage industries in rural areas.
2. High birthrate with high death rate. 4. Securing for all the workers reasonable leisure
3. High birthrate with low death rate. and cultural opportunities.
Select the correct order of the above stages using Which of the above are the Gandhian Principles
the codes given below : that are reflected in the Directive Principles of
(a) 1, 2, 3 (b) 2, 1, 3 State Policy.
(c) 2, 3, 1 (d) 3, 2, 1 (a) 1, 2 and 4 only (b) 2 and 3 only
Ans. (c): The theory of the demographic transition of (c) 1, 3 and 4 only (d) 1, 2, 3 and 4
economic development was conceived by W.S. Ans. (b): Gandhian ideology is reflected in many
Thomson in the year 1929, but it was given the provisions of the directive principles of state policy.
scientific form by Frank W Notestein in the year 1945. Article 40 of the Indian constitution describes the
‘Organisation of village panchayats’. Article 43
Specific stages of demographic transition related to
describes the promotion of cottage industries and
economic development are as follows– Uniform Civil Code comes under the article 44.
-High birth rate with high death rate. Securing a uniform civil code for Indian citizens (art.
-High birth rate with low death rate. 44) and liberal ideology and securing proper leisure
-Low birth rate with low death rate. and cultural opportunities for all workers comes under
socialist ideology.
INDIAN CONSTITUTION 35. Which of the following is/are among the
Fundamental Duties of citizens laid down in the
• Constitutional Development of India Indian Constitution ?
33. Which of the following provisions of the 1. To preserve the rich heritage of our composite
constitution of India have a bearing on culture.
Education? 2. To protect the weaker sections from social
1. Directive Principles of State Policy injustice.
2. Rural and Urban Local Bodies 3. To develop the scientific temper and spirit of
3. Fifth Schedule inquiry.
4. Sixth Schedule 4. To strive towards excellence in all spheres of
5. Seventh Schedule individual and collective activity.
IAS (Pre) GS 2012 Paper I 175 YCT
Select the correct answer using the codes given (a) 1 only (b) 2 and 3
below: (c) 1 and 3 (d) None
(a) 1 and 2 only (b) 2 only Ans. (d): None of the above statements is correct: (i)
(c) 1, 3 and 4 only (d) 1, 2, 3 and 4 Union Territories (Delhi and Puducherry) are
Ans. (c): The fundamental duties are defined as the represented in the Rajya Sabha. (ii) It is not within the
moral obligations of all citizens to help promote a purview of the Chief Election Commissioner to
spirit of patriotism and uphold India’s Unity. These adjudicate election disputes. It is the Supreme Court
duties set out in part IV-A of the constitution, concern and High Court which look into the disputes. (iii)
individuals and the nation. Like the directive According to the art. 79 of the Constitution of India,
principles, they are not legally enforceable. The correct the parliament consists of the Lok Sabha, the Rajya
option is (c). Sabha and the President of India.
39. Regarding the office of the Lok Sabha Speaker,
• Executive of Center and States, consider the following statements :
Emergency Provisions 1. He/She holds the office during the pleasure of the
36. According to the Constitution of India, it is the President
duty of the President of India to cause to be laid 2. He/She need not be a member of the House at the
before the Parliament which of the following? time of his/her election but has to become a
1. The Recommendations of the Union Finance member of the House within six months from the
Commission. date of his/her election.
2. The Report of the Public Accounts Committee. 3. If/he/she intends to resign, the letter of his her
3. The Report of the Comptroller and Auditor resignation has to be addressed to the Deputy
General. Speaker.
4. The Report of the National Commission for Which of the statements given above is/are correct?
Schedules Castes. (a) 1 and 2 only (b) 3 only
Select the correct answer using the codes given (c) 1, 2 and 3 (d) None
below: Ans. (b): Statements (1) and (2) are not correct. On the
(a) 1 only (b) 2 and 4 only dissolution of the Lok Sabha, although the speaker
(c) 1, 3 and 4 (d) 1, 2, 3 and 4 ceases to be a member of the house, he/she does not
Ans. (c) : It is not the duty of the president of India to vacate his/her office. The speaker may, at any time,
cause to be laid the report of public account committee resign from office by writing under his/her hand to the
before the parliament. Deputy Speaker.
37. A deadlock between the Lok Sabha and the 40. Which of the following special power have been
Rajya Sabha calls for a joint sitting of the conferred on the Rajya Sabha by the
Parliament during the passage of : Constitution of India ?
1. Ordinary Legislation (a) To change the existing territory of a State and to
2. Money Bill change the name of a State.
3. Constitution Amendment Bill (b)To pass a resolution empowering the Parliament
Select the correct answer using the codes given to make laws in the State List and to create one
below : or more All India Service.
(a) 1only (b) 2 and 3 only (c) To amend the election procedure of the President
(c) 1 and 3 only (d) 1, 2 and 3 and to determine the pension of the President
Ans. (a): Unlike a money bill, a bill can become law after his/her retirement.
only when both the houses agree to it with or without (d)To determine the functions of the Election
amendments. This question does not arise in relation to Commission and to determine the number of
the Money Bill because the Lok Sabha has the ultimate Election Commissioners .
power to pass it. It should be kept in mind that the Ans. (b): The privileges conferred on the Rajya Sabha
procedure for joint sitting prescribed by Article 108 is by the constitution of India are passing of a resolution
limited to general legislation. This does not apply to the to make rules in the state list and empower article 312
constitutional amendment. It is governed by Article of the constitutions to create one or more All-India
368(2). It should be passed by a separate special Services.
majority by each house.
38. Consider the following statements : • Legislature : Council of Ministers
1. Union Territories are not represented in the Rajya 41. The Prime Minister of India, at the time of
Sabha.
his/her appointment
2. It is within the purview of the Chief Election
Commissioner to adjudicate the election disputes. (a) need not necessarily be a member of one of the
3. According to the Constitution of India, the Houses of the Parliament but must become a
Parliament consists of the Lok Sabha and the member of one of the Houses within six months.
Rajya Sabha only. (b) need not necessarily be a member of one of the
Which of the statements given above is/are Houses of the Parliaments but must become a
correct? member of the Lok Sabha within six months.
IAS (Pre) GS 2012 Paper I 176 YCT
(c) must be a member of one of the Houses of the 1. CAG exercises exchequer control on behalf of the
Parliament. Parliament when the President of India declares
(d) must be a member of the Lok Sabha. national emergency/financial emergency.
2. CAG reports on the execution of projects or
Ans. (a): The Prime Minister is appointed by the programmes by the ministries are discussed by
president. The minimum age required is 25 years, as he the Public accounts Committee.
can be a member of either house if not, must be within 3. Information from CAG reports can be used by
6 months. In normal circumstances, the president can investigating agencies to press charges against
hardly exercise his discretion; the presidents choice to those who have violated the law while managing
appoint the Prime Minister is restricted to the leader of public finances.
the party with majority of Lok Sabha. The correct 4. While dealing with the audit and accounting of
option is (b). government companies, CAG has certain judicial
42. In the parliament of India, the purpose of an powers for prosecuting those who violate the law.
adjournment motion is- Which of the statements given above is/are correct?
(a) to allow a discussion on a definite matter of (a) 1, 3 and 4 (b) 2 only
urgent public importance. (c) 2 and 3 only (d) 1, 2, 3 and 4
(b) to let opposition members collect information Ans. (c) : CAG reports on execution of projects or
from the ministers. programmes by the Public Accounts Committee.
(c) to allow a reduction of specific amount in • Information from CAG reports can be used by
demand for grant. investigating agencies to press charges against
(d) to postpone the proceedings to check the those who have violated the law while managing
inappropriate or violent behaviour on the part public finance.
of some members. • Only (2) and (3) are correct statements.
Ans. (a): The purpose of bringing an adjournment 45. The distribution of powers between the Centre
motion in the Indian Parliament is to debate a certain and the States in the Indian Constitution is based
urgent issue of public importance. There are many on the scheme provided in the-
meetings per day in a session that are separated by (a) Morley-Minto Reforms, 1909
adjournment. By adjournment, the consideration of the (b) Montagu-Chelmsford Act, 1919
working of the house is deferred for a specified period (c) Government of India Act, 1935
of time. It can also be of a period of a few hours, days (d) Indian Independence Act, 1947
or weeks. Ans. (c): In the constitution of India, the separation of
43. Which of the following can be said to be powers between the center and the states is based on
essentially the parts of ‘Inclusive Governance’ ? the Government of India Act, 1935. The federal system
1. Permitting the Non-Banking Financial Companies consists of two governments and the division of
to do banking. powers between them. The federal polity depends on
2. Establishing effective District planning the co-ordination between these two governments.
Committees in all the districts. 46. Which of the following are the methods of
3. Increasing the government spending on public Parliamentary control over public finance in
health. India?
4. Strengthening the Mid-day Meal Scheme. 1. Placing Annual Financial Statement before the
Select the correct answer using the codes given Parliament.
below: 2. Withdrawal of money from Consolidated Fund of
(a) 1 and 2 only (b) 3 and 4 only India only after passing the Appropriation Bill.
(c) 2, 3 and 4 (d) 1, 2, 3 and 4 3. Provisions of supplementary grants and vote-on-
account.
Ans. (c): Inclusive governance refers to a regime in 4. A periodic or at least a mid-year review of
which the people belonging to the most vulnerable programme of the Government against
section of society get a fair share in the distribution of macroeconomic forecasts and expenditure by a
national income generated at a high economic growth Parliamentary Budget Office.
rate. From this perspective, empowering the 'mid-day 5. Introducing Finance Bill in the Parliament.
meal' scheme, increasing government expenditure on Select the correct answer using the codes given
public health, setting up effective district planning below:
committees in all districts will be called part of (a) 1, 2, 3, and 5 only (b) 1, 2 and 4 only
inclusive governance. Permitting the non-banking (c) 3, 4 and 5 only (d) 1, 2, 3, 4 and 5
financial companies to do banking is not the essential Ans. (a): Parliament exercises the following types of
part of the 'Inclusive Governance'. control over the expenditure by placing annual
financial statement before the parliament under Article
• Centre-State Relations,Administration, 112. Under Article 114, it is given that no money can
State of Jammu & Kashmir (now UT) be withdrawn from the Consolidated Fund of India
44. In India, other than ensuring that public funds, without passing Appropriation Bills.
are used efficiently and for intended purpose, Provisions of supplementary grants and vote-on-accounts
what is the importance of the office of the is given in Article 115 and Article 116 respectively.
Comptroller and Auditor General (CAG) ? Article 107 stipulate provisions for Finance Bill.
IAS (Pre) GS 2012 Paper I 177 YCT
• Category-Special Provisions, Select the correct answer using the codes given
below:
Scheduled/ Tribal Areas (a) 1 and 2 (b) 2 and 3
47. With reference to the Delimitation Commission. (c) 1 and 4 (d) 3 and 4
consider the following statements : Ans. (c): The original jurisdiction of the Supreme Court
1. The orders of the Delimitation Commission Subject to the provisions of Article 131 of the indian
cannot be challenged in a Court of Law. Constitution, the Supreme Court shall, to the exclusion
2. When the orders of the Delimitation Commission of any other court, have original jurisdiction in any
are laid before the Lok Sabha or State Legislative dispute: (a) between the Government of India and one or
Assembly, they cannot effect any modifications in more States; or (b) between the Government of India
the orders. and any State or States on one side and one or more
Which of the statements given above is/are correct? other States on the other; or (c) between two or more
(a) 1 only (b) 2 only States, if and in so far as the dispute involves any
(c) Both 1 and 2 (d) Neither 1 nor 2 question (whether of law or fact) on which the existence
Ans. (c) : The Government of India established the or extent of a legal right depends.
Delimitation Commission under the provisions of the
Delimitation Commission Act. Delimitation• Panchayati Raj System and
Commission has been set up four times in the past - Constitution Amendments and Schedules
1952,1962,1972 and 2002 Under Delimitation Acts of
1952,1962,1972, and 2002. 50. In the areas covered under the Panchayat
The commission is a powerful and independent body (Extension to the Scheduled Areas) Act, 1996,
whose orders cannot be challenged in any court of law. what is the role/power of Gram Sabha?
The orders are laid before the Lok Sabha and the 1. Gram Sabha has the power to prevent alienation
respective State Legislative Assemblies. However, of land in the Scheduled Areas.
modifications are not permitted. 2. Gram Sabha has the ownership of minor forest
• Judiciary (Centre and State) produce.
3. Recommendation of Gram Sabha is required for
48. What is the provision to safeguard the autonomy granting prospecting licence or mining lease for
of the Supreme Court of India ? any mineral in the Scheduled Areas.
1. While appointing the Supreme Court Judges, the Which of the statements given above is/are
President of India has to consult the Chief Justice correct?
of India. (a) 1 only (b) 1 and 2 only
2. The Supreme Court Judges can be removed by
the Chief Justice of India only. (c) 2 and 3 only (d) 1, 2 and 3
3. The salaries of the Judges are charged on the Ans. (d): As per the provisions of the Panchayats
Consolidated Fund of India to which the (Extensions to Scheduled Areas) Act, 1996, Gram
legislature does not have to vote. Sabha has ownership of minor forest produce and stops
4. All appointments of officers and staffs of the the transfer of land in scheduled areas mining for any
Supreme Court of India are made by the mineral in scheduled area. The recommendation of
Government only after consulting the Chief Gram Sabha is also necessary for grant of lease and
Justice of India. prospecting license.
Which of the statements given above is/are correct?
(a) 1 and 3 only
(c) 4 only
(b) 3 and 4 only
(d) 1, 2, 3 and 4
ECONOMICS
Ans. (a): Every judge of the Supreme Court shall be • Developing and Developed Economy
appointed by the President. The President may consult
other persons for this purpose in addition to the advice 51. Under, which of the following circumstances may
of ministers. In addition to the appointment of other ‘capital gains’ arise ?
judges of the Supreme Court, it is mandatory to consult 1. When there is an increase in the sales of product.
the Chief Justice of India (Article 124). It has been 2. When there is a natural increase in the value of
provided that the administrative expenses of the
Supreme Court and the salaries, allowances etc. of the the property owned.
Judges and other staff of the Supreme Court will be 3. When you purchase a painting and there is a
charged on the Consolidated Fund of India. i.e., growth in its value due to increase in its
Parliament cannot vote on them (Article 146). popularity.
49. Which of the following are included in the Select the correct answer using the codes given
original jurisdiction of the Supreme Court: below:
1. A dispute between the Government of India and (a) 1 only (b) 2 and 3 only
one or more States. (c) only 2 (d) 1, 2 and 3
2. A dispute regarding elections to either House of Ans. (b): Capital gains arise when there is a natural
the Parliament or that of Legislature of a State. increase in the value of the property owned and when
3. A dispute between the Government of India and a
Union Territory. you purchase a painting and there is a growth in its
4. A dispute between two or more states. value due to an increase in its popularity.
IAS (Pre) GS 2012 Paper I 178 YCT
52. With reference to the guilds (Shreni) of ancient 1. Deprivation of education, health, assets and
India that played a very important role in the services at household level.
country’s economy, which of the following 2. Purchasing power parity at national level.
statements is/are correct ? 3. Extent of budget deficit and GDP growth rate at
1. Every guild was registered with the central
national level.
authority of the State and the king was the chief
administrative authority on them. Select the correct answer using the codes given
2. The wages, rules of work, standards and price below :
were fixed by the guild. (a) 1 only (b) 2 and 3
3. The guild had judicial powers over its own (c) 1 and 3 (d) 1, 2 and 3
members. Ans. (a) : The Multidimensional Poverty Index (MPI)
Select the correct answer using the codes given based its estimation on education (years of schooling
below : and attendance in schools), health (child mortality and
(a) 1 and 2 only (b) 3 only nutrition) and standard of living (electricity, sanitization,
(c) 2 and 3 only (d) 1, 2 and 3 drinking water, flooring of the house, cooking fuel and
asset ownership). Equal weightage is assignments to
Ans. (c): The guilds (Shreni) that played very the three lead factors such that weightage is
important role in the ancient economy had their own proportionately split among the sub- factors.
seperate courts that settled their members’ disputes.
The 'guilds' itself ensured salaries, work rules, • Economic Planning and National
standards and prices. The craft guilds was headed by a Income/Budget
Chief called the Jetthaka. 55. The endeavour of ‘Janani Suraksha Yojana’
programm is -
• Rural Development 1. to promote institutional deliveries.
2. to provide monetary assistance to the mother to
53. How do District Rural Development Agencies meet the cost of delivery.
(DRDAs) help in the reduction of rural poverty in 3. to provide for wage loss due to pregnancy and
India ? confinement.
1. DRDAs act as Panchayati Raj Institution in Which of the statements given above is/are
certain specified backward regions of the country correct?
2. DRDAs undertake area- specific scientific study. (a) 1 and 2 only (b) 2 only
of the causes of poverty and malnutrition and (c) 3 only (d) 1, 2 and 3
prepare detailed remedial measures. Ans. (a) : The Endeavour of Janani Surakesha Yojna
3. DRDAs secure inter-sectoral and inter programmes are (i) to promote institutional deliveries,
(ii) to provide monetary assistance to the mother to
departmental coordination and cooperation for
meet the cost of delivery. Janani Suraksha Yojana
effective implementation of anti-poverty (JSY) is a safe motherhood intervention under the
programmes. National Rural Health Mission (NRHM) being
4. DRDAs watch over the ensure effective implemented with the objective of reducing maternal
utilization of the funds intended for anti poverty and neo-natal mortality by promoting institutional
programmes. delivery among the poor pregnant women. The Yojana
th
Which of the statements given above is/are was launched on 12 April 2005, by the Hon’ble Prime
correct ? Minister, Manmohan Singh.
(a) 1, 2 and 3 only (b) 3 and 4 only 56. Consider the following :
(c) 4 only (d) 1, 2, 3 and 4 1. Hotel and restaurants.
Ans. (b): In order to strengthen the district Rural 2. Motor transport undertakings.
Development Agencies and make them more 3. Newspaper establishments.
professional in their functioning, a program called 4. Private medical institutions.
District Rural Development Agency (DRDA) The employees of which of the above can have the
Administration was launched on 1 April 1999 as a ‘Social Security’ coverage under Employees’
centrally sponsored scheme. The center and the state State Insurance Scheme ?
provide funds for this program in the ratio of 75:25. (a) 1, 2 and 3 only (b) 4 only
DRDAs secure inter-sectoral and inter-departmental (c) 1, 3 and 4 (d) 1, 2, 3 and 4
coordination and co-operation for effective Ans. (d) : According to the latest decision of
implementation of anti-poverty programs. DRDAs Employees State Insurance Corporation (ESIC) Act-
monitor funds received for anti-poverty programs and 1948, now factories/institutions with 10 employees
ensured that they are used effectively. will also come under its purview, even if they are
using electricity in the construction process or not.
54. The Multi-dimensional Poverty Index developed Employees of hotel and restaurant, motor transport
by Oxford Poverty and Human Development industry, newspaper establishments and private
Initiative with UNDP support covers which of the medical institutions can get social security cover under
following ? the 'Employees Insurance Scheme'.
IAS (Pre) GS 2012 Paper I 179 YCT
57. With reference to National Rural Health Mission, Ans. (d) : To encourage the development of the
which of the following are the jobs of ‘ASHA’, manufacturing sector, the Government of India has
trained community health worker ? taken the initiative to establish national investment and
1. Accompanying women to the health facility for manufacturing sectors, facilitate single-window
antenatal care checkup. clearance and establishment of Technology
2. Using pregnancy test kits for early detection of Acquisition and Development Fund.
pregnancy.
3. Providing information on nutrition and 60. In India, in the overall Index of Industrial
immunization. Production, the Indices of Eight Core Industries
have a combined weight of 37.90%. Which of the
4. Conducting the delivery of baby.
following are among those Eight Core Industries?
Select the correct answer using the codes given
below : 1. Cement 2. Fertilizers
(a) 1, 2 and 3 only (b) 2 and 4 only 3. Natural gas 4. Refinery products
(c) 1 and 3 only (d) 1, 2, 3 and 4 5. Textiles
Ans. (a): One of the key components of the National Select the correct answer using the codes given
Rural Health Mission is to provide every village in the below:
country with a trained female community health (a) 1 and 5 only (b) 2, 3 and 4 only
activist, ‘ASHA’ or Accredited Social Health Activist. (c) 1, 2, 3 and 4 only (d) 1, 2, 3, 4 and 5
They are selected from the village itself and Ans. (c): Currently, the Eight Core Industries comprise
accountable to it, the ASHA will be trained to work as 40.27 (37.90% at that time) percent of the weight of
an interface between the community and the public items included in the Index of Industrial Production
health system. (IIP). The eight core industries include coal, crude oil,
58. ‘How does the National Rural Livelihood mission steel, electricity, cement, fertilizer, natural gas and
seek to improve livelihood options of rural poor ? refinery products.
1. By setting up a large number of new
manufacturing industries and agribusiness centers • Money/Banking, Tax System, Centre -
in rural areas. State Relations
2. By strengthening ‘self-help groups’ and providing
skill development. 61. The Reserve Bank of India (RBI) acts as a
3. By supplying seeds, fertilizers, diesel pumps sets banker’s bank .This would imply which of the
and micro-irrigation equipment free of cost to following ?
farmers. 1. Other banks retain their deposits with the RBI.
Select the correct answer using the codes given 2. The RBI lends funds to the commercial banks in
below: times of need.
(a) 1 and 2 only (b) 2 only 3. The RBI advises the commercial banks on
(c) 1 and 3 only (d) 1, 2 and 3 monetary matters.
Ans. (b): The National Rural Livelihood Mission is an Select the correct answer using the codes given
attempt to improve the livelihood options of the rural below:
regional poor. The Central Government launched the (a) 2 and 3 only (b) 1 and 2 only
National Rural Livelihood Mission (NRLM) in June (c) 1 and 3 (d) 1, 2 and 3
2011. This mission, launched for strengthening self-
help groups in rural areas and for the prevention of Ans. (d): The Reserve Bank of India is the Central
poverty of the poor through them, started in the Bank of India. It was established on 1st April 1925
Banswara district of Rajasthan. Under this mission, the with an authorized capital of 5 crore rupees. The
government plans to provide self-help groups at the Reserve Bank was nationalized on 1st January 1949.
village level as federations and provide them with a The Reserve Bank acts as a bankers bank i.e, the
suitable base of better livelihood by providing them central bank, which means other banks keep their
with profitable self-employment opportunities. deposits with the RBI to control the credit. It gives
loans to commercial banks and advises on monetary
• Agriculture, Industry and Trade matters. Apart from this, control of foreign exchange,
issue of notes, functioning as a bank of banks are also
59. What is /are the recent policy initiatives(s) of included in its functions.
Government of India to promote the growth of
manufacturing sector ? 62. Which of the following measures would result in
1. Setting up of National Investment and an increase in the money supply in the economy ?
Manufacturing Zones. 1. Purchase of government securities from the
2. Providing the benefit of ‘single window public by the Central Bank
2. Deposit of currency in commercial banks by the
clearance’.
public
3. Establishing the Technology Acquisition and 3. Borrowing by the government from the Central
Development Fund. Bank
Select the correct answer using the codes given 4. Sale of government securities to the public by the
below : Central Bank
(a) 1 only (b) 2 and 3 only Select the correct answer using the codes given
(c) 1 and 3 only (d) 1, 2 and 3 below :
IAS (Pre) GS 2012 Paper I 180 YCT
(a) 1 only (b) 2 and 4 only Ans. (c): The 13th Finance Commission report has an
(c) 1 and 3 (d) 2, 3 and 4 elaborate design for the GST. Statement (1) is correct
Ans. (c): The Reserve Bank carries out the task of while Statement (2) is incorrect. The report basically
controlling the quantity and direction of credit created discusses the improved implementation of the existing
by commercial banks. He uses quantitative and schemes. The statement (3) is correct. The report in its
qualitative measures to achieve this objective. When para 10.144 mentions: "Taking into account the
the RBI is looking to increase the quantity of money in demand of local bodies that they be allowed to benefit
circulation, it purchases government securities from from the buoyancy of central taxes and the
commercial banks and institutions. constitutional design of supplementing the resources of
Panchyayats and Municipalities through grant-in-aid,
63. Consider the following statements :
we recommend that local bodies be transferred a
The price of any currency in international percentage of the divisible pool of taxes (over and
market is decided by the- above the share of the states) as stipulated by as, after
1. World Bank. converting this share to grant-in-aid under article 275".
2. Demand for goods/services provided by the
country concerned. • Foreign Trade of India, Economic
3. Stability of the government of the concerned
country. Organizations & Share Market
4. Economic potential of the country in question. 66. Which of the following would include Foreign
Which of the statements given above are correct? Direct Investment in India ?
(a) 1, 2, 3 and 4 (b) 2 and 3 only 1. Subsidiaries of companies in India.
(c) 3 and 4 only (d) 1 and 4 only 2. Majority foreign equity holding in Indian
Ans. (b): Money supply increases when money flows companies.
out of the RBI. The purchase of Government securities 3. Companies exclusively financed by foreign
from the public by the central bank leads to the transfer companies.
of money to the public, thereby leading to an increase 4. Portfolio investment.
in money supply in the economy. Similarly, borrowing Select the correct answer using the codes given
by the government from the central bank leads to an below :
increased money supply. (a) 1, 2, 3 and 4 (b) 2 and 4 only
However, the deposit of currency in commercial banks (c) 1 and 3 only (d) 1, 2 and 3 only
by the public doesn’t increase money supply as the Ans. (d): Foreign Direct Investment (FDI) is a direct
money just gets transferred from the public to investment into production or business in a country by
commercial banks. an individual or company of another country, either by
64. The basic aim of Lead Bank Scheme is that- buying a company in the target country or expanding
(a) Big banks should try to open offices in each operations of an existing business in that country.
district. Foreign direct investment is in contrast to portfolio
(b) There should be stiff competition among the investment, which is a passive investment in the
various nationalized banks. securities of another country such as stocks and bonds.
(c) Individual banks should adopt particular
districts for intensive development. SCIENCE
(d) All the banks should make intensive efforts to
mobilize deposits. • Physics
Ans. (c): The basic aim of the Lead Bank Scheme is 67. A person stood alone in a desert on a dark night
that stiff competition among the various nationalized and wanted to reach his village which was
banks and big banks should try to open offices in each
situated 5 km east of the point where he was
district.
standing. He had no instruments to find the
65. Which of the following is/are among the direction but he located the polestar. The most
noticeable features of the recommendations of the convenient way now to reach his village is to walk
Thirteenth Finance Commission? in the –
1. A design for the Goods and Service Tax, and a (a) direction facing the polestar
compensation package linked to adherence to the (b) direction opposite to the polestar
proposed design. (c) direction keeping the polestar to his left
2. A design for a creation of lakhs of jobs in the (d) direction keeping the polestar to his right
next ten years in consonance with India‘s
demographic dividend. Ans. (c): Pole star's position with respect to rotating
3. Devolution of a specified share of central taxes to earth never changes and it always points to the north
local bodies as grants. direction. Thus, if the person keeps the pole star to his
Select the correct answer using the codes given left, he can be sure of walking in the eastern direction
below: in which his village is located. Polestar was used
(a) 1 only (b) 2 and 3 only widely for navigation in ancient and medieval times
(c) 1 and 3 only (d) 1, 2 and 3 due to its stable position.

IAS (Pre) GS 2012 Paper I 181 YCT


68. Consider the following statements : (b) Ozone layer around the Earth reflects them back
If there were no phenomenon of capillarity- to outer space.
1. It would be difficult to use a kerosene lamp. (c) Moisture in the upper layers of atmosphere
2. One would not be able to use a straw to consume prevents them from reaching the surface of the
a soft drink. Earth.
3. The blotting paper would fail to function. (d) None of the statements (a), (b) and (c) given
4. The big trees that we see around would not have above is correct.
grown on the Earth.
Which of the statements given above are correct? Ans. (a) : Electric charge particles travelling at a speed
(a) 1, 2 and 3 only (b) 1, 3 and 4 only of several hundred km/sec in space cannot reach the
(c) 2 and 4 only (d) 1, 2, 3 and 4 surface of the earth because the Earth's magnetic force
turns them towards the pole.
Ans. (b): If capillary phenomena were not present, the
use of the Kerosene lamp would have been difficult,
the blotting paper would have failed to function and • Chemistry
the large trees that we see all around would not grow 72. Which one of the following sets of elements was
on the earth because capillarity is fundamentally primarily responsible for the origin of life on the
responsible for all these phenomena. Earth ?
69. Graphene is frequently in news recently. What is (a) Hydrogen, Oxygen, Sodium
its importance? (b) Carbon, Hydrogen, Nitrogen
1. It is a two-dimensional material and has good (c) Oxygen, Calcium, Phosphorus
electrical conductivity. (d) Carbon, Hydrogen, Potassium
2. It is one of the thinnest but strongest materials
tested so far. Ans. (b): Carbon, Hydrogen, Nitrogen are basically
3. It is entirely made of silicon and has high optical responsible for the origin of life on Earth. Because of
transparency. these primary gases, the necessities of life, availability
4. It can be used as ‘conducting electrodes’ required of water, food and environment etc. are ensured.
for touch screens, LCDs and organic LEDs. 73. Recently, there has been a concern over the short
Which of the statements give above are correct? supply of group of elements called ‘rare earth
(a) 1 and 2 only (b) 3 and 4 only metals’. Why ?
(c) 1, 2 and 4 only (d) 1, 2, 3 and 4 1. China, which is the largest producer of these
Ans. (c): "Graphene" is a two-dimensional material elements, has imposed some restrictions on their
and has good electrical conduction. It is one of the export.
thinnest but strongest materials tested so far. It can be 2. Other than China, Australia, Canada and Chile,
used for conducting electrodes required for touch
these elements are not found in any country.
screens, LCDs and organic LEDs.
3. Rare earth metals are essential for the
70. Which of the following is /are cited by the manufacture of various kinds of electronic items
scientists as evidence/evidences for the continued and there is a growing demand for these elements.
expansion of universe ?
1. Detection of microwaves in space. Which of the statements given above is/are
2. Observation of red shift phenomenon in space. correct?
3. Movement of asteroids in space. (a) 1 only (b) 2 and 3 only
4. Occurrence of supernova explosions in space. (c) 1 and 3 only (d) 1, 2 and 3
Select the correct answer using the codes given Ans. (c): Origin of life started with the formation of
below: simple organic molecules like nucleoside. The building
(a) 1 and 2 blocks of nucleoside are Carbon, Hydrogen, Oxygen,
(b) 2 only Nitrogen and Phosphorus. Carbon is the most
(c) 1, 3 and 4 important of these because of its capability to form a
(d) None of the above can be cited as evidence wide variety of compounds.
Ans. (a): Scientists cite the observation of a red shift 74. What is the role of ultraviolet (UV) radiation in
phenomenon in space and the presence of microscopic the water purification systems.
waves in space as evidence of the continued expansion 1. It inactivates/kills the harmful microorganisms in
of the universe. In addition, supernova explosions in water.
space and the movement of asteroids etc. are not used 2. It removes all the undesirable odours from the
as an extension of the universe. water.
71. Electrically charged particles from space 3. It quickens the sedimentation of solid particles,
travelling at speeds of several hundred km/sec removes turbidity and improves the clarity of
can severely harm living beings if they reach the water.
surface of the Earth .What prevents them from Which of the statements given above is/are
reaching the surface of the Earth ? correct?
(a) The Earth’s magnetic field diverts them towards (a) 1 only (b) 2 and 3 only
its poles. (c) 1 and 3 only (d) 1, 2 and 3
IAS (Pre) GS 2012 Paper I 182 YCT
Ans. (a) : Ultraviolet radiation in water purification Ans. (d): A fungal biopesticide formulation has been
neutralizes/destroys harmful micro-organism present in developed by a faculty member of the department of
water. Certain gases located in the atmosphere (i.e. Biotechnology at Gitam University and the test results
Ozone) are allowed to come to the ground only after held at different districts showed that it could control
filtering the earth radiation waves. The gases are several pests affecting chilli, paddy and vegetable
present the ultraviolet radiation of the Sun in the crops.
middle and protect the surface from overheating. 78. Lead, ingested or inhaled is a health hazard.
75. A team of scientists at Brookhaven National After the addition of lead to petrol has been
laboratory including those from India created the banned, what still are the sources of lead
poisoning?
heaviest anti-matter (anti-helium nucleus).
1. Smelting units
What is/are the implication/implications of the 2. Pens and pencils
creation of anti-matter ? 3. Paints
1. It will make mineral prospecting and oil 4. Hair oils and cosmetics
exploration easier and cheaper. Select the correct answer using the codes given
2. It will help probe the possibility of the existence below :
(a) 1, 2 and 3 only (b) 1 and 3 only
of stars and galaxies made of anti matter.
(c) 2 and 4 only (d) 1, 2, 3 and 4
3. It will help understand the evolution of the
Ans. (b): Among the options in questions are sources
universe. of lead poisoning are smelting units and paints. The
Select the correct answer using, the codes given amount of lead found in paint used for painting in
below : homes is fatal to human health. The welding/smelting
(a) 1 only (b) 2 and 3 only units of zinc and copper release lead into the
(c) 3 only (d) 1, 2 and 3 atmosphere.
Ans. (b): The scientific team of the Brookhaven 79. With reference to ‘stem cells’, frequently in the
National Laboratory, which also included Indian news, which of the following statements is / are
scientists, produced the heaviest anti-matter in the correct?
form of anti-helium. This will be helpful in checking 1. Stem cells can be derived from mammals only.
the possibility of stars or galaxies formed from anti- 2. Stem cells can be used for screening new drugs.
3. Stem cells can be used for medical therapies.
matter and it will also be helpful in developing an
Select the correct answer using the codes given
understanding of the universe. below :
(a) 1 and 2 only (b) 2 and 3 only
• Botany (c) 3 only (d) 1, 2 and 3
76. Consider the following kinds of organisms: Ans. (b): 'Stem cells', often in the spotlight can be
used to test new drugs. In addition, they can also be
1. Bat used for medical therapy. It can independently perform
2. Bee such actions as nutrition, respiratory enhancement,
3. Bird reproduction and self-regulation.
Which of the above is/are pollinating agent/
agents? • Agriculture, Animal Husbandry and
(a) 1 and 2 only (b) 2 only Dairy
(c) 1 and 3 only (d) 1, 2 and 3
Ans. (d): Pollination, in the scientific sense, is the 80. Consider the following :
1. Assessment of land revenue on the basis of
process of pollen reaching the ovary or spore, which
nature of the soil and the quality of crops.
produces fruits and seed after conception. Bees acts as 2. Use of mobile cannons in warfare.
pollinators as well as birds in tropical regions are also 3. Cultivation of tobacco and red chillies.
commonly known as pollinators. Bats are also Which of the above was/were introduced in to
pollinators of many flowers. India by the English?
(a) 1 only (b) 1 and 2
• Medical Science, Anthropology (c) 2 and 3 (d) None
77. Consider the following kinds of organisms : Ans. (d): Land revenue was assessed only in the
1. Bacteria medieval period based on the nature of the soil and the
2. Fungi properties of the produce. In the war, mobile canons were
3. Flowering plants used by Babur in the first battle of Panipat. Apart from
Some species of which of the above kind of this, tobacco was first brought to India by the Portuguese
organisms are employed as biopesticides ? during the reign of Mughal Emperor Akbar.
(a) 1 only (b) 2 and 3 only 81. How does National Biodiversity Authority (NBA)
(c) 1 and 2 only (d) 1, 2 and 3 help in protecting the Indian agriculture ?
IAS (Pre) GS 2012 Paper I 183 YCT
1. NBA checks the biopiracy and protects the 84. Biomass gasification is considered to be one of the
indigenous and traditional genetic resources. sustainable solutions to the power crisis in India.
2. NBA directly monitors and supervises the In this context, which of the following statements
scientific research on genetic modification of crop is/are correct ?
plants.
3. Application for Intellectual Property Rights 1. Coconut shells, groundnut shells and rice husk
related to genetic/biological resources cannot be can be used in biomass gasification .
made without the approval of NBA . 2. The combustible gases generated from biomass
Which of the statements given above is /are gasification consist of hydrogen and carbon
correct ? dioxide only.
(a) 1 only (b) 2 and 3 only 3. The combustible gases generated from biomass
(c) 1 and 3 only (d) 1, 2 and 3 gasification can be used for direct heat generation
Ans. (c): An application for intellectual property right but not in internal combustion engines.
related to genetic/biological resources cannot be Select the correct answer using the codes given
applied without the recommendation of the National below:
Biodiversity Authority. This authority prohibits bio- (a) 1 only (b) 2 and 3 only
theft and treats native and traditional genetic resources.
(c) 1 and 3 only (d) 1, 2 and 3
The Genetic Engineering Appraisal Committee
(GEAC) is responsible for approval of proposal Ans. (a): Biomass gasification is considered one of the
relating of Genetically Engineered organisms and sustainable solutions to the energy crisis. In this
products into the environment including experimental context, coconut shell, peanut peel and paddy husk are
field trials as per the provisions of rules 1998. used for biomass gasification.
• Science and Technology 85. Consider the following statements :
Chlorofluorocarbons, known as ozone-depleting
82. What are the reasons for the people’s resistance substances, are used-
to the introduction of Bt brinjal in India? 1. in the production of plastic foams.
1. Bt brinjal has been created by inserting a gene
from a soil fungus into its genome. 2. in the production of tubeless tyres.
2. The seeds of Bt brinjal are terminator seeds and 3. in cleaning certain electronic components.
therefore, the farmers have to buy the seeds 4. as pressurizing, agents in aerosol cans.
before every season from the seed companies. Which of the statements given above is /are
3. There is an apprehension that the consumption of correct?
Bt brinjal may have adverse impact on health. (a) 1, 2 and 3 only (b) 4 only
4. There are some concerns that the introduction of (c) 1, 3 and 4 only (d) 1, 2, 3 and 4
BT brinjal may have adverse effect on the
biodiversity. Ans. (c): Chloroflurocarbons, popularly known as
Select the correct answer using the codes given ozone depleting substances are used to clean certain
below: electronic components. Aerosol is used as a
(a) 1, 2 and 3 only (b) 2 and 3 only compressive agent in cars and also in the manufacture
(c) 3 and 4 only (d) 1, 2, 3 and 4 of solvable foam. CFCs are simple compounds of
Ans. (c): The main reason for people opposing the chlorine, fluorine and carbon.
introduction of Bt-Brinjal in India is that introducing
Bt brinjal will have an adverse effect on biodiversity ENVIRONMENT AND ECOLOGY
and possibility that the use of Bt brinjal may have
adverse effects on health. Apart from this, the farmers 86. The increasing amount of carbon dioxide in the
will also have the opposite effect on the labourers air is slowly raising the temperature of the
working in fields and other tree plants. atmosphere, because it absorbs:
83. Other than resistance to pests, what are the (a) the water vapour of the air and retains its heat.
prospects for which genetically engineered plants (b) the ultraviolet part of the solar radiation.
have been created ? (c) all the solar radiations.
1. To enable them to withstand drought. (d) the infrared part of the solar radiation.
2. To increase the nutritive value of the produce. Ans. (d): Among the Greenhouse gases (GHGs), only
3. To enable them to grow and do photosynthesis in water vapour has the ability to absorb both incoming
spaceships and space stations. (UV) and outgoing (infrared) radiation.
4. To increase their shelf life.
Select the correct answer using the codes given 87. In which one among the following categories of
below: protected areas in India are local people not
(a) 1 and 2 only (b) 3 and 4 only allowed to collect and use the biomass ?
(c) 1, 2 and 4 (d) 1, 2, 3 and 4 (a) Biosphere Reserve
Ans. (c) : All statements are correct. Theoretically, (b) National Park
there are no limits to the extent of modification (c) Wetlands declared under Ramsar Conventions
produced by genetic engineering. (d) Wildlife Sanctuaries
IAS (Pre) GS 2012 Paper I 184 YCT
Ans. (b): In the national parks of India, local people Current Affairs
are not allowed to accure and use livestock in reserved
91. Consider the following protected areas:
areas. The National Wildlife Action plan was launched 1. Bandipur 2. Bhitarkanika
in 1983. Harvesting, grazing and habitat damages are 3. Manas 4. Sunderbans
not allowed in the park areas, while the sanctuary has Which of the above are declared Tiger Reserves?
some leeway. (a) 1 and 2 only (b) 1, 3 and 4 only
88. The Millennium Ecosystem Assessment describes (c) 2, 3 and 4 only (d) 1, 2, 3 and 4
the following major categories of ecosystem Ans. (b) : Bhitarkanika (Odisha) is reserved for
services-provisioning, supporting, regulating, crocodile while Bandipur (Karnataka), Manas (Assam)
preserving and cultural. Which one of the and Sundarbans (West Bengal) are tiger reserves. The
'Project Tiger' was launched in India on 1st April 1973.
following is supporting service ?
A total of 42 tiger reserves have been created in the
(a) Production of food and water country so far (when the questions was asked).
(b) Control of climate and disease Currently, 52 tiger reserves are in India (June 2021).
(c) Nutrient cycling and crop pollination 92. The National Green Tribunal Act, 2010 was
(d) Maintenance of diversity enacted in consonance with which of the
Ans. (c) : According to the 'Millennium Ecosystem following provisions of the Constitution of India ?
Assessment, Nutrient cycling and pollination is in the 1. Right to healthy environment, construed as a part
category of supporting services. These services are of Right to life under Article 21.
essential for all other services and the basic sustenance 2. Provision of grants for raising the level of
of the ecosystem. The Millennium Ecosystem administration in the Scheduled Areas for the
welfare of Scheduled Tribes under Article 275
Assessment (MEA) was called for by the United (1).
Nations Secretary-General Kofi Annan in 2000. 3. Powers and functions of Gram Sabha as
Initiated in 2001, the objective of the MA was to mentioned under Article 243 (A.)
assess the consequences of ecosystem change for Select the correct answer using the codes given
human well-being and the scientific basis for action below:
needed to enhance the conservation and sustainable (a) 1only (b) 2 and 3 only
use of those systems and their contribution to human (c) 1 and 3 only (d) 1, 2 and
well-being. Ans. (a) : The National Green Tribunal Act, 2010
embraces the objective that the right to a healthy
89. Which of the following can be threats to the environment has been construed as a part of the right
biodiversity of a geographical area ? to life under Article 21 of the constitution in the
1. Global warming judicial pronouncement in India.
2. Fragmentation of habitat 93. If National Water Mission is properly and
3. Invasion of alien species completely implemented, how will it impact the
4. Promotion of vegetarianism country ?
Select the correct answer using the code given 1. Part of the water needs of urban areas will be met
below: through recycling of wastewater.
(a) 1, 2 and 3 only (b) 2 and 3 only 2. The water requirements of coastal cities with
(c) 1 and 4 only (d) 1, 2, 3 and 4 inadequate alternative sources of water will be
Ans. (a): Global warming, fragmentation of habitat met by adopting appropriate technologies that
and Invasion of alien species can be threats to the allow for the use of ocean water.
biodiversity of a geographical area. 3. All the rivers of Himalayan origin will be linked
to the rivers of peninsular India.
90. Consider the following agricultural practices : 4. The expenses incurred by farmers for digging,
1. Contour bunding bore-wells and for installing motors and pump-
2. Relay cropping sets to draw groundwater will be completely
3. Zero tillage reimbursed by the Government.
Select the correct answer using the codes given
In the context of global climate change, which of below:
the above helps/help in carbon sequestration/ (a) 1 only (b) 1 and 2 only
storage in the soil? (c) 3 and 4 only (d) 1, 2, 3 and 4
(a) 1 and 2 only (b) 3 only
Ans. (b): If National Water Mission is properly and
(c) 1, 2 and 3 (d) None of them
completely implemented, then the part of the water needs
Ans. (b): In the context of global climate change, of urban areas will be met through recycling of waste
contour bunding, relay cropping and zero tillage all water. The water requirements of coastal cities with
three methods are helpful in the storage of carbon in inadequate alternative sources of water will be met by
the soil. But the Union Public Service Commission has adopting appropriate technologies that allow for the use
considered the answer to this question as an option (b). of ocean water.
IAS (Pre) GS 2012 Paper I 185 YCT
94. With reference to consumers rights/privileges • Environment and Ecology
under the provisions of law in India, which of the
following statements is/are correct ? 97. Which one of the following groups of animals
1. Consumers are empowered to take samples for belongs to the category of endangered species?
food testing. (a) Great Indian Bustard , Musk Deer, Red Panda
2. When a consumer files a complaint in any and Asiatic Wild Ass.
consumer forum, no fee is required to be paid. (b) Kashmir Stage, Cheetal, Blue Bull and Great
3. In case of death of a consumer, his /her legal heir Indian Bustard.
can file a complaint in the consumer forum in (c) Snow Leopard, Swamp Deer, Rhesus Monkey
his/her behalf. and Saras (Crane).
Select the correct answer using the codes given (d) Lion-tailed Macaque, Blue Bull, Hanuman
below: Langur and Cheetal.
(a) 1 only (b) 2 and 3 only Ans. (a): The Great Indian Bustard, Musk Dear, Red
(c) 1 and 3 only (d) 1, 2 and 3 Panda and Asiatic Wild Ass come under the category
Ans. (c) : The Consumer Protection Act, 1986 was of endangered species. Some such species of fauna
passed by the Government of India in the year 1986 for have not disappeared completely but are on the verge
the protection of the interests of consumers. Under its of extinction. Such as African elephant, vulture,
provisions, consumers have the right to take samples porcupine, black beer, reindeer, etc., are prominent.
for testing the food. If a disturbance is found, the 98. Vultures which used to be very common in Indian
Consumer can file his complaint in the consumer countryside some years ago are rarely seen
Forum. If the litigant consumer dies during the nowadays. This is attributed to-
litigation, then his legal heir can file his complaint in (a) the destruction of their nesting sites by new
the consumer forum on his/her behalf. invasive species.
(b) a drug used by cattle owners for treating their
• Indian Heritage and Culture diseased cattle.
95. With reference to Dhrupad, one of the major (c) scarcity of food available to them.
traditions of India that has been kept alive for (d) a widespread, persistent and fatal disease among
centuries, which of the following statements are them.
correct ? Ans. (b) : Until a few years ago, vultures used to
1. Dhrupad originated and developed in the Rajput appear in the Indian countryside but are rarely seen
kingdoms during the Mughal period. nowadays. The main reason for this is the drug
2. Dhrupad is primarily a devotional and spiritual (diclofenac) used by cattle owners for the treatment of
music. sick animals. About 100 million vultures have been
3. Dhrupad Alap uses Sanskrit syllables from killed in India and Pakistan in the last decade as a
Mantras. result of the use of diclofenac medicine.
Select the correct answer using the codes given
below: 99. Consider the following
(a) 1 and 2 only 1. Black-necked crane 2. Cheetah
(b) 2 and 3 only 3. Flying squirrel 4. Snow leopard
(c) 1, 2 and 3 Which of the above are naturally found in India?
(d) None of the above is correct (a) 1, 2 and 3 only (b) 1, 3 and 4 only
(c) 2 and 4 only (d) 1, 2, 3 and 4
Ans. (c) : Dhrupad is the oldest surviving form of
Indian classical music and traces its origin to the Ans. (b): Black-necked cranes, flying squirrels and
chanting of Vedic hymns and mantras during the Vedic snow leopards are found naturally in India. There are
age. Most Dhrupads are religious in nature praising about 1500000 fauna species in the world, of which
Hindu Gods (originally sung in temples), although there are 250000 flora species. Among all the world’s
some texts praise the kings. A Dhrupad performance animal species 91212 amphibian and insect species are
starts with the alap, which uses sacred Sanskrit found only in North India. The Cheetah is the only
syllables from Sanskrit Mantras. animal that is currently on the verge of extinction.
96. How do you distinguish between Kuchipudi and 100. Government of India encourages the cultivation
Bharatanatyam dances ? of ‘sea buckthorn’. What is the importance of
1. Dancers occasionally speaking dialogues is found this plant ?
in Kuchipudi dance but not in Bharatanatyam. 1. It helps in controlling soil erosion and in
2. Dancing on the brass plate by keeping the feet on preventing desertification.
2. It is a rich source of biodiesel.
its edges is a feature of Bharatanatyam but 3. It has nutritional value and is well-adapted to live
Kuchipudi dance does not have such a form of in cold areas of high altitudes.
movements. 4. Its timber is of great commercial value.
Which of the statements given above is/are correct? Which of the statements given above is/are
(a) 1 only (b) 2 only correct ?
(c) Both 1and 2 (d) Neither 1 nor 2 (a) 1 only (b) 2, 3 and 4
Ans. (a) : 'Bharatanatyam' is primarily a dance form of (c) 1 and 3 (d) 1, 2, 3 and 4
Tamil Nadu and has now taken PAN India form. Ans. (c) : The Indian government is encouraging 'Sea
Kuchipudi has its roots in Andhra Pradesh. Tambulam buckthorn' farming. The farming of this plant is helpful
(brass plate) is not used in Bharatanatyam, whereas in in controlling soil erosion and prevents desertification
Kuchipudi dance, the dancers use Tambulam in and has nutritional value and is well adapted to survive
popular form. in high altitude cold areas.
IAS (Pre) GS 2012 Paper I 186 YCT
UNION PUBLIC SERVICE COMMISSION
Civil Services (Preliminary Exam) - 2011
GENERAL STUDIES : PAPER-I
Time: 2 hours (Exam date : 12.06.2011) Maximum Number: 200

ANCIENT HISTORY • Buddhism, Jainism, Bhagavata,


• Indus Valley Civilization & Vedic Shaiva and Other Religion
3. The Jain philosophy holds that the world is
Civilization
created and maintained by -
1. Regarding the Indus Valley Civilization, consider (a) Universal law (b) Universal Truth
the following statements: (c) Universal Faith (d) Universal Soul
1. It was predominantly a secular civilization, Ans. (a) : Jain philosophy believes that the whole
and the religious element, though present, world is made up of two continual and independent
did not dominate the scene.
elements called Jiva and Ajiva. Jiva is a "living
2. During this period, cotton was used for substance" or sentient, while Ajiva is an "inanimate
manufacturing textiles in India. substance" or insentient. Here, Jiva means individual
Which of the statements given above is/are correct? soul and not the universal soul. According to Jainism,
(a) 1 only souls are many and creatures live in every particle of
the universe. Therefore, according to Jain philosophy,
(b) 2 only the creation and maintenance of the universe are done
(c) Both 1 and 2 by Universal Law.
(d) Neither 1 nor 2
• Revolt of 1857 and Mass Movement of
Ans. (c): The religious outlook of the Indus Valley
civilization was primarily idyllic and customary. In this 19th Century
civilization, there is evidence of worship of many 4. Which amongst the following provided a common
animals, birds, vegetation, trees etc., along with the
factor for tribal insurrection in India in the 19th
worship of the Mother Goddess and the male deity. The
Indus valley people knew the first knowledge of the century?
method of growing cotton. The Greeks named it 'Sindon'. (a) Introduction of a new system of land revenue
and taxation of tribal products.
2. The ‘’dharma’’ and ‘‘rita’’ depict a central idea
(b) Influence of foreign religious missionaries in
of ancient Vedic civilization of India. In this
tribal areas.
context, consider the following statements:
(c) Rise of a large number of money lenders,
1. Dharma was a conception of obligations and traders and revenue farmers as intermediaries in
of the discharge of one’s duties to oneself and tribal areas.
to others. (d) Complete disruption of the old agrarian
2. Rita was the fundamental moral law order of the tribal communities.
governing the functioning of the universe and Ans. (d) : The main reason for the 19th Century tribal
all it contained. movement in India were as follows– (i) The
Which of the statements given above is/are correct? government weakened their traditions by granting
(a) 1 only zamindar status to the chieftains of the tribes and by
implementing the new method of rest. (ii) Infiltration
(b) 2 only
of Christian missionaries by the government in tribal
(c) Both 1 and 2 areas, (iii) Among the tribals, a group of Mahajan,
(d) Neither 1 nor 2 Merchants and Revenue Collectors, who were in the
role of middlemen, were brought, (iv) To tighten
Ans. (c): The Rigveda, whose reign was from 1500
government control in forest areas, banned the use of
BC to 1000 BC. The word ‘dharma’ relates to
morality. The meaning of 'Rita' is the moral and the forest as fuel & livestock and ban on jhoom and
material system of the world. fallow farming.

IAS (Pre) GS 2011 Paper I 187 YCT


• Land Revenue Systems in British 8. What was the purpose with which Sir William
Wedderburn and W.S. Caine had set up the
India/Freedom Struggle and National
Indian Parliamentary Committee in 1893 ?
Movement (a) To agitate for Indian political reforms in the
5. The tendency for increased Litigation was visible House of Commons
after the introduction of the land settlement (b) To campaign for the entry of Indians into the
system of Lord Cornwallis in 1793. The reason imperial Judiciary
for this is normally traced to which of the (c) To facilitate a discussion on India’s
following provisions?
Independence in the British Parliament
(a) Making Zamindar’s position stronger vis-a-vis
(d) To agitate for the entry of eminent Indians into
the ryot
(b) Making East India Company an overlord of the British Parliament .
Zamindars Ans. (a): In 1893, the Indian Parliamentary
(c) Making judicial system more efficient Committee was setup by Sir William Wedderburn and
(d) None of the above WS Caine to organize an agitation in the House of
Ans. (d) : The Permanent Settlement was an agreement Commons for political reforms in India.
between the East India Company and Bengali
9. Mahatma Gandhi said that some of his deepest
landlords to fix revenues to be raised from land. It was
convictions were reflected in a book titled, “Unto
concluded in 1793 by the Company administration
this last” and the book transformed his life. What
headed by Charles Earl Cornwallis. The other two
systems prevalent in India were the Ryotwari system was the message from the book that transformed
and the Mahalwari system. The correct option is (d) Mahatma Gandhi?
(a) Uplifting the oppressed and poor is the moral
6. Which one of the following observations is not
responsibility of an educated man.
true about the Quit India Movement of 1942?
(a) It was a non–violent movement (b) The good of an individual is contained in the
(b) It was led by Mahatma Gandhi good of all
(c) It was a spontaneous movement (c) The life of celibacy and spiritual pursuit are
(d) It did not attract the labour class in general essential for a noble life.
Ans. (b) : The Quit India Movement started on 9 (d) All the statements (a), (b) and (c) are correct in
August, 1942. Initially this movement was non - this context.
violent. Most of popular leaders were arrested and Ans. (b) : Mahatma Gandhi had read John Ruskin's
imprisoned, and movement became leaderless this book 'Unto this Last' during his stay in Africa. This
movement led to the end of British Rule in India. book changed Gandhiji's life. The message in this book
7. What was the reason for Mahatma Gandhi to that changed the life of Gandhi was that the welfare of
organize a satyagraha on behalf of the peasants the individual lies in the welfare of all.
of Kheda?
1. The Administration did not suspend the land 10. With reference to Indian freedom struggle, Usha
revenue collection in spite of a drought. Mehta is well-known for –
2. The Administration proposed to introduce (a) Running the secret Congress Radio in the wake
Permanent Settlement in Gujarat. of Quit India Movement
Which of the statements given above is/are correct? (b) Participating in the Second Round table
(a) 1 only (b) 2 only
(c) Both 1 and 2 (d) Neither 1 nor 2 Conference
(c) Leading a contingent of Indian National Army
Ans. (a): At the beginning of 1917 and 1918, Gandhiji
(d) Assisting in the formation of Interim
participated in three struggles i.e. of Champaran Government under Pandit Jawaharlal Nehru
Satyagraha (Bihar), Ahmedabad and Kheda (Gujarat).
Ans. (a) : Quit India movement started on 9 August,
Gandhiji came to know that the farmers of the Kheda
1942; during this movement, secret radio was
district are in a lot of trouble. The crop is ruined, yet
the government is recovering tax from them, the broadcast by the Congress in many areas of Mumbai.
farmers demanded for tax exemption, but the While Dr. Ram Manohar Lohia used to broadcast on
government refused. This was the reason for Mahatma this radio, Usha Mehta was a member of a small
Gandhi to organize Satyagraha. organization operating underground radio.

IAS (Pre) GS 2011 Paper I 188 YCT


11. With reference to the period of Indian freedom 1. La Nina is characterized by unusually cold
struggle, which of the following was/were ocean temperature in equatorial Indian Ocean
recommended by the Nehru report ? whereas El Nino is characterized by unusually
1. Complete Independence for India. warm ocean temperature in the equatorial
2. Joint electorates for reservation of seats for Pacific Ocean.
minorities . 2. El Nino has adverse effect on south–west
3. Provision of fundamental rights for the people monsoon of India, but La Nina has no effect on
of Indian in the Constitution. monsoon climate.
Select the correct answer using the codes given Which of the statement given above is/are correct?
below: (a) 1 only (b) 2 only
(a) 1 only (b) 2 and 3 only (c) Both 1 and 2 (d) Neither 1 nor 2
(c) 1 and 3 only (d) 1, 2 and 3 Ans. (d): La Nina is a cold current flowing across the
Ans. (b): On 11 May, 1928, at the Second All Party west coast of South America. It is characterized by the
low and high pressure prevailing in the Pacific Ocean.
Conference in Mumbai, a seven-member committee
Low pressure in that region bring goods monsoon to
was set up under the chairmanship of Pt. Motilal Nehru
the Indian subcontinent.
to determine the principles of the Constitution of India.
The recommendation made in the report were as 14. Salinisation occurs when the irrigation water
follows– accumulated in the soil evaporates, leaving
(i) Dominion status to India. behind salts and minerals. What are the effects of
(ii) India will be a union under whose control there salinization on the irrigated land ?
will be a bicameral legislature at the centre, the (a) It greatly increases the crop production
cabinet will be responsible to the house. (b) It makes some soils impermeable
(iii) The position of Governor- General shall be that of (c) It raises the water table
a constitutional head. (d) It fills the air spaces in the soil with water
(iv) The demand for a separate electorate on the Ans. (b): The saline of the soil is generated from the
communal ground was rejected. salt and minerals left behind due to evaporation of the
(v) Fundamental rights are defined by defining irrigated water collected in the soil. Soil becomes
citizenship. inaccessible due to salinization on the irrigated land.
15. The lower Gangetic plain is characterized by
GEOGRAPHY OF INDIA humid climate with high temperature throughout
the year. Which one among the following pairs of
• Physical Structure/Drainage System/ crops is most suitable for this region ?
Human Geography (a) Paddy and Cotton (b) Wheat and Jute
12. Two important rivers–one with its source in (c) Paddy and Jute (d) Wheat and Cotton
Jharkhand (and known by a different name in Ans. (c) : The most important feature of the lower
Odisha), and another, with its source in Odisha gangetic plains is that the climate here remains humid
merge at a place only a short distance from the with high temperatures throughout the year, making
coast of Bay of Bengal before flowing into the this area the most suitable for paddy and jute crops.
Sea. This is an important site of wildlife and 16. The Brahmaputras, Irrawady and Mekong rivers
biodiversity and a protected area. Which one of originate in Tibet and flow through narrow and
the following could be this? parallel mountain ranges in their upper reaches.
(a) Bhitarkanika (b) Chandipur-on-sea
(c) Gopalpur-on-sea (d) Simlipal Of these rivers. Brahmaputra makes a ‘‘U’’ turn
in its course to flow into India .This ‘‘U’’ turn is
Ans. (a): The Bhitarkanika Mangroves are a
mangrove wetland in India's Odisha state. The due to :
2 (a) Uplift of folded Himalayan series
Bhitarkanika Mangroves cover an area of 650 km in
the river delta of the Brahmani and Baitarani rivers. (b) Syntaxial bending of geologically young
Himalayas’
13. La Nina is suspected to have caused recent floods
(c) Geo-tectonic disturbance in the tertiary folded
in Australia. How is La Nina different from El mountain chains
Nino. (d) Both (a) and (b)
IAS (Pre) GS 2011 Paper I 189 YCT
Ans. (b): The Brahmaputra, Irrawaddy and Mekong Ans. (b) : The state of Arunachal Pradesh has the best
rivers have their origins in Tibet. They flow through climatic conditions, which can lead to the cultivation
narrow and parallel mountain ranges. The of various varieties of orchids and export- oriented
Brahmaputra takes a U-turn before entering India; this industries can be developed here. Arunachal Pradesh is
U-turn is due to syntaxial bending of the geologically emerging as a major tourist region in north-eastern
young Himalayas. India. It is considered to be a treasure of nature and
home to a variety of orchids.
17. A state in India has the following characteristics :
1. Its northern part is arid and semiarid.
2. Its central part produces cotton. WORLD GEOGRAPHY
3. Cultivation of cash crops is predominant over
food crops. • Universe and Solar System
Which one of the following states has all of the 20. An artificial satellite orbiting around the Earth
above characteristics? does not fall down. This is so because the
(a) Andhra Pradesh (b) Gujarat attraction of Earth–
(c) Karnataka (d) Tamil Nadu (a) Does not exist at such distance.
Ans. (b): The northern part of the state of Gujarat is (b) Is neutralized by the attraction of the moon.
dry and semi-arid. Cotton is grown in its Central (c) Provides the necessary speed for its steady
region and here, cash or commercial crops are motion.
cultivated more than food crops. Its present existence (d) Provides the necessary acceleration for its
(Gujarat) came on 1 May, 1960. It is situated on the motion.
west coast of India. The main crops grown here are Ans. (d) : We observe weightlessness in satellites
Cotton and Groundnut. because the net magnitude of the force on the body is
almost zero. i.e. earth's gravitation is nullified in terms
• Monsoon, Forest, Soil, Irrigation
of magnitude by centrifugal force. But, earth's
Projects and Agriculture gravitation is necessary to change the direction of the
18. With reference to micro-irrigation , which of the satellite continuously and thus providing the
following statements is/are correct ? acceleration in terms of change in direction.
1. Fertilizer/nutrient loss can be reduced 21. What is the difference between asteroids and
2. It is the only means of irrigation in dry land comets?
farming 1. Asteroids are small rocky planetoids, while
3. In some areas of farming, receding of ground comets are formed of frozen gases held
water table can be checked together by rocky and metallic material.
Select the correct answer using the codes given 2. Asteroids are found mostly between the orbits
below : of Jupiter and Mars, while comets are found
(a) 1 only (b) 2 and 3 only mostly between Venus and Mercury.
(c) 1 and 3 only (d) 1, 2 and 3 3. Comets show a perceptible glowing tail, while
Ans. (c): The Micro irrigation system is one of the asteroids do not.
excellent modern techniques of irrigation. This can Which of the statements given above is/are correct?
reduce fertilizer/nutrient loss from the soil and prevent (a) 1 and 2 only (b) 1 and 3 only
groundwater levels in some agricultural areas. (c) 3 only (d) 1, 2 and 3
Ans. (b): The main difference between asteroids and
• Minerals, Power Resources, comets lies mainly in their composition. Both were
Industries and Trade formed during the early phase of the solar system.
Asteroids mainly consist of rocky and metallic
19. Among the following State, which one has the
material, while comets comprise ice, frozen gases, dust
most suitable climatic conditions for the
and rocky and metallic material. Asteroids are found
cultivation of a large variety of orchids with nearer to the Sun, while comets are found farther from
minimum cost of production, and can develop the Sun as the heat can melt any comet's ice. In fact,
and export oriented industry in this field ? when a comet approaches the Sun, it starts losing its
(a) Andhra Pradesh (b) Arunachal Pradesh ice because of the Sun's heat, as a result of which a
(c) Madhya Pradesh (d) Uttar Pradesh perceptible glowing tail can be seen.

IAS (Pre) GS 2011 Paper I 190 YCT


• Lithosphere 25. The 2004 Tsunami made people realize that
mangroves can serve as a reliable safety hedge
22. Westerlies in southern hemisphere are stronger against coastal calamities. How do mangroves
and persistent than in northern hemisphere. Why? function as a safety hedge ?
1. Southern hemisphere has less landmass as (a) The mangrove swamps separate the human
compared to northern hemisphere. settlements from the sea by a wide zone in which
2. Coriolis force is higher in southern hemisphere people neither live nor venture out.
as compared to northern hemisphere. (b) The mangroves provide both food and medicines
Which of the statements given above is/are correct? which people are in need of after any natural
(a) 1 only (b) 2 only disaster.
(c) Both 1 and 2 (d) Neither 1 nor 2 (c) The mangrove trees are tall with dense canopies
Ans. (a) : The higher speed and the greater persistence and serve as an excellent shelter during a
of the westerlies in the Southern Hemisphere are cyclone or tsunami
caused by the difference in the atmospheric pressure (d) The mangrove trees do not get uprooted by
patterns as well as its variation from that of the storms and tides because of their extensive roots
Northern Hemisphere. The landmass in the southern Ans. (d) : Mangrove forests stabilize the coastline
hemisphere is comparatively less and average annual reducing erosion from surf waves, current waves and
pressure decreases much more rapidly on the pole tides. The intricate root system of mangroves also
ward side of the high- pressure belt .Hence statement makes these forests attractive to fishes and other
(a) is correct. organisms seeking food and shelter from predators.
As far as Coriolis force is concerned, it is strongest in
the polar regions and zero at the equator. At the 26. What could be the main reason/reasons for the
intermediate level, it varies directly as the sine of the formation of Africa and Eurasian desert belt ?
latitude. 1. It is located in the sub-tropical high pressure
cells.
23. Between India and East Asia, the navigation-time 2. It is under the influence of warm ocean
and distance can be greatly reduced by which of currents.
the following? Which of the statements given above is/are
1. Deepening the Malacca straits between correct in this context?
Malaysia and Indonesia. (a) 1 only (b) 2 only
2. Opening a new canal across the Kra isthmus (c) Both 1 and 2 (d) Neither 1 nor 2
between the Gulf of Siam and Andaman Sea.
Ans. (a): The distance from the reach of the trade winds,
Which of the statements given above is /are correct?
the location of the continent or the western edge of the
(a) 1 only (b) 2 only
(c) Both 1 and 2 (d) Neither 1 nor 2 site and the effect of cold currents is the main reason for
the creation of the African and Eurasian desert region.
Ans. (b): Navigation time and distance between India
Apart from this, the location in sub-tropical high- pressure
and East Asia can be reduced by opening a new canal
across the Kra isthmus between the Gulf of Siam and cells is also an important reason.
the Andaman Sea. • Hydrosphere
24. Consider the following statements : 27. Southeast Asia has captivated the attention of
1. Biodiversity is normally greater in the lower global community over space and time as a
latitudes as compared to the higher latitudes. geostrategically significant region. Which among
2. Along the mountain gradient, biodiversity is
the following is the most convincing explanation
normally greater in lower altitudes as
for this global perspective ?
compared to the higher altitudes.
(a) It was the hot theatre during the Second World
Which of the statements given above is/are correct?
War.
(a) 1 only (b) 2 only
(c) Both 1 and 2 (d) Neither 1 nor 2 (b) Its location is between the Asian Powers of
China and India.
Ans. (c) : In mountain regions, biodiversity is generally
(c) It was the arena of superpower confrontation
higher in lower elevations than in higher altitudes and
during the Cold War period.
biodiversity is generally higher in lower latitudes than (d) Its location between the Pacific and Indian
in higher latitudes. oceans and its pre-eminent maritime character.

IAS (Pre) GS 2011 Paper I 191 YCT


Ans. (d) : As a geostrategic ally important area, South- (a) 1 only (b) 1 and 2 only
East Asia attracts the attention of the global (c) 3 only (d) 1, 2 and 3
community over a long period of time and is situated
Ans. (d) : Right to Education (Article 26), Right to
between the Pacific Ocean and the Indian Ocean and
equal access to public service [Article 21] and Right to
has an excellent pre-eminent maritime character.
food [Article 25] comes under the Universal
28. The surface of a lake is frozen in severe winter,
Declaration of Human Rights.
but the water at its bottom is still liquid. What is
the reason ? 31. In India, if a religious sect/ community is given
(a) Ice is a bad conductor of heat the status of a national minority, what special
(b) Since the surface of the lake is at the same advantages it is entitled to ?
temperature as the air, no heat is lost. 1. It can establish and administer exclusive
(c) The density of water is maximum at 40C. education institutions.
(d) None of the statements (a), (b) and (c) given
2. The President of India automatically
above is correct.
nominates a representative of the community
Ans. (c): The surface of a lake is frozen in severe
to Lok Sabah.
winter but the water at its bottom is still liquid because
the density of water is maximum at 4°C. The surface 3. It can derive benefits from the Prime
water freezes at 0°C, but due to the temperature of the Minister’s 15 Point Programme.
water below the surface being 4°C, the upper glacial Select the correct answer using the code given below
surface of the lake acts as an insulator and does not (a) 1 only (b) 2 and 3 only
allow the heat present in the water to go out. (c) 1 and 3 only (d) 1, 2 and 3

POPULATION AND Ans. (c): Minority education has the power to reserve
only up to 50% of seats for students belonging to its
URBANIZATION own community (Supreme Court Judgments on St.
29. India is regarded as a country with Demographic Stephens College Case, 1992, S.C.C. 550).
Dividend. This is due to –
(a) Its high population in the age group below 15 • Executive of Center and States,
years.
(b) Its high population in the age group of 15-64
Emergency Provisions
years. 32. The authorization for the withdrawal of funds
(c) Its high population in the age group above 65 from the Consolidated fund of India must come
years. from –
(d) Its high total population.
(a) The President of India
Ans. (b): India is counted as a country with the (b) The Parliament of India
demographic dividend. The reason for this is that the
(c) The Prime Minister of India
population in the age group of 15-64 years is more and
the number of children and old people is less. (d) The Union Finance Minister
Ans. (b) : Approval of the parliament of India is
INDIAN CONSTITUTION Mandatory to withdraw funds from the Consolidated
Fund of India. The Consolidated Fund of India is the
AND POLITY axis of parliamentary control regarding expenditure. It
is a reservoir in which all the revenue received by the
• Citizenship, Fundamental Rights, Government of India is deposited and all its borrowing
Fundamental Duties, Directive will be paid through it. According to Article 266 (3),
Principles of State Policies funds from consolidated fund of India will be spent
according to law.
30. Consider the following 33. All revenues received by the Union Government
1. Right to education by way of taxes and other receipts for the conduct
2. Right to equal access to public service of Government business are credited to the –
3. Right to food (a) Contingency fund of India
Which of the above is/are Human Right/ Human (b) Public Account
Rights under Universal Declaration of Human (c) Consolidated fund of India
Rights? (d) Deposits and Advances Fund
IAS (Pre) GS 2011 Paper I 192 YCT
Ans. (c): All revenues received by the Union 1. Free schooling till the age of 18 years in
Government by way of taxes and other receipts for the government-run schools.
conduct of government business are credited to the 2. Preferential allotment of land for setting up
consolidated fund of India. Consolidated Fund of India business.
is the most important of all government accounts. This 3. Ramps in public buildings.
fund was constituted under Article 266 (1) of the Which of the statements given above is/are correct?
Constitution of India. All revenues received by the (a) 1 only (b) 2 and 3
government by way of direct taxes and indirect taxes, (c) 1 and 3 only (d) 1, 2 and 3
money borrowed as receipts from loans given to the Ans. (d) : All the above provisions are mentioned in
government flow into the Consolidated Fund of India. the persons with Disabilities (equal opportunities,
protection of rights and full participation) Act, 1995.
• Legislature : Council of Ministers 37. With reference to Aam Admi Bima Yojana
34. Under the constitution of India, Which one of the Consider the following statements :
following is not a fundamental duty? 1. The member insured under the scheme must
(a) To vote in public elections be the head of the family or an earning
(b) To develop the scientific temper member of the family in a rural landless
house-hold.
(c) To safeguard public property
2. The member insured must be in the age group
(d) To abide by the constitution and respect its
of 30 to 65 years.
ideals 3. There is a provision for free scholarship for up
Ans. (a) : Under the constitution of India, to vote in to two children of the insured who are
public elections is not a fundamental duty. studying between classes 9 and 12.
Which of the statements given above is/are correct?
• Panchayati Raj System, (a) 1only (b) 2 and 3 only
Constitutional Amendments and (c) 1 and 3 only (d) 1, 2 and 3
Schedules, Acts Ans. (c) : Aam Admi Bima Yojna (AABY) is a
government of India social security scheme
35. The Constitution (Seventy-Third Amendment) administered through Life Insurance Corporation of
Act, 1992, which aims at promoting the India (LIC) that provides death and disability cover to
Panchayati Raj Institutions in the country, the head of the family between the age group of 18 to
provides for which of following? 59 years, under 48 identified vocational/occupational
1. Constitution of District Planning Committees. groups/rural landless households. The scheme also
2. State Election Commissions to conduct all provides an add-on-benefit wherein scholarship of Rs.
panchayat elections. 100 month per child is paid on a half-yearly basis to a
3. Establishment of State Finance Commissions. maximum of two children per member, studying in 9th
Select the correct answer using the codes to 12th standard.
given below
38. Among the following who are eligible to benefit
(a) 1 only (b) 1 and 2 only
from the Mahatma Gandhi National Rural
(c) 2 and 3 only (d) 1, 2 and 3
Employment Guarantee Act?
Ans. (*) : District Planning Committee (DPC) is a (a) Adult members of only the scheduled caste and
constitutional Body mentioned in 74th constitutional scheduled tribe households.
amendment 1992, article 243 (ZD). The committee (b) Adult members of below poverty line (BPL)
composed of elected member of local bodies of urban households.
or rural areas, and nominated members within districts. (c) Adult members of households of all backward
State Election commission in a constitutional body that communities.
comes under the article 243 (K). (d) Adult members of any household.
State finance Commission is also a constitutional body Ans. (d) : Mahatma Gandhi National Rural
that comes under article 243(I). Employment Guarantee Act is an Indian Labour law and
social security measure that aims to guarantee the 'right
Hence option (c) should be correct answer.
to work'. This Act was passed in September 2005.The
Note : According to UPSC official Answer Key (d) is MGNREGA was initiated with the objective of
correct answer. "enhancing livelihood security in rural areas by providing
36. India is a home to lakh of persons with at least 100 days of guaranteed wage employment in a
disabilities. What are the benefits available to financial year, to every household whose adult members
them under the law? volunteer to do unskilled manual work".
IAS (Pre) GS 2011 Paper I 193 YCT
39. Consider the following statements : (a) 1 only (b) 2 only
In India, a Metropolitan Planning Committee – (c) Both 1 and 2 (d) Neither1 and nor 2
1. Is constituted under the provisions of the Ans. (d): Except for the slowdown in the year 2008-
Constitution of India. 09, the Indian economy has consistently grown and at
2. Prepares the draft development plans for incrementing rate.
metropolitan area.
3. Has the sole responsibility for implementing 43. In India, which of the following have the highest
Government sponsored schemes in the share in the disbursement of credit to agriculture
metropolitan area. and allied activities?
Which of the statements given above is/are correct? (a) Commercial Banks
(a) 1 and 2 only (b) 2 only
(b) Cooperative Banks
(c) 1 and 3 (d) 1, 2 and 3
(c) Regional Rural Banks
Ans. (a): Article 243 ZE of part IX-A of the
(d) Microfinance Institutions.
constitution of India mandates the formation of
Metropolitan Planning Committees (MPCs) Ans. (a) : Commercial banks account for the largest
Accordingly, in each metropolitan area, a Metropolitan share of expenditure in agriculture and allied sectors.
Planning Committee will be set up to prepare a Commercial banks provided direct loans of Rs. 140380
development plan for the entire metropolitan area. crore to the agricultural sector along with the regional
40. With reference to the Finance Commission of rural banks in 2006-07, while the regional rural bank
India, which of the following statements is correct? provided an annual loan of about Rs. 20440 crore.
(a) It encourages the inflow of foreign capital for
infrastructure development. • Economic Planning & National
(b) It facilitates the proper distribution of finances
among the Public Sector Undertakings.
Income /Budget
(c) It ensures transparency in financial 44. In the Union Budget 2011-12, a full exemption
administration. from the basic customs duty was extended to the
(d) None of the statements (a), (b) and (c) given bio-based asphalt (bioasphalt). What is the
above is correct in this context. importance of this material?
Ans. (d): The President of India established the 1. Unlike traditional asphalt, bio-asphalt is not
Finance Commission of India in 1951, under Article based on fossil fuels.
280 of the Indian Constitution. It was formed with an
2. Bioasphalt can be made from non-renewable
objective to define the financial relations between the
central government of India and the state governments. resources.
There have been fifteen finance commissions till date. 3. Bioasphalt can be made from organic waste
The most recent was constituted in 2017 and is chaired materials.
by N.K. Singh. 4. It is eco-friendly to use bioasphalt for
surfacing of the roads.
ECONOMICS Which of the statements given above are correct
41. A ‘closed economy’’ is an economy in which – (a) 1, 2 and 3 (b) 1, 3 and 4
(a) the money supply is fully controlled. (c) 2 and 4 (d) 1, 2, 3 and 4
(b) deficit financing takes place. Ans. (b): Bio-asphalt is an alternative to conventional
(c) only exports take place. asphalt, which is not manufactured based on fossil
(d) neither exports nor imports take place. fuels. It is manufactured from non-petroleum based
Ans. (d) : A closed economy is an economy where renewable sources. These sources include maize,
there is neither import nor export. That is, a closed paddy, sugar, molasses, waste of white mustard oil,
economy is an economy that keeps off the import and waste of palm oil, lignin etc. Bio-asphalt is helpful in
export of goods and services. Hence, (d) is correct. keeping the surface temperature of the roads low.
Therefore, laying the top surface of roads with bio-
42. In the context of Indian economy, consider the
asphalt is eco-friendly.
following statements.
1. The growth rate GDP has steadily increased in 45. Both Foreign Direct Investment (FDI) and
the last five years. Foreign Institutional Investment (FII) are related
2. The growth rate in per capita income has to investment in a country. Which one of the
steadily increased in the last five years. following statements best represents an
Which of the statements given above is/are correct ? important difference between the two ?
IAS (Pre) GS 2011 Paper I 194 YCT
(a) FII helps bring better management skills and • Agriculture, Industry and Trade
technology, while FDI only brings in capital .
(b) FII helps in increasing capital availability in 48. Which of the following can aid in furthering the
Government’s objective of inclusive growth ?
general, while FDI only targets specific sectors.
1. Promoting Self-Help Groups
(c) FDI flows only into the secondary market, while
FII targets primary market. 2. Promoting Micro, Small & Medium
Enterprises
(d) FII is considered to be more stable than FDI.
3. Implementing the Right to Education Act
Ans. (b): FDI only targets a specific enterprise. It aims
Select the correct answer using the codes given
to increase the enterprises capacity or productivity or
below:
change its management control. The FII investment
(a) 1 only (b) 1 and 2 only
flows only into the secondary market. It helps in
(c) 2 and 3 only (d) 1, 2 and 3
increasing capital availability in general rather than
enhancing the capital of a specific enterprise. The FDI Ans. (d): Inclusive development is called development
is considered to be more stable than FII. FDI not only in which the people belonging to the most vulnerable
brings in capital but also helps is good governance section of the society get a fair share in the distraction
of national income generated at a high rate of
practice and better management skills and even
economic growth, i.e., the leakage of national income
technology transfer. is more downstream. Encouraging self-help groups,
46. When the annual Union Budget is not passed by promoting micro, small and medium enterprises and
the Lok Sabha – implementing the Right to Education Act can be
(a) The Budget is modified and presented again helpful to take forward the inclusive growth target of
(b) The Budget is referred to the Rajya Sabha for the government.
suggestions 49. A genetically engineered form of brinjal, known
(c) The union finance Minister is asked to resign as the Bt-brinjal, has been developed. The
(d) The Prime Minister submits the resignation of objective of this is –
Council of Ministers (a) To make it pest-resistant
(b) To improve its taste and nutritive qualities.
Ans. (d) : When the Lok Sabha does not pass the
(c) To make it drought-resistant
Annual Union Budget, the Prime Minister submits the
(d) To make its shelf-life longer
resignation of the Council of Ministers. If the budget is
not passed under any condition in Parliament, then it Ans. (a) : The Bt-brinjal has genetically modified
can be understood that the ruling party is in the strains to resist the pest bollworm and other pests,
especially against the lepidopteran insects.
minority. Technically, it means the government has
lost the confidence vote in the Lok Sabha and it has to 50. With what purpose is the Government of India
resign. In Indian history, this has not happened so far. promoting the concept of Mega Food Parks?
1. To provide good infrastructure facilities for
47. What is the difference between ‘‘vote–on- the food processing industry.
account’’ and ‘‘interim budget’’? 2. To increase the processing of perishable items
1. The provision of a vote-on-account is use by a and reduce wastage.
regular Government, while and interim budget is 3. To provide emerging and eco-friendly food
a provision used by a caretaker Government. processing technologies to entrepreneurs.
2. A vote-on-account only deals with the Select the correct answer using the codes given
expenditure in Government's budget while an below.
interim budget includes both expenditure and (a) 1 only (b) 1and 2 only
receipts. (c) 2 and 3 only (d) 1, 2 and 3
Which of the statements given above is/are correct? Ans. (b): The Eleventh Five Year Plan (2007-12) was
(a) 1 only (b) 2 only approved by the National Development Council on 19
(c) Both 1 and 2 (d) Neither 1 and nor 2 December, 2007.The"Mega Food Park" scheme was
launched by the Central Government in September,
Ans. (b) : Under Article 116 (a), Lok Sabha can grant
2008 under the five-year plan. The major objectives of
an advance amount to the government by a passing
the schemes are– (i) To increase the processing of
grant of accounts, about which it is not possible for the perishable materials from 6% to 20% and to reduce
government to give budget redress. It is related to the wastage. (ii) To provide innovative and eco-friendly
expenditure side of the budget of the government. The food processing for entrepreneurs. (iii) To provide
interim budget includes both expenditure and receipts value-added supply chain from work to market with
and can be presented by the caretaker before the excellent adequate infrastructure facilities for the food
election and the regular government after the election. processing industry.
IAS (Pre) GS 2011 Paper I 195 YCT
• Money/Banking, Tax System, (b) It is an intense affirmative action of the
Government to boost economic activity in the
Center- State Relation country.
51. India has experienced persistent and high food (c) It is Government’s intensive action on financial
inflation in the recent past. What could be the institutions of ensure disbursement of loans to
reason? agriculture and allied sectors to promote greater
1. Due to a gradual switchover to the cultivation food production and contain food inflation.
of commercial crops, the area under the (d) It is an extreme affirmative action by the
cultivation of food grains has steadily Government to pursue its policy of financial
decreased in the last five years by about 30%. inclusion.
2. As a consequence of increasing incomes, the Ans. (b) : 'Financial Stimulus' is a deeply
consumption patterns of the people have determinative action of the government which aims to
undergone a significant change. promote economic activities in the country.
3. The food supply chain has structural Government financial stimuli are provided to
constraints. overcome the slowdown of the economy or to save it
Which of the statements given above are correct? from the financial crisis. Under this, the save various
(a) 1 and 2 only (b) 2 and 3 only sectors of the economy are provided with financial
(c) 1 and 3 only (d) 1, 2 and 3 incentives to protect them from recession.

Ans. (b): Increased income has led to significant 54. Consider the following actions which the
changes in people's consumption patterns, leading to Government can take –
higher food inflation. Structural bottlenecks in supply 1. Devaluing the domestic currency.
chain demand for many food items have been one of 2. Reduction in the export subsidy.
the reasons for persistent and high food inflation in 3. Adopting suitable policies which attract
India in recent years. (According to UPSC 2011 greater FDI and more funds from FIIs.
commission's Answer key option (c) will be right Which of the above action/actions can help in
answer). reducing the current account deficit?
52. A company marketing food products advertises (a) 1 and 2 (b) 2 and 3
that its items do not contain trans-fats. What (c) 3 only (d) 1 and 3
does this campaign signify to the customers?
1. The food products are not made out of Ans. (d) : To reduce current account deficit,
hydrogenated oils. devaluation of domestic currency and adopting suitable
2. The food products are not made out of animal policies which attract greater FDI and more funds from
facts/oils. FIIs etc. are the actions which can be taken by a
3. The oils used are not likely to damage the government.
cardiovascular health of the consumers. 55. A rapid increase in the rate of inflation is
Which of the statements given above is/are above sometimes attributed to the base effect. What is
(a) 1 only (b) 2 and 3 only base effect ?
(c) 1 and 3 only (d) 1, 2 and 3 (a) It is the impact of drastic deficiency in supply
Ans. (c) : If a company marketing food product due to failure of crops.
advertises that its items do not contain transfats, the (b) It is the impact of the surge in demand due to
company wishes to inform the customers that the food rapid economic growth.
products are made out of animal fats/oils and (c) It is the impact of the price levels of previous
hydrogenated oils. The oil used are also not likely to year on the calculation of inflation rate.
damage the cardiovascular health of the consumers.
(d) None of the statements (a), (b) and (c) given
Hence, statement 1, and 3 is correct.
above is correct in this context.
53. Which one of the following statements
Ans. (c): The base effect is the effect of prior data on
appropriately describes the ‘‘fiscal stimulus’’?
the calculation of current data. The 'base effect' is the
(a) It is massive investment by the Government in
effect of the previous years prices on the calculation of
manufacturing sector to ensure the supply of
the current inflation rate in the context of an increase
goods to meet the demand surge cause by rapid
economic growth. in the inflation rate.

IAS (Pre) GS 2011 Paper I 196 YCT


56. Which one of the following is not a feature of 59. Why is the offering of ‘’teaser loans’’ by
Value Added Tax ? commercial banks a cause of economic concern?
(a) It is a multi-point destination-based system of 1. The teaser loans are considered to be an aspect
taxation. of sub-prime lending and banks may be
exposed to the risk of defaulters in future.
(b) It is a tax levied on value addition at each stage
2. In India, the teaser loans are mostly given to
of transaction in production-distribution chain. inexperienced entrepreneurs to set up
(c) It is a tax on the final consumption of goods or manufacturing or export units.
services and must ultimately be borne by the Which of the statements given above is /are correct?
consumer. (a) 1 only (b) 2 only
(d) It is basically a subject of the Central (c) Both 1 and 2 (d) Neither1 and nor 2
Government and the State governments are only Ans. (a): There are such Teaser loans offered by
commercial banks that are offered to the borrower
a facilitator for its successful implementation.
irrespective of the ability to choose the loan and other
Ans. (d): "Value-based tax" is a multi-point target parameters. The rate of interest on such loans is
based tax system. In this, a tax is levied on value attractively low in the initial years and increases in
addition at every stage of the transaction in the subsequent years.
production/distribution chain. It is a tax on the final 60. Economic growth is usually coupled with –
consumption of goods and services, which consumers (a) Deflation (b) Inflation
eventually have to bear. (c) Stagflation (d) Hyperinflation
57. Micro finance is the provision of financial Ans. (b): Inflation or currency expansion is the state
in which the value of a currency falls and prices rise.
services to people of low-income groups. This
Economic outlook committees and controlled inflation
includes both the consumers and the self- are beneficial to the underdeveloped economy as it
employed. The service/services rendered under increases production.
micro-finance is/are:
61. The lowering of Bank Rate by the Reserve Bank
1. Credit facilities of India leads to
2. Saving facilities (a) More liquidity in the market
3. Insurance facilities (b) Less liquidity in the market
4. Fund Transfer facilities (c) No change in the liquidity in the market
Select the correct answer using the codes given (d) Mobilization of more deposits by commercial
below the lists: bank
(a) 1 only (b) 1 and 4 only Ans. (a) : The liquidity in the market increases as a
result of the Reserve Bank of India reducing the bank
(c) 2 and 3 only (d) 1, 2, 3, and 4
rate. Bank rate cuts are intended to help reduce other
Ans. (d): Providing credit facilities to the rural poor, interest rates that encourage lending from banks.
especially women, for carrying out income- generating
activities without any collateral guarantee is called • Foreign Trade of India, Economic
micro-financing. The services provided under micro- Organizations & Share Market
finance are credit facilities, saving facilities, insurance 62. In terms of economy, the visit by foreign
facilities and fund transfer facilities. nationals to the XIX Common Wealth Games in
India amounted to –
58. Regarding the International Monetary Fund, (a) Export (b) Import
Which one of the following statements is correct? (c) Production (d) Consumption
(a) It can grant loans to any country Ans. (a): The income comes from "tourism" and that
(b) It can grant loans to only developed countries is an invisible export.
(c) It grants loans to only member countries
63. With reference to India, consider the following
(d) It can grant loans to the central bank of a country Central Acts :
Ans. (c): The International Monetary Fund (IMF) is an 1. Import and Export (Control) Act, 1947.
international monetary organization. It was established 2. Mining and Mineral Development (Regulation).
on 27 December, 1945 in Washington (United States) 3. Customs Act, 1962
4. India Forest Act, 1927
as decided by the Bretton Woods Conference. It
Which of the above Acts have relevance to/
provides loans only to member countries thereby bearing on the biodiversity conservation in the
correcting their adverse balance of payments. country?
IAS (Pre) GS 2011 Paper I 197 YCT
(a) 1 and 3 only (b) 2, 3 and 4 only Which of the statements given above is/are correct?
(c) 1, 2, 3 and 4 (d) None of the above act (a) 1only (b) 2 and 3 only
Ans. (c) : There are 35 Acts' of The Indian government (c) 1 and 3 (d) 1, 2 and 3
that have relevance to biodiversity conservation which Ans. (c): Ultraviolet light is generated in CFLs by
includes all the four options given in this question. passing electricity from mercury vapour, which is
64. Why is the Government of India disinvesting its absorbed by the phosphorus coating inside the lamp.
equity in the Central Public Sector Enterprises LED lamps use traditional semiconductor light-
(CPSEs) emitting diodes, organic LED or polymer LED
1. The Government intends to use the revenue technology. CFLs have an average life span of 6000 to
earned from the disinvestment mainly to pay 15000 hours, while LED lamps can generally last 25-
back the external debt. 30 years.
2. The Government no longer intends to retain
the management control of the CPSEs. • Chemistry
Which of the statements given above is/are correct? 67. Consider the following:
(a) 1 only (b) 2 only
1. Carbon dioxide
(c) Both 1 and 2 (d) Neither 1 nor 2
2. Oxides of Nitrogen
Ans. (d): The Government of India is disinvesting its 3. Oxides of Sulphur
equity in Central Public Sector Enterprises (CPSEs).
Which of the above is/are the emission/emissions
The proceeds from the disinvestment of public sector
from coal combustion at thermal power plants -
undertaking go to the 'National Investment Fund' set
(a) 1 only (b) 2 and 3 only
up in 2005, whose main objective is to invest in social
development projects. The government's share of (c) 1 and 3 only (d) 1, 2 and 3
citizens in public sector undertakings, better corporate Ans. (d) : Coal combustion in thermal power plants
governance and management control are the main provides the emission of carbon dioxide, oxides of
points of government disinvestment. nitrogen, oxides of sulphur and airborne inorganic
particles, etc.
SCIENCE 68. Regarding ‘‘carbon credits’’, which one of the
• Physics following statements is not correct ?
(a) The carbon credit system was ratified in
65. The function of heavy water in a nuclear reactor
conjunction with the Kyoto Protocol .
is to –
(a) Slow down the speed of neutrons. (b) Carbon credits are awarded to countries or
(b) Increase the speed of neutrons. groups that have reduced greenhouse gases
(c) Cool down the reactor below their emission quota.
(d) Stop the Nuclear reaction. (c) The goal of the carbon credit system is to limit
Ans. (a): The function of heavy water (D2O) in a the increase of carbon dioxide emission.
nuclear reactor is to reduce the speed of neutrons. (d) Carbon credits are traded at a price fixed from
Deuterium and tritium are isotopes of hydrogen (H2). time to time by the United Nations Environment
Heavy water is a compound of hydrogen, deuterium and Programme.
oxygen. Its relative density is 101 and the freezing point Ans. (d) : According to the carbon credit system, the
is slightly higher than ordinary water. The Heavy Water maximum carbon emissions made by that country are
Board, Mumbai, is responsible for the production of determined according to the industries available in a
nuclear energy as well as for the supply of the required
country. The carbon credit system was implemented
heavy water demand for research reactors.
under the Kyoto Protocol to reduce carbon emissions
66. What is the difference between a CFL and an LED in the environment. In international markets, carbon
lamp? credits are traded according to their current market
1. To produce light, a CFL uses mercury prices.
vapour and phosphorus while an LED lamp
uses semi-conductor material. 69. Aspartame is an artificial sweetener sold in the
2. The average life span of a CFL is much market. It consists of amino acids and provides
longer than that of an LED Lamps. calories like other amino acids. Yet it is used as a
3. A CFL is less energy-efficient as compared to low-calorie sweetening agent in food items. What
an LED lamp. is the basis of this use ?
IAS (Pre) GS 2011 Paper I 198 YCT
(a) Aspartame is as sweet as table sugar, but unlike Ans. (b): Two types of tissues are found in vascular
table sugar, it is not readily oxidized in human plants: phloem and xylem. Phloem is the innermost
body due to lack of requisite enzymes. layer of the bark of the tree. Its main function is to
(b) When aspartame is used in food processing , the transport soluble organic matter produced in the
sweet taste remains, but it becomes resistant to process of photosynthesis. Phloem will also be
oxidation. damaged by removing the bark of a tree from all four
(c) Aspartame is as sweet as sugar, but after sides near the base of the tree, due to which the
ingestion into the body, it is converted into nutrients produced by the leaves in the process of
metabolites that yield no calories photosynthesis will not reach the roots, the roots will
(d) Aspartame is several times sweeter than table be deprived of energy, and the tree will gradually dry
sugar, hence food items made with small and slowly will be dead.
quantities of aspartame yield fewer calories on
oxidation. • Zoology
Ans. (d) : Aspartame is an artificial sweetener sold in 72. A married couple adopted a male child. A few
the market. It is many times sweeter than normal sugar years later, twin boys were born to them. The
or sucrose. Therefore, a very small amount of blood group of the couple is AB positive and O
aspartame is sufficient to produce a sweet taste. negative. The blood group of the three sons is A
Therefore, its calorie fraction is negligible. positive, B positive, and O positive. The blood
group of the adopted son is –
• Botany
(a) O positive
70. Regular intake of fresh fruits and vegetables is (b) A positive
recommended in the diet since they are a good (c) B positive
source of antioxidants. How do antioxidants help (d) Cannot be determined on the basis of the given
a person maintain health and promote longevity? data.
(a) They activate the enzymes necessary for Ans. (a): The blood group of the parents are AB and
vitamins synthesis in the body and help prevent O; the child's blood group may be A or B and since the
vitamin deficiency. body is adopted and it is given in the question that "the
(b) They prevent excessive oxidation of blood group of the three sons are A+, B+, O+". The
carbohydrates, fats and proteins in the body and correct answer is O+.
help avoid unnecessary wastage of energy.
(c) They neutralize the free radicals produced in the • Agriculture Animal Husbandry & Dairy
body during metabolism. 73. At present, scientists can determine the
(d) They activate certain genes in the cells of the arrangement or relative positions of genes or
body and help delay and ageing process. DNA sequences on a chromosome. How does this
Ans. (c) : It is desirable to take fresh fruits and knowledge benefit us?
vegetables in the diet regularly as they are good source 1. It is possible to know the pedigree of livestock.
of anti-oxidant element. Anti-oxidant elements are 2. It is possible to understand the causes of all
helpful in keeping a person healthy and maintain human diseases.
longevity as they neutralize the free radicals produced 3. It is possible to develop diseases-resistant
in the body as a byproduct of metabolism. Oxidation is animal breeds.
a chemical reaction in which electrons transfer from a Which of the statements given above is/are correct?
substance to an oxidizing agent. Oxidation reactions (a) 1 and 2 only (b) 2 only
can initiate chain reactions that damage cells. (c) 1 and 3 only (d) 1, 2 and 3
71. When the bark of a tree is removed in a circular Ans. (c) : The DNA sequence help in understanding
fashion all around near its base, it gradually dries the genealogies of the animals. In addition, various
up and dies because– genetic diseases such as Alzheimer's, Myotonic
(a) Water from soil cannot rise to aerial parts dystrophy, cystic fibrosis and inability of gene and
(b) Roots are starved of energy
defects arising in them can be cured by finding out the
(c) Tree is infected by soil microbes.
cause of many diseases.
(d) Roots do not receive oxygen for respiration.
IAS (Pre) GS 2011 Paper I 199 YCT
• Computers (a) 1, 2 and 3 only (b) 1, 3 and 4 only
(c) 2 and 4 only (d) 1, 2, 3, and 4
74. What is the difference between Bluetooth and
Wi-Fi Devices? Ans. (a) : A geostationary orbit is a circular
(a) Bluetooth uses 2.4 GHz radio frequency band, geosynchronous orbit in which the satellite is placed at
whereas Wi-Fi can use 2.4 GHz or 5 GHz an altitude of approximately 35786 km above mean sea
frequency band. level. The orbit lies in the plane of the earth's equator.
(b) Bluetooth is used for Wireless Local Area An object in such an orbit that has an orbital period
Networks (WLAN) only, where Wi-Fi is used equal to earth's rotational period and thus appear
for Wireless Wide Area Networks (WWAN) motionless.
only. 77. Microbial fuel cells are considered a source of
(c) When information is transmitted between two sustainable energy. Why?
devices using Bluetooth technology, the devices
1. They use living organisms as catalysts to
have to be in a line of sight of each other, but
generate electricity from certain substrates.
when Wi-Fi technology is used the devices need
2. They use a variety of inorganic materials as
not be in the line of sight of each other
substrates.
(d) The Statements (a) and (b) given above are
correct in this context 3. They can be installed in waste water treatment
plants to cleanse water and produce electricity.
Ans. (a): Bluetooth uses a radio frequency spectrum in
Which of the statement given above is/are correct?
the range of 2.402 GHz to 2.480 GHz and Wi-Fi is
(a) 1only (b) 2 and 3
IEEE 802.11 standard rate based technology using 2.4
(c) 1 and 3 only (d) 1, 2 and 3
GHz, 35 GHz and 5 GHz frequency spectrum.
Ans. (c): Microbial fuel cell (MFC) converts the
75. A new optical disc format known as the Blu-ray
chemical energy available in a biodegradable substrate
Disc (BD) is becoming popular. In what way is it
directly into electrical energy. MFC is also considered
different from the traditional DVD ?
an attractive alternative to power generation because it
1. DVD supports Standard Definition video while
contains organic matter such as sucrose, glucose,
BD supports High Definition video.
acetate, starch format, methanol, protein etc. are used
2. Compared to a DVD, the BD format has
several times more storage capacity. as substrates. Also, some inorganic substances such as
3. Thickness of BD is 2.4mm while that of DVD is sulfides are also used as substrates.
1.2 mm. 78. The formation of ozone hole in the Antarctic
Which of the statements given above is/are correct? region has been a cause of concern. What could
(a) 1 only (b) 1 and 2 only be the reason for a formation of this hole
(c) 2 and only (d) 1, 2 and 3 (a) Presence of prominent troposphere turbulence,
Ans. (b) : The Blu-ray disc is the next- generation and inflow of chlorofluorocarbons.
optical disc dedicated to high definition video. DVD (b) Presence of prominent polar front and
discs can store a maximum of 4.7 GB of data and a stratospheric clouds, and inflow of
single-layer Blu-ray disc can store 25 GB of data on an chlorofluorocarbons.
average. (c) Absence of polar front and stratospheric clouds;
and inflow of methane and chlorofluorocarbons.
• Science & Technology (d) Increased temperature at polar region due to
76. Satellites used for telecommunication relay are global warming.
kept in a geostationary orbit. A satellite is said to Ans. (b) : The formation of an ozone hole in the
be in such an orbit when: Antarctic region is a matter of concern, the possible
1. The orbit is geosynchronous. reason for the formation of this hole in the presence of
2. The orbit is circular specific polar atmospheres and stratospheric clouds
3. The orbit lies in the plane of the Earth’s and the inflow of chlorofluorocarbons. Nitric acid
equator. present in polar stratospheric clouds reacts with
4. The orbit is at an altitude of 22,236 km. chlorofluorocarbons to form chlorine which is
Select the correct answer using the codes given responsible for the photochemical destruction of the
below : ozone layer.

IAS (Pre) GS 2011 Paper I 200 YCT


79. The jet aircrafts fly very easily and smoothly in What one of the following could be that animal ?
the lower stratosphere. What could be the (a) Indian wild buffalo (b) Indian wild ass
appropriate explanation? (c) Indian wild boar (d) Indian gazelle
1. There are no clouds or water vapour in the Ans. (b) Sandy and saline regions are the natural
lower stratosphere. habitats of an Indian animal species; such animal is an
2. There are no vertical winds in the lower Indian wild Ass. Rann of Kutch is its natural habitat.
stratosphere There are no predators of that animal in this area, but
Which of the statements given above/is are due to the destruction of the habitat, its existence is in
correct in this context? danger.
(a) 1 only (b) 2 only
82. The ‘’Red Data Books’’ published by the
(c) Both 1 and 2 (d) Neither 1 nor 2
International Union for Conservation of Nature
Ans. (c) : In the lower part of the stratosphere, jet, and Natural Resources (IUCN) contain lists of –
planes can fly very smoothly as it does not have
1. Endemic plant and animal species present in
seasonal incidents like thunderstorms, thunder clouds,
the biodiversity hotspots.
lightning strikes, water vapour and dust. The
2. Threatened plant and animal species.
stratosphere ranges from 18 to 32 km. The ideal
3. Protected sites for conservation of nature and
condition of flying for aircraft and jet aircraft is found
natural resources in various countries.
in this stratosphere.
Select the correct answer using the codes given
80. What is Virtual Private Network below :
(a) It is a private computer network of an (a) 1 and 3 (b) 2 only
organization where the remote users can transmit (c) 2 and 3 (d) 3 only
encrypted information through the server of the Ans. (b): The list of threatened plant and animal
organization . species are included in the ''Red Data Book" published
(b) It is a computer network across a public internet by the International Union for Conservation of Nature
that provides users access to their organization’s and Natural Resources (IUCN).
network while maintaining the security of the 83. Three of the following criteria have contributed
information transmitted. to the recognition of Western Ghats-Sri Lanka
(c) It is a computer network in which users can and Indo-Burma regions as hotspots of
access a shared pool of computing resources biodiversity:
through a service provider. 1. Species richness
(d) None of the statements (a), (b), and (c) given 2. Vegetation density
above is correct description of Virtual Private 3. Endemism
Network 4. Ethno- botanical importance
Ans. (b): A secure way to connect to a private Local 5. Threat perception
Area Network (LAN) located at an organization's 6. Adaption of flora and fauna to warm and
remote location is a virtual private network (VPN) in humid condition
which the internet or an unsecured public network is Which three of the above are correct criteria in
used to exchange private information by encryption this context?
technology. (a) 1, 2 and 6
(b) 2, 4 and 6
ENVIRONMENT & ECOLOGY (c) 1, 3 and 5
(d) 3, 4 and 6
81. A sandy and saline area is the natural habitat of Ans. (c): The Western Ghats-Sri Lanka and Indo-
an Indian animal species. The animal has no Burma regions have been recognized as biodiversity
predators in that area but its existence is hotspots based on three parameters called species
threatened due to the destruction of its habitat . richness, endemism and threat perception.

IAS (Pre) GS 2011 Paper I 201 YCT


84. Human activities in the recent past have caused 87. The Himalayan Range is very rich in species
the increased concentrations of carbon dioxide in diversity. Which one among the following is the
the atmosphere, but a lot of it does not remain in most appropriate reason for this phenomenon?
the lower atmosphere because of – (a) It has a high rainfall that supports luxuriant
1. Its escape into the outer stratosphere. vegetative growth.
2. The photosynthesis by phytoplankton in the (b) It is a confluence of different bio-geographical
oceans. zones.
3. The trapping of air in the polar ice caps. (c) Exotic and invasive species have not been
Which of the statements given above is/are correct? introduced in this region.
(a) 1 and 2 (b) 2 only (d) It has less human interference.
(c) 2 and 3 (d) 3 only Ans. (b): The Himalayan Mountains form the natural
Ans. (b): The concentration of carbon dioxide in the and political boundary of the Indian sub-continent.
Due to the geographical location of the Himalayas, the
atmosphere has increased in recent years due to
rest of the Indian subcontinent has become a distinct
globalization rapidly increasing population,
personality from Asia. The Himalayan mountains are
deforestation and other human activities, but most of it
extremely rich in terms of species. The main reason for
does not remain in the lower part of the atmosphere.
that is that it is a confluence of different bio-
One of the main reasons is that phyto plankton
geographical regions.
performs photosynthesis in the oceans.
88. A layer in the Earth’s atmosphere called Ionosphere
85. In the context of ecosystem productivity, marine
facilitates radio communication. Why?
upwelling zones are important as they increase
1. The presence of ozone caused the reflection of
the marine productivity by bringing the –
radio waves to Earth.
1. Decomposer microorganisms to the surface.
2. Radio waves have a very long wavelength.
2. Nutrients to the surface. Which of the statements given above is/are correct?
3. Bottom-dwelling organisms to the surface. (a) 1 only (b) 2 only
Which of the statements given above is/are correct? (c) Both 1 and 2 (d) Neither 1 nor 2
(a) 1 and 2 (b) 2 only
Ans. (d): A layer called the Ionosphere in the Earth's
(c) 2 and 3 (d) 3 only
atmosphere facilitates radio communication because of
Ans. (b) : In the context of productivity, the upwelling the presence of electrically charged ions in the
area is important because they increase marine ionosphere.
productivity by bringing nutrients to the surface. Under
89. There is a concern over the increase in harmful
the sea flow, the hot and nutritious water present on algal blooms in the seawaters of India. What
the sea surface is transferred by the airflow through could be the causative factors for this phenomenon?
cold and nutrient- rich water. 1. Discharge of nutrients from the estuaries.
86. If a tropical rain forest is removed, it does not 2. Run-off from the land during the monsoon.
regenerate quickly as compared to a tropical 3. Upwelling in the seas.
deciduous forest. This is because – Select the correct answer from the codes given
(a) The soil or rain forest is deficient in nutrients. below :
(b) Propagules of the trees in a rain forest have poor (a) 1 only (b) 1 and 2 only
viability.
(c) The rain forest species are slow-growing. (c) 2 and 3 only (d) 1, 2 and 3
(d) Exotic species invade the fertile soil of rain forest Ans. (d): Concern has been expressed over the increase
Ans. (a) : If a tropical rain forest is removed, it does in harmful algae bloom in India's seawater. The main
not re-developed as fast as a tropical deciduous forest; reason for this is the discharge of nutrients from the
the main reason being the lack of nutrients in the soil estuaries, run-off from the land during the monsoon and
of the rain forest. upwelling in the seas.

IAS (Pre) GS 2011 Paper I 202 YCT


90. Consider the following 2. The Australia group comprises predominantly
1. Photosynthesis of Asian, African and North American
2. Respiration countries, whereas the members countries of
3. Decay of organic matter
Wassenaar Arrangement are predominantly
4. Volcanic action
Which of the above add carbon dioxide to the from the European Union and American
carbon cycle on Earth? Continents.
(a) 1and 4 only (b) 2 and 3 only Which of the statements given above is/ are correct?
(c) 2, 3 and 4 (d) 1, 2, 3 and 4 (a) 1 only (b) 2 only
Ans. (c) : Carbon dioxide is a colourless, odourless (c) Both 1 and 2 (d) Neither 1 nor 2
gas. Carbon dioxide in the atmosphere is found to be
0.03 percent as required. Its aqueous solution is acidic. Ans. (d): The Australia Group (AG) is an informal
At atmospheric pressure, it transforms into solid -state association that works on the basis of consensus. It
at 78°C temperature, which is called 'dry ice'. 'Dry ice' aims to allow exporters or transshipment countries to
is used in refrigeration, respiration, weathering of
minimize the risk of further proliferation of Chemical
biomaterials and volcanic activity adds carbon dioxide
to the carbon cycle while photosynthesis, weathering and Biological Weapons (CBW). There are not many
of rocks etc., results in reduced carbon dioxide in the Asian/African 'countries in the Australian group. There
atmosphere. are 43 members, including India, in this group.
91. Biodiversity forms the basis for human existence 94. In the context of global oil prices, ‘‘Brent crude
in the following ways :
oil’’ is frequently referred to in the news. What
1. Soil formation
2. Prevention of soil erosion does this term imply?
3. Recycling of waste 1. It is a major classification of crude oil.
4. Pollination of crops 2. It is sourced from North Sea.
Select the correct answer using the code given 3. It does not contain sulphur.
below :
(a) 1, 2 and 3 only (b) 2, 3 and 4 only Which of the statements given above is/are correct?
(c) 1 and 4 only (d) 1, 2, 3 and 4 (a) 2 only (b) 1 and 2 only
Ans. (d): Biodiversity remains the basis of human (c) 1 and 3 only (d) 1, 2 and 3
existence through soil formation, soil erosion Ans. (b): In the context of global oil prices, 'Brent
prevention, crop pollination and recycling of waste.
crude oil' is often mentioned is the news. Brent crude
92. Which one of the following is not a site for in-situ
oil is the largest of the major classification of crude oil.
method of conservation of flora?
(a) Biosphere Reserve (b) Botanical Garden It is obtained from the North Sea. This includes Brent
(c) National Park (d) Wild life Sanctuary crude, Brent sweet, Light crude, Ecofisk and Oseberg.
Ans. (b): Botanical garden does not come under the This oil is used for pricing about two-thirds of the
in- situ method of conservation of flora, while the world's international trade of crude oil.
biosphere reserve, national park and wildlife sanctuary
are included in it. 95. Recently,’’ oilzapper’’ was in the news. What is
it?
CURRENT AFFAIRS (a) It is an eco-friendly technology for the
remediation of oily sludge and oil spills
93. Recently, the USA decided to support India’s
membership in multi-lateral export control (b) It is the latest technology developed for under
regimes called the "Australia Group" and the sea oil exploration
"Wassenaar Arrangement". What is the (c) It is a generically engineered high biofuel
difference between them ?
1. The Australia Group is an informal yielding maize variety
arrangement which aims to allow exporting (d) It is the latest technology to control the
accidentally caused flames oil wells.
countries to minimize the risk of assisting
Ans. (a) : Recently, 'oilzapper' was in the news. It is an
chemical and biological weapons proliferation,
ecologically developed technology for the treatment of
whereas the Wassenaar Arrangement is a
oily punk and scattered oil. It has been developed by
formal group under the OECD holding
'The Energy and Research Institute (TERI).
identical objectives.
IAS (Pre) GS 2011 Paper I 203 YCT
MISCELLANEOUS (a) 1 only (b) 1 and 3 only
(c) 3 only (d) 1, 2 and 3
96. The new START treaty was in the news. What is Ans. (b) : The main objective of the look east policy is
this treaty ? economic integration with east and southeast Asia.
Indian realized that it’s East Asian neighbours
(a) It is a bilateral strategic nuclear arms reduction
achieved rapid economic growth and that it was
treaty between the USA and the Russian lagging behind.
Federation. 99. India maintained its early cultural contacts and
(b) It is a multilateral energy security cooperation trade links with Southeast Asia across the Bay of
treaty among the members of the East Asia Bengal. For this pre-eminence of early maritime
Summit. history of Bay of Bengal, which of the following
(c) It is a treaty between the Russian Federation and could be the most convincing explanation/
the European Union for the energy security explanations?
cooperation.
(a) As compared to other countries, India had a
(d) It is a multilateral cooperation treaty among the
better ship-building technology in ancient and
BRICS countries for the promotion of trade.
medieval times.
Ans. (a) : The New Strategic Arms Reduction Treaty (b) The rulers of southern India always patronized
(New START) was signed in 2010 in Prague by Russia traders,Brahmin priests and Buddhist monks in
and the United States. This treaty continues the this context.
bipartisan process of verifiably reducing US and (c) Monsoon winds across the Bay of Bengals
Russian strategic nuclear arsenals begun by former Facilitated sea voyages.
Presidents Ronald Reagan and George H.W. Bush (d) Both (a) and (b) are convincing explanations in
through START-I in 1994. this context.
97. Karl Marx explained the process of class struggle Ans. (c): India had maintained its initial cultural
with the help of which one of the following contacts and trade relations with South-East Asia even
theories ? beyond the Bay of Bengal. The main reason for this
(a) Empirical liberalism was the ease of sea voyages through the monsoon
(b) Existentialism winds in the Bay of Bengal.
(c) Darwin’s theory of evolution 100.With reference to the period of colonial rule in
(d) Dialectical meterisalism India,’’ Home Charges’ formed an important
part of drain of wealth from India. Which of the
Ans. (d) : Darwin's 'struggle for existence' in natural
following funds constituted Home Charges?
history is analogous to the class struggle in "human 1. Funds used to support the India Office in
history". So option (c) is incorrect. The sketch of London
dialectical materialism given by Karl Marx explains 2. Funds used to pay salaries and pensions of
that Dialectical principle is of universal application British personnel engaged in India
3. Funds used for waging wars outside India by
and all the development is "struggle of opposites". the British
Thus option (d) is the correct answer. Select the correct answer using the codes given
98. With reference to Look East Policy of India, below :
(a) 1 only (b) 1 and 2 only
consider the following statements : (c) 2 and 3 only (d) 1, 2 and 3
1. India wants to establish itself as an important Ans. (b) : In India during the colonial rule, "Home
regional player in the East Asian Affairs Charges" was an important part of the exploitation of
2. India Wants to plug the vacuum created by the property in India. In Colonial India, Home Charges
termination of cold War. was the expenditure incurred by the secretary of state
3. India wants to restore the historical and in England on behalf of India. This included the India
cultural ties with its neighbors in Southeast office in London. Funds used for maintenance, salaries
and East Asia. of English employees working in India and funds used
Which of the statements given above is/are correct? for pension, etc., were included.

IAS (Pre) GS 2011 Paper I 204 YCT


UNION PUBLIC SERVICE COMMISSION
Civil Services (Preliminary Exam) - 2010
GENERAL STUDIES : PAPER-I
Time: 2 hours Maximum Number: 200
(a) He wanted to avenge the expulsion by
ANCIENT HISTORY Marathas of his viceroy timur Shah from
Lahore.
1. Why did Buddhism start declining in India in
(b) The frustrated governor of Jullundhar Adina
the early medieval times?
Beg Khan invited him to invade Punjab
1. Buddha was by that time considered as one of (c) He wanted to punish Mughal administration
the incarnations and Vishnu of thus became a for non-payment of the revenues of the
part of Vaishnavism. Chachar Mahal (Gujrah, Aurangabad, Sialkot
2. The invading tribes from Central Asia till the and Pasrur)
time of last Gupta king adopted Hinduism and (d) He wanted to annex all the fertile plants of
persecuted Buddhists. Punjab up to the borders of Delhi to his
3. The Kings of Gupta dynasty were strongly kingdom.
opposed to Buddhism. Ans. (a) : He wanted to avenge the expulsion by
Which of the statements given above is/are Marathas of his viceroy Timur Shah from Lahore.
correct?
(a) 1 only (b) 1 and 3 only MODERN HISTORY
(c) 2 and 3 only (d) 1, 2 and 3
Ans. (a) : Buddha was considered 9th incarnation of 5.
For the Karachi session of Indian National
Congress in 1931 presided over by Sardar Patel,
Vishnu. No tribe from Central Asia persecuted Buddhists.
who drafted the Resolution on Fundamental
Gupta dynasty did not oppose Buddhism. However, they
Rights and Economic Programme?
did not give them patronage.
(a) Mahatma Gandhi
2. Among the following, who was not a proponent (b) Pandit Jawaharlal Nehru
of bhakti cult? (c) Dr. Rajendra Prasad
(a) Nagarjuna (b) Tukaram (d) Dr. B. R. Ambedkar
(c) Tyagaraja (d) Vallabhacharya
Ans. (b) : Karachi session of Indian National Congress,
Ans. (a) : Nagarjuna was an Indian Buddhist 1931, presided over by Sardar Patel, Pandit Jawaharlal
philosopher who propounded the doctrine of emptiness or
Nehru drafted the Resolution on Fundamental Rights
Sunyavada. He is not related to the Bhakti movement.
and Economic Programme.
Tukaram was the greatest Marathi Bhakti poet. 6. Who among the following were official
Vallabhacharya and Tyagaraja were also followers of Congress negotiators with Cripps Mission?
the Bhakti movement. So the correct answer is (a) Mahatma Gandhi and Sardar Patel
Nagarjuna. (b) Acharya J.B. Kripalani and C. Rajagopalachari
3. There are only two known examples of cave (c) Pandit Nehru and Maulana Azad
paintings of the Gupta period in ancient India. One
(d) Dr. Rajendra Prasad and Rafi Ahmed Kidwai
of these is paintings of Ajanta caves. Where is the
Ans. (c) : The Cripps Mission was an attempt in 1942
other surviving example of Gupta paintings?by the British government to secure India’s cooperation
(a) Bagh caves (b) Ellora caves
in World War II. The Mission was headed by Sir
(c) Lomas Rishi cave (d) Nasik cavesStafford Cripps. Pandit Jawaharlal Nehru and Maulana
Azad were official congress negotiators with Cripps
Ans. (a) : There are only two known examples of cave
Mission.
paintings of the Gupta period in ancient India. One of these
is paintings of Ajanta caves and the other is Bagh caves.
7. Consider the following statements:
1. The common wealth has no charter, treaty
MEDIEVAL HISTORY or constitution.
2. All the territories/countries once under the
4. What was the immediate reason for Ahmad British empire (jurisdiction/rule/mandate)
Shah Abdali to invade India and fight the automatically joined the Commonwealth as
Third Battle of Panipat? its members.
IAS (Pre) GS 2010 Paper I 205 YCT
Which of the statements given above is/are 10. Who among the following Governor Generals
correct? created the Covenanted Civil Service of India
(a) 1 only (b) 2 only which later came to be known as the Indian
(c) Both 1 and 2 (d) Neither 1 nor 2 Civil Service?
Ans. (d) : The Charter of the Commonwealth was (a) Warren Hastings
adopted on 19th December, 2012 and was officially (b) Wellesley
signed by Queen Elizabeth II at Marlborough House, (c) Cornwallis
London, on the Commonwealth Day on 11th March, (d) William Bentinck
2013. The Commonwealth of Nations is a voluntary Ans. (c) : Lord Cornwallis created the Covenanted Civil
association of 53 independent sovereign states, most of Service of India which later came to be known as the
which were once part of the British Empire. The Indian Civil Service.
Commonwealth's membership includes both republics 11. What was the immediate cause for the launch
and monarchies. The head of the Commonwealth of of the Swadeshi movement?
Nations is Queen Elizabeth II, who reigns as monarch
(a) The partition of Bengal done by Lord Curzon
directly in 16 member states known as Commonwealth
realms. (b) A sentence of 18 months rigorous
imprisonment imposed on Lokmanya Tilak
8. With reference to Pondicherry (Now Puducherry), (c) The arrest and deportation of Lala Lajpat Rai
consider the following statements: and Ajit Singh, and passing of the Punjab
1. The first European power to occupy Colonization Bill
Pondicherry were the Portuguese. (d) Death sentence pronounced on the Chapekar
2. The second European power to occupy brothers
Pondicherry were the French.
Ans. (a) : The government’s decision to partition
3. The English never occupied Pondicherry.
Bengal was made public in December 1903.The official
Which of the statements given above is/are reason given was that Bengal with a population of 78
correct? million was too big to be administered. The government
(a) 1 only (b) 2 and 3 only announced partition in July 1905. To boycott this on 7th
(c) 3 only (d) 1, 2 and 3 Aug 1905 formal proclamation of Swadeshi movement
Ans. (a) : The first European power to occupy Pondicherry was made from the Calcutta townhall.
were the Portuguese. The first Europeans to exert power in 12. Consider the following statements:
India were the Portuguese who sailed round Southern 1. Dr. Rajendra Prasad persuaded Mahatma
Africa in search of the trade goods of the East such as Gandhi to come to Champaran to investigate
pepper, cloves, silk. When they reached the coast of India the problem of peasants.
and the islands further east, weapons technology and lack 2. Acharya J.B Kriplani was one of Mahatama
of moral scruples gave them advantage over the local Gandhi's colleagues in this Champaran
regimes they found. They began to occupy parts such as investigation.
Goa in India and towns in Indonesia.
Which of the statements given above is/are
9. Consider the following statements: correct?
1. The "Bombay Manifesto" signed in 1936 (a) 1 only (b) 2 only
openly opposed the preaching of socialist (c) Both 1 and 2 (d) Neither 1 and 2
ideals.
Ans. (b) : Acharya J.B Kriplani was one of Mahatama
2. It evoked support from a large section Gandhi's colleagues in this Champaran investigation.
community from all across India. The correct answer is Only (2). Rajkumar Shula
Which of the statements given above is/are persuaded Mahatma Gandhi to come to Champaran to
correct? investigate the problem of peasant.
(a) 1 only (b) 2 only 13. By a regulation in 1793, the District Collector
(c) Both 1 and 2 (d) Neither 1 nor 2 was deprived of his judicial powers and made
Ans. (a) : The 'Bombay Manifesto signed in 1936 by 21 the collecting agent only. What was the reason
Bombay businessmen, contained an open indictment of for such regulation?
Nehru's preaching of socialist ideals which were (a) Lord Cornwallis felt that the District Collector's
deemed prejudicial to private property and to the peace efficiency of revenue collection would
and prosperity of the country. enormously increase without the burden of other
Although, it did not evoked support from any other work
section of the business community, it strengthened the (b) Lord Cornwallis felt that judicial power should
hands of the moderates within the Congress like compulsorily be in the hands of Europeans
Bhulabhai Desai and G.B. Pant who put pressure on while Indians can be given the job of revenue
Nehru to tone down his socialist utterances. collection in the districts
IAS (Pre) GS 2010 Paper I 206 YCT
(c) Lord Cornwallis was alarmed at the extent of (b) Reconstitution of the Central Executive Council
power concentrated in the District Collector and in such a way that all its members, except the
felt that such absolute power was undesirable in Governor General and the Commander-in-Chief
one person. should be Indian leaders
(d) The judicial work demanded a deep knowledge (c) Fresh elections to the Central and Provincial
of India and a good training in law and Lord Legislatures to the held at the end of 1945 and
the Constitution making body to be convened as
Cornwallis felt the District Collector should be
soon as possible
only a revenue collector.
(d) A solution for the constitutional deadlock
Ans. (c) : Lord Cornwallis was alarmed at the extent of
Ans. (d) : “In the last years of the war Rajagopalachari
power concentrated in the District Collector and felt that was instrumental in negotiations between Gandhi and
such absolute power was undesirable in one person. Jinnah. In 1944 he proposed a solution to the Indian
14. With reference to Simon Commission’s Constitutional tangle. He prepared a formula for Congress-
recommendations, which one of the following League cooperation. It was a tacit acceptance of the
statements is correct? League’s demand for Pakistan. Gandhi supported the
(a) It recommended the replacement of diarchy formula. But Jinnah was in favour of two-nation theory
with responsible government in the provinces. only.
(b) It proposed the setting up of inter provincial
INDIAN GEOGRAPHY
council under the Home Department
(c) It suggested the abolition of bicameral 17. Consider the following pairs:
legislature at the Centre Protected area Well known for
(d) It recommended the creation of Indian Police 1. Bhiterkanika, Orissa ... Salt Water
Service with a provision for increased pay Crocodile
and allowances for British recruits as 2. Desert National Park, .... Great Indian
compared to Indian recruits Rajasthan Bustard
Ans. (a) : The Simon Commission was "to enquire into 3. Eravikulam, Kerala ... Hoolock Gibbon
working of the system of government, the growth of Which of the pairs given above is/are correctly
education and the development of representatives matched?
institution in British India and matters connected (a) 1 only (b) 1 and 2 only
therewith" and to report "as to whether and to what extent, (c) 2 only (d) 1, 2 and 3
it is desirable to establish the principle of responsible Ans. (b) : Eravikulam National Park is known for
government. It favoured the replacement of diarchy with Nilgiri Tahr. Hoolock Gibbon are found in the Manas
responsible government in provinces. National Park, Namdapha National Park and Kaziranga
National Park in the North-Eastern India.
15. Four resolution were passed at the famous
18. The approximate representation of land use
Calcutta session of Indian National Congress in
classification in India is:
1906. The question of either retention OR of
(a) Net area sown 25%, forests 33%; other areas
rejection of these four resolutions became the
42%
cause of a split in Congress at the next
(b) Net area sown 43%, forests 17%; other areas
Congress session held in Surat in 1907. Which 25%
one of the following was not one of those (c) Net area sown 43%, forests 29%; other areas
resolutions? 28%
(a) Annulment of partition of Bengal (d) Net area sown 47%, forests 23%; other areas
(b) Boycott 30%
(c) National education Ans. (d) : The approximate representation of land use
(d) Swadeshi classification in India is Net area sown 47%; forests 23%;
Ans. (a) : The correct answer is (a) Annulment of other areas 30%.
partition of Bengal. 19. In India, during the last decade the total
16. After Quit India Movement, C. Rajagopalachari cultivated land for which one of the following
issued a pamphlet entitled "The Way Out". crops has remained more or less stagnant?
Which one of the following was proposal in this (a) Rice (b) Oilseeds
pamphlet? (c) Pulses (d) Sugarcane
(a) The establishment of a "War Advisory Council" Ans. (c) : In India, during the last decade, the total
composed of representative of British India and cultivated land for pulses crops has remained more or less
the Indian States stagnant.

IAS (Pre) GS 2010 Paper I 207 YCT


20. When you travel in certain parts of India, you 25. Tamil Nadu is a leading producer of mill-made
will notice red soil. What is the main reason for cotton yarn in the country. What could be the
this colour? reason?
(a) Abundance of magnesium 1. Black cotton soil is the predominant type of
(b) Accumulated humus soil in the State.
(c) Presence of ferric oxides 2. Rich pool of skilled labour is available
(d) Abundance of phosphates Which of the above is/are the correct reasons?
Ans. (c) : Presence of ferric oxides. (a) 1 only
21. Which one of the following is the appropriate (b) 2 only
reason for considering the Gondwana rocks as (c) Both 1 and 2
most important of rock systems of India? (d) Neither 1 and 2
(a) More than 90% of limestone reserves of India Ans. (b) : The predominant soil in Tamil Nadu is red
are found in them which supports cotton cultivation and a rich pool of
(b) More than 90% of India's coal reserves are skilled labour is available in Tamil Nadu.
found in them 26. Rivers that pass through Himachal Pradesh are
(c) More than 90% of fertile black cotton soils (a) Beas and Chenab only
are spread over them (b) Beas and Ravi only
(d) None of the reasons given above is (c) Chenab, Ravi and Satluj only
appropriate in this context
(d) Beas, Chenab, Ravi, Satluj and Yamuna
Ans. (b) : More than 90% of India's coal reserves are
Ans. (d) : Rivers like Beas, Chenab, Ravi, Sutlej and
found in them.
Yamuna passes through Himachal Pradesh.
22. With reference to the river Luni, which one of
27. With reference to the mineral resources of
the following statements is correct?
India, consider the following pairs:
(a) It flows into Gulf of Khambhat
Mineral 90% Natural sources in
(b) It flows into Gulf of Kuchchh
1. Copper Jharkhand
(c) If flows into Pakistan and merges with a
tributary of Indus 2. Nickel Orissa
(d) It is lost in the marshy land of the Rann of 3. Tungsten Kerala
Kuchchh Which of the pairs given above is/are correctly
matched?
Ans. (d) : Luni river originates from Aravalli range and
flows in south west direction through the hills and finally (a) 1 and 2 only (b) 2 only
ends up in the marshy land of Rann of Kutch. (c) 1 and 3 only (d) 1, 2 and 3
23. Which one of the following pairs is not Ans. (b) : Most of nickel reserves of India are found in
correctly matched? Cuttack, Keonjhar and Mayurbhanj districts of Odisha
Dam/Lake River (92%). As per UNFC, the total resources of nickel ore
(a) Govind Sagar : Satluj have been estimated at 189 million tonnes.
(b) Kolleru Lake : Krishna About 175 million tonnes of nickel is estimated in Odisha.
Copper is mostly found in Bihar-Jharkhand (44%) and
(c) Ukai Reservoir : Tapi
Rajasthan (20%).
(d) Wular Lake : Jhelum
Ans. (b) : Govind Sagar Dam is on Sutlej (HP) near WORLD GEOGRAPHY
Punjab border. Kolleru Lake is located between Krishna
and Godawari river valleys and serves as a natural flood 28. Which one of the following can one come across
balancing reservoir for both the rivers. River Budaneru if one travels through the Strait of Malacca?
(sorrow of Vijayawada) drains into Kolleru Lake. Ukai (a) Bali (b) Brunei
reservoir on Tapi river (Gujarat) near Maharashtra (c) Java (d) Singapore
border. Wular Lake is on river Jhelum.
Ans. (d) : One can one come across to Singapore if one
24. The latitudes the pass through Sikkim also pass travels through the Strait of Malacca. Malacca strait is
through: between the Indonesian island of Sumatra and the
(a) Rajasthan Malay Peninsula. Singapore lies on the tip of the Malay
(b) Punjab Peninsula.
(c) Himachal Pradesh 29. A geographic region has the following distinct
(d) Jammu & Kashmir characteristics:
Ans. (a) : The latitude that pass through Sikkim also pass 1. Warm and dry climate
through Rajasthan. 27 degree latitude passes through 2. Mild and wet winter
Gangtok of sikkim and Alwar in Rajasthan. 3. Evergreen oak trees
IAS (Pre) GS 2010 Paper I 208 YCT
The above features are the distinct characteristic 32. In India, which type of forest among the
of which one of the following regions? following occupies that largest area?
(a) Mediterranean (a) Montane Wet Temperate Forest
(b) Eastern China (b) Sub-tropical Dry Evergreen Forest
(c) Central Asia (c) Tropical Moist Deciduous Forest
(d) Atlantic coast of North America (d) Tropical Wet Evergreen Forest
Ans. (a) : Mediterranean climatic type is associated Ans. (c) : In India, Tropical Moist Deciduous Forest
with all of the above features. The climate is known for occupies the largest area. The tropical monsoon
warm to hot, dry summers and mild to cool, wet deciduous forests are found in areas receiving annual
winters. The cause of this climate is directly related to rainfall of 100 to 200 cm in India, with a distinct dry
large bodies of water such as Mediterranean sea and and rainy seasons and a small range of temperature.
ocean currents. Almost no rainfall during the summer. 33. Consider the following statements:
Most of the rainfalls during the cooler winter. 1. On the planet Earth, the fresh water
30. What causes wind to deflect toward left in the available for use amounts to about less than
Southern hemisphere? 1% of the total water found.
(a) temperature (b) Magnetic field 2. Of the total fresh water found on the planet
(c) Rotation of the earth (d) Pressure Earth 95% is bound up in polar ice caps
Ans. (c) : Deflection of wind towards left in Southern and glaciers.
Hemisphere (SH) and right in North Hemisphere (NH) Which of the statements given above is/are
is mainly due to rotation of Earth (coriolis force) and correct?
Ferrel's Law talks about the diversion i.e., left and right (a) 1 only (b) 2 only
in South and North hemisphere respectively. (c) Both 1 and 2 (d) Neither 1 and 2
31. If there were no Himalayan ranges, what would Ans. (c) : About 2.75 percent of water is fresh water in
have been the most likely geographical impact earth and about 95% is frozen in glaciers. Less than 1%
on India? of the world’s fresh water is readily accessible.
1. Much of the country would experience the 34. Following are the characteristics of an area in
cold waves from Siberia. India:
2. Indo-genetic plain would be devoid of such 1. Hot and humid climate
extensive alluvial soils. 2. Annual rainfall 200 cm
3. The pattern of monsoon would be different 3. Hill slopes up to an altitude of 1100 metres
from what it is at present. 4. Annual range of temperature 15°C to 30°C
Which of the statements given above is/are Which one among the following crops are you
correct? most likely to find in the area described above?
(a) 1 only (b) 1 and 3 only (a) Mustard (b) Cotton
(c) 2 and 3 only (d) 1, 2 and 3 (c) Pepper (d) Virginia tobacco
Ans. (d) : All three statements mentioned above are Ans. (c) : Pepper is the crop most likely to find in the
correct. The Himalayas are the tallest mountain ranges area described above.
of the world and stand as unsurmountable wall. They 35. A geographic area with an altitude of 400
save us from the severe cold winds which would have metres has following characteristics:
entered India in the absence of these mountain ranges.
Month J F M A M J J A S O N D
Different places in the Himalayas are the sources of
Average
great rivers like the Indus, the Ganga and the
maximum 31 31 31 31 30 30 29 28 29 29 30 31
Brahmaputra as well as their tributaries. These mighty
temp.°C
rivers and their tributaries bring huge quantities of
Average
fertile alluvial soils from the mountain areas and deposit
maximum 21 21 21 21 21 21 20 20 20 20 20 20
these soils in the Indo Gangetic plain. At present this is
temp.°C
considered to be one of the most fertile plains of the
Rainfall
world. Had there been no Himalayas, the entire plain 51 85 188 158 139 121 134 168 185 221 198 86
would be devoid of alluvial soils and it would be totally (mm)
a barren plain. The Himalayas offer great obstruction to If this geographic area were to have a natural
the monsoon winds coming from the Bay of Bengal. forest, which one of the following would it most
These winds cannot cross the Himalayan wall and are likely be?
forced to move from east to west along the foothills of (a) Moist temperate coniferous forest
the Himalayas and cause rains in the entire Indus- (b) Montane subtropical forest
Ganga-Brahmaputra plain, in addition to the mountain (c) Temperate forest
areas. (d) Tropical rain forest
IAS (Pre) GS 2010 Paper I 209 YCT
Ans. (d) : The above temperature and rainfall represent Ans. (b) : State Finance Commission is an institution
tropical rain forest. This geographical region have double created by the 73rd and 74th constitutional
rainfall peak, coinciding with March and October and high Amendments (CAs) to rationalize and systematize State
average annual temperature with only 2-3°C annual /sub-State-level fiscal relations in India. Article 243I of
temperature range. Total annual average rainfall is 173.4 the Constitution mandated the State Governor to
cm. constitute a Finance Commission every five years.
Altitude of area is around 400 m, which has little impact 39. Consider the following statements:
for tropical areas because height of 400 m above sea level In India, taxes on transactions in Stock
is largely the height of Plain area. Exchanges and Futures Markets are
36. A new type of E1 Nino called E1 Nino Modoki 1. levied by the Union
appeared in the news. In this context, consider 2. collected by the State
the following statements: Which of the statements given above is/are
1. Normal E1 Nino forms in the Central Pacific correct?
ocean whereas E1 Nino Modoki forms in (a) 1 only (b) 2 only
Eastern Pacific ocean. (c) Both 1 and 2 (d) Neither 1 nor 2
2. Normal E1 Nino results in diminished Ans. (a) : As per Article 269 of the Constitution of
hurricanes in the Atlantic ocean but E1 Nino India, taxes other than stamp duties on transactions in
Modoki results in a greater number of stock-exchanges and futures markets are levied and
hurricanes with greater frequency. collected by the Union but assigned to the states.
Which of the statements given above is/are 40. With reference to the Consumer Disputes
correct? Redressal at district level in India. Which one
(a) 1 only (b) 2 only of the following statements is not correct?
(c) Both 1 and 2 (d) Neither 1 nor 2 (a) A State Government can establish more than
one District Forum in a district if it deems fit
Ans. (b) : Normal El Nino results in diminished (b) One of the members of the District Forum
hurricanes in the Atlantic ocean but El Nino Modoki shall be a woman.
results in a greater number of hurricanes with greater (c) The District Forum entertains the Complaints
frequency. where the value of goods or services does not
37. Though coffee and tea both are cultivated on hill exceed rupees fifty lakhs.
slopes, there is some difference between them (d) A complaint in relation to any goods sold or
regarding their cultivation. In this context, any service provided may be filed with a
consider the following statements: District Forum by the State Government as a
1. Coffee plant requires a hot and humid climate representative of the interests of the
of tropical areas where as tea can be cultivated consumers in general.
in both tropical and subtropical areas. Ans. (c) : As per section 21 of consumer protection act
2. Coffee is propagated by seeds but tea is 1986 the District Forum entertains the complaints where
propagated by stem cuttings only. the value of goods or services does not exceed rupees
Which of the statements given above is/are twenty lakhs.
correct? 41. In the context of governance, consider the
(a) 1 only (b) 2 only following:
(c) Both 1 and 2 (d) Neither 1 nor 2 1. Encouraging Foreign Direct Investment inflows
Ans. (a) : Coffee plant requires a hot and humid climate 2. Privatization of higher educational Institutions
of tropical areas whereas tea can be cultivated in both 3. Down-sizing of bureaucracy
tropical and subtropical areas. 4. Selling/offloading the shares of Public Sector
Undertakings
INDIAN CONSTITUTION AND POLITY Which of the above can be used as measures to
control the fiscal deficit in India?
38. Which one of the following authorities makes (a) 1, 2 and 3 (b) 2, 3 and 4
recommendation to the Governor of a State as (c) 1, 2 and 4 (d) 3 and 4 only
to the principles for determining the taxes and Ans. (b) : Privatization of higher educational Institutions
duties which may be appropriated by the and down-sizing of bureaucracy and selling/ offloading
Panchayats in that particular State? the shares of Public Sector Undertakings directly cause
(a) District Planning Committees reduction in fiscal deficit.
(b) State Finance Commission Without knowing the destination of Foreign Direct
(c) Finance Ministry of that State Investment inflows, we cannot determine the actual
(d) Panchayati Raj Ministry of that State effect of it on fiscal situation.

IAS (Pre) GS 2010 Paper I 210 YCT


42. Who of the following shall cause every Article 41 Right to work, to education and to public
recommendation made by the Finance assistance in certain cases.
Commission to be laid before each House of Article 42 Provision for just and humane conditions of
Parliament? work and maternity relief.
(a) The President of India All these schemes also fulfill the criteria of right to life,
(b) The Speaker of Lok Sabha given under the Article 21 of the Constitution
(c) The Prime Minister of India (Fundamental Right).
(d) The Union Finance Minister 45. Consider the following statements:
The Supreme Court of India tenders advice to
Ans. (a) : As per provisions given under Article 281 of the
the President of India on matters of law or fact.
Indian Constitution the recommendations of the finance
1. on its own initiative (on any matter of larger
commission go to the president who is constitutionally
public interest).
bound to place it before the two houses of the parliament.
2. If he seeks such an advice.
43. The "instrument of Instructions" contained in 3. Only if the matters relate to the Fundamental
the Government of India Act 1935 have been Rights of the citizens.
incorporated in the Constitution of India in the Which of the statement given above is/are
year 1950 as: correct?
(a) Fundamental Rights (a) 1 only (b) 2 only
(b) Directive Principles of State Policy (c) 3 only (d) 1 and 2
(c) Extent of executive power of State Ans. (b) : Article 143 of the Indian Constitution confers
(d) Conduct of business of the Government of upon the Supreme Court advisory jurisdiction. The
India President may seek the opinion of the Supreme Court on
Ans. (b) : The Directive Principles resemble the any question of law or fact of public importance on which
‘Instrument of Instructions’ enumerated in he thinks it expedient to obtain such an opinion.
the Government of India Act of 1935. In the words of 46. With reference to Lok Adalats, which of the
Dr. B.R. Ambedkar, ‘the Directive Principles are like following statements is correct?
the instrument of instructions, which were issued to the (a) Lok Adalats have the jurisdiction to settle the
Governor-General and to the Governors of the colonies matters at pre-litigative stage and not those
of India by the British Government under the matters pending before any court
Government of India Act of 1935. What is called (b) Lok Adalats can deal with matters which are
Directive Principles is merely another name for civil and not criminal in nature
the instrument of instructions. The only difference is (c) Every Lok Adalat consists of either serving or
that they are instructions to the legislature and the retired judicial officers only and not any other
executive’. person
44. With reference to the Constitution of India, (d) None of the statements given above is correct
consider the following: Ans. (d) : Lok Adalat is one of the modes of Alternative
1. Fundamental Rights Dispute Resolution. The Lok Adalat Mechanism is
available throughout the State through the Legal Services
2. Fundamental Duties
Authorities/Committees for amicable settlement. It is a
3. Directive Principles of State Policy very effective institution of alternative dispute resolution,
Which of the above provisions of the Constitution where the cases pending in the court of law or at pre-
of India is/are fulfilled by National social litigation stage are settled amicably. The decision or decree
Assistance programme launched by the of Lok Adalat is final and binding like a civil
government of India? court. It consists of a sitting or retired judicial officer and
(a) 1 only (b) 3 only other persons. (Two members and one social worker)
(c) 1 and 3 only (d) 1, 2, and 3 (Two members should be a lawyer).
Ans. (b) : NSAP is a social assistance programme for
poor households launched on 15th August, 1995 and
ECONOMY
represents a significant step towards the fulfillment of 47. In India, which of the following is regulated by
the Directive Principles in Articles 41 and 42 of the the Forward Markets Commission?
Constitution recognising the concurrent responsibility (a) Currency Futures Trading
of the Central and State Governments in the matter. (b) Commodities Futures Trading
The NSAP will include for the time being, three (c) Equity Futures Trading
benefits as its components, viz (d) Both Commodities Futures and Financial
1. National Old Age Pension Scheme (NOAPS) Futures Trading
2. National Family Benefit Scheme (NFBS) Ans. (b) : Commodities Futures Trading is regulated by
3. National Maternity Benefit Scheme (NMBS) the Forward Markets Commission.

IAS (Pre) GS 2010 Paper I 211 YCT


48. Which one of the following is not a feature of 52. In India, the tax proceeds of which one of the
Limited Liability Partnership firm? following as a percentage of gross tax revenue
(a) Partners should be less than 20. has significantly declined in the last five years?
(b) Partnership and management need not be (a) Service tax (b) Personal income tax
separate (c) Excise duty (d) Corporation tax
(c) Internal governance may be decided by Ans. (c) : The excise duty's share in the total tax revenue
mutual agreement among partners. which was 41.3% in 1991-92, declined to 25.1% in 2006-
(d) It is corporate body with perpetual succession. 07 It is the most that has declined in the last 5
Ans. (a) : Partners should be less than 20 is not a years. Corporation tax has maximum share followed by
feature of Limited Liability Partnership firm. income tax. Service tax was introduced very late so it’s
share would obviously increase.
49. With reference to the institution of Banking
Ombudsman in India, which one of the 53. With reference to Indian economy, consider the
statements is not correct? following statements:
(a) The Banking is appointed by the Reserve 1. The gross Domestic Product (GDP) has
Bank of India increased by four times in the last 10 years.
(b) The Banking Ombudsman can consider 2. The percentage share of Public Sector in
complaints from Non Resident Indians having GDP has declined in the last 10 years.
accounts in India. Which of the statements given above is/are
correct?
(c) The orders passed by the Banking
Ombudsman are final and binding on the (a) 1 only (b) 2 only
parties concerned (c) Both 1 and 2 (d) Neither 1 nor 2
(d) The service provided by the Banking Ans. (b) :
Ombudsman is free of any fee The Gross Domestic Product (GDP) in 2000-01 was Rs
Ans. (c) : The orders passed by the Banking Ombudsman 19.2 lakh crore which increased to Rs 58 lakh crore in
are final and binding on the parties concerned. 2009-10, with 6.7% growth rate in 2009-10. It is nearly
three times the GDP in 2000-01. With the rapid growth
50. With reference to India, consider the following: of the private sector after 1991, the share of public
1. Nationalization of Banks sector in GDP has gone down.
2. Formation of Regional Rural Banks 54. In the context of India's Five Year Plans, a shift
3. Adoption of Villages by Bank Branches in the pattern of industrialization with lower
Which of the above can be considered as steps emphasis on heavy industries and more on
taken to achieve the "financial inclusion" in infrastructure begins in
India? (a) Fourth Plan (b) Sixth Plan
(a) 1 and 2 only (b) 2 and 3 only (c) Eight Plan (d) Tenth Plan
(c) 3 only (d) 1, 2 and 3 Ans. (c) : Industrial sector was given more weightage
Ans. (d) : All of the above can be considered as steps from Second Five Year Plan to Fifth Five Year Plan.
taken to achieve the financial inclusion' in India. Since, the Eight Year Plan, allocation for industrial
Nationalization targeted expansion of bank branches in sector has been reduced in favour of infrastructure.
rural area and also a control on concentration of economic 55. Inclusive growth as enunciated in the Eleventh Five
powers in the hands of few. RRBs were planned to cater to Year Plan does not include one of the following:
the needs the rural areas. (a) Reduction of poverty
51. Consider the following statements: (b) Extension of employment opportunities
The functions of commercial banks in India (c) Strengthening of capital market
include (d) Reduction of gender inequality
1. Purchase and sale of shares and securities Ans. (c) : Inclusive growth is ‘broad-based growth’
on behalf of customers. ‘shared growth’ and ‘pro-poor growth’. It excludes the
2. Acting as executors and trustees of wills. capital markets which is left to operate by itself in the open
Which of the statements given above is/are market.
correct? 56. Which one of the following is responsible for
(a) 1 only (b) 2 only the preparation and presentation of Union
(c) Both 1 and 2 (d) Neither 1 nor 2 Budget to the Parliament?
Ans. (c) : Commercial banks are not directly engaged in (a) Department of Revenue
capital market. Capital market is the primary concern of (b) Department of Economic Affairs
investment banks. They act as executors and trustees of (c) Department of Financial Services
wills of their clients. (d) Department of Expenditure
IAS (Pre) GS 2010 Paper I 212 YCT
Ans. (b) : The Finance Ministry of India is an important 60. In the context of Indian economy, consider the
ministry concerned with the economy of India. It is following pairs:
comprised of five departments one of which is the Term Most appropriate description
Department of Economic Affairs. It is the nodal agency 1. Melt down............Fall in stock prices
of central government whose principal task is to prepare 2. Recession..............Fall in growth rate
and present the Union Budget before the Parliament. 3. Slow down............Fall in GDP
57. Consider the following actions by the Which of the pairs given above is/are correctly
Government: matched?
1. Cutting the tax rates (a) 1 only (b) 2 and 3 only
2. Increasing the government spending (c) 1 and 3 only (d) 1, 2 and 3
3. Abolishing the subsidies Ans. (a) : Melt down is the term used for weakness in the
In the context of economic recession, which of financial markets which implies security prices. Recession
the above actions can be considered a part of means negative growth and fall in GDP. Slow down means
reduction in the growth rate of GDP.
the "fiscal stimulus" package?
(a) 1 and 2 only (b) 2 only 61. In India, the interest rate on savings accounts
in all the nationalized commercial banks is
(c) 1 and 3 only (d) 1, 2 only 3
fixed by:
Ans. (a) : In the 'fiscal stimulus' package either taxes are (a) Union Ministry of Finance
reduced or government spending is increased to increase (b) Union Commission of Finance
the purchasing power of the people. (c) Indian Banks Association
58. When the Reserve Bank of India announces an (d) None of the above
increase of the Cash Reserve Ratio, what does Ans. (d) : The interest rate on savings accounts in all the
it mean? nationalised commercial banks is fixed by the Reserve
(a) The commercial banks will have less money to Bank of India (RBI). This is done to ensure that the banks
lend do not discriminate among the depositors and also to create
(b) The Reserve Bank of India will have less money a level-playing field in the banking sector.
to lend RBI, however, allows the nationalised commercial banks
(c) The Union Government will have less money to to slightly moderate the interest rates offer on term
lend deposits.
(d) The commercial banks will have more money to 62. Which of the following terms indicates a
lend mechanism used by commercial banks for
Ans. (a) : Increase in Cash Reserve Ratio (CRR) means providing credit to the government?
that the banks have to deposit more with the RBI and will (a) Cash Credit Ratio
have less to lend to the borrowers. (b) Debt Service Obligation
59. With reference to India, consider the following (c) Liquidity Adjustment Facility
statements: (d) Statutory Liquidity Ratio
1. The Wholesale Price Index (WPI) in India Ans. (d) : Statutory Liquidity Ratio (SLR) is the
is available on a monthly basis only. mechanism used by the RBI to regulate the liquidity of the
commercial banks. Under SLR, commercial banks invest a
2. As compared to Consumer Price Index for
certain percentage of their money in Government securities
Industrial Workers (CPI/(IW)), the WPI
and other assets like gold so as to comply with RBI
gives less weight to food articles. guidelines.
Which of the statements given above is/are
63. In order to comply with TRIPS Agreement, India
correct?
enacted the Geographical Indications of Goods
(a) 1 only (b) 2 only (Registration & Protection) Act, 1999. The
(c) Both 1 and 2 (d) Neither 1 and 2 difference/differences between a "Trade Mark"
Ans. (b) : As compared to Consumer price Index for and a Geographical Indication is/are
Industrial Workers (CPI/(IW)), the WPI gives less 1. A Trade Mark is an individual or a company's
weight to food articles this statement will be correct. right whereas a Geographical Indication is a
But acceding to present data both statement are correct community's right.
because earlier WPI was available on weekly basis. 2. A Trade Mark can be licensed whereas a
Now it has changed to monthly basis. Weight of food Geographical Indication cannot be licensed
items-WPI-food Articles-15.402% and food products- 3. A Trade Mark is assigned to the manufacture
11.538%, Consumer Price Index for Industrial workers goods whereas the Geographical Indication is
(CPI (IW)]-57%. The weightage of primary articles in assigned to the agricultural goods/products
new WPI is 22.62% in 2017. and handicrafts only.
IAS (Pre) GS 2010 Paper I 213 YCT
Which of the statements given above is/are (c) Elimination of primary deficit by the end of
correct? the fiscal year 2008-09
(a) 1 only (b) 1 and 2 only (d) Fixing government guarantees in any
(c) 2 and 3 only (d) 1, 2 and 3 financial year as a percentage of GDP
Ans. (b) : A trademark is a distinctive sign or indicator Ans. (c) : In the FRBM Act, the target has been set for
used by an individual, business organization, or other reduction in revenue deficit and fiscal deficit only and
not for primary deficit.
legal entity to identify that the products or services to
consumers with which the trademark appears originate 67. In the parlance of financial investments, the
from a unique source, and to distinguish its products or term 'bear' denotes
services from those of other entities. The owner of a (a) An investor who feels that the price of particular
security is going to fall
registered trademark may commence legal proceedings
(b) An investor who expects the price of particular
for trademark infringement to prevent unauthorized use
shares to rise
of that trademark. However, registration is not required.
(c) A shareholder or a bondholder who has an
64. Which one of the following statements is an interest in a company, financial or otherwise
appropriate description of deflation? (d) Any lender whether by making a loan or buying
(a) It is a sudden fall in the value of a currency a bond.
against other currencies Ans. (a) : The term bear denotes an investor who believes
(b) It is a persistent recession in both the financial that a particular security or market is headed downward.
and real sectors of economy Bears attempt to profit from a decline in prices. Bears are
(c) It is a persistent fall in the general price level of generally pessimistic about the state of a given market.
goods and services Bearish sentiment can be applied to all types of markets
(d) It is a fall in the rate of inflation over a period of including commodity markets, stock markets and the bond
time market.
Ans. (c) : When there is a continuous fall in the price PHYSICS
levels of goods and services over a period of time, it is
called deflation. Under such condition, the rate of inflation 68. What is the principle by which a cooling system
falls below 0%. (Radiator) in a motor car works?
65. With reference to the Non-banking Financial (a) Conduction only
Companies (NBFCs) in India, consider the (b) Convection
following statements: (c) Radiation only
1. They cannot engage in the acquisition of (d) Both conduction and radiation
securities issued by the government. Ans. (b) : In a car radiator, the coolant liquid flows
2. They cannot accept demand deposits like through the tubes inside combustion engine and absorbs
Savings the engine's heat and in-turn gets heated itself.
Which of statements given above is/are The heated fluid then makes its way through a rubber
hose to the radiator in the front of the car. As it flows
correct?
through the thin tubes in the radiator, the hot liquid is
(a) 1 only (b) 2 only cooled by the air stream entering the engine
(c) Both 1 and 2 (d) Neither 1 nor 2 compartment from the grill in front of the car. Once the
Ans. (b) : A Non-Banking Financial Company (NBFC) fluid is cooled, it returns to the engine to absorb more
is a company registered under the Companies Act, heat.
1956. NBFCs lend and make investments and hence The air picks up heat from the coolant by convection
their activities are akin to that of banks however process, i.e. heated air is blown away by the fan, and
they cannot accept demand deposits. Also, they do not colder air replaces it.
form part of the payment and settlement system of RBI
and cannot issue cheques drawn on itself. CHEMISTRY
66. Which one of the following was not stipulated 69. Excessive release of the pollutant carbon
in the Fiscal Responsibility and Budget monoxide (CO) into the air may produce a
Management Act, 2003? condition in which oxygen supply in the human
(a) Elimination of revenue deficit by the end of body decreases. What causes this condition?
the fiscal year 2007-08 (a) When inhaled into the human body, CO is
(b) Non-borrowing by the central government converted into CO2
from Reserve Bank of India except under (b) The inhaled CO has much higher affinity for
certain circumstances haemoglobin as compared to oxygen.
IAS (Pre) GS 2010 Paper I 214 YCT
(c) The inhaled CO adversely affects the (a) Potato is mainly made up of starch whereas
respiratory centre in the brain. paper is mainly made up of cellulose
(d) the inhaled CO adversely affects the (b) Patato transmits microwaves whereas paper
respiratory centre in the brain. reflects microwaves
Ans. (b) : Carbon Monoxide pollution occurs primarily (c) Potato contains water whereas paper does not
from emissions produced by fossil fuel powered contain water
engines. The incomplete reaction of air with fuel (d) Potato is a fresh organic material whereas
produces the colourless, odourless and highly toxic gas. paper is a dead organic material
The main issue with Carbon Monoxide is its health Ans. (c) : Potato contains water whereas paper does not
effects. It is capable of binding to the chemicals in our contain water.
blood, called haemoglobin. It does so far more
effectively than oxygen and also stays bound to the 74. Chlorination is a process used for water
haemoglobin for far longer than oxygen does. The purification. The disinfecting action of chlorine
effect is that the blood is starved of oxygen, which then is mainly due to
affects the rest of the body. (a) The formation of hydrochloric acid when
chlorine is added to water
70. Indiscriminate disposal of used florescent
electric lamps causes mercury pollution in the (b) The formation of hypochlorous acid when
environment. Why is mercury used in the chlorine is added to water
manufacture of these lamps? (c) The formation of nascent oxygen when
(a) A mercury coating on the inside of the lamp chlorine is added to water
makes the light bright white (d) The formation of hydrogen when chlorine is
(b) When the lamp is switched on, the mercury in added to water
the lamp causes the emission of ultra-violet Ans. (b) : Water chlorination is the process of adding
radiations chlorine or hypochlorite to water this method is used to
(c) When the lamp is switched on, it is the kill certain bacteria and other microbes as Chlorine is
mercury which converts the ultra-violet highly toxic.
energy into visible light.
(d) None of the statement given above is correct BIOLOGY
about the use of mercury in the manufacture
of fluorescent lamps 75. Widespread resistance of malarial parasite to
drugs like chloroquine has prompted attempts to
Ans. (c) : When the lamp is switched on, it is the
mercury which converts the ultra-violet energy into develop a malarial vaccine to combat malaria.
visible light. . Why is it difficult to develop an effective malaria
vaccine?
71. Hydrogen fuel cell vehicles produce one of the
(a) Malaria is caused by several species of
following as "exhaust"
plasmodium
(a) NH3 (b) CH4
(b) Man does not develop immunity to malaria
(c) H2O (d) H2O2
during natural infection
Ans. (c) : A hydrogen vehicle is an alternative fuel (c) Vaccines can be developed only against bacteria
vehicle that uses hydrogen as its onboard fuel for
(d) Man is only an intermediate host and not the
motive power. The hydrogen vehicle use hydrogen fuel
definitive host
cell for generation of motive power. These fuel cell in
which hydrogen serve as a fuel and oxygen as an Ans. (b) : It is difficult to develop an effective malaria
oxidant emits exhaust of water. vaccine, because repeated natural infection does not
72. Consider the following: produce detectable resistance to exo-erythrocytic cycle
of malaria in man.
1. Oxides of Hydrogen
2. Oxides of Nitrogen 76. Which one of the following processes in the bodies
3. Oxides of Sulphur of living organisms is a digestive process?
Which of the above causes/cause acid rain? (a) Breakdown of proteins into amino acids
(a) 1 and 2 only (b) 3 only (b) Breakdown of glucose into CO2 and H2O.
(c) 2 and 3 only (d) 1, 2 and 3 (c) Conversion of glucose into glycogen
Ans. (c) : Acid rain is caused by Oxides of Nitrogen, (d) Conversion of amino acids into proteins.
Oxides of Sulphur. Ans. (a) : Dietary protein is degraded into the amino
73. If a potato is placed on a pure paper plate which acids and their derivatives are then absorbed by the
is white and unprinted and put in a microwave gastrointestinal tract. Digestion is a catabolic process in
oven, the potato heats up but the paper plate does which large molecules of the food gets broken out into
not. This is because: small substances.

IAS (Pre) GS 2010 Paper I 215 YCT


It typically begins in the stomach when pepsinogen is (a) The chances of transmission from female to
converted to pepsin by the action of hydrochloric acid male are twice as likely as from male to
and continued by trypsin and chymotrypsin in the female.
intestine. During respiration breakdown of glucose (b) The chances of transmission are more if a
into CO2 and H2O takes place. person suffers from other sexually transmitted
Conversion of glucose into glycogen is a glycogenesis infections
process. It occurs in liver. (c) An infected mother can transmit the infection
During translation, amino acids translated into to her baby during pregnancy, at childbirth
proteins. and by breast feeding.
77. From the point of view of evolution of living (d) The risk of contracting infection from
organisms, which one of the following it the transfusion of infected blood is much higher
correct sequence of evolution? than an exposure to contaminated needle.
(a) Otter-Tortoise-Shark Ans. (a) : Human Immunodeficiency Virus (HIV) is the
(b) Shark-Tortoise-Otter retrovirus that causes Acquired Immunodeficiency
(c) Tortoise-shark-Otter Deficiency Syndrome (AIDS). The virus weakens a
person's ability to fight infections and cancer by infecting
(d) Shark-Otter-tortoise
and killing CD4 cells (also called T cells). Women account
Ans. (b) : Correct sequence of evolution: for more than half the number of people living with HIV
Shark-Tortoise-Otter. worldwide. Young Women (10-24 years old) are twice as
78. Consider the following statements: likely to acquire HIV as young men the same age. HIV is
1. Hepatitis B is several times more infectious transmitted by
than HIV/AIDS (i) Infected blood
2. Hepatitis B can cause liver cancer (ii) Unsterilised injections
Which of the statements given above is/are (iii) Unprotected sex
correct? (iv) Infants from infected mothers
(a) 1 only (b) 2 only 81. King Cobra is the only snake that makes its
(c) Both 1 and 2 (d) Neither 1 nor 2 own nest. Why does it make its nest?
Ans. (c) : Both 1 and 2 . So, the correct answer is option (a) It is a snake-eater and the nest helps attract
(c). other snakes
Hepatitis B is much more infectious than AIDS. It can (b) It is a viviparous snake and needs a nest to
cause liver cancer. give birth to its offspring
79. Consider the following statements: (c) It is an oviparous snake and lays its eggs in
1. Every individual in the population is equally the nest and guards the next until they are
susceptible host for Swine Flu. hatched
2. Antibiotics have no role in the primary (d) It is a large, cold blooded animal and needs a
treatment of Swine Flu. nest to hibernate in the cold season
3. to prevent the future spread of Swine Flue Ans. (c) : It is an oviparous snake and lays its eggs in
in the epidemic area, the swine (pigs) must the nest and guards the nest until they are hatched.
all be culled.
Which of the statements given above is/are ENVIRONMENT AND ECOLOGY
correct?
(a) 1 and 2 only (b) 2 only 82. What are the possible limitations of India in
mitigating the global warming at present and
(c) 2 and 3 only (d) 1, 2 and 3
in the immediate future?
Ans. (a) : Swine influenza is a respiratory disease of pigs
1. Appropriate alternate technologies are not
caused by type A influenza viruses that regularly cause
sufficiently available.
outbreaks of influenza in pigs. Influenza viruses that
commonly circulate in swine are called 'swine influenza 2. India cannot invest huge funds in research
viruses or swine flu viruses. H1N1 antiviral agents used for and development.
treatment and prevention of swine flu are oseltamiuir 3. Many developed countries have already set
(Tamiflu) and zanamivir (Relenza). Swine flu is resistant up their polluting industries in India.
to amantadine (Symmetrel) and rimantadine (Flumadine). Which of the statements given above is/are
80. With regard to the transmission of the Human correct?
Immunodeficiency Virus, which one of the (a) 1 and 2 only (b) 2 only
following statements is not correct? (c) 1 and 3 only (d) 1, 2 and 3
IAS (Pre) GS 2010 Paper I 216 YCT
Ans. (a) : The correct answer is (a) only 1 and 2. growth of cancer cells and slows their growth and
Possible limitations of India in mitigating the global spread in the body.
warming are lack of alternate technology to reduce Taxol is used to treat breast cancer, lung cancer and
emissions and being a developing country, India cannot ovarian cancer. It is also used to treat AIDS related
invest huge amount of fund in research and Kaposi's sarcoma.
development work. India itself contributes a lot in 85. Due to their extensive rice cultivation, some
global warming due to its own industries, vehicles, less regions may be contributing to global warming.
forest cover etc. To what possible reason/reasons is this
83. Consider the following which can be found in attributable?
the ambient atmosphere: 1. The anaerobic conditions associated with rice
1. Soot cultivation cause the emission of methane.
2. Sulphur hexafluoride 2. When nitrogen based fertilizers are used,
3. Water vapour nitrous oxide is emitted from the cultivated
soil.
Which of the above contribute to the warming
Which of the statements given above is/are
up of the atmosphere?
correct?
(a) 1 and 2 only (b) 3 only
(a) 1 only (b) 2 only
(c) 2 and 3 only (d) 1, 2 and 3
(c) Both 1 and 2 (d) Neither 1 nor 2
Ans. (d) : All above mentioned (soot, sulphur
Ans. (c) : Methane and nitrous oxide are the two green
hexafluoride, water vapour) contribute to the warming up
house gases emitted from rice field. In rice field, the
of the atmosphere.
flooding a field cuts off the oxygen supply from
Soot mostly from diesel engines is blocking snow and ice
atmosphere resulting anaerobic condition which emits
reflecting sunlight, which is contributing to near
methane and nitrogen fertilizer which generally used in
worldwide melting of ice and as much as a quarter of all rice field for high production, emits nitrous oxide by the
observed global warming. reaction with atmospheric oxygen.
SF6 (Sulphur hexafluoride) is the most potent as
86. Sustainable development is described as the
greenhouse gas that has been evaluated with a global
development that meets the needs of the present
warming potential of 23,900 times that is CO2 when
without compromising the ability of future
compared over a 100 years period.
generations to meet their own needs. In this
Greenhouse effect is caused by atmospheric trace gases
perspective, inherently the concept of sustainable
such as CO2, O3 (ozone) and water vapour (H2O). development is intertwined with which of the
Infrared radiation from the Earth is stored temporarily following concepts?
in the atmosphere. Of all these, trace gases, water
(a) Social justice and empowerment
vapour represents the most important constituent. It
(b) Inclusive Growth
contributes to the natural greenhouse warming process
by approximately 60%. (c) Globalization
(d) Carrying capacity
84. Consider the following statements:
1. The Taxus tree naturally found in the Ans. (d) : The correct answer is option (b).
Himalayas. 87. Given below are the names of four energy
2. The Taxus tree is listed in the Red Data crops. Which one of them can be cultivated for
Book. ethanol?
3. A drug called "taxol" is obtained from (a) Jatropha (b) Maize
Taxus tree is effective against Parkinson's (c) Pongamia (d) Sunflower
disease. Ans. (b) : Maize can be cultivated for ethanol.
Which of the statements given above is/are 88. Some species of plants are insectivorous. Why?
correct? (a) Their growth in shady and dark places does
(a) 1 only (b) 1 and 2 only not allow them to undertake sufficient
(c) 2 and 3 only (d) 1, 2 and 3 photosynthesis and thus they depend on
Ans. (b) : Taxus Wallichiana Zucc (Taxus baccata L.) insects for nutrition
naturally occurs in the Himalayas. It has been included (b) They are adopted to grow in nitrogen
in the Red Data Book. The Taxol molecule, however, is deficient soils and thus depend on insects for
very complex and therefore almost impossible to sufficient nitrogenous nutrition
reproduce in the laboratory. One would have to rely on (c) They cannot synthesize certain vitamins
extractions of the Taxol compound the Taxus tree. themselves and depend on the insects
Taxol is a cancer medication that interferes with the digested by them
IAS (Pre) GS 2010 Paper I 217 YCT
(d) They have remained in that particular stage of (a) Keep all the sites completely inaccessible to
evolution as living fossils, a link between man so that they will not be exploited
autotrophs and heterotrophs (b) Conserve all the sites through ecosystem
Ans. (b) : Insectivorous plants prey insects because approach and permit tourism and recreation
most of the insectivorous plants grow in those area only
where there is deficit of nitrogen in nutrients so they (c) Conserve all the sites through ecosystem
catch insects to fulfill its need of nutrients. approach for a period without any exploitation,
89. Which one of the following reflects back more with specific period for each site, and then allow
sunlight as compared to other three? sustainable use of them by future generations.
(a) Sand desert (d) Conserve all the sites through ecosystem
(b) Paddy crop land approach and allow their simultaneous
(c) Land covered with fresh snow sustainable use
(d) Prairie land Ans. (c) : Conserve all the sites through ecosystem
Ans. (c) : Land covered with fresh snow reflects back approach for a period without any exploitation with
more sunlight as compared to other three. Fresh snow and specific period for each site, and then allow sustainable use
ice reflect 80 - 85% of the sunlight. Therefore, the skin of of them by future generations.
skiers is irradiated by sunlight from all sides, since the
93. Other than Jatropha curcas, why is Pongamia
snow does not take up the light but reflects it and remains
pinnata also considered a good option for the
cold. Scientists give a name to this ability of surfaces to
production of bio diesel in India?
reflect light.
1. Pongamia pinnata grows naturally in most
90. Which bacterial strain, developed from natural
of the arid regions of India
isolates by genetic manipulations, can be used
for treating oil spills? 2. The sees of Pongamia pinnata are rich in lipid
(a) Agrobacterium (b) Clostridium content of which nearly half is oleic acid.
(c) Nitrosomonas (d) Pseudomonas Which of the statements given above is/are
correct?
Ans. (d) : Pseudomonas- bacterial strain, developed
from natural isolates by genetic manipulations, can be (a) 1 only (b) 2 only
used for treating oil spills. (c) Both 1 and 2 (d) Neither 1 nor 2
91. Which feature of some species of blue green Ans. (c)
algae helps promote them as bio-fertilizers? ⇒ Oil extracted from the Pongamia Pinnata seeds is
(a) They convert atmospheric methane into the best substitute for kerosene for lighting lamps
ammonia which the crop plants can absorb in rural areas.
readily ⇒ The oil extracted has yet not used on a commercial
(b) They induce the crop plants to produce the level, but due to an increase in awareness and
enzymes which help convert atmospheric growth in research, Pongamia can be developed as
nitrogen to nitrates an alternative source of fuel that can replace diesel
(c) They have the mechanism to convert from the market.
atmospheric nitrogen into a form that the crop ⇒ The oil extracted is commonly known as 'Karanja
plants can absorb readily
oil'.
(d) They induce the roots of the crop plants to
absorb the soil nitrates in larger quantities ⇒ Pongamia Pinnata is adaptable to arid zones, hence
grows naturally in most of the arid regions of India.
Ans. (c) : During the crop growth cycle, the blue-green
Hence statement 1 is correct.
algae grow, multiply, fix atmospheric nitrogen and
make it available to the crop by way of excretion and ⇒ Reasons for using Pongamia Pinnata as a source of
autolysis. biodiesel:
Cyanobacteria, also known as blue-green algae (BGn). • It has a good nitrogen- fixing ability.
They have specialised cell i.e., heterocyst- It fixes • The plant has the ability to grow in waterlogged,
nitrogen. BGn form a significant component of the saline, and alkaline soil, wasteland and can
marine nitrogen cycle and an important primary withstand harsh agro climates.
producer in many areas of the ocean, but are also found • Pongamia Pinnata seeds contain 30% - 40% oil and
in habitats other than the marine environment. little amount of free fatty acids. Monounsaturated
92. India is a party to the Ramsar Convention and and polyunsaturated fatty acids accounted for
has declared many areas as Ramsar Sites. approx. 63% and 23%, respectively, of the seed oil
Which of the following statements best and Oleic acid, is the major fatty acid found in the
describes as to how we should maintain these P. Pinnata seeds. Hence statement 2 is correct.
sites in the context of this Convention? Hence option (c) is correct.
IAS (Pre) GS 2010 Paper I 218 YCT
94. Consider the following statements: 97. With reference to soil conservation, consider
1. Biodiversity hotspots are located only in the following practices:
tropical regions 1. Crop rotation
2. Indian has four biodiversity hotspots i.e., 2. Sand fences
Eastern Himalayas, Western Ghats and 3. Terracing
Andaman and Nicobar Islands. 4. Wind breaks
Which of the statements given above is/are Which of the above are considered appropriate
correct? methods for soils conservation in India?
(a) 1 only (b) 2 only (a) 1, 2 and 3 only
(c) Both 1 and 2 (d) Neither 1 nor 2 (b) 2 and 3 only
Ans. (d) : Both the statements are incorrect. Currently, (c) 1, 3 and 4 only
36 biodiversity hotspots have been identified, most of (d) 1, 2, 3 and 4
which occur in tropical forests. They represent just Ans. (d) : Crop rotation, Sand fences, terracing and
2.3% of Earth's land surface, but between them they wind breaks all are used for sand conservation in India.
contain around 50% of the world's endemic plant Sand fences is a simple and cheap method of inducing
species and 42% of all terrestrial vertebrates. India hosts accretion of sands in arid and sandy areas.
4 biodiversity hotspots: the Himalayas, the Western
Ghats, the Indo-Burma region and the Sundaland CURREN AFFAIRS
(Includes Nicobar group of Islands).
95. Consider the following statements: 98. As per the UN-Habitat's Global Report on
1. The boundaries of a National Park are Human Settlements 2009, which one among the
defined by legislation. following regions has shown the fastest growth
2. A Biosphere Reserve is declared to conserve rate of urbanization in the last three decades?
a few specific species of flora and fauna. (a) Asia
3. In a Wildlife Sanctuary, limited biotic (b) Europe
interference is permitted. (c) Latin America and Caribbean
Which of the statements given above is/are (d) North America
correct? Ans. (a)
(a) 1 only (b) 2 and 3 only • UN- Habitat's Global Report on Human Settlement
(c) 1 and 3 only (d) 1, 2 and 3 2009.
Ans. (c) : Biosphere Reserve declared forests means to • According to the report, urban growth will be less
conserve some representative ecosystems as a whole for rapid in developed regions, in Latin America and the
long-term in situ. Since 1970, legislations like Wildlife Caribbean, and in transitional countries of East
Protection Act are used to specify borders of National Europe, all of which are already highly urbanized,
Parks and Wildlife Sanctuary. but rapid in Africa and Central, South and East Asia,
There are three zones in a wildlife sanctuary-core zone (no which are currently less urbanized. China is expected
interference allowed), buffer zone (eco-development and to double its urban population from about 40% of its
forest improvement, regulated pilgrimage, research national population from 2006 to 2030 to more than
activities are allowed) and transition zone (tourism is 70% by 2050.
allowed). • The report points out the fact that urbanization in
96. A pesticide which is a chlorinated hydrocarbon Asia is mainly due to the high level of mobility of the
is sprayed on a food crop. The food chain is: population from villages to the cities.
Food crop-Rat-Snake-Hawk. In this food chain, • The chart and the table are shown below will provide
the highest concentration of the pesticide would the global trends in urbanization:
accumulate in which one of the following? The correct answer is Asia (a).
(a) Food crop (b) Rat 99 In the context of bilateral trade negotiations
(c) Snake (d) Hawk between India and European Union, what is the
Ans. (d) : The correct answer is hawk. difference between European Commission and
Hawks are a group of medium-sized diurnal birds of European Council?
prey of the family Accipitridae. Hawks are widely 1. European Commission represents the EU in
distributed and vary greatly in size. the subfamily trade negotiations whereas European Council
Accipitrinae includes goshawks, sparrowhawks, sharp- participates in the legislation of matters
shinned hawks and others. This subfamily are mainly pertaining to economics policies of the
woodland birds with long tails and high visual acuity. European Union.
IAS (Pre) GS 2010 Paper I 219 YCT
2. European Commission comprises the Heads of financing selected social sector schemes which promote
State or government of member countries education, health and employment. The residual 25% of
where as the European Council comprises of the annual income of NIF was to be used to meet the
the persons nominated by European capital investment requirements of profitable and
Parliament. revivable PSUs.
Which of the statements given above is/are 101. Consider the following statements:
correct? 1. The Union Government fixes the Statutory
(a) 1 only (b) 2 only Minimum Price of sugarcane for each sugar
(c) Both 1 and 2 (d) Neither 1 nor 2 season.
Ans. (d) : The European Commission has four 2. Sugar and sugarcane are essential
responsibilities that are central to the EU’s operation; it commodities under the Essential Commodities
proposes legislation designed to implement the Act.
objectives outlined in EU treaties, it manages EU policy Which of the statements given above is/are
and the EU budget, it acts as the guardian of the treaties, correct?
monitoring compliance with EU law and referring (a) 1 only (b) 2 only
suspected cases of non-compliance to the Courts and it (c) Both 1 and 2 (d) Neither 1 nor 2
represents the EU in external trade negotiations. The
Ans. (c) :Statutory minimum price of sugar come for
European Council is the EU’s supreme political body.
every season is a policy provision while sugar and sugar
The European Council brings together the political
cane falls within essential commodities act.
leaders of the member states. The President of the
European Commission is also a member. It sets out 102. The International Development Association, a
general policy objectives for the EU to follow, deals lending agency, is administered by the:
with sensitive political matters and acts as a trouble (a) International Bank for Reconstruction and
shooter for areas of business where ministers are unable Development
to agree. (b) International Fund for Agricultural Development
100. With reference to the National Investment (c) United Nations Development Programme
Fund to which the disinvestment proceeds are (d) United Nations Industrial Development
routed, consider the following statements: Organization
1. The assets in the National Investment Fund are Ans. (a) : The International Development Association
managed by the Union Ministry of Finance. (IDA) is the part of the World Bank that helps the
2. The National Investment Fund is to be world’s poorest countries. It is headquartered in
maintained within the Consolidated fund of Washington, D.C., United States. It was established in
the India. 1960. Together the IBRD and the IDA are called as the
3. Certain Asset Management Companies are World Bank. The IBRD and IDA share the same staff
appointed as the fund managers. and Headquarters and evaluate projects with the same
4. A certain proportion of annual income is used rigorous standards.
for financing select social sectors. 103. With reference to the National Rehabilitation
Which of the statements given above is/are and Resettlement Policy, 2007 consider the
correct? following statements:
(a) 1 and 2 (b) 2 only 1. This policy is applicable only to the persons
(c) 3 and 4 (d) 3 only affected by the acquisition of land for
Ans. (c) : Only 3 and 4 correct. So, the correct answer projects and not to the involuntary
is (c). displacement due to any other reason.
Government of India had constituted the National 2. This policy has been formulated by the
Investment Fund (NIF) in November, 2005 into which Ministry of Social Justice and Empowerment.
the proceeds from disinvestment of Central Public Which of the statements given above is/are
Sector Enterprises were to be channelized. The corpus correct?
of NIF was to be of a permanent nature and NIF was to (a) 1 only (b) 2 only
be professionally managed to provide sustainable (c) Both 1 and 2 (d) Neither 1 nor 2
returns to the Government, without depleting the
Ans. (d) : It has been formulated by ministry of rural
corpus. Selected Public Sector Mutual Funds namely
UTI Asset Management Company Ltd., SBI Funds development.
Management Private Ltd. and LIC Mutual Fund Asset 104. Two of the schemes launched by the Government
Management Company Ltd. were entrusted with the of India for Women's development are Swadhar
management of the NIF corpus. As per this Scheme, and Swayam Siddha. As regards the difference
75% of the annual income of the NIF was to be used for between them, consider the following statements:
IAS (Pre) GS 2010 Paper I 220 YCT
1. Swayam Siddha is meant for those in difficult Ans. (c) : ISRO launched the beta version of its web-
circumstances such as women survivors of based 3-D satellite imagery tool, Bhuvan, in 2009. It
natural disasters or terrorism, women offers superior imagery of Indian locations compared to
prisoners released from jails, mentally other virtual Globe software. Bhuvan is a satellite
changed women etc. whereas Swadhar is mapping tool similar to Google Earth. It offers
meant for holistic empowerment of women resolution upto 10 metres and is considered as a rival to
through Self Help Groups. Google Earth and Wikimapia. On its 5th anniversary on
2. Swayam Siddha is implemented through 12th, August, 2014, Bhuvan has rolled our platform and
Local Self Government bodies or reputed plug-in independent Bhuvan 3D. It is available in
Voluntary Organizations whereas Swadhar is English, Hindi, Tamil and Telugu.
implemented through the ICDS units set up in
107. Which among the following do/does not
the state.
belong/belongs to the GSM family of wireless
Which of the statements given above is/are
technologies?
correct?
(a) EDGE
(a) 1 only (b) 2 only
(b) LTE
(c) Both 1 and 2 (d) Neither 1 nor 2
(c) DSL
Ans. (d) : Swayamsiddha is an integrated programme
(d) Both EDGE and LTE
for the empowerment of women through the network of
Self-Help Groups of women. The programme shall be Ans. (c) : DSL does not belong to the GSM family of
implemented through the State government in 650 wireless technology.
blocks throughout the country, but the State 108. With reference to the treatment of cancerous
governments will be at liberty to choose the tumours, a tool called cyber knife has been
implementing authority. making the news. In this context, which one of
Swadhar scheme, launched in 2001, provides for the following statements is not correct?
holistic rehabilitation of women in difficult (a) It is a robotic image guided system
circumstances. Twenty-two projects for the (b) It delivers an extremely precise dose of
rehabilitation of the widows, women affected by riots in radiation
Gujarat, and women and girls rescued from prostitution (c) It has the capability of achieving sub-
are in operation at present. So both the statements are millimetre accuracy
wrong.
(d) It can map the spread of tumour in the body
105. As a result of their annual survey, the National
Ans. (d) : Cyber knife has the capability of achieving sub-
Geographic Society and an international polling
millimeter accuracy is not correct.
firm Globe Scan gave India top rank in Greendex
2009 score. What it this score. 109. India-based Neutrino Observatory is included
(a) It is a measure of efforts made by different by the Planning commission as a mega science
countries in adopting technologies for reducing project under the 11th Five Year Plan. In this
carbon footprint context, consider the following statements:
(b) It is a measure of environmentally sustainable 1. Neutrinos are charge less elementary
consumer behaviour in different countries particles that travel close to the speed of
(c) It is an assessment of programmes/schemes light.
undertaken by different countries for improving 2. Neutrinos are created in nuclear reactions
the conservation of natural resources of beta decay.
(d) It is an index showing the volume of carbon 3. Neutrinos have a negligible, but nonzero
credits sold by different countries mass.
Ans. (b) : In the third annual survey of National 4. Trillions of Neutrinos pass through human
Geographic Society and Globe Scan, Indian top ranked in body every second.
Greendex 2009 due to environmentally sustainable Which of the statements given above are correct?
consumer behaviour. (a) 1 and 3 only (b) 1, 2 and 3 only
106. In the context of space technology, what is (c) 2, 3 and 4 (d) 1, 2, 3 and 4
"Bhuvan", recently in the news? Ans. (d) : Neutrinos are nearly massless, chargeless
(a) A mini satellite launched by ISRO for particles that travel at or near the speed of light. There
promoting the distance education in India are three types of neutrinos: electron neutrinos, muon
(b) The name given to the next Moon Impact Prove, neutrinos and tauon neutrinos. Electron neutrinos (or
for Chandrayan-II antineutrinos) are generated whenever protons change
(c) A geo-portal of ISRO with 3D imaging into neutrons (or neutrons into protons) the two forms
capabilities of India of beta decay. More than 50 trillion solar neutrinos pass
(d) A space telescope developed by India. through the human body every second.

IAS (Pre) GS 2010 Paper I 221 YCT


110. Recently, LASIK (Laser Assisted In Situ 113. An objective of the National Food Security
Keratomileusis) procedure is being made Mission is to increase the production of certain
popular for vision correction. Which one of the crops through area expansion and productivity
following statements in this context is not enhancement in a sustainable manner in the
correct? identified districts of the country. What are
(a) LASIK procedure is used to correct refractive those crops?
errors of the eye (a) Rice and wheat only
(b) It is a procedure that permanently changes the (b) Rice, wheat and pulses only
shapes of the cornea (c) Rice, wheat, pulses and oil seeds only
(c) It reduces a person's dependence on glasses or (d) Rice, wheat, pulses, oil seeds and vegetables
contact lenses Ans. (b) : The target is to increase production of rice,
(d) It is a procedure that can be done on the wheat and pulses by 10,8 and 2 million tonnes respectively
person of any age by end of the Eleventh Plan.
Ans. (d) : In order to undergo LASIK procedure one 114. Mon 863 is a variety of maize. It was in the
should be at least 18 years old, as the vision of people news for the following reason:
younger than 18 usually continues to change. (a) It is a genetically modified dwarf variety
111. With reference to BRIC countries, consider the which is resistant to drought
following statements: (b) It is a genetically modified variety which is
1. At present, China's GDP is more than the pest resistant
combined GDP of all the three other (c) It is a genetically modified variety with then
countries. times higher protein content than regular
2. China's population is more than the combined maize crop
population of any two other countries. (d) It is a genetically modified variety used
Which of the statements given above is/are exclusively for bio-fuel production
correct? Ans. (b) : Mon 863 is genetically modified variety of
(a) 1 only (b) 2 only maize, which is resistant of corn root worm.
(c) Both 1 and 2 (d) Neither 1 and 2 115. Consider the following statement:
Ans. (a) : With reference to BRIC countries at present, The satellite Oceansat-2 launched by India
China's GDP is more than the combined GDP of all the helps in:
three other countries. The GDP of China was 6.09 lakh 1. Estimating the water vapour content in the
crores USD (2010). While India has a GDP of 1.68 lakh atmosphere
crores USD (2010), 2.21 lakh crores USD, and Russia had 2. Predicting the onset of monsoons.
a GDP of 1.52 lakh crores USD (2010). So China's GDP 3. Monitoring the pollution of coastal waters.
was more than the sum of all three other countries. Hence Which of the statements given above is/are
statement 1 is correct. According to Current Data BRICS correct.
is an acronym for the grouping of the world's leading (a) 1 and 2 only (b) 2 only
emerging economies, namely Brazil, Russia, India, China,
(c) 1 and 3 only (d) 1, 2 and 3
and South Africa.
Ans. (d) : Oceansat-2 continues with the service of
Hence answer is (a) only 1.
oceansat-1. The main objectives of oceansat-2 are to study
112. Stiglitz Commission established by the surface winds and ocean surface strata, observation of
President of the United Nations General chlorophyll concentration, monitoring of phytoplanktion
Assembly was in the international news. The blooms, study of atmospheric aerosol and suspended
commission was supposed to deal with sediments in the water.
(a) The challenges posed by the impending
116. Genetically modified "golden rice" has been
global climate change and prepare a road map
engineered to meet human nutritional requirements.
(b) The workings of the global financial systems Which one of the following statements best qualifies
and to explore ways and means to secure a golden rice?
more sustainable global order (a) The grains have been fortified with genes to
(c) Global terrorism and prepare a global action provide three times higher grain yield per acre
plan for the mitigation of terrorism than other high yielding varieties
(d) Expansion of the United Nations Security (b) Its grains contain pro-vitamin A which upon
Council in the present global scenario ingestion is converted to vitamin A in the
Ans. (b) : The workings of the global financial systems human body
and to explore ways and means to secure a more (c) Its modified genes cause the synthesis of all
sustainable global order. the nine essential amino acids.
IAS (Pre) GS 2010 Paper I 222 YCT
(d) Its modified genes cause the fortification of • It is the most widely ratified human rights treaty in
its grains with vitamin D. the history of the world.
Ans. (b) : Its grains contain pro-vitamin A which upon • Under the articles of the convention, all parties to it
ingestion is converted to vitamin A in the human body, are required to ensure that children's basic needs are
117. A great deal of Foreign Direct Investment fulfilled and they are able to reach their full potential.
(FDI) to India comes from Mauritius than from The correct answer is option (d).
many major and mature economics like UK 119. Consider the following countries:
and France. Why? 1. Brazil
(a) India has preference for certain countries as 2. Mexico
regards receiving FDI 3. South Africa
(b) India has double taxation avoidance agreement According to UNCTAD, which of the above
with Mauritius is/are categorized as "Emerging Economics"?
(c) Most citizen of Mauritius have ethnic identity (a) 1 only (b) 1 and 3 only
with India and so the feel secure to invest in (c) 2 and 3 only (d) 1, 2 and 3
India
Ans. (d) : Emerging markets are nations with social or
(d) Impending dangers of global climatic change
business activity in the process of rapid growth and
prompt Mauritius to make huge investments
industrialisation. Currently, there 28 emerging markets in
in India
the world with the economies of China and India
Ans. (b) : India has a double tax avoidance treaty with considered to be the two largest.
Mauritius. Investors from Mauritius are exempted from
Some countries of Latin America (particularly Argentina,
tax in India and the tax rates are very low in Mauritius.
Brazil, Chile, Mexico, Colombia and Peru), some countries
So, it is favorable for investors to get registered in
in South-East Asia, most countries in Eastern Europe,
Mauritius and than invest in India as an entity from
Russia, some countries in the middle East (particularly in
Mauritius. This investment is not from Mauritius but
the Persian Gulf Arab state) and parts of Africa
through Mauritius.
(particularly South Africa) are emerging markets.
Emphasising the fluid nature of the category, political
MISCELLANEOUS Scientist Ian Bremmer defines an emerging market as "a
118. With reference to the United Nations country where politics matters atleast as much as
convention on the Rights of the Child, consider economics to the markets".
the following: 120. Which one of the following is not related to
1. The Right of Development United Nations?
2. The Right to Expression (a) Multilateral Investment Guarantee Agency
3. The Right to Recreation (b) International Finance Corporation
Which of the above is/are the Rights of the (c) International Centre for Settlement of Investment
child? Disputes
(a) 1 only (b) 1 and 3 only (d) Bank for International Settlements
(c) 2 and 3 only (d) 1, 2 and 3 Ans. (d) : The Bank for International Settlements is an
Ans. (d) : United Nations Convention on the Rights of international financial institution owned by Central Banks
the Child. which "fosters international monetary and financial
• The UNCRC is a human rights treaty that sets the cooperation and serves as a bank for Central Banks''. The
political, civil, economic, social, health, and cultural other 3 options are related to the United Nations.
rights of children. 121. Which of the following is/are treated as
• It is an international agreement that is legally binding artificial currency?
on the members. (a) ADR
• It consists of 54 articles that spell out various (b) GDR
children's rights and also the measures governments (c) SDR
should take in order to make these rights available to (d) Both ADR and SDR
children. Ans. (c) : Special Drawing Right (SDR) belongs to the
• The CRC was adopted by the United Nations in IMF. Member countries maintain their reserve tranches
1989. with it in SDR and get fiscal support from it in SDR.
• It entered into force in 1990 after receiving a 122. As regards the use of international food safety
minimum of 20 ratifications. standards as reference point for the dispute
• It has been ratified by all members of the UN except settlements which one of the following does
for the United States. WTO collaborate with?
IAS (Pre) GS 2010 Paper I 223 YCT
(a) Codex Alimentarius Commission 1. Development of infrastructure facilities.
(b) International Federation of Standards Users 2. Promotion of investment from foreign sources.
(c) International Organization for Standardization 3. Promotion of exports of services only.
(d) World Standards Cooperation Which of the above are the objectives of this
Ans. (a) : Codex alimentarius commission was created in Act?
1963 by FAO and WHO to develop food standards (a) 1 and 2 only (b) 3 only
guidelines and related texts such as codes of practice under (c) 2 and 3 only (d) 1, 2 and 3
to joint FAO/WHO food standards programme. The main Ans. (a) : The objectives of establishing SEZs (Special
purpose of this program are protecting health of the economic Zones) include making available goods and
consumer and ensuring fair trade practices in food trade. services free of taxes and duties supported by an
123. The United Nations framework Convention of integrated infrastructure for export production, and
Climate Change (UNFCCC) is an international single window approval mechanism and a package of
treaty drawn at: incentives to attract foreign and domestic investments
(a) United National Conference on the Human for promoting export-led growth.
Environment, Stockholm, 1972 127. A question paper had ten questions. Each
(b) UN Conference on Environment and question could only be answered as True (T) or
Development, Rio de Janeiro, 1992 False (F). Each candidate answered all the
(c) World Summit on Sustainable Development, questions. Yet, no two candidates wrote the
Johannesburg, 2002 answers in an identical sequence. How many
(d) UN Climate Change Conference, Copenhangen, different sequences of answer are possible?
2009 (a) 20 (b) 40
Ans. (b) : The United Nations Framework Convention (c) 512 (d) 1024
on Climate Change (UNFCCC) is an international Ans. (d) : Each question has to way to the answer.
environmental treaty negotiated at the Earth Summit in Each candidate answered the all question.
Rio de Janeiro from 3 to 14 June 1992, then entered into 2×2×2………. 10 times
force on 21 March 1994. 210 = 1024
124. Consider the following: 1024 total sequences of answers of possible.
1. Bluetooth device 128. Examine the following statements:
2. Cordless phone 1. All colours are pleasant.
3. Microwave oven 2. some colours are pleasant
4. Wi-Fi device 3. No colour is pleasant
Which of the above can operate between 2.4 4. Some colours are not pleasant
and 2.5 GHz range of radio frequency band? Given that the statement 4 is true, what can be
(a) 1 and 2 only (b) 3 and 4 only definitely concluded?
(c) 1, 2 and 4 only (d) 1, 2, 3 and 4 (a) 1 and 2 are true (b) 1 is false
Ans. (d) : Microwaves are electromagnetic waves with (c) 2 is false (d) 3 is true
wavelengths ranging from as long as one meter to as short Ans. (b) : As per statement 4: Some colours are not
as one millimeter or equivalently with frequencies between pleasant. It implies that there is atleast one colour that is
300 MHz 0.3 GHz and 300 GHz. A microwave oven not pleasant. It obviously means that statement 1 that
passes ionizing microwave radiation at a frequency near states that all colours are pleasant is definitely false.
2.45 GHz through food causing dielectric heating by 129. Three men start together to travel the same
absorption of energy in the water fats and sugar contained way around a circular track of 11 km. Their
in the food. Bluetooth and Wi-Fi have many applications: speeds are 4, 5.5 and 8 km/ph respectively.
setting up networks printing or transferring files. They also When will they meet at the starting point for
work in the given frequency range. All cordless sold in the the first time?
US use the 1.9 GHz or 2.4-GHz bands. (a) After 11 hours (b) After 21 hours
125. In the context of the affairs of which of the (c) After 22 hours (d) After 33 hours
following is the phrase "Special Safeguard Ans. (c) : Formula :-
Mechanisms" mentioned in the news frequently? Distance
(a) United Nations Environment Programme Speed =
(b) World Trade Organisation Time
(c) ASEAN-India Free Trade Agreement The time taken by three men.
(d) G-20 Summits 11 11 11
= , ,
Ans. (b) : The correct answer is World Trade Organisation. 4 5.5 8
126. The SEZ Act, 2005 which came into effect in Then LCM of numerator/HCF of denumerator.
February 2006 has certain objectives. In this 22
= = 22 hours
context, consider the following: 1
IAS (Pre) GS 2010 Paper I 224 YCT
130. P, Q, R and S are four men. P is the oldest but Ans. (a) : Formula
not the poorest. R is the richest but not the P×R ×T
oldest. Q is older than S but not than P or R. P SI =
100
is richer than Q but not than S. The four men P × (r1 − r2 ) × T
can be ordered (Descending) in respect of age SI =
100
and richness, respectively, as:
500 × (r1 − r2 ) × 2
(a) PQRS, RPSQ 2.5 =
(b) PRQS. RSPQ 100
(c) PRQS. RSQP 2.5
r1 − r2 = = 0.25%
(d) PRSQ, RSPQ 10
Ans. (b) : The four men can be ordered (Descending) in 134. When ten persons shake hands with one
respect of age and richness respectively. PRQS, RSPQ. another, in how many ways is it possible?
131. A man fills a basket with eggs in such a way that (a) 20 (b) 25
the number of eggs added on each successive day (c) 40 (d) 45
is the same as the number already present in the Ans. (d) :
basket. This way the basket gets completely filled n ( n – 1)
10 persons total way of handshake =
in 24 days. After how many days the basket was 2
1/4th full? 10 × 9
(a) 6 (b) 12 Then =
2
(c) 17 (d) 22 Hence option (d) is correct = 45
Ans. (d) :Let x be the no. of eggs already present in the 135. A candidate attempted 12 questions and
basket. secured full marks in all of them. If he obtained
Then no. of eggs on each successive days - 60% in the test and all questions carried equal
x + 2x + 4x + 8x + ………. up to 24 days. marks, then what is the number of questions in
Form here we can see that number of eggs are getting the test?
doubled on each successive days. on 24th day basket (a) 36 (b) 30
was filled completely. (c) 25 (d) 20
1 Ans. (d) : Suppose questions Number of test
∴ On 23rd day basket was filled =
2 =x
1 1 1 x×60% = 12
Hence on 22nd day basket was filled = × =
2 2 4 60
th
x× = 12
1 100
So the basket was tilled in 22 day. 12 ×100
4 x= = 20
132. The diameters of two circular coins are in the 60
ratio of 1 : 3. The smaller coin is made to roll 136. In how many ways can four children be made
around the bigger coin till it returns to the to stand in a line such that two of them, A and
position from where the process of rolling B are always together?
started. How many times coins the smaller (a) 6 (b) 12
rolled around the bigger coin? (c) 18 (d) 24
(a) 9 (b) 6 Ans. (b) : Total children = 4
(c) 3 (d) 1.5 condition = A & B always together (it means single
Ans. (c) : entity) The now children = 4 – 2 + 1 = 3
Perimeter of bigger coin : perimeter of smaller coin Then according to fraction method
2x (3) 2x (1) n!(n–1) × (n– 2) × …..
=3:1 Then ⇒ 3! × 2!
Then smaller coin will roll 3 times around the bigger = 3 ×2 × 1 × 2 × 1 = 12
coin. Then total of ways is = 12
133. The difference between the simple interest 137. In a meeting, the map of a village was placed in
received from two banks on Rs. 500 for two such a manner that south-east becomes north,
years is Rs. 2.50. What is the difference north-east becomes west and so on what will
between their rates? south become?
(a) 0.25% (b) 0.5% (a) North (b) North-east
(c) 1% (d) 2.5% (c) North-west (d) West

IAS (Pre) GS 2010 Paper I 225 YCT


Ans. (b) : According to question South becomes North - Ans. (a) : According to above instruction B is a
East because we move 135º in clockwise direction from professor.
south at will become north-East. A ⇒ unmarried lady
B ⇒ Professor
C ⇒ Lawyer/Doctor
D ⇒ Unmarried lady
E ⇒ Doctor /Lawyer
141. Half of the villagers of a certain village have
their own houses. One-fifth of the villagers
cultivate paddy. One-third of the villagers are
138. A person travelled a distance of 50 km in 8 literate. Four-fifth of the villagers are below
hours. He covered a part of the distance on foot twenty five. Then, which one of the following is
at the rate of 4 km per hour and a part on a certainly true?
bicycle at the rate of 10 km per hour. How (a) All the villagers who have their own houses
much distance did he travel on foot? are literate
(a) 10 km (b) 20 km
(b) Some villagers under twenty five are literate
(c) 30 km (d) 40 km
(c) A quarter of the villagers who have their own
Ans. (b) : According to question 50km is cover in 8 houses cultivate paddy
hours
(d) Half of the villagers who cultivate paddy are
by Foot speed = 4km/H
literate
by bicycle speed = 10 km/H
Dis tance Ans. (b) : Suppose total villagers are = 100x
Then speed = The villagers who have their own houses
Time
Supposes x km covered on foot. Then (50 – x) km 1
= × 100x = 50x
covered by bicycle. 2
x ( 50 − x ) 1
Then = + =8 The villagers who cultivated paddy = ×100x = 20x
4 10 5
( 5x + 100 − 2x ) = 8 1
⇒ Literate villagers = × 100x = 33.33x
2 3
⇒ 3x + 100 = 160 4
The villagers are below twenty five = × 100x = 80x
⇒ 3x = 60 5
x = 20 Some villagers under twenty five are literate as the
Then distance travelled on foot is 20 km. number of villagers under twenty five are 80x and
139. How many numbers from 0 to 999 are not number of literate are 33.33x Hence statement (b) is
divisible by either 5 or 7? true.
(a) 313 (b) 341 142. Each person's performance compared with all
(c) 686 (d) 786 other persons is to be done to rank them
Ans. (c) : The number from 0 to 999 are divisible by 5 subjectively. How many comparisons are
&7 needed in total, if there are 11 persons?
999 999 (a) 66 (b) 55
= 199 = 142
5 7 (c) 54 (d) 45
999 Ans. (b) : Total person = 11, Rank = 2
both 5 & 7 ⇒ 35 ⇒ = 28
35 11! 11× 10 × 9
Then 142 + 199 – 28 = 313 ∴11 C1 =
(11 – 9 )!9! 2!× 9!
Then 0 to 999 divisible by 5 or 7 is 999 – 313 = 686.
= 55
140. In a group of five persons A, B, C, D and E, there
is a professor, a doctor and lawyer. A and D are Hence Answer will be (b) correct.
unmarried ladies, and do not work. Of the 143 Six books A, B, C, D, E and F are placed side
married couple in the Group, E is the husband. B by side. B, C and E have blue cover and the
is the brother of A and is neither a doctor nor a other, books have red cover. Only D and F are
lawyer. Who is professor? new books and the rest are old. A, C, and D are
(a) B law reports and others are Gazetteers. What
(b) C book is a new law report with a red colour?
(c) A (a) A (b) B
(d) Cannot be determined with the available data (c) C (d) D
IAS (Pre) GS 2010 Paper I 226 YCT
Ans. (d) : According to question instruction Ans. (c) : Suppose Z = 100
Blue cover ⇒ B, C, F X = 80
Red cover ⇒ A, D, F Y = 72
New book ⇒ D, F difference between y & X = 80 – 72 = 8
law report ⇒ A, C, D 8
Then Y decreases % of X = × 100 = 10%
Other gazetteers ⇒ B, F, F 80
New law report with red cover is D. 148. A cuboid has size sides of different colours. The
144. Running at a speed of 60 km per hour, a train red side is opposite to black. The blue side is
passed through a 1.5 km long tunnel in two adjacent to white. The brown side is adjacent of
minutes. What is the length of the train? blue. The read side is face down. Which one of the
(a) 250 m (b) 500 m following would be the opposite to brown?
(c) 1000 m (d) 1500 m (a) Read (b) Black
Ans. (b) : Suppose train length x m (c) White (d) Blue
x + 1.5 × 1000 Ans. (c) : According to above question instruction. The
Then, = 2 × 60
5 opposite to brown is white. Hence option (c) will be
60 ×
18 correct.
2 × 60 × 60 × 5
x + 1500 =
18
x + 1500 = 2000
x = 2000 – 1500
x = 500m
145. Six persons M, N, O, P, Q and R are sitting in
two rows, three in each. Q is not at the end of
any row. P is second to the left of R. O is the
neighbour of Q and is sitting diagonally 149. In a tournament 14 teams play league matches.
opposite to P. N is the neighbour of R. One the If each team plays against every other team
basis of above information, who is facing N? once only then how many matches are played?
(a) R (b) Q (a) 105 (b) 91
(c) P (d) M (c) 85 (d) 78
Ans. (b) : According to above instruction Ans. (b) : If n teams play league matches against each
other, then total number of matches are played
n ( n – 1)
=
2
This is right sequence sitting six person. So according to So, in this tournament the total number of matches is
diagram Q. Just opposite of N. Hence option (b) is
14 (14 – 1)
correct. =
146. A person X has four notes of Rupee 1, 2, 5 and 2
10 denomination. The number of different 14 ×13
=
sums of money she can form them is: 2
(a) 16 (b) 15 = 91
(c) 12 (d) 8 150. Tow trains leave New Delhi at the same time.
Ans. (c) : The total number ways of different sums of One travels north at 60 kmph and the other
money she can from = (taken one at a time) + (taking travels south at 40 kmph. After how many
two at a time) + (taking three at a time) + (taking four at hours will the trains be 150 km apart?
a time) 3 4
= 4c1 + 4c2 + 4c3 + 4c4 (a) (b)
2 3
⇒ 4 + 6 + 4 + 1 = 15 3 15
∴ The number of different sums of money she can form (c) (d)
4 2
them is.
Ans. (a) : The first train speed = 60 km/ ph
147. Two numbers X and Y are respectively 20% second train speed = 40 km /pm
and 28% less than a third number Z. By what Both train are opposite side then total speed
percentage is the number Y less than the = 60 + 40 = 100 km/ph
number X?
150 3
(a) 8% (b) 9% time = = h
(c) 10% (d) 12% 100 2

IAS (Pre) GS 2010 Paper I 227 YCT


UNION PUBLIC SERVICE COMMISSION
Civil Services (Preliminary Exam) - 2009
GENERAL STUDIES : PAPER-I
Time: 2 hours Maximum Number: 200
Ans. (b) : Tabo Monastery is located in the Tabo
ANCIENT HISTORY village of Spiti Valley Himachal Pradesh.
1. Anekantavada is a core theory and philosophy 5. Mahamastakabhisheka, a great religious event,
of which one of the following? is associated with which of the following?
(a) Buddhism (b) Jainism (a) Bahubali (b) Buddha
(c) Sikhism (d) Vaishnavism (c) Mahavir (d) Nataraja
Ans. (b) : Anekantavada is one of the most important Ans. (a) : The Mahamastakabhisheka is a great
and fundamental doctrines of Jainism. It refers to the religious event is associated with lord Bahaubali.
notion that truth and reality are perceived differently Lard Bahubali was the son of lord Rishabha, who was
from diverse points of view, and that no single point of the first Jain Thirthankar.
view is the complete truth.
2. Match List-I with List-II and select the correct MODERN HISTORY
answer using the code given below the lists:
List-I List-II 6. With whose permission did the English set up
(Famous Temple) (State) their first factory in Surat?
A. Vidyashankara temple 1. Andhra Pradesh (a) Akbar (b) Jahangir
B. Rajarani temple 2. Karnataka (c) Shahjahan (d) Aurangzeb
C. Kandariya Mahadeo 3. Madhya Pradesh Ans. (b) : Jahangir gave permission to the East India
D. Bhimesvara temple 4. Orissa Company to set up their first factory in Surat.
Code:
7. In collaboration with David Hare and
A B C D
Alexander Duff, who of the following
(a) 2 4 3 1
established Hindu College at Calcutta?
(b) 2 3 4 1
(a) Henry Louis Vivian Derozio
(c) 1 4 3 2
(b) Ishwar Chandra Vidyasagar
(d) 1 3 4 2
Ans. (a) : None of the option is correct. (c) Keshab Chandra Sen
Vidhyashankar temple is situated in Sringeri, (d) Raja Rammohan Roy
Karnataka, Rajarani temple is situated in Bhubneswar, Ans. (d) : The foundation of Hindu College at Calcutta
Odisha, Kandariya Mahadeo temple is in Madhya was laid on January 20, 1817 by Raja Rammohan Roy.
Pradesh and Bhimesvara tempe is also in Karnataka. 8. In the context of the India freedom struggle,
3. Where is the famous Virupaksha temple 16th October, 1905 is well known for which one
located? of the following reasons?
(a) Bhadrachalam (b) Chidambaram (a) The formal proclamation of Swadeshi
(c) Hampi (d) Srikalahasti Movement was made in Calcutta town hall
Ans. (c) : Virupaksha temple is located on the south (b) Partition of Bengal Took effect
bank of the river Tungabadra in Hampi. It is dedicated
(c) Dadabhai Naoroji declared that the goal of
to Lord Shiva. It is acknowledged as one of the World
Indian National Congress was Swaraj
Heritage Sites of UNESCO. The temple was built by
Lakkan Dandesha, a chieftain under the ruler Deva (d) Lokmanya Tilak started Swadeshi Movement
Raya II of the Vijayanagara Empire. in Poona
4. In which State is the Buddhist site Tabo Ans. (b) : Partition of Bengal took place on 16th
Monastery located? October, 1905. This day was observed as a day of
(a) Arunachal Pradesh mourning throughout Bengal. People fasted, bathed in
(b) Himachal Pradesh the Ganga and walked barefoot in processions singing
(c) Sikkim Vande Mataram. People tied Rakhi as a symbol of unity
(d) Uttarakhand of the two halves of Bengal.
IAS (Pre) GS 2009 Paper I 228 YCT
9. Consider the following statements: 12. Who of the following Prime Ministers sent
1. The discussions in the Third Round Table Cripps Mission to India?
Conference eventually led to the passing of (a) James Ramsay MacDonald
the government of India Act of 1935. (b) Stanley Baldwin
2. The Government of India Act of 1935 (c) Neville Chamberlain
provided for the establishment of an a (d) Winston Churchill
Union of the provinces of British India and Ans. (d) : In March 1942, a mission headed by Stafford
the Princely States. Cripps was sent to India with constitutional proposals to
Which of the statements given above is/are seek Indian support for the war by British PM, Winston
Churchill.
correct?
(a) 1 only (b) 2 only 13. During the freedom struggle, Aruna Asaf Ali
was a major woman organizer of underground
(c) Both 1 and 2 (d) Neither 1 nor 2 activity in:
Ans. (c) : The key sources of the Government of India (a) Civil Disobedience Movement
Act of 1935 are report of Simon Commission, (b) Non-Cooperation Movement
discussion at the 2nd round table conference. The white (c) Quit India Movement
paper of 1933 and the report of the joint select (d) Swadeshi Movement
committees. Hence statement 1 is correct. Ans. (c) : Aruna Asaf Ali was one of the leading
Also this act ended the system of diarchy introduced by women organizers of underground activities during the
Government of Indian Act of 191 and provided for Quit India Movement.Her original name was Aruna
establishment of a federation of India to be made up of Ganguly. She married to noted lawyer Asaf Ali and
provinces of British India and some or all of the became active in politics. She went underground after
princely states. However, the federation never became hoisting the flag challenging the police at Gwalia Tank
the reality. Ground in Bombay.
10. During the Indian Freedom Struggle, why did 14. Consider the following statements:
Rowlett Act arouse popular indignation? The Cripps Proposals include the provision for
(a) It curtailed the freedom of religion. 1. Full independence for India
(b) It suppressed the Indian traditional education. 2. Creation of Constitution making body.
(c) It authorized the government to imprison Which of the statements given above is/are
correct?
people without trial.
(a) 1 only (b) 2 only
(d) It curbed the trade union activities.
(c) Both 1 and 2 (d) Neither 1 nor 2
Ans. (c) : Rowlett Act was passed in March 1919, even
Ans. (b) : Cripps Mission came to India in March 1942.
though every single Indian member of the Central Cripps, in consultation with Indian leaders, announced
legislative council opposed it. It authorized the his proposals -
government to imprison people without trial. It became (1) The aim of the British Government is to create a
a reason for Jallianwala Bagh tragedy of Amritsar on 13 new Indian Union, the status of which will be
April, 1919. dominion.
11. In the 'Individual Satyagraha' Vinoba Bhave (2) After the war, a Constituent Assembly would be
was chosen as the first Satyagrahhi. Who was constituted, which would consist of elected
the second? representatives of the British provinces and princely
(a) Dr. Rajendra Prasad states.
(b) Pandit Jawaharlal Nehru (3) During the war, an executive council would be
formed, which would include the leaders of the
(c) C. Rajagopalachari
main sections of the Indian people, but the defense
(d) Sardar Vallabhbhai Patel department would remain under the control of the
Ans. (b) : In September 1940, the resolution of British government. Therefore, there was no
Individual Satyagraha was passed by the All India provision for complete independence of India.
Congress Committee.Vinoba Bhave was chosen as first 15. Which one of the following began with the
‘Individual Satyagrahi’ on October 17, 1940. As soon as Dandi March?
a person or small group was arrested, his place was (a) Home Rule Movement
taken by someone else. Pandit Jawaharlal Nehru was (b) Non-Cooperation Movement
chosen as the second Satyagrahi in this movement. But (c) Civil Disobedience Movement
this movement could awaken only a very limited (d) Quit India Movement
enthusiasm, so on December 17, 1940, Gandhiji Ans. (c) : The Civil Disobedience Movement began
suspended it. with Dandi March by Mahatma Gandhi in 1930.
IAS (Pre) GS 2009 Paper I 229 YCT
16. With which one of the following movements is 21. In India, which one of the following State has
the slogan "Do or die" associated? the largest inland saline wetland?
(a) Swadesi Movement (a) Gujarat (b) Haryana
(b) Non-Cooperation Movement (c) Madhya Pradesh (d) Rajasthan
(c) Civil Disobedience Movement Ans. (d) : Rajasthan has the largest inland saline
(d) Quit India Movement wetland area of Sambhar Salt lake.
Ans. (d) : Gandhiji gave a call for ‘do or die’ in 1942 22. At which one of the following places do two
during Quit India Movement. He said we shall either important rivers of India's originate while one
free India or die in the attempt; we shall not live to see of them flows towards north and merges with
the perpetuation of our slavery.
another important river flowing towards Bay
17. Who of the following founded the Ahmedabad of Bengal, the other one flows towards Arabian
Textile Labour Association? Sea?
(a) Mahatma Gandhi
(a) Amarkantak (b) Badrinath
(b) Sardar Vallabhabhai Patel
(c) Mahabaleshwar (d) Nasik
(c) N. M. Joshi
Ans. (a) : Narmada and Son rivers originate from
(d) J. B. Kripalani
Amarkantak in Madhya Pradesh. The Son river joins the
Ans. (a) : Mahatma Gandhi founded the Ahmedabad
Ganges at Danapur (near Patna) and the Narmada river
Textile Labour Association in 1918.
opposite the normal flow direction (from east to west)
18. Who of the following is the author of a flows through rift valleys and falls to Arabian sea.
collection of poems called "Golden Threshold?
23. Which one of the following rivers does not
(a) Aruna Asaf Ali
originate in India?
(b) Annie Besant
(a) Beas (b) Chenab
(c) Sarojini Naidu
(c) Ravi (d) Sutlej
(d) Vijayalakshmi Pandit
Ans. (c) : Sarojini Naidu is the author of the poem Ans. (d) : The Sutlej River is the longest of the five
'Golden Threshold'. Smt. Naidu was an eminent poet rivers that flow through the historic crossroads region of
and nationalist leader. She also has the distinction of Punjab in northern India and Pakistan.The source of the
becoming the first Indian woman president of the 40th Sutlej is near Lake Rakshastal in Tibet.
annual session of the Indian National Congress held in 24. In India, the ports are categorized as major
Kanpur in 1925. and non-major ports. Which one of the
19. Consider the following statements: following is a non-major port?
1. The first telegraph line in India was laid (a) Kochi (Cochin)
between Kolkata (formerly Calcutta) and (b) Dahej
Diamond Harbour. (c) Paradip
2. The first Export Processing Zone in India was (d) New Mangalore
set up in Kandla.
Ans. (b) :India has about 7516.6 km long coastline.
Which of the statements given above is/are There are 12 major ports and 187 other ports. Major
correct?
ports come under the central government, while other
(a) 1 only (b) 2 only
ports (small/intermediate ports) come under the
(c) Both 1 and 2 (d) Neither 1 nor 2 jurisdiction of the respective state governments.
Ans. (c) : The first electric telegraph line in India was Mumbai, Nhavasheva (Jawahar Lal Nehru Port),
started between Kolkata and Diamond Harbour in 1850 Kandla, Marmagoa, New Mangalore and Kochi are the
and first export zone of India was set up in Kandla in major ports on the west coast, while the major ports on
1965. Hence both the statements are correct.
the east coast are Kolkata/Haldia, Paradip,
Visakhapatnam, Chennai, Ennorp and Tuticorin.
INDIAN GEOGRAPHY 25. Consider the following statements:
20. Which one of the following planets has largest 1. India does not have any deposits of Thorium.
number of natural satellites or moons? 2. Kerala's monazite sands contain Uranium
(a) Jupiter (b) Mars Which of the statements given above is/are
(c) Saturn (d) Venus correct?
Ans. (a) : Jupiter has maximum natural satellites (a) 1 only (b) 2 only
accounting to 79 hence option a will be correct but (c) Both 1 and 2 (d) Neither 1 nor 2
according to present data Saturn has 82, Mars has 2 and Ans. (b) : Monazite is a mixture of Thorium, Uranium,
Venus has none. Cerium and Lanthanum etc. It is found in abundance in
Note : Now, Saturn has the largest number of natural the coastal areas of Kerala. India has vast reserves of
satellites i.e. 82. Thorium deposits.
IAS (Pre) GS 2009 Paper I 230 YCT
26. Consider the following statements: 30. In which one of the following places is the
1. There are no east flowing rivers in Kerala. Shompen tribe found?
2. There are no west flowing rivers in Madhya (a) Nilgiri Hills (b) Nicobar Islands
Pradesh. (c) Spiti Valley (d) Lakshadweep
Which of the statements given above is/are Island
correct? Ans. (b) : The Shompen tribe is one of two Mongloid
(a) 1 only (b) 2 only tribes found in Nicobar Island.
(c) Both 1 and 2 (d) Neither 1 nor 2
Ans. (d) : Three east flowing rivers are found in Kerala WORLD GEOGRAPHY
which falls into Kavari, they are Kabani, Bhavani and
Pambar. Whereas, in Madhya Pradesh; Tapti, Narmada 31. Consider the following statements:
and Mahi rivers flows westward. Hence both the
1. In the world, the tropical deserts occur
statements are not correct.
along the western margins of continents
27. Consider the following regions:
within the trade wind belt.
1. Eastern Himalayas
2. In India, the East Himalayan region gets
2. Eastern Mediterranean region
high rainfall from North-East winds.
3. North-western Australia
Which of the statements given above is/are
Which of the above is/are Bio-diversity
correct?
Hotspot(s)?
(a) 1 only (b) 2 only
(a) 1 only (b) 1 and 2 only
(c) Both 1 and 2 (d) Neither 1 and 2
(c) 2 and 3 only (d) 1, 2 and 3
Ans. (a) : A biodiversity hotspot is a biogeographic Ans. (a) : In the world, tropical deserts are found in the
region that is both a significant reservoir of western frontiers of the continents in the Trade wind
Biodiversity and threatend with destruction. Eastern belt. The eastern Himalayan region in India receives
Himalayas is a biodiversity not spot. Mediterranean more rainfall from the south-west monsoon.
Basin is a hotspot, not eastern Mediterranean region and 32. Consider the following countries:
also south west part of Australia has not spot, not North 1. Australia 2. Namibia
west part of Australia 3. Brazil 4. Chile
28. Consider the following statements: Through which of the above does the Tropic of
1. The Baglihar Power Project had been Capricorn pass?
constructed within the parameters of the Indus (a) 1 only (b) 2, 3 and 4
water Treaty. (c) 1, 2 and 3 (d) 1, 2, 3 and 4
2. The project was completely built by the
Ans. (d) : The countries through which the Tropic of
Union Government with loans from Japan and
Capricorn passes are - Australia, Madagascar,
the World Bank.
Mozambique, South Africa, Botswana, Namibia, Brazil,
Which of the statements given above is/are
Argentina, Paraguay and Chile.
correct?
(a) 1 only (b) 2 only 33. Consider the following pairs:
(c) Both 1 and 2 (d) Neither 1 nor 2 Famous Place Country
Ans. (a) : Baglihar Dam also known as Baglihar 1. Cannes : France
Hydroelectric Power Project, is a run-of-the-river power 2. Davos : Denmark
project on the Chenab River in the southern Doda 3. Roland Garros : The Netherlands
district of the Indian controlled part of Jammu and Which of the pairs given above is/are correctly
Kashmir. This project was conceived in 1992, approved matched?
in 1996 and construction began in 1999. The project is (a) 1 only (b) 1 and 2 only
estimated to cost USD $1 billion. The first phase of the
(c) 2 and 3 only (d) 1, 2, and 3
Baglihar Dam was completed in 2004. Baglihar dam
project has been financed by Power Finance Ans. (a) : Cannes located in France which famed for
Corporation and J& K bank. International film festival. Davos is a town in
29. The Dul Hasti Power Station is based on which Switzerland and Roland Garros is in Paris (France).
one of the following rivers? 34. In the structure of planet Earth, below the
(a) Beas (b) Chenab mantle the core is mainly made up of which one
(c) Ravi (d) Sutlej of the following?
Ans. (b) : The Dul Hasti Power Station is on the (a) Aluminium (b) Chromium
Chenab river in Jammu and Kashmir. (c) Iron (d) Silicon
IAS (Pre) GS 2009 Paper I 231 YCT
Ans. (c) : The interior of the Earth is divided into three 40. Match List-I with List-II and select the correct
major zones - Crust, Mantle and Core. The volume of answer using the code given below the lists:
the core is only 16% of the whole earth, but its mass is List-I List-II
about 32% of the total mass of the earth. Although a (Geographic feature) (Country)
small amount of silicon is also present in the inner parts A. Great Victoria Desert 1. Australia
of the core, it is mainly composed of nickel and iron.
B. Grand Canyon 2. Canada
Hence it was also called Nife layer.
C. Lake Winnipeg 3. New Zealand
35. Which among the following has the world’s
largest reserves of Uranium? D. Southern Alps 4. USA
(a) Australia (b) Canada Code:
(c) Russian Federation (d) USA A B C D
Ans. (a) : Australia has the world’s largest uranium (a) 1 2 4 3
reserves. Approximately 24% of the planet’s uranium is (b) 1 4 2 3
present in Australia. (c) 3 2 4 1
36. Who of the following scientists proved that the (d) 3 4 2 1
stars with mass less than 1.44 times the mass of Ans. (b) : The Great Victoria Desert is located in
the Sun end up as White Dwarfs when they Australia. The Barberton, Nimpson, Gibson and Sourt
die? deserts are also located in Australia. Grand Canyon is
(a) Edwin Hubble (b) S. Chandrashekhar located in Lake U.S.A. Winnipeg, Great Slav,
(c) Stephen Hawking (d) Steven Weinberg Greatbear, Radier and Lake Athwaska are in Canada.
Ans. (b) : If the star's mass is less than 1.4 Ms (where Near Athavaska Lake is Uranium City. The Southern
Ms is the mass of the Sun), it loses its nuclear energy Alps Mountains are located in New Zealand.
and turns into a white dwarf. It is also called fossil star.
White dwarf cools down and turns into Black Dwarf. INDIAN CONSTITUTION AND POLITY
1.4 Ms is called Chandrasekhar limit. S. Chandrasekhar
was awarded with Nobel Prize in Physics for the study 41. With reference to Union Government consider
of the structure and evolution of stars. the following statements:
37. Which one of the following pairs is not 1. The number of Ministries at the Centre on
correctly matched? 15Th August 1947 was 18.
City River 2. Number of Ministries at the Centre at
(a) Berlin : Rhine present is 36.
(b) London : Thames Which of the statements give above is/are
(c) New York : Hudson correct.
(d) Vienna : Danube (a) 1 only (b) 2 only
Ans. (a) : Berlin (Germany) is situated on the banks of (c) Both 1 and 2 (d) Neither 1 nor 2
the river Spree while London (UK) is situated on the Ans. (a) : The number of ministries at the Centre can
banks of Thames, New York (USA) is situated on the vary based on factors such as volume of work,
banks of Hudson river and Vienna (Austria) is situated importance attached to different sectors, changes of
on the banks of Danube. orientation of policy etc. On 15 Aug, 1947, the number
38. Consider the following pairs: of ministries at the centre was 18.
Automobile Manufacturer Headquarters 42. With reference to Union Government,
1. BMW AG : USA Consider the following statements:
2. Daimler AG : Sweden 1. The Constitution of India provides that all
3. Renault S.A. : France Cabinet Ministers shall be compulsorily the
4. Volkswagen AG : Germany sitting members of Lok Sabha only.
Which of the pairs given above is/are correctly 2. The Union Cabinet Secretariat operates
matched? under the direction of the Ministry of
(a) 1, 2 and 3 (b) 3 and 4 Parliamentary affairs.
(c) 4 only (d) 1, 2 and 4 Which of the statements given above is/are
Ans. (b) : BMW AG and Daimler AG are Head correct?
quartered in Germany. (a) 1 only (b) 2 only
39. Cape Canaveral, the site from which space (c) Both 1 and 2 (d) Neither 1 nor 2
shuttles are launched is located on the coast of Ans. (d) : Statement 1 is incorrect as members of Rajya
(a) Florida (b) Virginia Sabha can also become cabinet ministers. Statement 2 is
(c) North Carolina (d) South Carolina incorrect as Cabinet secretariat is under the direct
Ans. (a) : Cape Canaveral is the part of Florida’s coast. charge of the Prime Minister.
IAS (Pre) GS 2009 Paper I 232 YCT
43. With reference to Union Government consider 47. Consider the following statements:
the following statements: 1. The Advocate General of a State in India is
1. The Ministries/Departments of the government appointed by the President of India upon
of India are created by the Prime Minister on the recommendation of the governor of the
the advice of the Cabinet Secretary. concerned State.
2. Each of the Ministries is assigned to a Minister 2. As provided in Civil Procedure Code. High
by the President of India on the advice of the Courts have original, appellate and advisory
Prime Minister. jurisdiction at the State level.
Which of the statements given above is/are Which of the statements given above is/are
correct? correct?
(a) 1 only (b) 2 only (a) 1 only (b) 2 only
(c) Both 1 and 2 (d) Neither 1 nor 2 (c) Both 1 and 2 (d) Neither 1 nor 2
Ans. (b) : According to Article 70 of the Constitution, Ans. (d) : Statement 1 is incorrect as Advocate General
the Prime Minister shall be appointed by the President of the state is appointed by the governor of the State.
and the other Ministers shall be appointed by the Statement 2 is also incorrect as High Courts have
President on the advice of the Prime Minister. The Original, Appellate and Writ jurisdiction (not advisory
government of India (Allocation of Business) Rules, jurisdiction) at the state level.
1961 is made by the President of India under Article 77 48. If a Panchayat is dissolved, elections are to be
of the constitution for the allocation of business of the held within
government of India. The Ministries/ Departments of (a) 1 month (b) 3 month
the government of India are created by the president on (c) 6 month (d) 1 year
the advice of the prime Minister under these Rules. Ans. (c) : Duration of Panchayats is five year. Fresh
44. Which one of the following Constitutional election to constitute a Panchayat shall be completed
Amendments states that the total number of before the expiry of its term; or in case of dissolution
Ministers, including the Prime Minister, in the fresh election is to be conducted before the expiry of a
Council of Minister shall not exceed fifteen period of 6 months from the date of its dissolution.
percent of the total number of members of the 49. In India, the first Municipal Corporation was
House of the people? set up in which one among the following?
(a) 90th (b) 91st (a) Calcutta (b) Madras
nd
(c) 92 (d) 93rd (c) Bombay (d) Delhi
Ans. (b) : The above provision has been added by 91st Ans. (b) : In 1688, the first Municipal Corporation of
Constitutional Amendment Act, 2003. India was set up in Madras.
45. During which Five Year Plan was the 50. Among the following Presidents of India, Who
Emergency clamped, new elections took place was also the Secretary General of Non-Aligned
and the Janata Party was elected? Movement for some Period?
(a) Third (b) Fourth (a) Dr. Sarvepalli Radhakrishnan
(c) Fifth (d) Sixth (b) Varahagiri Venkatagiri
Ans. (c) : This all happened from 1975-78 during fifth (c) Gianin Zail Singh
five year plan, the period of which is (1974-78) (d) Dr. Shanker Dayal Sharma
46. Consider the following statements: Ans. (c) : Giani Zail Singh was the 7th president of
1. The Governor of Punjab is concurrently the India. He also became 9th Secretary General of Non
Administrator of Chandigarh. Aligned Movement.
2. The Governor of Kerala is concurrently the 51. With reference to Lok Adalats, consider the
Administrator of Lakshadweep. following statements:
Which of the above statements is/are correct? 1. An award made by a Lok Adalat is Deemed
(a) 1 only (b) 2 only to be decree of a civil court and no appeal lies
(c) Both 1 and 2 (d) Neither 1 nor 2 against thereto before any court.
Ans. (a) : The Governor of Punjab is also the 2. matrimonial/Family disputes are not covered
administrator of Chandigarh along with his under Lok Adalat.
responsibility, but the Governor of Kerala is not the Which of the statements given above is/are
administrator of Lakshadweep along with his correct?
responsibility. Administration of all Union Territories (a) 1 only (b) 2 only
under Article 239 is governed by the Centre. (c) Both 1 and 2 (d) Neither 1 nor 2
IAS (Pre) GS 2009 Paper I 233 YCT
Ans. (a) : Lok Adalat is a forum where disputes / cases 56. Consider the following statements:
pending in court are settled in an amicable manner even 1. MMTC Limited is India's largest
before they are filed. Lok Adalats have been given international trading organisation.
legal status under the Legal Services Authority Act 2. Neelachal Ispact Nigam Limited has been
1987. Under this, the decision made by the Lok Adalat set up by MMTC jointly with the
has the same recognition as the decision of a civil court, government of Orissa.
it is final and binding on all the parties and no appeal Which of the statements given above is/are
can be made against it. Matrimonial/family matters are correct?
not covered under Lok Adalats. (a) 1 only (b) 2 only
52. Consider the following statements: (c) Both 1 and 2 (d) Neither 1 nor 2
1. Central Administrative Tribunal (CAT) was Ans. (c) : MMTC Limited is the largest international
set up during the Prime Ministership of Lal trading company in India and has been in existence for
Bahadur Shastri. almost five decades. It handles the export of primary
2. The Members for CAT are drawn from products such as coal, iron ore, and manufactured agro
both judicial and administrative streams. and industrial products. MMTC also imports important
Which of the statements given above is/are commodities such as ferrous and nonferrous metals for
correct? industry and agricultural fertilizers. Neelachal Ispat
(a) 1 only (b) 2 only Nigam Ltd, located at Kalinga Nagar Industrial
Complex, Duburi is a joint venture of MMTC and
(c) Both 1 and 2 (d) Neither 1 nor 2
Orissa Government.
Ans. (b) : Statement 1 is incorrect as CAT was set up
57. In which one of the following States has India's
in1985 during the prime ministership of Rajiv Gandhi.
largest Private sector sea port been
The Members for CAT are drawn from both judicial commissioned recently?
and administrative streams.
(a) Andhra Pradesh (b) Karnataka
53. In India, who is the Chairman of the National (c) Kerala (d) Tamil Nadu
Water Resources Council?
Ans. (a) : At the time when question was asked, India's
(a) Prime Minister largest private sector sea port was inaugurated at
(b) Minister of Water Resources Krishna patnam in Nellore district of Andhra Pradesh.
(c) Minister of Environment and Forests At present, port of Mundra in Gujrat in the largest
(d) Minister of Science and Technology private container port in India, Mundra in a major hub
Ans. (a) :National Water Resources Council was set up for containers and bulk cargo. It is run by the Adani
by the Government of India in March 1983. The Prime Ports and SEZ Limited (APSEZ) and began operations
Minister is the Chairman. in 2001..
54. Under the administration of which one of the 58. Consider the following statements:
following is the Department of Atomic Energy? 1. The commission for Agricultural Costs and
(a) Prime Minister’s Office Prices recommends the Minimum Support
(b) Cabinet Secretariat Prices for 32 crops.
(c) Ministry of Power 2. The Union Ministry of Consumer Affairs,
Food and Public Distribution has launched
(d) Ministry of Science and Technology
the National Food Security Mission.
Ans. (a) : The Department of Atomic Energy is under
Which of the statements given above is/are
the direct control of Prime Minister’s Office with its
correct?
Headquarter in Mumbai.
(a) 1 only (b) 2 only
(c) Both 1 and 2 (d) Neither 1 nor 2
ECONOMY Ans. (d) : The commission for Agricultural Costs and
55. In the context of independent India's economy. Prices recommends the Minimum Support Prices for 26
Which one of the following was the earliest crops. Therefore statement 1 is not correct. The
event to take place? National Development Council (NDC) in its 53rd
meeting held on 29th May, 2007 adopted a resolution to
(a) Nationalisation of Insurance companies
launch a Food Security Mission consisting of rice,
(b) Nationalisation of State Bank of India wheat and pulses and to increase the production of rice
(c) Enactment of Banking Regulation Act by 10 million tons, wheat by 8 million tons and pulses
(d) Introduction of First Five-Year Plan by 2 million tons by the end of the Eleventh Plan
Ans. (c) : Nationalization of State Bank of India – (2011–12). The project is under Ministry of Agriculture.
1955; Introduction of First Five-Year Plan – 1951; The project identifies districts and varieties which
Enactment of Banking Regulation Act – 1949; would be concentrated on. Hence 2nd statement is also
Nationalization of Insurance Companies – 1955-56. incorrect.
IAS (Pre) GS 2009 Paper I 234 YCT
59. Among other things, which one of the following 62. In the context of global economy, which one of
was the purpose for which the Deepak Parekh the following part is not correctly matched?
Committee was constituted? (a) JP Morgan Chase : Financial Services
(a) To study the current socio-economic (b) Roche Holding AG : Financial Services
conditions of certain minority. (c) WL ROSS & Co. : Private Equity Firm
(b) To suggest measures for financing the (d) Warburg Pincus : Private Equity Firm
development of infrastructure.
Ans. (b) : Roche Holding AG is a Switzerland based
(c) To frame a policy on the production of
pharmaceuticals and diagnostics company. Rest of the
genetically modified organisms
pairs are correctly matched.
(d) To suggest measures to reduce the fiscal
63. Consider the following Institutions:
deficit in the Union Budget.
Ans. (b) : The main objective to constitute Deepak 1. Christie's
Parekh Committee was to make suggestions in the 2. Osian's
matter of infrastructure financing. The committee has 3. Soptheby’s
said in the recommendation that the holding companies Which of the above is/are auctioneer
should be allowed to raise FDI under self-approved /auctioneers? Consider the following:
route for financing the infrastructure sector. Along with (a) 1 only (b) 1 and 3 only
this, the committee has also recommended tax (c) 2 and 3 only (d) 1, 2 and 3
exemption to the investors in ultra mega power projects. Ans. (d) : All the given institutions are auctioneers.
60. Consider the following statements regarding 64. In the context of Indian news in recent items,
Indian Planning:
what is MCS-SX?
1. The Second Five-Year Plan emphasized on
(a) A kind of supercomputer
the establishment of heavy industries.
(b) Title of Moon Impact probe
2. The Third Five-Year Plan introduced the
concept of import substitution as a strategy (c) Stock exchange
for industralisation. (d) Nuclear-powered submarine
Which of the statements given above is/are Ans. (c) : Metropolitan Stock Exchange of India
correct? Limited (MSEI), formerly known as MCX Stock
(a) 1 only (b) 2 only commodity Exchange Limited (MCX-SX), is India's
(c) Both 1 and 2 (d) Neither 1 nor 2 youngest and one of the three stock exchanges
Ans. (c) : India's Second Five-Year Plan (1956-61) was recognised by country's securities market regulator -
to set India on the path of industrialisation. P.C. Securities and Exchange Board of India (SEBI). It
Mahalanobis was the moving spirit behind the second offers an electronic, transparent and hi-tech platform for
five year plan. He gave the highest priority to trading in Capital Market, Futures & Options, Currency
strengthening the industrial base of the economy. India's Derivatives, Interest Rate Futures (IRF) and Debt
Third Five-Year Plan (1961-66) emphasised on long- Market segments.
term development. 65. Who among the following is the founder of
61. Consider the following: World Economic Forum?
1. Fringe Benefit Tax (a) Klaus Schwab
2. Interest Tax (b) John Kenneth Galbraith
3. Securities Transaction Tax (c) Robert Zoellick
Which of the above is/are Direct Tax/Taxes? (d) Paul Krugman
(a) 1 only (b) 1 and 3 only Ans. (a) : The World Economic Forum was founded in
(c) 2 and 3 only (d) 1, 2 and 3 1971 by Klaus Schwab, a German-born business
Ans. (d) : Fringe Benefits Tax (FBT) was imposed on professor at the University of Geneva. Originally named
employers by India’s Finance Act 2005 and was the European Management Forum, it changed its name
introduced for the financial year commencing April 1, to the World Economic Forum in 1987.
2005. The Fringe Benefit Tax was abolished in the 66. Which one of the following pair is not correctly
Finance Bill of 2009. Securities Transaction Tax (STT)
matched?
is the tax payable on the value of taxable securities
transaction. STT was introduced in India by the 2004 (a) Japan : Nikkei
budget and is applicable with effect from 1st October (b) Singapore : Shcomp
2004. Interest tax is also a direct tax. Hence all the (c) UK : FTSE
given taxes are direct tax. (d) USA : Nasdaq
IAS (Pre) GS 2009 Paper I 235 YCT
Ans. (b) : The major stock exchanges of the world are : (c) Magnesium chloride and Zinc chloride
Nikkei Tokyo (Japan) (d) Ammonium chloride and Calcium chloride
Shcomp or S.S.E. Composite Index Shanghai (China) Ans. (a) : A dry cell has the electrolyte immobilized as
FTSE UK a paste, with only enough moisture in the paste to allow
Nasdaq USA current to flow. The electrolyte is ammonium chloride
MDAX Germany in the form of a paste next to the zinc anode. In some
SEMEX,Straits Times Singapore more modern types of so called ‘high power’ batteries,
the ammonium chloride has been replaced by zinc
KOSPI Index Korea
chloride.
SET Thailand
TAIEXIndex Taiwan BIOLOGY
NASDAQ USA
71. In the context of genetic disorders, consider the
following:
PHYSICS A woman suffers from colour blindness while
67. Which one of the following types of waves are her husband does not suffer from it. They have
used in a night vision apparatus? a son and a daughter. In this context, which
(a) Radio waves one of the following statements is most
probably correct?
(b) Microwaves
(a) Both Children suffer from colour blindness.
(c) Infra-red waves (b) Daughter suffers from colour blindness while
(d) None of the above son does not suffer from it.
Ans. (c) : Night Vision as referenced here is an opto (c) Both child do not suffer from colour
electronic device that provides us with the miracle of blindness.
vision in total darkness and the improvement of vision (d) Son suffers from colour blindness while
in low light environments. Infra-red waves are used in daughter does not suffer from it.
night vision apparatus. Ans. (d) : Son suffers from colour blindness while
68. Which one among the following has the highest daughter does not suffer from it many of the genes
energy? involves in color vision are on X chromosomes, making
(a) Blue light (b) Green light colour blindness much more common in males because
(c) Red light (d) Yellow light males only have 1 X chromosome while females have
Ans. (a) : Energy of light E=hν two.
Since, blue light has the highest frequency, so blue light 72. The marine animal called dugong which is
has the highest energy. vulnerable to extinction is a/an
(a) Amphibian (b) Bony fish
(c) Shark (d) Mammal
CHEMISTRY Ans. (d) : Dugong is a large marine mammal. Dugong
69. Which one of the following is used as an has a fusiform body with no dorsal fin or hind limbs,
explosive? instead of possessing paddle. Dugong is heavily
dependent on seagrass for subsistence. It is also known
(a) Phosphorus trichloride
as sea Cow.
(b) Mercuric oxide
73. In making the saffron spice , which one of the
(c) Graphite following parts of the plant is used?
(d) Nitroglycerin (a) Leaf (b) Petal
Ans. (d) : Nitroglycerine (NG) also known as (c) Sepal (d) Stigma
nitroglycerine, trinitroglycerin (TNG), nitro, glyceryl Ans. (d) : Stigma of the plant is used in making the
trinitrate (GTN) or 1,2,3- trinitroxypropane and glyceryl saffron spices. Stigma is the top of the central part of a
trinitrate is a heavy, colorless, oily, explosive liquid flower. It receives the pollen that allows it to form new
obtained by nitrating glycerol. Alfred Nobel discovered seeds.
that while mixing nitroglycerin with diatomaceous earth 74. Consider the following statements:
would turn the liquid into a paste, called dynamite. An 1. Sweet orange plant is propagated by grafting
advantage of dynamite was that it could be cylinder- technique.
shaped for insertion into the drilling holes used for 2. Jasmine plant is propagated by layering
mining. technique.
70. In a dry cell (Battery), which of the following Which of the statements given above is/are
are used as electrolytes? correct?
(a) Ammonium chloride and Zinc chloride (a) 1 only (b) 2 only
(b) Sodium chloride and Calcium chloride (c) Both 1 and 2 (d) Neither 1 nor 2
IAS (Pre) GS 2009 Paper I 236 YCT
Ans. (c) : Grafting is a most common method of Ans. (a) : Bats of the genus Pteropus are commonly
vegetative propagation. In this method, parts of two known as the fruit bat or flying foxes among other
plants are joined in such a way that they grow as a one colloquial names
plant. Sweet orange plant is propagated by grafting 80. Where is the headquarters of Animal welfare
technique. Layering is another type of vegetative Board of India located?
propagation. In this method, roots are artificially (a) Ahmedabad (b) Chennai
induced on the stem branches before they are detached (c) Hyderabad (d) Kolkata
from the plant for propagation. Jasmine plant is Ans. (b) : The Animal Welfare Board of India was set
propagated by this technique.
up in 1962 with its headquarter at Chennai, under the
75. With reference to the evolution of living provisions of the "Prevention of Cruelty to Animals
organisms, which one of the following
Act. 1960" (PCA Act. 1960).
sequences is correct?
(a) Octopus-Dolphin-Shark
(b) Pangolin-tortoise-Hawk CURRENT AFFAIRS
(c) Salamander-Python-Kangaroo
81. Which one of the following Union Ministries
(d) Frog-Crab-Prawn
Ans. (c) : In the context of evolution of organisms, the implements the Cartagena Protocol on
correct sequence is Salamander (amphibian) - Python - Biosafety?
(reptile) Kangaroo (mammals). (a) Ministry of Science and Technology
76. Consider the following: (b) Ministry of Health and Family Welfare
1. Camphor 2. Chicory (c) Ministry of Environment and Forests
3. Vanilla (d) Ministry of Chemicals and Fertilizers
Which of the above is/are plant product (s)? Ans. (c) : The Cartagena Protocol on Biosafety to the
(a) 1 and 2 only (b) 3 only Convention on Biological Diversity is an international
(c) 1 and 2 only (d) 1, 2 and 3 agreement which aims to ensure the safe handling,
Ans. (d) : Camphor is a waxy, flammable, white or transport and use of living modified organisms (LMOs)
transparent solid with a strong aromatic odour. resulting from modern biotechnology that may have
Cinnamomum camphora is the source of camphor.
adverse effects on biological diversity, taking also into
Vanilla is a flavour obtained from the orchid of the
account risks to human health. It was adopted on 29
genus Vanilla, generally from Mexican Vanilla
January 2000 and entered into force on 11 September
planifolia. Chicory is a root extractor of Cichorium
2003. Ministry of Environment and Forest implements
intybusis used commercially as a substitute for coffee.
it in India.
77. In the context of alternative sources of energy,
ethanol as a viable bio-fuel can be obtained 82. In the context of CO2 emission and Global
from: Warming, what is the name of a market driven
(a) Potato (b) Rice device under the UNFCC that allows
(c) Sugarcane (d) Wheat developing countries to get funds/incentives
Ans. (c) : Sugarcane ethanol is an alcohol-based fuel from the developed countries to adopt better
produced by the fermentation of sugarcane juice and technologies that reduce greenhouse gas
molasses. Because it is a clean, affordable and emissions?
lowcarbon biofuel, sugarcane ethanol has emerged as a (a) Carbon Footprint
leading renewable fuel for the transportation sector. (b) Carbon Credit Rating
(c) Clean Development Mechanism
ENVIRONMENT AND ECOLOGY
(d) Emission Reduction Norm
78. The Panda belongs to the same family as that Ans. (c) : The Clean Development Mechanism (CDM),
of: defined in Article 12 of the Protocol, allows a country with
(a) Bear (b) Cat an emission-reduction or emission-limitation commitment
(c) Dog (d) Rabbit under the Kyoto Protocol (Annex B Party) to implement
Ans. (a) : Panda and bear belongs to family Ursidae. an emission-reduction project in developing countries.
Pandas are medium sized bears. Of all the endangered
bear species they have the most distinguished colour 83. The concept of carbon credit originated from
combination. which one of the following?
79. In the context of Indian wild life, the flying fox (a) Earth Summit, Rio de Janeiro
is a (b) Kyoto Protocol
(a) Bat (b) Kite (c) Montreal Protocol
(c) Stork (d) Vulture (d) G-8 Summit, Heiligendamm
IAS (Pre) GS 2009 Paper I 237 YCT
Ans. (b) : The Kyoto Protocol invented the concept of 87. In the latter half of the year 2008, which one of
carbon emissions trading, whereby carbon credits were the following countries pulled out of
a "flexibility mechanism". Under this flexibility Commonwealth of Independent States?
mechanism Annex 1 (developed countries) could use (a) Azerbaijan (b) Belarus
the carbon credits to meet their emission reduction (c) Georgia (d) Kazakhstan
commitments. A carbon credit is a generic term for any Ans. (c) : On August 18, 2008, Georgia's foreign
tradable certificate or permit representing the right to minister announced the termination of its membership
emit one tonne of carbon dioxide or the mass of another from the Commonwealth of Independent States.
greenhouse gas with a carbon dioxide equivalent (CO2)
88. With reference to the United Nations, consider
equivalent to one tonne of carbon dioxide.
the following statements:
84. With reference to the schemes launched by the
1. The Economic and Social Council (ECOSOC)
Union Government, consider the following
statements? of UN consists of 24 member States.
1. Ministry of Health and Family Welfare 2. It is elected by a 2/3rd majority of the General
launched the Rashtriya Swasthya Bima Assembly for a 3-year term.
Yojana. Which of the statements given above is/are
2. Ministry of Textiles launched the Rajiv correct?
Gandhi Shilpi Swasthya Bima Yojana. (a) 1 only (b) 2 only
Which of the statements given above is/are (c) Both 1 and 2 (d) Neither 1 nor 2
correct? Ans. (b) : The Economic and Social Council of the
(a) 1 only (b) 2 only United Nations is a permanent body of 54 members, in
(c) Both 1 and 2 (d) Neither 1 nor 2 which 1/3 of the members are released every year.
Ans. (b) : The National Health Insurance Scheme has Thus the term of each member is 3 years. But a retiring
been started by the Central Government from 1st April, member can be re-elected immediately. Each member
2008 for the families living below the poverty line. The nation has only one representative in the council.
central government will pay 75% of its premium, while 89. In the year 2008, which one of the following
the rest will be paid by the state governments. This conducted a complex scientific experiment in
scheme was started by the Ministry of Labor and which sub-atomic particles were accelerate to
Employment of the Central Government. Ministry of nearly the speed of light?
Textiles launched the Rajiv Gandhi Shilpi Swasthya (a) European Space Agency
Bima Yojana. Hence only 2nd statement is correct. (b) European Organization for Nuclear Research
85. NAMA-11 group of countries frequently (c) International Atomic Energy Agency
appears in the news in the context of the affairs (d) National Aeronautics and Space Administration
of which one of the following? Ans. (b) : The European Organization for Nuclear
(a) Nuclear Suppliers Group Research conducted a complex scientific experiment in
(b) World Bank 2008, in which non-atomic particles can be accelerated
(c) World Economic Forum to almost the speed of light
(d) World Trade Organization 90. India recently landed its Moon Impact probe
Ans. (d) : NAMA-11 (Non Agricultural Market Access) on the Moon. Among the following countries,
is an organization of developing countries. In the Hong which one landed such probe on the Moon
Kong Ministerial Conference of the World Trade earlier?
Organization, 2005 presented by the European Union (a) Australia (b) Canada
regarding tariff reduction(Non Agricultural Market (c) China (d) Japan
Access - NAMA) on industrial imports of developing
countries. The formula was accepted with Ans. (d) : On November 14, 2008, on the birth
anniversary of India's first Prime Minister Pandit
modifications brought by Argentina, Brazil and India.
Jawaharlal Nehru, the country's first fully India-built
86. Recently, which one of the following was unmanned space craft 'Moon Impact Probe' (MIP) was
included in the UNESCO's World Heritage landed on the surface of the Moon. M.I.P Separating
list? from Chandrayaan-1, landed in a deep crater located on
(a) Dilwara Temple the South Pole of the Moon and made a presence of
(b) Kalka-Shimla Railway India's national flag tricolor. India became one of the
(c) Bhiterkanika Mangrove Area countries after Soviet Russia, America and Japan who
(d) Visakhapatnam to Araku Valley railway line had brought their national flag to the moon with their
Ans. (b) : On July 8, 2008, the Kalka-Shimla Railway own effort. On 1st March, 2009 China’s change 1
became part of the mountain railways of India World executed a controlled crash onto the surface of the
Heritage site. Moon. Hence correct answer should be and (d).
IAS (Pre) GS 2009 Paper I 238 YCT
91. Consider the following countries: Ans. (d) : B.K. Chaturvedi committee was setup for
1. Brunei Darussalam 2. East Timor permanent and transparent method of pricing of
3. Laos petroleum products in India.
Which of the above is/are member/members of 97. A very big refugee camp called Dadaab,
ASEAN? recently in the news, is located in
(a) 1 only (b) 2 and 3 only (a) Ethiopia (b) Kenya
(c) 1 and 3 only (d) 1, 2 and 3 (c) Somalia (d) Sudan
Ans. (c) : The Association of Southeast Asian Nations, Ans. (b) : A refugee camp named 'Dadaab' is located in
is a geo-political and economic organization of 10 the country of Kenya.
countries located in Southeast Asia, which was formed 98. From which one of the following did Kosovo
on 8 August 1967 by Indonesia, Malaysia, the declare its independence its independence?
Philippines, Singapore and Thailand. Since then,
(a) Bulgaria
membership has expanded to include Brunei, Burma
(Myanmar), Cambodia, Laos, and Vietnam. (b) Croatia
92. Which one of the following pairs of countries (c) Macoedonia
fought wars over a region called Ogaden? (d) Serbia
(a) Eritrea and Sudan Ans. (d) : Kosovo recently declared its independence
(b) Ethiopia and Somalia from Serbia. Serbia broke away from the former
(c) Kenya and Somalia Yugoslavia.
(d) Ethiopia and Sudan 99. Which one of the following is not a
Ans. (b) : Ogaden is a Sudanese and Muslim majority drug/pharma company?
region. There was a fight between Ethiopia and (a) Chevron
Somalia for this area. (b) Nicholas Piramal
93. Elephant Pass, which is frequently in the news, (c) Pfizer
is mentioned the context of the affairs of which (d) Zydus Cadila
one of the following? Ans. (a) : Chevron Corporation is an American
(a) Bangladesh (b) India multinational energy corporation.
(c) Nepal (d) Sri Lanka 100. A present group of nations known as G-8
Ans. (d) :‘Elephant pass’ is located in the northern started first as G-7. Which one among the
province, gateway to Jaffna Pennisula Sri Lanka. following was not one of them?
94. Other than Venezuela, which one among the (a) Canada (b) Italy
following from South America is a member of (c) Japan (d) Russia
OPEC? Ans. (d) : The G-8 is a group of the world's most
(a) Argentina (b) Brazil prosperous nations. The United States, Britain, France,
(c) Ecuador (d) Bolivia Japan, Italy, Canada and Germany - these seven
Ans. (c) : The Organisation of the Petroleum Exporting developed and industrialized nations of the world are
Countries (OPEC) is a cartel of twelve developing called G-7. Russia was also included in this group at
countries made up of Algeria, Angola, Ecuador, Iran, the Denver summit in 1997, after which this group came
Iraq, Kuwait, Libya, Nigeria, Qatar, Saudi Arabia, the to be known as 'G-8'. In 2008, the 34th summit of the
United Arab Emirates, and Venezuela. G8 was held at the Rusutsu Resort in the Toyako-Ansen
95. Which one of the following is the country's region, located in the Hakedo Islands, Japan.
administrative capital/new federal Note: Russia is removed from the group following
administrative centre of Malaysia? American sanctions.
(a) Kota Bharu (b) Kuala Terengganu 101. In the middle of the year 2008 the parliament f
(c) Putrajaya (d) Taiping which one of the following countries became
Ans. (c) : The capital of Malaysia is Kuala Lumpur. the first in the world to enact a Climate Act by
But recently the city of 'Putrajaya' was being developed passing "The Climate Change Accountability
by the Government of Malaysia as its administrative Bill"?
capital and federal city of the country. (a) Australia (b) Canada
96. With which one of the following has the B.K. (c) Germany (d) Japan
Chaturvedi Committee dealt? Ans. (b) : The Parliament of Canada became the first
(a) Review of Centre-State relations parliament in the world to enact a climate act by passing
(b) Review of Delimitation Act 'The Climate Change Accountability Bill' in mid-2008.
(c) Tax reforms and measures to increase Canada is the first country in the world to pass such an
revenues act. The bill was proposed by Jack Layton of the New
(d) Price reforms in the oil sector Democratic Party of Canada.
IAS (Pre) GS 2009 Paper I 239 YCT
102. For outstanding contribution to which one of 2. The highest honour conferred by the
the following fields is Shanti Swarup Sahitya Akademi on a writer is by electing
Bhatnagar Prize given? him its Fellow.
(a) Literature (b) Performing Arts Which of the statements given above is/are
(c) Science (d) Social Service correct?
Ans. (c) :Shanti swarup Bhatnagar Award is given by (a) 1 only (b) 2 only
the Government of India for exceptional contribution in (c) Both 1 and 2 (d) Neither 1 nor 2
the field of science.This award is honoured in the name Ans. (c): The National School of Drama was
C.S.I.R Founder Director late Shanti Swaroop established by the Sangeet Natak Akademi in 1959 and
Bhatnagar. This award was started in 1957. the Kathak Center was established in 1964. Both these
103. Consider the following statements: institutes are in Delhi. The Academy honours great
1. V.K. Krishna Menon Foundation is based in artists of various arts by electing them as Fellows of the
New Delhi. Academy. The Sahitya Akademi gives the highest honor
2. the first recipient of the V.K. Krishna to writers by choosing them as their 'Fellow'.
Menon Award is the Chief Justice of India, 107. Among the following, which one is not a
K.G. Balakrishnan. football club?
Which of the statements given above is/are (a) Arsenal (b) Aston Villa
correct? (c) Chelsea (d) Monte Carlo
(a) 1 only (b) 2 only Ans. (d) : Arsenal Football Club from Halloway (North
(c) Both 1 and 2 (d) Neither 1 nor 2 London) is a football club; Aston Villa Football Club is
Ans. (b) : The V.K. Krishna Menon Foundation, based a professional football club based in Aston
in London was instituted in the memory of Vengalil (Birmingham), Chelsea Football Club is a professional
Krishna Kurup Krishna Menon. Former Chief Justice of football club based in west London, while Monte Carlo
India. K.G. Balakrishnan was given the first V. K. is one of the administrative regions of Monaco.
Krishna Menon award for his contribution to social 108. When Leander Paes won the "mixed doubles"
justice and fighting corruption in the Indian judicial at the US Open in the year 2008, who of the
system. following was his partner?
104. To integrate cultural leaders into its meetings, (a) Ana Ivanovic
which one of the following gives "Crystal (b) Cara Black
Award"? (c) Dinara Safina
(a) Asia Pacific Economic Cooperation (d) Jelena Jankovic
(b) International Bank for Reconstruction and Ans. (b) : Cara Black and Leander Paes won in the final
Development 7- 6(8-6), 6-4, against Liezel Huber and Jamie Murray.
(c) World Health Organization 109. Among the following, who are the Agaria
(d) World Economic Forum community?
Ans. (d) : The World Economic Forum's Crystal Award (a) A traditional today tappers community of
honours artists who have used their art to improve the Andhra Pradesh
state of the world. (b) A traditional fishing community of Maharashtra
105. With reference to Stree Shakti Puraskar, for (c) A traditional silk-weaving community of Karnataka
which one of the following is "Devi Ahilya Bai (d) A traditional salt pan workers community of
Holkar Award" given? Gujarat
(a) Administrative skills Ans. (d) : The Agaria are a Muslim community found
(b) Achievements in Scientific Research in the state of Gujarat. They are salt pan workers.
(c) Achievements in Sports and Games 110. Consider the following statements:
(d) Courage and Valour (1) INS Sindhughosh is an aircraft carrier.
Ans. (d) : Stree Shakti Puraskar is given by the Union (2) INS Viraat is a submarine.
Ministry of Women and Child Development. Under Which of the statements given above is/are
this award are Kannagi Award, Rani Laxmibai Award, correct?
Rani Gaudilyu Zeliang Award, Devi Ahilyabai Award (a) 1 only (b) 2 only
and Mata Jijabai Award. Devi Ahilyabai Holkar Award (c) Both 1 and 2 (d) Neither 1 nor 2
is given for gallantry and courage. Ans. (d) : INS Sindhughosh (S55) is the lead ship of the
106. Consider the following statements: class of diesel-electric submarines of the Indian Navy.
1. The National School of Drama was set up by INS Viraat is a Centaur-class aircraft carrier in service
Sangeet Natak Akademi in 1959. with the Indian Navy.
IAS (Pre) GS 2009 Paper I 240 YCT
111. In the context of Indian defence, consider the Which of the above pairs is/are correctly
following statements: matched?
1. The Shourya missile flies with a speed of (a) 1 only (b) 2 only
more than Mach. (c) 1 and 2 (d) 2 and 3
2. The range of Shourya missile is more than Ans. (b) : ABN AMRO Bank is a Dutch bank with
1600 km. headquartered in Amsterdam,Netherlands. Barclays
Which of the statements given above is/are PLC is a global financial services company
correct? headquartered in London, United Kingdom. KB
(a) 1 only (b) 2 only Kookmin Bank is the largest bank by both asset
(c) Both 1 and 2 (d) Neither 1 nor 2 value and market capitalisation in South Korea.
Ans. (d) : The missile flew at 7.5 Mach, that is 7.5 116. Consider the following statements:
times the speed of sound and covered its full range of 1. Between Census 1951 and Census 2001, the
700 km in 500 seconds. The speed of Shaurya Missiles density of the population of India has
is 7.5 Mach, and its range is approximately 700 km to increased more than three times.
1900 km. Hence both the statements are incorrect.
2. Between Census 1951 and Census 2001, the
112. Consider the following names: annual growth rate (exponential) of the
1. Ike population of India has doubled.
2. Kate Which of the statements given above is/are
3. Gustav correct?
Which of the above are the names of hurricanes (a) 1 only (b) 2 only
that had occurred very recently?
(c) Both 1 and 2 (d) Neither 1 nor 2
(a) 1 and 2 only (b) 2 and 3 only
Ans. (d) : The rate of population density increase in
(c) 1 and 3 only (d) 1, 2 and 3
India is as follows: Density 1951 – 117 per sq km, 2001
Ans. (c) : Ike killed more than 80 individuals across the
– 324 per sq km; Annual growth rate in population
Caribbean and the Bahamas and caused U.S. property
density : 1951 – 1.25%, 2001 – 1.93%.
danger of some $ 19.3 billion, while Gustav killed 112
individuals in 2008, including 77 in Haiti. 117. Consider the following statements:
113. The borther Umakant and Ramakant 1. Infant mortality rate takes into account the
Gundecha are death of infants within a month after birth.
(a) Dhrupad vocalists 2. Infant mortality rate is the number of infant
(b) Kathak dancers deaths in a particular year per 100 live birth
(c) Sarod maestros during that year.
(d) Tabla players Which of the statements given above is/are
Ans. (a) : Umakant and Ramakant Gundecha brothers correct?
are well known Dhrupad singers. Dhrupad is the oldest (a) 1 only (b) 2 only
and most important form of singing. It originated from (c) Both 1 and 2 (d) Neither 1 nor 2
the metaphor-management called 'Dhruv'. Ans. (d) : Infant Mortality Rate – Calculates the
number of dead babies under the age of 1 month in
MISCELLANEOUS proportion to the number of babies per 1000 live births,
whereas, in terms of the number of babies per 1000 live
114. WiMAX is related to which one of the births in a year, Child Mortality is estimated by the
following? number of children dead between 0-4 years.
(a) Biotechnology Infant mortality rate is the probability of a child born in
(b) Space technology a specific year or period dying before reaching the age
(c) Missile technology of one, it subject to age specific mortality rates of that
(d) Communication technology period. Infant mortality rate is not a rate but a
Ans. (d) : WiMAX is one of the hottest broadband probability of death derived from a life table and
wireless communication technology around today. expressed as rate per 1000 live births. Hence both the
WiMAX systems are expected to deliver broadband statements are not correct.
access services to residential and enterprise customers 118. Which one among the following South Asian
in an economical way. countries has the highest population density?
115. Consider the following pairs: (a) India (b) Nepal
Large Bank Country of origin (c) Pakistan (d) Sri Lanka
1. ABN Amro Bank : USA Ans. (a) : Overall population density of India is 324.
2. Barclays Bank : UK Nepal is 102, Pakistan is 146, Sri Lanka is 280 person
3. Kookmin Bank : Japan per square kilometers.
IAS (Pre) GS 2009 Paper I 241 YCT
119. Match List-I with List-II and select the correct Code:
answer using the code given below the lists: A B C D
List-I List-II (a) 2 1 4 3
(Person) (Organization) (b) 2 4 1 3
A. Anil Agarwal 1. Gujarat Heavy (c) 3 1 4 2
Chemicals Ltd. (d) 3 4 1 2
B. Gautam H. Singhania 2. Raymond Ltd. Ans. (b) : Amrit Sher Gill is a famous painter.
C. Sanjay Dalmia 3. Vadanta Resource Pt.Bhimsen Joshi is a pioneer of Indian classical music,
D. Venugopal Dhoot 4. Videocon Group he is associated with Kirana Gharana. He has been
Code: awarded 'Bharat Ratna', the highest civilian honour of
A B C D India in 2009. Rukmani Devi Arundale is a famous
(a) 3 1 2 4 dancer and Suryakant Tripathi 'Nirala' was a famous
(b) 4 1 2 3 poet know as the main pillar of Hindi literature.
(c) 3 2 1 4 122. Match List-I with List-II and select the correct
(d) 4 2 1 3 answer using the code given below the lists:
Ans. (c) : Individuals Organisation List-I List-II
Anil Agrawal Vedanta Resources (Book) (Author)
Gautam.H.Singhania Raymond Ltd. A. In Custody 1. Amartya Sen
Sanjay Dalmia Gujarat Heavy Chemicals Ltd. B. Sea of Poppies 2. Amitav Ghosh
Venugopal Dhoot Videocon Group
C. The Argumentative 3. Anita Desai
120. Match List-I with List-II and select the correct Indian
answer using the code given below the lists:
D. Unaccustomed Earth 4. Jhumpa Lahiri
List-I List-II
Code:
(Famous Person) (Well Known for)
A B C D
A. Mandakini Amete 1. Theatre direction
(a) 4 1 2 3
B. Neelam Mansingh 2. Social service and
(b) 4 2 1 3
Chowdhry community
leadership (c) 3 2 1 4
C. Romila Thapar 3. Dance (d) 3 1 2 4
D. Vanashree Rao 4. History writing Ans. (c) : Book Author
Code: In Custody Anita Desai
A B C D Sea of Poppies Amitabh Ghosh
(a) 2 1 4 3 The Argumentative Indian Amartya Sen
(b) 2 4 1 3 Unaccustomed Earth Jhumpa Lahiri
(c) 3 1 4 2 123. Match List-I with List-II and select the correct
(d) 3 4 1 2 answer using the code given below the lists:
Ans. (a) : Mandakini Amte is known for social service List-I List-II
and community leadership. She is the daughter-in-law (Famous person) (Well-known as)
of famous social worker Baba Amte. She has been A. Anna Hazare 1. Lawyer
jointly awarded the Ramon Magsaysay Award for B. Deepak Parekh 2. Banker
Community Service in 2008 along with her husband C. GVK Reddy 3. Gandhian and social
Prakash Amte. Neelam Mansingh Chowdhary is famous
activist
for theatrical direction. Romila Thapar is a famous
historian. She has written many books like 'History of D. Harish Salve 4. Industrialist
India', 'The decline of the Maurya Empire and Ashoka'. Code:
Vanshree Rao is associated with the field of dance. A B C D
121. Match List-I with List-II and select the correct (a) 1 2 4 3
answer using the code given below the lists: (b) 1 4 2 3
List-I List-II (c) 3 2 4 1
(Famous person) (Well-known as) (d) 3 4 2 1
A. Amrita Sher-Gil 1. Dancer Ans. (c) : Anna Hazare is a famous Gandhian and social
B. Bhimsen Joshi 2. Painter activist. Deepak Parekh is a famous banker. He is head
C. Rukmini Devi Arundale 3. Poet of the HDFC bank. G.V.K. Reddy is a famous
D. Suryakant Tripathi Nirala 4. Singer industrialist and Harish Salve is a famous lawyer.
IAS (Pre) GS 2009 Paper I 242 YCT
124. Match List-I with List-II and select the correct 127. Which one of the following brings out the
answer using the code given below the lists: publication called "Energy Statistics" from
List-I List-II time to time?
(Persons in news) (Countries they (a) Central Power Research institute
belong to) (b) Planning Commission
A. Luisa Dias Diogo 1. Kenya (c) Power Finance Corporation Ltd.
B. Mwai Kibaki 2. Mozambique (d) Central Statistical Organisation
C. Rupiah Banda 3. Zambia Ans. (d) : Energy Statistics is published by Ministry of
4. Zimbabwe Statistics and Programme Implementation and Central
Code: Statistical Organisation (CSO).
A B C 128. Consider the following pairs:
(a) 1 2 3 1. Ashok Leyland : Hinduja Group
(b) 2 1 3 2. Hindalco Industries : A.V. Birla Group
(c) 2 1 4 3. Suzlon Energy : Punj Lloyd Group
Which of the pairs given above is/are correctly
(d) 1 2 4
matched?
Ans. (b) : Popular People Affiliated Countries
(a) 1 and 2 only (b) 2 only
Luisa Dias Diago Mozambique
(c) 1 and 3 only (d) 1, 2 and 3
Mwai Kibaki Kenya
Ans. (a) : The Hinduja Group is a global conglomerate
Rupiah Banda Zambia company headquartered in London, United Kingdom.
125. Consider the following pairs: Ashok Leyland is one of its companies. Hindalco Industries
Organization Location of Limited, the metals flagship company of the Aditya Birla
Headquarters Group, is the world's largest aluminium rolling company and
1. Asian Development 1. Tokyo one of the biggest producers of primary aluminium in Asia.
Bank Suzlon energy limited is a Indian multinational turbine
2. Asia-Pacific 2. Singapore manufacturing organization, based in Pune. It is not
Economic Cooperation associated with Punj Lloyd Group.
3. Association of South 3. Bangkok 129. Consider the following pairs:
East Asian Nations Persons Sport
Which of the above Pairs is/are correctly 1. Anand Pawar : Chess
matched? 2. Akhil Kumar : Boxing
(a) 1 and 2 only (b) 2 only 3. Shiv Shankar : Golf
(c) 2 and 3 only (d) 3 only Prasad Chowrasia
Ans. (b) : The headquarters of Asia Pacific Economic Which of the pairs given above is/are correctly
matched?
Cooperation is in Singapore. The headquarters of
Association of Southeast Asia Nations (ASEAN) is in (a) 1 and 2 only (b) 2 and 3 only
Jakarta, Indonesia. (c) 3 only (d) 1, 2 and 3
The headquarters of Asian Development Bank is in Ans. (b) : Anand Pawar is a badminton player. Rest is
Manila. correctly matched.
126. Consider the following countries: 130. Who of the following is a football player?
1. Switzerland 2. Malta (a) Fernando Alonso (b) Kimi Raikkonen
3. Bulgaria (c) Lewis Hamiltion (d) Nicolas Anelka
Which of the above are members of European Ans. (d) : Nicolas Anelka is a French footballer. All
Union? others are related to Formula-1 (car race).
(a) 1 only (b) 2 and 3 only 131. Which one of the following pairs is not
(c) 1 and 3 only (d) 1, 2 and 3 correctly matched?
Ans. (b) : The EU countries are: Austria, Belgium, (a) Barboara Spotakova : Jevelin throw
Bulgaria, Croatia, Republic of Cyprus, Czech Republic, (b) Pamela Jelimo : Weightlifting
Denmark, Estonia, Finland, France, Germany, Greece, (c) Sanya Richards : Sprint
Hungary, Ireland, Italy, Latvia, Lithuania, Luxembourg, (d) Yelena Isinbayeva : Pole Vault
Malta, Netherlands, Poland, Portugal, Romania, Ans. (b) : Pamela Jelimo is a Kenyan middle-distance
Slovakia, Slovenia, Spain and Sweden. runner, specialising in the 800 metres.
IAS (Pre) GS 2009 Paper I 243 YCT
132. Consider the following pairs: 136. A person travels 12 km due North, then 15 km
Tradition State due East, after that 15 km due West and then
1. Gatka, a traditional : Kerala 18 km due South, how far is he from the
martial art starting point?
2. Madhubani, a traditional : Bihar (a) 6 km (b) 12 km
painting (c) 33 km (d) 60 km
3. Sindhe Khababs Sindhu : Jammu and Ans. (a) : Following is the track of movement of the
Darshan Festival Kashmir person.
Which of the pairs given above is/are correctly
matched?
(a) 1 and 2 only (b) 3 only
(c) 2 and 3 only (d) 1, 2 and 3
Ans. (c) : Gatka, a traditional martial art, belongs to the
state of Punjab. Madhubani painting is related to the
state of Bihar, it is also called Mithila painting style.
Gangadevi of Rashidpur is famous among the great
painters of this painting. Sindhe Khabbas Sindhu
Darshan Mahotsav is celebrated with great pomp in the
state of Jammu and Kashmir.
133. Consider the following famous names:
1.Amrita Sher-Gil
2. Bikash Bhattacharjee
3. N. S. Bendre Hence distance from starting point
4. Subodh Gupta 18 – 12 = 6 km.
Who of the above is/are well known as 137. Six persons A, B, C, D, E and F are standing in
artiest(s)? a row, C and D are standing close to each other
(a) 1 only (b) 1 and 4 only alongside E. B is standing be side A only. A is
(c) 2, 3 and 4 only (d) 1, 2, 3 and 4 fourth from F. Who are standing on the
Ans. (d) : Amrita Shergill was an eminent Indian extremes?
Painter. Bikash Bhattacharjee was an Indian painter (a) A and F (b) B and D
form Kolkata, West Bengal. Narayan Shridhan Bendre (c) B and F (d) None of the above
was one of India's celebrated artist of 20th century.
Subodh Gupta is an Indian contemporary artist based in Ans. (c) : C, D, E are together.
New Delhi. Also, A and B are together.
134. The Security Council of UN consists of 5 Possible arrangements,
permanent members, and the remaining 10 B A C D E or C D E A B
members are elected by the General Assembly Now, since A is fourth from F,
for a term of F must lie on the extreme end.
(a) 1 year (b) 2 year Thus, B and F are on the extreme positions.
(c) 3 years (d) 5 years
BACDEF or FCDEAB.
Ans. (b) : The United Nations Security Council
originally had 5 permanent members and 6 non- 138. There is a family of 6 persons A, B, C, D, E and
permanent members, a total of 11 members. But in F. There are two married couples in the family.
September 1965, the number of non-permanent the family members are lawyer, teacher,
members was increased to 10. The non-permanent salesman, engineer, accountant and doctor. D,
members are elected by the General Assembly by a the salesman is married to the lady teacher.
two-thirds majority for a two-year term. The permanent The doctor is married to the lawyer. F, the
members of the UN Security Council are the US, the accountant is the son of B and brother of E. C,
Soviet Union (now Russia), China, Britain and France. the lawyer is the daughter-in-law of A, E is the
135. Who of the following is the author of the book unmarried engineer. A is the grandmother of
"The Audacity of Hope"? F. How is E related to F?
(a) Al Gore (b) Barack Obama (a) Brother
(c) Bill Clinton (d) Hillary Clinton (b) Sister
Ans. (b) : The book 'The Audacity of Hope: Thought on (c) Father
Reclaiming the American Dream' was written in 2006
by the then US Senator and later US President Barack (d) Cannot be established (cannot be determined)
Obama. 'Dreams from My Father' is another work of Ans. (d) : The gender of E is not clear. F is brother of
Obama. E but it is not clear whether E is brother or sister of F.

IAS (Pre) GS 2009 Paper I 244 YCT


139. There are three cans A, B and C. The 141. How many three-digit numbers can be
capacities of A, B and C are 6 litres, 10 litres generated from 1, 2, 3, 4, 5, 6, 7, 8, 9 such that
and 16 litres respectively. The can C contains the digits are in ascending order?
16 litres of milk. The milk has to be divided in (a) 80 (b) 81
them using these three cans only. (c) 83 (d) 84
Consider the following statements: Ans. (d) : Any 3 numbers out of 9 can be selected in
9
1. It is possible to have 6 litres of milk each in C3 ways.
can A and can B. Now, these three numbers can be arranged among
2. It is possible to have 8 litres of milk each in themselves in ascending order in only 1 way.
can B and can C Hence, total no. of ways = 9C3 × 1 = 84.
Which of statements given above is/are 142. In a carrom board game competition, m boys
correct? and n girls (m>n>1) of a school participate in
(a) 1 only (b) 2 only which every student has to play exactly one
(c) Both 1 and 2 (d) Neither 1 nor 2 game with every other student. Out of the total
Ans. (a) : The first statement is correct, since the filling games played, it was found that in 221 games
capacity of vessel A and vessel B is 6 liters and 10 liters one player was a boy and the other player was
respectively, so it is possible to take 6 liters of milk in a girl.
vessel A and vessel B each. But, the second statement Consider the following statements:
is not correct, because the filling capacity of vessel B 1. The total number of students that
and vessel C is 10 liters and 16 liters respectively and participated in the competition is 30.
vessel C contains 16 liters of milk. Therefore, although 2. the number of games in which both players
it is possible to take 8 liters of milk in vessel B but not were girls is 78.
in vessel C, because 16 liters of milk is already present Which of the statements given above is/are
in vessel C. correct?
140. There are four persons A, B, C, D and A has (a) 1 only (b) 2 only
some coins. A gave half of the coins to B and 4 (c) Both 1 and 2 (d) Neither 1 nor 2
more besides. B gave half of the coins to C and
Ans. (c) : Since in 221 games each boy plays exactly
4 more besides. C gave half of the Coins to D
one game with each girl so
and 4 more besides. Both B and D end up with
same number of coins. How many coins did A m × n = 221 ………(i)
have originally? also m > n > 1 ∴ m must be 17 and n = 13 to satisfy the
(a) 96 (b) 84 equation 1.
(c) 72 (d) 64 ∴ total no. of students = m + n = 17 + 13 = 30
Ans. (c) : Let there are x coin originally with A Again no. of games in which both players are girl =
13
According to question C 2 = 78
A gave half + 4 coins to B Hence both the statements are correct.
x 143. A person has 4 coins each of different
∴ coins with B = + 4
2 denomination. What is the number of different
again B gave half of its coins + 4 to C sums of money the person can form (using one
1 x or more coins at a time)?

∴ coins with C =  + 4  + 4 (a) 16 (b) 15
2 2 
(c) 12 (d) 11
x   x   Ans. (b) : No. of different sums of money = any 1 coin
and coins left with B =  + 4  − 1/ 2  + 4  − 4 
2   2   at a time + any 2 coins + any 3 coins in all 4 coins
1 x  x = 4C1 + 4C2 + 4C3 + 4C4 = 4 + 6 + 4 + 1 = 15.
=  + 4 − 4 = − 2
2 2  4 144. How many numbers lie between 300 and 500 in
Now C gave half of its coins + 4 coins to D where 4 comes only one time?
(a) 99 (b) 100
1 1  x   x
∴ coins with D =   + 4  + 4 + 4 = + 7 (c) 110 (d) 120
2 2 2   8
Ans. (a) : Numbers between 300 and 500 in where 4
According to question now B and D have same no. of occurs only once
coins 300-399 = 18
x x x 400-410 = 10
∴ −2= +7⇒ =9
4 8 8 411-420 = 9
∴ x = 9 × 8 = 72 421-430 = 9
IAS (Pre) GS 2009 Paper I 245 YCT
431-439 = 8 480
⇒ 2a − (n − 1) + 4 =
450-460 = 10 n
461-470 =9 Since left hand side of the equation gives integer value,
471-480 = 9 right hand side of the equation should give an integer
481-490 = 9 value.
491-500 = 8 Thus 480 should be divisible by n.
Required numbers are - 18+ 10 + 9 + 9 + 8 + 10 + 9 + 9 Out of the given option 480 is not divisible by 7
+ 9 + 8 = 99. ∴ 7 cannot be the possible value of n.
145. In an examination, there are three subjects A, 148. Four metal rods of lengths 78 cm, 117 cm and
B and C. A student has to pass in each subject.
169 cm are to be cut into parts of equal length.
20% students failed in A, 22% students failed
.Each part must be as long as possible. What is
in B and 16% failed in C. The total number of
the maximum number of pieces that can be
students passing the whole examination lies
between. cut?
(a) 42% and 84% (b) 42% and 78% (a) 27 (b) 36
(c) 58% and 78% (d) 58% and 84% (c) 43 (d) 480
Ans. (a) : Pass % is minimum when failure % is Ans. (b) :The given lengths of four metal rods are 78 ,
maximum. 104 , 117 and 169 cm
Maximum % of failure = 20 + 22 + 16 = 58% Now 78 = 13x2x3
104 = 13 x 2 x 2 x2
∴ Minimum pass % = 100 – 58 = 42%
117 = 13 x 3 x3
Now, pass % is maximum, when failure % is
169 = 13 x 13
minimum. Minimum % of failure = 16%
The maximum length of each part is as possible.
(∵ 22 % failure covers both 16% and 20% i.e., 22% is = LCM = 13 cm
the minimum value of failure %). : . Number of pieces = 6 + 8 + 9 + 13 = 36
∴ Maximum % of passing = 100 – 16 = 84% 149. While adding the first few continuous natural
Hence, pass % range = 42% to 84%. numbers, a candidate missed one of the
146. How many letters of the English alphabet numbers and wrote the answer as 177. What
(capitals) appear same when looked at in a was the number missed?
mirror? (a) 11 (b) 12
(a) 9 (b) 10 (c) 13 (d) 14
(c) 11 (d) 12 Ans. (c) : Let there be n natural numbers and x be the
Ans. (c) : The letters (capital letters) of the English missed number.
alphabet are- n(n +1)
∴ − x = 177
ABCDEFGHIJKLMNOPQRSTUVWXYZ 2
The following letters look the same when viewed in the ⇒ n (n + 1) – 2x = 354
mirror,
⇒ n (n + 1) = 354 + 2x ……….(1)
AHIMOTUVWXY
Out of the given options only 13 as value of x satisfies
Required number of letters = 11
the equation (1) as
147. There are 240 balls and n number of boxes B1,
19 (19 + 1) = 354 + 2 (13) = 380
B2, By, ....... Bn. The balls are to be placed in the
boxes such that B1 should contain 4 balls more Hence option (c) is correct LHS = RHS
than B2' B2 should contain 4 balls more than 150. How many time are an hour hand and a minute
B2. B2 should contain 4 balls more than B3' and hand of a clock at right angles during their
so on. which one of the following cannot be the motion from 1.00 p.m. to 10.00 p.m.?
possible value of n? (a) 9
(a) 4 (b) 5 (b) 10
(c) 6 (d) 7 (c) 18
Ans. (d) : Common difference = 4 (d) 20
∴ it can be considered as Arithmetic Progression. Ans. (c) : In one hour, hour hand and minute hand are at
B2 – B1 = B3 – B2……… = 4 right angles 2 times.
Total no. of balls = 240 Time = 10 p.m – 1 p.m = 9 hr.
n ∴ No. of times, when both hands are perpendicular to
∴ (2a + (n − 1)d) = 240 each other in 9 hr = 9 × 2 = 18.
a
IAS (Pre) GS 2009 Paper I 246 YCT
UNION PUBLIC SERVICE COMMISSION
Civil Services (Preliminary Exam) - 2008
GENERAL STUDIES : PAPER-I
Time: 2 hours Maximum Number: 200

ANCIENT HISTORY MODERN HISTORY


1. What does Baudhayan theorem (Baudhayan 4. Which was the Capital of Andhra State when it
Sulva Sutra) relate to? was made a separate State in the Year 1953?
(a) Lengths of sides of a right angled triangle (a) Guntur (b) Kurnool
(b) Calculation of the value of pi (c) Nellore (d) Warangal
Ans. (b) : Kurnool was the Capital of Andhra State
(c) Logarithmic calculations when it was made a separate State in the year 1953.
(d) Normal distribution curve 5. Who among the following wrote the poem,
Ans. (a) : Baudhayana Theorem (Baudhayan Formula) Subhhe-Azadi?
is related to the lengths of the sides of a right triangle. (a) Sahir Ludhiyanvi
(b) Faiz Ahmed Faiz
According to this theorem, the square on the hypotenuse
(c) Muhammad Iqbal
of a right triangle is equal to the sum of the squares on (d) Maulana Abul Kalam Azad
the base and the perpendicular. Ans. (b) : Faiz Ahmed Faiz has written a poem called
2. Near which one of the following cities are Subahe Azadi. He was born in Sialkot. After the
partition of India, he remained in Pakistan. His main
Palitana Temples located?
poetry collections are 'Dast-e-Saba' and 'Jindanama'.
(a) Bhavnagar (b) Mount Abu 6. Where was the First Session of the Indian
(c) Nasik (d) Ujjain National Congress held in December 1885?
Ans. (a) : The Palitana temples of Jainism are located (a) Ahmadabad (b) Bombay
(c) Calcutta (d) Delhi
on Shatrunjaya by the city of Palitana in Bhavnagar
Ans. (b) : First session of Indian National Congress
district, Gujarat. was organized in the premises of Gokuldas Tejpal
Sanskrit College, Bombay on 28 December, 1885 AD.
MEDIEVAL HISTORY This session was presided by Womesh Chandra
Banerjee. A total of 72 delegates attended this session
3. During the time of which Mughal Emperor did of Congress, mostly lawyers and journalists.It is
noteworthy that the Congress was established in 1884 in
the English East India Company establish its the name of Indian National Union by A.O. Hume,
first factory in India? which was renamed as the Indian National Congress in
(a) Akbar (b) Jahangir the session held in Bombay. The demands of the first
session of the Congress mainly focused on
(c) Shahjahan (d) Aurangzeb
administrative reforms and reduction in military
Ans. (b) :During the reign of Mughal Emperor activities.
Jahangir, the English East India Company established 7. Which one of the following was a journal
its first factory in India at Surat. It is important in this brought out by Abul Kalam Azad?
regard that the Indian envoy of James I ( then British (a) Al-Hilal
(b) Comrade
Crown), William Hawkins (1608-11 AD) had requested
(c) The Indian Sociologist
to open a factory in Surat in the court of Jahangir. (d) Zamindar
Jahangir allowed Hawkins to open the factory. Ans. (a) : The Al-Hilal was a newspaper published by
However, Jahangir canceled permission after some time Indian leader Maulana Abul Kalam Azad and used as a
due to the opposition from the local merchants of Surat medium for criticism of the British Raj in India. The
and the Portuguese. The British were again allowed to newspaper also espoused the cause of the Indian
independence movement and exhorted Indian Muslims
open the factory during the period of Thomas Roe to join the movement. The newspaper was shut down
(1615-18 AD), who came after Hawkins. under the Press Act of 1914.
IAS (Pre) GS 2008 Paper I 247 YCT
8. Who among the following Gandhian followers (a) Simon Commission
was a teacher by profession? (b) Simla Conference
(a) A. N. Sinha (c) Cripps Proposal
(b) Braj Kishore Prasad (d) Cabinet Mission
(c) J. B. Kriplani Ans. (d) : In 1946 cabinet mission plan, the executive
(d) Rajendra Prasad council was expanded to consist of only Indian member
Ans. (c) : Following his education at Fergusson College except the viceroy. The mission, the the Simla
in Pune, Kriplani worked as a school teacher before conference attempted to facilitate an agreement between
joining the freedom movement following Gandhi’s the Muslim league and the congress. when this failed,
return from South Africa. He was a professor of English the mission came out with its own proposals known as
and History. Kripalani came in contact with Gandhi in the cabinet Mission plan.
1917 during Champaran Satyagraha. 13. Who among the following gave a systematic
critique of the moderate politics of the Indian
9. Who among the following translated the
National Congress in a series of articles entitled
Autobiography of Madam Curie in Hindi?
New Lamps for Old?
(a) Atal Bihari Bajpayee
(a) Aurobindo Ghosh
(b) Lal Bahadur Shastri
(b) R. C. Dutt
(c) Choudhary Charan Singh (c) Syed Ahmad Khan
(d) Gobind Vallabh Pant (d) Viraraghavachari
Ans. (b) : The former Prime Minister of India, Lal Ans. (a) : K.G. Deshpande, a friend of Aurbindo from
Bahadur Shastri translated the autobiography of the his Cambridge days, was in charge of a weekly,
inventor of radium, Madam Curie in Hindi. “Induprakash”, published from Bombay. He requested
10. Who among the following rejected the title of Sri Aurobindo to write upon the current political
Knighthood and refused to accept a position in situation. Sri Aurobindo began writing a series of fiery
the Council of the Secretary of State for India? articles under the title “New Lamps for Old”, strongly
(a) Motilal Nehru criticising the Congress for its moderate policy.
(b) M. G. Ranande 14. Match List-I with List-II and select the correct
(c) G. K. Gokhale answer using the code given below the Lists:
(d) B. G. Tilak List-I (Author) List-II (Work)
A. Bankimchandra 1. Shatranj Ke Khilari
Ans. (c) : Gopal Krishn Gokhle rejected the title of
B. Dinabandhu Mitra2.Debi Chaudhurani
Knighthood and refused to take office in the council of
C. Premchand 3. Nil-Darpan
the secretary of the State for India. He was the founder 4. Chandrakanta
of servant of India society and mentored Mahata Gandhi Code
in his early years. A B C
11. During the Indian Freedom Struggle, who of (a) 2 4 1
the following raised an army called 'Free (b) 3 4 2
Indian Legion'? (c) 2 3 1
(a) Lala Hardayal (d) 3 1 4
(b) Rashbehari Bose Ans. (c) : Devi Chaudharani, Durgesh Nandani and
(c) Subhas Chandra Bose Anand Math are famous works of Bankim Chandra
(d) V. D. Savarkar Chatterjee. The story of Anand Math is based on the
Sanyasi rebellion. Our national anthem Vande Mataram
Ans. (c) : In 1942, Subhash Chandra Bose recruited
is taken from Anand Math. Neeldarpan is the creation
Indian prisoners of war captured from Rommel in North
of Deenbandhu Mitra. Its story is based on the Nile
Africa and formed a team of 10 thousand soldiers and
rebellion. Munshi Premchand is credited with the
named it 'Free Indian Legion'.
creation of famous works like Shatranj Ke Khilari,
12. Which one of the following suggested the Buddhi Kaki, Godan, Gaban, Nirmala, Karmabhoomi
reconstitution of the Viceroy's Executive etc. Chandrakanta is a novel by Devki Nandan Khatri.
Council in which all the portfolios including 15. Who among the following used the phrase 'Un-
that of War Members were to be held by the British' to criticize the English colonial control
India leaders? of India?
IAS (Pre) GS 2008 Paper I 248 YCT
(a) Anandmohan Bose Ans. (c) : Centre for D.N.A Fingerprinting and
(b) Badruddin Tyabji Diagnostics is located in Hyderabad (Andhra Pradesh).
(c) Dadabhai Naoroji The National Brain Research Center is located in
(d) Pherozeshah Mehta Gurgaon (Haryana). The National Center for Cell
Ans. (c) : Criticizing the British colonial control of Science is located in Pune.
India, the term 'Un-British' was used by Dadabhai 19. Consider the following:
Naoroji. In his essay; Poverty and Un British Rule in 1. Rice fields 2. Coal mining
India, he clearly threw light on the economic plight of 3. Domestic animals 4. Wetlands
India during the British rule and first propounded the Which of the above are sources of methane, a
theory of drain (outflow) of wealth. He was the first major greenhouse gas?
Indian to be elected to the British House of Commons in (a) 1 and 4 only (b) 2 and 3 only
1892 on a Liberal Party ticket.
(c) 1, 2 and 3 only (d) 1, 2, 3 and 4
16. Who was the Viceroy of India when the
Ans. (d) : Rice fields, coal mining, domestic animal and
Rowlatt Act was passed?
wet lands all are the sources of the methane.
(a) Lord Irwin
20. As per India's National Population Policy,
(b) Lord Reading
2000, by which one of the following years is it
(c) Lord Chelmsford
our long-term objective to achieve population
(d) Lord Wavell stabilization?
Ans. (c) : During the viceroyalty of Lord Chelmsford, (a) 2025 (b) 2035
the Rowlatt act (1919) was passed. As per this act, the
(c) 2045 (d) 2055
British government was empowered to detain anyone
Ans. (c) : As per India’s National Population Policy -
without any trial. It was repealed by Viceroy Lord.
2000, by 2045 India will achieve population stability
which means that the size of population will not go up.
INDIAN GEOGRAPHY
21. Consider the following statements:
17. Where was the first desalination plant in India 1. Salt-water crocodile is found in the Andaman
to produce one lakh litres freshwater per day and Nicobar Islands.
based on low temperature thermal desalination 2. Shrew and tapir are found in the Western
principle commissioned? Ghats of the Malabar region.
(a) Kavaratti (b) Port Blair Which of the statements given above is/are
correct?
(c) Mangalore (d) Valsad
(a) 1 only (b) 2 only
Ans. (a) : Based on the principle of low temperature (c) Both 1 and 2 (d) Neither 1 nor 2
thermal desalination in India, the first desalination plant
Ans. (a) : Salt water crocodile are found throughout the
was installed in Kavaratti to produce one lakh liters of
Andaman and Nicobar Islands living in the mangrove
fresh water per day.
habitats. Shrew and Tapir are found in Himalayan
18. Which of the following pairs are correctly
range.
matched?
22. Consider the following pairs:
Institute Location
Tributary River Main River
1. National Centre for : Mysore
1. Chambal : Narmada
Cell Science
2. Sone : Yamuna
2. Centre for DNA : Hyderabad
3. Manas : Brahmaputra
Fingerprinting
Which of the pairs given above is/are correctly
& Diagnostics matched?
3. National Brain : Gurgaon (a) 1, 2 and 3 (b) 1 and 2 only
Research Centre (c) 2 and 3 only (d) 3 only
Select the correct answer using the code given Ans. (d) : The Manas river is a major tributary of
below: Brahmaputra River, which passes through the Manas
Code: national park. While Chambal is a major tributary of
(a) 1, 2 and 3 (b) 1 and 2 only river Yamuna and Sone is major tributary of river
(c) 2 and 3 only (d) 1 and 3 only Ganga
IAS (Pre) GS 2008 Paper I 249 YCT
23. Which of the following hills are found where 28. Which of the following pairs are correctly
the Eastern Ghats and the Western Ghats matched?
meet? Irrigation project State
(a) Anaimalai Hills 1. Damanganga : Gujarat
(b) Cardamom Hills 2. Girna : Maharashtra
(c) Nilgiri Hills 3. Pamba : Kerala
(d) Shevoroy Hills Select the correct answer using the code given
Ans. (c) : Nilgiri hills are at the junction of the eastern below:
and western ghats of the Sahayadri hills. The heights of Code:
the hills range varies between 2,280 and 2,290 metres. (a) 1 and 2 only (b) 2 and 3 only
24. Among the Indian languages, which one is (c) 1 and 3 only (d) 1, 2 and 3
spoken maximum in the world after Hindi? Ans. (d) : Damanganga water project is in Gujarat.
(a) Telugu (b) Tamil Girna irrigation project is in Jalgaon area of
(c) Bengali (d) Malayalam Maharashtra. Pamba river is located in Kerala.
Ans. (c) :As per 2011 Census, Bengali remains the 29. On which one of the following rivers is the
second most spoken language while Marathi has Tehri Hydropower Complex located?
replaced Telugu from third place. (a) Alaknanda
25. Out of all the Biosphere Reserves in India, four (b) Bhagirathi
have been recognized on the World Network by (c) Dhauliganga
UNESCO. Which one of the following is not (d) Mandakini
one of them? Ans. (b) : The Tehri Hydropower complex is located on
(a) Gulf of Mannar (b) Kanchenjunga the Bhagirathi river of Utarakhand.
(c) Nanda Devi (d) Sunderbans 30. Which of the following pairs are correctly
Ans. (b) : India currently has 18 Biosphere Reserve. matched?
Out of these, the Nilgiris, the Sundarbans, the Gulf of Waterfalls River
Mannar and Nanda Devi were recognized by UNESCO 1. Kapildhara Falls : Godavari
in the world network, while Kangchenjunga was not but
2. Jog Falls : Sharavati
now Kanchanjunga also recognised as world network
3. Sivasamudram Falls : Cauvery
by UNESCO.
Select the correct answer using the code given
26. Consider the following statements:
below:
1. Chikmagalur is well-known for sugar
Code:
production.
(a) 1 and 2 only (b) 2 and 3 only
2. Mandya is well-known as a coffee-producing
(c) 1 and 3 only (d) 1, 2 and 3
region.
Ans. (b) : Jog / Garsoppa / Mahatma Gandhi Waterfall
Which of the statements given above is/are
is situated on the Sharavati River, its height is 225 m.
correct?
Shivasamudram Falls is situated on the river Kaveri.
(a) 1 only (b) 2 only
(c) Both 1 and 2 (d) Neither 1 nor 2 While the Kapildhara waterfall situated at river
Narmada.
Ans. (d) : Both the statements are incorrect.
Chikmagalur is not well-known for sugar production 31. Which of the following minerals are found in a
and Mandya is not well-known for coffee-producing natural way in the State of Chhattisgarh?
region. Chikmagalur is famous for coffee while Mandya 1. Bauxite 2. Dolomite
is famous for sugar and paper production. 3. Iron ore 4. Tin
27. Where are Tapovan Vishnugarh Hydroelectric Select the correct answer using the code given
Projects located? below:
(a) Madhya Pradesh (b) Uttar Pradesh
Code:
(c) Uttarakhand (d) Rajasthan
(a) 1, 2 and 3 only (b) 1 and 3 only
Ans. (c) : Tapovan and Vishnugarh Hydroelectric
(c) 2 and 4 only (d) 1, 2, 3 and 4
Projects are located in Uttarakhand. The Tapovan
Vishnugad power plant is a 520MW run- of-river Ans. (d) : The following minerals are found in a natural
project being constructed on Dhauliganga River in way in the State of Chhattisgarh -
Chamoli District of Uttarakhand. Bauxite, Dolomite, Iron ore and Tin.
IAS (Pre) GS 2008 Paper I 250 YCT
32. With which one of the following rivers is the Ans. (a) : The Namchik-Namphuk coalfield is located
Omkareshwar Project associated? in Arunachal Pradesh. First attempt in a modern way to
(a) Chambal (b) Narmada extract coal in India was in Raniganj coalfield of
(c) Tapi (d) Bhima Bengal. 98% of the total coal is found in the ancient
Ans. (b) : The 520 MW Omkareshwar Hydroelectric Gondwana rocks in the country; The remaining 2% is
Project is related to the Narmada River. This project found in tertiary or tertiary age rocks.
has been inaugurated on 19 November 2007. This 37. Which one of the following is not essentially a
project has been constructed by Narmada Hydroelectric species of the Himalayan vagetatioin.
Development Corporation, which is a joint venture of
(a) Juniper (b) Mahogany
National Hydroelectric Power Corporation and
Government of Madhya Pradesh.The Omkareshwar (c) Silver fir (d) Spruce
Project is the first downstream project to utilize the Ans. (b) :Mahogany is a straight-grained, reddish-
water from the main reservoir of the Indira Sagar brown timber of tropical rain forest vegetation. It is not
Project. found in Himalayan region. Juniper, Silver fir and
33. In India, how many States share the coastline? spruce are species of Himalayan vegetation.
(a) 7 (b) 8
(c) 9 (d) 10 WORLD GEOGRAPHY
Ans. (c) : The total number of 9 states in India are
located along the coast line; Gujarat, Maharashtra, Goa, 38 Which one amongst the following has the
Karnataka, Kerala, Tamil Nadu, Andhra Pradesh, Orissa largest livestock population in the world?
and West Bengal. (a) Brazil (b) China
34. Which one among the following has the (c) India (d) USA
maximum number of National Parks? Ans. (c) : India has the largest cattle inventory in the
(a) Anadaman and Nicobar Islands world. India is followed by Brazil and China. Roughly
(b) Arunachal Pradesh 63% of the world's cattle are in India, Brazil & China.
(c) Assam 39. In order of their distances from the Sun, which
(d) Meghalaya of the following planets lie between Mars and
Ans. (a) : Andaman and Nicobar Islands and Madhya Uranus?
Pradesh both have the largest number-9 of national (a) Earth and Jupiter
parks.
(b) Jupiter and Saturn
35. Match List-I with List-II and select the correct
(c) Saturn and Earth
answer using the code given below the Lists:
List-I (Board) List-II (Headquarters) (d) Saturn and Neptune
A. Coffee Board 1. Bengaluru Ans. (b) : Jupiter and Saturn are located between Mars
B. Rubber Board 2. Guntur and Uranus.
C. Tea Board 3. Kottayam
D. Tobacco Board 4. Kolkata 40. Which of the following countries share borders
Code with Moldova?
A B C D 1. Ukraine 2. Romania 3. Belarus
(a) 2 4 3 1 Select the correct answer using the code given
(b) 1 3 4 2 below:
(c) 2 3 4 1
Code:
(d) 1 4 3 2
(a) 1 and 2 only (b) 2 and 3 only
Ans. (b) : The correct matching is as follows-
(c) 1 and 3 only (d) 1, 2 and 3
Board Headquarters
Ans. (a) : Ukraine and Romania share borders with
Coffee Board - Bangaluru
Rubber Board - Kottayam Moldova. Ukraine's border with the Moldova is the
Tea Board - Kolkata longest of the two, as it is 759 miles long compared to
Tobacco Board - Guntur the 423.3 miles long of the Romania-Moldova border.
36. In which one of the following States are 41. Which one of the following straits is nearest to
Namchik-Namphuk Coalfields located? the International Date Line?
(a) Arunachal Pradesh (b) Meghalaya (a) Malacca Strait (b) Bering Strait
(c) Manipur (d) Mizoram (c) Strait of Florida (d) Strait of Gibraltar
IAS (Pre) GS 2008 Paper I 251 YCT
Ans. (b) : Bering Strait is nearest to the International
Ans. (b) : M.Hidayatullah became Sixth Vice -
Date line, because the international Date line lies President of India in 1979 and Dr. Shankar Dayal
equidistant between the American continents, on its Sharma in 1987. Both were elected unopposed.
East and Asia, Australia, and Europe on its west.
46. Which Schedule of the Constitution of India
42. Which one of the following cities is nearest to contains special provisions for the
the equator?
administration and control of Scheduled Areas
(a) Colombo (b) Jakarta
in several States?
(c) Manila (d) Singapore
(a) Third (b) Fifth
Ans. (d) : Among the given options, Singapore is
closest to the equator. (c) Seventh (d) Ninth
43. Which of the following pairs is/are correctly Ans. (b) : Fifth Schedule of the Constitution of India
matched? contains special provisions for the administration and
Theory/Law control of Scheduled Areas in several States.
Associated Scientist
1. Continental Drift : Edwin Hubble 47. Department of Border Management is a
2. Expansion of Universe : Alfred Wegener Department of which one of the following
3. Photoelectric Effect : Albert Einstein Union Ministries?
Select the correct answer using the code given (a) Ministry of Defence
below: (b) Ministry of Home Affairs
Code: (c) Ministry of Shipping, Road Transport and
(a) 2 and 3 only
Highways
(b) 3 only
(d) Ministry of Environment and Forests
(c) 2 only
(d) 1 only Ans. (b) : Department of Border Management is a
Ans. (b) : Albert Einstein was a German-Swiss department of the Union Home Ministry.
48. For which one of the following reforms was a
physicist and mathematician famous for the 'theory of
relativity'. Albert Einstein received the Nobel Prize in Commission set up under the Chairmanship of
Physics in 1921 for the photoelectric effect. In 1933, Veerappa Moilly by the Government of India?
Einstein was expelled from Germany by the Nazis and (a) Police Reforms
he took refuge in America. He was famous for (b) Tax Reforms
propounding the relation of mass and energy (E = mc2).
While the theory of expansion of universe propounded (c) Reforms in Technical Education
by Edwin Hubble and theory of continental given by (d) Administrative Reforms
Albred Wegener. Ans. (d) : On August 31, 2005, under the chairmanship
44. In which one of the following is Malta located? of Veerappa Moilly, the former Chief Minister of
(a) Baltic Sea Karnataka, a six-member Second Administrative
(b) Mediterranean Sea Reforms Commission was formed. The first report of
(c) Black Sea the Second Administrative Reforms Commission was
(d) North Sea submitted to the Prime Minister on June 9, 2006, titled
Ans. (b) : Malta is a country located in the
'Right to Information - Master's Key to Good
Mediterranean Sea, its capital is Valletta.
Governance'. So far this commission has submitted six
INDIAN CONSTITUTION AND POLTY reports. It is to be known that the first Administrative
Reforms Commission was constituted in 1966 under the
45. Who among the following have held the office chairmanship of Morarji Desai. After Morarji Desai
of the Vice-President of India? became the Deputy Prime Minister, the first
1. Mohammad Hidayatullah
Administrative Reforms Commission was headed by K.
2. Farkhruddin Ali Ahmed
Hanumanthaiah.
3. Neelam Sanjiva Reddy
4. Shankar Dayal Sharma 49. Under which one of the following constitution
Select the correct answer using the code given Amendment Acts, four languages were added
below: to the languages under the Eight Schedule of
(a) 1, 2, 3 and 4 the Constitution of India, thereby raising their
(b) 1 and 4 only number to 22?
(c) 2 and 3 only (a) Constitution (Ninetieth Amendment) Act
(d) 3 and 4 only (b) Constitution (Ninety first Amendment) Act
IAS (Pre) GS 2008 Paper I 252 YCT
(c) Constitution (Ninety-second Amendment) Ans. (b) : Justice P. N. Bhagwati and Justice V. R.
Act Krishna Iyer were among the first judges to admit
(d) Constitution (Ninety-third Amendment) Act Public Interest litigation (PIL) in the court.
Ans. (c) :Constitution (Ninety-second Amendment) Act 53. Which one of the following is the largest
2003 included four more languages in the Eighth (areawise) Lok Sabha constituency?
Schedule .They are Bodo, Dogri (Dongri), (a) Kangra (b) Ladakh
Mathilli(Maithili)and Santhali With this, the total
(c) Kachchh (d) Bhilwara
number of constitutionally recognized Languages
increased to 22. Originally there were 14 languages is Ans. (b) : Ladakh is the largest constituency of Lok
8th Schedule. 21st amendment act added Sindhi Sabha in terms of area.
language 71st amendment at added Konkari, Manipuri 54. How many High Courts in India have
and Nepali language. jurisdiction over more than one State (Union
50. Consider the following statements: Territories not included)?
The Constitution of India provides that (a) 2 (b) 3
1. The Legislative Assembly of each State shall (c) 4 (d) 5
consist of not more than 450 members Ans. (b) : Bombay HC (Maharashtra & Goa); Guwahati
chosen by direct election from territorial (Assam, Manipur, Meghalaya, Nagaland, Tripura,
constituencies in the State. Mizoram and Arunachal Pradesh); Punjab and Haryana
2. A person shall not be qualified to be chosen HC (Punjab, Haryana).
to fill a seat in the Legislative Assembly of a
55. What is the number of spokes in the
State if he/she is less than 25 years of age.
Dharmachakra in the National Flag of India?
Which of the statements given above is/are
correct? (a) 16 (b) 18
(a) 1 only (b) 2 only (c) 22 (d) 24
(c) Both 1 and 2 (d) Neither 1 nor 2 Ans. (d) : Dharamchakra has 24 spokes, it is navy-blue
Ans. (b) : Statement 2 is correct as per provisions given in colour and lies in the middle of white band. Its design
under Article 173. Statement 1 is incorrect as according is that of the wheel which appears on the abacus of the
to article 170, the legislative assembly of each state Saranath Lion capital of Ashoka.
shall consist of not more than 500 and not less than 60
members chosen by direct election from territorial ECONOMY
constituencies in the state.
51. Which of the following is/are included in the 56. What is the name of the scheme which provides
Directive Principle of State Policy? training and skills to women in traditional and
1. Prohibition of traffic in human beings and non-traditional trades?
forced labour. (a) Kishori Shakti Yojna
2. Prohibition of consumption except for (b) Rashtri Mahila Kosh
medicinal purposes of intoxicating drinks (c) Swayamsidhha
and of other drugs which are injurious to
(d) Swawlamban
health
Select the correct answer using the code given Ans. (d) : Swavlamban' is the the name of the scheme
which provides training and skills to women in
below:
traditional and non-traditional trades. Swawlamban
(a) 1 only (b) 2 only
(NORAD) Scheme is being implemented by the
(c) Both 1 and 2 (d) Neither 1 nor 2 Department of Women and Child Development with
Ans. (b) :Statement 1 is incorrect as it is a Fundamental partial assistance from Norway since 1982.
Right under article 23 of Part III of the constitution. 57. Consider the following statements with
Statement 2 corresponds to Directive Principles of State reference to Indira Gandhi National Old Age
Policy under Article 47 under Part IV of the Pension Scheme (IGNOAPS):
Constitution. 1. All persons of 60 years or above belonging to
52. Consider the following statements: the households below poverty line in rural
1. Justice V.R. Krishna lyer was the Chief Justice areas are eligible.
of India. 2. The Central Assistance under this Scheme is at
2. Justice V.R. Krishna lyer is considered as one the rate of Rs 300 per month per beneficiary.
of the progenitors of public interest litigation Under the Scheme, States have been urged to
(PIL) in the Indian judicial system. give matching amounts.
Which of the statements give above is/are Which of the statements given above is are
correct? correct?
(a) 1 only (b) 2 only (a) 1 only (b) 2 only
(c) Both 1 and 2 (d) Neither 1 nor 2 (c) Both 1 and 2 (d) Neither 1 nor 2
IAS (Pre) GS 2008 Paper I 253 YCT
Ans. (d) : Indira Gandhi National Old Age Pension (a) Bandung Conference
Scheme (IGNOAPS) : The eligible age for IGNOAPS (b) Bretton Woods Conference
is 60 years and above. There is no such criteria for (c) Versailles Conference
households below poverty line. The pension is Rs.200 (d) Yalta Conference
p.m. for persons between 60 years and 79 years. For
persons who are 80 years and above the pension is Ans. (b) : United Nations Monetary and Financial
Rs.500/ - per month. Conference in which agreements were signed for the
58. In August 2006, the Government of India establishment of IBRD, GATT and IMF, commonly
notified the Rural Electrification policy. This called Bretton Woods Conference. The International
policy aims at provision of access to all Bank for Reconstruction and Development (IBRD) is a
households by which year? global development cooperative bank which supports
(a) 2008 (b) 2009 World Bank’s mission by providing loans to middle
(c) 2010 (d) 2012 income countries. At present the World Bank group is a
Ans. (b) : Goals include provision of access to grouping of the following institutions - 1. Bank for
electricity to all households by the year 2009, quality International Development and Reconstruction (IBRD),
and reliable power supply at reasonable rates, and 2. International Development Association (IDA), 3
minimum lifeline consumption of 1 unit/household/day International Finance Corporation (IFC), 4. Multilateral
as a merit good by year 2012. Investment Guarantee Organization (MIGA), 5.
59. Which of the following pairs about India's International Center for Settlement of Investment
economic indicator and agricultural production Disputes (ICSID). India is a member of all except
(all in rounded figures) are correctly matched? ICSID. General Agreement on Tariffs and
1. GDP per capita : Rs 37000 Trade (GATT) covers international trade in goods.
(current prices) GATT was established in 1947. The IMF (International
2. Rice : 180 million tons Monetary Fund) is an international monetary
3. Wheat : 75 million tons organization. According to the decision of the Bretton
Select the correct answer using the code given Woods Conference, it was established in Washington on
below: December 27, 1945, but it actually started functioning
(a) 1, 2 and 3 (b) 1 and 2 only from March 1, 1947.
(c) 2 and 3 only (d) 1 and 3 only 62. Which of the following pairs in respect of
Ans. (d) : GDP in India is Rs. 37,000 per person current power generation in India is/are
(present price).The total production of wheat in 2006-07 correctly matched?
was 74.9 million tonnes and that of rice was 92-7
(Rounded Figure)
million tonnes.
1. Installed electricity : 100000 MW
60. India is a member of which of the following?
Generation capacity
1. Asian Development Bank
2. Asia-Pacific Economic Cooperation 2. Electricity generation : 660 billion KWh
3. Colombo Plans Select the correct answer using the code given
4. Organization for Economic Cooperation and below:
Development (OECD) (a) 1 only (b) 2 only
Select the correct answer using the code given (c) Both 1 and 2 (d) Neither 1 nor 2
below. Ans. (b) : According to the estimates for the year 2006-
(a) 1 and 3 only (b) 2 and 4 only 07, the total installed capacity of power generation in
(c) 1, 2 and 3 only (d) 1, 2, 3 and 4 India is 132300 MW. Apart from this, the total
electricity generation in India was 660 billion kW/h.
Ans. (a) : India is a member of the Asian Development
Thus the first fact in question is false and the second
Bank and Colombo Plan from the above mentioned
fact is true.
institutions. The Asian Bank was established in 1967
with the cooperation of the United States and Japan for 63. What does S & P 500 relate to?
economic development in Asian countries.Its (a) Supercomputer
headquarter is located in Manila (Philippines). The (b) A new technique in e-business
Colombo Plan was established in 1950 with the aim of (c) A new technique in bridge building
providing assistance for the development of newly (d) An index of stocks of large companies
independent Asian countries.Its headquarter is in
Colombo (Sri Lanka). Ans. (d) : The S&P 500 is an indicator of the stock of
61. How is the United Nations Monetary and large companies. Its old name is Crisil - 500. This is
Financial Conference wherein the agreements the share price index of the Mumbai Stock Exchange.
were signed to set up IBRD, GATT and IMF, Dolex, Sensex and Bankex are also the stock base
commonly known? indices of the Mumbai Stock Exchange.
IAS (Pre) GS 2008 Paper I 254 YCT
64. Which of the following are the public sector Ans. (c) : Kinetic friction also known as dynamic or
undertakings of the Government of India? sliding friction occurs when a body is in relative motion
1. Balmer Lawrie and Company Ltd. with another with their surface in contact. It is the
2. Dredging Corporation of India friction force that slows down and eventually stops a
3. Educational Consultants of India Ltd. moving object. So, dynamic friction in any object is
Select the correct answer using the code give proportional to instant reaction and is less than static
below: friction. When a car turns on a straight level road, the
(a) 1 and 2 only (b) 2 and 3 only normal reaction either of the front wheels will be equal
(c) 1 and 3 only (d) 1, 2 and 3 to R.
Ans. (d) : Balmer Lawrie & Company Limited, 68. Consider the following statements:
Dredging Corporation of India and Educational 1. A widely used musical scale called diatonics
Consultants of India Limited are Public Sector scale has seven frequencies.
Undertakings of the Government of India. 2. The frequency of the note ‘Sa’ is 256 Hz and
that of ‘NI’ is 512 Hz.
PHYSICS Which of the statements given above is/are
correct?
65. Consider the following statements: (a) 1 only (b) 2 only
1. The albedo of an object determines its visual
(c) Both 1 and 2 (d) Neither 1 nor 2
brightness when viewed with reflected light.
Ans. (a) : Tones with different frequencies are found in
2. The albedo of Mercury is much greater than
all types of popular music systems. Musical scale called
the albedo of the Earth.
diatonics scale has seven frequencies. Hence statement
Which of the statements given above is/are
1 is correct. The frequency of the note ‘Sa’ is 240 Hz
correct?
and NI is 450 Hz. Therefore statement 2 is not correct.
(a) 1 only (b) 2 only
69. Which one of the following laser types is used
(c) Both 1 and 2 (d) Neither 1 nor 2
in a laser printer?
Ans. (a) :The albedo of an object will determine its
(a) Dye laser (b) Gas laser
visual brightness when viewed with reflected light. For
(c) Semiconductor laser (d) Excimer laser
example, the planets are viewed by reflected sunlight
Ans. (c) : A semiconductor laser (LD) is a device that
and their brightness depends upon the amount of light
received from the sun and their albedo. The albedo of causes laser oscillation by flowing an electric current to
mercury is 0.12 while albedo of earth is 0.30. Therefore semiconductor. It is used in Laser Printers.
statement 2 is not correct. 70. Consider the following statements in respect of
66. How can the height of a person who is six feet a jet engine and a rocket:
tall be expressed (approximately) in 1. A Jet engine uses the surrounding air for its
nanometres? oxygen supply and so is unsuitable for
6
(a) 183×10 nanometres motion in space.
6
(b) 234×10 nanometres 2. A rocket carries its own supply of oxygen in
7
(c) 183×10 nanometres the gas form and fuel.
(d) 234×107 nanometres Which of the statements given above is/are
7 correct?
Ans. (c) : 6 Feet=183 × 10 nanometres.
(a) 1 only (b) 2 only
67. A person is sitting in a car which is at rest. The
reaction from the road at each of the four (c) Both 1 and 2 (d) Neither 1 nor 2
wheels of the car is R. When the car runs on a Ans. (c) : A jet engine combines oxygen from the air
straight level road, how will the reaction at with fuel at high temperature. There is usually a spark
either of the front wheels vary? to ignite the fuel vapor, but once the engine begins
(a) It will be greater than R turning, it will continue running until it runs out of fuel
(b) It will be less than R or air. So it is not suitable for motion in space. A rocket,
(c) It will be equal to R in contrast, carries both fuel (which may be solid or
(d) It shall depend on the material of the road liquid) and oxygen.

IAS (Pre) GS 2008 Paper I 255 YCT


Chemistry 76.
Who among the following discovered heavy
water?
71. What is the pH level of blood of a normal (a) Heinrich Hertz
person? (b) H. C. Urey
(a) 4.5-4.6 (b) 6.45-6.55 (c) G. Mendel
(c) 7.35-7.45 (d) 8.25-8.35 (d) Joseph Priestley
Ans. (c) : The normal pH value of blood is slightly
Ans. (b) :Heavy water (D2O) was discovered by
basic. It ranges between 7.35 to. 7.45.
American scientist Harold Urey (H.C. Urey) in 1931.
72. Which one of the following pairs of metals
In all P.H.W.R. (Pressurized Heavy Water Reactor)
constitutes the lightest metal and the heaviest
metal, respectively? D2O (Heavy water) is used as a diluent or coolant.
(a) Lithium and mercury
(b) Lithium and osmium BIOLOGY
(c) Aluminium and osmium
77. Consider the following statements about
(d) Aluminium and mercury
probiotic food:
Ans. (b) : The lightest metal in the periodic table is
lithium (Li) with atomic number 3 and density 0.53 1. Probiotic food contains live bacteria which are
kg/L. Lithium metal is extremely soft (and highly considered beneficial to humans.
reactive) and so is unusable for many applications. 2. Probiotic food helps in maintaining gut flora.
Osmium is a hard metallic element which has the Which of the statements given above is/are
greatest density of all known elements. It is twice as correct?
heavy as lead, and has a specific gravity of 22.59. (a) 1 only (b) 2 only
73. Mixture of which one of the following pairs of (c) Both 1 and 2 (d) Neither 1 nor 2
gases is the cause of occurrence of most of the Ans. (c) : Probiotics are the living microorganisms,
explosions in mines? healthy for host organism. Lactic acid bacteria, Bifido
(a) Hydrogen and oxygen bacteria are common type of microbes used as
(b) Oxygern and acetylene probiotics. Certain yeast and Bacilli may also be useful.
(c) Methane and air Probiotics are commonly consumed as a part of
(d) Carbon dioxide and methane fermented food with specially added active live cultures
Ans. (c) : Cause of blast in a mine in generally mixture such as yoghurt or as diatary supplements.
of methane and air. CH4 is the main gas exerted from a 78. Among the following, which one lays eggs and
mine. When methane comes in contact with air, does not produce young ones directly?
explosions take place.
(a) Echidna (b) Kangaroo
74. Which one of the following is also called
Stranger Gas? (c) Porcupine (d) Whale
(a) Argon (b) Neon Ans. (a) : Echidna lays eggs, after which children
(c) Xenon (d) Nitrous oxide develop from them. It is such a mammal that does not
give babies directly.
Ans. (c) :Xenon is called the ‘stranger’ gas. The gas is
very un-reactive and heavier than air, that is why named 79. The release of which one of the following into
strange (in greek it means ‘xenon’). ponds and wells helps in controlling the
mosquitoes?
75. What are Rubies and Sapphires chemically
known as? (a) Crab (b) Dogfish
(c) Gambusia fish (d) Snail
(a) Silicon dioxide
(b) Aluminum oxide Ans. (c) : Gambusia affinis is a larvivorous fish and is
also known by the name mosquito fish. This fish is
(c) Lead tetroxide
released into ponds and wells to control the mosquitoes
(d) Boron nitride by eating mosquito larvae, insects and crustaceans.
Ans. (b) : Rubies and Sapphires are scientifically the
80. What is Bisphenol A (BPA)?
same stone, differing only in colour. Corundum, the (a) A medical test for detecting cancer
predominating mineral of both, is composed of nearly (b) A test for testing the use of drugs to improve
pure alumina (Al2O3).The colouring substance which performance by athletes
differentiates rubies and sapphires is believed to be (c) A chemical used for the development of food
chromium. Rubies and Sapphires are chemically known packaging materials
as aluminum oxide. (d) A special type of alloy steel

IAS (Pre) GS 2008 Paper I 256 YCT


Ans. (c) : Bisphenol A, commonly abbreviated as BPA, CURRENT AFFAIRS
is an organic compound with two phenol functional
groups. Bisphenol A is used primarily to make plastic 85. ISRO successfully conducted a rocket test using
which is used in making food packaging material. It is a cryogenic engines in the year 2007. Where is
key monomer in production of epoxy resins and used to the test-stand used for the purpose, located?
make polycarbonateplastic.Polycarbonate plastic, which (a) Balasore
is clear and nearly shatter-proof, is used to make a (b) Thiruvananthapuram
variety of common products including baby and water (c) Mahendragiri
bottles, sports equipment, medical and dental devices, (d) Karwar
dental fillings and sealants, eyeglass lenses, CDs and
Ans. (c) : India successfully tested the indigenous
DVDs, and household electronics.
cryogenic engine on 15 November 2007 at the Liquid
Propulsion System Center in Mahendragiri, Tamil
ENVIRONMENT AND ECOLOGY Nadu. The new version of the indigenous cryogenic
81. Among the following, which one is not an ape? engine for GSLV was successfully test fired for 720
seconds. Now GSLV will no longer have to rely on
(a) Gibbon (b) Gorilla
Russian cryogenic engines. Till now only America,
(c) Langur (d) Orangutan
Russia and Europe had the capability of cryogenic
Ans. (c) : Gibbon, Gorilla and Orangutan are apes, but engines.
Langur is not an ape, it is prosimians.
86. Nobel prize winning Scientist James D. Watson
82. Which one of the following is an insectivorous is known for his work in which area?
plant?
(a) Metallurgy
(a) Passion flower plant (b) Meterology
(b) Pitcher plant (c) Environmental protection
(c) Night queen (d) Genetics
(d) Flame of the forest Ans. (d) : Nobel Prize winning scientist James D.
Ans. (b) : Pitcher plant is an insectivorous plant. It Watson is best known for his work in the field of
feeds on living creatures including insects and small genetics. He was the first to use the name genetics in
mammals. These plant attracts the prey with a smell of 1905.
rotting meat. The victim is dissolved by chemical 87. In the context of the Indian defence, what is
enzymes. 'Dhruv'?
83. For which one of the following snakes is the (a) Aircraft-carrying warship
diet mainly composed of other snakes? (b) Missile-carrying submarine
(a) Krait (b) Russell's viper (c) Advanced light helicopter
(d) Intercontinental ballistic missile
(c) Rattle-snake (d) King cobra
Ans. (c) : Hindustan Aeronautics Limited (HAL) has
Ans. (d) : Nagraj (King Cobra) is a dark brown or black developed the Dhruv advanced light helicopter (ALH),
snake up to 2 meters long. The food of this snake is a light (5.5t class) multi-role and multi-mission
mainly other snakes. Karait is the most venomous snake helicopter for army, air force, navy, coastguard and civil
among Indian snakes, it is also called Chitkodia snake. operations for both utility and attack roles by day and
It mostly comes out in the evening. Rasal Pridaku is night.
found in plains and hilly areas. Its length is about 1.5 88. With which one of the following is BRIT
meters, its venom is toxic to the blood system. (Government of India) engaged?
84. In which one of the following kinds of (a) Railway Wagons
organisms is the phenomenon found wherein (b) Information Technology
the female kills the male after copulation? (c) Isotope Technology
(a) Dragonfly (b) Honeybee (d) Road Transport
(c) Spider (d) Pit viper Ans. (c) : Board of Radiation and Isotope Technology
in short known as "BRIT" is a unit of the Department of
Ans. (c) : Sexual cannibalism is identified only in Atomic Energy, headquartered in Navi Mumbai. It is
spiders in which female organism kills the male during involved in production, development, and supply of
or after copulation. In most species in which it occurs, radioisotope based products and provision of isotope
sexual cannibalism is related to the larger size of female applications, radiation processing, radio analytical
due to sexual dimorphism. services etc.

IAS (Pre) GS 2008 Paper I 257 YCT


89. What is the purpose of the US Space Agency's (a) Ministry of Agriculture
Themis Mission, which was recently in the (b) Ministry of Science and Technology
news? (c) Ministry of New and Renewable Energy
(a) To study the possibility of life on Mars (d) Ministry of Rural Development
(b) To study the satellites of Saturn Ans. (d) : The implementation of the Bio-Diesel
(c) To study the colourful display of high latitude Mission (as the nodal ministry) is being done by the
skies Union Ministry of Rural Development.
(d) To build a space laboratory to study the 94. Rajiv Gandhi National Flying Institute is being
stellar explosions established in which State?
Ans. (c) : The Themis mission, comprising five (a) Karnataka (b) Maharashtra
identical probes, aims to gain new insights into the (c) Kerala (d) Orissa
colourful displays in high-latitude skies. In particular, Ans. (b) : Rajiv Gandhi National Flying Institute was
scientists want to understand what triggers a sudden inaugurated on 9 February 2008 by the Prime Minister
brightening of the lights. Dr.Manmohan Singh at Birsi Airport, Gondia
90. In which one of the following locations is the (Maharashtra).
International Thermonuclear Experimental 95. Recently, the manuscripts of which one of the
Reactor (ITER) project to be built? following have been included in the UNESCO's
(a) Northern Spain Memory of World Register?
(b) Southern France (a) Abhidhamma Pitaka
(c) Eastern Germany
(b) Mahabharata
(d) Southern Italy
(c) Ramayana
Ans. (b) : ITER (International Thermonuclear
(d) Rig-Veda
Experimental Reactor) is an international nuclear fusion
research and engineering megaproject, which is Ans. (d) : In June 2007, UNESCO has included 30
currently building the world's largest experimental to manuscripts of Rigveda, the oldest Hindu text from
kamak nuclear fusion reactor adjacent to the Cadarache 1800 to 1500 BC, and the Red Fort, built in Delhi by the
facility in the south of France. Mughal emperor Shah Jahan, in its two separate
91. Which one of the following is a spacecraft? heritage lists. While the Red Fort has been included in
(a) Apophis (b) Cassini the list of World Heritage Sites, the Rigveda
(c) Spitzer (d) TechSar manuscripts have been included in the list of cultural
heritage. Then, total number of sites in India in the
Ans. (b) : Cassini-Hygens commonly known as Cassini.
Cassini is a probe developed by NASA, and Huygens is World Heritage List stands at 27.
a landed developed by E.S.A. Cassini mission is a joint NOTE: Now it is 38.
venture of 3 different space agencies- NASA, ESA, and 96. From which one of the following did India buy
ASI to visit planet Saturn. Cassini is an unmanned the Barak anti-missile defence systems?
spacecraft. (a) Israel (b) France
92. Selene-1, the lunar orbiter mission belongs to (c) Russia (d) USA
which one of the following? Ans. (a) : Barak is an Israeli surface-to-air missile
(a) China (b) European Union (SAM) designed to be used as a ship-borne point-
(c) Japan (d) USA defense missile system against aircraft, anti-ship
Ans. (c) :Mitsubishi Heavy Industries, Ltd. and the missiles, and UAVs. India and Israel jointly developed
Japan Aerospace Exploration Agency (JAXA) launched and produced it.
the Lunar Orbit Explorer "KAGUYA" (SELENE) on
97. Who among the following scientists shared the
September 14, 2007, from the Tanegashima Space
Nobel Prize in Physics with his son?
Center. The major objectives of the mission were to
understand the Moon's origin and evolution and to (a) Max Planck (b) Albert Einstein
observe the Moon in various ways in order to utilize the (c) William Henry Bragg (d) Enrico Fermi
facts in the future. Ans. (c) : Sir William Henry Bragg shared the Noble
93. Which one of the following Union Ministries is Prize in physics with his son Willion Lawrence Bragg
implementing the Biodiesel Mission (as Nodal in 1915 for their services in the analysis of crystal
Ministry)? structure by means of X-rays.

IAS (Pre) GS 2008 Paper I 258 YCT


98. Ogaden region has been a source of conflict Ans. (c) : Sarosh Zaiwalla is a renowned international
between countries? legal expert.
(a) Morocco and Algeria 104. Where was the World Summit on Sustainable
(b) Nigeria and Cameroon Development (Rio + 10) held?
(c) Angola and Zambia (a) Davos (b) Nova Scotia
(d) Ethiopia and Somalia (c) Johannesburg (d) Shanghai
Ans. (d) : The Ogaden region was the cause of dispute Ans. (c) : The Johannesburg Summit, 2002 was a
between Ethiopia and Somalia countries of Africa. specially organized global summit on sustainable
99. In the year 2007, an earthquake led to massive development. It is also called the Earth 10 (Rio + 10)
radioative water leakage in the largest nuclear conference because it was also a ten year review of the
plant in the world. In which country did it implementation of Agenda-21 given in 1992 AD in Rio-
occur? de Janeiro to control climate change. Emphasis was
(a) Germany (b) Canada laid on the mandatory and timely implementation of the
(c) Japan (d) USA Kyoto Protocol.
Ans. (c) : On July 16, 2007, a powerful 6.8 magnitude 105. Consider the following statements:
earthquake occurred in Niigata, Japan's northwestern 1. Clean Development Mechanism (CDM) in
province, due to which the Kasivazaki Nuclear Plant, respect of carbon credits is one of the Kyoto
one of the world's largest nuclear power plants, caught Protocol Mechanisms.
fire and radioactive emissions started leaking from it. 2. Under the CDM, the projects handled
100. Cristina Kirchner succeeded her husband to pertain only to the Annex-I countries
become President of a South American Which of the statements given above is/are
country. Which is that country? correct?
(a) Chile (b) Argentina (a) 1 only (b) 2 only
(c) Colombia (d) Venezuela (c) Both 1 and 2 (d) Neither 1 nor 2
Ans. (b) : President Bester Kirchner's wife Christina Ans. (a) : The Clean Development Mechanism (CDM)
was elected as President in Argentina. She is the first is one of the Flexible Mechanisms defined in the Kyoto
elected female President of the country. In the election Protocol (IPCC, 2007) that provides for emissions
reduction projects which generate Certified Emission
held on October 28, 2007, Christina got 45% and her
Reduction units which may be traded in emissions
rival Elisa Correa got 22.6% of the vote. trading schemes. The CDM is defined in Article 12 of
101. Abraham Alkazi is an eminent personality in the Protocol and is intended to meet two objectives: (1)
which one of the following areas? to assist parties not included in Annex I in achieving
(a) Heart surgery sustainable development and in contributing to the
ultimate objective of the United Nations Framework
(b) Civil aviation
Convention on Climate Change(UNFCCC), which is to
(c) Theatre training prevent dangerous climate change; and (2) to assist
(d) Transport engineering parties included in Annex I in achieving compliance
Ans. (c) : Abraham Alkazi is related to the field of with their quantified emission limitation and reduction
theater training. commitments (greenhouse gas (GHG) emission caps).
102. The term "Prisoner's Dilemma" is associated 106. What is Agent Orange?
with which one of the following (a) An ace spying group trained by the Police in
(a) A technique in glass manufacture London
(b) A term used in shipping industry (b) A weed-killing chemical used by the US
(c) A situation under the Game theory Military in the Vietnam War
(d) Name of a supercomputer (c) A technique used in the advanced railway
safety procedure
Ans. (c) : The term 'Prisoner's Dilemma' is related to
(d) A special chemical used in making alloy steel
situations within game theory
Ans. (b) : Agent Orange is a weed-killing chemical
103. How is Sarosh Zaiwalla well-known as? used by the United States military in the Vietnam War.
(a) A leading theatre personality The Vietnam War ended on 30 April 1975. The end of
(b) A leading wildlife photographer the war in Vietnam greatly facilitated the decline of the
(c) A leading international legal expert Cold War, as Vietnam remained the center of the Cold
(d) A leading orthopaedic surgeion War in international relations in the mid-sixties.

IAS (Pre) GS 2008 Paper I 259 YCT


107. Match List-I with List-II and select the correct 109. Consider the following statements:
answer using the code given below the Lists: 1. The Nuclear Suppliers Group has 24
List-I List-II countries as its members.
(Organization) (Associated 2. India is a member of the Nuclear Suppliers
Convention/person/Place Group.
Headquarters) Which of the statements given above is/are
A. Amnesty I 1. Viskuli correct?
international (a) 1 only (b) 2 only
B. Commonwealth of 2. Paris (c) Both 1 and 2 (d) Neither 1 nor 2
Independent State Ans. (d) : The Nuclear Suppliers Group has 45
(CIS) countries as members. India is not a member of the
Nuclear Suppliers Group.
C. Danube 3. Belgrade
110. C. Rangarajan has been actively involved in the
Commission
Pan-Asian Commission addressing to which
D. European Space 4. Belgrade Convention
one of the following issues?
Agency
(a) Promotion of tourism in the Asian region
Code:
(b) Impact of HIV/AIDS in the Asian region
A B C D (c) Promotion of free trade in the Asian region
(a) 3 1 4 2 (d) Higher technical education in the universities
(b) 4 2 3 1 in the Asian region
(c) 3 2 4 1 Ans. (b) : C. Rangarajan have been associated with the
(d) 4 1 3 2 Pan - Asian Commission activism of 'The Impact of
Ans. (a) : Amnesty International is an international HIV / AIDS’, in the Asian Region.
organization for the protection of human rights, its
headquarter is located in London.It began with an
111. Match List-I with List-II and select the correct
appeal in the newspapers on 28 May 1961 by a British answer using the code given below the Lists:
lawyer, Peter Benenson. Almost all the countries of the
List-I List-II
world are its members.
(Person) (Position)
Commonwealth of Independent States (CIS) - This is
A. Nagender Singh 1. Chief Election
the Commonwealth of Independent States, it was
Commissioner of India
established in 1991, it has a total of 12 members, its
B. A. N. Ray 2. President, International
headquarter is in Minsk (Viskuli).
Court of Justice
Danube Commission - It is related to Belgrade
C. R. K. Trivedi 3. Chief Justice of India
conference.
D. Ashok Desai 4. Attorney General of
European Space Agency – Its headquarter is in Paris
India
(France).
Code:
108. "Hand-in-Hand 2007', a joint anti-terrorism
A B C D
military training was held by the officers of the
(a) 1 4 2 3
Indian Army and officers of Army of which one
(b) 2 3 1 4
of the following countries?
(c) 1 3 2 4
(a) China (b) Japan
(d) 2 4 1 3
(c) Russia (d) USA
Ans. (b) : Nagendra Singh was the first Indian to
Ans. (a) : India and China started the first ever joint
become the President of the International Court of
military exercise as part of confidence building
Justice.
measures on December 21, 2007 at Kunming Military
A . N. Rai was the Chief Justice of India, his tenure
Academy in China's southeastern Yunan province. In
was from 26 April 1973 to 27 April 1977.
this exercise called Hand in Hand-2007. The Indian
R. K. Trivedi was the Chief Election Commissioner of
Army was led by Brigadier D.S. Dadwal. This exercise India, his tenure was from 18 June 1982 to 31
is an important step towards understanding each other December 1985.
and strengthening cooperation. Ashok Desai - was the Attorney General of India.

IAS (Pre) GS 2008 Paper I 260 YCT


112. Which one of the following is correct in respect 116. Kim Dae-yung won the Nobel Prize for Peace.
of Indian football team's performance in the He is from which one of the following
Olympic Games? countries?
(a) India has never participated in the Olympic (a) Vietnam (b) Cambodia
Football Tournament (c) South Korea (d) Japan
(b) India played only in the First Round Ans. (c) : Kim Dae-yung of South Korea was awarded
(c) India entered only up to the Quarter Finals the Nobel Peace Prize in 2000.
(d) India entered the Semi Finals 117. Consider the following statements:
Ans. (d) : In 1956 Olympic Games, India entered semi 1. Orange prize is awarded to a work of
final of Football tournament. published fiction in English to a woman.
113. Starting with the Australian Open Lawn 2. Pulitzer Prize is awarded by the
Tennis Tournament, Which one of the Commonwealth Foundation to a citizen of
following is the correct Chronological order of any commonwealth country for any literary
the other three major lawn Tennis work in English.
Tournaments? Which of the statements given above is/are
(a) French Open-US Open-Wimbledon correct?
(b) French Open-Wimbledon-US Open (a) 1 only (b) 2 only
(c) Wimbledon-US Open-French Open (c) Both 1 and 2 (d) Neither 1 nor 2
(d) Wimbledon-French Open-US Open Ans. (a) : Established in 1996, the 'Orange Award' is
Ans. (a) : Starting with the Australian Open Lawn the world's prestigious award given to a female writer.
Tennis Tournament, the correct chronology of the other The 2007 award has been given to Nigerian novelist
three major lawn tennis tournaments is - French Open - Chimamanda Ngojo. The Pulitzer Prize is awarded by
US Open - Wimbledon. The Australian Open took Columbia University in the field of journalism and
place in Melbourne in the last week of January 2008. literature in America since 1917 in memory of Joseph
Novak Djokovic of Serbia won the men's singles title Pulitzer, the publisher of 'New York World'. Pulitzer
while Maria Sharapova of Russia captured the women's was from Hungary and moved to America when he was
singles title. Novak is the first man from Serbia to win young. Awards are given for general reporting, national
a Grand Slam title. reporting, feature writing, poetry, autobiography,
114. Who among the following has been the Captain fiction, drama, history, special local reporting,
of the India team in Cricket Test Matches for international reporting and public service.
the maximum number of times?
(a) Sunil Gavaskar MISCELLANEOUS
(b) Mansur Ali Khan Pataudi
118. How much is one barrel of oil approximately
(c) Saurav Ganguly equal to?
(d) Mohammad Azharuddin (a) 131 litres (b) 159 litres
Ans. (c) : Saurav Ganguly became the captain of the (c) 257 litres (d) 321 litres
Indian team for the most number of times in cricket test Ans. (b) : One barrel of crude oil consist of approx 159
matches. Litres.
115. Which one of the following countries won the 119. Among the following, which one has the
Euro Football Tournament, 2004 held in minimum population on the basis of data of
Portugal? Census o f India, 2001?
(a) Italy (b) France (a) Chandigarh (b) Mizoram
(c) Greece (d) Portugal (c) Puducherry (d) Sikkim
Ans. (c) : Euro Football Tournament, 2004 was held in Ans. (d) : According to census report 2001. Population
Portugal between 12 June - 4 July 2004. On 4 July of Sikkim was 54,0851. Population of Chandigarh was
2004, Greece (Greece) won the Euro Cup by defeating 901,000, Population of Pondicherry was 974000 and
hosts Portugal 1–0 in the final held at the Stadium of population of Mizoram was 889,000.
Light in the Portuguese capital, Lisbon. Greece 120. What is the approximate percentage of persons
Theodoras Jagorakis was voted the best player of the above 65 years of age in India's current
Euro Cup, 2004. captain of population?

IAS (Pre) GS 2008 Paper I 261 YCT


(a) 14-15% (b) 11-12% 125. Consider the following statements:
(c) 8-9% (d) 5-6% 1. Ajman is one of the seven Emirates of the
Ans. (c) : When question was asked, percentage of UAE.
population of persons above 65 years was 8-9% but 2. Ras al-Khaimah was the last Sheikhdon to
according to the 2011 census, the present population of join the UAE.
India is about 5-6% of the total population of persons
Which of the statements given above is/are
above 65 years of age. In this view, option (d) of the
answer will be correct. correct?
121. For India, China, the UK and the USA, which (a) 1 only (b) 2 only
one of the following is the correct sequence of (c) Both 1 and 2 (d) Neither 1 nor 2
the median age of their populations? Ans. (c) : Ajman is one of the seven emirates
(a) China < India < UK < USA constituting the United Arab Emirates with an area of
(b) India < China < USA < UK 260 sq kilometer. Ras al-Khaimah was the 7th and last
(c) China < India < USA < UK Sheikhdom to join UAE in 1972.
(d) India < China < UK < USA 126. Which of the following pairs is/are correctly
Ans. (b) : India < China < USA < UK matched?
122. Which of the following are among the million- 1. Francis Collins : Mapping human genome
plus cities in India on the basis of data of the
2. Sergey Brin : Google Search Engine
Census, 2001?
1. Ludhiana 2. Kochi 3. Jimmy Wales : Wikipedia
3. Surat 4. Nagpur Select the correct answer using the code given
Select the correct answer using the code given below:
below: (a) 1 and 2 only (b) 2 and 3 only
Code: (c) 3 only (d) 1, 2 and 3
(a) 1, 2 and 3 only (b) 2, 3 and 4 only Ans. (d) : Francis Sellers Collins is an American
(c) 1 and 4 only (d) 1, 2, 3 and 4 physiciangeneticist noted for his discoveries of disease
Ans. (d) : According to the Census of 2001 of India, the genes and his leadership of the Human Genome Project.
population of Ludhiana is 1,398,467, Surat is 2,433,835 Sergey Mikhaylovich Brin is an American
and Nagpur is 2,052,066. computer scientist and internet entrepreneur who,
123. Which two countries follow China and India in together with Larry Page, co-founded Google, one of
the decreasing order of their populations? the world's most profitable Internet companies. Jimmy
(a) Brazil and USA (b) USA and Indonesia Donal "Jimbo" Wales is an American Internet
(c) Canada and Malaysia (d) Russia and Nigeria entrepreneur best known as the co-founder and
Ans. (b) : 10 largest nations in terms of population - promoter of the online non-profit encyclopedia
China (1304.2 million), India (1065.5 million), U.S.A.
Wikipedia.
(294.0 million), Indonesia (219.9 million), Brazil (178.5
million), Pakistan (153.6 million), Bangladesh (146.7 127. Elizabeth Hawley is well-known for her
million). The Russian Federation (143.2 million), Japan writings relating to which one of the following?
(127.7 million) and Nigeria (124.0 million). (a) Historical monuments in India
124. Amongst the following States, which one has (b) Regional dances in India
the highest percentage of rural population to its (c) Himalayan expeditions
total population (on the basis of the Census, (d) Wildlife in India
2001)?
(a) Himachal Pradesh Ans. (c) : Elizabeth Hawley is known for her writings
(b) Bihar related to the expedition to the Himalayas.
(c) Orissa 128. Match List-I with List-II and select the correct
(d) Uttar Pradesh answer using the code given below the Lists:
Ans. (a) : According to Census, 2001, the percentage of List-I (Person) List-II (Area)
rural population in the total population of Himachal A. Sabyasachi Mukherjee 1. Microfinance
Pradesh is highest. The total population of Himachal loans
Pradesh is 6,077,900 and its rural population is
B. Aniruddha Bahl 2. Pharmaceuticals
5,482,319 (90.2%), while the percentage of rural
population in Bihar is 89.5, in Orissa 85.0 percent of C. Vikram Akula 3. Fashion designing
rural population and in Uttar Pradesh is 79.2 percent of D. Yusuf Hamid 4. Investigative
rural population. journalism

IAS (Pre) GS 2008 Paper I 262 YCT


Code: Ans. (c) : The Yom Kippur War, also known as the
A B C D 1973 Arab-Israeli War, was a war fought by the
(a) 1 2 3 4 coalition of Arab states led by Egypt and Syria against
(b) 3 4 1 2 Israel from October 6 to 25, 1973.
(c) 1 4 3 2 Directions: Each of the following four (4) items
(d) 3 2 1 4 consists of two statements, one labelled as
Ans. (b) : Sabyasachi Mukherjee is related to Fashion 'Assertion (A)' and the other as 'Reason (R)'.
Designing. Aniruddha Bahl is related to Investigative You are to examine these tow statements
Journalism. Vikram Akula is related to Microfinance carefully and select the answers to these items
loans and Yusuf Hamid is related to Pharmaceuticals. using the codes given below.
Codes:
129. Bimbavati Devi is a well-known dance of which
type of dance? (a) Both A and R are individually true and R is
the correct explanation of A.
(a) Manipuri (b) Bharat Natyam
(b) Both A and R are individually true but R is
(c) Kuchipudi (d) Odissi
not the correct explanation of A
Ans. (a) : Bimbavati Devi is famous Manipuri dance.
(c) A is true but R is false
Jhaveri sisters Rita Devi, Sabita Mehta, Nirmala Mehta
and Thumbal Yaima are also famous Manipuri dancers. (d) A is false but R is truly
The basis of Manipuri dance is the ancient tribal dance 132. Assertion (A) : In human body, liver has an
of Manipur, but a special object of Vaishnavism. It got important role in fat digestion.
painted in the colours of classical tradition. Yamani Reason (R) : Liver produces two important
Krishna Murthy, Sonal Man Singh, Padma fat-digesting enzymes.
Subrahmanyam and Leela Samson are famous dancers Ans. (c) : The liver is the largest gland in the human
of Bharat Natyam dance. body, which is located in the upper part of the
130. Match List-I with List-II and select the correct abdominal cavity on the right side, its weight is about
answer using the code given below the Lists: 1.5-2 kg and it is dark grey in colour. It is divided into
List-I (Person) List-II (Area) two sections by a deep trough. In its lower part there is
A. K.P. Singh 1. Banking a small pear-shaped sac called gall bladder. The bile
secreted by the liver is stored in this sac. This bile
B. Vikram Pandit 2. Fiction writing
intensifies the action of enzymes present in the
C. Roopa Farooki 3. Retail Business
intestine. Apart from this, the liver takes an active part
D. Kishore Biyani 4. Real estate in carbohydrate, fat and protein metabolism and protects
Code: it by neutralizing the toxins produced in the body.
A B C D 133. Assertion (A) : In the visible spectrum of light,
(a) 4 2 1 3 red light is more energetic than
(b) 3 1 2 4
green light.
(c) 3 2 1 4
Reason (R) : The wavelength of red light is
(d) 4 1 2 3 more than that of green light.
Ans. (d) : K. P. Singh is related to the real estate sector; Ans. (d) : When sunlight passes through a prism, it
Vikram Pandit is related to the banking sector, recently.
bends towards the base of the prism after refraction and
He has been appointed as the Chief Executive Officer
splits into light of different colours. The group of colours
(CEO) of Citigroup.No one has reached this pinnacle in
the Indian financial sector before Vikram Pandit,a obtained in this way is called spectrum and the process
native of Nagpur, Maharashtra; Roopa Farooqui is of splitting of light into such constituent colours is
related to fiction writing; Kishore Biyani is related to called dispersion. Among the colours received from
the field of retail business. sunlight, the violet colour is at the bottom towards the
131. Yom Kippur War was fought between which base and the red colour is at the top i.e. the deflection of
sides/countries? violet is the highest and the deflection of red is the least.
(a) Turkey and Greece The order of the different colours from base to top is as
(b) Serbs and Croats
follows - Violet, Indigo, Blue, Green, Yellow, Orange,
(c) Israel and Arab countries led by Egypt and
Syria and Red. Violet light has the shortest wavelength and
(d) Iran and Iraq red light has the longest wavelength.

IAS (Pre) GS 2008 Paper I 263 YCT


134. Assertion (A) : Radio waves bend in a 137. Norman Ernest Borlaug who is regarded as the
magnetic field. father of the Green Revolution in India is from
Reason (R) : Radio waves are electro which country?
magnetic (a) United States of America
Ans. (a) : Assertion is false and reason is correct. (b) Mexico
Radio waves are electromagnetic in nature. Therefore it
(c) Australia
is unaffected by both magnetic and electric fields.
135. Assertion (A) : The Congress Ministries in all (d) New Zealand
the provinces resigned in the Ans. (a) : Norman Ernest Borlaug, who is considered
year 1939. the father of India's Green Revolution, belongs to the
Reason (R) : The Congress did not accept United States of America.The 'Green Revolution' started
the decision of the viceroy to in India in the mid-1960s in the field of agriculture,
declare war against Germany which resulted in self-sufficiency in food, thanks to
in the Context of the Second Nobel Prize-winning agricultural scientist Norman Borlaug
World War. and Indian agricultural scientist Dr. M. S. Swaminathan.
Ans. (a) : World War II started on September 3, 1939,
and on the same day then Viceroy Linlithgow, without 138.
consulting the Indian people, declared India one side of
the war and warring against Germany. Soon after,
emergency laws were enacted through the British
Parliament and the Indian Legislature, which curtailed
the autonomy of provincial governments and imposed
restrictions on the civil rights of the people. The
Congress was not opposed to giving its support and
support during the war, but instead demanded that
In how many different ways can all of 5
"India should be declared an independent nation" and
during the war a real representative government of the identical balls be placed in the cells shown
people should be established at the center. Wanted that above such that each row contains at least 1
the matter of war and peace for India should be decided ball?
by the Indian people, but due to the British government (a) 64 (b) 81
disagreeing with this idea of the Congress, the Congress (c) 84 (d) 108
ministries resigned in all the provinces. Ans. (d) : Method of placing a ball in the
136. Match List-I with List-II and select the correct first row = 3C1x3C3
answer using the code given below the Lists:
Method of placing a ball in all the rows including the
List-I List-II
other two rows = 3C3x3C1 x 3C1 x 3C = 27 ways.
(Service/Agency) (Country)
Ways of placing the ball in the remaining
A. Foreign Intelligence Service 1. Israel
two rows = 3C1x3C1x3C2x3C , = 81 ways
B. Ministry of State Security 2. Britain
So, Total number of balls placed according to the
C. Secret Intelligence Service 3.China
question = 81 +27 = 108.
D. The Mossad 4. Russia
Code: 139.
A B C D
(a) 4 1 2 3
(b) 2 3 4 1 How many different triangles are there in the
(c) 4 3 2 1 figure a shown above?
(d) 2 1 4 3 (a) 28 (b) 24
Ans. (c) : The Foreign Intelligence Service is Russia's (c) 20 (d) 16
external intelligence agency, mainly for civilian affairs.
Ans. (a) : The total number of triangles in the above
The Ministry of State Security (MSS) is the intelligence
figure is 28.
agency and security agency of the People's Republic of
China. The Secret Intelligence Service(commonly 140. There are 6 different letters and 6
known as MI6) is the British intelligence agency which correspondingly addressed envelopes. If the
supplies the British Government with foreign letters are randomly put in the envelopes, what
intelligence. Mossad is the national intelligence agency is the probability that exactly 5 letters go into
of Israel. the correctly addressed envelopes?
IAS (Pre) GS 2008 Paper I 264 YCT
(a) Zero (b) 1/6 144. What is the number of terms in the series 117,
(c) 1/2 (d) 5/6 120, 123, 126, ......., 333?
Ans. (a) : As there are 6 letters and envelopes, so if (a) 72 (b) 73
exactly 5 are into correctly addressed envelopes, then
the remaining 1 will automatically be placed in the (c) 76 (d) 79
correctly addressed envelope. Thus, the probability that
Ans. (b) : 117, 120, 123, 126, ........., 333
exactly 5 go into the correctly addressed envelope is
zero. Given series is an A.P series with first term, a =
117, last term l =333 and common difference, d = 3
141.
last term, l = a + (n – 1)d
There are two identical red, two identical black
where, n = number of terms.
and two identical white balls. In how many
117 + (n – 1)3 = 333
different ways can the balls be placed in the
cells (each cell to contain one ball) shown above (n – 1)3 = 216
such that balls of the same colour do not n = 73
occupy any two consecutive cells? 145. In how many different ways can four books A,
(a) 15 (b) 18 B, C and D be arranged one above another in a
(c) 24 (d) 30 vertical order such that the books A and B are
Ans. (c) : Let us start with Red colour where, R = Red, never in continuous position?
B = Black, W = White. There are eight such (a) 9 (b) 12
arrangements, if we start with Red ball similarly, there (c) 14 (d) 18
are 8 arrangements, if we start with Black or white ball.
Ans. (b) : Books A , B , C and D can be arranged in
Hence No. of arrangements = 8 + 8 + 8 = 24.
vertical order according to the method given in the
142. Which one of the following is the correct question in the following way -
sequence in respect of the Roman numerals C,
ACDB BCDA
D, L and M?
ADCB BDCA
(a) C > D > L > M (b) M > L > D > C
ACBD BCAD
(c) M > D > C > L (d) l > C > D > M
ADBC BDAC
Ans. (c) : Roman numerals are those used by the
ancient Romans. These numbers are letters that were CADB DACB
converted to numbers. Some Roman numerals are as CBDA DBCA
follows - So in total 12 ways can be arranged. Other methods
L = 50, C = 100, D = 500 , M = 1000 So the correct (i) There are 4 books, and we can arrange these 4 books
sequence of the given Roman numerals C , D , L , M is in 4! ways.
- M > D > C > L.
(ii) If we consider book A & B always together then we
143. In the series AABABCABCDABCDE can arrange the books is 3! × 2 ways.
.......which letter occupies the 100th position?
(iii) If we house to find out the ways of arrangement in
(a) H (b) I
which books A & B never be together then.
(c) J (d) K
Arrangement of all 4 book-arrangement of if A & B

Ans. (b) : always be together.

100th word = 4! – 3! × 2

1+2+3+4+5+6+7+8+9+10+11+12+13+[ 9] = 4 × 3 × 2 × 1 – 3×2×2
It means 100th word = 9th word of alphabet= I. = 24–12 = 12 ways.

IAS (Pre) GS 2008 Paper I 265 YCT


146. Carpenter A can make a chair in 6 hours, (a) 600 (b) 580
carpenter B in 7 hours and carpenter C in 8
(c) 560 (d) 540
hours. If each carpenter works for 8 hours per
day, how many chairs will be made in 21 days? Ans. (a) : Percentage of total students who failed in the
(a) 61 (b) 67 examination = 15 + 15 + 25 = 55 %
(c) 73 (d) 79
In addition, percentage of students who passed the total
Ans. (c) : Chair made by carpenter A in 1 hour = 8/6
Chair made by carpenter B in 1 hour 8/7 = 100 - 55 = 45
Chair made by carpenter C in 1 hour = 8/8 = 1 Total number of students assuming X
So total chairs made by carpenters A, B, C in 21 days
X x 45/100 = 270
= 21x 8/6 + 21x 8 /7 +21x 1 = 73
X = 270 x 100/ 45 = 60
147. A person purchases 100 pens at a discount of
10%. The net amount of money spent by the Total number of students on this basis = 60 x 10 = 600
person to purchase the pens is Rs. 600. The Failed percentage
selling expenses incurred by the person are
15% on the net cost price. What should be the
selling prices for 100 pens in order to earn a
profit of 25%?
(a) Rs. 802.50 (b) Rs. 811.25
(c) Rs. 862.50 (d) Rs. 875 So total failed percentage of student
Ans. (c) : Buys 100 pens at 10% discount, = 15 + 15 + 25 = 55%
Net expenditure on purchase of pens = 600 Hence student percentage who passed in both
Selling expenses = 15% of cost price = 15% of 600 = paper = 100–55 = 45%
Rs 90.
45% → 270 then total student
:: . Total expenditure = Rs.690.
Profit = 25% 270
= ×100 = 600
:: . 25% of 690 = 172.50 45

: . Selling price = 690 + 172.50 = Rs 862.50. 150. March 1, 2008 was Saturday. Which day was it
148. A school teacher has to select the maximum on March 1, 2002?
possible number of different groups of 3 (a) Thursday (b) Friday
students out of a total of 6 students. In how
(c) Saturday (d) Sunday
many groups any particular student will be
included? Ans. (b) : In a year, number of weeks
(a) 6 (b) 8 = 52 extra day = 1
(c) 10 (d) 12 From 2002 to 2008, there are 6 years.
Ans. (c) : Suppose any particular student is always So number of extra days = 6 (1) = 6
selected. Now, remaining 2 students are to be selected
While 2004 and 2008 are leap years,
out of the remaining 5 students.
having one more extra day apart from the normal extra
It can be done in 5C2 ways.
day.
= 5!/ 2! x 3! = 10
149. In an examination, 70% of the students passed Thus, number of extra days = 6 + 1 + 1 = 8
in the Paper I, and 60% of the students passed Out of these 8 extra days, 7 days form a week and so 1
in the Paper II. 15% of the students failed in day remains.
both the papers while 270 students passed in
Hence, March 1, 2002 is 1 day less then March 1, 2008
both the papers. What is the total number of
i.e., it is Friday.
students?
IAS (Pre) GS 2008 Paper I 266 YCT
UNION PUBLIC SERVICE COMMISSION
Civil Services (Preliminary Exam) - 2007
GENERAL STUDIES : PAPER-I
Time: 2 hours Maximum Number: 200
(a) 1 and 2 (b) 3 only
MIEDIEVAL HISTORY (c) 1 only (d) 1 and 3
1. Where is the famous Vijaya Vittala temple Ans. (d) : Terms of Jawaharlal Nehru were 1947-52;
having its 56 carved pillars emitting musical 1952-57; 1957-62; 1962-64. He died in 1964 during his
notes locates? 4th Prime Ministerial term. He represented Phulpur
(a) Belur (b) Bhadrachalam constituency in UP. The first non-congress Prime
(c) Hampi (d) Srirangam Minister of India was Morarji Desai (Janta Party) from
Ans. (c) : Vijaya Vittala temple is located at Hampi, the1977-1979. Hence, statements 1 and 3 are correct.
capital of Vijayanagar kingdom during the reign of 5. Which one of the following was the first fort
Krishnadev Raya. constructed by the British in India?
2. Who among the following wrote the book (a) Fort William (b) Fort St. George
Bahubivah? (c) Fort St. David (d) Fort. St. Angelo
(a) Raja Rammohan Roy
Ans. (b) : In 1639, Damarella Venkatappa, a feudatory
(b) Ishwar Chandra Vidyasagar
of Vijayanagara ruler Venkata II, allowed the
(c) Pandita Ramabai Englishman Francis Day to build a fort in Madras. This
(d) Rabindranath Tagore fort later came to be known as Fort St. George.
Ans. (b) : Ishwar Chandra Vidyasagar wrote the book In 1698 AD, the British got the zamindari of Sutanati,
Bahubivah. It was written in protest of the evil of Kalita and Govindpur in exchange for 1200 rupees. By
polygamy. Bahu (many or more than one)- bivah combining these three, a fortified factory named Fort
(marriage) meaning more than one marriage. William was formed. Sir Charles Eyre became the first
3. The First Factory Act restricting the working head of this Fort William.
hours of women and children, and authorizing
6. Consider the following statements:
local governments to make necessary rules was
1. Robert Clive was the first Governor-General
adopted during whose time?
of Bengal.
(a) Lord Lytton
2. William Bentinck was the first Governor-
(b) Lord Bentinck
General of India.
(c) Lord Ripon
Which of the statements given above is/are
(d) Lord Canning
correct?
Ans. (c) : The First Factory Act was passed in 1881
(a) 1 only (b) 2 only
during the time of Lord Ripon. It was passed to improve
(c) Both 1 and 2 (d) Neither 1 nor 2
the service condition of the factory workers in India. It
fixed the working hours for women and children above Ans. (b) : Statement 1 is incorrect as Warren Hastings
the age of seven years. was the first governor-general of Bengal.
4. Consider the following statements: 7. Who among the following Europeans were the
1. Jawaharlal Nehru was in his fourth term last to come to pre-independence India as
as the Prime Minister of India at the time traders?
of this his death. (a) Dutch (b) English
2. Jawaharlal Nehru was represented by Rae (c) French (d) Portuguese
Bareli constituency as a Member of
Ans. (c) : Following the Portuguese, English, and
Parliament.
Dutch, the French also established trading bases in
3. The first non-Congress Prime Minister of
India assumed the Office in the year 1977. India. Pondicherry on the Coromandel Coast was the
Which of the statements given above is/are first establishment of French power in south-eastern
correct? India in 1674 CE.
IAS (Pre) GS 2007 Paper I 267 YCT
8. The ruler of which one of the following states Ans. (d) : The song Amar Sonar Bangla was written in
as has removed from power by the British on 1905 by Rabindranath Tagore. He is the only person to
the pretext of misgovernance? write National Anthem of two countries. i.e., India and
(a) Awadh (b) Jhansi Bangladesh.
(c) Nagpur (d) Satara
13. At which one of the following places did
Ans. (a) : Lord Dalhousie took over Awadh in 1856 on Mahatma Gandhi first start his Satyagraha in
charges of misgovernance. He merged Jhansi, Satara
India?
and Nagpur as well as Jaitpur, Sambalpur, Baghat,
Udaipur, under the Doctrine of Lapse. (a) Ahmedabad (b) Bardoli
9. Who among the following started the (c) Champaran (d) Kheda
newspaper Shome Prakash? Ans. (c) : Mahatma Gandhi returned to India from
(a) Dayanand Saraswati South Africa on January 9, 1915. In 1917, he started
(b) Ishwar Chandra Vidyasagar the first Satyagraha movement from Champaran.
(c) Raja Rammohan Roy Gandhiji had gone there on the request of Rajkumar
Shukla of Champaran.
(d) Surendranath Banerjee
After Champaran, Gandhi ended the dispute between
Ans. (b) : Newspapers were an important medium for
the mill owners and workers in Ahmedabad and then
the dissemination of information on various problems. It
led the peasant movement in Kheda.
is great help in the campaigns for social reform and to
influence activities of the state. The first newspaper in 14. Which one the following aroused a wave of
India was the Bengal-Gazette started in 1780. Ishwar popular indignation that led to the massacre by
Chandra Vidyasagar started Shome Prakash in 1890. the British at Jallianwala Bagh?
(a) The Arms Act
10. Who wrote the book- "The Story of the
(b) The Public Safety Act
Integration of the Indian States"?
(c) The Rowlatt Act
(a) B. N. Rau
(d) The Vernacular Press Act
(b) C. Rajagopalachari
Ans. (c) : Jallianwala Bagh massacre took place on 13
(c) Krishna Menon
April, 1919 during anti-Rowlatt agitation in Amritsar.
(d) V.P. Menon
Rowlatt Act was passed in March 1919 which
Ans. (d) : V. P Menon authored a book on the political authorized the government to imprison any person
integration of India, "The Story of the Integration of without trial and conviction in a court of law. Tagore
Indian States" and on the partition of India, "Transfer of renounced his Knighthood in protest against Jallianwala
Power". Bagh massacre.

INDIAN NATIONAL MOVEMENT 15. Assertion (A) : According to the Wavell Plan,
the number of Hindu and
11. Which one of the following places was Muslim members in the
associated with Acharya Vinoba Bhave's Executive Council were to be
Bhoodan Movement at the beginning of the equal.
movement? Reason (R) : Wavell thought that this
(a) Udaygiri (b) Rapur arrangement would have avoided
(c) Pochampalli (d) Venkatagiri the partition of India.
Ans. (c) : Bhoodan Movement was a voluntary land (a) Both A and B are true and R is the correct
reform movement in India started by Acharya Vinoba explanation of A
Bhave on April 18, 1951 started at Pochampalli village, (b) Both A and R all true, but R is not correct
in Telangana (Andhra Pradesh). explanation of A
(c) A is true, but R is false
12. The song 'Amar Sonar Bangla' written during
the Swadeshi Movement of India inspired the (d) A is false, but R is true
liberation struggle of Bangladesh and adopted Ans. (c) : In October 1943, the British Government
as the National Anthem of Bangladesh. Who decided to replace Lord Linlithgow with Lord Wavell as
wrote this song? the Viceroy of India. Right after assuming charge as
(a) Rajni Kanta Sen Viceroy, Wavell’s most important task was to present a
(b) Dwijendra Lal Roy formula for the solution of the Indian problem which
(c) Mukunda Das was acceptable for both the Congress and the Muslim
(d) Rabindranath Tagore League.
IAS (Pre) GS 2007 Paper I 268 YCT
INDIAN GEOGRAPHY Ans. (b) : The correct match of Aluminium company
and that location.
16. Where are Shevaroy Hills located? BALCO Korba
(a) Andhra Pradesh HINDALCO Renukoot
(b) Karnataka Indian Aluminium Hirakund
(c) Kerala NALCO Koraput
(d) Tamil Nadu 20. Which one of the following is located in the
Ans. (d) : Shevaroy Hills are located in Tamil Nadu. Bastar region?
The Servarayan hills, with the anglicised name (a) Bandhavgarh National Park
Shevaroy Hills, are a towering mountain range near the (b) Dandeli Sanctuary
city of Salem, in Tamil Nadu. It is one of the major hill (c) Rajaji National Park
stations in Tamil Nadu and in the Eastern Ghats. The (d) Indravati National Park
local Tamil name comes from a local deity, Servarayan.
Ans. (d) : Indravati National Park is located in the
17. Which one of the following is the correct Bastar region. Indravati National Park is a national park
sequence in the decreasing order of production located in Bijapur district of Chhattisgarh. It derives its
(in million tonnes) of the given food grains in name from the nearby Indravati River.
India?
21. Out of the four southern States : Andhra
(a) Wheat-Rice-Pulses-Coarse cereals
Pradesh, Karnataka, Kerala and Tamil Nadu,
(b) Rice-Wheat-Pulses-Coarse cereals which share boundaries with the maximum
(c) Wheat-Rice-Coarse cereals-Pulses number of India States?
(d) Rice-Wheat-Coarse cereals-Pulses (a) Andhra Pradesh only
Ans. (d) : Production of rice is 96.43 MT, wheat is 78.4 (b) Karnataka only
MT, pulses is 15.11 MT and coarse cereals are 40.73 (c) Each of Andhra Pradesh and Karnataka
MT in India. So, the correct sequence in decreasing (d) Each of Tamil Nadu and Kerala
order is Rice – Wheat – Coarse cereals – Pulses(2007).
Ans. (c) : When commission asked question both state
18. Which one of the following is also known as top Andra Pradesh & Karnataka shares its boundary with
Slip? five state. Hence option (c) is correct.
(a) Simlipal National Park Note : According to new data among the southern
(b) Periyar Wildlife Sanctuary states; Karnataka shares its boundary with 6 states
(c) Manjira Wildlife Sanctuary namely Andhra Pradesh, Telangana, Goa, Maharashtra,
(d) Indira Gandhi Wildlife Sanctuary and Kerala and Tamil Nadu.
National Park Whereas Andhra Pradesh shares its boundary with 5
states.
Ans. (d) : Indira Gandhi Wildlife Sanctuary and
National Park is also known as Top Slip. It is located in 22. Dalbergia species is associated with which one
of the following?
Annamalai hills of Tamil Nadu.
(a) Cashew nut (b) Coffee
19. Match List-I with List-II and select the correct
(c) Tea (d) Rosewood
answer using the codes given below the Lists:
Ans. (d) : Rosewood is associated with Dalbergia
List-I List-II
species. Dalbergia species are used as food plants and
(Aluminium Company) (Location)
fragrant wood in aromatic oils.
A. BALCO 1. Hirakund
23. Which one among the following major Indian
B. HINDALCO 2. Korba
cities is most eastward located?
C. Indian Aluminium 3. Koraput
(a) Hyderabad
Company
(b) Bhopal
D. NALCO 4. Renukoot (c) Lucknow
Code: (d) Benguluru (Bangalore)
A B C D Ans. (c) : Among the given options, Lucknow is most
(a) 3 1 4 2 eastward located. Lucknow, the capital of Uttar
(b) 2 4 1 3 Pradesh. It is home to extraordinary monuments
(c) 3 4 1 2 depicting a fascinating blend of ancient, colonial and
(d) 2 1 4 3 oriental architecture.

IAS (Pre) GS 2007 Paper I 269 YCT


24. Which one of the following rivers originates at 29. Shahgarh area in Jaisalmer district of
Amarkantak? Rajasthan was in news in the year 2006 because
(a) Damodar (b) Mahanadi of which one of the following?
(c) Narmada (d) Tapti (a) Finding high quality gas reserves
Ans. (c) : The Narmada River originates from west of (b) Finding uranium deposits
Amarkantak plateau in Madhya Pradesh. River (c) Finding zinc deposits
Damodar originates from Chhota Nagpur plateau,
(d) Installation of wind power units
Mahanadi originates from Bastar plateau and Tapti
Originates from Satpura hills. Ans. (a) : High quality gas reserves have been found at
25. Which one among the following States of India a depth of 3,161 metres in an exploratory well drilled in
has the lowest density of population? Shahgarh block in Jaisalmer district of Rajasthan. The
(a) Himachal Pradesh (b) Meghalaya gas was estimated to have 88 to 91 per cent
(c) Arunachal Pradesh (d) Sikkim hydrocarbon.
Ans. (c) : Population density of Arunachal Pradesh is 30. Consider the following statements:
13, Himachal Pradesh is 109, Meghalaya is 103, and 1. In India, Red Panda is naturally found in the
Sikkim is 76. Western Himalayas only.
26. Consider the following statements: 2. In India, Slow Loris lives in the dense forests
1. Balaghat is known for its diamond mines. of the North East.
2. Majhgawan is known for its manganese Which of the statements given above is/are
deposits. correct?
Which of the statements given above is/are (a) 1 only (b) 2 only
correct? (c) Both 1 and 2 (d) Neither 1 nor 2
(a) 1 only (b) 2 only Ans. (b) : Red Panda in India is found in Sikkim,
(c) Both 1 and 2 (d) Neither 1 nor 2 western Arunachal Pradesh, Darjeeling district of West
Ans. (d) : Both the statements are incorrect. Balaghat of Bengal and parts of Meghalaya. It is also the state
Madhya Pradesh is known for its manganese mines. animal of Sikkim. Hence statement 1 is not correct. In
Majhgawan is famous for its diamond mines. India, Slow Loris lives in the dense forests of the North
East. The Bengal slow loris (Nycticebus bengalensis) or
27. Which one of the following National Highways
northern slow loris is a strepsirrhine primate and a
passes through Maharashtra, Chhattisgarh and
species of slow loris native to the Indian subcontinent
Orissa?
and Indo-China.
(a) NH4 (b) NH5
31. Match List-I with List-II and select the correct
(c) NH6 (d) NH7
answer using the codes given below the Lists:
Ans. (c) : NH6 passes through Maharashtra,
List-I List-II
Chhattisgarh and Orissa. NH6 passes through Gujarat,
Maharashtra, Chhattisgarh, Odisha, Jharkhand and West (Town) (River Nearer to it)
Bengal state in India. A. Betul 1. Indrawati
28. With reference to the steel industry in India in B. Jagdalpur 2. Narmada
the recent times, consider the following C. Jabalpur 3. Shipra
statements: D. Ujjain 4. Tapti
1. Vizag Steel Plant (RINL) has been declared Code:
Mini Ratna. A B C D
2. Merger of IISCO with SAIL has been (a) 1 4 2 3
completed.
(b) 4 1 2 3
Which of the statements given above is/are
(c) 4 1 3 2
correct?
(d) 1 4 3 2
(a) 1 only (b) 2 only
(c) Both 1 and 2 (d) Neither 1 nor 2 Ans. (b) : Betul, a town in Madhya Pradesh is situated
on the banks of river Tapti. Jagdalpur is a town in
Ans. (c) : Vizag Steel Plant was declared as a Mini
Ratna company on 11th Feb 2006. Merger of IISCO Chhattisgarh on the banks of the river Indravati.
with SAIL has been completed on 29th September Jabalpur is on the sides of the river Narmada and Ujjain
2004. is situated along the banks of the river Shipra.

IAS (Pre) GS 2007 Paper I 270 YCT


32. In which State is the Guru Shikhar Peak Ans. (d) : Greenwich (0° longitude) passes through
located? London. Lisbon (Portugal) and Accra (Ghana) lie near
(a) Rajasthan (b) Gujarat the Greenwich Line whereas Addis Ababa (Ethiopia) is
(c) Madhya Pradesh (d) Maharashtra located at 38.42° East longitude. Therefore, the time of
Ans. (a) : The Guru Shikhar Peak is located in Addis Ababa will not be the same as that of the above
Rajasthan. Guru Shikar is located at a height of 1772 three cities.
meters above sea level, and one can enjoy the view of 36. Which one among the following rivers is the
Mount Abu region from this peak. It is 15 km from longest?
Mount Abu and a road from there leads almost to the (a) Amazon (b) Amur
top of the mountain. (c) Congo (d) Lena
33. The largest coral reef in the world is found Ans. (a) : The Amazon is considered as the world's
near the coast of which one of the following largest river by volume, but scientists have believed it is
countries? slightly shorter than Africa's Nile. The Brazilian
(a) Australia (b) Cuba scientists' 14-day expedition extended the Amazon's
(c) Ghana (d) Philippines length by about 176 miles (284 kilometers), making it
Ans. (a) : The largest coral reef in the world is found 65 miles (105 kilometers) longer than the Nile.
near the coast of Australia. The Great Barrier Reef is Nile (4132 miles) is the longest river in the world
the world's largest coral reef system composed of over followed by Amazon (4000 miles) and Yangtze (3915
2,900 individual reefs and 900 islands stretching for miles).
over 2,300 kilometres (1,400 mi) over an area of 37. Consider the following statements:
approximately 344,400 square kilometres (133,000 sq 1. Either of the two belts over the oceans at
mi). The reef is located in the Coral Sea, off the coast of about 30° to 35° N and S Latitudes is known as
Queensland, Australia. Horse Latitude.
34. Assertion (A) : River Kalinadi is an east- 2. Horse latitudes are low pressure belts.
flowing river in the southern Which of the statements given above is/are
part of India. correct.
Reason (R) : The Deccan Plateau is higher (a) 1 only (b) 2 only
along its western edge and (c) Both 1 and 2 (d) Neither 1 nor 2
gently slopes towards the Bay Ans. (a) : Either of the two belts over the oceans at
of Bengal in the east. about 30° to 35° N and S latitudes is known as Horse
(a) Both A and R are individually true and R is Latitude. Horse latitude, either of two subtropical
the correct explanation of A atmospheric high-pressure belts that encircle Earth
(b) Both A and R are individually true but R is around latitudes 30°– 35° N and 30°– 35° S and that
not the correct explanation of A generate light winds and clear skies. Because they
(c) A is true but R is false contain dry subsiding air, they produce arid climates
(high pressure) in the areas below them. The belt
(d) A is false but R is true contain several separate high pressure centers and shift
Ans. (d) : River Kalinadi is a west flowing river in the a few degree away from the equator in summer.
southern part of India. So, the assertion is wrong. 38. Match List-I with List-II and select the correct
Deccan Plateau has elevation ranging from 1,500 to answer using the codes given below the Lists:
2,500 ft and gently slopes towards the Bay of Bengal in List-I List-II
the east. So, the reason is correct. (City) (River)
A. Bangkok 1. Irrawaddy
WORLD GEOGRAPHY B. Phnom-Penh 2. Mekong
C. Hanoi 3. Menam
35. Which one of the following cities does not have D. Yangon 4. Red River
the same clock time as that of the other three Code:
cities at any given instant? A B C D
(a) London (U.K.) (a) 3 2 4 1
(b) Lisbon (Portugal) (b) 4 1 3 2
(c) Accra (Ghana) (c) 3 1 4 2
(d) Addis Ababa (Ethiopia) (d) 4 2 3 1
IAS (Pre) GS 2007 Paper I 271 YCT
Ans. (a) : 43. Assertion (A) : There are no tea plantations in
Bangkok Menam any African country.
Phnom-Penh Mekong Reason (R) : Tea plants need fertile soil
with high humus.
Hanoi Red River
(a) Bothe A and R true and R is the
Yangon Irrawaddy
correct explanation of A
39. Consider the following statements:
(b) Both A and R are true but R is not a
1. The annual range of temperature is greater
in the Pacific Ocean than that in the correct explanation of A
Atlantic Ocean. (c) A is true but R is false
2. The annual range of temperature is greater (d) A is false but R is true
in the Northern Hemisphere than that in the Ans. (d) : In the 20th century, tea is being grown in 3
Southern Hemisphere. African countries; Tanzania, Kenya and Uganda. So, the
Which of the statements given above is/are assertion is wrong. But the reason is correct because tea
correct? plants need highly organic or loamy soils.
(a) 1 only (b) 2 only
(c) Both 1 and 2 (d) Neither 1 nor 2 INDIAN CONSTITUTION AND POLTY
Ans. (b) : The annual range of temperature is greater in
44. Consider the following statements:
the Northern Hemisphere than that in the Southern
1. The mode of removal of a Judge of a High
Hemisphere as there is more land in Northern
Court in India is same as that of removal of
Hemisphere compared to more waters in Southern
a Judge of the Supreme Court.
hemisphere. There is a big difference between the
specific heat of land and water; the loss of heat from the 2. After retirement from the office, a
continents is bigger than the oceans. Therefore annual permanent Judge of a High Court cannot
range of temperature is higher in northern hemisphere. plead or act in any court or before any
The annual range of temperature is greater in the authority in India.
Atlantic Ocean than that in the Pacific Ocean. Which of the statements given above is/are
correct?
Thus only 2 is correct.
(a) 1 only (b) 2 only
40. Through which one of the following Straits, (c) Both 1 and 2 (d) Neither 1 nor 2
does a tunnel connect the United Kingdom and
Ans. (a) : A judge of the Supreme Court or High court
France? is removed by the Process of impeachment. The
(a) Davis Strait (b) Denmark Strait President is authorized to remove the judge from his
(c) Strait of Dover (d) Strait of Gibraltar office only after an address by the Parliament has been
Ans. (c) : Through Strait of Dover, a tunnel connects presented to him in the same session for such removal.
The address must be supported by a special majority of
the United Kingdom and France. Strait of Dover
each house of the Parliament. (Majority of the total
connects United Kingdom and France. It is about 32 km membership of that House and a majority of not less
in length. than two-thirds of the member of that House present
41. Where is Copacabana Beach located? and voting).
(a) Buenos Airs (b) Hawaiian Islands The retired permanent judges of a High court are
(c) Rio de Janerio (d) Valletta prohibited from pleading or acting in any court or
Ans. (c) : Copacabana Beach is located in Rio de before any authority in India except the Supreme Court
Janeiro. Located in Zona Sul, Copacabana Beach is one and other high courts.
of the most famous and most beautiful beaches in the 45. Which one of the following is the correct
world. chronological order of the formation of the
following as full States of the Indian Union?
42. In which one of the following districts, have
(a) Sikkim - Arunachal Pradesh - Nagaland-
large reserves of diamond-bearing kimberlite
Haryana
been discovered in the recent past?
(b) Nagaland – Haryana – Sikkim – Arunachal
(a) Hoshangabad (b) Raipur
Pradesh
(c) Sambalpur (d) Warangal
(c) Sikkim – Haryana – Nagaland – Arunachal
Ans. (b) : Payali and Behradein in Raipur, Chhattisgarh Pradesh
has large reserves of diamond bearing kimberlite. It has (d) Nagaland – Arunachal Pradesh – Sikkim -
28% of the total diamond reserve of India. Haryana
IAS (Pre) GS 2007 Paper I 272 YCT
Ans. (b) : Nagaland-1962, Haryana-1966, Sikkim-1975, Which of the statements given above is/are
Arunachal Pradesh-1986 correct?
46. Who among the following have been the Union (a) 1 only (b) 2 only
Finance Ministers of India? (c) Both 1 and 2 (d) Neither 1 nor 2
1. V.P. Singh Ans. (a) : The members of the Public Accounts
2. R. Venkataraman Committee are elected by the Parliament every year
3. Y. B. Chavan amongst its members according to the principle of
4. Pranab Mukherjee proportional representation by means of the single
Select the correct answer using the code given transferable vote so that all parties get equal
below: representation. The Chairman of the committee is
(a) 1, 2 and 3 only appointed by the speaker from amongst its members.
(b) 1, 3 and 4 only Currently, it consists of 22 members (15 from Lok
(c) 2 and 4 only
Sabha and 7 from Rajya Sabha). Its function broadly is
(d) 1, 2, 3, and 4
to examine public expenditure. Hence only statement 1
Ans. (d) : VP Singh 1985-87, R Venkataraman 1980- is correct.
82, YB Chavan 1971-75, Pranab Mukherjee 1982-85,
2009-12 49. Which of the following Constitution
Amendment Acts seeks that the size of the
47. Consider the following statements:
Councils of Ministers at the Centre and in a
1. The Judges (Inquiry) Bill 2006 contemplates
to establish a Judicial Council which will State must not exceed 15 percent of the total
receive complaints against Judges of the number of members in the Lok Sabha and the
Supreme Court including the Chief Justice total number of members of the Legislative
of India, High Court Chief Justices and Assembly of that State, respectively?
Judges. (a) 91st (b) 93rd
th
2. Under the Protection of Women from (c) 95 (d) 97th
Domestic Violence Act, 2005, a woman can Ans. (a) : In the 91st Constitutional Amendment Act.
file a petition before a 1st Class Judicial 2003, the above provision was added.
Magistrate.
The Constitution (Ninety-First Amendment) Act, 2003
Which of the Statements given above is/are amended the Article 75 of the Constitution by inserting
correct. clause 1(A) which says the total number of Ministers,
(a) 1 only (b) 2 only including the Prime Minister, in the Council of
(c) Both 1 and 2 (d) Neither 1 nor 2 Ministers shall not exceed fifteen percent of the total
Ans. (b) : Statement 1 is incorrect as The Judges number of members of the House of the People (Lok
Inquiry Bill 2006 establishes a National Judicial Sabha).
Council NJC to conduct inquiries into allegations of 50. Consider the following statements in respect of
incapacity or misbehaviour by High Court and Supreme financial emergency under Article 360 of the
Court judges. The proposed NJC would consist of the Constitution of India:
Chief Justice of India, two Supreme Court judges and 1. A Proclamation of financial emergency issued
two High Court Chief Justices to investigate High Court shall cease to operate at the expiration of two
judges or the Chief Justice of India and four Supreme
months, unless before the expiration of the
Court judges to investigate Supreme Court judges. The
period it has been approved by the resolutions
NJC shall investigate complaints submitted by any
of both Houses of Parliament.
person or upon receiving a reference from Parliament
based on a motion moved by 50 Rajya Sabha or 100 2. If any Proclamation of financial emergency is
Lok Sabha MPs. in operation, it is competent for the President
of India to issue directions for the reduction of
48. Consider the following statements:
salaries and allowances of all or any class of
1. The Chairman of the Committee on Public
Accounts is appointed by the Speaker of the persons service connection with the affairs of
Lok Sabha. the Union but excluding the Judges of the
2. The Committee on Public Accounts Supreme Court and the High Court.
comprises Members of Lok Sabha, Which of the statements given above is/are correct?
Members of Rajya Sabha and a few eminent (a) 1 only (b) 2 only
persons of industry and trade. (c) Both 1 and 2 (d) Neither 1 nor 2
IAS (Pre) GS 2007 Paper I 273 YCT
Ans. (a) : Clause 1 of the Article 360 laying down the ECONOMY
provisions of Financial Emergency, it state that the
proclamation of Financial Emergency shall cease to 54. Tarapore Committee was associated with
operate at the expiration of two months, unless before which one of the following?
the expiration of that period it has been approved by (a) Special Economic Zones
resolutions of both Houses of Parliament. (b) Fuller capital account convertibility
The president is empowered to reduce the salaries of all (c) Foreign exchanges reserves
government officials, including judges of the Supreme (d) Effect of oil-prices on the Indian economy
court and High courts, in cases of a financial
Ans. (b) : The Tarapore Committee Report on Fuller
emergency.
Capital Account Convertibility has gone beyond just
51. Who was the Speaker of the First Lok Sabha? recommending differential capital adequacy for banking
(a) Hukam Singh conglomerates. It has suggested that industrial houses
(b) G.V. Mavalankar should be allowed to set-up banks and that non-banking
(c) K.M. Munshi financial companies convert themselves into banks. In
(d) U.N. Dhebar 1997, the first Tarapore Committee on Capital Account
Ans. (b) : G.V. Mavalankar was the speaker of the first Convertibility (CAC) was released. Now, they have
Lok Sabha (15 May, 1952) . released their second report detailing roadmap towards
52. Which one of the following pairs is not bringing free convertibility of rupees.
correctly matched? 55. Who among the following served as the Chief
(a) T.S. Krishanamurthy : Former Chief Election Economist of the International Monetary
Commissioner of India Fund?
(b) K.C. Pant : Chairman, Tenth (a) Ashok Lahri
Finance Commission
(b) Sumantra Ghoshal
of India
(c) A. M. Khurso : Former Chairman, (c) Saumitra Chaudhuri
Union Public Service (d) Raghuram Govinda Rajan
Commission Ans. (d) : Raghuram Govind Rajan is currently the Eric
(d) R.C. Lahoti : Former Chief Justice J. Gleacher Distinguished Service Professor of Finance
of India at the Booth School of Business at the University of
Ans. (c) : A.M. Khusro was Chairman of eleventh Chicago. He was also an honorary economic adviser
Finance Commission of India. to Prime Minister of India Manmohan Singh (appointed
Directions: The following 5 (Five) items consists of 2008). He previously was the chief economist of the
two statements, one labelled as the 'Assertion (A)' and International Monetary Fund and headed a committee
the other as Reason (R)'. You are to examine these appointed by the Planning Commission on financial
two statements carefully and select the answers to reforms in India.
these items using the codes given below:
(a) Both A and R are individually true and R is 56. Participatory Notes (PNs) are associated with
the correct explanation of A which one of the following?
(b) Both A and R are individually true but R is (a) Consolidated Fund of India
not the correct explanation of A (b) Foreign Institutional Investors
(c) A is true but R is false (c) United Nations Development Programme
(d) A is false but R is true
(d) Kyoto Protocol
53. Assertion (A) : The council of Ministers in the
Union of India is collectively Ans. (b) : Participatory Notes (PNs) are instruments
responsible both to the Lok issued by SEBI registered foreign institutions to entities
Sabha and the Rajya Sabha. that want to invest in Indian markets but do not want to
Reason (R) : The Members of both the Lok directly register with the market. This results in
Sabha and the Rajya Sabha are concealment of the investor's identity.
eligible to be the Ministers of
57. The National Housing Bank was set up in India
the Union Government.
as a wholly-owned subsidiary of which one of
Ans. (d) : Assertion is false because the Council of the following?
Ministers in the Union of India is collectively (a) State Bank of India
responsible to the Lok Sabha only. The Members of
both the Lok Sabha and the Rajya Sabha are eligible to (b) Reserve Bank of India
be the Ministers of the Union Government. Hence (c) ICICI Bank
reason is correct. (d) Life Insurance Corporation of India

IAS (Pre) GS 2007 Paper I 274 YCT


Ans. (b) : National Housing Bank was established in July PHYSICS
1988 as a subsidiary of Reserve Bank of India. This bank
is the apex bank for housing finance in the country. 61. What is the average distance (approximate)
58. Which one of the following is the correct between the Sun and the Earth?
sequence in the decreasing order of (a) 70×105 km (b) 100×105 km
6
contribution of difference sectors to the Gross (c) 110×10 km (d) 150×106 km
Domestic Product of India? Ans. (d) : The average distance between the sun and the
(a) Services-Industry-Agriculture earth is 152 × 106 km which is near to 150 × 106 km.
(b) Services-Agriculture-Industry 62. Consider the following statements:
(c) Industry-Services-Agriculture 1. If magenta and yellow coloured circles
(d) Industry-Agriculture-Services intersect, the intersected area will have red
Ans. (a) : The contribution of the given sectors in the colour.
GDP of India in descending order is as follows - 2. If cyan and magenta coloured circles
intersect, the intersected area will have blue
Services, Industry and Agriculture.
colour.
According to the Economic Survey 2006-07, the share Which one of the statements given above is/are
of service sector in GDP is 55.1%, the share of correct?
agriculture sector is 18.5% and the share of industrial (a) 1 only (b) 2 only
sector is 26.4%. (c) both 1 and 2 (d) Neither 1 nor 2
The service sector is the largest sector of India. Gross Ans. (c) : The intersected area of magenta and yellow-
Value Added (GVA) at current prices for the service sector coloured circles will have red colour. The
is estimated at 96.54 lakh crore INR in 2020-21. The intersected area of cyan and magenta coloured circles
service sector accounts for 53.89% of total India's GVA of will have blue colour.
179.15 lakh crore Indian rupees. With GVA of Rs. 46.44 Magenta = Red + Blue
lakh crore, the Industry sector contributes 25.92%. While Cyan = Blue + Green
Agriculture and allied sector share 20.19%. 63. Consider the following statements:
59. Consider the following statements: 1. A flute of smaller length produces waves of
1. The repo rate is the rate at which other lower frequency.
banks borrow from the Reserve Bank of India. 2. Sound travels in rocks in the form of
2. A value of 1 for Gini Coefficient in a country longitudinal elastic waves only.
implies that there is perfectly equal income for Which one of the statements given above is/are
everyone in its population correct?
(a) 1 only (b) 2 only
Which of the statements given above is/are
(c) Both 1 and 2 (d) Neither 1 nor 2
correct?
Ans. (b) : 1. Frequency produced by a string is
(a) 1 only (b) 2 only
inversely proportional to its length.
(c) Both 1 and 2 (d) Neither 1 nor 2 f=2Lv
Ans. (a) : Repo Rate is the rate at which commercial 2. Sound always travels as longitudinal wave through
banks borrow funds from RBI. A reduction in the repo the propagating material.
rate will help banks to get money from the central bank at So only statement 2 is correct.
a cheaper rate. When the repo rate increases borrowing 64. Four wires of same material and of dimensions
from RBI becomes more expensive. A value of (0) for as under are stretched by a load of same
Gini Coefficient in a country implies that there is perfect magnitude separately. Which one of them will
equality in the country. If the value is 1 then there is be elongated maximum?
complete inequality of income in the country. (a) Wire of 1 m length and 2 mm diameter.
60. Basel II relates to which one of the following? (b) Wire of 2 m length and 2 mm diameter.
(a) International standards for safety in civil (c) Wire of 3 m length and 1.5 mm diameter.
aviation. (d) Wire of 1 m length and 1 mm diameter.
(b) Measures against cyber crimes. Ans. (c) : As per Hookes law
(c) Measures against drug abuse by sportspersons. L
∆L ∝ 2 , So ∆L for 1m length and 2 mm dia –
(d) International standards for measuring the D
adequacy of a bank's capital. 1000
= 250mm
Ans. (d) : Basel II is the revised international capital 4
framework. The Basel II framework describes a more 2000
∆L for 2m length and 2mm dia = = 500mm
comprehensive measure of minimum standard for 4
capital adequacy that national supervisory authorities ∆L for 1 m length and 1 mm dia = 1333.33mm
are now working to implement through domestic rule- 1000
= = 1000mm
making and adoption procedures. 1

IAS (Pre) GS 2007 Paper I 275 YCT


65. Which of the following types of light are CHEMISTRY
strongly absorbed by plants?
(a) Violet and orange (b) Blue and red 70. Which one of the following does not contain
(c) Indigo and yellow (d) Yellow and violet
silver?
Ans. (b) : Photosynthesis occurs between wavelengths
(a) Horn silver
of about 400 nm and 750 nm. Red and blue colour
wavelength is categories in this wavelength. So plants (b) German silver
absorb these colours. Photosynthesis does not occur in (c) Ruby silver
the infra-red or in ultraviolet light. (d) Lunar caustic
66. Which of the following types is used by
Ans. (b) : German Silver is an alloy of copper, zinc, and
computed tomography employed for
visualisation of the internal structure of human nickel, sometimes also containing lead and tin. It does
body? not contain silver metal and is extensively used due to
(a) X-rays (b) Sound waves its hardness and resistance to corrosion.
(c) Magnetic resonance (d) Radioisotopes 71. Which one among the following is called
Ans. (a) : Computed tomography is a diagnostic philosopher's wool?
imaging procedure that uses X-rays to build cross-
(a) Zinc bromide
sectional images ("slices") of the body.
CT is based on the fundamental principle that the (b) Zinc nitrate
density of the tissue passed by the X-ray beam can be (c) Zinc oxide
measured from the calculation of the attenuation of X- (d) Zinc chloride
ray intensity. Using this principle, CT allows the
Ans. (c) : Philosophers wool is a oxide of zinc, a white
reconstruction of the density of the body.
powder used as a pigment cosmetics glass inks and in
67. Which one of the following pairs is not
correctly matched? zinc ointment.
(a) William Dickson : Motion Picture film 72. Which one of the following non-metals is not a
(b) Charles Babbage : Programmable computer poor conductor of electricity?
(c) Nicholas Stern : Construction technology (a) Sulphur
(d) Brian Greene : String theory
(b) Selenium
Ans. (c) : Nicholas Stern is a British economist and
academic. (c) Bromine
68. Which one of the following types of glass can (d) Phosphorus
cut off ultraviolet rays? Ans. (b) : Sulphur is the best electrical insulating material
(a) Soda glass (b) Pyrex glass known, with a resistivity of about 2 × 1023 µΩ-cm. The
(c) Jena glass (d) Crookes glass
reason for this large resistivity is probably the electron
Ans. (d) : Crookes glass can intercept ultraviolet rays.
traps produced by thermal breaking of S8 rings. Electrical
Ultraviolet rays were discovered by Ritter. The
wavelength of these waves ranges from 10–8 m to 10–7 m. conductivity of sulphur is 5.0 × 10–14 S m–1. Electrical
These waves are generated by sunlight, electric discharge, conductivity of selenium is 8 × 106 S m–1. Electrical
vacuum spark etc. conductivity of bromine is 1.0 × 10–10 S m–1. Electrical
69. Assertion (A) : A jet aircraft moving at Mach conductivity of phosphorous 1.0 × 10–9 S m–1. Thus from
Number equal to 1 travels this data it is clear that electrical conductivity of selenium
faster at an altitude of 15 km is maximum.
than while moving at Mach 73. Which one of the following is another name of
Number equal to 1 near the
RDX?
sea level.
Reason (R) : The velocity of sound depends (a) Cyanohydrin
on the temperature of the (b) Dextran
surrounding medium. (c) Cyclohexane
Ans. (d) : Mach number, a useful quantity in aerodynamics, (d) Cyclonite
is the ratio of air speed to the local speed of sound. The
Ans. (d) : RDX, an initialism for Research Department
speed of sound varies with temperature. Since temperature
Explosive, is an explosive nitroamine widely used in
and sound velocity normally decrease with increasing
altitude, sound is refracted upward. Mach number is a military and industrial applications. It is also known as
function of temperature at altitude. With decrease in sound cyclonite. Its chemical name is cyclotrimethylene
velocity, Mach number increases. trinitramine.

IAS (Pre) GS 2007 Paper I 276 YCT


BIOLOGY Ans. (c) : The medulla oblongata is a major structure
located in the lower half of the brainstem. It is
74. Which one of the following parts of the pitcher responsible for the regulation of heart rate, breathing
plant becomes modified into a pitcher? and blood pressure, as well as reflexes, such as
(a) Stem (b) Leaf vomiting, sneezing and coughing. This section of the
(c) Stipule (d) Petiole brain helps transfer messages to the spinal cord from the
body and the thalamus, which is in the brain. The main
Ans. (b) : In pitcher plant, the lamina of leaf is
function of the thalamus is to process information to
modified into pitcher and the leaf apex gives rise to a and fro the spinal cord and the cerebellum.
coloured lid for attracting the insects. The medulla oblongata helps regulate breathing, heart
75. In human body, which one of the following and blood vessel function, digestion, sneezing and
hormones regulates blood calcium and phosphate? swallowing. This part of the brain is a center for
(a) Glucagon respiration and circulation. Sensory and motor neurons
(b) Growth hormone (nerve cells) from the forebrain and midbrain travel
through the medulla.
(c) Parathyroid hormone
79. Production of which one of the following is a
(d) Thyroxin
function of the liver?
Ans. (c) : Parathyroid hormone secrets from parathyroid
(a) Lipase (b) Urea
gland. This hormone regulates calcium and phosphate
(c) Mucus (d) Hydrochloric acid
level in blood. Parathyroid hormone raises the level of
Ans. (b) :Urea is the main nitrogenous excretory
calcium in the blood and decreases the level of
product of Ureotelic animals, produced by liver cells
phosphorous in the blood.
from de-aminated excess amino-acids via the urea
76. How do most insects respire? cycle.
(a) Through skin 80. Which one of the following is not a digestive
(b) Through gills enzyme in the human system?
(c) By lungs (a) Trypsin (b) Gastrin
(d) By tracheal system (c) Ptyalin (d) Pepsin
Ans. (d) : Most insects respire through tracheal system Ans. (b) : Gastrin is a hormone, which stimulates
that lies on a network of small tubes that channel secretion of digestive enzyme in stomach. Trypsin,
oxygen directly to the different part of the body. The Pepsin and Ptylin are the digestive enzymes of human
tracheal system is composed of chitin-ringed tubes system.
called trachea that connect directly to the air through 81. In the human body, which structure is the
opening in the body wall called spiracles. appendix attached to?
77. In human beings, normally in which one of the (a) The large intestine
(b) The small intestine
following parts, does the sperm fertilize the
(c) The gall bladder
ovum?
(d) The stomach
(a) Cervix
Ans. (a) : The appendix is a small tube-shaped structure
(b) Fallopian tube attached to the first part of the colon, also known as the
(c) Lower part of uterus colon.
(d) Upper part of uterus 82. Robert Webster is known for his work
Ans. (b) : Fallopian tube is a tube like structure that associated with which one of the following?
carries egg from the ovary to uterus. In this tube the (a) Cardiology (b) Influenza virus
sperm meets ovum and fertilization occurs. (c) HIV/AIDS (d) Alzheimer
Ans. (b) : Robert Gordon Webster is an avian
78. Which one of the following parts of human
brain is the regulating centre for swallowing influenza authority who correctly posited that
and vomiting? pandemic strains of flu arise from genes in flu virus
(a) Cerebellum strains in nonhumans for example, via a reasortment
(b) Cerebrum of genetic segments between viruses in humans and
(c) Medulla oblongata nonhumans rather than by mutations in annual human
(d) Pons flu strains.
IAS (Pre) GS 2007 Paper I 277 YCT
83. Which one of the following is the correct ENVIRONMENT AND ECOLOGY
sequence in the order of decreasing length of
the three structural parts given below of small 87. How is Steve Fossett known as?
intestine in the human body?
(a) As a crocodile hunter.
(a) Jejunum-Duodenum-Ileum
(b) For completing the longest nonstop flight
(b) Ileum-Duodenum-Jejunum
around the globe.
(c) Jejunum-Ileum-Duodenum
(c) For swimming across Atlantic Ocean.
(d) Ileum-Jejunum-Duodenum
(d) For climbing to Mt. Everest without any co-
Ans. (d) : The correct sequence in the order of
climber.
decreasing length of the three structural parts are Ileum
– Jejunum – Duodenum of small intestine. Ans. (b) : Steve Fossett is known for completing the
longest nonstop flight around the globe
SCIENCE AND TECHNOLOGY
CURRENT AFFAIRS
84. Which one of the following pairs is not
correctly matched? 88. Which one of the following organisations won
the CSIR Award for Science and Technology
(a) Cosmic Background : Satellite
(S&T) innovations for Rural Development,
Explorer (COBE) Programme
2006?
(b) Falcon : Under-sea cable system
(a) CLRI (b) IARI
(c) Discovery : Space Shuttle
(c) NDDB (d) NDRI
(d) Atlantis : Space station
Ans. (a) : Central Leather Research Institute (CLRI),
Ans. (d) : Atlantis was a Space Shuttle orbiter of
received the first ever CSIR Award for S&T
National Aeronautics and Space Administration
Innovations for Rural Development in 2006.
(NASA).
89. What is the Galileo Project which has been in
85. NASA's Deep Impact space mission was
news recently?
employed to take detailed pictures of which
(a) An inter country programme of missile shield
comet nucleus?
developed by the United States of America.
(a) Halley's Comet
(b) A project developed by India with assistance
(b) Hale-Bopp
from Canada.
(c) Hyakutake
(c) An environmental protection project being
(d) Tempel 1
developed by Japan.
Ans. (d) : NASA’s Deep Impact space mission was (d) A multi-satellite navigation project being
employed to take detailed pictures of Tempel 1 comet developed by the European Union.
nucleus. It is a periodic Jupiter-family comet that was Ans. (d) : Galileo is the global navigation satellite
discovered in 1867 by Wilhelm Tempel. As per the system (GNSS) that is currently being created by the
research, it completes an orbit of the sun every 5.5 years European Union (EU) and the European Space Agency
of time. (ESA).
86. Where was the first conference of the Pugwash 90. Consider the following statements.
Conferences on Science and World Affairs held 1. The series of the International Paper Sizes is
in the year 1957? based on A0 size whose area is 0.5 m2
(a) Minnowbrook (USA) (approximately).
(b) Rhode Island (USA) 2. The area of A4 size paper is 1/8th of that of
the A0 size paper.
(c) Nova Scotia (Canada)
Which of the statements given above is/are
(d) Nagasaki (Japan) correct?
Ans. (c) : The Pugwash Conferences on Science and (a) 1 only (b) 2 only
World Affairs is an international organization that (c) Both 1 and 2 (d) Neither 1 nor 2
brings together scholars and public figures to work
Ans. (d) : The International Paper Quantity Series is based
toward reducing the danger of armed conflict and to on the magnitude of A0, which has an area of 1.0 square
seek solutions to global security threats. It was founded meters (approximately). The area of A4 magnitude paper is
in 1957 by Joseph Rotblat and Bertrand Russell in 1/16th of the area of A0 magnitude paper, not 1/8th.
Pugwash Nova Scotia Canada. Hence both the given statements are wrong.
IAS (Pre) GS 2007 Paper I 278 YCT
91. Which one of the following pairs of countries 96. Consider the following statements:
joined the European Union in January, 2007? 1. China has the observer's status at the South
(a) Bulgaria and Romania Asian Association for Regional Cooperation.
(b) Bulgaria and Belgium 2. India has the observer's status at the
(c) Romania and Slovenia Shanghai Cooperation Organisation.
(d) Hungary and Croatia Which of the statements give above is/are
Ans. (a) : Bulgaria and Romania joined European correct?
Union in January 2007. (a) 1 only (b) 2 only
92. Raghu Rai is well-known for which one of the (c) Both 1 and 2 (d) Neither 1 nor 2
following areas? Ans. (c) : States with observer status in South Asian
(a) Research in Mathematics Association for Regional Cooperation (SAARC)
(b) Photography include Australia, China, the European Union, Iran,
(c) Water harvesting Japan, Mauritius, Myanmar, South Korea and the
(d) Pollution control United States. India has observer status in the Shanghai
Ans. (b) : Raghu Rai is an Indian photographer and Cooperation Organization (SCO), India and Pakistan
photojournalist. became its permanent member in 2017.
93. Who among the following is Chile's first 97. In the latter half of the year 2006, in which one
woman President? of the following countries did a military coup
(a) D. Ortega (b) M. Bachelet take place?
(c) E. Morales (d) A. Garcia (a) Cambodia (b) Laos
(c) Thailand (d) Vietnam
Ans. (b) : Michelle Bachelet was the first female president
of Chile from 2006 to 2014. In December 2013, Bachelet Ans. (c) : The Thai military launched a coup against
Prime Minister Thaksin Shinawatra in September 2006,
was re-elected as President of Chile with over 62% of the
declaring martial law throughout Thailand. The military
vote. She is the first person since 1932 to win the
also announced the formation of an interim regime, as
presidency of Chile twice in competitive elections.
indicated by Lieutenant General Sonthi Boonyaratglin,
94. What was the purpose of the Operation Sukoon the commander-in-chief of the army.
launched by the Government of India?
98. Which one of the following is not a member of
(a) Helping Indonesia in its efforts to rehabilitate
the Shanghai Cooperation Organisation?
the victims of earthquake in the country.
(a) Russia (b) Kazakhstan
(b) Evacuating the Indian Nationals from Lebanon
(c) Ukraine (d) Uzbekistan
during the conflict in the Middle East.
(c) Assisting United Nations in its efforts to help Ans. (c) : The Shanghai Cooperation Organization
the civil war victims in the Darfur region of (SCO) currently comprises eight member States - China,
North Africa. India, Kazakhstan, Kyrgyzstan, Russia, Pakistan, Tajikistan
(d) Providing a relief package to farmers after a and Uzbekistan.
spate of suicides by other farmers in Andhra 99. Who among the following wrote the book
Pradesh. Ayodhya : 6 December 1992?
Ans. (b) : Operation Sukoon was an operation launched (a) Chandra Shekhar (b) PV. Narasimha Rao
by the Indian Navy to evacuate Indian, Sri Lankan and (c) Jaswant Singh (d) Arun Shourie
Nepalese nationals, as well as Lebanese nationals with Ans. (b) : In the book Ayodhya, the author PV
Indian spouses, from the conflict zone during the 2006 Narasimha Rao has explained the Ayodhya issue with a
Lebanon War. political perspective. Pamulaparti Venkata Narasimha
95. Which one of the following countries recently Rao (28 June 1921 – 23 December 2004) was an Indian
upgraded its defence agency to a full defence lawyer and politician who served as the tenth Prime
ministry? Minister of India (1991–1996).
(a) Italy (b) Japan 100. With reference to the international meetings
(c) Switzerland (d) Poland held in the year 2006, which of the following
Ans. (b) : For the first time since World War II, Japan pairs is/are correctly matched?
upgraded its Defence Agency to a full ministry in 2007. 1. NAM Summit : Havana
The upgrade of the Defence Agency, formerly under the 2. APEC Meeting : Bangkok
Cabinet Office, was propelled by deep concern in Japan 3. EU-India Summit : Helsinki
over the development of North Korean missiles and 4. UN Climate Change : Geneva
nuclear weapons. Conference
IAS (Pre) GS 2007 Paper I 279 YCT
Select the correct answer using the code given 104. Consider the following statements:
below: 1. The nation-wide scheme of the National
(a) 1 only (b) 1 and 3 only Child Labour Projects (NCLP) is run by the
(c) 1, 2 and 3 (d) 2, 3 and 4 Union Ministry of Social Justice and
Ans. (b) : The 14th NAM Summit was held on 16 Empowerment.
September 2006 in Havana (Cuba). The 17th EU India 2. Gurupadswamy Committee dealt with the
Summit took place in Helsinki on 13 October 2006. The issues of child labour.
14th APEC Summit took place in Hanoi on 18-19
Which of the statements given above is/are
November 2006 (Vietnam). The United Nations
Climate Change Conference 2006 took place in Nairobi, correct?
Kenya, between 6 and 17 November 2006. (a) 1 only (b) 2 only
101. The Stilwell Road, built in 1940's, which was (c) Both 1 and 2 (d) Neither 1 nor 2
recently in news, connects which of the Ans. (b) : National Child Labour Projects (NCLF) is
following? run by the Union Ministry of Labour and Employment.
(a) Agartala in India and Yangon in Myanmar via In 1979, Government formed the first committee called
Bangladesh. Gurupadswamy Committee to study the issue of child
(b) Ledo in India and Kumming in China via labour and to suggest measures to tackle it. Hence only
Myanmar. statement 2 is correct.
(c) Kalimpong in India and Lhasa in Tibet via
Bhutan. 105. Which of the following countries signed the
(d) Imphal in India and Bangkok in Thailand via Tshwane Declaration in October, 2006?
Myanmar. (a) China and South Africa.
Ans. (b) : The Stilwell Road built in 1940s by (b) India and South Africa.
Americans to connect Ledo in India and Kunming in (c) South Africa and Botswana.
China via Myanmar during Second World War.
(d) Saudi Arabia and South Africa.
102. MCA-21 is a major initiative taken up by the
Ans. (b) : The Tshwane Declaration was signed by
Government of India in which one of the
following areas? India and South Africa in 2006. Through this
(a) Foreign direct investment in India. declaration both the parties reaffirmed their Strategic
Partnership.
(b) Attracting international tourists.
(c) e-governance. 106. Recently, the European Union and other six
(d) Modernization of airports. countries including India signed the
Ans. (c) : International Thermonuclear Experimental
1: This is related to the e-filing of taxes by the Reactor (ITER) Project. Which one of the
companies in India. following was not a signatory to it?
2: MCA21 is an e-Governance initiative of the Ministry (a) Canada (b) China
of Company Affairs (MCA), Government of India (c) Japan (d) USA
that enables easy and secure access of
the MCA services to the corporate entities, Ans. (a) : India, European Union, along with 5 other
professionals and citizens of India. nations (China, Japan, Korea, Russia and United States)
103. Near the end of the year 2006, which one of the signed the ITER Project. Canada was not among the
following countries was suspended from the signatories as it stayed out of the agreement due to
Commonwealth after a military coup? scarcity of necessary resources.
(a) Kenya (b) Myanmar
107. Which one of the following Himalayan Passes
(c) Fiji (d) Tanzania was reopened around in the middle of the year
Ans. (c) : In December 2006, the Commonwealth 2006 to facilitate trade between Indian and
suspended Fiji in protest against the military coup. Fiji's China?
military commander, Commodore Frank Bainimarama (a) Chang La (b) Jara La
announced that he had taken control of the country (c) Nathu La (d) Shipki La
against the Prime Minister, Laisenia Qarase.
Ans. (c) : Nathu La Himalayan Passes was reopened
It is for the third time in 20 years that the Pacific nation
has been suspended from Commonwealth of Nations around in the middle of the year 2006 to facilitate trade
because of military coup. between India and China.

IAS (Pre) GS 2007 Paper I 280 YCT


108. Consider the following statements: Ans. (c) : Prithvi Air Defence( (PAD) missile was
1. Republicans won the majority in the U.S. tested in November 2006, followed by PAD in
House of Representatives Elections held in December 2007. With the test of the PAD missile, India
the year 2006. became the fourth country to have successfully
2. Republican Bobby Jindal, won a seat in the developed an Anti-ballistic missile system, after United
U.S. House of Representatives for the States, Russia, and Israel. On 6 March 2009, India again
second time. successfully tested its missile defence shield, during
Which of the statements given above is/are which an incoming "enemy" missile was intercepted at
corrected? an altitude of 75 km. Prithvi is a tactical surface-to-
(a) 1 only (b) 2 only surface short-range ballistic missile (SRBM) developed
(c) Both 1 and 2 (d) Neither 1 nor 2 by DRDO of India under the Integrated Guided Missile
Ans. (b) : In 2006, Democrats won the majority of Development Program (IGMDP) .It is also a single-
seats in US elections. Hence statement 1 is not correct. stage liquid fuelled missile having a maximum warhead
Second statement is correct. mounting capability of 500 kg, but with an extended
109. Which one of the following countries is range of 250 km (160 mi).
planning to construct a rival to the Panama
112. Who among the following was chosen as the
Canal to link to construct a rival to the Panama
FIFA World Player of the year for the year
Canal to link the Pacific and Atlantic Oceans?
2006?
(a) Colombia
(a) Zinedine Zidane
(b) Costa Rica
(c) Guatemala (b) Fabio Cannavaro
(d) Nicaragua (c) Ronaldinho
Ans. (d) : Nicaragua is planning to construct a rival to (d) Thierry Henry
the Panama Canal to link the Pacific and Atlantic Ans. (b) : Fabio Cannavaro was chosen as the FIFA
Oceans. World Player of the year for the year 2006.
110. Consider the following statements: 113. Wangari Maathai, the Nobel Prize winner from
1. In the year 2006, India successfully tested a Kenya is known for her contribution to which
full-fledged cryogenic stage in rocketry. one of the following?
2. After USA, Russia and China, India is the (a) Journalism
only country to have acquired the capability (b) International economics
for use of cryogenic stage in rocketry. (c) Sustainable development
Which of the statements given above is/are
(d) Child development
correct?
Ans. (c) : Wangari Maathai, the Nobel Prize winner
(a) 1 only (b) 2 only
from Kenya is known for her contribution to
(c) Both 1 and 2 (d) Neither 1 nor 2
Sustainable development
Ans. (a) : In the year 2006, India successfully tested a
114. For which one of the following books did Kiran
full-fledged cryogenic stage in rocketry. India Desai win the Man Booker prize 2006?
developed its own cryogenic rocket technology in 2018. (a) The Secret River
111. Consider the following statements: (b) In the Country of Men
1. In November, 2006, DRDO successfully (c) The Inheritance of Loss
conducted the interception test using (d) Mother's Milk
Prithvi-II missile. Ans. (c) : The Inheritance of Loss is the second novel
by Indian author Kiran Desai. It won the Man Booker
2. Prithvi-II is a surface-to-surface missile and
Prize for the year 2006.
can be deployed to guard the metros against
115. The Pulitzer Prize is associated with which one
air attacks. of the following?
Which of the statements given above is/are (a) Environmental protection
correct? (b) Olympic Games
(a) 1 only (b) 2 only (c) Journalism
(c) Both 1 and 2 (d) Neither 1 nor 2 (d) Civil Aviation
IAS (Pre) GS 2007 Paper I 281 YCT
Ans. (c) : The Pulitzer Prize is an award for MISCELLANEOUS
achievements in newspaper and online journalism,
literature, and musical composition in the United States. 120. Three identical vessels A, B and C are filled
It was established in 1917 by provisions in the will of with water, mercury and kerosene respectively
American (Hungarian-born) publisher Joseph Pulitzer up to an equal height. The three vessels are
and is administered by Columbia University in New provided with identical taps at the bottom of
York City. the vessels. It the three taps are opened
116. Which one of the following cities has been the simultaneously, then which vessel is emptied
venue of the Asian Games for the maximum first?
number of times from the year 1951 to the year (a) Vessel B
2006? (b) All the vessels A, B and C will be emptied
(a) Delhi (b) Bangkok simultaneously
(c) Tokyo (d) Beijing (c) Vessel A
Ans. (b) : Bangkok has been the venue of the Asian (d) Vessel C
Games for the maximum number of times from the yearAns. (d) : Kerosene has least viscosity among all three
1951 to the year 2006. liquids. So it has more tendency to flow and its vessel
117. Who was Leander Paes's partner when he won will get emptied first.
Doubles Final in the US Open tennis 121. Match List-I with List-II and select the correct
Tournament, 2006? answer using the codes given below the Lists:
(a) Max Mirnyi List-I List-II
(b) Martin Damm (Eminent Person) (Known As)
(c) Bob Bryan A. Bhanu Bharti 1. Music composer
(d) Mike Bryan B. Mike Pandey 2. Poet and literature
Ans. (b) : Martin Damm was Leander Paes's partner C. Mohd. Zahur 3. Theatre director
when he won Doubles Final in the US Open tennis Khayyam
Tournament, 2006. D. Vinda Karandikar 4. Wildlife film maker
118. What is the broad area in which the Nobel Code:
Prize winners for the year 2006 in Physiology A B C D
or Medicine, worked to get the Prize? (a) 1 4 3 2
(a) Prevention of weakening due to ageing. (b) 3 2 1 4
(b) Flow of genetic information. (c) 1 2 3 4
(c) Immunology and disease resistance. (d) 3 4 1 2
(d) Adult stem cell research. Ans. (d) : The correct match of given lists are:
Ans. (b) : Nobel prize for Physiology or medicine
was awarded to two scientists for Flow of genetic Bhanu Bharti - Theatre director
information. Mike Pandey - Wildlife film maker
119. Who among the following bowlers have taken Mohd. Zahur Khayyam – Music composer
more than 500 wickets in Test Cricket? Vinda Karandikar - Poet and literature
1. Wasim Akram 122. Match List-I with List-II and select the correct
2. Richard hadlee
answer using the codes given below the Lists:
3. Glen McGroth
List-I List-II
4. Courtney Walsh
Select the correct answer using the code given (Person) (Company)
below: A. Vishwapati Trivedi 1. Essar Group
(a) 1 and 2 only B. Tulsi R. Tanti 2. Info Edge India
(b) 3 and 4 only (which runs
(c) 1, 2 and 3 only naukri.com)
(d) 1, 2, 3 and 4 C. Shasti Ruia 3. Indian (Indian
Ans. (b) : Glen McGroth and Courtney Walsh have Airlines)
taken more than 500 wickets in Test Cricket. D. S. Bikhchandani 4. Suzlon Energy
IAS (Pre) GS 2007 Paper I 282 YCT
Code: Code:
A B C D A B C D
(a) 2 4 1 3 (a) 2 3 4 1
(b) 3 1 4 2 (b) 4 1 2 3
(c) 2 1 4 3 (c) 2 1 4 3
(d) 3 4 1 2 (d) 4 3 2 1
Ans. (d) : Ans. (a) :
Vishwapati Trivedi Indian (Indian Airlines) V. S. Naipaul In a free State
Tulsi R. Tanti Suzlon Energy Salman Rushdie Midnigth's Children
Paul Scott Staying on
Shasti Ruia Essar Group
J. G. Farrell The Siege of Krishnapur
Bikhchandani Info Edge India (which runs
naukri.com) 126. Match List-I with List-II and select the correct
answer using the codes given below the Lists:
123. Consider the following statements:
List-I List-II
1. North Atlantic Co-operation Council (Company) (Major Area/Product)
(NACC) is the name of the new organization
A. Chevron 1. Wind energy
which has replaced the North Atlantic
B. AT & T 2. Oil
Treaty Organization (NATO).
C. AMD 3. Telephone, Internet
2. The United States of America and the
D. Enercon GmbH 4. Micro processor
United Kingdom became the members of
Code:
the NATO when it was formed in the year
A B C D
1949.
(a) 2 1 4 3
Which of the statements given above is/are
(b) 4 3 2 1
correct.
(c) 2 3 4 1
(a) 1 only (b) 2 only
(d) 4 1 2 3
(c) Both 1 and 2 (d) Neither 1 nor 2
Ans. (c) : Chevron Corporation is an American
Ans. (b) : The North Atlantic Cooperation Council multinational energy corporation and one of the
(NACC) was established by the Allies on 20 December successor companies of Standard Oil headquartered in
1991 as a forum for dialogue and cooperation with California. AT and T Inc. is an American multinational
NATO’s former Warsaw Pact adversaries. The NATO telecommunications corporation, headquartered at
has 29 members currently. In 1949, there were 12 Whitacre Tower in downtown Dallas, Texas. Advanced
founding members of the Alliance: Belgium, Canada, Micro Devices, Inc. is an American worldwide
Denmark, France, Iceland, Italy, Luxembourg, the semiconductor company based in Sunnyvale,
Netherlands, Norway, Portugal, the United Kingdom California, United States, that develops computer
and the United States. processors. Enercon GmbH is the fourth-largest wind
124. How is Gabriel Garcia Marquez well known turbine manufacturer in the world, based in Germany.
as? 127. Where are the headquarters of the
(a) Known for research in agriculture. Organization of the Islamic Conference (OIC)
located?
(b) A renowned football coach.
(a) Dubai (b) Jeddah
(c) A great writer who won the Nobel Prize for (c) Islamabad (d) Ankara
literature.
Ans. (b) : The Organization of Islamic Cooperation
(d) Known for research in railway engineering. (OIC) is the second largest inter-governmental
Ans. (c) : Gabriel Garcia Marquez was a Colombian organization after the United Nations. Its headquarters
novelist, short-story writer, screenwriter and journalist. is situated in Jeddah, Saudi Arabia.
He was awarded with Nobel Prize in Literature in 1982. 128. Match List-I with List-II and select the correct
125. Match List-I with List-II and select the correct answer using the codes given below the Lists:
answer using the codes given below the Lists: List-I List-II
List-I List-II (Person) (Major Area/Product)
(Writer) (Book) A. Bhajan Sopori 1. Bharatnatyam dancer
A. V. S. Naipaul 1. The Siege of B. Birju Maharaj 2. Exponent of Santoor
Krishnapur C. Priyadarsini 3. Mridangam maestro
B. Salman Rushdie 2. In a free State Govind
C. Paul Scott 3. Midnigth's Children D. T.V. 4. Kathak dance
D. J. G. Farrell 4. Staying on Gopalakrishanan
IAS (Pre) GS 2007 Paper I 283 YCT
Code: List-I List-II
A B C D (Person) (known as)
(a) 2 1 4 3 A. John C. Mather 1. Co-founder of
(b) 3 1 4 2 Microsoft
(c) 2 4 1 3 B. Michael Griffin 2. Space Walker
C. Paul G. Allen 3. Administrator of
(d) 3 4 1 2
NASA
Ans. (c) : Bhajan Sopori Santoor is an excellent artist. D. Piers Sellers 4. Nobel Prize Winner,
Birju Maharaj is a Kathak dancer. Priyadarshini 2006 in Physics
Govind is a Bharatanatyam dancer. T.V. Gopal Code:
Krishnan is an excellent artist of Mridangam. A B C D
129. Match List-I with List-II and select the correct (a) 4 1 3 2
answer using the codes given below the Lists: (b) 2 3 1 4
List-I List-II (c) 4 3 1 2
(Person) (Position/Organization) (d) 2 1 3 4
A. Nancy Pelosi 1. WTO Ans. (c) :
B. Margaret Chan 2. Speaker, U.S. House John C. Mather Nobel Prize Winner, 2006 in
Physics
of Representatives
Michael Griffin Administrator of NASA
C. Pascal Lamy 3. WHO
Paul G. Allen Co-founder of Microsoft
D. Steve Ballmer 4. Microsoft Piers Sellers Space Walker
Code:
133. Who among the following is considered as the
A B C D inventor of the World Wide Web (WWW)?
(a) 2 1 3 4 (a) Edward Kasner (b) Bill Gates
(b) 4 3 1 2 (c) Time Tim Berners-Lee(d) Vinod Dham
(c) 2 3 1 4 Ans. (c) : Sir Timothy John Tim Berners-Lee, also
(d) 4 1 3 2 known as TimBL, was an English computer scientist,
best known as the inventor of the World Wide Web. He
Ans. (c) :
made a proposal for an information management system
Nancy Pelosi Speaker, U.S. House in March 1989, and he implemented the first successful
of Representatives communication between a Hypertext Transfer Protocol
Margaret Chan WHO (HTTP) client and server via the Internet.
Pascal Lamy WTO
Steve Ballmer Microsoft MATHS AND REASONING
130. In which one of the following cities is the 134. A train completes a journey with a few
Global Automotive Research Centre being set stoppages in between at an average speed of 40
up? km per hour. If the train had not stopped
(a) Chennai (b) Hyderabad anywhere, it would have completed the journey
(c) Pune (d) Gurgaon at an average speed of 60 km per hour. On an
average, how many minutes per hour does the
Ans. (a) : The Global Automotive Research Center train stop during the journey?
(GARC) ,Chennai, is situated in the SIPCOT Industrial (a) 20 minutes per hour
Growth Center at Orgadam near Chennai in the close (b) 18 minutes per hour
proximity of manufacturing facility of Indian and (c) 15 minutes per hour
Global automotive giants. (d) 10 minutes per hour
131. Parimarjan Negi has excelled in which one of Ans. (a) : With Trick 40 and 60 LCM = 120
the following games? Suppose total distance 120 km
(a) Billiards (b) Swimming
(c) Chess (d) Weightlifting
Ans. (c) : Parimarjan Negi is a chess Grandmaster from
India. 3–2=1
132. Match List-I with List-II and select the correct 60
Then stoppage time per hour = = 20 min
answer using the codes given below the Lists: 3
IAS (Pre) GS 2007 Paper I 284 YCT
135. The average salary of 100 employees in an ⇒ (Time taken to walk 2x km) + (Time taken to
office is Rs. 16,000 per month. The drive 2x km) =27/2 hrs.
management decided to raise salary of every But time taken to drive 2x km =19/4 hrs.
employee by 5% but stopped a transport Therefore, time taken to walk 2x km is:
allowance of Rs. 800 per month which was paid 27/2−19/4=54/4−19/4=54−19/4 =35/4 =8 3/4
earlier to every employee. What will be the new Hence, it would take 8 hours 45 minutes for him to walk
average monthly salary? both ways.
(a) Rs. 16,000
138. A and B can complete work together in 5 days.
(b) Rs. 16,500 If A works at twice his speed and B at half of
(c) Rs. 16,800 his speed, this work can b finished in 4 days.
(d) Cannot be known since data are insufficient How many days would it take for A alone to
Ans. (a) : Since, salary of each employee is increased complete the job?
by 5%, so net average increase in salary = 5%. (a) 10 (b) 12
∴ New average monthly salary (c) 15 (d) 18
= 16000 ( 1+ 5/100 ) Ans. (a) : Let alone time for A and B be x and y
= 16800 respectively.
But stopped transport allowance 800 per month Now, 1/x + 1/y = 1/5 …….(i)
Then average salary of 100 employ Also, 2/x + 1/2y = 1/4 …...(ii)
= 16800 – 800 =16000 Solving (i) and (ii), we get, x = 10 and y = 10
Hence option (a) is correct. 139. If all the numbers from 501 to 700 are written,
136. A person has to completely put each of three what is the total number of times does the
liquids: 403 litres of petrol, 465 litres of diesel digits 6 appear?
and 496 litres of Mobil Oil in bottles of equal (a) 138 (b) 139
size without mixing any of the above three (c) 140 (d) 141
types of liquids such that each bottle is Ans. (c) : Numbers from 501 to 599 which have 6 as
completely filled. What is the least possible digit are 506, 516, 526, 536, 546, 556, 560, 561, 562,
number of bottles required? 563, 564, 565, 566, 567,568, 569, 576, 586 and 596,
(a) 34 (b) 44 i.e., 6 occurs 20 times.
(c) 46 (d) None of the above Number of times 6 occurs from 600 to 699 = 100 + 20 =
Ans. (b) : Maximum capacity of each bottle can be 120
found by taking the H.C.F of the three given liquids. ∴ Total number of times 6 occurs = 20 + 120 = 140
Maximum capacity of each bottle = HCF of 403, 465 140.
and 496 = 31
Number of bottles for 403 l of petrol = 403 / 31 = 13
Number of bottles for 465 l of diesel =465 / 31 = 15
Number of bottles for 496 l of mobile oil = 496 / 31=16 In how many maximum different ways can 3
Hence, total number of bottles = 13 + 15 + 16 = 44 identical balls be placed in the 12 squares (each)
137. Amit starts from a point A and walks to ball to be placed in the exact centre of the squares
another point B, and then returns from B to A and only one ball is to be placed in one square
by his car and thus takes a total time of 6 hours shown in the figure given above such that they do
and 45 minutes. If he had driven both ways in
not lie along the same straight line?
his car, he would have taken 2 hours less. How
long would it take for him to walk both ways? (a) 144 (b) 200
(a) 7 hours 45 minutes (c) 204 (d) 216
(b) 8 hours 15 minutes Ans. (c) : 3 balls can be placed in any of the 12 squares
(c) 8 hours 30 minutes in 12C3 ways.
(d) 8 hours 45 minutes
Total no. of arrangements = 12C3 = 220
Ans. (d) : Let the distance be x km.
Now, assume that balls lie along the same line
(Time taken to walk x km) + (Time taken to
There can be 3 cases
drive x km) =27/4 hrs.

IAS (Pre) GS 2007 Paper I 285 YCT


(i) When balls lie along the straight horizontal line. 3 1

balls can be put in any of the 4 boxes along the 4 2 3


horizontal row in 4C3 ways. Since there are 3 rows,
5
so number of ways = 4C3 × 3 = 12
6
(ii) When balls lie along the vertical straight line 3 balls
can be put in any of the 3 boxes along the vertical The face 2 is adjacent to the face 3.
row in 3C3 ways. Now as there are 4 vertical rows, 143. Each of the 3 persons is to be given some
so number of ways in 2nd case = 3C3 × 4 = 4 identical items such that product of the
numbers of items received by each of the three
(iii) Balls lie along the 2 diagonal lines towards the left persons is equal to 30. In how many maximum
and 2 diagonal lines towards the right. different ways can this distribution be done?
No. of ways = 2 + 2 = 4 (a) 21 (b) 24
(c) 27 (d) 33
∴ No. of ways when balls lie along the line = 12 + 4 + 4 = 20
Ans. (c) : 30 = 2 × 3 × 5 and all the positive divisor of
∴ Number of ways when balls don’t lie along the line = 30 are – 1, 2, 3, 5, 6, 10,15, 30
Total No. of ways – number of ways when ball lie along we have to find the number of triples (a, b, c) such that
a × b × c = 30.
the line = 220 – 20 = 200.
Now, 30 = 15 × 2 = 10 × 3 = 6 × 5 = 2 × 3 × 5. Each of
141. Groups each containing 3 boys are to be these can be made into triples of divisors by introducing
formed out of 5 boys A, B, C, D and E such that factors of 1 suitably if required. Thus we have the
no one group contains both C and D together. following triples–
30 = (30, 1, 1), (1, 30, 1) and (1, 1, 30) → 3
What is the maximum number of such 15 × 2 = 6 permutations of (1, 2, 15)
different groups? 10 × 3 = 6 permutations of ( 1, 3, 10)
(a) 5 (b) 6 6 × 5 = 6 permutations of (1, 5, 6)
2 × 3 × 5 = 6 permutations of (2, 3, 5)
(c) 7 (d) 8
∴ altogether the distribution can be done in (4 × 6) + 3
Ans. (c) : 5C3 = 27 ways.
Total number of arrangements, when any 3 boys are 144. 6 equidistant vertical lines are drawn on a
selected out of 5= 5C3 board. 6 equidistant horizontal lines are also
Now, when groups contain both C and D, then their drawn on the board cutting the 6 vertical lines,
and the distance between any two consecutive
selection is fixed and the remaining 1 boy can be
horizontal lines is equal to the between any two
selected out of the remaining 3 boys. It can be done consecutive vertical lines. What is the
in 3C1 ways. maximum number of squares thus formed?
So, number of groups, when none contains (a) 37 (b) 55
both C and D= total number of arrangements-number of (c) 126 (d) 225
5
arrangements when group contains both C and D = C3 Ans. (b) :
– 3C1 =10–3 = 7
142. Six faces of a cube are numbered from 1 to 6, each
face carrying one different number. Further:
1. The face 2 is opposite to the face 6.
2. The face 1 is opposite to the face 5.
3. The face 3 is between the face 1 and the face 5
4. The face 4 is adjacent to the face 2.
Which one of the following is correct?
(a) The face 2 is adjacent to the face 3
Counting number of square from figure
(b) The face 6 is between the face 2 and the face 4
Case 1 : Single square box = 5 × 5 = 25 ……(i)
(c) The face 1 is between the face 5 and the face 6 Case 2 : Double square box = 4 × 4 = 16 ……(ii)
(d) None of the above Case 3 : Triple square box = 3 × 3 = 9 …….(iii)
Ans. (a) : It is very clear from figure. Case 4 : Square with 4 square = 2 × 2 = 4 ……(iv)
Case 5 : Square with 5 square box = 1 ……..(v)
Adding all the square boxes from (i), (ii), (iii), (iv) and (v)
We get
25 + 16 + 9 + 4 + 1 = 55 square boxes.
145.
IAS (Pre) GS 2007 Paper I 286 YCT
148. Five balls of different colours are to be placed in
three different boxes such that any box contains at
least one ball. What is the maximum number of
In the figure shown above, what is the different ways in which this can be done?
maximum number of different ways in which 8 (a) 90 (b) 120
identical balls can be placed in the small (c) 150 (d) 180
triangles 1, 2, 3 and 4 such that each triangle
contains at least one ball? Ans. (c) : Let the boxes be x, y, z
(a) 32 (b) 35 I – x, y, z : 1, 1, 3
(c) 44 (d) 56 Total 3 case possible – 3× 5C3 2C1 1C1 = 3 × 10 × 2 = 60
Ans. (b) : There can be 5 cases–
1. First triangle can have 5 balls and rest three 1 each II – x, y, z : 1, 2, 2
4! Total 3 case possible – 3× 5C2 3C2 1C1 = 90
So no. of ways for (5,1,1,1) = = 4
3! Total ways = I + II = 60 + 90 = 150
2. First triangle can have 4 balls, 2nd can have 2 balls
and rest 1 each 149. Amit has five friends : 3 girls and 2 boys.
4! Amit's wife also has 5 friends: 3 boys and 2
So no. of ways for (4, 2, 1,1) = = 12 girls. In how many maximum number of
2! different ways can they invite 2 boys and 2 girls
3. Similarly no. of ways for combination
such that two of them are Amit's friends and
4! two are this wife's?
(2, 2, 2, 2) = = 1
4! (a) 24 (b) 38
4! (c) 46 (d) 58
4. No. of ways for (3, 3, 1, 1) = =6
2!× 2! Ans. (c) :
4! 2 boys from Amit friend and 2 girls from his wife
5. No. of ways for (3, 2, 2, 1) = = 12
2! friend
As either of these 5 cases are possible, Hence total no. OR 1 boy & 1 girl from Amit friend and 1 boy
of ways = 4 + 12 + 1 + 6 + 12 = 35 and 1 girl from his wife friend
146 Three dice (each having six faces with each face OR 2 girls from Amit friend and 2 boys from his wife
having one number from 1 to 6) are rolled. What friend
is the number of possible outcomes such that at
Number of ways =1×1+3×2×3×2+3×3=1+36+9 = 46
least one dice shows the number 2?
(a) 36 (b) 81 150.
(c) 91 (d) 116
Ans. (c) : Total number of outcome when 3 dice are
rolled = 6 × 6 × 6 = 216
Number of way in which 2 does not appear on any of
the dice = 5 × 5 × 5
= 125 (Each small circle represents a different station)
So, the required possibility to show 2 atleast one time What is the maximum number of different paths
will be = 216 – 125 = 91 that exist between the station A and the station B?
147. All the six letters of the name SACHIN are (a) 28 (b) 31
arranged to form different words without (c) 33 (d) 35
repeating any letter in any one word. The
words so formed are then arranged as in a
Ans. (b) :
dictionary. What will be the position on the
word SACHIN in the sequence?
(a) 436 (b) 590
(c) 601 (d) 751
Ans. (c) : In S A C H I N
S is fixed since it is the starting letter There are 4 roots between A to b vio P, Q, R and S
No. of words that start with A Case 1 : rout via P →
=A−−−− =5×4×3×2×1 =5! =120 A to P = 3 and P to B = 3
No. of words starting with C Routes via P = 3 × 3 = 9
=C−−−− =5!=120 Case 2 : A to Q = 4 and Q to B = 3
Similarly, no. of words starting from H,I,N will be 120
∴ Total No. of words=120+120+120+120+120=600 Routes via Q = 4 × 3 = 12
Since SACHIN will be ranked after 600 words, Case 3 : Similarly routes via R = 3 × 2 = 6
therefore it's rank will be 601th . Case 4 : Routes via S = 2 × 2 = 4
Total no. of routes = 9 + 12 + 6 + 4 = 31
IAS (Pre) GS 2007 Paper I 287 YCT
UNION PUBLIC SERVICE COMMISSION
Civil Services (Preliminary Exam) - 2006
GENERAL STUDIES : PAPER-I
Time: 2 hours Maximum Number: 200
Which of the statements given above is/are
ANCIENT HISTORY correct?
(a) 1 only (b) 2 only
1. The Allahabad Pillar inscription is associated (c) Both 1 and 2 (d) Neither 1 nor 2
with which one of the following? Ans. (b) : Most of the inscriptions of the Andhra
(a) Mahapadma Nanda Ikshvaku period record either the construction of the
(b) Chandragupta Maurya Buddhist viharas or the gifts made to them which show
(c) Ashoka that the Ikshvaku rulers of Southern India were
supporters of Buddhism. The Pala rulers of Eastern
(d) Samudragupta India were patrons of Buddhism. They established
Ans. (d) : The Allahabad Pillar inscription is associated famous Vikramshila vihar.
with Samudragupta's court poet Harisena. 5. Who among the following laid the foundation
2. Between which of the following was the ancient of Rashtrakuta Empire?
town of Takshsila located? (a) Amoghavarsha I (b) Dantidurga
(a) Indus and Jhelum (b) Jhelum and Chenab (c) Dhruva (d) Krishna I
(c) Chenab and Ravi (d) Ravi and Beas Ans. (b) : Dantivarman or Dantidurga was the founder
of the Rashtrakuta dynasty. The dynasty ruled large
Ans. (a) : Taxila was located between River Indus and parts of the Indian subcontinent between the sixth and
Hydaspes, river Hydaspes is now known as river 10th centuries. Dantidurga was a feudatory of the
Jhelum, it is a tributary of river Indus. It is located in Chalukyas. In the mid-8th century, Dantidurga rose to
Rawalpindi,Punjab. importance and established himself in place of the
3. With reference to the invaders in ancient India, declining Chalukya dynasty.
which one of the following is the correct 6. Which one of the following pairs is correctly
chronological order? matched?
(a) Vikramsila Monastery : Uttar Pradesh
(a) Greeks-Sakas-Kushans (b) Hemkund Gurudwara : Himachal
(b) Greeks-Kushans-Sakas (c) Udayagiri : Maharashtra
(c) Sakas-Greeks-Kushans (d) Amaravati Buddhist : Andhra Pradesh
(d) Sakas-Kushans-Greeks Stupa
Ans. (a) : The invasion in ancient India was firstly done Ans. (d) : Amaravati Buddhist Stupa : Andhra Pradesh
by Greeks, then Sakas and then Kushanas. 7. Match List-I (Finding/Invention/Calculation)
The Greek conquests of India took place in the years with List-II (Ancient Indian Scholar) and select
before the Common Era, and a rich trade flourished the correct answer using the code given below
the Lists:
between India and Greece, especially in silk, spices, and
List-I List-II
gold. The Greeks invaded India several times, starting (Finding/Invention (Ancient Indian
with the conquest of Alexander the Great between the /Calculation) Scholar)
years 327 to 326 B.C.E. The Indo-Greek rule lasted A. Time taken by the Earth 1. Aryabhatta
from about 180 BC till about 55 BC. The Sakas were to orbit the Sun
alternatively known as Indo-Scythians, invaded B. Calculation of the value 2. Bhaskaracharya
northwest India in the first century BC onwards. The of π ('pi')
Kushan Empire fragmented into semi-independent C. Invention of the digit 3. Budhayana
kingdoms in the 3rd century AD, which fell to the zero
Sasanians invading from the west, establishing the D. The game of snakes
Kushano-Sasanian Kingdom in the areas of Sogdiana, and ladders 4. Gyandev
Bactria and Gandhara. In the 4th century, the Guptas, an Codes:
Indian dynasty also pressed from the east. A B C D
4. Consider the following statements: (a) 2 4 1 3
1. The Ikshvaku rulers of Southern India were (b) 1 3 2 4
antagonistic towards Buddhism. (c) 2 3 1 4
2. The Pala rulers of Eastern India were (d) 1 4 2 3
patrons of Buddhism. Ans. (c) : A-2 , B-3, C- 1, D- 4
IAS (Pre) GS 2006 Paper I 288 YCT
MEDIEVAL HISTORY (a) An important military commander during
Akbar's reign
8. When Raja Wodeyar founded the Kingdom of (b) An official historian of the reign of Shah
Mysore, who was the ruler of the Vijayanager Jahan
Empire? (c) An important noble and confidant of
(a) Sadasiva (b) Tirumala Aurangazeb
(d) A chronicler and poet during the reign of
(c) Ranga II (d) Venkata II
Muhammad shah
Ans. (d) : Indigenous chieftains from Karnataka rose to
Ans. (b) : Abdul Hamid Lahori was a traveler and
become the founders of a dynasty after a series of official historian during the period of Mughal Emperor
internal bickerings with other local chieftains. Vijaya Shah Jahan. He is best known for his work called
and Krishna two brothers were responsible for the Padshahnamah. It is divided into two volumes and is a
establishment of this small feudatory state of genre of works written as the official visual history of
Vijayanagara Empire. Vijaya ascended the throne under Mughal Emperor, Shah Jahan’s reign.
the assumed name of Yaduraja in 1399 A.D. To begin
13. Assertion (A) : Muhammad bin Tughluq issued
with this was a very small state containing only parts of
Mysore district. Mysore was the capital city. a new gold coin which was called
Dinar by Ibn Batutah.
9. Which one of the following is the correct
Reason (R) : Muhammad bin Tughluq
chronological order of the Afghan rulers to the
wanted to issue token currency
throne of Delhi?
in gold coins to promote trade
(a) Sikandar Shah Ibrahim Lodi-Bahlol Khan
with West Asian and North
Lodi
African countries.
(b) Sikandar Shah – Bahlol Khan Lodi- Ibrahim
Ans. (c) : Muhammad bin Tughluq issued a new gold
Lodi
coin, which was called Dinar by Ibn Batutah.
(c) Bahlol Khan Lodi – Sikandar Shah – Ibrahim Mohammed-bin-Tughlaq introduced the "token
Lodi currency system". Since money is merely a medium of
(d) Bahlol Khan Lodi – Ibrahim Lodi – Sikandar exchange, token currencies do not have to depend on
Shah the supply of gold and silver. There was a shortage of
Ans. (c) : Bahlol (1451–1489); Sikandar (1489–1517); silver in the world during fourteenth century and that is
Ibrahim (1517–1526). why Mohammad-bin-Tughlaq introduced token
currency. So the reason is not correct.
10. Bhakta Tukaram was a contemporary of which
Mughal Emperor?
(a) Babar (b) Akbar
MODERN HISTORY
(c) Jahangir (d) Aurangzeb 14. Which portfolio was held by Dr. Rajendra
Ans. (c) : Best answer is c as Tukaram (1608–1650) Prasad in the Interim Government formed in
was a Marathi Bhakti poet and a devotee of Lord the year 1946?
Krishna. Time period of Jahangir was 1605-1627. (a) Defence
11. The initial design and construction of which (b) External Affairs and Commonwealth
massive temple took place during the reign of Relations
Suryavarman II? (c) Food and Agriculture
(d) None
(a) Sri Mariamman Temple
Ans. (c) : Food and Agriculture portfolio was held by
(b) Angkor Vat
Dr. Rajendra Prasad in the Interim Government formed
(c) Batu Caves Temple in the year 1946. After the Indian independence in
(d) Kamakhya Temple 1947, Dr. Rajendra Prasad became the first President of
Ans. (b) : Angkor Vat was built by the Khmer King India.
Suryavarman II in the early 12th century in 15. Consider the following statements:
Yaśodharapura, the capital of the Khmer Empire, as his 1. The charter Act 1853 abolished East India
state temple and eventual mausoleum. Breaking from Company's monopoly of Indian trade.
the Shaiva tradition of previous kings, Angkor Wat was 2. Under the Government of India Act 1858, the
dedicated to Vishnu. British Parliament abolished the East India
12. In Indian history, who was Abdul Hamid Company altogether and undertook the
Lahori? responsibility of ruling India directly.

IAS (Pre) GS 2006 Paper I 289 YCT


Which of the statements given above is/are Ans. (c) : They also celebrated the first Independence
correct? day on January 26, 1930, which was the date later
(a) 1 only (b) 2 only selected for the adoption of the Indian constitution.
(c) Both 1 and 2 (d) Neither 1 nor 2
The Lahore Session of the Indian National Congress
Ans. (b) : The Charter Act of 1813 ended the trade
was held under the presidency of Jawahar Lal Nehru in
monopoly of East India Company in India. Hence
statement 1 is not correct. Under the Government of the year 1929 wherein a resolution was adopted to gain
India Act 1858, the British Parliament abolished the complete independence from the British.
East India Company altogether and undertook the 20. Who was the Governor-General of India
responsibility of ruling India directly. Hence statement during the Sepoy Mutiny?
2 is correct.
(a) Lord Canning (b) Lord Dalhousie
16. Which one of the following revolts was made
(c) Lord Hardinge (d) Lord Lytton
famous by Bankim Chandra Chatterjee in his
novel Anand Math? Ans. (a) : Lord Canning was Governor General of India
(a) Bhil uprising during the sepoy mutiny.
(b) Rangpur and Dinapur uprising 21. Consider the following statements about
(c) Bishunpur and Birbhum rebellion
Madam Bhikaji Cama:
(d) Sanyasi rebellion
1. Madam Cama unfurled the National Flag at
Ans. (d) : Anand Math is a Bengali novel, written by
Bankim Chandra Chatterjee and published in 1882, set the International socialist Conference in
in the background of the Sanyasi Rebellion in the late Paris in the year 1907.
18th century. 2. Madam Cama served as private secretary to
17. In the year 1613, where was the English East Dadabhai Naoroji.
India Company given permission to set up a 3. Madam Cama was born to Parsi parents.
factory (trading post)? Which of the statements given above are
(a) Bangalore (b) Madras
correct?
(c) Masulipattam (d) Surat
Ans. (d) : In 1612, Sir Thomas Roe visited Mughal (a) 1, 2 and 3 (b) 2 and 3 only
Emperor Jahangir to arrange for a commercial treaty (c) 1 and 2 only (d) 3 only
that would give the company exclusive rights to reside Ans. (b) : Madam bhikajai Cama was born to Parsi
and build factories in Surat and other areas. In return, parents. Bhikaji Cama unfurled the first version of
the company offered to provide the Emperor with goods national flag- a tricolour of green, saffron and red strips,
and rarities from the European market. Jahangir
made by herself at international socialist conference at
accepted the proposal and the company created trading
posts in Surat (where a factory was built in 1613), Stuttgart, Germany in 1907. Not in Paris.
Madras (1639), Bombay (1668), and Calcutta (1690). Hence, statement 2 and 3 is correct.
18. With reference to the revolt of the year 1857,
who of the following was betrayed by a friend; INDIAN GEOGRAPHY
captured and put to death by the British?
(a) Nama Sahib (b) Kunwar Singh 22. Consider the following statements:
(c) Khan Bahadur Khan (d) Tantia Tope 1. Assam shares a border with Bhutan and
Ans. (d) : Tatya Tope (1814 - 1859) or Tantia Tope Bangladesh.
(birth name: Ramachandra Pandurang Tope) was a
2. West Bengal shares a border with Bhutan
close associate of Nana Saheb, and with him, led the
and Nepal.
revolt at Kanpur. He was handed over to British by Man
Singh, Raja of Narwar, who negotiated with British to 3. Mizoram shares border with Bangladesh
hand over Tope in return for protection to his life and and Myanmar.
also settling a dispute with Maharaja of Gwalior. Which of the statements given above are
correct?
19. Under whose presidency was the Lahore
Session of the Indian National Congress held in (a) 1, 2 and 3 (b) 1 and 2 only
the year 1929 wherein a resolution was adopted (c) 2 and 3, only (d) 1 and 3, only
to gain complete independence from the Ans. (a) : All the statements given above are correct.
British? Assam shares a border with Bhutan and Bangladesh,
(a) Bal Gangadhar Tilak West Bengal shares a border with Bhutan and Nepal
(b) Gopal Krishna Gokhale and Mizoram shares a border with Bangladesh and
(c) Jawaharlal Nehru
Myanmar.
(d) Motilal Nehru
IAS (Pre) GS 2006 Paper I 290 YCT
23. Assertion (A) : The percentage of net sown 26.
Which one of the following statements is not
area in the total area of correct?
Andhra Pradesh is less as (a) Mahanadi River rises in Chhattisgarh
compared to that of West (b) Godavari River rises in Maharashtra
Bengal. (c) Cauvery River rises in Andhra Pradesh
(d) Tapti River rises in Madhya Pradesh
Reason (R) : The soil of most of Andhra
Pradesh is laterite. Ans. (c) : Cauvery river originates in the foothills of
Western Ghats at Talakaveri, Kodagu in Karnataka. It
(a) Both A and R are true and R is the correct
flows generally south and east through Karnataka and
explanation of A.
Tamil Nadu and across the southern Deccan plateau
(b) Both A and R are true but R is not the correct
through the southeastern lowlands, emptying into the
explanation of A. Bay of Bengal through two principal mouths in
(c) A is correct but R is wrong. Poompuhar, Tamil Nadu.
(d) A is wrong but R is correct. 27. Where is the Central Water and Power
Ans. (c) : Assertion is true but Reason is false. Net Research Station located?
sown area of Andhra Pradesh is 40% and West Bengal (a) Khadakwasla (b) Sileru
has 60%. So the assertion is correct. But the reason is (c) Jamnagar (d) Srisailam
false because the alluvial soil is only found across delta
Ans. (a) : Central Water and Power Research Station
of rivers of the Godavari the Krishna and the Cauvery
was established in the year 1916 in Khadakwasla, Pune
of Andhra Pradesh.
by the then Bombay Presidency as a Special Irrigation
24. Which one of the following pairs is not Cell. It is the Premier Research and Development (R &
correctly matched? D) organisation and is a subordinate office under
Monastery State Ministry of Water Resources, River Development and
(a) Dhankar Monastery : Himachal Pradesh Ganga Rejuvenation.
(b) Rumtek Monastery : Sikkim 28. Which one of the following companies is
(c) Tabo Monastery : Himachal Pradesh associated with the exploration and commercial
(d) Kye Monaster : Arunachal Pradesh production of oil in Barmer-Sanchore basin of
Rajasthan?
Ans. (d) : Dhankar Monastery is present at 12,774 feet
above sea level of Lahaul and Spiti of Himachal (a) Cairn Energy
Pradesh. Rumtek monastery is important and largest (b) Unocal Corporation
monastery of Sikkim, 24 km away from Gangtok. Tabo (c) Reliance Energy Ventures
monastery is situated at a distance of about 50 (d) ONGC
kilometres from Kaza in Lahual district of Himachal
Ans. (a) : Cairn Energy PLC is one of the leading
Pradesh. Kye Monastery is situated 12 kms north of independent exploration and development companies
Kaza of Himachal Pradesh, not in Arunachal Pradesh. for oil and gas in Europe and is listed on the London
25. Match List-I (Valley) with List-II (State) and Stock Exchange. It is linked to the exploration and
select the correct answer using the codes given commercial production of oil in Rajasthan's Barmer-
below the Lists: Sanchore basin. The 25-year contract for oil and gas
exploration and production from the Cairn India Barmer
List-I List-II
oil block in Rajasthan, in which ONGC holds a 30%
(Valley) (State) interest, was due to be renewed in 2020.
A. Markha Valley 1. Sikkim 29. Which of the following substances are found in
B. Dzukou Valley 2. Himachal Pradesh the beach sands of many parts of Kerala?
C. Sangla Valley 3. Jammu and 1. Ilmenite
Kashmir 2. Zircon
D. Yumthang Valley 4. Nagaland 3. Sillimanite
Codes: 4. Tungsten
A B C D Select the correct answer using the code given
(a) 2 4 3 1 below:
(b) 3 1 2 4 (a) 1, 2, 3 and 4 (b) 1, 2 and 3 only
(c) 2 1 3 4 (c) 3 and 4 only (d) 1 and 2 only
(d) 3 4 2 1 Ans. (b) : Ilmenite, Zircon and Silimanite are found in
Ans. (d) : A-3; B-4; C-2; D-1 the beach sands of many parts of Kerala.
IAS (Pre) GS 2006 Paper I 291 YCT
30. Match List-I (Centre of Handicrafts) with List- WORLD GEOGRAPHY
II (State) and select the correct answer using
33. Bermuda triangle extend up to which of the
the code given below the Lists:
following places?
List-I List-II 1. Southern Florida
(Centre of Handicrafts) (State) 2. Puerto Rico
A. Mon 1. Arunachal Pradesh 3. Hawali Islands
B. Nalbari 2. Assam Select the correct answer using the code given
C. Pasighat 3. Meghalaya below:
D. Tura 4. Nagaland (a) 1, 2 and 3 (b) 1 and 2, only
(c) 2 and 3, only (d) 1 and 3, only
Codes:
Ans. (b) : The Bermuda Triangle, a mysterious stretch
A B C D
of ocean between Bermuda, Puerto Rico and the tip of
(a) 5 3 2 3 Florida, has allegedly, throughout the years, swallowed
(b) 1 3 4 2 a horde of unsuspecting ships, planes and people.
(c) 4 2 1 3 34. Which one of the following countries is not a
(d) 1 2 4 3 member of the Nordic Council?
Ans. (c) : A-4; B-2; C-1; D-3 (a) Norway (b) Denmark
(c) Iceland (d) United Kingdom
31. Match List-I (National Park/Wildlife
Ans. (d) : The Nordic Council is a geo-political inter-
Sanctuary) with List-II (Nearby Town) and
parliamentary forum for co-operation between the
select the correct answer using the code given Nordic countries. It was formed after the Second World
below the Lists: War in 1952 to promote co-operation between the five
List-I List-II Nordic countries - Denmark, Finland, Iceland, Norway
(National Park/Wildlife (Nearby Town) and Sweden.
Sanctuary) 35. Which one of the following pairs is not
A. Chandra Prabha 1. Jaipur correctly matched?
B. Karera 2. Jhansi (a) Slovenia : Bratislava
C. Jaisamand 3. Agra (b) Seychelles : Victoria
D. Nahargarh 4. Varanasi (c) Sierra Leone : Freetown
5. Udaipur (d) Uzbekistan : Tashkent
Codes: Ans. (a) : Slovenia - Ljubljana, Seychelles - Victoria
A B C D Sierra Leone-Freetown, Uzbekistan - Tashkent.
(a) 4 1 5 2 Bratislava is the capital of Slovakia.
(b) 5 2 3 1 36. Consider the following statements:
(c) 4 2 5 1
1. Length of a terrestrial mile is lesser than that
(d) 5 1 3 2
of a nautical mile.
Ans. (c) : A-4; B-2; C-5; D-1
2. Harmattan is a dusty land-wind of the East
32. From north toward south, which one of the African Coast.
following is the correct sequence of the given
rivers in India? 3. Greece and Albania form a part of the
(a) Shyok-Spiti-Zaskar-Satluj Iberian Peninsula.
(b) Shyok-Zaskar-Spiti-Satluj Which of the statements given above is/are
(c) Zaskar-Shyok-Satluj-Spiti correct?
(d) Zaskar-Satluj-Shyok-Spiti (a) 1, 2 and 3 (b) 2 and 3 only
Ans. (b) : The correct sequence of the rivers in India (c) 3 only (d) 1 only
from north towards south is Shyok – Zaskar – Spiti – Ans. (d) : One nautical mile is 1852 meters and one
Satluj. Shyok flows through northern Ladakh in India. terrestrial mile is 1609 metres. So, statement '1' is
Zaskar River flows in north-eastern part of Ladakh. correct. Harmattan is a dry and dusty wind of West
Spiti River is situated at Kaza in Himachal Pradesh at Africa. So, statement '2' is wrong. Iberian peninsula
an elevation of 12500 feet. Satluj River rises from includes Portugal, Spain, Andorra and Gibraltar. Greece
slopes of Kailash and flows in south-westerly direction and Albania are not a part of Peninsula. So, statement '3'
to Himachal Pradesh & Punjab. is wrong.

IAS (Pre) GS 2006 Paper I 292 YCT


37. Assertion (A) : To orbit around the Sun, the Ans. (b) : Earth is moving about our sun in a very
planet Mars takes lesser time nearly circular orbit. It travels with the mean velocity of
than the time taken by the Earth. 30 kms.
Reason (R) : The diameter of the planet Mars 42.
Through which one of the following groups
is less than that of the Earth. countries does the Equator pass?
(a) Both A and R are true and R is the correct (a) Barzil, Zambia and Malaysia
explanation of A. (b) Colombia, Kenya and Indonesia
(b) Both A and R are true but R is not the correct
(c) Barzil, Sudan and Malaysia
explanation of A.
(d) Benezuela, Ethiopia and Indonesia
(c) A is correct but R is wrong.
Ans. (b) : Among the following countries, equator
(d) A is wrong but R is correct.
passes through Colombia, Kenya and Indonesia. The
Ans. (d) : A is incorrect as the mars revolution round equator passes through 13 countries: Ecuador,
the sun takes 1.88 years of Earth’s revolution. The Colombia, Brazil, Sao Tome & Principe, Gabon,
diameter of earth is 12,742 km (7,918 miles) and that ofRepublic of the Congo, Democratic Republic of the
mars is 6,779 km (4,220 miles). Congo, Uganda, Kenya, Somalia, Maldives, Indonesia
38. In which one of the following oceans is and Kiribati.
Diamantina Trench situated? 43. Which one of the following pairs is not
(a) Pacific Ocean (b) Atlantic Ocean correctly matched?
(c) Indian Ocean (d) Arctic Ocean Well-known Place Country
Ans. (c) : The Diamantina Trench is a deep sea trench (a) Baikonour : Russia
in the South East Indian Basin. With a maximum depth (b) Kourou : French Guiana
of 8,047 meters, it is the deepest trench of the Indian
(c) Borobudur : Indonesia
Ocean, deeper than the better-known Java Trench.
(d) Cannes : France
39. Which one of the following countries is the Ans. (a) : Baikonur is world's first and largest
leading producer of uranium? operational space launch facility. It is located in
(a) United States of America Kazakhstan and not in Russia.
(b) Canada 44. Other than India and China, which of the
(c) Germany following groups of countries border
(d) Zambia Myanmar?
(a) Bangladesh, Thailand and Vietnam
Ans. (b) : Canada is the largest producer of uranium,
(b) Cambodia, Laos and Malaysia
accounting for about 22% of world output. Production
(c) Thailand, Vietnam and Malaysia
of uranium in Canada is mainly from the Mc Arthur
(d) Thailand, Laos and Bangladesh
River mines in northern Saskatchewan province, which
is the largest mine in the world. Ans. (d) : Other than India and China; Thailand, Laos
• In 2020 leading producer of Uranium- Kazakhastan > and Bangladesh borders Myanmar.
Canada > Australia. 45. Huangpu River flows through which one of the
following cities?
40. Where is Bundala Biosphere Reserve which has
(a) Beijing (b) Ho chi Minh City
been recently added to the UNESCO's Man
(c) Shanghai (d) Manila
and Biosphere (MAB) network, located?
Ans. (c) : Formerly romanized as Whangpoo is a 113-
(a) Russia (b) India kilometer (70 mi) long river flowing through Shanghai
(c) Sri Lanka (d) Bangladesh that was first excavated and created by Lord Chunshen,
Ans. (c) : The Bundala Biosphere Reserve is located on one of the Four Lords of the Warring States. It is the
the Southeast Coast of Sri Lanka. The core area of last significant tributary of the Yangtze before it
Bundala Biosphere is national park and includes empties into the East China Sea.
brackish water lagoon. It has been added to UNESCO's 46. Claims to the historical Macedonian territory
Man and Biosphere. have been a bone of contention between which
41. What is the approximate mean velocity with of the following countries?
which the Earth moves round the Sun in its (a) Portugal and Spain
orbit? (b) Bulgaria and Greece
(a) 20 km/s (b) 30 km/s (c) Romania and Bulgaria
(c) 40 km/s (d) 50 km/s (d) Portugal and Greece
IAS (Pre) GS 2006 Paper I 293 YCT
Ans. (b) : Macedonian territory is disputed by Bulgaria 50. Assertion (A) : In India, every state has a
and Greece. North Macedonia, officially the Republic High Court in its territory.
of North Macedonia, is a country in the Balkan Reason (R) : The Constitution of India
Peninsula in Southeast Europe. It is one of the successor provides for a High Court in
states of Yugoslavia, from which it declared each state.
independence in September 1991 under the name (a) Both A and R are true and R is the correct
Republic of Macedonia.
explanation of A.
(b) Both A and R are true but R is not the correct
INDIAN CONSTITUTION AND POLTY explanation of A.
47. Which one of the following pairs is not (c) A is correct but R is wrong.
correctly matched? (d) A is wrong but R is correct.
(a) States Reorganization Act : Andhra Ans. (d) : Assertion is incorrect but reason is correct .
Pradesh
(b) Treaty of Yandabu : Assam Assertion is incorrect as it's not necessary that every
(c) State of Bilaspur : Himanchal state has a high court of its own. There are several
Pradesh examples of states which doesn't have respective high
(d) year 1966 : Gujarat court's of their own are :- Punjab , Mizoram, Goa, etc .
becomes a The states which doesn't have their own high court,
state shares jurisdiction of other High Courts.
Ans. (d) : Agitation by both Gujarati and Marathi 51. What does the 104th Constitution Amendment
nationalists for their own linguistic states led to the split Bill relate to?
of Bombay state on linguistic lines and on 1 May 1960, (a) Abolition of Legislative Councils in certain
two new states of Gujarat and Maharashtra were states
formed.
(b) Introduction of dual citizenship for persons of
48. Which one among the following commissions Indian origin living outside India
was set up in pursuance of a definite provision
(c) Providing quota to socially and educationally
under an Article of the Constitution of India?
(a) University Grants Commission backward classes in private educational
(b) National Human Rights Commission institutions
(c) Election Commission (d) Providing quota for religious minorities in the
(d) Central Vigilance Commission services under the Central Government
Ans. (c) : Election commission was set up in pursuance Ans. (c) : 104th Constitutional Amendment Bill is
of a definite provision under Article 324 of the related to provide quota to socially and educationally
Constitution of India. According to Article 324 of the backward classes in private educational institutions.
Constitution, Superintendence, direction and control of
52. Which one of the following subjects is under
elections are to be vested in an Election Commission.
the Union List in the Seventh Schedule of the
49. Consider the following statements :
1. A person who has held office as a permanent Constitution of India?
Judge of a High Court cannot plead or act (a) Regulation of labour and safety in mines and
in any court or before any authority in oilfields
India except the Supreme Court. (b) Agriculture
2. A person is not qualified for appointment as (c) Fisheries
a Judge of a High Court in India unless he (d) Public health
has for at least five years held a judicial
office in the territory of India. Ans. (a) : Regulation of labour and safety in mines and
Which of the statements given above is/are oilfields is under the Union List in the Seventh
correct? Schedule of the Constitution of India. The Concurrent
(a) 1 only (b) 2 only List or List-III (Seventh Schedule) is a list of 52 items
(c) Both 1 and 2 (d) Neither 1 nor 2 (though the last item is numbered 47) given in the
Ans. (d) : Statement 1 is incorrect because after Seventh Schedule to the Constitution of India. It
retirement, a permanent Judge of High Court shall not includes the power to be considered by both the central
plead or act in a Court or before any authority in India, and state government.
except the Supreme Court and a High Court other than 53. Consider the following statements:
the High Court in which he had held his office (article
220). Statement 2 is also incorrect as according to 1. There is no provision in the constitution of
article 217, a person is not qualified for appointment as India to encourage equal pay for equal work
a Judge of a High Court in India unless he has for at for both men and women.
least ten years held a judicial office in the territory of 2. The constitution of India does not define
India. backward classes.
IAS (Pre) GS 2006 Paper I 294 YCT
Which of the statements given above is/are Ans. (c) : 86th amendment to the Constitution of India
correct? made free and compulsory Education to the Children of
(a) 1 only (b) 2 only 6-14 years age group, a Fundamental Right. The
(c) Both 1 and 2 (d) Neither 1 nor 2 programme also seeks to provide computer education to
Ans. (b) : Statement 1 is incorrect as equal pay for bridge the digital divide. Sarva Shiksha Abhiyan covers
equal work for both men and women is provided under the entire country with special focus on educational
Article 39(d) in Directive Principles of the State Policy needs of girls, SC s/ ST s and other children in difficult
under Part IV of the Constitution. The constitution of circumstances. The SSA also seeks to provide computer
India does not define backward classes. education even in the rural areas. Effected in 1976
54. Consider the following statements: during the Emergency by then Prime Minister Indira
1. The Rajya Sabha alone has the power to Gandhi, the amendment restructured the Seventh
declare that it would be in national interest Schedule ensuring that State List subjects like
for the parliament to legislate with respect education, forest, protection of wild animals and birds,
to a matter in the State List. administration of justice, and weights and
2. Resolutions approving the Proclamation on measurements were transferred to the Concurrent List.
Emergency are passed only by the Lok 56. Who was the Chief Justice of India when
Sabha. Public interest litigation (PIL) was introduced
Which of the statements given above is/are to the Indian Judicial system?
correct. (a) M. Hidayatullah
(a) 1 only (b) 2 only (b) A. M. Ahmadi
(c) Both 1 and 2 (d) Neither 1 nor 2
(c) A. S. Anand
Ans. (a) : The Council of States (Rajya Sabha) is the (d) P. N. Bhagwati
Upper House of our Parliament. A lasting body, Rajya
Sabha isn't subjected to dissolution. The Constitution Ans. (d) : Public Interest Litigation is the judicial
gives that Rajya Sabha may pass a resolution, by a innovation which enables any person who is public-
larger part of at the very least two-thirds of the spirited or who wants to act on behalf of some people to
Members present and casting a ballot such that it is file a case in the court. The Concept of PIL was
fundamental or convenient in the national interest that introduced by Justice PN Bhagwati.
Parliament should make law as for any issue counted in
the State List. At that point, Parliament is engaged to ECONOMY
make a law regarding the matter indicated in the goals
for the entire or any piece of the domain of India. Such 57. Which one of the following statements is correct?
resolution stays in power for the greatest time of one Fiscal responsibility and Budget Management
year however this period can be reached out by one year Act (FRBMA) concerns:
on end by passing a further resolution. (a) fiscal deficit only
The proclamation of emergency must be approved by (b) revenue deficit only
both Houses of the Parliament, this is a constitutional (c) both fiscal deficit and revenue deficit.
requirement. (d) neither fiscal deficit nor revenue deficit.
55. Consider the following statements: Ans. (c) : Fiscal Responsibility and Budget
1. Free and compulsory education to the Management Act (FRBMA) concerns both fiscal deficit
children of 6-14 years age group by the State and revenue deficit. A fiscal deficit is a shortfall in a
was made a Fundamental Right by the 76th government's income compared with its spending. The
Amendment to the Constitution of India. government that has a fiscal deficit is spending beyond
2. Sarva Shiksha Abhiyan seeks to provide its means. A fiscal deficit is calculated as a percentage
computer education even in rural areas.
of gross domestic product (GDP), or simply as total
3. Education was included in the Concurrent
dollars spent in excess of income. A revenue deficit
List by the 42th Amendment, 1976 to the
occurs when realized net income is less than the
Constitution of India.
projected net income. This happens when the actual
Which of the statements given above are
amount of revenue and/or the actual amount of
correct?
expenditures do not correspond with budgeted revenue
(a) 1, 2 and 3 (b) 1 and 2 only
and expenditures.
(c) 2 and 3 only (d) 1 and 3 only
IAS (Pre) GS 2006 Paper I 295 YCT
Directions: The following six (6) items consist of two 61. Which one of the following is the correct
statements, one labelled as the 'Assertion (A)' and the statements?
other as 'Reason (R)'. Your are to examine these two Service tax is a/an
statements carefully and select the answers to these (a) direct tax levied by the Central Government
items using the code given below: (b) indirect tax levied by the Central Government
Code: (c) direct tax levied by the State Government
(a) Both A and R are individually true and R is (d) indirect tax levied by the State Government
the correct explanation of A Ans. (b) : Service Tax is an indirect tax levied by the
(b) Both A and R are individually true but R is Central Government
not the correct explanation of A
(c) A is true but R is false PHYSICS
(d) A is false but R is true
62. What does the term 'Dolby B' or 'Dolby C'
58. Assertion (A) : 'Balance of Payments' represents
printed on tape recorders and other sound
a better picture of a country's
systems refer to?
economic transactions with the
(a) Frequency modulated system
rest of the world than the
'Balance of Trade'. (b) Amplitude modulated system
(c) Noise reduction circuit
Reason (R) : 'Balance of Payments' takes into
account the exchange of both (d) Both DC and AC power can be used
visible and invisible items Ans. (c) Term 'Dolby B' or 'Dolby C' printed on tape
whereas 'Balance of Trade' does recorders and other sound systems refer to noise
not. reduction circuit.
Ans. (a) : Balance of payments of a nation consists of 63.
In which one among the following is the speed
visible account, invisible account and capital account. of sound maximum?
Whereas balance of trade = Export of goods – import of (a) Air at 0°C (b) Air at 100°C
goods. If imports of a country are more than exports, the (c) Water (d) Wood
deficit can be made up by invisibles remittance by NRI
Ans. (d) : The speed of sound is the distance travelled
and FDI investment. during a unit of time by a sound wave propagating
59. Which one of the following India banks is not a through an elastic medium. In dry air at 20 °C (68 °F),
the speed of sound is 343.2 metres per second (1,126
nationalized bank?
ft/s). In fresh water, sound travels at about 1497 m/s at
(a) Corporation Bank 25 °C. The value of 331.3 m/s which represents the 0 °C
(b) Dena Bank speed. The speed of sound at 100 °C is 386 m/s. The
(c) Federal Bank speed of sound in wood is 3300 - 3600 m/s.
(d) Vijaya Bank
Ans. (c) : Among the given banks, Federal bank is not a CHEMISTRY
nationalized bank.
64. Which are the material generally employed as
60. Consider the following statements: solder in soldering operation in electronics?
1. In India, during the financial year 2004-2005 (a) Iron and tin
an increase of below 10% over the value of (b) Lead and tin
exports (in rupee terms) in the financial (c) Aluminium and lead
year 2003-2004 was reported. (d) Aluminium and iron
2. According to the WTO, India's share in the Ans. (b) : Solder is a fusible metal alloy with a melting
point or melting range of 90 to 450 degree Celsius (190
world merchandise exports crossed 2% in
to 840 °F), used in a process called soldering where it is
the year 2005. melted to join metallic surfaces. It is especially useful in
Which of the statements given above is/are electronics and plumbing. Solder is an alloy of lead and
correct? tin.
(a) 1 only (b) 2 only 65. Which one of the following is printed on a
(c) Both 1 and 2 (d) Neither 1 nor 2 commonly used fluorescent tube light?
(a) 220 K (b) 273 K
Ans. (d) : Neither 1 nor 2 (c) 6500 K (d) 9000 K
IAS (Pre) GS 2006 Paper I 296 YCT
Ans. (c) : A fluorescent lamp or fluorescent tube is a BIOLOGY
gas discharge lamp that uses electricity to excite
68. Consider the following statements:
mercury vapour. The excited mercury atoms produce 1. ELISA test is employed as the first and most
shortwave ultraviolet light that then causes a phosphor basic test for an individual to detect cancer.
to fluoresce, producing visible light. 6500 K is usually 2. Almost 50% of human beings have Rh+
printed on fluorescent tubelight. blood while the remaining have Rh- blood.
Which of the statements given above is/are
66. Consider the following chemicals: correct?
1. Benzene (a) 1 only (b) 2 only
2. Carbon tetrachoride (c) Both 1 and 2 (d) Neither 1 nor 2
3. Sodium carbonate Ans. (d) : ELISA (Enzyme-Linked Immunosorbent
Assay) is an important test for diagnosing viral diseases
4. trichloroethylene like AIDS (caused by HIV). Majority of the human
Which of the above is/are used as dry cleaning beings are Rh+ while very few are Rh–.
chemicals? 69. Consider the following statements with
(a) 1 only reference to human body:
1. The common bile duct releases its contents
(b) 2 only
into stomach.
(c) 1, 2 and 4 only 2. The pancreatic duct release its contents into
(d) 1, 2, 3 and 4 duodenum
Ans. (c) : Dry cleaning is a cleaning process for Which of the statements given above is/are
correct?
clothing and textiles involves using a chemical solvents
(a) 1 only (b) 2 only
other then water. The solvent used is typically (c) Both 1 and 2 (d) Neither 1 nor 2
trichloroethylene while carbon tetra chloride and tri Ans. (b) : Bile duct and pancreatic duct both releases its
chloroethane is used historically. Benzene is other contents into duodenum. The digestive enzyme
common agent used for this purpose. (pancreatic juices) releases to duodenum from
pancreatic duct.
67. Consider the following statements:
70. Which one among the following is not a good
1. Caffeine, a constituent of tea and coffee is a source of nutritional calcium?
diuretic. (a) Rice (b) Ragi
2. Citric acid is used in soft drinks. (c) Skimmed milk (d) Egg
3. Ascorbic acid is essential for the formation of Ans. (a) :
Rice is not a good source of nutritional calcium. It has
bones and teeth.
only 10 mg calcium in 100 mg of rice.
4. Citric acid is a good substitution for ascorbic 71. Assertion (A) : Cellulose is used in making
acid in our nutrition. shatter-proof glass.
Which of the statements given above are Reason (R) : Polysaccharides are not soluble
correct? in water.
(a) Both A and R are true and R is the correct
(a) 1 and 2 only (b) 1, 2 and 3 only
explanation of A.
(c) 3 and 4 only (d) 1, 2, 3 and 4
(b) Both A and R are true but R is not the correct
Ans. (d) : Caffeine also spelled caffei is most explanation of A.
commonly consumed by humans in infusions extracted (c) A is correct but R is wrong.
from the bean of the coffee plant and the leaves of the (d) A is wrong but R is correct.
tea bush, as well as from various foods and drinks Ans. (a) : Cellulose is not soluble in water that is why it
containing products derived from the kola nut. Acids is used in making shatter–proof glass. Cellulose is a
poly saccharide.
are added to soft drinks for extra bite and mouth feel.
The primary acid used in colas is phosphoric acid, while 72. What is the name of the vessel that delivers the
nutrient rich blood from the stomach and small
the one used in citrus flavoured drinks is usually citric
intestine to the liver?
acid. Citric acid is a good substitution of ascorbic acid (a) Left hepatic artery (b) Hepatic vein
as per nutritional value. (c) Right hepatic artery (d) Hepatic portal vein
IAS (Pre) GS 2006 Paper I 297 YCT
Ans. (d) : Hepatic portal vein deliver the nutrient rich 75. Consider the following statements in respect of
blood from the stomach and small intestine to the liver. India's advanced satellite INSAT-4A:
It is the largest gland in the body. 1. INSAT-4A was launched in December, 2005
from the Mexico
73. Consider the following statements:
2. The European Commercial launch service
1. Meningococcal Meningitis is transmitted provider Arianespace was associated with
from person to person by mosquito bites. the launch of INSAT-4A.
2. Vomiting and neck pain area two of the 3. Tata Sky- a digital cable service provider is
symptoms of Meningococcal Meningitis. associated with DTH television broadcasting
Which of the statements given above is/are services from INSAT-4A
correct? Which of the statements given above are
correct?
(a) 1 only (b) 2 only
(a) 1, 2 and 3 (b) 1 and 2, only
(c) Both 1 and 2 (d) Neither 1 nor 2
(c) 2 and 3, only (d) 1 and 3, only
Ans. (b) : Meningococcal meningitis is transmitted from Ans. (c) : INSAT-4A was launched at French Guyana,
person to person through cough and having a body contact in December, 2005 by the European Ariane launch
on a regular basis. It is called droplet transmission. vehicle, INSAT-4A which is positioned at 83 E
longitude along with GSAT-12, has high power Ku-
CURRENT AFFAIRS band transponders with footprint covering Indian
74. Consider the following statements: mainland and C-band transponders covering Indian
1. Appellate Tribunal for Electricity has been geographical boundary and area beyond India in South
established by each State Government in East and North West regions. Tata Sky- a digital cable
India. service provider is associated with DTH television
2. One of the components of the Accelerate broadcasting services from INSAT-4A.
Power Development and Reforms 76. Consider the following statements:
Programme (APDRP) is up-gradation of sub- 1. Kofi Annan-the UN Secretary General is
transmission and distribution system for from Nigeria.
electricity in India. 2. Kofi Annan is the first UN Secretary
Which of the statements given above is/are General to be appointed from the ranks of
correct? the United Nations staff.
(a) 1 only (b) 2 only 3. Kofi Annan was appointed for a second
(c) Both 1 and 2 (d) Neither 1 nor 2 term to the office of the UN Secretary
Ans. (b) : The Appellate Tribunal for Electricity was General
constituted in the year 2005 to hear appeals against the Which of the statements given above are
orders of the adjudicating officer or the Central and correct?
State Electricity Regulatory Commissions under the (a) 1, 2 and 3 (b) 1 and 2, only
Electricity Act, 2003. The second appeal lies before the (c) 2 and 3, only (d) 1 and 2, only
Supreme Court on substantial questions of law Ans. (c) : The seventh Secretary-General of the United
only. The Appellate Tribunal for Electricity comprises a Nations was Kofi Annan of Ghana. As the first
Chairperson who has been a Judge of the Supreme Secretary-General to be elected from the ranks of the
Court or Chief Justice of a High Court, one judicial staff of the United Nations, his term began on 1 January
member who has been or qualified to be a judge of High 1997. By acclamation, the General Assembly, acting on
Court, two technical members who are electricity sector a recommendation by the Security Council on 29 June
experts and one technical member who is an expert 2001, appointed him to a second term of office,
from petroleum and natural gas sector. Principal bench beginning on 1 January 2002 and ending on 31
of The Appellate Tribunal for Electricity is in Delhi December 2006.
while there are three circuit benches at Kolkata, 77. Which one of the following countries is not a
Mumbai and Delhi. Hence, statement 1 is not correct. participating team in the Football World Cup-
Accelerated Power Development Reforms Program 2006 to be held in Germany in July, 2006?
(APDRP) is intended for up gradation of sub (a) Russia (b) Togo
transmission and distribution in densely electrified (c) Ivory Coast (d) Switzerland
zones in the urban and industrial areas and improvement Ans. (a) : The 2006 FIFA World Cup was the 18th
in commercial viability of State Electricity Boards by FIFA World Cup, the quadrennial international football
reducing the aggregate technical and commercial losses world championship tournament. It was held from 9
to around 15 percent as against the existing over 50 June to 9 July, 2006 in Germany. Russia did not
percent. Hence, statement 2 is correct. participate in this event.
IAS (Pre) GS 2006 Paper I 298 YCT
78. What is Medicines Sans Frontiers (MSF)? 82. Which one of the following pair is not correctly
(a) An agency formed by the International matched?
Olympic Committee (IOC) to check misuse
Recent Scientific Country
of drugs by sportspersons
(b) A non-government organization which Achivement
specializes in international humanitarian aid (a) Creating the world : United Kingdom
and emergency medical assistance first mouse with a full
(c) An organization to develop applications of human chromosome
nanotechnology in medicine.
(d) An organization of medical practitioners (b) Cloning a human : Germany
funded by the European Union which carries embryo for the first
out research against spread of AIDS. time
Ans. (b) : Medicines Sans Frontiers (MSF) translates to (c) Guiding a spacecraft : United States of
Doctors without Borders. It provides medical assistance America for collision
to people affected by conflict, epidemics, disasters, or
with a comet
exclusion from healthcare. MSF was founded in 1971 in
Paris by a group of journalists and doctors. (d) Landing a spacecraft : Japan
79. In which one of the following films did the on an asteroid
Nobel Laureate Peari S. Buck Collaborate? Ans. (b) : Cloning a human embryo for the first time
(a) Shatranj Ke Khilari
(b) Guide took place in U.SA. Cloning is a process when identical
(c) Lawrence of Arabia types of progenies are produced from one individual.
(d) Titanic 83. Who among the following was presented with
Ans. (b) : The script of the film Guide had been written the Sangeet Natak Akademi Ratna Award,
by Nobel Laureate Pearl S Buck who was signed for the 2005 for her lifetime achievement is dance?
film perhaps for no other reason but the fact that she
was a bestselling writer worldwide whose books set in (a) Chandralekha
China were also known in India. (b) Tenjavur Balasaraswati
80. Which one among the following was the first to (c) Uma Sharma
legalize euthanasia? (d) Yamini Krishnamurthy
(a) Austria (b) Switzerland
(c) Netherlands (d) Canada Ans. (a) : Chandralekha, Bharatnatyam dancer and
Ans. (c) : The Netherlands became the first country in Choreographer was conferred with Sangeet Natak
the world to formally legalize euthanasia, when the Akademi Ratna Award, 2005 for her contribution to
lower house of parliament approved a "mercy killing" contemporary Indian dance.
Bill by a two-thirds majority. 84. Which one of the following countries is the first
81. Consider the following statements: country in the world to propose a carbon tax
1. The agreement on South Asian Free Trade for its people to address global warming?
Area (SAFTA) came into effect from 1st
(a) Australia (b) Germany
December, 2005.
2. As per SAFTA agreement terms, India, (c) Japan (d) New Zealand
Pakistan and Sri Lanka have to decrease Ans. (d) : New Zealand is the first country in the world
their custom duties to the level of 0 to 5 to propose a carbon tax for its people to address global
percent by the year 2013. warming in the year 2005.
Which of the statements given above is/are 85. Consider the following statements:
correct? 1. Petronet LNG Ltd. is setting up another
(a) 1 only (b) 2 only LNG terminal at Mangalore.
(c) Both 1 and 2 (d) Neither 1 nor 2 2. The Head Office of the Dredging
Ans. (d) : The South Asian Free Trade Area (SAFTA) Corporation of India is at Visakhapatnam.
is the free trade arrangement of the South Asian 3. The Narwapahar Mine is operated by the
Association for Regional Cooperation (SAARC). The Uranium Corporation of India Limited.
agreement came into force in 2006, succeeding the 1993 Which of the statements given above are
SAARC Preferential Trading Arrangement. SAFTA correct?
signatory countries are Afghanistan, Bangladesh, (a) 1, 2 and 3 (b) 1 and 2, only
Bhutan, India, Maldives, Nepal, Pakistan and Sri Lanka.
(c) 2 and 3 only (d) 1 and 3 only
In the first phase of the two-year period ending in 2007,
SAFTA requires developing countries in South Asia Ans. (c) : Petronet LNG Ltd. is setting up another LNG
(India, Pakistan and Sri Lanka) to bring their duties terminal at Kochi, Kerala. Hence first statement is not
down to 20 percent. correct. Rest of the two statements is correct.

IAS (Pre) GS 2006 Paper I 299 YCT


86. Consider the following statements in respect of Ans. (a) : The Indira Gandhi Centre has established a
welfare schemes launched by the Ministry of comprehensive R&D infrastructure over the entire range
Rural Development, Government of India: of FBR technology, spanning reactor engineering,
1. Drinking water for rural areas is one of the metallurgy and materials, chemistry of fuels and
components of the Bharat Nirman Plan. materials, fuel reprocessing and reactor safety studies.
Development of the indigenous plutonium-uranium
2. In the National Rural Employment
mixed carbide nuclear fuel for the fast breeder test
Guarantee Act, 2005 there is a provision that reactor. The fuel has performed satisfactorily and
the statutory minimum wage applicable to produced nuclear electricity from a fast reactor for the
agricultural workers in the State has to be paid first time in India.
to the workers under the Act. 89. Consider the following statements in respect of
Which of the statements given above is/are the National Rural Employment Guarantee
correct? Act, 2005 :
(a) 1 only (b) 2 only 1. Under the provisions of the Act, 100 days of
(c) Both 1 and 2 (d) Neither 1 nor 2 employment in a year to every household
Ans. (c) : Statement 1 as well as statement 2 regarding whose adult members volunteer to do
the welfare schemes launched by Ministry of Rural unskilled manual work has become a
Development are correct. fundamental right.
2. Under the provisions of the Act, Women are
87. What is Indo Next which was launched in
to get priority to the extent that one-half of
January, 2005?
persons who are given employment are
(a) A new scheme to promote Indian tourism
women who have asked for work.
(b) A new scheme to promote export of Indian
Which of the statements given above is/are
handicrafts
correct?
(c) An association of the Non-Resident Indians to
(a) 1 only (b) 2 only
organize Pravasi Bhartiya Divas every year in
(c) Both 1 and 2 (d) Neither 1 nor 2
India
Ans. (d) : Both the given statements are incorrect.
(d) An alternative trading platform being
Provision given in statement 1 is a legal guarantee not
promoted by the Bombay Stock Exchange the Fundamental Right. In statement 2 it is not 1/2 but it
and Regional Stock Exchanges is 1/3 women.
Ans. (d) : BSE Indo Next is a joint initiative of BSE 90. Whom did Croatia defeat in the Davis Cup-
and Participating RSEs (Regional Stock Exchanges). In 2005 Finals to win the Davis Cup-2005?
order to instill higher sense of involvement on the issues (a) United States of America
relating to management, business development, (b) Slovak Republic
marketing and general operations of the BSE IndoNext (c) Agrentina
platform, a "BSE Indo Next Council" has been (d) Spain
constituted. This Council's duties have been defined in Ans. (b) : The 2005 Davis Cup was the 94th edition of
the By-laws of BSE and participating RSEs and operate the most important tournament between nations in
under the overall supervision and control of the men's tennis. The final took place on 2-4 December at
Governing Board of BSE. the Sibamac Arena in Bratislava, Slovakia, with Croatia
defeating Slovakia for their first title.
88. In which one of the following areas did the 91. Indian Airlines (New name : Indian) have
Indira Gandhi Centre of Atomic Research redesigned their logo which is a graphic wheel.
make significant progress in the year 2005? This logo has been inspired from which one of
(a) Reprocessing the uranium-plutonium mixed the following?
carbide fuel of the Fast Breeder Test Reactor (a) Hampi Temples
(b) New applications of radioisotopes in (b) Mamalapuram Temple
metallurgy (c) Sun Temple, Konark
(c) A new technology for production of heavy (d) Khajuraho Temple
water Ans. (c) : After the name change to Indian, the
(d) A new technology for high level nuclear company's aircraft sported a new look inspired by the
waste management Sun Temple at Konark in Odisha.
IAS (Pre) GS 2006 Paper I 300 YCT
92. Where is Davos-the venue of the annual Ans. (b) : Recently Uttar Pradesh and Madhya Pradesh
meeting of World Economic Forum, located? Governments signed a memorandum of understanding
for the linking of river Betwa and Ken. Both Ken and
(a) France (b) Germany
Betwa are the tributaries of Yamuna. According to the
(c) Switzerland (d) Luxembourg proposal, 1020 million cubic metres of water will be
Ans. (c) : Davos is host to annual meeting of World diverted from Ken Basin in a link canal that will be 231
Economic Forum, an annual meeting of global political km long. The canal will transverse both states to
and business elites and the home of huge ski resort is a provide irrigation and drinking water.
town of Switzerland. 97. With reference to the Government of India's
various programmes, what is Nirmal Gram
93. Recently with which country's government did Puraskar?
ONGC-Mittal Energy Limited (OMEL) sign a (a) It is an incentive scheme of scholarships for
joint venture deal of $6 billion dealing with the single girl child in families in villages.
infrastructure, refinery and power? (b) It is an incentive scheme of scholarships for
female sportspersons from villages who
(a) Colombia (b) Venezuela represent their states in any game.
(c) Nigeria (d) Saudi Arabia (c) It is an incentive scheme for schools in the
Ans. (c) : The right option is (c) Nigeria. villages for computer education.
(d) It is an incentive scheme for Panchayati Raj
94. Which one among the following was awarded institutions.
the CSIR Diamond Jubilee Technology Award Ans. (d) : Nirmal Gram Puraskar is associated with
2004 in September, 2005 by the Prime Minister rural sanitation, the award was first given in 2005.
of India? 98. What is the new administrative capital
(a) Infosys Technology Ltd. proposed for Myanmar?
(a) Bassein (b) Mandalay
(b) Midas Communication Technologies (c) Myitkyina (d) Pyinmana
(c) Tata Consultancy Services Ans. (d) : Pyinmana is a logging town and sugarcane
(d) Biocon India Ltd. refinery center in the Naypyidaw Union Territory of
Ans. (b) : The "Midas Communications Technologies Myanmar. The administrative capital of Myanmar was
officially moved to a militarized Greenfield site two
Private Limited" has been conferred with the CSIR miles west of Pyinmana on November 6, 2005.
Diamond Jubilee Technology Award 2004 for design,
99. Consider the following statements:
development and commercialization of core DECT 1. Sikkim has the minimum area among the 28
WLL (Wireless in Local Loop) access system and Indian States (Delhi and Pondicherry not
optima, a Fiber-in-Local Loop (FLL) access system. included).
Midas Communication developed these products in 2. Chandigarh has the highest literacy rate
collaboration with TeNeT (Telecommunication and among Pondicherry, NCT of Delhi and
Computer Network), lIT Madras (Chennai). other Union Territories.
95. Who among the following directed the film chosen 3. Maharashtra has the highest population after
Uttar Pradesh among the 28 Indian States
as India/s official entry for the year 2006 Academy
(Delhi and Pondicherry not included).
Awards? Which of the statements given above is/are
(a) Adoor Gopalakrishnan correct?
(b) Amol Palekar (a) 1 and 2 (b) 2 and 3
(c) Sanjay Leela Bhansali (c) 1 only (d) 3 only
(d) Kunal Kohli Ans. (d) : Maharashtra is ranked 2nd after Uttar
Pradesh among 28 Indian states. So, option '3' is correct.
Ans. (b) : Amol Palekar's directed film Paheli was But option '1' and '2' are incorrect, because Goa is the
India's official entry for Best Foreign Film at the 2006 smallest state based on area (i.e., 3702 sq km), not the
Oscars. Sikkim. Highest literacy rate is of Union territory is
96. Recently Uttar Pradesh and Madhya Pradesh Lakshadweep i.e. 86.7%.
Governments signed a Memorandum of 100. What was the main reason for which Niger was
Understanding for the linking of two rivers as a in the news in recent times?
link project. Which are these two rivers? (a) Many deaths due to AIDS
(b) Fierce civil war
(a) Betwa and Chambal (c) Intense fighting with Algeria
(b) Betwa and Ken (d) Famine and starvation
(c) Chambal and Son Ans. (d) : Niger was in the news recently for famine
(d) Ken and Narmada and starvation.

IAS (Pre) GS 2006 Paper I 301 YCT


MISCELLANEOUS Ans. (c) : CMYK refers to the primary colors of
pigment: Cyan, Magenta, Yellow, and key (Black).
101. Who is the President of the Council of These are the inks used on the press in "4-color process
Scientific and Industrial Research? printing", commonly referred to as "full color printing"
(a) President of India or "four color printing".
(b) Vice-President of India 105. Consider the following statements:
(c) Prime Minister of India 1. The Nobel Price Awarding Ceremony takes
(d) Union Minister of Science and Technology place on December 10 of every year
Ans. (c) : Prime Minister of India is the chairman of the 2. The Nobel Price for Literature was added
Council of Scientific and Industrial Research. Council later on the other five areas-Physics,
of Scientific and Industrial Research (CSIR) established Chemistry, Physiology or Medicine, peace
in 1942, is an autonomous body and India’s largest and Economics.
research and development (R&D) organisation, with 37 Which of the statements given above is/are
laboratories and 39 field stations or extension centres correct?
spread across the nation, with a collective staff of over (a) 1 only (b) 2 only
17,000. It operates as an autonomous body registered (c) Both 1 and 2 (d) Neither 1 nor 2
under the Registration of Societies Act of 1860.
Ans. (a) : Since 1901, the Nobel Prizes have been
102. Which one of the following countries is not a presented to the Laureates at ceremonies on 10
member of ASEAN? December every year, the anniversary of Alfred Nobel's
(a) Vietnam death. As stipulated in the will of the Swedish-born
(b) Brunei Darussalam inventor and international industrialist Alfred Nobel,
(c) Bangladesh which was opened after his death in 1896, the Nobel
(d) Myanmar Prizes in Physics, Chemistry, Physiology or Medicine
Ans. (c) : The Association of Southeast Asian Nations and Literature are awarded in Stockholm, Sweden,
(ASEAN) is a regional intergovernmental organization while the Nobel Peace Prize is awarded in Oslo,
comprising ten countries in Southeast Asia. It aimed Norway.
primarily at promoting economic growth and regional 106. A watch showed a time of fourteen minutes
stability among its members. There are currently 10 past nine (9 hrs and 14 minutes). The positions
member states: Indonesia, Malaysia, Philippines, of the hour-hand and the minute-hand of the
Singapore, Thailand, Brunei, Laos, Myanmar, watch are exactly interchanged. The new time
Cambodia and Vietnam. shown by the watch is closest to which one of
the following?
103. Match List-I (Person) with List-II
(a) Twelve minutes to three
(Organization/Area of work) and select the
correct answer using the code given below the (b) Thirteen minutes to three
Lists: (c) Fourteen minutes to three
List-I List-II (d) Fifteen minutes to three
(Person) (Organization Ans. (c) : According to question, if the time is 9 : 14
/Area of Work) then
A. Chanda Kochhar 1. Advertising
B. Amrita Patel 2. Banking
C. Indra Nooyi 3. Diary Development
D. Piyush Pandey 4. Pepsi Co
Codes:
A B C D
(a) 2 1 4 3
(b) 4 3 2 1 and when the hour and minute hands are interchanged
then
(c) 2 3 4 1
(d) 4 1 2 3
Ans. (c) : A-2; B-3; C-4, D-1
104. To which one of the following processes in the
term CMYK related?
(a) Railway Signaling
(b) Navigation
(c) Offset printing
(d) Electronic voting machine The new time would be close to 14 minutes to three.

IAS (Pre) GS 2006 Paper I 302 YCT


107. Which of the following pairs is/are correctly 110. Who is Wole Soyinka?
matched? (a) A well-known economist
Inventor Invention (b) A well-known football player
1. Christopher Cockerell : Hovercraft (c) A well-known industrialist owning steel
2. David Bushnell : Submarine plants in many parts of the world
3. J.C. Perrier : Steamship (d) A Nobel Prize winner for literature
Select the correct answer by using the code Ans. (d) : Akinwande Oluwole "Wole" Babatunde
given below : Soyinka is a Nigerian playwright and poet. He was
(a) 1, 2 and 3 (b) 1 and 2, only awarded the 1986 Nobel Prize in Literature, the first
(c) 2 and 3, only (d) 1 and 3, only African to be honored in that category.
Ans. (a) : The Hovercraft invented by Christopher 111. In a tournament each of the participants was to
Cockerell is officially launched in Southampton. Sir play one match against each of the other
Christopher Cockerell invented the hovercraft, the participants. 3 players fell ill after each of them
vehicle which can travel across land or water on a had played three matches and had to leave the
cushion of air. David Bushnell is credited as the tournament. What was the total number of
inventor of the Submarine, which was first used to participants at the beginning, if the total
launch explosives against British ships during the number of matches played was 75?
American Revolution (1775-83). Steam ship was (a) 8 (b) 10
invented by J.C Perrier in the year 1775. (c) 12 (d) 15
108. Who among the following wrote "The Ans. (d) : Let the total no. of participants be x at the
Communist Manifesto" along with Karl Marx? beginning.
(a) Emile Durkheim After 3 players fell in, remaining participants = x – 3
(b) Friedrich Engels Now,
x −3
(c) Robert Owen C 2 + 3×3 = 75
(d) Max Weber ( x − 3)!
Ans. (b) : The Communist Manifesto was written by + 9 = 75
2!( x − 5)!
Karl Marx with the assistance of Friedrich Engels.
x2 – 7x – 120 = 0
109. (x + 8) (x – 15) = 0
Neglecting the negative value of x we get
x = 15
112. P, Q, R, S and T reside in a 5-storeyed (Ground
+ 4) building and each of them resides on a
separate floor. Further,
1. T does not reside on the topmost floor.
2. Q does not reside on the ground floor.
3. S resides on one storey above that of P and
one storey below that of R.
Each of 8 identical balls is to be placed in the To know as to which one of the 5 persons
squares shown in the figure given above in a resides on the ground floor which of the above
horizontal direction such that one horizontal statements are sufficient/insufficient?
row contains 6 balls and the other horizontal (a) 1 and 3 are sufficient
row contains 2 balls. In how many maximum (b) 2 and 3 are sufficient
different ways can this be done? (c) 1, 2 and 3 are sufficient
(a) 38 (b) 28 (d) 1, 2 and 3 are insufficient
(c) 16 (d) 14 Ans. (d) : On the basis of the facts given in the
Ans. (a) : There can be two cases: question, there can be 3 arrangements.
(i) when x row contains 6 balls: 1 2 3
Then the 2 balls can be arranged in y row in 6p2 R Q Q
ways = 15 or the 2 balls can be arranged in any of
he 4 two box row in 4 ways. S R T
So, no of ways, when x contains 6 balls = 15 + 4 P S R
= 19 Q P S
(ii) Similarly, no. of ways, when y row contains 6 balls
= 19 T T P
As, either of case (i) or case (ii) is possible, As either of T or P can reside on the ground floor, so to
Hence, total no. of ways = 19 + 19 exactly find who resides on the ground floor, given
= 38 Ans. statements are insufficient.

IAS (Pre) GS 2006 Paper I 303 YCT


113. A box contains 5 sets of balls while there are 3 116. In an office, the number of persons who take
balls in each set. Each set of balls has one tea is twice the number of persons who take
colour which is different from every other set.
What is the least number of balls that must be only coffee. The number of persons who take
removed from the box in order to claim with coffee is twice the number of persons who take
certainty that a pair of balls of the same colour only tea.
has been removed? Consider the following statements:
(a) 6 (b) 7
(c) 8 (d) 9 1. The sum of the number of persons who take
Ans. (a) : Set 1 2 3 4 5 either tea or coffee or both is four times the
number of persons who take both coffee and
Balls 1 1 1 1 1
tea.
Now any further removal of balls from any set will
ensure that removed ball is of the same colour as one of 2. The sum of the number of persons who take
the already removed balls, thus constituting a pair of the only coffee and those who take only tea is
removed balls of the same colour. twice the number of persons who take both
Hence, minimum no. of removed balls = 6
tea and coffee.
114. There are three parallel straight lines. Two
points A and B are marked on the first line, Which of the statements given above is/are
points C and D are marked on the second line correct?
and points E and F are marked on the third (a) 1 only (b) 2 only
line. Each of these 6 points can move to any
position on its respective straight line. (c) Both 1 and 2 (d) Neither 1 nor 2
Consider the following statements: Ans. (c) : Let x number of persons take tea,
1. The maximum number of triangles that can y number of persons take coffee and
be drawn by joining these points is 18. z number of persons take both coffee and tea.
2. The minimum number of triangles that can
be drawn by joining these points is zero. Now,
Which of the statements given above is/are x + z = 2y …….(i)
correct? y + z = 2x …….(ii)
(a) 1 only (b) 2 only
From equation (i) and (ii)
(c) Both 1 and 2 (d) Neither 1 nor 2
Ans. (b) : Maximum number or triangles can be formed x = y = z
by selecting 3, 4 or 5 points out of 6 at a time. So
∴ Maximum number of triangles = C3 + C 4 + C5 = 41 x + y + z = z + z + z = 3z
6 6 6

Which is more than 18. Hence 1st statement is incorrect. Thus 1st statement is wrong
Now, minimum no. of triangle can be zero, when the Again
points overlap on the same line and also when all points x + y = z + z = 2z
are along the same vertical line. Hence 2nd statement is
Hence 2nd statement is correct.
correct.
115. A mixed doubles tennis game is to be played 117. 3 digits are chosen at random from 1, 2, 3, 4, 5,
between two teams (each team consists of one 6, 7, 8 and 9 without repeating any digit. What
male and one female.) There are 4 married is the probability that their product is odd?
couples. No team is to consist of a husband and
his wife. What is the maximum number of (a) 2/3 (b) 5/108
games that can be played? (c) 5/42 (d) 7/48
(a) 12 (b) 21 Ans. (c) : Total number of 3-digit no. = 9 × 8 × 7 = 504
(c) 36 (d) 42
Ans. (d) : Let married couples are – For product to be odd, we have to choose only from odd
MF MF MF MF numbers.
∴ Total no. of 3-digit no. whose product are odd
ab cd ef and gh
= 5 × 4 × 3 = 60
∴ possible teams ad cb eb gb af cf ed gd
60 5
ah ch eh and gf ∴ Required probability = =
Now team ad can play with cd, cg, cb, eb, eh, gb and gf 504 42
i.e. 7 118. 16 64 64
The same will be the case with other teams i.e. 7
So no of total matches 12 × 7 = 84 9 18 25 25 18 1 9 18 1
Since every match includes two teams, 36 16 X
84 (I) (II) (III)
The number of matches = = 42 What is the value of X in figure III?
2
IAS (Pre) GS 2006 Paper I 304 YCT
(a) 4 (b) 16 122. Who among the following is a Hindustani
(c) 25 (d) 36 classical singer?
Ans. (d) : In every figure the sum of square route of (a) Geeta Chandran (b) Leela Samson
corner digit comes in middle box. (c) Gangubai Hangal (d) Swapnasundari
Here, underfoot of 9 = 3 Ans. (c) : Gangubai Hangal was a Hindustani classical
And, underfoot of 64 = 8, underfoot of 1 = 1 singer.
Now, 3 + 8 + 1 = 12 123. Match List-I (Person) with List-II (Area of
Then, 18 – 12 = 6 Distinction) and select the correct answer using
So, X = (6)2 the code given below the Lists:
= 36 Ans. List-I List-II
119. Consider the following statements: (Person) (Area of Distinction)
1. According to the Census 2001, Kerala has A. E.C.G. Sudarshan 1. Theatre
the smallest gap in male and female literacy B. V. Shanta 2. Violin
rates among the 28 states of India (Delhi
C. T.N. Krishanan 3. Research in Physics
and Pondicherry not included).
D. Mahesh Dattani 4. Cancer treatment
2. According to the Census 2001, Rajasthan
5. Water harvesting
has literacy rate above the national average
Codes:
literacy rate.
A B C D
Which of the statements given above is/are
(a) 3 1 2 4
correct?
(a) 1 only (b) 2 only (b) 2 4 5 1
(c) 3 4 2 1
(c) Both 1 and 2 (d) Neither 1 nor 2
(d) 2 1 5 4
Ans. (d) : Mizoram is the state where male literacy is
90.7% and female literacy is 86.7%. So, Mizoram is the Ans. (c) : A-3; B-4; C-2; D-1
state of smallest gap in male and female literacy, where A. E.C.G. Sudarshan Research in Physics
Kerala has 94.2% in male and 87.7% in female. In B. V. Shanta Cancer treatment
statement 2 literacy of Rajasthan is 60.4%, where C. T.N. Krishanan Violin
national literacy is 64.8%. D. Mahesh Dattani Theatre
120. Zayed bin Sultan-al-Nahyan was a long serving 124. What is the Universal Product Code (UPC)
President of which of the following?
adopted for?
(a) Oman
(a) Fire safety code in buildings
(b) Kuwait (b) Earthquake-resistant building code
(c) United Arab Emirates
(c) Bar code
(d) Saudi Arabia
(d) Against adulteration in eatables
Ans. (c) : Zayed bin Sultan Al Nahyan was the Ans. (c) : The Universal Product Code UPC is a
principal driving force behind the formation of the
barcode symbology i.e. a specific type of barcode that is
United Arab Emirates (UAE), the ruler of Abu Dhabi
widely used in the United States, Canada, United
and first President of the United Arab Emirates, a post
Kingdom, Australia, New Zealand and in other
which he held for over 33 years (1971-2004).
countries for tracking trade items in stores.
121. In which State is the Rajiv Gandhi National
125. Consider the following statements:
Institute of Youth Development located?
(a) Tamil Nadu (b) Karnataka 1. The Richter scale is a logarithmic scale and
(c) Himachal Pradesh (d) Uttaranchal so an increase of 1 magnitude unit represents a
Ans. (a) : The Rajiv Gandhi National Institute of Youth factor of 10 times in amplitude.
Development, Sriperumbudur, Tamil Nadu, is an 2. Each integer reading of the Richter scale has
Institution of National Importance by the Act of an energy 100 times that of the previous integer
Parliament No. 35/2012 under the Ministry of Youth reading.
Affairs & Sports, Government of India. The Rajiv Which of the statements given above is/are
Gandhi National Institute of Youth Development was correct?
set up in 1993 under the Societies Registration Act, (a) 1 only (b) 2 only
XXVII of 1975. (c) Both 1 and 2 (d) Neither 1 nor 2
IAS (Pre) GS 2006 Paper I 305 YCT
Ans. (a) : The magnitude of intensity of energy released Ans. (d) : Lectures from Colombo to Almora is a book
by an earthquake is measured by Richter Scale. The of Swami Vivekananda based on the lectures he
number indicating magnitude on Richter Scale ranges delivered in Srilanka and India after his return from the
between 0 and 9 but infact the scale has no upper West.
limited of number because it is a logarithmic scale. But 130.
each integer reading of the Richter Scale has in energy
32 times of previous integer reading. So statement 2 is
not correct.
126. For which sport is the Val Barker Cup
awarded in the Olympic Games?
Each of the six faces of a cube is numbered by
(a) Swimming (b) Boxing
one of the six digits from 1 to 6. This cube is
(c) Long Jump (d) High Jump shown in its four different positions in the
Ans. (b) : Val Barker Cup is awarded for boxing in the figures I, II, III and IV. Consider the following
Olympic Games. statements:
127. How many numbers are there in all from 6000 1. Figures II and III are sufficient to know as to
to 6999 (Both 6000 and 6999 included) having which face is opposite to the face numbered 6.
at least one of their digits repeated? 2. Figures II and III are sufficient to know as to
(a) 216 (b) 356 which face is opposite to the face numbered 4.
(c) 496 (d) 504 3. Figures I and IV are sufficient to know as to
which face is opposite to the face numbered.
Ans. (c) : If we subtract the number where no digits are Which of the statements given above are
repeating from total number between 6000 and 6999 we correct
get the answer.
(a) 1 and 3 only (b) 1 and 2 only
Total No = 1000
(c) 2 and 3, only (d) 1, 2 and 3
6 is fixed in the 1st place and the second place can have
9 different possibilities, then 3rd place can have 8 and Ans. (d) : 1 is correct as it clearly shows that 1 and 5 are
4th place 7. This will result in numbers where no digit is the top and bottom 2 and 6 are on the sides 4 so 2 is
repeating i.e. opposite 6.
1
1 × 9 × 8 × 7 = 507
6 4 2
Subtracting from 1000 we get 496.
5
Hence there will be 496 such possibilities. • 2 is correct on the same lines, 3 is opposite 4.
128. Each of 2 women and 3 men is to occupy one • 3 is correct as I & IV clearly tells that 4 is apposite
chair out of 8 chairs, each of which is 3. All the three statements regarding the figure
numbered from 1 to 8. First, women are to
given in the question are correct. Hence, the answer
occupy and two chairs from those numbered 1
is (d)
to 4; and then the 3 men would occupy any
three chairs out of the remaining 6 chairs. 131. Which one of the following pairs is not
What is the maximum number of different correctly matched?
ways in which this can be done? (a) Sigmund Freud : Psychoanalysis
(a) 40 (b) 132 (b) Anna Freud : Child psychiatry
(c) 1440 (d) 3660 (c) Milton Friedman : Economics
Ans. (c) : Two women can occupy 2 chairs out of the (d) Eric R. Kandel : Literature
first four chairs in 4 P2 ways. Ans. (d) : Eric Richard Kandel is an American
neuropsychiatrist. He was a recipient of the 2000 Nobel
3 men can occupy the remaining 6 chairs in 6 P3 ways Prize in Physiology or Medicine for his research on the
∴ total no. of ways = 4 P2 × 6 P3 = 1440 physiological basis of memory storage in neurons.
132. Projects and Development India Limited is mainly
129. "Lectures from Colombo to Almora" is based
engaged in design engineering, procurement and
on the experiences of which one of the
supervision of construction/commissioning in
following?
which area?
(a) Veer Savarkar (a) Pharmaceutical plants
(b) Annie Besant (b) Petroleum refineries and sugar factories
(c) Ramkrishna Paramhansa (c) Ship-building yards
(d) Swami Vivekanand (d) Fertilizer and allied chemical plants
IAS (Pre) GS 2006 Paper I 306 YCT
Ans. (d) : Projects and Development India Limited Codes:
(PDIL), an ISO-9001-2008 certified company, is a A B C D
leading design engineering and consultancy (a) 4 3 2 1
organization having experience of 40 years in design, (b) 2 1 4 3
detailed engineering, procurement, project management, (c) 4 1 2 3
construction supervision and comissioning of fertilizer (d) 2 3 4 1
and allied chemical, power etc.
Ans. (a) : The correct match is as follows :
133. Which is the correct chronological sequence of C N R Rao – Solid State Chemistry
the major events given below?
Jagdish Natwarlal Bhagwati – Economist
1. SLV-3 Launch
G. N Ramchandran – Physics
2. Formation of Bangladesh
Ashok Jhunjhunwala – Telecommunications
3. Sikkim becomes 22nd state of the Indian
Union 137. Match List-I (Place of Archaeological
Monument) with List-II (State) and select the
4. Pokharan-I test
correct answer using the code given below the
Select the correct answer using the code given
Lists:
below:
List-I List-II
(a) 2 - 4 - 3 - 1 (b) 3 - 1 - 2 - 4
(Place of Archaeological (State)
(c) 2 - 1 - 3 - 4 (d) 3 - 4 - 2 - 1
Monument
Ans. (a) : The correct order is as follows : Formation of A. Sisupalgarh 1. Assam
Bangladesh - March, 1971 SLV-3 Launch July 18, 1980,
B. Piprahwa 2. Manipur
Pokhran First Nuclear Test - 18 May 1974. Sikkim
becomes twenty second State of the Indian Union- 15 May C. Goalpara 3. Orissa
1975. So the correct sequence is 2–4–3–1. D. Bishnupur 4. Uttar Pradesh
Codes:
134. In which Olympics Games did India last win a
A B C D
Gold Medal?
(a) 2 4 1 3
(a) Montreal (1976) (b) Moscow (1980) (b) 2 1 4 3
(c) Los Angeles (1984) (d) Atlanta (1996) (c) 3 4 1 2
Ans. (b) : India defeated Spain in the final Hockey (d) 3 1 4 2
match held in Moscow Olympics (1980). Ans. (c) : A-3; B-4; C-1; D-2
135. In which country is the committee which selects 138. Which one of the following pairs is not
winners for Nobel Peace prize located? correctly matched?
(a) Norway (b) Sweden (Well-known Company) (Major Area of Work)
(c) Finland (d) Denmark (a) Adobe Systems India : Software
Ans. (a) : The Nobel peace prize Committee is (b) Sasken : Communication
composed of five members appointed by the Storting technology
(Norwegian parliament). The Nobel Peace Prize is (c) Genpact : Power generation
awarded in Oslo, Norway, not in Stockholm, Sweden, (d) Synthite : Export of spices
where the Nobel Prizes in Physics, Chemistry,
Ans. (c) : Genpact Limited is a multinational business
Physiology or Medicine, Literature and the Economics
process outsourcing and information technology
Prize are awarded. services company, domiciled in Bermuda with
136. Match List-I (Indian Scientist/Scholar) with executive headquarters in New York.
List-II (Area of Work) and select the correct 139. Match List-I (Author) with List-II (Book) and
answer using the code given below the Lists: select the correct answer using the code given
List-I List-II below the Lists:
(Indian Scientist (Area of Work) List-I List-II
/Scholar) (Author) (Book)
A. C.N.R. Rao 1. Telecommunication A. Amartya Sen 1. An Ordinary Person's
B. Jagdish Bhagwati 2. Physics Guide to Empire
B. Bimal Jalan 2. The Argumentative
C. G. N. 3. Economics
Indian
Ramachandran
C. Arundhati Roy 3. The Future of India
D. Ashok 4. Solid state chemistry
D. Mani Shankar 4. Confessions of a Secular
Jhunjhunwala and Material Science Aiyar Fundamentalist
IAS (Pre) GS 2006 Paper I 307 YCT
Codes: after his 81st birthday. José Manuel Barroso is a
A B C D Portuguese politician and university teacher, currently
(a) 3 2 1 4 serving as non-executive Chairman of Goldman Sachs
International. He previously served as the 11th
(b) 2 3 1 4
President of the European Commission and the 115th
(c) 3 2 4 1 Prime Minister of Portugal. After construction began in
(d) 2 3 4 1 1999, Pakistan claimed that design parameters of
Ans. (b) : A-2; B-3; C-1; D-4 Baglihar project violated the Indus Water Treaty of
140. Which one of the following countries is not a 1960. During 1999-2004 India and Pakistan held several
rounds of talks on the design of projects, but could not
member of the Commonwealth of Independent
reach an agreement. After failure of talks on 18 January
States (CIS)? 2005, Pakistan raised six objections to the World Bank,
(a) Armenia (b) Belarus a broker and signatory of Indus Water Treaty. In April
(c) Estonia (d) Georgia 2005 the World Bank determined the Pakistani claim as
Ans. (c) : Estonia is not a member of the a ‘Difference’, a classification between the less serious
Commonwealth of Independent States (CIS). ‘Question’ and more serious ‘Dispute’, and in May
2005 appointed Professor Raymond Lafitte, a Swiss
141. Which one of the following is not correctly
civil engineer as a neutral expert, to adjudicate the
matched? difference.
Prominent Indian Writer Language 143. Consider the following statements:
(a) Raja Rao : Telugu 1. Life Insurance Corporation of India is the
(b) Govind Trimbuk : Marathi oldest insurance company in India
Deshpande 2. National Insurance Company Limited was
(c) Subramaniyam Bharati : Tamil nationalized in the year 1972 and made a
subsidiary of General Insurance Corporation
(d) Tara Shankar Joshi : Gujarati
of India.
Ans. (a) : Raja Rao was an Indian writer of English 3. Headquarters of United India Insurance
language novels and short stories, whose works are Company Limited are located at Chennai.
deeply rooted in Hinduism. Which of the statements given above are
142. Match List-I with List-II and select the correct correct?
answer using the code given below the Lists: (a) 1, 2 and 3 (b) 1 and 2 only
(c) 2 and 3 only (d) 1 and 3 only
List-I List-II
Ans. (c) : Oriental Life Insurance company (1818) is
A. Harold Pinter 1. Director-General FAO the oldest insurance company in India. Rest of the
B. Jacques Diouf 2. President, European statements is correct.
Commission 144. Which one of the following pairs is not
C. Jose Manuel 3. World Bank appoint correctly matched?
Barroso expert, for adjudicating Railway Zone Headquarters
(a) North Eastern Railway : Gorakhpur
on the Baglihar Hydel (b) South Eastern Railway : Bhubaneshwar
Project (c) Eastern Railway : Kolkata
D. Raymond 4. Litterateur (d) South-East Central : Bilaspur
Railway
Lafitte
Ans. (b) :
Codes: Bhubaneswar Railway Station (BBS) serves
A B C D Bhubaneswar, the capital of the Indian state of Odisha
(a) 4 3 2 1 and it is the headquarters of the East Coast Railway
(b) 2 1 4 3 zone of the Indian Railways.
(c) 4 1 2 3 145. Match List-I (Leading Woman Lawn Tennis
Player) with List-II (Country) and select the
(d) 2 3 4 1
correct answer using the code given below the
Ans. (c) : Harold Pinter was a British playwright, Lists:
screenwriter, director and actor. A Nobel Prize winner, List-I List-II
Pinter was one of the most influential modern British (Leading Woman Lawn (Country)
dramatists with a writing career that spanned more than
Tennis Player)
50 years. Jacques Diouf was a Senegalese diplomat and
the Director-General of the United Nations' Food and A. Daniela Hantuchova 1. Russia
Agriculture Organization from January 1994 to 31 B. Patty Schnyder 2. Slovakia
December 2011. He died on 17 August 2019, 16 days C. Nadia Petrova 3. France
D. Amelie Mauresmo 4. Switzerland
IAS (Pre) GS 2006 Paper I 308 YCT
Codes: Since the two persons starts from A and B respectively,
A B C D So they shall meet each other when there is difference
(a) 2 1 4 3 7
(b) 3 4 1 2 of round between the two. Relative speed of A and B
8
(c) 2 4 1 3
3 1 1
(d) 3 1 4 2 = − =
8t 4t 8t
Ans. (c) : Correct match is as follows:
Daniela Hantuchova – Slovakia 7 7
Time takes to cover round at this speed = ×8t = 7t
Patty Schnyder – Switzerland 8 8
Nadia Patrova – Russia 148. Which one of the following is not a Central
Amelie – France University?
146. Each of the five persons A, B, C, D and E (a) Pondicherry University
possesses unequal number (< 10) of similar (b) Maulana Azad National Urdu University
items. A, B and C possess 21 items in all, while Hyderabad
C, D and E possess 7 items in all. How many (c) Visva Bharti, Shanti Niketan
items in all. How many items do A and B
(d) University of Madras, Chennai
possess in all?
(a) 15 (b) 17 Ans. (d) : University of Madras, Chennai
(c) 18 (d) Data is insufficient 149. In a question paper, there are four multiple-
Ans. (b) : According to question choice questions. Each question has five choices
A + B + C = 21 ..……..(i) with only one choice for its correct answer.
And C + D + E = 7 ………(ii) What is the total number of ways in which a
For equation (ii) ‘c’ can take values 1, 2 ad 4 candidate will not get all the four answers
As correct?
1+2+4=7 (a) 19 (b) 120
For C=1 (c) 624 (d) 1024
A + B + 1 = 21 Ans. (c) : Since, every question has five options,
∴ A + B = 20 So, no. of choices for each question = 5
Similarly for C = 2, A + B = 21 – 2 = 19 ∴ total no. of choices = 5 × 5 × 5 × 5 × 5 = 625
and for C = 4, A + B = 21 – 4 = 17 Now, no. of Choices of all correct answer = 1
147. There are 8 equidistant points A, B, C, D, E, F, Hence, no. of choices for all the four answer not correct
G and H in the clockwise direction on the = total no. of choices – no. of
periphery of a circle. In a time interval t, a choices of all correct anwer
person reaches from A to C with uniform = 625 – 1 = 624 Ans. (c)
motion while another person reaches the point
150. Match List-I (Person) with List-II (Area of
E from the point B during the same time
Work) and select the correct answer using the
interval with uniform motion. Both the persons
code given below the Lists:
move in the same direction along the
List-I (Person) List-II (Area of work)
circumference of the circle and start at the
same instant. How much time after the start, A. Ela Bhatt 1. Theatre training
will the two persons meet each other? B. Mahashweta Devi 2. Women's labour
(a) 4t (b) 7t sector
(c) 9t (d) Never C. Ebrahim Alkazi 3. Management teaching
Ans. (b) : Distance covered by first person in time D. Vijay Govindarajan 4. Litterateur and social
worker for the
2 1 country's tribal
t = = round
8 4 communities
3 Codes:
Distance covered by second person in time t = round A B C D
8
(a) 3 1 4 2
1
Speed of 1st person = (b) 2 4 1 3
4t (c) 3 4 1 2
3 (d) 2 1 4 3
Speed of 2nd person =
8t Ans. (b) : A-2; B-4; C-1; D-3

IAS (Pre) GS 2006 Paper I 309 YCT


UNION PUBLIC SERVICE COMMISSION
Civil Services (Preliminary Exam) - 2005
GENERAL STUDIES : PAPER-I
Time: 2 hours Maximum Number: 200
(a) 1 and 2 (b) 2 and 3
ANCIENT HISTORY (c) 1 and 3 (d) 1, 2 and 3
1. Who among the following was not a Ans. (b) : Statement 1 is incorrect as Lord Cornwallis
contemporary of the other three? was the first Governor General who established a
(a) Bimbisara (b) Gautama Buddha regular police force in India on the British pattern. A
(c) Milinda (d) Prasenjit Supreme Court was established at Fort Williams by the
Ans. (c) : In the given options except Milinda, all three Regulating Act, 1773 with jurisdiction over Bengal,
are related with Pre-Mauryan times. Milinda existed Bihar and Orissa. The Indian Penal Code came into
during post Mauryan times. Bimbisara belonged to the effect in the year 1860.
Haryanka dynasty and Magadha empire came into 5. Which one of the following is the correct
prominence under him. He was contemporary to the chronological order of the battle fought in
Buddha. He strengthened his position by marriage India in the 18th Century?
alliances. His first wife was sister of Prasenjit, who was
(a) Battle of Wandiwash – Battle of Buxar –
also contemporary of Buddha.
Battle of Ambur – Battle of Plassey
MODERN HISTORY (b) Battle of Ambur – Battle of Plassey – Battle
of Wandiwash – Battle of Buxar
2. Consider the following statements: (c) Battle of Wandiwash – Battle of Plassey –
The Government of India Act, 1935 provided Battle of Amburt – Battle of Buxar
for: (d) Battle of Ambur – Battle of Buxar – Battle of
1. the provincial autonomy. Wandiwash – Battle of Plassey
2. the establishment of Federal Court.
Ans. (b) :Battle of Ambur-1749
3. All India Federation at the Centre.
Which of the statements given above are Battle of Plassey-1757
correct? Battle of Wandiwash-1760
(a) 1 and 2 (b) 2 and 3 Battle of Buxar-1764
(c) 1 and 3 (d) 1, 2 and 3 6. Which one of the following is the correct
Ans. (d) : The Government of India Act, 1935 provided statement?
for the provincial autonomy, the establishment of (a) The modern Kochi was a Dutch colony till
federal court and all India Federation at the centre. India's Independence.
3. Which party was founded by Subhash (b) The Dutch defeated the Portuguese and built
Chandra Bose in the year 1939 after he broke
Fort Williams in the modern Kochi.
away from the Congress?
(a) Indian Freedom Party (c) The modern Kochi was first a Dutch colony
(b) Azad Hind Fauz before the Portuguese took over from them.
(c) Revolutionary Front (d) The modern Kochi never became apart of the
(d) Forward Block British colony.
Ans. (d) : Forward Block was founded by Subhash Ans. (b) : The Dutch defeated the Portuguese and built
Chandra Bose in the year 1939 after he broke away Fort Williams in the modern Kochi in 1663. Kochi
from the Congress. became the part of British colonial rule in the year
4. Consider the following statements: 1814.
1. Warren Hastings was the first Governor 7. Consider the following statements:
General who established a regular police 1. Lord Mountbatten was the Viceroy when
force in India on the British pattern. Simla Conference took place.
2. A Supreme Court was established at 2. Indian Navy Revolt, 1946 took place when
Calcutta by the Regulating Act, 1773. the Indian sailors in the Royal Indian Navy
3. The Indian Penal Code came into effect in at Bombay and Karachi rose against the
the year 1860. Government.
Which of the statements given above are Which of the statements given above is/are
correct? correct?
IAS (Pre) GS 2005 Paper I 310 YCT
(a) 1 only (b) 2 only 12. In October 1920, who of the following headed a
(c) Both 1 and 2 (d) Neither 1 nor 2 group of Indians gathered at Tashkent to set up
Ans. (b) : Statement 1 is incorrect as Lord Wavell was the a Communist Party of India?
Viceroy when Shimla Conference took place in June, (a) H.K. Sarkar (b) P.C. Joshi
1945. On February 18, 1946, some 1,100 Indian sailors or (c) M.C. Chagla (d) M.N. Roy
“ratings” of the HMIS Talwar and the Royal Indian Navy Ans. (d) : In October 1920, M.N. Roy headed a group
Signal School in Bombay declared a hunger strike, of Indians gathered at Tashkent to set up a Communist
triggered by the conditions and treatment of Indians in Party of India.
the Navy. On the very next day, between 10,000 to
13. At which Congress Session was the Working
20,000 sailors from Karachi, Madras, Calcutta,
Committee authorised to launch a programme
Mandapam, Visakhapatnam and the Andaman Islands
of Civil Disobedience?
joined the strike. Hence statement 2 is not correct.
(a) Bombay (b) Lahore
8. Which one of the following territories was not (c) Lucknow (d) Tripuri
affected by the Revolt of 1857?
Ans. (b) : At Lahore session (1929, President - J.L.
(a) Jhansi (b) Chittor
Nehru) the working committee was authorised to launch
(c) Jagdishpur (d) Lucknow
a programme of Civil Disobedience.
Ans. (b) : The territory of Chittor was not affected by the
14. Who among the following was not associated
Revolt of 1857. Jhanshi, Jagdishpur and Lucknow were the
with the formation of U.P. Kisan Sabha in
prominent hotbeds for the revolt of 1857.
February 1918?
9. Which one of the following places did Kunwar (a) Indra Narain Dwivedi
Singh, a prominent leader of the Revolt of 1857
(b) Gauri Shankar Misra
belong to?
(c) Jawaharlal Nehru
(a) Bihar (b) Madhya Pradesh
(d) Madan Mohan Malviya
(c) Rajasthan (d) Uttar Pradesh
Ans. (c) : In February 1918, Kisan Sabha was established
Ans. (a) : Kunwar Singh was a notable leader during the
in Uttar Pradesh with the efforts of Indra Narain Dwivedi,
Indian Rebellion of 1857. He belonged to a royal Ujjainiya
Gauri Shankar Misra and Madan Mohan Malviya. Jawahar
(Parmar) Rajput house of Jagdishpur, currently a part of
Lal Nehru was not related to Kisan Sabha.
Bhojpur district, Bihar.
15. In which one of the following provinces was a
10. Which of the following pairs are correctly
Congress ministry not formed under the Act of
matched?
1935?
Movement/Satyagraha Person Actively (a) Bihar (b) Madras
Associated with
(c) Orissa (d) Punjab
1. Champaran : Rajendra Prasad
Ans. (d) : Under Act of 1935 congress ministries were
2. Ahmedabad Mill : Morarji Desai formed in Bombay, Madras, Central Provinces, Odisha,
Worker United provinces, Bihar and later in NWFP and Assam
3. Kheda : Vallabhbhai Patel also.
Select the correct answer using the codes given 16. Consider the following statements : On the eve of
below: the launch of Quit India Movement, Mahatma
(a) 1 and 2 (b) 2 and 3 Gandhi
(c) 1 and 3 (d) 1, 2 and 3 1. Asked the government servants....to resign.
2. Asked the soldiers to leave the posts.
Ans. (c) : Morarji Desai is not related to Ahmedabad
3. Asked the Princes of the Princely States to
mill worker strike. Rest is correctly matched.
accept the sovereignty of their own people.
11. Who among the following drafted the Which of the statements given above is/are correct?
resolution of fundament rights for the Karachi
(a) 1 and 2 (b) 2 and 3
Session of Congress in 1931?
(c) 3 only (d) 1, 2 and 3
(a) Dr. B.R. Ambedkar
Ans. (c) : Gandhiji asked the government servants not
(b) Pandit Jawaharlal Nehru
to resign but declare their allegiance to the Congress.
(c) Dr. Rajendra Prasad Soldiers were asked not to leave their posts but not to
(d) Sardar Vallabhbhai Patel fire. Gandhi asked the Princes of the Princely States to
Ans. (b) : Sardar Vallabhbhai Patel presided, the accept the sovereignty of their own people.
congress session of 1931 but Jawaharlal Nehru drafted 17. Where were the Ghadar revolutionaries, who
the resolution on Fundamental Rights for the Karachi became active during the outbreak of the
Session of Congress. World War I based?
IAS (Pre) GS 2005 Paper I 311 YCT
(a) Central America (b) North America RTC. Poona Pact was signed by B.R. Ambedkar in
(c) West America (d) South America September, 1932. In the Poona Act, special provisions
Ans. (b) : Ghadar revolutionaries were based in San for representation of the depressed people in the local
Francisco and branches along the US West Coast and bodies and civil services were made. Hence all the
in the far East. statements are correct.
The Ghadar Party was an Indian revolutionary 21. Consider the following statement:
organisation primarily founded by Indians. The party 1. Ishwar Chandra Vidyasagar founded the
was multi-ethnic and had Sikh, Hindu and Muslim Bethune School at Calcutta with the main
leaders. The party was headquartered in San Francisco, aim of encouraging education for women.
United States. 2. Bankim Chandra Chattopadhyay was the
18. What was Komagata Maru? first graduate of the Calcutta University.
(a) A political party based in Taiwan 3. Keshav Chandra Sen's Campaign against
(b) Peasant communist leader of China. Sati led to the enactment of a law to ban
(c) A naval ship on voyage to Canada. Sati by the then Governor General.
(d) A Chinese village where Mao Tre Lung Which of the statements given above is/are
began his long march. correct?
(a) 1 only (b) 1 and 2
Ans. (c) : The Komagata Maru incident (1914) is about
a Japanese naval ship on which 376 passengers from (c) 2 and 3 (d) 1, 2 and 3
Punjab were boarded for voyage from Hong Kong to Ans. (b) : Statement 3 is incorrect as Raja Ram Mohan
Canada. The incident got highlighted because out of all Roy's campaign against Sati led to the enactment of
the passengers, only 24 were admitted to Canada but the Bengal Regulation Act of 1829 to ban Sati by the then
other 352 passengers were not allowed inside Canada Governor General William Bentick. Bankim Chandra
and the ship was forcefully returned to India. Chattopadhyay and Joddy Nath Bose were the first
graduates of the Calcutta University on January 30,
19. Who among the following was a proponent of
1858. JED Bethune founded the Bethune School in
Fabianism as a movement?
Calcutta in 1849. Ishwar Chandra Vidyasagar was also
(a) Annie Besant
associated with this.
(b) Michael Madhusudan Dutt
22. Who among the following repealed the
(c) A.O. Hume
Vernacular Press Act?
(d) R. Palme Dutt
(a) Lord Dufferin (b) Lord Ripon
Ans. (a) : The Fabian Society is a British socialist (c) Lord Curzon (d) Lord Hardinge
intellectual movement best known for its initial ground-
breaking work beginning in the late 19th century and Ans. (b) : Lord Lytton brought Vernacular Press Act
then up to World War I. Fabianism focused on the which came in to force in 1878. It was repealed by Lord
advancement of socialist ideas through gradual Ripon in 1882.
influence and patiently insinuating socialist ideology
into intellectual circles and groups with power. INDIAN GEOGRAPHY
20. Consider the following statements: 23. Match List I with List II and select the correct
1. In the First Round Table Conference, Dr. answer using the codes given below the lists:
Ambedkar demanded separate electorates List I List II
for the depressed classes. (National Park/Wild-life (State)
2. In the Poona Act, special provisions for Sanctuary)
representation of the depressed people in A. Bondla Wildlife 1. Orissa
the local bodies and civil services were Sanctuary
made.
B. Kangerghat National 2. Assam
3. The Indian National Congress did not take
Park
part in the Third Round Table Conference.
C. Orang Sanctuary 3. Chhattisgarh
Which of the statement given above are
correct? D. Ushakothi Wildlife 4. Goa
(a) 1 and 2 (b) 2 and 3 Sanctuary
(c) 1 and 3 (d) 1, 2 and 3 5. Tripura
Ans. (d) : First Round Table Conference was held from A B C D
Nov, 1930 - Jan 1931. Second Round Table Conference (a) 4 3 2 1
was held in Dec 1931 in London. Third Round Table (b) 4 3 2 1
Conference was held from Nov 1932 - Dec 1932. Indian (c) 2 3 5 1
National Congress did not participate in first and third (d) 4 1 2 3
IAS (Pre) GS 2005 Paper I 312 YCT
Ans. (b) :Bondla Wildlife Sanctuary Goa (a) 1, 2 and 3 (b) 1, 3 and 4
Kangerghat National Park Chhattisgarh (c) 2 and 4 (d) 1, 2, 3 and 4
Orang Sanctuary Assam Ans. (d) : All the monuments/sites are included in
Ushakothi Wildlife Sanctuary Odisha World Heritage list of UNESCO. Sun Temple of
24. Consider the following statements: Odisha, group of monuments at Mahabalipuram were
1. India is the only country in the world included in 1985. Champaner – Pavagadh
producing all the five known commercial Archeological Park of Gujarat and Chhatrapati Shivaji
varieties of silk. Terminus of Maharastra are included in year 2004 in
UNESCO list.
2. India is the largest producer of sugar in the
world. 28. Which one of the following statements is not
Which of the statements given above is/are correct?
correct? (a) The Western Ghats are relatively higher in
(a) 1 only (b) 2 only their northern region.
(c) Both 1 and 2 (d) Neither 1 or 2 (b) The Anai Mudi is the highest peak in the
Western Ghats.
Ans. (a) : India produces five varieties of silk namely
(c) Tapi river lies to the south of Sapura
Mulberry silk, Muga silk, Tasar silk, Oak tasar silk and
Eri silk. India is the 2nd largest producer of sugar in the (d) The Narmada and the Tapi river valley are
world. said to be old rift valleys.
25. Match List I with List II and select the correct Ans. (a) : The statement that is not correct is that the
answer using the codes given below the lists: Western Ghats are relatively higher in their northern
region. The Western Ghats are relatively higher in their
List I List II southern region due to the presence of peaks like
(Power Station) (State) Dodabeta, Annaimudi. Rest of the statements are
A. Kothgudem 1. Andhra Pradesh correct.
B. Raichur 2. Gujarat 29. Ghandhi Sagar Dam is a part of which one of
C. Mettur 3. Karnataka the following?
D. Wanakbori 4. Tamil Nadu (a) Chambal Project
A B C D (b) Kosi Project
(a) 4 2 1 3 (c) Damodar Valley Project
(b) 1 3 4 2 (d) Bhakra Nangal Project
(c) 4 3 1 2 Ans. (a) : Gandhi Sagar Dam is a part of the Chambal
(d) 1 2 4 3 project. The Gandhi Sagar dam is one of the four dams
built on the Chambal river. Other three dams are Rana
Ans. (b) : Pratap Sagar dam, Jawahar Sagar dam and Kota
Power Station State Barrage. The dam is located in Mandsaur district of
Kothagudem : Andhra Pradesh Madhya Pradesh.
Raichur : Karnataka 30. Which one of the following is the correct
Mettur : Tamil Nadu sequence of the given hills starting from the
Wanakbori : Gujarat north and going toward the South?
26. For which one of the following, is Satara well- (a) Nallamalai Hills - Nilgiri Hills - Javadi Hills-
known? Anaimalai Hills
(a) Thermal power plant (b) Anaimalai Hills - Javadi Hills - Nilgiri Hills -
(b) Wind energy plant Nallamalai Hills
(c) Hydro-electric plant (c) Nallamalai Hills - Javadi Hills - Nilgiri Hills -
(d) Nuclear power plant Anaiimalai Hills
Ans. (a) : Satara is well known for Thermal Power (d) Anaimalai Hiills - Nilgiri Hill - Javadi Hills -
Plant. Nallamalai Hills
27. Consider the following sites/monuments: Ans. (c) : The correct sequence of the given hills
1. Champaner-Pavagadh Archeological Park. starting from the north to south are Nallamalai Hills -
2. Chhatrapati Shivaji Railway Station, Javadi Hills - Nilgiri Hills - Anaimalai Hills.
Mumbai.
3. Mamallapuram. 31. Which one of the following is not a Biosphere
4. Sun Temple (Konark Temple). Reserve?
Which of the above are included in the World (a) Agasthyamalai (b) Nallamalai
Heritage List of UNESCO? (c) Nilgiri (d) Panchmarhi

IAS (Pre) GS 2005 Paper I 313 YCT


Ans. (b) : Nallamalai is not a biosphere reserve. It is 3. Fairy Queen is a train using the world's oldest
hill of Eastern ghats which stretches over Kurnool, working engine and the Indian Railways
Mahabubnagar, Guntur and Kadapa districts of Andhra conduct a journey of wildlife and heritage sites
Pradesh. on it.
32. Match List-I (Atomic Power Plants/Heavy Which of the statements given above is/are
Water Plants) with List-II (State) and select the correct?
correct answer using the codes given below the (a) 2 only (b) 3 only
lists: (c) 1 and 2 (d) None
List-I List-II Ans. (b) : The headquarter of the North Western
(Atomic Power (State) Railway is located at Jaipur. Hence option 1 is not
Plants/Heavy Water correct. 'Indrail pass' is issued by Indian Railways for
Plants) the foreign tourists and NRI’s. Hence option 2 is also
not correct.
A. Thal 1. Andhra Pradesh
The Fairy Queen is the oldest working Steam
B. Manuguru 2. Gujarat Locomotive in the world, built in 1855. It was
C. Kakrapar 3. Maharashtra completely over hauled in 1996 and resumed
D. Kaiga 4. Rajasthan commercial operations in 1997 from Delhi to Alwar
with overnight stay in Sariska Tiger Reserve. It was
5. Karnataka
certified as the world's oldest working locomotive by
Codes: the Guinness Book of World Records.
A B C D 35. Which one of the following is the correct sequence
(a) 2 1 4 5 of the given Indian cities the decreasing order of
(b) 3 5 2 1 their normal annual rainfall?
(c) 2 5 4 1 (a) Kochi–Kolkata–Delhi–Patna
(d) 3 1 2 5 (b) Kolkata–Kochi–Patna–Delhi
Ans. (d) : (c) Kochi–Kolkata–Patna–Delhi
(Atomic Power Plants (State) (d) Kolkata–Kochi–Delhi–Patna
/Heavy Water Plants) Ans. (c) : Kochi-3228mm,
Thal Maharashtra Kolkata-1582mm
Manuguru Andhra Pradesh Patna-1000mm
Kakrapar Gujarat Delhi-707mm
Kaiga Karnataka 36. Consider the following statements:
33. Which one of the following pairs is not 1. Silent Valley National Park is in the
correctly matched? Nallamalai range.
(a) Southern Air : Thiruvanatha puram 2. Pathrakkadavu Hydroelectric Project is
Command proposed to be built near the Silent Valley
(b) Eastern Naval : Visakhapatnam national Park.
Command 3. the Kunthi river originates in Silent Valley's
rainforests.
(c) Armoured Corps : Jabalpur
Which of the statements given above is/are
Centre and School
correct?
(d) Army Medical : Lucknow
(a) 1 and 3 (b) 2 only
Crops Centre and
(c) 2 and 3 (d) 1, 2 and 3
School
Ans. (c) : Silent Valley National Park is situated in
Ans. (c) : Indian Army's Armoured Corps Centre and Nilgiri Hills of Western Ghats. The park is bound by
School is located in Ahmed Nagar, Maharashtra. Rest is Attappadi reserved forest to the east and vested forest
correctly matched. of Palaghat division and Nilamber division to the south-
34. Consider the following statements concerning west respectively. Rest of the statements is correct.
the Indian Railways: 37. Consider the following statements
1. The Head Quarters of the North Western 1. Area wise, Chhattisgarh is larger than West
Railway is located at Jodhpur. Bengal.
2. According to the Population 2001 census,
2. 'Indrail pass'– a travel-as-you-please ticket has population of West Bengal is larger than
been created especially for freedom fighters that of Chhattisgarh.
and sportspersons who have represented India Which of the statements given above is/are
in any game/sport. correct?
IAS (Pre) GS 2005 Paper I 314 YCT
(a) 1 only (b) 2 only (a) Seikan Rail Tunnel : China
(c) Both 1 and 2 (d) Neither 1 nor 2 (b) Petronas Towers : Malaysia
Ans. (c) : Area of Chhattisgarh is 135,192 square km (c) Appalachian Trail : United State of
and Area of West Bengal is 88,572 sq km but the America
population density in Chhattisgarh is 154, whereas the (d) Rogun Dam : Tajikistan
population density in West Bengal is 904. Ans. (a) : Seikan Rail Tunnel is world's longest tunnel
38. Which one of the following statements is not with length of 34 miles, is located in Japan. Seikan
correct? railway tunnel is Japan's 53.85 km long tunnel beneath
(a) There is no definition of the Scheduled Tribe the Tsugaru Strait and connects the Aomori Prefecture
in the Constitution of India on Honshu Island and the Hokkaido Island. With its
(b) North-East India accounts for a little over half track located 140m below the seabed, the Seikan tunnel
of the country's tribal population. is the world's deepest and longest railway tunnel.
(c) The people known as Todas live in the Nilgiri 42. Which one of the following is the correct
area. sequence of the given Continents in the
(d) Lotha is a language spoken in Nagaland. decreasing order of their percentage of Earth's
Ans. (b) : Largest population of Schedule Tribes in land?
India resides in Madhya Pradesh. In addition to that, (a) North America – Africa – South America –
schedule tribes also live in Maharashtra, Jharkhand and Europe
Bihar. Highest population density of schedule tribes are (b) Africa – North America – South America –
found in North-eastern states of India. Rest of the Europe
statements is correct. (c) North America – Africa – Europe – South
America
WORLD GEOGRAPHY (d) Africa – North America – Europe – South
America
39. Itaipu Dam built on the River Parana is one of Ans. (b) : The correct sequence of the given Continents
the largest dams in the world. Which one of the in the decreasing order of their percentage of Earth's
following two countries have this as a joint land is Africa – North America – South America –
project? Europe.
(a) Brazil and Peru
43. Match List I with List II and select the correct
(b) Paraguay and Ecuador
answer using the codes given below the lists:
(c) Brazil and Paraguay
List I List II
(d) Colombia and Paraguay
(Country) (Name of Parliament)
Ans. (c) : River Parana originates in the Brazilian
A. Netherlands 1. Diet
Plateau and forms the boundary between Brazil and
Paraguay and then between Paraguay and Argentina. It B. Ukraine 2. States General
ultimately falls in the Atlantic Ocean at Rio de la Plata. C. Poland 3. Supreme Council
Therefore it is a joint project of Brazil and Paraguay. D. Japan 4. Sejm
40. Consider the following statements: A B C D
1. Total land area of Earth is approximately (a) 4 1 2 3
1475 lakh square kilometres. (b) 2 3 4 1
2. Ratio of land area to water area of Earth is (c) 4 3 2 1
approximately 1 : 4. (d) 2 1 4 3
3. Maximum percentage of Earth's water is
Ans. (b) : Country Name of Parliament
in the Pacific Ocean.
Which of the statements given above is/are A. Netherlands 1. States General
correct? B. Ukraine 2. Supreme Council
(a) 1 and 3 (b) 2 and 3 C. Poland 3. Sejm
(c) 1 only (d) 3 only D. Japan 4. Diet
Ans. (a) : Total land area is 148940000 km2 square 44. Consider the following statements
kilometers. The ratio of land area to water area of earth 1. A geostationary satellite is at an approximate
is approximately 1: 3. height of 10,000 km.
The Pacific Ocean is the largest ocean, so maximum 2. FM transmission of music is of very good
percentage of earth's water is present in it. quality because the atmospheric or man-made
41. Which one of the following pairs is not noises which are generally frequent variations
correctly matched? can do little harm.
IAS (Pre) GS 2005 Paper I 315 YCT
Which of the statements given above is/are (a) Islamabad – Gujranwala – Peshawar - Multan
correct? (b) Peshawar – Gujranwala – Multan – Islamabad
(a) 1 only (b) 2 only (c) Peshawar – Islamabad – Gujranwala – Multan
(c) Both 1 and 2 (d) Neither 1 nor 2 (d) Islamabad Multan – Peshawar – Gujranwala
Ans. (b) :Geostationary satellite need to stay constantly Ans. (c) : The correct sequence of the given towns of
above one particular place on earth such as Pakistan while moving from the North towards the
telecommunication satellite. This way, an antenna on South is Peshawar - Islamabad - Gujranwala - Multan.
earth can be fixed to always stay pointed towards that 48. Where are the Balearic Islands located:
satellite without moving. In order to perfectly match (a) Mediterranean Sea (b) Black Sea
earth’s rotation, the speed of geostationary satellite
(c) Baltic Sea (d) North Sea
should be about 3 km per second at an altitude of 35786
km. Hence statement 1 is not correct. FM transmission Ans. (a) : The Balearic Islands is a Spanish archipelago
of music is of very good quality because the located in the Mediterranean Sea. It is an autonomous
atmospheric or man-made noises which are generally community of Spain. It has four major islands and
frequent variations can do little harm. several minor ones.
45. Consider the following statements: 49. Which one of the following countries does not
1. The axis of the earth's magnetic field is border Lithuania?
inclined at 23½º to the geographic axis of (a) Poland (b) Ukraine
the earth. (c) Belarus (d) Latvia
2. The earth's magnetic pole in the northern Ans. (b) : Ukraine does not border Lithuania. Belarus
hemisphere is located on a peninsula in lies in the east of the Lithuania, Baltic ocean in west,
northern Canada. Latvia in north and Poland in south.
3. Earth's magnetic equator passes through 50. Consider the following statements:
Thumba is South India. 1. Great Britain comprises England, Wales,
Which of the statements given above is/are Scotland and Northern Ireland.
correct? 2. England covers less than 60% of the total
(a) 1, 2 and 3 (b) 2 and 3 area of the United Kingdom.
(c) 2 only (d) 3 only Which of the statements given above is/are
Ans. (b) : The axis of earth is inclined to the plane of correct?
the ecliptic at an angle of 66 ½ degree, giving rise to (a) 1 only (b) 2 only
different seasons and varying lengths of day and night. (c) Both 1 and 2 (d) Neither 1 nor 2
1 Ans. (c) : The total area of United Kingdom is 93, 628
Earth rotates on its own axis, tilted at an angle of 23
2 sq miles (242,500 sq km), where England covers 50,
degrees, giving rise to Day and Night. 301 sq miles (130, 365 sq km) i.e. less than 60 % of
The North Magnetic Pole moves over time due to total land area of United Kingdom. The United
magnetic changes in Earth's core. In 2001, it was Kingdom (UK) is made up of England, Scotland, Wales
determined by the Geological Survey of Canada to lie and Northern Ireland.
west of Ellesmere Island in northern Canada at 81.3°N 51. Where is the volcanic mountain, Mount St.
110.8°W. It was situated at 83.1°N 117.8°W in 2005. Helens located?
The Thumba Equatorial Rocket Launching Station (a) Chile
(TERLS) is an Indian spaceport operated by the Indian (b) Japan
Space Research Organisation. It is located in Thumba, (c) Philippines
Thiruvananthapuram which is near the southern tip of (d) United States of America
mainland India, very close to earth's magnetic equator.
Ans. (d) : Volcanic mountain. St. Helen is located in
46. Which one of the following pairs is not United States of America.
correctly matched?
(a) Bahamas : Nassau Directions:
The following 7(Seven) items consist of two
(b) Costa Rice : San Jose
statements: one labelled as the 'Assertion (A)'
(c) Nicaragua : Belmopan
and the other as 'Reason (R)'. You are to
(d) Dominican Republic : Santo Domingo
examine these two statements carefully and
Ans. (c) : Managua is the capital of Nicaragua, not select the answers to these items using the code
Belmopan. Balmopan is the capital city of Balize. given below:
47. Which one of the following is the correct Codes:
sequence of the given towns of Pakistan while (a) Both A and R are individually true and R is
moving from the North towards the South? the correct explanation of A
IAS (Pre) GS 2005 Paper I 316 YCT
(b) Both A and R are individually true but R is Ans. (d) : A−4;B−3;C−2;D−1
not the correct explanation of A
(City) (River)
(c) A is true but R is false
Washington D.C. River Potomac
(d) A is false but R is true
52. Assertion (A) : The same face of the Moon is Berlin River Spree
always presented to the Earth. Paris River Seine
Reason (R) : The Moon rotates about its own Madrid River Manzanares
1
axis in 23 days which is about the same time INDIAN CONSTITUTION &POLTY
2
that it takes to orbit the Earth.
56. Consider the following statements:
Ans. (a) : The same face of the Moon is always 1. There are 25 High Courts in India.
presented to the Earth because the period of the rotation 2. Punjab, Haryana and the Union Territory
of the moon is same as the period of its revolution of Chandigarh have a common High
around the earth. So the same face of the moon is Court.
always presented to earth. 3. National Capital Territory of Delhi has a
53. Assertion (A) : Existence of human life on High Court of its own.
Venus is highly improbable. Which of the statements given above is/are
Reason (R) : Venus has extremely high level correct?
of carbon dioxide in its atmosphere. (a) 2 and 3 (b) 1 and 2
(c) 1, 2 and 3 (d) 3 only
Ans. (a) : The atmosphere of Venus has carbon dioxide
primarily followed by nitrogen, making it denser and Ans. (c) : Separate High Courts have started functioning
for the states of Telangana and Andhra Pradesh from 1st
hotter for the existence of human life.
January 2019. The High Court for Telangana is the 25th
54. Assertion (A) : Wind patterns are clockwise in High Court in the country. Telangana was carved out of
the northern hemisphere and anticlockwise in Andhra Pradesh in 2014 and the two states continued to
the southern hemisphere. have a common high court in Hyderabad. However when
Reason (R) : The directions of wind patterns in question was asked at that time there were 21 High Courts.
the northern and the southern hemisphere are • High Court of Punjab and Haryana is the
governed by the Coriolis effect. common High Court for states of Haryana and
(a) Both A and R are true and R is the correct Punjab and Union Territory of Chandigarh based in
explanation of A. Chandigarh.
(b) Both A and R are true but R is not the correct • Delhi is the only Union Territory in India, which
explanation of A. has its own High Court. As far as the other Union
Territories are concerned, they share their
(c) A is correct but R is wrong.
jurisdiction with some other States.
(d) A is wrong but R is correct.
57. Consider the following statements:
Ans. (a) : Assertion is correct and reason is the correct 1. The Parliament cannot enlarge the
explanation of assertion. Due to the rotation of the jurisdiction of the Supreme Court of India
earth, winds do not flow in the perpendicular direction as its jurisdiction is limited to that
but flows in the opposite direction. This is due to the conferred by the Constitution.
effect of Coriolis force. Due to Coriolis effect, wind 2. The officers and Servants of the Supreme
patterns are clockwise in the northern hemisphere and Court and High Courts are appointed by
anticlockwise in the southern hemisphere. the concerned Chief Justice and the
55. Match List I (City) with List II (River) and administrative expenses are charged on the
select the correct answer using the codes given Consolidated Fund of India.
below the lists: Which of the statements given above is/are
correct?
List I List II (a) 1 only (b) 2 only
(City) (River) (c) Both 1 and 2 (d) Neither 1 nor 2
A. Washington D.C. 1. River Manzanares Ans. (d) : According to Article 138 (1) of the Constitution,
B. Berlin 2. River Seine the Supreme Court shall have such further jurisdiction and
C. Paris 3. River Spree powers with respect to any of the matters in the Union List
as Parliament may by law confer.
D. Madrid 4. River Potomac • The officers and Servants of the Supreme Court and
Codes: High Courts are appointed by the concerned Chief
A B C D Justice. However the administrative expenses of
(a) 2 3 4 1 Supreme Court is charged on the Consolidated Fund
(b) 2 1 2 3 of India, whereas administrative expenses of High
(c) 2 1 3 3 Court is charged on the consolidated fund of the
(d) 4 3 2 1 state.

IAS (Pre) GS 2005 Paper I 317 YCT


58. Consider the following statements: Which of the statements given above is/are
1. Article 301 pertains to the Right to correct?
Property. (a) 1 only (b) 2 only
2. Right to Property is a legal right but not a (c) Both 1 and 2 (d) Neither 1 nor 2
Fundamental Right. Ans. (a) : Among the given statements, statement 1 is
3. Article 300 A was inserted in the correct but 2 is not correct. Reasons are as follows
Constitution of India by the Congress The Seventy-third Amendment Act (1992) which got
Government at the Centre by the 44th President's assent on 25th April, 1993 after ratification
Constitutional Amendment. of the required number of states, provided constitutional
Which of the statements given above is/are guarantee for formation of Panchayats, inserted Part IX
correct? and the 11th Schedule in the Constitution.
(a) 2 only (b) 2 and 3 It is correct that Part IXA of the Constitution of India
(c) 1 and 3 (d) 1, 2 and 3 contains provisions for municipalities but Article 243
Ans. (a) : Article 301 pertains to Freedom of Trade, envisages 3 types (not 2 types) of municipalities.
Commerce and Intercourse. (a) Nagar Panchayats
• In the original constitution right to property was a (b) A Municipal Council for Smaller Urban Areas
Fundamental Right under Article 31 and Article (c) A Municipal Corporation for a Larger Urban Area
19(1) (f). But 44th Amendment Act, 1978 omitted
sub clause f, and inserted Article 300A to make 61. Consider the following statements"
right to property a legal right. 1. The constitution of India has 20 parts.
• The Government at that time was Janta Party 2. There are 390 Articles in the Constitution of
government. India in all.
59. Consider the following statements: 3. Ninth, Tenth, Eleventh and Twelfth
1. Article 371 A to 371 I were inserted in the Schedules were added to the constitution of
Constitution of India to meet regional India by the Constitution (Amendment)
demands of Nagaland, Assam Manipur, Acts.
Andhra Pradesh, Sikkim, Mizoram, Which of the statements given above is/are
Arunachal Pradesh and Goa. correct?
2. Constitutions of India and the United (a) 1 and 2 (b) 2 only
States of America can envisage a dual (c) 3 only (d) 1, 2 and 3
policy (The Union and the States) but a Ans. (c) : The Constitution of India has 24 Parts, 12
single citizenship. Schedules and more than 444 articles at present. In the
3. A naturalized citizen of India can never be original Constitution, there were 22 parts, 8 Schedules
deprived of his citizenship. and 395 articles. Ninth Schedule was added by 1st
Which of the statements given above is/are Constitutional Amendment Act, 1951. Tenth Schedule
correct? was added by 52nd Constitutional Amendment Act,
(a) 1, 2 and 3 (b) 1 and 3 1985, Eleventh Schedule was added by 73rd
(c) 3 only (d) 1 only Constitutional Amendment Act. 1992, and Twelfth
Ans. (d) : Article 371(A) to 371(1) deals with special Schedule was added by 74th Constitutional Amendment
provisions to Nagaland, Assam, Manipur, Andhra Act, 1992.
Pradesh, Sikkim, Mizoram, Arunachal Pradesh and 62. Who among the following was the Chairman of
Goa. The Constitution of India envisages a single policy the Union Constitution Committee of the
for both Union and the States. Further, a naturalized Constituent Assembly?
citizen is one who acquires citizenship either by (a) B.R. Ambedkar
Naturalization or by Registration. They can be deprived (b) J.B. Kripalani
of citizenship if they acquired citizenship by using (c) Jawaharlal Nehru
fraudulent means. Hence, only statement 1 is correct. (d) Alladi Krishnaswami Ayyar
60. Consider the following statements: Ans. (c) : The Constituent Assembly was constituted on
1. Part IX of the Constitution of India November 1946 under Cabinet Mission Plan. The total
contains provisions for Panchayats and strength of the Constituent Assembly was 389. It
was inserted by the Constitution (73rd comprised of several Committees to deal with different
Amendment) Act, 1992. tasks. One such Union Constitution Committee was
2. Part IX A of the Constitution of India headed by Jawahar Lal Nehru.
contains provisions for municipalities and 63. Which one of the following cities is the seat of
the Article 243Q envisages two types of Parliament of South Africa?
municipalities- Municipal Council and a (a) Pretoria (b) Durban
Municipal Corporation for every State. (c) Johannesburg (d) Cape Town
IAS (Pre) GS 2005 Paper I 318 YCT
Ans. (d) : Since, 1910 as the seat of Parliament, Cape (a) 1 only (b) 2 only
Town has been the legislative capital of South Africa, (c) Both 1 and 2 (d) Neither 1 nor 2
Pretoria the Administrative capital and Bloemfontein Ans. (b) : The series of index number of wholesale
the Judicial Capital. price introduced from April 2000 has the year 1993-94
64. Consider the following statements: as base year.
1. The constitution of The United States of 67. Consider the following statements:
America came into force in the year 1810. 1. Poverty Reduction and Growth Facility
2. All revenue bills must originate in the House (PRGF) has been established by the
of Representatives of the US Congress. International Development Association
3. Gerge W. Bush is the only President in the (IDA) to provide further assistance to low-
history of the United States of America income countries facing high level of
whose father was also the President to the indebtedness.
United States of America. 2. Singapore Regional Training Institute
Which of the statements given above is/are (STI) is one of the institutes that provides
correct? training in macroeconomic analysis and
(a) 1 only (b) 2 only policy, and related subjects as a part of
(c) 1 and 2 (d) 2 and 3 programme of the IMF Institute.
Ans. (b) : The Constitution of the United States of Which of the statements given above is/are
America is the supreme law of the country. Since the correct?
Constitution came into force in 1789, it has been (a) 1 only (b) 2 only
amended 27 times, including one amendment that (c) Both 1 and 2 (d) Neither 1 nor 2
repealed a previous one, in order to meet the needs of a Ans. (b) : In 1999, the IMF (International Monetary
nation that has profoundly changed since the eighteenth Fund) established the Poverty Reduction and Growth
century. Facility (PRGF) to make the objectives of poverty
All Bills for raising revenue shall originate in the House reduction and growth more central to lending operations
of Representatives; but the Senate may propose or in its poorest member countries. Hence statement 1 is
concur with amendments as on other Bills. not correct. The IMF – Singapore Regional Training
It has now happened twice in USA’s history that father Institute (STI), located in Singapore, serves as the
and son have been president: the Adamses (John Adams International Monetary Fund’s regional training center
(1797-1801) and John Quincy Adams (1825-1829)) and for the Asia-Pacific region. It provides training on
the Bushes (George H.W. Bush (1989-1993) and macroeconomic and financial management and related
George W. Bush (2001-2009)). legal and statistical issues to government officials from
65. The Constitution (98th Amendment) Act is 37 countries. Hence statement 2 is correct.
related to: 68. Consider the following statements:
(a) Empowering the Centre to levy and 1. Sensex is based on 50 of the most
appropriate service tax important stocks available on the Bombay
(b) The constitution of the National Judicial Stock Exchange (BSE).
Commissions 2. For calculating the Sensex, all the Sensex
(c) Readjustment of electoral constituencies on stocks are assigned proportional
the basis of the Population Census 2001 weightage.
(d) The demarcation of new boundaries between 3. New York Stock Exchange is the oldest
States stock exchange in the world
Ans. (b) : The Constitution (98th Amendment) Act, Which of the statements given above is/are
2012 is related to the constitution of the National correct?
Judicial Commission. (a) 2 only (b) 1 and 3
(c) 2 and 3 (d) None
ECONOMY Ans. (a) : The ‘BSE SENSEX’ is a value-weighted
index composed of 30 stocks and was started in 1
66. Consider the following statements
January, 1986. The origin of the New York Stock
1. During the year 2004, India's foreign
Exchange (NYSE) can be traced to 17 May, 1792, when
exchange reserves did not exceed the 125
the Buttonwood Agreement was signed by 24 stock
billion U.S. Dollar mark.
brokers outside 68 Wall Street in New York under a
2. The series of index numbers of wholesale
prices introduced from April, 2000 has the buttonwood tree. Amsterdam stock exchange (1602) is
year 1993-94 as base year. considered as the oldest stock exchange in the world
Which of the statements given above is/are correct? and was established by the Dutch East India company.

IAS (Pre) GS 2005 Paper I 319 YCT


69. Consider the following statements: PHYSICS
1. The Headquarters of the International
Organization for Standardization are 72. What is the order of magnitude of electric
located in Rome. resistance of the human body (dry)?
2. ISO 9000 relates to the quality (a) 102 ohm (b) 104 ohm
management system and standards. (c) 106 ohm (d) 108 ohm
3. ISO 14000 relates to environmental Ans. (b) : Human body has different resistances, when
management system standards. dry, resistance is 100,000 ohms. When wet because of
sweat or water, resistance is only 1,000 ohms.
Which of the statements given above is/are
correct?
CHEMISTRY
(a) 1, 2 and 3 (b) 3 only
(c) 2 and 3 (d) None 73. Consider the following statements
Ans. (c) : International Organization for 1. Anhydrous sodium carbonate is commonly
Standardization (ISO) is an independent known as baking soda.
nongovernmental international organization with a 2. Baking soda is used in fire extinguishers.
membership of 165 national standards bodies. It was 3. Bleaching powder is manufactured in
established in London in 1946. Its central secretariat is Hasenclever plant.
in Geneva, Switzerland. Hence statement 1 is not Which of the statements given above is/are
correct. Rest statements are correct. correct?
70. Consider the following statements: (a) 1, 2 and 3 (b) 2 and 3
1. India is the second country in the world to (c) 3 only (d) 1 and 2
adopt a National Family Planning Ans. (b) : Anhydrous sodium carbonate is also known
Programme. as calcined soda. It is formed by heating the hydrates. It
2. The National Population Policy of India is also formed when sodium hydrogen carbonate is
2000 seeks to achieve replacement level of heated (calcined).
fertility by 2010 with a population of 111 • Baking soda, (sodium bicarbonate) is used in class
crores. C fire extinguishers. When sodium bicarbonate
3. Kerala is the first State in India to achieve (baking soda) is heated by the fire it becomes
replacement level of fertility. sodium carbonate. During this process water and
Which of the statements given above is/are carbon dioxide is released.
correct? • Hasenclever Plant: Dry slaked lime is moved
(a) 1 only (b) 1 and 2 forward by the revolving blades of the rotating
(c) 2 and 3 (d) 1, 2 and 3 shaft. In the meantime chlorine gas is passed
Ans. (c) : India is not the second country in the world to through the lowermost cylinder which moves up to
adopt a national family planning program. Family the upper cylinders and reacts with the dry slaked
Planning programme was started in 1952 by lime coming downward to form bleaching powder.
Government of India. Kerala is the first state in India to 74. Consider the following statements:
achieve replacement level of fertility. 1. During the process of osmosis, the solvent
travels from the concentrated solution to
UTTAR PRADESH: G.K. the dilute solution.
2. In the reverse osmosis, external pressure is
71. Which of the following states border Uttar
applied to the dilute solution.
Pradesh?
Which of the statements given above is/are
1. Punjab 2. Rajasthan correct?
3. Chhattisgarh 4. Jharkhand (a) 1 only (b) 2 only
Select the correct answer using the code given (c) Both 1 and 2 (d) Neither 1 nor 2
below.
Ans. (d) : During the process of osmosis, the solvent
(a) 1, 2, 3 and 4 (b) 2, 3 and 4 travels from dilute solution to concentrated solution and
(c) 2, 3 and 4 (d) 1 and 3 thus, the 1st statement is incorrect. Reverse osmosis is
Ans. (b) : Uttar Pradesh is bordered by Haryana, Delhi, reverse of the process of osmosis where external energy
Rajasthan in west; Madhya Pradesh in the south and or pressure is applied to the more saline or concentrated
Bihar in the east. The northern boundary is with Nepal. solution. Hence statement 2 is also not correct.

IAS (Pre) GS 2005 Paper I 320 YCT


75. Assertion (A) : The main constituent of the (c) Volume of water saved by using rain-water
Liquefied Petroleum Gas is methane. harvesting.
Reason (R) : Methane can be used directly (d) Volume of water utilized by an effective
for burning in homes and factories where it can flood control.
be supplied through pipelines. Ans. (b) : Virtual water trade (also known as trade in
(a) Both A and R are true and R is the correct embedded or embodied water) refers to the hidden flow of
explanation of A. water if food or other commodities are traded from one
(b) Both A and R are true but R is not the correct place to another. It refers to the sum of the water use in the
explanation of A. various steps of the production chain.
(c) A is correct but R is wrong.
(d) A is wrong but R is correct.
BIOLOGY
Ans. (d) : The assertion is incorrect as Liquefied 79. Pneumoconiosis affects the workers who work
Petroleum Gas (LPG) consists mainly of propane, mainly in:
propylene, butane, and butylene in various mixtures. (a) Tanneries
These gases are present at normal temperature and (b) Coal mining industry
pressure. It does not contain methane. Methane cannot (c) Distilleries
be used directly for burning in homes and factories. (d) Glass industry
76. Consider the following statements: Ans. (b) : Pneumoconiosis, a lung disease, mostly
1. Liquefied Natural Gas (LNG) in liquefied affects workers who work in the mining and
under extremely cold temperatures and construction sector and deals with soil, silica, coal dust
high pressure to facilitate storage or and asbestos. The disease includes asbestosis, silicosis
transportation in specially designed vessels. and coal workers' pneumoconiosis.
2. First LNG terminal in India was built in 80. Assertion (A) : All the proteins in our food are
Hassan. digested in small intestine only.
3. Natural Gas Liquids (NGL) are separated Reason (R) : The protein-digesting enzymes
from LPG and these include ethane, from pancreas are released into small intestine.
propane, butane and natural gasoline. (a) Both A and R are true and R is the correct
Which of the statements given above is/are explanation of A.
correct? (b) Both A and R are true but R is not the correct
(a) 1 only (b) 1 and 3 explanation of A.
(c) 2 and 3 (d) 1, 2 and 3 (c) A is correct but R is wrong.
Ans. (b) : First LNG terminal in India was built in (d) A is wrong but R is correct.
Dahej, Gujarat and another terminal at Kochi, Kerala. Ans. (d) : The digestion of protein begins in stomach.
Hence statement 2 is not correct. Rest of the statements The enzyme pepsin acts on protein and brings about their
is correct. breakdown in the stomach. The protein-digesting
77. Which one of the following is the correct enzymes from pancreas are released into small intestine.
sequence of the given substances in the 81. Assertion (A) : Ameoba reproduces by fission.
decreasing order of their densities?
Reason (R) : All unicellular organisms reproduced
(a) Steel > Mercury > Gold by asexual methods.
(b) Gold > Mercury > Steel (a) Both A and R are true and R is the correct
(c) Steel > Gold > Mercury explanation of A.
(d) Gold > Steel > Mercury (b) Both A and R are true but R is not the correct
Ans. (b) : Gold > Mercury > Steel explanation of A.
Density of gold is 19.30 g/cm3. (c) A is correct but R is wrong.
The density of ultra pure liquid mercury is 13.534 (d) A is wrong but R is correct.
g/cm3. Ans. (a) : Assertion is correct and reason is the correct
The density of steel is 7.80 g/cm3. explanation of assertion. Amoeba is a unicellular
78. Virtual water trade is being looked at by animal. It reproduce by the process of asexual
experts as a solution to the world's water crisis. reproduction. Fission is one of the processes of asexual
What does virtual water (VW) imply. reproduction, where amoeba divides in half to produce
off spring.
(a) Volume of heavy water required to replace
82. Assertion (A) : The person with diabetes
ordinary water.
insipidus feels thirsty.
(b) Volume of water required to produce a Reason (R) : A person with diabetes insipidus
commodity or service. suffers from excess secretion of vasopressin.
IAS (Pre) GS 2005 Paper I 321 YCT
(a) Both A and R are true and R is the correct (a) 1 only (b) 2 only
explanation of A. (c) Both 1 and 2 (d) Neither 1 nor 2
(b) Both A and R are true but R is not the correct
explanation of A. Ans. (b) : Out of the total forest cover, dense forest
constitutes around 2.54%; very dense forest and 8.77% are
(c) A is correct but R is wrong.
moderately dense forest. National Forestry Action
(d) A is wrong but R is correct.
Programme aims a long term plan to achieve the target of
Ans. (a) : Diabetes Insipidus (DI) is a condition 33% forest cover.
characterized by excessive thirst and excretion of large
amounts of severely diluted urine. Diabetes mellitus is a
group of metabolic diseases in which a person has high CURRENT AFFAIRS
blood sugar, due to two causes either the pancreas does
not produce enough insulin, or because cells do not 86. Consider the following statements:
respond to the insulin that is produced. The most 1. Kyoto Protocol came into force in the year
common type in humans is the neurological form which 2005.
involves a deficiency of arginine vasopressin or 2. Kyoto Protocol deals primarily with the
antidiuretic hormone. depletion of the Ozone layer.
83. Which of the following diseases of milching 3. Methane as a greenhouse gas is more
animals are infectious? harmful than carbon dioxide.
1. Foot and Mouth disease 2. Anthrax Which of the statements given above is/are
3. Black Quarter 4. Cowpox correct?
Select the correct using the code given below: (a) 1 and 2 (b) 1 and 3
(a) 1, 2 and 3 (b) 2, 3 and 4 (c) 1 only (d) 3 only
(c) 1 and 4 (d) 1, 2, 3 and 4
Ans. (b) : The Kyoto Protocol is an international
Ans. (c) : Black Quarter is a disease which is found in agreement that aimed to reduce carbon dioxide (CO2)
horse. emissions and the presence of greenhouse gases (GHG) in
Anthrax is a serious infectious disease caused by gram-
the atmosphere. The essential tenet of the Kyoto Protocol
positive, rod-shaped bacteria known as Bacillus
was that industrialized nations needed to lessen the amount
anthracis. Anthrax can be found naturally in soil and
commonly affects domestic and wild animals around of their Green House Gases emissions.
the world. The Kyoto Protocol is an international agreement linked
84. Consider the following statements: to the United Nations Framework Convention on
1. Dengue is a protozoan disease transmitted Climate Change, which commits its parties by setting
by mosquitoes. internationally binding greenhouse gases emission
2. Retro-orbital pain is not a symptom of reduction targets. Therefore statement 2 is incorrect.
Dengue. The Kyoto Protocol was adopted in Kyoto, Japan on 11
3. Skin rash and bleeding from nose and gums December, 1997 and entered into force on 16 February,
are some of the symptoms of Dengue. 2005. Hence statement 1 is correct. Methane traps up to
Which of the statements given above is/are 100 times more heat in the atmosphere than carbon
correct? dioxide, but the fact is that methane also leaves the
(a) 1 and 2 (b) 3 only atmosphere within a decade. This makes methane a
(c) 2 only (d) 1 and 3 short-lived but intense climate changer. So methane
warms the planet rapidly, but it dissipates from the
Ans. (c) : Dengue is a viral disease transmitted through atmosphere more quickly than carbon dioxide.
mosquito. Fever, headache, pain in joints, back muscles Therefore statement 3 is also correct.
and eyeballs, skin rash for a few days, bleeding from
87. Who among the following is not a recipient of
nose and gums are some of the symptoms of the dengue
the Bharat Ratna Award?
fever.
(a) Ustad Bismillah Khan
(b) Satyajit Ray
ENVIRONMENT AND ECOLOGY (c) Lata Mangeshkar
85. Consider the following statements: (d) Raj Kapoor
1. The forest cover in India constitutes around Ans. (d) : Raj Kapoor is not a recipient of Bharat Ratna
20% of its geographical area. Out of the award. Lata Mangeshkar 2001, Bismillah Khan 2001, and
total forest cover, dense forest constitutes Satyajit Ray 1992 are the holders of Bharat Ratna Award.
around 40% 88. Who among the following is the President of
2. The National Forestry Action Programme the FICCI?
aims at Bringing one-third of the area of (a) Sunil Mittal
India under tree/forest cover. (b) Brijmohan Lal Munjal
Which of the statements given above is/are (c) Onkar S. Kunwar
correct? (d) Vivek Burman
IAS (Pre) GS 2005 Paper I 322 YCT
Ans. (c) : When question was asked, Onkar S. Kunwar Ans. (b) : Cochin International Airport, Kochi, Kerala
was the President of FICCI. Mr. Rajesh Shah took over is the first airport in India developed under a public-
as the President of Federation of Indian Chambers of private partnership.
Commerce & Industry (FICCI) for 2017-18. Media 94. Recently, to which of the following countries
executive Uday Shankar has taken over as FICCI did India offer to build a Buddha Temple?
President for 2020-2021. (a) China (b) Myanmar
89. Which one of the following is the landmark (c) Thailand (d) Vietnam
performance of Dr. V. Mohan Reddy, an
Ans. (a) : Top religious officials and local people in
Indian born doctor working in U.S.A., during
central China’s Henan province have warmly welcomed
February, 2005?
India’s initiative to build a Buddhist temple in the
(a) Research on human stem cell which is likely
historic city of Luoyang. During Chinese Premier Wen
to revolutionise treatment of deadly diseases
Jiabao’s recent visit to India, the two governments
like cancer.
signed the Memorandum on Construction of an Indian
(b) Successful open heart procedure called
style Buddhist temple on the western side of the White
arterial switch performed on an infant.
House temple in Luoyang, Henan province.
(c) Research on genetic engineering which can
help in treatment of deadly disease like AIDS. 95. Consider the following:
(d) Successful brain surgery to revive the 1. Disputes with mobile cellular companies
function of nearly dead brain of a premature 2. Motor accident cases
born infant. 3. Pension cases
Ans. (b) : Dr. V. Mohan Reddy in February 2005 For which of the above are Lok Adalats held
performed successful open heart surgery of premature (a) 1 only (b) 1 and 2
baby of 700 gms. (c) 2 only (d) 1, 2 and 3
90. Which countries played in the Lawn Tennis Ans. (c) : Lok Adalat is one of the alternative dispute
Davis Cup Final in the year 2004? redressal mechanisms. It is a forum where
(a) Switzerland and United States of America. disputes/cases pending in the court of law or at pre-
(b) Spain and United States of America. litigation stage are settled/ compromised amicably.
(c) Australia and Argentina. Lok Adalats have been given statutory status under the
(d) Sweden and Belgium. Legal Services Authorities Act, 1987. Matters such as
Ans. (b) : Spain and U.S.A. played the Lawn Tennis matrimonial/family disputes, criminal (compoundable
Davis Cup Final in the year 2004. offences) cases, land acquisition cases, labour disputes,
workmen’s compensation cases, bank recovery cases,
91. Where do the Sahariya tribals, who were etc. are being taken up in Lok Adalats.
recently in the news, live?
(a) Andhra Pradesh (b) Assam 96. Consider the following statements:
(c) Rajasthan (d) Orissa 1. Second World Buddhist Summit was held
in Bangkok in November-December, 2004.
Ans. (c) : The Saharias are mainly found in Morena,
Sheopur, Bhind, Gwalior, Datia, Shivpuri, Vidisha and 2. World Punjabi Conference was held in
Guna districts of Madhya Pradesh and Baran district of Jalandhar in December, 2004.
Rajasthan. 3. 4th Meeting of the SAARC Trade Ministers
92. Which one of the following companies has was held in Islamabad in November, 2004.
started a rural marketing network called 'e- Which of the statements given above is/are
chaupals'? correct?
(a) ITC (a) 3 only (b) 1 and 2
(b) Dabur (c) 2 and 3 (d) 1 only
(c) Proctor and Gamble Ans. (a) : The Second World Buddhist Summit was
(d) Hindustan Lever held at Lumbini, a city in south western Nepal regarded
Ans. (a) : ITC has started a rural marketing network as Buddha's birthplace, during November December
called e-Chaupals. 2004. World Punjabi Conference was held at Punjab
93. Which one of the following airports in India is University in Patiala during December 2004. 4th
the first to be owned by a public limited Meeting of the SAARC trade ministers was held in
company? Islamabad during November, 2004.
(a) Dabolim Airport, Goa 97. Consider the following statements:
(b) Cochin airport 1. Vigyan Rail is a Science Exhibition on
(c) Hyderabad airport wheels organized by the Council of
(d) Bangalore airport Scientific and Industrial Research.
IAS (Pre) GS 2005 Paper I 323 YCT
2. Vigyan Prasar is an autonomous body Ans. (b) : (Location) (Known For/In News for)
under the Ministry of Human Resource A. Kakinada 1. Bio-diesel plant
Development.
B. Dundigal 2. Indian Air Force Academy
3. EDUSAT, the ISRO's educational satellite
was launched from French Guyana in C. Margao 3. Skybus Netri rauk test run
2004. D. Bhadrachalam 4. ITC paper board unit
Which of the statements given above is/are 101. Under which one of the Ministries of the
correct? Government of India does the Food and
(a) 1, 2 and 3 (b) 1 and 2 Nutrition Board work?
(c) 2 and 3 (d) None (a) Ministry of Agriculture
Ans. (d) : Vigyan Rail is a Science Exhibition on (b) Ministry of Health and Family Welfare
wheels organised by "Vigyan Prasar jointly with the
support of Ministry of Railway and Department of (c) Ministry of Human Resource Development
Science and Technology. (d) Ministry of Rural Development
Vigyan Prasar is an autonomous body under the Ans. (c) : The food & Nutrition board was set up in
Ministry of science and Technology. EDUSAT the 1964. It is an attached office of the Ministry of women
ISRO's educational satellite was launched from & Child Development and has a countrywide set up.
Sriharikota in 2004. Originally with Ministry of Food, the FNB was shifted
98. Of which one of the following games is to Ministry of Women and Child Development in 1993.
Shanmugham Venkatesh an outstanding
player? 102. Who among the following is not a member of
(a) Table tennis (b) Hockey the Investment Commission formed in
(c) Football (d) Basketball December, 2004?
Ans. (c) : Shanmugham Venkatesh is an outstanding (a) Ratan Tata
player in Indian football team. (b) Deepak Parekh
99. In which country is Bandung, where the (c) Ashok Ganguly
Conference of African and Asian nations was (d) Kumamangalam birlaj
held which led to establishing Non-Aligned
Ans. (d) : Investment Commission formed in
Movement (NAM), situated?
December, 2004 was headed by Tata Group chairman
(a) Thailand (b) Egypt
Ratan Tata. The other two members were HDFC
(c) Indonesia (d) Philippines
chairman Deepak Parekh and chairman of I-one-Source
Ans. (c) : Bandung is at Indonesia where the
and Former Chairman of HLL Ashok Ganguly.
Conference of African and Asian nation was held which
led to establishing Non-Aligned Movement (NAM). 103. In which one of the following countries did
100. Match items in the List I (Location) with those Hundreds of People die in year 2004 as a result
in List II (Known For/In News for) and select of flooding and mudslides triggered by
the correct answer using the codes given below Tropical Storm Jeanne?
the lists: (a) Colombia (b) Haiti
List I List II (c) Sudan (d) Ghana
(Location) (Known For/In News Ans. (b) : Hundreds of People died in year 2004 in
for) Haiti as a result of flooding and mudslides triggered by
A. Kakinada 1. Skybus Netri rauk Tropical Storm Jeanne Haiti.
test-run 104. Which one of the following statements is
B. Dundigal 2. ITC paper board correct?
unit (a) The First Meeting of the Parties (MOP 1) to
C. Margao 3. bio-diesel plant the Cartagena Protocol on Biosafety was held
D. bhadrachalam 4. Indian Air Force in Philippines in the year 2004.
Academy (b) India is not a signatory to the Biosafety
Codes: Protocol/Convention on Biological Diversity.
A B C D (c) The biosafety Protocol deals with genetically
(a) 2 1 4 3 modified organisms.
(b) 3 4 1 2 (d) The United States of America is members of
(c) 2 4 1 3 the Biosafety Protocol/ Convention on
(d) 3 1 4 2 biological Diversity.

IAS (Pre) GS 2005 Paper I 324 YCT


Ans. (c) : The biosafety Protocol deals with genetically Codes:
modified organisms. A B C D
Cartagena Protocol on Bio safety was adopted on 29th (a) 5 1 4 2
January and came into force on 11th September 2003. (b) 2 3 1 5
As on June 15, 2020, it has 173 members. India along (c) 5 3 1 2
with United States of America are signatory to this (d) 2 1 4 5
protocol. The Cartagena Protocol on Bio safety/
Ans. (b) : (Lawn Tennis (Winner of Women Signals)
Convention on Biological Diversity is an international
Tournament)
agreement which aims to ensure the safe handling,
transport and use of living modified organisms, French Open-2004 Anastasia Myskina
resulting from modern biotechnology that may have Wimbledon-2004 Maria Sharapova
adverse effects on biological diversity, taking also into US Open-2004 Svetlana Kuznetsova
account risks to human health. Hence third statement is Australian Open-2004 Justine Henin-Hardene
not correct. 108. Consider the following statements:
105. Where was world's largest ever Meet on HIV 1. The Man Booker Prize is awarded to citizen
AIDS held in July, 2004? of any of the countries of the British
(a) Bangkok (b) Singapore Common wealth or the Republic of Ireland.
(c) New York (d) Rome 2. A leading London-Based software company
underwrites the Man Booker Prize
Ans. (a) : AIDS has undoubtedly emerged as the
presently.
history’s biggest epidemic. To address this menace,
around 20,000 policy makers, scientist, activists and 3. The winner of the Man Booker Prize in year
celebrities attended the 15th International Aids 2004 is a South Asian.
Conference at Bangkok. Which of the statements given above is/are
correct?
106. Consider the following statements: (a) 1 only (b) 1 and 3
1. Global Trust Bank has been amalgamated (c) 2 and 3 (d) 1, 2 and 3
with the Punjab National Bank.
Ans. (a) : The Man Booker Prize is awarded to citizen
2. The second report of the Kelkar Committee
of any of the countries of the British Common wealth or
dealing with direct and indirect taxes has the Republic of Ireland. Rest of the statements is not
maintained its original recommendations correct.
including the abolition of exemptions
relating to housing loans. 109. Which of the following pairs are correctly
matched?
Which of the statements given above is/are
Person Award
correct?
1. Champadevi Shukla : Goldman
(a) 1 only (b) 2 only
Environmental
(c) Both 1 and 2 (d) Neither 1 nor 2 Prize
Ans. (b) : Global Trust Bank Ltd has been amalgamated 2. Dr. P. Sri : Vachaspati
with the Oriental Bank of Commerce. The merger took Ramachanrudu
place on 14th August, 2004. Hence statement 1 is not Puraskar
correct. Second statement is correct.
3. Ela Ramesh Bhatt : Lal Bahadur
107. Match List-I (Law Tennis Tournament) with Shastri National
List-II (Winner of Women Singles) and select Award for
the correct answer using the codes given below excellence in
the lists: Public
List I List II Administration,
(Lawn Tennis (Winner of Women Academics and
Tournament) Singles) Management
A. French Open-2004 1. Svetlana 4. Upamanyu : Lalit Kala Ratna
Kuznetsova Chatterjee Award
B. Wimbledon-2004 2. Anastasia Myskina Select the correct answer using the code given
C. US Open-2004 3. Maria Sharapova below.
(a) 1 and 2 (b) 1, 2 and 3
D. Australian Open- 4. Serena Williams
(c) 3 and 4 (d) 1, 2, 3 and 4
2004
5. Justine Henin- Ans. (b) : Upamanyu Chatterjee is the winner of Sahitya
Hardene Award winner and not Lalit Kala Award winner.

IAS (Pre) GS 2005 Paper I 325 YCT


MISCELLANEOUS (b) Salem Steel Plant is a premier producer of
stainless steel in India.
110. According to Census 2001, which one of the (c) Maharashtra Elektrosmelt Ltd. is a subsidiary
following Indian States has the maximum of the Steel Authority of India Ltd.
population in India after Uttar Pradesh? (d) Visakhapatnam Steel Plant is a unit of the
(a) West Bengal (b) Maharashtra Rashtriya Ispat Nigam Ltd.
(c) Bihar (d) Tamil Nadu Ans. (a) : Rourkela Steel Plant, the first integrated steel
Ans. (b) : According to 2001 census, Maharashtra had plant in the Public sector of India was set-up with the
the maximum population in India after Uttar Pradesh. collaboration of West Germany.
The total population in Maharashtra was 96,878,627. 116. Match List I with List II and select the correct
answer using the codes given below the lists:
111. Which one of the following cities is not a
List I (Person) List II (Organization)
former capital of the given country (Country
given in the brackets)? A. V.R.S. Natarajan 1. Bharat Heavy
(a) Karachi (Pakistan) Electricals Limited
(b) Auckland (New Zealand) B. A.K. Puri 2. Air India
(c) Kyoto (Japan) C. V. Thulasidas 3. Maruti Udyog Limited
(d) Brisbane (Australia) D. Jagdish Khattar 4. Bharat Earth Movers
Ans. (b) : Karachi was the capital of Pakistan from Limited
1947 – 1959. 5. Indian Space Research
Kyoto was the capital of Japan before Tokyo. Organization
Brisbane was the capital of Australia before Cannbera A B C D
in 1842 – 1859. (a) 2 3 5 1
112. For which one of the following countries, is (b) 4 1 2 3
Spanish not an official language? (c) 2 1 5 3
(d) 4 3 2 1
(a) Chile (b) Columbia
(c) Republic of Congo (d) Cuba Ans. (b) : V.R.S. Natarajan Bharat Earth
Movers Limited
Ans. (c) : French is the official language of Republic of A.K. Puri Bharat Heavy
Congo, not Spanish. But Spanish is the official language Electricals Limited
of Chile, Columbia and Cuba. V. Thulasidas Air India
113. Whose autobiography is the book "My Music, Jagdish Khattar Maruti Udyog
My Life"? Limited
(a) Pandit Shiv Kumar Sharma 117. Which one of the following was probed by the
(b) Ustad Amjad Ali Khan Liberhan Commission?
(c) Pandit Ravi Shankar (a) Test Cricket match fixing.
(d) Ustad Zakir Hussain (b) Best Bakery Case.
Ans. (c) : ‘My Music, My life’ is the autobiography of (c) Tehelka tapes Case.
Pandit Ravi Shankar. (d) Demolition of the disputed structure at
114. Which one of the following pairs is not Ayodhya.
correctly matched? Ans. (d) : The Liberhan Commission was commissioned
(a) Arjun : Indigenously produced by the Government of India to investigate the demolition
Main Battle Tank of the disputed structure Babri Masjid in Ayodhya in 1992.
(MBT) 118. Match List I with List II and select the correct
(b) Phalcon : Cruise missile supplied answer using the codes given below the lists:
by Russia to India List I List II
(c) Saras : Indigenously developed (Distinguished (Achievement/
civilian passenger Person) Known as)
aircraft A. Guenter Grass 1. First woman Prime
(d) Operation Seabird : New Indian Naval Base Minister of Canada
at Karwar B. Trevor 2. Nobel Prize Winner for
Ans. (b) : Phalcon is a radar system provided by Israel Huddleston literature
to India. C. Dicky Dolma 3. Leading campaigner
against apartheid in
115. Which one of the following statements is not
South Africa
correct?
D. Kim Campbell 4. Youngest woman to
(a) Rourkela Steel Plant, the first integrated steel
climb the Mt. Everest
plant in the Public Sector of India was set up
5. American violinist
with the Soviet Union collaboration.
IAS (Pre) GS 2005 Paper I 326 YCT
A B C D Ans. (d) : B.V. Rao was an Indian Agriculturalist and
(a) 5 3 2 1 Poultry Farmer, considered by many as the Father of
(b) 2 3 4 1 Poultry Farming in India. C.K. Prahalad widely
(c) 5 3 4 2 recognized as one of the world's 10 most influential
Management Gurus, John Kurian is related to
(d) 2 4 1 3
Fisheries Economy. Lastly, Kiran Karnik was
Ans. (b) : Guenter Grass: The 71-year old German President of NASSCOM, the premier trade body and
novelist, who confronted his countrymen with their 'Chamber of Commerce for the IT Software and
guilty silence after the Holocaust, won the Novelist Services Industry in India.
Prize for Literature. 121. Who among the following invented Lasers?
Archbishop Trevor Huddleston: The Anglican monk (a) Theodore Maiman (b) Denis Papin
who led the British campaign to end apartheid in South (c) William Moton (d) Francis Crick
Africa.
Ans. (a) : Theodore Maiman invented Lasers.
Dicky Dolma : Youngest woman in the world to climb
122. Which one of the following is not an ASEAN
Mt. Everest in 1993.
member?
Kim Campball : The first and only female Prime (a) Cambodia (b) China
Minister of Canada.
(c) Laos (d) Philippines
119. Consider the following statements: Ans. (b) : The Association of Southeast Asian Nations,
1. The Charter of the United Nations or ASEAN, was established on 8 August 1967 in
Organization was adopted at Geneva, Bangkok, Thailand, with the signing of the ASEAN
Switzerland in June, 1945. Declaration (Bangkok Declaration) by the Founding
2. India was admitted to the United Nations Fathers of ASEAN, namely Indonesia, Malaysia,
Organization in the year 1945. Philippines, Singapore, and Thailand. It is a regional
3. The Trusteeship Council to the United intergovernmental organization comprising ten
Nations Organization was established to Southeast Asian countries - Indonesia, Cambodia,
manage the affairs of territories detached Myanmar (Burma), Thailand, Vietnam, Laos, Brunei,
from Japan and Italy after the Second Malaysia, Singapore, Philippines.
World War or such territories not under 123. Consider the following statements:
the control of a country at that time. 1. The Parliament of Russia is called Federal
Which of the statements given above is/are Assembly.
correct? 2. The Council of the Federation in the
Russian Parliament is the lower house.
(a) 1, 2 and 3 (b) 2 only
3. The name of the upper house in the
(c) 2 and 3 (d) 3 only
Russian Parliament is State Duma.
Ans. (c) :Statement (1) is not correct. The charter of Which of the statements given above is/are
UNO was adopted at San Francisco in June 1945. Rest correct?
of the statements are correct. (a) 1, 2 and 3 (b) 1 and 2
120. Match List I with List II and select the correct (c) 2 and 3 (d) 1 only
answer using the codes given below the lists: Ans. (d) : The name of the parliament of Russian
List I List II Federation is the Federal Assembly of Russia.
(Distinguished (Area of Work) • The term for its lower house, State Duma (which is
person) better known than the Federal Assembly itself, and
A. B.V. Rao 1. Automobiles is often mistaken for the entirety of the parliament)
Manufacture comes from the Russian word dumat, "to think".
B. C.K. Prahalad 2. Fisheries Economy • The Federation Council is the upper house of the
Federal Assembly of according to the 1993
C. John Kurien 3. Information Technology
Constitution of the Russian Federation.
and Software
D. Kiran Karnik 4. Poultry Farming
Directions:
(For the next FOUR items): Based on the
5. Management Science
information given below, answer the four items
A B C D which follow it:
(a) 2 5 1 3 Gopal, Harsh, Inder, Jai and Krishnan have
(b) 4 3 2 5 Ahmedabad, Bhopal, Cuttack, Delhi and
(c) 2 3 1 5 Ernakulam as their hometowns (Not
(d) 4 5 2 3 necessarily in that order). They are studying in
IAS (Pre) GS 2005 Paper I 327 YCT
Engineering, Medical, Commerce, Economics 128. Left pan of a faulty balance weighs 100 gram
and History Colleges (Not necessarily in that more than its right pan. A Shopkeeper Keep
order). None of the five boys is studying in his the weight measure in the left pan while buying
goods but keeps it in the right pan while selling
hometown, but each of them studies Further, it his goods. He uses only 1 kg weight measure. If
is given that: he sells his goods at the listed cost price, what is
(i) Gopal's hometown is Ernakulam. his gain?
(ii) Harsh is not studying in Ahmedabad or (a) 200/11% (b) 100/11%
Bhopal. (c) 100/9% (d) 200/9%
Ans. (d) : Let the cost price of 1 kg goods be Re 1
(iii) Economics College is in Bhopal. Then he buys 1100gm of goods for Re 1 and sells 900
(iv) Inder's hometown in Cuttack gm of goods for Re 1
(v) Krishnan is studying in Delhi. ∴ Cost price of 1100 gm of goods = Re 1
(vi) Jai is studying in Ernakulam and the History 1
⇒ Cost price of 900 gm of goods = × 900
College is in his hometown Ahmedabad. 1100
(vii) Engineering College is situated in 9
Ernakulam. =
11
Based on the information given above answer Selling price of 900 gm of goods = Re 1
the next FOUR items: 9 2
Gain = 1 − =
The given information can be tabulated as– 11 11
Student Home College in Studying Sub 2 11 200
Town Home Town ject Gain% = × ×100 = %
Gopal Ernakulum Engineering Bhopal/
11 9 9
Ahmedabad 129. On a railway route between two places A and
Harsh Delhi Commerce/ Cuttuck b, there are 10 stations on the way. If 4 new
Medical stations are to be added how many types of new
Inder Cuttuk Commerce/ Bhopal/ tickets will be required if each ticket is issued
Medical Ahmedabad for a one-way journey?
Jai Ahmedabad Ernakulum Eng (a) 14 (b) 48
inee (c) 96 (d) 108
ring
Ans. (d) : There are 10 + 2 - stops along the line before
Krishnan Bhopal Delhi
the 4 new stops.
124. Which is Krishnan's hometown? ∴ Each of the 4 new stations that can go back and forth
(a) Ahmedabad between these 12 stops = 4 × 12 × 2 = 96 new one
(b) Cuttack way ticket would be needed.
(c) Bhopal Again
For each 4 stations that can go back and forth between
(d) Cannot be determined each other there are 4!/2! = 12 ways to select
Ans. (c) : Bhopal ∴ Total one way ticket required = 96 + 12 = 108
125. Which College is situated in Inder's hometown? 130. Aryan runs at a speed of 40 metre/minute.
(a) Commerce Rahul follows him after an interval of 5
(b) Medical minutes and runs at a speed of 50
(c) Economics metre/minute. Rahul's dog runs at a speed of
60 metre/minute and starts along with Rahul.
(d) Commerce or Medical
The dog reaches Aryan and then comes back to
Ans. (d) : Commerce or Medical Rahul and continues to do so till Rahul reaches
126. Who studies in Bhopal? Aryan. What is the total distance covered by
(a) Gopal (b) Harsh the dog?
(c) Gopal or Inder (d) Inder or Harsh (a) 600 metres (b) 750 metres
(c) 980 metres (d) 1200 metres
Ans. (c) : Gopal or Inder
Ans. (d) : Distance covered by Aryan in 5 min = 40 × 5
127. If Inder studies in Ahmedabad, then which one = 200 m.
of the following is the correct combination of Relative speed of Rahul with reference to Aryan
Person–Hometown–Place of Study? 50 – 40 = 10 metres/minute
(a) Gopal–Ernakulam–Delhi ∴ Time taken by Rahul to cover 200m at relative
(b) Jai–Ahmedabad–Ernakulam 200
(c) Krishanan–Delhi–Ernakulam Speed = = 20 minutes
10
(d) Harsh–Bhopal–Delhi ∴ Distance covered by the dog = 60 × 20
Ans. (b) : Jai–Ahmedabad–Ernakulam = 1200 metres
IAS (Pre) GS 2005 Paper I 328 YCT
131. A big rectangular plot of area 4320 m2 is 133. Which of the following pairs are correctly
divided into 3 square-shaped smaller plots y matched?
fencing parallel to the smaller side of the plot. Publication Group
However some area of land was still left as a 1. Sportstar : The Hindu publication
square could not be formed. So, 3 more square- group
shaped plots were formed by fencing parallel to 2. Business world : ABP group
the longer side of the original plot such that no 3. The Week : Malayala Manorama
area of the plot was left surplus. What are the publication group
dimensions of the original plot? 4. Reader's Digest : Indian Express
(a) 160 m × 27 m (b) 240 m × 18 m publication group
(c) 120 m × 36 m (d) 135 m × 32 m Select the correct answer using the codes given
below:
(a) 1, 2, 3 and 4 (b) 1, 2 and 3
Ans. (c)
(c) 2 and 4 (d) 1 and 3
Ans. (b) : Readers Digest Magazine in India belongs to
Times of India group. Rest is correctly matched.
Let the side of each large square be x and that of each 134. Which of the following pairs are correctly
small square be y matched?
∴ 3 (x2) + 3 (y2) = 4320 Enterprise Industrial Group
2 2
x + y = 1440 ……….(i) 1. VSNL : Bharati Group
and area of original plot be 2. Mundra Special : Adani group
x × (3x + y) = 4320 ………(ii) Economic Zone Ltd.
be equ. (i) and (ii) we get 3. CMC Ltd. : Tata Group
2 2 2 4. IPCL : Reliance Group
3 x + xy = 3 ( x + y )
xy = 3y2 Select the correct answer using the codes given
x = 3y ……………(iii) below:
Now from equ. (i) and (iii) (a) 1, 2 and 3 (b) 3 and 4
(3y) + y = 1440 ⇒ 10y = 1440
2 2 2 (c) 1, 2 and 4 (d) 2, 3 and 4
y = 144 ⇒ y = 12
2 Ans. (d) : Videsh Sanchar Nigam Limited belongs to
Tata group. Rest is correctly matched.
So x = 36
135. Match items in the List I (Business woman)
∴ Length of the plot = 3 × 36 + 12 = 120m
with List II (Company) and select the correct
Width of the plot = x = 36 m.
answer using the codes given below the lists:
132. 2 men and 1 woman board a bus in which 5 List I List II
seats are vacant. One of these five seats is (Businesswoman) (Company)
reserved for ladies. A woman may or may not
A. Zia Mody 1. Venkateshware
sit on the seat reserved for ladies but a man
Hatcheries
cannot sit on the seat reserved for ladies. In
how many different ways can the five seats be B. Anuradha J. Desai 2. AZB & Partners
occupied by these three passengers? C. Villoo Morawala 3. Quantum Market
(a) 15 (b) 36 Patell Research
(c) 48 (d) 60 D. Meena Kaushik 4. Avestha Gengraine
Ans. (b) : Case 1 – Technologies
If lady sits on reserved seat then 5. Biocon India
4
2 men can occupy seats from 4 vacant seat in P2 ways Codes:
A B C D
= 4 × 3 = 12 ways
(a) 4 1 5 3
Case 2 –
(b) 2 3 4 1
If lady does not sits on reserved seat then the lady can
(c) 4 3 5 1
occupy a seat from 4 vacant seat in 4 ways 1 and 1 man
can occupy a seat from 3 seats in 3 ways and another (d) 2 1 4 3
man can occupy a seat from remaining 2 seats in two Ans. (d) : Zia Mody AZB & Partners
ways. Anuradha J. Desai Venkateshware Hatcheries
Total ways 4 × 3 × 2 = 24 ways Villoo Morawala Patel Avestha Gengraine
Hence from both the cases total number of ways = 12 + 24 Technologies
= 36 ways Meena Kaushik Quantum market research

IAS (Pre) GS 2005 Paper I 329 YCT


136. A square is divided into 9 identical smaller 141. There are 6 persons : A, B, C, D, E and F, A
squares. Six identical balls are to be placed in has 3 items more than C
these smaller squares such that each of the D has 4 items less than B
three rows gets at least one ball (one ball in one E has 6 items less than F
square only). In how many different ways can
this be done? C has 2 items more than E
(a) 27 (b) 36 F has 3 items more than D
(c) 54 (d) 81 Which one of the following figures cannot be
equal to the total number of items possessed by
Ans. (d) :
all the 6 persons?
137. There are 6 persons– A, B, C, D, E and F. They
(a) 41 (b) 47
are to be seated in a row such that B never sits
anywhere ahead of A and C never sits (c) 53 (d) 58
anywhere ahead of B. In how many different Ans. (d) : According to question–
ways can this be done? A=C+3
(a) 60 (b) 72 D=B–4
(c) 120 (d) None of the above E=F–6
Ans. (d) : C=E+2
138. For which one of the following items, is F=D+3
Tirupur well-known as a huge exporter to Also it can be inferred that A = B – 2
many parts of the world? ∴ total no. of items = A + B + C + D + E + F
(a) Gems and Jewellery (b) Leather goods
= A + (A + 2) + (A – 3) + (A – 2) + (A – 5) + (A + 1)
(c) Knitted garments (d) Handicrafts
= 6A – 7
Ans. (c) : Tirupur in Tamil Nadu has gained universal
recognition as the leading source of Hosiery, Knitted
If A = 8 then total no. of items = 8 × 6 – 7 = 41
Garments, Casual Wear and Sportswear. For A = 9 then total no. of items = 9 × 6 – 7 = 47
139. Which one of the following pairs is not For A = 10 then total no. of items = 10 × 6 – 7 = 53
correctly matched? ∴ 58 can not be equal to the total number of items
Project Company possessed by all the 6 persons.
(a) Integrated Steel Plant : Steel Authority of 142. Ten identical particles are moving randomly
at Jajpur (Orissa) India inside a closed box. What is the probability that
(b) Power Plant at : Essar Power at any given point of time all the ten particles
Jamnagar (Gujarat) will be lying in the same half of the box?
(c) Nabinagar Power Plant : Indian Railways (a) 1/2 (b) 1/5
(Bihar) (c) 1/2 9
(d) 2/11
(d) Kayamkulam Power : National Thermal
Plant (Kerala) Power Corporation Ans. (c) : Probability of a particle lying in any
particular half = 1/2
Ans. (a) : Integrated steel power plant at Jajpur, Orissa
belongs to Tata group. Rest is correctly matched. ∴ Probability of all 10 particles lying in either
10 10 10
140. 300 persons are participating in a meeting, out 1 1 1 1
half =   +   = 2   = 9
of which 120 are foreigners and the rest are 2 2 2 2
Indians. Out of the Indians, there are 110 men
who are not judges; 160 are men or judges and 143. An equilateral triangular plate is to be cut into
35 are women judges. There are no foreign n number of identical small equilateral
judges. How many Indian women attended the triangular plates. Which one of the following
meeting? can be possible value of n?
(a) 35 (b) 45 (a) 196 (b) 216
(c) 55 (d) 60 (c) 256 (d) 296
Ans. (c) : Foreigner = 120 Ans. (c) : An equilateral triangle whenever divided into
Indian = 180 equal parts, gives 4 equilateral triangles. Hence it is
always its power of 4 whenever it is equally divided
further.
Thus only 44 = 256 satisfy the above condition.
144. There are 10 identical coins and each one of
them has 'H' engraved on its one face and 'T'
160 men or judge and 35 women judges engraved on its other face. These 10 coins are
∴ men = 110 + 15 = 125 lying on a table and each one of them has 'II'
women = 160 – 125 = 55 face as the upper face.
IAS (Pre) GS 2005 Paper I 330 YCT
In one attempt, exactly four (neither more nor Ans. (d) : Pondicherry a Union Territory has the sex
less coins can be turned upside down.) What is ratio of 1001. Chandigarh has 777, Delhi has 861
the minimum total number of attempts in Daman and Diu has 710, Dadra and Nagar Haveli has
which the 'T' faces of all the 10 coins can be
812.
brought to be the upper faces?
(a) 4 (b) 7 149. Match List-I (Famous Former Sports-person)
(c) 8 (d) Not possible with List-II (Game/sport) and select the correct
answer using the codes given below the lists:
Ans. (a) : On the 1st attempt and coins are over turned.
Now, 6 coins are left List I List II
2nd the next turn 4 more are overturned and only two are (Famous Former (Game/Sport)
left. Sportsperson)
Now in 3rd attempt we take one more from remaining A. Wally Gruout 1. Svetlana
two coins and any 3 from previously turned ones. Kuznetsova
Finally in 4th attempt the 1 left coin and 3 coins from B. Eusebio 2. Anastasia Myskina
previous steps which have already over turned twice can
be over turned. C. Rod Laver 3. Maria Sharapova
Hence in 4 attempts required process can be completed. D. Dark Spitz 4. Serena Williams
145. Which one of the following pairs is not 5. Justine Henin-
correctly matched? Hardene
British Prime Minister Political Party
Codes:
(a) John Major : Conservative Party
(b) James Callaghan : Labour Party A B C D
(c) Harold wilson : Conservative Party (a) 3 5 2 4
(d) Margaret Thatcher : conservative Party (b) 2 4 3 1
Ans. (c) : Harold Wilson was from labour Party. Rest is (c) 3 4 2 1
correctly matched. (d)
2 5 3 4
146. In which one of the following countries, Tamil Ans. (c) : (Famous Former (Game/Sport)
is a major language?
Sportsperson)
(a) Myanmar (b) Indonesia
Wally Gruout Maria Sharapova
(c) Mauritius (d) Singapore
Eusebio Serena Williams
Ans. (d) : In Singapore, Tamil is a major language.
Rod Laver Anastasia Myskina
147. Which one of the following pairs is not
Dark Spitz Svetlana Kuznetsova
correctly matched?
Current Name Old Name 150. Consider the following statements:
(a) Harare : Salisbury 1. The number of post offices in India is in
(b) Ethiopia : Abyssinia excess of 1.5 lakhs.
(c) Ghana : Dutch Guiana 2. Bharat Sanchar Nigam Limited (BSNL) was
(d) Kinshasa : Leopoldville formed in the year 1997.
Ans. (c) : Ghana - Dutch Guiana 3. Telecom Regulatory Authority of India
148. Which one of the following is the correct (TRAI) was established in the year 2000.
statement on the basis of Census-2001? Which of the statements given above is/are
(a) Bihar has the highest percentage of the correct?
Scheduled Castes of its population (a) 1, 2 and 3 (b) 1 and 2
(b) The decadal growth of population of India (c) 1 only (d) 3 only
(1991-2001) has been below 20%
Ans. (c) : India has world’s largest postal network with
(c) Mizoram is the Indian State with the least
1,55,837 post office (1,39,280 in rural areas). A post
population
office serves an area of 21.09 sq kms and population of
(d) Pondicherry has highest sex ratio among the
6,602.
Union Territories.
IAS (Pre) GS 2005 Paper I 331 YCT
UNION PUBLIC SERVICE COMMISSION
Civil Services (Preliminary Exam) - 2004
GENERAL STUDIES : PAPER-I
Time: 2 hours Maximum Number: 200
(a) 1 only (b) 2 only
ANCIENT HISTORY (c) Both 1 and 2 (d) Neither 1 nor 2
1. Which one of the following sequences indicates Ans. (d) :The fourth Buddhist Council was held in
the correct chronological order? Kashmir during the reign of Kanishka under the
(a) Shankaracharya – Ramanuja – Chaitanya presidentship of Vasumitra and Ashvaghosh. Fa-Hien
(b) Ramanuja – Shankaracharya – Chaitanya visited India during the reign of Chandra Gupta-II and
(c) Ramanuja – Chaitanya – Shankaracharya not during Kanishka's reign. Harsha was not
(d) Shankaracharya – Chaitanya – Ramanuja antagonistic to Buddhism when Hiuen -Tsang met him.
Ans. (a) : The correct chronological order is 4. With reference to ancient Jainism, Which one
Shankaracharya (9th Century CE) - Ramanuja (1017- of the following statements is correct?
1137 CE) - Chaitanya (1436-1533 CE) (a) Jainism was spread in South India under the
2. Consider the following statements: leadership of Sthulabahu
1. Narasimha Saluva ended the Sangama (b) The Jainas who remained under the
dynasty and seized the throne for himself leadership of Bhadrabahu were called
& started the Saluva dynasty. Shvetambaras after the Council held at
2. Vira Narasimha deposed the last Saluva Pataliputra
ruler and seized the throne for himself. (c) Jainism enjoyed the patronage of the Kalnga
3. Vira Narasimha was succeeded by his king Kharavela in the first century BC
younger brother, Krishnadeva Raya. (d) In the initial stage of Jainism, the Jainas
4. Krishnadeva Raya was succeeded by his worshipped images unlike Buddhists
half brother, Achyuta Raya. Ans. (c) : Jainism enjoyed the patronage of Kalinga
Which of the statements given above are king Kharavela in the first century BC.
correct? Jainism spread in South India under the leadership of
(a) 1, 2 and 3 (b) 2, 3 and 4 Bhadrabahu (not Sthalabahu). The Jainas who remained
(c) 1 and 4 (d) 1, 2, 3 and 4 under the leadership of Bhadrabahu were called
Ans. (d): In the Vijayanagara Empire, the Sangam Digambaras (not Shvetambaras) after the first council
dynasty, Saluva dynasty, Tuluva dynasty and Aravidu held at Pataliputra (modern Patna) by Sthulbahu. Like
dynasty ruled respectively.The rulers were Harihar I, Buddhism, Jain also did not worshipped images during
Bukka I, Harihar II, Devaraya I, Devaraya II, initial stage.
etc.Narasimha Saluva established the Saluva dynasty,
5. Which one of the following four Vedas contains
Vir Narasimha established the Tuluva dynasty by
killing Narasimha Raya II the last ruler of the Saluva an account of magical charms and spells?
dynasty. Krishnadeva Raya was the ruler of the Tuluva (a) Rig-veda (b) Yajur-veda
dynasty. Other rulers of the Tuluva dynasty were (c) Atharvaveda (d) Sama-veda
Achyuta Rai, Sadashiv Rai etc. The famous battle of Ans. (c) :Atharvaveda contains mantras on three topics
Talikota took place during the time of Sadashiv Rai - Gyana (Knowledge), Karma (deeds), and Upasana
during which Vijayanagara was defeated. (Source (invocation). It is important from the point of view of
Medieval India - Harishchandra Varma). knowing the history of science in India. It is also
3. Consider the following statements: collection of spells and charms which are popular
1. The Chinese pilgrim Fa – Hien attended among the people. This Veda throws light on the beliefs
the fourth Great Buddhist Council held by of the people, some of the Mantras are meant to bring
Kanishka. success in life, while some were used to ward off evil
2. The Chinese pilgrim Hiuen – Tsang met spirits responsible for disease and sufferings. This Veda
Harsha and found him to be antagonistic to is believed to be a later composition and contains some
Buddhism. non-Aryan material. It seems to have been composed
Which of the statements given above is/are when a synthesis of Aryan and non-Aryan cultures took
correct? place.
IAS (Pre) GS 2004 Paper I 332 YCT
MEDIEVAL HISTORY MODERN HISTORY
6. Who was the last ruler of the Tughluq dynasty
9. The Montague-Chelmsford Report formed the
of the Delhi Sultanate? basis of
(a) Firuz Shah Tughlaq (a) the Indian Council Act, 1909
(b) Ghiyas-ud-din Tughlaq Shah II (b) the Government of India Act, 1919
(c) Nasir-ud-din Mahmud (c) the Government of India Act, 1935
(d) Nusrat Shah (d) the Indian Independence Act, 1947
Ans. (b) : Government of India Act, 1919, was passed
Ans. (c) :Nasiruddin Mahmud was the last ruler of the
by British Parliament to further expand the participation
Tughlaq dynasty who ruled from 1394 to 1412. In the of Indians in the Government of India. Since the act
following years, the Delhi Sultanate had disintegrated embodied reforms as recommended by a report of
further. Many provinces like Malwa and Gujarat Edwin Montagu (Secretary of State for India) and Lord
declared their independence. Timur invaded India in Chelmsford (Viceroy and Governor General), it is also
1398 A.D., which further worsened the situation. His called as Montague-Chelmsford Reforms or simply
invasion delivered a death blow to the Tughlaq dynasty. Mont-Ford Reforms. The most notable feature of the
Act was “end of benevolent despotism” and
7. How did the dynasty of Nizam Shah is of introduction of responsible government in India.
Ahmadanagar come to an end? 10. Consider the following statements:
(a) Ahmadnagar was annexed into Mughal Some of the main features of the Government
empire and Husain Shah was consigned to life of India Act, 1935 were the
imprisonment 1. Abolition of diarchy in the Governor's
(b) Mughal troops destroyed Daulatbad fort and provinces
killed Nizam-ul Mulk of Ahmadnagar 2. Power of the Governors to veto legislative
(c) Fateh Khan usurped the throne from Nizam- action and to legislate on their own
ul-Mulk 3. Abolition of the principle of communal
representation which of the statements
(d) Malik Ambar was defeated in a battle with given above is/are correct?
Mughals in 1631 and the entire royal family (a) 1 only (b) 1 and 2
was killed by the Mughal troops (c) 2 and 3 (d) 1, 2 and 3
Ans. (a) : Ahmadnagar was founded by Malik Ahmad Ans. (b) : By the Act of 1919, the rule of diarchy in the
of Nizam Shahi dynasty in 1490. It was annexed by provinces and bicameral system in the center was
Shahjahan in 1633 into the Mughal Empire. implemented. By the Government of India Act of 1935,
8. Consider the following statements about Sikh the diarchy in the provinces was abolished and
Gurus: implemented at the center. Bicameral system was
implemented in some provinces. In this act, the
1. Banda Bahadur was appointed as the military
Governors were given veto power in legislative
leader of the Sikhs by Guru Tegh Bahadur
activities and they were given the power to legislate
2. Guru Arjun Dev became the Sikh Guru after themselves. The basis of the communal electoral
Guru Rama Das. system was further expanded. (Source- NCERT -
3. Guru Arjun Dev gave to Sikhs their own Modern India - SK Pandey)
script-Guru mukhi. 11. Consider the following statements:
Which of the statements given above is/are 1. The First Session of the Indian National
correct? Congress was held in Calcutta.
(a) 1 only (b) 2 and 3 2. The Second Session of the Indian National
(c) 1 and 3 (d) 1 and 2 Congress was held under the presidentship
Ans. (*) : Banda Bahadur was appointed as the military of Dadabhai Naoroji.
3. Both Indian National Congress and
leader of the Sikhs by the 10th Sikh Guru, Guru Gobind
Muslim League held their sessions at
Singh and not Guru Tegh Bahadur. The 2nd Sikh Guru,
Lucknow in 1916 and concluded the
Guru Angad gave to Sikh their own script- 'Guru Lucknow Pact.
Mukhi', not Guru Arjun Dev. Hence statement 1 and 3 Which of the statements given above is/are
are incorrect. Guru Arjun Dev became the Sikh Guru correct?
after Guru Rama Das. Hence statement 2 only correct (a) 1 and 2 (b) 2 only
answer. (c) 2 and 3 (d) 3 only
IAS (Pre) GS 2004 Paper I 333 YCT
Ans. (c) : Founded by A.O. Hume, The first session of 14. Consider the following princely States of the
the Indian National Congress was held on 28–31 British rule in India:
December 1885 at Gowalia Tank in Mumbai. It was 1. Jhansi 2. Sambalpur
chaired by Womesh Chandra Banerjee (W.C. Banerjee). 3. Satara
The second session of the Indian National Congress The correct chronological order in which they
took place in Calcutta in 1886, which was presided over were annexed by the British is
by Dadabhai Naoroji. In this session, Surendranath (a) 1, 2, 3 (b) 1, 3, 2
Banerjee's National Conference was merged with the (c) 3, 2, 1 (d) 3, 1, 2
Indian National Congress. After the formation of the
Ans. (c) : These princely states were annexed by Lord
Muslim League by Nawab Salimullah in Dhaka in
Dalhousie under Doctrine of Lapse namely Jhansi -
1906, communal harmony began to deteriorate. With
1854, Sambalpur- 1849 and Satara - 1848.
the efforts of Bal Gangadhar Tilak and Mrs. Annie
Besant, in 1916 A.D, a joint session of Congress and 15. The name of the famous person of India who
Muslim League took place in Lucknow. It was chaired returned the Knighthood conferred on him by
by Ambikacharan Majumdar. The resolution passed the British Government as a token of protest
during this session is known as the Lucknow Pact, against the atrocities at Punjab in 1919 was
accepted the demand for communal representation of (a) Tej Bahadur Sapru
the Muslim League. Source - Modern India - NCERT + (b) Ashutosh Mukherjee
BL Grover and Yashpal) (c) Rabindra Nath Tagore
12. Which one of the following statements is (d) Syed Ahmad Khan
correct? Ans. (c) : Rabindra Nath Tagore returned the
(a) The Constituent Assembly of India was Knighthood conferred on him by the British
elected by the Provincial Assemblies in the Government as a token of protest against the Jallianwala
year 1946 Bagh Massacre (April 13, 1919).
(b) Jawaharlal Nehru, M.A. Jinnah and Sardar 16. Which one of the following pairs is not
Vallabhbhai Patel were members of the correctly matched?
Constituent Assembly of India. (a) Pitt's India Act : Warren Hastings
(c) The First Session of the Constituent (b) Doctrine of Lapse : Dalhousie
Assembly of India was held in January, 1947 (c) Vernacular Press Act : Curzon
(d) The Constitution of India was adopted on 26th (d) Ilbert Bill : Ripon
January, 1950 Ans. (c) : Vernacular Press Act was passed by Lord
Ans. (a) : The election was held for the provincial Lytton in 1878 and it was repealed by Ripon in 1882.
assemblies in July 1946 with the aim to form a 17. Consider the following events during India's
Constituent Assembly under the Cabinet Mission freedom struggle:
Scheme. The Muslim League participated in the 1. Chauri-Chaura Outrage
elections but was not involved at the time the
2. Minto-Morley Reforms
Constituent Assembly was held. On the basis of this
election, the first meeting of the Constituent Assembly 3. Dandi March
was held on 9 December 1946 in Delhi under the 4. Montague-Chelmsford Reforms
chairmanship of Sachchidananda Sinha. Later Rajendra Which one of the following is the correct
Prasad was made the permanent chairman and Dr. chronological order of the events above?
Bhimrao Ambedkar as the chairman of the drafting (a) 1-3-2-4 (b) 2-4-1-3
committee. Constitution of India was adopted on 26 (c) 1-4-2-3 (d) 2-3-1-4
January, 1950. (Source - Indian Constitution and Polity Ans. (b) : Minto–Morley Reforms-1909; Montague–
- Youth Competition Times) Chelmsford Reforms–1918; Chauri–Chaura–Feb 1922;
13. During the Indian Freedom Struggle, who Dandi March – 1930
among the following proposed that Swaraj 18. Consider the following statements:
should be defined as complete independence 1. In the Third Battle of Panipat, Ahmed Shah
free from all foreign control? Abdali defeated Ibrahim Lodi.
(a) Mazharul Haque 2. Tipu Sultan was killed in the Third Anglo-
(b) Maulana I-Israt Hasrat Mohani Mysore War.
(c) Hakim Ajmal Khan 3. Mir Jafar entered in a conspiracy with the
(d) Abul Kalam Azad English for the defeat of Nawab Siraj-ud-
Ans. (b) : In 1921, Maulana Hasrat Mohani proposed daulah in the Battle of Plassey.
that Swaraj should be defined as complete Which of the statements given above is/are
independence, free from all foreign control when he correct?
was presiding over the session of All India Muslim (a) 1, 2 and 3 (b) 3 only
League in Lucknow. (c) 2 and 3 (d) None
IAS (Pre) GS 2004 Paper I 334 YCT
Ans. (b) : The Third Battle of Panipat took place on 14 GEOGRAPHY
January 1761 at Panipat, about 60 miles (97 km) north
of Delhi, between a northern expeditionary force of the 21. Which one of the following statements is not
Maratha Empire and invading forces of the King of correct?
Afghanistan, Ahmad Shah Abdali, supported by two (a) The largest Buddhist monastery in India is in
Indian allies—the Rohillas Najib-ud-daulah, Afghans of Assam.
the Doab. (b) The language Konyak is spoken in Nagaland
Tipu Sultan was killed in the Fourth Anglo-Mysore (c) The largest river island in the world is in
Assam.
War.
(d) Sikkim is the least-populated State of the
Mir Jafar served as the commander of Bengali forces
Indian Union.
under Sirajud-Daulah, the Nawab of Bengal, but
Ans. (a) : The largest Buddhist Monastry is in Tawang,
betrayed him during the Battle of Plassey and succeeded
Arunachal Pradesh not in Assam. Rest of the statements
him after the British victory in 1757.
is correct.
19. Which of the following pairs are correctly 22. Which among the following National Highway
matched? routes in the longest?
List I (Period) List II (Wars) (a) Agra-Mumbai
(b) Chennai-Thane
A. AD 1767-69 First Anglo-Maratha War (c) Kolkata-Hajira
(d) Pune-Machilipatnam
B. AD 1790-92 Third Mysore War Ans. (c) : Length of Agra-Mumbai NH – 1161 km,
Length of Chennai-Thane NH – 1235 km, Length of
C. AD 1824-26 First Anglo-Burmese War Kolkata-Hajira NH – 1946 km, Length of Pune-
Machilipatnam NH – 926 km.
D. AD 1845-46 Second Sikh War
23. Which of the following is institutes have been
recognised as the Institutes of National
Select the correct answer using the codes given Importance (by an Act of Parliament)?
below: 1. Dakshina Bharat Hindi Prachar Sabha,
(a) 2 and 4 (b) 3 and 4 Chennai.
(c) 1 and 2 (d) 2 and 3 2. National Institute of Pharmaceutical
Education and Research, Mohali.
Ans. (d) : First Anglo Maratha War (1775–1782 AD)
ended with the Surat treaty, the Marathas were defeated 3. Sree Chitra Tirunal Institute of Medical
Services and Technology, Thiruvananthapuram.
by the British army under the leadership of Colonel
Keating. The Third Anglo-Mysore War (1790–92 AD) 4. Lakshmibai National Institute of Physical
Education, Gwalior.
took place between the British and Tipu Sultan in which
Select the correct answer using the codes given
Tipu Sultan was defeated and the war ended with the
below:
Treaty of Srirangapatam. The First Anglo-Burmese
(a) 1, 2 and 3 (b) 3 and 4
War (1824–1826 AD) was ended by the Yandabo
(c) 1, 2 and 4 (d) 1, 2, 3 and 4
Treaty. It was fought at the time of Governor General
Lord Amhert, who died in 1826 AD. The Second Ans. (a) : Dakshina Bharat Hindi Prachar Sabha
Chennai, National Institute of Pharmaceutical Education
Anglo-Sikh War (1848–1849 AD) led to the complete
and Research, Mohali and Sree Chitra Tirunal Institute
control of Punjab by the Britishers. (Source Modern
for Medical Services and Technology,
India - SK Pandey
Thiruvanantpuram are institutes of national importance.
20. Consider the following Viceroys of India 24. Consider the following crops:
during the British rule 1. Cotton 2. Groundnut
1. Lord Curzon 2. Lord Chelmsford 3. Maize 4. Mustard
3. Lord Hardinge 4. Lord Irwin Which of the above are Kharif crops?
Which one of the following is the correct (a) 1 and 2 (b) 1, 2 and 3
chronological order of their tenure? (c) 3 and 4 (d) 1, 2, 3 and 4
(a) 1-3-2-4 (b) 2-4-1-3 Ans. (b) : Cotton, Groundnut and Maize are kharif crop
(c) 1-4-2-3 (d) 2-3-1-4 whereas Mustard is a rabi crop.
Ans. (a) : Lord Curzon (1899-1905); Lord Chelmsford 25. In which one of the following Union
(1916- 1921); Lord Hardinge (1910-1916); Lord Irwin Territories, do the people of the Onge tribe
(1926- 1931). live?
IAS (Pre) GS 2004 Paper I 335 YCT
(a) Andaman and Nicobar Island Ans. (c) : Minerals Location
(b) Dadra and Nagar Haveli A. Coal 1. Giridih
(c) Daman and Diu B. Copper 3. Alwar
(d) Lakshadweep C. Manganese 4. Dharwar
Ans. (a) : In Union Territory of Andaman and Nicobar D. Lignite 2. Jayankondam
Islands people of the Onge tribe live. The Onge are one 29. Consider the following
of the Andamanese indigenous peoples of the Andaman 1. Mahadeo Hills 2. Sahyadri Parvat
Islands. Traditionally hunter-gatherers, they are a 3. Satpura Range
designated as a Scheduled Tribe of India. They belong What is the correct sequence of the above from
to the Negrito race. the north to the south?
26. Consider the following statements: (a) 1, 2, 3 (b) 2, 1, 3
1. Damodar Valley Corporation is the first (c) 1, 3, 2 (d) 2, 3, 1
multipurpose river valley project of
Ans. (c) : The correct sequence of the above from the
independent India.
north to the south is Mahadeo Hills, Satpura Range and
2. Damodar Valley Corporation includes Sahyadri Parvat.
thermal and gas power stations.
30. Lake Sambhar is nearest to which one of the
Which of the statements given above is/are
following cities of Rajasthan?
correct?
(a) Bharatpur (b) Jaipur
(a) 1 only (b) 2 only
(c) Jodhpur (d) Udaipur
(c) Both 1 and 2 (d) Neither 1 nor 2
Ans. (b) : The largest salt lake of India, the Sambhar
Ans. (a) : Damodar Valley Corporation's is first lake, is about 65 km west of Jaipur; which is the nearest
multipurpose river valley project of independent India.
of all the cities mentioned above.
It was started in the year 1948. The joint venture
projects are Maithon Power Limited, Panchat and Directions: The following 8 (eight) items consist of two
Tilayiya which are hydroelectric power station. It does statements, one labelled as the 'Assertion (A)' and
not include a gas power station but only thermal and the other as 'Reason (R). You are to examine these
hydel power stations. two statements carefully and select the answers to
27. Consider the following statements: these items using the code given below:
Among the Indian States (a) Both A and R are individually true and R is
1. Andhra Pradesh has the longest coastline the correct explanation of A
2. Gujarat has the highest number of airports (b) Both A and R are individually true but R is
Which of the statements given above is/are
NOT the correct explanation of A
correct?
(a) 1 only (b) 2 only (c) A is true but R is false
(c) Both 1 and 2 (d) Neither 1 or 2 (d) A is false but R is true
Ans. (b) : The length of coastline in India is 7516 km. 31. Assertion (A) :Bangalore receives much higher
The state of Gujarat has the longest length of coastline. average annual rainfall than that of Mangalore.
The highest airport is in Gujarat. Reason (R) : Banglore has the benefit of
28. Match List I (Minerals) with List II (Location) receiving rainfall both from south-west and north-
and select the correct answer using the codes east monsoons.
given below the lists: (a) Both A and R are individually true and R is
the correct explanation of A
List I List II
(b) Both A and R are individually true but R is
A. Coal 1. Giridih NOT the correct explanation of A
(c) A is true but R is false
B. Copper 2. Jayamkondam (d) A is false but R is true
Ans. (d) : Bangalore receives lesser rainfall than
C. Manganese 3. Alwar
Mangalore. Mangalore is located in windward slope and
D. Lignite 4. Dharwar receives 2000 mm of rainfall annually whereas
Bangalore is located in rain shadow area therefore
A B C D receives less than 500 mm of water annually.
(a) 1 4 3 2 32. Assertion (A) :West-flowing rivers of Peninsular
(b) 2 3 4 1 India have no deltas.
(c) 1 3 4 2 Reason (R) : These rivers do not carry any
(d) 2 4 3 1 alluvial sediments.
IAS (Pre) GS 2004 Paper I 336 YCT
(a) Both A and R are individually true and R is WORLD GEOGRAPHY
the correct explanation of A
(b) Both A and R are individually true but R is 35. Latvia does not share its borders with which
NOT the correct explanation of A one of the following countries?
(c) A is true but R is false (a) Russia (b) Estonia
(d) A is false but R is true (c) Lithuania (d) Poland
Ans. (a) : West flowing rivers Narmada and Tapti do Ans. (d) : Latvia does not share its borders with Poland.
not form delta, because topography of western Latvia is bordered by the 4 neighboring nations of
peninsular India is rocky. Due to this, these rivers do Russia (eastwards), Estonia (northwards), Belarus
not carry alluvial sediments. (southeastwards), and Lithuania (southwards).
33. Assertion (A) : The thickness of the atmosphere 36. Which one of the following statements is
correct?
is maximum over the Equator.
(a) Cirrus clouds are composed of ice crystals
Reason (R) : High insolation and strong
(b) Cirrus clouds exhibit a flat base and have the
convection currents occur over the Equator. appearance of rising domes
(a) Both A and R are individually true and R is (c) Cumulus clouds are white and thin and form
the correct explanation of A delicate patches and give a fibrous and
(b) Both A and R are individually true but R is feathery appearance
NOT the correct explanation of A (d) Cumulus clouds are classified as high clouds
(c) A is true but R is false Ans. (a) : The most common form of high-level clouds
(d) A is false but R is true are thin and often wispy cirrus clouds. Typically found
Ans. (b) : The circulation of atmosphere and oceans is at heights greater than 20,000 feet (6,000 meters), cirrus
fundamentally caused by the fact that the amount of clouds are composed of ice crystals that originate from
incoming solar radiation varies from a maximum at the the freezing of super cooled water droplets. Cirrus
equator to a minimum at the poles. More reflection and generally occurs in fair weather and points towards the
absorption of incoming radiation takes place in high direction of air movement at their elevation.
latitude. 37. Which one of the following statements is not
The thickness of the atmosphere is maximum over the correct?
equator and high insolation and strong convection (a) Gulfs with narrow fronts and wider rears
experience high tides.
currents occur over the Equator.
(b) Tidal currents take place when a gulf is
34. Match List I (Beaches in India) with List II connected with the open sea by a narrow
(States) and select the correct answer using the channel.
codes given below: (c) Tidal bore occurs when a tide enters the
List I List II narrow and shallow estuary of a river.
(d) The tidal nature of the mouth of the river
A. Gopnath Beach 1. Andhra Pradesh Hooghly is of crucial importance to Kolkata
as port.
B. Lawsons Bay 2. Kerala Ans. (a) : Gulfs with narrow fronts and wider rears
Beach experience high tides is incorrect. The gulfs with wide
fronts and narrow rears experience higher tides. The in
C. Devbagh Beach 3. Gujarat and out movement of water into a gulf through channels
is called tidal current.
D. Sinquerim Beach 4. Goa 38. Consider the following geological phenomena:
1. Development of a fault
5. Karnataka 2. Movement along a fault
3. Impact produced by a volcanic eruption
A B C D 4. Folding of rocks
(a) 5 4 2 1 Which of the above cause earthquakes?
(b) 3 1 5 4 (a) 1, 2 and 3 (b) 2 and 4
(c) 5 1 2 4 (c) 1, 3 and 4 (d) 1, 2, 3 and 4
(d) 3 4 5 1 Ans. (d) :An earthquake is caused by tectonic plates
Ans. (b) : Beach in India States getting stuck and putting a strain on the ground. The
strain becomes so great that rocks give way by breaking
A. Gopinath Beach : Gujarat
and sliding along fault planes. Plate boundaries lock as
B. Lawsons Bay Beach : Andhra Pradesh the plates move past each other, creating frictional
C. Devbagh Beach : Karnataka stress.Volcanic activity and volcanic eruption is usually
D. Sinquerim Beach : Goa triggered by alterations of tectonic plates, resulting in
IAS (Pre) GS 2004 Paper I 337 YCT
landslides or earthquakes. Volcanic eruptions can be
C. Gibraltar 3. Canada
extremely damaging to the environment, particularly
because of number of toxic gases possibly present in D. Rhode Island 4. Germany
pyroclastic material. Early in the folding, earthquakes
occur on the slipping fault. But as time goes on, the 5. Denmark
quakes are increasingly caused by the slipping rock
layers above and around the fault as the folds buckle A B C D
under pressure. (a) 1 2 5 3
39. The great Asian river Mekong does not run (b) 3 4 2 1
through (c) 1 4 2 3
(a) China (b) Malaysia (d) 3 2 5 1
(c) Cambodia (d) Laos Ans. (b) : British Colombia is western most provinces
Ans. (b) : Mekong river does not flow through of Canada, Bavaria is a state of Germany. Gibraltar is
Malaysia but runs through China, Laos, Cambodia and the overseas territory of UK and Rhoda Island is the
Vietnam. smallest state in USA.
40. Match List I with List II and select the correct
answer using the codes given below the lists: INDIAN CONSTITUTION & POLITY
List I (Sea) List II (Country) 43. Which one of the following statements is not
correct?
A. Black Sea 1. Bulgaria (a) In Lok Sabha, a no-confidence motion has to
set out the grounds on which it is based
B. Red Sea 2. China
(b) In the case of a no-confidence motion in Lok
C. Yellow Sea 3. Eritrea Sabha, no conditions of admissibility have
been laid down in the Rules
D. Caspian Sea 4. Kazakhstan (c) A motion of no-confidence once admitted,
has to be taken up within ten days of the leave
being granted
A B C D
(a) 1 4 2 3 (d) Rajya Sabha is not empowered to entertain a
motion of no-confidence
(b) 2 3 1 4
(c) 1 3 2 4 Ans. (a) : If any member of the House feels that the
(d) 2 4 1 3 government does not have majority then he/she can
move a no confidence motion. The member need not
Ans. (c) : Black Sea coastal countries - Ukraine, Russia,
Georgia, Turkey, Bulgaria. Coastal countries of the give a reason for moving no confidence motion. As per
Red Sea - Egypt, Sudan, Eritrea, Djibouti, Somalia, rule 198 of the Rules of Procedure and Conduct of Lok
Yemen, Saudi Arabia. Yellow Sea - China. Caspian Sabha a minimum of 50 members have to accept the
Sea - Russia, Kazakhstan, Turkmenistan, Iran, motion and accordingly the speaker will announce the
Azerbaijan. (Source - New School Atlas) date for discussion for the motion. The allotted date has
41. Which one of the following does not border to be within 10 days from the day motion is accepted.
Panama? 44. With reference to Indian Public Finance,
(a) Costa Rica (b) Pacific Ocean consider the following statements:
(c) Colombia (d) Venezuela 1. Disbursements from Public Accounts of
Ans. (d) : Venezuela does not border Panama. Panama India are subject to the Vote of Parliament.
is a country located in Central America, bordering both 2. The Indian Constitution provides for the
the Caribbean Sea and the Pacific Ocean, between establishment of a Consolidated Fund, a
Colombia and Costa Rica. Public Account and a Contingency Fund
42. Match List I (State/Province/Overseas for each State.
Territory) with List II (Country) and select the 3. Appropriations and disbursements under
correct answer using the codes given below the the Railway Budget are subject to the same
lists: form of parliamentary control as other
List I List II appropriations and disbursements.
Which of the statements given above are
A. British Colombia 1. USA correct?
(a) 1 and 2 (b) 2 and 3
B. Bavaria 2. UK (c) 1 and 3 (d) 1, 2 and 3
IAS (Pre) GS 2004 Paper I 338 YCT
Ans. (d) : All the three statements are correct. Any sort 2. Unless sooner dissolved or there is an
of disbursement from or appropriation of Indian Public extension of the term, there is an automatic
Finance is subject to the approval and regulation of the dissolution of the Lok Sabha by efflux of
Parliament of India. Under Article 266 (1) of the time, at the end of the period of five years,
Constitution of India, a Consolidated Fund of State (a even if no formal order of dissolution is
separate fund for each state) has been established. The issued by the President.
Consolidated Fund receives all state government 3. The Speaker of Lok Sabha continues in
revenues as well as all loans raised by issue of treasury office even after the dissolution of the
bills. The Public Account was constituted by Article House and until 'immediately before the
266(2)of the Indian Constitution. It deals with the first meeting of the House.'
transactions that are related to debt and not covered Which of the statements given above are
under the Consolidated Fund of India. The Contingency correct?
Fund of India established under Article 267 (1) of the (a) 1 and 2 (b) 2 and 3
Constitution is in the nature of an imprest (money (c) 1 and 3 (d) 1, 2 and 3
maintained for a specific purpose) which is placed at the Ans. (d) : Article 85— (1) The President shall, from
disposal of the President to enable him/her to make time to time, session every House of Parliament at such
advances to meet urgent unforeseen expenditure, time and place as he thinks fit.
pending authorisation by the Parliament. Appropriations (2) The President may from time to time
and disbursements under the Railway Budget need to be (a) Prorogue the Houses or any House
approved by the Parliament like other appropriations (b) Dissolve the Lok Sabha.
and disbursements. Article 94 - The Speaker shall not vacate his post
45. Consider the following statements: immediately before the first session of the Lok Sabha to
1. The highest criminal court of the district is be held after the dissolution of the Lok Sabha.
the Court of District and Session Judge. The Speaker of the Lok Sabha has the power to adjourn
2. The District Judges are appointed by the the House indefinitely. (Source - Constitution and
Governor in consultation with the High Polity of India - Youth Competition Times)
Courts. 47. Which one of the following statements correctly
3. A person to be eligible for appointment as describes the Fourth Schedule of the
a District Judge should be an advocate or a Constitution of India?
pleader of seven year's standing or more, (a) It contains the scheme of the distribution of
or an officer in judicial service of the powers between the Union and the States
Union of the State. (b) It contains the languages listed in the
4. When the Sessions Judge awards death Constitution
sentence, it must be confirmed by the High (c) It contains the provisions regarding the
Court before it is carried out. administration of tribal areas
Which of the statements given above are correct? (d) It allocates seats in the Council of States
(a) 1 and 2 (b) 2, 3 and 4 Ans. (d) : The Fourth Schedule of the Constitution
deals with the allocation of seats in the Rajya Sabha to
(c) 3 and 4 (d) 1, 2, 3 and 4
the States and Union Territories. The allocation is based
Ans. (d) : The District and Sessions Judge's Court is the upon the population of states and UTs. So far, all states
highest judicial court in the district. Article 233 (1) — and only Union Territory of Delhi and Puducherry have
The appointment of a District Judge in a State is made representation in Rajya Sabha.
and promoted by the Governor of that State in 48. The resolution for removing the Vice-President
consultation with the High Court having the jurisdiction of India can be moved in the
over the State concerned. Article 233 (2) - A person (a) Lok Sabha alone
who is not already in the service of the Union or a State, (b) either House of Parliament
shall be eligible to be appointed as a District Judge only (c) Joint Sitting of Parliament
if he has been an advocate for at least 7 years. The
(d) Rajya Sabha alone
decision of the death sentence of the session judge will
Ans. (d) : According to Article 67 of the Constitution of
be valid only after the confirmation of the High Court.
India, the Vice-President can be removed by a
(Source - Constitution and Polity of India - Youth
resolution of the Rajya Sabha passed by an effective
Competition Times) majority and agreed to by a simple majority of the Lok
46. Consider the following statements: Sabha. But no such resolution may be moved unless at
1. The Speaker of Lok Sabha has the power least 14 days advance notice has been given. Notably,
to adjourn the House sine die but, on the Constitution does not list grounds for removal. No
prorogation, it is only the President who Vice-President or ex officio Vice-President has ever
can summon the House. faced removal proceedings.
IAS (Pre) GS 2004 Paper I 339 YCT
49. With reference to the Constitution of India, (a) 1 and 2 (b) 2 and 3
which one of the following pairs is not correctly (c) 2 only (d) 3 only
matched? Ans. (b) : The highest decision making body for
(a) Forests : Concurrent List planning in India is the Parliament. The Secretary of the
(b) Stock Exchanges : Concurrent List Planning Commission of India is also the Secretary of
(c) Post Office Savings : Union List National Development Council. Economic and social
Bank planning is placed under Entry 20 in the Concurrent list.
(d) Public Health : State List
52. With reference to Indian Parliament, which or
Ans. (b) : Stock Exchanges are listed in the Seventh
one of the following is not correct?
Schedule (Article 246) List I-Union List, item no. 90
that reads, taxes other than stamp duties on transactions (a) The Appropriation Bill must be passed by
in stock exchanges and futures markets.Forests- both the Houses of Parliament before it can
Concurrent List, 17-A, Post Office Savings Bank - be enacted into law.
Union List-3, Public health and sanitation; hospitals and (b) No money shall be withdrawn from the
dispensaries - State List -6. Consolidated Fund of India except under the
50. Consider the following tasks: appropriation made by the Appropriation Act.
1. Superintendence, direction and conduct of (c) Finance Bill is required for proposing new
free and fair elections. taxes but no another Bill/Act is required for
2. Preparation of electoral rolls for all making changes in the rates of taxes which
elections to the Parliament, State are already under operation.
Legislature and the Office of the President (d) No Money Bill can be introduced except on
and the Vice-President. the recommendation of the President.
3. Giving recognition to political parties and
allotting election symbols to political Ans. (c) : Appropriation Bill gives power to the
parties and individuals contesting the government to withdraw funds from the Consolidated
election. Fund of India for meeting the expenditure during the
4. Proclamation of final verdict in the case of financial year. Under Article 114(3) of the Constitution,
election disputes. no amount can be withdrawn from the Consolidated
Which of the above are the functions of the Fund without the enactment of such a law by
Election Commission of India? Parliament. According to Article 265 of the
(a) 1, 2 and 3 (b) 2, 3 and 4 Constitution, no tax shall be levied or collected except
(c) 1 and 3 (d) 1, 2 and 4 by authority of law. A Finance Bill is a Money Bill as
Ans. (a) : The functions of the Election Commission of defined in Article 110 of the Constitution. Finance Bill
India are superintendence, direction and control of the is part of Union Budget, stipulating all legal
preparation of the electoral rolls for the conduct of all amendments required for changes in taxation. Hence
elections to the Parliament and to the Legislature of statement 3 is not correct. Money Bill could be
every State and of elections to the offices of President introduced on the recommendation of the President. If a
and Vice President. Election Commission of India question arises Whether a Bill is Money Bill or not, the
prepares electoral rolls for all above said elections. decision of the Speaker of the Lok Sabha is final.
Election Commission recognizes political parties and
3. Which one of the following Articles of the
provides them election symbols. Proclamation of final
verdict in case of election dispute lies with Supreme Constitution of India says that the executive
Court of India which in turn shall obtain the opinion of power of every State shall be so exercised as
the Election Commission and shall act according to not to impede or prejudice the exercise of the
such opinion. executive power of the Union?
51. Consider the following statements: (a) Article 257 (b) Article 258
1. The highest deciding body for planning in (c) Article 355 (d) Article 356
India is the Planning Commission of India. Ans. (a) : Art.257 (1) : The executive power of every
2. The Secretary of the Planning Commission State shall be so exercised as not to impede or prejudice
of India is also the Secretary of National the exercise of the executive power of the Union, and
Development Council. the executive power of the Union shall extend to the
3. The Constitution includes economic and giving of such directions to a State as may appear to the
social planning in the Concurrent List in Government of India to be necessary for that purpose.
the Seventh Schedule of the Constitution of 54. Match List I (Articles of the Constitution of
India. India) with List II (Provision) and select the
Which of the statements given above is/are correct answer using the codes given below the
correct? lists:
IAS (Pre) GS 2004 Paper I 340 YCT
List I List II Ans. (c) : Central Rural Sanitation Programme (CRSP)
was launched in 1986 primarily with the objective of
A. Article 14 1. The State shall not improving the quality of life of the rural people and also
discriminate against any to provide privacy and dignity to women. Rural
citizen on ground only of sanitation is not a subject in the Concurrent List. Public
religion, race, caste, sex, Health and Sanitation comes under the State List.
place of birth or any of 56. Which Article of the Constitution of India says,
them. 'No child below the age of fourteen years shall
be employed to work in any factory or mine or
B. Article 15 2. The State shall not deny to engaged in any other hazardous employment?
any person equality before (a) Article 24 (b) Article 45
the law or the equal (c) Article 330 (d) Article 368
protection of laws within Ans. (a) : Article 24 of the Constitution of India says,
the territory of India. 'No child below the age of fourteen years shall be
employed to work in any factory or mine or engaged in
C. Article 16 3. 'Untouchability' is abolished any other hazardous employment.’ It concerns
and its practice in any form prohibition of employment of children in factories,
is forbidden. etc. It comes under Fundamental Right.
57. According to the National Human Rights
D. Article 17 4. There shall be equality of Commission Act, 1993, who amongst the
opportunity for all citizens following can be its Chairman?
in matters relating to
(a) Any serving Judge of the Supreme Court.
employment or appointment
(b) Any serving Judge of the High Court.
to any office under the
State. (c) Only a retired Chief Justice of India.
(d) Only a retired Chief Justice of a High Court.
Ans. (c) : According to the National Human Rights
A B C D Commission Act, 1993, only a retired Chief Justice of
(a) 2 4 1 3 India can be chairman of the National Human Rights
(b) 3 1 4 2 Commission (NHRC). However by 2019 amendment
(c) 2 1 4 3 now any retired judge of Supreme Court can be its
(d) 3 4 1 2 chairman.
Ans. (c) : Fundamental Rights have been mentioned in 58. Who among the following was never the Lok
Sabha Speaker?
Part III of the Constitution. Article 14 to 18 of the
(a) K.V.K. Sundaram (b) G.S. Dhillon
Constitution provides ‘Right of Equality’ to every
citizen. Article 14 embodies equality expressed in the (c) Baliram Bhagat (d) Hukam Singh
Preamble of the Constitution. The succeeding Articles Ans. (a) : K.V.K. Sundaram was never Speaker of the
15, 16, 17 and 18 lays down specific applications of the Lok Sabha. Hukum Singh (April 1962-March 1967),
general rules laid down in Article 14. Article 15 relates G.S. Dhillon (April 1969-Dec 1975), Baliram Bhagat
(Jan 1976-March 1977) were Lok Sabha Speaker.
to prohibition of discrimination on grounds of religion,
Kalyan Vaidya Nathan Kuttur Sundaram, also referred
race, caste, sex or place of birth. Article 16 guarantees
as K. V. K. Sundaram, was an Indian civil servant, who
equality of opportunity in matters of public holds the record as the first Law Secretary of
employment. Article 17 abolishes untouchability. independent India and second Chief Election
55. Assertion (A) : The Central Rural Sanitation Commissioner of India. He also chaired the Fifth Law
Programme was launched in 1986 to improve the Commission of India for the period 1968–71.
quality of life of rural people in India. 59. Consider the following events:
Reason (R) : Rural sanitation is a subject in the 1. Fourth general elections in India
Concurrent List in the Constitution of India. 2. Formation of Haryana State
(a) Both A and R are individually true and R is 3. Mysore named as Karnataka State
the correct explanation of A 4. Meghalaya and Tripura become full State
(b) Both A and R are individually true but R is Which one of the following is the correct
NOT the correct explanation of A chronological order of the above?
(c) A is true but R is false (a) 2-1-4-3 (b) 4-3-2-1
(d) A is false but R is true (c) 2-3-4-1 (d) 4-1-2-3

IAS (Pre) GS 2004 Paper I 341 YCT


Ans. (a) : Haryana was founded in 1966 when the Ans. (c) : Tata Consultancy Services (TCS) was the
former state of Punjab was divided into Haryana and the largest IT software and services exporter in India during
modern Punjab. The 1967 general elections or the 4th the year 2002-03.
Lok Sabha was held in February, 1967. Tripura,
Manipur and Meghalaya became full-fledged states on 63. Consider the following statements:
Jan 21, 1972. Mysore state was renamed Karnataka in 1. The Oil Pool Account of Government of
1973. India was dismantled with effect from 1-4-
60. Which one of the following is the correct 2002.
sequence in the descending order of precedence 2. Subsidies on PDS kerosene and domestic
in the warrant of precedence?
(a) Attorney General of India-Judge of the LPG are borne by Consolidated Fun of
Supreme Court-Members of Parliament- India.
Deputy Chairman of Rajya Sabha 3. An expert committee headed by Dr R.A.
(b) Judges of the Supreme Court-Deputy Mashelkar to formulate a national auto
Chairman of Rajya Sabha-Attorney General fuel policy recommended that Bharat
of India-Member of Parliament
Stage-II Emission Norms should be applied
(c) Attorney General of India Deputy Chairman
of Rajya Sabha-Judge of the Supreme Court- throughout the country by 1. April, 2004.
Members of Parliament Which of the statements given above are
(d) Judges of the Supreme Court-Attorney correct?
General of India-Deputy Chairman of Rajya (a) 1 and 2 (b) 2 and 3
Sabha-Members of Parliament
(c) 1 and 3 (d) 1, 2 and 3
Ans. (b) : Judges of the Supreme Court , Deputy
Chairman of Rajya Sabha, Attorney General of India, Ans. (a) : An expert committee headed by Dr. R. A.
Members of Parliament Mashelkar to formulate a national auto fuel policy
recommended that Bharat Stage - II Emission Norms
ECONOMY should be applied throughout the country by 1 April,
2005. Hence statement 3 is not correct. Rest of the two
61. Which one of the following statements is not
correct? statements is correct.
(a) Under the Targeted Public Distribution 64. In the last one decade, which one among the
System, the families Below Poverty Line are following sectors has attracted the highest
Provided 50 kg of food grains per month per Foreign Direct Investment inflows into India?
family at subsidised price.
(b) Under Annapurna Scheme, indigent senior (a) Chemical other than fertilizers
citizens of 65 years of age or above eligible (b) Services sector
for National Old Age Pension but not getting (c) Food processing
pension can get 10 kg of food grains per (d) Telecommunication
person per month free of cost.
(c) Ministry of Social Justice and Empowerment Ans. (b) : Service sector received 21% of total FDI
has scheme in which indigent people living in flow between 2006 to 2010.
welfare institutions like orphanages are given 65. Consider the following statements:
15 kg of foodgrains per person per month at 1. The National Housing Bank the apex
BPL rates. institution of housing finance in India, was set
(d) Ministry of Human Resource Development up as a wholly-owned subsidiary of the
gives financial support to Mid-day Meal Reserve Bank of India.
Scheme for the benefit of class I to V students
2. The small Industries Development Bank of
in Government or Government aided Schools.
Ans. (a) : Under the Public Distribution System (PDS), India was established as a wholly owned
Government of India increased the allocation to BPL subsidiary of the Industrial Development
families from 10 kg to 20 kg of food grains per family Bank of India.
per month at 50% of the economic cost and allocation to Which of the statements given above is/are
APL families at economic cost w.e.f. 1.4.2000. correct?
62. Which one of the following was the largest IT (a) 1 only (b) 2 only
software and services exporter in India during (c) Both 1 and 2 (d) Neither 1 nor 2
the year 2002-03?
(a) Birlasoft Ans. (c) : National Housing Bank was set up as a
(b) Infosys Technologies wholly owned subsidiary of RBI in 1988 under the
(c) Tata Consultancy Services National Housing Bank Act 1987. It is the apex
(d) Wipro Technologies institution in India for housing. The Small Industries

IAS (Pre) GS 2004 Paper I 342 YCT


Development Bank of India was established in 1990 as Ans. (c) : The Government’s price policy for
a wholly owned subsidiary of the Industrial agricultural commodities seeks to ensure remunerative
Development Bank of India with the aim to aid in the prices to the growers for their produce with a view to
growth and development of micro, small and medium- encourage higher investment and production and to
scale enterprises (MSME). Currently, the ownership is safeguard the interest of consumers by making available
held by 34 Government of India owned/controlled supplies at reasonable prices with low cost of
institutions. intermediation. Government announces, Minimum
66. Assertion (A) : India does not export natural Support Prices (MSP) for 25 major agricultural
rubber.
commodities each year in both the Crop seasons after
Reason (R) : About 97% of India's demand for taking into account the recommendations of the
natural rubber is met from domestic production.
Commission for Agricultural Costs and Prices (CACP).
(a) Both A and R are individually true and R is
Hence statement 2 is not correct. Rest of the statements
the correct explanation of A
is correct.
(b) Both A and R are individually true but R is
NOT the correct explanation of A 69. Consider the following statements:
(c) A is true but R is false India continues to be dependent on imports to
(d) A is false but R is true meet the requirement of oilseeds in the country
Ans. (a): India does not export natural rubber as most because
of the natural rubber produced in India is been 1. Farmers prefer to grow food grains with
consumed within the country. Therefore most of the highly remunerative support prices
demand of natural rubber within the country is fulfilled 2. Most of the cultivation of oilseed crops
by domestic production. continues to be dependent on rainfall
67. Assertion (A) : For the first time, India had no 3. Oils from the seeds of tree origin and rice
trade deficit in the year 2002-03 bran have remained unexploited
Reason (R) : For the first time, India's exports 4. it is far cheaper to import oil seeds than to
crossed worth $ 50 billion in the year 2002-03. cultivate the oilseed crops
(a) Both A and R are individually true and R is Which of the statements given above are
the correct explanation of A correct?
(b) Both A and R are individually true but R is (a) 1 and 2 (b) 1, 2 and 3
NOT the correct explanation of A
(c) 3 and 4 (d) 1, 2, 3 and 4
(c) A is true but R is false
Ans. (b) : India continues to be dependent on imports
(d) A is false but R is true
to meet the requirement of oilseeds in the country due
Ans. (d) : Assertion is not correct as the trade deficit
to the fact that agriculture in India is highly dependent
fell from 2000-01 to 2002-03 but there was trade deficit
on rain. Cost of production of oil seeds is more
in 2002-2003.
compared to price offered to them that is why farmers
68. Consider the following statements:
prefer to grow food grains with highly remunerative
1. Regarding the procurement of food grains,
support prices. India still needs to work upon to
Government of India follows a
extract oil from seeds of tree and rice bran. Every year
procurement target rather than an open-
ended procurement policy. India pays high amount of foreign exchange to import
2. Government of India announces minimum oil seeds.
support prices only for cereals. 70. Consider the following statements:
3. For distribution under Targeted Public 1. Reserve Bank of India was nationalised on
Distribution System (TPDS), wheat and 26 January, 1950.
rice are issued by the Government of India 2. The borrowing programme of the
Government of India is handled by the
at uniform central issue prices to the
Department of Expenditure, Ministry of
States/Union Territories.
Finance.
Which of the statements given above is/are Which of the statements given above is/are
correct? correct?
(a) 1 and 2 (b) 2 only (a) 1 only (b) 2 only
(c) 1 and 3 (d) 3 only (c) Both 1 and 2 (d) Neither 1 nor 2
IAS (Pre) GS 2004 Paper I 343 YCT
Ans. (d) : The Reserve Bank of India was nationalised Ans. (b) : A body moving with a uniform angular
with effect from 1st January, 1949 on the basis of the velocity ω on a circular path of radius r has radial
Reserve Bank of India (Transfer to Public Ownership) acceleration equal to ω2r directed towards the centre of
Act, 1948. All shares in the capital of the Bank were the path.
deemed transferred to the Central Government on 74. A weightless rubber balloon is filled with 200cc
payment of a suitable compensation. The finance
of water. Its weight in water is equal to
ministry is likely to set up a full-fledged independent
public debt management agency by the end of next year
to manage government borrowing programme that runs (a) (b)
into lakhs of crore. At present, the government debt,
including market borrowings, is managed by the
Reserve Bank of India. (c) (d) Zero
Ans. (d) : In this case, balloon is filled up with water
PHYSICS and then put into water. Initially, after some
oscillations, it becomes stable because the downward
71. Assertion (A) : In our houses, the Current in directing force exerted by weight of water in balloon is
A.C. electricity line changes direction 60 times balanced by the upward directing buoyancy force of
per second. water. Hence, the resultant weight in water is zero as
Reason (R) : The frequency of alternating there is no resultant force acting upon it.
voltage supplied is 50 Hz. 75. A person stands at the middle point of a
(a) Both A and R are individually true and R is wooden ladder which starts slipping between a
the correct explanation of A. vertical wall and the floor of a room, while
(b) Both A and R are individually true but R is plane. The path traced by a person standing at
NOT the correct explanation of A. the middle point of the slipping ladder is
(c) A is true but R is false. (a) a straight line (b) an elliptical path
(d) A is false but R is true. (c) a circular path (d) a parabolic path
Ans. (d) : An Alternating Current (A.C.) is one which is Ans. (c) : The path traced by a person standing at the
constantly changing direction. Alternating current have middle point of the slipping ladder is a circular path.
two cycles namely positive half cycle and negative half
cycle. Mains electricity is an A.C. supply which is CHEMISTRY
supplied to homes in India at 220 V at 50 Hz. Thus, in
our houses the current in A.C. electricity line changes 76. Consider the following statements:
its direction 50 times. 1. Baking soda is used in fire extinguishers.
72. A car is running on a road at a uniform speed 2. Quicklime is used in the manufacture of
glass.
of 60 km/hr, the net resultant force on the car
is 3. Gypsum is used in the manufacture of
Plaster of Paris.
(a) driving force in the direction of car's motion
Which of these statements given above is/are
(b) resistance force opposite to the direction of correct?
car's motion (a) 1 and 2 (b) 2 and 3
(c) an inclined force (c) 1 only (d) 1, 2 and 3
(d) equal to zero Ans. (d) : Baking soda is used as a fire extinguisher for
Ans. (d) : By Newton's second law F = ma. If a = 0, electrical fires and grease fires. When baking soda is
Fnet = 0 Thus, the resultant force is zero. heated, it releases carbon dioxide and produces water
73. A spherical body moves with a uniform 2NaHCO3 → H2O + CO2 + Na2CO3. Since carbon
angular velocity 'w' around a circular path of dioxide is heavier than air and does not support
radius 'r'. Which one of the following combustion like oxygen does, it smothers the fire while
statements is correct? water that is formed cools the fire to below ignition
(a) The body has no acceleration. temperature. Calcium oxide, CaO is also called lime,
(b) The body has a radial acceleration w2 r quicklime, or caustic lime. Calcium oxide is widely
directed towards the centre of the path. used in making porcelain and glass. Plaster of Paris is a
(c) The body has a radial acceleration 2/5 w2 r type of building material based on calcium sulphate
directed away from the centre of the path. hemihydrate nominally CaSO4.1/ 2H2O. It is created by
(d) The body has an acceleration w2 r tangential heating gypsum to about 300oF (150ºC).
to its path. 2CaSO4.2H2O→ 2CaSO4.1/ 2H2O + 3H2O

IAS (Pre) GS 2004 Paper I 344 YCT


77. Match List I (Fuel Gases) with List II (Major BIOLOGY
Constituents) and select the correct answer
using the codes given below the lists: 80. Consider the following statements:
List I List II 1. Non-function of Lacrimal gland is an
important symptom deficiency of Vitamin
A. CNG 1. Carbon Monoxide, A.
Hydrogen 2. Deficiency of Vitamin B1 can lead to
indigestion and heart enlargement.
B. Coal gas 2. Butane, Propane
3. Vitamin C deficiency can lead to pain in
C. LPG 3. Methane, Ethane the muscles.
4. Deficiency of Vitamin D causes increased
D. Water gas 4. Hydrogen, Methane, loss of Ca++ in urine.
Carbon Monoxide
Which of the statements given above are
A B C D correct?
(a) 2 1 3 4 (a) 1 and 2 (b) 2, 3 and 4
(b) 3 4 2 1 (c) 1, 3 and 4 (d) 1, 2, 3 and 4
(c) 2 4 3 1 Ans. (d) : Lacrimal is a tear producing gland which
(d) 3 1 2 4 does not function due to deficiency of vitamin -A.
Ans. (b) : Compressed Natural Gas (CNG) is made by Deficiency of vitamin - B1 causes loss of appetite, heart
compressing natural gas which is composed of methane disorders, muscular weakness etc.
(CH4), it also contain small amount of ethane. Coal gas 81. The hormone insulin is a
typically contains hydrogen, methane and carbon (a) Glycolipid (b) Fatty acid
monoxide. LPG is the abbreviation of Liquefied (c) Peptide (d) Sterol
Petroleum Gas. The major constituents of LPG are Ans. (c) : Insulin is a peptide hormone composed of 51
propane and butane. Water gas is a synthesis gas, amino acids. Insulin is secreted from pancreas (Islets of
containing carbon monoxide and hydrogen. Langerhans).
78. Standard 18-carat gold sold in the market 82. Consider the following statements:
contains
1. Toothless mammals such as pangolins are
(a) 82 parts gold and 18 parts other metals.
not found in India.
(b) 18 parts gold and 82 parts other metals.
2. Gibbon is the only ape found in India.
(c) 18 parts gold and 6 parts other metals.
Which of the statements given above is/are
(d) 9 parts gold and 15 parts other metals.
correct?
Ans. (c) : 24 carat is pure gold with no other metals.
(a) 1 only (b) 2 only
Lower cartages contain less gold; 18 carat gold contains
(c) Both 1 and 2 (d) Neither 1 nor 2
75 per cent gold and 25 per cent other metals, often
copper or silver. Ans. (b) : Pangolins (Manis crassicaudata) are found in
India, Sri Lanka, Nepal, Africa and some parts of
79. Salts of which of the following elements provide
colours to fireworks? Pakistan. Gibbons are the only apes found in India. The
(a) Zinc and sulphur Hoollongapar Gibbon Sanctuary is in Assam which is a
(b) Potassium and mercury safe recluse for gibbons.
(c) Strontium and barium 83. In which organ of the human body are the
(d) Chromium and nickel lymphocyte cells formed?
Ans. (c) : Creating firework colors is a complex (a) Liver (b) Long bone
endeavor, requiring considerable art and application (c) Pancreas (d) Spleen
of physical science. Strontium and barium both are Ans. (b) : Lymphocytes are the cells that determine the
alkaline earth metal and are extremely reactive. They specificity of the immune response to infectious
both impart characteristic color to flame. Strontium microorganisms and other foreign substances. In
salts impart a red color to fireworks. Strontium
humans, lymphocytes make up 25 to 33% of the total
compounds are also important for stabilizing
fireworks mixtures. Barium is used to create green number of leukocytes. They are found in central
colors in fireworks, and it can also help stabilize other lymphoid organs and tissues such as the spleen, tonsils
volatile elements. and lymph nodes and large bones (Bone Marrow).

IAS (Pre) GS 2004 Paper I 345 YCT


84. Assertion (A) :Fatty acids should be a part of the A B C D
balanced human diet. (a) 3 2 1 4
Reason (R) : The cells of the human body cannot (b) 1 4 3 2
synthesize any fatty acids. (c) 3 4 1 2
(a) Both A and R are individually true and R is (d) 1 2 3 4
the correct explanation of A. Ans. (b) : National Park/ Sanctuary
(b) Both A and R are individually true but R is A. Kanger Ghati National Park - Chhattisgarh
NOT the correct explanation of A. B. Nagarahole National Park - Karnataka
(c) A is true but R is false. C. Kugti Wildlife Sanctuary - Himachal Pradesh
(d) A is false but R is true. D. Sultanpur Bird Sanctuary - Haryana
Ans. (c) : Assertion is true because a balance diet is a 87. Match List I (Biosphere Reserve) with List II
diet which contains the correct amount of (States) and select the correct answer using the
carbohydrates, proteins and fats. But the reason is false, codes given below the lists:
because human body cells synthesize various fatty acids List I List II
except linoleic and linolenic fatty acids.
85. Consider the following statements: A. Similipal 1. Sikkim
1. Femur is the longest bone in the human body.
B. Dihang Dibang 2. Uttaranchal
2. Cholera is a disease caused by bacteria.
3. 'Athlete's foot' is a disease caused by virus. C. Nokrek 3. Arunachal Pradesh
Which of the statements given above are correct?
D. Kanchenjunga 4. Orissa
(a) 1 and 2 (b) 2 and 3
(c) 1 and 3 (d) 1, 2 and 3 5. Meghalaya
Ans. (a) : The head of the femur articulates with the
A B C D
acetabulum in the pelvic bone forming the hip joint,
(a) 1 3 5 4
while the distal part of the femur articulates with the
tibia and kneecap forming the knee joint. By most (b) 4 5 2 1
measures the femur is the strongest bone in the body. (c) 1 5 2 4
The femur is also the longest bone in the human body. (d) 4 3 5 1
Cholera is caused by a bacteria Vibrio Ans. (d) : Similipal biosphere reserve is in Orissa.
cholerae.Athlete’s foot disease is caused by parasificDihang Dibang biosphere reserve is in Arunachal
Pradesh, Nokrek biosphere reserve is in Meghalaya and
fungus of genus Trichophyton.
Kanchenjunga biosphere reserve is situated in Sikkim.
88. Amongst the following Indian States which one
ENVIRONMENT & ECOLOGY has the minimum total forest cover?
86. Match List I (National Park/Sanctuary) with (a) Sikkim (b) Goa
List II and (State) and select the correct answer (c) Haryana (d) Kerala
Ans. (c) : Haryana has the minimum total forest cover
using the codes given below the lists:
(till date 6.79%).
List I List II
CURRENT AFFAIRS
A. Kanger Ghati 1. Chhattisgarh
National Park 89. Which of the following is not a
recommendation of the task force on direct
B. Nagarahole 2. Haryana taxes under the chairmanship of Dr. Vijay L.
National Park Kelkar in the year 2002?
(a) Abolition of Wealth Tax.
C. Kugti Wildlife 3. Himachal Pradesh (b) Increase in the exemption limit of personal
Sanctuary income to Rs. 1.20 lakh for widows.
(c) Elimination of standard deduction.
D. Sultanpur Bird 4. Karnataka (d) Exemption from tax on dividends and capital
Sanctuary gains from the listed equity.
IAS (Pre) GS 2004 Paper I 346 YCT
Ans. (b) : Increase in the exemption limit of personal (a) 1 only (b) 2 only
income to Rs.1.50 lakh for widows was not one of the (c) Both 1 and 2 (d) Neither 1 nor 2
recommendations of the Kelkar commission in 2002. Ans. (a) : Statement '1' is correct, because National
90. Match List I (Book) with List II (Authors) and Thermal Power Corporation has diversified into the
select the correct answer using the codes given hydro projects. Presently it has undertaken Koldem
below the lists: project in Himachal Pradesh. Power Grid Corporation
of India has not been diversified into telecom sector.
List I List II
Hence statement 2 is not correct.
A. My Presidential 1. S. Radhakrishnan 93. The Ramon Magsaysay Award winner Shanta
Year Sinha is known as
(a) a campaigner for urban sanitation.
B. The Hindu View 2. V.V. Giri (b) an anti-child labour activist.
of Life (c) an organizer of rain-water harvesting schemes.
(d) an activist for the welfare of poor rural
C. Voice of 3. N. Sanjiva Reddy
women.
Conscience
Ans. (b) : The Ramon Magsaysay Award winner
D. Without Fear or 4. R. Venkataraman Shanta Sinha is an anti-child labour activist who had
Favour done considerable work to rescue child labour from
hazardous industries.
94. Match List I (persons) with List II (Positions)
A B C D and select the correct answer using the codes
(a) 2 1 4 3 given below the lists:
(b) 4 3 2 1 List-I List-II
(c) 2 3 4 1 A. B.P. 1. Executive Director,
(d) 4 1 2 3 Mishra IMF
Ans. (d) : My Presidential Years - R Venkataraman, B. Suresh 2. Chairman, Steal
The Hindu View of Life - S. Radhakrishnan's, Voice of Kalmadi Authority of India Ltd.
conscience – V.V. Giri, Without fear or favor - N. C. Praful 3. President Indian
Sanjiva Reddy. Patel Olympic Association
91. What was the reason for 5 lakh people of Hong D. V.S. Jain 4. Vice-President, South
Kong to make a demonstration around the Asian Region, World
middle of 2003? Bank
(a) They were demanding tax relief. A B C D
(b) They were against the Hong Kong (a) 4 2 1 3
Government's plan to impose an internal (b) 1 3 4 2
security law. (c) 4 3 1 2
(c) They were the of Falun Gong group who (d) 1 2 4 3
were demanding religious freedom. Ans. (b) : B.P. Mishra is the Executive Director of IMF,
(d) They were demanding more direct elections Suresh Kalmadi is the president of Indian Olympic
in Hong Kong. Association, Praful Patel is the Vice President, South
Ans. (b) : Nearly 5 lakh people of Hong kong Asian Region, World Bank and V.S. Jain is the
demonstrated around the middle of 2003 against the chairman of Steel Authority of India Limited.
Hong kong Government plan to impose an internal 95. Who among the following is well known as an
security law. exponent of flute?
92. Consider the following statements: (a) Debu Choudhary (b) Madhup Mudgal
1. National Thermal Power Corporation has (c) Ronu Majumdar (d) Shafaat Ahmad
diversified into hydropower sector. Ans. (c) : Flute - Hari Prasad Chaurasia, V Kunjamani,
2. Power Grid Corporation of India has Rajendra Prasanna - Renu Majumdar. Tabla player -
diversified into telecom sector. Zakir Hussain, Latif Khan, Fayyaz Khan, Sukhwinder
Which of the statements given above is/are Singh, Devu Chaudhary. Santurists - Bhajan Sopori,
correct? Shiv Kumar Sharma, Madhup Mudgal.

IAS (Pre) GS 2004 Paper I 347 YCT


96. Tarun Bharat Sangh, an organization based in 100. Consider the following statements:
a village near Alwar (Rajasthan) has become 1. The loans disbursed to farmers under Kisan
famous because of. Credit Card Scheme are covered under
(a) Cultivation of genetically modified cotton. Rashtriya Krishi Bima Yojana of Life
(b) rehabilitation of women victims of AIDS. Insurance Corporation of India.
(c) Livelihood projects for destitute rural women. 2. The Kisan Credit Card holders are provided
(d) rain-water harvesting. personal accident insurance of Rs 50,000 for
Ans. (d) : Tarun Bharat Sangh (TBS) is a non-profitable accidental death and Rs 25,000 for permanent
environmental NGO; with headquarter in Bheekampura, disability.
Alwar, Rajasthan. It has became famous due to Rain- Which of the statements given above is/are
water harvesting project. correct?
97. Param Padma, which was in news recently, is (a) 1 only (b) 2 only
(a) a new Civilian Award instituted by the (c) Both 1 and 2 (d) Neither 1 nor 2
Government of India Ans. (b) : The Kisan Credit Card Scheme was
(b) the name of a supercomputer developed by introduced in 1998-99, which facilitated the farmers to
India take loans from commercial banks. Such farmers, who
(c) the name given to a proposed network of are eligible for production loan of Rs. 5000 or more, are
canals linking northern and southern rivers of entitled to it. The loans under this are governed by the
India General Insurance Corporation (GIC). In the budget for
(d) a software programme to facilitate e- the year 2002-03, the government had also introduced
governance in Madhya Pradesh the facility of personal insurance package by the banks
to the Kisan Credit Card holders, which is given to the
Ans. (b) : Param Padma, a supercomputer developed in
other credit card holders. Under this, an amount of Rs
India, was introduced in April 2003. It had a peak speed
50,000 can be given on death due to accident and up to
of 1024 GFLOPS (about 1 TFLOP) and peak storage of
Rs 25,000 on permanent disqualification. The card
1 TB.
issuing institutions have to pay the premium amount of
98. Consider the following statements: this insurance. Source Indian Economy - S.N. Lal +
1. Hema Malini is the Chairperson of the NCERT.
Children's Film Society, India.
101. Liberia was in the international news in the
2. Yash Chopra is the Chairman of the Central recent times for.
Board of Film Certification of India.
Harboring terrorists associated with religious
(a)
3. Dilip Kumar, Raj Kapoor and Dev Anand
fundamentalism.
have all been recipients of Dada Saheb Phalke
Award. (b) Supplying raw uranium to North Korea.
Which of the statements given above is/are (c) Its long-running civil war killing or
correct? displacing thousands of people.
(a) 1 and 3 (b) 2 and 3 (d) Cultivation of drug-yielding crops and
smuggling of drug.
(c) 2 only (d) 3 only
Ans. (c) : Civil war has been going on in many
Ans. (d) : Indian Children's Film Committee - Raveena
countries of North Africa like Sudan, Liberia, Nigeria,
Tandon, Censor Board - Asha Parikh (currently
Cameroon etc. for years. Lakhs of people have died in
PRASOON JOSHI), Dada Saheb Phalke, Dilip Kumar,
this and the number and status of the displaced is also
Raj Kapoor and Devanand have been recipients of Dada
pathetic. Recently, agreement has been reached in the
Saheb Phalke Award.
disputed countries Nigeria and Cameroon on the
99. Famous Golf player Vijay Singh is from which
Bakassi Peninsula.
one of the following countries?
102. Consider the following statements:
(a) Fiji (b) Mauritius 1. P.V. Narasimha Rao's government
(c) Malaysia (d) Poland established diplomatic relations between
Ans. (a) : Golf player Vijay Singh is an Indo-Fijian India and Israel.
belongs to Fiji. He was born in 1963, who was number 2. Ariel Sharon is the second Prime Minister
of Israel to have visited India.
1 in the Official World Golf Ranking for 32 weeks in
Which of the statements given above is/are
2004 and 2005.
correct?
IAS (Pre) GS 2004 Paper I 348 YCT
(a) 1 only (b) 2 only (b) celebrate the tercentenary of the city of St.
(c) Both 1 and 2 (d) Neither 1 nor 2 Petersburg.
Ans. (a) : The diplomatic relation between India and (c) Convene a meeting of European and CIS
Israel was established in 1992 under the Narsimha Rao countries to discuss the issues of missile
regime. Ariel Sharon happened to be the first Prime shield for European and CIS countries.
Minister of Israel to visit India. (d) develop the strategies for containing global
103. The reason for Chechnya to be in the news terrorism.
recently is. Ans. (b) : More than 40 head of State Government were
(a) discovery of huge reserves of oil. invited by Vladimir Putin to celebrate the tercentenary
(b) separatist rebellious activities by the local of the city of St. Petersberg.
people. 108. Consider the following statements:
(c) continuous conflict between the government 1. Indira Gandhi Centre for Atomic Research
troops and the narcotic mafia resulting in a uses fast reactor technology.
great loss of human life. 2. Atomic Minerals Directorate for Research
(d) intense cold wave killing hundreds of people. and Exploration is engaged in heavy water
production.
Ans. (b) : The reason for Chechnya to be in the news
recently is separatist rebellious activities of the local 3. Indian Rare Earths Limited is engaged in
the manufacture of Zircon for India's
people against the Russian government.
Nuclear Programme beside other rare
104. The Prime Minister of which one of the earth products.
following countries was assassinated in the year Which of the statements given above are
2003? correct?
(a) Czech Republic (b) Romania (a) 1, 2 and 3 (b) 1 and 2
(c) Serbia (d) Slovenia (c) 1 and 3 (d) 2 and 3
Ans. (c) : The Prime Minister of Serbia was Ans. (a) : All the statements are correct according to
assassinated in the year 2003. Atomic developments in India.
105 Consider the following companies: 109. Consider the following international language:
1. Arabic 2. French
1. Voltas 2. Titan Industries
3. Spanish
3. Rallis India 4. Indian Hotels
The correct sequence of the language given
Which of the above companies are in the Tata
above in the decreasing order of the number of
Group of industries? their speakers is-
(a) 1 and 2 (b) 2, 3 and 4 (a) 3-1-2 (b) 1-3-2
(c) 1, 3 and 4 (d) 1, 2, 3 and 4 (c) 3-2-1 (d) 1-2-3
Ans. (d) : Voltas, Titan Industries, Rallis India and Ans. (a) : Total speaker of Spanish in world is 329
Indian Hotels are all companies under Tata Group. million, Arabic speakers are 220 million and French
106. In which one of the following countries, did an speakers are 200 million. So, the correct sequence in
decreasing order of the number of speakers is Spanish,
ethnic violence between the communities of
Arabic and French.
Hema and Lendu result in the death of
110. Match List I (Distinguished Ladies) with List II
hundreds of people?
(Area of Work) and select the correct answer
(a) Democratic Republic of Congo using the codes given below the lists:
(b) Indonesia
List I List II
(c) Nigeria
(d) Zambia A. Jhumpa Lahiri 1. Science and
Ans. (a) : The ethnic violence between the communities environment
of Hema and Lendu results in the death of hundred
people in Democratic Republic of Congo. B. Sunita Narain 2. Novel-writing
107. More than 40 Heads of States/Governments C. Naina Lal Kidwai 3. Film industry
were invited by Vladimir Putin in May, 2003
to. D. Ravina Raj Kohli 4. Banker
(a) discuss the issues related to the rehabilitation
5. Television media
of Iraq.
IAS (Pre) GS 2004 Paper I 349 YCT
A B C D Ans. (c) : Mahesh Bhupati’s partner Max Mirnyi is
(a) 4 5 3 1 from Belarus.
(b) 2 1 4 5 116. Which of the following Cricketers holds the
(c) 4 1 3 5 record for the highest score in a Cricket Test
(d) 2 5 4 1 Match innings by an Indian?
Ans. (b) : Jhumpa Lahiri is a famous novel writer. His (a) Sunil Gavaskar (b) Vinod Mankad
famous novel is 'Namesake', Sunita Narain is a famous (c) Sachin Tendulkar (d) V.V.S. Laxman
environmentalist. The name of his organization is'
Center for Science and Environment (CSE), Naina Lal Ans. (d) : Among the given options, Highest score in a
Kidwai - is a famous banker. Raveena Raj Kohli – cricket match innings by an Indian is 281 runs of V.V.S.
Television media (Doordarshan). (Source - India Laxman.
Today) 117. INS Trishul acquired by the Indian Navy in
111. There research work of Paul Lauterbur and 2003 has been built by
Peter Mansfield, the Nobel Prize winners for (a) Israel (b) USA
Medicine in 2003, relates to. (c) Russia (d) France
(a) the control of AIDS. Ans. (c) : INS Trishul acquired by Indian Navy in 2003
(b) magnetic resonance. has been built by Russia.
(c) respiratory diseases. 118. INSAT-3E, India's communication satellite,
(d) genetic engineering. was launched in 2003 from
Ans. (b) : The research work of Paul Lauterbur and (a) French Guiana (b) Seychelles
Peter Mansfield, the Nobel Prize winners for medicine (c) Mauritius (d) Mauritania
in 2003 relates to magnetic resonance imaging. Ans. (a) : INSAT–3E, India’s communication satellite
112. In 2003, Alison Richard took over as the first was launched in 2003 from French Guiana.
ever women Vice-Chancellor of. 119. Match List I (Person) with List II (Position)
(a) Oxford University and select the correct answer using the codes
(b) Cambridge University given below the lists:
(c) Harward University
List I List II
(d) Purdune University
Ans. (b) : In 2003, Alison Richard took over as the A. Anil Kakodar 1. Chief of the
second women Vice-Chancellor of Cambridge University. Integrated Defense
113. George W. Bush, the President of America, Staff
comes from which of the following American
B. Raman Puri 2. Chairman, 17th Law
States?
Commission
(a) California (b) Texas
(c) Virginia (d) Indian C. M. Jagannadha Rao 3. Chairman, ISRO
Ans. (b) : George W. Bush, the President of America,
comes from the American state of Texas. D. G. Madhavan Nair 4. Chairman Atomic
Energy Commission
114. Which of the following authors won the Booker
Prize twice? A B C D
(a) Margaret Atwood (b) J.M. Coetzee (a) 3 1 2 4
(c) Graham Swift (d) Ian McEwan
(b) 4 2 1 3
Ans. (b) : J.M. Coetzee, a South African writer and
(c) 3 2 1 4
scholar won the Booker Prize twice. He got his first
(d) 4 1 2 3
Booker Prize in 1983 for his book “Life and times of
Michael K” and got second Booker Prize in 1999 for his Ans. (d) : Anil Kakodar is the president of the Indian
book “Disgrace”–a novel about life in post apartheid Nuclear Energy Commission; Raman Puri is the
South Africa. President of the Integrated Defense Staff, whose
115. In the well-known Lawn Tennis doubles team, headquarters is in Andaman; 17th Law Commission
Max Mirnyi-the partner of Mahesh Bhupati, President - M. Jagannath Rao; Indian Space Research
comes from which of the following countries? Organization (ISRO) Madhavan Nayar (current- Dr. K.
(a) Italy (b) Sweden Sivan) is the President of Indian Space Research
(c) Belarus (d) Croatia Organization (ISRO)

IAS (Pre) GS 2004 Paper I 350 YCT


120. Which one of the following statements is Ans. (d) : Shikha Tandon Swimming
correct? 'Deccan Odyssey' is Ignace Tirkey Hockey
(a) a book on Chatrapati Shivaji. Pankaj Advani Billiards & Snooker
(b) a warship recently acquired by the Indian Rohan Bopanna Lawn Tannis
Navy. 124. The record for the highest score in an innings
(c) a recently started air service between Mumbai in test Cricket is now being held by M. Hyden.
and Colombo. Immediately prior to him, the three record
holders were.
(d) a luxury train which travels through
(a) Don Bradman, Sunil Gavaskar & Colin
Maharashtra.
Cowdrey
Ans. (d) : 'Deccan Odyssey' is a luxury train which (b) Len Hutton, Peter May & Vivian Richards.
travels through Maharashtra and includes Goa in its (c) Hanif Mohammed, Garfield Sobers and Brian
journey. Lara.
121. Which one of the following is the correct (d) Bob Cowper, Bill Lawry and Brian Lara
decreasing sequence in terms of the value (in Ans. (c) : Hanif Mohammaed of Pakistan scored 356
rupees) of the minerals produced in India in Runs which was broken by Garfilled Sobers with a
the year 2002-03? score of 365 runs, this record was broken by Brian Lara
(a) Metallic minerals-Fuel minerals-Non-metallic of West Indies with 375 runs, this was broken by
minerals. Mathew Hayden with 380 runs but recently Lara broke
his record with 400 runs in a test inning.
(b) Fuel minerals-metallic minerals-Non-metallic
125. Consider the following statements:
minerals.
1. Smart Card is a plastic card with an
(c) Metallic minerals-Non-metallic minerals-Fuel
embedded microchip.
minerals.
2. Digital technology is primarily used with
(d) Fuel minerals-Non-metallic minerals-Metallic new physical communication medium such
minerals. as satellite and fibre optics transmission.
Ans. (b) : Fuel minerals- Metallic minerals- Non 3. A digital library is a collection of
metallic minerals. documents in an organized electronic form
122. Which one of the following cities (they were in available on the Internet only.
the news in recent times) is not correctly Which of the statements given above is/are
matched with its country? correct?
(a) Salamanca : Spain (a) 3 only (b) 1 and 2
(b) Cannes : Italy (c) 2 and 3 (d) 1, 2 and 3
(c) Cancun : Mexico Ans. (b) : A digital library is a special library with a
(d) Bruges : Belgium focused collection of digital objects that can include
Ans. (b) : Cannes (France) is a city organising film text, visual material, audio material, video material,
festival every year. stored as electronic media formats (as opposed to print,
123. Match List I (Sports-person) with List II microform, or other media), along with means for
(Sport/Game) and select the correct answer organizing, storing, and retrieving the files and media
using the codes given below the lists: contained in the library collection. Digital libraries can
vary immensely in size and scope, and can be
List I List II maintained by individuals, organizations, or affiliated
with established physical library buildings or
A. Shikha Tandon 1. Badminton institutions, or with academic institutions. The digital
content may be stored locally, or accessed remotely via
B. Ignace Tirkey 2. Swimming computer networks. An electronic library is a type of
information retrieval system
C. Pankaj Advani 3. Lawn Tennis
126. Shirin Ebadi, who won the Nobel Peace in
D. Rohan Bopanna 4. Snooker Hockey 2003, is from.
5. Hockey (a) Iraq (b) Nigeria
(c) Iran (d) Libya
A B C D
Ans. (c) : Shirin Ebadi is a lawyer from Iran and has
(a) 3 5 4 2 contributed significantly for the democratisation of
(b) 2 4 1 3 polity in Iran. She also participated in the world social
(c) 3 4 1 2 forum held in Mumbai. She won Nobel Peace Prize in
(d) 2 5 4 3 2003.
IAS (Pre) GS 2004 Paper I 351 YCT
MISCELLANEOUS Which of the statements given above are
correct?
127. Which of the following pairs is not correctly (a) 1 and 2 (b) 1, 2 and 3
matched? (c) 2, 3 and 4 (d) 1, 3 and 4
(a) Reinhold Messner : Computer Ans. (a) : Claims to Macedonia territory have been a
Technology source of contention between Bulgaria and Greece.
In 1991, Slovenia declared independence from
(b) Harlow Shapley : Astronomy
Yugoslavia.
(c) Gregor Mendel : Hereditary Theory
131. The Archaeological Survey of India is an
(d) Godfrey Hounsfield : CT Scan attached office of the Department/Ministry of
Ans. (a) : Reinhold Andreas Messner is an Italian (a) Culture
mountaineer, explorer, and author. (b) Tourism
128. Consider the following statements: (c) Science and Technology
1. Adam Osborne produced the first portable (d) Human Resource Development
computer Ans. (a) : Archaeological Survey of India (ASI) was
2. Ian Wilmut created the first cloned sheep established in 1861. It functions as an institution
Which of the statements given above is/are associated with the Department of Culture. Its main
correct? functions are - maintenance, preservation,
(a) 1 only (b) 2 only archaeological discovery work and excavation of
(c) Both 1 and 2 (d) Neither 1 nor 2 monuments, excavation of sites and ruins preserved by
Ans. (c) : Adam Osborne introduced the first portable the center, chemical preservation of monuments and
computer in 1981 and Ian Wilmut, an English archaeological remains, etc. Presently, there are 3693
embryologist first cloned a mammal (Lamb) in year monument sites of national importance, out of which 40
1996. monuments are included in the World Heritage Sites.
129. Consider the following statements: (Source - India-2005)
As per 2001 Census 132. Consider the following statements:
1. the two States with the lowest sex ratio are 1. The Islamic Calendar is twelve days
Haryana and Punjab shorter than the Gregorian Calendar.
2. the two States with the lowest population 2. The Islamic Calendar began in AD 632
per sq km of area are Meghalaya and 3. The Gregorian Calendar is a solar
Mizoram calendar
3. Kerala has both the highest literacy rate Which of the statements given above is/are
and sex ratio correct?
Which of the statements given above is/are (a) 1 only (b) 1 and 2
correct? (c) 1 and 3 (d) 3 only
(a) 3 only (b) 2 and 3 Ans. (d) : The Islamic calendar began in 622 CE during
(c) 1 and 2 (d) 1 and 3 which the emigration of Muhammad from Mecca to
Ans. (d) : Two states with lowest density of population Medina known as Hijri occurred. It is a lunar calendar
are Arunachal Pradesh (13) and Mizoram (52). Hence consisting of 12 lunar months in a year of 354 or 355
statement 2 is not correct. Rest of the statements is days. It is also called Hijri calendar. The Gregorian
correct. calendar is a lunar calendar.
130. Consider the following statements: 133. World's longest ruling head of government is
1. Montenegro and Serbia agreed to a new from.
structure for the Yugoslavia Federation. (a) Switzerland (b) Cuba
2. Croatia remained under the Hungarian (c) Zimbabwe (d) New Zealand
Administration until the end of First Ans. (b) : World’s longest ruling government is Fidel
World War. Castro of Cuba. Fidel Castro came in power in 1959
3. Claims to Macedonia Territory have long
following the Cuban revolution and became President
been a source of contention between
Belgium and Greece. Minister of Cuba in 1961 and from 1976; he became the
4. In 1991, Slovenia declared independence President of Cuba. Fidel Castro died on November 25,
from Czechoslovakia. 2016.
IAS (Pre) GS 2004 Paper I 352 YCT
134. Match List I (Institute) with List II (Located 136. Match List I with List II and select the correct
At) and select the correct answer using the answer using the codes given below the lists:
codes given below the lists: List I List II
List I List II (Agency) (Headquarters)

A. United Nations Development 1. Nairobi


A. Indian Institute of 1. Coimbatore Programme (UNDP)
Geomagnetism
B. United Nations Environment 2. Vienna
B. International 2. Mumbai Programme (UNEP)
Advanced Research
C. United Nations Industrial 3. Bern
Centre for Powder
Development Organization
Metallurgy and New
(UNIDO).
Materials
D. Universal Postal Union 4. New York
C. Salim Ali Centre for 3. Jabalpur (UPU)
Ornithology & Natural
History
A B C D
D. Tropical Forest 4. Hyderabad (a) 2 3 4 1
Research Institute (b) 4 1 2 3
(c) 2 1 4 3
A B C D
(d) 4 3 2 1
(a) 2 3 1 4
Ans. (b) : The United Nations was established on
(b) 1 4 2 3
October 24, 1945. It is headquartered in New York. It
(c) 2 4 1 3
is on the island of Manhattan. Under this, various
(d) 1 3 2 4 institutions were formed in different years.
Ans. (c) : Indian Institute of Geomagnetism - Located UNDP 1965 New York
in Mumbai (Maharashtra); International Advanced UNEP 1972 Nairobi
Institute related to Powder Metallurgy - Hyderabad.
UNIDO 1966 Vienna
Salim Ali Center for Ornithology and Natural History -
UPU 1874 Bern
Coimbatore (Tamil Nadu). Tropical Forest Research
Institute - Jabalpur (MP). (Source - India, 2005) WMO 1950 Geneva
UNESCO 1945 Paris
135. Which of the following pairs is correctly
matched? IMF 1944 Washington (D.C.)
IAEA 1957 Vienna
Departments Ministry of the
Government of India 137. Which one of the following pairs is not
correctly matched?
1. Department of : Ministry of Health
Women and Child and Family Welfare Unit of Indian Railway Location
Development
2. Department of : Ministry of Human (a) Railway Staff College : Vadodara
Official Language Resource Development
3. Department of : Ministry of Water (b) Central Organization for : Varanasi
Drinking Water Resources Railway Electrification
Supply
Select the correct answer using the codes given (c) Wheel and Axle Plant : Bangalore
below:
(d) Rail-coach Factory : Kapurthala
(a) 1 (b) 2
(c) 3 (d) None
Ans. (d) : Department of Child Development : Ministry Ans. (b) : There is a Diesel Locomotive Works (DLW)
of Women and Child Development; Department of at Maduadih, Varanasi. While the Central Railway
Official language : Ministry of Home Affairs; Electrification Organization is in Allahabad.
Department of Drinking Water : Ministry of Jal Shakti Other institutes of Railways are-
and Sanitation. Rail Engine Factory - Chittaranjan.
IAS (Pre) GS 2004 Paper I 353 YCT
Rail Box Factory - Kapurthala (Punjab). Ans. (c) : Since, answers of no. two examinees are
Rail Wheel Manufacturing Factory - Chapra identical, so first item in List-A can be matched with
Railway Staff College - Vadodara (Gujarat). any of the 5 items in List-B. It can be done in 5 ways.
Major Research Branch of Indian Railways - Lucknow. Similarly, 2nd item in List-A can be matched with any
of the remaining 4 items in List-B. It can be done in 4
Indian Railways is organized into 18 zones.
ways. Continuing in the same way, No of ways of
138. Match List I (New Names of the Countries) arranging the items = 5 × 4 × 3 × 2 × 1=120 Now, in
with List II (Old names of the countries) and this arrangement there is one such arrangement, which
select the correct answer using the codes given is the correct answer. ∴ Maximum number of
below the lists: examinees = no. of ways of arrangement of items = 120
List I List II – 1 = 119
The details given below relate the FOUR items that
A. Benin 1. Nyasaland follow:
Amit wishes to buy a magazine. Four magazines-one
B. Belize 2. Basutoland each on politics, sports, sciences and films are available
to choose from, they are edited by Feroz, Gurbaksh,
C. Botswana 3. Bechuanaland
Swami and Ila (no necessarily in that order) and
D. Malawi 4. British Honduras published by Aryan, bharat, Charan and Dev Publishers
(not necessarily in that order). Further, it is given that
5. Dohomey (i) Dev Publishers have published the magazine
edited by Feroz
A B C D
(ii) The magazine on politics in published by
(a) 3 1 2 4 Aryan Publishers
(b) 5 4 3 1 (iii) The magazine on films is edited by Swami
(c) 3 4 2 1 and is not published by Charan Publishers
(d) 5 1 3 4 141. The magazine on politics is
Ans. (b) : Country Old name (a) edited by Ila
Benin Dahomey (b) edited by Gurbaksh
(c) published by Dev Publishers
Belize British Honduras
(d) published by Charan Publishers
Botswana Bechuanaland
Ans. (b) :edited by Gurbaksh
Malawi Nyasaland
142. The magazine on science is published by
139. Two cars X and Y starts from two places A and (a) Aryan Publishers (b) Bharat Publishers
B respectively which are 700 km apart at 9 a.m. (c) Charan Publishers (d) Dev Publishers
both the cars run at an average speed of 60 Ans. (c) : Charan Publishers
km/hr. Car X stops at 10 a.m. and again starts 143. The magazine on sports is:
at 11 a.m. while the other car Y continues to (a) edited by Feroz
run without stopping. When do the two cars (b) edited by Gurbaksh
cross each other? (c) published by Bharat Publishers
(a) 2 : 40 p.m. (b) 3 : 20 p.m. (d) Published by Charan Publishers
(c) 4 : 10 p.m. (d) 4 : 20 p.m. Ans. (a) : edited by Feroz
Ans. (b) : 144. The magazine on films is:
140. In a question of a test paper, there are five (a) published by Dev Publishers
items each under List-A and List-B. The (b) published by Bharat Publishers
examinees are required to match each item (c) edited by Gurbaksh
under List-A with its corresponding correct (d) published by Charan Publishers
item under List-B. Further, it is given that Ans. (b) :published by Bharat Publishers
(i) no examinee has given the correct answer 145. A and B start from the same point and in the
same direction at 7 a.m. to walk around a
(ii) answers of no two examinees are identical
rectangular field 400 m x 300 m. A and R walk
What is the maximum number of examinees at the rate of 3 km/hr and 2.5 km/hr
who took this test? respectively. How many times shall they cross
(a) 24 (b) 26 each other if they continue to walk till 12:30
(c) 119 (d) 129 p.m.?
IAS (Pre) GS 2004 Paper I 354 YCT
(a) Not even once (b) Once 148. In how many different ways can six players be
(c) Twice (d) Thrice arranged in a line such that two of them, Ajit
Ans. (b) and Mukherjee, are never together?
Perimeter of the filed (a) 120 (b) 240
= 2 (400+300)m (c) 360 (d) 480
=1400m Ans. (d) : Total no of ways of arrangement for six
= 1.4 Km players = 6! Let us take Ajit and Mukerjee as one entity.
Since A and B move in the same direction, so they will So now there are (6 – 2 + 1) = 5 players .
first meet each other when there is a difference of one These 5 players can be arranged in 5! ways and Ajit and
round i.e., 1.4 Km between the two Relative speed of A Mukerjee can be arranged among themselves in 2!
and B = (3-2.5) Km ways. Thus, no of ways, when Ajit and Mukerjee are
= 0.5 Km/ hr always together = 5 ! × 2!
Time take to cover 1.4 Km at the this speed: Hence, no of ways when they are never together =
Total no of ways – no of ways when they are always
 1.4 
=  hr together
 0.5  = 6 ! – (5 ! × 2 !) = 6 × 5 ! – (5 ! × 2 !) = 5 ! (6 – 2) =
4 480
= 2 hr
5 149. 50 men or 80 women can finish a job in 50
= 2 hr 48 min days. A contractor deploys 40 men and 48
So, they shall first cross each other at women for this work, but after every 'duration
9.48m of 10 days, 5 men and 8 women are removed till
the work is completed. The work is completed
And again 2 hr 48 min after 9.48 am i,e.,
in
12.36 pm (a) 45 days (b) 50 days
Thus, till 12.30 pm they will cross each other once. (c) 54 days (d) 62 days
146. Nine different letters are to be dropped in three Ans. (b) : Total number of working day = 50 × 50 =
different letter boxes. In how many different 2500 days for men, For women it is equal to 4000 days
ways can this be done? For 1st 10 day means 40 men × 10 = 400 day, 48
(a) 27 (b) 3 9 women × 10 = 480 days 2nd after 10 days means 35
(c) 93 9
(d) 3 – 3 men × 10 = 350 day, 40 women × 10 = 400 days 3rd 10
day 30 men × 10 = 300 days, 32 women × 10 = 320
Ans. (b) : First letter can be dropped into any of the 3 days 4th 10 day 25 men × 10 = 250 days, 24 women ×
boxes. It can be done in 3 ways. Similarly second letter 10 = 240 days 5th 10 day 20 men × 10 = 200 days, 16
can also be dropped into any of the 3 boxes in 3 ways women × 10 = 160 days Men = 1500 days, Women =
and so on. Hence, total no of ways = 3×3×3×.........upto 1600 days 1 man = 1.6 women 1600 days of women =
9 times = 3(raise the power 9) 1000 day of men So, 2500 days need to complete work
147. How many three-digit even numbers are there mean after 50 day i.e., Man of 1600 working days +
such than 9 comes as a succeeding digit in any woman of 1600 days i.e., 50 days.
number only when 7 is the preceding digit only 150. Three students are picked at random from a
when 9 is the succeeding digit? school having a total of 1000 students. The
(a) 120 (b) 210 probability that these three students will have
(c) 365 (d) 405 identical date and month of their birth is
(a) (b)
Ans. (d) : Since, we need three digit 100th , 10 th ,& unit .
There is 0, 1, 2, 3, 4, 5, 6, 7, 8, 9 are possible digits of
required no. now, since required number is even no. (c) (d)
Thus at unit place there is five possibilities - 0, 2, 4, 6
and 8. Ans. (c) : For 1st student, Probability of selecting any
Now at 100th place there is - 1, 2, 3, 4, 5, 6, 7, 8, 9 are one day as his birthday =365/365 =1 Now, the
possible and 10th place, 1, 2, 3, 4, 5, 6, 7, 8 and 9 are remaining two students to be selected must have same
possible numbers. day as their birthday as for the 1st student. Probability
Then by "fundamental theorem of counting", of rest two students, having the same birthday as that of
9×9×5 = 405 the 1st student = 1/365 x 1/365 Hence, required
Hence, option (d) - 405 will be correct answer. probability = 1 x 1/(365)2 = 1/(365)2

IAS (Pre) GS 2004 Paper I 355 YCT


UNION PUBLIC SERVICE COMMISSION
Civil Services (Preliminary Exam) - 2003
GENERAL STUDIES : PAPER-I
Time: 2 hours Maximum Number: 200
portrays his early life and his military conquests. In the
ANCIENT HISTORY Battle of Venni, he defeated the mighty confederacy
consisting of the Cheras, Pandyas and eleven minor
1. Emperor Harsha's southward march was chieftains. The Chola emperor launched a successful
stopped on the Narmada river by naval expedition against the Sailendra kingdom.
(a) Pulakesin-I (b) Pulakesin-II 4. 'Mrichchhakatika' and an ancient Indian book
(c) Vikramaditya-I (d) Vikramaditya-II written by Shudraka deals with
Ans. (b) : Emperor Harsha's southward march was (a) the love affair of a rich merchant with the
stopped on the Narmada river by Pulakesin II, the daughter of a courtesan.
Chalukya king of Vatapi in northern Mysore. (b) the victory of Chandragupta II over the Shaka
2. Assertion (A) : Saluva Narasimha put an end to Kshatrapas of western India.
the old dynasty and assumed to royal life. (c) the military expeditions and exploits of
Reason (R) : He wanted to save the kingdom Samudragupta.
from further degeneration and disintegration. (d) the love affair between a Gupta King and a
(a) Both A and R are individually true and R is princess of Kamarupa.
the correct explanation of A. Ans. (a) : Mrichchhakatika (The Clay Court) is a
(b) Both A and R are individually true but R is Sanskrit play written by Shudraka in 2nd century BC. It
NOT the correct explanation of A. is about a young man named Charudatta, and his love
(c) A is true but R is false. for Vasantasena, a rich courtesan.
(d) A is false but R is true. 5. Consider the following statements:
Ans. (a) : The founders of Vijayanagar, Harihara and 1. Vardhaman Mahavira's mother was the
Bukka, laid the foundation of the Sangam dynasty. daughter of Lichhavi chief Chetaka.
Virupaksha II, the last ruler of this dynasty, was a 2. Gautama Buddha's mother was a princess
completely unfit person. There was chaos all around from the Koshalan dynasty.
the empire. In this situation Narasimha Saluva, a 3. Parshvanatha, the twenty-third Tirthankara,
powerful feudal lord of the empire, took power by belonged to Banaras.
killing Virupaksha. Narasimha Saluva strengthened the Which of these statements is/are correct?
army to reorganize the empire. For that he encouraged (a) Only 1 (b) Only 2
Arab traders to import more and more horses. He (c) 2 and 3 (d) 1, 2 and 3
appointed an ambitious man named Narsa Nayak as the Ans. (c) : Vardhaman Mahavira's mother, Trishala was
guardian of his son. Narsa Nayak took over the power the sister (not the daughter) of Lichhavi chief Chetaka.
and greatly expanded the empire. Later, his son Veer
6. Consider the following statements:
Narasimha laid the foundation of the Tuluva dynasty.
1. The last Mauryan ruler, Brihadratha was
3. Consider the following statements: assassinated by his commander-in-chief,
1. The Cholas defeated Pandya and Chera Pushyamitra Sunga.
rulers and established their domination
2. The last Sunga king, Devabhuti was
over peninsular India in the early medieval
assassinated by his Brahmana minister
times.
Vasudeva Kanva who usurped the throne.
2. The Cholas sent an expedition against
3. The last ruler of the Kanva dynasty was
Sailendra empire of South East Asia and
deposed by the Andhras.
conquered some of the areas.
Which of these statements is/are correct? Which of these statements is/are correct?
(a) Only 1 (b) Only 2 (a) 1 and 2 (b) Only 2
(c) Both 1 and 2 (d) Neither 1 nor 2 (c) Only 3 (d) 1, 2 and 3
Ans. (c) : The Chola kingdom of the Sangam period Ans. (d) : All statements are correct. The last Mauryan
extended from modern Tiruchi district to southern ruler, Brihadratha was assassinated by his commander-
Andhra Pradesh. Their capital was first located at in-chief, Pushyamitra Sunga in 185 BC. The last ruler
Uraiyur and then shifted to Puhar. Karikala was a of Kanva dynasty, Susharman was deposed by the
famous king of the Sangam Cholas. Pattinappalai Andhras and Satavahana dynasty came into being.

IAS (Pre) GS 2003 Paper I 356 YCT


MEDIEVAL HISTORY Reason (R) : Marathas were the first to have a
clear concept of a united India nation.
Direction: The following 15 (fifteen) items consist of
(a) Both A and R are individually true and R is
two statements, one labelled as the Assertion
the correct explanation of A.
‘(A)' and the other as Reason ‘(R)’. You are to
examine these two statements carefully and (b) Both A and R are individually true but R is
select the answers to these items using the code NOT the correct explanation of A.
given below: (c) A is true but R is false.
(a) Both A and R are individually true and R is (d) A is false but R is true.
the correct explanation of A. Ans. (c) : Mughal empire declined due to
(b) Both A and R are individually true but R is misgovernance and foreign intruders like Britishers.
NOT the correct explanation of A. During this period, Marathas resisted well and emerged
(c) A is true but R is false. as the strongest native power in India. Marathas never
(d) A is false but R is true. had the concept of a United Indian Nation. They are
7. Assertion (A) : Emperor Akbar marched towards generally criticized for this reason.
Afghanistan in 1581 with a huge army:
10. Alam Khan, one of those who invited Babur to
Reason (R) : He was on his way to reclaim his
invade India was
ancestral country of Ferghana in Central Asia.
(a) Both A and R are individually true and R is (a) An uncle of Ibrahim Lodi and a pretender to
the correct explanation of A. the throne of Delhi.
(b) Both A and R are individually true but R is (b) A cousin of Ibrahim Lodi who was ill-treated
NOT the correct explanation of A. and expelled from the country.
(c) A is true but R is false. (c) The father of Dilawar Khan to whom cruel
(d) A is false but R is true. treatment was meted out by Ibrahim Lodi.
Ans. (c) : Akbar marched towards Afghanistan in 1581 (d) A high official in Punjab province who was
to conquer Kabul to expand his kingdom. R is incorrect very much discontented with Ibrahim Lodi's
as he had no intention to reclaim his ancestral country treatment to his tribe.
of Ferghana. Ans. (a) : Babur fought three important battles in India
8. Assertion (A) : Shah Alam II spend the initial viz, the battles of Panipat, Khanwa and Ghaghara and
years as an Emperor far away from his capital. won all of them. He came to India from Ferghana to
Reason (R) : There was always a lurking fight Ibrahim Lodi in the battle of Panipat and establish
danger of foreign invasion from the north-west his rule in India. Due to disaffection with Ibrahim,
frontier. Daula Khan Lodi along with the Ibrahim Lodi's own
(a) Both A and R are individually true and R is uncle, Alam Khan invited Babur to invade the
the correct explanation of A.
kingdom.
(b) Both A and R are individually true but R is
NOT the correct explanation of A. 11.
The battle of Dharmat was fought between
(c) A is true but R is false. (a) Muhammad Ghori and Jai Chand
(d) A is false but R is true. (b) Babur and Afghans
Ans. (c) : Aligauhar ascended the throne of Shah Alam (c) Aurangzeb and Dara Shikoh
II after the assassination of Alamgir II by Wazir (d) Ahmad Shah Durrani and the Marathas
Imadulmulk in the course of the destabilization of the
Ans. (c) : The battle of Dharmat was fought between
Aurangzeb and Dara Shikoh in 1658 for Shahjahan's
later Mughal rulers. He spent the initial years as an
throne. Dara Shikoh was defeated by Aurangzeb.
emperor away from his capital due to the fear of Najib
Khan Rohila, who became very powerful in Delhi, and
12. How did the Mughal Emperor Jahandarshah's
not due to forigh invasion. He was re-established in reign come to an early end?
1772 AD with the help of Mahadji Scindia. Finally (a) He was deposed by is Wazir.
after the Delhi victory in 1803 by the British, he was(b) He died due to a slip while climbing down
assassinated by Ghulam Qadir in 1806 AD. steps.
9. Assertion (A) : Marathas emerged as the (c) He was defeated by his nephew in a battle.
strongest native power in India after the (d) He died of sickness due to too much
decline of Mughal empire. consumption of wine.
IAS (Pre) GS 2003 Paper I 357 YCT
Ans. (c) : Jahandarshah was defeated in the battle of MODERN HISTORY
Agra on 10 January, 1713 by Farrukhsiyar, his nephew
and the second son of Azim-ush-Shan, with the support 15. Who headed the Interim Cabinet formed in the
year 1946?
of the Sayyid Brothers. Jahandarshah fled to Delhi
(a) Rajendra Prasad
where he was captured and handed over to the new
(b) Jawaharlal Nehru
Emperor, who confined him along with Lal Kunwar. He
(c) Sardar Vallabhbhai Patel
lived in confinement for a month, until 11 February
(d) Rajagopalachari
1713, when professional stranglers were sent to murder
Ans. (b) : Interim government was formed on
him.
September 2, 1946 as per Cabinet mission proposal. It
13. Consider the following statements: was headed by Jawahar Lal Nehru.
1. Kitab-i-Nauras, a collection of songs in 16. The leader of the Bardoli Satyagraha (1928)
praise of Hindu deities and Muslim saints, was :
was written by Ibrahim Adil Shah II. (a) Sardar Vallabhbhai Patel
2. Amir Khusrau was the originator in India (b) Mahatma Gandhi
of the early from of the musical style (c) Vaithalbhai J. Patel
known as Qawali. (d) Mahadev Desai
Which of these statements is/are correct? Ans. (a) : Bardoli Satyagraha of 1928, in the state of
(a) Only 1 (b) Only 2 Gujarat was led by Vallabhbhai Patel. The women of
(c) both 1 and 2 (d) neither 1 nor 2 Bardoli gave him the title of Sardar.
Ans. (c) : Ibrahim Adilshah was the ruler of Bijapur. 17. During the colonial period in India, what was
He made Hindavi the official language in place of the purpose of the Witley Commission?
Persian. Ibrahim Adilshah appointed Hindus to various (a) To review the fitness of India for further
political reforms.
posts. He composed the Hindi song collection Kitab-e-
Nauras, which praised Muslim saints and Hindu deities. (b) To report on existing conditions of labour and
to make recommendations.
At this time, Farishta wrote a historical book called
(c) To draw up a plan for financial reforms for
'Tithi-e-Farishta'. The initiator of Qawali was Amir
India.
Khusro.
(d) To develop a comprehensive scheme for Civil
14. How did Sultan Qutb-ud-din Aibak die? Services in India.
(a) He was treacherously stabbed to death by one Ans. (b) : The Royal Commission on Labour or the
of his ambitious nobles. Whitley Commission on Labour was set up in 1929 to
(b) He was killed in a battle with Taj-u-din inquire into the existing conditions of labour in
Yildiz, the ruler of Ghazni who entered into a industrial undertakings and plantations in India. The
contest with him over the capture of Punjab. Commission was chaired by John Henry Whitley.
(c) He sustained injuries while besieging the 18. With reference to the entry of European
powers into India, which one of the following
fortress of Kalinjar in Bundelkhand and
statement in NOT correct?
succumbed to them later. (a) The Portuguese captures Goa in 1499.
(d) He died after a fall from his horse while (b) The England opened their first factory in
playing Chaugan. South India at Masulipatam.
Ans. (d) : Qutbuddin Aibak was given authority over (c) In Eastern India, the English Company
opened its first factory in Orissa in 1633.
Indian territories while returning after Mohammad
(d) Under the leadership of Dupleix, the French
Ghori took control of Indian territories. Qutbuddin
occupied Madras in 1746.
Aibak only assumed the title of 'warlord'. He did not try
Ans. (a) : The Portuguese conquest of Goa occurred
to mint coins in his own name. In the memory of
when the governor of Portuguese India, Alfonso de
Qutubuddin Bakhtiyar Kaki, the foundation of Qutub Albuquerque captured the city in 1510. Goa was not
Minar was laid, which was completed by Iltutmish. His among the cities Albuquerque had received orders to
capital was Lahore where he died from falling from a conquer: he had only been ordered by the Portuguese
horse while playing Chaugan in 1210 AD. king to capture Hormuz, Aden and Malacca.

IAS (Pre) GS 2003 Paper I 358 YCT


19. With reference to Indian freedom struggle, Ans. (a) : The poem was first sung by Rabindranath
which one of the following statements in NOT Tagore in 1896. The first two verses of the song were
correct? adopted as the National Song of India in October 1937
(a) The Rowlett Act aroused a wave of popular by the Congress Working Committee prior to the end of
indignation and led to the Jallianwala Bagh
colonial rule in August 1947. An ode to the Motherland,
Massacre.
it was written in Bengali script in the novel Anandmath.
(b) Subhas Chandra Bose formed Bloc.
(c) Bhagat Singh was one of the founders of The original language of Vande Mataram, the national
Hindustan Republican Socialist Association. song of India, composed by Bankimchandra Chatterji
(d) In 1931, the Congress Session at Karachi was Sanskrit but was written in Bengali script. Hence
opposed Gandhi-Irwin Pact. first statement is not correct. Rest of the given
Ans. (d) : Gandhi Irwin Pact was endorsed by the statements is correct.
Congress during the Karachi Session of 1931, which 23. The aim of education as stated by the Wood's
was held from March 26-31. Gandhi was nominated to dispatch of 1854 was:
represent Congress in the Second Round Table (a) The creation of employment opportunities for
Conference. Rest of the given options is correct. native Indians.
20. In India, among the following locations, the (b) The spread of western culture in India.
Dutch established their earliest factory at: (c) The promotion of literacy among the people
(a) Surat (b) Pulicat using English medium of language.
(c) Cochin (d) Cassim Bazar (d) The introduction of scientific research and
Ans. (b) : As Dutch established their first factory at rationalism in the traditional Indian education.
Masulipatam (Andhra Pradesh) in 1605. Their second
Ans. (b) : The Wood's Dispatch was issued, with Sir
factory was established at Pulicat in 1610.
Charles Wood as the President of the Board of Control
21. Consider the following statements regarding
of Education. The main objective of the Dispatch was to
the relations between India and Pakistan:
impart Western knowledge to the Indian people and also
1. During Shimla Agreement, Indira Gandhi
to develop their intellect and moral character.
and Zulfikar Bhutto agreed to maintain
the sanctity of LOC. 24. Which one of the following statements is NOT
2. Lahore Summit took place in the year correct?
1997. (a) Ali Mardan Khan introduced the system of
3. Islamabad Summit was held between Rajiv revenue farming in Bengal.
Gandhi and Nawaz Sharif. (b) Maharaja Ranjit Singh set up modern
Which of these statements is/are correct? foundries to manufacture cannons at Lahore.
(a) 1, 2 and 3 (b) 1 and 3 (c) Sawai Jai Singh of Amber had Euclid's
(c) Only 2 (d) Only 1 'Elements of Geometry' translated into
Ans. (d) : The Lahore Declaration is a bilateral Sanskrit.
agreement and governance treaty between India and (d) Sultan Tipu of Mysore gave money for the
Pakistan. The treaty was signed on February 21, 1999,
construction of the idol of Goddess Sarda in
at the conclusion of a historic summit in Lahore and
the Shringeri temple.
ratified by the parliaments of both countries the same
year. Islamabad Summit was held between Rajiv Ans. (a) : Ali Mardan Khan, who was a Turkish ruler,
Gandhi and Benazir Bhutto. began to behave as an independent ruler in Bengal at the
22. Which one of the following statement is NOT time of Iltutmish. But after two years he was killed by
correct? his soldiers. He did not develop a new system of
(a) The National Song Vande Mataram was revenue in Bengal. Ranjit Singh set up artillery in
composed by Bankimchandra Chhatterji Lahore, which was his administrative capital. Other
originally in Bengali. options of the question are also correct.
(b) The National Calendar of India based on 25. Which one of the following provisions was
Sakaera has its 1st Chaitra on 22nd March NOT made in the Charter Act of 1833?
normally and 21st March in a leap year. (a) The trading activities of the East India
(c) The design of the National Flag of India was Company were to be abolished.
adopted by the Constituent Assembly on 22nd (b) The designation of the supreme authority was
July, 1947. to be changed as the Governor-General of
(d) The song 'Jana-gana-mana', composed India in Council.
originally in Bengali by Rabindranath Tagore (c) All law-making powers to be conferred on
was adopted in its Hindi version by the governor-General in Council.
Constituent Assembly on 24th January, 1950 (d) An Indian was to be appointed as a Law
as the National Anthem of India. Member in Governor-General's Council.
IAS (Pre) GS 2003 Paper I 359 YCT
Ans. (d) : Charter Act of 1833 was the final step 28. When Congress leaders condemned the
towards centralization in British India. This was also Montague-Chelmsford Report, many
called the Saint Helena Act, 1833. It made the moderates left the party to form the
Governor-General of Bengal as the Governor-General (a) Swarajya Party
of India and granted him all civil and military powers. (b) Indian Freedom Party
The Governor-General of India was given exclusive (c) Independence Federation of India
legislative powers for the entire British India. There was (d) Indian Liberal Federation
no provision regarding an Indian to be appointed as
Ans. (d) : On 20 August 1917, the Montague
Law Member in Governor General’s Council. Chelmsford Report was released. On the basis of which
26. With reference to colonial rule in India, what the Government of India Act of 1919 was enacted. The
was sought by the Ilbert Bill in 1883? report was welcomed by some liberals, while the
(a) To bring Indians and Europeans on par as far Congress opposed it. As a result, the Indian National
as the criminal jurisdiction of courts was Liberal Federation was established under the
concerned. chairmanship of Surendranath Banerjee. But indirectly
(b) To impose severe restrictions on the freedom it continued to work under the Congress.
of the native press as it was perceived to be 29. Assertion (A) : In 1916, Maulana Mohammad
hostile to colonial rulers. Ali and Abul Kalam Azad resigned from the
(c) To encourage the native Indians to appear for Legislative Council.
civil service examinations by conducting Reason (R) : The Rowlett Act was passed by
them in India. the Government in spite of being opposed by all
(d) To allow native Indians to possess arms by Indian members of the Legislative Council.
amending the Arms Act. (a) Both A and R are individually true and R is
Ans. (a) : Ilbert Bill, in the history of India, a the correct explanation of A.
controversial measure proposed in 1883 that sought to (b) Both A and R are individually true but R is
allow senior Indian magistrates to preside over cases NOT the correct explanation of A.
involving British subjects in India. The bill, severely (c) A is true but R is false.
weakened by compromise, was enacted by the Indian (d) A is false but R is true.
Legislative Council on Jan. 25, 1884.
Ans. (d) : In 1917, a committee was set up under the
27. An important aspect of the Cripps Mission of chairmanship of Sir Sydney Rowlett to end the
1942 was influence of the revolutionaries of India and identify the
(a) that all Indian states should join the Indian level of revolutionary conspiracy. Based on this, on
Union as a condition to consider any degree March 18, 1919, the Revolutionary and Anarchy Act
of autonomy for India. was passed, which was called Rowlett Act. This law
(b) the creation of an Indian Union with was passed despite opposition from Indian members.
Dominion status very soon after the Second Maulana Mohammad Ali and Abul Kalam Azad
World War. resigned from the Legislative Council during the
(c) the active participation and cooperation of the Khilafat Movement.
Indian people, communities and political
parties in the British was effort as a condition INDIAN GEOGRAPHY
for granting independence with full sovereign 30. The thermal power plant of Bokaro is located
status to India after war. in
(d) the framing of a constitution for the entire (a) Bihar (b) Chhattisgarh
Indian Union, with no separate constitution
for any province, and a Union Constitution to (c) Jharkhand (d) Orissa
be accepted by all provinces. Ans. (c) : Bokaro thermal power plant is located in
Ans. (b) : The Cripps Mission was a failed attempt in Jharkhand, near Konai Dam.
late March 1942 by the British government to secure 31. Which amongst the following States has the
full Indian cooperation and support for their efforts in highest population density as per Census-2001?
World War II. The mission was headed by a senior (a) Kerala (b) Madhya Pradesh
minister Sir Stafford Cripps. Cripps Mission was sent (c) Uttar Pradesh (d) West Bengal
by the British Government in March 1942 to India with Ans. (d) : As per the census 2001, West Bengal had the
key objective to secure Indian cooperation and support highest population density of 904, Kerala had 819,
for British War Efforts. Headed by Sir Stafford Cripps, Madhya Pradesh had 196 and Uttar Pradesh had 689.
this mission sought to negotiate an agreement with But at present, Bihar has the highest population density
Indian leaders. followed by West Bengal.

IAS (Pre) GS 2003 Paper I 360 YCT


32. Assertion (A) : The eastern coast of India 36. Among the following cities, which one is at the
produces more rice than the western coast. highest altitude above mean sea level?
Reason (R) : The eastern coast receives more (a) Bangalore (b) Delhi
rainfall than the western coast. (c) Jodhpur (d) Nagpur
(a) Both A and R are individually true and R is Ans. (a) : Bangalore is at the highest altitude above
mean sea level. Bangalore is the city which is 937
the correct explanation of A.
metres above sea level, Delhi is 218 metres above sea
(b) Both A and R are individually true but R is level, Jodhpur is 230 metres above sea level and Nagpur
NOT the correct explanation of A. is 247.5 metres above the sea level.
(c) A is true but R is false. 37. Consider the following statements:
(d) A is false but R is true. 1. Longitude of Jabalpur's location is between
Ans. (c) : Assertion is true, but reason is false. The those of Indore and Bhopal.
western coast receives more rainfall than eastern coast, 2. Latitude of Aurangabad's location is between
but the temperature between 20-27ºC, heavy clayed soil those of Vadodara and Pune.
quality and rainfall is more suitable for rice cultivation. 3. Bangalore is situated more southward than
Chennai.
33. What is the correct sequence of the rivers-
Which of these statements is/are correct?
Godavari, Mahanadi, Narmada and Tapi in
(a) 1 and 3 (b) Only 2
the descending order of their lengths? (c) 2 and 3 (d) 1, 2 and 3
(a) Godavari - Mahanadi - Narmada - Tapi Ans. (c) : Jabalpur's longitude line (79.95 ° E) is not
(b) Godavari - Narmada - Mahanadi - Tapi between Indore (76 ° E) and Bhopal (77.42 ° E), hence
(c) Narmada - Godavari - Tapi - Mahanadi statement (1) is incorrect. The latitude of Aurangabad
(d) Narmada - Tapi - Godavari - Mahanadi (19 ° 53 '47 "N) lies between the latitudes of Vadodara
(22.30 ° N) and Pune (18 ° 31 ° N). Hence the statement
Ans. (b) : Narmada, Godavari, Tapi and Mahanadi are
(2) is correct. The latitude of Bangalore is 12.97 ° N and
peninsular rivers. the latitudinal of Chennai is 13.04 ° N, so Bangalore is
Narmada and Tapi are west-flowing rivers. more south-facing city than Chennai, hence statement
Godavari and Mahanadi are east flowing rivers. (3) is correct.
The sequence of these rivers in descending order as per 38. Which one among the following States is
their Length smallest in area?
1. Godavari - 1465 KM (a) Andhra Pradesh (b) Gujarat
2. Narmada - 1312 KM (c) Karnataka (d) Tamil Nadu
3. Mahanadi - 851 KM Ans. (d) : Tamil Nadu is the smallest state in area. In
4. Tapi - 724 KM all, Tamil Nadu has 32 districts and out of them,
34. Nanda Devi peak forms a part of Chennai - the capital of the state is the smallest district
(a) Assam Himalayas (by area). Chennai has an area of 426 square km and is
one of the original 13 districts of Tamil Nadu. Despite
(b) Kumaon Himalayas
being the smallest district, it is the most densely
(c) Nepal Himalayas
populated one.
(d) Punjab Himalayas
39. Among the following cities, which one is
Ans. (b) : Nanda Devi peak forms a part of Kumaon nearest to the Tropic of Cancer?
Himalayas located in Chamoli district of Uttaranchal.
(a) Delhi (b) Kolkata
35. Consider the following statements:
(c) Jodhpur (d) Nagpur
1. India is the original home of the cotton
plant. Ans. (b) : Among the following cities, Kolkata is
2. India is the first country in the world to nearest to the Tropic of Cancer. Distance between
develop hybrid cotton variety leading to Kolkata, India and the Tropic of Cancer is 97 km = 61
increased production. miles.
Which of these statements is/are correct? 40. Which one among the following States has the
(a) Only 1 (b) Only 2 highest female literacy rate as per the Census
(c) Both 1 and 2 (d) Neither 1 nor 2 2001?
Ans. (c) : Both the statements are correct. India is the (a) Chhattisgarh (b) Madhya Pradesh
original home of the cotton plant and India is the first (c) Orissa (d) Rajasthan
country in the world to develop hybrid cotton variety Ans. (a) : As per the census of 2001, Chhattisgarh has
leading to increased production. the highest female literacy of 51.85, Orissa has 50.51%.

IAS (Pre) GS 2003 Paper I 361 YCT


WORLD GEOGRAPHY Ans. (d) : A is false and R is true because in tropical
rainforest areas, heavy rainfall occurs throughout the
41. The waterfall 'Victoria' is associated with the
year and there is a lot of leaching of minerals. These
river areas also have high temperature and bacterial activity
(a) Amazon (b) Missouri is enormous which leads to decrease in humus content.
(c) St. Lawrence (d) Zambeji Thus the fertility of soil is reduced and no intensive
Ans. (d) : The waterfall 'Victoria' is associated with the cultivation is possible without application of chemical
river Zambeji. Victoria Falls, spectacular waterfall fertilizers. But due to these geographical conditions,
located about midway along the course of the Zambezi primary productivity of the tropical rain forest is very
high when compared to that of temperate forests.
River, at the border between Zambia to the north and
Zimbabwe to the south. 45. Assertion (A) : Areas lying within five to eight
degrees latitude on either side of the equator
42. Which among the following countries has the
receive rainfall throughout the year.
largest population?
Reason (R) : High temperatures and high
(a) Indonesia (b) Japan
humidity cause convectional rain to fall mostly
(c) Pakistan (d) Sudan
in the afternoons near the equator.
Ans. (a) : Among the given options, Indonesia has the (a) Both A and R are individually true and R is
largest population. the correct explanation of A.
43. Assertion (A) : The amount of moisture in the (b) Both A and R are individually true but R is
atmosphere is related to latitude. NOT the correct explanation of A.
Reason (R) : The capacity to hold moisture in (c) A is true but R is false.
the form of water vapour is related to (d) A is false but R is true.
temperature.
Ans. (a) : Areas lying within 5-8 degree latitude on
(a) Both A and R are individually true and R is either side of equator receive conventional rainfall due
the correct explanation of A. to high temperature and high humidity throughout the
(b) Both A and R are individually true but R is year. High temperatures and high humidity causes
NOT the correct explanation of A. convectional rain to fall mostly in the afternoons near
(c) A is true but R is false. the equator.
(d) A is false but R is true. So, the correct option is 'Both A and R are true but R is
the correct explanation of A'.
Ans. (a) : The wide spread of air around the earth is
called the atmosphere. N2, O2, CO2, etc. as well as 46. Consider the following statements:
water vapor are found in the atmosphere. Due to the 1. In Macedonia, ethnic Albanians are a minority.
extreme temperature and extreme precipitation at the 2. In Kosovo, Serbians are a majority.
equator, the amount of water vapor in the atmosphere is Which of these statements is/are correct?
also very high, while the amount of water vapor (a) Only 1 (b) Only 2
decreases gradually from the equator to the polar
(c) Both 1 and 2 (d) Neither 1 nor 2
regions.
Ans. (b) : In Macedonia, ethnic Albanian population is
44. Assertion (A) : Unlike temperate forests, the
23%, which is a good number in Macedonia. But in
tropical rain forests, if cleared, can yield Kosovo 92% Albanians are present where Serbians are
productive farmland that can support intensive the minority.
agriculture for several years even without
47. Israel has common borders with
chemical fertilizers.
(a) Lebanon, Syria, Jordan and Egypt.
Reason (R) : The primary productivity of the
(b) Lebanon, Syria, Turkey and Jordan.
tropical rain forest is very high when compared
(c) Cyprus. Turkey, Jordan and Egypt.
to that of temperate forests.
(d) Turkey, Syria, Iraq and Yemen.
(a) Both A and R are individually true and R is
Ans. (a) : Israel is located in the Middle East along the
the correct explanation of A.
eastern end of the Mediterranean Sea. It is bound by the
(b) Both A and R are individually true but R is Mediterranean Sea to the west, Lebanon to the north,
NOT the correct explanation of A. Syria to the northeast, Jordan to the east and Egypt to
(c) A is true but R is false. the southwest. The country is divided into three
(d) A is false but R is true. topographical regions.

IAS (Pre) GS 2003 Paper I 362 YCT


48. The Basque people who are frequently in the 2. The first joint sitting of Lok Sabha and
news for their separatist activity live in. Rajya Sabha was held in the year 1961.
(a) France (b) Italy 3. The second joint sitting of the two Houses of
(c) Portugal (d) Spain Indian Parliament was held to pass the
Ans. (d) : The Basque people live in the north central Banking Service Commission (Repeal) Bill.
Spain and frequently in news for their separatist activity
Which of these statements is correct?
in Spain.
(a) 1 and 2 (b) 2 and 3
49. Which one of the following countries is land
locked? (c) 1 and 3 (d) 1, 2 and 3
(a) Bolivia (b) Peru Ans. (d) : The joint sitting of the Parliament is called by
(c) Suriname (d) Uruguay the President (Article 108) and is presided over by the
Ans. (a) : The countries which are surrounded by land Speaker or, in his absence, by the Deputy Speaker of the
from four directions are called landlocked countries. Lok Sabha or in his absence, the Deputy-Chairman of the
Bolivia is a landlocked country in South America. Rajya Sabha. The first joint sitting was held on 6 May
Bolivia is surrounded by Peru, Brazil, Paraguay, 1961 following a disagreement between the two Houses
Argentina and Chile. Brazil is the largest country in over certain amendments to the Dowry Prohibition Bill,
South America in terms of area and French Guyana is
1959. This was followed by another sitting on 9 May
the smallest country. Kauru is the launch site in French
Guiana. 1961 when the Bill, as amended, was finally passed.
Banking service commission repeal act, 1978.
50. Which one of the following countries does NOT
border the Caspian Sea? 54. Under which Article of the Indian Constitution
(a) Armenia (b) Azerbaijan did the President give his assent to the
(c) Kazakhstan (d) Turkmenistan ordinance on electoral reforms when it was
Ans. (a) : Russia Azerbaijan, Tajikistan Iran and sent back to him by the Union Cabinet without
Kazakhstan are bordered by the Caspian Sea. making any changes (in the year 2002)?
The border countries of Armenia - Azerbaijan, Georgia, (a) Article 121 (b) Article 122
Turkey. (c) Article 123 (d) Article 124
51. Among the following which planet takes Ans. (c) : Article 123 of the Constitution of India grants
maximum time for one revolution around the power to the President to promulgate ordinances when
Sun? either of the two Houses of Parliament is not in session.
(a) Earth (b) Jupiter
(c) Mars (d) Venus 55. Which one of the following statements is
correct?
Ans. (b) : Jupiter takes maximum time for one
(a) Only the Rajya Sabha and not the Lok Sabha
revolution around the sun.
can have nominated members.
(b) There is a constitutional provision for nominating
INDIAN CONSTITUION & POLITY two members belonging to the Anglo-Indian
52. The Ninth Schedule to the Indian Constitution community to the Rajya Sabha.
was added by (c) There is no constitutional bar for a nominated
member to be appointed as a Union Minister.
(a) First Amendment
(d) A nominated member can vote both in the
(b) Eighth Amendment Presidential and Vice Presidential elections.
(c) Ninth Amendment Ans. (c) : Nominated members enjoy all powers,
(d) Forty Second Amendment privileges and immunities available to an elected
Ans. (a) : First constitutional amendment, 1951, member of Parliament. They take part in the
included Ninth Schedule in constitution of India, which proceedings of the House as any other member. They
provided that by incorporating any law into it, the state are not entitled to vote in the election of the President of
would make it immune from Judicial scrutiny. It India. But in the election of the Vice-President of
provided for validation of certain acts and regulations, India, they have a right to vote. There is no
constitutional bar for a nominated member to be
mostly relating to land reform.
appointed as a Union Minister.
53. Consider the following statements:
56. The power to enlarge the jurisdiction of the
1. The joint sitting of the two houses of the Supreme Court of India with respect to any
Parliament in India is sanctioned under matter included in the Union List of Legislative
Article 108 of the Constitution. Powers rests with
IAS (Pre) GS 2003 Paper I 363 YCT
(a) The President of India. it. The Speaker is empowered to appoint the Chairman
(b) The Chief Justice of India. of the Committee from amongst its members. The
(c) The Parliament. Committee on Public Accounts is constituted by the
Parliament each year. Estimates Committee is a
(d) The Union Ministry of Law, Justice and
parliamentary committee consisting of 30 members
Company Affairs. elected every year by the Lok Sabha from amongst its
Ans. (c) : The power to enlarge the jurisdiction of the members to examine the Budget estimates of the Union
Supreme Court of India with respect to any matter Government. No minister shall be appointed as a
included in the Union List of Legislative Powers rests member of Estimates Committee. Ministry of
with the Parliament. Parliamentary affairs is entrusted with the responsibility
57. Which one of the following High Courts has the of coordinating, planning and arranging Government
Territorial Jurisdiction over Andaman and business in both the Houses of Parliament. For this
Nicobar Islands? purpose, the ministry works under the overall direction
of Cabinet Committee on Parliamentary Affairs. The
(a) Andhra Pradesh (b) Calcutta
Minister of Parliamentary Affairs nominates Members
(c) Madras (d) Orissa of Parliament on committees, councils, boards and
Ans. (b) : The Calcutta High Court is the oldest High commissions etc set up by the Government in various
Court in India. It has jurisdiction over the state of West ministries. Therefore all the statements are correct.
Bengal and the Union Territory of the Andaman and 60. As per Indian Protocol, who among the
Nicobar Islands. following ranks highest in the order of
58. Under which Article of the Indian Constitution precedence?
did the President make a reference to the (a) Deputy Prime Minister
Supreme Court to seek the Court's opinion on the (b) Former President
constitutional validity of the Election (c) Governor of a State within his State
Commission's decision on deferring the Gujarat (d) Speaker of Lok Sabha
Assembly elections (in the year 2002)? Ans. (c) : Order of precedence is President, Vice
(a) Article 142 (b) Article 143 President, PM, Governor of State within their respective
(c) Article 144 (d) Article 145 states, Former Presidents and Deputy PM, Chief Justice
of India and Speaker of LS.
Ans. (b) : Article 143 of the Constitution is regarding
power of the President to consult the Supreme Court 61. Which one of the following Articles of the
related to a matter of public importance. Indian Constitution provides that 'It shall be
the duty of the Union to protect every State
59. Consider the following statements: against external aggression and internal
1. While members of the Rajya Sabha are disturbance'?
associated with committees on Public (a) Article 215 (b) Article 275
Accounts and Public Undertakings (c) Article 325 (d) Article 355
members of committee on Estimates are
Ans. (d) : If president is satisfied that grave emergency
drawn entirely from Lok Sabha.
exists whereby the security of India or of any part of the
2. The Ministry of Parliamentary Affairs territory thereof is threatened or whether by war or
works under the overall direction of external aggression or armed rebellion, he may by
Cabinet Committee on Parliamentary proclaimation made a declaration to that effect in respect
Affairs. of the whole of India or of such part of territory thereof as
3. The Minister of Parliamentary Affairs may be specified in the proclamation.
nominates Members of Parliament on
62. Match List I with List II and select the correct
Committees, Councils, Board and
Commissions etc. set up by the Government answer using the codes given below the lists:
of India in the various ministries. List I List II
Which of these statements are correct? (Item in the India (Country
Constitution) from which)
(a) 1 and 2 (b) 2 and 3
A. Directive Principles 1. Australia
(c) 1 and 3 (d) 1, 2 and 3
of State Policy
Ans. (d) : The Committee on Public Accounts is the B. Fundamental Rights 2. Canada
oldest parliamentary committee and was first
constituted in 1921. The committee consists of 22 C. Concurrent List in 3. Ireland
members, 15 members are elected from Lok Sabha and Union-State
Relations
7 members of the Rajya Sabha are associated with
IAS (Pre) GS 2003 Paper I 364 YCT
D. India as a Union of 4. United Ans. (b) : Article 280 of the Constitution of India
States with greater Kingdom provides for a Finance Commission as a quasi-judicial
powers to the Union body. It is constituted by the President of India every
5. United fifth year or at such earlier time as he considers
States of necessary. The commission makes recommendations to
the president with regard to the distribution of the
America
proceeds of taxes between the union and the states.
A B C D Finance Commission lays down the principle which
(a) 5 4 1 2 should govern the grants-in-aid to be given to the states.
(b) 3 5 2 1 65. Consider the following statements:
(c) 5 4 2 1 In the electoral college for Presidential Election
(d) 3 5 1 2 in India,
Ans. (d) : Borrowed features from different countries 1. the value of the vote of an elected Member
are correct. of Legislative Assembly equals
Directive Principle of State State Population
Ireland
Policy Number of Elected MLAs of the State × 100
2. the value of the vote of an elected Member
Fundamental Rights USA
of Parliament equals
Concurrent List of Union- Australia Total Value of the votes of all elected MLAs
State relations
Total Number of elected MPs
India as Union of States with Canada 3. there were more than 5000 members in the
greater powers to center latest elections.
63. Which of the following Bills must be passed by Which of these statements is/are correct?
each House of the Indian Parliament (a) 1 and 2 (b) Only 2
separately, by special majority? (c) 1 and 3 (d) Only 3
(a) Ordinary Bill Ans. (b) : Statement 1 is not correct because there
(b) Money Bill should be 1000 instead of 100.
(c) Finance Bill Statement 2 is correct. The President is indirectly
(d) Constitution Amendment Bill elected by means of a electoral college consisting the
Ans. (d) : According to Article 368, an amendment of elected members of the Parliament of India and the
Legislative Assemblies of the states and the Union
this Constitution may be initiated only by the
Territories of Delhi and Puducherry.
introduction of a Bill for the purpose in either House of
Statement 3 is incorrect. Presently 776 MPs and 4120
Parliament, and when the Bill is passed in each House
members of Legislative Assemblies participated as
by a majority of the total membership of that House
electors in Presidential election 2017. So their total
present and voting, it shall be presented to the President number is 4896 Therefore (b) is the correct option.
who shall give his assent to the Bill and thereupon the
66. Which of the following Constitutional
Constitution shall stand amended in accordance with the
Amendments are related to raising the number
terms of the Bill.
of Members of Lok Sabha to be elected from
64. Consider the following statements: the States?
The function (s) of the Finance Commission (a) 6th and 22nd (b) 13th and 38th
is/are st
(c) 7th and 31 (d) 11th and 42nd
1. to allow the withdrawal of the money out
of the Consolidated Fund of India. Ans. (c) : The Amendment Act 1956 provided for
2. to allocate between the States the shares of composition of the House of the people an readjustment
proceeds of taxes. after every census. Accordingly, the seats in the central
3. to consider applications for grants-in-aid and state legislatures were rearranged. The number of
from States. Lok Sabha members was increased from 525 to 545 and
4. to supervise and report on whether the the representation of Union Territories was reduced
Union and State governments are levying from 25 to 20 by the 31st Amendment.
taxes in accordance with the budgetary 67. Survey of India is under the ministry of:
provisions. (a) Defence
Which of these statements is/are correct? (b) Environment & Forests
(a) Only 1 (b) 2 and 3 (c) Home Affairs
(c) 3 and 4 (d) 1, 2 and 4 (d) Science & Technology
IAS (Pre) GS 2003 Paper I 365 YCT
Ans. (d) : Survey of India, the National Survey and Reason (R) : During the year 2001-02, negative
Mapping Organization of the country under the growth in exports was witnessed in respect of
Department of Science and Technology, is the oldest iron and steel, coffee, textiles and marine
scientific department of the Govt. of India. It was set up products.
in 1767.
(a) Both A and R are individually true and R is
68. Which one of the following schedules of the
the correct explanation of A.
Indian Constitution lists the names of states
and specifies their territories? (b) Both A and R are individually true but R is
(a) First (b) Second NOT the correct explanation of A.
(c) Third (d) Fourth (c) A is true but R is false.
Ans. (a) : The 1st Schedule of the Constitution mentions (d) A is false but R is true.
the list of the states and union territories of the Union of Ans. (d) : During the year 2001-02, there was a decline
India. Presently, there are 28 states and 8 Union of 1.6% and not 2.17% in Indian exports. Whereas
territories in the country. reason is correct.
73. Assertion (A) : The new EXIM policy is liberal,
ECONOMY market-oriented and favours global trade.
69. Which one among the following countries has Reason (R) : GATT has played a significant role
the lowest GDP per capita? in the liberalization of the economy.
(a) China (b) India (a) Both A and R are individually true and R is
(c) Indonesia (d) Sri Lanka the correct explanation of A.
Ans. (b) : India among the given countries in option has (b) Both A and R are individually true but R is
the lowest GDP per capita. GDP per capita is as NOT the correct explanation of A.
follows-India 1509 USD, China 6959 USD, Sri Lanka (c) A is true but R is false.
3204 USD, Indonesia 3510 USD. (d) A is false but R is true.
70. Among the following commodities imported by Ans. (b) : The new Export-Import Policy (EXIM
India during the year 2000-01, which one was POLICY) is liberal and market oriented and suited to
the highest in terms of Rupee value? global trade and the GATT (General Agreement on
(a) Edible oil Tariff and Trade) agreement has played an important
role in the liberalization of the economy. Hence both
(b) Fertilizers
the assertion and the reason are correct. But the reason
(c) Organic and inorganic chemicals is not the correct explanation of the assertion.
(d) Pearls, precious and semi-precious stones
74. Consider the following statements:
Ans. (d) : Pearls, precious and semi-precious stones
1. In the last five years, Indian software exports
comprises approx 8.3% of the total were the
have increased at a compound annual growth
commodities imported which had highest rupee value.
rate of about 60%.
71. Consider the following statements
2. The software and service industry in India
1. The maximum limit of shareholding of India registered an overall growth of about 28% in
promoters in private sector banks in India is rupee terms during the year 2001-2000.
49 percent of the pad up capital. Which of these statements is/are correct?
2. Foreign Direct Investment upto 49 percent (a) Only 1 (b) Only 2
from all sources is permitted in private sector (c) Both 1 and 2 (d) Neither 1 nor 2
banks in India under the automatic route. Ans. (b) : Software exports had a compound annual
Which of these statements is/are correct? growth rate of 41% in the five-year period from 1997-
(a) Only 1 (b) Only 2 98 to 2002-03. Not 60%, so statement 1 is false. The
(c) Both 1 and 2 (d) Neither 1 nor 2 total rupee equivalent growth in the software and
services industry in India was around 28% in the year
Ans. (d) : Currently, FDI in private sector banks is
2001-02. Hence statement 2 is correct.
raised to 74% under the automatic route including
investment by FIIs. 75. Which one of the following statements is NOT
72. Assertion (A) : During the year 2001-02, the correct?
value of India's total exports declined, (a) India is the second largest producer of
registering a negative growth of 2.17% nitrogenous fertilizers in the world.

IAS (Pre) GS 2003 Paper I 366 YCT


(b) India is the ninth largest steel producing (a) Cement (b) Coal
country in the world. (c) Electricity (d) Steel
(c) India is the second largest producer of silk in Ans. (d) : The steel industry recorded highest growth
the world. rate of 7%, electricity generation growth rate was 4%,
(d) India ranks third in the world in coal coal industry growth was 3.3%.
production. 80. Debenture holder of a company are its
(a) Shareholders (b) Creditors
Ans. (a) : India is the second largest importer of
nitrogenous fertilizers in the world not the producer. (c) Debtors (d) Directors
Hence first statement is not correct. Rest of the Ans. (b) : Shares, debentures etc. are the main sources
statements is correct. of capital raising of any company. Share means a
shareholder of a company and is a part of the company.
76. With reference to Government of India's They are directly affected by the profit and loss of the
decisions regarding Foreign Direct Investment company. The company issues debentures for the
(FDI) during the year 2001-02, consider the requirement of capital. In a way, the company takes
following statements: loans. It is issued on a fixed term and at a fixed rate of
1. Out of the 100% FDI allowed by India in tea interest. The amount is refunded by the company on
sector, the foreign firm would have to completion of time.
disinvest 33% of he equity in favour of an 81. Consider the following statements:
Indian partner within four years. In India stamp duties on financial transaction
are :
2. Regarding the FDI in print media in India,
1. Levied and collected by the State Government.
the single largest Indian shareholders should
2. Appropriated by the Union Government.
have a holding higher than 26%. Which of these statements is/are correct?
Which of these statements is/are correct? (a) Only 1 (b) Only 2
(a) Only 1 (b) Only 2 (c) Both 1 and 2 (d) Neither 1 nor 2
(c) Both 1 and 2 (d) Neither 1 nor Ans. (d) : Stamp duty is a form of tax that is levied on
Ans. (c) : Both the statements are correct. documents. It is levied by the union or the central
government and collected and appropriated by the state
77. In India, the first bank of limited liability governments. (Entry 91 in the union list; entry 63 in the
managed by Indians and founded in 1881 was state list of Seventh Schedule of Constitution of India).
(a) Hindustan Commercial Bank 82. Which one of the following statements is
(b) Oudh Commercial Bank correct with reference to FEMA in India?
(c) Punjab National Bank (a) The Foreign Exchange Regulating Act
(d) Punjab and Sind Bank (FERA) was replaced by Foreign Exchange
Management Act (FEMA) in the year 2001.
Ans. (b) : The first bank in India was established in
(b) FERA was given a sunset clause of one year
1806 under the name Bank of Bengal. The first till 31st May, 2002 to enable Enforcement
commercial bank in India was established in 1881. It Directorate to complete the investigation of
was named Awadh or Oudh Commercial Bank. The pending issues.
Imperial Bank of India was established in 1921 and it (c) Under FEMA, violation of foreign exchange
was nationalized and renamed SBI in 1955 on the rules has ceased to be a criminal offence.
recommendation of the Gorwala Committee. (d) As per the new dispensation, Enforcement
78 The Government holding in BPCL is Directorate can arrest and prosecute the
(a) more than 70% people for the violation of foreign exchange
(b) between 60% and 70% rules.
(c) between 50% and 60% Ans. (b) : FERA was for two years until May 31, 2002.
(d) less than 50% A fixed time (sunset) clause was given so that the
Enforcement Directorate completes the investigation of
Ans. (b) : Government holds stake of 66.21% in BPCL. undecided subjects. The remaining statements a, c and
Last year, the government had decided to disinvest its d are incorrect.
10% stake but this was not possible due to political
83. Consider the following statements:
deadlock.
1. India's import of crude and petroleum
79. During the year 2002-01, which one of the products during the year 2001-02
following industries recorded highest growth accounted for about 27% of India's total
rate in India? imports.
IAS (Pre) GS 2003 Paper I 367 YCT
2. During the year 2001-02, India's exports 1
had increased by 10% as compared to the ⇒ gα
R2
previous year. ∆g ∆R
Which of these statements is/are correct? = −2 = −2( −0.01) = 0.02
g R
(a) Only 1 (b) Only 2
(c) Both 1 and 2 (d) Neither 1 nor 2 ∆g = 0.02g
Ans. (a) : India's import of crude and petroleum So, the value of g increases by 2 percent.
products during the year 2001-02 accounted for about 87. A hydrogen-inflated polythene balloon is released
27% of India's total imports. Hence first statement is from the surface of the earth. As the balloon rises
correct while during the year 2001-02, India's exports to an altitude up in the atmosphere, it will:
had increased nearly by 20% as compared to the
(a) Decrease in size
previous year. Hence second statement is not correct.
(b) Flatten into a disc-like shape
84. Consider the following statements:
1. India ranks first in the world in fruit (c) Increase in size
production. (d) Maintain the same size and shape
2. India ranks second in the world in the Ans. (c) : As a weather balloon rises into high altitudes,
export of tobacco. where air pressure decreases, the helium or hydrogen
Which of these statements is/are correct? pressure inside the balloon increases and expands the
(a) Only 1 (b) Only 2 balloon.
(c) Both 1 and 2 (d) Neither 1 nor 2 88.Consider the following statements:
Ans. (d) : India ranked 2nd in the fruit production in the 1. Steam at 100ºC and boiling water at 100ºC
world and ranked 6th in export of tobacco. contain same amount of heat.
2. Latent heat of fusion of ice is equal to the
PHYSICS latent heat of vaporization of water.
3. In an air-conditioner, heat is extracted
85. Assertion (A) : The weight of a body decreases from the room-air at the evaporator coils
with the increase of latitude on earth. and is rejected out at the condenser coils.
Reason (R) : The earth is not a perfect sphere. Which of these statements is/are correct?
(a) Both A and R are individually true and R is (a) 1 and 2 (b) 2 and 3
the correct explanation of A. (c) Only 2 (d) Only 3
(b) Both A and R are individually true but R is Ans. (d) : The heat required to change the state of
NOT the correct explanation of A. matter at constant temperature is called latent heat. The
(c) A is true but R is false. latent heat of melting for ice is 80 calories/gram. The
(d) A is false but R is true. latent heat value of evaporation for water is 540
calories/g. Burning with steam is more painful than
Ans. (d) : The weight of the body has nothing to do boiling water because the secret heat of steam is more
with the latitude of the earth. We experience the weight than that of water.
of any object with the gravitational acceleration (g)
present on the earth, irrespective of the mass of the 89. Consider the following statements: A 4-wheel
object. Without gravitational acceleration, we vehicle moving a sharp circular path at high
experience weightlessness. This is why we find a state speed will
of weightlessness in space. The value of gravitational 1. Overturn about its outer wheels
acceleration increases when the equator moves from the 2. Overturn about its inner wheels
equator to the poles. 3. Skid outward
86. If the radius of the earth were to shrink by one 4. Skid inwards
percent, its mass remaining the same, the value Which of these statements are correct?
of 'g' on the earth's surface world. (a) 1 and 3 (b) 2 and 4
(a) Increase by 0.5% (b) Increase by 2% (c) 2 and 3 (d) 1 and 4
(c) Decrease by 0.5% (d) Decrease by 2% Ans. (c) : In case of object moving along a circular
path, two types of forces namely centripetal and
Ans. (b) : The value of ‘g’ or the acceleration due to
centrifugal acts upon it. As they have opposite impact,
gravity is given by thus the object remains in balance. When centrifugal
GM force gets disturbed then the object will leave the
g= 2
R balance and skid towards an outward direction.

IAS (Pre) GS 2003 Paper I 368 YCT


90. Diffusion of light in the atmosphere takes place Ans. (a) : Ionic compound is a chemical compound in
due to which ions are held together in a lattice structure by
(a) Carbon dioxide ionic bonds. Ionic compounds dissolve in polar
(b) Dust particles solvents, especially those that ionize, such as water and
(c) Helium ionic liquids. They are usually appreciably soluble in
(d) Water vapour other polar solvents such as alcohols, acetone. Solid
Ans. (b) : When a ray of light enters a medium where ionic compounds cannot conduct electricity because
there is a multiplicity of dust and other microscopic there are no mobile ions or electrons present in the
particles, the ray of light spreads all around. This is lattice.
called scattering or diffusion of light. For this reason, 94. Which one of the following statements is
the color of the sky appears blue. correct?
91. An oil tanker is partially filled with oil and (a) The presence of NaCl increases the rate of
moves forward on a level road with uniform setting of Plaster of Paris.
acceleration. The free surface of oil then (b) Gypsum is added to the cement to slow down
(a) remains horizontal. its rate of setting.
(b) is inclined to the horizontal with smaller (c) All alkaline earth metals from hydrated salts.
depth at the rear end.
(d) Barium and Strontium are found free in
(c) is inclined to the horizontal with larger depth
nature.
at the rear end.
(d) Assumes parabolic curve. Ans. (d) : Strontium is a relatively abundant element in
the Earth's crust. It is as abundant as fluorine and its
Ans. (c) : The surface of tanker will move in forward alkaline earth partner, barium. The most common
direction with increasing velocity (as acceleration in minerals containing strontium are celestine and
uniform), so depth will be larger at the rear end. strontianite.
92. Which one of the following statements is NOT 95. Regarding the atom of a chemical element, the
correct? magnetic quantum number refers to
(a) The velocity of sound in air increases with the (a) orientation (b) shape
increase of temperature. (c) size (d) spin
(b) The velocity of sound in air is independent of
Ans. (a) : Magnetic quantum number represents the
pressure.
number of orbitals present in the sub-shell magnetic
(c) The velocity of sound in air decreases as the
humidity increases. quantum number about the orientation of the orbital.
(d) The velocity of sound in air is not affected by An electron due to its orbital motion around the nucleus
the change in amplitude and frequency. generates an electric field. This electric field generates
Ans. (c) : The speed of sound in air directly proportional to magnetic field which in turn interacts with external
square root of temperature as Cair ∝ T magnetic field. Thus under the influence of external
Here Cair = speed of sound in air magnetic field, electrons of a sub shell can orient
T = temperature of medium themselves in a certain preferred region of space around
Therefore, as humidity increases speed of sound also the nucleus called orbital. The magnetic quantum
increases. number determines the number of preferred orientation
Hence option (c) is not correct statement. of electron present in a sub shell.

CHEMISTRY 96. Assertion (A) : In the periodic table of chemical


elements, electron affinity is always found to
93. With reference to ionic compounds, consider increase from top to bottom in a group.
the following statements: Reason (R) : In a group, the atomic radii
1. Ionic compounds are insoluble in alcohol. generally increase from top to bottom.
(a) Both A and R are individually true and R is
2. Ionic compounds in the solid state are good
the correct explanation of A.
conductors of electricity.
(b) Both A and R are individually true but R is
Which of these statements is/are correct? NOT the correct explanation of A.
(a) Only 1 (b) Only 2 (c) A is true but R is false.
(c) Both 1 and 2 (d) Neither 1 nor 2 (d) A is false but R is true.
IAS (Pre) GS 2003 Paper I 369 YCT
Ans. (d) : A trend of decreasing electron affinity is BIOLOGY
found by going down the groups in the periodic table.
The additional electron will be entering in an orbital 100. Octopus is
farther away from the nucleus and thus would (a) an arthropod (b) an echinoderm
experience a lesser effective nuclear charge. It is fairly (c) a hemichordate (d) a mollusc
obvious that the atoms get bigger by going down group Ans. (d) : Octopus is an animal of class-Cephalopoda
with the increase in atomic number. The reason is and phylum Mollusca. The shell is absent. It is found at
equally obvious, addition of extra layers of electrons. bottom of the sea. It kills its prey with poisonous saliva.
97. Assertion (A) : Coal-based thermal power It can change its colour.
stations contribute to acid-rain. 101. Consider the following conditions of a sick
Reason (R) : Oxides of carbon are emitted human body:
when coal burns.
1. Swollen lymph nodes
(a) Both A and R are individually true and R is
the correct explanation of A. 2. Sweating at night
(b) Both A and R are individually true but R is 3. Loss of memory
NOT the correct explanation of A. 4. Loss of weight
(c) A is true but R is false. Which of these are symptoms of AIDS?
(d) A is false but R is true. (a) 1 and 2 (b) 2, 3 and 4
Ans. (b) : Coal based thermal power plants contribute (c) 1, 3 and 4 (d) 1, 2, 3 and 4
to acid rain because SO2 and NO2 are emitted from Ans. (d) : AIDS is a viral disease. In this disease the
these plants which form H2SO4 and HNO3 in immune system weakens and the body becomes prone
atmosphere, that cause acid rain. Oxides of carbon are to different type of infection by pathogens. Sweating at
emitted when coal burns, but it does not contribute to night, swollen lymph nodes, loss of memory and loss of
acid rain. weight are also symptoms of AIDS.
98. Which one of the following statements is 102. Consider the following statements:
correct? 1. The common blue green algae, Spirogyra and
(a) Liquid Sodium is employed as a coolant in Ulothrix are found in both fresh water ponds
nuclear reactors. and oceans.
(b) Calcium carbonate is an ingredient of tooth 2. The chameleon can look ahead with one eye,
paste. and at the same time look behind with
(c) Bordeaux mixture consists of Sodium another.
sulphate and lime. Which of these statements is/are correct?
(d) Zinc amalgams are used as a dental filling. Which of these statements is/are correct?
Ans. (b) : Toothpaste usually contains 20–42% water. (a) Only 1 (b) Only 2
The main ingredients of toothpaste are abrasive (c) Both 1 and 2 (d) Neither 1 nor 2
Chloride and humid. Generally, the amount of abrasive Ans. (b) : Spirogyra and Ulothrix are green algae not
in toothpaste is up to 50%. Ablative contains insoluble blue algae. The chamelean can see in both the direction
particles of aluminum hydroxide (Al (OH) 3) and forward and backward at the same time.
calcium carbonate (CaCo3). 103. Assertion (A) : Unsaturated fats are more reactive
99. Consider the following statements: compared with the saturated fats.
The purpose of adding sodium sulphate and Reason (R) : Unsaturated fats have only single
sodium silicate to the detergent in a washing bonds in their structure.
powder is (a) Both A and R are individually true and R is
the correct explanation of A.
1. to keep the washing powder dry
2. to maintain the alkalinity of the powder (b) Both A and R are individually true but R is
NOT the correct explanation of A.
Which of these statements is/are correct?
(a) Only 1 (b) Only 2 (c) A is true but R is false.
(c) Both 1 and 2 (d) Neither 1 nor 2 (d) A is false but R is true.
Ans. (c) : The addition of silicates to synthetic Ans. (c) : Unsaturated fats are more reactive due to the
detergents has proved very beneficial. Silicates soften presence of the multiple bonds between carbon atoms.
water by the formation of precipitates that can be easily They undergo a chemical reaction (like hydrolysis,
rinsed away. Soluble silicates contribute to detergents as hydrogenation etc.) easily due to the presence of
cleaning aids, processing aids, and corrosion inhibitors. multiple bonds. Hence, the unsaturated fats are more
As cleaning aids, soluble silicates provide alkalinity and reactive compared with the saturated fats
promote soil suspension. Sodium sulphate is simply a Unsaturated fats contain one or more double bonds so
filler in detergent powder, in much the same way as these molecules are said to be unsaturated.
fillers are used in many medical tablets, it doesn't play So, the correct answer is 'Assertion is correct but
any active role as such. Reason is incorrect'.

IAS (Pre) GS 2003 Paper I 370 YCT


104. With reference to the work of human kidney, ENVIRONMENT & ECOLOGY
consider the following statements:
1. After the waste is removed in the kidney, 107. Consider the following statements:
the cleaner blood is sent back through 1. Molasses is a by-product of sugar production
renal artery. process.
2. From Bowman's capsule, the filtered liquid 2. Bagasse obtained in the sugar mills is used
passes through tiny tubes where much of as a fuel in the boilers to generate steam
the glucose is reabsorbed and sent back to for the sugar factories.
the blood in the renal vein. 3. Sugar can only be produced from sugarcane
Which of these statements is/are correct? as the raw material.
Which of these statements are correct?
(a) Only 1 (b) Only 2 (a) 1 and 2 (b) 2 and 3
(c) Both 1 and 2 (d) Neither 1 nor 2 (c) 1 and 3 (d) 1, 2 and 3
Ans. (b) : A nephron consists of a network of tiny blood Ans. (a) : To make molasses, sugar cane is harvested
vessels, the glomerulus surrounded by Bowman's and stripped of leaves. Its juice is extracted, usually by
cutting, crushing, or mashing. The juice is boiled to
capsule. Pressure forces much of the blood plasma concentrate it, promoting sugar crystallization.
through the glomerulus and into Bowman's capsule. The For every 100 tons of Sugarcane crushed, a Sugar
resulting tubular fluid, which contains water and factory produces nearly 30 tons of wet Bagasse.
dissolved chemicals then passes into convoluted tubule Bagasse is often used as a primary fuel source for Sugar
mills; when burned in quantity, it produces sufficient
and loop of Henle. This capillary network provides a heat and electrical energy to supply all the needs of a
mean for pickup and delivery of materials that are typical Sugar mill, with energy to spare.
reabsorbed and secreted by the kidney. Sugar could be produced from sugarcane and sugar
beet, Out of 120 nations, nearly 65 nations produce
105. Which one of the following statements is not
from sugarcane, nearly 40 are from sugar beet and 10
correct? are from both.
(a) Milk contains none of the B-vitamins. 108. Which one among the following covers the
(b) Vitamin-A (retinol) deficiency leads to dry highest percentage of forest area in the world?
and scaly skin. (a) Temperate Coniferous forests
(b) Temperate Deciduous forests
(c) One of the symptoms of scurvy is pain in the
(c) Tropical Monsoon forests
joints. (d) Tropical Rain forests
(d) Vitamin B1 (thiamine) deficiency can lead to Ans. (a) : Temperate Coniferous forests cover the
heart failure. highest percentage of forest cover in the world.
Ans. (a) : Due to absence of vitamin A (retinol), the Temperate coniferous forest is a terrestrial biome found
in temperate regions of the world with warm summers
skin becomes dry and night blindness occurs. Vitamins
and cool winters and adequate rainfall to sustain a
found in milk are - vitamin A, B2, B3, B7, B12, D, E. forest.
106. With reference to normal human beings, 109. Consider the following animals of India:
consider the following statements: 1. Crocodile 2. Elephant
1. In response to the presence of HCI, Which of these is/are endangered species?
(a) Only 1 (b) Only 2
secretin is produced from the duodenum.
(c) Both 1 and 2 (d) Neither 1 nor 2
2. Enterogastrone is produced in the small Ans. (c) : Crocodile and Elephant both are endangered
intestine in response to the presence of species.
fatty acids. 110. Which one of the following is produced during
Which of these statements is/are correct? the formation of photochemical smog?
(a) Only 1 (b) Only 2 (a) Hydrocarbons
(c) Both 1 and 2 (d) Neither 1 nor 2 (b) Nitrogen Oxides
(c) Ozone
Ans. (c) : When the food reaches the stomach HCl is (d) Methane
recreated, it provide an acidic medium for pepsin to Ans. (b) : photochemical smog formed by a complex
work, and kills most of the bacterial present in the food. series of chemical reactions involving sunlight oxide of
When the food enters the duodenum (small intestine), nitrogen and volatile organic compounds that are
recretin is produced by intestine juice. present in atmosphere as a result for air pollution.
Note : During the photochemical smog no and O3 both
Also Enterogastrone is produced in small intestine. but major production NO2 (Nitrogen Oxides).
IAS (Pre) GS 2003 Paper I 371 YCT
CURRENT Ans. (c) : Barry Richards was a famous batsmen of
South Africa, Tex Dexter was hard hitting batsmen of
111. With reference to India, which one of the England, Alan Davidson was a renowned wicket keeper
following statements is NOT correct? of Australia and Charlie Griffth was a fast bowler of
(a) IPCL is India's largest petrochemical West Indies.
company. 115. Which one among the following has the largest
(b) RIL is the largest private sector company in shipyard in India?
India. (a) Kolkata (b) Mumbai
(c) MTNL is listed on NYSE. (c) Kochi (d) Visakhapatnam
(d) BSNL is the first telecom service Ans. (c) : Cochin Shipyard Limited (Kochi) (CSL) is
organization in India to launch as nationwide the largest shipbuilding and maintenance facility in
cellular service at one time. India. It is part of a line of maritime-related facilities in
Ans. (a) : Jamnagar in Gujarat is the largest petrochemical the port-city of Kochi, Kerala. Services provided by the
industrial center of India. The city is famous for the shipyard are building platform supply vessels and
petrochemical industry due to the presence of Reliance Ltd double-hulled oil tankers.
which is the biggest petrochemical industry in India at 116. Match List I with List II and select the correct
present. RIL is India's largest private sector company with answer using the codes given below the lists:
a leadership position in the petrochemicals industry List-I List-II
while IPCL (Indian Petrochemical Corporation Limited) (Organisation) (Location)
is India's second largest company in that sector. A. National Sugar 1. Dehradun
112. The Kelkar proposals which were in the news Institute
recently were the: B. Mishra Dhatu 2. Kamptee
(a) Recommendations for reform in the power Nigam Ltd.
sector. C. Institute of Military 3. Pune
(b) Recommendations for tax reforms. Law
(c) Guidelines for the privatization of public D. Institute of National 4. Hyderabad
sector undertakings. Integration 5. Kanpur
(d) Guidelines for reducing vehicular pollution, A B C D
and the promotion of CNG use. (a) 3 1 4 2
Ans. (b) : The Vijay Kelkar Committee on Direct Taxes (b) 5 1 4 3
first submitted its report in year 2003. (c) 3 4 5 2
113. Dilip Kaur Tiwana is well known as a (d) 5 4 2 3
(a) Literateur (b) Musician Ans. (d) : Correct match of List I and List II is –
(c) Scientist (d) Sportsperson List I List II
Ans. (a) : Dr. Dalip Kaur Tiwana is a foremost novelist National Sugar Institute - Kanpur
and short-story writer of contemporary Punjabi Mishra Dhatu Nigam Ltd. - Hyderabad
literature. Institute of Military Law - Kamptee
114. Match List I with List II and select the correct Institute of National Integration - Pune
answer using the codes given below the lists: 117. Tim Montgomery set a new world record at the
List I List II IAAF Grand Prix final in the year 2002 in.
(Cricketers) (Country) (a) Long Jump (b) 110 m Hurdle Race
(c) 100 m Dash (d) High Jump
A. Barry Richards 1. England
Ans. (c) : Tim Montgomery set a new world record at
B. Tex Dexter 2. West Indies
the IAAF Grand Prix Final in the year 2002 in 100 m
C. Alan Davidson 3. South Africa Dash.
D. Charlie Griffith 4. New Zealand 118. Consider the following statements:
5. Australia 1. India launched its first full-fledged
A B C D meteorological satellite (METSAT) in
(a) 3 2 5 1 September, 2002.
(b) 5 1 4 2 2. For the first time, the space vehicle PSLV-
(c) 3 1 5 2 C4 carried a payload of more than 1000 kg
(d) 5 2 4 1 into a geosynchronous orbit.
IAS (Pre) GS 2003 Paper I 372 YCT
Which of these statements is/are correct? List I List II
(a) Only 1 (b) Only 2 (Name of the (Associated with)
(c) Both 1 and 2 (d) Neither 1 nor 2 Person)
Ans. (c) : METSAT was the first Indian (ISRO), A. Brijmohan Lal 1. Biotechnology
Meteorological geostationary satellite designed to Munjal
simultaneously obtain atmospheric cloud cover, water B. Kiran Karnik 2. Automobile
vapour and temperature data. PSLV-C4 carried a industry
payload of 1060 kg in to a geosynchronous orbit. C. Kiran Mazumdar 3. Software industry
Shaw
119. Who is the author of the book' New Dimensions
D. Sunil Mittal 4. Telecom industry
of India's Foreign policy?
5. Film Industry
(a) A.B. Vajpayee (b) Jaswant Singh
A B C D
(c) P.C. Alexander (d) Yashwant Sinha
(a) 4 1 5 2
Ans. (a) : “New Dimensions of India’s Foreign Policy” (b) 4 3 1 2
was written by Atal Bihari Vajpayee. (c) 2 3 1 4
120. Match List I with List II and select the correct (d) 2 1 5 4
answer using the codes given below the lists: Ans. (c) : Brijmohan Lal Munjal is chief of Hero Honda
List I (High List II i.e., he is related to Automobile Industry. Kiran Karnik
Officials) (Organisation) is chief of NASCOM i.e., he is related to Software
A. C.P. Jain 1. Indian Airlines Industry. Kiran Mazumdar Shaw is related to
biotechnology and Sunil Mittal is chief of Bharti Group
B. S. Ramadorai 2. NTPC i.e., related to Telecom industry.
C. Sunil Arora 3. TCS 124. Whom did Mahesh Bhupati and Max Mirnyi
D. Vivek Paul 4. Wipro Technologies beat in the US Open men's doubles tennis
A B C D Championship in the year 2002 to win the title?
(a) 4 1 3 2 (a) Wayne Black and Kevin Ullyet
(b) Mike Bryan and Bob Bryan
(b) 4 3 1 2
(c) Jonas Bjorkman and Todd Woodbridge
(c) 2 3 4 1
(d) Jiri Novak and Radek Stepanek
(d) 2 1 3 4
Ans. (d) : Mahesh Bhupati and Max Mirnyi beat Jiri
Ans. (c) : C.P. Jain is in NTPC, S. Ramodorai is in Tata Novak and Radek Stepanek to win the U.S. Open Men’s
Consultancy Service (TCS), Sunil Arora is in Indian Double Title.
Airlines and Vivek Paul is in Wipro Technologies. 125. Who among the following was selected for the
121. In December 2002, the Prime Minister of India Data Saeb Phalke Award?
signed 'Delhi Declaration' with the. (a) Dev Anand (b) Lata Mangeshkar
(a) President of Indonesia (c) Ramanand Sagar (d) Yash Chopra
(b) President of Russia Ans. (d) : Dada Saheb Phalke award was given to Yash
(c) Prime Minister of Cambodia Chopra for 2001, for his eminent contribution to the
film industry. Rajnikanth was awarded for the year
(d) Prime Minister of Laos 2019.
Ans. (b) : Prime Minister (Atal Bihari Vajpayee) of 126. As per the Human Development Index given by
India signed “Delhi Declaration” with the president UNDP, which one of the following sequences of
(Vladimir Putin) of Russia. By this agreement, India got South Asian countries is correct, in the order of
military weapons and technologies from Russia. higher to lower development?
122. The Indira Gandhi Peace Prize, 2002 was (a) India – Sri Lanka – Pakistan – Maldives
awarded to: (b) Maldives – Sri Lanka – India – Pakistan
(a) John Hume (b) Nelson Mandela (c) Sri Lanka – Maldives – India – Pakistan
(d) Maldives – India – Pakistan – Sri Lanka
(c) Norodom Sihanouk (d) Sadako Ogata
Ans. (b) : The Human Development Index (HDI) is a
Ans. (*) : The 2002, Indira Gandhi Peace Prize was
composite statistic of life expectancy, education, and
awarded to Shridath Rampal, a Guyanese politician who per capita income indicators, which are used to rank
was the second commonwealth secretary-General. countries into four tiers of human development. Sri
123. Match List I with List II and select the correct Lanka, Maldives, India, Pakistan rank 73,104,130,147
answer using the codes given below the lists: respectively according to recent HDI ranking.

IAS (Pre) GS 2003 Paper I 373 YCT


127. Who won the title in the finals of French Open Ans. (b) : Argentina faced economic crisis due to huge
men's singles tennis championship in the year public debt which caused a great civil strife and people
2002? became anxious about their future financial condition.
(a) Albert Costa (b) Juan Carlos Ferrero 132. In the FIFA World Cup Football event in the
(c) Andra Agassi (d) Pete Sampras year 2002, the number of goals scored by
Ronaldo of Brazil was.
Ans. (a) : Albert Costa won the French Open Title of (a) 6 (b) 7
Men in the year 2002. (c) 8 (d) 9
128. The sportsperson Soma Biswas is associated Ans. (c) : Ronaldo of Brazil scored eight goals in the
with FIFA World Cup Football event in 2002. He also been
(a) Sailing (b) Hockey golden Shoes.
(c) Golf (d) Athletics 133. In the year 2002, the President of India
Ans. (d) : Soma Biswas is related to athletics. She is presented the national award for excellence in
public administration and management
champion of women’s 200 m Race. sciences to.
129. Match List I with List II and select the correct (a) Kumar Manglam Birla
answer using the codes given below the lists: (b) N.R. Narayana Murthy
List I List II (c) Rahul Bajaj
(Distinguished (Organisation/ (d) Ratan Tata
Lady) Industry) Ans. (b) : Infosys Technologies chief mentor N R
Narayana Murthy has been chosen for Lal Bahadur
A. Amrita Patel 1. National Dairy Shastri National Award for Excellence in Public
Development Board Administration and Management Sciences for the year
B. Anu Aga 2. Park Hotels 2001.
C. Mallika 3. Pfizer Limited 134. The Genetic Engineering Approval Committee,
Srinivasan whose permission is required for cultivation of
D. Priya Paul 4. Thermax Limited any genetically modified crop such as Bt-
Cotton in India, is under the Union Ministry of
5. Tractors and Farm
(a) Agriculture
Equipment Limited
(b) Environment & Forests
A B C D (c) Commerce and Industry
(a) 1 4 5 2 (d) Rural Development
(b) 2 4 1 5 Ans. (b) : The genetic Engineering approval committee
(c) 1 3 5 2 work under ministry of environment & forest and climate
(d) 2 3 1 5 chance. In its report on genetic modified crops and its
impact on environment recommended, the GM crops
Ans. (a) : Amrita Patel is related to National Dairy should be introduced in the country only after critical
Development Board. Anu Aga is related to Thermax scientific evaluation of its benefit and safety, also
Limited, Mallika Srinivasan is related to Tractors and recommended restructuring of regulatory framework for
Farm Equipment and Priya Paul is related to Park unbiased assessment of GM crops.
Hotels.
130. Sandeep Pandey, the winner of Raman MISCELLANEOUS
Magsysay Award, is mainly an activist in:
135. 'A' and 'B' are two fixed point in a field. A
(a) prevention of child marriages cyclist 'c' moves such that & ACB is always a
(b) promotion of communal harmony right angle. In this context, which one of the
(c) environmental protection following statements is correct?
(d) education and livelihood projects for Dalits (a) The path followed by the cyclist is an ellipse.
Ans. (d) : Sandeep Pandey is mainly an activist in (b) The path followed by the cyclist is a circle.
education and livelihood projects for Dalits. (c) The path followed by the cyclist is an
131. What was the main reason for a great civil exponential curve.
strife in Argentina recently? (d) This type of motion is not possible.
(a) Ethnic conflict between the native Red Indian Ans. (b) : According to question ∠ACB = 90º
tribes and others.
(b) Economic crisis due to huge public debt.
(c) The issue of using native languages as
medium of instruction in schools.
(d) Clashes between government troops and
secessionist groups.
IAS (Pre) GS 2003 Paper I 374 YCT
According to condition right Angle. A right Angle can 139. A car travels the first one-third of a certain
be in any orientation or rotation as long as internal angle distance with a speed of 10km/hr, the next one-
is 90º. So correct statement the path followed by the third distance with a speed of 20 km/hr and the
cyclist in a circle.
last one-third distance with a speed of 60
136. Life expectancy is highest in the world in
km/hr. The average speed of the car for the
(a) Canada (b) Germany
whole journey is :
(c) Japan (d) Norway
(a) 18 km/hr (b) 24 km/hr
Ans. (c) : Life expectancy in Canada is 79.7 years,
Germany - 77.8 years, Japan - 80.9, Norway - 78.9 (c) 30 km/hr (d) 36 km/hr
years. Hence Japan has the highest life expectancy in Ans. (a) : Let the total distance be x km
the world. Time = distance/speed
137. Three flags, each of different colour, are 1 1 1 1 1 1
available for a military exercise. Using these = × x×+ × x×+ × x
10 3 20 3 60 3
flag, different codes can be generated by
waving x
=
(i) Single flag of different colours 18
(ii) any two flags in a different sequence of Now
colours x
(iii) three flags in a different sequence of Average speed =
x /18
colours. = 18 km/hr
The maximum number of codes that can be
generated is 140. 'A' walks around a circular field at the rate of
(a) 6 (b) 9 one round per hour while 'B' runs around it at
(c) 15 (d) 18 the rate of six rounds per hour. They start in
Ans. (c) : (i) Number of ways of arranging 3 colours the same direction from the same point at 7.30
taking- a.m. They shall first cross each other at
3×2! (a) 8.30 a.m. (b) 8.10 a.m.
1 at a time 3p1 = =3
2! (c) 7.48 a.m. (d) 7.42 a.m.
(ii) Number of ways of arranging three colours taking- Ans. (d) :
3! Since, A and B move in the same direction along the
2 at a time 3p1 = = 6
1! circle, so they will first meet, each other when there is a
(ii) Number of ways of arranging 3 colours taking- difference of one round between the two.
3 at a time 3p3 = 6 Relative speed of A and B = (6−1) =5 rounds per hour.
∴ Maximum no. of codes = 3 + 6 + 6 = 15 Ans Time taken to complete one round at this speed =1/5
hr=12min. 7.30 + 12 min = 7.42 am.
138. Which one of the following statements is NOT
Hence, they shall first cross each other at 7.42 am.
correct?
(a) The Press Council of India is an autonomous 141. Which one of the following statements is
quasi-judicial body established under an Act correct?
of Parliament. (a) East Timor was a British colony for four
(b) The Press Information Bureau provides centuries.
accreditation to media person so as to have (b) The European colonial rulers handed over
easy access to information form government East Timor to Indonesia in 1975.
sources. (c) The United Nations took over East Timor in
(c) Among all the states of India Maharashtra 1999 to prepare it for independence.
publishes the largest number of newspapers. (d) East Timor finally declared its independence
(d) Press Trust of India is the largest news in December 2001.
agency in the country.
Ans. (c) : East Timor got independence on 20th May,
Ans. (c) : India's largest news agency PTI is a non- 2002. The people of East Timor suffered some of the
profit co-operative society of Indian newspapers, whose worst atrocities of modern times in their struggle for
responsibility is to provide efficient and unbiased news
self determination. When their Portuguese colonial
to its customers. It was established on 27 August, 1947
and started its services from 1949 AD. It is offering its masters withdrew in 1975, Indonesia claimed the
services in both English and Hindi. 'Bhasha' is the Hindi territory for itself and ruthlessly suppressed the
news service of the agency. Uttar Pradesh has the independence movement. Eventually the UN took over
highest number of newspapers published in all states of the administration in 1999 and supervised the territory's
India. transition to independence.

IAS (Pre) GS 2003 Paper I 375 YCT


142. With reference to Indian defence, which one of 146. In which one of the following countries Rupees
the following statement is NOT correct? is currency:
(a) With the induction of Prithvi-II, the IAF is the (a) Bhutan (b) Malaysia
only air force in the world with surface-to-
surface ballistic missiles under its command. (c) Maldives (d) Seychelles
(b) Sukhoi-30 MKI jet fighters can launch air-to- Ans. (d) : The rupee is the currency of Seyhelles. It is
air and air-to-surface precision missiles. locally called roupi, it is subdivided into 100 cents.
(c) Trishul is a supersonic surface-to-air missile Currency of Bhutan is Ngultrum, Currency of Malaysia
with a range of 30 km.
is Malaysian Ringgit and Currency of Maldives is
(d) The indigenously built INS Prabal can launch
surface-to-surface missiles. Maldives Rufiyaa.
Ans. (c) Trishul is a supersonic surface-to-air missile 147. Three bells toll at intervals of 9, 12 and 15
with a range of 9 km with payload of 5.5 Kg. minutes respectively. All the three begin to toll
143. Four different candles, which can last for 5 at 8 a.m. At what time will they first toll
hours, 4 hours, 3 hours and 2 hours of burning, together again?
respectively, are lit in a room at the same (a) 8.45 a.m. (b) 10.30 a.m.
instant and allowed to burn till such time that
the three candles in the room get extinguished. (c) 11.00 (d) 1.30 p.m.
The cost of burning each one of these candles is Ans. (c) : Bells will toll together again at a time, which
75 paise per hour. The cost involved is: is obtained by taking L.C.M. of their individual tolling
(a) Rs. 2.75 (b) Rs. 3.75 intervals. L.C.M. of 9, 12 and 15 = 180 min. They will
(c) Rs. 9.75 (d) Rs. 12.50
toll together again after 180 min, i.e. 3 hours. Time = 8
Ans. (c) : Total hours lit candles until three candles are
+ 3 = 11 a.m.
extinguished = 2 + 3 + 4 + 4 = 13 hours
So the cost of burning = 13 × .75 148. Which one among the following languages has
= 9.75 rupees largest number of speakers in the world?
144. Which one of the following statements is (a) Bengali (b) French
correct? (c) Japanese (d) Portuguese
(a) Alliance Air is a wholly-owned subsidiary of
Ans. (a) : Among the given options, Bengali has largest
Indian Airlines.
number of speakers in the world.
(b) The Airports Authority of India manages
seven of the country's international airports. 149. Which among the following countries was the
(c) The Airports Authority of India is the earliest to give women the right to vote?
regulatory organisation for enforcing civil air (a) Iceland (b) India
regulations in India. (c) New Zealand (d) U.S.A.
(d) It is the function of Directorate General of Ans. (c) : New Zealand gave the right to vote to women
Civil Aviation to plan and construct runways in 1893.
and terminal buildings and to provide air 150. Total time taken by a person in going to a place
safety services. by walking and returning on cycle is 5 hours 45
Ans. (a) : Alliance Air is a wholly owned subsidiary of minutes. He would have gained 2 hours by
Indian Airlines. Rest of the given statements is not cycling both ways. The time taken by him to
correct. walk both ways is
145. A two member committee comprising of one (a) 6 hrs. 45 minutes (b) 7 hrs. 45 minutes
male and one female member is to be (c) 8 hrs. 15 minutes (d) 8 hrs. 30 minutes
constituted out of five males and three females.
Ans. (b) : Given that time taken for riding both ways
Amongst the females, Ms. A refused to be a
will be 2 hours lesser than the time needed for waking
member of the committee in which Mr.
one way and riding back
...................taken as the member. In how many
From this, we can understand that
different ways can the committee b
time needed for riding one way = time needed for
constituted?
waking one way - 2 hours
(a) 11 (b) 12
Given that time taken in walking one way and riding
(c) 13 (d) 14
back = 5 hours 45 min
Ans. (d) 5C1 × 3C1 – 1
Hence, The time he would take to walk both ways = 5
= 15 – 1
= 14 hours 45 min + 2 hours = 7 hours 45 min

IAS (Pre) GS 2003 Paper I 376 YCT


UNION PUBLIC SERVICE COMMISSION
Civil Services (Preliminary Exam) - 2002
GENERAL STUDIES : PAPER-I
Time: 2 hours Maximum Number: 200

Ans. (a) : The statue of Gomateshwara was built by the


ANCIENT HISTORY Ganga dynasty minister and commander Chavundaraya.
It is a 57-foot (17 m) monolith (statue carved from a
1. Match List I with List II and select the correct single piece of rock) situated above a hill in
answer using the codes given below the lists: Shravanabelagola in the Hassan district, Karnataka.The
List I List II Gommateshwara statue is dedicated to the Jain figure
(Ancient site) (Archaeological Bahubali. It was built around 983 A.D. and is one of the
finding) largest free-standing statues in the world. Mahavira was
the last Tirthankara of Jains.
A. Lothal 1. Ploughed field 3. In ancient Indian Buddhist monasteries, a
ceremony called Pavarana used to be held, It
B. Kalibangan 2. Dockyard was the
C. Dholavira 3. Terracotta replica of a (a) Occasion to elect the Sanghaparinayka and
plough two speakers one on Dhamma and the other
on Vinaya.
D. Banawali 4. An inscription (b) Confession by monks of their offences
comprising ten large committed during their stay in the
sized signs of the monasteries during the rainy season.
Harappan script (c) Ceremony of initiation of new person into the
Buddhist Sangha in which the head is shaved
and when yellow robes are offered.
A B C D
(d) Gathering of Buddhist monks on the next day
(a) 1 2 3 4 to the full-moon day of Ashadha when they
(b) 1 2 4 3 take up a fixed abode for the next four months
(c) 2 1 4 3 of rainy season.
(d) 2 1 3 4 Ans. (b) : Pavarana is a Buddhist holy day celebrated
Ans. (c) : Lothal was the only Indus valley site with on the full moon of the eleventh lunar month. It marks
the end of the month of Vassa, sometimes called
artificial brick dockyard. In Kalibangan evidence of
"Buddhist Lent." This day marks the end of the rainy
ploughed agriculture fields have been found. An season in some Asian countries like Thailand, where
inscription comprising 10 large sized sign of the Theravada Buddhism is practiced. On this day, each
Harappan script was excavated from the city of monk must come before the community of monks and
Dholavira. Also a terracotta replica of plough has been atone for an offense he may have committed during the
discovered from Banwali. Vassa.
2. Which one of the following statements is not 4. The ancient Indian play Mudrarakshasa of
correct? Visakhadutt has its subject on:
(a) The statue of Gomateshwara at Sravanabelagola (a) a conflict between Gods and Demons of
represents the last Tirthankara of Jains. ancient Hindu lore.
(b) India's largest Buddhist monastery is in (b) a romantic story of an Aryan prince and a
Arunachal Pradesh. tribal woman.
(c) The Khajuraho temples were built under (c) the story of the power struggle between two
Aryan tribes.
Chandela Kings.
(d) the court intrigues at the time of
(d) The Hoysalesvara temple is dedicated to Siva.
Chandragupta Maurya.
IAS (Pre) GS 2002 Paper I 377 YCT
Ans. (d) : The Mudrarakshasa ("The Signet of the Ans. (a) : The historian Barani did not considered the
Minister"), a historical play in Sanskrit by rule in India under the Sultans of Delhi to be truly
Vishakhadatta (4th century CE) narrates the ascent of Islamic because most of the population of the Sultanate
the king Chandragupta Maurya’s power in Northern period did not follow Islam.
India. It narrates the court intrigues at the time of 8. With reference to Sufism in Indian history,
Chandragupta Maurya. consider the following statements:
5. Who among the following was the first Bhakti 1. Shaikh Ahmad Sarhandi was a
saint to use Hindi for the propagation of his contemporary of Ibrahim Lodi.
message? 2. Shaikh Nasiruddin Chirg-i-Dehlavi was a
(a) Dadu (b) Kabir disciple of Shaikh Nizamuddin Auliya.
(c) Ramananda (d) Tulsidas 3. Aurangzeb was a contemporary of Shaikh
Ans. (c) : Ramananda was the first Bhakti saint to use Salim Chisti.
Hindi for the propagation of his message. He was 4. The Qadiri order of Sufis was first
known for communicating in vernacular Hindi, and introduced in India by Shaikh Niamtullah
accepting disciples of all castes. and Makhdum Muhammad Jilani.
Which of these statements are correct:
MEDIEVAL HISTORY (a) 1 and 2 (b) 1 and 3
(c) 2 and 3 (d) 2 and 4
6. Assertion (A) : Muhammad bin Tughlaq left
Delhi, and, for two years lived in a camp called Ans. (d) : Sheikh Ahmad Sarhandi of Naqshbandi order
Swarga-dwari. was a contemporary of Akbar and Jahangir. He was
disciple of Shaikh Nizamuddin Auliya. The Qadiri order
Reason (R) : At that time, Delhi was ravaged
of Sufis was first introduced in India by Sheikh
by a form of plague and many people died.
Nizamatullah (He died in 1430 AD) and Makhdum (or
(a) Both A and R are individually true and R is Nasiruddin) Muhammad Jilani (died in 1517).
the correct explanation of A.
9. With reference to medieval Indian rulers,
(b) Both A and R are individually true but R is
which one of the following statements is
NOT the correct explanation of A.
correct?
(c) A is true but R is false.
(a) Alauddin Khilji first set up a separate ariz's
(d) A is false but R is true. department.
Ans. (a) : Mohammad -bin-Tughlaq was the Sultan of (b) Balban introduced the branding system of
Delhi from 1325 to 1351. From his accession to the horses in his military.
throne in 1325 until his death in 1351, Muhammad (c) Muhammad bin Tughlaq was succeded by his
contended with 22 rebellions, pursuing his policies, uncle to the Delhi throne.
consistently and ruthlessly. He left Delhi for two years, (d) Firoz Shah Tughlaq set up a separate
lived in a camp called Swarga - dwari. This was department of slaves.
because at that time Delhi was ravaged by a form of Ans. (d) : Firoz Shah Tughlaq established a seperate
plague (bubonic) and many people died. It wiped out department for slaves which was called 'Diwan-i-
almost half of Sultan's army. The plague attacked the Bandagan. Muhammad-bin- Tughlaq's cousin Firoz
city in 1334. Shah Tughlaq succeeded him to the throne. Alauddin
7. The historian Barani refused to consider the Khilji introduced the branding system of horses in his
military. So only option D is correct.
state in India under Delhi Sultans as truly
Islamic because 10. The motive behind Shah Jahan's Balkh campaign
was to:
(a) the majority of the population did not follow
Islam. (a) Secure a friendly ruler in Balkh and
Badakshan which bordered Kabul.
(b) the Muslim theologists were often
(b) conquer Samarqand and Farghana, the
disregarded. Mughal homelands.
(c) the Sultan supplemented the Muslim law by (c) fix the Mughal frontier on the 'scientific line',
framing his own regulations. the Amu Daria.
(d) religious freedom was accorded to non- (d) expand the Mughal Empire beyond the
Muslims. subcontinent.
IAS (Pre) GS 2002 Paper I 378 YCT
Ans. (a) : The motive behind Shah Jahan's Balkh and Ans. (c) : Cabinet Mission (1946) was the last
Badakshan campaign in central Asia was to secure the opportunity to avoid the partition of India with the
defence of North-West India. He wanted to secure a formation of interim Government. But Muslim League
friendly ruler in Balkh and Badakshan as these two rejected the plan even though they won 73 out of 78
provinces bordered Kabul. That's why the statement (a) seats in the interim government. They did not attend the
first meeting of Constituent Assembly on Dec 9, 1946.
is correct.
14. With which one of the following mountain
tribes did the British first come into contact
MODERN HISTORY with after the grant of Diwani in the year 1765?
(a) Garos (b) Khasis
11. During the Indian freedom struggle, the
Khudai Khidmatgars, also known as Red (c) Kukis (d) Tipperahs
Shirts, called for Ans. (b) : The British first came into contact with
mountain tribe called 'Khasis' after the Diwani Grant in
(a) The Union of Pashtun tribal area in north-
the year 1765
west with the Afghanistan.
15. With reference to the period of extremist
(b) The adoption of terrorist tactics and methods
nationalist movement in India with its spirit of
for terrorising and finally ousting the colonial
Swadeshi, which one of the following
rulers.
statements is not correct?
(c) The adoption of communist revolutionary
(a) Liaqat Hussain led the Muslim peasants of
ideology for political and social reform.
Barisal in their agitations.
(d) The Pathan regional nationalist unity and a
(b) In 1898, the scheme of national education was
struggle against colonialism.
formulated by Satish Chandra Mukherjee.
Ans. (d) : Khudai Khidmatgars were organized by (c) The Bengal National College was founded in
Khan Abdul Gaffar Khan also known as Frontier 1906 with Aurobindo as the Principal.
Gandhi. Khudai Khidmatgar literally translates as the (d) Tagore preached the cult of Atmasakti, the
servants of God who led a non-violent freedom struggle main plank of which was social and economic
against the British Empire by the Pashtuns also known regeneration of the villages.
as Pathans or Afghans of the North-West Frontier Ans. (a) : Liaqat Hussain did not led the Muslim
Province. peasants of Barisal in their agitation. Rest of the
12. Assertion (A) : The effect of labour participation statements is correct.
in the Indian nationalist upsurge of the early 1930s 16. With reference to the Indian freedom struggle,
was weak. which one of the following statements is not
Reason (R) : The labour leaders considered the correct?
ideology of Indian National Congress as bourgeois (a) Hakim Ajmal Khan was one of the leaders to
and reactionary. start a nationalist and militant Ahrar
movement.
(a) Both A and R are individually true and R is
(b) When the Indian National Congress was
the correct explanation of A.
formed, Sayyid Ahmad Khan opposed it.
(b) Both A and R are individually true but R is (c) The All-Indian Muslim League which was
NOT the correct explanation of A. formed in 1906 vehemently opposed the
(c) A is true but R is false. partition of Bengal and separate electorates.
(d) A is false but R is true. (d) Maulana Barkatullah and Maulana
Obeidullah Sindhi were among those who
Ans. (a) : The labour leaders were influenced by
formed a Provisional government of Indian in
communist ideology and considered INC as bourgeoisie
Kabul.
and reactionary. That's why their participation in the
Ans. (c) : The All-India Muslim League founded at
Indian nationalist upsurge was limited.
Dacca now Dhaka Bangladesh in the Bengal Presidency
13. The last opportunity to avoid the partition of in 1906. It supported the partition of Bengal to create a
India was lost with the rejection of: majority of Muslims after partition of the province.
(a) Cripps Mission 17. Which one of the following submitted in 1875 a
(b) Rajagopalachari Formula petition to the House of Commons demanding
(c) Cabinet Mission India's direct representation in the British
(d) Wavell Plan Parliament?

IAS (Pre) GS 2002 Paper I 379 YCT


(a) The Deccan Association
D. Pitt's India Act 4. The Company's
(b) The Indian Association directors were asked
(c) The Madras Mahajan Sabha to present to the
British government
(d) The Poona Sarvajanik Sabha
all correspondence
Ans. (d) : In 1875, the Poona Sarvajanik Sabha and documents
submitted a petition to the House of Commons signed pertaining to the
by 21,713 persons demanding direct representation of administration of the
India in Parliament. company.

18. The real intention of the British to include the A B C D


princely states in the Federal Union proposed (a) 2 4 3 1
by the India Act of 1935 was to: (b) 1 3 4 2
(a) Exercise more and direct political and (c) 2 3 4 1
administrative control over the princely states. (d) 1 4 3 2
(b) Involved the princes actively in the Ans. (a) : Charter Act, 1813, ended company’s trade
administration of the colony. monopoly in India. Through the Regulating Act,
(c) Finally effect the complete political and company's directors were asked to present to the British
administrative take-over of all the princely government all correspondence and documents
states by the British. pertaining to the administration of the company. By the
(d) Use the princes to counter-balance the anti- Act of 1858, the power to govern India was transferred
imperialist doctrine of the nationalist leaders. from the East India Company to the British Crown and
Pitt's India Act set up a Board of Control in Britain to
Ans. (d) : The real intention of the British to include the
fully regulate the East India Company's affairs in India.
princely states in the Federal Union proposed by the
India Act of 1935, was to use the princes to counter- 20. Which one of the following Acts of British India
balance the anti-imperialist doctrines of the nationalist strengthened the Viceroy's authority over his
executive council by substituting "portfolio" or
leaders.
departmental system for corporate functioning?
19. Match List I with List II and select the correct (a) Indian Councils Act, 1861
answer using the codes given below the lists: (b) Government of India Act, 1858
List I List II (c) Indian Councils Act, 1892
(Acts of (Provisions) (d) Indian Councils Act, 1909
Colonial Ans. (a) : The Indian Councils Act of 1861 transformed
Government of the Viceroy's Executive Council into a miniature
India) cabinet run on the portfolio system, and each of the five
ordinary members was placed in charge of a distinct
A. Charter Act, 1. Set up a Board of department of Calcutta's government.
1813 1813 Control in 21. The President of Indian National Congress at
Britain to fully the time of partition of India was
regulate the East (a) C. Rajagopalachari
India Company's (b) J.B. Kripalani
affairs in India. (c) Jawaharlal Nehru
(d) Maulana Abul Kalam Azad
B. Regulating Act 2. Company's trade Ans. (b) : Partition of India was mentioned in the
monopoly in India Mountbatten Plan. It was also known as Indian
was ended. Independence Act 1947 and was put forward by Lord
Mountbatten on June 3, 1947. It resulted in the creation
C. Act of 1858 3. The power to govern of two independent dominions, India and Pakistan.
was transferred from Acharya J.B Kriplani was the President of Indian
the East India National Congress at the time of partition of India. He
Company to the also served in the interim government of India (1946–
British Crown. 1947) and the Constituent Assembly of India.

IAS (Pre) GS 2002 Paper I 380 YCT


22. With reference to colonial period of Indian Ans. (d) : Open stunted forests with bushes and having
history, Match List I with List II and select the long roots and sharp thorns or spines are commonly
correct answer using the codes given below the found in Western Andhra Pradesh. Open stunted forests
lists-
with bushes and small trees having long roots and sharp
List I (Person) List II (Event) thorns or spines are commonly found in the area where
the rainfall is less than 80 cm. Such areas are found in
A. Macdonald 1. Doctrine of Lapse
Rajasthan, Gujarat, Haryana and western Andhra
B. Linlithgow 2. Communal Award Pradesh.

C. Dalhousie 3. August Offer 25. Which one of the following is not a lagoon?
(a) Ashtamudi lake (b) Chilka lake
D. Chelmsford 4. Diarchy (c) Periyar lake (d) Pulicat lake
A B C D Ans. (a) : Chilka, Periyar and Pulicat lake are lagoons.
(a) 3 2 1 4 They are formed due to separation of sea water by
(b) 3 2 4 1 deposition and clay, but Ashtamudi lake is not a lagoon.
(c) 2 3 1 4 26. With reference to Indian agriculture, which
(d) 1 4 3 2 one of the following statements is correct?
Ans. (c) : The Communal Award was announced by (a) About 90 percent of the area under pulses in
British PM, Ramsay McDonald in August 1932. August India is rainfed.
Offer (1940)- Linlithgow ; Under Doctrine of Lapse, (b) The share of pulses in the gross cropped area
Dalhousie annexed Satara, Jaitpur, Sambalpur, Udaipur,
at the national level has doubled in the last
Jhansi, Nagpur and Awadh. Government of India Act
two decades.
1919, introduced diarchy during the viceroyality of
Chelmsford. (c) India accounts for about 15 percent of the
total area under rice in the world.
INDIAN GEOGRAPHY (d) Rice occupies about 34 percent of the gross
cropped area of India.
23. With reference to India, which one of the
following statements is not correct? Ans. (a) : In Indian agriculture about 90% of the area
(a) About one-third of the area of the country under pulse is rainfed but pulses need less water for
records more than 750 millimeters of annual cultivation so the production of pulse hampers.
rainfall. 27. HINDALCO, an aluminium factory located at
(b) The dominant source of irrigation in the Renukut owes its site basically to:
Country is wells. (a) Proximity of raw materials.
(c) Alluvial soil is the predominant type of soil in
(b) Abundant supply of power.
the northern plains of the country.
(d) The mountain areas account for about thirty (c) Efficient transport network.
percent of the surface area of the country. (d) Proximity to the market.
Ans. (b) : Due to the network of rivers in the northern Ans. (b) : HINDALCO, an aluminium factory is located
parts of the country, the soil here is alluvial soil. About at Renukut due to abundant supply of power from
35% of the total irrigated agricultural area in the whole National Thermal Power Station, Rihand and a captive
country is irrigated by canals, which is the maximum. thermal power plant at Renu Sagar. It has a
Rest of the statements is correct. cogeneration unit at the plant itself.
24. Open stunted forests with bushes and small 28. The correct sequence of the eastward flowing
trees having long roots and sharp thorns or rivers of the peninsular India from north to
spines are commonly found in south is
(a) Eastern Orissa (a) Subarnarekha, Mahanadi, Godavari, Krishna,
(b) North-Eastern Tamil Nadu Pennar, Cauvery and Vaigai.
(c) Siwaliks and Terai region (b) Subarnarekha, Mahanadi, Krishna, Godavari,
(d) Western Andhra Pradesh Cauvery, Vaigai and Pennar.

IAS (Pre) GS 2002 Paper I 381 YCT


(c) Mahanadi, Subarnarekha, Godavari, Krishna, 32. Consider the following high yielding varieties
Cauvery, Pennar and Yaigai. of crops in India:
(d) Mahanadi, Subarnarekha, Krishna, Godavari, 1. Arjun 2. Jaya
Cauveri, Vaigai and Pennar. 3. Padma 4. Sonalika
Ans. (a) : The correct sequence of the eastward flowing Which of these are wheat?
rivers of the peninsular India from north to south is (a) 1 and 2 (b) 2 and 3
Subarnarekha, Mahanadi, Godavari, Krishna, Pennar,
(c) 1 and 4 (d) 3 and 4
Cauvery and Vagai.
Ans. (c) : Arjun and Sonalika are hybrid variety of
29. The average annual temperature of a
wheat produced by Indian Council of Agricultural
meteorological station is 26ºC, its average
Research.
annual rainfall is 63cm and the annual range of
temperature is 9ºC. The station in question is
(a) Allahabad (b) Chennai WORLD GEOGRAPHY
(c) Cherrapunji (d) Kolkata
33. Which one of the following statements is
Ans. (b) : The climatic condition of Chennai is tropical correct with reference to our solar system?
savanna type where the average annual temperature of
meterological station is 26ºC and average rainfall is 63 (a) The Earth is the densest of all the planets in
cm and annual range temperature is 9ºC. our solar system.
30. Chapchar Kut is a festival celebrated in the (b) The predominant element in the composition
state of of Earth is silicon.
(a) Arunachal Pradesh (b) Assam (c) The Sun contains 75 percent of the mass of
(c) Mizoram (d) Sikkim the solar system.
Ans. (c) :The Chapchar Kut is a festival of Mizoram. (d) The diameter of the Sun is 190 times that of
It is celebrated during March after completion of their the Earth.
most arduous task of jhum cultivation i.e., jungle- Ans. (a) : With an average density of 5.52, the Earth is
clearing (clearing of the remnants of burning). It is a the densest of all the planets of our solar system. The
spring festival celebrated with great favour and gaiety. predominant element of the Earth's composition is iron
31. Match List I with List II and select the correct (35%) while that of the earth's crust is oxygen (46%).
answer using the code given below the lists: The Sun contains more than 99% of the mass of the
solar systems. The diameter of the sun is about 109
List I (Indian List II (Scientific
times that of the Earth.
Wildlife names) Species)
34. Consider the following statements:
A. Asiatic Wild Ass 1. Boselaphus 1. In equatorial regions, the year is divided
tragocamelus into four main seasons.
2. In Mediterranean region, summer receives
B. Barasingha 2. Rucervus
more rain.
duvauceli
3. In China type climate, rainfall occurs
C. Chinkara 3. Equus hemionus throughout the year.
4. Tropical highlands exhibit vertical
D. Nilgai 4. Gazella bennetti zonation of different climates.
A B C D Which of these statements are correct:
(a) 2 3 1 4 (a) 1, 2, 3 and 4 (b) 1, 2 and 3
(b) 3 2 4 1 (c) 1, 2 and 4 (d) 3 and 4
(c) 2 3 4 1
Ans. (d) : Equatorial regions are found near equator
(d) 3 2 1 4 which includes Central America, South America,
Ans. (b) : List-I List-II Central Africa and South East Asia. Rainforest are
(Animal) (Scientific name) located in the equatorial climate region. They are hot
A. Asiatic wild Ass – Equus hemionus and wet throughout the year which creates humid
climate. Hence statement 1 is not correct.
B. Barasingha – Rucervus duvaucelii
Mediterranean region has dry climate. There is no
C. Chinkara – Gazella bennetti rainfall during summer while this region gets rain
D. Nilgai – Boselaphus during winters. Hence statement 2 is also incorrect. Rest
tragocamelus of the two given statements is correct.
IAS (Pre) GS 2002 Paper I 382 YCT
35. Match List I with List II and select the correct 38. Assertion (A) : Artificial satellites are always
answer using the code given below the lists: launched from the earth in the eastward direction.
List I List II Reason (R) : The earth rotates from west to east
(Country) (President) and so the satellite attains the escape velocity.
A. Cango 1. Levy Mwanawasa (a) Both A and R are individually true and R is
B. Nigeria 2. Joseph Kabila the correct explanation of A.
C. Uganda 3. Olusegum Obasanjo (b) Both A and R are individually true but R is
D. Zambia 4. Yoweri Museveni NOT the correct explanation of A.
A B C D (c) A is true but R is false.
(a) 2 3 4 1 (d) A is false but R is true.
(b) 3 2 1 4 Ans. (c) : Artificial satellites are always launched from
(c) 2 3 1 4 the earth in the eastward direction as the earth rotates
from west to east. It makes the satellite geosynchronous.
(d) 3 2 4 1
But the escape velocity of 11.2 km/s is given by the
Ans. (a) : When the question was asked the President of booster of the rocket and has nothing to do with the
Congo was Joseph Kabila, President of Nigeria was direction of launching the satellite
Olusegum Obasanjo. Yoweri Museveni is president of 39. Assertion (A) : The surface winds spiral inwards
Uganda since 1986 and Levy Mwanawasa was upon the centre of the cyclone.
president of Zambia. Reason (R) : Air descends in the centre of the
36. Consider the following ecosystems: cyclone
1. Taiga 2. Tropical evergreen (a) Both A and R are individually true and R is
3. Tropical decidous 4. Tundra the correct explanation of A
The correct sequence in decreasing order of the (b) Both A and R are individually true but R is
albedo values of these ecosystems is NOT the correct explanation of A
(a) 1, 4, 3, 2 (b) 4, 1, 2, 3 (c) A is true but R is false
(c) 4, 1, 3, 2 (d) 1, 4, 2, 3 (d) A is false but R is true
Ans.(c): Surface albedo is a property of the earth’s Ans. (a) : The surface wind spiral inwards the centre of
surface that provides an important climate regulating the cyclone, because air descends at the centre of the
ecosystem service through the reflection of incoming cyclone.
solar radiation. The correct sequence in decreasing 40. Consider the following climatic and
order of the albedo values of these ecosystems are geographical phenomena:
Tundra, Taiga, Tropical deciduous and Tropical 1. Condensation
evergreen. 2. High temperature and humidity
37. Assertion (A) : 60º–65º latitudes in both the 3. Orography
hemispheres have a low pressure belt instead of 4. Vertical wind
high pressure. Thunder Cloud development is due to which of
Reason (R) : The low pressure areas are these phenomena?
permanent over oceans rather than on land. (a) 1 and 2 (b) 2, 3 and 4
(a) Both A and R are individually true and R is (c) 1, 3 and 4 (d) 1, 2, 3 and 4
the correct explanation of A.
Ans. (d) : Thunder cloud can develop by all the above
(b) Both A and R are individually true but R is mentioned climatic and geographical phenomena. High
NOT the correct explanation of A. temperature and humidity cause the wind to rise
(c) A is true but R is false. vertically up where the winds get condensed forming
(d) A is false but R is true. cumulonimbus clouds or thunder clouds. These clouds
Ans. (c) : Sub-polar low pressure is located between cause rain in the presence of orographical features such
as mountains.
60– 65º latitude in both hemisphere. Low pressure is
produced in this area, because the surface air spreads 41. For short-term climatic predictions, which one
outward from this zone due to rotation of earth. Salt of the following events, detected in the last
water takes time to release the heat as compared to land. decade, is associated with occasional weak
So high pressure areas are permanent on over oceans monsoon rains in the Indian sub-continent?
rather than on land. Hence, A is true but R is false. So (a) La Nina
option (c) is correct answer. (b) Movement of Jet Streams

IAS (Pre) GS 2002 Paper I 383 YCT


(c) EI Nino and Southern Oscillations 44. Consider the following countries:
(d) Greenhouse effect on global level 1. Brazil 2. Indonesia
Ans. (c) : For short-term climatic predictions, El Nino 3. Japan 4. Russia
and southern oscillations, detected in the last decade, is What is the descending order of the size of the
associated with occasional weak monsoon rains in the following countries population-wise?
Indian subcontinent. El Niño and the Southern (a) 1, 2, 4, 3 (b) 2, 3, 1, 4
Oscillation, also known as ENSO is a periodic
(c) 2, 1, 4, 3 (d) 1, 2, 3, 4
fluctuation in sea surface temperature (El Niño) and the
air pressure of the overlying atmosphere (Southern Ans. (c) : According to data of 2010, population of
Oscillation) across the equatorial Pacific Ocean. Indonesia is 242 million, population of Brazil is 201
million, population of Japan is 126.8 million and
42. In the map given below, four island of Indian
population of Russia is 139.3 million. So the
Ocean region i.e., (A) Seychelles, (B) Chagos,
descending order of the size of the country is Indonesia
(C) Mauritius and (D) Socotra are marked as 1,
2, 3 and 4. Match them and select the correct Brazil Russia and Japan.
answer from the codes given below: 45. Match List I with List II and select the correct
answer using the codes given below the lists:
List I (Ethnic List II
Community) (Country)

A. Apatani 1. China

B. Dayak 2. India

C. Dinka 3. Indonesia

D. Uighur 4. Sudan
A B C D
(a) 2 3 4 1
A B C D
(b) 3 2 4 1
(a) 1 3 4 2
(c) 2 3 1 4
(b) 3 1 2 4
(d) 3 2 1 4
(c) 1 3 2 4
(d) 3 1 4 2 Ans. (a) : Human Ethnic Community – Country
Ans. (d) : In the given figure, four islands of Indian 1. Dinka- Sudan
ocean region are Chagos, Socotra, Seychelles and 2. Apatani -India
Mauritius which are marked as 1, 2, 3 and 4 3. Dayak- Indonesia
respectively. 4. Uighur- China
43. Consider the following statements: 46. Consider the following countries:
1. Ocean currents are the slow-surface 1. Albania 2. Bosnia
movement of water in the ocean. Herzegovina
2. Ocean currents assist in maintaining the 3. Croatia 4. Macedonia
Earth's heat balance. Which of these countries has/have Adriatic Sea
3. Ocean currents are set in motion primarily as a boundary?
by prevailing winds. (a) 1 and 2 (b) 1, 2 and 3
4. Ocean current are affected by the
(c) 4 only (d) 3 and 4
configuration of the ocean.
Ans. (b) : The Adriatic Sea is a body of water
Which of these statements are correct?
separating the Italian Peninsula from the Balkan
(a) 1 and 2 (b) 2, 3 and 4
Peninsula. The Adriatic is the northernmost arm of the
(c) 1, 3 and 4 (d) 1, 2, 3 and 4 Mediterranean Sea, extending from the Strait of Otranto
Ans. (b) : Ocean currents are not slow most of the time (where it connects to the Ionian Sea) to the northwest
and can be surface current also, so statement 1, is and the Po Valley. The countries with coasts on the
wrong, and Rest of others are correct. Hence, option (b) Adriatic are Albania, Bosnia and Herzegovina, Croatia,
will be right answer. Italy, Montenegro and Slovenia.

IAS (Pre) GS 2002 Paper I 384 YCT


INDIAN CONSTITUTION AND POLITY C. Article 30(1) 3. All minorities whether
based on religion or
47. In the case of election to the Lok Sabha, the language shall have
amount of security deposited for general the fundamental right
category candidates and SC/ST category to establish and
candidates respectively is administer educational
(a) Rs. 5,000 and Rs. 2,500 institutions of their
(b) Rs. 10,000 and Rs. 2,500 choice.
(c) Rs. 10,000 and Rs. 5,000
D. Article 31(1) 4. No citizen shall be
(d) Rs. 15,000 and Rs. 7,500
denied admission into
Ans. (c) :In India, candidates contesting election to the any educational
Lok Sabha have to pay a security deposit of Rs 25,000. institution maintained
For Scheduled Caste and Scheduled Tribe candidates, by the State, or
the amount is Rs 12,500 and Rs 5,000 respectively. Any receiving State aid, on
candidate who fails to secure more than one-sixth (16.6 grounds of religion,
per cent) of the total valid votes casted would have to race, caste, language
forfeit his or her deposit. or any of them.
When the question was asked it was 10,000 for general
cateogy and 5000 for SC/ST category. A B C D
(a) 2 4 3 1
48. The 93rd Constitutional Amendment Bills deals
with the: (b) 3 1 2 4
(a) Continuation of reservation for backward (c) 2 1 3 4
classes in government employment. (d) 3 4 2 1
(b) Free and compulsory education for all Ans. (a) : Article 16(2) - No discrimination on grounds
children between the age of 6 and 14 years.only of religion, race, caste, sex, origin, place of birth,
(c) Reservation of 30 percent posts for women in
residence in relation to any employment or office under
government recruitments.
the State. Article-29(2) - No discrimination against
(d) Allocation of more number of parliamentary
citizens on the basis of religion, race, caste, sex,
seats for recently created States.
language only on admission to educational institutions
Ans. (b) : According to Constitution 93rd Amendment
declared or subsidized by the state. Article 30 (1) -
Bill, every Child of the age group of 6-14 years shall
have right to free and compulsory Education. No child Freedom of all minorities to establish educational
institutions of their choice based on religion or
is liable to pay any kind of fee/ capitation fee/charges.
A collection of capitation fee invites a fine up to 10 language. Article-31(1) - No person can be deprived of
times the amount collected. property by law except otherwise. (Source - Bayer Act
49. Match List I with List II and select the correct - Constitution)
answer using the codes given below the lists: 50. The salaries and allowances of the Judges of
List I (Article List II the High Court are charged to the
of Indian) (Provisions) (a) Consolidated Fund of India
(b) Consolidated Fund of the State
A. Article 16(2) 1. No person shall be
deprived of his (c) Contingency Fund of India
property save by the (d) Contingency Fund of the State
authority of law. Ans. (b) : The Consolidated Fund of India includes
revenues which the government generates through taxes
B. Article 29(2) 2. No person can be
and expenses incurred in the form of borrowings and
discriminated against
loans. The salary of the Judge of High Court are also
in the matter of public
charged from the Consolidated Fund of the respective
appointment on the
state.
ground of race,
religion or caste. 51. The members of the Constituent Assembly
which drafted the Constitution of India were
IAS (Pre) GS 2002 Paper I 385 YCT
(a) Nominated by the British Parliament. (c) Article 14 to Article 18
(b) Nominated by the Governor General. (d) Article 13 to Article 17
(c) Elected by the Legislative Assemblies of Ans. (c) : There are 6 groups of Fundamental Rights:
various provinces. Right to Equality 14-18, Right to Freedom19-22 ,Right
(d) Elected by the Indian National Congress and against Exploitation 23& 24, Right to Freedom of
Muslim League. Religion 25-28, Cultural & Educational Rights 29 &30,
Ans. (c) : The members of the Constituent Assembly Right to Constitutional remedies 32.
which drafted the Constitution of India were elected by 55. Which one of the following amendments to the
the Legislative Assemblies of various provinces. Indian Constitution empowers the President to
52. The purpose of the inclusion of Directive send back any matter for reconsideration by
Principles of State Policy in the Indian the Council of Ministers?
Constitution is to establish (a) 39th (b) 40th
(a) political democracy (c) 42nd (d) 44th
nd
(b) social democracy Ans. (d) : 42 Constitutional Amendment Act, 1976
(c) Gandhian democracy made it explicit that the President shall act in
accordance with the advice of Council of Ministers. The
(d) social and economic democracy amendment went into effect from 3 January 1977. The
th
Ans. (d) : The Directive Principles of State Policy has 44 Amendment Act, 1978 however added that the
been mentioned under Part IV (Article 36 to Article 51) President can send the advice back for reconsideration
of the Constitution of India. Though not justiciable, once. But if the Council of Ministers sends the same
they are fundamental in governance of the country. The advice again to the President then the President must
purpose of the inclusion of Directive Principles of State accept it. The amendment went into effect from 20 June
1979.
Policy in the Indian Constitution is to establish Social
and Economic Democracy. 56. The term of the Lok Sabha
(a) Cannot be extended under any circumstances.
53. With reference to Indian Polity, which one of
(b) Can be extended by six months at a time.
the following statements is correct?
(c) Can be extended by one year at a time during
(a) Planning Commission is accountable to
the proclamation of emergency.
Parliament.
(d) Can be extended for two years at a time
(b) President can make ordinance only when during the proclamation of emergency.
either of the two Houses of Parliament is not
Ans. (c) : The term of the Lok Sabha, unless dissolved
in session. earlier, is five years from the date appointed for its first
(c) The minimum age prescribed for appointment meeting. However, while a proclamation of emergency
as a Judge of the Supreme Court is 40 years. (Article 352) is in operation, this period may be
(d) National Development Council is constituted extended by Parliament by law for a period not
of Union Finance Minister and the Chief exceeding one year at a time and not extending in any
Ministers of all the States. case beyond a period of six months after the
proclamation has ceased to operate.
Ans. (b) : President can issue an ordinance when either
57. Which one of the following Articles of the
of two houses of Parliament are not in session (Art
Directive Principles of State Policy deals with
123). No minimum age is prescribed for appointment as
the promotion of international peace and
a judge of the Supreme Court in the Constitution. The
security?
age of a Judge of the SC shall be determined by such
(a) 51 (b) 48 A
authority and in such manner as parliament may by law
(c) 43 A (d) 41
provide. National Development Council is composed of
the PM as its head, all Union Cabinet ministers, the Ans. (a) : This comes under Directive Principles of
CMs of all states, CMs/ Administrators of all UTs and State Policy under part IV of the Constitution. Article
the members of the NITI Aayog (erstwhile Planning 51 states that the state shall endeavour to promote
Commission). There is no constitutional provision international peace and security.
regarding the accountability of the planning commission 58. Which one of the following rights was
to the parliament. described by Dr. B.R. Ambedkar as the heart
54. In the Indian Constitution, the Right to and soul of the Constitution?
Equality is granted by five Articles. They are (a) Right to freedom of religion
(b) Right to property
(a) Article 16 to Article 20 (c) Right to equality
(b) Article 15 to Article 19 (d) Right to Constitutional remedies
IAS (Pre) GS 2002 Paper I 386 YCT
Ans. (d) : Dr. B.R. Ambedkar called Article 32 of the 61. Which one of the following authorities
Indian Constitution i.e. Right to Constitutional remedies recommends the principles governing the
as ' the heart and soul of the Constitution’. It was made grants-in-aid of the revenues to the states out of
so because mere declaration of the fundamental rights the Consolidated Fund of India?
without an effective machinery for enforcement of the
fundamental rights would have been meaningless. Also, (a) Finance Commission
a right which does not have a remedy is a worthless (b) Inter-State Council
declaration. Thus, the framers of our constitution
(c) Union Ministry of Finance
adopted the special provisions in the article 32 which
provided remedies to the citizen, in case of violation of (d) Public Accounts Committee
Fundamental Rights. Ans. (a) :Finance Commission is a constitutional Body
59. Consider the following statements with and it is constituted by the President. It is set up every
reference to India:
five years or before. It lays down rules by which Centre
1. The Chief Election Commissioner and
other Election Commissioners enjoy equal should provide grants in aid to states out of consolidated
powers but receive unequal salaries. fund of India and suggest measures to increase the
2. The Chief Election Commissioner is resources of State.
entitled to the same salary as is provided to
a judge of the Supreme Court.
3. The Chief Election Commissioner shall not ECONOMY
be removed from his office except in like
manner & on like grounds as a judge of the 62. Consider the following financial institutions of
Supreme Court. India:
4. The term of office of the Election 1. Industrial Finance Corporation of India
Commissioner is five years from the date
(IFCI)
he assumes his office or till the day he
attains the age of 62 years, whichever is 2. Industrial Credit and Investment
earlier. Corporation of India (ICICI).
Which of these statements are correct? 3. Industrial Development Bank of India
(a) 1 and 2 (b) 2 and 3
(c) 1 and 4 (d) 2 and 4 (IDBI)
Ans. (b) : The three commissioners, including the Chief 4. National Bank for Agriculture and Rural
Election Commissioner, are entitled to a salary which is Development (NABARD).
"equal to the salary of a judge of the Supreme Court", The correct chronological sequence of the
according to the provisions of a law governing their
establishment of these institutions is:
conditions of service. Chief Election Commissioner of
India like Judge of the Supreme Court can be removed (a) 1, 2, 3, 4 (b) 2, 3, 4, 1
from his office only by the Parliament with a two-thirds (c) 3, 4, 1, 2 (d) 4, 1, 2, 3
majority in both the Lok Sabha and the Rajya Sabha on
Ans. (a) : IFCI – 1948
the grounds of proved misbehavior or incapacity.The
President appoints Chief Election Commissioner and ICICI – 1955
Election Commissioners. They have tenure of six years, IDBI – 1964
or up to the age of 65 years, whichever is earlier. NABARD – 1982
60. The Consultative Committee of Members of
63. A country is said to be in a debt trap if:
Parliament for Railway Zones is constituted by
(a) It has to abide by the conditionalities imposed
the
by the International Monetary Fund.
(a) President of India
(b) It has to borrow to make interest payments on
(b) Ministry of Railways outstanding loans.
(c) Ministry of Parliamentary Affairs (c) It has been refused loans or aid by creditors
(d) Ministry of Transport abroad.
Ans. (c) : The Consultative Committee of members of (d) The World Bank charges a very high rate of
Parliament for Railway Zones in constituted by the interest on outstanding as well as new loans.
Ministry of Parliamentary Affairs. Main purpose of this Ans. (b) : Debt trap is a situation in which a debt is
Committee is to provide a forum for informal difficult or impossible to repay, typically because high
discussions between the Government and Members of interest payment prevents repayment of the principle.
Parliament on policies and programmes of the A country is said to be in a debt trap if it has to borrow
Government and the manner of their implementation. to make interest payments on outstanding loan.

IAS (Pre) GS 2002 Paper I 387 YCT


64. Consider the following statements: The decreasing order of the contribution of
Full convertibility of the rupee may mean these sectors to the Gross Domestic Product
1. Its free float with other international (GDP) at factor cost at constant prices (2000-
currencies. 01) is
2. Its direct exchange with any other (a) 3, 1, 2, 4 (b) 1, 3, 4, 2
international currency at any prescribed (c) 3, 4, 1, 2 (d) 1, 3, 2, 4
place inside and outside the country.
Ans. (c) : The correct order is Trade - Hotel, Transport
3. It acts just like any other international and Communication > Financing, Insurance, Real
currency.
Estate and Business services > Agriculture, forestry
Which of these statements are correct? and fishing > Manufacturing.
(a) 1 and 2 (b) 1 and 3
68. With reference to the Public Sector
(c) 2 and 3 (d) 1, 2 and 3
Undertakings in India, consider the following
Ans. (d) : Capital account convertibility means free statements:
conversion of cross-border capital flows, implying that
anybody can convert domestic currency into hard 1. Minerals and Metals Trading Corporation
currency at the market rate and take the hard currency of India Limited is the largest non-oil
out of the country. importer of the country.
65. India has the maximum volume of foreign 2. Project and Equipment Corporation of
trade with India Limited is under the Ministry of
(a) USA (b) Japan Industry.
(c) Germany (d) UAE 3. One of the objectives of Export Credit
Ans. (a) : In the given question maximum trade was Guarantee Corporation of India Limited is
with U.S.A. Foreign Trade in 2004-05. to enforce quality control and compulsory
Import Export pre-shipment inspection of various
exportable commodities.
Asia and Oceania Asia, Oceania
Western Europe Western Europe Which of these statements is/are correct?
(a) 1 only (b) 1 and 2
America America
(c) 2 and 3 (d) 3 only
(Source - India - 2006)
Ans. (a) : Minerals and metal trading corporation of
66. India's external debt increased from US $ India limited is the largest non-oil importer of the
98,158 million as at the end of March 2000 to country. Hence 1st statement is correct.
US $ 1,00 225 million as at the end of March
Project and equipment corporation of India works under
2001 due to increase in:
ministry of commerce and industry. Hence 2nd statement
(a) Multilateral and bilateral debt.
is incorrect.
(b) Rupee debt.
ECGC or the Export Credit Guarantee Corporation of
(c) Commercial borrowings and NRI deposits. India Limited was set up in 1957 with the objective of
(d) Borrowings from International Monetary promoting exports by providing credit risk insurance
Fund. and related services for exports.
rd
Ans. (c) : The external debt growth Contribution during Hence 3 statement is also incorrect.
March 2000-2001.- Commercial Credit - 190,074- 69. With reference to the Indian Public Finance,
24,074, NRI Deposits 13,500-15,437, During this consider the following statements
period there was relatively no increase in multilateral
and bilateral credit. ( Source - India - 2004 ) 1. External liabilities reported in the Union
Budget are based on historical exchange
67. With reference to the Indian economy, consider
rates.
the following activities:
1. Agriculture, Forestry and Fishing 2. The continued high borrowing has kept the
2. Manufacturing real interest rates high in the economy.
3. Trade, Hotels, Transport and Communication 3. The upward trend in the ratio of Fiscal
4. Financing, Insurance, Real Estate and Deficit to GDP in recent years has an
Business services adverse effect on private investments.
IAS (Pre) GS 2002 Paper I 388 YCT
4. Interest payments is the single largest 72. Which one of the following committees
component of the non-plan revenue recommended the abolition of reservation of
expenditure of the Union Government. items for the small scale sector in industry?
Which of these statements are correct? (a) Abid Hussain Committee
(a) 1, 2 and 3 (b) 1 and 4 (b) Narasimhan Committee
(c) Nayak Committee
(c) 2, 3 and 4 (d) 1, 2, 3 and 4
(d) Rakesh Mohan Committee
Ans. (c) : External liabilities or debt refers to money
borrowed from a source outside the country. External Ans. (a) : Abid Hussain Committee, 1997 recommended
debt has to be paid back in the currency in which it is total abolition and desired replacing policy of
borrowed. Hence statement 1 is not correct. Rest of the reservation with a policy of promotion and strengthening
statements is correct. of small scale units.
70. Consider the following statements: 73. India's population growth during the 20th
1. NTPC is the largest power utility in India century can be classified into four distinct
phases. Match List I with List II and select the
2. ONGC accounts for half of the LPG
correct answer using the codes given below the
production in India.
lists:
3. Indian Oil Corporation operates all the oil
List I (Period) List II (Phase)
refineries in India.
4. The Indian Ordinance Factory is the A. 1901-1921 1. Steady growth
largest departmentally run industrial
undertaking in the country. B. 1921-1951 2. Rapid high growth
Which of these statements is/are correct? C. 1951-1981 3. Stagnant growth
(a) 1 only (b) 2 and 3
D. 1981-2001 4. High growth with
(c) 2, 3 and 4 (d) 1 and 3
definite signs of
Ans. (*) : In the given statements only 1 &4 are correct slowdown
because ONGC, IOC, HPCL are the major LPG
A B C D
producers. But 3 is wrong because all the refineries are
(a) 3 1 4 2
not run by Indian oil corporation. Many private
(b) 1 3 2 4
companies also have their own refineries.
(c) 3 1 2 4
71. Consider the following: (d) 1 3 4 2
1. Currency with the public
Ans. (c) : Period Phase
2. Demand deposits with banks
A. 1901–1921 Stagnant Growth Phase
3. Time deposits with banks
B. 1921–1951 Steady Growth Phase
Which of these are included in Broad Money
(M 3) in India? C. 1951–1981 Rapid High Growth Phase
(a) 1 and 2 (b) 1 and 3 D. 1981–2001 High Growth with definite
(c) 2 and 3 (d) 1, 2 and 3 74. With reference to the Wholesale Price Index
Ans. (d) : Narrow money is the most liquid part of the (WPI), consider the following statements:
money supply because the demand deposits can be 1. The new WPI series with base 1993–94 = 100
withdrawn anytime during the banking hours. Time became effective from April 1998.
deposits on the other hand have a fixed maturity period 2. In the new WPI series, the weight for
and hence cannot be withdrawn before expiry of this primary articles has gone down by 10
period. When we add the time despots into the narrow
percentage points.
money, we get the broad money, which is denoted by
M3. M3 = Narrow money + Time Deposits of public 3. The weight for electricity has increases in the
with banks. Broad money does not include the interbank new WPI series.
deposits such as deposits of banks with RBI or other Which of these statements are correct:
banks. At the same time, time deposits of public with all (a) 1, 2 and 3 (b) 2 and 3
banks including the cooperative banks are included in
the Broad Money. (c) 1 and 3 (d) 1 and 2

IAS (Pre) GS 2002 Paper I 389 YCT


Ans. (b) : The new WPI series with base 1993-94 = 100 Ans. (a) : Only in the year 2000-01, Bombay Stock
became effective from April 2000. In the new WPI Exchange accounted 46.1% of total contracts and 41.1%
series, the weight for primary articles has gone down by of total turnover of derivative segment of India.
10 percentage points from 32 -22%. The weight for
electricity has increased in the new WPI series from 11- PHYSICS
14%.
75. Global capital-flows to developing countries 79. Assertion (A) : An iron ball floats on mercury but
increased significantly during the nineties. In gets immersed in water.
view of the East Asian financial crisis and Latin Reason (R) : The specific gravity of iron is more
American experience, which type of inflow is than that of mercury.
good for the host country? (a) Both A and R are individually true and R is
(a) Commercial loans the correct explanation of A.
(b) Foreign Direct Investment (b) Both A and R are individually true but R is
(c) Foreign Portfolio Investment NOT the correct explanation of A.
(d) External Commercial Borrowings (c) A is true but R is false.
Ans. (b) : There are two forms of foreign investment in (d) A is false but R is true.
any country - foreign direct investment and portfolio Ans. (c) : Iron with specific gravity less than that of
investment. In foreign direct investment, investors mercury but more than that of water, floats in the
bring in technical knowledge, machinery and capital former but gets immersed in the second. Hence, option
goods along with investing capital directly in various C is correct.
sectors. Whereas in portfolio investment, only money is 80. Assertion (A) : With the increase of temperature,
invested. Therefore, considering the needs of the the viscosity of glycerin increases.
developing country, foreign direct investment seems
Reason (R) : Rise of temperature increases
appropriate. (Source - Economy - Datta - Sundaram
kinetic energy of molecules.
76. In terms of value, which one of the following (a) Both A and R are individually true and R is
commodities accounted for the largest the correct explanation of A.
agricultural exports by India during the three
(b) Both A and R are individually true but R is
year period from 1997-1998 to 1999-2000?
NOT the correct explanation of A.
(a) Cereals (b) Marine products
(c) A is true but R is false.
(c) Spices (d) Tea
(d) A is false but R is true.
Ans. (b) : Largest agricultural exports were of marine
Ans. (d) : Increase in temperature decreases the
products followed by tea and spices. viscosity of a liquid and increases its fluidity. Thus with
77. Five Year plan in India is finally approved by: the increase of temperature viscosity of glycerine will
(a) Union Cabinet decrease. Hence (A) is false. But R is correct as with the
(b) President on the advice of Prime Minister rise of temperature the kinetic energy of the liquid
increases as kinetic energy of liquid molecules is
(c) Planning Commission
directly proportional to absolute temperature.
(d) National Development Council
81. Consider the following natural phenomena:
Ans. (d) :In India plans were formulated by Planning
1. Terrestrial heating 2. Reflection of light
Commission and were finally approved by National
Development Council. All state Chief Ministers are 3. Refraction of light 4. Diffraction of light
members of NDC. The NDC is headed by Prime Due to which of these phenomena is mirage
Minister of India. formed?
78. Among the following major stock exchanges of (a) 1 and 2 (b) 2, 3 and 4
India, the exchange which recorded highest (c) 1 and 3 (d) 4 only
turnover during the year 200-01 is Ans. (c) : Convection currents of hot air rising off a
(a) Bombay Stock Exchange surface have a lower density than the air directly above
(b) Calcutta Stock Exchange it. The difference in density cause light passing through
(c) Delhi Stock Exchange the air to refract differently, which causes the formation
of mirage.
(d) National Stock Exchange
IAS (Pre) GS 2002 Paper I 390 YCT
82. Sun's halo is produced by the refraction of light 85. Match List I with List II and select the correct
in: answer using the code given below the lists:
(a) Water vapour in Stratus clouds. List I List II (The element)
(b) Ice crystals in Cirro-Cumulus clouds. (Oxidation
No.)
(c) Ice crystals in Cirrus clouds.
(d) Dust particles in Stratus clouds. A. 2 1. Oxidation number of Mn
Ans. (c) : 'Sun Halo', a circle of light with 22º radius, is in MnO2.
Produced when sun light is refracted in hexagonal B. 3 2. Oxidation number of S
suspended ice crystals and super cool water droplets in H2 S2 O7.
formed in the cirrus clouds.
C. 4 3. Oxidation number of Ca
in CaO2.
CHEMISTRY
D. 6 4. Oxidation number of AI
83. Which one of the following statements is not in NaAlH4.
correct?
A B C D
(a) The boiling point of an aqueous solution is
(a) 3 4 1 2
higher than that of pure water.
(b) 4 3 1 2
(b) Addition of solutes to a solution causes an (c) 3 4 2 1
increase in its water potential. (d) 4 3 2 1
(c) The vapour pressure of the water in a solution Ans. (a) : The oxidation number of an element is the
is lower than that of pure water. number which tells the number of charges on that atom
(d) When a solution is separated from water by a in a molecule or ion, if all the remaining atoms from
semi permeable membrane, water movement that molecule or ion are dissociated as potential ions.
can be prevented by applying pressure to the Oxidation number of Mn in MnO2 is 4, oxidation
solution. number of S in H2S2O7 is 6, Oxidation number of Ca in
CaO2 is 2 and Oxidation number of AL in NaALH4 is 3.
Ans. (b) : On adding a solute or other soluble substance
to water, the boiling point increases because the solute BIOLOGY
molecule present on the surface reduces the rate of
evaporation of water. Due to this, the vapour pressure 86. With reference to the latest developments in
of water in the solution is lower than that of pure water. stem cell research, consider the following
The entry of solvent into the solution can be prevented statements:
by osmotic pressure, that is, external pressure on the 1. The only source of human stem cells are
solution. Thus the statements a, c, d are true and (b) the embryos at blastocyst state.
false. 2. The stem cells can be derived without
84. Assertion (A) : Synthetic detergents can lather causing destruction to blastocysts.
well in hard water. 3. Then stem cells can regenerate themselves
Reason (R) : Synthetic detergents form soluble in vitro virtually forever.
calcium and magnesium salts with hard water. 4. Indian research centres also created a few
(a) Both A and R are individually true and R is cell lines which can be developed into many
the correct explanation of A. types of tissues.
(b) Both A and R are individually true but R is Which of these statements are correct?
NOT the correct explanation of A. (a) 1, 2, 3 and 4 (b) 1, 2 and 3
(c) A is true but R is false. (c) 1, 2 and 4 (d) 3 and 4
(d) A is false but R is true. Ans. (a) : Embryos at the cystic stage are the only
Ans. (a) : Synthetic detergents are used in the case of source of human stem cells. Stem cells can be obtained
hard water because the calcium and magnesium salts of without destroying the cysts. Stem cells can literally
detergents are soluble in water. Detergents are more survive on their own for eternity. Indian research
soluble than soaps and hence form more lather than centers have also created some cell lines that can be
soaps. Therefore both Assertion and Reason are correct grown into many types of tissues. Hence all the four
and Reason is the correct explanation for Assertion. statements are correct.

IAS (Pre) GS 2002 Paper I 391 YCT


87. Which one of the following statements is not Ans. (c) : Viroid is a small infective segment of nucleic
correct? acid, usually RNA. Prion is an infectious pathogenic
(a) All echinoderms are viviparous. agent devoid of nucleic acid and composed mainly of
(b) Roundworm has no circulatory system. proteins. Mycoplasmas are smallest free living cell and
(c) In bony fishes, swim bladder is usually they lack a cell wall while Rickettsiase possess a cell
present. wall.
(d) In cartilaginous fishes, fertilization is internal. Direction : The following 10 (ten) items consist of two
statements, one labelled as the 'Assertion (A) and the
Ans. (a) : The main characteristics of the organisms of
other as 'Reason (R). You are to examine these two
Echinodermata Union are as follows: (i) All organisms
statements carefully and select the answers to these items
of this union are marine. (ii) Water convection system
using the code given below:
is found in it. (iii) The brain does not develop in the
nervous system. (iv) Special ability to reproduce. (a) Both A and R are individually true and R is
the correct explanation of A.
Examples - star fish, sea urchin, sea cucumber, feathers,
britrale star, (v) Most echinoderms are oviparous. (b) Both A and R are individually true but R is
NOT the correct explanation of A.
88. Match List I with List II and select the correct
(c) A is true but R is false.
answer using the codes given below the lists:
(d) A is false but R is true.
List I List II (Achievement) 90. Assertion (A) : Drinking of whisky increases the
(Scientists) frequency of urination.
A. Arber and 1. Developed transgenic Reason (R) : Alcohol intake speeds up the
Smith plants with Agro secretion of vasopressin in the body.
bacterium T-DNA. (a) Both A and R are individually true and R is
the correct explanation of A.
B. Feldman 2. Discovered (b) Both A and R are individually true but R is
endonucleases. NOT the correct explanation of A.
(c) A is true but R is false.
C. Mullis 3. Discovered reverse (d) A is false but R is true.
transcriptase. Ans. (c) : Alcohol acts as a diuretic. It actually
promotes urine production by inhibiting the release of
D. Temin and 4. Discovered polymerase antidiuretic hormone, or vasopressin from the pituitary
Baltimore chain reaction. gland. In turn, reduced levels of antidiuretic hormone,
prevent the kidneys from reabsorbing water and thereby
increase urine production.
A B C D 91. Assertion (A) : Human diet should compulsorily
(a) 2 1 4 3 contain Glycine, Serine and Tyrosine.
(b) 1 2 4 3 Reason (R) : Essential amino acids cannot be
(c) 2 1 3 4 synthesized in the human body.
(a) Both A and R are individually true and R is
(d) 1 2 3 4
the correct explanation of A.
Ans. (a) : Arber and Smith explored the restriction (b) Both A and R are individually true but R is
enzymes. Feldman developed parasitic plants containing NOT the correct explanation of A.
Agro bacterium (T - DNA). Mullis detected the (c) A is true but R is false.
polymerase chain reaction (PCR), and Tamin and (d) A is false but R is true.
Baltimore detected reverse transcriptase. Ans. (d) : Glycine, Serine and Tyrosine are non-
89. Which one of the following statements is essential Amino Acids. That means human body can
correct? produce them even if it is not provided from food.
(a) Prions are the smallest free-living cells. Hence (A) is false. But R is true as essential Amino
(b) The cell wall of Mycoplasmas is made up of Acids cannot be made by the body. Hence they must
amino sugars. come from food.
(c) Viroids consist of a single stranded RNA 92. Ticks and mites are actually
molecule. (a) Arachnids (b) Crustaceans
(d) Rickettsias lack cell wall. (c) Insects (d) Myriapods
IAS (Pre) GS 2002 Paper I 392 YCT
Ans. (a) : Ticks and mites are categorized under order 97. Foot-and-Mouth disease in animals, a current
Arachnids of phylum Arthopoda of animal kingdom. epidemic in some parts of the world, is caused
by:
93. With reference to the blood in a normal person,
(a) Bacterium (b) Fungus
which one of the following statements is
(c) Protozoan (d) Virus
correct?
(a) Compared to arteries, veins are less numerous Ans. (d) : Foot and mouth disease in animals (hoof and
and hold less of the body's blood at any given mouth disease) is a viral infection affecting cloven
hoofed domestic and wild animals. The viruses belong
time.
to the same family picornaviridae.
(b) Blood cells constitute about 70 percent of the
98. When one gene controls two or more different
total volume of the blood.
characters simultaneously, the phenomenon is
(c) White Blood Cells (WBC) are made by called:
lymph nodes only. (a) Apomixis (b) Pleiotropy
(d) The blood has more platelets than WBC. (c) Polyploidy (d) Polyteny
Ans. (d) : Blood has more platelets than corpuscles. A Ans. (b) : Polyploidy occurs in cells and organisms
microlitre of blood contains 5,000-10,000 of White when there are more than two paired (homologous) set
blood cells (WBC) and 150,000-500,000 platelets. of chromosome found which controls more than one
94. Consider the following plants: characters simultaneously.
1. Bougainvillea 2. Carnations • Apomix is the process of asexual reproduction in
which the embryo grows from egg cells without
3. Cocoa 4. Grapes
being fertilized.
Which of these plants are propagated by stem
• Pleiotropy occurs when a single gene influences
cuttings?
multiple phenotypic traits.
(a) 1 and 2 (b) 2, 3 and 4
• Polytene is a giant chromosome found in some two-
(c) 1, 3 and 4 (d) 1, 2, 3 and 4 winged flies.
Ans. (d) : The process of stem cutting is also known as 99. With reference to the human body, consider
striking or cloning. It is a vegetative mode of the following statements:
propagation of plants. Stems, roots or both can be used
1. The production of somatotropin goes up when
to propagate the plants. Bougainvillea, carnations,
a person exercises.
cocoa and grapes can all be propagated with stem
cuttings 2. Men's testes produce progesterone.
3. Women's adrenal glands secrete testosterone.
95. In the context of organic evolution, the loss of
limbs in snakes is explained by the 4. Stress causes the adrenals to release very less
phenomenon of: amount of cortisol than usual.
(a) Use and disuse of organs. Which of these statements are correct?
(b) Adaptation to living in burrows. (a) 1, 2, 3 and 4 (b) 1, 2 and 3
(c) Natural selection. (c) 2, 3 and 4 (d) 1 and 4
(d) Inheritance of acquired characters. Ans. (d) : Somatotropin is a growth hormone and
Ans. (a) : In organic evolution, Lamarck believed that exercise is a potentst physiological stimulus for growth
any particular part of the body that is put into more hormones. Hence 1 statement is correct.
frequent use, has tendency to develop stronger and Testes secrete testosterone andnd female’s rd
adrenal gland
better while that part of body which is put into less or secrete progesterone. Hence 2 and 3 statements are
no use has tendency to gradually disappear. So, loss of incorrect.
th
limbs in snake is an example of use and disuse of 4 statement is correct as stress causes the adrenal
organs. glands to release very less amount of cortisol (a steroid)
than usual.
96. In a bisexual flower, if androecium and
gynoecium mature at different times, the
phenomenon is known as ENVIRONMENT AND ECOLOGY
(a) Dichogamy (b) Herkogamy 100. The sea coast of which one of the following
(c) Heterogamy (d) Monogamy states has become famous as a nesting place for
Ans. (a) : In bisexual flower, the production of male the giant Olive Ridley turtles from South
and female flower (androecium and gynoecium) mature India?
at different times to avoid self-fertilization. That is (a) Goa (b) Gujarat
called dichogamy. (c) Orissa (d) Tamil nadu

IAS (Pre) GS 2002 Paper I 393 YCT


Ans. (c) : The sea coast of Orissa is famous as a nesting Which of these countries has/have been
place for giant olive Ridley turtles from South India. suffering from civil war?
Majority of Olive Ridleys turtles nest (in two or three (a) 1 only (b) 2 and 3
large groups) near Gahirmatha Beach in Odisha. The (c) 4 only (d) 1, 2, 3 and 4
beach in Odisha is the largest mass nesting place for the
Ans. (d) : All the four countries Angola, Columbia,
Olive Ridley turtles followed by the coasts of Mexico
and Costa Rica. In 1991, over 600,000 turtles nested Congo and Sudan are suffering from civil war.
along the coast of Odisha in one week. 105. With reference to Indian satellites and their
101. Among living organisms, which one of the launchers, consider the following statements:
following is the most responsible factor for 1. All the INSAT-series of satellite were
bringing about the origin of a new species? launched abroad.
(a) Isolation 2. PSLVs were used to launch IRS-series of
(b) Mutation satellites.
(c) Natural Selection
3. India used the indigenously built cryogenic
(d) Sexual Reproduction
engines for the first time for powering the
Ans. (a) : Origin of new species depends on isolation of third stage of GSLV.
gene flow between new species and develop significant
4. GSAT, launched in the year 2001, has
genetic divergence.
payloads to demonstrate digital broadcasts
102. Match List I with List II and select the correct
and internet services.
answer using the codes given below the lists:
Which of these statements are correct?
List I List II
(a) 1, 2, 3 and 4 (b) 2, 3 and 4
(Mangrove) (State)
(c) 1, 2 and 4 (d) 1 and 3
A. Achra Ratnagiri 1. Karnataka Ans. (b) : Statement I is incorrect, Rest statements 2, 3
B. Coondapur 2. Kerala and 4 are correct about the Indian satellites and their
launches.
C. Pichavaram 3. Andhra Pradesh
106. In the year 2001, NASA launched a spacecraft
D. Vembanad 4. Maharashtra to Mars named:
(a) Mars Climate Orbiter
5. Tamil Nadu
(b) Mars Global Surveyor
A B C D (c) Mars Odyssey
(a) 2 1 5 4
(d) Mars Polar Lander
(b) 4 5 3 2
(c) 2 5 3 4 Ans. (c) : In the year 2001, NASA launched a
(d) 4 1 5 2 spacecraft named Mars Odyssey which costed Rs. 40
Ans. (d) : The correct mangroves match is : crore Dollars on 7th April.
A. Achra Ratnagiri – Maharashtra 107. A World Summit with representation from all
B. Coondapur – Karnataka
the countries will be held in Johannesburg in
C. Pichavaram – Tamil Nadu
September 2002 on the subject of:
D. Vembanad – Kerala
103. Which one of the following is monogamous? (a) AIDS control
(a) Wolf (b) Walrus (b) Global terrorism
(c) Deer (d) Seal (c) Human rights
Ans. (a) : The Grey wolf is generally monogamous, (d) Sustainable development
with mated pairs usually remaining together for life.
Ans. (d) : In September 2002 in Johannesburg “World
Source :"National Geographic"
Conference on Sustainable Development” was held in
which nearly 200 countries participated.
CURRENT AFFAIRS
108. Berber speaking community, frequently in the
104. Consider the following countries: news, live in
1. Angola 2. Colombia (a) Afghanistan (b) Algeria
3. Congo 4. Sudan (c) Argentina (d) Australia

IAS (Pre) GS 2002 Paper I 394 YCT


Ans. (b) : Berber, self name Amazigh, are desendants 112. Consider the following statements:
of the pre-Arab inhabitants of North Africa. They live in 1. The World Intellectual Property
scattered communities across, Morocco, Algeria, Organisation (WIPO) is a specialized
Tunisia, Libya, Egypt, Mali, Niger and Mauritania. agency of United Nations System of
109. With reference to Power Sector in India, Organisations.
consider the following statements:
2. WIPO has its headquarters at Rome.
1. Rural electrification has been treated as a
Basic Minimum Service under the Prime 3. The Trade Related Aspects of Intellectual
Minister's Gramodaya Yojana. Property Rights (TRIPS) Agreement is
2. 100 percent Foreign Direct Investment in binding on all WTO members.
power is allowed without upper limit. 4. Least developed country members of WTO
3. The Union Ministry of Power has signed a are not required to apply the provisions of
Memorandum of Understanding with 14 TRIPS Agreement for a period of 20 years
States. from the general date of application of the
Which of these statements is/are correct? Agreement.
(a) 1 only (b) 1 and 2 Which of these statements are correct:
(c) 2 and 3 (d) 3 only (a) 1, 2, 3 and 4
Ans. (c) : Up to 100% FDI allowed in respect of (b) 2, 3 and 4
projects relating to electricity generation transmission (c) 1, 2 and 4
and distribution other than atomic reactor power plants.
There is no limit on the project cost and quantum of (d) 1 and 3
foreign direct investment. When the question was asked Ans. (d) : WIPO is a specialized agency of the United
3rd statement was also correct. Nations. WIPO was established in 1967, WIPO's
110. "World Development Report" is an annual headquarter is in Geneva (Switzerland).
publication of : The TRIPS Agreement is a minimum standards
(a) United Nations Development Programme. agreement, which allows members to provide more
(b) International Bank of Reconstruction and extensive protection of intellectual property if they wish
Development. so. The basic obligation on each member country is to
(c) World Trade Organisation. accord the treatment in regard to the protection of
(d) International Monetary Fund. intellectual property. Hence TRIPS Agreement is
Ans. (b) : International Bank of Reconstruction and binding on all the WTO members. The TRIPS
Development (IBRD)/World Bank. Agreement gives all WTO Members transitional periods
The International Bank of Reconstruction and so that they can meet their obligations under it. For
Development or World Banks World Development developing countries, the general transitional period
Report, published annually since 1978, is an invaluable was five years, i.e. until 1 January 2000. For those
guide to the economic, social and environmental state of countries on the United Nations list of least developed
the world today. countries, the transitional period was initially eleven
111. Consider the following statements: years with the possibility of extension upon duly
The Draft Electricity Bill, 2001 seeks to replace motivated request. It has been extended three times and
1. Indian Electricity Act, 1910. is now set to run until 1 July 2034.
2. Electricity (Supply) Act, 1948. 113. Who among the following set a new national
3. Electricity Regulatory Commission Act, record in the women's Pole vault event in the
1998. 7th edition of Senior Federation Cup held in
4. State Electricity Boards. Karnataka in the year 2001?
Which of these statements are correct: (a) G.G. Pramila
(a) 1, 2 and 3 (b) 2, 3 and 4 (b) Jyothna Deka
(c) 1, 2 and 4 (d) 1, 3 and 4 (c) Karamjeet Kaur
Ans. (a) : The draft Electricity Bill 2001 came to force (d) Manisha Dey
on June 10, 2003. With the coming into force of this
Act, Indian Electricity Act 1910, Electricity Supply Act Ans. (c) : Karamjit Kaur of Punjab improved her own
1948 and Electricity Regulatory Commission Act 1988 national record in Women's Pole Vault by clearing a
have been repealed. The Central Government has height of 3.15m on the concluding day of the three day
established the Electricity Appellate Tribunal in April Federation Cup Senior Athletic Meet held in Karnataka
2004. Its headquarter is in New Delhi. in 2001.

IAS (Pre) GS 2002 Paper I 395 YCT


114. The main reason for unrest and violence in Ans. (b) : The Concept of the National renewal Fund
Zimbabwe in recent years is due to: was announced by the Government as a part of the New
(a) Conflict between different native ethnic Industrial Policy, 1991. The Government formally
groups. established the National Renewal Fund (NRF) by a
(b) Prolonged crisis overland reforms. Government of India resolution on 3rd February, 1992.
During the period 1992-93 to 1998-99, assistance from
(c) Economic crisis due to continuous drought
the NRF has been provided for implementation of
and famine.
voluntary retirement scheme in Central Public Sector
(d) Power struggle between political group of Undertakings and counseling/redeployment scheme for
white European settlers and native black workers rationalised from the organised sector.
community.
117. Which country among the following has been
Ans. (c) : The main reason for unrest and violence in involved in two ties in Test cricket?
Zimbabwe in recent years is because of economic crisis (a) Australia (b) India
due to continuous drought and famine. (c) West Indies (d) England
115. Match List I with List II and select the correct Ans. (a) : Australia has been involved in two test ties in
answer using the codes given below the lists: test cricket against England and India (1960 & 1986).
List I (Artist) List II (Art) 118. The world's highest ground based telescopic
observatory is located in
A. Hiren 1. Bharatanatyam
(a) Colombia (b) India
Bhattacharya dance
(c) Nepal (d) Switzerland
B. Malini Rajurkar 2. Hindustani vocal Ans. (b) : The world's highest ground based telescopic
music observatory is located at Leh in India.
119. Consider the following names:
C. Pratibha Prahlad 3. Kuchipudi dance
1. Archbiship Desmond tutu
D. Vempati Chinna 4. Puppetry 2. Lech Wales
Satyam 3. Shimon Peres
A B C D 4. Yasser Arafat
(a) 4 2 1 3 Who among these won the Nobel Peace Prize?
(a) 1 and 2 (b) 1, 3 and 4
(b) 3 1 2 4
(c) 2, 3 and 4 (d) 1, 2, 3 and 4
(c) 4 1 2 3
(d) 3 2 1 4 Ans. (d) : The Nobel Peace Prize had been won by
Archbishop Desmond Tutu (1984), Shimon Peres
Ans. (a) : The correct match is as follows- (1994), Yaser Arafat (1994) and Lech Walesa in (1983).
Artist Art
120. With reference to Indian transport systems,
Hiren Bhattacharya Puppet Art consider the following statements:
Malini Rajurkar Hindustani Vocal Sangeet 1. Indian railway system is the largest in the
Pratibha Prahlad Bharatnatyam Dance world.
Vempati Chinna Satyam Kuchipudi Dance 2. National Highways cater to 45 percent of
the total road transport demand.
116. Consider the following statements: 3. Among the states, Kerala has the highest
The objectives of the National Renewal Fund density of surface road.
set up in February 1992 were 4. National Highway No. 7 is the longest in
1. to give training and counseling for the country.
workers affected by retrenchment or VRS. Which of these statements are correct?
(a) 1 and 2 (b) 1 and 3
2. Redeployment of workers.
(c) 2 and 3 (d) 2 and 4
Which of these statements is/are correct?
Ans. (d) : Indian railway is the fourth largest railway in
(a) Neither 1 nor 2 (b) Both 1 and 2
the world after USA, Russia and China. National
(c) 1 only (d) 2 only Highways is nearly 2% (1.96%) of total roads, but it

IAS (Pre) GS 2002 Paper I 396 YCT


carries nearly 40 – 45% of road traffic of India. Ans. (d) : On 25 Dec, 2000, (then) Prime Minister Atal
Maharashtra has the highest density of the surface Bihari Vajpayee started Antyodaya Anna Yojana.
roads, not Kerala, National Highway No. 7 (now NH44) Under this scheme, 25 kg grains per month at
is the longest in the country which connects Varanasi to
subsidized price to the poorest families, was made
Kanyakumari.
available. Under this 2 Rs./kg. Wheat and 3 Rs./kg rice
121. Match List I with List II and select the correct was provided. Under the National Old Age Pension
answer using the codes given below the lists: Scheme, there is a provision of giving 75 rupees as
List I (Name of List II central pension. (Source – India-2004)
the lady) (Achievement) 123. Recently, several hundred South African
A. Chinna Pillai 1. Winner of Neerja soldiers were sent to Burundi.
Bhanot Award. (a) To side with Tutsi tribe in the country's civil
war.
B. Kaveri Thakur 2. Recipient of Stree (b) To side with the Hutu tribe in the ongoing
Shakti Puraskar. ethnic conflict.
C. Mira Nair 3. Became the fastest (c) As a part of United Nations special force to
swimmer in Asian at keep control on the ethnic groups revolting
the age of 16 years. against the government.
(d) As South Africa's protection force to help
D. Yasoda 4. Recipient of the
mediate an agreement between warring
Ekambaram Golden Lion at
groups of civil war.
Venice Film festival.
Ans. (d) : To stop the ongoing civil war in Burundi,
A B C D South Africa sent its peace keeping force to mediate an
(a) 3 2 1 4 agreement between warring groups of civil war.
(b) 2 3 4 1 124. The Asia – Pacific Economic Co-operation
(c) 3 2 4 1 forum was held in the year 2001 in.
(d) 2 3 1 4 (a) Bangkok (b) Hong kong
(c) Jakarta (d) Shanghai
Ans. (b) : Chinna Pillai was awarded the Stree Shakti
Puraskar of 2001. Kaveri Thakur became the fastest Ans. (d) : The Asia-Pacific Economic Cooperation
swimmer in Asia at the age of 16. Mira Nair - Indian forum was held on 20-21 October 2001 at Sanghai in
filmmaker, who made films like Monsoon Wedding, China.
became the first woman to receive the Golden Lion 125. In the year 2001, Germany approved a $ 32
award at the Venice Film Festival. Yashoda million credit to India.
Ekambaram - Conferred with the Neerja Bhanot Award. (a) To promote primary education in selected
122. With reference to the government's welfare state.
schemes, consider the following statements: (b) For Tehri dam project.
1. Under the Antyodaya Anna Yojana, the
food grains are avaialble to the poorest of (c) To assist in the development of nuclear power
the poor families of Rs. 2 per kg for wheat generation projects.
and Rs. 3 per kg for rice. (d) For oceanographic research.
2. Under the National Old Age Pension Ans. (d) : In the year 2001 Germany approved $32
Scheme, the old and destitute are provided
million credit to India for oceanographic research.
Rs. 75 per month as Central pension, in
addition to the amount provided by most 126. Who among the following won six gold medals
State Governments. in the 9th FINA-World Swimming Championship
3. Government of India has allocated 25 kg held in Fukuoka, Japan in the year 2001?
food grains per Below Poverty Line family
(a) Grant Hackett (b) Ian Thorpe
per months, at less than half the economic
cost. (c) Michael Phelps (d) Romam Sloudnow
Which of these statements are correct? Ans. (b) : In 9th FlNA–World Swimming Championship
(a) 1 and 2 (b) 1 and 3 held at Fukuoka in Japan in 2001, Ian Thorpe of Australia
(c) 2 and 3 (d) 1, 2 and 3 won six gold medals.

IAS (Pre) GS 2002 Paper I 397 YCT


127. In the year 2001, India offered a grant of five MISCELLANEOUS
million dollars to Tajikistan to:
(a) Tackle the drought situation 132. Match List I with List II and select the correct
(b) Set up a Software technology Park answer using the codes given below the lists:
(c) Promote mineral exploration
(d) Procure defence equipment List I (Organisation) List II
Ans. (a) : To tackle the drought situation in Tajikistan, (Headquarters)
India offered a grant of five million dollars in the year
2001. A. International Atomic 1. Brussels
Energy Agency
128. Who was the first Indian lady actress to receive
the Padma Shri Award?
B. International 2. Geneva
(a) Smita Patil
Telecommunication
(b) Nargis Dutt
Union
(c) Meena Kumari
(d) Madhubala C. Council of the 3. Paris
Ans. (b) :The first Indian lady actress to receive the European Union
Padma Shri Award was Nargis Dutt. She was awarded
in 1958. D. Organisation for 4. Vienna
129. In the year 2001, the Prime Minister Economic Cooperation
announced a five-year excise duty holiday for and Development
industries in.
(a) Cyclone prone coastal Andhra Pradesh A B C D
(b) Border states of North-East (a) 1 2 4 3
(c) Earthquake ravaged Kutch district (b) 4 3 1 2
(d) Recently formed states of Chattisgarh and (c) 1 3 4 2
Jharkhand (d) 4 2 1 3
Ans. (c) : In the year 2001, the Prime Minister Atal
Ans. (d) : The International Atomic Energy
Bihari Vajpayee announced a five year excise duty
holiday for industries in earthquake ravaged Kutch Commission was established on 29 July 1957. Its
district in Gujarat. headquarter is in Vienna (Austria). The International
Telecommunication Union (ITU) was established in
130. In the Year 2001, in which one of the following
events of Women's Singles did Venus Williams 1865, with its headquarters in Geneva (Switzerland).
defeat Serena Williams to win the title? Council of the European Union (CEV) - Brussels,
(a) Australian Open 2001 Organization for Economic Cooperation and
(b) 7 French Open 2001 Development - Paris (France).
(c) Wimbledon 2001 133. Match List I (Commission) with List II (Matter
(d) US Open 2001 of Enquiry) and select the correct answer using
Ans. (d) : In U.S. Open 2001, Venus William defeated the codes given below the lists:
Serena Williams to lift the title. This was first time in List I List II
the history of grand slam that two sisters have played
the final. A. Wadhwa 1. The assassination
131. Consider the following countries of South Asia: Commission of Rajiv Gandhi
1. Bangladesh 2. India
3. Pakistan 4. Sri Lanka B. Liberhan 2. Killing of Graham
The descending order of literacy status of these Commission Staines
countries is:
(a) 4, 2, 1, 3 (b) 2, 4, 3, 1 C. Sri Krishna 3. Demolition of a
Commission religious structure
(c) 4, 2, 3, 1 (d) 2, 4, 1, 3
at Ayodhya
Ans. (c) : Descending order of countries in literacy
rate:- Sri Lanka > India > Pakistan > Bangladesh (in D. Jain Commission 4. Riots in Mumbai in
2006). At present, Sri Lanka (92%)> India (72%)>
1993
Bangladesh (61%)> Pakistan(56%).
IAS (Pre) GS 2002 Paper I 398 YCT
A B C D Ans. (b) : Let the side of cube is x then the volume of
(a) 3 2 4 1 the 2 cm part of the cube = 2 × x × x = 2x2
(b) 2 3 4 1 Mass displaced by the portion = 0.2 kg = 200 gm.
(c) 3 2 1 4 Now mass = volume × density ⇒ 200 = 2x2 × 1 ⇒ x2
(d) 2 3 1 4 = 100 or x = 10 cm.
Ans. (b) : Wadhwa Commission was appointed for 137. Who among the following won the men's single
killing of Graham Staines, Liberhan Commission for title at the World Badminton Championship in
Demolition of Babri Masjid at Ayodhya, Sri Krishna the year 2001?
Commission appointed for riots in Mumbai and Jain (a) Gopichand (b) Hendrawan
Commission for assassination of Rajiv Gandhi. (c) Ji Xin Peng (d) Peter Gade
134. Assume that the moon takes exactly 30 days to Ans. (b) : Hendrawan of Indonesia won the men’s
complete the cycle and also assume that it rises singles title at the world Badminton Championship in
in the East exactly at 6.48 p.m. on the first day. the year 2001, defeating Peter Gade of Sweden in the
On the fourth day, at what time will it rise? final.
(a) 8:24 p.m. (b) 9.12 p.m. 138. A trader fixed the price of an article in such a
(c) 10.00 p.m. (d) 11.48 p.m. way that by giving a rebate of 10% on the price
fixed the made a profit of 15%. If the cost of
Ans. (b) : Total time taken by the moon to complete the
the article is Rs. 72, the price fixed on it is:
cycle = 30 days.
(a) Rs. 82.80 (b) Rs. 90.00
24
∴ consecutive moon rise takes a time of (c) Rs. 92.00 (d) Rs. 97.80
30
Ans. (c) : Let the fixed price of the article be x
20× 60
= minutes = 48 minutes 9x
39 then S.P. = 90% of x =
10
Now first day moon rise at 6 : 48 pm
115 × 72
Then 2nd day = 6 : 48 + 0 : 48 = 7 : 36 Also the S.P. is 115% of 72 =
100
3rd = 7 : 36 + 0 : 48 = 7 : 84 = 8 : 24
9 x 115 × 72
4th = 8 : 24 + 0 : 48 = 8 : 72 ⇒ =
10 100
= 9 : 12 pm
115 × 72 ×10
135. A complete cycle of the traffic light takes 60 ⇒ x = = 92
9 × 100
seconds. During each cycle the light is green for
25 seconds, yellow for 5 seconds, and red for 30 139. Consider the volumes of the following:
seconds. At a randomly chosen time, the 1. A parallelepiped of length 5 cm, breadth 3
probability that the light will not be green is: cm and height 4 cm.
(a) 1/3 (b) 1/4 2. A cube of each side 4 cm.
(c) 5/12 (d) 7/12 3. A cylinder of radius 3 cm and length 3 cm.
Ans. (d) : Probability that the light is not green 4. A sphere of radius 3 cm.
The volumes of these in the decreasing order is
Time for which light is not Green
= (a) 1, 3, 2, 4 (b) 4, 2, 3, 1
Time taken for the entire cycle
(c) 1, 2, 3, 4 (d) 4, 3, 2, 1
5 + 30 35 7
= = = Ans. (d) : Volume of parallelepiped = l × b ×h
60 60 12
= 5 ×3 × 4 = 60 cm3
136. A solid cube just gets completely immersed in
Volume of cube = 4 × 4 × 4 = 64 cm3
water when a 0.2 kg mass is placed on it. If the
2
mass is removed, the cube is 2 cm above the Volume of cylinder = πr h = π × 32 × 3 = 84.7 cm3
water level. What is the length of each side of 4 4 22
the cube? Volume of sphere = πr 3 = × ×33 = 113.04 cm 2
3 3 7
(a) 12 cm (b) 10 cm
∴ required order is 4, 3, 2, 1.
(c) 8 cm (d) 6 cm
IAS (Pre) GS 2002 Paper I 399 YCT
140. A train of length 150 metres, moving at a speed ⇒ x (x + 2) = 48
of 90 km/hr can cross a 200 metre bridge in ⇒ x2 + 2x – 48 = 0
(a) 8 seconds (b) 14 seconds ⇒ x2 + 8x – 6x – 48 = 0
(c) 6 seconds (d) 15 seconds x (x + 8) – 6 (x + 8) = 0
Ans. (b) : Total distance to be crossed by the train (x + 8) (x – 6) = 0
= 150 + 200 = 350 m Neglecting the negative value of x
90×1000 x=6
Speed = 90 km/hr = m/s = 25 m/s
3600 Hence speed of 1st lady 6 + 2 = 8 km/hr
350 and speed of 2nd lady = 6 km/hr
Time taken = = 14sec.
25 143. Amit started a business by investing Rs. 30,000.
141. Match List I with List II and select the correct Rahul joined the business after some time and
answer using the codes given below the lists: invested Rs. 20,000. At the end of the year,
List-I List-II profit was divided in the ratio of 2 : 1. After
(Book) (Author)
how many months did Rahul join the Business?
A. Autobiography of 1. Mulk Raj Anand (a) 2 (b) 3
unknown Indian (c) 4 (d) 5
B. India : A wounded 2. Nirad C. Chaudhari Ans. (b) : Let after t months Rahul joined the business
civilization
∵ Amit does the business for 1 year and Rahul for (12 – t)
C. Confession of a lover 3. R.K. Narayan
months
D. The English Teacher 4. V.S. Naipaul
∴ They will share the profit in ratio
A B C D
30000 × 12 : 20000 × (12 – t) = 2 : 1
(a) 1 3 2 4
360000 2
(b) 2 4 1 3 =
24000 − 20000t 1
(c) 1 4 2 3
40000 t = 48000 – 36000
(d) 2 3 1 4
Ans. (b) :
= 120000
List-I List-II 12000
t= = 3months
(Book) (Author) 40000
A. Autobiography of 1. Mulk Raj Anand 144. When the time in the wall-clock is 3.25 p.m. the
unknown Indian acute angle between the hours-hand and the
B. India : A wounded 2. Nirad C. Chaudhari minutes-hand is
civilization
C. Confession of a lover 3. R.K. Narayan (a) 60º (b)
D. The English Teacher 4. V.S. Naipaul
142. Two ladies simultaneously leave cities A and B (c) (d) 42º
connected by a straight road and travel
towards each other. The first lady travels 2 Ans. (c) : In a clock, the angle between two successive
km/hr faster than the second lady and reaches 360
number is = 30°
B one hour before the second lady reaches A. 12
The two cities A and Bare 24 km apart. How When the time is 3.25 pm, the minute hand will be on 5
many kilometers does each lady travel in one and will have moved 60° from 3 and hour hand would
hour? be between 3 and 4 and as it moves 30° in 60 minutes,
(a) 5 km, 3 km (b) 7 km, 5 km so in 25 minutes it would move
(c) 8 km, 6 km (d) 16 km, 14 km 30°× 25
= 12.5
Ans. (c) : Let the speed of 2nd lady be x km/hr 60
Then speed of 1st lady = (x + 2) km/hr So the angle between the two hands
24 24 1
°
∴ − =1 60° − 12.5° = 47.5 = 47
x ( x + 2) 2

IAS (Pre) GS 2002 Paper I 400 YCT


145. A hollow sphere of radius R, a hollow cube of 40× 60
side R and a thin circular plate of radius R, = 24
100
made up of the same material, are all heated to
20º C above room temperature. When left to Since no. of total employees drawing more than
cool in the room, which of them will reach the 36×100
room temperature first? 5000 = = 36
100
(a) Circular plate
No. of women who draw more than 50000
(b) Cube
(c) Sphere = 36 – 24 = 12
(d) All of them will reach the room temperature ∴ No. of women who draw less than 5000
at the same time = 40 – 12 = 28
Ans. (c) : The loss of heat is directly proportional to the ∴ Percentage of such women
surface area, hence loss of heat will be fast in case of
sphere because it has the maximum surface area. 28×100
= = 70%
146. The age of a man is three times the sum of the 40
ages of his two sons. Five years hence, his age 149. Consider the following statements:
will be double of the sum of the ages of his sons.
The father's present age is 1. Light of longer wavelength is scattered
(a) 40 years (b) 45 years much more than the light of shorter
(c) 50 years (d) 55 years wavelength.
Ans. (b) : Let the sum of ages of two sons be x years 2. The speed of visible light in water is 0.95
∴ Age of the man = 3x years times the speed in vacuum.
After 5 years age of man = (3x + 5) 3. Radio waves are produced by rapidly
After 5 years age of two sons combined = (x + 10) years oscillating electrical currents.
Given (3x + 5) = 2x + 20 4. To detect the over speeding vehicles, police
x = 15 use the Doppler effect of reflected short
Hence 3x = 45 radio waves.
∴ Current age of father is 45 years. Which of these statements are correct?
147. The length of the longest pole than can be (a) 1 and 2 (b) 1 and 3
placed in a room 12m long and 8 m wide and 9 (c) 2 and 4 (d) 3 and 4
m high is:
Ans. (d) : Light of longer wavelength (lower refractive
(a) 12 m (b) 14 m
index) is scattered less than light of shorter wavelength.
(c) 17 m (d) 21 m
Speed of visible light in water = 2.25 x 108 M/Sec. And
Ans. (c) : Length of the longest pole = length of the
the speed of light in vacuum is 3.0 x 108 M/Sec. It is
diagonal of the room = (12) 2 + (8) 2 + (9) 2 clear that the speed of visible light in water is 0.75 times
= 289 = 17m. the speed of light in vacuum. Thus statement 1 and 2
148. In a company, 60% of the employees are men. are false and statement 3 and 4 are correct.
Of these 40% are drawing more than Rs. 150. A bus is moving with a speed of 30 km/hr
50,000 per years. If 36% of the total employees ahead of a car with a speed of 50 km/hr. How
of the company draw more than Rs. 50,000 per many kilo meters apart are they if it takes 15
year, what is the percentage among women minutes for the car to catch up with the bus?
who are drawing less than Rs. 50,000 per year? (a) 5 km (b) 7.5 km
(a) 70 (b) 60 (c) 12.5 km (d) 15 km
(c) 40 (d) 30
Ans. (a) : According to question relative speed of the
Ans. (a) : Let total no. of employees be 100 car with respect to the bus = 50 – 30 = 20 km/hr.
60×100 Now distance between the car and the bus, when care
∴ No. of men = = 60
100 catches the bus = distance travelled by
40×100 The car in 15 minutes (with relative speed of 20 km/hr.)
No. of women = = 40
100 15
× 20 = 5km
No. of men drawing more than Rs. 50000 60

IAS (Pre) GS 2002 Paper I 401 YCT


UNION PUBLIC SERVICE COMMISSION
Civil Services (Preliminary Exam) - 2001
GENERAL STUDIES : PAPER-I
Time: 2 hours Maximum Number: 200

Hoysala temples have features of both open (outer


ANCIENT HISTORY mantapa) and closed mantapa (innner mantapa). The
1. Which one of the Chola kings conquered ceilings of the mantapa are highly ornate bearing
Ceylon? mythological figures and floral design.
(a) Aditya I (b) Rajaraja I 5. Match List I with List II and select the correct
(c) Rajendra (d) Vijayalaya answer using the code given below the lists:
Ans. (c) Rajendra Chola I (1014-1044 AD) continued his List I List II
father’s (Rajaraja) policy of aggressive conquests and (Bhakti Saint) (Profession)
expansion. Half part of Ceylon was conquered by Rajaraja A. Namdev 1. Barber
while Rajendra conquered the whole of Ceylon. B. Kabir 2. Weaver
2. Who among the following presided over the C. Ravidas 3. Tailor
Buddhist Council held during the reign of D. Sena 4. Cobbler
Kanishka at Kashmir?
A B C D
(a) Parsva (b) Nagarjuna
(a) 2 3 1 4
(c) Sudraka (d) Vasumitra
(b) 3 2 4 1
Ans. (d) : The fourth Buddhist council was held in 72
(c) 3 2 1 4
AD at Kundalvana, Kashmir. It was presided by
Vasumitra, while Asvaghosa was his deputy. The (d) 2 3 4 1
council was held under the patronage of Kushan King Ans. (b) : Kabir, Namdev, Ravidas, Sena were the
Kanishka. Buddhism was divided into two sects namely saints of Bhakti movement.
Mahayana and Hinayana in that council. Namdev - He was a tailor from a lower caste family. He
3. Which one of the following animals was NOT was the great devotee of Lord Vitthala of Pandharpur.
represented on the seals and terracotta art of He belonged to Varakari Sect.
the Harappan culture? Kabir - He was Weaver. He was brought up by a
(a) Cow (b) Elephant Muslim couple. His songs and couplets are known as
(c) Rhinoceros (d) Tiger Doha, Sakhi or Sabad.
Ans. (a) : From the seals, terracotta figurines, and copper Ravidas - He followed the path of Nirguna Brahma. He
tablets we get an idea on the religious life of the was a cobbler and founder of Ramdasi Sampradaya. He
Harappans. The chief male deity was Pasupati, (proto- was the disciple of Saint Ramananda.
Siva) represented in seals as sitting in a yogic posture with Sena- He was a barber and belonged to the Varkari sect
three faces and two horns. He is surrounded by four dedicated to Lord Vithoba. He was also the disciple of
animals (elephant, tiger, rhino, and buffalo each facing a Saint Ramananda.
different direction). Two deer appear on his feet.
6. Which one of the following pairs is correctly
4. Hoysala monuments are found in: matched?
(a) hampi & Hospet
(a) Harappan Civilization : Painted Grey Ware
(b) Halebid & Belur
(b) The Kushans : Gandhara School of Art
(c) Mysore & Bangalore
(c) The Mughals : Ajantha Paintings
(d) Sringeri & Dharwar
(d) The Marathas : Pahari School of Painting
Ans. (b) : It is the largest monument in Halebidu. The
temple was built on the banks of a large man-made lake, Ans. (b) : Painted Greyware belonged to later Vedic
and sponsored by King Vishnuvardhana of the Hoysala period (1000–600BC). Ajanta paintings belong to the
Empire. Its construction started around 1121 CE and Gupta period. Pahari School came into existence during
was complete in 1160 CE. the Mughal period. Hence only option (b) is correct.

IAS (Pre) GS 2001 Paper I 402 YCT


7. The largest number of Buddhists is found in 10. The Mongols under Chengiz Khan invaded
(a) Bihar (b) Karnataka India during the reign of
(c) Maharashtra (d) Uttar Pradesh (a) Balban
(b) Feroz Tughlaq
Ans. (c) : The largest concentration of Buddhism is in
(c) Iltutmish
Maharashtra (58.3%), where (73.4%) of the total
(d) Muhammad bin Tughlaq
Buddhists in India reside. Karnataka (3.9 lakh) ,Uttar
Pradesh (3.0 lakh), West Bengal (2.4 lakh) and Madhya Ans. (c) : Mongols under Chengiz Khan (died in 1227)
invaded India during the reign of Iltutmish (1211-36)
Pradesh (2.0 lakh) are other states having large Buddhist
but did not enter deep in India as Iltutmish refused to
population Sikkim (28.1%), Arunachal Pradesh (13.0%)
give shelter to the Persian king, Khwarizm Shah, whom
and Mizoram (7.9%) have emerged as top three states in Chengiz Khan was chasing.
terms of having maximum percentage of Buddhist
11. Which of the following pairs is correctly
population. matched?
8. Assertion (A) : Harshavardhana convened the (a) Dewan-i-Bandagan : Tughlaq
Prayag Assembly. (b) Dewan-i-Mustakhraj : Balban
Reason (R) : He wanted to popularise only the (c) Dewan-i-Kohi : Alauddin Khilji
Mahayana form of Buddhism. (d) Dewan-i-Arz : Muhammad Tughlaq
(a) Both A and R are individually true and R is Ans. (a) : Diwan-i-Bandagani was a department created
the correct explanation of A by Feroz Shah Tughlaq. It was a department made for
(b) Both A and R are individually true but R is slaves. It was important because at that time Delhi had a
NOT the correct explanation of A lot of slaves. Diwan-i-Bandagan had the responsibility
to take care of them.
(c) A is true but R is false
(d) A is false but R is true 12. In which one of the following cities is the
Lingaraja Temple located?
Ans. (b) : Harshavardhana convened the Prayag (a) Bhubaneswar (b) Bijapur
Assembly to popularise himself among his people. He
(c) Kolkata (d) Shravanabelagola
wanted to popularise Mahayana form of Buddhism in
influence of Hiuen-Tsang and he called a conference in Ans. (a) : Lingaraj is temple dedicated to Harihara, a
Kannauj for that. form of Shiva and Vishnu. It is a 10 century temple
surrounded by high walls and is also called Harihara
temple. The temple is believed to be built by the kings
MEDIEVAL HISTORY from the Somavamsi dynasty. It is situated in
9. The shaded area in the above map shows the Bhubaneswar, Odisha.
empire of: 13. Assertion (A) : The Battle of Khanwa was
certainly more decisive and significant than the
First Battle of Panipat.
Reason (R) : Rana Sanga, the Rajput hero, was
certainly a more formidable adversary than
Ibrahim Lodi.
(a) Both A and R are individually true and R is
the correct explanation of A
(b) Both A and R are individually true but R is
NOT the correct explanation of A
(a) Ala-ud-din Khalji (c) A is true but R is false
(b) Mohammed Tughlaq (d) A is false but R is true
(c) Shahjahan Ans. (a) : The Mughal dynasty in India was established
(d) Aurangzeb after Babur defeated Ibrahim Lodi in the first battle of
Ans. (c) : Shah Jahan assumed the Mughal throne on Panipat in 1526 AD. But the erstwhile Rajput kingdom
24 January 1628 in Agra, a few days after the death of was still undivided. One ruler of this undivided kingdom
Jahangir. He inherited a vast and rich empire; and at was Rana Sanga. It invited Babur to India with the aim
mid-century this was perhaps the greatest empire in the that he would go back after defeating Ibraham Lodi like
world, exhibiting a degree of centralized control rarely
other Muslim invaders but it did not happen.
matched before. Shah Jahan expanded his empire in all
directions: he annexed the Rajput kingdoms of Baglana In 1527 AD, there was a Khanwa war between Babur and
and Bundelkhand to the west, and in 1635 he captured Rana Sanga, in which Rana Sanga was defeated. Thus
the kingdoms of Bijapur and Golconda in the Deccan. after this the Mughal Empire actually began to expand.
IAS (Pre) GS 2001 Paper I 403 YCT
MODERN HISTORY (a) 1 and 2 (b) 2 and 3
(c) 2 and 4 (d) 3 and 4
14. Who among the following Indian rulers Ans. (d) : Arya Samaj was founded in 1875 by Swami
established embassies in foreign countries on Dayanand Saraswati in Bombay. Dayanand Saraswati
modern lines? is called the 'Martin Luther' of India. Lala Lajpat Rai
(a) Haider Ali (b) Mir Qasim was also a famous Arya Samaji.
(c) Shah Alam II (d) Tipu Sultan
The Brahmo Samaj was founded by Raja Rammohan
Ans. (d) : Tipu Sultan established embassies in France, Roy in 1828 AD. After them, Devendranath Tagore
Turkey and Egypt on modern lines. Tipu Sultan, is known and Keshav Chandra Sen carried forward this
for his valiant acts in several wars and the sacrifices he
movement. Keshav Chandra Sen launched a movement
made to save his land from foreign invaders. He is
remembered for saving Deccan India from the British for a for women's education. Vinoba Bhave founded
long period of time. He was the ruler of the Kingdom of Sarvodaya Samaj to work among refugees.
Mysore from 1782 to 1799. 18. Who among the following leaders proposed to
15. The Hunter Commission was appointed after adopt Complete Independence as the goal of
the the Congress in the Ahmedabad session of
(a) Black-hole incident 1920?
(b) Jalianwala Bagh massacre (a) Abdul Kalam Azad
(c) Uprising of 1857 (b) Hasrat Mohani
(d) Partition of Bengal (c) Jawahar Lal Nehru
Ans. (b) : Hunter Commission was appointed after the (d) Mohandas Karamchand Gandhi
Jallianwala Bagh Massacre (13 April 1919). A separate Ans. (b) : The Swadeshi movement started after the
Hunter Commission (1882-83) emphsised on primary Partition of Bengal in 1905. After this the concept of
and secondary education. Swarajya began to take a wide form. But the complete
16. Under the Permanent Settlement, 1793, the self-rule was still in the form of imagination. In the
Zamindars were required to issues pattas to the 1920 Ahmedabad Congress Session, Hasrat Mohani
farmers which were not issues by many of the proposed to make the Swarajya the goal of the Congress
Zamindars. The reason was for the first time, but this proposal was unsuccessful.
(a) The Zamindars were trusted by the farmers Lahore session of 1929 (which was presided over by Pt.
Jawaharlal Nehru) passed the proposal of complete self-
(b) There was no official check upon the
government.
Zamindars
(c) It was the responsibility of the British 19. Who among the following organized the
government famous Chittagong armoury raid?
(d) The farmers were not interested in getting (a) Laxmi Sehgal (b) Surya Sen
pattas (c) Batukeshwar Datta (d) J.M. Sengupta
Ans. (b) : John Shore planned Permanent Settlement Ans. (b) : The Chittagong armoury raid was an attempt
and it was introduced in 1793 by Lord Cornwallis. on 18th April, 1930, to hold the armoury of police and
Zamindars were made the owners of the land and the
auxiliary forces from the Chittagong armoury in the
British got a fixed share of 10/11th of the revenue
collected by the zamindars. Bengal presidency. It was organised by Surya Sen and
There was no official checkup on the Zamindars so they his followers.
did not issued pattas to farmers. 20. A London branch of the All India Muslim
17. Consider the following statements: League was established in 1908 under the
1. Arya Samaj was founded in 1835. presidency of:
2. Lala Lajpat Rai opposed the appeal of (a) Aga Khan (b) Ameer Ali
Arya Samaj to the authority of Vedas in (c) Liaquat Ali Khan (d) M.A. Jinnah
support of its social reform programmes. Ans. (b) : In 1908, a branch of the Muslim League was
3. Under Keshab Chandra Sen, the Brahmo established in London under the chairmanship of Ameer
Samaj campaigned for women's education.
Ali.
4. Vinoba Bhave founded the Sarvodaya
Samaj to work among refugees. Which of 21. Who among the following, was the President of
these statements are correct? the All-India States Peoples Conference in
Which of these statements are correct? 1939?

IAS (Pre) GS 2001 Paper I 404 YCT


(a) Jaya Prakash Narayan List I List II
(b) Jawahar Lal Nehru (Institute) (Location)
(c) Sheikh Abdullah A. Central Institute of 1. Chandigarh
(d) Sardar Vallabhhai Patel Medicinal &
Ans. (b) : There were about 562 princely states in India. Aromatic Plants
Among them Hyderabad was the largest and Bilawari was
the smallest native princely state. B. Centre of DNA Finger 2. Hyderabad
All-India States Peoples Conference was established in Printing and
1920 in various princely states. In 1927 an All-India States Diagnostics
Peoples Conference was organized. At the Lahore session
C. Institute of Microbial 3. New Delhi
of the Congress, Jawaharlal Nehru announced a clear
policy towards indigenous settlements that the Indian Technology
princely states would not be separated from the rest of D. National Institute of 4. Lucknow
India and would have the right to determine their future. In Immunology
1939, the All-India States Peoples Conference was A B C D
presided over by Pt. Jawaharlal Nehru. (a) 2 4 1 3
22. Who amongst the following Englishmen, first (b) 4 2 1 3
translated Bhagvad-Gita into English? (c) 2 4 3 1
(a) William Jones (d) 4 2 3 1
(b) Charles Wilkins
Ans. (b) : The Central Institute of Medicinal and
(c) Alexander Cunningham
Aromatic Plants is located in Lucknow, Centre for DNA
(d) John Marshall
Finger Printing and Diagnostics is located in Hyderabad,
Ans. (b) : Charles Wilkins was a member of Asiatic Institute of Microbial Technology is located in Chandigarh
Society of Bengal founded by William Jones. He and National Institute of Immunology is located in New
translated Bhagavad Gita into English in 1794. Delhi.
25. The approximate age of the Aravallis range is:
INDIAN GEOGRAPHY
(a) 370 million years (b) 470 million years
23. (c) 570 million years (d) 670 million years
Ans. (a) : Geologists say that the mountain range is
as old as 370 million years which is older than the
Himalayan Range. Thus, making it the oldest range of
fold Mountains in India.
26.

The annual agricultural production of a


product for the period 1991-92 to 1998-99 is
shown in the figure given above. Which one of
the following is the product in question? In the above map, the black maps show the
(a) Pulses (b) Wheat distribution of
(c) Oilseeds (d) Rice (a) Asbestos (b) Gypsum
Ans. (d) : In the given bar diagram the annual (c) Limestone (d) Mica
production of “Rice” for the period 1991–92 to 1998–99 Ans. (d) : In the given figure of India, Mica is found in
is shown. all the states of India like Jharkhand, Orissa, Bihar,
24. Match List I with List II and select the correct Rajasthan, Tamil Nadu, Karnataka, West Bengal and
answer using the codes given below the lists: Andhra Pradesh.

IAS (Pre) GS 2001 Paper I 405 YCT


27. (a) Chandigarh
(b) Daman and Diu
(c) Dadra and Nagar Haveli
(d) Pondicherry
Ans. (c) : When the question was asked it was a map of
Union Territory of Dadra and Nagar Haveli. Currently
the UTs of Daman and Diu and Dadra and Nagar Haveli
are merged to create UT of Dadra and Nagar Haveli and
Daman and Diu.
In the Shaded area of the above map, the mean
temperature for the month of July varies WORLD HISTORY
between
(a) 22.5ºC–25.0ºC (b) 25.0ºC–27.5ºC 30. Match List I with List II and select the correct
(c) 27.5ºC–30.0ºC (d) 30.0ºC–32.5ºC answer using the code given below the lists:
List I List II
Ans. (b) : The shaded area displayed in the map shows
(Country) (President)
the change in temperature between 25 ° - 27.5 ° C for
A. Columbia 1. Vicente Fox
the month of July.
B. Philippines 2. Hugo Chavez
28. The location of the space organisation units C. Mexico 3. Gloria Macapagal
have been marked in the given map as 1, 2, 3 Arroya
and 4. Match these units with the list given
D. Venezuela 4. Andres Pastrana
below and select the correct answer using the
A B C D
codes given below:
(a) 4 3 2 1
List
(b) 3 4 2 1
A. I.S.R.O. (c) 4 3 1 2
B. I.I.R.S. (d) 3 4 1 2
C. N.R.S.A. Ans. (c) : Columbia is situated in North-Western part of
South America whose president was Andres Pstrana
D. S.A.C. from 1998-2002. Gloria Macapagal Arroyo became
President of Philippines on 20th January 2001. Mexico
A B C D is situated in North America whose president was
(a) 4 1 2 3 Vincent Fox whereas the president of Venezuela was
(b) 1 4 3 2 Hugo Chavez.
(c) 1 4 2 3
31. Consider the following statements:
(d) 4 1 3 2
1. Most magmas are a combination of liquid,
Ans. (d) : ISRO is situated in Banglore, which is solid and gas.
marked as '4', Indian Institute of Remote Sensing (IIRS)
2. Water vapour and carbon dioxide are the
is located in Dehradun which is marked as '1', National
principle gases dissolved in a Magma.
Remote Sensing Agency (NRSA) is situated in
Hyderabad, marked as '3' and Space Application Centre 3. Basaltic magma is hotter than the silicic
(SAC) is situated in Ahmedabad which is marked as '2' magma.
in the map. 4. The magma solidified between sedimentary
29. rocks in a horizontal position is known as
like.
Which of these statements are correct?
(a) 1, 2 and 3 (b) 2, 3 and 4
(c) 1 and 4 (d) 1, 2 and 4
Ans. (a) : The basic composition of Magma released
because of volcanic eruption is liquid, solid and gas.
Water vapor and carbon di-oxides are principal gases
dissolved in magma. Basaltic Magma is hotter than the
Silicic magma. However horizontal sedimentation of
magma is called sill, Dyke is vertical sedimentation of
The above map is the Union Territory of: Magma.

IAS (Pre) GS 2001 Paper I 406 YCT


32. Consider the following statements regarding (a) Equator (b) Tropic of Cancer
the earthquakes: (c) South Pole (d) North Pole
1. The intensity of earthquake is measured on Ans. (a) : If the stars are seen to rise perpendicular to
Mercalli Scale. the horizon by an observer then the observer is at the
2. The magnitude of an earthquake is a equator because celestial equator is an imaginary circle
measure of energy released. around the sky directly above the earth's equator. It is
always 90º from the pole. All the stars rotate in a path
3. Earthquake magnitudes are based on direct
that is parallel to the celestial equator.
measurements of the amplitude of seismic
36. The high density of population in Nile Valley
waves.
and Island of Java is primarily due to:
4. In the Richter Scale, each whole number
(a) Intensive agriculture
demonstrates a hundredfold increase in the
amount of energy released. (b) Industrialization
Which of these statements are correct: (c) Urbanization
(a) 1, 2 and 3 (b) 2, 3 and 4 (d) Topographic constraints
(c) 1 and 4 (d) 1 and 3 Ans. (a) : The Nile is the main river of Africa. It is
known as the Nile River after the white and blue Nile
Ans. (a) : On a ritcher scale each increase of one unit on
rivers meet at Khatun in Sudan. The Nile River is
the scale represents a Ten fold increase in the magnitude of
called the boon of Egypt because of its fertile
an earthquake. Each increase of one unit also represent 31
agricultural land characteristic of the Nile region. Most
times more energy than the previous whole number on the
th of Africa's population is inhabited in the areas around
scale. Hence 4 statement is incorrect. Rest of the
the Nile River.
statements are correct.
The island of Java, which is close to the equator, also
33. Cloudy nights are warmer compared to clear
receives high temperatures as well as high rainfall. As a
cloudless nights, because clouds:
result, the agricultural land here is also fertile.
(a) Prevent cold waves from the sky from
37. Consider the following statements made about
descending on earth
the sedimentary rocks:
(b) Reflect back the heat given off by earth
1. Sedimentary rocks are formed at earth's
(c) Produce heat and radiate it towards earth
surface by the hydrological system.
(d) Absorb heat from the atmosphere and send it
2. The formation of sedimentary rocks involves
towards earth.
the weathering of pre existing rocks.
Ans. (b) : Cloudy nights are warmer compared to clear 3. Sedimentary rocks contain fossils.
cloudless nights, because clouds Reflect back the heat 4. Sedimentary rocks typically occur in layers.
given off by earth. At night cloud cover has the opposite Which of these statements are correct?
effect. If skies are clear, heat emitted from the earth's (a) 1 and 2 (b) 1 and 4
surface freely escapes into space, resulting in colder (c) 2, 3 and 4 (d) 1, 2, 3 and 4
temperatures. However, if clouds are present, some of Ans. (d) : Sedimentary rocks are formed by aggregation
the heat emitted from the earth's surface is trapped by of sediments. Due to the formation of different layers of
the clouds and reemitted back towards the earth. sediments, they are called flattened rocks. Sedimentary
34. Which one of the following weather conditions is rocks are composed in a certain order.
indicated by a sudden fall in barometer reading? They are formed by aggregation of rock powder, fauna and
(a) Stormy weather flora. Hence fossils are found in sedimentary rocks.
Sedimentary rocks cover 75% of the Earth's surface.
(b) Calm weather
Most of the mineral and petroleum products are found in
(c) Cold & dry weather these rocky areas.
(d) Hot & sunny weather
38. Volcanic eruptions do not occur in the:
Ans. (a) : Sudden fall in barometer reading indicate (a) Baltic Sea (b) Black Sea
stormy weather. Storms are created when a center of low (c) Caribbean Sea (d) Caspian Sea
pressure develops with a system of high pressure Ans. (a) : Volcanic eruptions occur only in certain places
surrounding it. It may be marked by significant disruptions and do not occur randomly. This is because the Earth's crust
to normal conditions such as strong wind, tornados, hail, is broken into a series of slabs known as tectonic plates.
thunder and lightning. Sixty percent of all active volcanoes occur at the boundaries
35. If the stars are seen to rise perpendicular to the between tectonic plates. Volcanic eruptions do not occur in
horizon by an observer, he is located on the: the Baltic Sea.
IAS (Pre) GS 2001 Paper I 407 YCT
39. A class of animals known as Marsupials is a Ans. (b) : Fohn-Alpes, Simoom-Kurdistan, Santana
characteristic feature of California, Zonda-Argentina, Chinook-USA, Siracoo-Italy,
(a) Africa (b) Australia Mistral-France, Harmattan-Guinea Coast, Blizzard-Polar
(c) South America (d) South-east Asia regions, Brickfilder – Australia , Berg wind - South Africa
Ans. (b) : Marsupials are a group of mammals 42. The given maps shows four towns of Central
commonly having pouches of skin where mothers Asian region marked as 1, 2, 3 and 4. Identify
murses their pre mature babies. These marsupians are these from the following list and select the
found in Australia. An example of this is the kangaroo.
correct answer using the codes given below:
Most kangaroos in the world are found in Australia.
Town:
40. The temperature and rainfall of a meteorological
1. Bishkek 2. Ashgabat
station are given below:
3. Tashkent 4. Dushanbe
Temperature ºC Rainfall (cm)
J 9.4 12.2
F 10.6 9.1
M 11.7 7.9
A 12.2 2.5
M 13.3 1.8
J 13.9 0.3
J 13.9 – A B C D
(a) 3 1 2 4
A 14.4 –
(b) 3 1 4 2
S 15.6 0.8
(c) 1 3 2 4
O 15.0 2.5
(d) 1 3 4 2
N 13.3 6.1
D 10.6 11.7 Ans. (a) : Ashgabat is marked as '1' Tashkent is marked
as '2' Bishkek is marked as '3' Dushanbe is marked as '4' in
Average Temperature : 12.8ºc
the given map.
Average Rainfall : 54.9cm per annum
43. Assertion (A) : During the Neap Tides, the high
Identify the region having the above climatic
pattern from amongst the following: tide is lower and the low tide is higher than usual.
(a) Mediterranean region Reason (R) : The Neap Tide, unlike the Spring
(b) Monsoon region Tide, occurs on the New Moon instead of on the
(c) Steppe region Full Moon.
(d) N.W. European region (a) Both A and R are individually true and R is
Ans. (a) : The above mentioned climatic pattern the correct explanation of A
indicates the Mediterranean region with average (b) Both A and R are individually true but R is
temperature 12.8°C and average rainfall 59.9 cm. NOT the correct explanation of A
41. Match List I with List II and select the correct (c) A is true but R is false
answer using the code given below the lists: (d) A is false but R is true
List I List II Ans. (b) : The main reason for the origin of tides is the
(Local Wind) (Region) attraction power of the sun and the moon relative to the
A. Fohn 1. Argentina earth. Daily tides come once in 12 hours 26 minutes.
B. Simoom 2. Kurdistan The time between one tide and the next ebb is 6 hours
C. Santa Ana 3. California 15 minutes.
D. Zonda 4. Alps When the sun, moon and earth are in a straight line,
A B C D there is a state of great tide. In this case, the height of
(a) 2 4 1 3 the tide is maximum. The maximum time for the tide to
(b) 4 2 3 1 be at maximum height is 6 seconds.
(c) 2 4 3 1 However both (A) and (R) are individually correct, but
(d) 4 2 1 3 R does not explains A.

IAS (Pre) GS 2001 Paper I 408 YCT


44. Assertion (A) : Anticyclonic conditions are 6% of the total valid votes or 2% of the total valid votes
formed in winter season when atmospheric with 4 Lok Sabha seats in different states. There were
pressure is high and air temperatures are low. six national and 48 state level political parties during
Reason (R) : Winter rainfall in Northern India the general election of 1999.
causes development of anticyclonic conditions
47. Match List I with List II and select the correct
with low temperatures.
answer using the codes given below the lists:
(a) Both A and R are individually true and R is
the correct explanation of A List I List II
(b) Both A and R are individually true but R is (Article of the (Content)
NOT the correct explanation of A Constitution)
(c) A is true but R is false A. Article 54 1. Election of the
(d) A is false but R is true President of India
Ans. (c) : Anti-cyclonic condition develops due to
winter rainfall so, they are formed when atmospheric
B. Article 75 2. Appointment of the
pressure is high and air temperature is low.
Prime Minister and
In northern India winter rainfall occur due to
Council of
anticyclone and the condition of anticyclone already
Ministers
exist here.
C. Article 155 3. Appointment of the
So reason (R) is wrong hence option c is correct answer.
Governor of a state
INDIAN CONSTITUTION AND POLITY D. Article 164 4. Appointment of the
Chief Minister and
45. If a new state of the Indian Union is to be Council of
created, which one of the following schedules of Ministers of a state
the Constitution must be amended? 5. Composition of
(a) First (b) Second Legislative
(c) Third (d) Fifth Assemblies
Ans. (a) : The first schedule will have to be amended to A B C D
create a new territory. Article 2 and 3 of the Indian (a) 1 2 3 4
Constitution provides for it. (b) 1 2 4 5
46. Consider the following statements regarding (c) 2 1 3 5
the political parties in India: (d) 2 1 4 3
1. The Representation of the People Act, 1951 Ans. (a) : The correct match of this is option (a).
provides for the registration of political 48. Which Article of the Constitution provides that it
parties. shall be the endeavor of every state to provide
2. Registration of political parties is carried adequate facility for instruction in the mother
out by the Election Commission tongue at the primary state of education?
3. A national level political party is one which (a) Article 349 (b) Article 350
is recognised in four or more states. (c) Article 350 A (d) Article 351
4. During the 1999 general elections, there Ans. (c) : Article 350A states that it shall be the
were six national and 48 state level political
endeavour of every State and of every local authority
parties.
within the State to provide adequate facilities for
Which of these statements are correct?
instruction in the mother-tongue at the primary stage of
(a) 1, 2 and 4 (b) 1 and 3
education to children belonging to linguistic minority
(c) 2 and 4 (d) 1, 2, 3 and 4
groups; and the President may issue such directions to
Ans. (d) : According to the Representation of the any State as he considers necessary or proper for
People Act, 1951, registration of every political party is securing the provision of such facilities.
necessary and this registration is done by the Election
49. The Supreme Court of India tenders advice to
Commission. The earlier definition for being a national
the President on a matter of law or fact:
level party was to be recognized in four or more states
(a) On its own initiative
but at present some changes have been made in its
definition. A party can become a national party if it has (b) Only if he seeks such advice

IAS (Pre) GS 2001 Paper I 409 YCT


(c) Only if the matter relates to the Fundamental Ans. (a) : The Parliament of India carries out its
Rights of citizens functions through the Parliamentary Committees.
(d) Only if the issue poses a threat to the unity These are of two types - Regular Committees and Select
and integrity of the country Committees to be appointed every year.
Presently 24 standing parliamentary committees are
Ans. (b) : Article 143 of the Indian Constitution confers
functioning.
upon the Supreme Court’s advisory jurisdiction. The
President may seek the opinion of the Supreme Court 53. In which one of the following areas does the
State Government NOT have control over its
on any question of law or fact of public importance on
local bodies?
which he thinks it expedient to obtain such an opinion.
(a) Citizens grievances
50. Which one of the following duties is NOT (b) Financial matters
performed by the Comptroller and Auditor (c) Legislation
General of India? (d) Personnel matters
(a) To audit and report on all expenditure from Ans. (a) : The 11th Schedule was added to the
the Consolidated Fund of India Constitution by the 73rd Constitutional Amendment. It
(b) To audit and report on all expenditure from included provisions related to Panchayati Raj Institutions.
the Contingency Funds and Public Accounts In this, state governments have been given authority over
(c) To audit and report on all trading, local bodies over certain areas. Among them, personnel,
manufacturing, profit and loss accounts law, economic sector etc. are prominent.
(d) To control the receipt and issue of public 54. Consider the following statements regarding
the High Courts in India:
money, and to ensure that the public revenue
1. There are eighteen High Courts in the
is lodged in the exchequer
Country.
Ans. (d) : Under Article 148, there will be a Comptroller 2. Three of them have jurisdiction over more
and Auditor General of India, appointed by the President. than one state.
Article 149 describes the powers and functions of the 3. No Union Territory has a High Court of its
Comptroller and Auditor General. own
In India CAG only audits the accounts while it has no 4. Judges of the High Court hold office till the
control on receipt and issue of public money. age of 62.
51. Which one of the following statements correctly Which of these statement is/are correct?
describes the Fourth Schedule of the (a) 1, 2 and 4 (b) 2 and 4
Constitution of India? (c) 1 and 4 (d) 4 only
(a) It lists the distribution of powers between the Ans. (d) : There are a total of 21 High Courts in the
Union and the states. country. 6 of them have jurisdiction over more than one
state. The Union Territory of Delhi also has its own
(b) It contains the languages listed in the High Court. High Court judges can continue in office
Constitution for 62 years. Hence it is clear that only statement 4 is
(c) It contains the provisions regarding the correct.
administration of tribal areas Note- Currently (2021) the number of High Courts in
(d) It allocates seats in the Council of States India is 25.
Ans. (d) : The Fourth Schedule of the Constitution 55. Match List I with List II and select the correct
deals with the allocation of seats in the Rajya Sabha to answer using the codes given below the lists:
the States and Union Territories. The allocation is based List I List II
upon the population of states and UTs so far all states (Amendments (Contents)
and only Delhi and Puducherry are the Territories to the
Constitution)
having representation in Rajya Sabha.
A. The Constitution 1. Establishment of
52. In what way does the Indian Parliament (sixty-ninth state level Rent
exercise control over the administration? Amendment) Tribunals
(a) Through Parliamentary Committees Act, 1991
(b) Through Consultative Committees of various
B. The Constitution 2. No reservations
ministries
(Seventh-fifth for Scheduled
(c) By making the administration send periodic
Amendment) Castes in
reports Act. 1994 Panchayats in
(d) By compelling the executive to issue writs Arunachal Pradesh
IAS (Pre) GS 2001 Paper I 410 YCT
C. The Constitution 3. Constitution of Ans. (d) : Internal debt is that part of the total debt that is
(Eightieth Panchayats in owned to lenders within the country. It is the money that
Amendment) Villages or at government borrows from its own citizens. The
Act, 2000 other local levels government borrows by issuing the Government bonds,
D. The Constitution 4. Accepting the treasury bills and special securities issued to RBI. When
(Eighty-third recommendations Government borrows from the domestic sources, the
Amendment) of the Tenth increase in inflation is less in comparison to simply
Act, 2000 Finance printing the money.
Commission 58. Consider the following statements regarding
5. According the Reserve Bank of India:
status of National 1. It is a banker to the Central Government.
Capital Territory 2. It formulates and administers monetary
to Delhi
policy.
A B C D 3. It acts as an agent of the Government in
(a) 5 1 4 2 respect of India's membership of IMF.
(b) 1 5 3 4 4. It handles the borrowing programme of
(c) 5 1 3 4 government of India.
(d) 1 5 4 2 Which of these statements are correct?
Ans. (a) : 69th Constitution Amendment - 1991 - Delhi (a) 1 and 2 (b) 2, 3 and 4
was made the National Capital Territory (c) 1, 2, 3 and 4 (d) 3 and 4
75th Constitution Amendment - 1994 - Establishment of
Ans. (c) : Functions of RBI: sole authority to issue
State Level Rent Tribunals
currency; government's bank; banker's bank; guardian
80th Constitution Amendment - 2000 - Accepting the
of money market; lender of the last resort; sole reservoir
recommendations of the Tenth Finance Commission
of Foreign exchange reserves; controller of credit;
83rd Constitution Amendment -1944 - Arunachal Pradesh
clearing house for settling inter bank transactions,
was exempted from reservation in Panchayati Raj
handles burrowing programme of government of India.
Institutions as there was no Scheduled Caste.
It also acts as an agent of the Government in respect of
India’s membership of IMF.
ECONOMY
59. Consider the following factors regarding an
56. The prices at which the government purchases industry:
food grains for maintaining the public 1. Capital investments 2. Business turnover
distribution system and for building up buffer-
3. Labour force 4. Power
stocks is knows as :
consumption
(a) Minimum support prices.
Which of these determine the nature and size of
(b) Procurement prices
the industry?
(c) Issue prices
(a) 1, 3 and 4 (b) 1, 2 and 4
(d) Ceiling prices
(c) 2, 3 and 4 (d) 2 and 3
Ans. (a) : Minimum Support Price is the price at which
government purchases crops from the farmers. Ans. (b) : Among various factors entrepreneurial skills,
Minimum Support Price is an important part of India's availability of finance turnover and power consumption
agricultural price policy. The MSP helps to incentivize etc. decides the natural and size of an industry.
the farmers and thus ensures adequate food grains 60. The most appropriate measure of a country's
production in the country. economic growth is its:
57. Consider the following (a) Gross Domestic Product
1. Market borrowing
(b) Net Domestic Product
2. Treasury bills
(c) Net National Product
3. Special securities issued to RBI
(d) Per Capita Real Income
Which of these is/are component (s) of internal
debt? Ans. (d) : Per capita real income is nothing but NNP at
(a) 1 only (b) 1 and 2 factor cost. It means national income is sum total of all
(c) 2 only (d) 1, 2 and 3 factor incomes adjusted for increase in prices.

IAS (Pre) GS 2001 Paper I 411 YCT


61. Match List I with List II and select the correct (a) 1 only (b) 2 and 4
answer using the codes given below the lists: (c) 1 and 3 (d) 2 and 3
List I List II Ans. (b) : Indirect taxes are the charges that are levied
(Term) (Explanation) on a taxpayer for goods and services. Some of the
A. Fiscal deficit 1. Excess of total significant indirect taxes include Value Added Tax,
Expenditure over Central Sales Tax, Central Excise Duty, Customs Duty,
Total Receipts stamp duties and expenditure tax. Property tax,
B. Budget deficit 2. Excess of Revenue Corporation tax and Wealth tax are examples of direct
Expenditure over taxes.
Revenue Receipts 64. The largest share of Foreign Direct Investment
C. Revenue 3. Excess of Total (1997-2000) went to:
deficit Expenditure over (a) Food and food-product sector
Total Receipts less (b) Engineering sector
borrowings (c) Electronics and electric equipment sector
D. Primary 4. Excess of Total (d) Services sector
deficit Expenditure over Ans. (d) : The largest share of Foreign Direct Investment
Total Receipts less (1997-2000) went to the Services sector.
borrowings & 65. Match List I with List II and select the correct
Interest Payments answer using the codes given below the lists
A B C D List I List II
(a) 3 1 2 4 (Publisher) (Publication)
(b) 4 3 2 1 A. Ministry of 1. Report on
(c) 1 3 2 4 Industry Currency and
(d) 3 1 4 2 Finance
Ans. (a) : A. Fiscal deficit - Excess of total expenditure B. Central 2. Economic Survey
over total receipts less borrowings Statistical
B. Budget deficit - Excess of total expenditure over Organisation
receipts. C. Reserve Bank of 3. Wholesale Price
C. Revenue deficit - Excess of revenue expenditure. India Index
D. Primary deficit - Excess of total expenditure over D. Ministry of 4. National Accounts
total receipts less borrowings and interest payments. Finance Statistics
A B C D
62. Consider the following organisations:
(a) 4 3 2 1
1. International Bank for Reconstruction and
(b) 3 4 1 2
Development.
(c) 4 3 1 2
2. International Finance Corporation.
(d) 3 4 2 1
3. International Fund for Agricultural
Ans. (b) : Ministry of Industry – Wholesale Price
Development.
Index
4. International Monetary Fund.
Central Statistical Organisation – National Accounts
Which of these are agencies of the United Nations?
Statistics
(a) 1 and 2 (b) 2 and 3
Reserve Bank of India – Report on Currency
(c) 3 and 4 (d) 1, 2, 3 and 4
and Finance
Ans. (d) : At present the UNO has in total 15
Ministry of Finance – Economic Survey
specialised agencies that carry out various functions on
behalf of the UN all the given agencies are associated 66. Which of the following committees examined
with United Nations Organization. and suggested Financial Sector reforms?
63. Consider the following taxes: (a) Abid Hussain Committee
1. Corporation tax 2. Customs duty (b) Bhagwati Committee
3. Wealth tax 4. Excise duty (c) Chellaih Committee
Which of these is/are indirect taxes? (d) Narsimhan Committee
IAS (Pre) GS 2001 Paper I 412 YCT
Ans. (d) : Abid Hussain Committee - 1997 on Small (c) A is true but R is false
Scale Industries (d) A is false but R is true
Mahajan Committee - Sugar Industry 1997 Ans. (c) : This market where foreign exchange practices
Bhagwati Committee - Unemployment take place, the foreign exchange market and the rate at
Chelliah Committee - Tax Reforms which it happens is called the foreign exchange rate. It
Narasimham Committee - Financial Reforms varies in different regions.
67. The new series of Wholesale Price Index (WPI) In 2000, the foreign exchange rate expired on current
released by the Government of India is with account transactions.
reference to the base prices of: Presently, there is a steady increase in the foreign exchange
(a) 1981-82 (b) 1990-91 reserves.
(c) 1993-94 (d) 1994-95
Ans. (c) : When the question was asked Base year for PHYSICS
WPI was 1993-94. Earlier 1981-82 was being used as
the base year currently it is 2011-12. 71. When light waves pass from air to glass, the
68. The term National Income represents: variables affected are:
(a) Gross national product at market prices minus (a) Wavelength, frequency and velocity
depreciation (b) Velocity and frequency
(b) Gross national product at market prices minus (c) Wavelength and frequency
depreciation plus net factor income from (d) Wavelength and velocity
abroad
Ans. (d) : Upon refraction from one medium to another,
(c) Gross national product at market prices minus
depreciation and indirect taxes plus subsidies only the wavelength and velocity change, while
(d) Gross national product at market prices minus frequency remains the same.
net factor income from abroad Option D is the correct one.
Ans. (c) : Net National Product at Factor Cost (NNPFC) 72. When water is heated from 0º C to 10º C, its
is called the national income of a country. volume
Net national product = (GDP + net income earned from (a) Increases
foreign countries). (b) Decreases
69. Assertion (A) : There was an increase in (c) Does not change
industrial production during 1999-2000. (d) First decreases and then increases
Reason (R) : The period witnessed a stable Ans. (d) : The highest density of water is at 4°C. Unusual
exchange rate and improved business sentiments. spread is seen in water. On heating the water, its volume
(a) Both A and R are individually true and R is decreases to 0°C to 4 °C. Thereafter, it starts increasing.
the correct explanation of A 73. Match List I with List II and select the correct
(b) Both A and R are individually true but R is answer using the code given below the lists:
NOT the correct explanation of A List I List II
(c) A is true but R is false (Characteristic) (Particle)
(d) A is false but R is true A. Zero mass 1. Positron
Ans. (a) : The development of industries has started in B. Fractional charge 2. Neutrino
India only after 1980. The rapid economic growth that C. Fractional spin 3. Quark
began after the 1991 economic liberalization is still on D. Integral spin 4. Photon
the path. The industrial growth rate during 1999-02 was A B C D
6.7%. (a) 4 3 1 2
70. Assertion (A) : Ceiling on foreign exchange for a (b) 3 2 4 1
host of current account transaction heads was (c) 2 3 4 1
lowered in the year 2000. (d) 3 2 1 4
Reason (R) : There was a fall in foreign currency Ans. (a) : Quarks have fractional electric charge values–
assets also. either 1⁄3 or 2⁄3 times the elementary charge. The positron
(a) Both A and R are individually true and R is has an electric charge of +1e, a spin of 1/2, and has the
the correct explanation of A same mass as an electron. A neutrino is an electrically
(b) Both A and R are individually true but R is neutral, weakly interacting elementary subatomic particle
NOT the correct explanation of A with half integer spin.
IAS (Pre) GS 2001 Paper I 413 YCT
74. The mass of a body on earth is 100 kg 77. Which one of the following distance-time graph
(acceleration due to gravity, go = 10 m/s2). If (x-t) represents one-dimensional uniform motion?
acceleration due to gravity on the moon is (a) (b)
(ge/6), then the mass of the body on the moon is
(a) 100/6 kg (b) 60 kg
(c) 100 kg (d) 600 kg
Ans. (c) : Mass is a universal constant. The mass of a
body remains unchanged in any part of universe. Mass (c) (d)
of a body does not change with respect to gravity. It is
the weight that changes with gravity.
75. Who amongst the following was the first to
state that the earth was spherical?
(a) Aristotle (b) Copernicus Ans. (d) : In one dimensional uniform motion equal
(c) Ptolemy (d) Strabo distance is travelled at equal interval of time. So
Ans. (a) : Aristotle in 340 B.C first stated that the earth velocity is constant and hence acceleration is zero. So
was spherical in his book “On the Heaven”. graph (d) represents one dimensional uniform motion.
76. Consider the following statements: 78. Assertion (A) : A stick is dipped in water in a
A simple pendulum is set into oscillation. Then slanting position. If observed sideways, the stick
1. The acceleration is zero when the bob passes appears short and bent at the surface of water.
through the mean position. Reason (R) : The light coming from the stick
2. In each cycle the bob attains a given velocity undergoes scattering from water molecules giving
twice. the stick a short and bent appearance.
3. Both acceleration and velocity of the bob are (a) Both A and R are individually true and R is
zero when it reaches its extreme position
the correct explanation of A
during its oscillation.
4. The amplitude of oscillation of the simple (b) Both A and R are individually true but R is
pendulum decreases with time. NOT the correct explanation of A
Which of these statements are correct? (c) A is true but R is false
(a) 1 and 2 (b) 3 and 4 (d) A is false but R is true
(c) 1, 2 and 4 (d) 2, 3 and 4 Ans. (c) : When a ray of light passes from one medium
Ans. (c) : to another, it deflects from its path. When it goes from
sparse to dense medium, it goes towards the normal and
away from the opposite position.

CHEMISTRY
79. Which one of the following is not radioactive?
(a) Astatine (b) Francium
The bob of a simple pendulum is attached to a string which
(c) Tritium (d) Zirconium
pulls the bob along its length. Here B and C are extreme
positions whereas A is mean position. The speed of the Ans. (d) : Out of the given options zirconium is not
bob increases as it approaches the mean position A and radioactive. Generally elements which have atomic
continues to move till it reaches C. At C the speed number above 80, shows radioactivity. However Tritium
becomes zero. Due to the unbalanced force the bob moves (atomic no. 1) is radioactive isotope of hydrogen due to
towards the mean position. The speed of the bob is presence of a high number of neutrons compared to the
maximum at the mean position and is zero at the extreme number of protons.
positions. Thus, it is clear that in each cycle bob velocity
increases from zero to maximum. This means that it attains 80. Consider the following.
a given velocity twice. Suppose the bob of the pendulum In a nuclear reactor, self-sustained chain reaction
reaches up to B while oscillating, then AB is the is possible, because.
amplitude. For the next oscillation the bob fails to reach B 1. More neutrons are released in each of the
but it will reverse the direction from point BC' instead of fission reactions.
B. The amplitude of oscillation in the second case is ABC' 2. The neutrons immediately take part in the
which is less than AB. That means, a retarding force is fission process.
acting on the bob thereby reducing the amplitude of 3. The fast neutrons are slowed down by
oscillation. This retarding force is nothing but air- Graphite.
resistance or air-friction. At extreme position, acceleration 4. Every neutron released in the fission reaction
is maximum. So statement 3 is not correct. initiates further fission.
IAS (Pre) GS 2001 Paper I 414 YCT
Which of these statements are correct? 1. Ionisation potential gradually decreases
(a) 1, 2 and 3 (b) 1 and 3 along a period.
(c) 2 and 4 (d) 2, 3 and 4 2. In a group of elements, electron affinity
decreases as the atomic weight increases.
Ans. (b) : Self-Sustainable Chain Reaction is possible
3. In a given period, electronegativity decreases
in a nuclear reactor because each fission reaction has
as the atomic number increases
relatively more neutrons released and is slowed down
Which of these statements(s) is/are correct?
by rapid neutron graphite.
(a) 1 only (b) 2 only
81. Which one of the following is the correct sequence (c) 1 and 3 (d) 2 and 3
in increasing order of molecular weights of the Ans. (b) : In a periodic table, in a group (vertical
hydrocarbons? column) of element, electron affinity decreases as
(a) Methane, ethane, propane and butane atomic weight increases. In a given period (horizontal
(b) Propane, butane, ethane and methane row) electronegativity (tendency of atom to acquire
(c) Butane, ethane, propane and methane negative charge) increases with increase in atomic
(d) Butane, propane, ethane and methane number. Ionisation potential (the energy required to
remove an electron) gradually increases along a period.
Ans. (a) : The molecular formulas and weights for
85. A radioactive substance has a half-life of four
given hydrocarbons can be written as: months. Three-fourth of the substance would
Methane; CH4, Molecular Weight; 16 decay in:
Ethane; C2H6, Molecular weight; 30 (a) 3 months (b) 4 months
Propane; C3H8, Molecular weight; 44 (c) 8 months (d) 12 months
Butane:C4H10, Molecular weight: 58 Ans. (c) : As it is given that half life of given substance
Thus, the increasing order of molecular weights can be is 4 months.
written as; Methane, ethane, propane and butane. The amount of substance left after 4 months = 1/2
82. In an atom, the order of filling up of the orbitals The amount of substance left after 8 months = 1/4
Therefore the amount of substance decay in 8 months =
is governed by:
(1 – 1/4) = 3/4
(a) Aufbau principle
86. Quartzite is metamorphosed from:
(b) Heisenberg's uncertainty principle
(a) Limestone (b) Obsidian
(c) Hund's rule (c) Sandstone (d) Shale
(d) Pauli's exclusion principle Ans. (c) : Quartzite is metamorphosed from Sandstone.
Ans. (a) : Aufbau principle states that in the ground Quartzite is a nonfoliated metamorphic rock composed
state of the atom, the orbitals are filled in order of their almost entirely of quartz. It forms when a quartz-rich
increasing energies, starting with the orbital of lowest sandstone is altered by the heat, pressure, and chemical
energy. The word Aufbau is German word which means activity of metamorphism. These conditions recrystallize the
building up. The increasing order of energy and hence sand grains and the silica cement that binds them together.
that of filling of orbitals is as follows: 1s, 2s, 2p, 3s, 3p, 87. Assertion (A) : A piece of copper and a piece of
4s, 3d, 4p, 5s, 4d, 5p, 6s, 4f, 5d, 6p. glass are heated to the same temperature. When
touched, thereafter, the copper piece appears
83. An aqueous solution of copper sulphate is
hotter than the glass piece.
acidic in nature because the salt undergoes:
Reason (R) : The density of cooper is more than
(a) Dialysis (b) Electrolysis that of glass.
(c) Hydrolysis (d) Photolysis (a) Both A and R are individually true and R is
Ans. (c) : Aqueous solution of copper sulphate is acidic the correct explanation of A
in nature because copper sulphate on dissolving with (b) Both A and R are individually true but R is
water, following chemical reaction takes place NOT the correct explanation of A
CuSO4(s) + 2H2O - Cu(OH)2(l) + H2SO4(aq) Now as (c) A is true but R is false
we can see in above chemical equation the sulphuric (d) A is false but R is true
acid generated is strong mineral acid which results into Ans. (b) : Although the density of copper (8920–8960
increase in acidity of solution. Thus aqueous solution of Kg / m3) is greater than the density of glass (2400–2800
copper sulphate is acidic in nature. Kg / m3), its conductivity is due to the copper being
84. Consider the following statements with warmer. Hence both the statement and the reason are
reference to the Periodic Table of chemical true but the reason is not the correct explanation of the
elements: statement.
IAS (Pre) GS 2001 Paper I 415 YCT
88. Assertion (A) : A chemical reaction becomes A B C D
faster at higher temperatures. (a) 4 1 3 2
Reason (R) : At higher temperatures, molecular (b) 1 4 3 2
motion becomes more rapid. (c) 1 4 2 3
(a) Both A and R are individually true and R is (d) 4 1 2 3
the correct explanation of A Ans. (d) : Sternum is the breast bone. Clavicle is the
(b) Both A and R are individually true but R is collar bone. Patella is the knee cap. Scapula is the shoulder
NOT the correct explanation of A blade.
(c) A is true but R is false
92. Consider the following statements:
(d) A is false but R is true
1. Tapeworm is a hermaphrodite
Ans. (a) : Temperature is the amount of kinetic energy 2. Round-worm has separate sexes.
possessed by the particles of an element and a reaction 3. Filaria is caused by a nematode.
occurs when two atoms or particles collide with each
4. Guinea-worm is and annelid.
other. For example, when an atom of Na collides with
Which of these are correct?
an atom of Cl, NaCl is formed. Now, when the
(a) 1 and 2 (b) 1, 2 and 3
temperature raises, the particles have more -kinetic
energy and thus move faster. Moving faster implies that (c) 3 and 4 (d) 2, 3 and 4
there will be more collisions per seconds Thus, the Ans. (b) : Tape worms are hermaphrodite (both male
reaction occurs faster. Hence, Assertion and Reason and female reproductive organ present).
both are correct and R is the correct explanation of A. Round worms are unisexual (separate sex).
Filaria is caused by Wuchereria bancrofti which is
BIOLOGY a parasitic nematode.
Guinea worm is a long and very thin nematode, not
89. The American multinational company, Monsanto an annelid.
has produced an insect-resistant cotton variety 93. Which organelle in the Cell, other than
that is undergoing field-trials in India. A toxin nucleus, contains DNA?
gene from which ONE of the following bacteria (a) Centriole (b) Golgi apparatus
has been transferred to this transgenic cotton?
(c) Lysosome (d) Mitochondrion
(a) Bacillus : substilis
Ans. (d) : Mitochondria contain DNA other than
(b) Bacillus : thuringiensis
nucleus. Mitochondria are thought to be the parasites in
(c) Bacillus : amyloliquijaciens the eukaryotic cells that got inside the latter as a result
(d) Bacillus : globullii of the endo-symbiotic event that took place earlier in
Ans. (b) : The cotton incorporated the cry1Ac gene from the evolution. Centriole is a small set of microtubules.
the soil bacterium Bacillus thuringiensis (Bt), making Golgi apparatus is a complex of vesicles and folded
the cotton toxic to bollworms. membranes within the cytoplasm of most eukaryotic
90. The cellular and molecular control of programmed cells involved in secretion and intracellular transport.
cell death is known as: Lysosome is an organelle in the cytoplasm of eukaryotic
(a) Apoptosis (b) Ageing cells containing degradative enzymes enclosed in a
(c) Degeneration (d) Necrosis membrane.
Ans. (a) : The action of apoptosis involves cellular and 94. "Athlete's Foot" is a disease caused by:
molecular control of programmed cell death. It occurs in (a) Bacteria (b) Fungus
multi cellular organism. Apoptosis is a highly regulated (c) Protozoan (d) Nematode
and controlled process that confers advantages during life Ans. (b) : Athlete's Foot disease is caused by parasitic
cycle of an organism. fungus of genus Trichophyton. Scaling, flaking and
91. Match List I with List II and select the correct itching of affected areas are the symptoms of this
answer using the code given below the lists: disease. This disease transmitted in moist areas where
List I (Bone) List II (Name) people walk bare foot.
A. Breast-bone 1. Clavicle 95. In the eye donation, which part of the eye is
B. Collar-bone 2. Patella transplanted from the donor?
C. Knee-cap 3. Scapula (a) Cornea (b) Lens
D. Shoulder blade 4. Sternum (c) Retina (d) The whole eye
IAS (Pre) GS 2001 Paper I 416 YCT
Ans. (a) : Cornea, the main focusing part, is the clear front A B C D
surface of the eye. Like a window, it allows light to enter (a) 2 3 1 5
the eye. Vision could be markedly reduced or lost if the (b) 3 4 2 5
cornea becomes cloudy or scarred. This condition is (c) 2 3 5 1
known as corneal blindness. (d) 3 1 2 1
Eye donation is an act of donating one’s eyes after his/her Ans. (a) : The correct match is as follows:
death. Only corneal blinds can be benefited through this List-I List-II
process not other blinds. (Substance) (Physiological role)
Cornea transplant is the surgical procedure which replaces 1. Ptyaline A. Digests starch
a disc-shaped segment of an impaired cornea with a 2. Pepsin B. Breakdown proteins into
similarly shaped piece of a healthy donor cornea. smaller peptides i.e. digest
96. A man whose blood group is not known meets proteins
with a serious accident and needs blood 3. Renin C. Convert angitensinogen in
transfusion immediately. Which one of the blood blood into angiotensis
groups mentioned below and readily available in 4. Oxytocin D. Induces contraction of
the hospital will be safe for transfusion? smooth muscles.
(a) O, Rh– (b) O, Rh+ 99. Epiphytes are plants which depend on other

(c) AB, Rh (d) AB, Rh+ plants for:
Ans. (a) : There is no A and B antigen on RBC of “O– (a) food (b) mechanical support
”bloodgroup , So O, Rh– is known as universal blood (c) shade (d) water
donor. Ans. (b) : They are an important source of food for
97. "Metastasis" is the process by which: many species. Typically, the older parts of a plant will
(a) Cells divide rapidly under the influence of have more epiphytes growing on them. Epiphytes differ
drugs from parasites in that they grow on other plants for
(b) Cancer cells spread through the blood or physical support and do not necessarily affect the host
lymphatic system to other sites or organs negatively.
(c) The chromosomes in cell nuclei are attached 100. Antigen is a substance which:
to the spindle before moving to the anaphase (a) Destroys harmful bacteria
poles (b) Is used to treat poisoning
(d) Cancer cells are successfully inhibited to (c) Lowers body-temperature
divide any further (d) Stimulates formation of antibody
Ans. (b) : Cancer cells spread through the blood or Ans. (d) : Antigens are glycoproteins found in RBCs.
lymphatic system to other sites or organs. Due to this, there is a difference in human blood groups.
"Metastasis" the spread of cancer cells from the place On the basis of presence of antigen, four types of blood
where they first formed to another part of the body. In groups are found in humans - O, A, B, AB.
metastasis, cancer cells break away from the original
(primary) tumor, travel through the blood or lymph 101. Which of the following features of DNA makes
system, and form a new tumor in other organs or tissues it uniquely suited to store and transmit genetic
of the body. information from generation to generation?
(a) Complementarily of the two strands
98. Match List I with List II and select the correct
answer using the codes given below the lists: (b) Double helix
(c) Number of base-pairs per turn
List I List II
(d) Sugar-phosphate backbone
(Substance) (Physiological role)
A. Ptyalin 1. Converts Ans. (b) : Double helix makes it uniquely suited to
angiotensinogen in store and transmit genetic information from generation
blood into angiotensin to generation.
B. Pepsin 2. Digests starch 102. Which of the following cell organelles play the
most significant role in protein synthesis?
C. Renin 3. Digests proteins
(a) Lysosome and Centrosome
D. Oxytocin 4. Hydrolyses fasts
(b) Endoplasmic reticulum and Ribosome
5. Induces contraction (c) Golgi apparatus and Mitochondria
of smooth muscles
(d) Lysosome and Mitochondria
IAS (Pre) GS 2001 Paper I 417 YCT
Ans. (b) : The main function of endoplasmic reticulum is (a) 1, 2 and 4 (b) 1, 2, 3 and 4
to synthesize all the fats and proteins that make up various (c) 2 and 3 (d) 1, 3 and 4
membranes such as cell membranes, nucleus membranes, Ans. (b) : Gulf of Mannar is one of India’s 18 biosphere
etc. reserves declared so far. It is one of the world’s richest
region from a marine biodiversity perspective.
103. Assertion (A) : The boiling point of water
Ganga Action Plan Phase-II was merged with National
decreases as the altitude increases.
River Conservation Plan in December, 1996.
Reason (R) : The atmospheric pressure increases National Museum of Natural History, established in 1972
with altitude. and opened in 1978 was a museum located in New Delhi.
(a) Both A and R are individually true and R is The NMNH’s mission was to promote environmental
the correct explanation of A education. The museum was destroyed by fire on 26 April,
(b) Both A and R are individually true but R is 2016.
NOT the correct explanation of A Environmental Information System (ENVIS) is a one stop,
web enabled and comprehensive portal which provide
(c) A is true but R is false
information on environment and related subject areas.
(d) A is false but R is true
106. Identify the correct order of the processes of
Ans. (c) : The boiling point of water decreases as the soil-erosion from the following:
altitude increase. As altitude increases atmospheric (a) Splash erosion, Sheet erosion, Rill erosion,
pressure decreases. Hence option (c) is correct. Gully erosion
104. Assertion (A) : Scientists can cut apart and paste (b) Sheet erosion, Splash erosion, Gully erosion,
together DNA molecules at will, regardless of the Rill erosion
source of the molecules. (c) Rill erosion, Gully erosion, Sheet erosion,
Reason (R) : DNA fragments can be manipulated Splash erosion
using restriction endonucleases and DNA ligases (d) Gully erosion, Rill erosion, Sheet erosion,
Splash erosion
(a) Both A and R are individually true and R is
Ans. (a) : The correct order of soil erosion process is
the correct explanation of A
splash erosion, sheet erosion, rill erosion and gully erosion.
(b) Both A and R are individually true but R is Splash occurs when the raindrop hits the surface; explosive
NOT the correct explanation of A impact soil into individual particles. Sheet erosion is the
(c) A is true but R is false detachment of soil particles by rain drop and their removal
(d) A is false but R is true down slope by water flowing overland as a sheet in
Ans. (a) : The complementary properties of nucleotides definite channels. Rill erosion refers to the development of
small ephemeral concentrated flow paths which function as
of DNA are powerful tools for genetic engineering. DNA
both sedimentary source and sediment delivery systems for
or any segment of a gene can be combined with other erosion on hill slopes. Sheet erosion removes the this top
organisms' DNA using restriction endonuclease and DNA layer by raindrop Rill erosion occurs in lines which are less
lipase. Such organisms that carry external DNA are called than 30 cm deep. When these rills are deeper than 30 cm
transgenic organisms. resulting channels are called as gully erosion. Gully
erosion occurs when water flows in narrow channels after
ENVIRONMENT AND ECOLOGY heavy rain and melting snow.

105. Consider the following statements regarding CURRENT AFFAIRS


environmental issues of India:
1. Gulf of Mannar is one of the biosphere 107. The theme of Indian Science Congress, 2001
reserves. was:
(a) "Food, nutrition & environmental security"
2. The Ganga Action Plan, phase II has been
(b) "Arrest declining interest in pure sciences"
merged with the National River Conservation
(c) "Make India energy self-sufficient"
Plan.
(d) Make India I.T. Superpower
3. The National Museum of Natural History at
Ans. (a) : The 88th Indian Science Congress was
New Delhi imparts non-formal education in
organised on 3-7 January 2001. The focus theme of this
environment and conservation. Congress was “Food nutrition and environment security”.
4. Environmental Information System (ENVIS) 108th Indian Science Congress from 3-7th January, 2022 at
acts as a decentralised information network Symbiosis International University, Pune. Focal Theme -
for environmental information. Science and Technology for "Sustainable Development
Which of these statements are correct? with Women Empowerment."

IAS (Pre) GS 2001 Paper I 418 YCT


108. National Agricultural Insurance Scheme (a) 1 and 2 (b) 2 and 4
replacing Comprehensive Crop Insurance (c) 3 and 4 (d) 1, 2 and 3
Scheme was introduced in the year: Ans. (d) : Antyoday Anna Yojna is a sponsored scheme
(a) 1997 (b) 1998 of government of India to provide highly subsidised
(c) 1999 (d) 2000 food to poor families. It was launched in December,
Ans. (c) : The first crop insurance scheme in the country 2000.
was run as a test from 1973 to 1984. The Union Pradhanmantri Gram Sadak Yojna is a nationwide plan
to provide all weather road connectivity to unconnected
Agriculture Ministry started Comprehensive Crop
villages. It was also launched in year 2000.
Insurance Scheme from April 1985.
Sarvapriya Yojna - 2002 – by consumer affairs, food
In place of this, a ‘National Agricultural Insurance and publi distribution ministry.
Scheme: 1999-2000’ has been implemented from Rabi Jawahar Gram Samridhi Yojna - April, 1999.
year 1999-2000.
112. A great Landslide caused by an earthquake killed
The main objective of this scheme is to protect the farmers hundreds of people in January 2001 near:
from crop damage due to natural calamities like drought,
(a) San salvador (b) San Jose
flood, hailstorm, cyclone, fire, pests, diseases, so that their
(c) Mangula (d) Guatemala City
credit rating can be restored in the coming season.
Ans. (a) : Hundreds of people were killed near San
109. Consider the following states:
Salvador (EI Salvador) due to landslide caused by an
1. Gujarat 2. Karnataka
earthquake measuring (7.7 to 7.9) on Richter scale in
3. Maharashtra 4. Tamil Nadu year 2001.
The descending order of these states with
113. Falun Gong is:
reference to their level of Per Capita Net State
(a) An ethnic minority in Eastern China
Domestic Product is:
(b) An insurgency outfit in Western China
(a) 1, 3, 4, 2 (b) 3, 1, 2, 4
(c) A pro-democracy movement in China
(c) 1, 3, 2, 4 (d) 3, 1, 4, 2
(d) A spiritual movement in China
Ans. (d) : Net State Domestic Product Per Capita at
current prices in descending order Goa, Chandigarh, Ans. (d) : Falun Gong or Falun Dafa is a Chinese religious
Punjab, Haryana, Maharashtra, Gujarat, Tamil Nadu, spiritual practice that combines meditation and qigong
Karnataka. exercises with a moral philosophy centered on the tenets of
truthfulness, compassion, and forbearance. Through moral
110. The range of Agni-II missile is around:
rectitude and the practice of meditation, practitioners of
(a) 500 km (b) 2000 km Falun Gong aspire to eliminate attachments, and ultimately
(c) 3500 km (d) 5000 km to achieve spiritual enlightenment. Falun Gong originated
Ans. (b) : Range of Agni II missile is around 2000 km. and was first taught publicly in northeastern China in 1992
This air-to-air ballistic missile is based on indigenous by Li Hongzhi.
technology. 114. Who is the scientist in whose honour the
SLV-4 rockets are used in Agni missiles. India is one of "Chandra" X-ray telescope has been named?
the 6 superpowers that has developed the medium-range (a) Chandrasekhar Venkat Raman
missile capability. (b) Jagdish Chandra Bose
Agni-I – 1500Km (c) Prafulla Chandra Roy
Agni-II – 2000-2500Km (d) Subramanyam Chandrasekhar
Agni-III– 3000-3500Km Ans. (d) : “Chandra” X-ray telescope has been named
Agni-IV– 4000Km after famous physicist of Indian origin Subramanyam
Agni V – 5000 Km Chandrasekhar.
111. Consider the following schemes launched by 115. The Union Budget 2000 awarded a Tax Holiday
for the North-Eastern Region to promote
the Union Government:
industrialisation for:
1. Antyodaya Anna Yojana (a) 5 years (b) 7 years
2. Gram Sadak Yojana (c) 9 years (d) 10 years
3. Sarvapriya Ans. (d) : The then Finance Minister Yashwant Sinha,
4. Jawahar Gram Samridhi Yojana while presenting the budget of 1999-2000, made
Which of the above were announced in the year provision for tax holiday for 10 years to provide
2000? industrialization incentives in the Northeast region.

IAS (Pre) GS 2001 Paper I 419 YCT


116. Match List I with List II and select the correct Ans. (c) : “Attacking Poverty” was the theme of the
answer using the codes given below the lists: World Development Report 2001.
List I List II The World Bank has published the World Development
(Award/Prize) (Recipient) Report 2021: Data for Better Lives.
A. Indira Gandhi 1. Archibiship 120. Mekong Ganga co-operation project is:
Prize for Peace, Desmond Tutu (a) An irrigation project involving India and
Myanmar
Disarmament
(b) A joint tourism initiative of some Asian
and
countries
Development
(c) A hydroelectric power project involving India,
B. Jamnalal Bajaj 2. Dr. Gurudev Bangladesh and Myanmar
Award Singh Kush (d) A defence and security agreement of India with
its eastern neighbours
Ans. (b) : Mekong Ganga Co-operation Project is a
C. International 3. Dr. M.S. joint tourism initiative of some Asian countries. It
Gandhi Peace Swaminathan comprises six member countries, namely India (Look-
Prize East connectivity projects), Thailand, Myanmar,
D. World Food 4. Nelson Mandela Cambodia, Laos and Vietnam. The four areas of
Prize cooperation are tourism, culture, education, and
transportation. The organization takes its name from the
A B C D Ganga and the Mekong, two large rivers in the region.
(a) 1 3 4 2
121. Who amongest the following was the Chairman
(b) 3 1 4 2 of I.S.R.O. when INSAT-3B was launched?
(c) 1 3 2 4 (a) Anil Kakodkar (b) Abdul Kalam
(d) 3 1 2 4 (c) K. Kasturirangn (d) U.R. Rao
Ans. (b) : 1. Indira Gandhi – Dr. M.S. Swaminathan Ans. (c) : K. Kasturirangan was the chairman of
Prize for Peace, I.S.R.O when INSAT-3B was launched.
Disarmament Directions : The following 12 (twelve) items consist of
and Development two statements, one labelled as the 'Assertion (A)' and
the other as 'Reason (R). You are to examine these two
2. Jamnalal Bajaj Award – Archibiship Desmond Tutu
statements carefully and select the answer to these items
3. International Gandhi – Nelson Mandela
using the code given below:
Peace Prize
(a) Both A and R are individually true and R is
4. World Food Prize – Dr. Gurudev Singh Kush the correct explanation of A
117. Which among the following ports was called (b) Both A and R are individually true but R is
Babul Makka (Gate of Makka) during the NOT the correct explanation of A
Mughal Period? (c) A is true but R is false
(a) Calicut (b) Borach (d) A is false but R is true
(c) Cambay (d) Surat 122. Assertion (A) : India's software exports increased
Ans. (d) : Surat had an important place as a port in at an average growth rate of 50% since 1995-96.
Mughal economic activities. This port was called the Reason (R) : Indian software companies were cost-
Mecca Gate because Muslims used to go for the effective and maintained international quality.
pilgrimage from this port. (a) Both A and R are individually true and R is
118. The Chess player Alexi Shirov represents: the correct explanation of A
(a) Albania (b) Kazhakstan (b) Both A and R are individually true but R is
(c) Russia (d) Spain NOT the correct explanation of A
(c) A is true but R is false
Ans. (d) : The chess player Alexi Shirov represents
(d) A is false but R is true
spain. He was ranked 2nd in the World in 1994.
Ans. (a) : The impact of the 1991 economic reforms
119. The theme of the World Development Report ranged from the service sector to the information
2001, is: technology sector. The Indian software industry is also
(a) From Plan to market growing with a rapid export rate of around 60%. The
(b) Knowledge for Development reason for this is the quality at the international level as
(c) Attacking Poverty well as the low price, which is the general rule of the
(d) The State in the Changing world economy.
IAS (Pre) GS 2001 Paper I 420 YCT
MISCELLANEOUS 1. The installed capacity of power generation
is around 95000 MW.
123. A city has a population of 3,00,000 out of which 2. Nuclear plants contribute nearly 15% of
1,80,000 are males. 50% of the population is total power generation.
literate. If 70% of the males are literate, the 3. Hydroelectricity plants contribute nearly
number of literate females is: 40% of total power generation.
(a) 24,000 (b) 30,000 4. Thermal plants at present account for
(c) 54,000 (d) 60,000 nearly 80% of total power generation.
Ans. (a) : Total population = 300000 Which of these statements is/are correct?
Total number of males = 180000
(a) 1 only (b) 2 and 3
Total Literates = 50% of total population = 150000
(c) Third (d) Fifth
Number of literate males = 70% of males = 126000
∴ Number of literate females = 150000 – 126000 = Ans. (a) :
24000 According to the data of 2000, the power generation is
124. In a survey, it was found that 80% of those 95000 MW. Nuclear plants contribute 2% of total
surveyed owned a car while 60% of those power generation. Hydroelectricity plants contribute
surveyed owned a mobile phone. If 55% owned 25% of total production and 73% is contributed by
both a car and a mobile phone, what percent of thermal power plants.
those surveyed owned a car or a mobile phone 127. In 1930, a person's age was 8 times that of his
or both?
son. In 1938, the father's age became ten times
(a) 65% (b) 80%
that of his son's age in 1930. The ages of the son
(c) 85% (d) 97.5%
and father in 1940 were, respectively:
Ans. (c) : Percentage of car owners = 80%
(a) 16 years, 59 years (b) 15 years, 50 years
Percentage of mobile phone owners = 60%
(c) 14 years, 42 years (d) 13 years, 34 years
Percentage of people having both car and mobile phone
= 55% Ans. (c) : Let the father's age and his son's age in 1930
Percentage of people having only car = 80 – 55 = 25% be 8x and x respectively.
Percentage of people having only mobile phone = 60 – In 1938, father's age = (8x+ 8) = 10x
55 = 5% 2x = 8
Percentage of people having a car or a mobile phone or x=4
both = 55% + 25% + 5% = 85% Age of the son in 1940 = (x+10) = 4 + 10 = 14
125. Consider the following statements regarding Age of the father in 1940 = (8x+10) = 8×4 + 10 = 42
the Armed Forces:
1. First batch of women pilots was
commissioned in Indian Air Force in 1996.
2. Officers' Training Academy is located in 128.
Nagpur.
3. Southern Command of Indian Navy has its
Headquarters at Chennai.
4. One of the Regional Headquarters of Coast
Guard is located at Port Blair.
Which of these statements are correct? A circle of 1 m radius is drawn inside a square
(a) 1, 2, 3 and 4 (b) 2, 3 and 4 as shown in the figure given above. What is the
(c) 3 only (d) 4 only area of the shaded portion in m2?
Ans. (*) : First batch of women pilot was (a) (4 –π)
commissioned in 1996. Gunjan Saxena was one of the 6  π
women pilot inducted in 1996. Officer’s training (b)  1 − 
Academy (OTA) is situated in Chennai and not in  2
Nagpur. Headquarter of Southern Command of Indian 1 π
(c) −
navy is in Cochin. Regional Headquarter of a coast 4 4
guard is located at Mumbai, Chennai and Port Blair.  π
126. Consider the following statements regarding (d)  1 − 
 4
power sector in India:
Ans. (d) : Since radius of circle = 1 m
∴ side of the square = 1 + 1 = 2m
IAS (Pre) GS 2001 Paper I 421 YCT
So area of circle = πr2 = π List I List II
2 2
And area of square = (2) = 4m (Achievement in (Scientists)
1 genetics)
Area of shaded region = (Area of square − Area of circle)
4 A. Discovery of 1. Khurana
1 transduction and
= (4 − π)
4 conjugation in
 π 2 bacteria
= 1 −  m
 4 B. Establishing the sex- 2. Kornberg
129. Match List I with List II and select the correct linked inheritance
answer using the code given below the lists: C. Isolation of DNA 3. Lederberg
List I List II polymerase from E.
(Important Day) (Date) Coli
A. World Environment Day 1. March 20
D. Establishing the 4. Morgan
B. World Forestry Day 2. June 5
C. World Habitat Day 3. September 16 complete genetic code
D. World Ozone Day 4. October 3 5. Ochoa
5. December 10 A B C D
A B C D (a) 4 3 2 1
(a) 2 1 4 5 (b) 3 4 1 5
(b) 1 2 4 3 (c) 4 3 1 5
(c) 1 2 3 4
(d) 3 4 2 1
(d) 2 1 4 3
Ans. (d) : The correct match is - Ans. (d) : The correct match is option (d)
A.World Environment Day - June 5 A. Discovery of 1. Lederberg
B. World Forestry Day - March 20 transduction and
C. World Habitat Day - October 3 conjugation in bacteria
D. World Ozone Day - September 16 B. Establishing the sex- 2. Morgan
130. Match List I with List II and select the correct linked inheritance
using the codes given below the lists:
C. Isolation of DNA 3. Kornberg
List I List II
(Books) (Author) polymerase from E.
A. The Struggle in 1. Lech Walesa Coli
my life D. Establishing the 4. Khurana
B. The Struggle & 2. Nelson Mandela complete genetic code
the Triumph 132. Anand must be a vegetarian because he is a
C. Friends & Foes 3. Leonid Brezhnev Buddhist, The argument assumed that:
D. Rebirth 4. Zulfikar Ali Bhutto
(a) Most Buddhists are vegetarians
5. Sheikh Mujibur
Rehman (b) All Buddhists are vegetarians
A B C D (c) Only Buddhists are vegetarians
(a) 1 2 5 3 (d) Most vegetarians are Buddhists
(b) 2 1 5 3 Ans. (b) : Anand must be a vegetarian because he is a
(c) 1 2 3 4 Buddhist, The argument assumed that all Buddhists are
(d) 2 1 3 4 vegetarians.
Ans. (b) : The Struggle in My Life is written by Nelson 133. Most guitarists are bearded males. If a
Mandela, " The Struggle and the Triump" is written by represents all males, B represents beared males
Lech Walesa, "Friends and Foes" is written by Sheikh and C represents all male guitarists, then the
Mujibur Rahman and "Rebirth" is written by Leonid correct diagram for their relation (shaded
Brezhenev, the former president of U.S.S.R portion) is
131. Match List I with List II and select the correct
answer using the codes given below the lists:
IAS (Pre) GS 2001 Paper I 422 YCT
In the above figure, ABCD is a cyclic
quadrilateral, AB = BC and ∠ BAC = 70º, then
(a) (b) ∠ ADC is
(a) 40º (b) 80º
(c) 110º (d) 140º
Ans. (d) :

(c) (d)

In ∆ABC, AB = BC (Isosceles triangle)


Ans. (a) : As all species comes under the male category, ∴ ∠BAC = ∠BCA = 70°
so B and C comes under A. Some guitarists are beared, Now, ∠BAC + ∠ABC + ∠BCA = 180°
so B and C have some common elements between them. 70° + ∠ABC + 70° = 180°
134. Which of the following pairs is NOT correctly ∠ABC = 180° – 140°
matched? = 40°
(a) India's first technicolor : Jhansi Ki Rani As ABCD is a cyclic quadrilateral and sum of opposite
film angle pair is equal to 180°
(b) India's first -3D film : My Dear ∴ ∠ ADC + ∠ ABC = 180°
Kuttichethan ∠ADC = 180° – 40°
(c) India's first insured film : Tall = 140°
(d) India's first actress to : Meena Kumari 137. Match List I with List II and select the correct
win the Bharat Ratna answer using the codes given below the lists:
Ans. (d) : Meena Kumari is not the recipient of Bharat List I List II
Ratna. Indira Gandhi was the first woman recipient of (Person) (Achievements )
this award. No actress have been awarded Bharat Ratna A. Deep Sen 1. India's youngest
so far. Gupta International Master in
135. Match List I with List II and select the correct the Sangli International
Chess
answer using the code given below the lists:
List I List II B. P. 2. The first Indian ever to
Harikrishna win a gold in discus in
(Dancer) (Dance)
the World Junior
A. Kalamandalam 1. Kathakali Athletic Championship
Kshemavathy in santiago, 2000
B. Kottakkal Sivaraman 2. Manipuri C. Seema Antil 3. Won the title in the
C. Lakshmi Viswanathan 3. Mohiniattam Asian Junior Chess
D. N. Madhabi Devi 4. Bharatnatyam Tournament in Mumbai,
A B C D 2000
(a) 1 3 2 4 D. Tejas Bakre 4. Won the under-12 title in
(b) 3 1 4 2 the World Youth Chess
(c) 1 3 4 2 Festival in Oropesa, 2000
(d) 3 1 2 4 A B C D
Ans. (b) : The correct match is as follows: (a) 4 3 2 1
Kalamandalam Kshemavaty – Mohiniattam Kottakkal (b) 2 4 1 3
Sivaraman – kathakali (c) 4 1 2 3
Lakshmi Vishwanathan – Bharatanatyam (d) 2 1 3 4
N. Madhabi Devi – Manipuri Ans. (c) : Deep Sen Gupta won the under 12 title in the
World Youth Chess Festival in Oropesa, 2000. P.
136.
Harikrishna is India’s youngest international master in
the Sangli International Chess, Tejas Bakre won the title
in the Asian chess Tournament in Mumbai 2000.
Seema Punia-Antil was first Indian ever to win a gold in
discuss throw in the World Junior Athletics
championship is Santiago 2000.

IAS (Pre) GS 2001 Paper I 423 YCT


138. Match List I with List II select the correct x
answer using the codes given below the lists: time taken =
60
List I List II
Totaldistance
(Person) (Distinguished as) average speed for complete journey =
A. Santosh Yadav 1. T.V. host Total time
B. Oprah Winfrey 2. Journalist 2x 4800
C. Oscar Wilde 3. Mountaineer = = = 48km/hr.
x x 100
D. P. Sainath 4. Dramatist and +
40 60
Author 141. Match List I with List II and select the correct
A B C D answer using the code given below the lists:
(a) 3 1 4 2
List-I List-II
(b) 1 3 2 4 (Sports women) (Sports)
(c) 3 1 2 4 A. Anjali Vedpthak 1. Athletics
(d) 1 3 4 2 B. Mouma Das 2. Gymnastics
Ans. (a) : C. Neelam Singh 3. Table Tennis
List I List II D. Tumpa Debnath 4. Chess
(Person) (Distinguished as) A B C D
1. Santosh Yadav - Mountaineer (a) 3 4 2 5
2. Oprah Winfrey - T.V. host (b) 4 3 1 2
3. Oscar Wilde - Dramatist and Author (c) 4 3 2 5
4. P. Sainath - Journalist (d) 3 4 1 2
139. The following table shows the percentage Ans. (c) : Anjali Vedpathak is related to Rifle Shooting,
change in the consumption of electricity by five Mouma Das is related to Table Tennis, Neelam Singh is
towns? P, Q, R, S, T from 1986 to 1988: related to Atheletics and Tumpa Debnath is related to
Town From 1986 to From 1987 to Atheletics.
1987 1988 142. The earlier name of WTO was:
P +8 – 18 (a) UNCTAD (b) GATT
Q – 15 + II (c) UNIDO (d) OECD
R +6 +9 Ans. (b) : GATT – General Agreement on Tariff and
S –7 –5 Trade. GATT came into force on 1 January 1948 with
T + 13 –6 HQ at Geneva. WTO was set up as a result of aid from
If town T consumed 500,000 units in 1986, how 8th round of GATT talks (Uruguay round, 1986-93).
much did it consume in 1988? WTO was formed on 1 Jan 1995 at Marrakesh,
(a) 371,00 units (b) 531,100 units Morocco.
(c) 551,100 units (d) 571,100 units 143. Solve the given equations:
Ans. (b) : If there are two percentage changes of x% x2 + y2 = 34 x4 – y4 = 544
and y%. The values of x and y are
xy (a) ± 4, ± 3 (b) ± 5, ± 3
then net % change = x + y + (c) ± 3, ± 5 (d) ± 3, ± 4
100
Net % change for T from 1986 to 1988 Ans. (b) : According to question
(13)(6) x2 + y2 = 34 ………(1)
13 − 6 − = 6.22 and x4 – y4 = 544
100
or (x2)2 – (y2)2 = 544
∴ Consumption of electricity by T in 1988
or (x2 + y2) (x2 – y2) = 544 ………(2)
 6.22  Putting value of (x2 + y2) from equation 1 to equation 2
= 50,00,00  1+  = 531100
 100  34 (x2 – y2) = 544
140. A person travels from X to Y at a speed of 40 544
x2 – y2 = = 16
kmph and returns by increasing his speed by 34
50%. What is his average speed for both the x2 – y2 = 16
trips? Now checking it with the given options, only
(a) 36 kmph (b) 45 kmph x = 5 and y = 3 satisfy it.
(c) 48 kmph (d) 50 kmph 144. Consider the decadal Census data given below:
Ans. (c) : Let the distance from x to y = x km Decadal population (in millions)
Speed towards y from x = 40 km/hr Year Population
x 1961 10.7
time taken =
40 1971 14.3
Returning speed from y to x = 40 + (50% of 40) 1981 16.2
= 40 + 20 = 60 km/hr 1991 18.9
IAS (Pre) GS 2001 Paper I 424 YCT
The above data refer to which one of the 3. The smallest religious minority in India are
'Population by Religion' Groups? the Zoroastrians.
(a) Sikhs (b) Jains 4. The Constitution of India recognizes and
(c) Christians (d) Buddhists
protects religious and linguistic minorities.
Ans. (c) : The given data refers to the decadal
population of Christians in India. In 2001 it was 24.08 Which of these statements are correct?
million and in census 2011 it was 2.78 crores. (a) 2 and 3 (b) 1 and 4
145. A worker reaches his factory 3 minutes late if (c) 2, 3 and 4 (d) 1, 2, 3 and 4
his speed from his house to the factory is Ans. (d) : The Indian Constitution recognizes and protects
5km/hr. If he walks at a speed of 6 km/hr, then minorities on religious and linguistic grounds.
he reaches the factory 7 minutes early. The In 1993, the National Minorities Commission was given
distance of the factory from his house is:
constitutional status. The Chairman and members of the
(a) 4 km (b) 5 km
(c) 6 km (d) 7 km Minorities Commission are appointed by the President.
Ans. (b) : Let the distance be x km then Parsi community is the most talented but least in
x 3 x 7 x x 10 1 population in India. When the question was asked there
− = + ⇒ − = = are 6 minority communities, namely Muslims (14.2%),
5 60 6 60 5 6 60 6
x 1 30 Christians (2.3%), Sikhs (1.7%), Buddists (0.7%), Jains
= ⇒x= =5km. (0.4%) and Paris (0.006%).
30 6 6
146. Two wires have their lengths, diameters and 149. A conveyer belt delivers baggage at the rate of
resistivities, all in the ratio of 1 : 2. If the 3 tons in 5 minutes, and a second conveyer belt
resistance of the thinner wire is 10 ohms, the delivers baggage at the rate of 1 ton in 2
resistance of the thicker wire is: minutes. How much time will it take to get 33
(a) 5 ohms (b) 10 ohms
tons of baggage delivered, using both the
(c) 20 ohms (d) 40 ohms
conveyer belts?
l
Ans. (b) : Resistance of the wire = R = ρ (a) 25 minutes and 30 seconds
A (b) 30 minutes
rd 2 (c) 35 minutes
where A =
4 (d) 40 minutes and 45 seconds
R 2 ρ 2l2d12 Ans. (b) : Baggage delivered by first belt in 1 minute
=
R1 ρ1l1d 22
3
R 2 1 1 22 =   tons
= × × =1 5
10 2 2 12
R2 = 10Ω 1
Baggage delivered by second belt in 1 minute = tons
147. Consider the following organizations 2
1. Atomic Minerals Directorate for Research Both deliver baggage in 1 minute
& Exploration 3 1 11
2. Heavy Water Board =  +  tons = tons
 5 2  10
3. Indian Rare Earths Limited
4. Uranium Corporation of India ∴ Required time
Which of these/are under the Department of 33 10
Atomic Energy? ⇒ 33× = 30 minutes.
11 11
(a) 1 only (b) 1 and 4 10
(c) 2, 3 and 4 (d) 1, 2, 3 and 4
Ans. (d) : The Department of Atomic Energy constituted 150. Water is filled in a container in such a manner
on 3 August 1954 is involved in the development of that its volume doubles after every five
nuclear power technology, radiation, use of technology in minutes. If it takes 30 minutes for the container
agriculture, medicine and industry, and basic research. All to be full, in how much time will it be one-
four of the above options are related to the Department of fourth full?
Atomic Energy. (a) 7 minutes (b) 10 minutes
148. Consider the following statements about the (c) 20 minutes (d) 25 minutes
minorities in India:
1. The Government of India has notified five Ans. (c) : Take 1 litre for first 5 minutes. Then it
communities, namely, Muslims, Sikhs, becomes double afterwards every five minutes. We now
Christians, Buddists and Zoroastrians as get, 1, 2, 4, 8, 16, 32 for 5 mins, 10 mins, 15 mins, 20
minorities. mins, 25 mins, 30 mins. Total litres we get is “32” and
2. The National Commission for Minorities
1/4th of this will be “8” which occurs at 20 mins.
was given statutory status in 1993.
IAS (Pre) GS 2001 Paper I 425 YCT
UNION PUBLIC SERVICE COMMISSION
Civil Services (Preliminary Exam) - 2000
GENERAL STUDIES : PAPER-I
Time: 2 hours Maximum Number: 200

Ans. (a) : Both Assertion and Reason are correct and


ANCIENT HISTORY Reason is the correct explanation for Assertion.
1. Assertion A : The origin of Feudal system in 4. Assertion A : The Aham and Puram Poems of the
ancient India can be traced to military campaigns. Padinen Kilukanakku group formed a continuation
Reason R : There was considerable expansion of of the Sangam composition.
the Feudal system during the Gupta period. Reason R : They were included under the Post-
(a) Both A and R are individually true and R is Sangam works as against the Sangam works
the correct explanation of A proper.
(b) Both A and R are individually true but R is (a) Both A and R are individually true and R is
not the correct explanation of A the correct explanation of A
(c) A is true but R is false (b) Both A and R are individually true but R is
(d) A is false but R is true not the correct explanation of A
Ans. (b) : The origin of feudal system in ancient India (c) A is true but R is false
can be traced to military campaigns because the feudal (d) A is false but R is true
lords provided military help to the king during the war. Ans. (a) : On the basis of interpretation and context, the
There was considerable expansion of feudal system Sangam literature can be described into two types viz.
during Gupta period. Aham (inner) and Puram (outer). The topics of Aham
2. Assertion A : Ashoka annexed Kalinga to the are related to personal and human aspects such as love
Mauryan Empire. and sexual things. The topics of Puram are related to
Reason R : Kalinga controlled land and sea routes human experiences and emotions such as Heroism,
to South India. Valor, Ethics and Philanthropy. The poems have also
(a) Both A and R are individually true and R is been classified on nature themes which are known as
the correct explanation of A ThinatPatinen. kilkanakku is the post Sangam work that
(b) Both A and R are individually true but R is is of Aham as well as Puram context.
not the correct explanation of A
5. The practice of military governorship was first
(c) A is true but R is false introduced in India by the:
(d) A is false but R is true (a) Greeks (b) Shakas
Ans. (a) : Ashoka annexed kalinga to the Mauryan (c) Parthians (d) Mughals
empire in 261 BC. Though Ashoka abandoned the Ans. (a) : Greeks first introduced military governorship in
policy of Physical occupation in favour of Policy of India. They appointed their Governors called strategos.
Cultural conquest after the Kalinga War, yet he retained
6. Which one of the followings dynasties was
Kalinga after its conquest and incorporated it into his
ruling over North India at the time of
empire as Kalinga controlled land and sea routes to Alexander's invasion?
South India. Thus R explains A. (a) Nanda (b) Maurya
3. Assertion A : The emphasis of Jainism on non- (c) Sunga (d) Kanva
'violence (ahimsa) prevented agriculturalists from
Ans. (a) : Alexander invaded India in 326 BC. Nandas
embracing Jainism.
were ruling over India at that time.
Reason R : Cultivation involved killing of insects The chronological order of these 4 dynasties are:
and, pests. Nanda, Maurya, Sunga and Kanvas. The Nanda dynasty
(a) Both A and R are individually true and R is ruled in northern part of the Indian subcontinent during
the correct explanation of A the 4th century BCE, and possibly during the 5th
(b) Both A and R are individually true but R is century BCE. The Nandas overthrew the Shishunaga
not the correct explanation of A dynasty in the Magadha region of eastern India, and
(c) A is true but R is false expanded their empire to include a larger part of
(d) A is false but R is true northern India.
IAS (Pre) GS 2000 Paper I 426 YCT
MEDIEVAL HISTORY 10. Which one of the following Muslim rulers was
hailed as the 'Jagadguru' by his Muslim
7. The given map refers to the kingdom of subjects because of his belief in secularism?
(a) Husain Shah (b) Zain-ul-Abidin
(c) Ibrahim Adil Shah (d) Mahmud II
Ans. (c) : Ibrahim Adil shah was a secular ruler of the
Mughal period. Hindu king during his reign referred
him as 'Jagadguru'. In addition to Jagadguru, his
subjects also used to call him 'Adil Baba'.

MODERN HISTORY
11. Who among the following streamlined the
(a) Akbar at the time of capture of Khandesh in Maratha administration after Sambhaji?
1601
(a) Raja Ram (b) Balaji Viswanath
(b) Akbar at the time of his death in 1605
(c) Aurangzeb at the time of capture of (c) Ganga Bai (d) Nanaji Deshmukh
Hyderabad Ans. (b) : Peshwa Vishwanath was from the Chitpavan
(d) Aurangzeb at the time of his death in 1707. Brahmin family of a place called Srivardhan in Konkan,
Ans. (d) : Bijapur in 1686 AD and after the conquest of He had a great knowledge of both revenue and military
Golconda in 1687 AD, the Mughal empire spread from subjects. After Shambha Ji, Rajaram organized the
the Himalayas in the north to the Kaveri river in the Marathas and strengthened them.
south during the reign of Aurangzeb. In 1707, the map 12. "In this instance we could not playoff the
represents the kingdom of Aurangzeb. Mohammedans against the Hindu." To which
8. Consider the following events: one of the following events did this remark of
1. Reign of Krishna Deva Raya of Aitchison relate?
Vijayanagara. (a) Revolt of 1857
2. Construction of Qutub Minar. (b) Champaran Satyagraha (1917)
3. Arrival of Portuguese in India.
(c) Khilafat & Non-Cooperation Movement (1919-
4. Death of Firuz Tughlaq.
The correct chronological sequence of these 22)
events is: (d) August Movement of 1942
(a) 2, 4, 3, 1 (b) 2, 4, 1, 3 Ans. (c) : The above comment of Delhi Collector
(c) 4, 2, 1, 3 (d) 4, 2, 3, 1 Aitchison was made in the context of the Khilafat and
Ans. (a) : Construction of QutubMinar - 1206-1210 Non-Cooperation Movement (1919-22).
AD, Firuz Tughlaq's death 1388 AD, Portuguese arrival 13. Which one of the following is NOT a feature of
1498 AD, Vijayanagara reign of Krishnadeva Raya- the Government of India Act of 1935?
1509-1529 AD. (a) Diarchy at the Centre as well as in the
9. Match List-I with List-II and select the correct provinces
answer using he codes given below the lists: (b) A bicameral legislature
List I List II (c) Provincial autonomy
A. Iqta 1. Marathas (d) An All-India Federation
B. Jagir 2. Delhi Sultans
C. Amaram 3. Mughals Ans. (a) : Salient Features of the Government of India
D. Mokasa 4. Vijayanagara Act, 1935 were as follows: Abolition of provincial
A B C D dyarchy and introduction of dyarchy at centre, Abolition
(a) 3 2 1 4 of Indian Council and introduction of an advisory body
(b) 2 3 4 1 in its place, Provision for an All India Federation with
(c) 2 3 1 4 British India territories and princely states.
(d) 3 2 4 1 14. The Indian National Army (I.N.A.) came into
Ans. (b) : Iqta was introduced by Delhi Sultans while existence in 1943 in
Jagir was introduced by Mughals. (a) Japan (b) Burma
Amaram was introduced by Vijaynagar empire. (c) Singapore (d) Malaya
The land pieces assigned to military chiefs were Ans. (c) : On 4 July 1943, two days after reaching
called as Amaram during Vijayanagar empire. Singapore, Bose assumed the leadership of the INA in a
Mokasa was introduced by Maratha. ceremony at Cathay Building. Bose's influence was
‘Iqta’ is an Arabic word, which became a tool of notable. His appeal re-invigorated the INA, which had
administration in Islamic traditions.
IAS (Pre) GS 2000 Paper I 427 YCT
previously consisted mainly of prisoners of war. It also Ans. (d) : The native State of Tripura became involved
attracted Indian expatriates in South Asia. The Indian in the Freedom Movement early in the 20th century
National Army (INA) was originally founded by Capt. because there were already some groups fighting
Mohan Singh in Singapore in September 1942 with against the kingship and its protector, the British.
Indian POWs (prisoners of war). This was along the 19. That the per capital income in Indian was Rs.
concept of and with support of, what was then known as 20/- in 1867-68, was ascertained for the first
the Indian Independence League, headed by expatriate
time by
nationalist leader Rash Bihari Bose.
(a) M.G. Ranade (b) Sir W. Hunter
15. The last major extension of British Indian (c) R.C. Dutta (d) DadabhaiNaoroji
territory took place during the time of
Ans. (d) : The 'Drain Theory' was first developed by
(a) Dufferin (b) Dalhousie
DadabhaiNaoroji in a series of speeches and writings
(c) Lytton (d) Curzon
subsequently published in 1901 in a volume entitled
Ans. (b) : The last major extension of British Indian "Poverty and un-British Rule in India". He ascertained
Territory took place during the tenure of Lord per capita income in India was Rs. 20 in 1867-68.
Dalhousie under his Doctrine of Lapse (Jhansi, Satara,
20. After returning from South Africa, Gandhiji
Jaitpur, Sambalpur, Udaipur and Nagpur) and on the
launched his first successful Satyagraha in
basis of misgovernance (Awadh).
(a) Chauri-Chaura (b) Dandi
16. Which one of the following pairs is not
(c) Champaran (d) Bardoli
correctly matched?
(a) Baki Itihas : Badal Sarkar Ans. (c) : Gandhiji launched his first successful
Satyagraha in Champaran after returning from South
(b) Sita Swayamvar : Vishnu Das Bhave
Africa. Mahatma Gandhi began his experiments with
(c) Yayati : Girish Karnad
Satyagraha against the oppressive European indigo
(d) Giddha : Jabbar Patel
planters at Champaran in Bihar in 1917.
Ans. (c) : Yayati is not written by Girish Karnad but is
21. Match List I with List II and select the correct
written by Vishnu Sakharam Khandekar.
answer using the codes given below the lists:
17. As an alternative to the partition of India, List I List II
Gandhiji suggested to Mountbatten that he:
A. Chittagong Armoury 1. Kalpana Dutt
(a) postpone granting of independence
Raid
(b) invite Jinnah to form the government
B. Abhinav Bharat 2. Guru Ram Singh
(c) invite Nehru and Jinnah to form the
government together C. Anushilan Samiti 3. Vinayak Damodar
(d) invite the army to take over for some time Savarkar
D. Kuka Movement 4. Aurobindo Ghosh
Ans. (b) : Mountbatten Plan came into existence on
A B C D
June 3, 1947. Gandhi ji suggested that he should invite
Jinnah to form the government rather than partition of (a) 1 3 4 2
India. (b) 1 3 2 4
3 June plan was also known as the Mountbatten Plan. (c) 3 1 2 4
The British government proposed a plan announced on (d) 3 1 4 2
3 June 1947 that included these principles: Principle of Ans. (a) : Chittagong Armoury Raid – April 1930;
the Partition of British India was accepted by the British Abhinav Bharat–1904; Anushilan Samiti – (1902,
Government. Successor governments would be given Calcutta).
dominion status. 22. Match List I with List II and select the correct
18. The native state of Tripura became involved in answer using the codes given below the lists:
the Freedom movement early in the 20th List-I List-II
century because A. Land allotted to big 1. Jagirdari
(a) The kings of Tripura were always anti-British feudal landlords System
(b) The Bengal revolutionaries took shelter in B. Land allotted to revenue 2. Ryotwari
farmers or rent collectors System
Tripura
C. Land allotted to each 3. Mahalwari
(c) The tribes of the state were fiercely freedom peasant with the right to system
loving sublet, mortgage, transfer,
(d) There were already some groups fighting gift or sell
against the kingship and its protector, the D. Revenue settlements made 4. Zamindari
British at village level System

IAS (Pre) GS 2000 Paper I 428 YCT


Codes: 25. While delivering the presidential address, the
A B C D Congress President who advocated the
(a) 1 3 2 4 introduction of Roman script for Hindi language
(b) 1 4 2 3 was:
(c) 3 4 1 2 (a) Mahatma Gandhi
(d) 2 1 3 4 (b) Jawaharlal Nehru
Ans. (b) : The system in which Land allotted to big (c) Abul Kalam Azad
landlords was called Jagirdari system. The system in (d) Subhash Chandra Bose
which land allotted to revenue farmers or rent collectors Ans. (d) : Subhash Chandra Bose (in 1938, Haripura
was Zamindari system. The land allotted to each Session), while delivering the presidential address has
peasant with the right to sublet, mortgage transfer was advocated the introduction of Roman script for Hindi
called Ryotwari system. The settlement made at village language.
level was called Mahalwari system. 26. At the time of partition of India, which one of
23. Assertion A : Lord Linlithgo described the the following provinces of British India came
August Movement of 1942 as the most serious forward with a plan for a united and
independent existence?
revolt after the Sepoy mutiny.
(a) Punjab
Reason R : Peasants joined the movement in large
(b) Assam
number in some places.
(c) Bengal
(a) Both A and R are individually true and R is
(d) Bihar
the correct explanation of A
Ans. (a) : Punjab provinces of British India came
(b) Both A and R are individually true but R is forward with a plan for a united and independent
not the correct explanation of A existence.
(c) A is true but R is false 27. The Balkan Plan for fragmentation of India
(d) A is false but R is true was the brain-child of:
Ans. (a) : This movement placed the demand for (a) W. Churchill
independence on the immediate agenda of national (b) M.A. Jinnah
movement. After Quit India, there could be no going (c) Lord Mountbatten
(d) V.P. Menon
back. During this struggle, common people including
Ans. (c) : Balkan Plan (1947) was the brain child of
peasants displayed unparalleled heroism. That’s why
Mountbatten. This plan envisaged the transfer of power
Lord Linlithgo (1936–44) described the Quit India to separate provinces with Punjab and Bengal given the
Movement as the most serious revolt after the sepoy option to vote for partition of their provinces. (The
mutiny. partitioned units thus formed will have the choice to
Ans. (b) : Quit India movement was the most serious join India or Pakistan or remain independent).
revolt after sepoy mutiny not because large number of 28. Consider the following statements about the
peasant participated but because all the strata of Indian Indian National Congress:
society (accept a few) participated in the movement. 1. Sarojini Naidu was the first woman to be
the President of the Congress
24. Assertion A : The basic weakness of the early 2. C. R. Das was in prison when he functioned
nationalist movement lay in its narrow social base. as the President of the Congress
Reason R : It fought for the narrow interests of 3. The first Britisher to become the President
the social groups which joined if. of the Congress was Alan Octavian Hume
Codes: 4. Alfred Webb was the President of the
(a) Both A and R are true and R is the correct Congress in 1894
explanation of A. Which of these statements are correct?
(b) Both A and R are true but R is not a correct (a) 1 and 3 (b) 2 and 4
explanation of A. (c) 2, 3 and 4 (d) 1, 2, 3 and 4
(c) A is true but R is false Ans. (b) : Annie Besant became the first woman
(d) A is false but R is true president of the Calcutta Congress session in 1917.
Alan Octavian Hume was never made president at any
Ans. (c) : R is incorrect as early nationalists fought for
Congress session.
the rights of common people.
IAS (Pre) GS 2000 Paper I 429 YCT
INDIAN GEOGRAPHY Name of institutes
A. Central Marine Fisheries Research Institute
29. Consider the following statements about the B. Central Sheep Breeding Farm
megacities of India? C. National Dairy Research Institute
1. Population of each megacity is more than 5 D. National Institute of Agricultural Extension
million. Management
2. All the megacities are have importance sea A B C D
ports. (a) 5 1 3 2
3. Megacities are either national or state capitals. (b) 5 2 1 4
Which of these statements are correct? (c) 4 2 1 3
(a) 1, 2 and 3 (b) 1 and 2 (d) 1 2 3 4
(c) 2 and 3 (d) 1 and 3 Ans. (b) : Central Marine Fisheries Research Institute is
Ans. (d) : Megacities of India have more than 5 million located in Cochin, Kerala, which is marked as ‘5’ in the
population. But not all the major megacities are located given map.
on the sea coast or are sea ports. Delhi, which is the Central sheep breeding farm is located in Hissar,
national capital is not a sea port. Haryana, which is marked as ‘2’.
India currently has five megacities namely New Delhi National Dairy Research Institute is situated in Karnal,
(Population of 26.5 million), Mumbai (Population of
Haryana is marked by ‘1’ in the map. National Institute
21.4 million), Kolkata (Population of 15 million),
Bengaluru (Population of 10.5 million) and Chennai of Agricultural Extension management is situated at
(Population of 10.2 million). All the major megacities Hyderabad, Telangana and is marked by ‘4’ in the given
are not located on the seacoast or have seaports. For map.
example, Delhi, the national capital does not have a 32. Consider the following statements:
seaport. Delhi is the State capital, Mumbai is the capital 1. Maharashtra has the highest acreage
of Maharashtra, Kolkata is the capital of West Bengal, under Jawar in India.
Bengaluru is the capital of Karnataka and Chennai is the 2. Gujarat is the largest producer of
capital of Tamil Nadu.
groundnut in India.
30. The correct sequences in decreasing order of 3. Rajasthan has the largest area of cultivable
the four sugarcane producing States in India is: wasteland in India.
(a) Maharashtra U.P., Tamil Nadu, Andhra
4. Andhra Pradesh has the highest per
Pradesh
hectare yield of maize in India.
(b) U.P., Maharashtra, Tamil Nadu, Andhra
Which of these statements are correct?
Pradesh
(a) 1 and 4 (b) 2 and 3
(c) Maharashtra, U.P., Andhra Pradesh, Tamil
Nadu (c) 1 and 3 (d) 2 and 4
(d) U.P. Maharashtra, Andhra Pradesh, Tamil Ans. (c) : Maharashtra has the highest acreage area
Nadu under sorghum in India (Maximum area of 5351000
Ans. (b) : The decreasing order of the sugarcane hectares in India). Karnataka has the highest production
producing States in India is: Uttar Pradesh- per hectare of corn. Andhra Pradesh has become the
132427.6842, Maharashtra- 69648.0768, Karnataka- largest producer of groundnut in India. Rajasthan has
35732, Tamil Nadu- 33919.17, Andhra Pradesh-15567 the largest area of agricultural waste land in India due
31. Match the cities labelled as 1, 2, 3, 4 and 5 in to Thar desert. Here agriculture could only be done by
the given map with the names of the institutes supplying water.
located in these cities and select the correct 33. Match List I with List II and select the correct
answer using the codes given below the names answer using the codes given below the lists:
of the institutes? List I List II
(Industrial Unite) (Centre)
A. Atlas Cycle Company 1. Bangalore
Ltd.
B. Bharat Earth Movers 2. Bhubaneswar
Ltd.
C. Indian Farmers 3. Kalol
Fertilizers Co-
operative Ltd.
D. National Aluminium 4. Sonepat
Company Ltd.
IAS (Pre) GS 2000 Paper I 430 YCT
A B C D
(a) 1 4 2 3
(b) 1 4 3 2
(c) 4 1 2 3
(d) 4 1 3 2
Ans. (d) : Atlas cycle company limited is situated in
Sonepat, Bharat Earth Movers Ltd is located in
Bengaluru, Indian Farmers Fertilizers Cooperative Ltd
is located in Kalol, National Aluminium Company Ltd
is located in Bhubaneswar.
34. Match the locations of ports labelled as A, B, C
and D in the given map with the names of those List (Ports of Pondicherry)
ports and select the correct answer using the 1. Karaikal 2. Mahe
codes given below the names of the ports: 2. Pondicherry 4. Yanam
A B C D
(a) 2 1 3 4
(b) 1 2 3 4
(c) 2 1 4 3
(d) 1 2 4 3
Ans. (a) : A – Mahe
B – Karaikal
C – Podicherry
D – Yanam
Name of Ports
37. Which one of the following pairs of primitive
1. Kakinada 2. Karwar
tribes and places of their inhabitation is NOT
3. Mangalore 4. Tuticorin
correctly matched?
5. Veraval
(a) Buksa : Pauri-Garhwal
A B C D
(a) 4 2 3 5 (b) Kol : Jabalpur
(b) 5 2 4 1 (c) Munda : Chhotanagpur
(c) 1 3 4 2 (d) Korba : Kodagu
(d) 5 3 2 1 Ans. (d) : Korba tribe is found in chhotanagpur of
Ans. (b) : In the given map A marked port is Veraval of Chhattisgarh and Jharkhand, whereas Kodagu is a
Gujarat, B marked port is Karwar of Karnataka, C district in Karnataka.
marked port is Tuticorin of Tamil Nadu and D marked 38. Assertion A : The frequency of floods in North
port is Kakinada of Andhra Pradesh.
Indian plains has increased during the last couple
35. Which one of the following ports of India
of decades.
handles the highest tonnage of import cargo?
Reason R : There has been reduction in the depth
(a) Calcutta (b) Kandla
of river valleys due to deposition of silt.
(c) Mumbai (d) Visakhapatnam
(a) Both A and R are individually true and R is
Ans. (d) : Vishakhapatnam is the port of India which
the correct explanation of A
handles the highest tonnage of import cargo. It is on the
(b) Both A and R are individually true but R is
eastern coast of Andhra Pradesh. In 2009 – 10 the
shipping ministry planned Vishakhapatnam port to not the correct explanation of A
handle 70 million tonnes of cargo. (c) A is true but R is false
Ans. (b) : Presently, Kandla port handle the highest (d) A is false but R is true
tonnage of Import cargo. Ans. (a) : Both assertion and reasons are individually
36. Match the different ports of Union Territory of true and R is the correct explanation of A. The
Pondicherry labelled as A, B, C and D in the deposition of silt on the river bottom is because of many
given map with their respective names and reasons including the pollution which make the river
select the correct answer using the codes given shallower than before, promoting easy flooding of the
below the lists of ports: nearby areas.

IAS (Pre) GS 2000 Paper I 431 YCT


39. Assertion A : Ganga Plain is the most densely 41. Match List I with List II and select the correct
populated part of India. answer using the codes given below the lists:
Reason R :Ganga is the most harnessed river of List I (Institute) List II
India. (Location)
(a) Both A and R are individually true and R is A. Central Institute of 1. Hyderabad
the correct explanation of A Higher Tibetan Studies
(b) Both A and R are individually true but R is B. Indira Gandhi Institute 2. Mumbai
not the correct explanation of A of Development
Research
(c) A is true but R is false
C. National Institute of 3. Bangalore
(d) A is false but R is true
Mental Health and
Ans. (b) : The Ganga plain is most important plain of Neuro-sciences
India. The gangetic plain covers more than a fourth of
D. Central Institute of 4. Dharmshala
the country’s total surface. The alluvial fertile soil is English & Foreign
very suitable for cultivation of different cereals, pulses Languages
and nuts. So bulk of the population resides in this plain
area. Due to population burst across the Ganga, it is 5. Varanasi
most harnessed river of the country.
A B C D
40. Consider the following statements: (a) 5 3 4 1
1. Tides are of great help in navigation and (b) 5 2 3 1
fishing. (c) 3 2 4 5
2. High tide enables big ships to enter or leave (d) 4 5 1 2
the harbour safely. Ans. (b) : Central institute of Higher Tibetan studies is
3. Tide prevents siltation in the harbours. located at Varanasi, Uttar Pradesh since the year 1967.
4. Kandla and Diamond harbour are tidal Indira Gandhi Institute of Development Research is
ports. located in Mumbai, Maharashtra.
Which of these statements are correct? National Institute of Mental Health and
(a) 1 and 4 (b) 2, 3 and 4 Neurosciences is located in Bengaluru, Karnataka.
(c) 1, 2 and 3 (d) 1, 2, 3 and 4 Central institute of English and foreign languages is
Ans. (d) : Tidal ebb is an important phenomenon in the located in Hyderabad, Andhra Pradesh.
oceans that has a direct and indirect impact on humans
and the environment. Hence all the statements in WORLD GEOGRAPHY
question are correct.
42. Match List I with List II and select the correct
The periodical rise and fall of the sea level, once or answer using the code given below the lists:
twice a day, mainly due to the attraction of the sun and List-I List-II
the moon is called a tide. Since tides are caused by the (Minerals) (Major Producer)
earth-moon-sun positions, which are known accurately, A. Mineral Oil 1. Zambia
the tides can be predicted well in advance. This helps B. Copper 2. Guyana
the navigators and fishermen plan their activities. Tidal C. Manganese 3. Venezuela
flows are of great importance in navigation. Tidal D. Bauxite 4. Gabon
heights are very important, especially harbours near A B C D
rivers and within estuaries having shallow ‘bars’ at the (a) 3 1 4 2
entrance, which prevent ships and boats from entering (b) 3 1 2 4
into the harbour. Tides are also helpful in desilting the (c) 1 3 2 4
sediments and in removing polluted water from river (d) 1 3 4 2
estuaries. Tides are also used to generate electrical Ans. (a) : The major producer of mineral oils is
power. Kandala port in Gujarat and Diamond Harbour Venezuela, major producer of copper is Zambia, major
port in Kolkata are the tidal sea ports in India. Hence all producer of Manganese is Gabon and major producer of
the given statements are correct. Bauxite is Guyana.

IAS (Pre) GS 2000 Paper I 432 YCT


43. Which one of the following is known as the 46. Consider the following provinces of former
"Coffee port" of the world? Yugoslavia:
(a) Sao Paulo (b) Santos 1.Bosnia 2. Croatia
(c) Rio de Janeiro (d) Buenos Aires 3.Slovenia 4. Yugoslavia
Ans. (b) : Santos is known as coffee port of the world. The correct sequence of these provides from
Sao Paulo is also called the world's coffee market. The the east to the west is:
city is also the first in Brazil to produce coffee
(a) 4, 1, 3, 2 (b) 4, 1, 2, 3
significantly. Santos is a port which is known as the
Brazilian port. Rio-de Janeiro is the city of Brazil (c) 1, 4, 3, 2 (d) 1, 4, 2, 3
which is a center of attraction for tourists due to its Ans. (b) : After World War II, the monarchy became a
location on the coast of the sea. Buenos Aires is the communist republic under Prime Minister Tito, now
capital of Argentina. It is famous for shipbuilding, as called the Federal People's Republic of Yugoslavia. It
well as for meat production and export. was composed of six republics: Serbia,
44. At which one of the cities labeled as A,B,C and Croatia, Bosnia and Herzegovina, Macedonia, Slovenia,
D on the given map of Europe was the historic and Montenegro, as well as two provinces, Kosovo and
treaty between NATO and Warsaw Pact Vojvodina. The correct sequence from east to west is:
countries signed in 1998? Yugoslavia, Bosnia, Croatia and Slovenia.
47. Which one of the following lakes forms an
international boundary between Tanzania and
Uganda?
(a) Chad (b) Malawi
(c) Victoria (d) Zambezi
Ans. (c) : Lake Victoria acts as the international
boundary between Tanzania and Uganda. It is the
(a) A (b) B
largest lake in Africa and second widest fresh water
(c) C (d) D
body in the world.
Ans. (c) : In the given figure of Europe the 'c' marked Lake Victoria is one of the African Great Lakes. The
city is Brussels, where historic treaty between NATO lake was renamed Lake Victoria after Queen Victoria
and Warsaw pact was signed in 1998. Brussels, by the explorer John Hanning Speke, in his reports, the
officially the Brussels-Capital Region, is a region of first Briton to document it. Speke accomplished this in
Belgium comprising 19 municipalities, including the 1858, while on an expedition with Richard Francis
City of Brussels, which is the capital of Belgium.
Burton to locate the source of the Nile River.
45. Consider the following statements about the
48. Assertion A : In Australia, cattle rearing is done
'Roaring Forties':
more for meat than for milk.
1. They blow uninterrupted in the Northern and
Southern Hemispheres. Reason R : Australians are traditionally non
2. They blow with great strength and constancy. vegetarians.
3. Their direction is generally from North-West to (a) Both A and R are individually true and R is
East in the Southern Hemisphere. the correct explanation of A
4. Overcast skies, rain and raw weather are (b) Both A and R are individually true but R is
generally associated with them. not the correct explanation of A
Which of the statements are correct? (c) A is true but R is false
(a) 1, 2 and 3 (b) 2, 3 and 4 (d) A is false but R is true
(c) 1, 3 and 4 (d) 1, 2 and 4
Ans. (b) : Both A and R is correct but R is not the
Ans. (b) : The Roaring Forties is and 500 westerly wind
correct explanation of A. Though the Australians are
moving at latitude 40° in the southern hemisphere. It
does not run in the northern hemisphere because it has traditionally non-vegetarian but they rear cattle mainly
land area at latitude 40°. Rainfall is intense here due to for exporting meat to other countries.
the high volume of aquifers in the southern hemisphere. 49. Along which one of the following meridians did
Its direction is generally from northwest to east. It India experience the first light of the sunrise of
brings with it excessive moisture due to coming from the new millennium?
the tropical region and crossing the sea, causing heavy
(a) 2º 30' W (b) 82º 30' E
rainfall on the western coasts of the continents, while
the eastern coasts are mostly unaffected by it. (c) 92º 30' W (d) 92º 30' E

IAS (Pre) GS 2000 Paper I 433 YCT


Ans. (d) : Along 92°30'E meridians, India experienced A B C D
the first light of the sunrise of the new millennium. The (a) 2 1 4 3
first light of the sunrise of new millennium in India was (b) 1 2 4 3
experienced at Kathal island of Andaman and Nicobar, (c) 3 2 1 4
which is located at 92°30 E meridian. (d) 2 1 3 4
50. Match List I with List II and select the correct Ans. (a) : Local Bodies Local Bodies States as
answer using the codes given below the lists:
States as in 1999 in 1999
List I List II
(Oceanic Trench) (Location) Zila Parishad at the sub- Assam
A. Aleutian 1. Indian Ocean divisional level
B. Kermadec 2. North Pacific Ocean Mandal Praja Parishad Andhra Pradesh
C. Sunda 3. South Pacific Ocean Tribal Councils Meghalaya
D. S. Sandwich 4. South Atlantic Ocean Absence of Village Mizoram
A B C D Panchayats
(a) 2 4 1 3 53.
The Parliament can make any law for the
(b) 2 3 1 4 whole or any part of India for implementing
(c) 1 3 2 4 International treaties:
(d) 1 4 2 3 (a) With the consent of all the States
Ans. (b) : Aleutian trench is located in North Pacific (b) With the consent of the majority of States
Ocean, Kermadec trench is located in South Pacific (c) With the consent of the States concerned
Ocean, Sunda trench is located in Indian Ocean and (d) Without the consent of any State
Sandwich trench is located in South Atlantic Ocean. Ans. (d) : Article 253 of the Constitution of India states
51. Match the drainage basins labelled as A, B, C that, Parliament has power to make any law for the
and D with the names listed below and select whole or any part of the territory of India for
the correct answer using the codes given belowimplementing any treaty, agreement or convention with
the drainage basins any other country or countries or any decision made at
Name of drainage basins: any international conference, association or other body.
1. Ganga-Brahmputra 2. Indus 54. Which one of the following statements about a
3. Parana 4. Zambezi Money Bill is not correct?
(a) A Money Bill can be tabled in either House of
Parliament
(b) The Speaker of Lok Sabha is the final
authority to decide whether a Bill is a Money
Bill or not
(c) The Rajya Sabha must return a Money Bill
passed by the Lok Sabha and send it for
A B C D consideration within 14 days
(a) 3 1 2 4
(d) The President cannot return a Money Bill to
(b) 1 3 4 2
the Lok Sabha for reconsideration
(c) 1 3 2 4
(d) 3 1 4 2 Ans. (a) : Article 110 of the Constitution defines the
Money Bill. The Money Bill could only be introduced
Ans. (d) :A. Parana River (South America), B. Ganges-
in the Lok Sabha only. The recognition of a bill as a
Brahmaputra River (Asia), C. Zambezi River (Africa),
D. Indus River (Asia). money bill is conferred by the Speaker of the Lok
Sabha. The Rajya Sabha has very little power in
relation to the Money Bill. If the Rajya Sabha does not
INDIAN CONSTITUTION AND POLITY pass a money bill within 14 days, it will automatically
52. Match List I with List II and select the correct deemed to be passed by the Rajya Sabha. Under the
answer using the codes given below the lists: Constitution, a Money Bill cannot be returned to the
House by the President for reconsideration.
List I List II
(Local bodies) (States as in 1999) 55. The 73rd Constitution Amendment Act, 1992
A. ZilaParishads at the 1. Andhra Pradesh refers to the:
(a) Generation of gainful employment for the
sub-divisional level
unemployed and the under-employed men
B. Mandal Praja Parishad 2. Assam and women in rural area
C. Tribal Councils 3. Mizoram (b) Generation of employment for the able
D. Absence of Village 4. Meghalaya bodied adults who are in need and desirous of
Panchayats work during the lean agricultural season
IAS (Pre) GS 2000 Paper I 434 YCT
(c) Laying the foundation for strong and vibrant Ans. (c) : Chief Justice of India, Union Cabinet
Panchayati Raj Institutions in the country Minister, Chief Election Commissioner and Cabinet
(d) Guarantee of right to life, liberty and security Secretary is the correct order of precedence.
of person, equality before law and equal 59. The primary function of the Finance
protection without discrimination. Commission in India is to:
Ans. (c) : 73rd Amendment Act added Eleventh (a) Distribute revenue between the Centre and
Schedule to the Constitution and Part IX, which the States
provides for the Panchayati Raj System. (b) Prepare the Annual Budget
The Constitution (73rd Amendment) Act, 1992 has (c) Advise the President on financial matters
added a new part IX consisting of 16 Articles and the (d) Allocate funds to various ministries of the
Eleventh Schedule to the Constitution. The 73th
Union and State Governments
Amendment envisages the Gram Sabha as the
foundation of the Panchayat Raj System to perform Ans. (a) : Distribute revenue between the Centre and
functions and powers entrusted to it by the State the States.
Legislatures. The Finance Commission of India has been set up under
56. The Speaker can ask member of the House to Article 280 of the Constitution. The first Finance
stop speaking and let another member speak. Commission was set up in 1951 and there have been
This phenomenon is known as: fifteen (2017) so far. Its core responsibility is to
(a) Decorum (b) Crossing the floor evaluate the state of finances of the Union and State
Governments, recommend the sharing of taxes between
(c) Interpolation (d) Yielding the floor
them, lay down the principles determining the
Ans. (d) : Decorum - Parliamentary etiquette distribution of these taxes among States.
Crossing the floor - Changing the party Interpolation -
60. The state which has the largest number of seats
Seeking clarification through ruling
reserved for the Scheduled Tribes in the Lok
Yielding the floor - Respecting speaker's order
Sabha is:
57. Consider the following statements about the (a) Bihar (b) Gujarat
Attorney General of India: (c) Uttar Pradesh (d) Madhya Pradesh
1. He is appointed by the President of India
Ans. (d) : In the given options. Madhya Pradesh, with 6
2. He must have the same qualifications as are seats, has the largest number of seats reserved for the
required for a Judge of the Supreme Court. Scheduled Tribes (STs) in the Lok Sabha, Bihar and
3. He must be a member of either House of Uttar Pradesh have no seats reserved for STs and
Parliament. Gujarat has 4 seats reserved for STs.
4. He can be removed by impeachment by
Parliament. ECONOMY
Which of these statements are correct?
(a) 1 and 2 (b) 1 and 3 61. Consider the following statements:
(c) 2, 3 and 4 (d) 3 and 4 The Indian rupee is fully convertible
1. in respect of Current Account of Balance
Ans. (a) : According to Article 76 of the Constitution, of Payment
The President shall appoint a person who is qualified to
2. in respect of Capital Account of Balance of
be appointed a Judge of the Supreme Court to be
Payment
Attorney-General for India. Hence statement 3 is not
correct. Attorney-General hold office during the 3. into gold
pleasure of the President. Hence statement 4 is also not Which of these statements is/are correct?
correct. (a) 1 alone (b) 3 alone
58. Consider the following functionaries: (c) 1 and 2 (d) 1, 2 and 3
1. Cabinet Secretary Ans. (a) : On 19 August 1994, India attained the full
convertibility of the rupee on the current account.
2. Chief Election Commissioner
Indian rupee payment balance is convertible into current
3. Union Cabinet Ministers account and gold. The Tarapore Committee was
4. Chief Justice of India formed for the full convertibility of Rupees.
Their correct sequence, in the Order of Precedence 62. Resurgent India Bonds were issued in US
is: dollar, Pound, Sterling.
(a) 3, 4, 2, 1 (b) 4, 3, 1, 2 (a) Japanese Yen (b) Deutsche Mark
(c) 4, 3, 2, 1 (d) 3, 4, 1, 2 (c) Euro (d) French Franc
IAS (Pre) GS 2000 Paper I 435 YCT
Ans. (b) : The Resurgent India Bonds (RIBs) are bank (c) Automobile shares have shown a steady
instruments issued by SBI representing foreign currency improvement in price, unaffected by large
denominated deposits in the form of promissory notes fluctuations in BSE Sensex.
offered to Non-Resident Indians and Overseas (d) None of the above
Corporate Bodies. Bonds being denominated in foreign Ans. (d) : According to above graph analyses that none
of the above option given correct.
currency i.e., in US dollar, Pound Sterling and Deutsche
Mark. Bonds are free from forex risk. BSE, the first ever stock exchange in Asia established in
63. The growth rate of per capita income at 1875 and the first in the country to be granted
current prices is higher than that of per capita permanent recognition under the Securities Contract
income at constant prices, because the later Regulation act, 1956, has had an interesting rise to
takes into account the rate of: prominence over the past 143 years.
(a) Growth of population 65. In an open economy, the national income (Y) of
(b) Increase in price level the economy is:
(c) Growth of money supply (C, I, G, X, M stand for Consumption,
(d) Increase in the wage rate Investment, Govt. Expenditure, total exports
and total imports respectively)
Ans. (b) : The growth rate of per capita income at
(a) Y = C + I + G + X
current values is relatively higher than the growth rate
of per capita income at constant values, because the (b) Y = C + I + G – X + M
growth rate of per capita income at constant values is (c) Y = C + I + G + (X–M)
taken into account for the growth rate of the price level. (d) Y = C + I – G + X – M
64. The upper part of the given graph is a Ans. (b) : National income = total consumption
hypothetical movement in the BSE Sensex over Expenditure +Total Invest Total Govt. Expenditure –
a few months and the lower part is the (Export – Import
fluctuation in the average value of automobile Y = C + I – G + (X – M)
shares in the same period (actual values not Y=C+1+G–X+M
given). So that option (b) is correct answer.
66. A rise in 'SENSEX' means
BSE SENSEX
(a) A rise in prices of shares of all companies
registered with Bombay Stock Exchange
(b) A rise in prices of all companies registered
with National Stock exchange
(c) An overall rise in prices of shares of group of
companies registered with Bombay Stock
Exchange
(d) A rise in prices of shares of all companies
belonging to a group of companies registered
with Bombay Stock Exchange
Ans. (c) : The Sensory Index of the Bombay Stock
Exchange (BSE) is called the SENSEX for short. Rise
in the sensitive index refers to the overall rise in the
value of shares of companies listed in the BSE.
67. The new Gross Domestic Product (GDP) series
released by the Central Statistical Organisation
(CSO) in February, 1999 is with reference to
base price of:
Which one of the following inferences can be (a) 1991-92 (b) 1992-93
drawn from the graphs? (c) 1993-94 (d) 1994-95
(a) The automobile-share market has been as Ans. (c) : The new GDP series released by the (CSO) in
unstable as BSE Sensex in that period February, 1999 is with reference to base price of 1993-
(b) There has been a major political change in 94. At present, GDP is calculated keeping 2011-12 as
the base year.
June/July
IAS (Pre) GS 2000 Paper I 436 YCT
68. Match List I with List II and select the correct relative currency value fluctuations (often due to
answer using the codes given below the lists: currency devaluations). Price deflation, financial
List I List II crises and bank failures are also common elements
A. Boom 1. Business activity at high of a depression that do not normally occur during a
level with increasing recession.
income, output and An economic recovery is the phase of the business
employment at macro cycle following a recession, during which an
level economy regains and exceeds peak employment and
output levels achieved prior to downturn.
B. Recession 2. Gradual fall of income,
output and employment 69. Assertion A : The rate of growth of India's
with business activity in exports has shown an appreciable increase after
a low gear 1991.
C. Depression 3. Unprecedented level of Reason R : The Govt. of India has resorted to
under employment and devaluation.
unemployment, drastic (a) Both A and R are true and R is the correct
fall in income, output explanation of A
and employment (b) Both A and R are true but R is NOT the
D. Recovery 4. Steady rise in the correct explanation of A
general level of prices, (c) A is true but R is false
income, output and (d) A is false but R is true
employment Ans. (a) : Devaluation of currency is conscious decision
A B C D taken by Central bank of country to lower the external
(a) 1 2 3 4 value of domestic currency. As a result of this, Indian
(b) 1 2 4 3 goods become cheaper for Foreigners. Import would
(c) 2 1 4 3 become costlier.
(d) 2 1 3 4 70. ".....instill into the vast millions of workers,
Ans. (a) : An economic boom is the expansion and men and women, who actually do the job, a
peak phases of the business cycle. It is also known as an sense of partnership and of cooperative
upswing, upturn, and a growth period. During a boom, performance....." The above passage related to:
key economic indicators will rise. Gross domestic (a) Planned Development
product (GDP), which measures a nation's economic (b) Community Development
output. (c) Panchayati Raj System
Economic recession is a period of general (d) Integrated Development Programme
economic decline and is typically accompanied by a Ans. (b) : The above given passage relates to the
drop in the stock market, an increase in Community Development Programme. This programme
unemployment, and a decline in the housing market. was started in 1952. All over development of rural areas
Generally, a recession is less severe than a was the prime objective of this programme.
depression. 71. The standing Committee of State Finance
In economics, a depression is a sustained, long- Ministers recommended in January, 2000
term downturn in economic activity in one or more uniform rates across the states in respect of:
economies. It is a more severe economic downturn (a) Value added tax
than a recession, which is a slowdown in economic (b) Sales tax
activity over the course of a normal business cycle. (c) Stamp duty and registration fees
Depressions are characterized by their length, by (d) Agricultural income tax
abnormally large increases in unemployment, falls Ans. (b) : A sales tax is a consumption tax charged at
in the availability of credit, shrinking output as the point of purchase for certain goods and services.
buyers dry up and suppliers cut back production Sales Tax is levied on purchase and sale of goods
and investment, more bankruptcies including in India and is levied under the authority of both Central
sovereign debt defaults, significantly reduced Legislation (Central Sales Tax) and State Governments
amounts of trade and commerce (especially Legislations (Sales Tax). The Central government
international trade), as well as highly volatile levies Sales Tax principally on intra-state sale of goods.

IAS (Pre) GS 2000 Paper I 437 YCT


72. Economic Liberalisation in India started with: 75. Insect-resistant cotton plants have been
(a) Substantial changes in industrial licensing genetically engineered by inserting a gene from
policy a/an:
(b) The convertibility of Indian Rupees (a) Virus (b) Bacterium
(c) Doing away with procedural formalities for (c) Insect (d) Plant
foreign direct investment Ans. (b) : Insect resistant cotton plants have been
(d) Significant reduction in tax rates genetically engineered by inserting a gene from
Ans. (a) : In 1991, the Government of India announced bacterium Bacillus thuringiensis.
a new economic policy in relation to various aspects of 76. Which one of the following does a TV remote
the national economy. 'Economic liberalization' was the control unit use to operate a TV set?
main feature of the new policy. It started with the (a) Light waves (b) Sound waves
abolition of the 'licensing system' in respect of all (c) Microwaves (d) Radio waves
industries except industries of national importance.
Ans. (d) : Waves of GHz frequency are used to operate
73. Gilt-edged market means TV sets. Light, sound and microwaves cannot be
(a) bullion market transmitted at GHz, while radio waves can be
(b) market of Government securities transmitted. Radio waves movement depends on the
(c) market of guns direction. This is why the remote has to be placed in
(d) market of pure, metals front of the TV set.
Ans. (b) : Government and semi-government securities 77. A noise level of 100 decibels would correspond
are traded through the Reserve Bank in the 'Gilt Edged' to
market. 'Gilt edged' means the best. It is called (a) Just audible sound
excellent because it keeps the price of government and (b) Ordinary conversation
semi-government securities stable and does not (c) Sound from a noisy street
fluctuate like other securities in this sector. This is the (d) Noise from a machine-shop
reason that banks and other institutions have special
Ans. (d) : The decibel is commonly used in acoustics to
attraction for these securities. quantify sound levels relative to a 0 dB reference which
has been defined as a sound pressure level of .0002
PHYSICS microbar. The noise level of 100 decibel would
74. Consider the following features of newer corresponds to noise from a machine shop.
models of motor cars: 78. A 'black hole' is a body in space which does not
1. Radial tyres allow any radiation to come out. This property
2. Streamlined body is due to its.
3. Multipoint fuel injection (a) very small size (b) very large size
4. Catalytic converter with exhaust (c) very high density (d) very low density
Which of these features make the newer
Ans. (c) : A black hole is a region of spacetime
models of motor cars more fuel efficient?
(a) 1 and 2 (b) 2 and 3 exhibiting gravitational acceleration so strong that
nothing, no particles or even electromagnetic radiation
(c) 2, 3 and 4 (d) 1, 3 and 4
such as light, can escape from it. Temperature is on the
Ans. (b) : A "streamlined" design is one in which order of billionths of a kelvin for black holes of stellar
objects that move through a gas or liquid are shaped to mass, making it essentially impossible to observe. The
match these lines, and therefore, reduce the energy
theoretical density of a black hole depends on how big it
required to produce that motion. Multi point fuel
is and how you define its size. The density at the center
injection system injects fuel into individual cylinders,
of a black hole is infinite, which leads to difficulties to
based on commands from the on board engine
measure density in modern cosmology.
management system computer, popularly known as the
Engine Control Unit/ECU. These techniques result not 79. Fluorescent tubes are fitted with a choke. These
only in better 'power balance' amongst the cylinders but choke oil
also in higher output from each one of them, along with (a) Steps up the line voltage
faster throttle response. The other two options radial (b) Steps down the line voltage
tyres and catalytic converter with exhaust do not (c) Reduces current in the circuit
contribute to make the car more fuel efficient. (d) Chokes low frequency currents
IAS (Pre) GS 2000 Paper I 438 YCT
Ans. (c) : A fluorescent lamp or fluorescent tube is a 82. Assertion A : A man standing on a completely
gas discharge lamp that uses electricity to excite frictionless surface can propel himself by
mercury vapour. The excited mercury atoms produce whistling.
shortwave ultraviolet light that then causes a phosphor Reason R :If no external force acts on a system,
to fluoresce, producing visible light .In fluorescent its momentum cannot change.
tubes the simplest ballast for alternating current use is
(a) Both A and R are individually true and R is
an inductor (choke coil) placed in series, consisting of a
winding on a laminated magnetic core. The inductance the correct explanation of A
of this winding limits the flow of AC current on account (b) Both A and R are individually true but R is
of its large impedance. The only waste of energy is due not the correct explanation of A
to the hysteresis loss in the iron core, which is much (c) A is true but R is false
less than the waste of energy in comparison to (d) A is false but R is true
resistance which can also reduce the current if placed
Ans. (b) : First statement can be explained on the laws
instead of the choking coil.
of motion. First statement involves the use of Newton.
80. For reproducing sound, a CD (Compact Disc)
An initial force is required to start and continue a
audio player uses a
motion at a frictionless surface. Therefore, a man
(a) Quartz crystal
(b) Titanium needle standing on a completely frictionless surface can propel
(c) Laser beam himself by whistling. As per the Newton’s second law
(d) Barium titanate ceramic of motion, The rate of change of momentum of a body
is directly proportional to the applied force and takes
Ans. (c) : In compact disc digital data is scanned by
laser beam for the reproduction of recorded sound. place in the direction in which the force acts. Hence, if
no external force acts on a system, its momentum
81. When a CD (compact disc used in audio and
cannot change.
video systems) is seen in sunlight, rainbow like
colours are seen. This can be explained on the 83. Assertion A : Small glass beads fixed on traffic
basis of the phenomenon of : signals glow brightly when light falls upon them.
(a) Reflection and diffraction Reason R :Light is totally reflected when the
(b) Reflection and transmission angle of incidence exceeds a certain critical value
(c) Diffraction and transmission and light travelling in a denser medium is
(d) Refraction, diffraction and transmission reflected from a rarer medium.
Ans. (d) : The reason CDs reflect rainbow colors is (a) Both A and R are individually true and R is
because they have a clear plastic coating on top of a the correct explanation of A
mirrorized surface. Light refracts (bends) when it moves (b) Both A and R are individually true but R is
from one medium (such as air) to another with a not the correct explanation of A
different optical density (such as the clear plastic
(c) A is true but R is false
surface of a CD). Different wavelengths of light (every
color has a different wavelength) travel at different (d) A is false but R is true
speeds, so that full spectrum appears when white light Ans. (a) : Both the statements are correct and statement
passes from the air through the plastic surface of a CD, 2 is also correct explanation for statement 1. Total
separated light rays which are then reflected back to us internal reflection is an optical phenomenon that
by the mirrorized center surface of a CD. Here the happens when a ray of light strikes a medium boundary
diffraction and transmission also takes place because at an angle larger than a particular critical angle with
diffraction of light rays occurs when it strikes the
respect to the normal to the surface. If the refractive
surface of CD and transmission is obvious when light
index is lower on the other side of the boundary, no
enters from one medium to another. The thickness of
light can pass through and all of the light is reflected.
the different optical media, angle of source light, and
brightness of source light reflects rainbow pattern on a Thus because of this phenomena traffic light beads glow
CD. when light falls on them.

IAS (Pre) GS 2000 Paper I 439 YCT


CHEMISTRY 87. Match List I (Industrial processes) with List II
(Industry with which associated) and select the
84. Soft drinks such as colas contain significant correct answer using the codes given below:
quantities of List I List II
(a) Caffeine (b) Nicotine A. Cracking 1. Rubber
(c) Tannin (d) Renin B. Smelting 2. Petroleum
C. Hydrogenation 3. Copper
Ans. (a) : Caffeine is a xanthine alkaloid compound that
D. Vulcanization 4. Edible Fats
acts as a stimulant in humans. Caffeine is sometimes
A B C D
called as guaranine, when found in guarana, mateine
(a) 3 2 1 4
when found in mate and theine when found in tea. It is
(b) 2 3 4 1
found in the leaves and beans of the coffee plant, in tea,
(c) 2 3 1 4
yerba mate and guarana berries and in small quantities
(d) 3 2 4 1
in cocoa.
Ans. (b) : Cracking is the process whereby complex
85. Aluminium surfaces are often 'anodized'. This organic molecules such as kerosene or heavy
means the deposition of a layer of hydrocarbons are broken down into simpler molecules
(a) Chromium oxide (b) Aluminium oxide such as light hydrocarbons, by the breaking of carbon-
(c) Nickel oxide (d) Zinc oxide carbon bonds. Smelting is a form of extractive
metallurgy. Its main use is to produce a metal from its
Ans. (b) : Anodizing is an electrolytic passivation
ore. In this process, the oxide ore is reduced to free
process used to increase the thickness of the natural
metal by using reducing agents like coke or charcoal.
oxide layer on the surface of metal parts. The process is Copper is also obtained from its oxide by this method in
called "anodizing" because the part to be treated forms free state. Hydrogenation is a reduction reaction which
the anode electrode of an electrical circuit. Anodizing results in an addition of hydrogen (usually as H2). If an
increases corrosion and withstand resistance. The organic compound is hydrogenated, it becomes more
anodized aluminium layer is grown by passing a direct 'saturated'. Hydrogenation results in the conversion of
current through an electrolytic solution, with the liquid vegetable oils to solid (edible) fats.
aluminium object serving as the anode. The current Vulcanization is a chemical process for converting
releases hydrogen at the cathode and oxygen at the rubber into more durable materials via the addition of
surface of the aluminium anode, creating a buildup of sulphur. Sulphur modify the rubber by forming
aluminium oxide layer. crosslinks (bridges) between individual polymer chains.
Vulcanized material is less sticky and has superior
86. Consider the following statements: mechanical properties.
Hard water is not suitable for 88. Assertion A : Large cold storage plants use
1. Drinking ammonia as refrigerant while domestic
2. Washing clothes with soap refrigerators use chlorofluoro-carbons.
3. Use in boilers Reason R : Ammonia can be liquified at ambient
4. Irrigating crops temperatures at low pressures.
Which of these statements are correct? (a) Both A and R are individually true and R is
(a) 1 and 3 (b) 2 and 3 the correct explanation of A
(c) 1, 2 and 4 (d) 1, 2, 3 and 4 (b) Both A and R are individually true but R is
not the correct explanation of A
Ans. (d) : Hardness of water is due to presence of
(c) A is true but R is false
chlorides, sulphates and nitrates of calcium and
magnesium. Hard water cannot be used for washing (d) A is false but R is true
purposes as soap do not form lather with hard water Ans. (a) : Both A and R are individually true and R is
easily which results in to wastage of soap in laundries. the correct explanation of A.
Moreover use of hard water in industries results in to Large cold storage plants use ammonia as refrigerant
scale formation in inner layers of boilers on which while domestic refrigerators use chlorofluorocarbons
insoluble calcium and magnesium carbonates gets because ammonia could only be liquefied at ambient
temperature and low pressure, which is not possible at
deposited and being a bad conductor of heat result in to
the domestic refrigerators. That is why
wastage of energy. Hard water is not fit for drinking
chlorofluorocarbons (CFC) is used in domestic
purpose as it hinders ionic imbalance in body. Repeated refrigerators as a coolant though it is harmful to the
irrigation of crops by hard water increases calcium and environment. Hence, both A and R are individually true
magnesium ions in soil which increases soil alkalinity. and R is the correct explanation of A.
IAS (Pre) GS 2000 Paper I 440 YCT
89. Most of the explosions in mines occur due to BIOLOGY
the mixing of:
92. Hybridoma technology is a new biotechnological
(a) Hydrogen with oxygen
approach for commercial production of:
(b) Oxygen with acetylene
(a) Monoclonal antibodies
(c) Methane with air (b) Interferon
(d) Carbon dioxide with ethane (c) Antibiotics
Ans. (c) : Methane explosions are one of the major (d) Alcohol
source of explosion in coal mines. It occurs due to the Ans. (a) : Hybridoma technology is being used for
presence of build up of methane (a by product of coal), commercial production of monoclonal antibodies. The
when it comes in contact with a heat source. When air monoclonal antibodies are protective protein produced
contains 9.5% of , it reaches the perfect oxidation point by the clone of single immune cell.
and explodes. So, the correct answer is 'Methane and 93. The sensation of fatigue in the muscles after
air'. prolonged strenuous physical work is caused by:
(a) A decrease in the supply of oxygen
90. Match List I with List II and select the correct
(b) Minor wear and tear of muscles fibres
answer using the codes given below the lists:
(c) The depletion of glucose
List I List II
(d) The accumulation of lactic acid
A. German Silver 1. Tin
Ans. (d) : After prolonged rigorous physical work, the
B. Solder 2. Nickel body experiences fatigue due to accumulation of lactic
C. Bleaching Powder 3. Sodium acid in the muscles.
D. Hypo 4. Chlorine 94. A small pouch containing, silica gel is often
A B C D found in bottles of medicine in tablet or powder
(a) 1 2 4 3 form because silica gel:
(b) 2 1 3 4 (a) Kills bacteria
(c) 1 2 3 4 (b) Kills germs and spores
(d) 2 1 4 3 (c) Absorbs moisture
Ans. (d) : Nickel silver, also known as German silver, (d) Absorbs all gases present inside the bottle
is a copper alloy with nickel and often zinc. Solders are Ans. (c) : Silica gel granule is a network of inter-
typically made from tin or lead or a combination of both connecting microscopic pores, which attract and hold
in the ratio of 63:37 respectively. Calcium hypochlorite, moisture by the phenomena known as absorption.
also known as bleaching powder, is a chemical 95. Canola refers to special type of oil seed mustard
compound with formula Ca(ClO)2. It is widely used for varieties bred for human consumption. The main
water treatment and as a bleaching agent. This chemical characteristic of these varieties is that the:
is considered to be relatively stable and has greater (a) Seeds have very high oil content
(b) Oil is rich in unsaturated fatty acids
available chlorine than sodium hypochlorite (liquid
(c) Oil has long shelf-life
bleach). Hypo solution is the abbreviation for sodium
(d) Oil has very low Erucic acid content
thiosulphate or sodium hyposulphite, a chemical used to
Ans. (b) : Canola oil is the lowest in saturated fat, it is
fix the image on photographic film after it has been high in certain omega 3 fatty acid and mono saturated
developed. fats, which are found in nuts and are healthy for the
91. Which one of the following materials is very heart.
hard and very ductile? 96. Assertion A : In human beings, the females play a
(a) Carborundum (b) Tungsten major role in determining the sex of the offspring.
(c) Cast iron (d) Nichrome Reason R : Women have two 'X' chromosomes.
Ans. (d) : Nichrome is a non-magnetic alloy of nickel, (a) Both A and R are individually true and R is
chromium, and iron, usually used as a resistance wire. A the correct explanation of A
(b) Both A and R are individually true but R is
common alloy is 80% nickel and 20% chromium, by
not the correct explanation of A
mass. This alloying providenichrome properties like (c) A is true but R is false
hardness and ductility. (d) A is false but R is true
IAS (Pre) GS 2000 Paper I 441 YCT
Ans. (d) : Assertion is false, but R is true. In human Ans. (a) : Stones are typically classified by their
being, male plays the major role in determining the sex location: nephrolithiasis (in the kidney), ureterolithiasis
of the offspring because male has XY-chromosome, (in the ureter), cystolithiasis (in the bladder), or by what
while the female has two X-chromosome. XY they are made of (calcium oxalate, uric acid, struvite,
combination of chromosome produces male child while cystine).
XX combination produces female child. 100. At which stage in its life cycle does the silk
97. Assertion A : "DNA Finger printing" has become worm yield the fibre of commerce?
a powerful tool to establish paternity and identity (a) Egg (b) Larva
of criminals in rape and assault cases. (c) Pupa (d) Imago
Reason R : Trace evidences such as hairs, saliva Ans. (c) : Silk worm yield the fibre from pupa stage,
and dried semen are adequate for DNA analysis. which is commercially used to produce clothes.
(a) Both A and R are individually true and R is 101. Match List I (Diseases) with List II (Types of
the correct explanation of A disease) and select the correct answer using the
(b) Both A and R are individually true but R is codes given below the lists:
not the correct explanation of A List I List II
(c) A is true but R is false A. Haemophilia 1. Deficiency disease
(d) A is false but R is true B. Diabetes 2. Genetic disease
Ans. (a) : DNA fingerprinting is a laboratory technique C. Rickets 3. Hormonal
used to establish a link between biological evidence and disorder
a suspect in a criminal investigation. A DNA sample D. Ringworm 4. Fungal infection
taken from a crime scene is compared with a DNA A B C D
sample from a suspect. If the two DNA profiles are (a) 2 3 4 1
matched, then the suspect is proved culprit. For rape (b) 2 3 1 4
and assault cases, body fluid identification of stains (c) 3 2 1 4
such as differentiation of semen, saliva, vaginal fluid (d) 3 2 4 1
and menstrual blood can be critical in identifying
Ans. (b) : Ringworm is a fungal infection in humans.
probative evidence and corroborating the events of an
Rickets is caused by deficiency of Vitamin D, thus a
assault.
deficiency disease. Hemophilia is a genetic disorder and
98. Assertion A : In a motion picture, usually 24 Diabetes is hormonal disorder that is caused by
frames are projected every second over the whole irregularities of Insulin, a vital hormone secreted by
length of the film. pancreas.
Reason R :An image formed on the retina of eye
102. The blood glucose level is commonly expressed
persists for about 0.1s after the removal of
as:
stimulus.
(a) mm. of HG
(a) Both A and R are individually true and R is
(b) Milligram per deciliter
the correct explanation of A
(b) Both A and R are individually true but R is (c) Parts per million
not the correct explanation of A (d) Grams per litre
(c) A is true but R is false Ans. (b) : The level of blood glucose level is measured
(d) A is false but R is true by insulin and glucagon secreted by the pancreas. The
Ans. (c) : In a motion picture, usually 24 frames are amount of glucose is measured in mg/dL. A normal
projected every second over the whole length of the fasting (no food for eight hours) blood sugar level is
film because it was determined to be the minimum between 70 and 99 mg/dL. A normal blood sugar level
speed needed to capture video while still maintaining two hours after eating is less than 140 mg/dL.
realistic motion. Hence A is true. The image of an 103. Match List I (Endocrine glands) with List II
object seen by our eyes persists on retina for about (Hormones secreted) and select the correct
1/16th of a second even after object has disappeared answer:
from our view. Hence R is false. List I List II
99. The 'stones' formed in human kidney consist A. Gonads 1. Insulin
mostly of B. Pituitary 2. Progesterone
(a) Calcium oxalate (b) Sodium acetate C. Pancreas 3. Growth hormones
(c) Magnesium sulphate (d) Calcium D. Adrenal 4. Cortisone
IAS (Pre) GS 2000 Paper I 442 YCT
A B C D CURRENT AFFAIRS
(a) 3 2 4 1
(b) 2 3 4 1 107. The Swarna Jayanti Shahari Rozgar Yojana
(c) 2 3 1 4 which came into operation from 1-12-1997 aims
to provide gainful employment to the urban
(d) 3 2 1 4
unemployed or underemployed poor but does
Ans. (c) : Progesterone secrets from gonads. Growth not include:
hormone secrets from pituitary gland. Insulin hormone (a) Nehru RozgarYojana
secrets from pancreas. Cortisone secrets from adrenal (b) Urban Basic Services Programme
gland. (c) Prime Minister's Integrated Urban Poverty
Eradication Programme
ENVIRONMENT AND ECOLOGY (d) Prime Minister's RozgarYojana
Ans. (d) :Two sub schemes of Swarna Jayanti Shahari
104. Within biological communities, some species
Rozgar Yojana (SJSRY) are Urban self employment
are important in determining the ability of a programme and Urban wage employment programme.
large number of other species to persist in the Three schemes Nehru RozgarYojana (NRY), Urban
community. Such species are called Basic Services Programme (UBSP), Prime Minister's
(a) Keystone species (b) Allopatric species Integrated Urban Poverty Eradication Programme
(c) Sympatric species (d) Threatened species (PMIUPEP) were merged in this programme.
Ans. (a) : Within biological communities, some species 108. The given map shows locations of airports
may be important in determining the ability of large labelled as 1, 2, 3, 4 and 5. What is the correct
number of other species to persist in the community. sequence of the airports in which the hijacked
It has a disproportionately large effect on its Indian Airlines place IC-814 landed after its
initial take off from Kathmandu in December
environment relative to its abundance. These crucial
1999?
species are known as key stone species.
Allopatric species are species having exclusive areas of
geographic distribution.
Sympatric species are species having overlapping area
of geographical distribution.
Threatened species are those which are under threat of
extinction because of human beings destruction.
Therefore, the correct answer is option A. (a) 3, 1, 2, 4 (b) 2, 4, 1, 3
105. Which one of the following organisms can serve (c) 5, 4, 2, 3 (d) 5, 1, 3, 2
as a biofertilizer for rice crop? Ans. (c) : In December 1999, the hijacked aircraft IC-
(a) Blue-green algae (b) Rhizobium sp 814 landed in Amritsar, Lahore, Dubai and Kandahar
(c) Mycorrhizal fungi (d) Azotobactersp respectively, shown in the order of 5, 4, 2, 3
respectively.
Ans. (a) : Blue green algae is used as a biofertilizer in
rice crop. Blue green algae has the nitrogen fixing 109. Consider the following statements:
The Ministerial Meeting of WTO held in
ability which enhances the production of rice.
December 1999 was unsuccessful because it
106. Phytoron is a facility to: attempted to link trade with
(a) Grow plants under disease-free conditions 1. Labour related issues
(b) Conserve endangered species of plants 2. Environment related issues.
(c) Grow plants under controlled conditions 3. Terrorism related issues.
(d) Induce mutations 4. Debt related issues.
Ans. (c) : Phytotron is a type of chamber, designed to Which of these statements are correct?
provide a controlled environment for the study of plant (a) 1,3 and 4 (b) 1 and 2
growth. (c) 2 and 3 (d) 2 and 4
IAS (Pre) GS 2000 Paper I 443 YCT
Ans. (b) : Seattle WTO protests, 1999, sometimes Ans. (b) : According to Global Competitive Report of
referred to as the Battle of Seattle or the Battle in 'World Economic Forum' in 1999, Singapore occupied
nd
Seattle, were a series of protests related to labour and the first place with Index 2.12, India was at 52 with
environment issues. Protesters surrounded the WTO Index - 1.30. Russia was at the bottom with Index -
Ministerial Conference of 1999, when members of the 2.02.
World Trade Organization (WTO) convened at the 114. Match List I with List II and select the correct
Washington State Convention and Trade Center in answer using the codes given below the lists:
Seattle, Washington on November 30, 1999. List I List II
110. The following news item appeared in a A. Development 1. UN India
National daily dated 1-12-1999: Programme Human
".....Parliament today rejected a Bill to grant Development
women the right to vote and stand for office in Report
parliamentary elections, by a margin of 32 to B. National 2. India
30. Council of Development
The National Assembly was split between Applied Report
liberal, pro-government and Shiite Muslim Economic
deputies who were in favour of women's right, Research
while the opposition camp grouped Sunni C. Indira Gandhi 3. Word
Muslim fundamentalists and tribal MPs. Institute of Development
A total of 64 MPs and Ministers were present, Development Report
of whom two abstained." Research
The Parliament referred to in this quotation is D. World Bank 4. Human
that of Development
(a) Kuwait (b) Iran Report
(c) Bahrain (d) Saudi Arabia A B C D
Ans. (a) : The Parliament referred to in this quotation is (a) 4 1 2 3
that of Kuwait. (b) 4 2 1 3
111. The first Indian state to have its Human (c) 2 3 4 1
Development Report prepared and released by (d) 2 1 4 3
Amartya Kumar Sen in Delhi is Ans. (a) : The 'UN-India Human Development Report'
(a) West Bengal (b) Kerala is released by the National Council of Applied
(c) Madhya Pradesh (d) Andhra Pradesh Economic Research, NCAER. The 'India Development
Ans. (c) : Madhya Pradesh was the first state to have Report,' is presented by the Indira Gandhi Institute of
calculated Human Development Report in 1995, under Development Research, ICIDR. The World
the guidance of Prof. Amartya Sen. Development Report is released every year by the
World Bank. Similarly, the 'Human Development
112. The best performance in terms of Human
Report, (HDR) is related to the' Development Program '.
Development among the Asian countries is by
(a) China (b) Malaysia 115. Indian Human Development Report does not
(c) Korea (d) Philippines give for each sample village.
(a) Infrastructure and Amenities Index
Ans. (c) : Human development in a country depends on
life expectancy of the individual and standard of living (b) Education Related Index
of the population. Among Asian countries, Korea is the (c) Health Related Index
best performer in human development index. Its infant (d) Unemployment Related Index
mortality rate is 6% and literacy rate is 98%. Ans. (d) : Indian Human Development Report does not
113. Which one of the following countries occupies give Unemployment Related Index for each sample
the first place in the "Global Competitive village. The Human Development Index (HDI) is a
Report" of World Economic Forum? composite statistic of life expectancy, education and per
(a) U.S.A. (b) Singapore capita income indicators, which are used to rank
(c) Hong Kong (d) France countries into four tiers of human development.

IAS (Pre) GS 2000 Paper I 444 YCT


116. Match the international events listed below MISCELLANEOUS
with their respective places labelled as 1, 2, 3 4
and 5 in the given map and select the correct 118. Consider the following statements regarding
the Chakiarkoothu form of dance:
answer using the codes given below the lists:
1. It is performed by Chakiar caste.
2. It cannot be traditionally witnessed by the
higher caste Hindus.
3. Mizhavu is the accompanying instrument.
4. It theatre form is called koothambalam.
Which of these statements are correct?
(a) 1, 3 and 4 (b) 1, 2 and 3
(c) 2, 3 and 4 (d) 1, 2 and 4
Ans. (a) : Chakiarkoothu is the famous folk dance of
Kerala. two instruments accompany the performance, a.
Mizhavu and a pair of ilathalam regarding which
International Events: statement 1, 2, 3 and 4 are correct. This dance form is
believed to been introduced to Kerala by the early
1. Venue of Commonwealth Conference held
Aryan immigrants & is performed only by the members
in 1999 of the Chakiar caste.
2. Venue of World Trade Organisation 119. Consider the following pictures of a dice:
Meeting held in 1999
3. Place of Israel-Syria Peace talks held in
January, 2000
4. Place of military action by Russian troops
in January, 2000
A B C D
(a) 2 1 5 3 What is the number opposite 3?
(a) 1 (b) 4
(b) 3 4 2 1
(c) 5 (d) Data insufficient
(c) 4 1 2 3
Ans. (b) : From figure (i) & (ii). It is clear that 2 is
(d) 4 3 5 2 adjacent to 3 and 4 is adjacent to 2, hence 4 must be
Ans. (c) : A. Conference site of the Commonwealth opposite to 3.
Conference held in 1999- Durban, B. Conference site of 120. Match List I (Books) with List II (Authors)
and select the correct answer using the codes
the World Trade Organization meeting in 1999- Seattle,
given below the lists:
C. Israel Syria Peace Dialogue in January, 2000- List I List II
Washington, D. Russian soldiers in January, 2000, A. My Music, My Life 1. Laxman Gaikwad
Location of military action - Dagestan to Chechnya. B. Adha Gaon 2. Rahi Masoom Raza
C. Radha 3. Ramakanta Rath
117. To reach the final of first grand slam of the D. The Pilferer 4. Ravi Shankar
year 2000, Martina Hingis defeated A B C D
(a) Lindsay Davenport (a) 3 2 4 1
(b) 4 2 3 1
(b) Jennifer Capriati
(c) 4 1 3 2
(c) SarenaWilliems (d) 3 1 4 2
(d) Conchita Martinez Ans. (b) : “My Music my Life” is written by famous
Ans. (d) : To reach the final of Australian open which is sitar player Pandit Ravi Shankar “Adha Goon” is
written by Rahi Masoom Raza. “Radha” is a novel of
the first grandslam of the year 2000, Martina Hingis
Ramakanta Rath “The Pilferer” is written by Laxman
defeated Conchita Martinez 6-3,6-2. Gaikwad.

IAS (Pre) GS 2000 Paper I 445 YCT


121. Examine the following relationships among 123. Match List I (Power generation plant) with List
members of a family of six persons A, B, C, D, II (Feed material) and select the correct answer
E and F: using the codes given below the lists:
1. The number of males equals that of females. List I List II
2. A and E are sons of F. A. M/s Gowthami Solvent 1. Rice Husk
Oil Limited,
3. D is the mother of two, one boy and one girl.
Andhra Pradesh
4. B is the son of A.
B. M/s K.M. Sugar Mills, 2. Slaughter-
5. There is only one married couple in the
Uttar Pradesh house waste
family at present.
C. M/s Satia Paper Mills, 3. Distillery
Which one of the following inferences can be Punjab spent wash
drawn from the above? D. M/s Al Kabeer Exports 4. Black Liquor
(a) A, B and C are all females Ltd. Andhra Pradesh
(b) A is the husband of D A B C D
(c) E and F are children of D (a) 3 1 2 4
(d) D is the granddaughter of F (b) 3 1 4 2
Ans. (b) : Clearly, there are three males and three females. (c) 1 3 4 2
Now, A, E and B are sons and hence male. So, C, D and F (d) 1 3 2 4
Ans. (c) : M/S Gowthami Solvents Oil Limited, Andhra
are females. D has one male and one female child. Clearly,
Pradesh power generating plant uses Rice husk as feed
A and D form a couple having two children, namely B and
material, M/s KM Sugar Mills, Uttar Pradesh uses
C. Hence, A is the husband of D.
distillery spent wash as feed material, M/S Satia Paper
122. The given diagram shows the number of Mills, Punjab uses black liquor as feed material and
students who failed in an examination M/S Al Kabeer Exports Limited Andhra Pradesh uses
comprising papers in English, Hindi and slaughter house wash as feed material.
Mathematics. The total number of students 124. In a certain code, MARCH is written as
who took the test is 500. What is the percentage OCTEJ, how is RETURN written in that code?
of students who failed in at least two subjects? (a) TFUVSM (b) QGSTQM
(c) TGVWTP (d) TGRVSO
Ans. (c) : 1 2 3 4 5 6 7 8 9 10 11 12 13
A B C D E F G H I J K L M
14 15 16 17 18 19 20 21 22 23 24 25 26
N O P Q R S T U V W X Y Z

(a) 6.8 (b) 7.8


(c) 34 (d) 39

Ans. (b) :
125. A, B, C, D, E and F not necessarily in that
order are sitting in six chairs regularly placed
around a round table. It is observed that
A is between D and F
C is opposite D
→ No of student who failed in atleast two test D and E are not on neighbouring chairs
= 10 + 12 + 12 + 5 → student who failed in 3 subject Which one of the following must be true?
= 39 (a) A is opposite B
(b) D is opposite E
39
→ %= ×100 = 7.8% (c) C and Bare neighbours
500 (d) B and E are neighbours
IAS (Pre) GS 2000 Paper I 446 YCT
Ans. (d) : 1. A is between D & F (a) 6 (b) 12
(c) 18 (d) 24
Ans. (b) : Let the number of boys be x. Then, (3/4) x =
18 or x = 18 x(4/3) = 24.
If total number of students is y, then (2/3) y = 24 or
y = 24 x (3/2) = 36.
Therefore Number of girls in the class = (36 - 24) = 12.
128. A college student desires to get elected to the
2. C is opposite to D
Municipal Council of his city. The validity of his
nomination would dependent on the important
condition, among others, that:
(a) He obtains permission from the Principal of
his college
(b) He is a member of a political party
(c) His name figures in the Voter's List
(d) He files a declaration owing allegiance to the
Constitution of India
3. D & E are not neighbors
Ans. (c) : A person needs to be 21 years of age for
contesting in municipal elections. Unless one's name is
not in the voters' list, he is not eligible to contest
Municipal Council in his city.
129. Which one of the following statements is
incorrect?
(a) Goa attained full statehood in 1987
(b) Diu is an island in Gulf of Khambhat
(c) Daman and Diu were separated from Goa by
By seeing this career we can say that B and E are
the 56th Amendment of the Constitution of
neighbors.
India
126. Match List-I with List-II and select the correct (d) Dadra and Nagar Haveli were under French
answer using the codes given below the lists colonial rule till 1954
List I List II
Ans. (d) : The Annexation of Dadra and Nagar Haveli,
(Artist) (Medium of also known as the Invasion of Dadra and Nagar Haveli,
music delivery) was the conflict in which the territories of Dadra and
A. Balamurali Krishna 1. Hindustani vocal Nagar Haveli passed from Portuguese rule to Indian
B. MitaPandit 2. Ghatam Union rule in 1954. The territories were subsequently
C. Kanyakumari 3. Sitar merged into the Indian Union in 1961.
D. Nikhil Bannerjee 4. Carnatic vocal 130. A rectangular piece of iron sheet measuring 50
5. Violin cm by 100 cm is rolled into a cylinder of height
A B C D 50 cm. If the cost of painting the cylinder is Rs.
(a) 5 1 2 3 50/- per square meter, then what will be the
(b) 4 3 1 5 cost of painting the outer surface of the
(c) 3 1 5 2 cylinder?
(d) 5 4 1 3 (a) Rs. 25.00 (b) Rs. 37.50
(c) Rs. 75.00 (d) Rs. 87.50
Ans. (a) : Balamurali Krishna is famous carnatic
vocalist, Mita Pandit is famous Hindustani vocalist,
Kanyakumari is famous for Ghatam from of singing and Ans. (a) :
Nikhil Banerjee is famous sitar player.
127. In a class there are 18 boys who are over 160
cm tall. If these boys constitute three-fourths of
the boys and the total number of boys is two-
third of the number of students in the class,
then what is the number of girls in the class?
IAS (Pre) GS 2000 Paper I 447 YCT
Here we take only curved surface area, not total surface (c) 11 (d) 16
area because the upper and bottom surface are opened Ans. (d) : A bag contains 20 balls, 8 balls are Green,
which we can’t paint it). Fairy White and 5 are Red. We have to pick at least one
ball of each colour.
Earned surface area = 2πrh Now considering this–
∵ 2πr = 100 cm  Even if we pick 8 balls, there is probability that all
= 100 × 50   of them may be green. Hence we must pick more
 h = 50 cm 
than 8 balls for sure.
= 5000 cm2 Now, if we pick 8 + 7 balls, that is 15 balls, there is
5000 a probability that all of them may be green and
= = .50m 2 {By converting cm2 into m2} white only with no red ball picked at all.
100 ×100
Hence, taking a worst possible condition we must
Cost of painting = 50/per meter pick = 15 + 1 = 16 balls to be sure that at least one
∴ total cost of painting = 0.5 × 50 = 25 Rs. ball of all the three colour is picked.
131. Which one of the following statements is not 135. Which one of the following has a greater
perimeter than the rest?
true?
(a) A square with an area of 36 sq. cm.
(a) Ghaggar's water is utilised in the Indira (b) An equilateral triangle with a side of 9 cm.
Gandhi Canal (c) A rectangle with 10 cm. as length and 40 sq.
(b) Narmada rises from Amarkantak region cm. as area
(c) Nizam Sagar is situated on the Manjira river (d) A circle with a radius of 4 cm.
(d) Penganga is a tributary of the Godavari Ans. (c) : Side of the square 36 = 6cm Perimeter of the
Ans. (a) : Indira Gandhi canal originated from Harike square = 4(6) = 24 cm (b) Perimeter of the triangle = 9
+ 9 + 9 = 27 cm (c) Area of the rectangle = 40 lb = 40
barrage at Sultanpur on Sutlej but Ghaggar is a tributary b = 40/10 = 4 Perimeter = 2(l + b) = 2(4 + 10) = 28 cm
of river Saraswati, which ends in the Thar desert. (d) Perimeter of the circle = 2πr = 2(3.14)(4) = 25.12
132. In an examination, every candidate took cm Clearly, Perimeter of the rectangle is maximum.
physics or mathematics or both 65.8% took 136.
physics and 59.2% took mathematics. The total
number of candidates was 2000. How many
candidates took both physics and mathematics?
(a) 750 (b) 500
(c) 250 (d) 125
In the given figure angle OQP=30º, and angle
Ans. (b) : Let x% candidates take both the subjects. ORP = 20°, angle QOR is equal to:
Percentage of candidates who opted Physics (a) 100º (b) 120º
= 65.8% and percentage of candidates who opted (c) 130º (d) 140º
Mathematics = 59.2% Ans. (a) : In ∆POQ
∴ x = (65.8 + 59.2 - 100)% = (125 - 100)% = 25% Side OQ = OP = radius
Also, total number of candidates = 2000 Then ∠PQO = ∠OPQ = 30
∴ Number of candidates who opted both the subjects In ∆POR
= 25 x 2000/100 = 500 OR = OP = Radius
133. The memory of a computer is commonly
∴ ∠ORP = ∠RPO = 20°
expressed in terms of kilobytes or Megabytes.
A byte is made up of: ∴ ∠QPR = ∠QPO + ∠RPO = 30 + 20 = 80
∵ Angle at the centre = 2 × (Angle of the perimeter)
(a) eight binary digits (b) eight decimal digits
(c) two binary digits (d) two decimal digits ∠QOR = 2× 50 = 100°
Ans. (a) : The byte is a unit of digital information in 137. A club has 108 members. Two-thirds of them
computing which consists of eight bits. are men and the rest are women. All members
134. A bag contains 20 balls. 8 balls are green, 7 are
white and 5 are red. What is the minimum are married except for 9 women members.
number of balls that must be picked up from How many married women are there in the
the bag blind-folded (without replacing any of club?
it) to be assured of picking atleast one ball of
each colour? (a) 20 (b) 24
(a) 4 (b) 7 (c) 27 (d) 30
IAS (Pre) GS 2000 Paper I 448 YCT
Ans. (c) : No. of women = 1/3 (108) = 36
∴ No. of unmarried women = No of women – No of (a) (b)
unmarried women = 36 – 9 = 27
138. The following figure represents times vs.
learning curves of two students, Q and R for
learning a mathematics lesson (c) (d)

Ans. (c) :

Which one of the following inferences can be


drawn from the graph?
140. A person starts from a point A and travels 3
(a) R started slowly in the beginning but go
km eastwards to B and then turns left and
ahead of Q to complete learning the lesson travels thrice that distance to reach C. He
(b) Q started slowly and finished learning the again turns left and travels five times the
distance be covered between A and B and
lesson earlier then R
reaches his destination D. The shortest distance
(c) R was always faster than Q in learning between the starting point and destination is
mathematics (a) 18 km (b) 16 km
(d) Q was always faster than R in learning (c) 15 km (d) 12 km
mathematics Ans. (c) : The movement of the person are as shown in
fig.
Clearly AB = 3 km
BC = 3AB = (3×3) KM = 9KM CD = 5AB = (5×3) KM
= 15KM
Ans. (a) :
Draw AE ⊥ CD.
Then CE = AB = 3 km and AE = BC = 9 Km DE =
(CD-CE) = (15-3) Km = 12 Km.
In ∆ AED, AD2 = AE2+DE2 = AD = (92+(12)2) km =
15km
∴ Required distance = AD = 15 Km.
141. The distribution of 1,00,000 tourists who visited
R start slowly in the beginning but go ahead of Q to India during a particular year is shown in the
compete learning the lesson. given charts. Based on this, the number of
Japanese tourists below the age of 39 who
139. A goat is tied to two poles P and Q with ropes visited India in the year concerned is
that are 15 meters long, P and Q are 20 meters
apart as shown in the given diagram:

(a) 10,000 (b) 8,000


(c) 6,000 (d) 4,000
Which one of the following shaded portions
indicates the total area over which the goat can Ans. (d) : Tourists from Japan = 20/100 (100000)
graze? =20000
% below 39 years = 10 + 10 = 20%

IAS (Pre) GS 2000 Paper I 449 YCT


∴ % of Tourists from Japan below 39 years = 20/100 144. Which one of the following statements is false?
(20000) =4000 (a) All countries other than India have over 80%
142. What is maximum number of pieces of 5cm ⋅ literacy
5cm ⋅ 10cm cake that can be cut from a big (b) Malaysia and Korea have life expectancy
cake of 5cm ⋅ 30cm ⋅ 30cm size? higher than all other countries
(a) 10 (b) 15 (c) Higher the adult literacy lower is the infant
(c) 18 (d) 30 mortality
Ans. (c) : Maximum number of pieces that can be cut (d) The life expectancy at birth in India is almost
out the same as that of Indonesia
= (Volume of cake) / (Volume of Each piece of cake)
Ans. (c) : The false statement is (c), higher the adult
= (5 × 30 × 30)/(5 × 5 × 10) = 18
literacy lower is the infant morality.
143. Which one of the following statements is true
145. A rectangular water tank measures 15 m × 6 m
according to 1991 census data?
at top and is 10 m deep. It is full of water. If
(a) U.P. has the highest density of population in
water is drawn out lowering the level by 1 meter,
India
how much of water has been drawn out?
(b) Himachal Pradesh has the highest female to
male sex ratio in India (a) 90,000 litres
(c) West Bengal has the highest growth rate of (b) 45,000 litres
population in India
(c) 4,500 litres
(d) Bihar has the lowest literacy rate in India
(d) 900 litres
Ans. (d) : According to 1991 census data, Bihar has the
Ans. (a) : Water drawn out = Initial volume of the tank
lowest literacy rate in India i.e, 37.49% (63.82% in
– Final volume of the tank = [15 × 6 × 10 – 15 × 6 × (10
2011).
– 1)] = 15 × 6 × (10 – 9) = 90 m3 = 90000 litres
Directions: The next two items are based on the
146. An accurate clock shows 8 O'clock in the
following table. Study the same carefully and attempt morning. Through how many degrees will the
the two items that follow it: hour hand rotate when the clock shows 2
Indicators of development for some Asian countries O'clock in the afternoon?
Country Life Infant Adult (a) 150º (b) 144º
expectancy mortality literacy (c) 168º (d) 180º
at birth rate (Per rate Ans. (d) : Angle traced be the hour hand in 6 hours
(year) 1995 1000 live (Percent)
 360 
births) = ×6°
1995  12 
1996
= 180°
India 62.4 72 52
147. The monthly income of Komal and Asha are in
China 69.2 38 82 the ratio of 4 : 3. Their monthly expenses are in
the ratio of 3 : 2. However, both save Rs. 600/-
Indonesia 64.0 47 84
per month. What is their total monthly
Malaysia 71.4 11 84 income?
Thailand 69.5 31 94 (a) Rs. 8,400/- (b) Rs. 5,600/-
(c) Rs. 4,200/- (d) Rs. 2,800/-
Korea 71.7 6 98
Ans. (c) : Let monthly income of Komal and Asha be
Philippines 67.4 32 95
4x and 3x

IAS (Pre) GS 2000 Paper I 450 YCT


Also, let monthly expenses of Komal and Asha be 3y
and 2y.
Now, 4x – 3y =600 .....(i)
3x – 2y = 600 .....(ii)
Solving (i) and (ii), x = 600 and y = 600
∴ Total monthly income = (4 + 3)(600) = Rs.4200
148. If X = –2, then
X3 – X2 – X – 1 is equal to
(a) 1 (b) –3
(c) –11 (d) –15 According to the question all line segment are same
length.
Ans. (c) : x = – 2
GE + XA + BC + DY + FX = 10
= x3 – x2 – x – 1 = 0 So
GE = XA = BC = DY = FX = 2cm each
By putting x = – 2
Area of shaded position = (Area of ABCD + Area
(– 2)3 – (2)2 – (–2) – 1
OPQR)
=–8–4+2–1
+ (Area EXDY + Area JMNK) + Area GILH
= – 11
[2 × 2 + 2 × 2] + [6 × 2 + 6 × 2] + [2 × 10]
149. The Raga which is sung early in the morning is:
= 8 + 24 + 20 = 52 cm2
(a) Todi (b) Darbari
Hence area of shaded portion = 2 (Area of square) +
(c) Bhopali (d) Bhimpalasi
2 (Area of rectangle EFGH IJKL) + Area of rectangle
Ans. (a) : Raga Todi belongs to Todi Thaat. It is a ABCD = 2 (4 + 6 × 2) + 10 × 2 = 52 cm2.
morning Raga, and uses all seven nodes in the ascent
and descent Rishabh. Gandhar and Dhaivat are komal
(flat), Madhyam-is Tivra while Nishad is Shuddha. The
derivative ragas out of this structure are grouped under
the broud head of Todi Thaat.
150. In the given figure, all line segments of the
shaded portions are of the same length and at
right angles to each other. The same can be cut
out of a board of side 10 cm. What is the area
of the shaded portion?

(a) 36 cm2 (b) 48 cm2


(c) 52 cm2 (d) 64 cm2
Ans. (c) :

IAS (Pre) GS 2000 Paper I 451 YCT


UNION PUBLIC SERVICE COMMISSION
Civil Services (Preliminary Exam) - 1999
GENERAL STUDIES : PAPER-I
Time: 2 hours Maximum Number: 200
5. One consistent feature found in the history of
ANCIENT HISTORY southern India was the growth of small
1. From the third century AD when the Hun regional kingdoms rather than large empires
invasion ended the Roman Empire, the Indian because of :
merchants relied more and more on the: (a) The absence of minerals like iron
(a) African trade (b) Too many divisions in the social structure
(b) West-European trade (c) South-East Asian
(c) The absence of vast areas of fertile land
trade
(d) Middle-Eastern trade (d) The scarcity of manpower
Ans. (c) : After the 3rd century AD, Roman Empire Ans. (c) : One consistent feature found in the history of
came to an end. Indian merchants meanwhile had begun southern India was the growth of small regional
to rely more heavily on the south-east Asian Trade. kingdoms rather than large empires was due to the fact
2. The following persons came to India at one that in southern India, fertile lands were scattered and
time or another: the population was centered around those fertile land.
1. Fa-Hien 2. I-Tsing So due to the absence of large fertile land, small
3. Megasthenese 4. Hieum-Tsang
regional kingdoms were developed in southern India.
The correct chronological sequence of their
visits is: 6. The term 'Aryan' denotes:
(a) 3, 1, 2, 4 (b) 3, 1, 4, 2 (a) An ethnic group
(c) 1, 3, 2, 4 (d) 1, 3, 4, 2 (b) A nomadic people
Ans. (b) :Fa-Hien – 405-411 CE (c) A speech group
I-Tsing – 671-695 CE (d) A superior race
Megasthanese – 302-298 BC Ans. (c) : The term Aryan denotes a linguistic group
Hieun-Tsang – 630-645CE
and not a race. Their language known as Vedic Sanskrit
3. Which one of the following was intitially the
has definite relationship with major languages of
most powerful city state of India in the 6th
Europe and Asia. Scholars call this group of languages
century B.C.?
(a) Gandhar (b) Kamboj as Indo-European and the people speaking these
(c) Kashi (d) Magadh languages as Indo-Europeans or Indo-Aryans.
Ans. (d) : Of the 16 Mahajanapadas, Magadha, Kosala, 7. Which one of the following ports handled the
Vatsa and Avanti were more powerful. They fought north Indian trade during the Gupta period?
amongst themselves for years and ultimately Magadha (a) Tamralipti
emerged victorious under Bimbisara (Haranyak dynasty)
(b) Broach
during 6th Century BC.
4. The Indo-Greek Kingdom set up in north (c) Kalyan
Afghanistan in the beginning of the second (d) Cambray
century B.C. was: Ans. (a) : Tamralipta or Tamralipti was the name of an
(a) Bactria (b) Scythia ancient city on the Bay of Bengal corresponding with
(c) Zedrasia (d) Aria Tamluk in modern-day India. Tamralipta may had been
Ans. (a) : Bactria was the ancient civilizations of one of the most important urban centres of trade and
Iranian peoples. Ancient Bactria was located between
commerce of early historic India, trading along the Silk
the Hindu Kush mountain range and the Amu Darya
river, covering the flat region that straddles modern-day Road with China. Broach was the main port of western
Afghanistan and Tajikistan. India where sea trade done with western countries.

IAS (Pre) GS 1999 Paper I 452 YCT


MEDIEVAL HISTORY 10. Assertion (A) : During the time of Akbar, for
every ten Cavalrymen, the
8. The first writer to use Urdu as the medium of mansabdars had to maintain
poetic expression was: twenty horses.
(a) Amir Khusrau (b) Mirza Ghalib Reason (R) : Horses had to be rested while
(c) Bahadur Shah Zafar (d) Faiz on March and replacements
Ans. (a) : Amir Khusrau, was a Persian musician, poet, were necessary in times of
and scholar associated with the rulers of Delhi war.
Sultanate. Besides being the first writer to use Urdu as a Ans. (a) : During the time of Akbar, for every ten
medium of poetic expression, he also wrote poetry cavalrymen, the mansabdars had to maintain twenty
primarily in Persian, Hindi, and Urdu languages. horses because Horses had to be rested while on march
9. To which Lodi Sultan does the given map and replacements were necessary in times of war, so
related and what town does the site marked A both A and R are true and R is the correct explanation
on the map represent? of A.
11. 'The king was freed from his people and they
from their king'. On whose death did Badauni
comment thus?
(a) Balban
(b) Ala-ud-din Khaliji
(c) Muhammad-bin-Tughlaq
(d) Firoz Shah-Tughlaq
Ans. (c) : Muhammad Bin Tughlaq died in 1351 CE.
On his death, Badauni commented this. Tughlaq was
succeeded by his cousin Firoz Tughlaq.
12. Which one of the following pairs is not
(a) Behlol Lodi, Jaunpur correctly matched?
(b) Sikandar Lodi, Aligrah (a) Jahangir : William Hawkins
(c) Ibrahim Lodi, Jaunpur (b) Akbar : Sri Thomas Roe
(d) Ibrahim Lodi, Aligarh (c) Shahjahan : Tavernier
Ans. (a) : Bahlol Lodi defeated Hussain Shah Sharqi of (d) Aurangzeb : Manucci
Jaunpur in 1479. Bahlul Khan Lodi was the chief of the Ans. (b) : Sir Thomas Roe visited the court of Jahangir
Pashtun Lodi tribe. Bahlol Lodi (1451-1489) was the to initiate trade with India. He was an ambassador of
founder of the Lodi dynasty. Bahlul became sultan of James – I, king of England. Tavernier’s account covers
the dynasty on 19 April 1451. the reign of Shahjahan and Aurangzeb.
Directions: The following 10 items consist of two
MODERN HISTORY
statements. One labelled as 'Assertion A' and the
other labelled as 'Reason R'. You are to examine 13. Which Indian nationalist leader looked upon a
these two statements carefully and decide if the war between Germany and Britain as a greatest
opportunity which would enable Indian to
Assertion A and the reason R are individually true
exploit the situation to their advantage?
and if so, whether the Reason is a correct explanation
(a) C. Rajagopalachari
of the Assertion. Select your answer to these items
(b) M. A. Jinnah
using the codes given below and mark your answers
(c) Subhash Chandra Bose
sheet accordingly.
(d) Jawaharlal Nehru
Codes: Ans. (c) : S.C. Bose and the socialists argued that the
(a) Both A and R are True and R is the correct war was an imperialist one since both sides were
explanation of A fighting for gaining or defending colonial territories.
(b) Both A and R are True but R is not a correct Therefore the question of supporting either of the two
explanation of A sides did not arise. Instead advantage should be taken of
(c) A is true but R is False the situation to attain freedom by immediately starting a
(d) A is false but R is True Civil Disobedience Movement.
IAS (Pre) GS 1999 Paper I 453 YCT
14. ‘Abhinava Bharat’ a secret society of 17. There was no independent development of
revolutionaries was organised by: industries in India during British rule because
of the:
(a) Khudiram Bose
(a) Absence of heavy industries
(b) V.D. Savarkar
(b) Scarcity of foreign capital
(c) Prafulla Chaki (c) Scarcity of natural resource
(d) Bhagat Singh (d) Preference of the rich to invest in land.
Ans. (b) : ‘Abhinava Bharat’ a secret society of Ans. (a) : There was no independent development of
revolutionaries was organised in 1904 by VD Savarkar. industries in India during British rule because during that
15. The paintings of Abanindranath Tagore are period Britishers used India as market and used to export
classified as: raw material from India to England to manufacture. There
were no heavy industries available in India during that
(a) Realistic (b) Socialistic
period.
(c) Revivalistic (d) Impressionistic
18. "It made its proposals in May. It still wanted a
Ans. (c) : Abanindranath Tagore (brother of Ravindranath united India. There was to be a Federal Union
Tagore) is known as the leader of the Revivalist composed of British provinces."
Movement in the field of modern Indian painting in The above quotation is related to–
Bengal. Abanindranath Tagore’s works reflect the (a) Simon Commission
synthesis of Ajanta murals and Mughal painting. (b) Gandhi-Irwin Pact
Essentially a romantic painter, he delighted in painting the (c) Cripps Mission
hoary past. He along with his disciples viz. Nandalal Bose
(d) Cabinet Mission
and Ashit Kumar Haldar are said to be the Pre-Raphaelites
Ans. (d) : Cabinet Mission reached Delhi on March 24,
of Bengal.
1946. It put forward its own proposals in May 1946. The
16. Match List-I with List-II and select the correct main objective of Cabinet Mission was to find out ways
answer using the codes given below the lists: and means for the peaceful transfer of power in India, to
List-I List-II suggest measures for the formation of a Constitution
making machinery and also to set up the Interim
A. Pandit Vishnu 1. Introduced the
Government 1st provided a plan to a avoid the partition of
Digamber Paluskar scheme of Raga India and to support a federal union comprising of British
classification of provinces.
Indian music 19. The Governor-General who followed a spirited
B. Venkatamahi 2. Proponent of "Forward" policy towards Afghanistan was:
(a) Minto (b) Dufferin
Carnatic music
(c) Elgin (d) Lytton
C. Shyama Shastri 3. Proponent of the Ans. (d) : The three Governor Generals have
Khayal form of contributed mainly to the relationship between the
Hindustani music company and Afghanistan. Lord Auckland (1836-42)
D. Amir Khusrau 4. Wrote the music for was the one who initiated the forward policy against
the song ‘Vande Afghanistan. Sir John Lawrence (1864-68) adopted a
Mataram’ policy of skilful inaction towards Afghanistan. Lord
Lytton (1876-80) again implemented the forward policy
Codes:
towards Afghanistan.
A B C D
(a) 4 1 3 2 20. At a time when empires in Europe were
crumbling before the might of Napoleon, which
(b) 4 1 2 3
one of the following Governors-General kept
(c) 1 4 3 2
the British flag flying high in India?
(d) 1 4 2 3 (a) Warren Hastings (b) Lord Cornwallis
Ans. (b) : Pandit Vishnu Digamba0r Paluskar (c) Lord Wellesley (d) Lord Hastings
introduced the scheme of Raga classification of Indian
Ans. (c) : Lord Wellesley came to India in 1798 as
music, He sang the original version of the Bhajan "
Raghupati Raghav Raja Ram" and founded the Governor General. At a time when the British were
Gandharva Mahavidyalaya on 5 May, 1901. fighting with Frome all over the world. He helped in
Venkatamahi wrote the music for the song ‘Vande bringing as many Indian state as possible under British
Matram’, Shyama Shastri was a proponent of Karnatic control, through his policies of subsidiary alliance, wars
music and Amir Khusrau was the proponent of Khyal and annexations of the territories of previously
form of Hindustani music. subordinated rulers.
IAS (Pre) GS 1999 Paper I 454 YCT
21. Which of the following leaders of the Congress Codes:
was totally in favour of Cabinet Mission Plan? A B C D
(a) Mahatma Gandhi
(b) Jawaharlal Nehru (a) 4 3 2 1
(c) Sardar Patel (b) 3 4 1 2
(d) Maulana Abul Kalam Azad (c) 4 3 1 2
Ans. (c) : Patelji was completely in favour of the (d) 3 4 2 1
Cabinet Mission Plan, 1946. In the context of this plan,
Ans. (a) : (Books) (Authors)
Patelji had said - "This plan is the most excellent plan in
the context of the circumstances of the time, it contains A. The First Indian war Vinayak Damodar Savarkar
such seeds, from which the land of India can be freed B. Anand Math Bankim Chandra Chatterji
from suffering." C. Life Divine Sri Aurobindo
22. Which one of the following statements is not D. Sadhana Rabindra Nath Tagore
correct? 25. Consider the following events:
(a) 'Neel Darpan' was a play based on the
exploitation of the indigo farmers. 1. Indigo Revolt 2. Santhal Rebellion
(b) The author of the play 'Ghashiram Kotwal is 3. Deccan Riot 4. Mutiny of the Sepoys
Vijay Tendulkar The correct chronological sequence of these
(c) The play ‘Nabanna’ by Nabin Chandra Das events is:
was based on the famine of Bengal
(a) 4, 2, 1, 3 (b) 4, 2, 3, 1
(d) Urdu theatre used to depend heavily on Parsi
theatre (c) 2, 4, 3, 1 (d) 2, 4, 1, 3
Ans. (c) : Ghashiram Kotwal is a Marathi play written Ans. (d) : The correct sequence of the movements given
by Vijay Tendulkar in 1972. Neel Darpan, the Mirror of in the question is as follows: Santhal Rebellion - 1856,
Indigo; translated as Neel Darpan; or, the Indigo Sepoy Mutiny - 1857, Neel Rebellion 1859, Deccan
Planting Mirror is a Bengali play written by Riots - 1875.
Dinabandhu Mitra in 1858-1859. The play was essential 26. Match List-I with List-II and select the correct
to Neel bidroha, or Indigo Revolt. The play Nabanna answer using the codes given below the lists:
about the famine of Bengal was written by Bijon
Bhattacharya. Hence third statement is not correct. Urdu List-I List-II
theatre used to depend heavily on Parsi theatre for basic (Year) (Event)
infrastructure. A. 1775 1. First Anglo-Burmese war
23. Which one of the following Indian leaders was B. 1780 2. First Anglo-Afghan war
dismissed by the British from the Indian Civil
C. 1824 3. First Anglo-Maratha War
Service?
(a) Satyendranath Tagore D. 1838 4. Second Anglo-Mysore War
(b) Surendranath Banerji Codes:
(c) R.C. Dutt A B C D
(d) Subhash Chandra Bose (a) 4 3 2 1
Ans. (b) : Surendranath Banerji cleared the Indian civil
(b) 4 3 1 2
services examination in 1869, but was barred owing to a
dispute over his exact age. After clearing the matter in (c) 3 4 1 2
the courts, Banerjee cleared the exam again in 1871 and (d) 3 4 2 1
was posted as assistant magistrate in Sylhet. However, Ans. (c) : The First Anglo-Burmese War, also known as
Banerjee was dismissed soon from his job owing to the First Burma War, (5 March 1824 – 24 February
racial discrimination. Satyendranath Tagore in 1863 1826) was the first of three wars fought between the
became the first Indian to qualify the ICS. British and Burmese empires in the 19th century. The
24. Match List-I with List-II and select the correct First Anglo-Afghan War (also known by the British as
answer using the codes given below the lists: the Disaster in Afghanistan) was fought between the
List-I List-II British East India Company and the Emirate of
(Books) (Authors) Afghanistan from 1839 to 1842.The First Anglo-
A. The First Indian war 1. Rabindranath Maratha War (1775–1782) was the first of three Anglo-
of Independence Tagore Maratha wars fought between the British East India
B. Anand Math 2. Sri Aurobindo Company and Maratha Empire in India. The war began
C. Life Divine 3. Bankim Chandra with the Treaty of Surat and ended with the Treaty of
Chatterji Salbai. The Second Anglo–Mysore War was a conflict
D. Sadhana 4. Vinayak Damodar between the Kingdom of Mysore and the British East
Savarkar India Company from 1780 to 1784.
IAS (Pre) GS 1999 Paper I 455 YCT
27. Assertion (A) : Lord Linlithgow described 30. Which one of the following statements is
the August Movement of correct?
(a) Kacchative and Tin Bigha were territories
1942 as the most serious
acquired by the Indian Republic from the
rebellion since Sepoy Mutiny. French
Reason (R) : There was massive upsurge (b) Kacchativu and Tin Bigha are territories
of the peasantry in certain handed over to Sri Lankan and Bangladeshi
areas. sovereignty respectively by the Government
of India
(a) Both A and R are True and R is the correct (c) Kacchativu and Tin Bigha are areas that were
explanation of A annexed by the Chinese in the 1962 Sino-
(b) Both A and R are True but R is not a correct Indian War
explanation of A (d) Kachativu and Tin Bigha are englaves which
were transferred to India by lease
(c) A is true but R is False arrangements with Sri Lanka and Pakistan
(d) A is false but R is True respectively
Ans. (a) : The biggest movement after the Sepoy Ans. (b) : A territorial dispute arose in regard to the
Mutiny of 1857, the Quit India Movement of 1942 was ownership of a one square mile uninhabited island,
described by Lord Linlithgow as a serious rebellion. called Kacchativu, off the Jaffna coast in the Palk
Apart from the general public and farmers, a large straits. Pilgrims from both India and Sri Lanka used to
number of women also participated in the movement. go to Kacchativu Island every year in the month of
This movement was the last great battle of the Indian March during the four-day St. Anthony’s festival for
worship at the local Roman Catholic Church. India
freedom struggle. This movement is also known as
protested over the presence of Sri Lankan police during
"August Revolution". This movement placed the the festival in 1968. This caused conflict. Both India
demand for independence on the immediate agenda of and Sri Lanka were keen to avoid a serious situation.
national movement. after Quit India movement there The Prime Ministers of India and Sri Lanka met twice
could be no going back. In this struggle, the common and reached to a final decision on the issue of island’s
people displayed unparallel heroism. That's why lard title, resolved to maintain status quo in and around the
linlithgow (1934-44) described the Quit India island. Neither India nor Sri Lanka would send
movement as the most serious revolt after the Sepoy its policemen in uniform or custom officials, or resort to
mutiny. aerial reconnaissance or naval patrolling of adjacent
28. The First Venture of Gandhi in all-India politics waters during the St. Anthony’s festival. Finally,
was the: through a comprehensive agreement India accepted Sri
Lanka’s ownership of the Kacchativu Island. The Teen
(a) Non-Cooperation Movement
Bigha Corridor is a strip of land formerly belonging to
(b) Rowlatt Satyagraha India on the West Bengal–Bangladesh border which has
(c) Champaran Movement been leased indefinitely to Bangladesh so that it can
(d) Dandi March access its Dehgram– Angalpota enclaves, one of the
Indo-Bangladesh enclaves. This Corridor was formally
Ans. (b) : Rowlatt Satyagraha was the first venture of transferred to Bangladesh on June 26, 1992.
Gandhi on all India level. Satyagraha was to be 31. Match List-I with List-II and select the correct
launched on April 6, 1919. But after the Jallianwala answer using the codes given below the lists:
Bagh massacre, Gandhi ji was alarmed by the List-I List-II
atmosphere of violence and with drew the movement on
A. Shyamji Krishna 1. Bande Matram
April 18,1919. Rowlatt Act permitted few political
B. Madam Bhikaji Cama 2. Indian Sociologist
cases to be tried without Juries and allowed internment
C. Annie Besant 3. The Talwar
of suspects without trial.
D. Aurobindo Ghosh 4. Common weal
29. The Congress policy of pray and petition Codes:
ultimately came to an end under the guidance A B C D
of: (a) 2 3 4 1
(a) Aurobindo Ghosh (b) 3 2 1 4
(b) Bal Gangadhar Tilak (c) 2 3 1 4
(c) Lala Lajpat Rai (d) 3 2 4 1
(d) Mahatma Gandhi Ans. (a) : Shyamji Krishna Verma was associated to the
Ans. (b) : Bal Gangadhar Tilak described the method of Journal 'Indian Sociologist". Madam Bhikaji Cama was
three P’s – Prayer, petition and protest as political related to the 'Talwar Journal'. Annie Besant published
mendicancy. He propagated militancy and not the journal 'Commonweal'. Aurobindo Ghosh associated
mendicancy. with 'Vande Matram'.
IAS (Pre) GS 1999 Paper I 456 YCT
INDIAN GEOGRAPHY Ans. (a) : Trans chart is the chartering wing of the
Minister of Surface Transport. This wing also looks
32. Match List-I with List-II and select the correct after trading through ships by private sectors.
answer using the codes given below the lists: 35. Which one of the following statements is not
List-I List-II true or correct about the Konkan Railway?
A. Cotton 1. Rainfall 1000-1500 mm; (a) The total length is about 760 km
(b) It runs through the states of Karnataka, Goa,
Temperature 40°-60°C
Maharashtra and Kerala
B. Flax 2. Rainfall 1500-2000 mm; (c) It is the only rail route that cuts across the
Temperature 25°-35° C Western Ghats
C. Sugar beet 3. Rainfall 600-800 mm; (d) The Konkan Railway Construction Company
Temperature 5°-18° C which came into being raised money through
D. Jute 4. Rainfall 500-1000 mm; Public Issues.
Ans. (d) : Konkan Railway (741 km line) connects
Temperature 18°-22° C
Indian’s commercial capital Mumbai and Mangalore
5. Rainfall 500-600 mm; (Karnataka). Hence first statement is correct. The first
Temperature 18°-22° C train on the completed track of Konkan Railway was
Codes: flagged off on January 26, 1998, Republic Day. Konkan
A B C D Railway connects the Konkan region of Maharashtra to
Kerala via Goa and Karnataka. Hence second statement
(a) 1 3 4 2
is also correct. Konkan Railway is a vital link between
(b) 2 3 5 4 Mumbai and Mangalore, traversing through picturesque
(c) 4 5 2 1 Western Ghats with more than 90 tunnels, about 2000
(d) 4 3 5 2 bridges and 564 deep cuttings. Hence third statement is
Ans. (a) : The correct match is as follows– also correct. Konkan Railway is a joint venture between
the Central government and the State governments of
Crop Climatic condition
Maharashtra, Karnataka, Kerala and Goa with the
A. Cotton : Rainfall 1000 mm to 1500 government of India taking a 51 per cent share, while
mm : Temperature 40° to 60° the rest went to Maharashtra (22 per cent), Karnataka
B. Flax : Rainfall 600-800 mm; Temp (15 per cent), Kerala (6 per cent) and Goa (6 per cent).
5°C-18°C Hence fourth statement is not correct.
C. Sugar beet : Rainfall 500-600 mm; Temp 36. The given figure shows a portion of Southern
18°C-22°C India. The proposed site (Koodankulam) for
D. Jute : Rainfall 1500-2000 mm; the construction of two 1000 MW nuclear
Temp. 25°C-35°C power plants has been labelled in the map as:
33. Which one of the following agriculture
practices is eco-friendly?
(a) Organic farming
(b) Shifting cultivation
(c) Cultivation of high yielding varieties
(d) Growing plants in glass-houses.
Ans. (a) : Organic farming is a production system of
crops which avoids the use of synthetic and chemical
inputs like fertilizers, pesticides, growth regulators and
livestock feed additives. Therefore organic farming
does not harm environment and hence it is eco-friendly.
34. Trans chart is the:
(a) Chartering wing of the Minister of Surface
Transport
(b) Container service established by Indian (a) A (b) B
Railways (c) C (d) D
(c) Training institute of maritime studies and Ans. (b) : Koodankulam is the site for 1000 MW
research nuclear power plant under construction in Tirunelveli
(d) Passenger insurance scheme of India district of Tamil Nadu. In the given figure
Railways Koodankulam plant is labelled as ‘B’.
IAS (Pre) GS 1999 Paper I 457 YCT
37. Which one of the following types of coal Ans. (c) : In the given map the site of Tehri Dam is
contains a higher percentage of carbon than labeled as “C” which lies in Uttaranchal. The Tehri
the rest? Dam is the highest dam in India and one among the
(a) Bituminous coal (b) Lignite highest in the world. It is a multi-purpose rock and
earth-fill embankment dam on the Bhagirathi River near
(c) Peat (d) Anthracite
Tehri in Uttarakhand. It is the primary dam of the
Ans. (d) : Anthracite, often referred to as hard coal, is a THDC India Ltd. and the Tehri hydroelectric complex.
hard, compact variety of coal that has a submetallic Phase 1 was completed in 2006.
luster. It has the highest carbon content, fewest
40. In which one of the following areas in the given
impurities and the highest energy density of all types of
coal and is the highest ranking of coals. map was there a recent discovery of copper
deposits by the Atomic Minerals Division of
38. Match List-I with List-II and select the correct
Department of Atomic Energy?
answer using the codes given below the lists:
List-I List-II
(Resorts) (States)
A. Chakrata 1. Assam
B. Haflong 2. West Bengal
C. Kalimpong 3. Uttarakhand
D. Kufri 4. Himachal Pradesh
Codes:
A B C D
(a) 1 3 2 4
(b) 3 1 4 2
(c) 3 1 2 4
(d) 1 3 4 2
Ans. (c) : Chakrata holiday resort is situated in
Uttarakhand. Haflong resort is situated in Assam,
Kalimpong resort is situated in West Bengal and Kufri
resort is situated in Himachal Pradesh. (a) 1 (b) 2
(c) 3 (d) 4
39. In the given figure, the site of the Tehri dam
has been labelled as: Ans. (d) : In the given map, copper deposits discovered
by Atomic Minerals Division of Department Atomic
energy, is at Hassan in Karnataka and is marked as ‘4’.
41. Match List-I with List-II and select the correct
answer using the codes given below the lists:
List-I List-II
(Industries) (Industrial Centres)
A. Pearl fishing 1. Pune
B. Automobiles 2. Tuticorin
C. Ship building 3. Pinjore
D. Engineering goods 4. Marmagao
Codes:
A B C D
(a) 2 1 4 3
(b) 2 1 3 4
(c) 1 2 4 3
(d) 1 2 3 4
Ans. (a) : Industries Industrial centres
A. Pearl fishing- Tuticorin
B. Automobile industry- Pune
(a) A (b) B C. Ship building -Marmagao
(c) C (d) D D. Engineering goods- Pinjore.
IAS (Pre) GS 1999 Paper I 458 YCT
42. The population growth rate in Kerala is the Ans. (c) : ‘A’ marked area in the given map is Punjab
lowest among major Indian States. Which one and Haryana where Sikhs are the largest religious
of the following is the most widely accepted minorities. ‘B’ marked area denoted part of Rajasthan
reason for this? and Gujarat where Jains are the largest religious
minority. ‘C’ marked area denoted Sanchi area of
(a) Kerala has made the highest investment in
Madhya Pradesh where Buddhist are the religious
family planning.
minority. ‘D’ refers the part of Andhra Pradesh where
(b) Kerala has the highest literacy rate in India. Christians are religious minority.
(c) Kerala has invested heavily in promoting 44. Which one of the following pairs of states and
literacy and public health and placed high tribes is not correctly matched?
priority on social policies.
(a) Assam : Miri
(d) The population pyramid in Kerala has (b) Nagaland : Konyak
relatively fewer women in the reproductive
(c) Arunachal Pradesh : Apatani
age group.
(d) Madhya Pradesh : Lambada
Ans. (b) : Literacy and population growth are inter-
Ans. (d) : Madhya Pradesh tribes and tribal culture
related. Literate people understand the disadvantage of
information. Information about Central India Tribes like
high population. Fewer resources are needed for a small
Gonds, Baiga, Bhil, Korku, Abujhmar, Bharia, Sahariya
family and they could be better taken care of. As Kerala
are tribes of Madhya Pradesh.
has highest literacy rate in India, it reflects on its
population also. Lambada tribe belongs to Telangana. Rest is correctly
matched.
43. Match the areas shown as A, B, C and D on the
45. As per 1991 Census, which one of the following
given map showing with the largest religious
minorities. Select the correct answer using the groups of Union Territories had the highest
literacy rate?
codes given below the list of minorities.
(a) Chandigarh and Dadra & Nagar Haveli
(b) Delhi and Andaman & Nicobar Islands
(c) Andaman & Nicobar Islands and Pondicherry
(d) Pondicherry and Delhi
Ans. (d) : As per 1991 census, Pondicherry and Delhi
had the literacy rates of 75.3 and 74.7 respectively.
46. In the rough outline map of a part of Jammu
and Kashmir shown in the figure, places
marked A, B, C and D represent respectively:

Largest Religious Minorities–


1. Buddhists 2. Christians
3. Jains 4. Muslims
5. Sikhs
Codes:
A B C D
(a) 5 1 3 2
(b) 4 3 2 1
(c) 5 3 1 2
(d) 4 2 1 3
IAS (Pre) GS 1999 Paper I 459 YCT
(a) Anantnag, Baramula, Srinagar and Kargil 49. If it is 10.00 a.m. I.S.T., then what would be the
(b) Baramula, Srinagar, Kargil and Anatnag local time at Shillong on 90° E longitude?
(c) Baramula, Srinagar, Anantnag and Kargil (a) 9.38 a.m. (b) 10.38 a.m.
(d) Srinagar, Baramula, Kargil and Anantnag (c) 10.22 a.m. (d) 9.22 a.m.
Ans. (c) :In map the places A , B , C and D are Ans. (b) : Standard time meridian passes through 82.5
representing Baramulla, Srinagar, Anantnag and Kargil degree. Difference between two given longitudes = 9.5
respectively. The order of these districts in terms of area degrees. As we know 1 degree= 4 minutes, therefore 9.5
is as follows - Kargil (14036 km.), Baramulla (4588 degrees = 4×9.5=38 minutes. Thus, time is 10+38
km.), Anantnag (3984 km.) and Srinagar (2228 km.) minutes= 10:38 AM.
(Source - Oxford Atlas) 50. The Ravva offshore block, with great potential
47. Match List-I with List-II and select the correct for oil, is located in:
answer using the codes given below the lists: (a) Krishna-Godavari basin
List-I List-II (b) Cauvery basin
(Rivers) (Dams) (c) Mahanadi basin
A. Cauvery 1. Alamatti (d) Palar-pennar basin
B. Krishna 2. Mettur Ans. (a) : The Ravva off shore block, with great
C. Narmada 3. Gandhi Sagar potential for oil, is located in the Krishna-Godavari
D. Chambal 4. Sardar Sarovar basin in Andhra Pradesh.
Codes:
WORLD GEOGRAPHY
A B C D
(a) 1 4 2 3 51. Which one of the following port cities in
(b) 2 1 4 3 Venezuela has been developed as an oil port?
(c) 2 1 3 4 (a) Caracas (b) Maracaibo
(d) 1 3 4 2 (c) Maracay (d) Capuano
Ans. (b) :The Mettur Dam is one of the largest dams in Ans. (b) : Maracaibo port is an oil port situated on the
India and the largest in Tamil Nadu, located across the east side of Lake Maracaibo in Venezuela opposite the
river Cauvery where it enters the plains. The Lal city of Maracaibo and is operated by the Venezuelan
Bahadur Shastri Dam, also known as Almatti Dam, is a State Oil Company (PDVSA PETROLEO, S.A). It is
hydroelectric project on the Krishna River in North the largest crude oil export port in South America.
Karnataka. The Sardar Sarovar Dam is a gravity dam on 52. Match the cities labelled as A, B, C and D in
the Narmada river near Navagam, Gujarat. The Gandhi the given map with the names of cities and
Sagar Dam is one of the four major dams built select the correct answer using the codes given
on India's Chambal River. The dam is located in below the names of cities:
the Mandsaur, Neemuch districts of Madhya Pradesh.
48. Assertion (A) : According to statistics, more
female children are born
each year than male children
in India.
Reason (R) : In India, the death rate of a
male child is higher than that
of the female child.
(a) Both A and R are True and R is the correct Name of Cities–
explanation of A 1. Darwin 2. Kuala Lumpur
(b) Both A and R are True but R is not a correct 3. Lagos 4. Nairobi
explanation of A 5. Singapore
Codes:
(c) A is true but R is False
A B C D
(d) A is false but R is True (a) 1 2 4 3
Ans. (c) : Assertion is correct, but the reason is wrong. (b) 2 1 4 3
In India, the sex ratio of female is lower than male, (c) 1 4 5 2
because female feticides is common across India. (d) 4 3 5 2
IAS (Pre) GS 1999 Paper I 460 YCT
Ans. (a) ‘A’ marked city in map is Darwin in Australia. 55. Consider the following statements about the
‘B’ marked city in the given map is Kualalumpur in European Union:
Malayasia. 1. The European Union was known earlier as
‘C’ marked city in map is Nairobi in Kenya. the European Community.
The city marked ‘D’ in the map is Lagos in Nigeria. 2. The Single European Act (1986) and the
53. A person of mixed European and Indian blood Maastricht Treaty were milestones in its
in Latin America is called a: formation.
(a) Mulatto (b) Mestizo 3. Citizens of European Union countries enjoy
(c) Meiji (d) Mau-Mau dual citizenship.
Ans. (b) : A person of mixed European and Indian 4. Switzerland is a member of the European
blood in Latin America is called a Mestizo. The term Union
mestizo means mixed in Spanish, and is generally used Which of the above statements are correct?
throughout Latin America to describe people of mixed (a) 2 and 4 (b) 1 and 3
ancestry with a white European and an indigenous
(c) 3 and 4 (d) 1, 2 and 3
people.
Ans. (d) : Switzerland is not a member of the European
54 The physical regions marked as 1, 2, 3 and 4 on
Union. Apart from statement 4, all the other three
the given map are respectively: statements are true.
In 1986, the Single European Act is signed. This was a
treaty which provided the basis for a vast six-year
program aimed at sorting out the problems with the free
flow of trade across Europian borders and thus creates
the ‘Single Market’. ‘Maastricht’ Treaty on European
Union in 1993 and the Treaty of Amsterdam in 1999
were signed to protect environment. EU citizenship was
established in 1991 by the Treaty on European Union in
order to promote European values and identity. The
Treaty confers on EU citizens a set of rights, such as the
right of free movement, the right of diplomatic
protection, the right to vote in and stand for elections to
the European Parliament. Some of these rights can be
exercised only when moving from one Member State to
another. EU citizenship depends strictly on national
citizenship, since EU citizens are only those who
(a) Andes, Brazilian Shield, Guyana Highlands already hold the citizenship of an EU Member State.
and Amazon Basin Member States reserve the right to regulate the
acquisition and loss of national citizenship in ways that
(b) Andes, Guyana Highlands, Brazilian Shield
reflect their interests and identities. Currently, there are
and Amazon Basin
27 member countries in European Union. Switzerland is
(c) Amazon Basin, Guyana Highlands, Brazilian not a member of EU. Hence statement 4 is not correct.
shield and Andes
56. The language spoken by the largest number of
(d) Guyana Highlands, Brazilian Shield Andes
people in the world is:
and Amazon Basin
(a) Hindi (b) English
Ans. (a) : In the given figure: Andes is marked as ‘1’,
which is located at the western coast of South America. (c) Mandarin (d) Spanish
Brazilian shield is marked as ‘2’; this area is located at Ans. (c) :Ethnologue (2019, 22nd edition)
the eastern part of South America. The shield extends Rank Language Percentage of world
over 8.5 million square area. Guyana highland is pop. (March 2019)
marked as ‘3’; Guyana highland is a plateau and low 1. Mandarin Chinese 11.922%
mountain region of South America located north of
Amazon river and South of Orinoco river. Amazon 2. Spanish 5.994%
basin is marked as ‘4’; it is the largest tropical rain 3. English 4.922%
forest in the world. Amazon River drains more than half 4. Hindi (sanskritised 4.429%
of Brazil, part of Bolivia, Peru and Ecuador. Hindustani)
IAS (Pre) GS 1999 Paper I 461 YCT
57. Assertion (A) : Chile continues to be an Ans. (d) : O Schmidt suggests the earth's origin is from
important producer of gases and dust particles. O. Schmidt in 1943 gave Inter-
copper in the world. Stellar Dust Hypothesis for the origin of the earth and
solar system.
Reason (R) : Chile is endowed with the
60. Match List-I with List-II and select the correct
world's largest deposit of
answer using the codes given below the lists:
porphyry copper. List-I List-II
(a) Both A and R are True and R is the correct (Volcanic Mountain) (Country)
explanation of A A. Mt. Rainier 1. Italy
(b) Both A and R are True but R is not a correct B. Etna 2. Mexico
explanation of A C. Paricutin 3. Philippines
(c) A is true but R is False D. Taal 4. U.S.A.
(d) A is false but R is True Codes:
Ans. (a) : Chile is an important producer of copper in A B C D
the world and it is endowed with the world's largest (a) 4 2 1 3
deposit of porphyry copper. (b) 4 1 2 3
58. A ship from the eastern extremity of the (c) 2 1 4 3
Aleutian Islands to Dutch harbour crosses 180° (d) 4 3 2 1
meridian at 23.30 hrs. On January 1, 1999. Ans. (b) : Mount Rainier, also known as Tahoma or
Tacoma, is a large active stratovolcano located 59 miles
what time and date will be recorded by the
south-southeast of Seattle, USA in Mount Rainier
captain of the ship in his diary after one-hour
National Park. Mount Etna, or Etna, is an active
journey from the point of crossing of the stratovolcano on the east coast of Sicily, Italy, in the
meridian? Metropolitan City of Catania, between the cities of
(a) January 1, 0030 hrs Messina and Catania. It lies above the convergent plate
(b) January 2, 0030 hrs margin between the African Plate and the Eurasian
Plate. Parícutin is a cinder cone volcano located in the
(c) January 3, 0030 hrs
Mexican state of Michoacán, near the city of Uruapan
(d) January 4, 0030 hrs and about 322 kilometers west of Mexico City. The
Ans. (b) : The eastern end of the Aleutian Islands is volcano surged suddenly from the cornfield of local
located west on the 180º of longitude (172º E. farmer Dionisio Pulido in 1943, attracting both popular
Longitude) while Dutch Harbor is located to the east of and scientific attention. Taal Volcano is a complex
the 180º of longitude (166º 31' 55" W. longitude) of the volcano located on the island of Luzon in the
ship's 180º longitude. On moving from west to east of Philippines. It is the second most active volcano in the
the line, the date will decrease by 1 day i.e. on crossing Philippines with 33 historical eruptions. All of these
180º meridian on January 1, 1999 at 23 : 30, the day eruptions are concentrated on Volcano Island, an island
will become December 31,1998 according to the rules near the middle of Taal Lake.
and after 1 hour (23 : 30 + 60 : 00 + 24 : 00 + 30) The 61. In the given map, which one of the following
captain of the ship will enter the day at 00.30 as January pairs of ocean currents is shown?
1, 1999. However, in reality the International Date Line
does not exactly follow 180ºof longitude and is
sometimes referred to as 180º of longitude in order to
keep the politically and economically relevant regions
together. Accordingly, the entire Alaska Territory
(Which includes both The Aleutian Islands and Dutch
Harbor) is placed east of the International date Line. In
this situation, the International date Line will not be
crossed by ship at all and even after crossing 180º
meridian, the day will remain the same (January 1,
1999) and an hours there after would be recorded by the
ship's captain at January 2, 00: 30 hrs.
59. Which one of the following scholars suggests the
earth's origin from gases and dust particles?
(a) James Jeans (b) H. Alfven
(c) F. Hoyle (d) O. Schmidt
IAS (Pre) GS 1999 Paper I 462 YCT
(a) Benguela and Falkland Which of the above statements are correct?
(b) Canary and Humboldt (a) 1, 2 and 4 (b) 3 and 4
(c) 2 and 3 (d) 1, 3 and 4
(c) Agulhas and Guinea
Ans. (b) : From the graph, it can be seen that yield rate
(d) Benguela and Guinea is not zero at B and C. With no fertilizer input, yield is
Ans. (d) : Benguela Current, oceanic current is a branch 20. Yield is clearly minimum at D, i.e., 8. At C, yield is
of the West Wind Drift of the Southern Hemisphere. It 20 and it is neither maximum nor minimum. So clearly
flows northward in the South Atlantic Ocean along the option 3 and 4 are correct.
west coast of southern Africa nearly to the Equator 64. Which one of the areas marked as A, B, C and D
before merging with the westward-flowing Atlantic in the given figure of the cyclone, witnesses heavy
South Equatorial Current. Benguela is a cold current. torrential sort-duration rainfall accompanied by
Guinea Current, surface oceanic current of the Atlantic thunderstorms?
Ocean, the eastward continuation of the Atlantic Equatorial
Countercurrent, off the western coast of Africa near the
Gulf of Guinea. The warm, highly saline Guinea Current
reaches a depth of less than 660 feet (200 m). Guinea is a
warm current flowing east of Guinea.
62. At which one of the following positions shown
in the diagram will the height of the ocean tide
be maximum?

(a) A (b) B
(c) C (d) D
Ans. (b) : In the given figure ‘B’ lies between two
cyclone zone ‘A’ and ‘D’. At the ‘B’ marked region
rainfall would be accompanied by thunder storms.
(a) M1 (b) M2 65. Consider the following temperature an rainfall
(c) M3 (d) M4 data:
Ans. (d) : At ‘M4’ position of the diagram the height of Month Temperature °C Rainfall cm
the tide is at maximum, because in this position earth,
moon and sun lies in a straight line. January 6.7 14.0
63. The yield versus fertilizer input is shown in the
February 6.7 13.2
graph.
March 7.2 11.4

April 8.9 9.4

May 11.1 8.1

June 13.9 8.1

July 15.0 9.6

August 15.0 12.2


Consider the following statements based on this
graph: September 13.9 10.4
1. Yield rate is zero at B and C. October 11.1 14.4
2. There is no Yield with no fertilizer input.
3. The yield is minimum at D. November 8.9 14.0
4. The yield is neither minimum nor
December 7.8 16.8
maximum at C.
IAS (Pre) GS 1999 Paper I 463 YCT
The climate to which this data pertains is The National Human Rights Commission consists of a
(a) St. Lawrence type Chairperson, four full-time members and ex-officio
(b) China type members of various commissions members. The
Chairperson is a retired Chief Justice of the Supreme
(c) West European type
Court. Hence statement 1 is correct. It is not mandatory
(d) Mediterranean type for the each State to have its own Human Rights
Ans. (d) : The given data pertains to Mediterranean Commission. Hence statement 2 is incorrect. As
type of climate. Dry summer and wet winter is the main National Human Rights Commission is a statuary body,
characteristic feature of the Mediterranean type of hence its power is recommendatory in nature. Hence
climate and in the given data, maximum rainfall occurs statement 3 is correct. The National Human Rights. It is
in the winter season from October to February. not mandatory to appoint a woman as a member of the
66. Match List-I with List-II and select the correct commission. Hence statement 4 is not correct.
answer using the codes given below the lists: 68. The most short-lived of all of the Britain's
List-I List-II constitutional experiments in India was the:
(Timber) (Country) (a) Indian Councils Act of 1861
A. Cedar 1. Myanmar (b) Indian Councils Act of 1892
B. Douglas Fir 2. Canada (c) Indian Councils Act of 1909
C. Mahogany 3. Mexico (d) Government of India Act of 1919
D. Teak 4. Honduras Ans. (c) : Time span of given acts:
Codes: Indian Councils Act of 1861 – 31 years
A B C D Indian Councils Act of 1892 – 17 years
(a) 3 2 1 4 Indian Councils Act of 1909 – 10 years
(b) 3 2 4 1 Government of India Act 1919 – 16 years
(c) 2 3 4 1 69. The Constitution of India recognises:
(a) Only religious minorities
(d) 2 3 1 4
(b) Only linguistic minorities
Ans. (c) : The correct matching would be –
Timber Country (c) Religious and linguistic minorities
Ceda Canada (d) Religious, linguistic and ethnic minorities
Douglas fir Mexico Ans. (c) : The Constitution of India uses the word
Mahogany Honduras ’minorities’ in some Articles (Articles 29 to 30 and 350
A to 350) but does not define it anywhere. Article 30
Teak Myanmar
speaks specifically of two categories of minorities;
INDIAN CONSTITUTION AND POLITY religious and linguistic.
70. In the new Panchayati Raj Bill enacted in 1993,
67. Consider the following statements regarding there are several fresh provisions deviating
the National Human Rights Commission of from the past. Which one of the following is not
India: one such provision?
1. Its Chairman must be retired Chief Justice (a) A number of added responsibilities in the area
of India. of agriculture, rural development, primary
2. It has formations in each state as State education and social forestry among others
Human Rights Commission (b) Elections being made mandatory for all posts
3. Its power are only recommendatory in at the time they are due
nature (c) A statutory representation for women in the
4. It is mandatory to appoint a woman as a panchayats, upto a third of the strength
member of the Commission (d) Regular remuneration to the panchayat
Which of the above statements are correct? members, so as to ensure their punctuality
(a) 1, 2, 3 and 4 (b) 2 and 4 and accountability
(c) 2 and 3 (d) 1 and 3 Ans. (d) : “Regular remuneration to the panchayat
Ans. (d) : The National Human Rights Commission members, so as to ensure their punctuality and
(NHRC) of India was established on 12th October, 1993. accountability” was not included under 73rd
National Human Rights Commission is a statuary body. Amendment Act 1993.
IAS (Pre) GS 1999 Paper I 464 YCT
71. Consider the following statements: ECONOMY
An amendment to the constitution of India can
be initiated by the: 74. The planning process in the industries sector in
1. Lok Sabha 2. Rajya Sabha India has assumed a relatively less important
3. State Legislatures 4. President position in the nineties as compared to that in
Which of the above statements is/are correct?
(a) 1 alone (b) 1, 2 and 3 the earlier period. Which one of the following is
(c) 2, 3 and 4 (d) 1 and 2 not true in this regard?
Ans. (d) : Under Part-20 of the Constitution, there is a (a) With the advent of liberalisation, industrial
detailed provision of the procedure related to the investments/development have largely been
amendment of the Constitution under Article 368. placed within the domain of private and
From the point of view of amending the constitution, multinational sectors.
the Articles of the Indian Constitution can be divided
into three sections – those articles, which can be (b) With markets assuming a central place, the
amended by a simple majority in the Parliament. (ii) role of central planning in many sectors has
Articles which can be amended by a two-third majority been rendered redundant
in Parliament. (iii) Those articles which can be amended (c) The focus of planning has shifted to sectors like
by the acceptance of resolutions of the legislatures of human resource development, infrastructure,
half the states of India with a two-third majority in the
population control and welfare.
Parliament. A motion for amendment can be moved in
either House (Lok Sabha, Rajya Sabha) of Parliament (d) The nation's priorities have shifted away from
but it will be considered passed when it is approved by industrial development to rural development.
both the Houses of Parliament by a two-third majority. Ans. (d) : Statement (a), (b) and (c) are true which
72. Consider the following statements about the explains the declining important position of industries
recent amendments to the Election Law by the sector after nineties in the planning process.
Representation of the people (Amendment) Act 75. Consider the following statements:
1996:
1. Any conviction for the offence of insulting Small-scale industries are, in most cases, not as
the Indian National flag or the Constitution efficient and competitive as the large-scale
of India shall entail disqualification for ones. Yet the Government provides preferential
contesting elections to Parliament and State treatment and reservations in a range of
legislatures for six years from the date of products to the small firms because small-scale
conviction.
industries:
2. There is an increase in the security deposit
which a candidate has to make to contest 1. Provide higher employment on a per unit
the election to the Lok Sabha capital deployment basis.
3. A candidate cannot now stand for election 2. Promote a regional dispersion of industries
from more than one Parliamentary and economic activities.
constituency.
3. Have performed better in export of
4. No election will now be countermanded on
the death of a contesting candidate. manufactured products than the large scale
Which of the above statements are correct? ones.
(a) 2 and 3 (b) 1, 2 and 4 4. Provide jobs to low-skill workers, who
(c) 1 and 3 (d) 1, 2, 3 and 4 otherwise may not find employment avenues
Ans. (b) : As per the Representation of People Act elsewhere.
(RPA), 1951, more constituencies were allowed to Which of the above statements are correct?
contest until in 1996 when the RPA was amended to set (a) 1 and 4 (b) 1 and 2
the cap at two constituency. Hence statement 3 is not (c) 2 and 3 (d) 3 and 4
correct. Rest of the given statements is correct. Ans. (b) : Small-scale industries contribution to Indian
73. A British citizen staying in India cannot claim exports is 35% and in the total manufacturing turnover
Right to: contributes approximately 40%. Small scale industries
(a) Freedom of trade and profession have a low capital output ratio and give employment to
(b) Equality before the Law larger number of workers. As they use raw material they
(c) Protection of life and personal liberty
can be dispersed over a wider area.
(d) Freedom of religion
Ans. (a) : Fundamental rights under Article 15, 16, 76. The Economic crisis in the latter half of 1990s
19,29 and 30 are exclusive to the citizens of India. most seriously affected Indonesia, Thailand,
Freedom of Trade and Profession comes under article Malaysia and South Korea. The cause of the
19(1) (g). crisis was:
IAS (Pre) GS 1999 Paper I 465 YCT
(a) Mismanagement of the financial resource and Codes:
financial sector, in general A B C D
(b) The prolonged over-valuation of local (a) 1 4 2 3
(b) 1 4 3 2
currencies vis-a-vis the western currencies.
(c) 4 1 2 3
(c) The downswing and recession in the western (d) 4 1 3 2
economies which earlier provided export Ans. (b) : A - 1, B - 4, C - 3, D - 2
market to these export oriented countries. 79. Which one of the following is the objective of
(d) None of the above National Renewal Fund?
Ans. (a) : Mismanagement of the financial resources (a) To safeguard the interests of workers who
and financial sector were major cause for the economic may be affected by technological upgradation
crisis during later half of 1990s that affected these of industry or closure of sick units
countries. (b) To develop the core sector of the economy
77. The farmers are provided credit from a (c) For the development of infrastructure such as
number of sources for their short and long energy, transport communications and irrigation
term needs. The main Source of credit to the (d) For human resource development such as full
literacy, employment population control,
farmers include:
housing and drinking water
(a) The Primary Agricultural Cooperative Ans. (a) : The concept of the National Renewal Fund
Societies, Commercial Banks, RRBs and was announced by the Government as a part of the New
private money lenders Industrial Policy, 1991. The Government established the
(b) The NABARD, RBI, commercial banks and National Renewal Fund (NRF) by a Government of
private money lenders India resolution on 3rd February, 1992.
(c) The District Central Cooperative Banks Assistance from NRF has been provided for the
(DCCB), the lead banks, IRDP and JRY Voluntary Retirement Scheme (VRS) and the scheme of
counseling, retraining etc. of workers rationalised from
(d) The Large Scale Multi-Purpose Adivasis
the organised sector. The main objective of the National
Programme, DCCB, IFFCO and commercial
Renewal Fund was to provide a social safety net to the
banks. workers who are likely to be affected by technological
Ans. (a) : Regional rural banks were established under up-gradation and modernisation in the Indian industry.
RRB Act 1976. They provide credit to agriculture and 80. Match List-I with List-II and select the correct
other rural activities. answer using the codes given below the lists:
78. The product life cycle from inception to demise List-I List-II
is shown in the graph. Match List-I with List-II A. WTO 1. Provides loans to address short-
and select the correct answer using the codes term balance of payment
given below the list: problems
List-I B. IDA 2. Multi-lateral trade negotiation
body
(Stage)
C. IMF 3. Sanction of soft loans
A. Product D. IBRD 4. Facilitating lending and
Development borrowing for reconstruction and
B. Maturity Development
C. Growth Codes:
A B C D
D. Introduction
(a) 2 3 4 1
List-II (b) 2 3 1 4
(Zone) (c) 3 2 4 1
(d) 3 2 1 4
Ans. (b) : The correct match is as follows–
List-I List-II
A. WTO Multi-Lateral Trade negotiation body
B. IDA Sanction of soft loan
C. IMP Provides loans to address short-term
balance of payment problems
D. IBRD Facilitating lending and borrowing
for reconstruction and development.
IAS (Pre) GS 1999 Paper I 466 YCT
81. Since 1980, the share of the tertiary sector in (c) A is true but R is False
the total G.D.P. of India has (d) A is false but R is True
(a) Shown an increasing trend Ans. (a) : Budgetary deficit is the difference of total
receipts of the government and total expenditure of the
(b) Shown a decreasing trend
government. Whereas fiscal deficit is the difference of
(c) Remained Constant total expenditure and total receipts except borrowing.
(d) Been fluctuating That’s why the fiscal deficit is greater than budgetary
Ans. (a) : The share of the tertiary sector (service sector) in deficit.
when the question was asked total GDP or Gross Domestic Fiscal deficit = Revenue receipts + non-debt creating
Product of India was increasing which was a sign of capital receipts – Total expenditure ;
economic development. This reproduces the trend shown Budget deficit = Total receipts – Total expenditure.
by western countries as they were developing. 85. Assertion (A) : Information technology is
82. The term "imperial preference" was applied to fast becoming a very
the: important field of activity in
India.
(a) Special privileges on British imports in India
Reason (R) : Software is one of the major
(b) Racial discrimination by the Britishers exports of the country and
(c) Subordination of Indian interest to that of the India has a very strong base
British in hardware.
(d) Preference given to British political agents (a) Both A and R are True and R is the correct
over Indian Princes explanation of A
Ans. (a) : Imperial Preference (later Commonwealth (b) Both A and R are True but R is not a correct
Preference) was a proposed system of reciprocally explanation of A
leveled tariffs or free trade agreements between (c) A is true but R is False
different Dominions and Colonies within the British (d) A is false but R is True
Commonwealth of Nations. The purpose of such Ans. (c) : Yes, it is true that information technology is
practices was to promote the mutual prosperity, and fast becoming a very important field of activity in India
thus unity of allied imperial nations. because now a days it is used everywhere in
development of every sector. It is applied in field of
83. Assertion (A) : Devaluation of a currency may
medicine, research, banking etc. It is also true that
promote export. software is one of major exports of the country but India
Reason (R) : Price of the country's products does not have strong base in hardware.
in the international market 86. From the balance sheet of a company, it is
may fall due to devaluation. possible to:
(a) Both A and R are True and R is the correct (a) Judge the extent of profitability of this
explanation of A company
(b) Both A and R are True but R is not a correct (b) Assess the profitability and size of the
company
explanation of A
(c) Determine the size and composition of the
(c) A is true but R is False assets and liabilities of the company
(d) A is false but R is True (d) Determine the market share, debts and assets
Ans. (a) : Devaluation is a conscious decision taken by of the company
Central Bank of the country to lower the external value Ans. (c) : In financial accounting, a balance sheet or
of domestic currency. After devaluation of the rupee statement of financial position is a summary of the
Indian goods would become cheaper for foreigners. financial status of an organisation which can be a sole
84. Assertion (A) : Fiscal deficit is greater than proprietorship, a business partnership or a company.
budgetary deficit. Assets, liabilities and ownership of equity are listed as
Reason (R) : Fiscal deficit is the borrowing on a specific date, which is normally the end of the
from the Reserve Bank of financial year. A balance sheet is the “snapshot of a
India plus other liabilities of company’s financial condition”.
the government to meet its 87. Persons below the poverty line in India are
expenditure. classified as such based on whether:
(a) Both A and R are True and R is the correct (a) They are entilied to a minimum prescribed
explanation of A food basket.
(b) Both A and R are True but R is not a correct (b) They get work for a prescribed minimum
explanation of A number of days in a year
IAS (Pre) GS 1999 Paper I 467 YCT
(c) They belong to agricultural labourer 4. Some areas have faced continuous political
household and the scheduled caste/tribe social instability.
group Which of the above statements are correct?
(a) 1, 2 and 3 (b) 1, 2 and 4
(d) Their daily wages fall below the prescribed
(c) 1, 3 and 4 (d) 2, 3 and 4
minimum wages
Ans. (a) : Regional disparities in India are high and
Ans. (a) : Poverty line in India is classified on the basis have been increasing in recent years, because only in
of calories per day available to a person. It is a simple selected sites, frequent investments has being made.
average and does not show the actual availability of Some areas are less agro-climatically developed and
food to a person or a household. some areas are completely oriented towards agricultural
88. In a business concern, there are four functions transformation and as a result, they are facing a lack of
namely, Production (PR), Finance (FN) social and economic opportunities. Hence, the required
answer will be option (a).
Personnel (PS) and Marketing (MK). The
90. Which one of the following statements
Customer (C) may also play a role in the
regarding the levying, collecting and
prosperity of the concern. Match List-I with distribution of Income Tax is correct?
List-II and select the correct answer using the (a) The Union levies, collects and distributes the
codes given below the lists: proceeds of income tax between itself and the
List-I List-II states
(Pie Diagram) (Statement) (b) The Union levies, collects and keeps all the
proceeds of income tax to itself
(c) The Union levies and collects the tax but all
A. 1. Customer is the the proceeds are distributed among the states
controlling factor (d) Only the surcharge levied on income tax is
shared between the Union and the states
B. 2. Marketing is an Ans. (a) : Article 270 of the Constitution permitted
integrating factor while mandatory sharing of the net proceeds of income tax
customer is the levied and collected by the Union with the States. Such
controlling factor proceeds assigned to States did not form part of the
Consolidated Fund of India. Article 272 provided for
C. 3. Marketing is the sharing of Union excise duties, but Parliament by law so
controlling factor provides.
91. Consider the following statements:
D. 4. Marketing is more Industrial development in India, to an extent, is
important than others constrained by–
1. Lack of adequate entrepreneurship and
5. Marketing is as equal a leadership in business.
function as the others 2. Lack of savings to invest.
Codes: 3. Lack of technology skill and infrastructure.
A B C D 4. limited purchasing power among the larger
masses.
(a) 4 3 1 2
Which of the above statements are correct?
(b) 5 4 1 2 (a) 1, 2 and 3 (b) 1, 3 and 4
(c) 4 3 2 5 (c) 2, 3 and 4 (d) 1, 2 and 4
(d) 5 4 2 1 Ans. (b) : Industrial development in India is constrained
Ans. (b) : The correct match is (b) A-5, B-4, C-1, D-2 due to several factors. India is a developing country
89. Consider the following statements: with world’s second largest population. Most of the
Regional disparities in India are high and have population is being employed in agriculture sector in
been rising in recent years because: rural areas. Most of the businesses are owned by big
business houses. Some of the key factors which are
1. There is persistent investment over time
responsible for the slow industrial growth in India are
only in select locales
lack of business leadership and entrepreneurship
2. Some areas are agro-climatically less because people don’t want to take risk. Along with that
conducive to development India has to highly depend upon western countries for
3. Some areas continue to face little or no technology. As a developing country, per capita income
agrarian transformation and the consequent is low in India and due to this, people have not much
lack of social and economic opportunities. money to spend.
IAS (Pre) GS 1999 Paper I 468 YCT
92. Among which one of the following sets PHYSICS
social/religious groups is the extent of poverty
the highest, as per government statistics for the 95. Barium in a suitable form is administered to
nineties? patients before an X-ray examination of the
(a) Muslims in Kerala, Gujarat and A.P. stomach, because:
(b) Tribals in Bihar, Orissa, M.P. and Maharashtra (a) Barium allows X-ray to pass through the
(c) Scheduled Caste in Punjab, Western UP, stomach on account of its transparency to X-
Northern Rajasthan and Tamil Nadu
rays
(d) Christians in Gujarat, and Assam
(b) Barium compound, like magnesium sulphate
Ans. (b) : According to the recent data made available by
Central Statistical Organisation, the incidence of poverty is helps in cleaning the stomach before X-ray
highest in tribes of Bihar, Orissa and Madhya Pradesh. examination
93. Tourism industry in India is quite small (c) Barium is a good absorber of X-rays and this
compared to many other countries in terms of helps the stomach to appear clearly in contrast
India's potential and size. Which one of the with the other regions in the picture.
following statements is correct is this regard? (d) Barium salts are white in colour and this
(a) Distances is India are too for apart and its helps the stomach to appear clearly in contrast
luxury hotels are too expensive for western
with other regions in the picture.
tourists
(b) For most of the months India is too hot for Ans. (c) : Barium is given in adequate amount to
western tourists of feel comfortable patients before X-ray examination. The gastrointestinal
(c) Most of the picturesque resorts in India such tract does not show up very well on ordinary X-ray
as in the North-East and Kashmir are, for all pictures. However, if a patient drinks a white liquid
practical purposes, out of bounds which contains a chemical called barium sulphate, the
(d) In India, the infrastructure required for outline of the upper parts of the gut (esophagus,
attracting tourists is inadequate stomach and small intestines) shows up clearly on X ray
Ans. (d) : India due to its handicrafts, culture and pictures as X-rays do not pass through barium.
monuments is a preferred destination for tourism. 96. Endoscopy, a technique used to explore the
However, there is a lack of tourist infrastructure. stomach or other inner parts of the body is
94. The Employment Assurance Scheme envisages based on the phenomenon of:
financial assistance to rural areas for (a) Total internal reflection
guaranteeing employment to at least. (b) Interference
(a) 50 percent of the men and women seeking
(c) Diffraction
jobs in rural areas
(b) 50 percent of the men seeking jobs in rural (d) Polarization
areas Ans. (a) : Endoscopy, a technique used to explore the
(c) One man and one woman in a rural family stomach or other inner parts of the body is based on the
living below the poverty line principle of total internal reflection. The endoscopy
(d) One person in a rural landless household procedure uses an endoscope to examine the interior of
living below the poverty line a hollow organ or cavity of the body. Unlike many other
Ans. (c) : Employment Assurance Scheme was medical imaging techniques, endoscopes are inserted
launched on 2nd October, 1993 for implementation in directly into the organ.
1778 identified backward blocks of different States. 97. A hemispherical bowl is filled to the brim with a
Later, the scheme was extended to the remaining blocks beverage. The contents of the bowl are
of the country in a phased manner. At present, the transferred into a cylindrical vessel whose radius
is 50% more than its height. If the diameter is
scheme is being implemented in all the rural blocks of
same for both bowl and cylinder, then the volume
the country. The programme has been restructured from
of the beverage in the cylindrical vessel will be:
1.4.99. The primary objective of the EAS is to provide
gainful employment during the lean agricultural season (a)
to all able bodied adults in rural areas who are in need
and desirous of work, but cannot find work. The (b)
secondary objective is the creation of community, social
and economic assets for sustained employment and (c) 100%
development. Internal debt comprises loans raised in the (d) More than 100% (that is, some liquid will still
open markets. be left in the bowl)
IAS (Pre) GS 1999 Paper I 469 YCT
Ans. (c) : Let the radius of the hemispherical bowl be ‘r’ 3. Wires supporting the filament melt at
2 thigh temperature.
∴ volume of the bowl = πr 3
3 Which of the above statements are correct?
as the beverage is filled to the brain (a) 1 and 3 (b) 2 and 3
2 (c) 1 and 2 (d) 1, 2 and 3
∴ volume of the beverage = πr 3 ………(i) Ans. (d) : An electric light bulb uses a glowing wire
3
filament heated by electrical resistance to generate light
Now radius of the cylindrical vessel = r (given that (a process known as thermal radiation). The 'bulb' is the
diameter is same as the bowl). glass enclosure which keeps the filament in a vacuum or
and height of the vessel = low-pressure noble gas. Complete evacuation of bulb is
50 not possible. A small quantity of gases left behind
r = h+ h
1w which expands on heating through filament. Moreover,
the wires supporting the filament sometimes melt due to
1 3
= h+ h⇒ h high heat generated as a result of high resistance of
2 2 filament. Thus, these factors leads to short life span of
2 ordinary bulb.
h= r
3 101. Consider the following statement regarding a
Now volume of the vessel = πr 2 h motor car battery:
2 1. The voltage is usually 12V.
= πr 2 × r
3 2. Electrolyte used in hydrochloric acid.
2 3. Electrodes are lead and copper.
= πr 3 ………(ii)
3 4. Capacity is expressed in ampere-hour.
as the volume is equal to the volume of beverage. Which of the above statements are correct?
100% beverage can be transferred to the vessel. (a) 1 and 2 (b) 2 and 3
98. A hunter aims his gun at a point between the (c) 3 and 4 (d) 1 and 4
eyebrows of a monkey sitting on a branch of a Ans. (d) : In case of motor car battery or automotive
tree. Just as he fires, the monkey jumps down. batteries a nominal 12-volt potential difference is
The bullet will: provided by connecting six galvanic cells in series.
(a) Hit the monkey at the point aimed Capacity of these batteries is expressed in ampere-hour.
(b) Hit the monkey below the point aimed Electrolyte used is a solution of about 35% sulphuric
(c) Hit the monkey above the point aimed acid and 65% water and electrodes used are plates of
(d) Miss the monkey altogether lead and separate plates of lead dioxide.
Ans. (a) : As monkey and bullet both fall under the 102. Match List-I (Quantity) with List-II (Units)
same gravitational force, so bullet will hit exactly at the and select the correct answer using the codes
same point where it has been aimed.
given below the lists:
99. Cobalt-60 is commonly used in radiation
therapy because it emits: List-I List-II
(a) Alpha rays (b) Beta rays A. High speed 1. Mach
(c) Gamma rays (d) X-rays B. Wavelength 2. Angstrom
Ans. (c) :The most common radioactive isotope of C. Pressure 3. Pascal
cobalt is cobalt-60. It is an important gamma-emitting D. Energy 4. Joule
radionuclide produced as a fission product. The gamma Codes:
ray (photon) energy can be used in radiotherapy such as A B C D
for the treatment of cancer, in food irradiation, or in
(a) 2 1 3 4
industrial gauges or sensors. Cobalt-60 is used in the
radiotherapy of cancer. Gamma rays penetrating power (b) 1 2 4 3
is vey great and can pass through several centimeters of (c) 1 2 3 4
Lead. (d) 2 1 4 3
100. Consider the following statements: Ans. (c) : Mach number (Ma or M) is Commonly used
to represent the speed of an object when it is travelling
An ordinary light bulb has a rather short life
close to or above the speed of +sound. Pressure is the
because the– force per unit area applied in a direction perpendicular
1. Filament wire is not uniform to the surface of an object. The SI unit for pressure is
2. Bulb cannot be evacuated completely. the Pascal (Pa), equal to one Newton per square meter
IAS (Pre) GS 1999 Paper I 470 YCT
(N/m2). In physics, the wavelength of a sinusoidal wave Ans. (c) : Glass is etched by applying a caustic
is the spatial period of the wave—the distance over compound to the glass surface. The compound
which the wave's shape repeats. The unit for wavelength chemically deposits at the glass substrate (borosilicate),
is angstrom. In physics, energy is a quantity that is the and causes etching in the glass surface. Another method
ability to do work. In the International System of Units involves physically grinding the glass surface with an
(SI), energy is measured in joules, but in some fields
abrasive, like a grinding wheel. The surface is worn
other units such as kilowatt-hours and kilocalories are
away where the wheel hits, and the etched surface
also used.
remains. Some very artistic designs can be created with
103. For which one of the following is capillarity not
both methods. Glass can be etched or scratched by
the only reason?
diamond and hydrofluoric acid.
(a) Blotting of ink
(b) Rising of underground water 107. Assertion (A) : To dilute sulphuric acid, acid
(c) Spread of water drop on a cotton cloth is added to water and not
(d) Rising of water from the roots of a plant to its water to acid.
foliage Reason (R) : Specific heat of water is quite
Ans. (b) : Capillary action, or capillarity, is a large.
phenomenon where liquid spontaneously rises in a (a) Both A and R are True and R is the correct
narrow space such as a thin tube, or in porous materials
explanation of A
such as paper or in some non-porous materials such as
liquefied carbon fibre. This effect can cause liquids to (b) Both A and R are True but R is not a correct
flow against the force of gravity or the magnetic field explanation of A
induction. In blotting of ink, spread of water drop on a (c) A is true but R is False
cotton cloth and the rising of water from the roots of a (d) A is false but R is True
plant to its foliage etc. are example of it whereas for
rising of underground water capillarity is not the only Ans. (a) : Sulphuric acid (H2SO4) reacts very
reason. It can also rise due to water recharge also. vigorously with water, in a highly exothermic reaction.
Thus if you add water to concentrated sulfuric acid, it
104. Consider the following statements:
can boil and you may get a nasty acid burn. That's why
1. If a person looks at a coin which is in a
for dilution, acid is added to water not water to acid as
bucket of water, the coin will appear to be
closer than it really is. specific heat of water is quite large and it can absorb
large quantity of heat produced by sulphuric acid.
2. If a person under water looks at a coin above
Moreover water is less dense than sulphuric acid, so if
the water surface, the coin will appear to be at
a higher level than it really is. water is poured on the acid, the reaction occurs on top
Which of the above statements is/are correct? of the liquid. If acid is added to the water, it sinks and
any wild and crazy reactions have to get through the
(a) 1 and 2 (b) 1 alone
water or beaker.
(c) 2 alone (d) Neither 1 nor 2
Ans. (a) : When the person looks at a coin which is in a 108. Match List-I (Naturally occurring substances)
bucket of water, the coin will appear closer and at a with List-II (Elements) and select the correct
higher level than it really is. µ2 = real depth / apparent answer using the codes given below the lists:
depth. List-I List-II
A. Diamond 1. Calcium
CHEMISTRY B. Marble 2. Silicon
105. When ants bite, they inject C. Sand 3. Aluminium
(a) Glacial acetic acid (b) Methanol D. Ruby 4. Carbon
(c) Formic acid (d) Satiric acid Codes:
Ans. (c) : When ants bite, they inject formic acid in the A B C D
muscle tissue. This formic acid is found at the mandible (a) 3 1 2 4
of ants. (b) 4 2 1 3
106. Consider the following statements: (c) 2 1 3 4
Glass can be etched or scratched by (d) 4 1 2 3
1. Diamond 2. Hydrofluoric acid Ans. (d) : Diamonds is the polymorph of the element
3. aquaregia 4. conc. sulphuric acid carbon. Calcium is the basic element of naturally
Which of these statements are correct? occurring marble. Sand is formed by Silicon and
(a) 1 and 4 (b) 2 and 3 Aluminium is the basic element of naturally occurring
(c) 1 and 2 (d) 2 and 4 Ruby.
IAS (Pre) GS 1999 Paper I 471 YCT
BIOLOGY Codes:
A B C D
109. Assertion (A) : Insect resistant transgenic (a) 3 1 4 2
cotton has been produced by
(b) 1 3 2 4
inserting Bt gene.
Reason (R) : The Bt gene is derived from a (c) 1 3 4 2
bacterium. (d) 3 1 2 4
(a) Both A and R are True and R is the correct Ans. (c) : Adrenalines are secreted in the situations of
explanation of A exercise, fear or any kind of dangerous situation.
(b) Both A and R are True but R is not a correct Oestrogens are steroid hormones or female primary sex
explanation of A hormones. Insulin is responsible for the metabolism of
(c) A is true but R is False sugar in the body. Pheromones are the compounds
(d) A is false but R is True which are secreted generally from insect. Some of the
Ans. (a) : Bacillus thuringiensis is the bacterium which pheromones include alarm pheromones, food trail
is used in Bt gene of cotton. Bt cotton is insect resistant. pheromones, and sex pheromones.
110. Assertion (A) : Dolly was the first cloned 113. Neem tree has acquired industrial importance
mammal. as a source of:
Reason (R) : Dolly was produced by in (a) Biopesticide and antifertility compound
vitro fertilization. (b) Antifertility compound, biofertilizer and anti-
(a) Both A and R are True and R is the correct cancer drug
explanation of A (c) Biofertilizer, biopesticide and antifertility
(b) Both A and R are True but R is not a correct compound
explanation of A (d) Anti-cancer drug, biopesticide and biofertilizer
(c) A is true but R is False Ans. (c) : Neem has been patented by India for its use
as biofertilizer, biopesticide and antifertility compound
(d) A is false but R is True
as contraceptive. Neem is a natural herb that comes
Ans. (d) : Cloning is a process, which is artificially
from the neem tree, other names for which include
performed in laboratory ( in vitro). Dolly was a cloned
Azadirachta indica and Indian lilac. The extract comes
animal, but not the first cloned mammal. In 1984, a
from the seeds of the tree and has many different
mice was cloned first.
traditional uses. Neem is known for its pesticidal and
111. In eye donation, which one of the following
insecticidal properties, but people also use it in hair and
parts of donor's eye is utilized?
dental products
(a) Iris (b) Lens
(c) Cornea (d) Retina 114. Match List-I (Drugs/Chemicals) with List-II
Ans. (c) : Cornea, the main focusing part , is the clear (Their uses) and select the correct answer using
front surface of the eye. Like a window, it allows light the codes given below the lists:
to enter the eye. Vision could be partially reduced or List-I List-II
lost if the cornea becomes cloudy or scarred. This A. Atropine 1. Local anesthesia
condition is known as corneal blindness. Eye donation B. Ether 2. Heart trouble
is an act of donating one’s eyes after his/her death. Only C. Nitroglycerine 3. Dilation of pupil
corneal blinds can be benefited through this process not
D. Pyrethrin 4. Mosquito control
other blinds. Cornea transplant is the surgical procedure
which replaces a disc-shaped segment of an impaired Codes:
cornea with a similarly shaped piece of a healthy donor A B C D
cornea. (a) 1 3 2 4
112. Match the hormones in List-I with items in (b) 1 3 4 2
List-II and select the correct answer using the (c) 3 1 4 2
codes given below the lists: (d) 3 1 2 4
List-I List-II Ans. (d) : Ether is used as an anesthesia at the time of
A. Adrenalin 1. Anger, fear, danger surgery to prevent pain. Atropine is used as eye drop to
B. Estrogen 2. Attracting partners through dilate the pupil before eye examination. Nitroglycerine
sense of smell is widely used in the treatment of heart. Pyrethrin is
C. Insulin 3. Females biodegradable, non-persistent pesticide which is used to
D. Pheromones 4. Glucose kill mosquitoes, fleas and other insects.
IAS (Pre) GS 1999 Paper I 472 YCT
115. Indian farmers are unhappy over the ENVIRONMENT AND ECOLOGY
introduction of "Terminator Seed Technology"
118. The minimum land area recommended for forest
because the seeds produced by this technology
cover to maintain proper Ecological balance in
are expected to:
India is:
(a) Show poor germination (a) 25% (b) 33%
(b) Form low-yielding plants despite the high (c) 43% (d) 53%
quality Ans. (b) : For proper ecological balance, 33% of forest
(c) Give rise to sexually sterile plants land is recommended. In 2019, total forest cover in
(d) Given rise to plants incapable of forming India was 21.67% only.
viable seeds 119. "India has the largest population of the Asian
X. Today, there are just about 20,000 to 25,000
Ans. (c) : The Terminator gene technology is also
X in their natural habitat spreading across the
called, 'Genetic Use Restriction Technology (GURT), evergreen forests, dry thorn forests, swamps
terminator gene is a specific genetic sequence inserted and grasslands. Their prime habitats are
into a seed’s DNA. Once activated by a synthetic however the moist deciduous forests. The X
chemical catalyst of the manufacturer’s choosing, the population in India ranges from North-West
sequence renders the seed and crop which produces India where they are found in the forest
sterile Plant. This means that the farmer cannot retain divisions of Dehradun. Bijnor and Nainital
districts of UP to the Western Ghats in the
seeds for future crops as is the practice in India. The
states of Karnataka and Kerala and in Tamil
farmer has to repurchase the seeds at a heavy cost.
Nadu.
116. Which one of the following genetic diseases is In Central India, their population is distributed
sex-linked? is southern Bihar and Orissa. In the East, they
(a) Royal haemophilia are seen in North Bengal, Assam and a few
(b) Tay-Sachs disease other states."
(c) Cystic fibrosis The animal "X" referred to in this quotation is:
(d) Hypertension (a) Lion
(b) Elephant
Ans. (a) : Royal hemophilia is sex- linked disease.
(c) Tiger
Clotting of blood is abnormally delayed even small cut
(d) One-horned rhinoceros
will result in non stop bleeding in the affected
Ans. (b) : The animal X referred in question is elephant.
individual. Queen Victoria was a carrier of this disease The population of elephants in Northern India ranges in
and produce hemophilic descendents, so hemophilia is a the western Himalayan foothills from Dehradun to
royal disease. Nepal in Uttarakhand and Uttar Pradesh. The North-
Eastern distribution is found from Sikkim, north
Bengal, Assam eastwards till Mishmi Hills in Arunachal
SCIENCE AND TECHNOLOGY Pradesh, Nagaland, Meghalaya, Manipur, Mizoram and
Tripura. The central distribution occurs in south Bihar,
117. Low temperatures (Cryogenics) find
south Bengal, Jharkhand, Chhattisgarh and Orissa. The
application in : southern distribution has its presence in Western Ghats
(a) space travel, surgery and magnetic levitation and parts of Karnataka, Tamil Nadu and Kerala.
(b) surgery, magnetic levitation and telemetry 120. Which one of the following is a useful
functional association between fungi and the
(c) space travel surgery and telemetry roots of higher plants?
(d) space travel, magnetic levitation and (a) Bio-fertilizer (b) Coralloid root
telemetry. (c) Lichen (d) Mycorrhiza
Ans. (a) : Cryogenics is the study of the production and Ans. (d) : Mycorrhiza is a useful functional association
behavior of materials at very low temperatures. between fungi and the roots of higher plants in which
fungi absorb nutrients from the soil as their roots are not
Cryogenic processes are used in Space Travel, surgery
such strong and the trees provide food to the fungi. The
and transportation like magnetic levitation. best example is Orchid mycorrhiza.
IAS (Pre) GS 1999 Paper I 473 YCT
121. The first marine sanctuary in India, having Contentious Issues
within its bounds coral reefs, mollusca, 1. Disengagement of troops.
dolphins, tortoises and various kinds of sea
birds, has been established in: 2. Settlement of boundary dispute.
(a) Sundarbans (b) Chilka Lake 3. Sharing river water.
(c) Gulf of Kachchh (d) Lashadweep Match the issues with the areas marked in the
Ans. (b) : The first marine sanctuary in India, having
map as A, B and C and select the correct
within its bounds coral reefs, mollusca, dolphins,
tortoises and various kinds of sea birds, has been answer using the codes given below:
established in Chilka Lake. Codes:
122. Which one of the following legislations does not A B C
deal with the protection of environment?
(a) The Water (Cess) Act, 1977 (a) 2 1 3
(b) The Forest (Conservation) Act, 1980 (b) 2 3 1
(c) The Public Liability Insurance Act, 1991 (c) 1 3 2
(d) The Port Laws Amendment, 1997
(d) 3 2 1
Ans. (a) : The Water (Cess) Act, 1977 is related to
water and irrigation, not with the protection of Ans. (b) : During composite dialogue process between
environment other acts in the question deal with India and Pakistan in November 1998, three contentious
protection of environment. issues were discussed i.e. settlement of Boundary issues
in Rann of Kutch, marked A in the map, sharing of river
CURRENT AFFAIRS water of Indus tributary marked ‘B’ map and
123. Which one of the countries labelled as 1, 2, 3 disengagement of troops in Siyachin marked “C” in the
and 4 on the given map was grantee map.
membership of G-15 in its VII summit at 125. Match List-I with List-II and select the correct
Kuala-Lumpur? answer using the codes given below the lists:
Africa List-I List-II
(Writers) (Books)
A. Sashi Tharoor 1. Clear Light of Day
B. Amitav Ghosh 2. Circle of Reason
C. Anita Desai 3. Love and Longing in
Bombay
D. Vikram Chandra 4. Show Business
(a) 1 (b) 2 Codes:
(c) 3 (d) 4 A B C D
Ans. (d) : Kenya was granted as the 16th member of G–
(a) 4 2 1 3
15 countries which is marked as ‘4’ in the given map.
124. In the November 1998 Composite Dialogue (b) 4 2 3 1
Process between India and Pakistan, three (c) 2 4 1 3
contentious issues listed below as 1, 2 and 3 (d) 2 4 3 1
were discussed.
Ans. (a) : Shashi Tharoor is the writer of “Show
Business, Amitav Ghosh wrote “Circle of Reason”,
Anita Desai wrote “Clear light of Day” and Vikram
Chandra wrote” Love and longing in Bombay”. These
were best selling novels.

MISCELLANEOUS
126. In a group of Five people, K, L and M are
ambitious, M, N and R are honest, L, M and N
are intelligent and K, N and R are industrious.
Among these neither industrious nor ambitious
person (s) would include:
(a) K alone (b) L and R
(c) M and N (d) none in the group
IAS (Pre) GS 1999 Paper I 474 YCT
Ans. (d) : People Traits Ans. (b) : Let us assume radius of sphere = r m
Honest Ambitious Intelligent Industrious
AO = OB = 10m
OC = 3 m
K
OD = 2r – 3
L ∴ AO × OB = CO × DO
M ⇒ 10 × 10 = 3 (2r – 3)
N ⇒ 6r = 109
R 109
⇒ r= m
Hence Industrious or ambitious are: 6
K, L, M, N and R.
Hence there is none in the group who are neither industrious
nor ambitious.
127. A man is standing on the 8 m long shadow of a
6 m long pole. If the length of his shadow is 2.4
m, what is the height of the man?
(a) 1.4 m (b) 1.6 m
(c) 1.8 m (d) 2.0 m
Ans. (c) : 8 m long shadow is costed for a height of 6
m. ∴ surface area of spherical dome - shaped roof
2
6  109 
∴ 1 m long shadow is costed for a height for = m . 4π   ×3
= 
8 6 
= 109πm 2
6  109 
∴ 2.4 m long shadow is costed for a height for = × 24 2 
8  6 
= 1.8m 130. Strips of two metals A and B are firmly jointed
128. If the angle of a triangle are in the ratio of 4 : 3 together as shown in the figure.
: 2, than the triangle:
(a) is obtuse
(b) has one angle greater than 80° On heating, A expands more than B does. If this
(c) is a right triangle joined strips is heated, then it will appear as:
(d) is acute
Ans. (d) : Angles of the triangles may be 4x°, 3x° and (a) (b)
2x°
∴ 4x + 3x + 2x = 180 ⇒ 9x = 180 (c) (d)
x = 20° Ans. (b) : Thermal expansion is the tendency of matter
So the angles 80°, 60° and 40°. Hence the triangle will to change in volume in response to a change in
be an acute triangle. temperature. In this case A expands more than B.
129. The surface area of spherical dome-shaped roof Therefore, expansion of volume will be more for a in
of a cylindrical water tank shown in the figure comparison to B on application of same amount of heat.
is: Therefore, on heating it will appear like figure (b).
131. The first feature film (talkie) to be produced in
India was:
(a) Hatimatai (b) Alam Ara
(c) Pundalik (d) Raja Harishchandra
Ans. (b) : Alam Ara (1931) was the first Indian feature
film (talkie) produced and directed by Ardeshir Irani.
The first Indian talkie was so popular that police aid had
to be summoned to control the crowds.
132. If X + 2Y = 2X + Y, then X2/Y2 is equal to:
(a) 60π m2 (b) 109πm2 (a) 0 (b) 1
(c) 120π m2 (d) 300πm2 (c) 2 (d) 4
IAS (Pre) GS 1999 Paper I 475 YCT
Ans. (b) : Given that 2. The percentage of those who worked 35 or
2x + y = x + 2y more hours is less than 25
∴ 2x – x = 2y – y 3. At least 5 staff members worked more than
⇒ x=y 44 hours.
x2 x2 Which of these inferences is/are valid?
∴ = =1
y2 x2 (a) 1 alone (b) 2 alone
133. Match List-I with List-II and select the correct (c) 1 and 2 (d) 1, 2 and 3
answer using the codes given below the lists: Ans. (c) : Average no. of hour
List-I List-II 5(17) +1(22) + 25(27) + 40 (32) +15(37) + 8(42.5)
=
(Libraries) (Location) 5 +1+ 25 + 40 +15 + 8
A. Saraswati Mahal 1. Patna 2957
= = 31.45 ~ 30
Library 94
B. Library of Tibetan 2. Dharamshala No. of persons who worked 35 or more hours
Work and Archives 15 + 8 = 23
C. Raza Library 3. Thanjavur
23
D. Khuda Baksh Oriental 4. Rampur % of the such persons = ×100 = 24.46
94
Public Library
⇒ < 25
Codes: st nd
Hence 1 and 2 inferences are valid.
A B C D
135. In a factory quality assurance test is conducted
(a) 3 2 4 1
on various samples for a specific characteristic
(b) 3 2 1 4
value of the product. The values and the
(c) 2 3 1 4
number of samples are as given in the following
(d) 2 3 4 1
table:
Ans. (a) : Library of Tibetan work and archives.
Characteristic value, X Number of Samples
A. Saraswati – Thanjavur Tamil Nadu
10 3
Mahal Library
11 7
B. Library of – Dharamshala
12 10
Tibetan work Himachal Pradesh
13 15
and archives
14 28
C. Raza Library – Rampur , Uttar Pradesh
15 33
D. Khuda Baksh – Patna Bihar
16 24
Oriental public
Library 17 11

134. In an office, the distribution of work hours is as 18 10


shown in the following table: 19 6
No. of Staff Members No. of hours worked 20 3
5 0-19 Consider the following statements based on the
1 20-24 table
25 25-29 1. The probability that X ≤ 15 is 0.64
40 30-34
2. The probability that 13 < X ≤ 17 is greater
15 35-39
than 0.64
8 40-45
3. The probability that X = 15 is less than 0.22
Consider the following inferences drawn from
the table: Which of the above statements is/are not true?
1. The average number of hours worked by a (a) 1 alone (b) 1 and 2
staff member is about 30. (c) 2 and 3 (d) 1, 2, and 3
IAS (Pre) GS 1999 Paper I 476 YCT
Ans. (a) : Number of samples = 150 138. Which one of the following statements is not
correct? Dinar/New Dinar is the currency of :
No.of samples for ( x )
So probability = (a) Sudan (b) Yugoslavia
Total no.of sample (150) (c) U.A.E (d) Tunisia
When we consider the given statements Ans. (c) : Currency of Sudan, Yugoslavia and Tunisia
is dinar, whereas currency of U.A.E. is Dirham.
3 + 7 +10 +15 + 28 + 33 96
(i) P( x ≤ 15) = = = 0.64 139. Which one of the following pairs is not
150 150
correctly matched?
Hence statement 1 is correct. (a) Jim Laker : Highest number of
28 + 33 + 24 +11 96 wickets in a Cricket
(ii) P (13 < x ≤ 17) = = = 0.64
150 150 Test Match.
Hence statement 2 is incorrect. (b) Brain Lara : Highest score in an
33 innings in Test
(iii) P( x = 15) = = 0.22
150 Cricket.
Hence statement 3 is also incorrect. (c) Sanath Jayasurya
and Roshan
136. At a given time, two players are standing on a
Mahanama : Highest partnership in
play-field. The Cartesian coordinates of their
a Test Cricket.
locations are (20, 60) and (-40, –20) units. What
(d) Sunil M. Gavaskar : Highest aggregate of
is the distance the players?
runs in Test Cricket.
(a) 60 units (b) 80 units
Ans. (d) : Among the four given option, option (d) is
(c) 100 units (d) 140 units
not correctly matched because the highest total of runs
Ans. (c) : A = (20, 60) and B (– 40, – 20)
in Test cricket which was scored by Sunil Gavaskar was
AB = 20 − ( −40) 2 + 60 − ( −20) 2 broken by Allan Border of Australia. Sachin Tendulkar
is currently the batsman with the highest total of test
= 3600 + 6400 = 10000 = 100
cricket runs.
137. The area of an ellipse is twice that of a circle. 140. Which one of the following pairs of folk dance
The major diameter of the ellipse is twice that of forms and states is not correctly matched?
the minor diameter. The radius of the circle is: (a) Korku : Maharashtra
(b) Jhummar : Haryana
(a) 50% of minor diameter of the ellipse
(c) Thali : Himachal Pradesh
(b) 50% of major diameter of the ellipse
(d) Mukna : Manipur
(c) Minor diameter of the ellipse
Ans. (c) : Thali is the folk dance of Rajasthan, not the
(d) Major diameter of the ellipse
folk dance of Himachal Pradesh.
Ans. (a) : Let the minor diameter of ellipse be 2x 141. In order to win the Grand Slam in Tennis, a
major diameter of ellipse = 2y which is equal to 2 (2x) player must win which one of the following
∴ 2y = 4x groups of tournaments?
y = 2x ……..(i) (a) Australian Open, Wimbledon, French Open,
Let the radius of circle be r U.S. Open
Then π × x × y = 2πr2 (b) Wimbledon, French Open, U.S. Open
π × x × 2x = 2πr2 (c) Wimbledon, French Open, Paegas Czech
Open, U.S. Open
2πx2 = 2πr2
(d) Davis Cup, Wimbledon, French Open
x=r
Ans. (a) : In order to win the Grand Slam in Tennis, a
1 player has to win four events in a calendar year, which are
∴ radius of the circle is of the diameter of minor
2 as follows - Australian Open, French Open, Wimbledon
ellipse or 50% of minor diameter of the ellipse. Championship and U.S open.
IAS (Pre) GS 1999 Paper I 477 YCT
142. Amar, Akbar and Anthony are friends, being Ans. (d) : On another planet - soil, water, light, air, sky
looked after by a Farah, Amar weighs 50% Local name - sky, light, air, water, soil, so if a person
more than Akbar and Anthony weights 25% feels thirsty on that planet, he will drink the light.
less than Amar. Farah weighs a third of the 145. The average speed of a train in the onward
combined weight of the three boys. All four journey is 25% more than that of the return
together weigh 232 kg. The correct journey. The train halts for one hours on
arrangement of the persons in the ascending reaching the destination. The total time taken
order of their weights is: for the complete to and fro journey is 17 hours
(a) Anthony, Akbar, Farah, Amar covering a distance of 800 km. The speed of
(b) Anthony, Akbar, Amar, Farah train in the onward journey is:
(c) Akbar, Anthony, Amar, Farah (a) 45 km per hour
(d) Akbar, Anthony, Farah, Amar (b) 47.06 km per hour
Ans. (d) : Let Akbar’s weight be x (c) 50.00 km per hour
3x (d) 56.25 km per hour
∴ Amar’s weight 50% more than x = ..…..(i)
2 Ans. (d) : Let the speed in return journey be x km/hr.
3x 3 x 9 x 25 x 5 x
And Anthony’s 25% less than Amar’s = − = ∴ speed in onward journey = x + = km/hr.
2 8 8 100 4
……(ii) 5
2× x × x
1 3x 9 x  ∴ average speed = 4 km/hr
Farah’s weight =  x + +  ……..(iii) 5
3 2 8  x+x
4
1  8 x + 12 x + 9 x  29 x 10 x
= 
3  = 24 = km./hr.
8  9
3x 9 x 29 x Now time taken to complete to and fro journey minus
ATQ = x + + + = 232 rest = 16 hr. and distance is 800 km.
2 8 24
9
= 116x = 232 × 24 ⇒ x = 48 Then 800× = 16
10x
Akbar = 48 kg.
800×9
Amar = 72 kg. x= = 45
[from equ.(i), (ii) and (iii)] 16×10
Anthony = 54 kg.
5
Farah = 58 kg. ∴ speed in on ward journey = × 45 = 56.25km/hr.
4
Hence the correct ascending order is Akbar < Anthony 146. Consider the following statements:
< Farah < Amar. The striking feature of the Jama Masjid in
Kashmir completed by Zainul-Aidin include (s)
143. In the sequence of numbers 5, 8, 13, X, 34, 55,
1. Turret.
89 ........ the value of 'X' is "
2. Similarity with Buddhist pagodas.
(a) 20 (b) 21 3. Persian style
(c) 23 (d) 29 Which of the above statements is/are correct?
Ans. (b) : The given series is formed by adding the (a) 1 alone (b) 1, 2 and 3
previous 2 terms, i.e 8+5=13. Similarly 8+13=21, which (c) 2 and 3 (d) 1 and 3
further gives 21+13=34, hence B is right answer. Ans. (b) : Jama Masjid is a mosque in Srinagar, Jammu
144 On another planet, the local terminology for & Kashmir. The Jama Masjid of Srinagar is situated at
earth, water, light, air and sky, are 'sky', 'light', Nowhatta, in the middle of the old city. It was built by
'air', 'water' and 'earth' respectively. If someone Sultan Sikandar in 1400 AD. Later, the son of Sultan
is thirsty there, what would he drink"? Sikandar, Zain-ul-Abidin had extended the mosque. It is
(a) Sky (b) Water similar to buddist Pagoda with Persian style and also
(c) Air (d) Light includes turret in its architecture.
IAS (Pre) GS 1999 Paper I 478 YCT
147. In a town 25% families own a phone and 15%
own a car. 65% families own neither a phone nor
a car. 2000 families own both a car and a phone.
Consider the following statements in this regard:
1. 10% families own both a car and a phone
2. 35% families own either a car or a phone
3. 40,000 families live in the town.
Which of the above statements are correct?
(a) 1 and 2 149. Lathyrism is caused by excessive consumption
(b) 1 and 3 of:
(c) 2 and 3 (a) Kesari dal
(d) 1, 2 and 3 (b) Mustard oil
Ans. (c) : ATQ (c) Polished rice
25% family have phone (d) Mushrooms
15% family have care Ans. (a) : Lathyrism is caused by excessive
and 65% don’t have a car or a phone consumption of khesari dal. Lathyrism gets its name
∴ percent of family having either phone or care from scientific name of khesari dal which is Lathyrus
= 100 – 65 = 35% …….(i) sativus. The symptom of the disease is paralysis of both
the lower limbs.
∴ percentage of family having both phone and care
150. A company manufacturing air-conditioners has
(25 + 15) – 35 = 5%
set a monthly target. The target and realised
Hence 1st statement is incorrect.
values are shown in the bar chart.
and second statement is correct [from equn. (i)]
ATQ
5% = 2000 family
2000×100
∴ 100% = = 40, 000
5
Hence 3rd statement is also correct.
148. In a code language, "SOLID" is written as
'WPSLPIMFHA' What does the code word
'ATEXXQIBVO' refer to?
(a) EAGER
(b) WAFER
(c) WAGER
(d) WATER Consider the following statements based on the
chart:
Ans. (d)
1. The targeted sales on a monthly basis have
been achieved.
2. The overall target value has been exceeded
by 7.5%
3. The Sales Department deserves a pat on the
back.
Which of the above statement is/are correct?
(a) 1 alone (b) 2 alone
(c) 1 and 2 (d) 2 and 3
Ans. (b) : The right option is (b)- 2 alone.
Similarly
IAS (Pre) GS 1999 Paper I 479 YCT
UNION PUBLIC SERVICE COMMISSION
Civil Services (Preliminary Exam) - 1998
GENERAL STUDIES : PAPER-I
Time: 2 hours Maximum Number: 200

Ans. (b) : Buddhacharita is an epic poem in the Sanskrit


ANCIENT HISTORY mahakavya style on the life of Gautama Buddha by
1. Many of the Greeks, Kushanas and Sakas Aśvaghoṣa, composed in the early second century CE.
embraced Buddhism rather than Hinduism Rest of the options are correctly matched.
because: 4. What is the correct chronological order in
(a) Buddhism was in the ascendant at that time which the following appeared in India?
(b) They had renounced the policy of war and 1. Gold coins
violence 2. Punch marked silver coins
(c) Caste-ridden Hinduism did not attract them 3. Iron Plough
(d) Buddhism provided easier access to Indian
4. Urban culture
society
Select the correct answer using the codes given
Ans. (c) : Greek, Kushanas and Sakas entered India
through North Western Frontier and established their rule below:
or kingdom. They needed legitimacy for their rule for Codes:
people to accept them as their king. Accepting religion of (a) 3, 4, 1, 2 (b) 3, 4, 2, 1
local area was the best way to achieve legitimacy. (c) 4, 3, 1, 2 (d) 4, 3, 2, 1
Buddhism provided easiest entry in comparison to
Ans. (d) : First urban culture referes to Indus Valley
Hinduism which had strict birth related caste structure. So
these rulers embraced Buddhism in order to strengthen Civilisation (2250-1750 BC). Iron Plough were used in
their dominance over the local people. Vedic times. First coins issued in India were punch
marked. The largest number of punch marked (mostly
2. The Ashokan major rock edicts which tell us silver) coins which have been found can be assigned to
about the Sangam Kingdom include rock edicts: the Mauryan period. Gold coins are issued by Indo
(a) I and X (b) I and XI Greeks and Kushan rulers who ruled after Mauryans.
(c) II and XIII (d) II and XIV 5. Which of the following pairs are correctly
Ans. (c) : The Major Rock Edicts of Indian Emperor matched?
Ashoka refer to 14 separate major Edicts of Ashoka 1. Lothal : Ancient dockyard
which are significantly detailed and represent some of 2. Sarnath : First Sermon of Buddha
the earliest dated rock inscriptions of any Indian
3. Rajagir : Lion capital of Ashoka
monarch.
The ll and Xlll rock edicts mention the southern 4. Nalanda : Great seat of Buddhist learning
kingdoms of Chola, Pandya, Satyaputra, Keralaputra Select the correct answer using the codes given
and Tambapanni. below:
3. Which of the following pairs are correctly Codes:
matched? (a) 1, 2, 3 and 4 (b) 2 and 4
1. Mrichchakatikam – Shudraka (c) 1, 2 and 4 (d) 1 and 2
2. Buddhacharita – Vasuvandhu Ans. (c) : Lion capital was originally placed on the top
3. Mudrarakshasha – Vishakhadatta of the Ashoka pillar at the important Buddhist site of
4. Harshacharita – Banabhatta Sarnath by the Emperor Ashoka, in about 250 BCE. The
Select the correct answer using the codes given pillar, sometimes called the Aśoka Column, is still in its
below: original location, but the Lion Capital is now in the
Codes: Sarnath Museum, in the state of Uttar Pradesh, India.
(a) 1, 2, 3 and 4 6. Which one of the following ancient Indian
(b) 1, 3 and 4 records in the earliest royal order to preserve
(c) 1 and 4 food-grains to be utilised during the crises in
(d) 2 and 3 the country?
IAS (Pre) GS 1998 Paper I 480 YCT
(a) Sohgaura Copper-plate MEDIEVAL HISTORY
(b) Rummindei pillar-edict of Ashoka
9. The loss of Qandhar was a big blow to the
(c) Prayaga-Prasasti
Mughal Empire from the view point of:
(d) Mehrauli Pillar inscription of Chandra
(a) Natural resource
Ans. (a) : The earliest known copper-plate called
(b) Buffer territory
Sohgaura is a Maurya record that mentions famine relief
efforts. It is one of the very few pre-Ashoka Brahmi (c) Communication
inscriptions in India. (d) Strategic stronghold
7. The concept of Eight-fold path forms the theme Ans. (d) : Shah Jahan recovered Kandhar in 1638 from
of: the Iranians but lost it again in 1649 despite
(a) Dipavamsa three campaigns. The loss of Kandhar was a big blow as
(b) Divyavadana it was a strategic stronghold.
(c) Mahaparinibban Sutta 10. Fawazil in the Sultanate period meant:
(d) Dharma Chakara Pravartana Sutta (a) Extra payment made to the nobles
Ans. (d) : The Noble Eight fold Path, is one of the (b) Revenue assigned in lieu of salary
principal teachings of the Buddha, who described it as (c) Excess amount paid to the exchequer by the
the way leading to the cessation of suffering (Dukkha) iqtadars
and attainment of self-awakening. In Buddhist
(d) Illegal exactions extracted from the peasants
symbolism, the Noble Eight fold Path is often
represented by means of the Dharma wheel Ans. (c) : From the time of Balban the iqtadars were
(Dharmachakra), whose eight spokes represent the eight expected to send the excess of the income to the center
elements of the path. Eight fold path includes right after meeting his and the army’s expenses this was
understanding, right speech, right livelihood; right known as Fawazil. It was done to maintain a balance
mindfulness, right thought, right action, right effort and between the income and expenditure of Iqta-holders.
right concentration.
11. The Sultan of Delhi who is reputed to have built
8. The given map relates to:
the biggest network of canals in India was:
(a) Iltutmish
(b) Ghiyasuddin Tughlaq
(c) Firoz Shah Tughlaq
(d) Sikandar Lodi
Ans. (c) : The Sultan of Delhi who is reputed to have
built the biggest network of canals in India was Firoz
Shah Tughlaq.
12. Consider the following statements:
Ahadis were those troopers who
(a) Kanishka at the time of his death 1. Offered their services singly.
(b) Samudra Gupta after the close of his south 2. Did not attach themselves to any chief.
India compaigan 3. Had the emperor as their immediate conlonel
(c) Asoka towards close of his reign 4. Attached themselves to Mirzas.
(d) Empire to Thaneswar on the eve of Harsha's Of these statements:
accessin (a) 1, 3 and 4 are correct
Ans. (c) : The given map is related to the reign of (b) 1, 2 and 3 are correct
Ashoka towards end of his rule. Ashoka 304 - 232 BC (c) 2 and 3 are correct
popularly known as Ashoka the Great was an Indian (d) 1 and 4 are correct
emperor of the Maurya Dynasty who ruled over the
Indian Sub continent from 269 BC to 232 BC. His Ans. (b) : Ahadis were the gentlemen troopers
empire stretched from present day Pakistan, who were recruited individually and were under the
Afghanistan in the west to the present-day Bangladesh command of a separate mansabdar or officer, and had a
and the Indian state of Assam in the east and as far diwan and a bakhshi of their own. Ahadis were considered
South as Northen Kerala and Andhra. very efficient and loyal troops and were paid high salaries.

IAS (Pre) GS 1998 Paper I 481 YCT


13. Consider the following: 16. What is the correct sequence of the following
1. Tughlaquabad fort 2. Lodi garden events?
3. Qutb Minar 4. Fatehpur Sikri 1. Tilak’s Home Rule League
The correct chronological order in which they 2. Kamagatamaru Incident
were built is: 3. Mahatma Gandhi's arrival in India
(a) 3, 1, 4, 2 (b) 3, 1, 2, 4 Select the correct answer using the codes given
(c) 1, 3, 2, 4 (d) 1, 3, 4, 2 below:
Ans. (b) : Tughlaqabad fort - built by Ghiyas-ud-din (a) 1, 2, 3 (b) 3, 2, 1
Tughlaq - in 1321 (c) 2, 1, 3 (d) 2, 3, 1
Lodhi garden - built by Ala-ud-din Alam Shah - in Ans. (d) : Tilak’s Home Rule League – April, 1916;
1444 Karnagatamaru Incident – September, 1914; Mahatma
Qutub Minar - construction started by Qutu-ud-din Gandhi’s arrival in India from South Africa –1915
Aibak and completed by Iltutmish - 1192 17. Simon Commission of 1927 was boycotted
Fatehpur Sikri - founded by Akbar - in 1569. because:
14. In the given map, the shaded part represents (a) There was no Indian Member in the
Akbar's empire at a certain juncture; A stands
Commission
for an independent country and 'B' marks the
site of a city. Which one of the following (b) It supported the Muslim League
alternatives gives all correct information? (c) Congress felt that the people of India are
entitled to Swaraj
(d) There were differences among the members
Ans. (a) : In 1928, Simon commission came to India to
look after the working of the govt. of India Act, 1919 and
to suggest changes. But it consisted only of Englishmen &
did not have a single Indian representative. That’s why it
received Pan India boycott.
18. The Indian Muslims, in general, were not
attracted to the Extremist movement because
of the:
(a) Influence of Sir Sayyid Ahmed Khan
(b) Anti-Muslim attitude of extremist leaders
(a) Akbar in 1557 : (A) Golkunda, (B) Lahore
(c) Indifference shown to Muslim aspirations
(b) Akbar in 1557 : (A) Khandesh, (B) Multan
(d) Extremists policy of harping on Hindu past
(c) Akbar in 1605 : (A) Gondwana, (B) Multan
Ans. (d) : In general, the Indian Muslims were not
(d) Akbar in 1605 : (A) Gondwana, (B) Lahore
attracted to the extremist movement because of the
Ans. (d) : The shaded part represents Akbar’s territory religious policy of harping on Hindu past like
at the time of his death in 1605. A – Gondwana; B – celebrating Shivaji Jayanti and Ganesh Utsav by Tilak
Lahore and holy dip in sacred rivers during Swadeshi
movement.
MODERN HISTORY
19. Which one of the following events, was
15. The member of Shivaji's Ashta Pradhana who characterised by Montague as 'preventive
looked after foreign affairs was Murder'?
(a) Peshwa (b) Sachiv (a) Killing of INA activists
(c) Pandit Rao (d) Sumant (b) Massacre of Jallianwala Bagh
(c) Shooting of the Mahatma
Ans. (d) : There was a Council of Ministers consisting
of eight ministers to advise the Maratha ruler Shivaji. (d) Shooting of curzon-wylhe
This council was called 'Ashta Pradhan'. The foreign Ans. (b) : Jalianwala Bagh massacre of 13th April, 1919
minister of this council who oversees foreign affairs was carried out by Reginald Edward Harry Dyer. Edwin
was called Sumant or Dabir. The main work of Sumant Montague condemned the act and called it a
or Dabir was to advise the king in respect of foreign ‘‘Preventive Murder’’. Whereas C.F. Andrews called it
states and in all matters of war and peace. a ‘‘ruthless murder’’.

IAS (Pre) GS 1998 Paper I 482 YCT


20. What is the correct sequence of the following 22. Which of the following pairs are correctly
events? matched?
1. The August offer 1. Theodore Beck : Mohammadan Anglo-
2. The I.N.A. trial Oriental
2. Ilbert Bill : Ripon
3. The Quit India Movement
3. Pherozeshah : Indian National
4. The Royal Indian Naval Rating's Revolt
Mehta Congress
Select the correct answer using the codes given 4. Badruddin Tyabji : College, Aligarh
below: Muslim League
Codes: Select the correct answer using the codes given
(a) 1, 3, 2, 4 (b) 3, 1, 2, 4 below:
(c) 1, 3, 4, 2 (d) 3, 1, 4, 2 Codes:
Ans. (a) : The August Offer was a proposal made by the (a) 1, 2, 3 and 4 (b) 2 and 4
British government in 1940 promising the expansion of (c) 1, 3 and 4 (d) 1, 2 and 3
the Executive Council of the Viceroy of India to include Ans. (d) : Theodore Beck was a British educationalist.
more Indians, the establishment of an advisory war From 1883 until his death he was principal of
council, giving full weight to minority opinion. Muhammadan Anglo-oriental college.
The Indian National Army trials (INA trials), which Ilbert Bill was introduced on 9 February, 1883 during
the viceroyship of the Ripon.
are also called the Red Fort trials, were the British
Pherozeshah Mehta was a member of Indian National
Indian trial by courts-martial of a number of officers of
Congress.
the Indian National Army (INA) between November
Badruddin Tyabji was an Indian Lawyer, activist and
1945 and May 1946, for charges variously for treason, politician. He was not associated with college, Aligarh
torture, murder and abetment to murder during World. Muslim league.
The Quit India Movement, or the August Movement, Hence only 1, 2 and 3 are correctly matched.
was a movement launched at the Bombay session of 23. "The congress is tottering to its fall and one of
the All-India Congress Committee by Mahatma my great ambitions while in India, is to assist it
Gandhi on 8 August 1942, during World War II, to a peaceful demise." This statement is
demanding an end to British Rule of India. attributed to:
On February 18, 1946, a section of non-commissioned (a) Lord Dufferin (b) Lord Curzon
officers and sailors known as Ratings, serving in the (c) Lord Lytton (d) None of the above
Royal Indian Navy, mutinied against the British Ans. (b) : Lord Curzon was the Viceroy of India from
Officers. The mutiny started as a strike by the ratings 1899 to 1905. In a letter to the British secretary of state
to protest against the hardships regarding pay, food in 1900, Curzon wrote, ‘‘In my belief congress is
and racial discrimination. tottering to its fall and one of my greatest ambition
while in India is to assist it to a peaceful demise.’’
21. Which one of the following defines extremist
24. Who was the leader of the Ghaddar Party?
ideology during the early phase of Indian
(a) Bhagat Singh (b) Lala Hardayal
freedom movement?
(c) Bal Gangadhar Tilak (d) V.D. Savarkar
(a) Stimulating the production of indigenous
Ans. (b) : Ghadar party was an extremist revolutionary
articles by giving them preference over
organisation founded by Sikhs in the US and Canada
imported commodities
with the aim of securing India's independence from
(b) Obtaining self-government by aggressive British rule. It was founded by Lala Hardayal and Sohan
means in place of petitions and constitutional Singh Bhakna in the year 1913. The party had its
ways. headquarters in San Francisco. Its key members
(c) Providing national education according to the were Bhai Parmanand, Kartar Singh Sarabha, Abdul
requirements of the country Hafiz Mohamed Barakatullah, Rashbehari Bose and
(d) Organising coups against the British Empire Gulab Kaur.
though military revolt 25. The educated middle class in India:
Ans. (b) : Out of the given options Obtaining self- (a) Opposed the revolt of 1857
government by aggressive means in place of petitions (b) Supported the revolt of 1857
and constitutional ways, defines extremist ideology (c) Remained neutral to the revolt of 1857
during the early phase of Indian Freedom Movement. (d) Fought against native rulers
IAS (Pre) GS 1998 Paper I 483 YCT
Ans. (c) : The revolt did not spread to all parts of the (a) Both A and R are true and R is the correct
country. Nor was it supported by all groups and sections explanation of A
of the Indian society. South and West India remained (b) Both A and R are true but R is not the
largely outside the fold of the revolt. Many Indian rulers
correct explanation of A
refused to help the rebels and some were openly hostile
to the rebels and helped the British in suppressing the (c) A is true but R is false
revolt. The middle and upper classes and the modern (d) A is false but R is true
educated Indians also did not support the revolt. Ans. (a) : The Khilafat Movement was a pan-
26. Lord Mountbatten came to India as Viceroy Islamist political protest campaign launched by
along with specific instruction to: Muslims of British India. The Khilafat Movement
(a) Balkanize the Indian sub-continent did bring the Urban Muslims into the fold of the
(b) Keep India united if possible National Movement. There was a predominant
(c) Accept Jinnah's demand for Pakistan element of anti-imperialism in both the National
(d) Persuade the Congress to accept partition and Khilafat Movements.
Ans. (b) : For the purpose of taking necessary steps for 30. Match List-I with List-II and select the correct
the transfer of power to the Indians Lord Wavell was answer using the codes given below the lists:
recalled and Lord Mountbatten was appointed the new
List-I List-II
Viceroy of India. Lord Mountbatten assumed the office
of the Viceroy and Governor-General in 1947. His A. 1556 1. Battle of Haldi Ghati
immediate task was to restore peace among the two B. 1600 2. Nadir Shah's capture of Delhi
warring sections and keep India united if possible. C. 1686 3. Death of Shivaji
27. When the Indian Muslims League was inducted D. 1739 4. Grant of Charter to East India
into the Interim government in 1946, Liyaqat Ali Company
Khan was assigned the portfolio of: 5. Accession of Akbar
(a) Foreign affairs (b) Home
Codes:
(c) Finance (d) Defence
A B C D
Ans. (c) : The Muslim League declared the 'Direct
(a) 3 4 2 1
Action Day' on August 16, 1946, by rejecting the
'Cabinet Mission Plan'. On 6 August 1946, the Viceroy (b) 5 4 3 2
invited Jawaharlal Nehru to form the Interim (c) 5 2 1 4
Government and on 2 September 1946, he formed a 12- (d) 1 5 3 2
member Interim Government in which Liaquat Ali
Ans. (b) : The correct match is :
Khan was entrusted with the Finance Department.
1556 – Accession of Akbar
28. The Indian National Congress agreed in 1947
to the partition of the country mainly because: 1600 – Grant of Charter to East India Company
(a) The principle of two-Nation theory was then 1680 – Death of Shivaji
acceptable to them 1739 – Nadirshah’s capture of Delhi
(b) It was imposed by the British Government
and the Congress was helpless in this regard 31. At the time of India's Independence, Mahatma
(c) They wanted to avoid large-scale communal Gandhi was:
riots (a) A member of Congress Working Committee
(d) India would have otherwise lost the (b) Not a member of the Congress
opportunity to attain freedom
(c) The President of the Congress
Ans. (c) : The Indian National Congress agreed in 1947
to the partition of the country mainly because they (d) The General Secretary of the Congress
wanted to avoid communal violence. Ans. (b) : At the time of India’s independence Gandhiji
29. Assertion (A) : The Khilafat movement did was not a member of congress.
bring the urban Muslims into the fold of the
Gandhi did not hold any political status after the
National Movement.
Independence had been achieved. After a long struggle
Reason (R) : There was predominant element
of anti-imperialism in both the National and for independence he finally rested without any position
Khilafat Movements. politically.

IAS (Pre) GS 1998 Paper I 484 YCT


INDIAN GEOGRAPHY Codes:
A B C D
32. Some people in Manipur live in houses built on
floating islands of weeds and decaying (a) 4 3 1 2
vegetation held together by suspended silt. (b) 4 5 1 3
These Islands are called: (c) 4 5 2 1
(a) Tipis (b) Barkhans (d) 2 1 4 3
(c) Phumdis (d) Izba
Ans. (b) : A. Jamnagar became a town when it housed
Ans. (c) : Some people in Manipur live in houses built a cement factory in the 1940s. Digvijay Cement
on floating Islands of weeds and decaying vegetation
Company means DCC was set up. And Sikka became
held together by suspended slit. These islands are called
Phumdis. Phumdis cover about two-thirds of Loktak's one of the prominent towns of the district.
surface. The two metre thick Phumdis are buoyant B. Hospete is also noted for its iron ore mining and
enough to support bamboo hutments. steel plants.
33. Which one of the following east flowing rivers C. Korba is the power capital of the newly formed state
of India has rift valley due to down warping? Chhattisgarh. It also has power plants such as
(a) Damodar (b) Mahanadi NTPC, CSEB, and Bharat Aluminium Company
(c) Son (d) Yamuna (BALCO).
Ans. (a) : Damodar river, out of the given options, D. Fertilizer products of Haldia are branded as Tata
forms rift valley due to down warping. Downward PARAS which is established as the most preferred
denotes a segment of the earth’s crust that is broadly Fertilizer brand in the market.
bent downward. 36. In the vicinity of Mumbai, a number of
34. Commercial production of mineral oil has specialised towns have been developed. Match
started recently in which one of the areas of lists of specialisation with towns and select the
India, labelled 1, 2, 3 and 4 in the rough map
correct answer using the codes given below the
given below:
lists:
List-I List-II
(Towns) (Specialisation)
A. Alibag 1. Fishing Centre
B. Balapur 2. Holiday resort
C. Nhava Sheva 3.Petro-chemical complex
D. Ratnagiri 4. Port
(a) 1 (b) 2 Codes:
(c) 3 (d) 4 A B C D
Ans. (a) : According to present data area 1 is the (a) 1 3 2 4
Mangala which located in Rajasthan Baramer district. (b) 2 3 4 1
The Mangala Area sits in Baramer Basin and is thought (c) 3 4 2 1
to contain oil in place volumes of 3.6 billion barrels of (d) 2 1 4 3
which 1 billion barrels are thought to be recoverable. Ans. (b) : Alibag – Holiday Resort
Cairn India is the operator of the field, a subsidiary of Balapur – Petrochemical complex
Vadanta Group.
Nhave Sheva – Port
35. Match List-I with List-II and select the correct Ratnagiri – Fishing Centre
answer using the codes given below the list: 37. Match List-I with List-II and select the correct
List-I List-II answer using the codes given below the lists:
(Places) (Industries) List-I List-II
A. Jamnagar 1. Aluminium (Agricultural products) (Foremost producer)
B. Hospet 2. Woolen Textile A. Cotton 1. Madhya Pradesh
C. Korba 3. Fertilizers B. Gram 2. Gujarat
D. Haldia 4. Cement C. Black pepper 3. West Bengal
5. Iron and Steel D. Pine-apple 4. Kerala
IAS (Pre) GS 1998 Paper I 485 YCT
Codes: 40. The four railway junctions shown by numerals
A B C D 1, 2, 3, 4 on the rough outline map of Gujarat
(a) 2 1 4 3 are respectively.
(b) 2 1 3 4
(c) 1 2 4 3
(d) 1 2 3 4
Ans. (a) : Gujarat is the foremost producer of cotton.
Gram is produced in Madhya Pradesh, Black pepper is
produced in Kerala and Pineapple is produced in West
Bengal.
38. The rough outline map given shows centres of
cement industry labelled 1, 2, 3 and 4. Match (a) Palanpur, Mahesana, Ahmedabad and Vadodara
these centres with the following sets of names: (b) Mahesana, Surendranagar, Rajkot and Junagarh
(c) Palanpur, Kanda, Bhuj and Okla
(d) Ahmedabad, Vadodara, Bhavnagar and Broach
Ans. (b) : Mahesana, Surendranagar, Rajkot and
Junagarh.
41. Which one of the following countries has
replaced Italy as the major importer of Bauxite
from India?
Names: (a) Canada
A. Katni B. Tirunelveli (b) Greece
C. Sikka D. Churk (c) Ukraine
Select the correct answer using the codes given (d) United Arab Emirates
below: Ans. (a) : Canada is the largest importer of bauxite
Codes: from India. India fulfills its bauxite import requirements
A B C D by importing from 19 countries. Major trading partners
(a) 3 4 2 1 for import of bauxite in India are Brazil (59.34 USD
(b) 2 4 1 3 Million), Guinea (58.37 USD Million), China (21.37
(c) 1 2 4 3 USD Million), Trinidad and Tobago (21.15 USD
(d) 2 3 1 4
Million), Ghana (11.3 USD Million).
Ans. (b) : Katni in Madhya Pradesh, Tirunelveli in
Kerala, Sikka in Gujarat and Churk in Uttara Pradesh 42. Which one of the following is the correct
are the important centres of Cement Industry. sequence of the states (labelled 1, 2, 3 and 4) of
India shown on the map in descending order in
39. Which one of the following ports shown on the
rough outline map of India is a riverine port? terms of their available ground water resource
for irrigation?

(a) 1 (b) 2 (a) 3, 4, 1, 2 (b) 3, 4, 2, 1


(c) 3 (d) 4 (c) 4, 3, 1, 2 (d) 4, 3, 2, 1
Ans. (d) : The 1, 2, 3 and 4 inscribed points in the given Ans. (d) : Uttar Pradesh is the state where 9,384,000
map of India are the ports of Kandla, Cochin, Paradip
hectare are covered by ground water irrigation.
and Calcutta respectively. Kandla, Cochin and Paradip
are the sea ports. Calcutta harbor is a coastal port of the Maharashtra covers 1,912,000 hectare, West Bengal
river. The reason is that it is situated on the Hooghly covers 1,397,000 hectares, Assam covers hectare for
River. ground water irrigation.

IAS (Pre) GS 1998 Paper I 486 YCT


43. Among the Indian states shown labelled 1, 2, 3 A. Central Drug Research Institute
and 4 in the rough outline map given, the B. National Atlas and Thematic Mapping
correct sequence of descending order of percent Organisation
of scheduled tribe population to their total
populations is: C. National Institute of Ocean Technology
D. Temperate Forest Research Centre
Select the correct answer using the codes given
below.
Codes:
A B C D
(a) 4 3 2 6
(b) 4 5 1 6
(c) 2 3 4 5
(d) 1 6 2 3
Ans. (a) : ‘Central Drug Research Institute’ is marked
by ‘4’ which is situated in Lucknow of Uttar Pradesh.
‘National Atlas and Thematic Mapping Organisation’ is
(a) 1, 3, 2, 4 (b) 3, 1, 2, 4
(c) 3, 1, 4, 2 (d) 1, 3, 4, 2 situated in Calcutta which is marked as ‘3’. National
Institute of Ocean Technology is situated in Chennai,
Ans. (b) : According to Census of India 2001, schedule
tribe population in Mizoram is 94.75%, Arunachal which is marked ‘2’ and Temperate Forest Research
Pradesh has 63.66% as schedule tribe, Manipur with Centre is situated in Shimla which is marked as ‘6’ on
34.4% and Orissa with 22.21% of tribal population. the map.
44. Which one of the following sets of states stands 47. Which one of the following pairs of cities has
to benefit the most from the Konkan Railway? recently been connected through a 6-lane
(a) Goa, Karnataka, Maharashtra, Kerala express way?
(b) Madhya Pradesh, Maharashtra, Tamil Nadu, (a) Ahmedabad : Vadodara
Kerala
(b) Dhaka : Chittagong
(c) Tamil Nadu, Kerala, Goa, Maharashtra
(d) Gujarat, Maharashtra, Goa, Tamil Nadu (c) Islamabad : Lahore
Ans. (a) : Konkan Railway route connects Roha in (d) Mumbai : Pune
Maharashtra with Mangalore in Karnataka. Konkan Ans. (d) : Recently Mumbai and Pune has been
Railway established with partnership of three states connected through a 6–lane express way.
Maharashtra, Goa and Karnataka. The state of Goa,
Karnataka, Maharashtra and Kerala benefit most from
Konkan railway. WORLD GEOGRAPHY
45. Which one of the following languages belongs 48. The misery index is the sum of a country s
to the Austric group? unemployment and inflation rate. The higher
(a) Marathi (b) Ladakhi the index, the more miserable is the country to
(c) Khasi (d) Tamil live in. The figure given below is the Misery
Ans. (c) : Austric group (Nishada) spoken by 1.38% of Index for various countries in Europe:
total Indian population, this group includes mainly tribal
languages and dialects of the central tribal belt, Khasi
and Jaintia hills of Meghalaya and Nicobar island.
46. Match the following research institutes A, B, C
and D with their respective location labelled as
1 to 6 in the given rough outline map:

Which of the following conclusions can be


drawn from the misery index given above?
1. Britain is the most miserable country to
live in.
2. The inflation rate in Spain is less than that
in Belgium and Britain.
IAS (Pre) GS 1998 Paper I 487 YCT
3. Italy and France seem to have almost 1. St. Lawrence 2. Orinoco
identical unemployment rate. 3. Mackenzie 4. Amazon
4. The higher the misery index, the higher the
5. Yukon
inflation rate. Select the correct answer
using the codes given below: Codes:
Codes: A B C D
(a) 1 alone (b) 2 and 3 (a) 4 3 2 1
(c) 1, 2, 3 and 4 (d) None
(b) 5 3 1 2
Ans. (b) : The catastrophe of any country is estimated
by the rate of unemployment prevailing in that country (c) 5 4 1 3
and the rate of inflation. Poverty is equal to the sum of (d) 3 1 4 2
these two rates. A study of the poverty indexes of Ans. (b) : In the given diagram of North America and
European countries displayed in the rectangle given in
the question leads to the conclusion that the inflation South America
rate in Spain is lower than in Belgium and Britain. ‘a’ marked river is Yukon
Spain has the highest unemployment rate, while France ‘b’ marked river is Mackenzie
and Italy have roughly the same unemployment rate. ‘c’ marked river is St. Lawrence
High rates of inflation are indicative of the condition of ‘d’ marked river is Orinoco
high poverty index.
49. Which one of the following regions of world 52. When there is noon at I.S.T. meridian people
supplies the maximum of our imported on another place of the Earth are taking their 6
commodities (in terms of rupee value): O'clock morning tea. The longitude of the
(a) Africa (b) America places:
(c) Asia and Oceania (d) Europe
(a) 17°30' E (b) 7°30' W
Ans. (d) : Europe is the region that supplies the
maximum of our imported commodities (in terms of (c) 172°30' E (d) 90° W
rupee value). Ans. (b) : If the time of the place is 6 hours behind IST,
50. The correct sequence of different layers of the it means it is located to the west of Prime Meridian i.e.
atmosphere from the surface of the Earth O degree meridian. A difference of 15 degrees in
upwards is: longitude represents the difference of 1 hour. So
(a) Troposphere, Stratosphere, Ionosphere, Mesosphere difference of hour = 6 so difference of longitude from
(b) Stratosphere, Troposphere, Ionosphere, Mesosphere IST = 6 × 15 = 90°.
(c) Troposphere, Stratosphere, Mesosphere, Ionosphere Longitude of IST = 82°30'E so longitude of the place =
(d) Stratosphere, Troposphere, Mesosphere, Ionosphere
– 90° + 82° 30' = – 7°30' i.e. 7° 30' west.
Ans. (c) : The correct sequence of different layers of the
atmosphere from the surface of the Earth upwards is 53. Match List-I with List-II and select the correct
Troposphere, Stratosphere, Mesosphere and answer using the codes given below the lists:
Thermosphere. The troposphere is the layer that we live List-I List-II
in. The troposphere starts at ground level and extends (Special characteristic) (Name of planet)
about 12 km upwards. From about 12 km to 50 km
A. Smallest planet of 1. Mercury
upwards is the stratosphere. In this layer, the jet stream
blows strong winds eastward. The mesosphere is the the solar system
coldest region of Earth’s atmosphere. It’s somewhere close B. Largest planet of 2. Venus
to -100 degrees Celsius. Thermosphere is the thickest layer the solar system
in the atmosphere starting at about 80 km. C. Planet second from 3. Jupiter
51. Match rivers labelled A, B, C and D on the the Sun in the solar
given map with their names given in the list
and select the correct answer using the codes system
given bellow the list: D. Planet nearest to 4. Pluto
the Sun
5. Saturn
Codes:
A B C D
(a) 2 3 5 1
(b) 3 5 1 2
(c) 4 1 2 3
(d) 4 3 2 1

IAS (Pre) GS 1998 Paper I 488 YCT


Ans. (d) : The correct match is: ECONOMY
List-I List-II
57. According to the World Development Report,
A. Smallest planet of the 1. Pluto low income economies are those for which the
solar system per capital GNP in 1994 was:
B. Largest planet of the 2. Jupiter (a) US $ 924 or less
solar system (b) US $ 825 or less
C. Planet second from 3. Venus (c) US $ 725 or less
system (d) US $ 525 or less
D. Planet nearest to the 4. Mercury Ans. (c) : According to world development report 1994
Sun low income economics are those with a GNP per capita
* Now, mercury is the smallest planet. of $725 or less. It was $ 675 or less in 1992.
58. The current price index (base 1960) in nearly
INDIAN CONSTITUTION AND POLITY 330, this means that:
(a) All items cost 3.3 times more than what they
54. The Indian parliamentary system is different
did in 1960
from the British parliamentary system in that
India has: (b) The prices of certain selected items have gone
up to 3.3 times
(a) Both a real and a nominal executive
(c) Weighted mean of prices of certain item has
(b) A system of collective responsibility
increased 3.3 times
(c) Bicameral legislature
(d) Gold price has gone up 3.3 times
(d) The system of judicial review
Ans. (c) : The current price index (base 1960) is nearly
Ans. (d) : The parliamentary system of government in
330. This means that weighted means of prices of
India is largely based on the British parliamentary
certain item has increased 3.3 times.
system. However, it never became a replica of the
British system and differs in some respects. The British 59. The banks are required to maintain a certain
system is based on the doctrine of the sovereignty of ratio between their cash in hand and total
Parliament, while the Parliament is not supreme in India assets. This is called:
and enjoys limited and restricted powers due to a (a) SBR (Statutory Bank Ratio)
written Constitution, federal system, judicial review and (b) SLR (Statutory Liquid Ratio)
fundamental rights. (c) CBR (Central Bank Reserve)
55. Panchayat Raj was first introduced in India in (d) CLR (Central Liquid Reserve)
October, 1959 in:
Ans. (b) : The banks are required to maintain a certain
(a) Rajasthan ratio between their cash in hand and total assets. This is
(b) Tamil Nadu called SLR (Statutory Liquid Ratio).
(c) Kerala The ratio of liquid assets to net demand and time
(d) Karnataka liabilities (NDTL) is called statutory liquidity ratio
(SLR). Banks have to report to the RBI every alternate
Ans. (a) : Panchayati Raj System was first introduced in
Friday their SLR maintenance, and pay penalties for
Nagaur district of Rajasthan on October 2, 1959 failing to maintain SLR as mandated.
followed by Andhra Pradesh in 1959. 60. The accounting year of the Reserve Bank of
56. Which one of the following schedules of the India is:
Constitution of India contains provisions (a) April-March
regarding anti-defection Act? (b) July-June
(a) Second Schedule (c) October-September
(b) Fifth Schedule (d) January-December
(c) Eighth Schedule Ans. (b) : When the question was asked the accounting
(d) Tenth Schedule year of the Reserve Bank of India was July1 - June 30.
But in September 2019, a decision at RBI directors meet
Ans. (d) : Then tenth schedule inserted in the
was taken to change the RBI financial accounting year
constitution by 52nd (1985) amendment Act deals with to March-April to align itself with the central
Anti-defection law. government calendar.
IAS (Pre) GS 1998 Paper I 489 YCT
61. Which one of the following is the correct sequence 64. Economic Survey in India is published
of decreasing order of the given currencies in officially, every year by the:
terms of their value in Indian Rupees? (a) Reserve Bank of India
(a) US dollar, Canadian dollar, New Zealand (b) Planning Commission of India
dollar, Hong Kong dollar (c) Ministry of Finance, Govt. of India
(b) US dollar, New-Zealand dollar, Canadian (d) Ministry of Industries, Govt. of India
dollar, Hong-Kong dollar Ans. (c) : The Department of Economic Affairs, Finance
(c) US dollar, Hong-Kong dollar, New Zealand Ministry of India presents the Economic Survey in the
dollar, Canadian dollar parliament every year, just before the Union Budget. It is
(d) US dollar, Hong-Kong dollar, Canadian prepared under the guidance of the Chief Economic
dollar, New-Zealand dollar Adviser, Finance Ministry. It is the ministry’s view on the
Ans. (a) : Foreign currencies Value of INR annual economic development of the country.
US Dollar – 75.04 65. The currency of the proposed European
Canadian Dollar – 60.70 Monetary Union will be:
NZ Dollar – 53.01 (a) Dollar (b) Euro
Hong Kong Dollar – 09.65 (c) Guider (d) Mark
62. 'Eco mark' is given to the Indian products that Ans. (b) : When the question was asked Euro was
are: proposed to be the currency of European Monetary
(a) Pure and unadulterated Union.
(b) Rich in proteins 66. Consider the following statements:
(c) Environment friendly The price of any currency in international
(d) Economically viable market is decided by the:
Ans. (c) : To increase consumer awareness, in the year 1. World Bank
1991, the Bureau of Indian Standards launched the eco- 2. Demand for goods/services provided by
labeling scheme known as Ecomark' for easy
the country concerned.
identification of environment-friendly products. Any
product which is made, used or disposed of in a way 3. Stability of the government of the
that significantly reduces the harm to the environment concerned country.
could be considered environment friendly as product. 4. Economic potential of the country in
The criteria follow cradle-to-grave approach, i.e., from question.
raw material extraction to manufacturing and to Of these statements–
disposal. The Ecomark' label is awarded to consumer (a) 1, 2, 3 and 4 are correct
goods which meet the specified environmental criteria
(b) 2 and 3 are correct
and the quality requirements of Indian Standards.
(c) 3 and 4 are correct
63. Capital Account Convertibility of the Indian
(d) 1 and 4 are correct
Rupee implies:
(a) That the Indian Rupee can be exchanged by Ans. (b) : The price of any currency is international
the authorised dealers for travel market is determined by the forces of demand and
(b) That the Indian Rupee can be exchanged for supply. Demand for a currency increases if the demand
for the goods and services provided by that country
any major currency for the purpose of trade in
increases. Moreover, a stable Govt gives confidence to
goods and services
investors and raises the demand for currency of that
(c) That the Indian Rupee can be exchanged for
country.
any major currency for the purpose of trading
financial assets 67. A consumer is said to be in equilibrium, if:
(a) He is able to fulfill his need with a given level
(d) None of the above
of income
Ans. (c) : Capital account convertibility means free (b) He is able to live in full comforts with a given
conversion of cross-border capital flows. Any entity can level of Income
convert domestic currency into hard currency at the (c) He can fulfill his needs without consumption
prevailing market rate and take hard currency out of the of certain items
country without the need of offering any explanation. (d) He is able to locate new source of income

IAS (Pre) GS 1998 Paper I 490 YCT


Ans. (a) : A consumer will be said to be in equilibrium PHYSICS
when he is able to meet his needs at a certain level of
his income. In case of deviation of equilibrium, there 70. Match the names of outstanding Indian
has been a situation of imbalance between the income scientists given in List-I with area of their
and needs of the consumer. specialized work given in List-II and select the
68. correct answer by using the codes given below
the lists:
Production of Rice and Wheat
List-I List-II
Year (In millions of Tonnes)
A. Dr. Raja 1. Plant Chemistry
Rice Wheat Percentage of
Ramanna
Wheat to Rice
B. Dr. M.S. 2. Nuclear Physics
1950-51 20.58 6.46 31.4
Swaminathan
1960-61 34.58 11.00 31.8
C. Prof. U.R. Rao 3. Thermodynamics
1970-71 42.22 23.83 56.4
and astrophysics
1980-81 58.63 36.31 67.7
D. Prof. Meghnad 4. Space research
1990-91 74.29 55.14 74.2
1994-95 81.81 65.77 80.4 Saha
1995-96 79.62 62.62 78.6 5. Agricultural
The above table indicates the performance of sciences
India in rice and wheat production form 1950- Codes:
51 to 1995-96. Which of the following A B C D
conclusions arrived at from the above table (a) 3 5 2 1
would be valid? (b) 2 1 4 3
1. Record production of rice as well as wheat (c) 2 5 4 3
has been in 1994-95. (d) 3 1 4 2
2. The ratio of wheat to rice production seems Ans. (c) : Raja Ramanna was an Indian physicist who is
to have steadily increased over 16 years.
best known for his role in India’s nuclear program
3. Wheat has not been popular among the during its early stages. Having joined the nuclear
Indian population before 1980. program in 1964, Ramanna worked under Homi
4. India became self-sufficient in rice and Jahangir Bhabha, and later became the director of this
wheat only after 1990. program in 1967.
Select the correct answer using the codes give Mankombu Sambasivan Swaminathan is an Indian
below:
geneticist and administrator, known for his role in
Codes: India’s Green Revolution, a program under which high-
(a) 1 and 2 (b) 1, 2, 3 and 4 yield varieties of wheat and rice were developed.
(c) 3 and 4 (d) None Prof U.R. Rao was an internationally renowned space
Ans. (a) : It is clear from the table that in the year scientist who had made original contributions to the
1994-95, the record production of rice and wheat in development of space technology in India and its
comparison to the previous year was 81.81 million
extensive application to communications and remote
tonnes (rice) and 65.77 million tonnes respectively. The
sensing of natural resources.
ratio of rice production to wheat production has steadily
increased over a period of 16 years. Meghnad Saha FRS was a Indian astrophysicist best
known for his development of the Saha ionization
69. The supply-side economics lays greater equation, used to describe chemical and physical
emphasis on the point of view of: conditions in stars.
(a) Producer (b) Global economy 71. One Astronomical Unit is the average distance
(c) Consumer (d) Middle-man between:
Ans. (a) : Supply side economics lays greater emphasis (a) Earth and the Sun
on the point of view of the producer whereas the (b) Earth and the Moon
demand side economics lays emphasis interest of the (c) Jupiter and the Sun
consumer. (d) Pluto and the Sun
IAS (Pre) GS 1998 Paper I 491 YCT
Ans. (a) : We can say that one astronomical unit (AU) processing of leather and textiles.
represents the mean distance between the Earth and our Magnesium sulphate is a chemical compound
sun. An AU is approximately 93 million miles (150 containing magnesium, sulphur and oxygen, with the
million km). It's approximately 8 light-minutes. More formula MgSO4. It is often encountered as the
exactly, one astronomical unit (AU) = 92,955,807 miles heptahydrate epsomite (MgSO4•7H2O), commonly
(149,597,871 km).
called "Epsom salt".
72. Consider the following statements regarding
Sodium bicarbonate or sodium hydrogen carbonate is the
asteroids:
chemical compound with the formula NaHCO3. The salt
1. Asteroids are rocky debris of varying sizes
orbiting the Sun. has many related names such as baking soda, bread soda,
2. Most of the asteroids are small but some cooking soda, bicarbonate of soda. Caustic soda or sodium
have diameter as large as 1000 km. hydroxide is an essential ingredient in an array of industrial
3. The orbit of asteroids lies between the orbits applications. In addition, consumers use caustic soda when
of Jupiter and Saturn. using cleaners, such as oven and drain cleaners.
Of these statements– 75. Some time back, the Government of India,
(a) 1, 2 and 3 are correct decided to delicense 'white goods' industry.
(b) 2 and 3 are correct 'White goods' include:
(c) 1 and 2 are correct
(a) Stainless steel and aluminium utensils
(d) 1 and 3 are correct
(b) Milk and milk products
Ans. (c) : Asteroids are rocky debris of varying sizes
with diameter from a few kilometres to as large as 1000 (c) Items purchased for conspicuous
km. Their orbit is between Mars and Jupiter and not consumption
between Jupiter and Saturn. (d) Soaps, detergents and other mass
73. A ball is dropped from the top of a high consumption goods
building with a constant acceleration of 9.8 Ans. (c) : White goods are large electrical goods used
m/s2. What will be its velocity after 2 seconds? domestically such as air conditioners, refrigerators,
(a) 9.8 m/s (b) 19.6 m/s cooking range etc., which had a white enamel finish.
(c) 29.4 m/s (d) 39.2 m/s Despite their availability in varied colours now, they are
Ans. (b) : The velocity after time t is given by v= u + gt still called white goods. The term is used where British
Here, u = 0, t = 2s, g = 9.8 m/s2 English is spoken. In the year 1993 the govt. of India
v = 0 + (9.8 × 2) = 19.6 m/s had decided to de-licence these goods.
76. Examine the following three statements:
CHEMISTRY 1. Processed meat a perishable food.
74. Match List-I with List-II and select the correct 2. All perishable foods are packed in sealed
answer using the codes given below the lists: tins.
List-I List-II 3. Sealed tins sometimes do not contain
A. Blue Vitriol 1. Sodium bicarbonate processed meat.
B. Epsom salt 2. Sodium hydroxide Which one of the following inferences can be
C. Baking soda 3. Magnesium sulphate drawn from the above statements?
D. Caustic soda 4. Copper sulphate (a) Sealed tins always contain perishable food
Codes:
(b) Processed meat is sometimes not packed in
A B C D
sealed tins processed meat is always packed
(a) 3 4 1 2
in sealed tins
(b) 4 3 2 1
(c) 3 4 2 1 (c) Processed meat is always packed in sealed
tins
(d) 4 3 1 2
(d) Non-perishable foods are never packed in
Ans. (d) : Blue vitriol is blue, crystalline hydrous
sealed tins
solution of copper sulphate, CuSO4•5H2O, one of the
Ans. (c) : We can draw the inference that processed
most important industrial copper salts, used in
meat is always packed in sealed tins.
insecticides, germicides, and hair dyes and in the
IAS (Pre) GS 1998 Paper I 492 YCT
77. Match List-I with List-II and select the correct 80. Which one of the following metals does not
answer using the codes given below the lists: form amalgams?
List-I List-II (a) Zinc (b) Copper
A. Potassium bromide 1. Fertilizer (c) Magnesium (d) Iron
B. Potassium nitrate 2. Photography Ans. (d) : Amalgam is an alloy consisting of mercury
C. Potassium sulphate 3. Bakery and any other element. Iron being exceptional in nature
D. Monopotassium 4. Gun-power not form amalgam with mercury. That's why mercury is
tartarate stored in vessels made up of iron. Small quantities of an
iron amalgam have, however, been formed by
Codes:
immersing sodium amalgam (containing 1 percent
A B C D sodium) in a clear, saturated solution of ferrous
(a) 2 4 1 3 sulphate, While rest three options zinc, copper and
(b) 2 4 3 1 magnesium combine with mercury to form there
(c) 4 2 3 1 respective amalgam.
(d) 4 2 1 3 81. Consider the following statements about
Ans. (a) : Potassium bromide is used in photography as acetylene:
a restrainer in black and white developer formulas. Gun 1. It is used in welding industry.
powder, also called black powder, is a mixture of 2. It is a raw material for preparing plastics.
sulphur, charcoal, and potassium nitrate. Gun powder 3. It is easily obtained by mixing silicon carbide
can be made by just using potassium nitrate and and water.
charcoal (or alternatively without charcoal), but without Of these statements
the sulphur (or coal), the powder is not as strong. (a) 1 and 2 are correct
Potassium sulfate is primarily used as a fertilizer. Mono (b) 1 and 3 are correct
potassium tartrate is used in bakery by combination (c) 2 and 3 are correct
with baking soda it results in to evolution of CO2 which (d) 1, 2 and 3 are correct
is used for baking cakes.
Ans. (a) : Welding refers to the process of joining two
78. Which one of the following elements is essential or more metals together. Approximately 20% of
for the construction of nuclear reactors? acetylene is consumed for oxyacetylene gas welding
(a) Cobalt (b) Nickel and cutting due to high temperature of flame. Acetylene
(c) Zirconium (d) Tungsten can be semihydrogenerated to form ethylene, providing
Ans. (c) : Zirconium is a chemical element with symbol a feedstock for a variety of Polyethylene plastics. When
Zr and atomic number 40. The name zirconium is taken silicon carbide and water is mixed it produces Silica and
rd
from the name of the mineral zircon, the most important methane. Hence 3 statement is incorrect.
source of Zirconium. Zirconium is used in nuclear
reactors to provide the dadding or outer covering for the BIOLOGY
cylindrical fuel rods that power a nuclear reaction.
82. Which one of the following types of
79. Consider the following statements:
microorganisms is most widely used in
Coke is one of the materials of the charge
Industries?
added to blast furnace for the production of
steel/iron. Its function is to: (a) Bacteria
1. Act as the reducing agent (b) Bacteria and Fungi
2. Remove silica associated with the iron ore (c) Bacteria and Algae
3. Function as fuel, to supply heat (d) Bacteria, Microalgae and Fungi
4. Act as an oxidizing agent Ans. (d) : Bacteria, micro-algae and fungi are most
Of these statements– widely used in industries.
(a) 1 and 2 are correct 83. Match List-I with List-II and select the correct
(b) 2 and 4 are correct answer using the codes given below the lists:
(c) 1 and 3 are correct List-I List-II
(d) 3 and 4 are correct
A. Fruit 1. Ovule
Ans. (c) : Coke is used as a charge in the iron-
B. Seed 2. Leaf
producing furnace, which aids in the blasting in the iron
furnace. Coke is used as a reducing agent and fuel in an C. Wood 3. Stem
iron furnace. Coke is not used in iron ore to separate D. Starch 4. Ovary
mixed silica (sand) and to act as an oxidant. Codes:
IAS (Pre) GS 1998 Paper I 493 YCT
A B C D 86. Ergotism is due to consumption of:
(a) 2 1 3 4 (a) Contaminated grains
(b) 4 1 3 2 (b) Rotting vegetables
(c) Contaminated water
(c) 2 3 1 4
(d) Stale cooked food
(d) 4 3 1 2
Ans. (a) : Ergotism is the effect of long term ergot
Ans. (b) : The correct match is as follows: poisoning, due to ingestion of alkaloids produced by
List-I List-II fungus Claviceps purpurea which is found in infected
cereals and ryes.
A. Fruit – Ovary 1. (Ripen ovary of a plant)
87. The complete conversion of glucose in the
B. Seed – Ovule 2. (Fertilized ovule develop
presence of oxygen into carbon dioxide and
into a seed) water with release of energy is called:
C. Wood – Stem 3. (Wood can be obtained (a) Aerobic respiration
from stem of a tree) (b) Anaerobic respiration
D. Starch – Leaf 4. (Formed as a result of (c) Glycolysis
photosynthesis by leaves) (d) Hydrolysis
84. Match List-I with List-II and select the correct Ans. (a) : Aerobic respiration takes place in the
presence of oxygen and finally degrades glucose to
answer using the codes given below the lists:
carbon dioxide and water with release of energy in form
List-I List-II of ATP.
A. Theory of Mutation 1. Beadk and Tatum 88. The major chemical compound found in human
B. Theory of Evolution 2. Jacob and Monod kidney stones is:
C. One gene one 3. Darwin (a) Uric acid
enzyme hypothesis (b) Calcium carbonate
D. Operon concept 4. Devries (c) Calcium oxalate
Codes: (d) Calcium sulphate
A B C D Ans. (c) : A kidney stone or renal calculus is also
known as nephrolith is a solid material formed in the
(a) 3 4 1 2
kidneys from the minerals found in the urine. While
(b) 4 3 1 2 most of the cases of kidney stones, this is caused due to
(c) 4 3 2 1 genetic reasons, environmental factors or a combination
(d) 3 4 2 1 of the two, factors such as obesity, intake of certain
foods, medications and lack of water intake also
Ans. (b) : The theory of mutation was formulated by
attribute to the formation of kidney stones. The major
Hugo de vries. Charles Darwin is credited with chemical compound found in human kidney stones is
proposing the theory of the evolution of organisms. The calcium oxalate or oxalate of lime. With the chemical
concept of one gene one enzyme was proposed by formula CaC2O4 or Ca (COO)2.
George Beadle and Edward Tatum. The operon concept 89. Consumption of fish is considered to be healthy
was proposed by Francois Jacob and Jacques Monod. when compared to flesh of other animals because
85. Which one of the following statements fish contains:
regarding starch and cellulose is NOT correct? (a) Polyunsaturated fatty acids
(a) Both of them are of plant origin (b) Saturated fatty acids
(c) Essential vitamins
(b) Both of them are polymers
(d) More carbohydrates and proteins
(c) Both of them give colour with iodine
Ans. (a) : Polyunsaturated fatty acid found in fish is an
(d) Both of them are made up of glucose anti oxidant, which helps in building of tissues.
molecules
90. Match List-I with List-II and select the correct
Ans. (c) : Starch is the reserve substance in plant cells answer using the codes given below the lists:
whereas cellulose is the most important structural List-I List-II
component of the cell wall of plants. Both starch and (Disease) (Organism)
cellulose are polymers of glucose. In contrast to starch A. Malaria 1. Fungi
and glycogen, cellulose is insoluble in ordinary solvents B. Poliomyelitis 2. Bacteria
and is not hydrolysed by boiling dilute acids. It gives no C. Tuberculosis 3. Virus
colour with iodine. D. Tingworm 4. Protozoane

IAS (Pre) GS 1998 Paper I 494 YCT


Codes: 93. Forest areas have been labelled as 1, 2, 3 and 4
A B C D in the rough outline map given:
(a) 4 3 2 1 Among these, those which were threatened in
(b) 4 2 3 1 1997 by a serious epidemic include:
(c) 3 4 2 1
(d) 3 4 1 2
Ans. (a) : Malaria is a mosquito-borne infectious
disease of humans and other animals caused by parasitic
protozoans belonging to the genus Plasmodium.
Poliomyelitis is a disease caused by infection with the
poliovirus. It is a highly infectious viral disease, which
mainly affects young children.
Tuberculosis, or TB, is an infectious bacterial disease is
caused by Mycobacterium tuberculosis, which most
commonly affects the lungs.
Ringworm is a common fungal infection that can cause
a red or silvery ring-like rash on the skin. Ringworm
commonly affects arms and legs. (a) Teak forests of 3 and 4
So, the correct answer is option A. (b) Oaks forests of 1 and sal forest of 2
91. Hemophilia is a genetic disorder which leads (c) sal forests of 3
to: (d) Sandalwood forest
(a) Decrease in hemoglobin level Ans. (c) : Sal forest of Madhya Pradesh indicates ‘3’ in
(b) Rheumatic heart disease the given India map. In 1997 Sal forest were badly
(c) Decrease in W.Bc. affected by the insects which cause destruction of Sal
(d) Non-clotting of blood forest by turning the trees dry.
Ans. (d) : Haemophilia is a bleeding disorder that slows 94. The discovery of Oak flora in 1966 added a
the blood clotting process. People with this condition new chapter to the history of Indian
experience prolonged bleeding or oozing following an Sericulture. Which one of the following states is
injury, surgery, or having a tooth pulled. In severe cases the leading producer of Oak tasar silk?
of haemophilia, continuous bleeding occurs after minor (a) Assam (b) Bihar
trauma or even in the absence of injury (spontaneous
bleeding). Serious complications can result from (c) Manipur (d) Orissa
bleeding into the joints, muscles, brain, or other internal Ans. (a) : The Oak Tasar silk is produced in Assam.
organs. Milder forms of haemophilia do not necessarily The Oak trees were discovered in Assam in 1966 which
involve spontaneous bleeding, and the condition may is used in Sericulture.
not become apparent until abnormal bleeding occurs 95. The water pollution in river is measured by the
following surgery or a serious injury.
dissolved amount of
ENVIRONMENT AND ECOLOGY (a) Chlorine (b) Ozone
(c) Nitrogen (d) Oxygen
92. A tree species in Mauritius failed to reproduce
Ans. (d) : Water pollution in river or any other water
because of the extinction a fruit-eating bird.
body is measured by amount of dissolved oxygen.
Which one of the following was that bird?
(a) Dove (b) Dodo 96. Estuaries possess district blooms of excessive
(c) Condor (d) Skuas growth of a pigmented dinofagellates. These
Ans. (b) : The dodo (Raphus cucullatus) is an extinct blooms are called
flightless bird that was endemic to the island (a) Red tides (b) Sea tides
of Mauritius, east of Madagascar in the Indian ocean. (c) Black tides (d) Sea flowers
It became extinct in 14th century from Mauritius
Ans. (a) : Red tides occurs when the population of
because of large-scale hunting. Dodo was helpful in
pollinating and propagating seed of this tree species. certain kinds of algae known as dinoflagellates
Subfossil remains show the dodo was about 1 metre explodes, creating what's called an "algal bloom."
(3 ft 3 in) tall and may have weighed 10.6–21.1 kg (23- Scientists sometimes refer to red tides as harmful algal
47 lb) in the wild. blooms.

IAS (Pre) GS 1998 Paper I 495 YCT


97. Consider the following climatic conditions 100. World Environment Conference to discuss
(northen hemisphere): global warming was held in 1997 in:
Months Temp. °C Rainfall Cm. (a) Stockholm (b) Rio de Janeiro
J 3.9 4.7
(c) Paris (d) Kyoto
F 4.4 5.7
M 8.3 8.2 Ans. (d) : World Environment Conference 1997
conference was held in Kyoto, Japan. The conference is
A 14.3 9.2
also known as "Rio+5" conferences on sustainable
M 20.0 9.2
development". It was a landmark conference sustainable
J 23.3 17.7
development for on mankind.
J 27.8 14.5
A 27.8 14.0 101. What is the correct sequence of the following
S 22.3 12.7 Indian states in descending order of their
O 18.3 7.0 length of surface roads per 100 km2 of their
N 12.2 5.0 area?
D 6.7 3.5 1. Haryana 2. Maharashtra
These are most likely to be found in the natural 3. Punjab 4. Tamil Nadu
regions of: Select the correct answer using the codes given
(a) China type (b) Equatorial type below:
(c) Hot desert type (d) Monsoon type
Codes:
Ans. (a) : This is most likely to be found in the natural
(a) 4, 3, 2, 1 (b) 4, 3, 1, 2
region of China type, which is chartacterised by a
warm, moist summer and a cool, dry winter. (c) 3, 4, 1, 2 (d) 3, 4, 2, 1
Temperatures strongly modified by maritime influence. Ans. (a) : Arrangement of descending order of the
surface roads per 100 sq. km of their area: Tamil Nadu
CURRENT AFFAIRS has 158.78 km, Punjab has 127.78 km, Maharashtra has
117.61 km, and Haryana has 63.70 km.
98. The satellites of which one of the following
countries have helped in the preparation of a 102. The damage to the Spektr Module of the
detailed and complete map of Antarctica? Russian Space Station Mir was due to:
(a) Canada (b) France (a) Collision with a soyuz cargo ship
(c) Russia (d) U.S.A. (b) Faulty material design of the Spektr Module
Ans. (a) : Antarctica is the seventh continent in the (c) Explosion inside the space station
world, devoid of population. It is called 'White (d) Collision with an asteroid
Continent' due to being covered with snow all around.
Ans. (a) : Due to collision with soyuz cargo ship, the
The continent of Antarctica was discovered in 1820
Spektr Module of the Russian space station Mir was
AD. The 'Queen Maud Range' divides this continent
into two parts. Antarctica at the South Pole is the center damaged.
of many scientific researches and studies. That is why 103. The meeting of G-15 countries held in
this continent is also called a continent dedicated to Malayasia in 1997 was attended by:
science. 'Dakshin Gangotri' and 'Maitri' are research (a) 14 member countries
centers in India located in Antarctica. A detailed map (b) 15 member countries
of the continent of Antarctica has been created through
(c) 16 member countries
images transmitted by a Canadian satellite.
(d) 17 member countries
99. Nobel Prize in Economics for the year 1997 was
awarded for contribution in the area of: Ans. (c) : The G-15 meet of 1997 was held 3-5th
(a) International Economics November, 1997 at Qualalmpur in Malaysia. Kenya was
(b) Financial Economics admitted as its 16th member but the organisation name
(c) Public Economics will remain G-15.
(d) Development Economics 104. Olympics 2004 is to be held in:
Ans. (b) : The Bank of Sweden Prize in Economic (a) Johannesburg (b) Capetown
Sciences in Memory of Alfred Nobel 1997 was awarded (c) Rome (d) Sydney
jointly to Robert C. Merton and Myron S. Scholes “for a Ans. (*) : The 2004 Olympic was held in Athens in
new method to determine the value of derivatives”.
Greece which is ancient Rome. (Hence none of the
Besides their valuation method, they have made
significant contributions to finance economics. given option is correct).

IAS (Pre) GS 1998 Paper I 496 YCT


105. Which one of the following was the venue for Codes:
the preliminary talks between the Sri Lankan (a) 1 and 2 (b) 2 and 3
Government and representative of Tamil (c) 1, 2 and 3 (d) 1, 2 and 4
United Liberation Front and other militant Ans. (c) : During STS-87, she was responsible for
groups? deploying the Spartan satellite which malfunctioned,
(a) New Delhi (b) Colombo necessitating a spacewalk by Winston Scott and Takao
(c) Thimpu (d) Madras Doi to capture the satellite. Other statements are correct.
Ans. (c) : The preliminary talks between the Sri Lankan 110. According to Meadows (1972), if the present
government liberation front and other militant group trends in world population, indusrialization,
was held at Thimpu, the capital of Bhutan. population, food production and resource
106. 'MERCOSUR' consists of group of countries depletion continue unchanged, the "Limits to
of: Growth" on our planet will he reached in the
(a) Africa (b) Asia next:
(c) Latin America (d) South East Asia (a) 50 years (b) 100 years
Ans. (c) : Mercosur is an economic and political block (c) 150 years (d) 200 years
comprising Argentina, Brazil, Paraguay, Uruguay, and Ans. (b) : In 1972 the book "Limits to Growth" was
Venezuela. published Its authors were Donella H. Meadows,
107. Indonesian forest fire in 1997 was caused by: Dennis L. Meadows, Jorgen Randers. Its sent out shock
waves around the world. It contained a warning that by
(a) Greenhouse effect
the year 2100 the world might be on a collision course
(b) Depletion of ozone layer with catastrophe if then current rates of growth in such
(c) El Nino effect areas as resource use industrial output food production
(d) None of the above and population expansion continued on their then
Ans. (d) : It's a game of blame, with everyone pointing current course This warning was the result of computer
their finger at someone else. WWF Indonesia modeling of a variety of future scenarios, based on
environmental group, which has been highlighting the different assumptions concerning the future state of the
problem of the recurring fires in Indonesia for years, world applied to the best data available regarding
says that the fires are caused by the collective various growth parameters.
negligence of businesses, 111. Human Poverty Index was introduced in the
Smallholders and government (which does not invest Human Development Report of the year:
adequately in preventive measures). Many blame big (a) 1994 (b) 1995
business. According to a recent report, big business is (c) 1996 (d) 1997
responsible.
Ans. (d) : The Human Poverty Index (HPI) was an
108. Bharat Ratna was awarded 1997 to: indication of the standard of living in a country,
(a) Dr. Homi Bhabha (Posthumous) developed by the United Nations (UN) to complement
(b) Former Presidnet R. Venkataraman the Human Development Index (HDI) and was first
(c) Satyajit Ray (Posthumous) reported as part of the Human Development Report in
(d) Dr. Abdul Kalam 1997. It was considered to better reflect the extent of
Ans. (d) : Abdul Kalam was the recipient of Bharat deprivation in developed countries compared to the
Ratna 1997. HDI. In 2010 it was supplanted by the UN's
109. Which of the following statements are true Multidimensional Poverty Index. The HPI concentrates
about the Indian-born woman astronaut on the deprivation in the three essential elements of
Kalpana Chawla? human life already reflected in the HDI: longevity,
1. She was born in Karnal. knowledge and a decent standard of living.
2. She flew on board the shuttle flight STS-87 112. The recent Land Mines Conference to sign the
3. She was trained at the Kennedy Space historic treaty was held in the Capital city of:
Centre. (a) Canada (b) Japan
4. She did a space-walk to retrieve the Spartan (c) Sweden (d) Zimbabwe
Satellite. Ans. (a) : The historic treaty on land mines was signed on
Select the correct answer using the codes given 3rd to 5th December at the conference held in Ottawa, the
below. capital of Canada. The Ottawa treaty banned landmines.

IAS (Pre) GS 1998 Paper I 497 YCT


113. Which of the following pairs are correctly Directions:
matched? The following fourteen items consists of two
1. Dow Jones : New York statements, one labelled as 'Assertion A' and
2. Hang Seng : Seoul the other labelled as 'Reason R'. Your are to
3. FTSE-100 : London
examine these two statements carefully and
Select the correct answer using the codes given
decide if the Assertion A and the Reasons R are
below:
individually true and if so, whether the Reason
Codes:
(a) 1, 2 and 3 (b) 2 and 3 is a correct explanation of the Assertion. Select
(c) 1 and 2 (d) 1 and 3 your answers to these items using the codes
given below and mark your answer sheet
Ans. (d) : Hang Seng is the stock market of Hong Kong
not Seoul. Rest 1 and 3 are correctly matched. accordingly.
114. The Economist who was associated with the Codes:
W.T.O. draft document is: (a) Both A and R are true and R is the correct
(a) A. K. Sen (b) T. N. Srinivasan explanation of A
(c) J. N. Bhagwati (d) Avinash Dixit (b) Both A and R are true but R is NOT a correct
Ans. (c) : J.N. Bhagwati is the Economist who is explanation of A
associated with the drafting of W.T.O. He is an (c) A is true but R is false
American economist who has an immense role to play
(d) A is false but R is true
in the reshaping of the world economy.
118. Assertion (A) : The Gandhara School of Art
115. The number of economically active women
bears the mark of Hellenistic influence.
(excluding students and those doing domestic
duties in their homes) as a percentage of all Reason (R) : Hinayana form was influenced by
women of working age (generally those aged that art.
15-64 years) was the highest in 1996 in: Ans. (c) : Gandhara School of Art was highly
(a) U.S.A. (b) China influenced by Greeks. But, it was Mahayana form of
(c) Russia (d) S. Korea Buddhism that was influenced by that art.
Ans. (b) : In 1996 it was 43% in China. Hence when the 119. Assertion (A) : Formic acid is a stronger acid
question was asked the correct answer was (b) China. than Acetic acid.
Reason (R) : Formic acid is an organic acid.
MISCELLANEOUS Ans. (b) : Formic acid is stronger than acetic acid due
116. What are the official languages of the U.N.O.? to its chemical composition. Formic acid is an organic
(a) English, French and Russian acid as well as Acetic acid also an organic acid.
(b) English, French, German and Russian 120. Assertion (A) : At first the Turkish
(c) English, French, Russian, Chinese and Hindi administration in India was essentially
(d) English, French, Chinese, Russian, Arabic military.
and Spanish Reason (R) : The country was parcelled out as
Ans. (d) : English, French, Chinese, Russian, Arabic 'Iqtas' among leading military leaders.
and Spanish are the official language of U.N.O.
Ans. (a) : Turkish administration in India was
117. Which of the following parties were not a part essentially military when the conquered the country,
of the United Front which was in power during they divided it into a number of tracts called Iqtas which
'96-97'? were paracelled out among the leading Turkish nobles.
1. Bahujana Samaj Party Hence both (A) and (R) are true and (R) is correct.
2. Samata Party explanation of (A).
3. Haryana Vikas Party 121. Assertion (A) : According to Ashoka's edicts
4. Asam Gana Parishad
social harmony among the people was more
Select the correct answer using the codes given
important than religious devotion.
below:
Codes: Reason (R) : He spread ideas of equity instead
(a) 1, 2, 3 and 4 (b) 1, 2 and 3 of promotion of religion.
(c) 3 and 4 (d) 1 and 2 Ans. (a) : By spreading ideas of equity instead of
Ans. (b) : Bahujan Samaj Party, Haryana Vikas party promoting religion, Ashoka's edicts stress social
and Samta Party were not the part of the United Font. harmony among people.

IAS (Pre) GS 1998 Paper I 498 YCT


122. Assertion (A) : The temperature of a metal Ans. (b) : Main reasons for the failure of the Cripps
wire rises when an electric current is passed mission of 1942 was - (1) Gandhi's opposition led the
through it. Indian National Congress to reject the British offer. (2)
Cripps modification of the original British offer, which
Reason (R) : Collision of metal atoms with each provided for no real transfer of power. (3) The behind-
other releases heat energy. the-scenes efforts of the Viceroy and Secretary of State
Ans. (c) : The temperature of a metal wire rises when for India to sabotage the mission.
an electric current is passed through it Electricity is the 128. Assertion (A) : The United States of America
movement of electrons, when the electrons passes has threatened to ask the World Trade
through the wire they collide with the atoms of the wire. Organisation (W.T.O.) to apply sanctions
During such collisions electrons transfer some of their against the developing countries for the non-
kinetic energy to the atoms of the wire therefore the observances of I.L.O. conventions.
temperature of the wire increases. Hence 'A' is true but Reason (R) : The United States of America
'R' is false. (U.S.A.) itself has adopted and implemented
those I.L.O. conventions.
123. Assertion (A) : Phenyl is used as a household
Ans. (c) : International labour law is not much known
germicide.
in USA. It works in its own context. Hence R is worng
Reason (R) : Phenyl is a phenol derivative and
129. Assertion (A) : During the reign of Shahjahan,
phenol is an effective germicide. Dara Sikoh was sent on expedition to Balkha,
Ans. (a) : Phenyl being a phenol derivative have Badakhshan and Qandahar.
effective germicidal properties because phenol is Reason (R) : The expedition sent by Shahjahan
germicidal in form of strong solutions. That’s why to the Middle-East was a marvellous success.
phenyl is used as a household germicide. Ans. (c) : Shah Jahan sent Dara Shikoh, with a large
124. Assertion (A) : partition of Bengal in 1905 army and two of the heaviest artillery pieces of the
brought to an end the Moderates' role in the Mughal at that time, on expedition of Balkha,
Badakhshan and Kandhar. But that expedition was a
Indian freedom Movement.
failure. In fact it was shah Jahan’s rule when Mughal
Reason (R) : The Surat session of Indian lost Kandhar forever. Hence Assertion A is correct but
National Congress separated the extremists Reason R is false.
from the Moderates. 130. Assertion (A) : Gandhi stopped the non-co-
Ans. (d) : Moderates continued to participate in the operation movement in 1922.
struggle for freedom even after 1905 Bengal Partition. Reason (R) : Violence at Chauri-Chaura led
Later in 1916 both the groups re-united in 1916 at him to stop the movement.
Lahore session. Ans. (a) : The Chauri-Chaura incident took place on
125. Assertion (A) : The first ever bill to make February 4, 1922 when 22 policemen trapped inside
primary education compulsory in India was their station during the protest in a town called Chauri-
Chaura, were burnt to death Hearing of this incident,
rejected in 1911. Gandhi called a meeting of Congress Working
Reason (R) : Discontent would have increased Committee (CWC) on February 12, 1922 (popularly
if every cultivator could read. known as Bardoli Resolution) and declared the
Ans. (d) : The British rejected the first ever bill in 1993 withdrawal of Non-Cooperation Movement. However
to make primary education compulsory in India because CR Das and Motilal Nehru strongly opposed the
suspension of this movement.
they feared it would spread discontent all over the
131. Match List-I with List-II and select the correct
country.
answer using the codes given below the lists:
126. Assertion (A) : Sodium metal is stored under List-I List-II
Kerosene. A. Ringgit 1. Indonesia
Reason (R) : Metallic sodium metls when B. Baht 2. South Korea
exposed to air.
C. Rupiah 3. Thailand
Ans. (a) : Sodium metal is stored under kerosene D. Won 4. Malaysia
because it is highly explosive and catches fire when Codes:
comes in contact with O2 . A B C D
127. Assertion (A) : The congress rejected the (a) 1 3 4 2
Cripps proposals. (b) 4 3 1 2
Reason (R) : The Cripps Mission consisted only (c) 1 2 4 3
of whites. (d) 4 2 1 3
IAS (Pre) GS 1998 Paper I 499 YCT
Ans. (b) : 1. Ringgit = Malaysia
2. Baht = Thailand
3. Rupiah = Indonesia
4. Won = South Korea
Hence, option (c) will be correct answer.
132. There are 50 students admitted to a nursery
135. Examine the following statements:
class. Some students can speak only English
1. All members of Mohan's family are honest.
and some can speak only Hindi. 10 students can
2. Some members of Mohan's family are not
speak both English and Hindi. If the number of
employed.
students who can speak English is 21, then how
many students can speak Hindi, how many can 3. Some employed persons are not honest.
speak only Hindi and how many can speak only 4. Some honest persons are not employed.
English? Which one of the following inferences can be
(a) 21, 11 and 29 respectively drawn from the above statements?
(b) 28, 18 and 22 respectively (a) All members of Mohan's family are employed
(c) 37, 27 and 13 respectively (b) The employed member of Mohan's family are
(d) 39, 29 and 11 respectively honest
(c) The Honest members of Mohan's family are
Ans. (d) : Number of students who speak only English =
(Number of students who speak English – Number of employed
students who speak both Hindi and English) =21 –10=11 (d) The employed member of Mohan's family are
Number of students who speak Hindi = (Total no of not honest
students – No of students who speak only English) = 50 Ans. (b) : According to question following diagram can
– 11 = 39 be made
∴ Number of students who speak only Hindi = (Number Honest
of Hindi speaking students – no of students who speak M’F
both languages) = 39 – 10 = 29. Mohan’s Family
Employed
Employed members
133. An accurate clock shows the times as 3.00. After of Mohan’s family
hour hand has moved 135°, the time would be:
(a) 7.30 (b) 6.30 Therefore option (b) is correct answer to the question.
(c) 8.00 (d) 9.30 136. Match List-I with List-II and select the correct
Ans. (a) : Hour hand covers an angle of 360° in 12 answer using the codes given below the lists:
hours. ∴Time taken to cover an angle of 135° A. Graphite 1. Bellary
135°×12 B. Lead 2. Didwana
= = 4.5hr.
360° C. Salt 3. Rampa
∴ Required time = 3 + 4.5 = 7.5 = 7:30 D. Silver 4. Zawar
134. Codes:
A B C D
(a) 3 4 1 2
(b) 1 4 2 3
(c) 3 1 4 2
In the above set of figures (I) to (IV), some
parts are shown to change their position in a (d) 2 3 1 4
regular direction. Following the same sequence, Ans. (b) : Correct match is :
which one of the following will appear at the List-I List-II
fifth stage? (Minerals) (Mining areas)
A. Graphite 1. Bellary
B. Lead 2. Zawar
C. Salt Didwana
3.
(a) (b) (c) (d)
D. Silver 4. Rampa
Ans. (c) : In each alternative figure, one extra side of
137. In a dinner party both fish and meat were
square extended in anticlockwise direction comes out
and in remaining figure, one inside circle comes out. served. Some took only fish and some only
IAS (Pre) GS 1998 Paper I 500 YCT
meat. There were some vegetarians who did not
accept either. The rest accepted both fish and
meat.
Which one of the following logic diagrams
correctly reflects the above situation?

(a)
(a) 0.75 : 1 (b) 1 : 1
(c) 1 : 0.75 (d) 1.25 : 1
(b) Ans. (b) :

(c)

(d)
Let LR = R
2πR
Perimeter of LMNOP = = πR
Ans. (a) : 2
Perimeter of LSRQP = Perimeter of LSR +
Perimeter of RQP
= πR + πR = πR
2 2

∴ Perimeter  LMNOP  = πR = 1 : 1
 LSRQP  πR
138. A, B, C, D, E, F and G are members of a family 140. A man purchases two clocks A and B at a total
consisting of 4 adults and 3 children, two of cost of Rs. 650. He sells A with 20% profit and B
whom, F and G are girls. A and D are brothers at a loss of 25% and gets the same selling price
and A is a doctor. E is an engineer married to for both the clocks. What are the purchasing
one of the brothers and has two children. B is prices of A and B respectively?
married to D and G is their child. Who is C? (a) Rs. 225; Rs. 425 (b) Rs. 250; Rs. 400
(c) Rs. 275; Rs. 375 (d) Rs. 300; Rs. 350
(a) G's brother (b) F's father
(c) E's father (d) A's son Ans. (b) : Let the cost price of A be x
and that of B be y.
Ans. (d) : E is married to A or D, but B is married to D,
so E is married to A. ∴ x + y = 650 (Total cost) ……..(i)
Now ABED are four adult and C, F, G are three For A → Profit = 20%
children B and D have child G SP − CP
∴ 20 = ×100
A and E have two children they are C & F now only F CP
and G are girls, so C is a boy. 20 SP − x SP 1
Hence C is A's and E's son. = ⇒ =
100 x x 5
139. LMNOP, is a semicircle with center at R and 6x
diameter LP, LSR and RQP are also semicircles = SP of A =
5
with centres at T and U and diameters LR = RP =
1/2 LP. The ratio of perimeter of LMNOP and For B → Loss = 25%
LSRQP is: CP − SP
∴ 25 = ×100
CP

⇒ 1 = y − SP
4 y
3y
= SP of B =
4
IAS (Pre) GS 1998 Paper I 501 YCT
As SP of A = SP of B (Given) overall average of 60%, how many marks does
6 x 3y the student need to secure in the third annual
= examination?
5 4
(a) 450 (b) 400
6 x 3y
⇒ − =0 (c) 350 (d) 300
5 4
Ans. (b) : Total marks for three examinations = 3 ×
⇒ 8x – 5y = 0 .……..(ii)
500 = 1500
By multiplying equn. (1) with 5 we get Total required marks in three examinations = 60% of
5x + 5y = 3250 ………(iii) 1500 = ( 3 × 500 × 60 )/100 = 900
Adding equn. (ii) & (iii) Marks secured in first examination = 45 % of 500
13x = 3250 = ( 500 × 45 ) / 100
x = 250 =225
So, Marks secured in third examination = 55 % of 500
250 + y = 650 ⇒ y = 650 – 250 = 400 = ( 500 × 55 ) / 100 =275
141. A fuse is used in mains electric supply as a Thus, the required marks in third examination
safety device. Which one of the following =900 – ( 225 + 275 ) = 900 – 500
statements about the fuse is correct? = 400
(a) It is connected in parallel with the main 144. Which one of the following satisfies the
relationship Dda : aDD : : Rrb : ?
switch
(a) DDA (b) RRR
(b) It is made mainly from siver alloys (c) bRR (d) BBR
(c) It must have a low melting point Ans. (c) : First and third letters have been interchanged.
(d) It must have a very high resistance Middle letter becomes capital.
Ans. (c) : As fuse is a safety device used in domestic 145. A square pond has 2 m sides and is 1 m deep. If
electric circuit to prevent the house from short circuit. it is to be enlarged the depth remaining the
same, into a circular pond with the diagonal of
The fuse wire is made up of the metal having low the square as diameter as shown in the figure,
melting point. then what would be the volume of earth to be
142. If 15 pumps of equal capacity can fill a tank in removed?
7 days, then how many extra pumps will be
required to fill the tank in 5 days?
(a) 6 (b) 7
(c) 14 (d) 21
Ans. (a) : No. of pumps required to fill a tank in 7 days
= 15 (a) ( 2π − 4 ) m3 (b) ( 4π − 4 ) m3
∴ No. of pumps required to fill a tank in 1 day = 15 × 7
(c) ( 4π − 2 ) m3 (d) ( 2π − 2 ) m3
= 105 ........(i)
Let the extra pumps required be n to till the tant in 5 Ans. (a) :
days.
Now, no of pumps required to fill the tank in 1 day
= (n + 15) 5 ........(ii)
From (i) and (ii), (n + 15) 5 = 105
n + 15 = 21
n=6
143. Out of the three annual examinations, each
with a total of 500 mark, a student secured Diagonal of the square = ( 2 ) + ( 2 ) = 2 2
2 2

average marks of 45% and 55% in the first and


second annual examinations. To have an Radius of the circle = 2 2 = 2
2
IAS (Pre) GS 1998 Paper I 502 YCT
∴ Volume to be removed
= (Volume of the circle – Volume of the square) Ans. (a) :

{
= π
 ( 2 ) ×1 − ( 2 × 2 ×1)}
2

= 2π – 4
146. One local and another express train were Similarly
proceeding in the same direction on parallel
tracks at 29 km/hour and 65 km/hour
respectively. The driver of the former noticed
that it took exactly 16 seconds for the faster train
to pass by him. What is the length of the faster
train?
(a) 60 m (b) 120 m
149. In a family, a couple has a son and a daughter.
(c) 160 m (d) 240 m
The age of the father is three times that of his
Ans. (c) : Relative speed of express train to local train = daughter and the age of the son is half of this
65 – 29 = 36 km/h mother. The wife is nine years younger to their
= 36 × (5/18) m/s = 10 m/s husband and the brother is seven years older
∴ Length of faster train = 10 × 16 = 160 m than his sister. What is the age of the mother?
147. A, B, C, D, E and F, not necessarily in that order (a) 40 years (b) 45 years
are sitting on six chairs regularly placed around a (c) 50 years (d) 60 years
round table. It is observed that A is between D Ans. (d) : Let the daughter’s age be x
and F, C is opposite D, and D and E are not on Then father age = 3× x = 3x
neighbouring chairs. Which one of the following
Mothers age = 3x – 9
pairs must be sitting on neighbouring chairs.
Son’s age = x + 7
(a) A and B (b) C and E
(c) B and F (d) A and C So x + 7 = 3 x − 9 ⇒ 2 x +14 = 3 x − 9
2
Ans. (b) : Based on the information given in the ⇒ x = 23
question following sitting arrangement can be made–
So mothers age = 23 × 3 – 9 = 60 years
150. The missing fraction in the series given below
is:
4 9 39
, , .....
9 20 86
17 19
(a) (b)
40 42
20 29
So C and E are sitting on the neighbouring chairs. (c) (d)
45 53
148. If in a certain code SAND is VDQG and BIRD
Ans. (b) : The pattern in the series is:
is ELUG, then what is the code for MOVE?
4× 2 +1 9
(a) PRYH =
9× 2 +1 20
(b) ORTG
9× 2 +1 19
(c) NPUH =
20× 2 + 2 42
(d) ORYH
19× 2 +1 39
=
42× 2 +1 86
19
Hence the missing fraction would be
42

IAS (Pre) GS 1998 Paper I 503 YCT


UNION PUBLIC SERVICE COMMISSION
Civil Services (Preliminary Exam) - 1997
GENERAL STUDIES : PAPER-I
Time: 2 hours Maximum Number: 200

Ans. (d) : Pampa was a Kannada poet whose works


HISTORYHH reflected his philosophical beliefs. Vikramarjuna
Vijaya, also known as Pampa Bharata, is a Kannada
1. Match List-I with List-II and select the correct version of the Mahabharata of Vyasa.
answer: 4. Which one of the following scripts of ancient
List-I List-II India was written from right to left?
A. Gupta 1. Badami (a) Brahmi (b) Nandagari
B. Chandella 2. Panamalai (c) Sharada (d) Kharoshti
C. Chalukya 3. Khajuraho Ans. (d) : Most of the inscriptions of Ashoka in ancient
D. Pallava 4. Deogarh India are in Brahmi script. This script was written from
Codes: left to right. Some of his inscriptions have also been
A B C D found in Kharoshthi script which was written from right
to left.
(a) 4 3 1 2
5. The famous dialogue between Nachiketa and
(b) 4 2 3 1
Yama is mentioned in the:
(c) 2 3 4 1
(a) Chhandogyopanishad
(d) 3 4 1 2
(b) Mundakopanishad
Ans. (a) : The Gupta had their capital at Deogarh,
Chandella had their capital at Khajuraho, Chalukya’s at (c) Kathopanishad
Badami and Pallava’s capital was at Panamalai. (d) Kenopanishad
2. Which one of the following statements regarding Ans. (c) : Kathopanishad is the conversation between
Ashoka stone pillars is incorrect? Nachiketa and Yamraj (God of death). Nachiketa was a
seeker, and Yamraj was a Guru (knower of secrets). The
(a) These are highly polished
Katha Upanishad is one of the mukhya Upanishads,
(b) These are monolithic embedded in the last short eight sections of the Kaṭha
(c) The shaft of pillars is tapering in shape school of the Krishna Yajurveda. It is also known as
(d) These are parts of architectural structures Kāṭhaka Upanishad, and is listed as number 3 in the
Ans. (d) : The number of these pillar articles is seven, Muktika canon of 108 Upanishads
which have been found in different places in the form of 6. Milindapanha is in the form of a dialogue
independent pillars. These stone pillars are well between the king Menander and the Buddhist
polished. Delhi-Topra is important in these pillar monk:
articles, initially it was buried in Saharanpur district of
(a) Nagasena
Uttar Pradesh. Brought to Delhi by Firoz Shah Tughlaq
in the Middle Ages. Seven inscriptions of Ashoka are (b) Nagarjuna
engraved on it. Other pillar inscriptions of Ashoka are (c) Nagabhatta
Delhi, Meerut, Lauria-Arraj, Lauria Nandangarh, (d) Kumarilabhatta
Rampurwa, Prayag. The stone pillars on which the state Ans. (a) : Milindpanha is a famous Buddhist scripture
proclamations of Ashoka are engraved are generally written in Pali language, in which the question-answer
called small pillar inscriptions. between the Indo-Greek king Menander or Milanda and
3. Which one of the following pairs of composers the Buddhist teacher Nagasena is compiled. It was
in different languages and their works on the
composed in the first century AD.
Mahabharata theme is correctly matched?
(a) Sarladasa – Bengali 7. Which one of the following edicts mentions the
(b) Kasirama – Oriya personal name of Asoka?
(c) Tikkana – Marathi (a) Kalsi (b) Rummindei
(d) Pampa – Kannada (c) Special Kalinga Edict(d) Maski

IAS (Pre) GS 1997 Paper I 504 YCT


Ans. (d) : The history of Ashoka is known to us mainly 11. Match List-I with List-II and select the correct
from his inscriptions. More than 40 records have been answer:
received so far. For the first time the name Ashoka is List-I List-II
found in the Maski inscription. This inscription is
(Author) (Text)
located in Raichur district of Karnataka state. Its name
is also found in Gurjara inscriptions.The Kalsi A. Varahamihira 1. Prabandha-
inscription is located in Dehradun in the state of Chintamani
Uttarakhand. This inscription is in Brahmi script. It was B. Visakhadatta 2. Mrichchha-Katikam
discovered in 1837. The Ashokan inscription was first C. Sudraka 3. Brihat-Samhita
read by James Princep in 1837. The Rummindei short D. Bilhana 4. Devi-Chandraguptam
column inscription was discovered in 1896 by Fehrer in 5. Vikramankadeva-
the Terai of Nepal. This small inscription shows that
charita
Ashoka visited here in the 20th year of his rule and
abolished the tax called Bali and reduced the land tax Codes:
from 1/6 to 1/8. The Maski inscription was discovered A B C D
by Borden in 1915. (a) 3 4 5 2
8. The following map shows four of the sixteen (b) 3 4 2 5
mahajanapadas that existed in ancient India: (c) 5 3 4 1
(d) 1 3 5 2
Ans. (b) : The correct match is mentioned as below:
Varahamihira – Brihat-Samhita
Visakhadatta – Devi-Chandraguptam
Sudraka – Mrichhkatikam
Bilhana – Vikramankadeva
Charita
The places marked A, B, C and D respectively 12. Which one of the following was a corporation
are of merchants in ancient India?
(a) Matsya, Cedi, Kosla, Anga (a) Chaturvedimanagalam
(b) Surasena, Avanti, Vastsa, Magadha (b) Parishad
(c) Matsya, Avanti, Vatsa, Anga (c) Ashtadikgaja
(d) Surasena, Cedi, Kosala, Magadha (d) Manigrama
Ans. (c) : Matsya, Avanti, Vatsa, Anga Ans. (d) : Manigramam refers to a medieval merchant
guild, organized by itinerant ethnic Indian traders,
9. In the Mahayana Buddhism, the Bodhisatya primarly active in the south India. The manigramam
Avalokitesvar was also known as: played a major role in the commercial activities of the
(a) Vajrapani (b) Manusri region. The manigramam is found both in the port-
(c) Padmapani (d) Maitreya towns and in the hinter land trade centres.
Ans. (c) : Padmapani is another name in Sanskrit for
Bhodisattva Avalokiteswara, who represents the MEDIEVAL HISTORYHH
compassion. The glit bronze image of the Bhodisattva
Padmapani [Lotus bearer] shows him as a graceful 13. After consolidating his power, Balban assumed
princely figure, endowed with special attributes the grand title of:
symbolizing his power and functions. He is described as (a) Tute-i-Hind (b) Kaisr-i-Hind
the embodiment of the pure, impartial and limitless (c) Zil-i-llahi (d) Din-i-IIahi
compassion. Ans. (c) : Ghiyasuddin Balban of the Balbani dynasty
10. The silver coins issued by the Guptas were abolished the group of forty Turkish chieftains
called: established by Iltutmish. Balban reorganized a new
(a) Rupaka (b) Karshapana military department 'Diwan-i-Arz', he followed a strict
(c) Dinara (d) Pana 'iron and blood' policy towards his opponents. Balban
Ans. (a) : The coin issued by the Guptas were mainly of was the first Sultan of the Delhi Sultanate who
silver and gold. explained his kingship theory in detail. After
Silver coin was called Rupaka based on Sakas of Ujjain consolidating his power, Balban assumed the title of 'Zil
weighing 32-46 grans. 6 Gold coins are known as -i-Ilahi’. Amir Khusro was the court poet during the
Dinnars. time of Alauddin Khilji. Amir Khusrau was the best

IAS (Pre) GS 1997 Paper I 505 YCT


Indian poet of Persian language. He was born in Ans. (d) :Abul Fazl's was court poet. Who wrote
Patiyali village of Etah (Uttar Pradesh). He was a important texts like Akbarnama and Ain-e-Akbari. It
disciple of Nizamuddin Auliya. He was the first Indian was also included in the nine gems of Akbar's court. He
writer to use Hindi words and idioms. Amir Khusrau was the chief advisor and secretary of Akbar.He had
was known as Tuti-e-Hind (Parrot of India). ample knowledge of history, philosophy and literature.
Abul Fazl was the head priest of Din-i-Ilahi. Among Abul Fazl was the chief priest of Akbar's Din-i-Ilahi
the important Hindu mansabdars, Birbal accepted this religion. He has mentioned the discovery of America in
religion. his works. In 1602, on the instructions of Jahangir,
Abul Fazl, who was coming from the south to Agra,
The British Government had honoured Mahatma
was assassinated by a Bundela chieftain. Amir Khusro
Gandhi with the title 'Kaiser-e-Hind'. It was returned by was the best Indian poet of Persian language. He was
Gandhiji in response to Jalliwalabagh massacre. born in Etah (Uttar Pradesh). Amir Khusrau's childhood
14. The head of the military department under the name was Abul Hasan. He was a disciple of
recognised central machinery of administration Nizamuddin Auliya. It was patronized by the Sultans
during Akbar's reign was from Balban to the time of Ghiyasuddin Tughlaq. In
(a) Diwan (b) Mir Bakshi the field of music, he invented the sitar by combining
the Iranian tambura and the Indian veena. Raskhan, a
(c) Mir Saman (d) Bakshi
contemporary of Vitthalnath, was a Muslim poet.He
Ans. (b) : The head of the military department under was engrossed in devotion to Krishna. His two main
the reorganized central machinery of administration works on Krishna are Premvatika, Sujan Raskhan.
during Akbar’s reign was Mir Bakshi, appointed from Malik Muhammad Jayasi is the representative poet of
among the leading nobles of the court. The Mir the Premashrayi branch of Bhaktikal. Sheikh Mehndi
Bakshi was in charge of intelligence gathering, and also and Mubarak Shah wereinitiation gurus. Their main
made recommendations to the emperor for military works are - Padmavat, Akhravat, Aakhri Kalam etc.
appointments and promotions.
15. Given below are two statements, one labelled as MODERN HISTORYHH
Assertion (A) and the other labelled as Reason
17. Match the List-I with List-II ad select the
(R) :
correct answer:
Assertion (A) : The sponsor and the most List-I List-II
prominent figure of the Chishti order of Sufis A. Butler Committee 1. Jallianwala Bagh
in India is Khwaja Moinuddin Chishti. Report massacre
Reason (R) : The Chishti order takes its name B. Hartog 2. Relationship between
from a village Chishti in Ajmer. Committee Report the Indian states and
In the context of the above two statements, paramount power
which one of the following is correct? C. Hunter Inquiry 3. Working of Dyarchy as
(a) Both A and R are true and R is the correct Committee Report laid down in the
explanation of A Montagu-Chelmsford
Reforms
(b) Both A and R are true but R is NOT a correct
D. Muddiman 4. The growth of
explanation of A
Committee Report education in British
(c) A is true but R is false
India ad potentialities
(d) A is false but R is true
of its further progress
Ans. (c) : The Chishti order is a Sufi order within the
mystic branches of Islam which was founded in Chishti, Codes:
a small town near Herat, Afghanistan about 930 CE. A B C D
The order was founded by Abu Ishaq Shami (“the (a) 3 2 1 4
Syrian”). The most famous of the Chishti saints is (b) 1 4 2 3
Moinuddin Chishti popularly known as Gharib Nawaz (c) 2 1 3 4
meaning “Benefactor of the Poor” who settled in Ajmer, (d) 2 4 1 3
India. Ans. (d) : During the tenure of Lord Irwin (1926-31
16. The Medieval Indian writer who refers to the AD), the Butler Committee (Harcourt Butler) was
discovery of America is: formed in 1927 to examine the relations between the
(a) Malik Muhammand Jayasi native states and the British Indian government and
(b) Amir Khusrau suggest ways to establish peaceful relations between
(c) Raskhan them. In 1929, during the tenure of Lord Irwin (1926-31
AD), the Indian Statutory Commission (Philip Hartog)
(d) Abul Fazl
IAS (Pre) GS 1997 Paper I 506 YCT
was formed to give its report on the state of education Codes:
and make suggestions for its development. During the A B C D
tenure of Lord Chelmsford (1916-21), the Hunter (a) 3 4 5 2
Commission was formed to investigate the Jallianwala (b) 4 1 2 3
Bagh massacre of April 13, 1919. It was during the (c) 2 3 4 5
tenure of Lord Chelmsford that in 1919, the Muddiman
(d) 3 1 2 5
Committee was formed for the working of the diarchy
Ans. (d) : The Morley Minto Reforms (Indian Councils
system laid down in the Montagu-Chelmsford Reforms.
18. "A graduate at 18, professor and associate editor Act-1909) provided for the appointment of an Indian
of the Sudharkarak at 20, Secretary of the member to the Executive Council of the Governor-
Sarvajanik Sabha and of the Provincial General. In this act, the members were given the right to
Conference at 25, Secretary of the National move and question and the right to vote on budget
Congress at 29, leading witness before an proposals. In this a heinous act, the division of
important Royal Commission at 31, Provincial
legislator at 34, Imperial legislator at 36, President constituencies was done on the basis of caste and
of the Indian National Congress at 39 _____ a religion. Some seats were reserved for Muslims. A
patriot whom Mahatma Gandhi himself regarded commission was announced on 8 November 1927 under
as his master". the chairmanship of Sir John Simon. All its members
This is how a biographer describes: were Englishmen. This commission was to recommend
(a) Pandit Madan Mohan Malaviya whether the people of India should be given more
(b) Mahadev Govind Ranade
constitutional rights. This commission reached India
(c) Gopal Krishna Gokhale
(Mumbai) on 3 February 1928. All the parties opposed
(d) Bal Gangadhar Tilak
this commission. Jinnah supported it. On February 5,
Ans. (c) : The above description is about Gopal Krishna
Gokhale, who was born in Kolhapur in 1866. He graduated 1922, a procession of Congress and Khilafat took out in
from Elphinstone College in 1884. In 1886, he was Chauri-Chaura, some policemen misbehaved with them,
appointed as Professor of Economics and History at the as a result, a group involved in the procession attacked
Ferguson College, Poona. In 1902, he was appointed as a the police and set fire to the police station, killing 22
member of the Imperial Legislative Council. policemen. On receiving this information, Gandhiji
19. Who among the following was associated with suspended the non-cooperation movement on February
suppression of Thugs? 12, 1922. On March 12, 1930, Gandhiji, along with 78
(a) General Heny Prendergast of his associates from the Sabarmati Ashram, left for the
(b) Captain Sleeman Dandi March from the Ahmedabad headquarters. On 6
(c) Alexander Burnes April 1930, Gandhiji started the Civil Disobedience
(d) Captain Robert Pemberton Movement after reaching Dandi with a handful of salt in
Ans. (b) :Colonel Sleeman did the work of suppression his hand.
of thugs on the orders of Lord William Bentick. As a
21. The Poona Pact which was signed between the
result of his efforts and some political efforts, this
British Government and Mahatma Gandhi in
problem almost ended in 1837
1932 provided for:
20. Match List-I with List-II and select the correct
answer: (a) The creation of dominion status for India
List-I List-II (b) Separate electorates for the Muslims
(Events) (Results) (c) Separate electorate for the Harijans
A. Morley Minto 1. Country-wise (d) Joint electorate with reservation for Harijans
Reforms agitation Ans. (d) : - In the Poona Pact of 26 September 1932,
B. Simon 2. Withdrawal of Dr. Ambedkar withdrew the demand for separate
Commission Movement representation of Harijans and the principle of joint
C. The Chauri- 3. Communal electorate was accepted. The seats reserved for
Chaura Incident Electorates depressed classes were increased from 71 to 141 in
D. The Dandi 4. Communal provincial legislature and 10% of total in the central
March outbreaks legislature.
5. Illegal manufacture Poona Pact was excepted by government as an
of salt amendment to communal award.
IAS (Pre) GS 1997 Paper I 507 YCT
22. Match List-I with List-II and select the correct December 17, 1928 AD. Assistant Superintendent of
answer: Police Saunders was assassinated by Bhagat Singh,
List-I List-II Chandrashekhar Azad and Rajguru at Lahore Railway
A. Surat Split 1. 1929 Station. Under this, Jatindas was caught and put in jail.
During the hunger strike in jail on September 13, 1929
B. Communal Award 2. 1928
AD. He died. Ghadar Party was founded in 1913 AD.
C. All party Convention 3. 1932
by Lala Hardayal in San Francisco (USA).
D. Poorna Swaraj Resolution 4. 1907
24. M.C. Setalvad, B.N. Rao and Alladi Krishnaswamy
5. 1905 Iyer were distinguished members of the:
Codes: (a) Swaraj Party
A B C D (b) All India National Liberal Federation
(a) 4 3 1 5 (c) Madras Labour Union
(b) 4 3 2 1 (d) Servants of Indian Society
(c) 2 5 4 1 Ans. (d) : 1905 AD. In Poona, Gopal Krishna Gokhale
(d) 1 4 2 3 founded the Bharat Sevak Mandal (Servants of India
Ans. (b) :Surat split in 1907 session of the INC was Society), whose purpose was to prepare campaigners for
presided by Ras Behari Bose. the service of India and to promote the true interests of the
Communal Award was announced by Ramsay Indian people in a constitutional manner. M.C. Shetalvad,
B. N. Rao and Alladi Krishnaswamy Iyer were eminent
McDonald in August, 1932.
members of this organization.
Poorna Swaraj resolution was passed in 1929
25. What is the correct chronological sequence of
session of INC at Lahore presided over by Nehru.
the following?
The Nehru Committee Report of 10 August 1928 was 1. Wood's Education Dispatch.
a memorandum outlining a proposed new dominion
2. Macaulay's minute on education.
status for the constitution of India. It was prepared by
3. The Sargent Education Report
a committee of the All Parties Conference.
4. Indian Education (Hunter Commission)
23. Match List-I with List-II and select the correct (a) 2, 1, 4, 3 (b) 2, 1, 3, 4
answer: (c) 1, 2, 4, 3 (d) 4, 3, 1, 2
List-I List-II Ans. (a) : On 19 July 1854, Charles Wood, head of the
A. Chittagaon Armoury raid 1. Lal Hardayal Interpretation Board of Control, presented a
B. Kakori Conspiracy 2. Jatin Das comprehensive plan on Indian education, which was
called 'Wood's Dispatch'. In this proposal, views on the
C. Lahore Conspiracy 3. Surya Sen
purpose, medium, reforms etc. of education were
D. Ghadar Party 4. Ram Prasad Bismil
expressed. This manifesto was also called the Magna
5. Vasudeo Phadke Carta of Indian Education. Emphasis was also given on
Codes: giving grant-in-aid in the manifesto. According to the
A B C D proposal, arrangements were made to establish one
(a) 3 4 1 5 university each in Calcutta, Bombay and Madras on the
(b) 4 3 2 5 model of the University of London. Macaulay laid
more emphasis on English education in India. The
(c) 3 4 2 1
Hunter Commission was appointed in 1882 to review
(d) 2 4 3 1 the progress made in education through Wood's
Ans. (c) : The Indian Republican Army (I.R.A.) manifesto. This commission consisted of 8 members.
founded by Surya Sen has a special place in the new The suggestions of the commission were as follows:
revolutionary organizations of Bengal. Surya Sen emphasis on providing vocational and business
actively participated in the non-cooperation movement. education at the high school level, secondly, private
Under the leadership of Surya Sen, the young men and efforts should be welcomed in the field of such literary
women were tried to revolt at a port called Chittagong education. In 1944, the Central Education Advisory
in East Bengal. Hindustan Republican Association Board presented the Sargent Education Plan, in which
successfully robbed down goods train going to Kakori there was a provision to establish primary schools,
on Saharanpur Lucknow line. In this 29 people were higher secondary schools and to provide free
tried, in which Ram Prasad Bismil, Ashfaq Ulla, compulsory education to the children of 6 to 11 years of
Roshan Lal and Rajendra Lahiri were hanged. age.
IAS (Pre) GS 1997 Paper I 508 YCT
26. Match List-I with List-II and select the correct Ans. (c) : Madras is hotter than Calcutta because
answer: Madras is closer to the equator. The maximum heat falls
List-I List-II on the equator, because here the sun's rays fall directly
A. Moplah revolt 1. Kerala throughout the year. Madras is hot, reason for this is
B. Pabna revolt 2. Bihar the difference in latitude. Snow in the Himalayas is
C. Eka Movement 3. Bengal caused by altitude. While coming from West Bengal to
D. Birsa Munda revolt 4. Awadh Punjab, the distance of Punjab from the sea increases,
Codes: due to which the amount of rainfall decreases. In the
A B C D Sutlej and Ganges plains, some rainfall is received
(a) 1 3 4 2 during the winter from western depression.
(b) 2 3 4 1 29. Match the List-I with List-II ad select the
(c) 1 2 3 4 correct answer:
(d) 3 4 1 2 List-I List-II
Ans. (a) : Moplah rebellion in 1921 in Malabar A. Deccan Traps 1. Late cenozoic
(Kerala), Pabna rebellion in Bengal from 1872-1876, B. Western Ghats 2. Pre-cambrian
Eka movement in Awadh in 1921, Munda rebellion in
Bihar in 1899. C. Aravali 3. Cretaciuos-Eocene
D. Narmada-Tapi 4. Cambrian
ART & CULTUREHH alluvial deposits
5. Pleistocene
27. Which one of the following dances involves solo
Codes:
performance?
(a) Bharatanatyam (b) Kuchipudi A B C D
(c) Mohiniattam (d) Odissi (a) 3 5 1 4
Ans. (a) : Bharatanatyam is a major form of Indian (b) 3 1 2 5
classical dance that originated in the state of Tamil (c) 2 1 3 4
Nadu. It is one of eight forms of dance recognized by (d) 1 4 2 5
the Sangeet Natak Akademi and it expressed South Ans. (b) : In the given list, the Aravali mountain is the
Indian religious themes and spiritual ideas, particularly oldest, which belong to the pre-cambrian period, the
of Shaivism, Vaishnavism and Shaktism Deccan trap is of Cretacious-Eocene and they are of the
new period. The Western Ghats are placed under the
INDIAN GEOGRAPHYHH late Cenozoic period, while the Narmada-Tapti alluvial
deposits are of Pleistocene.
28. Match List-I with List-II and select the correct
30. The January isotherm taken as a basis for
answer:
dividing India into tropical and subtropical
List-I List-II
zones is:
(Climatic conditions) (Reasons)
(a) 21°C (b) 18°C
A. Madras is warmer 1. North-East Monsoon
(c) 12°C (d) 15°C
than Calcutta
Ans. (b) : As per stamp and Kendrew's climatic
B. Snowfall in 2. Altitude classification of India, 18°C January isotherm was taken
Himalayas as basis for dividing India into tropical and sub-tropical
C. Rainfall decreases 3. Western depression zone/temperature.
from West Bengal 31. What is the correct sequence of the descending
to Punjab order of the following states in respect of
D. Sutlej-Ganga Plain 4. Distance from sea females literacy rates as per the 1991 census?
gets some rain in 1. Mizoram 2. Kerala
winter 5. Latitude 3. Goa 4. Nagaland
Codes: Choose the correct answer using the codes
A B C D given below:
(a) 1 2 4 5 (a) 2, 3, 1, 4
(b) 4 5 1 3 (b) 2, 1, 3, 4
(c) 5 2 4 3 (c) 2, 3, 4, 1
(d) 5 1 3 4 (d) 1, 2, 4, 3
IAS (Pre) GS 1997 Paper I 509 YCT
Ans. (b) : Female literacy rate (in %) of the states in Ans. (d) : The Himachal Pradesh Maintenance of
question as follows. Parents and Dependents Bill, 1996 was passed to
State 1991 2011 provide speedy relief to the old parents who are unable
Kerala 86.1 92.1 to maintain themselves. It makes mandatory for children
Mizoram 78.6 89.3 to maintain their elderly parents.
Goa 67.1 84.7 35. Which one of the following factors is responsible
Nagaland 54.75 76.1 for the change in the regular direction of the
32. Match List-I with List-II and select the correct ocean currents in the Indian Ocean?
answer: (a) Indian Ocean is half an ocean
List-I List-II (b) Indian Ocean has Monsoon drift
(Minerals) (Typical areas of occurrence) (c) Indian Ocean is a land-locked ocean
A. Coal 1. Bhandara (d) Indian Ocean has greater variation is salinity
B. Gold 2. Karanpura Ans. (b) : Indian Ocean has monsoon drift due to which
C. Mica 3. Hutti it is responsible for the change in the regular direction
of the ocean currents in the Indian Ocean. Due to the
D. Manganese 4. Nellore
monsoon drift of Indian Ocean, the regular direction of
Codes: the north Indian ocean currents changes twice a year.
A B C D 36. The canal-carrying of Farrakka is:
(a) 1 3 2 4 (a) 75000 Cusecs
(b) 2 3 4 1 (b) 70000 Cusecs
(c) 3 4 2 1 (c) 40000 Cusecs
(d) 2 1 4 3 (d) 35000 Cusecs
Ans. (b) : Coal - Karanpura (Bihar), Gold - Hutti mine Ans. (c) : The canal carrying capacity of Farakka
(Karnataka), Manganese - Bhandara (Maharashtra) and barrage is 40,000 cusec. Farakka barrage is a 2245
Mica - is found in Nellore district (Andhra Pradesh). metre long barrage across the river Ganga.
33. The Alamatti is on the river: 37. Which one of the following Pairs is correctly
(a) Godavari (b) Cauvery matched?
(c) Krishna (d) Mahanadi (a) Teak : Jammu and Kashmir
Ans. (c) : Almatti Dam is built on the Krishna River. (b) Deodar : Madhya Pradesh
The Krishna River originates from the Western Ghats
(c) Sandalwood : Kerala
near Mahabaleshwar. The plain between Tungabhadra
and Krishna river is known as Raichur doab, 17 km (d) Sundari : West Bengal
from the delta of the Krishna river, canals have been Ans. (d) : Sundari tree is found in Sundarban area of
taken out by making former dams, With which 4 lakh West Bengal. Sundarban is a largest mangrove area in
hectares of land is irrigated, the two main branches of the world. Sundarban got its name itself due to
Godavari become Vasistha and Gautami. Roli and abundance of ‘Sundari’ tree in this area.
Dowlais dams on Gautami and Munnar Dam and 38. Which one of the following is cultivated by
Vijjeswaram waram on Vasistha and 3220 km. Long transplanting seedlings?
canals have been taken out from which 5 lakh hectares (a) Maize (b) Sorghum
of land is irrigated. Irrigation facilities and power (c) Onion (d) Soyabeen
generation are being made available by building Metur
Ans. (c) : Onion is cultivated by transplanting seedlings.
dam on river Kaveri. Hirakund Dam Project is an
ambitious project of Orissa. The Mahanadi was once 39. Consider the map given below:
called the 'Sorrow of Orissa' due to its floods. The main
dam has been built at Hirakund place, 14 km upstream
from Sambalpur at a cost of Rs. 100 cr. It is the longest
dam in the world.
34. Which one of the following states of India has
passed a legislation (in 1996) making the
maintenance of one's parents mandatory?
(a) Kerala (b) West Bengal
(c) Maharashtra (d) Himachal Pradesh

IAS (Pre) GS 1997 Paper I 510 YCT


The places marked A, B, C and D is the map (a) Groundnut, ragi, tobacco and sugarcane
are respectively– (b) Groundnut, sugarcane, ragi and tobacco
(a) Rift valley region, Chhattisgarh plain, Rain (c) Ragi, sugarcane, groundnut and tobacco
Shadow region and Chota nagpur plateau (d) Ragi, groundnut, sugarcane and tobacco
(b) Chhattisgarh plain, Chota nagpur plateau, Rift Ans. (b) : The place marked in the given map of
valley region, and Rain shadow region India. ‘A’ refers to Gujarat where groundnut is a major
(c) Rift Valley region, Chhattisgarh plain, chota crop. ‘B’ refers to western part of Maharashtra which is
nagpur plateau and Rain shadow region known for sugarcane. ‘C’ refers to southern part of Orissa
(d) Chhattisgarh plain, Rain shadow region, and northern part of Andhra Pradesh, where ‘Ragi’ is
Chota nagpur plateau and Rift valley region cultivated abundantly. ‘D’ refers Andhra Pradesh and
Ans. (c) : Rift valley region, Chhattisgarh plain, Chota tobacco is cultivated in this area.
Nagpur plateau and Rain shadow region. 41. Which of the following places are known for
• There is further spreading of crust and subsequent paper manufacturing industry?
erosion deepens the rift. Rivers that flow thorough rift 1. Yamunanager 2. Guwahati
valley in India are: Narmada River, Tapti, Mahi (all
3. Shahabad 4. Ballarpur
flowing west), and Damodar River in Chota Nagpur
plateau. Choose the correct answer using the codes
• Chhattisgarh plain: A vast undulating tract, the plain given below:
consists of rich rice fields. Cotton and oilseeds are the (a) 1, 2 and 3
important commercial crops of the region. Extensive (b) 1, 2 and 4
coal deposits and substantial deposits of iron ore, (c) 1, 3 and 4
bauxite, manganese, and commercial clays have aided (d) 2, 3 and 4
its development.
Ans. (c) : Yamuna Nagar, Guwahati and Ballarpur are
• The Chota Nagpur Plateau is a plateau in eastern famous for paper manufacturing industry. Yamuna
India, which covers much of Jharkhand state as well Nagar is in Haryana, Guwahati is in Assam and
as adjacent parts of Odisha, West Bengal, Bihar and Ballarpur is in Maharashtra.
Chhattisgarh. The Indo-Gangetic plain lies to the
north and east of the plateau, and the basin of the 42. The tribal population in Andaman and Nicobar
Mahanadi River lies to the south. Islands belongs to the:
(a) Australoid race
• A rain shadow is a dry area on the leeward side of a
mountainous area. The mountains block the passage (b) Caucasoid race
of rain-producing weather systems and cast a (c) Mongoloid race
\"shadow\" of dryness behind them. Eastern Side of (d) Negroid race
Sahyadri ranges on Deccan e.g. Northern Karnataka Ans. (c) : The tribal population in Andaman and
& Sholapur, Beed, Osmanabad and Vidarbha Plateau Nicobar Islands particularly the Onge and Jarawa
of India.
belongs to Negroid race and Great Andaman's and
40. Consider the map given below: sentinels and shompen and Nicobar belongs to
Mongoloid race.
43. The following are the major oilseeds produced
in India
1. Sesamum 2. Mustard
3. Groundnut 4. Soyabeen
Which one of the following is the correct sequence
of the descending order of the quantity of their
production?
(a) 1, 2, 3, 4 (b) 3, 2, 4, 1
(c) 2, 4, 3, 1 (d) 4, 3, 2, 1
Ans. (b) : The descending order of production of the
The places marked A, B, C and D is the map oilseed crops as per question in India will be as follows
are respectively known for the cultivation of: Soyabean, Groundnut, Rye and Mustard and Sesamum.

IAS (Pre) GS 1997 Paper I 511 YCT


44. In the map shown in the given figure, rivers
labelled as 1, 2, 3 and 4 respectively: WORLD GEOGRAPHY
46. About 50% of the world population is
concentrated between the latitudes of:
(a) 5°N and 20°N (b) 20°N and
40°N
(c) 40°N and 60°N (d) 20°S and
40°S
Ans. (b) : For the solution of the presented question,
pay attention to the map of the world, where you will
find that between 20 ° N and 40 ° N, the most populous
countries of the world China, India, the countries of
North America and Southern Europe are located.
(a) Kosi, Gomati, Ghaghara and Gandak About 50% of the world's population resides in this
(b) Kosi, Ganga, Gomati and Ghaghara area.
(c) Gandak, Ganga, Gomati and Ghaghara 47. "_They are people of yellow complexion,
(d) Teesta, Gomati, Ghaghara and Kosi oblique eyes, high check bones, sparse hair and
Ans. (a) : Kosi, Gomati, Ghaghara and Gandak. medium height," The reference here is to:
45. Match List-I with List-II and select the correct (a) Nordic Aryans (b) Austrics
answer: (c) Negroids (d) Mongoloids
List-I List-II Ans. (d) : Mongloids are inhabitants of northern,
(Hazardous industries (Located at) eastern and south-eastern Asia. Their eyes has a
characteristic fold of skin on the upper lid, hair is lank
using child labour
and straight and the height is medium. The group also
leading to the filling
includes the Chinese, Japanese, Burmese, Thais,
of a public interest
Vietnamese and Malays.
petition in the
48. The tail of a comet is directed away from the
Supreme Court) sun because:
A. Glass Industry 1. Moradabad (a) As the comet rotates around the sun, the
B. Brassware Industry 2. Marakpur lighter mass of the comet is pushed away due
C. State Industry 3. Ferozabad to the centrifugal force alone
D. Handmade carpet 4. Mirzapur (b) As the comet rotates, the lighter mass of the
Industry comet is attracted by some star situated in the
Codes: direction of its tail
A B C D (c) The radiation emitted by the sun exerts a
radial pressure on the comet throwing its tail
(a) 3 1 2 4
(b) 1 3 4 2 away from the sun
(c) 3 1 4 2 (d) The tail of the comet always exists in the
(d) 1 3 2 4 same orientation

Ans. (a) : The correct match for List-I and List-II is Ans. (c) : Comets are found in the outer margin made
up of dust particles, gas, ice etc. They revolve around
mentioned as below:
the Sun. The head of a comet is formed by the
Glassy Industry – Ferozabad
evaporation of the material in the part located towards
Brassware Industry – Moradabad
the Sun as it approaches the Sun. The radiation emitted
State Industry – Marakpur by the Sun exerts a pressure on the comet, so that its tail
Hand made carpet – Mirzapur is always away from the Sun. Hellbop, Forbes, Haley,
Industry Temple-1, etc. are examples of orbiters. Halley's Comet

IAS (Pre) GS 1997 Paper I 512 YCT


is visible every 76th year. The orbits of comets are (a) A (b) B
elliptical. (c) C (d) D
49. Consider the map given below: Ans. (a) : Due to the force of gravity, all the planets
The river shown on the map is– revolve around the Sun in an anti-clockwise direction.
According to Kepler's law, the planet's field current is
fixed for a planet revolving around the Sun. When the
planet is closest to the Sun, its speed in orbit (V) is
maximum. In this case its kinetic energy (1/2mv) is
maximum.
52. "From Aceh in the far north west to Torees
Strait in the east is 5000 miles, almost as far as
from London to Baghdad. The archipelago has
(a) Irrawaddy (b) Mekong
14,000 islands some mere equatorial rocks,
(c) Chao Phraya (d) Salween
others some of the largest in the world." This
Ans. (d) : The Salween river is located in the eastern
description best fits:
part of Myanmar. This river is situated at Shan plateau
of Myanmar. Its basin covers 320,000 sq km. This river (a) West Indies (b) Japan
forms the border between Burma and Thailand. (c) Philippines (d) Indonesia
50. Match List-I with List-II and select the correct Ans. (d) : The above statement is related to Indonesia.
answer: The country called Indonesia is a group of islands in
List-I List-II which there are about 14000 islands. Some of these are
(Crops) (Geographical large islands (Java, Sumatra) and some are only
conditions) equatorial rock areas.
A. Barley 1. Hot and dry climate with 53. The group of small pieces of rock revolving
poor soil round the sun between the orbits of Mars and
B. Rice 2. Cool climate with poorer soil
Jupiter are called:
C. Millets 3. Warm and Moist climate with
high attitude (a) meteors (b) comets
D. Tea 4. Hot and Moist climate with (c) meteorites (d) asteroids
rich soil Ans. (d) : Apart from the stars, planets and satellites,
Codes: there are numerous tiny bodies which also move around
A B C D the sun. These bodies are called asteroids. They are
(a) 2 4 1 3 found between the orbits of Mars and Jupiter.
(b) 3 4 1 2 54. If the earth's direction of rotation is reversed,
what would be the I.S.T. when it is noon at the
(c) 3 2 4 3
International Date Line?
(d) 3 2 4 1
(a) 06.30 hrs. (b) 05.30 hrs.
Ans. (a) : Barley requires cool climate with poor soil
(c) 18.30 hrs. (d) 17.30 hrs.
quality. Barley can be grown in high latitude even
Ans. (c) : If the direction of rotation of the earth is
beyond the Arctic circle. Whereas rice needs hot and
reversed, then the relation of Indian Standard Time with
moist climate with rich soil. Millets are grown in hot
the International Date Line will be inverted. That means
and dry climate with poor soil, whereas tea needs warm
and moist climate with high altitude. the one which is +5.30 hrs now it will be -5.30 hrs.
55. Which one of the following stars is nearest to
51. The planet mercury is revolving in an elliptical
orbit around the sun as shown in the given the Earth?
figure. The kinetic energy of mercury is (a) Polaris (b) Alpha Centauri
greatest at the point labeled. (c) Sun (d) Sirius
Ans. (c) : The Sun is the closest star to the Earth, which
is 109 times bigger than the Earth. The Sun is
considered the father of the solar family. Who Provides
light and heat to the solar system. The distance between
the Sun and the Earth is about 152.09 million km. Earth
is also a satellite among many satellites of the Sun. The

IAS (Pre) GS 1997 Paper I 513 YCT


Sun is the basis of life on Earth. It takes 8 minutes 20 1 in the presented picture is indicating the California
seconds for the light of the Sun to reach the Earth. region of the United States of America. The other areas
56. Which one of the following conditions is most climatic conditions are different.
relevant for the presence of life on Mars? 59. Which one of the following countries is the
(a) Atmospheric composition largest producer of fuel wood in the world?
(b) Thermal conditions (a) Indonesia (b) Russia
(c) Occurrence of ice caps and frozen water
(d) Occurrence of ozone (c) India (d) China
Ans. (c) : Mars appears like a red ball. Due to the Ans. (b) : Fuel wood refers to various forms of wood
possibility of life, man has been more interested in this that are used as fuel for cooking heating or to drive stem
planet than other planets. The possibility of life on Mars powered engines or turbines for electricity generation.
has been proved by the presence of ice components and As Russia is having largest percentage of forest cover
frozen water. This planet has a thin layer of atmosphere, and thus it is the largest producer of fuel wood in the
in which nitrogen and argon gases are found.Nix
world.
Olympia is the highest mountain on Mars, which is
three times higher than Mount Everest. At present, the 60. One will NOT have to pass through the Suez
exploration of this planet is going on by the European Canal while going from Bombay to:
Union's Beagle-2 and NASA's Spirit mission. (a) Alexandria (b) Suez
57. Consider the geographical details given in the
following figure: (c) Port Said (d) Benghazi
The point marked by A in the above figure Ans. (b) : The Suez canal had been constructed in 1869
indicates a country in in Egypt between port said in the north and port Suez in
the south linking the Mediterranean sea and the Red sea.
So, while going from Bombay through Suez people
will come across Alexandria port, port said and
Benghazi port. Thus out of given options one will not
pass through suez.
(a) North America 61. Which one of the following is present in the
(b) South America largest amount in terms of percent by mass in
(c) Europe the earth's crust?
(d) Asia (a) Silicon (b) Oxygen
Ans. (c) : In the geographical details given in the above (c) Carbon (d) Calcium
figure, the point 'A' located between 50 ° north latitude Ans. (b) : Oxygen is the most abundant element in the
to 51 ° north latitude and 5 ° east longitude to 6 ° east earth's crust. It is estimated that oxygen comprises
longitude, is showing the country located in the nearly half of the mass of the crust. It accounts for 21%
continent of Europe. of Earth atmosphere.
58. Consider the map given below: Oxygen – 46.6% of Earth Mass
Silicon – 27.7% of Earth Mass
Aluminum – 8.1% of Earth Mass
Calcium – 3.6% of Earth mass

INDIAN CONSTITUTION AND POLITY


62. Which one of the following was NOT proposed
by the 73rd constitutional amendment in the
area of Panchayati Raj?
Of the four shaded areas in the map, that (a) Thirty percent seats in all elected rural local
which is characterised by hot dry summers, bodies will be reserved for women candidates
mild and moist winters and seasonal reversal at all levels
of winds is the area labbeded:
(b) The states will constitute their Finance
(a) 1 (b) 2
(c) 3 (d) 4 Commissions to allocate resource to
Ans. (a) : Studying the climate of 1, 2, 3 and 4 shaded Panchayati Raj institutions
areas in the given maps, it is clear that only in the (c) The Panchayati Raj elected functionaries will
shaded part (1) there is hot, dry, summer and mild be disqualified to hold their offices if they
winter and seasonal reversal of winds. The shaded part have more than two children
IAS (Pre) GS 1997 Paper I 514 YCT
(d) The elections will be held in six months time (d) He has to become a member of the Lower
if Panchayati Raj bodies are superseded or House within six months after being sowrn in
dissolved by the state government as the Prime Minister
Ans. (c) : According to the 73rd Constitutional Ans. (a) : If the Prime Minister of India belonged to the
Amendment, the provision of (a) out of the four answer upper house of parliament then he will not be able to
options of the above question is in Article 243D, (3) (b) is vote in his favour in the event of a nor confidence
motion. A no-confidence motion is a parliamentary
provided in Article 243-I (i) and (d) is provided in Article motion which is moved in Lok Sabha against the entire
243k (3). Only option (c) is not there in the 73rd council of ministers, stating that they are no longer
Constitution amendment act. deemed fit to hold positions of responsibility due to
their in adequacy in some respect or their failure to
63. Proportional representation is NOT necessary carry out their obligation. No prior reason needs to be
in a country where: stated for its adoption in the Lok Sabha.
(a) There are no reserved constituencies 66. Given below are two statements, one labelled as
(b) A two-party system has developed Assertion (A) and the other labelled as Reason
(R):
(c) The first-past-post system prevails
Assertion (A) : The reservation of thirty-three
(d) There is a fusion of presidential and Parliamentary percent of seats for women in Parliament and
forms of government State Legislatures does not require Constitutional
Ans. (b) : The main purpose of the proportional amendment.
representation system is to assure the representation of all Reason (R) : Political parties contesting elections
can allocate thirty-three percent of seats they
groups and ideologies. So, it is usually adopted in a multi- contest to women candidates without any
party system representing multiple ideologies. Constitutional amendment.
In a two party system, the seats are ultimately going to be The correct option among the following is
divided between the two parties and there is negligible (a) Both A and R are true and R is the correct
chance of leaving out a group as compared to the multi- explanation of A
(b) Both A and R are true but R is NOT the
party system.
correct explanation of A
So, first past the post can be adopted in two party system, (c) A is true, but R is false.
avoiding the complexities of proportional representation. (d) A is false, but R is true.
64. The concept of Public Interest Litigation Ans. (d) : Provisions have been made from Article 330
originated in: to 334 in Part 16 of the Constitution of India for the
reservation of seats for certain classes in Parliament and
(a) The United Kingdom
State Legislatures. If a similar place is to be reserved
(b) Australia for women, then this part must be amended. Hence
(c) The United States statement (a) is wrong. On the other hand reason (R) is
(d) Canada correct, because for example Asom Gana Parishad is a
Ans. (c) : The concept of public interest litigation political party, which reserves 35% seats for.
originated in the United States. The general rule of 67. In the Presidential election in India, every
elected member of the Legislative Assembly of
public interest in the Indian Constitution is that the
a State shall have as many votes as there are
remedy mentioned in Article 226 can be applied only by multiples of one thousand in the quotient
a person whose legal rights are violated by the state or obtained by dividing the population of the state
any person. But in its recent judgments, the Supreme by the total number of the elected members of
Court has held that any person or association of the the Assembly. As at present (1997) the
expression "population" here Means the
society can apply for remedy against the violation of the
population as ascertained by the:
legal rights of any person or class who, due to his (a) 1991 Census (b) 1981 Census
poverty or any other reason, cannot go to the court. (c) 1971 Census (d) 1961 Census
65. If the Prime Minister of India belonged to the Ans. (c) : In India, the allocation of seats in the lower
Upper House of Parliament: legislative houses (Lok Sabha and State Legislative
(a) He will not be able to vote in his favour in the Assemblies) is done by the 'Delimitation Commission'
even of a no-confidence motion constituted by the Parliament after every ten year census
(b) He will not be able to speak on the budget in last time in 1974 AD. The Delimitation Commission
the Lower House had allocated seats on the basis of the 1971 census.
Later, by the 42nd Constitutional Amendment, 1976, it
(c) He can make statements only in the Upper
was decided that the number of seats in the Lok Sabha
House
IAS (Pre) GS 1997 Paper I 515 YCT
and the State Legislative Assemblies would be 70. In which one of the following countries are 75
determined on the basis of the 1971 census till 2001. At percent of seats in both Houses of Parliament
present, on the basis of the national census, the
filled on the basis of first-past the post system
delimitation has been has been suspended till 2026 AD.
and 25 percent on the basis of Proportional
68. Which of the following are/is stated in the
Constitution of India? Representation system of electrons?
1. The president shall not be a member of (a) Germany (b) Italy
either House of Parliament. (c) France (d) Russia
2. The Parliament shall consist of the President Ans. (b) : Italy, troubled by the instability of the contract
and two houses. governments, has established this new system in line with
Choose the correct answer from the codes the Bicameral proposals to solve this problem. Under this,
given below: parliamentary elections will be held in two phases. The
(a) Neither 1 nor 2 (b) Both 1 and 2 two parties that get the most votes in the first phase will
(c) 1 alone (d) 2 alone participate in the second phase. In this, the winning party
Ans. (b) : Article 79 of the Indian Constitution provides will have the right to 75% of the seats in the House. The
for the Union that there shall be a Parliament consisting remaining 25% seats will be filled through the list system
of the President, the Rajya Sabha and the Lok Sabha. on the basis of proportional representation. Although this
The President is not a member of either house of the system will harm the smaller parties, they can also gain
Parliament. some space in the alliance by bargaining with the major
69. Match List-I with List-II and select the correct parties for the second phase of voting.
answer: 71. In the following quotation,
List-I List-II
'WE THE PEOPLE OF INDIA,
(Functionaries) (Oaths or
having solemnly resolved to constitute India into
affirmations)
A. President of India 1. Secrecy of a Sovereign, Socialist, Secular, Democratic
Information Republic and to secure to all its citizens
B. Judges of Supreme 2. Faithful JUSTICE, social economic and political,
Discharge of LIBERTY of thought, expression, belief faith and
Duties worship;
C. Members of 3. Faith and EQUALITY of status and of opportunity;
parliament Allegiance to the and to promote among them all;
Constitution of FRATERNITY assuring the dignity of the
India individual and the unity and the integrity of the
D. Minister for the 4. Upholding the
Nation; In our Constituent Assembly this 'X'do
Union Constitution and
hereby adopt, enact and give to ourselves this
the law
Constitution.", 'X' stands for
Codes:
A B C D (a) Twenty-sixth day of January, 1950
(a) 3 4 1 2 (b) Twenty-sixth day of November, 1949
(b) 4 3 2 1 (c) Twenty-sixth day of January, 1949
(c) 3 4 2 1 (d) None of the above
(d) 4 3 1 2 Ans. (b) :This is the Preamble of the Indian
Ans. (c) : The above list is mentioned in the Third Constitution, on which the date of November 26, 1949
Schedule of the Constitution of India, the details of the is inscribed. On this day Dr. Bhimrao Ambedkar in the
oath of the persons appointed to various constitutional Constituent Assembly passed the resolution for the
posts, which are as follows - passage of the Constitution. With the passage in the
Constituent Assembly, some of the provisions of the
Union Ministers - take the oath of secrecy of
Constitution (Citizenship, Elections and Interim
information.
Parliament etc.) came into force. The remaining
President of India - Faith and allegiance to the provisions came into force on 26 January 1950.
Constitution of India.
72. Given below are two statements one labelled as
Member of Parliament - Discharging duties faithfully.
Assertion (A) and the other labelled as
Judge of the Supreme Court - Upholding the dignity of
the Constitution and law. (Reason):
Assertion (A) : Willful disobedience or non-
compliance of Court order and use of
IAS (Pre) GS 1997 Paper I 516 YCT
derogatory language about judicial behaviour Ans. (c) : As per world banks national account data,
amount to Contempt of Court. India’s average domestic saving rate was 25.059% of
Reason (R) : Judicial activism cannot be GDP, but in 2007 it was 34.377% and in 2020 it was
practiced with arming the judiciary with punitive 28.349% of GDP. So correct option for both 1997 and
powers to punish contemptuous behaviour. 2020 will be (c).
In the context of the above two statements, 76. The number of industries for which industrial
which one of the following is correct? licensing is required has now been reduced to:
(a) Both A and R are true and R is the correct
(a) 15 (b) 6
explanation of A
(c) 35 (d) 20
(b) Both A and Ra re true but R is not correct
Ans. (a) : In the new industrial policy announced on
explanation of A
July 24, 1991, the number of industries for which it was
(c) A is true, but R is false.
mandatory to take license was 18. Again, with the
(d) A is false, but R is true.
elimination of the requirement of license for 3 other
Ans. (b) : Assertion and Reason as independent
industries, the number of industries requiring license
statements are true but does not explain each other.
was reduced to 15 before November 13, 1996. At
73. The Dinesh Goswami Committee recommended: present the number of such industries is reduced to 5.
(a) The constitution of state level election These 5 industries are as follows:
commission
1. Aerospace and defence related electronics
(b) List system of election to the Lok Sabha
2. Gun powder, industrial explosives and detonating
(c) Governmental funding of parliament elections
fuse.
(d) A ban on the candidature of independent
3. Dangerous chemicals
candidates in the parliamentary elections
4. Tobacco, cigarette and related products
Ans. (c) : Dinesh Gosami committee report, 1990 is
5. Alcoholic drinks.
associated with electoral reform in India. It focused
77. The sum of which of the following constitutes
mainly on the role of money and muscle power at
Broad Money in India?
elections, rapid criminalization of politics and also it
recommended governmental funding of parliament 1. Currency with the public.
elections. 2. Demand deposits with banks.
3. Time deposits with banks.
ECONOMYHH 4. Other deposits with RBI.
Choose the correct answer using the codes
74. The economic and monetary union of 15
given below:
European countries is proposed to be made by
(a) 1 and 2 (b) 1, 2 and 3
1999. But the currencies of two countries,
Franc, have already the same value and (c) 1, 2, 3 and 4 (d) 1, 2 and 4
circulate freely in both the countries. The Ans. (b) Narrow money is the most liquid part of the
countries are: money supply because the demand deposits can be
(a) France and Switzerland withdrawn anytime during the banking hours. Time
deposits on the other hand have a fixed maturity period
(b) Switzerland and Luxembourg
and hence cannot be withdrawn before the expiry of this
(c) Luxembourg and Belgium
period. When we add the time deposits into the narrow
(d) France and Belgium
money, we get the broad money, which is denoted by
Ans. (c) : Luxembourg and Belgium are the two
M3. M3 = Narrow money + Time Deposits of public
countries where 'Franc' is the currency, having same
with banks. We note here that the Broad money does
value and is in free circulation. In 2002, it was replaced
not include the interbank deposits such as deposits of
by Euro.
banks with RBI or other banks. At the same time, time
75. The average rate of domestic savings (gross)
deposits of public with all banks including the
for the Indian economy is currently estimated
cooperative banks are included in the Broad Money.
to be in the range of:
78. According to the latest census figures sex ratio
(a) 15 to 20 percent (b) 20 to 25 percent
in India is :
(c) 25 to 30 percent (d) 30 to 35 percent (a) declining (b) stable
IAS (Pre) GS 1997 Paper I 517 YCT
(c) increasing (d) fluctuating
Ans. (a) :Option (a) was the correct answer at the time
when question was asked an increase of 10 points has
been recorded in the sex ratio/sex ratio between 2001
and 2011.
79. Match List-I with List-II and select the correct The behaviour of a fictitious stock market
answer: index [comprising a weighted average of the
List-I List-II market prices of a selected list of companies
(Committees) (Chaired by) including some multinational corporations
A. Disinvestment of 1. Rajah Chelliah (MNC's) over a 15 day period is shown in the
graph. ?the behaviour of the MNC's in the
shares in Public
same period is also shown in the second graph.
Sector Enterprises Which one of following is a valid conclusion?
B. Industrial Sickness 2. Onkar Goswami (a) MNC's fell steeper in the period depicted
C. Tax Reforms 3. R. Malhotra (b) Share prices of every non-MNC company
D. Reforms in 4. C. Rangaraja soared by over 5% on the 12th day
Insurance Sector (c) Government announced a police disfavouring
MNC's on the 11th day
Codes:
(d) Whatever be the reason favouring market
A B C D revival on the 12th day. It appears to be relevant
(a) 1 4 2 3 only to non-MNC companies.
(b) 4 2 1 3 Ans. (d) :In the context of the question, option (d)
(c) 4 1 2 3 whatever the reason may have been in favour of market
(d) 1 3 4 2 resurgence on the 12th day, which appears to be relevant
Ans. (b) : Chelliah Committee on Tax Reforms – 1993 only to non-MNCs, would be a meaningful conclusion.
Omkar Goswami Committee on Industrial Sickness – 82. Match List-I with List-II and select the correct
1994. answer:
List-I List-II
Rangarajan Committee on Disinvestment of Shares in (Commodities exported (Countries of
PSUs – April 1993. from India) destination)
RN Malhotra Committee on Reforms in Insurance A. Iron Ore 1. Russia
Sector – 1993 B. Leather goods 2. U.S.A.
80. Human Development Index comprises literacy C. Tea 3. Japan
D. Cotton fabrics 4. U.K.
rates, life expectancy at birth and: 5. Canada
(a) Gross Domestic Product per head in the US Codes:
dollars A B C D
(b) Gross Domestic Product per head at real (a) 5 1 2 3
(b) 3 1 4 2
purchasing (c) 1 5 4 3
(c) Gross National Product in US dollars (d) 3 4 1 2
(d) National Income per head in US dollars Ans. (b) : Iron ore from India is exported to Japan.
Ans. (d) : HDI is a statistical tool which measures the The largest export of tea, to leather goods, is in the UK.
(Islands of England) and cotton fabrics are mainly
country’s overall achievement in its socio-economic
exported to the United States of America.
dimension. It is being prepared and published by the United
83. In India, inflation is measured by the:
Nations Development Programme. (a) Wholesale Price Index number
The basic components of HDI are Health (longevity of life), (b) Consumers Price Index urban non-manual workers
Education and living standards (Gross national income per (c) Consumers Price Index for agricultural workers
capita). (d) National Income Deflation
81. Consider the following graphs: Ans. (a) : Inflation is measured by general price
coordinate. Price coordinates measure changes in the
average prices of goods. The base year's coordinate is
assumed to be 100 and in comparison the current year's
coordinate is calculated. If the current year's coordinate
is less than 100, it shows a decrease in prices and if the
current year's coordinate is more than 100, it shows an
increase in prices. Since there is an inverse relationship
between the value of money (purchasing power of
IAS (Pre) GS 1997 Paper I 518 YCT
money) and inflation. Therefore, with the help of 87. The contribution of India's small scale sector to
coordinate the value of money can also be measured. the gross turnover in the manufacturing sector
The value of money (purchasing power) decreases with since 1992 has been of the order of:
an increase in the price level, while the value of money
(purchasing power) increases with a fall in the price (a) 40% (b) 34%
level. (c) 30% (d) 45%
84. The Sixth and the Eight Five Year Plans Ans. (a) : The contribution of small scale industries to
covered the Period 1980-1985 and 1992-1997 employment and exports is substantial. The sector
contributes over 40 per cent of the gross turnover in the
respectively. The Seventh Five Year Plan
manufacturing sector, 45 per cent of manufactured
covered the period:
exports and 25 per cent of total exports. As in 2015.
(a) 1987-1992 (b) 1986-1991
88. Which of the following come under Non-plan
(c) 1985-1990 (d) 1988-1994 expenditure?
Ans. (c) : The time of the Seventh Five Year Plan was 1. Subsidies
from 1st April, 1985 to 31st March, 1990 AD. After
2. Interest payments
this the Eighth Five Year Plan which was to start from
1st April, 1990 could not be started on time due to 3. Defence expenditure
political changes in the center in two years. Thus the 4. Maintenance expenditure for the
time of the Eighth Five Year Plan was from 1st April, infrastructure created in the previous plans
1992 to 31st March, 1997 AD. Choose the correct answer using the codes
85. Schemes of (i) Urban Micro-Enterprises, (ii) given below:
Urban Wage Employment and (iii) Housing and (a) 1 and 2 (b) 1 and 3
Shelter Up-gradation are parts of: (c) 2 and 4 (d) 1, 2, 3 and 4
(a) Integrated Rural Development Programme Ans. (d) : Non-plan expenditure is what the government
(b) Nehru Rojgar Yojana spends on the so-called non productive areas, such as
(c) Jawahar Rojgar Yojana salaries, subsidies, loan and interest payments, defence
(d) Prime Minister's Rojgar Yojana expenditure and maintenance expenditure for the
infrastructure created in the previous plans. So all the
Ans. (b) : The Nehru Rozgar Yojana (1989) has been
given codes are correct and thus option (d) is correct.
designed to provide employment to the urban
unemployed and under-employed poor. 89. National Income is the:
86. Given below are two statements one labelled as (a) Net National Product at market price
Assertion (A) and the other labelled as (b) Net National Product at factor cost
(Reason): (c) Net Domestic Product at market price
Assertion (A) : The emergence of economic (d) Net Domestic Product at factor cost
globalism does not imply the decline of socialist
ideology.
Reason (R) : The ideology of socialism believes
in Universalism and globalism.
In the context of the above two statements,
which one of the following is correct?
(a) Both A and R are true and R is the correct
explanation of A
(b) Both A and Ra re true but R is not correct
explanation of A Ans. (b) : National Income of countries is the total
(c) A is true but R is false. amount of income that is accrued by it through various
economic activities in one year. It is used in
(d) A is false but R is true.
determining the progress of the country.
Ans. (b) : Globalism is the policy of placing the
interests of the world above those of individual nations. It is also defined as Net National Product at factor cost.
This also means that there would be free movement of National Income = Net National Product + Subsidies –
goods and services across national boundaries. Interest Taxes.
Socialism is an economic system based on state 90. The Minimum Alternative Tax (M.A.T.) was
ownership of means of production including capital. introduced in the Budget of the Government of
Universalism means application of certain basic India for the year:
principles at universal level. (a) 1991-92 (b) 1992-93
IAS (Pre) GS 1997 Paper I 519 YCT
(c) 1995-96 (d) 1996-97 94. A girl is swinging on a swing in sitting position.
Ans. (d) : Minimum Alternate Tax (MAT) is applied when When the same girl stands up, the period of
the taxable income calculated according to the I-T Act swing will:
provisions is found to be less than 15.5 percent of the book (a) Be shorter
profit under the companies Act. It was introduced by the (b) Be longer
finance act, 1996.
(c) Depend on the height of the girl
PHYSICS (d) Not change
91. The tendency of a liquid drop to contract and
Ans. (a) : A swing is a example SHM,
occupy minimum area is due to:
(a) Surface tension (b) Viscosity Time period for it will be :
(c) Density (d) vapour pressure ρ
T = 2π
Ans. (a) : The surface area of a sphere is minimum for a g
given volume. Therefore, from the spherical shape of
the liquid droplets, it can be concluded that the liquid T ∝ l
wants to keep its surface area minimum. Thus it is clear When girl stands up, centre of mass of her body will go
that there is a kind of tension in the surface of the liquid up, so will be less then ‘T’.
which is called surface tension. The surface tension of a 95. A body standing at the point 'O' in the given
liquid is the force that acts F on the unit length of an diagram, throws a ball three times with the
imaginary line drawn on the surface of the fluid in the
same force, but projecting it along different
plane of the surface and perpendicular to the line. Its
unit is Newton/Meter. inclinations from the ground. The results of the
Surface tension T = F /L throws have been plotted in the diagram.
where, F= force, L = length of line. Which one of the following is a valid
conclusion?
92. Which one of the following is a vector
quantity?
(a) Momentum (b) Pressure
(c) Energy (d) Work
Ans. (a) : Scalars quantity are quantities that have
magnitude only and they are independent of direction.
Vectors quantity have both magnitude and direction.
Momentum is the product of the mass and velocity of an
object (p = mv). Momentum is a vector quantity, since
it has a direction as well as a magnitude. The rest of
(a) The larger the initial inclination, the longer
quantities in option pressure, work and energy have
the throw
magnitude but not direction.
(b) The larger the height reached, the longer the
93. A smooth inclined at an angle q with the
throw
horizontal as shown in the above figure. A body
(c) The larger the height reached, the shorter the
starts from rest and slides down the inclined
throw
surface. The time taken by the body to reach
(d) The larger the initial inclination, the greater
the bottom is:
the height reached
2h 2l Ans. (c) : In given figure ball is shown thrice with same
(a) (b)
g g force but with different inclination.
1 2h 2h Horizontal range = v cosθ
(c) (d) sinθ
sin θ g g Vertical range = v sinθ
Ans. (c) : If body is projected with higher altitude, it means value
Component of acceleration along the inclined plane will of ‘θ’ is greater than 45°
be gsinθ In such situation, horizontal range will be lesser because
l= gsinθ×t2 / 2 it depends upon value of cosθ.
sinθ= h / l ⇒ l = sinθ / h
As value of cosθ for angle greater than 45° is lesser
2h compare to sinθ so, horizontal range (throw) will be
t = sinθ
g lesser.
96. The working principle of a washing machine:

IAS (Pre) GS 1997 Paper I 520 YCT


(a) Centrifugation (b) Dialysis Ans. (c) : Oxygen is paramagnetic because the oxygen
(c) Reverse osmosis (d) Diffusion molecule contains two unpaired electrons. Hence,
Ans. (a) : The mechanical process which involves the option C is correct.
use of centrifugal force to separate particles from a 102. Which one of the following has the highest fuel
solution according to their seize, shape density, medium value?
viscosity and rotor speed is known as centrifugation. (a) Hydrogen (b) Charco
The working (c) Natural gas (d) Gasoline
Ans. (a) : Fuel value can be expressed in terms of
CHEMISTRY calorific value of fuel. The calorific value of a fuel is
97. The characteristic odour of garlic is due to : the amount of heat produced by burning 1 kg of fuel.
(a) A chloro compound Hydrogen has the highest calorific value (141,790
(b) A sulphur compound KJ/kg) thus have highest fuel value. Calorific value of
(c) A florine compound charcoal, natural gas and gasoline are (29,600) 43,000
(d) Acetic acid 47,300 kJ/kg) respectively. Natural gas majorly consists
Ans. (b) : The reason for the characteristic smell of of methane.
garlic is due to the 0.1% essential oil present in it. The 103. Which one of the following is used as an anti-
main constituents of this oil are diallyl disulphide, freeze for the automobile engines?
diallyl trisulphide and ally propel disulfide, which are (a) Propyl alcohol (b) Ethanol
all compounds of sulphur. (c) Methanol (d) Ethylene glycol
98. Which one of the following is an active Ans. (d) : Ethylene glycol is used as an antifreeze in
component of oil clove? automobile engines. It is prepared by the oxidation of
(a) Menthol (b) Eugenol ethylene. Where the cold is more, alcohol is used in place
(c) Methanol (d) Benzaldehyde of mercury for the thermometer. Whereas ethylene glycol
Ans. (b) : Oil of cloves, also known as Clove oil, is an is used in the form of antifreeze in vehicles.
essential oil obtained from the clove plant. It is a natural 104. The major component of honey is:
analgesic and antiseptic used primarily in dentistry for (a) Glucose (b) Sucrose
its main ingredient eugenol. The oil produced by cloves (c) Maltose (d) Frustose
can be used in many things from flavouring medicine to Ans. (d) : The major component of honey is fructose.
remedies for bronchitis, common cold, a cough, fever, Composition of honey (in percentage) is.
sore throat and tending to infections. Fructose – 38.2
99. The most reactive among the halogens is : Sucrose – 1.5
(a) Fluorine (b) Chlorine Glucose – 31
(c) Bromine (d) Iodine Minerals – 0.5
Ans. (a) : The most reactive gas among halogens is Water – 17.1
fluorine. Fluorine (F) has the most negative electric Maltose – 7.2
charge of all the other halogens. Due to this quality, it Carbohydrate – 4.2
is most active. The order of their reactivity is as follows
- Fluorine → Chlorine → Bromine → Iodine .
100. Which one of the following was used as a
BIOLOGYHH
chemical weapon in the First World War? 105. The pituitary gland by virtue of its tropic
(a) Carbon monoxide (b) Hydrogen cyanide hormones controls the secretary activity of
(c) Mustard gas (d) Water gas other endocrine glands. Which one of the
Ans. (c) : Mustard gas is a poisonous gas which is following endocrine glands can function
powerful as well as explosive. Its chemical name is independent of the pituitary gland.
dichloro-diethyl -sulfide [S(CH,CH,CI] .It is a colorless (a) Thyroid (b) Gonads
gas. Mustard gas was used as a chemical weapon in (c) Adrenals (d) Parathyroid
World War I, while gases called Phosphine and Sarin Ans. (d) : Parathyroid gland is such a gland, which
were used in World War II. works completely independently. The hormones
101. Which one of the following is paramagnetic in secreted by the pituitary gland do not play any role in its
nature? activity. The pituitary gland is also called the master
(a) Iron (b) Hydrogen gland because it controls the secretion of the endocrine
(c) Oxygen (d) Nitrogen
IAS (Pre) GS 1997 Paper I 521 YCT
glands. The personality of a person also motivates taken from each of them. DNA finger printing can solve
health, growth and sexual development. cases of rape murder etc.
106. Oxygen transportation in a human body takes 111. Arteries supplying blood to the heart are
place through? called:
1. Blood 2. Lungs 3. tissue (a) Carotid arteries
The correct sequence of transportation is (b) Hepatic arteries
(a) 1, 2, 3 (b) 3, 1, 2 (c) Coronary arteries
(c) 2, 1, 3 (d) 1, 3, 2 (d) Pulmonary arteries
Ans. (c) : The first organ in the human body where Ans. (c) : The arteries which supply blood to the heart
oxygen is acquired is the lungs. Oxygen enters the are known as coronary arteries. The right coronary
blood after the lungs and uptake from the blood to the artery supplies blood to right side of heart while left
tissues. Oxygen is mainly transported by the red coronary artery supplies blood to left side of heart.
pigment found in blood, hemoglobin. Hemoglobin is Hepatic artery is a short blood vessel that supplies
purple in color whereas oxy hemoglobin is bright red in oxygenated blood to the liver, pylorus (a part of the
colour. The amount of oxygen that will combine with stomach), duodenum (a part of the small intestine) and
hemoglobin is based on the pressure of oxygen and the pancreas. The carotid arteries are major blood vessels in
pH of the blood. the neck that supply blood to the brain, neck, and face.
107. Corpus luteum is a mass of cells found in: Thus, correct answer is option C.
(a) brain (b) ovary 112. Recommended daily intake of proteins for a
(c) pancreas (d) spleen moderately active women in
Ans. (b) : Corpus luteum is a mass of cells found in (a) 30 g (b) 37 g
ovary. It produces progesterone hormone. If the ovary is (c) 40 g (d) 46 g
not fertilised, the corpus-luteum degenerates after about
Ans. (d) : Daily intake of protein for active woman is
two weeks and is gradually replaced by ordinary ovary
46 gram. But a pregnant women’s protein requirement
tissue.
increase to 47 – 60 gm per day.
108. Alpha-keratin is a protein preset in:
113. Match List-I with List-II and select the correct
(a) blood (b) skin answer:
(c) wool (d) eggs List-I List-II
Ans. (b) : Alpha-keratin, or α-keratin, is a type A. Malaria 1. Bone-marrow
of keratin found in vertebrates. This protein is the
B. Filaria 2. Brain
primary component in hairs, horns, mammalian claws,
nails and the epidermis layer of the skin α-keratin is a C. Encephalitis 3. Muscle
fibrous structural protein, meaning, it is made up of D. Leukaemia 4. Lymph node
amino acids that form a repeating secondary structure. 5. Blood cells
109. What is the average fat content of buffalo Codes:
milk? A B C D
(a) 7.2% (b) 4.5% (a) 5 3 2 1
(c) 9.0% (d) 1.0% (b) 5 4 2 1
Ans. (a) : Average fat content in buffalo milk is 7.2% (c) 4 3 5 1
and average fat content in cow milk is 4.4%. As per the (d) 5 4 1 2
nutrient components, buffalo milk contains all the
Ans. (b) : The correct match is mentioned as below–
nutrients in higher proportion than cow’s milk.
Malaria – Blood cells
110. Which one of the following techniques can be
Filaria – Lymph node
used to establish the paternity of a child?
Encephalitis – Brain
(a) Protein analysis
Lukaemia – Bone marrow
(b) Chromosome counting
(c) Quantitative analysis of D.N.A.
ENVIRONMENT AND ECOLOGY
(d) D.N.A. finger printing
Ans. (d) : DNA finger printing technology is the process 114. Which one of the following organisms is likely
of establishing the biological paternal relationship between to show the highest concentration of D.D.T.
individual and his/her child on the analysis of sample cells

IAS (Pre) GS 1997 Paper I 522 YCT


once it has been introduced into the on the book 'Apprentices of the Mahatma' by Indian-
Ecosystem? origin South African writer Fatima Mir.
(a) Grasshopper (b) Toad 118. A major health mission carried out in 1996
(c) Snake (d) Cattle was:
Ans. (c) : After the introduction of DDT into the (a) War against leprosy
ecosystem, the concentration of DDT in the food chain (b) Small-pox eradication
will be lowest at the first level, higher than that at the (c) Pulse polio immunization
second level and highest at the third level. Since snake (d) Popularisation of oral rehydration therapy
is a tertiary consumer, the concentration of DDT will be
Ans. (c) : The pulse polio immunization was a major
highest in snake.
health mission carried out in 1996 to make India polio
free. The programme is still being carried out.
CURRENT AFFAIRSH 119. State funding of elections takes place in:
115. The Prime Minister of which one of the following (a) U.S.A. and Canada
countries is chosen by the ruling prince from a (b) Britain and Switzerland
slate of three candidates put up to him by the (c) France and Italy
President of France? (d) Germany and Austria
(a) San Marius (b) Liechtenstein Ans. (d) : State funding of elections takes place
(c) Malta (d) Monaco in Germany and Austria.
Ans. (d) : The Minister of State is the head of the 120. Bishop Carlos Felipe Xinenes Belo and Jos
government of Monaco, being appointed by and Remos Horta who shared the 1996 Nobel Peace
subordinate to the Prince or Princess of Monaco. The Prize are known for their work for the cause
office was created in 1911 with the adoption of of:
Monaco's constitution. Until the revision of the (a) East Timor (b) Gautemala
constitution of 2002, he had to be a French citizen,
(c) Bosnia (d) Barunda
selected from several senior civil servants proposed by
the French government. Since 2002, the Minister of Ans. (a) : These individuals have been awarded the
State can be either French or Monegasque and is chosen Nobel Peace Prize in the year 1996 for their efforts in
and appointed by the prince, after consultation with the peaceful and proper solution to the conflict in East
French government. Timor.
116. One of the important agreements reached in 121. Match List-I with List-II and select the correct
the 1996 Ministerial Conference of W.T.O. answer:
refers to: List-I List-II
(a) Commerce in Information Technology A. Rashtriya Mahila 1. Empowerment of
(b) Multilateral Agreement on Investment women
(c) Multi-fibre Agreement B. Mahila Samriddhi 2. Education for
(d) Exchange of technical personal Yojana women's equality
C. Indira Mahila 3. Promotion of
Ans. (a) : The 5-day meeting of the World Trade
Yojana savings among
Organization (W.T.O.) was held in Singapore from
December 9, 1996 to December 13, 1996. 128 countries rural women
participated in this meeting and announced an D. Mahila Samakhya 4. Meeting credit
agreement on duty free trade of information technology Programme needs of the poor
products by the year 2000. women
Codes:
117. The film "The Making of the Mahatama" has
A B C D
been directed by:
(a) 3 2 1 4
(a) Peter Ustinov (b) 1 3 4 2
(b) Richard Attenborough (c) 4 3 2 1
(c) Shyam Benegal
(d) 4 1 2 3
(d) Mira Nair
Ans. (c) : The Rashtriya Mahila Kosh (National Credit
Ans. (c) : The film 'The Making of Mahatma' is directed
Fund for Women) was set up in 1993 to make credit
by Shyam Vikalpa Benegal. The film's theme is based
available for lower income women in India. The Mahila
Samridhi Yojana (MSY) was launched on 2nd October,
IAS (Pre) GS 1997 Paper I 523 YCT
1993 with the objective of empowering the rural women 123. The average monthly income of a person in a
through building thrift habit, self-reliance and confidence. certain family of 5 is Rs. 1000/. What will be
The Indira Mahila Yojana (IMY) aims at organizing at the the monthly average income of person in the
same family if the income of one person
grass-root level to facilitate their participation in decision- increased by Rs. 12000/- per year?
making and their empowerment was launched on 20 (a) 1200 (b) 1600
August 1995. Pursuant to the objectives of the NPE (c) 2000 (d) 3400
(National Policy on Education), 1986, the Mahila Ans. (a) : Total income of the family per month = 5 ×
Samakhya Scheme was started in 1989 to translate the 1000 = Rs.5000 per month
goals enshrined in the NPE into a concrete programme for Increased amount = 12000/12 = Rs.1000 per month
the education and empowerment of women in rural areas Now, total amount of the family per month = Rs.6000
particularly those from socially and economically So, the average income of the family per month (after
marginalized groups. increasing) = 600/5 = 1200.
124. In the given figure, if QRS is an equilateral
MISCELLANEOUS triangle and QTS is an isosceles triangle and x
122. A rectangular plot of lawn shown in the figure = 47°, then value (in degrees) of y will be:
has dimension 'x' and 'y' and is surrounded by
a gravel pathway of width 2m. What is the total
area of the Pathway?

(a) 13° (b) 23°


(c) 33 (d) 43°
Ans. (a) :
In equilateral triangle, each angle is 60 degrees.
QTS is isosceles triangle hence angle QTS = 47degrees.
Angle RQS =60 degrees.
(a) 2x + 2y + 4
Y is difference of both these angles hence 13
(b) 2x + 2y + 8 125. In which one of the following countries will the no
(c) 4x + 4y + 8 confidence motion to bring down the government
(d) 4x + 4y + 16 passed by the legislature be valid only when the
legislature is able to find simultaneously a
Ans. (d) : Length of the rectangular lawn plot = X m. majority to elect successor government?
and width = Y m. (a) France (b) Germany
(c) Italy (d) Portugal
Ans. (c) : In Italy, the government can fall only when
the legislature can choose a successor government by
getting a majority immediately. The no-confidence
motion is considered passed only when the formation of
an alternative government is certain. This type of
provision is in Italy.
126. In the series POQ, SRT, VUW, ____, the blank
space refers to:
(a) XYZ (b) XZY
(c) YXZ (d) YZX
:: . Area of rectangular lawn = xy metre Square.
Ans. (c) : The order of the series is:
Length of the plot along the path = ( x +4 ) m .
Width of plot along the path = (Y + 4) m. [R is the next letter after Q]
Area of the plot along the path = ( x +4 ) ( y + 4 )
= xy + 4y + 4x + 16 sq. m. [U is the next letter after T]
::: Area of path = Area of plot with path - Area of plot
without path [X is the next letter after Q]
= [(xy + 4y + 4x + 16 ) – xy]
= 4x + 4y + 16 So,
= 4x + 4y + 16 sq.km.

IAS (Pre) GS 1997 Paper I 524 YCT


127. There are three drawers in a table. One
contains two gold coins, another two silver
coins and the third, a silver coin and a gold
coin. One of the drawers is pulled out and a
coin is taken out. It turns out to be a silver coin
What is the probability of drawing a gold coin,
if one of the other two drawers is pulled out
next and one of the coins in it is drawn at
random?
The predominant languages spoken in the
(a) 37.5% (b) 50%
areas marked A, B,C and D are respectively :
(c) 62.5% (d) 75%
Ans. (a) : According to above instruction after a silver (a) Brajbhasha, Bhojpuri, Maithili and Awadhi
coin pulled out reamed two drawers in table is gold (b) Awadhi, Maithili, Bhojpuri and Brajbhasha
coin. For finding the silver coin only drawer 1 and 3 (c) Brajbhasha, Awadhi, Bhojpuri and Maithili
remains in consideration because the open drawer in (d) Maithili, Bhojpuri, Brajbhasha and Awadhi
any case cannot be the drawer that have only gold coins.
Ans. (c) : According to the given map the area marked
1
Now the probability of next coin being a gold coin = ‘A’ is Mathura area of Uttar Pradesh, where Brajbhasha
2
is spoken. ‘B’ is marked by Faizabad of U.P., and the
= 50%.
language spoken is ‘Awadhi’. ‘C’ area is ‘Arah’ area of
128. In the following figure.
Bihar where ‘Bhojpuri’ is spoken. ‘D’ refers to north
Bihar where ‘Maithili’ is predominantly spoken
131. The number of students in two sections, A and
B having different heights is shown in the
following Table:
High Number of students with
P is 300 km Eastward of O and Q is 400 km North of O.R is exactly in the middle of Q and P. The distance between Q and R is:
(in meters) that height
(b) 250 km (c) 300 km
(d) 350 km (a) in section A in section B
Ans. (a) : OP2+OQ2
1.55 3 2
( 300 ) + ( 400 )
2 2
= 1.60 7 6
= 1002 ( 32 + 42 ) 1.62 12 14
1.65 15 14
= 1002 × 52 1.68 8 9
= 100 × 5 = 500 1.71 6 5
R is being in the midway of PQ so QR = 250 km. 1.75 3 4
129. When three coins are tossed together the
probability that all coins have the same face up The ration of the number of students of a
is: particular height in Section A to that in Section
1 1 B is the maximum for height of
(a) (b)
3 6 (a) 1.55 m (b) 1.60 m
1 1 (c) 1.65 m (d) 1.71 m
(c) (d)
8 12 Ans. (a) : Ratio of students of section A to B is
Ans. (c) : Probability of Head or Tail on the upper side maximum for 1.55 category height, it is 3/2 = 1.55
for a coin = 1/2
132. The world's fastest computer has been able to
∴ Probability of same side on the upper side
perform (as of Dec. 1996)
for the (a) 106 operations per second
3

Three coins = × × =   =
1 1 1 1 1 (b) 109 operations per second
2 2 2 2 8 (c) 1012 operations per second
130. Consider the map give below: (d) 1015 operations per second

IAS (Pre) GS 1997 Paper I 525 YCT


Ans. (c) : As of Dec. 1996 Hitachi Machine, the SR 136. The length, breadth and height of a room are'/'
2201 made by the Hitachi corporation of Japan was the 'b' and 'h' respectively. The perimeter of the
ceiling expressed as a percentage of the total
fastest computer and was able to perform more than area of the four walls is:
220 billion mathematical operations a second, which is (a) 100 h (b) 100/h
almost equal to 1012 operations per second. (c) h (d) h/100
133. The Basque separatist organisation is active in: Ans. (b) : The area of room covered by 4 walls of
length ‘L’ breadth ‘b’ and height ‘h’ is.
(a) Russia (b) Cyprus
A = 2 [l + b] × h
(c) Portugal (d) Spain Whereas perimeter of ceiling is 2 (l + b)
Ans. (d) : The Basque conflict, also known as the Spain– Perimeter of ceiling
ETA conflict, was an armed and political conflict from So, ×100
Area of the room
1959 to 2011 between Spain and the Basque National 2(l + b)
Liberation Movement, a group of social and = ×100
political Basque organizations which sought independence 2(l + b) × h
from Spain and France. 100
=
134. The following table shows the percent change h
in the amount of sales (in rupees) at different 137. Seven persons P, Q, R, S, T U and V participate
retail stores in a given neighbourhood market in and finish all the events of a series of swimming
in the period 1993 to 1995: races. There are no ties at the finish of any of the
events. V always finishes somewhere ahead of Q.
Retail Store Percent Change Either R finishes first and T finishes last or S
finishes first and U or Q finishes last. If in a
1993 to 1994 1994 to particular race V finished fifth, then which one of
1995 the following would be true?
(a) S finishes firsts (b) R finishes second
Anshu +10 (c) T finishes third (d) R finishes fourth
Borna –20 +9 Ans. (a) : According to question the right sequence is
Calpo +5 +12 First ⇒S
Dilip –7 –15 Second ⇒ U
Elegant +17 –8 Third ⇒ R
Fourth ⇒ T
Fifth ⇒ V
If the sales at Anshu Store amounted to Rs. 8 Sixth ⇒ P
Lakhs in 1993. Then the amount of sales (in Seventh ⇒ Q
lakhs of rupees) at that store in 1995 was: So S finishes first hence correct option is (a).
(a) 7.92 (b) 8.00 138. Consider the following:
(c) 8.80 (d) 9.68 1. Saxena, David, Jain and Kumar were District
Ans. (a) : If there are two changes of X% & Y% Collectors at places P, Q, R and S respectively
XY in 1970.
Then net changes = X + Y + 2. In 1972 they were transferred. Saxena and
100 Jain interchanged places. Kumar and David
10 × ( –10 ) also interchanged places.
Net Change = 10 – 10 + = –1% 3. One year later in 1973, they were again
100
transferred such that David and Jain
 1 
Amount of sales in 1995 = 8  1 –  = 7.92 interchanged places and Saxena and Kumar
 100  were also interchanged.
Hence option (a) will be correct. What should be the next round of transfers so
135. If the numbers representing volume and that all the four persons could have been posted
at all the four places?
surface area of a cube are equal, then the (a) Interchange Saxena and David as well as Jain
length of the edge of the cube in terms of the and Kumar
unit of measurement will be: (b) Interchange Saxena and Kumar as well as
(a) 3 (b) 4 David and Jain
(c) 5 (d) 6 (c) Interchange David and Kumar as well as
Ans. (d) : Surface area of cube = 6a2 [a = side of cube] Saxena and Jain
Volume of cube = a3 (d) It is not possible for all the four persons to
So, as per question, have been posted at all the four places
Surface Area of cube = volume of cube Ans. (c) : When David and Kumar interchange their
positions and Saxena and Jain interchange their
6a2 = a3 positions, the four people will be able to be stationed at
So, a = 6 all four places.
IAS (Pre) GS 1997 Paper I 526 YCT
139. In a group of travelling in a bus, 6 persons can 1km
speak Tamil, 15 can speak Hindi and 6 can [5 − V] =
15
speak Gujarati. In that group none can speak hour
any other language. If 2 persons in the group 60
can speak two languages and one person can 60
5−V =
speak all the three languages, then how many 15
persons are there in the group? 5 − V = 4km / hour
(a) 21 (b) 22 V = (5 − 4) km / hr
(c) 23 (d) 24 V = 1km / hr
Ans. (d) : Total number of Hindi, Tamil and
Gujarati speaking people = 15 + 6 + 6 = 27 143. A survey was conducted on a sample of 1000
Two persons of this group can speak 2 languages while persons with reference to their knowledge of
one can speak 3 languages. English. French and German. The results of
Whereas restriction is that no person of this group can the survey are presented in the given Benn
speak any other language. diagram. The ratio of the number of the
persons who do not know any of the three
Hence, number of persons = 27 – 3 = 24. languages to those who know all the three
140. Examine the following statements regarding a languages is
set of balls:
1. All balls are black
2. All balls are white
3. Only some balls are black
4. No balls are black
Assuming that the balls can only be black or
white, which two of the statements given above
can both be true, but cannot both be false?
Choose the correct answer from the codes
given below:
(a) 1 and 4 (b) 1 and 3 1 1
(a) (b)
(c) 2 and 3 (d) 2 and 4 27 25
Ans. (a) : As per the statement given above the 1 175
condition for ball being only black or only be white is. (c) (d)
(i) All balls are black which means there is no white 550 1000
balls and all are black balls. Ans. (b) : Total number of persons who know only
(iv) No balls are black means all balls are white. English or French or German = 200+ 180 +170 = 550
141. Daily weather map showing isobars is an The number of persons who know any two languages =
example of: 105 + 85 +78 = 268
(a) Choropleth map The number of persons who know all the three
languages. = 175
(b) Isopleth map
∴ The number of persons who know any of the
(c) Chorochromatic map languages = 550+268+175 = 993
(d) Choroschematic map Then :- The number of persons who do not know any of
Ans. (b) : Isopleths is a line on map connecting places the language = 1000 – 993 = 7
registering the same amount or ratio of some
7 1
geographical phenomenon or phenomena. The example Then the ratio of know not any languages = =
of isopleths is daily weather map showing Isobars. 175 25
142. A boat which has a speed of 5 km/hr in still know all language.
water crosses a river of width 1 km along the 144. Consider the figures given below:
shortes possible path in 15 minutes. The
velocity of the river water in km/hr is:
(a) 1 (b) 3
(c) 4 (d) 41
Ans. (a) : The speed of boat is 5 km/hr in still water = U
The velocity of river water = V
As, boat is going against the stream of river water– [U –V]
distance
Speed =
time
1km
[U − V] = In the given sequence of figures, the figure that
15min ute
would logically appear at SI no. 5 is

IAS (Pre) GS 1997 Paper I 527 YCT


Ans. (b) : The flight landing airports while travelling
from Delhi to Tokyo are as follow.
Bangkok → Hong kong → Hanoi → Taipei
(a) (b) So, correct sequence will be 4, 2, 1, 3.
148. Antigen is a substance which:
(a) Lowers body temperature
(b) Destroys harmful bacteria
(c) Triggers the immune system
(c) (d) (d) Iss used as an antidote to poison
Ans. (c) : The term antigen originally referred to a substance
that is an antibody generator. The presence of antigens in the
Ans. (a) : According to above figure analysis the fifth body may trigger an immune response. Antigens can be
number figure is - proteins, peptides, polysaccharides and lipids.
149. The number of times in a day the Hour-hand
and the Minute-hand of a clock are at right
angles is:
(a) 44 (b) 48
(c) 24 (d) 12
Hence option (a) is correct answer. Ans. (a) : (i) Minute hand moves 360 degrees in 60
145. The sequencing of the entire genome (the minute.
totality of all genes) of an organism was So, angle is let say ‘6t’[t is number of minute past
completed in 1996. The organism was: midnight]
(a) Albino mouse (b) Yeast (ii) The hour hand moves 30 degree in 60 minute.
(c) Human being (d) Plasmodium vivax So, angle is ‘0.5t’
Ans. (c) : The sequencing of all the entire genome (the The first time they make a 90° degree angle is when the
totality of all genes) of an organism was completed in
minute hand has moved 90 degree further than hour
1996. The organism was none other than human being.
hand.
Human whole genome genome sequencing (WGS) offers
the most detailed view into our genetic code. WGC has the So
ability to evaluate every base in the genone and navigate 6t = 0.5t + 90 ……..(i)
the complexity of genomic variants that make us unique. .90 4
t= t = 16
146. Which one of the following is NOT a principle 5.5 11
of "Panch-sheel"? Next time when the minute hand has gained another
(a) Non-alignment
180° on the hour hand and is 90° behind it.
(b) Peaceful Co-existence
6t = 0.5t + 270
(c) Mutual respect for each other's territorial
integrity and sovereignty 270 1
t= = 49 ……..(ii)
(d) Mutual non-interference in each other's 5.5 11
internal affairs
 1 4 8
Ans. (a) : The five principles of peaceful coexistence, So every  49 − 16  = 32 minutes
also known as panchsheel, were mentioned in the  11 11  11
preamble of the Sino Indian Agreement, 1954. For every 180° that minute hand gains on the hour hand
The five principles are as follows: there will be one 90° angles.
(i) Mutual respect for each other's territorial integrity So far every 24 hour, which is 1440 minutes the hour
and sovereignty. hand and minute hand at right angle will be.
(ii) Mutual non-aggression.  32.8 
(iii) Mutual non-interference in each other's internal 1440   = 44 times
 11 
affairs.
(iv) Equality and mutual benefit. 150. The Badami rock inscription of Pulakesin I is
(v) Peaceful co-existing. dated in the Sake year 465. If the same were to
147. During a flight from Delhi to Tokyo the be dated in Vikrama Samvat, the year would be:
following are the landing airports: (a) 601 (b) 300
1. Hongkong 2. Hanoi (c) 330 (d) 407
3. Taipei 4. Bangkok Ans. (a) : As Saka era started in 78 AD while Vikram
The correct sequence of the landing at these era started from 58 BC. So if we want to convert.
airports during an onward journey is Badami Rock inscription of Pulakesin I to Vikram era,
(a) 1, 2, 3, 4 (b) 4, 2, 1, 3 then
(c) 3, 4, 1, 2 (d) 4, 1, 2, 3 465 + 78 + 58 = 601 Vikram Samvat.
IAS (Pre) GS 1997 Paper I 528 YCT
UNION PUBLIC SERVICE COMMISSION
Civil Services (Preliminary Exam) - 1996
GENERAL STUDIES : PAPER-I
Time: 2 hours Maximum Number: 200

Ans. (d): Chanakya (Kautilya) also known as


ANCIENT HISTORY Vishnugupta wrote the famous text known as
Arthashastra in which he mentioned divorce to a wife
1. The river most mentioned in early vedic deserted by her husband.
literature is:
(a) Sindhu (b) Shutudri 6. Who among the following is known for his
(c) Saraswati (d) Ganga work on medicine during the Gupta period?
(a) Saumilla (b) Sudraka
Ans. (a): In early vedic literature many rivers have
(c) Shaunaka (d) Sushruta
been mentioned however river sindhu is mentioned
frequently. Ans. (d): Sushruta is known for his work on
medicine. He was famous surgeon. The work is
2. In the context of ancient India Society which attributed to Sushruta, a historical physician of 6th
one of the following terms does not belong to the century BCE, although the text as preserved dates to
category of the other three? the 3rd or 4th century CE. It is one of the three
(a) Kula (b) Vamsa foundational texts of Ayurveda (Indian traditional
(c) Kosa (d) Gotra medicine).
Ans. (c): In the context of Indian Society the terms
7. Match List-I with List-II and the correct
‘Kula’, ‘Vamsa’ and Gotra are related with family
answer:
while word ‘Kosa’ is related to wealth.
List-I List-II
3. Which one of the following was a Shaiva sect in A. Visakhadutta 1. Medicine
ancient India? B. Varahmihira 2. Drama
(a) Ajivika C. Charaka 3. Astronomy
(b) Mattamayura D. Brahmagupta 4. Mathematics
(c) Mayamata Codes:
(d) Ishana Sivaguru Deva-Paddhati A B C D A B C D
Ans. (b): Mattamayura was Shaiva sect in ancient (a) 1 3 4 2 (b) 2 1 3 4
India mentioned in Cheti’s inscriptions. (c) 2 3 1 4 (d) 3 4 1 2
Ans. (c): Visakhadatta – Drama
4. Which of the following were common to both Varahmihira – Astronomy
Buddhism and Jainism? Charaka – Medicine
1. Avoidance of extremities of penance and Brahmagupta – Mathematics
enjoyment.
2. Indifference to the authority of Vedas. MEDIEVAL HISTORY
3. Denial of efficacy of rituals.
4. Non-injury to animal life. 8. The Sufi saint who maintained that devotional
Select the answer using the codes given below: music was one way of coming close to god was
(a) 1, 2, 3 and 4 (b) 2, 3 and 4 (a) Muin-Ud-din Chishti
(c) 1, 3 and 4 (d) 1 and 2 (b) Baba Farid
Ans. (b): Avoidance to extremities of enjoyment and (c) Saiyad Muhammad Gesudaraz
penance was not common in Buddhism and Jainism. (d) Shah Alam Bukhari
Jain doctrine supported extremities of penance. Ans. (a): The Suif saint Muin-Ud-din Chishti
established Chishti Silsila in India. He maintained
5. Which one of the following texts of ancient that devotional music was one war of coming close to
India allows divorce to a wife deserted by her God. He worked in Ajmer.
husband.
(a) Kamasutra 9. Prem Vatika, poems on the life of Krishna,
(b) Manavadharmashastra was composed by:
(c) Sukra Nitisara (a) Bihari (b) Surdas
(d) Artha-Shastra (c) Raskhan (d) Kabir
IAS (Pre) GS 1996 Paper I 529 YCT
Ans. (c): The poetry Prem Vatika was composed by Ans. (a): The above description is related to Danish
Raskhan. His tenure matched with the reign of settlements at Tamil Nadu Coast. They established
Mughal emperor Akbar. He also composed ‘Sujan their settlements at Tamil Nadu and Serampore.
Raskhan’. However commercially they were not successful.

10. In Medieval India, Mansabdari system was 14. Who among the following was the First
introduced mainly for European to initiate the policy of taking part in
(a) Making recruitment to the army the quarrels of Indian princes with a view to
(b) Facilitating revenue collection acquire territories.
(c) Ensuring religious harmony (a) Clive (b) Dupliex
(d) Effecting clean administration (c) Albuquerque (d) Warren Hastings
Ans. (d): The profounder of Mansabdari system was Ans. (b): The First European to interfere in quarrels
Changez Khan, however in medieval period the of Indian princes was French Governor Dupleix. He
system was implemented in India by Emperor Akbar. interfered in quarrels of Hyderabad and Arcot.
This system was based on decimal system and perhaps
it was started by Akbar in 1575 and its objective was 15. His ‘principal forte was social and religious
the making effective military administration and for reform. He relied upon logistics to do away
the territorial command to sustain the parts of the with social ills and worked unceasingly for the
army. eradication of child marriage, the purdah
system etc. To encourage consideration of
11. Mughal Painting reached its zenith under: social problems on a national scale, he
(a) Humayun (b) Akbar inaugurated the Indian National Social
(c) Jahangir (d) Shahjahan conference, which for many years met for its
Ans. (c): During the Jahangir period Mughal painting annual session along site the India National
reached its zenith because in his court there were Congress. The reference in this passage is
many famous painters like Vishandas, Manohar, Nadir to________
Hussain, Mansoor etc. Jahangir also took very keen in (a) Ishwar Chandra Vidyasagar
the painting. (b) Behramji Merwanji Malabari
(c) Mahadev Govind Ranade
MODERN (d) B.R. Ambedkar
Ans. (c): Mahadev Govind Ranade was social
12. Which one of the following pairs is correctly reformer of Maharashtra. He Founded Prathna Samaj
matched? in 1867. He worked for widows. He was also related
(a) Guru Amar Das – Miri and Piri to establishment of Deccan Education Society. He
(b) Guru Arjun Dev – Adi Granth founded India Nations Social Conference in 1887.
(c) Guru Ram Das – Dal Khalsa
16 The following advertisement
(d) Guru Gobind Singh – Manji
THE MARVEL OF THE CENTURY
Ans. (b): Guru Arjundev made collection of the holy THE WONDER OF THE WORLD
book of Sikhs called ‘Adi Grantha’ in 1604, this LIVING PHOTOGRAPHIC PICTURES
comprises of the teachings of 5 Sikhs gurus, 18 Hindu IN
priest and Saints and also teachings of Kabir, Farid, LIFE-SIZED REPRODUCTIONS BY
Namdev, Raidas etc. MESSERS LUMIERE BROTHERS
CINEMATOGRAPHIE
13. “You might see a few carious Danes around, A FELI EXHIBITIONS WILL BE GIVEN
but that is because ______ used to be Danish AT
WATSON’S HOTEL TONIGHT
outpost. The quaint town with its Fort and a
Appeared in the Times of India dated 7th
beautiful church, the new Jerusalem, empty August
streets and deserted beachfront is a quaint (a) 1931 (b) 1929
gem”. The place referred to in this quotation lies (c) 1921 (d) 1896
on the: Ans. (d): It was an advertisement of first Indian
(a) Tamil Nadu Coast (b) Kerala Coast cinema. It was organised in 1896 at Watson Hotel of
(c) Karnataka Coast (d) Goa Coast Bombay.

IAS (Pre) GS 1996 Paper I 530 YCT


17. Consider the following landmarks in Indian Ans. (c):
education: 1. First session of National conference at Calcutta-
1. Hindu College, Calcutta 1883.
2. University of Calcutta 2. Foundation of Muslim League Dacca – 1906
3. Adam’s Report 3. Formation of All India States Peoples conference –
1927
4. Wood’s Dispatch 4. Announcement of Communal Award from White
The correct chronological order of these Hall – 1932
landmarks is -
(a) 1, 3, 4, 2 (b) 1, 4, 3, 2 20. Match List-I with List-II and the correct
(c) 3, 1, 4, 2 (d) 3, 2, 4, 1 answer:
List-I List-II
Ans. (a): Hindu College Calcutta – 1817 A. Governor General 1. Archibald Percival
University of Calcutta – 1857 of Presidency of Wavell, viscount
Adam’s Report – 1835 Fort William in and Earl; Wavell
Wood’s Dispatch – 1854 Bengal (Under
Regulating Act,
18. Match List-I with List-II and the correct 1773)
answer: B. Governor-General 2. James Andrew Broun
List-I List-II of India (Under Ramsay, Earl and
Charater Act, Marquess of
A. Abhinav Bharat 1. Sri Aurobindo Ghosh
1833) Dalhousie
Society C. Governor-General
B. Anushilan Samiti 2. Lala Hardayal and Viceroy of India 3. Charles Cornwallis
C. Ghadar Party 3. C.R. Das (Under Indian 2nd Earl and First
D. Sawaraj Party 4. V.D. Savarkar Councils Act, 1858) Marquess of
Codes: Cornwallis
D. Governor-General 4. Gibert John Elliot
A B C D A B C D
and Crown Murray-Kynymond
(a) 4 1 3 2 (b) 1 4 3 2 Representative Earl of Mintos.
(c) 1 4 2 3 (d) 4 1 2 3 (Under Government 5. Louis Mountbatten
Ans. (d): Abhinav Bharat – V.D. Savarkar of India Act, 1935) Earl Mountbatten
Society of Burma
Anushilan Samiti – Sri Aurobindo Ghosh Codes:
A B C D A B C D
Ghadar Party – Lala Hardayal
(a) 3 2 4 1 (b) 3 4 1 5
Sawaraj Party – C.R. Das (c) 2 3 4 5 (d) 4 2 3 1
Ans. (b):
19. Match List-I with List-II and the correct
A. Governor General Charles
answer: of presidency of Cornwallis 2nd
List-I List-II Fort William in Earl and first
(Period) (Exent) Bangal (Under Marquess of
A. 1883 1. Announcement of Regulating Act, Cornwallis
communal Award 1773)
from White Hall. B. Governor-General of Gibert John
India (Under Elliot Murray-
B. 1906 2. Formation of the all
Charater Act, Kynymond Earl
India states people 1833) of Mintos.
conference. C. Governor - Archibald
C. 1927 3. Foundation of General and Percival
Muslims league Viceroy of India Wavell,
Dacca. (Under Indian viscount and
D. 1932 4. First session of Councials Act, Earl; Wavell
National conference 1858)
at Calcutta. D. Governor-Grneral Louis
and Crown Mountbatten
Codes: Earl
Representative
A B C D A B C D (Under Mountbatten of
(a) 4 3 1 2 (b) 3 4 1 2 Government of Burma
(c) 4 3 2 1 (d) 3 4 2 1 India Act, 1935)
IAS (Pre) GS 1996 Paper I 531 YCT
21. Who among the following leaders did not 25. Match List-I with List-II and the correct
believe in the drain theory of Dadabhai Naoroji? answer:
(a) B.G.Tilak List-I List-II
(b) R.C. Dutt A. Surendra Nath 1. Hind Sawaraj
(c) M.G. Ranade Banerjee
(d) Sir Syed Ahmed Khan
B. M.K. Gandhi 2. The Indian struggle
Ans. (d): Sir Syed Ahmed Khan remain faithful to C. Subhash Chandra Bose 3. Autobiographical
English rule in India till the last. So he did not believe
writings
in the Drain Theory of Dada Bhai Naoroji.
D. Lajpat Rai 4. A Nation in making
22. The Anarchical and Revolutionary Crime Act Codes:
(1919) was popularly known as the: A B C D A B C D
(a) Rowlatt Act (b) Pitt’s India Act (a) 4 1 3 2 (b) 1 4 3 2
(c) Indian Arms Act (d) Ilbert Bill (c) 4 1 2 3 (d) 1 4 2 3
Ans. (a): The Anarchical and Revolutionary Crime Ans. (d):
Act 1919 was also called Rowlatt Act. This Act Surendra Nath – A Nation in making
provided the government to sent any Indian in prison Banerjee
without any trial and punishment. Thus the objective Subhash Chandra Bose – Hind Sawaraj
of Rowlatt Act was to suppress Indian nationalist. Lajpat Rai – Autobiographical
23. The meeting of Indian and British political writings
leaders during 1930-32 in London has often M.K. Gandhi – The Indian struggle
been referred to as the first, second and third 26. Consider the following statements about
round table conferences. It would be incorrect Jawaharlal Nehru:
to refer them as such because: 1. He was the president of the congress party in
(a) The Indian National congress did not take part 1947.
in two of them 2. He presided over the constituent Assembly.
(b) Indian parties other than Indian National 3. He formed the first congress ministry in
Congress participating in the conference United Province before India’s
represented sectional interests and not the whole independence.
of India. Of these statements-
(c) The British Labour Party had withdrawn from (a) 1, 2 and 3 are correct
the conference thereby making the proceeding (b) 1 and 3 are correct
of the conference partisan (c) 1 and 2 are correct
(d) It was an instance of a conference held in three (d) none is correct
session and not that of three separate Ans. (d): President of congress party in 1947 was J.B.
conferences Kriplani
Ans. (d): British Government organised Round Table • Dr. Rajendra Prasad president over Constituent
conferences in London for discussion on Assembly.
recommendations of Simon Commission. These • The first congress ministry in United Provinces
conferences were organised in 3 session, thus were was formed under leadership of Govind Ballabh Pant.
not different conferences but different sessions of a
27. Which one of the following is not correct
same conference.
about the cabinet mission plan?
24. Who among the following was a prominent (a) Provincial grouping
leader of the congress socialist party? (b) Interim cabinet of Indians
(a) M.N. Roy (c) Acceptance of Pakistan
(b) Ganesh Shankar Vidyarthi (d) Constitution Framing right
(c) Pattom Thanu Pillai Ans. (c): Cabinet Mission rejected the Muslim
(d) Acharya Narendra Dev League’s demand of Pakistan. Initially Muslim
Ans. (d): Jai Prakash , Narayan Acharya , Narendra League accepted the demand for Pakistan, however
Dev and Meenu Masani were the prominent founders late on it rejected and gave call of Direct Action Day.
of congress socialist party. It was established in 28. Consider the following statements:
October 1934 at Bombay. The non-cooperation movement led to the-
IAS (Pre) GS 1996 Paper I 532 YCT
1. Congress becoming a mass movement for the In the context of above two statements, which
first time one of the following is correct?
2. Growth of Hindu-Muslim unity. (a) Both A and R are true and R is the correct
3. Removal of fear of the British ‘might’ from explanation of A
the minds of the people. (b) Both A and R are true but R is not a correct
4. British government willingness grant
explanation of A
political concessions to Indians of these
statements (c) A is true but R is false
(a) 1, 2, 3 and 4 are correct (b) 2 and 3 are correct (d) A is false but R is true
(c) 1 and 3 are correct (d) 3 and 4 are correct Ans. (d): The last Governor General of free India was
Ans. (b): In non cooperation movement there was appointed by British sovereign. However British
growth of Hindu-Muslim unity in initial phase, sovereignty did not exist in free India.
however later on condition worsened after Moplah
rebellion. 33. Which one of the following first mooted idea of
• Due to non-cooperation movement, fear of British a constituent assembly to frame a constitution for
‘might’ was removed from the minds of people. India?
• Congress became mass movement during (a) Sawaraj Party in 1934
Swadeshi Andolan. (b) Congress Party in 1936
Thus option (b) is correct. (c) Muslims League in 1942
(d) All parties conference in 1946
29. Who among the following suggested the
winding up of the Indian National congress Ans. (a): The idea of the constituent Assembly to
after India attained independence? Frame a constitution for India was given for the first
(a) C. Rajagopalchari (b) Acharya Kriplani time in 1934 by the Sawaraj Party.
(c) Mahatma Gandhi (d) Jayaprakash Narayan 34. B.R. Ambedkar was elected to the constituent
Ans. (c): Gandhiji opined that aim of congress was to Assembly from:
achieve independence, which was achieved. Thus he (a) West Bengal (b) Bombay Presidency
suggested winding up of congress. (c) Madhya Bharat (d) Punjab
30. Of the four form of political protests mentioned Ans. (b): B.R. Ambedkar was a resident of Bombay
below, which one is derived from the name of the Presidency, but was elected to constituent Assembly
person who used it as a political weapon for the from West Bengal.
first time?
(a) Boycott (b) Gherao ART & CULTURE
(c) Banth (d) Hartal
Ans. (a): Captain Charles Cunningham Boycott was a 35. Which one of the following is not a part of
British land agent whose ostracism by the local early Jain literature?
community in Ireland as part of a campaign for (a) Therigatha (b) Acarangasutra
agrarian tenants' rights in 1880 gave the English (c) Sutra Kritanga (d) Brihatkalpsutra
language the verb to boycott, meaning "to ostracise".
Ans. (a): Therigatha is part of Buddhist literature
31. The Sarabandi (no tax) campaign of 1922 was led where as rest all are part of early Jaina literature.
by
(a) Bhagat Singh (b) Chittaranjan Das 36. Which one of the following sculptures
(c) Rajguru (d) Vallabhbhai Patel invariable used green schist as medium?
Ans. (d): The Sarabandi (no tax) compaign of 1922 in (a) Maurya sculptures (b) Mathura sculptures
Bardoli, Gujarat was led by Vallabhbhai Patel against (c) Bharhut sculptures (d) Gandhara sculptures
increase of land tax.
Ans. (d): Bharhut sculptures used green schist as the
32. Given below are two statements, one labeled as medium means in this sculpture Green schist
Assertion (A) and the other labeled as Reason metamorphic rocks were used. In Mathura sculptures,
(R):
red sandstone was used. In Gandhara Sculptures,
Assertion (A) : The British sovereignty
continued to exist in Free India bluish-grey sandstone was used. The Mauryan,
Reason (R) : The British sovereign appointed sculptures were rock-cut sculptures. In Maurya,
the last Governor General of sculptures buff-coloured fined grained hard sandstone
Free India and spotted red and white sandstone were used.

IAS (Pre) GS 1996 Paper I 533 YCT


37. Which one of the following temples figured the INDIAN GEOGRAPHY
news regarding the institution of the Devadasis?
(a) Jagannath temple, Puri 42. The Palk Bay lies between:
(b) Pashupatinath temple, Kathmandu (a) Gulf of Kachchh and Gulf of Khambat
(c) Kandariya Manadev temple, Khajuraho (b) Gulf of Mannar and bay of Bengal
(d) Chausath Yogini temple, Bhedaghat (c) Lakshadweep and Maldive Islands
Ans. (a): Jagannath temple situated in Puri, Orissa (d) Andaman and Nicobar Islands.
(Odisha) was in news for institution of Devadasis. Ans. (b): Palk bay lied between Gulf of Mannar and
This was a kind of religious practice in which a girl in Bay of Bengal which Gulf of Kachchh and Bay of
her puberty period was dedicated to worship and Bengal lies in Arabian sea near Gujarat Coast,
Andaman and Nicobar Islands are situated in Bay of
service of a deity or a temple for the rest of her life by
Bengal. Lakshadweep and Maldives Island are
her parents.
situated in Arabian sea near Kerala coast.
38. The term ‘apabhramsa’ was used in medieval
43. Consider the following rivers:
Sanskrit texts denote
1. Kishenganga 2. Ganga
(a) Outcastes among the Rajputs 3. Wainganga 4. Penganga
(b) Deviations from Vedic rituals The correct sequence of these rivers when
(c) Early forms of some of the modern Indian arranged in North-South direction is
languages (a) 1, 2, 3, 4 (b) 2, 1, 3, 4
(d) Non-Sanskrit verses (c) 2, 1, 4, 3 (d) 1, 2, 4, 3
Ans. (c): Apabhramsa is a terms used since time of Ans. (a): Kishenganga is tributary of river Jhelum.
patanjali to refer to languages spoken in north India Ganga originates from Himalaya. Wainganga river
before rise of the modern languages. originates from Mahadeo hils in Madhya Pradesh
Penganga originates from Sahayadri hills in
39. Which one of the following is an important Maharashtra.
historical novel written during latter half of the
nineteenth century? 44. Which one of the following rivers thrice forks
(a) Rost Goftar (b) Durgesh Nandini into two streams and reunites a few miles
(c) Maharatha (d) Nibandhamala farther on, thus forming the islands of
th Srirangappattanam, Sivasamudram and
Ans. (b): In the late of 19 century the important
Srirangam
historical novel was written by Bankim Chandra
(a) Cauvery (b) Tungabhadra
Chatterjee called Durgesh Nandini. This novel was
(c) Krishna (d) Godavari
based on the fight between Mughal Pathans for
Ans. (a): Cauvery rivers thrice forks into two streams
possession over Orissa.
and reunites a few miles farther on, thus forming the
40. Which one of the following pairs is correctly islands of Srirangapatanam, Sivasamudram and
matched? Srirangam.
(a) Naggal – Bihar (b) Tamasha – Orissa 45. In the India context the term De-notified tribes
(c) Ankia Naat – Assam (d) Baha – Punjab refers to:
Ans. (c): Naggal – Punjab (a) tribes which are aboriginals
Tamasha – Maharashtra (b) nomadic tribes
Ankia Nat – Assam (c) tribes practicing shifting cultivation
Baha – Jharkhand (d) tribes which were earlier classified as
criminal tribes
41. Nastalgia was: Ans. (d): British notified some tribes as criminal
(a) a Persian script used in medieval India tribes, however after independence they were
(b) a raga composed by Tasen Denotified Tribes.
(c) a cess levied by the Mughal rulers
46. ‘Saddle Peak’ the highest peak of Andaman
(d) a manual code of conduct for Ulemas
and Nicobar Islands is located in:
Ans. (a): It was a Persian script during Mughal (a) Great Nicobar (b) Middle Andaman
period. (c) Little Andaman (d) North Andaman

IAS (Pre) GS 1996 Paper I 534 YCT


Ans. (d): Andaman and Nicobar Islands are located in Ans. (b): Due to heating of Tibetan plateau, low
Bay of Bengal. Saddle peak which is highest peak of pressure is created in Indian sub-continent which
Islands is located in North Andaman. leads to in blow of South West monsoon Winds.

47. Which one of the following is an important 51. Local supply of coal is not available to:
crop of the Barak Valley? (a) TISCO, Jamshedpur
(a) Jute (b) Tea (b) VSL, Bhadravati
(c) Sugarcane (d) Cotton (c) HSL, Durgapur
(d) HSL, Bhilai
Ans. (b): Tea is the important economic activity and
Barak Valley have also its proportional share of tea Ans. (b): Bhadravati steel plant does not have supply
garden is comparison to Brahmaputra valley to at coal from local area. Here coal is obtained by
sustains its economy form time to time. There are burning wood of trees, also it has supply at electric
power from sivasamudram water falls.
plenty of oil and natural gas under the surface of
Barak valley as well to run the economy as a separate 52. Match List-I with List-II (States where they are
state. mined) and the correct answer:
List-I List-II
48. Which one of the following area of India
A. Manganese 1. Madhya Pradesh
produces largest amount of cotton
B. Nickel 2. Orissa
(a) North-Western India and Gangetic West Bengal C. Lead-zinc 3. Rajasthan
(b) North-Western and Western India D. Asbestos 4. Andhra Pradesh
(c) Western and Southern India Codes:
(d) Plains of Northern India A B C D A B C D
Ans. (b): In India Gujarat produces most of the (a) 1 3 2 4 (b) 4 3 2 1
(c) 1 2 3 4 (d) 4 2 3 1
cotton. Maharashtra, Tamil Nadu, Karnataka Madhya
Ans. (c):
Pradesh, Punjab and Haryana are other important
Ores State
cotton producing area. Manganese – Madhya Pradesh
Nickel – Orissa
49. Which of the following pair of states and their
Lead-zinc – Rajasthan
important crops are correct matched?
Asbestos – Andhra Pradesh
1. Kerala – Tapioca
2. Maharashtra – Cotton 53. Consider the map given below:
3. W. Bengal – Jute
4. Gujarat – Groundnut
(a) 1, 2 and 3 (b) 1, 2 and 4
(c) 1, 3 and 4 (d) 2, 3 and 4
Ans. (c):
State Important Crop
1. Kerala – Tapioca (First Position)
2. Maharashtra – Cotton (2nd position, 1st
is Gujarat)
3. W. Bengal – Jute (First position)
4. Gujarat – Groundnut
(First position)

50. High temperature and low pressure over the The dotted (broken) line in the map is the-
Indian sub-continental during the summer season (a) Durand line
draw air from Indian ocean leading to in blowing (b) Mac Mohan line
of the (c) Line of demarcation between India and
(a) South-East monsoon Pakistan suggested by the Boundary
(b) South-West monsoon Commission (1947)
(c) Trade Winds (d) Route followed by the young husband
(d) Westerlies expedition
IAS (Pre) GS 1996 Paper I 535 YCT
Ans. (a): The dotted line in the map shows Durand WORLD GEOGRAPHY
line which is the separation line between India and
Afghanistan. Mac Mohan line marks boundary 57. Diamond Ring is a phenomenon observed:
between India and China. Young husband expedition (a) At the start at total solar eclipse
(b) At the end of solar eclipse
was sent for Tibet during tenure of viceroy Curzon.
(c) Only along the peripheral regions of totality
54. Which of the following pairs are correctly trail
matched? (d) Only in the central regions of totality trail
1. Idukki : Thermal Power station Ans. (c): Diamond ring is a phenomenon associated
with the solar eclipse when only the peripheral parts
2. Sabarigiri : Hydro-electric project
of the sun are visible and sun look like a diamond
3. Ghatprabha : Irrigation project
ring. Thus diamond ring is witnessed during the solar
4. Ramganga : Multipurpose project eclipse only along peripheral regions of the totality
(a) 2, 3 and 4 (b) 1, 2, 3 and 4 trails.
(c) 3 and 4 (d) 1 and 2
58. Given below are two statements one labeled as
Ans. (a): Idukki is hydroelectric project of Kerala,
Assertion (A) and the other labeled as Reason
thus not correct, rest all are correctly matched.
(R):
Assertion (A) : Area near the equator receive
CENSUS rainfall throughout the year.
Reason (R) : High temperature and high
55. As per 1991 census, the average size of humidity cause convectional
households in terms number of persons per rain in most afternoon near the
household in respect of the given states equator.
follows the sequence (highest first, lowest In the context of the above two statements which
last): one of the following is correct?
(a) Haryana, Uttar Pradesh, Kerala, Tamil Nadu (a) Both A and R are true and R is the correct
explanation of A
(b) Uttar Pradesh, West Bengal, Gujarat, Kerala
(b) Both A and R are true and R is not the
(c) Gujarat, Haryana, Uttar Pradesh, West Bengal
correct explanation of A
(d) Uttar Pradesh, West Bengal, Tamil Nadu, (c) A is true but R is false
Kerala (d) A is false but R is true.
Ans. (b): As per 1991 census on basis of number of Ans. (a): Areas lying within 5-8 latitude on either side
persons per household average size of households is of equator receive conventional rainfall due to high
highest in Uttar Pradesh followed by West Bengal, temperature and high humidity throughout the year.
Thus both the statements are correct and Reason is
Gujarat and Kerala. correct explanation of Assertion.
56. As per the 1991 census, which one of the 59. A rough outline map of northern Sri Lanka is
following states has a lower population shown in the given figure Jaffna is located at the
density than other three? point marked
(a) Sikkim (b) Nagaland
(c) Meghalaya (d) Manipur
Ans. (a): As per census 1991 and 2011 population
density of above states are
State Population Population
Density Density
1991 2011
Sikkim 57 86 (a) A (b) B
Nagaland 73 119 (c) C (d) D
Meghalaya 79 132 Ans. (d): Point D marked in above map shows the
Manipur 82 115 exact location of Jaffna.

IAS (Pre) GS 1996 Paper I 536 YCT


60. The rough outline map shows a portion of the Ans. (b): Both statements are correct but do not
Middle-East. The countries labeled A, B, C explain the assertion. At present the Minorities
and D respectively Commission has a statutory status. 103rd constitutional
Amendment Bill provides for granting constitutional
status to minorities commission. However the Bill has
not passed by the Houses of Parliament.

63. Consider the following statements:


No one can be compelled to sing the National
Anthem since
1. It will be violative of Right to Freedom of
speech and expression.
(a) Syria, Iraq, Jordan and Saudi Arabia 2. It will be violative of the Right to Freedom of
(b) Syria, Iraq, Saudi Arabia and Jordan conscience and practice and propagation of
(c) Iraq, Syria, Saudi Arabia and Jordan religion.
(d) Iraq, Syria, Jordan and Saudi Arabia 3. There is no legal provision obliging any one to
sing the National Anthem.
Ans. (c): The countries marked in the map as A, B, C
(a) 1 and 2 are correct
and D are Iraq, Syria, Saudi Arabia and Jordan
(b) 2 and 3 are correct
respectively.
(c) 1, 2 and 3 are correct
61. “They are Fantastically diverse. They speak (d) none is correct
hundreds of languages and dialects. They Ans. (c): In India no one can be compelled to sing the
comprise scores of ethnic groups. They national anthem because under Article 19(1) (a) the
include highly industrialized economics and Freedom of speech and expression also includes right
upcoming economies. They span half of the to keep silence. However it is duty to show respect for
surface of the earth and are home to two- National Anthem but there is no legal provision
fifths of the world’s population”. The group of obliging constitution of India.
countries referred to here belongs to:
(a) S.A.F.T.A. (b) A.P.E.C. 64. Which one of the following is part of the
(c) E.C. (d) C.I.S. electoral college for the president but, not the
forum for his impeachment?
Ans. (b): The said diversity is found in A.P.E.C (Asia
(a) Lok Sabha
Pacific Economic Co-operation). The group includes
(b) Rajya Sabha
21 countries of North America, South America, Asia
(c) State Legislative Councils
and Australia.
(d) State Legislative Assemblies
Ans. (d): According to Article 54, electoral college of
INDIAN CONSTITUTION AND POLITY
president includes elected members of Lok Sabha,
62. Given below are two statements, on e labeled as Rajya Sabha and State legislative Assemblies. Under
Article 61, members of State Legislative Assemblies
Assertion (A) and the other labeled as
do not take part in the process of impeachment of the
Reason (R) president.
Assertion (A) : The word minority is not
defined in the constitution of 65. Which one of the following statement about the
India. duties of Prime Minister are correct?
Reason (R) : The minorities commission is (a) Is free to choose his minister only from
not a constitutional body. among members of either house of
parliament.
In the context of above two statements, which
(b) Can choose his cabinet after consulting the
one of the following is correct?
president of India.
(a) Both A and R are true and R is the correct
(c) Has full discretion in the choice of person
explanation of A
who are serve as his ministers in his cabinet
(b) Both A and R are true and R is not the
(d) Has only limited power in the choice of his
correct explanation of A
cabinet colleagues because of the discretionary
(c) A is true but R is false
powers vested in the president of India.
(d) A is false but R is true
IAS (Pre) GS 1996 Paper I 537 YCT
Ans. (c): Prime Minister has complete discretion to Ans. (c): Under Article 236 of the constitution, the
choose his ministers in the cabinet not necessarily term “District Judge” includes judges of a district civil
from the two houses of parliament but can also choose court, additional district judge, joint district judge,
any other person. That person should become member assistance district judge, chief judge of a small cause
of either house within 6 months from the date he court, chief presidency magistrate additional chief
enters office. presidency magistrate, session judge and assistance
sessions judge and additional sessions judges.
66. If the number of seats allocated to state in the
Lok Sabha is 42, then the number of seats 70. What is the system of governance in the
Panchayati Raj set up?
reserved for the scheduled castes in that state
(a) Single tier structure of local self government
will be
at the village level
(a) 21 (b) 14
(b) Two tier system of local self government at
(c) 7 (d) 6 the village and block levels
Ans. (*): Incomplete question. In this question not (c) Three tier structure of local self government
data is provided for percentage of SC population. at the village block and district levels
That’s why the number of reserved seats for SCs (d) Four tier system of local self government at
cannot be predicted. the village, block district and in the state
levels.
67. The power of the Supreme Court of India to Ans. (c): According to the 73rd Amendment Act, three
decide disputes between the centre and the tier system of Panchayat exists: Village Panchayat at
state falls under its: village level, District Panchayat at district level, the
(a) Advisory Jurisdiction Intermediate Panchayat which stands between the
(b) Appellate Jurisdiction village and District Panchayats in the states where the
(c) Original Jurisdiction population is above 20 lakhs.
(d) Constitutional Jurisdiction
Ans. (c): According to Article 131, The SC has ECONOMY
original jurisdiction in any dispute- (a) between the
71. A redistribution of income in a country can be
Government of Indian & one or more states; or (b)
best brought about through
between the Governmentt of India and any state or
(a) Progressive taxation combined with progressive
states on the side and one or more other states, or (c) expenditure.
between two or more states: (b) Progressive taxation combined with regressive
expenditure.
68. When the Chief Justice of High Court acts in an
(c) Regressive taxation combined with regressive
administrative capacity, he is subject to: expenditure.
(a) The writ jurisdiction of any other judges of (d) Regressive taxation combined with progressive
the High Court. expenditure.
(b) Special control exercised by the Chief Ans. (a): Progressive taxation combined with
Justice of India. progressive expenditure. It is the best way
(c) Discretionary powers of the governors of the redistribution of income in a country.
state. Redistribution of income and wealth is transfer of
(d) Special powers provided to the Chief income and wealth (including physical property) some
Minster. individuals to other through a social mechanism. Such
as taxation, welfare, public services lender form
Ans. (a): Article 226 of the constitution empowers of
monetary policies, confiscation, divorce etc.
a High Court to issue writs.
Also when the chief justice of a High Court acts in an 72. Which one of the following is correct regarding
administrative capacity, he is subject to the writ stabilization and structural adjustment as two
jurisdiction of any other judges of the High Court. components of the new economic policy adopted
in India.
69. According to the constitution of India the (a) Stabilization is a gradual, multi step process
term district judge shall not include: while structural adjustment is a quick adaption
(a) Chief Presidency Magistrate process
(b) Session Judges (b) Structural adjustment is a gradual multi-step
(c) Tribunal Judge process, while stabilization is a quick adoption
(d) Chief judge of a small cause court process.
IAS (Pre) GS 1996 Paper I 538 YCT
(c) Stabilization and structural adjustment are very 75. Hawala transactions relate to payments
similar and complimentary policies. It is (a) received in rupees against overseas
difficult to separate one from other currencies and vice versa without going through
(d) Stabilization mainly deals with a set of policies the official channels
which are to be implemented by the central (b) received for sale/transfer of shares without
government while structural adjustment is to be going through the established stock exchanges
set in motion by the state governments. (c) received as commission for services
Ans. (a): New Economic Policy was adopted in 1991 rendered to overseas investors/buyers/ sellers in
based on Rao-Manmohan model. Stablization assisting them to get over the red tape and/or in
component of any economy is essentially a shorter getting preferential treatment
term programme while the structural readjustment (d) made to political parties or to individuals for
component is a long term process. meeting election expenses
Ans. (a): Hawala is an illegal method of remittance
73. Assertion (A): An important policy instrument of across countries. There are money brokers who are the
economic liberalization is reduction in import middlemen who undertake hawala transfers. This
duties on capital goods. method of remittance does not involve the physical
Reason (R): Reduction in import duties would movement of cash. It is also known as Hundi. The
help the local entrepreneurs to improve word Hawala means trust. The Hawala system works
technology to face the global markets. as it is based on mutual trust between the hawala
In the context of the above two statements, agents. It works outside the banking system and legal
which one of the following is correct ? financial systems. The remittance happens based on
(a) Both A and R are true and R is the correct communication between the hawala agents. It is an
explanation of A alternate to the traditional remittance system.
(b) Both A and R are true and R is not a correct 76. The Eighth FiveYear Plan is different from the
explanation of A earlier ones. The critical difference lies in the fact
(c) A is true but R is false that
(d) A is false but R is true (a) it has a considerably larger outlay compared
Ans. (a): Both A and R are true and R is the correct to the earlier plans
explanation of A. (b) it has a major thrust on agricultural and rural
74. One of the important goals of the economic development
liberalisation policy is to achieve full (c) considerable emphasis is placed on
convertibility of the Indian rupee. This is being infrastructure growth
advocated because (d) industrial licensing has been abolished
(a) convertibility of the rupee will stabilize its Ans. (a): Eighth Five Year Plan (1992-97) had a
exchange value against major currencies of the bigger outlay with energy being given 26.6% of total
world outlay to a cheque a targeted growth rate of 6.78% per
(b) it will attract more foreign capital inflow in annum.
India
(c) it will help promote exports 77. In India, rural incomes are generally lower than
(d) it will help India secure loans from the the urban incomes. Which of the following
world financial markets at attractive terms reasons account for this ?
Ans. (b): The full convertibility of the Indian I. A large number of farmers are illiterate and
currency means that the rupee is freely exchangeable know little about scientific agriculture.
into other international currencies and vice versa. II. Prices of primary products are lower than
Also, this would mean that international investors can those of manufactured products.
buy and sell Indian assets at will. After 1994, the
III. Investment in agriculture has been low when
rupee has been partially convertible which means that
the currency is changed freely into foreign currency compared to investment in industry.
for business and trade expenses. But it cannot be Select the correct answer by using the codes
converted freely for acquiring overseas assets. Experts given below :
feel full convertibility of rupee would facilitate Codes :
growth and higher foreign investments. (a) I, II and III (b) I and II
Hence option (b) is correct . (c) I and III (d) II and III
IAS (Pre) GS 1996 Paper I 539 YCT
Ans. (a): In India, rural incomes are generally lower Items Years
than the urban incomes, because large number of 1992-93 1993 1994-
farmers are illiterate and know little about scientific -94 95
agriculture, Prices of primary products are lower than (Percentag
those of manufactured products and Investment in e to total)
agriculture has been low when compared to Agriculture & 16.9 18.0 15.9
investment in industry. allied products
78. Assertion (A): Though India’s national income Ores A Minerals 4.0 4.0 3.7
has gone up several fold since 1947, there has Manufactured 75.5 75.6 78.0
been no marked improvement in the per capita foods
income level. Petroleum 2.6 1.8 1.9
Reason (R): Sizeable proportion of the Products
population of India is still living below the The changing composition of the export trade is
poverty line. indicative of structural transformation of Indian
In the context of the above two statements which economy in favour of modernisation. The best
one of the following is correct ? indicator of the trend is the
(a) Both A and R are true and R is the correct (a) relative shape of petroleum products in
explanation of A exports
(b) Both A and R are true but R is not a correct (b) decline in the share of agricultural products
explanation of A in exports
(c) A is true but R is false (c) constant share of ores and minerals in exports
(d) A is false but R is true (d) increase in the share of manufactured
Ans. (d): Sizeable proportion of the population of products in exports
India is still living below the poverty line. Ans. (d): The changing composition of the export
Hence option (d) is correct answer. trade is indicative of structural transformation of
79. Which one of the following sets of commodities Indian economy in favour of modernisation. The best
are exported to India by arid and semiarid indicator of the trend is the increase in the share of
countries in the Middle East ? manufactured products in exports.
(a) Raw wool and carpets
(b) Fruits and palm oil PHYSICS
(c) Precious stones and pearls
(d) Perfume and coffee 82. Consider the following statements: A person in
Ans. (b): Fruits and palm oil are exported to India by a spaceship located half way between the
arid and semi arid countries of Middle East. earth and the sun will notice that the:
1. Sky is jet black
80. Consider the following items imported by India :
2. Stars do not twinkle
I. Capital goods
3. Temperature outside the spaceship is much
II. Petroleum
higher than that on the surface of earth
III. Pearls and precious stones
(a) 3 alone is correct (b) 1 and 2 are correct
IV. Chemicals
(c) 1 and 3 are correct (d) 1, 2 and 3 are correct
V. Iron and Steel
Ans. (d): There is a presence of atmosphere at earth’s
The correct sequence of the decreasing order of
surface which consists of thick and moving layers of
these items (as per 94-95 figures), in terms of
air. The dust particles, particulates, smog, water
value, is
vapour and smoke are also present in air. Stars twinkle
(a) I, II, III, IV, V
when we see them from the Earth’s surface because
(b) I, II, IV, III, V
we are viewing them through thick layer of turbulent
(c) II, I, III, IV, V
(moving) air in the Earth’s atmosphere. As their light
(d) II, I, IV, V, III
travels through the many layers of the Earth’s
Ans. (b): According to 1994&95 data that items India atmosphere, the light of the star is bent (reflected)
imported which decreasing order is - I, II, IV, III, V many times in random directions (light is bent when it
81. The table given below depicts the composition of hits a change in density-like a pocket of cold air or hot
India’s exports between 199293 and 199495 : air). This random reflection results in the star as its
IAS (Pre) GS 1996 Paper I 540 YCT
twinkling but for a person in spaceship no such Ans. (a): As
refraction are possible as there is vaccume in space.
α = UT + 1 gt 2
Thus, star will not twinkle. 2 d
Similarly, sky will appear blue to us due to Rayleigh U = 0 (given)
scattering which is again due to presence of α = 1 gt 2
2 t
atmosphere on earth’s surface. The shorter
wavelength light is absorbed by the gas molecules of α ∝ +2
atmosphere. The absorbed blue light is then radiated
86. Optical Fibre works on the principle of:
in different directions. It gets scattered all around the
(a) total internal reflection
sky. Some of this scattered blue light reaches you.
(b) interference
Since, you see, the sky looks blue. Whereas no
(c) scattering
atmosphere is there in space thus, no absorbing and
(d) interference
scattering is possible that’s why for a person in
spaceship sky appears black. Ans. (a): An optical fibre is a thin, flexible,
Temperature outside the spaceship is higher in transparent fibre that acts as waveguide or “light pipe”
comparison to earth’s surface because of being nearer to transmit light between the two ends of the fibre. An
to sun. optical fibre transmits light along its axis by the
process of total internal reflection. When light
83. When a mirror is rotated by an angle θ, the traveling in a dense medium hits a boundary at an
reflected way will rotate by angle larger than the “critical angle” for the boundary,
(a) 00 (b) θ/2 the light will be completely reflected. This effect is
(c) θ (d) 2θ used in optical fibres to confine light in the core.
Ans. (d): Consider a plane and a fixed incident ray of
87. Given below are to statements, one labeled as
light. Before the mirror has rotated, the angle of
assertion (A) and other labeled as reason (R)
incidence is θ as is the angle of reflection. If the
Assertion (A): Transformer is useful for
mirror is rotated through an angle ϕ the normal is stepping up or stepping down voltages.
rotated by an angle f and thus the angle of incidence Reason (R): Transformer is a device used in
increases to θ + ϕ. Therefore, the angle of reflection D.C. circuits. In the context of above two
statements, which one of the following is correct?
must also increase by ϕ to θ + ϕ. The difference
(a) Both A and R are true and R is the correct
between the final angle of reflection and the initial
explanation of A
angle of reflection is 2ϕ. Thus for a fixed incident ray (b) Both A and R are true and R is not the
the angle of the reflection is twice the angle through correct explanation of A
which the mirror has rotated. (c) A is true but R is false
(d) A is false but R is true
84. Which one of the following layers of atmosphere
is responsible for the deflection of radio waves. Ans. (c): A transformation is a device that transfers
(a) Troposphere (b) Stratosphere electrical energy from one circuit to another through
(c) Mesosphere (d) Ionosphere inductively coupled conductor-the transformer’s coils.
Ans. (d): Ionosphere is the top layer of atmosphere. A varying current in the first or primary winding
Radio waves are deflected in the ionosphere of the creates a varying magnetic flux in the transformer’s
atmosphere ionosphere is composed of DE and F core and thus a varying magnetic field through the
layers. D layers is good at absorbing AM radio waves. secondary winding. This varying magnetic field
induces a voltage in the secondary winding. By
85. The variation of displacement d with time t in the appropriate selection of the ratio of turns, a
case of a particle falling freely under gravity transformer thus allows an alternating current (AC) or
from rest is correctly shown in Graph: voltage to be “stepped up” by making Ns greater than
Np or “stepped down” by making Ns less than Np.
d d Here Np represents number of turns in primary
(a) (b) winding. Ns represents the number of turns in
t t secondary winding. As it is clear from principle
involved that transformer can be used to step up and
d d and step down A.C. voltage only. This device and not
(c) (d) be used for D.C. voltage as its working is based on the
t t change of magnetic flux with varying current.

IAS (Pre) GS 1996 Paper I 541 YCT


88. Domestic electrical wiring is basically a: CHEMISTRY
(a) Series connection
(b) Parallel connection 92. The alpha particle carries two positive charge.
(c) Combination of series and parallel connection Its mass is very nearly equal to that of
(d) Series connection within each room and (a) two protons
parallel connection elsewhere (b) an atom of helium
(c) sum of masses of two positrons and two
Ans. (b): In a parallel circuit, the voltage across each
neutrons
of the components is the same, and the total current is
(d) Two positrons as can positron carries a
the sum of currents through each component. The single positive charge
wiring for most homes is parallel. Ans. (b): Each Alpha particle contains two protons
89. A simple machine helps a person in doing: and two neutrons.
(a) less work Total number of nucleons = Number of protons +
(b) the same amount of work with lesser force Number of neutrons
(c) the same amount of work slowly =2+2=4
(d) the same amount of work much faster. Total number of nucleons = mass number of an
element
Ans. (b): A simple machine is advice that changes the
Thus mass number of alpha particle is 4 which is
direction or magnitude of a force. In general, they can
equal to molecular weight of helium atom
be defined as the simplest mechanisms that use
mechanical advantage to multiply force. Thus, simple 93. Which one of the following statements is
machine helps us in doing same amount of work with alloyed with iron to produce steel which can
lesser force. Few examples of simple machines are resist high temperatures and also have high
pulkey, lever, wheel, screw, etc. hardness and abrasion resistance?
(a) Aluminum (b) Chromium
90. When an air bubble at the bottom of lake
(c) Nickel (d) Tungsten
rises to the top it will
(a) increase in size (b) decrease in size Ans. (b): Steel is an alloy of iron and carbon on
(c) maintain its size (d) flatten into disc like mixing with carbon its strength and toughness got
shape increased. In order to make it further more
temperature and abrasion resistant chromium is
Ans. (a): When air bubble is at bottom of lake there
will be more atmospheric pressure on it and when it generally mixed with iron. It also increases its
will come at the surface of water, there will be lower hardness and load bearing capacity. Stainless steel is a
pressure on it and thus it will be increased in size. category of steel consists (75%) Fe 18% Cr and and
Ni is corrosion resistant.
91. A liquid is flowing in an streamlined manner
through a cylindrical pipe. Along a section 94. Living organism require at least 27 elements, of
containing the axis of the pipe, the flow profile which 15 are metals. Among these, those
will be: required in major quantities include.
(a) Potassium, Manganese, Molybdenum and
calcium
(a) (b) (b) Potassium, Molybdenum, Copper and
calcium
(c) Potassium, Sodium, Magnesium, and
calcium
(c) (d) (d) Sodium, Magnesium, Copper and
manganese
Ans. (a): If a fluid flows such that its velocity at a Ans. (c): Living organism require about 27 elements
point is always the same in magnitude and direction, of which 15 are metals K, Mg, Na, and Ca are
the fluid is said to have a streamline flow. The type of required in major. Minor quantity of Mn, Fe, Co, Cu,
flow in a fluid system is characterized by the presence Zn and Mo and trade amount of Cr, Sn, Ni and Al are
of laminae, or parallel stream of fluid. A streamline is required by some organism.
a straight or curved path such that tangent to it and Calcium, magnesium and phosphorus are essential
point gives the direction of flow of liquid at that point. parts of the bones and teeth.
Thus, according to question the flow profile is best Calcium is necessary for blood clotting sodium helps
represented by figure (a). in metal activity and transmission of nerve.

IAS (Pre) GS 1996 Paper I 542 YCT


BIOLOGY 99. Which of the following are associated with
Diabetes mellitus, a common disease in
95. Which one of the following sets of conditions is adults?
necessary for a good cultivation of Wheat? 1. Higher sugar level in blood
(a) Moderate temperature and moderate rainfall (b) 2. Lower sugar level in blood
High temperature and heavy rainfall 3. Lower insulin level in blood
(c) High temperature and moderate rainfall 4. Higher insulin level in blood
(d) Low temperature and low rainfall Select the correct answer by using the codes
Ans. (a): Wheat is a temperate crop. It can be grown given below:
well in sub-tropical regions. It requires moderate (a) 2 and 4 (b) 1 and 2
temperature of- 100C to 150C and moderate rainfall of (c) 2 and 3 (d) 1 and 3
50 to 75 cm. Ans. (d): Diabetes Mellitus is called due to deficiency
of insulin hormone. The hormone insulin maintains
96. Physico chemical characteristics of water in the blood sugar in body. Deficiency of insulin in
water sources undergo changes due to human body causes increase in sugar level in blood,
(a) aquatic macrophysics which leads to diabetes mellitus.
(b) aquatic fungi
(c) effluents 100. Consider the following statements AIDS – is
(d) evapotranspiration transmitted
Ans. (c): Aquatic macrophytes, aquatic fungi are 1. By sexual intercourse
natural organism and evapotranspiration is a natural 2. By blood transfusion
process by which plant losses water. Thus being 3. By mosquitoes and other blood sucking
natural they do not result into any change in physico- insects
chemical characteristics of water while effluents 4. Across to placenta
discharges into water bodies by industrial units and (a) 1, 2 and 3 correct (b) 1, 2 and 4 correct
(c) 1, 3 and 4 correct (d) 1 and 3 are correct
domestic sewage results into change in physic-
chemical characteristics like pH, conductivity, Ans. (b): AIDS is a viral disease. The virus is called
HIV. This disease is transmitted by sexual intercourse,
temperatures electrical conductivity, dissolved oxygen
blood transfusion and from pregnant mother to baby
Biological oxygen demand, chemical oxygen demand, through placenta.
NH +4 and NH -3 ion contents etc.
101. Which of the following would lead to
97. It is possible to produce seedless tomato fruits by malnutrition?
(a) applying trace elements in tomato fruits 1. Over nutrition
(b) spraying mineral solution on plants 2. Under nutrition
3. Imbalanced nutrition
(c) spraying hormones on Flowers
Select the correct answer by using the codes
(d) applying fertilizers containing radioactive
given below.
elements. Codes:
Ans. (c): Seedless tomato fruit can be grown without (a) 2 alone (b) 2 and 3
fertilization if the plant growth hormone is sprayed on (c) 1 and 3 (d) 1, 2 and 3
them. This will result in their parthenocarpy. This is Ans. (d): Malnutrition is defined as the insufficient,
the technique of induced parthenocarpy where the excessive or imbalanced consumption of nutrients. So
vegetables are sprayed with plant growth under nutrition, over nutrition and imbalanced
98. Ecologist have so far regulators. nutrition leads to malnutrition.
Show the correct option is 'Spraying hormones 102. People drinking water from a shallow
on flowers'. hand pumps, are likely to suffer from all of the
known, found and identified a large number following disease except.
of species in the plant and animal (a) Cholera (b) Typhoid
kingdom. In terms of numbers, the largest (c) Jaundice (d) Fluorosis
found and identified so far is from among the: Ans. (d): Cholera, Typhoid and Jaundice are
(a) Fungi (b) Plants waterborne disease. In shallow hand pumps there is
(c) Insects (d) Bacteria possibility of microbial contamination (bacteria) to
Ans. (c): In plant and animal kingdom largest number water. But fluorosis occurs due to excess amount of
of species identified are insects. fluorine found in water.
IAS (Pre) GS 1996 Paper I 543 YCT
103. Fat present below the skin surface in our body 107. Besides Proteins and carbohydrates, other
acts as a barrier against elements of nutritional value found in milk,
(a) Loss of heat from body include
(b) Loss of essential body fluids (a) calcium, potassium and iron
(c) Loss of salt from body (b) calcium and potassium
(d) Entry of harmful micro-organisms from the (c) potassium and iron
environment. (d) calcium and iron
Ans. (a): Fat present below the skin surface in our Ans. (b): Besides carbohydrates an protein calcium,
body, acts as a insulator against loss of heat from magnesium an phosphorus is found in milk.
body. The deposited fat works a barrier, which 108. Match List-I (Physiological process) with List- II
prevents loss of heat. (cell organelles) and select the correct answer by
using the codes given below:
104.Which of the following professional(s) are
List-I List-II
more likely to run the risk of permanent A. Photosynthesis 1. Plasma membrane
change in their cell’s DNA? B. Mineral uptake 2. Chloroplast
1. Researchers using carbon 14 isotope C. Respiration 3. Mitochondria
2. X-ray technician D. Protein synthesis 4. Ribosomes
3. Coal miner Codes:
4. Dyer and Painter (a) A-1; B-2; C-3; D-4 (b) A-2; B-2; C-4; D-3
Select the correct answer by using the codes (c) A-2; B-1; C-3; D-4 (d) A-2; B-1; C-4; D-3
given below Ans. (c): Photosynthesis occurs in green plants by
Codes: help of chloroplast plasma membrane helps in
(a) 2 alone (b) 1, 2 and 3 transport of minerals cellular respiration is related to
(c) 1, 2 and 4 (d) 1, 3 and 4 mitochondria, by which energy releases. Ribosome
Ans. (a): X-Ray technique, C14 and Dyer technique is helps in protein synthesis.
very important technique for permanent change in 109. Which one of the following is a modified stem?
DNA. But in coal miner C14 amount is very less so, (a) Carrot (b) Sweet potato
there is no risk of change in DNA. (c) Coconut (d) Potato
Ans. (d): Potato tubers bears buds in small pits known
105.Of the four land marks in medical history as eyes. Buds develops to branches some of the
given below, which one was the first to take branches become green, erect an leafy stems that grow
place? horizontally underground.
(a) Organ Transplant (b) Bypass surgery 110. Which one of the following is not an essential
(c) Test Tube Baby (d) Plastic surgery micronutrient for plants?
Ans. (d): Organ Transplant – 1950 (a) Boron (b) Zinc
Bypass surgery – 1960 (c) Sodium (d) Copper
Test Tube Baby – 1978 Ans. (c): There are 13 essential nutrients required by
Plastic Surgery – 1827 plants for its healthy an proper growth. Now these
106. Match List-I with List-II and the correct answer nutrients are divided into two categories:
Macronutrients (iron, copper manganese, zinc, boron,
by using the codes given below:
molybdenum and chloride) Additional mineral
List-I List-II
A. Vitamin 1. Pepsin nutrient elements which are beneficial but not
B. Enzyme 2. Carotene necessary are sodium, cobalt, vanadium, nickel,
C. Hormone 3. Keratin selenium, aluminum and silicon.
D. Protein 4. Progesterone Thus boron, zinc and copper falls into category of
Codes: essential macronutrients while sodium does not.
(a) A-1; B-2; C-3; D-4 (b) A-2; B-1; C-4; D-3 111. The nutritional deficiency condition the needs to
(c) A-2; B-1; C-4; D-4 (d) A-1; B-2; C-4; D-3 be given top priority for remedial action in India
Ans. (b): Vitamin A group includes retinal. Their today is
parent carotene is a vitamin substance as B-carotene. (a) Scurvy (b) Rickets
Pepsin is an enzyme produced in stomach, which (c) Xerophthalmia (d) Pellagra
digest protein. Ans. (c): Xerophthalmia is caused by deficiency of
Progesterone is a hormone, produced in human ovary vitamin A. Generally this disease is found in poorer
Keratin is a protein. The main component of hair and section of society. The symptom includes high
nail is keratin. blindness and eye irritation.

IAS (Pre) GS 1996 Paper I 544 YCT


ENVIRONMENT AND ECOLOGY CURRENT AFFAIRS
112. Which one of the following regions of India is 115. The following table shows the percentage
now regarded as an ‘Ecological hot spot’? distribution of revenue expenditure of
(a) Western Himalayas (b) Eastern Himalayas Government of India in 1989-90 and 1994-99
(c) Western Ghats (d) Eastern Ghats Expenditure Heat (Percentage total)
Ans. (c): Biodiversity hotspots in India includes Years Defence Interest Subsidies State/UTs
Western Ghats, the Himalayas, Indo-Burma border Grands
and Nicobar Islands (Sundaland hotspot). Hence Payments to others
1989-90 15:1 27.7 16.3 13.6 27.4
option (c) is correct.
1994-95 13.6 38.7 8.0 16.7 23.0
113. Given below are two statements, one labeled as Based on this table, it can be said that the Indian
Assertion (A) and other labeled as Reason (R): economy is in poor shape because the central
Assertion (A) : Mangroves are very specialized government continues to be under pressure to:
forest ecosystems of tropical (a) reduce expenditure on defence
and sub-tropical regions (b) spend more and more on interest payments
bordering certain sea coasts (c) reduce expenditure on subsidies
Reason (R) : They stablise the shoreline and (d) spend more and more as grants-in-aid to
act as bulwark against state government/Union Territories.
encroachments by sea.
Ans. (b): According to the data showing revenues
In the context of above two statements, which
expenditure of Indian government, it can be said that
one of the following is correct?
the Indian economy is in poor shape due to it’s
(a) Both A and R are true and R is the correct
explanation of A increasing dept payments due to burden of debt
(b) Both A and R are true and R is not the payments there remains little scope to utilize this
correct explanation of A money for any other purpose.
(c) A is true but R is false
116. Which one of the following statements is to be
(d) A is false but R is true
launched from India in 1996
Ans. (a): Mangroves are special type of vegetation (a) IRS-P2 (b) IRS-P3
and are found in intertidal regions where freshwater
(c) IRS-EN (d) IRS-ID
and saltwater intermixes in the bays, estuaries, creeps
and lagoons. They are found in tropical and Ans. (b): IRS-P3 was an experimental earth
subtropical regions. They stablise shoreline and act as observation mission undertaken by ISRO. The
bulwark against encroachments by sea. objectives of the mission were processing and
interpretation of data generated by its two payloads,
114. Consider the following programmer: the wide field sensor and Modular Opt electric sensor,
1. Afforestation and development of Wastelands development by the German Aerospace centre. The
2. Reforestation and replantation in existing mission was completed during January 2006 after
forests serving for a years 10 months.
3. Encouraging the wood substitutes and
supplying other types of fuel. 117. Consider the following statements:
4. Promotion of wide use of insecticides and Most international agencies which Fund
pesticides to restrict the loss of forest area Development Programme in India on
from degradation caused by pests and intergovernmental bilateral agreements mainly
insects. include:
It can also effect central nervous system and 1. Technical assistance
liver. 2. Soft loans which are required to be paid back
The National forest policy of 1988 includes: with interest
(a) 1, 2, 3 and 4 (b) 2 and 4 3. Grants, not required to be paid back
(c) 1, 3 and 4 (d) 1, 2 and 3 4. Food assistance to alleviate poverty
(a) 2 and 4 are correct
Ans. (d): National Forest Policy, 1988 describes the (b) 1, 2 and 3 are correct
protection of forest. It does not describe the use of (c) 1, 2 and 4 are correct
insecticide and pesticide in forest areas. (d) 3 and 4 are correct
IAS (Pre) GS 1996 Paper I 545 YCT
Ans. (b): A soft loan is a loan with a below market to reduce tariff barriers, remove trade barriers and
rate of interest. It also includes concession to facilitate international trade in goods and services.
borrowers such as long repayment periods or interest Over the years, GATT held eight rounds of
holidays. multilateral trade negotiations, the last and most
Technical assistance is aid involving highly educated extensive being the Uruguay Round (1986 - 1994).
or trained personnel, such as doctors, who are moved The WTO came into being at Marrakesh in 1995,
into a developing country to assist with the program following the conclusion of the Uruguay Round. After
of development. Food assistance is given to countries this GATT ceased to exists and its legal texts were
in urgent need of food supplies, especially if they have incorporated into WTO as GATT-1994. The eastern
just experienced a Natural disaster. Grant is usually block nations have no special status as WTO member.
given to governments through individual countries,
121. Which of the following were aims behind the
international aid agencies and through multilateral
institutions such as the World Bank and through setting up of the World Trade Organization
development charities. (WTO)?
1. Promotion of free trade and resource flow
118. Consider the following statements: across countries.
Towards the close of 1995, the fortunes of 2. Protection of intellectual property rights.
Indian Hokey were believed to be on the 3. Managing balanced trade between difference
upswing because. countries.
1. India had won the Azlan Shah Cup 1995. 4. Promotion of trade between the former east
2. India beat Pakistan in South-Asian Federation Block countries and the western world.
games final. Select the correct answer by using the codes
3. India had by then qualified for the Atlantic given below:
Olympics. (a) 1, 2, 3 and 4 (b) 1 and 2
(a) 1, 2 and 3 are correct (c) 2 and 3 (d) 1 and 4
(b) 1 and 2 are correct Ans. (a): The WTO has six key objective.
(c) 1 and 3 are correct 1. To set up and enforce rules for international.
(d) 2 and 3 are correct 2. To provide a forum for negotiating and monitoring
Ans. (b): India qualified for Atlanta Olympic in further trade liberalization.
Hockey January 1996 in Barcelona, so till 1995 India 3. To resolve trade disputes.
had not qualified for Atlanta Olympic. 4. To increase the transparency of decision- making
process.
119. According to WHO the disease which causes the 5. To cooperate with other major international
death of the largest number of people today is economic institutions involved in global economic
(a) AIDS (b) Tuberculosis management.
(c) Malaria (d) Ebola 6. To helping developing countries benefit fully from
Ans. (b): As per WHO disease responsible for the global trading system.
maximum death is Tuberculosis. It is caused by
bacteria Mycobacterium tuberculosis. 122. Consider the following statements:
At the present level of technology in India,
120. The emerging trading block in the world, Solar energy can be conveniently used to:
such as NAFTA, ASEAN and the like are 1. Supply hot water to residential buildings.
expected to:
2. Supply water for minor irrigation projects.
(a) Act as constrictions in free trade across
3. Provide street lighting.
world.
4. Electrify a cluster of villages and small towns.
(b) Promote free trade on the lines laid down by
the WTO (a) 1, 2, 3 and 4 are correct
(c) Permit transfer of technology between (b) 2 and 4 are correct
member countries. (c) 1 and 3 are correct
(d) Promote trade in agricultural commodities (d) 1, 2 and 3 are correct
between the countries of North and South. Ans. (a): Solar energy, radiation light and heat form
Ans. (b): World Trade Organization (WTO) is a body the sun has been harnessed by humans since ancient
making global trade rules with binding effect on its times using a range of ever evolving technologies. At
members. It is not only institution, but also a set of present solar energy in our country is used for solar
agreements. The WTO regime is known as rules- water heater. Supply water for minor irrigation
based multilateral trading system. The history of the projects, Provide street lighting and Electrify a cluster
organization dates back to 1947, when the General of villages and small towns. Hence option (a) is
Agreement on Tariffs and Trade (GATT) was set up correct answer.
IAS (Pre) GS 1996 Paper I 546 YCT
123. In the 1996 Cricket World Cup, the non test 129. Examine the following statements-
playing countries which participated included: 1. George attends Music classes on Monday.
(a) UAE, Kenya and Canada 2. He attends Mathematics classes on Wednesday.
(b) UAE, Kenya and Hong Kong 3. His literature classes are not on Friday.
(c) UAE, Kenya and Holland 4. He attends, history classes on the day
(d) Canada, Kenya and Hong Kong following the day of his mathematics classes.
Ans. (c): UAE, Kenya and Holland are the non-test 5. On Tuesday he attends his sports classes.
playing nations who participated in 1996 World Cup. If he attends just one subject in a day and his
Sunday is free, then he is also free on
124.Which one of the following had legalized (a) Monday (b) Thursday
Euthanasia? (c) Saturday (d) Friday
(a) Texas in the USA Ans. (d):
(b) Northern Territory in Australia Day Activity
(c) Quebec in Canada Monday Music
(d) Maharashtra in India Tuesday Sports
Ans. (a): Texas State in U.S.A has legalized Wednesday Maths
Euthanasia. Euthanasia is process in which if a person Thursday History
who is willing to die, he is given peaceful death by the Friday ………
doctor through an injection. Saturday Literature
Friday cannot have literature classes, so it must be on
125. Which one of the following is a modern tank? Saturday as Sunday is free. Only remaining day to be
(a) Bhim (b) Akash free is Friday, so Friday is free.
(c) Arjun (d) Prithvi
Ans. (c): Arjun is the modern tank produced by 130. Two important characteristics of a hypothesis are
Defence Research and Development organization that is should be testable and that it should be
(DRDO). stated in a manner that it can be refute. Which
one of the following hypothesis fulfils these
126.Which one of the following works has recently characteristics?
been made into a full length feature film? (a) Intelligent persons have good memory.
(a) R.K. Narayan’s Malgudi days (b) Some birds are animals
(b) Salman Rushdie’s Midnight’s Children (c) Some businessman are dishonest
(c) Vikram Seth’s A suitable Boy (d) All men are mortal
(d) Upmanyu Chatterjee’s English August. Ans. (a): According to both the characteristics of the
Ans. (d): Director Dev Bengal has made a full length hypothesis, (a) only satisfies the criteria because in
Feature film, starring Rahul Bose, on Upmanyu case of being testable, intelligent person can be teste
Chatterjee’s novel “English August”. positively and in case of refutation also it can be
shown that all the persons, having sound memory are
127. Which one of the following countries had not intelligent. While hypothesis (d) neither is testable
more or less evolved a two party system: not refutable two to its universal truthness, the
(a) Sri Lanka (b) Bangladesh hypothesis (b) can not be refuted because no bird can
(c) Pakistan (d) Myanmar be claimed non animal. In case of hypothesis (c), it is
Ans. (b): Bangladesh is the country which has testable but not refutable.
evolved a two party system. Power rotates between
131. The give pie charts show the proportion of
two parties which are Nationalist Party and Awami
literates and illiterates in a country in the year
League.
1970 and 1990.
Literates
MISCELLANEOUS 30
0
120
0 Literates
0
F M 1050 120
128. Mouse is to cat as Fly is to: F M
(a) rat (b) animal
Illiterates Illiterates
(c) spider (d) horse
Ans. (c): cats eat mouse, similarly spider eats flies. 1970 1990

IAS (Pre) GS 1996 Paper I 547 YCT


And also the proportion of males (M) and 135. In the given figure, the triangle represents
Females (F) among the literates. Which one of girls the square represents sports persons and
the following statements can be said to be true the circle represents coaches. The portion in the
beyond doubt?
figure which represents girls who are sports persons
(a) In 1970 half of the illiterates were women
(Females). but not coaches is the one labeled as
(b) The proportion of literate males to the total
population of males remained the same over F
A
the years. BD E
(c) Males literacy did not improve over this
C
period
(d) The ratio of female literates to male (a) A (b) B
literates improved significantly over this (c) E (d) E
period.
Ans. (b): It is the common area between the triangle
Ans. (c): According to above pie chart very concisely.
and square but excluding the circle i.e. B.
Analyses that male literacy did not over this period.
Hence option (c) is correct answer. 136. The next pair of letters in the series AZ, CX, FU
_________ is
132. Six roads lead to a country. They may be
(a) JQ (b) KP
indicated by letters X, Y, Z and digits 1, 2, 3.
When there is storm, Y is blocked. When (c) IR (d) IV
there are floods X, 1 and 2 will be affected. Ans. (a): First letter in each pair increase by 2, 3, 4.
When road 1 is block, Z also is blocked. At a While the second letter decreases by 2, 3, 4_______
time when there are floods and a storm also
respectively. So 4 letters after F is J and before U is Q.
blows, which road (s) can be used?
Hence, letter is JQ.
(a) Z and 2 (b) only Z
(c) only 3 (d) only Y +2 +3 +4
Ans. (c): Y is blocked for storms. For flood X, 1 and AZ CX FU JQ
2 are blocked. Z is blocked whenever 1 is blocked. So,
-2 -3 -4
for floods and storms, only unblocked road left is 3.
137. The following figure contains three squares
133. A truck, a car and a motor cycle have equal with area of 100, 16 and 49 sq. units
kinetic energies. If equal, stopping forces are respectively laying side by side as shown. By
applied and they stop after travelling a
how much should the area of the middle
distance of X, Y and Z respectively, then:
(a) X > Y > Z (b) X < Y < Z square be reduced in order that the total
(c) X = Y = Z (d) X ≃ 4Y ≃ 8Z length PQ of the resulting three square is 19?
Ans. (c): According to the question the kinetic energy
100 49
of all three vehicles is equal and stopping force is also 16
equal, therefore X = Y = Z because work done by P Q
stopping force = loss of kinetic energy. (a) 12 (b) 4
(c) 2 (d) 2
134. Which one of the following venn diagrams
correctly illustrates the relationship among the Ans. (a): Let the final length of the side of the smaller
classes: carrot, food vegetable? square be a
Now, a + 10 + 7 = 19
(a) (b) A = 19 – 17 = 2
2
∴ Area of the smaller square = (2) = 4
∴ Decrease in the area of the smaller
(c) (d) square = 16 – 4 = 12 units.

Ans. (a): Both carrot and vegetables come under food 138. The average of x1, x2 and x3 is 14. Twice the
category, while carrot comes under vegetable sum of x2 and x3 is 30. What is the value of x1.
category. (a) 20 (b) 27

IAS (Pre) GS 1996 Paper I 548 YCT


(c) 16 (d) 2
x1 + x 2 + x 3
Ans. (b): = 14
3
x1 + x2 + x3 = 42………(i)
x2 + x3 = 42 – x1………(ii) OA2 = AB2 + OB2
2(x2 + x3) = 30 (500)2 = (400)2 + (OB)2
x2 + x3 = 15 OB2 = 25000 – 16000
then, (x1 + x2 + x3 –x2+x3) = 42 – 15 = 27 OB2 = 9000
OB = 300 meter
139. A rectangle has perimeter of 50 meters. If its
142. Consider the following figures:
length is 13 meteres more than its breadth,
then its area is: 9 6
9
(a) 124 m2 (b) 144 m2
4 6
(c) 114 m2 (d) 104 m2 8
Ans. (c): Let the breath be b. (i) (ii) (iii)
Then, length = b + 13 Which one of the following conclusions can be
Perimeter = 50 = 2(l + b) drawn from these figures?
(a) The area of the three figure are all different
2(b + 13 + b) = 50
(b) The area of all three figure are equal
L = 6 + 13 = 19m
(c) The perimeter of the three figures are all equal
Area = length × breath (d) The perimeters of figures I and II are equal.
= 19 × 6 = 114 m2
Ans. (b): Area of rectangle = 9 × 4 = 36
140. Two packs of cards are thoroughly mixed and Area of square = 6 × 6 = 36
shuffled and two cards are drawn at random, Area of triangle = ½ × 9 × 8 = 36
one after the other. What is the probability ∴ Area of all three figure are equal.
that both of them are jacks? 143. The following figure represents sales (is
(a) 1/13 (b) 2/13 thousands). Over the period 1978 to 1983.
(c) 7/1339 (d) 1/169 The sales in 1981 exceeded that in 1979 by
Ans. (c): Total number of cards = 104 = 2×52
and total number of jacks = 8 = 2×4
∴ probability for the jack in first
Draw = 8/104
and probability for the jack in
second draw = 7/103
Since both the events are independent events.
Hence the probability that both of them are jacks
−8 7 7
× = .
104 103 1339 (a) Rs. One hundred (b) Rs. Then thousand
(c) Rs. One lakh (d) Rs. Ten lakh
141. A man starts walking in the north easterly
direction from a particular point. After waling a Ans. (c): Sales in year 1979 = 320,000
distance of 500 meters, he turns South Wards Sales in year 1981 = 420,000
and walks a distance of 400 meters. At the end of Thus the required difference
this walk, he is situated: = 420,000 – 320,000
(a) 300 meters north of the starting point. = Rs. 100,000
(b) 100 meters north-east of the starting point.
144. In an accurate clock, in a period of 2 hours
(c) 300 meters east of the starting point and 20 minutes (140 minutes), the minutes
(d) 100 meters north of the starting point hand will move over.
Ans. (c): According to pythagoras theorem (a) 5200 (b) 3200
0
(c) 840 (d) 1400
Ans. (c): The minute hand will move over in
IAS (Pre) GS 1996 Paper I 549 YCT
in 60 minutes = 3600 Reduction in price to bring it back to its original value
So, angle made by a minute hand 5x x
= − x=
4 4
3600
in a minute = = 60 x/4
60 % Reduction = × 100 = 20%
5x/4
So, in a period of 2 hours 20 minutes the minute hand
will move over 147.If A = x2 – y2, B = 20 and x + y = 10, then
= 6 × 1400 = 8400 (a) A is grater than B
(b) B is grater than A
145. Distance time graph in respect of a race
among four persons is shown in the given (c) A is equal to B
figure. Consider the following statements in (d) It is not possible to compare A and B as the
this regard: data provided is inadequate
Ans. (d): A = x2 – y2 = (x + y) (x – y) = 10(x – y)
C
B B = 20
A Now, it is not possible to compare A and B, as the
Time D value of x and y is not known.
148. When the frequency distribution is normal:
(a) median, mode and mean are all difference
Distance from one another
1. ‘A’ stood first in the race (b) mean, mode and median are identical
2. ‘C’ let all the way (c) mean is greater than mode
3. ‘D’ ran faster than other in later part of the race (d) mean is greater than median
of these statements Ans. (b): When frequency distribution is normal
(a) 1 and 3 are false and 2 is true. mean, median and mode are identical.
(b) 1 and 2 are false and 3 is true. 149. According to ancient Indian cosmogonic ideas
(c) 1 and 3 are true and 2 is false. the sequential order of the cycle of four
acrons (yugas) is:
(d) 1 is true and 2 and 3 are false.
(a) Dwapara, Krita, Treta and Kali
Ans. (b): ‘A’ never stood 1st in the race, it was ‘D’ (b) Krita, Dwapara, Treta and Kali
who stood 1st. From the graph it can be seen that ‘C’ (c) Krita, Treta, Dwapara and Kali
do not lead all the way. While ‘D’ after starting (d) Treta, Dwapara, Kali and Krita
slower, ran faster than others in the later part of the
Ans. (c): A complete Yuga starts with the Satya Yuga
race.
(Krita), Via Treta Yuga and Dwapara Yuga into a Kali
146. If the price of television set is increased by Yuga.
25%, then by what percentage should the new
150. The offending substance in the liquor tragedies
price be reduced to bring the price back to the
leading to blindness etc is-
original level? (a) ethyl alcohol
(a) 15% (b) amyl alcohol
(b) 25% (c) benzyl alcohol
(c) 20% (d) methyl alcohol

(d) 30% Ans. (d): Methyl alcohol is very dangerous and may
be fatal or cause blindness if swallowed. Harmful, if
Ans. (c): Let the original price be x. inhaled or absorbed through skin. It cannot be made
Increased price = x(1 + 25/100) = 5x/4 non poisonous some other harmful effects are results
into irritation of skin, eyes, and respiratory track.

IAS (Pre) GS 1996 Paper I 550 YCT


UNION PUBLIC SERVICE COMMISSION
Civil Services (Preliminary Exam) - 1995
GENERAL STUDIES : PAPER-I
Time: 2 hours Maximum Number: 200

Ans. (d) : The association of "Devanampriya Priyadarsin"


ANCIENT HISTORY with Ashoka was reinforced through various inscriptions,
and especially confirmed in the Minor Rock Edict
1. The word ‘Hindu’ as reference to the people of
inscription discovered in Maski.
Hind (India) was first used by
(a) the Greeks The title "Priyadarsin" is often associated with the title
(b) the Romans "Devanampriya" ("Beloved of the Gods").
(c) the Chinese 5. The term Yavanapriya, mentioned in ancient
(d) the Arabs Sanskrit texts, denoted:
Ans. (d) : The word ‘Hindu’ originates from the (a) A fine variety of Indian muslin
Sanskrit word for river, Sindhu. The Indus River (b) Ivory
running through northwest India into Pakistan received (c) Damsels sent to the Greek court for dance
its names from the sanskrit term sindhu. The Persian performance
designated the land around the river as Hindu, a (d) Pepper
mispronunciation of the Sanskrit sindhu. Later Arab Ans. (d) : Pepper [kali mirch] was one of the most
people influenced by Persian people started calling requiradable thing imported by greek trader in ancient
Hindu as a reference to the people of Hindi (India). India tradition, so in Sanskrit literature it is denoted as
2. Who among the following was a Brahmavadini Yavanpriya.
who composed some hymns of the Vedas? 6. Zero was invented by:
(a) Lopamudra (b) Gargi (a) Aryabhata (b) Varahamihira
(c) Leelavati (d) Savitri (c) Bhaskara I (d) an unknown Indian
Ans. (a) : In the Rig Veda, most hymns are Ans. (d) : Zero was invented by an unknown Indian
composed by male sages. But there are a few by Brahmgupta. The first modern equivalent of numeral
women, the rishikas, some independent and others zero comes from a Hindu astronomer and
wives of sages. Lopamudra, Vishwawara, Nivavari mathematician Brahmagupta in 628. His symbol to
and Sikata were important women sages. depict the numeral was a dot underneath a number.
Lopamudra is the wife of Agastya and both compose 7. Match List I with List II and select the correct
hymns. In her hymn, she expresses her sexual longing answer by using the codes given below the lists.
and asks him to stop his rituals and focus on her. List I List II
Commentators equate Lopamudra with Rati, goddess (Eras) (Reckoned from)
of pleasure. I. Vikrama era (A) 3102 B.C.
3. The concept of Anuvrata was advocated by: II. Saka era (B) 320 A.D.
(a) Mahayana Buddhism III. Gupta era (C) 78 A.D.
(b) Hinayana Buddhism IV. Kali era (D) 58 B.C.
(c) Jainism (E) 248 A.D.
(d) The Lokayata school Codes:
Ans. (c) : The anuvrata was advocated by Jainism. (a) IB, IID, IIIE, IVA
The anuvratas are vows to abstain from violence, (b) IA, IIC, IIIB, IVD
falsehood, and stealing; to be content with one's (c) ID, IIE, IIIB, IVC
own wife; and to limit one's possessions. The other (d) ID, IIC, IIIB, IVA
vows are supplementary and meant to strengthen Ans. (d) : The correct match of List I and List II is :
and protect the anuvratas. Vikram era 58B.C.
4. The name by which Asoka is generally referred Saka era 78A.D
to in his inscriptions is:
Gupta era 320B.C
(a) Chakravarti (b) Dharmadeva
(c) Dharmakirti (d) Priyadarsi Kali era 3102 B.C

IAS (Pre) GS 1995 Paper I 551 YCT


8. Who among the following anticipated Newton by Ans. (c) : Ancient Indian temples are classified in
declaring that all things gravitate to the earth? three broad types. This classification is based on
(a) Aryabhata (b) Varahamihira different architectural styles, employed in the
(c) Buddhagupta (d) Brahmagupta construction of the temples. Three main style of
Ans. (d) : The first modern equivalent of numeral temple architecture are the Nagara or the Northern
zero comes from a Hindu astronomer and
style, the Dravida or the Southern style and the Vesara
mathematician Brahmagupta in 628. His symbol to
or Mixed style.
depict the numeral was a dot underneath a number. He
anticipated that all things gravitate it the earth. Nagara style temple is well developed in the region of
Brahmagupta in AD 628, wrote that "bodies fall north India. The distinctive feature of these type of
towards the earth as it is in the nature of the earth to temple is long shikhara as compared to mandapa.
attract bodies, just as it is in the nature of water to Originally in Nagara style, there were no pillars.
flow". Dravida style advanced in the South, throughout the
9. According to the Mimamsa system of philosophy, Chola Empire, between 9th-12th Century AD. In
liberation is possible by means of: Dravida style temple is situated within an ambulatory
(a) Jnana (b) Bhakti hall. There is multiple storeys called Vimana, built
(c) Yoga (d) Karma above the Garbhagriha. Pillars and pilasters are
Ans. (d) : This philosophy encompasses the Nyaya- massively used in this architectural style.
vaisheshika systems and emphasises the concept of Many temples in Central India and Deccan have used
valid knowledge. According to Purva Mimamsa, the Vesara style with regional modifications. It is
Vedas are eternal and possess all knowledge. It says a fusion style of both Nagara and Dravida styles of
that the essence of the Vedas is dharma. By the temple architecture .Temples built but later Chalukyas
execution of dharma one earns merit which leads one of Kalyani and Hoysalas are considered as the
to heaven after death. Mimamsa philosophy was examples of Vesara style.
founded by Jaimini. According to Karma Mimamsa
system of philosophy, liberation is possible by means MEDIEVAL HISTORY
of action (karma).
10. In Sanskrit plays written during the Gupta 12. Bronze icons of Nataraja cast during the Chola
Period women and sudras speak: period invariably show the deity with:
(a) Sanskrit (b) Prakrit (a) Eight hands (b) Six hands
(c) Pali (d) Sauraseni (c) Four hands (d) Two hands
Ans. (b) : In the Gupta period, the languages used by Ans. (c) : Bronze icons of Nataraja cast during chola
the people of low society level and women was period invariably show the deity with four hands.
prakrit. The word, derived from its Indian root The making of bronze sculptures mainly made from
"Parikrit", itself has a flexible definition, being wax casting reached a high stage of development in
defined sometime as, "original, natural, artless, South India during the Medieval Period. The dancing
normal, ordinary, usual", or "vernacular", in contrast figure of Shiva as Natraja evolved and fully developed
to the literary and religious orthodoxy of Sanskrit. during the Chola Period and since then many variation
Sanskrit language reaches its peak during Gupta of this complex bronze image have been modeled.
period .Many play written during Gupta period were 13. Which one of the following monuments has a
in Sanskrit language. dome which is said to be one of the largest in the
Famous Sanskrit writer in time of Gupta reign were world?
Kalidasa,Vishakhadutta and Sudrak. (a) Tomb of Sher Shah, Sasaram
11. The Nagara, the Dravida and the Vesara are: (b) Jama Masjid, Delhi
(a) The three main racial groups of the Indian (c) Tomb of Ghiyas-ud-din Tughlaq, Delhi
subcontinent (d) Gol Gumbaz, Bijapur
(b) The three main linguistic divisions into which Ans. (d) : Gol Gumbaz is the most famous monument
the languages of India can be classified in Vijayapura (Bijapur). It is the tomb of Mohammed
(c) The three main styles of Indian temple Adil Shah (ruled 1627-1657). It is one of the largest
architecture dome ever built in the world. A particular attraction in
(d) The three main musical gharan as prevalent in this monument is the central chamber, where every
India sound is echoed several times.

IAS (Pre) GS 1995 Paper I 552 YCT


14. Who among the following were famous jurists of 17. The Mughal School of Painting formed the spinal
medieval India? column of the various schools of Indian miniature
I. Vijnanesvara II. Hemadri art. Which one of the following painting styles
III. Rajasekhara IV. Jimutavahana was not affected by Mughal painting?
Choose the correct answer from the codes given (a) Pahari (b) Rajasthani
below: (c) Kangra (d) Kalighata
Codes: Ans. (d) : Kalighata painting was not affected by
(a) I, II and III (b) II, III and IV Mughal school of painting as it originated in the 19th
(c) I, II and IV (d) I and IV century in West Bengal, India in the vicinity of Kalighat
Ans. (c) : Rajashekhar was the Sanskrit poet in the court Kali Temple, calcutta.
of Mahendrapala I. He wrote the Kavyamimasa. It mainly depicted Hindu gods and other Mythological
Vijnanesvara, Hemadri and Jimutavahana were prominent characters, the Kalighat painting developed to reflect a
jurist in Medieval India. variety of subjects, including many depictions of
15. Ashtapradhan was a Council of Ministers: everyday life. Mughal School of painting originated in
(a) In the Gupta Administration the reign of Akbar in 1560 C.E. who was keenly
(b) In the Chola Administration interested in the art of painting and architecture. The
(c) In the Vijayanagar Administration mughal style evolved as a result of a happy synthesis of
(d) In the Maratha Administration the indigenous Indian style of painting and safavid
Ans. (d) : The eight prominent officials of Maratha school of Persian Painting.
were known as Astha Pradhan. There was a prominent affect of Mughal paintings on
They were as follows: Pahari, Rajasthan, Kangra painting styles but it does not
Peshwa – (Prime Minister) affected Kalighata style of paintings.
Amatya or – Finance Minister
Mazumdar MODERN HISTORY
Waqia-Navis – (Home Minister)
Samant or Dabir – (Dealing with the outside) 18. The ‘Modi script’ was employed in the
Sachiv – (The official documents of the:
correspondence) (a) Wodeyars (b) Zamorins
Pandit Rao – (The official religious (c) Hoysalas (d) Marathas
officer) Ans. (d) : Modi script emerged in the 1400 as a
Sar-i-naubat – (army affairs) shorthand variant of the Devanagri used by scribes. It
Nyayadhish – (Judiciary) was used for writing Marathi. This script was once an
16. Consider the, map given below: essential part of Marathi literary culture but currently it
is on verge of extinction and are used on very limited
scale.
19. In the interim government formed in 1946, the
Vice-President of the Executive Council was:
(a) Jawaharlal Nehru
(b) Dr. S. Radhakrishnan
(c) C. Rajagopalachari
(d) Dr. Rajendra Prasad
Ans. (a) : In the interim government formed in 1946,
the vice President of the executive council was
Jawaharlal Nehru.
C. Rajgopalchari was incharge of education department.
The route indicated in the map was followed, Dr. Rajendra Prasad was incharge of food and
during the course of his military exploits, by agriculture department. Dr. S. Radhakrishnan became
(a) Chandragupta II (b) Harshavardhana the 1st Vice President of independent India and later he
(c) Rajendra Chola (d) Malik Kafur became President of India.
Ans. (d) : The route indicated in the map was followed, 20. The radical wing of the Congress Party, with
during the course of his military exploits by Malik Jawaharlal Nehru as one of its main leaders,
Kafur. founded the ‘Independence for India League’ in
As a commander of Alauddin’s force, Malik Kafur opposition to:
defeated the Mongol invaders in 1306 and subsequently, (a) The Gandhi-Irwin Pact
be led a series of expeditions in the southern part of (b) The Home Rule Movement
India, against the Yadavas, the Kakatiyas, the Hoysalas (c) The Nehru Report
and the Pandays. (d) The Montford Reforms
IAS (Pre) GS 1995 Paper I 553 YCT
Ans. (c) : The radical wing of the congress party, with Codes:
Jawaharalal Nehru and Subhash Chandra Bose founded (a) I, III, II, IV (b) IV, I, III, II
the Independence for India league in opposition to the (c) I, II, III, IV (d) IV, III, II, I
Nehru Report. As Nehru Report demanded dominion Ans. (b) : The correct sequence based on chronology
status but they believed in an idea of Poorna Swaraj. are as follows:
21. Which one of the following was an emigree The partition of Bengal – 1905
Communist Journal of M.N. Roy? The Lucknow Pact – 1916
(a) Kisan Sabha (b) The Worker The Rowlatt Act – 1919 (March)
(c) Vanguard (d) Anushilan The Introduction of Diarchy – 1919
Thus option (b) is correct.
Ans. (c) : M.N. Roy was prominent Indian philosopher
25. The word Adivasi was used for the first time to
of a twentieth century. He was famous as the father of
refer to the tribal people by:
Indian Communism and viewed as the first
(a) Mahatma Gandhi (b) Thakkar Bappa
revolutionary leader of India. Vanguard was an emigree
(c) Jyotiba Phule (d) B.R. Ambedkar
communist journal of M.N. Roy.
In 1939 he founded league of radical congressmen. Ans. (b) : The word Adivasi was used for the first time
He wrote famous book like ‘India in transition’ and to refer to the tribal people by Thakkar Bappa. He had
authored the book ‘Tribes of India’ which was
‘The future of Indian Politics’.
published in 1950. He was a firm follower of Mahatma
22. Hughly was used as a base for piracy in the Bay Gandhi and was made the secretary of the Harijan
of Bengal by: Sevak Sangh. He founded the Gond Sevak Sangh in
(a) The Portuguese (b) The French 1944. This organization was later renamed Varanasi
(c) The Danish (d) The British Seva Mandal.
Ans. (a) : Hughly was used as a base for piracy in the 26. The Barrah dacoity was the first major venture
Bay of Bengal by the Portuguese. During the reign of of the revolutionary terrorist of the freedom
Shahjahan in 1631-1632 Governor of Bengal Qasim movement in:
Khan quashed the uprising. (a) Bombay-Karnataka
23. Which one of the following pairs is correctly (b) Punjab
matched? (c) East Bengal
(a) Battle of Buxar ..............Mir Jafar vs. Clive (d) The Madras Presidency
(b) Battle of Wandiwash ...............French vs. East Ans. (c) : The Barrah dacioty was the first major
India Company venture of the revolutionaries of the freedom movement
(c) Battle of Chilianwala ...............Dalhousie vs. in East Bangal in 1908. Pulin Behari Das organised the
Marathas dacoity which was conducted in the day light, when
(d) Battle of Kharda ............... Nizam vs. East India revolutionaries attacked the residence of the Zamindar
Company of Barrah.
Ans. (b) : Battle of Wandiwash was fought between 27. In 1930 Mahatma Gandhi started Civil
French and East India company in 1760 French were Disobedience Movement from:
(a) Sevagram (b) Dandi
defeated heavily by the Sir Eyre Coote led East India
(c) Sabarmati (d) Wardha
Company.
Battle of Buxar was fought in 1764 between the Ans. (b) : On 12th March, 1930 Mahtma Gandhi
combined army of Nawab of Awadh, Mughal Empire launched civil disobedience movement from Sabarmati
and Nawab of Bengal with East India Company led by Ashram. The civil disobedience movement commenced
Hector Munro. with the infamous dandi salt march. Gandhiji broke the
salt laws imposed by the British Government.
Battle of Chilian wala was fought in 1849 between East
India Company and Sikh during the second Anglo-Sikh 28. Which one of the following pairs is not correctly
war. Battle of Kharda was fought between Nizams and matched?
Marathas during 1795. (a) Jamnalal Bajaj...................Satyagraha Ashram at
Thus only option (b) is correctly matched. Wardha
(b) Dadabhai Naoroji.................Bombay Association
24. What is the correct sequence of the following
events? (c) Lala Lajpat Rai ................National School at
Lahore
I. The Lucknow Pact
(d) Bal Gangadhar Tilak.....................Satya Shodhak
II. The Introduction of Dyarchy
Sabha
III. The Rowlatt Act
IV. The Partition of Bengal Ans. (d) :
Satyasodhak Samaj was established by Jyotiba Phule in
Choose the correct answer from the codes given
below: 1873 A.D .
IAS (Pre) GS 1995 Paper I 554 YCT
29. Examine the map given below: The place where there is no thermal power
station is marked.
(a) 1 (b) 2
(c) 3 (d) 4
Ans. (b) : The places marked by 1, 3, 4 have thermal
power station.
1. Ukai thermal power plant (Gujarat)
2. Shrisailam hydroelectric power plant/Andhra Pradesh
3. Korba thermal power plant (Chhattisgarh)
The places marked 1, 2, 3 and 4 were respectively 4. Obra thermal power plant (U.P.)
the seats of powers of the: Place marked by (2) is hydroelectric power plant.
(a) Scindias, Holkars, Gaekwads and Bhonsles 32. In the map given below four areas are differently
(b) Holkars, Scindias, Gaekwads and Bhonsles shaded, three of which indicate cereal crops
(c) Gaekwads, Bhonsles, Scindias and Holkars production areas and one indicates non-cereal
(d) Scindias, Holkars, Bhonsles and Gaekwads crop production area.
Ans. (a) : The place marked 1, 2, 3, 4 were the
respectively the seats of powers of the:
Scindias (Gwalior was the capital of Scindias)
Holkars (Indore was the capital of Holkars)
Gaekwads (Baroda was the capital of Gaekwads)
Bhonsle (Nagpur was the capital of Bhonsle)

INDIAN GEOGRAPHY
30. Consider the map given below:

As per the index given, the non-cereal crop


production area is:
(a) 1 (b) 2
(b) 3 (d) 4
Ans. (d) : From the map are 1, 2, 3 shown are cereals
crops production areas whereas area shown under 4,
The divisions along India’s coastal region are non-cereal crop production area.
indicate. 33. Of the four places marked 1, 2, 3 and 4 on the
(a) Coastal pollution zones map, the one indicating a tidal port is:
(b) Salinity density isopleths
(c) The areas upto which sovereignty extends
(d) Underwater relief contours
Ans. (d) : From the map, the divisions along India’s
coastal region indicate underwater relief contours.
31. In the map given below, three out of the four
places marked indicate places where a thermal
power station is located:

(a) 1 (b) 2
(c) 3 (d) 4
Ans. (a) : Kolkata port is a tidal port situated on the
banks of river Hooghly. It is marked by 1 on map.
Places marked as 2, 3, 4 are Paradip Port (Orissa),
Kochi Port (Kerala), Kandla Port (Gujarat).
34. Which one of the following mountain ranges is
spread over only one State in India?
(a) Aravalli (b) Satpura
(c) Ajanta (d) Sahyadri
IAS (Pre) GS 1995 Paper I 555 YCT
Ans. (c) : Ajanta mountain ranges is spreaded over only 38. The standard time of the following countries is
Maharashtra. ahead or behind Greenwich Mean Time depending
Aravali mountain ranges is spreaded from Gujarat to on whether they are east or west of the longitude
Delhi including Rajasthan State. Guru Shikar is highest passing through Greenwich:
peak of Aravali Mountain ranges. I. Cuba
Satpura mountain ranges is spreaded over Gujarat, II. Greece
Madhya Pradesh, Chhattisgarh and some part of
Jharkhand. Dhupgarh is highest peak of Satpura range. III. Iraq
Sahydari mountain is spread over Maharastra and IV. Costa Rica
Karnataka. V. Japan
Cardamom hills is highest peak of Sahyadri mountain Which one of the following sequential orders
range. gives the correct arrangement of the countries
35. The national highway from Delhi to Calcutta via according to their standard time from ahead to
Mathura and Varanasi is numbered: behind GMT?
(a) 1 (b) 2 (a) V, III, II, I, IV (b) II, IV, I, III, V
(c) 4 (d) 8 (c) IV, I, III, II, V (d) III, V, IV, I, II
Ans. (b) : National Highway 2 commonly reffered as Ans. (a) : As the sun rises in the east and sets in the
Delhi, Kolkata Road is a busy Indian National Highway west. Sunrise occurs in the east of Greenwich Line and
that runs through the state of Delhi Haryana, Uttar
Pradesh, Bihar, Jharkhand and West Bengal. then later in west of Greenwich line. So the time as well
36. Examine the map of Jammu and Kashmir given moves back and forth respectively. The countries
below: located from east to west in the alternative are Japan,
Iraq, Greece, Cuba and Costa Rice.
39. The original home of the gypsies was:
(a) Egypt (b) Russia
(c) India (d) Persia
Ans. (c) : The gypsies are a member of a traditionally
itinerant people who originated in northen part of India.
The gypsies are believed to trace their origin to nomadic
The mountain ranges marked 1, 2, 3 and 4 are communities like the Dom, Banjara, Gujjar Sansi from
respectively the North West parts of India.
(a) Ladakh, Zanskar, Karakoram and PirPanjal 40. Which one of the following is the continent with
(b) Karakoram, Ladakh, Zanskar and PirPanjal the highest mean elevation in the world?
(c) Karakoram, Zanskar, PirPanjal and Ladakh (a) Antarctica (b) North America
(d) Ladakh, PirPanjal, Karakoram and Zanskar
(c) Asia (d) South America
Ans. (b) : From the map of Jammu Kashmir, the Ans. (a) : Antartica also known as white continent is
correct location for 1, 2, 3, 4 are as follows:
Karakoram the fifth largest continent in the world. It is located in
Laddakh Southern Hemisphere. This continent is unique place for
Zanskar science explorartion and scientists. It is the continent
Pir Panjal with the highest mean elevation in the world.
41. The graph given below shows the mean average
WORLD GEOGRAPHY monthly temperatures (in °C) and mean monthly
rainfall (in cms) of a place:
37. Arakan Yoma is the extension of the Himalayas
located in
(a) Baluchistan
(b) Myanmar
(c) Nepal
(d) Kashmir
Ans. (b) : Arakan Yoma mountain range is also known
as the Rakhine mountains located in the region of This graph is indicative of which one of the
Western Myanmar. The specific location of the Arakan climatic zones of the world?
Yoma mountain is between the Irrawaddy river valley (a) Wet and dry tropical
and the Arakan coast. The Arakan Yoma mountains are (b) Rainy tropical
a part of the great Himalayas, specifically speaking they (c) Semi-arid tropical
are a part of the Eastern Himalayas. (d) Temperate marine
IAS (Pre) GS 1995 Paper I 556 YCT
Ans. (a) : The graph shown is indicative of wet and dry Ans. (a) : The correct match for List-I and List-II are.
tropical climate zone of the world. Hence, option (a) is List-I List-II
correct. European transcontinental – Paris to Warsaw
42. Given below is a map of some countries which railway
were parts of the erstwhile Soviet Union, with Trans-Andean railway – Buenos Aires to
water bodies shown by shaded areas : Valparaiso
Trans-Siberian railway – Leningrad to
Vladivostok
Orient Express – Paris to Istanbul
44. Coffee cultivation in Sri Lanka was abandoned
because of the disease.
(a) Leaf blight (b) Leaf spot
(c) Leaf rust (d) Rot
Ans. (c) : Coffee cultivation in Srilanka was banned
The countries marked 1, 2, 3, 4 and 5 are respectively mainly due to leaf rust.
(a) Tajikistan, Turkmenia, Uzbekistan, Kirgizia, Coffee leaf rust is devastating foliar disease of coffee
Kazakhstan plant caused by the fungus Hemileia Vastatrix. The rust
(b) Turkmenia, Kirgizia, Tajikistan, Uzbekistan, pustules are powdery and orange-yellow on the
Kazakhstan underleaf surface.
(c) Kazakhstan, Uzbekistan, Tajikistan, Kirgizia, 45. Match List I with List II and select the correct
Turkmenia answer by using the codes given below the lists:
(d) Kazakhstan, Turkmenia, Uzbekistan, Kirgizia, List I List II
Tajikistan I. Ernesto Zedillo (A) Uruguay
II. Alberto Fujimori (B) Brazil
Ans. (d) : From the given map, the countries marked as III. Julio Maria (C) Mexico
1, 2, 3, 4 and 5 are respectively Kazakhstan, Sanguinetti
Turkmenistan, Uzbekistan, Kyrgyzstan and Tazakistan.
IV. Fernando H. (D) Bolivia
Their capital city are as follows. Cardoso
Kazakstan – Astana
(E) Peru
Turkmenistan – Ashgabat
Codes:
Uzbekistan – Tashkent
(a) I-C, II-E, III-A IV-B
Kyrgyzstan – Bishkek (b) I-D, II-E, III-C, IV-A
43. Match List I with List II and select the correct (c) I-C, II-D, III-V, IV-B
answer by using the codes given below the lists. (d) I-E, II-C, III-A, IV-D
List I List II Ans. (a) : The correct match for the List-I and List-II is:
I. European (A) Paris to Istanbul Ernesto Zedillo – Mexico
transcontinental Alberto Fujimori – Peru
railway Julio Maria Sanguinetti – Uruguay
II. Trans-Andean (B) Leningrad to Fernando H. Cardoso – Brazil
railway Vladivostok 46. The palaeomagnetic results obtained from India
III. Trans-Siberian (C) Leningrad to indicate that in the past, the Indian land mass has
railway Volvograd moved:
(a) Northward (b) Southward
IV. Orient Express (D) Buenos Aires to
(c) Eastward (d) Westward
Valparaiso
Ans. (a) : The palaeomagnetic results obtained from
(E) Paris to Warsaw
India indicate that in the past, the Indian land mass has
Codes: moved in north direction with average speed of 12
(a) I-E, II-D, III-B, IV-A cm/year after sepration from Gondwanaland. The
(b) I-A, II-D, III-C, IV-B continental plate move with a average speed of 2
(c) I-E, II-A, III-B, IV-C cm/year ad oceanic plates moves with a average speed
(d) I-A, II-B, III-C, IV-D of 5 cm/year.
IAS (Pre) GS 1995 Paper I 557 YCT
47. Willy-Willy is: Ans. (d) : Originally the India subcontinent was a part
(a) A type of tree grown in temperate regions of a huge land mass called Gondwana. India is a
(b) A wind that blows in a desert subcontinent located in South of Asian continent. It is
(c) A tropical cyclone of the north-west Australia considered a subcontinent because it covers an
(d) A kind of common fish found near Lakshadweep expansive area of land that includes the Himalaya
islands region in the north, the gangetic plain as well as the
Ans. (c) : Willy-Willies is a severe tropical cyclone or a plateau region in the South.
whirlwind that take place over a desert. Harmless
Australian dust devils, vertical coloumn of air that kick INDIAN CONSTITUITON AND POLITY
up a lot of dust and create a strong whirlwind over 51. Which one of the following is correct in respect of
desert areas are an example of Willy-Willies. the commencement of the election process in
Hurricane is a name of cyclone in America, Typhoon is India ?
a name of cyclone in China and Japan. (a) The recommendation for election is made by the
48. Consider the map given below indicating four government and the notification for election is
places frequently figuring in the news: issued by the Election Commission
(b) The recommendation for election is made by the
Election Commission and the notification for
election is issued by the Home Ministry at the
Centre and Home Departments in the States
(c) The recommendation for election is made by the
Election Commission and the notification for
election is issued by the President and
Governors of the States concerned
(d) Both the exercises of making a recommendation
for election and that of issuing a notification in
Which one of them is Chechanya?
respect of it are done by the Election
Choose the correct answer from the codes Commission
given below:
Ans. (c) : As per People Representation Act, 1951 the
(a) 1 (b) 2 recommendation for election is made by the election
(c) 3 (d) 4 commission and the notification for election is issued by
Ans. (c) : The correct location for Chechanya in the the president and the Governors of the states concerned.
map is shown by point 3. 52. Which of the following is/are extra-constitutional
Chechanya is the Southern Federal district of Russia. and extra-legal device(s) for securing cooperation
Politically it has the status of an autonomus republic. Its and coordination between the States in India?
capital is ‘Grozny’. I. The National Development Council
49. In which one of the following crops international II. The Governors’ Conference
trade is low in the context of total produce? III. Zonal Councils
(a) Rice (b) Coffee IV. The Inter-State Council.
(c) Rubber (d) Wheat Choose the correct answer from the codes given
below:
Ans. (a) : In 1995, the rice production was highest in Codes:
India among cereals but its quality of rice cannot (a) I, II and III (b) I, III and IV
compete with other rice producing countries. Most of (c) III and IV (d) IV only
the rice produced in India was meant for internal Ans. (a) : National Development Counil is an extra
consumption. India is major producer and exporter of constutitional body established by the Government of
Basmati Rice. India in 1952. Prime Minister of India is the ex-officio
As per 2021, rice occupy the major share in India’s total chairman of the council. It prescribes guidelines for the
cereal export with 87.6% during the same period. formulation of national plans, assesses resource and five
50. The Indian subcontinent was originally part of a year plans and also recommends social and economic
huge land mass called: measure to increase development. The Governor’s
(a) Jurassic Land Mass conference is an extra constitutional body, headed by
president of India as its chairman. Its aim is to secure
(b) Aryavarta
cooperation and coordination among the state as of
(c) Indiana
India.
(d) Gondwana Continent
IAS (Pre) GS 1995 Paper I 558 YCT
Zonal councils were established by the state (a) Ministers who are not Members of Parliament
Reorganisation Act, 1956 to advise on matters of but who have to get themselves elected to either
common interest to each of five zone in which India is House of Parliament within six months after
divided. Presently there are six zonal council.
assuming office.
The Interstate council is a constitutional body
established under article 263 of Indian constitution. (b) Not more than 20 nominated members
The sole purpose of interstate council is to facilitate the (c) Not more than 20 representatives of Union
cooperation and coordination between states and the Territories
centre.
Thus I, II, III are extra-constitutional and extra legal (d) The Attorney-General who has the right to
device, whereas IV (Interstate council) is constitutional speak and take part in the proceedings of either
device. House of Parliament
53. Which of the following are matters on which a Ans. (c) : Article 81 of the constitution defines the
constitutional amendment is possible only with
composition of the house of the people of Lok Sabha. It
the ratification of the legislatures of not less than
one-half of the States? states that the house shall not consist of more than 550
I. Election of the President elected members of whom not more than 20 will
II. Representation of States in Parliament represent Union Territories. At present the strength of
III. Any of the Lists in the 7th Schedule the Lok Sabha is 543, of which 530 have been allocated
IV. Abolition of the Legislative Council of a State
Choose the correct answer from the codes given to the states and the rest to Union Territories.
below: Thus option (c), not more than 20 repreentatives of
Codes: Uuion Territories will fit in the place marked ‘x’.
(a) I, II and III (b) I, II and IV
(c) I, III and IV 55. Who among the following have the right to vote
(d) II, III and IV
Ans. (a) : Article 368 in Part XX of the constitution in the elections to both Lok Sabha and Rajya
deals with the provision of amendment of the Sabha?
constitution and its procedure. (a) Elected members of the Lower House of the
Under Article 368(2) in the constitution of India, states Parliament
that an amendment of this constitution may be initiated
(b) Elected members of the Upper House of the
only by the introduction of a Bill for the purpose in
either house of parliament and when the bill is passed in Parliament
majority of the total membership of that house present (c) Elected members of the Upper House of the
and voting. State Legislature
The ratification by state legislature of not less than one(d) Elected members of the Lower House of the
half of the states by resolution to that effect passed by
those legislature before the bill making provision for State Legislature
such amendment is presented to the President for assent.
Ans. (d) : Elected members of the lower house of the
The provisions are related to: state legislative are entitled to take part in both the
Article 54, Article 55, Article 73, Article 162. election of Lok Sabha and Rajya Sabha.
Chapter IV of part V, Chapter V of Part VI, or Chapter
Process of Rajya Sabha Election–
I of part XI.
Any of the lists in the seventh schedule The representative of the states and of the union
The representation of states in parliament. territories in the Rajya Sabha are elected by the method
Thus option (a), statement I, II and III are correct.of indirect election.
54. Consider the table given below: The representatives of each state and two union
territories are elected by the elected members of the
legislative Assembly of that state and members of the
Electoral College of that Union Territory, as the case
may in accordance with the system of proportional
representation by means of the single transferable vote.
Process of Lok Sabha Election:
Members of the Lok Sabha are elected by an adult
universal suffrage and a first past the post system.
56. Which one of the following States of India does
not have a Legislative Council so far even
though the Constitution (Seventh Amendment)
Act, 1956 provides for it?
Which one of the following will fit in the place (a) Maharashtra (b) Bihar
marked ‘X’? (c) Karnataka (d) Madhya Pradesh
IAS (Pre) GS 1995 Paper I 559 YCT
Ans. (d) : Madhya Pradesh of India does not have a 59. In which one of the following States of India it is
legislative council so far even though the constitution legal for a Hindu male and illegal for a Muslim
(Seventh Amendment) Act, 1956 contains provisions male to have more than one living wife?
for the same. Presently, there are six Indian states (a) Nagaland (b) Mizoram
where legislative council are present. They are U.P., (c) Goa (d) Arunachal Pradesh
Bihar, Maharashtra, Karnataka, Andhra Pradesh and Ans. (c) : Goa is the state of India where it is legal for
Telangana. Hindu male to have more than one living wife but
where Muslim male are not allowed to have the same.
57. Article 156 of the Constitution of India provides
that a Governor shall hold office for a term of 60. Agricultural income-tax is assigned to the State
Governments by:
five years from the date on which he enters upon
(a) The Finance Commission
his office. Which of the following can be deduced
(b) The National Development Council
from this?
(c) The Inter-State Council
I. No Governor can be removed from office till
(d) The Constitution of India
completion of his term.
Ans. (d) : Under Article 269, income other than
II. No Governor can continue in office beyond a
agricultural income shall be levied and collected by the
period of five years.
Government of Indian and it is distributed among states
Select the correct answer from the codes given and centre. Whereas agricultural income tax is assigned
below: to the state government by the Constitution of India.
Codes: 61. The Dinesh Goswami Committee was concerned
(a) I only (b) II only with:
(c) Both I and II (d) Neither I nor II (a) De-nationalisation of banks
Ans. (d) : Article 156 of Indian constitution states that: (b) Electoral reforms
(a) The Governor shall hold office during pleasure of the (c) Steps to put down insurgency in the North-East
president. (d) The problem of the Chakmas
(b) The Governor may by writing address his resignation Ans. (b) : Dinesh Goswami Committee was concerned
to the President. with electoral reform in 1990.
(c) The Governor shall hold his office for a term of 5 He recommended following measures:
years from the date on which he enters upon his office. Extensive restructuring of the accounting of election
(d) The Governor shall, notwithstanding the expiration of expenses.
his terms continues to hold office until his successor Electronic voting machine should be used to put an end
enters upon his office. to manipulating and tempering.
Thus, both the statement are wrong as it mentions that Speedy trial of election dispute through the help of
Governor holds office during pleasure of president and adhoc judges.
also he continue his office until his successor enters There should be fresh delimitation on the basis of 1981
upon his office. census.
62. Which one of the following is not explicitly stated
58. Prohibition of discrimination on grounds of
in the Constitution of India but followed as a
religion, etc. (Article 15 of the Constitution of India)
is a Fundamental Right classifiable under: convention?
(a) The Right to Freedom of Religion (a) The Finance Minister is to be a Member of the
(b) The Right against Exploitation Lower House.
(c) The Cultural and Educational Rights (b) The Prime Minister has to resign if he loses
(d) The Right to Equality majority in the Lower House.
Ans. (d) : Article 15 of Indian constitution states that (c) All the parts of India are to be represented in the
there is prohibition of discrimination on grounds of Council of Ministers.
religion, race, caste, sex and place of birth. (d) In the event of both the President and the Vice-
It comes under the Right to Equality (Article 14-18) of President demitting office simultaneously
Indian constitution. before the end of their tenure the Speaker of the
Right to freedom of Religion comes under Article (25- Lower House of the Parliament will officiate as
28). Right against exploitation comes under Article (23- the President.
24). Ans. (b) : The Prime Minister has to resign if he loses
The cultural and Educational Rights comes under majority in the Lower House is nowhere stated in the
Article (29-30). constitution of India but followed as a convention.
IAS (Pre) GS 1995 Paper I 560 YCT
Under Article 75(3), the council of Minister shall be Sabha or to the legislative assembly; and, in addition, it
collectively responsible to the House of the people. As wins four seats in the Lok Sabha from any state or
Prime Minister is head of council of Minister and he states; or
resigns if the majority is lost. -If it wins two per cent of seats in the Lok Sabha at a
63. Which of the following are the States in which the general election; and these candidates are elected from
Lok Ayukta Act includes the Chief Minister in its three states; or
ambit? -If it is recognised as a state party in four states..
(a) West Bengal and Kerala
Thus, no party is qualified to be national party. The
(b) Gujarat and Maharashtra
correct answer is option (d).
(c) Madhya Pradesh and Orissa
(d) Rajasthan and Karnataka 66. Consider the table given below providing some
Ans. (c) : Lokayukta investigates cases of corruption details of there results of the election to the
committed at state level and once proven recommends Karnataka State Legislative Assembly held in
action. December, 1994.
At the time when question was asked, the only states in
which the Lok Ayukta Act, 2013 includes the Chief
Minister in its ambit were Madhya Pradesh and Orissa.
64. Which one of the following is incorrect in respect
of Local Government in India?
(a) According to the Indian Constitution, local
government is not an independent tier in the In terms of electoral analysis, the voter-seat
federal system distortion is to be explained as the result of the
(b) 30% of the seats in local bodies are reserved for adoption of the
women (a) Hare system
(c) Local government finances are to be provided (b) Cumulative vote system
by a Commission (c) First-past-the-post system
(d) Elections to local bodies are to be determined
(d) Plural vote system
by a Commission
Ans. (c) :In term of electoral analysis, the voter-seat
Ans. (a) : According to 73rd constitutional Amendment
Act. distortion is to be explained as the result of the adoption
Under Article 243 (d), there will be reservation of of the first-past-the-post system. It explains that who
30% seats for woman in local bodies. gets more votes in terms of number is victorious.
Under Article 243 (i), local government finances are .
to be provided by a commission. 67. If in an election to a State Legislative Assembly the
Under Article 243 (k), election to local bodies are to candidate who is declared elected loses his deposit,
be determined by a commission. it means that:
Thus, option (b), (c), (d) are correct and option (a) is (a) The polling was very poor
incorrect in respect to local government in India. (b) The election was for a multi-member constituency
65. Which of the following political parties is/are (c) The elected candidate’s victory over his nearest
national political parties? rival was very marginal
I. Muslim League
(d) A very large number of candidates contested the
II. Revolutionary Socialist Party
election
III. All-India Forward Block
IV. Peasants and Workers Party of India Ans. (d) : According to the representation of people act
Choose the correct answer from the codes given 1951, it is mandatory for every candidate contesting
below: election (Parliamentary or Assembly election) to deposit
Codes: a certain security amount.
(a) I, II and III (b) II and IV Now, if candidate gets fewer than 1/6th of total valid
(c) III only (d) None of the above votes caste in the constituency, his security deposit will
Ans. (d) : A party is recognised as a national party if be lost.
any of the following conditions is fulfilled: If in a case a candidate who is declared elected and
-If it secures six per cent of valid votes polled in any loses his deposit, it means that a very large number of
four or more states at a general election to the Lok candidates contested the election.

IAS (Pre) GS 1995 Paper I 561 YCT


ECONOMY Ans. (a) : The Commission for Agricultural Costs and
Price (CACP), set up in 1965, is a decentralized agency
68. To prevent recurrence of scams in Indian Capital of the Government of India. It is an expert body which
Market, the
recommends the Minimum Support Prices (MSPs) by
Government of India has assigned regulatory taking into consideration various factors.
powers to
The objective of the commission are:
(a) SEBI
(b) RBI To recommend Minimum Support Price (MSPs).
(c) SBI To stabilize agricultural prices.
(d) SBI To ensure meaningful real income level to the farmers.
Ans. (a) : To prevent recurrence of scams in Indian To protect the interest of the consumer by providing
Capital Market, the government of India has assigned essential agricultural commodities at reasonable rates
regulatory powers to SEBI (Security and Exchange through public distribution system.
Board of India). 71. As part of the liberalisation programme and
The Security and Exchange Board of India is the with a view to attracting foreign exchange, the
regulatory body for securities and commodity in India Government and the RBI have devised two
under the jurisdiction of ministry of finance, schemes known as FCNR-’A’ and FCNR-’B’.
Government of India. It was established on 12th April, Which of the following is/are true regarding
1992 and given statutory powers o 30th January, 1992
these two schemes?
through the SEBI Act, 1992.
I. Under scheme ‘A’ RBI bears exchange rate
69. Which one of the following Five-Year Plans
fluctuations.
recognised human development as the core of all
developmental efforts? II. Under scheme ‘B’, other banks are to meet out
(a) The Third Five-Year Plan the difference in exchange rate fluctuations.
(b) The Fifth Five-Year Plan III. Both the schemes stand withdrawn now.
(c) The Sixth Five-Year Plan IV. Only scheme ‘A’ has been withdrawn.
(d) The Eighth Five-Year Plan Select the correct answer from the codes given
Ans. (d) : Eighth Five Year Plan (1992-1997), has below:
recognized human development as the core of all Codes:
development efforts. (a) III only (b) I and II
th
The major objective of 8 five year plan was. (c) I, II and III (d) I, II and IV
(i) Creation of employment, check population Ans. (d) : FCNR account stands for Foreign Currency
growth and overall human development. Non-Resident Account. FCNR accounts are term
(ii) Primary health facilities, drinking water and deposit account, which are maintained by the NRI’s and
vaccination in all villages. PIOS in the form of foreign currencies.
(iii) Strength the basic infrastructure. FCNR-B was introduced with an aim to replace the
(iv) 8th plan was a plan for managing the transition already prevailing FCNR-A. The foreign exchange risk
from a centrally planned economy to market led related to FCNR-A was borne by RBI and subsequently
economy through indicative planning. by the government of India. Whereas under Scheme
70. Which of the following are the objectives of the FCNR-B, other banks are to meet out the difference in
Commission for Agricultural Costs and Prices exchange rate fluctuations.
(CACP)? Thus statement 3 is incorrect and all other are correct.
I. To stabilise agricultural prices.
72. Which of the following are among the non-plan
II. To ensure meaningful real income levels to
expenditures of the Government of India?
the farmers.
I. Defence expenditure
III. To protect the interest of the consumers by
providing essential agricultural commodities II. Subsidies
at reasonable rates through public III. All expenditures linked with the previous
distribution system. plan periods
IV. To ensure maximum price for the farmer. IV. Interest payment
Select the correct answer from the codes Choose the correct answer from the codes
given below: given below:
Codes: Codes:
(a) I, II and III (b) I, II and IV (a) I and II (b) I and III
(c) I, III and IV (d) II, III and IV (c) II and IV (d) I, II, III and IV
IAS (Pre) GS 1995 Paper I 562 YCT
Ans. (d) : Non-plan expenditure is what the government 76. Corporation tax:
spends on the so called non-productive areas and is (a) is levied and appropriated by the States
mostly obligatory in nature. It includes salaries, (b) is levied by the Union and collected and
subsidies, loans and interest payment defence appropriated by the States
expenditure. (c) is levied by the Union and shared by the Union
and the States
Thus all of the codes given are correct and hence,
(d) is levied by the Union and belongs to it
option (d) is correct.
exclusively
73. Which of the following pairs are correctly matched? Ans. (d) : Corporation tax is a direct tax imposed on the
I. Increase in foreign exchange reserves net income or profit that enterprises make from their
................. Monetary expansion business.
II. Low import growth rate in India ................. Company registered under companies Act 1956, are
Recession in Indian Industry liable to pay corporation tax.
III. Euro-issues ................ Shares held by Indian As corporation tax is direct tax and it levied by the
companies in European countries union and it belongs to union exclusively.
IV. Portfolio investment ...................... Foreign 77. Which of the following constitute the World
institutional investors Bank?
Select the correct answer by using the following I. International Bank for Reconstruction and
codes: Development
Codes: II. International Finance Corporation
(a) I, II and IV (b) III and IV III. International Development Association
(c) I, II and III (d) I, II, III and IV IV. International Monetary Fund.
Choose the correct answer from the codes given
Ans. (d) : The correctly matched pairs are
below:
mentioned as below.
Codes:
(i) Increase in foreign – Monetary expansion (a) I, II and III (b) I and II
reserve exchange (c) III and IV (d) I, II, III and IV
(ii) Low import growth – Recession in Indian Ans. (a) : The World Bank is an international financial
rate in India Industry institution that provides loans and grants to the
(iii) Euro-issues – Shares held by Indian government of low and middle income countries for the
companies in European purpose of pursuing capital projects.
countries The banks which constitutes the World Bank are:
(iv) Portfolio investment – Foreign institutional (i) International Bank for Reconstruction and
Development.
Investors
(ii) International Finance Corporation.
Thus all are correctly matched. (iii) International Development Association.
74. What is the annual growth rate aimed at in the (iv) The Multilateral Investment Guarantee Agency.
Eighth Five-Year Plan? The international centre for settlement.
(a) 5.6% (b) 6% Thus code (i), (ii) and (iii) are correct.
(c) 6.5% (d) 7% 78. Which one of the following is not an instrument
Ans. (a) : The annual growth rate targeted during eighth of selective credit control in India?
five year plan was 5.6%. But actual return at the end of (a) Regulation of consumer credit
year was 6.6%. (b) Rationing of credit
75. Which one of the following is the largest mutual (c) Margin requirements
fund organisation in India? (d) Variable cost reserve ratios
(a) SBI Mutual Fund Ans. (d) : The instrument of selective credit in India
are:
(b) GIC Mutual Fund
1. Consumer credit regulation
(c) Industrial Bank Mutual Fund
2. Issue of directives
(d) Unit Trust of India 3. Rationing of credit
Ans. (d) : In 1963, the Unit Trust of India (UTI) was 4. Moral suasion and publicity
established by an act of parliament. It was set up by the 5. Direct Action
Reserve Bank of India and functional under the 6. Securing Loan Regulation by fixation of Margin
regulatory and administrative control of it. Requirements.
In 1964, UTI launched its first mutual fund scheme. Its Thus, option (a), (b), (c) are instrument of selective
headquarter is in Mumbai and have 41 regional office credit control, whereas option (d) is not a instrument of
throughout the country. selective credit control.

IAS (Pre) GS 1995 Paper I 563 YCT


79. Bank Rate implies the rate of interest: Ans. (d) : The main reason for low growth rate in India,
(a) Paid by the Reserve Bank of India on the insipte of high rate of savings and capital formation is
Deposits of Commercial Banks. high capital/output ratio.
(b) Charged by Banks on loans and advances. Capital/output ratio is a frequent used tool to explain the
(c) Payable on Bonds. relationship between the level of investment made in the
(d) At which the Reserve Bank of India discounts economy and the consequent increase in GDP.
the Bills of Exchange. A lower capital output ratio shows that only low level of
Ans. (d) : Bank rate is the rate charged on the loans investment is needed to produce a given growth rate in
offered by the central bank to the commercial banks the economy. Whereas high capital output ratio shows
without any collateral. that high level of investment required to produce a
It is also considered as the rate at which the central bank given growth rate which accounts for low growth rate in
discounts the bills of the commercial banks. country.
Bank rates controls the overall supply of money in the
economy. 83. The largest source of financing the public sector
outlay of the Eighth Five-Year Plan comes from:
80. The Narasimham Committee for Financial Sector
(a) Balance from current revenue
Reforms has suggested reduction in:
(b) Contribution of public enterprises
(a) SLR and CRR
(c) Government borrowings
(b) SLR, CRR and Priority Sector Financing
(d) Deficit financing
(c) SLR and Financing to capital goods sector
(d) CRR, Priority Sector Financing and Financing Ans. (d) : The largest source of financing the public
to capital goods sector sector outlay of eighth five year plan comes from deficit
financing.
Ans. (a) : Narasimham Committee recommendation on
financial sector reforms has suggested reduction in CRR Deficit financing means generating funds to finance the
and SLR. deficit which results from excess of expenditure over
The SLR then was 38.5% and CRR was 15%. The high revenue. The gap being covered by borrowing from the
amount of SLR and CRR meant locking the bank public by the sale of bonds or printing new money.
resources for government uses which created 84. The new Exim Policy announced in 1992, is for a
hinderance in the productivity of the bank, thus the period of:
committee recommended their gradual reduction. SLR (a) 3 years (b) 4 years
was recommended to reduce from 38.5% to 25% and (c) 7 years (d) 5 years
CRR from 15% to 3 to 5%. Ans. (d) : The Exim Policy or Export Policy is set of
81. One of the reasons for India’s occupational guidelines and instructions related to the import and
structure remaining more or less the same over the export of goods. The Government of India notifies the
years has been that: Exim Policy for a period of five years (1997-2002),
(a) Investment pattern has been directed towards under Foreign Trade Act, 1992.
capital-intensive industries 85. Consider the following:
(b) Productivity in agriculture has been high I. Industrial Finance Corporation of India
enough to induce people to stay with agriculture
II. Industrial Credit and Investment Corporation
(c) Ceilings on land holdings have enabled more of India
people to own land and hence their preference
III. Industrial Development Bank of India
to stay with agriculture
IV. Unit Trust of India
(d) People are largely unaware of the significance
The correct sequence in which the above were
of transition from agriculture to industry for
established is
economic development
(a) I, II, IV, III (b) I, III, II, IV
Ans. (a) : The reason for India’s occupational structure
(c) IV, III, II, I (d) I, IV, III, II
remaining more or less the same over the years has been
the investment pattern, that has been mainly directed Ans. (a) : Organisation Established Year
towards capital intensive industries. Industrial Finance Corporation of India 1st July, 1948
82. The main reason for low growth rate in India, in Industrial credit and Investment 5th January, 1955
spite of high rate of savings and capital formation Corporation of India
is: Unit Trust of India 1st Feb. 1964
(a) High birth rate Industrial Development Bank 1st July 1964
(b) Low level of foreign aid Of India
(c) Low capital/output ratio So, the correct sequence will be (i), (ii), (iv), (iii), and
(d) High capital/output ratio thus option (a) is correct.
IAS (Pre) GS 1995 Paper I 564 YCT
PHYSICS 90. Assertion (A): A diamond sparkles more than a
glass imitation cut to the same shape.
86. The clouds float in the atmosphere because of Reason (R): The refractive index of diamond is
their low: less than that of glass.
(a) Temperature (b) Velocity In the context of the above two statements,
(c) Pressure (d) Density which one of the following is correct?
(a) Both A and R are true, and R is the correct
Ans. (d) : The cloud float in the atmosphere because of explanation of A
their low density. (b) Both A and R are true, but R is not a correct
According to law of flotation, when a body floats in a explanation of A
liquid or gas, the weight of the liquid or gas by its (c) A is true, but R is false
immersed part is equal to the total weight of the body. (d) A is false, but R is true
87. The technique used to transmit audio signals in Ans. (c) : The flicker of the diamond is due to the high
television broadcasts is: refractive index. As a result its critical angle decreases
(a) Amplitude Modulation and there is complete internal reflection of light from its
various point. This is the reason why diamond sparkles
(b) Frequency Modulation
more than a glass imitation cut to the same shape.
(c) Pulse Code Modulation
Thus Assertion (A) is correct and Reason (R) is
(d) Time Division Multiplexing incorrect.
Ans. (b) : Frequency modulation is used to transmit 91. Optical fibre works on the principle of:
audio signals in television broadcasts. The signal (a) Total internal reflection
transmitted by this is free from noise and other (b) Refraction
distortions. (c) Scattering
88. A parachutist jumps from a height of 5000 (d) Interference
metres. The relationship between his falling speed Ans. (a) : Optical fibre works on the principle of total
‘v’ and the distance fallen through ‘d’ is best internal reflection.
represented as: Total internal reflection is a phenomenon in which the
angle of incidence is slightly higher than the critical
angle and the light does not pass through the rarer
medium but entire lights is reflected back to the denser
medium.
For total internal reflection of light, it is necessary that
light is passing from denser medium to rarer medium or
the angle of incidence is greater than the critical angle.
Ans. (d) : The initial gravity is ‘g’ when parachutist 92. Suppose a rocket ship is receding from the earth
falls down from the air ship. But when parachute is at a speed of 2/10th the velocity of light. A light
opened the viscosity of air acts in a place and it opposes in the rocket ship appears blue to the passengers
the motion so the gravity will fall down to zero. Thus on the ship. What colour would it appear to an
parachutist descend at a constant marginal speed. Thus observer on the earth?
graph ‘d’ shows the correct plot for speed vs distance of (a) Blue (b) Orange
parachutist. (c) Yellow (d) Yellow-orange
Ans. (c) : As the portion of light source reflected back
89. When the same note is played on a sitar and a from the earth, its frequency decreases and thus
flute, the sound produced can be distinguished wavelength increases as a result of which spectrum line
from each other because of the difference in: is shifted toward the red of spectrum.
(a) Pitch, loudness and quality Vs× d
(b) Pitch and loudness Wavelength change – ∆λ =
C
(c) Quality only
 v
(d) Loudness only ∆λ = f 
2
Ans. (c) : In the case of playing the same note on the =  
10 f = c 
sitar and the flute, the difference between the sound  λ 
produced by them can be recognized due to the sound
quality. 2  λ of blue light 
= × 4700°A  = 4700°A 
The quality of sound depends on the number of 10  
harmonic and supersonic present in the sound. = 940°A

IAS (Pre) GS 1995 Paper I 565 YCT


Wavelength visible to earth observe CHEMISTRY
(λ visible) = 47000 + 940 = 5640°A
96. Which one of the following is a mixed fertilizer?
λ yellow light 5640°A
(a) CAM (b) Urea
So, it falls in range of yellow spectrum.
(c) Ammonium Sulphate (d) NPK
93. Cryogenic engines find applications in:
Ans. (c) : Mixed fertilizers are fertilizers which contains
(a) Sub-marine propulsion
two or more of the elements of nitrogen, phosphorus and
(b) Frost-free refrigerators
potassium, which are essential for promoting plant growth
(c) Rocket technology and high crop yields. They are obtained by through mixing
(d) Researches in superconductivity the ingredients either manually or mechanically
Ans. (c) : Cryogenic engine finds most of its application Ammonium sulfate is inorganic salt and is used as soil
in rocket technology. fertilizers. It contains 21% Nitrogen and 24% sulphur.
A cryogenic engine is a rocket engine that uses a 97. Which one of the following polymers is widely
cryogenic fuel or oxidizer that is, its fuel or oxidizer (or used for making bullet proof material?
both) are gases liquefied and stored at very low (a) Polyvinyl chloride (b) Polyamides
temperature. (c) Polyethylene (d) Polycarbonates
A cryogenic rocket stage is more efficient and provides Ans. (d) : Polycarbonates are widely used for making
more thrust for every kilogram of propellant it burns bullet proof material.
compared to solid and earth-storable liquid propellant Polycarbonates are extremely durable and has high
rocket stages. Specific impulse achievable with cryogenic impact resistance, event against violent blows. Bullet
propellants is much higher compared to earth storable resistant polycarbonate is more suitable when protecting
liquid and solid propellants, giving it a substantial payloadagainst ballistics, including heavy automatic fire armor.
advantage. Polyvinyl chloride are used for making sheet covers,
94. The variations in temperatures from 0°C to sheets, film etc.
100°C with respect to time of two liquids P, Q are Polyethylene are used for making plastic covers, plastic
shown in the graph given below. Which one of the bottle, pipes and packaging covers.
following statements is correct? 98. The chemical used as a ‘fixer’ in photography
is:
(a) Sodium sulfate
(b) Sodium thiosulfate
(c) Ammonium persulfate
(d) Borax
Ans. (b) : The chemical used as a fixer in photography
is sodium thiosulfate. It is also known as sodium
(a) During heating, liquid P remained hotter than hyposulfite and is used by photographers to fix
developed negatives and prints. It acts by dissolving the
liquid Q throughout
part of the silver salts coated onto film which remain
(b) (b) At no point of time during heating did
unchanged by exposure to light.
the two liquids have the same temperature
(c) P attained the temperature of 100°C faster 99. Which one of the following can be used to
than Q confirm whether drinking water contains a
(d) Q attained the temperature of 100°C faster gamma emitting isotope or not?
than P (a) Microscope
(b) Lead plate
Ans. (d) : From graph it is clear that Q attained the
(c) Scintillation counter
temperature of 100°C faster than P.
(d) Spectrophotometer
95. An air bubble in water will act like a:
Ans. (c) : The presence of gamma emitting isotopes in
(a) Convex mirror (b) Convex lens
drinking water is detected by the scintillation counter.
(c) Concave mirror (d) Concave lens
100. Which one of the following pairs of materials
Ans. (d) : When lens is immersed in a medium whose serves as electrodes in rechargeable batteries
refractive index is greater than that of the material of the commonly used in devices such as torch lights,
lens, the nature of lens is reversed and thus convex lens electric shavers, etc?
behave like concave ad vice-versa. The air bubble is (a) Nickel and cadmium
convex lens, but when it is submersed into water, which (b) Zinc and carbon
is having higher refractive index than air and thus lens (c) Lead peroxide and lead
behave like concave lens. (d) Iron and cadmium
IAS (Pre) GS 1995 Paper I 566 YCT
Ans. (a) : Nickel and Cadmium rod act as electrodes in Ans. (b) : A person with ‘AB’ blood group is
the rechargeable batteries commonly used in devices sometimes called universal recipient because it have
such as torch light, electric shavers etc. antigens of ‘A’ and ‘B’ blood group whereas it does not
Nickel rod act as cathode and cadium rod act as anode have any antibodies in his body. So ‘AB’ blood group
and they react in presence of electrolyte solution of can receive bloods from person of ‘A’, ‘B’, ‘AB’ and
Potassium Hydro oxide in Ni-Cl batteries. ‘O’ blood groups.
101. The difference between a nuclear reactor and an 105. Which one of the following is a membrane that
atomic bomb is that: protects the developing embryo from
(a) No chain reaction takes place in nuclear reactor desiccation?
while in the atomic bomb there is a chain reaction (a) Amnion (b) Allantois
(b) The chain reaction in nuclear reactor is controlled (c) Chorion (d) Yolk sac
(c) The chain reaction in nuclear reactor is not Ans. (a) : The amnion is a thin, tough membrane that
controlled protects a developing child. It allows for nutrients to
(d) No chain reaction takes place in atomic bomb reach the fetus and wastes to be removed. Amniotic
while it takes place in nuclear reactor fluid is found inside amnion which will protect the
Ans. (b) : In nuclear reactors, the reaction is moderated developing embryo from desiccation.
and controlled for use of peaceful and constructive 106. Which one of the following antimicrobial drugs is
purpose. Where as in nuclear bomb uncontrolled chain suitable for treatment of both tuberculosis and
reaction takes place and it is used for destructive leprosy?
purposes. In nuclear bomb high amount of energy is (a) Isoniazid
(b) p-aminosalicylic acid
released as compared to nuclear reactor. Nuclear
(c) Streptomycin
reactors uses cadmium rod as moderator and water and
(d) Rifampicin
graphite as coolant.
Ans. (b) : P-amino salicylic acid is an antimicrobial
BIOLOGY drugs is suitable for treatment of both tubereculosis and
leprosy.
102. The normal temperature of human body on the It is antituberculosis antibiotic and works by stopping or
Kelvin scale is: slowing the growth of bacterial infections.
(a) 280 (b) 290 107. Most of the desert plants bloom during night time
(c) 300 (d) 310 because:
Ans. (d) : The normal temperature of human body is (a) Their blooming is controlled by low temperature.
37°C (b) They are sensitive to the phases of moon.
To convert °C into Kelvin (c) The desert insects eat away flowers during
day time.
= [273 + °C] Kelvin
(d) The desert insects are active during night
= 273 + 37 time.
= 310K
Ans. (d) : Most of the deserts plants bloom during the
103. Which one of the following sets is correctly matched? night time because desert insects are active in the night.
(a) Diphtheria, Pneumonia and Leprosy: Hereditary
108. Which of the following elements are present in
(b) AIDS, Syphilis and Gonorrhoea : Bacterial
all proteins?
(c) Colour blindness, Hemophilia and Sickle cell
I. Carbon II. Hydrogen
anaemia : Sex linked
III. Oxygen IV. Nitrogen
(d) Polio, Japanese B encephalitis and plague: Viral
Select the correct answer from the codes given
Ans. (c) : Syphilis, Gonorhoea – Sexually transmited below:
diseases Diphteria Leprosy and Plague – bacterial Codes:
diseases AIDS, Polio, Japanese B encephalitis – Viral (a) II and III
colour blindness, Hemophillia and sickle cell anaemia – (b) I, II and IV
sex linked. (c) I, III and IV
Thus only option (c) is matched correctly. (d) I, II, III and IV
104. A person with ‘AB’ blood group is sometimes Ans. (d) : Proteins were first described by Gerardus
called a universal recipient because of the: Johannes Mulder and named by Jons Jacob Berzelius
(a) Lack of antigen in his blood. in1838.
(b) Lack of antibodies in his blood. Proteins contain the element carbon, hydrogen and
(c) Lack of both antigens and antibodies in his oxygen, but proteins are the only macronutrient that
blood. contains nitrogens. So option (d) is correct, as it
(d) Presence of antibodies in his blood. includes all of the above mentioned elements.
IAS (Pre) GS 1995 Paper I 567 YCT
109. Which one of the following hormones contains 112. Which one of the following does not belong to
iodine? biosphere reserves set-up so far?
(a) Thyroxine (b) Testosterone (a) Great Nicobar (b) Sunderbans
(c) Insulin (d) Adrenaline (c) Nanda Devi (d) Gulf of Kutch
Ans. (a) : Thyroxine is a hormone secreted by the Ans. (d) : At the time, when question was asked Gulf of
thyroid gland in the bloodstream. It then travels to the Kutch was not a biosphere reserve, but as per 2021 it is
organs such as kidneys and liver where it gets converted a biosphere reserve with the largest area covered.
into its active form triiodothyronine. Biosphere Reserve is an international designation by
The thyroid gland uses iodine from food to make United Nations Educational Scientific and cultural
thyroid hormones thyroxine and triiodothyronine. Organization for representative parts of natural and
110. “It begins as a single cell and grows into a merciless cultural landscapes extending over large areas of
disease that claims millions of lives year after year. terrestrial or coastal/marine ecosystem.
But scientists are steadily unlocking its mysteries, 113. Which one of the following fuels causes minimum
and the fight against it may now have reached a
environmental pollution?
dramatic turning point. New discoveries promise
(a) Diesel (b) Coal
better therapies and hope in the war against ...” The
(c) Hydrogen (d) Kerosene
disease referred to in the above quotation is:
(a) Cancer (b) AIDS Ans. (c) : Hydrogen fuel does not emit any pollution
(c) Tuberculosis (d) Alzheimer’s disease and it is the most powerful fuel with high calorific and
valve.
Ans. (a) : Cancer is deadly diseases which occurs due
to uncontrolled division and growth of cells. This fatal Diesel, Coal, Kerosene fuel are obtained from fossils
disease can occur in all those organs whose cells have fuel and they emit pollution on combustion.
the ability to divide. The most common treatments of
the cancer are surgery, chemotherapy and radiation. CURRENT AFFAIRS
114. The lead character in the film The Bandit Queen
ENVIRONMENT AND ECOLOGY has been played by:
111. “Monoculture of commercially viable trees is (a) Pratibha Sinha (b) Rupa Ganguly
destroying the unique natural profile of (c) Seema Biswas (d) Shabana Azmi
............Thoughtless exploitation of timber, Ans. (c) : The lead character in the film ‘‘The Bandit
deforesting vast tracts for palm cultivation, Queen’’ directed by Shekhar Kapoor was played by
destruction of mangroves, illegal logging by tribals Seema Biswas. The story of ‘The Bandit Queen’ was
and poaching only compound the problem. Fresh based on the life of Phoolan Devi. Seema Biswas was
water pockets are fast drying up due to awarded national award for her role in ‘The Bandit
deforestation and destruction of mangroves.” The Queen.’
place referred to in this quotation is:
115. In the field of space technology, India has
(a) Sunderbans
demonstrated during 1994, her capability to:
(b) Kerala Coast
(a) Design advance intercontinental ballistic missile
(c) Orissa Coast warning systems
(d) Andaman and Nicobar Islands (b) Build satellites
Ans. (a) : Sundarbans contains the world’s largest (c) Build stealth missiles
mangrove forest. It is a vast contiguous mangrove forest
(d) Launch geo-stationary satellites
ecosystem in the coastal region of Bay of Bengal spread
over India and Bangladesh and Meghna rivers. Ans. (d) : The stretched Rohini satellite series (SROSS)
The quotation mentioned above refers to Sundarban. was launched from Sriharikota on May 4, 1994 A.D.
Mangroves are the plant communities occurring in into the lower orbit of Earth. After this, on Oct 15, 1994
intertidal zones along the coasts of tropical and India launched PSLV from Sriharikota and was placed
subtropical countries. into polar solar orbit. With this India has demonstrated
during 1994, her capability to launch geo-stationary
They perform many ecological functions such as
satellites from the country itself.
production of woody trees, provision of habitat, food
and spawning grounds for fin fish and shell fish, 116. Which one of the following countries had in 1994
provision of habitat for birds and other valuable fauna; voted against joining the European Union?
protection of coastlines and acceration of sediment to (a) Norway (b) Sweden
form new land. (c) Finland (d) Austria
IAS (Pre) GS 1995 Paper I 568 YCT
Ans. (a) : Norway had voted against joining the 121. Which one of the following is true regarding the
European Union. Jawahar Rozgar Yojana (JRY)?
The European Union is a political and economic union (a) It was launched during the Prime Ministership
of 27 member states that are located primarily in of Indira Gandhi.
st
Europe. The European union was established on 1 (b) It aims at creating one million jobs annually.
November, 1993 in Maastricht, Netherlands. (c) The target group of JRY are the urban poor
As of 31st January, 2020 the U.K. formally left the living below the poverty line.
European Union. (d) Under the scheme 30% of the employment
generated is reserved for women.
MISCELLANEOUS Ans. (d) : Jawahar Rozgar Yojana (JRY) was started on
1st April, 1989 with the aim to provide employment
117. The signatories to the treaty banning chemical opportunities to people.
weapons include: Jawahar Rozgar Yojana was combination of National
(a) USA, Russia, India, Iran and Israel Rural Employment Program (NREP) and Rural
(b) Iraq, Libya, India, Russia and China Landless Employment Guarantee Program (RLEGP).
(c) Brazil, Angola, North Korea, Pakistan and USA It is centrally sponsored scheme and run by state
(d) Syria, Sri Lanka, Japan, Singapore and France governments.
Ans. (a) : The treaty on ban on chemical weapon The Expenditure is shared by the centre and the state in
became effective worldwide on 25th April, 1997. 160 the ratio 80 : 20.
countries including U.S.A., Russia, Iran, Israel and There is provision of reservation of 30% employment
India had signed the treaty. generated to women.
As of 30th April 1997, 87 countries had signed the treaty Thus option (d) is correct.
and also ratified it in December, 1996. 122.Which of the following are correctly matched?
118. A meteor is I. Hamas-Palestinian extremist group
(a) a rapidly moving star II. Sinn Fein-IRA’s political wing
(b) a piece of matter which has entered the earth’s III. True Path Party-A major constituent of the
atmosphere from outer space ruling coalition in Turkey
(c) part of a constellation Choose the correct answer from the codes given
(d) a comet without a tail below:
Ans. (b): A meteor is a small body of matter from outer Codes:
space that enters the Earth's atmosphere and becomes (a) I and II (b) I and III
incandescent as a result of friction and appear as a (c) I, II and III (d) II and III
streak of light. Ans. (c) : The given statements are correctly matched
119. According to the 1991 Census, the highest as:
percentage of population in India is to be found Hamas – Palestinian extremist group
in the age-group of Sinn Fein – IRA’s political wing
(a) 60 years and above True Path Party – A major constituent of the
(b) 35 to 55 years ruling coalition in Turkey.
(c) 25 to 34 years 123. Which of the following benefits are likely to accrue
(d) 5 to 14 years to India from the World Trade Organisation?
Ans. (d) : According to the 1991 census, the percentage I. India’s share in the world trade is to go up from
of population in the age group between 5 and 14 years the present 600 million US dollars to 5 billion US
was maximum in India. The geographical area of India dollars by 2000 A.D.
is about 2.4% of the world, but 16% of the world’s II. It will help boost exports of agricultural commodities
population is found here. The population as per 1991 from India.
census was 838, 583, 988. III. India’s share in the world trade is likely to triple
120. The Nobel Prize for deciphering the language of by the year 2000 A.D.
bees was awarded to: Select the correct answer using the codes given
(a) H.G. Khurana (b) K.V. Frisch below:
(c) Julian Huxley (d) Dorothy Hodgkins Codes:
Ans. (b) : The nobel prize for deciphering the language (a) I, II and III (b) I and II
of bees in 1973 was given to K.V. Frisch. (c) I and III (d) II and III
IAS (Pre) GS 1995 Paper I 569 YCT
Ans. (d) : The benefits which are likely to accrue to the world... He touted the wonderful properties
India from the World Trade Organisation is it will help of Vitamin C first as a cure of common cold and
boost exports of agricultural commodities from India later as a preventive agent against cancer.” One
and India’s share in the world trade in likely to triple by of science’s major figures of all time referred to
the year 2000 A.D. above is:
(a) G.N. Lewis
124. Comet Shoemaker-Levy 9 hit the planet: (b) Linus Carl Pauling
(a) Pluto (b) Mars (c) Fritz London
(c) Jupiter (d) Saturn (d) Walter Heitler
Ans. (c) : Comet Shoemaker – Levy 9 was a comet that Ans. (b) : In 1970 Linus Pauling claimed that vitamin C
broke apart in July 1992 and collided with Jupiter in prevents and alleviates the episodes of the common
July 1994, which was the first direct observation of an cold.
extra terrestrial collision of solar system objects. After Pauling the only person to ever win two unshared Nobel
colloiding, it created a huge dark scars in the planet Prizes. Also in 1949, Pauling with his team established
atmosphere and lofting superheated plumes into its sickle cell Anemia as a genetic disease. In 1954 and
stratosphere. 1962 he received noble prize for chemistry and nobel
125. Examine the following statements: prize for peace.
I. All children are inquisitive. 129. ‘The final Agent Orange raid in Vietnam took
II. Some children are inquisitive. place in 1970; areas have begun to bloom again.
III. No children are inquisitive. But 19 years after the war’s end, it seems plain
IV. Some children are not inquisitive. that Agent Orange is killing and maiming human
Among these statements, the two statements beings, something it never intended to do. The
which cannot both be true simultaneously but apparent toxic fallout from those clouds ... is a
can both be false would be: crop of human miseries including cancers,
(a) I and III (b) I and IV miscarriages and birth defects—that may persist
(c) II and III (d) III and IV for decades.”
The offensive substance referred to in the above
Ans. (a) : From the given statement, the two statement
quotation is:
which cannot be true simultaneously but can both be
(a) DDT used as insecticide
false would be. (b) A complex mixture of herbicides and weedicides
All children are inquisitive and no children are used to increase agricultural output in the then
inquisitive. South Vietnam under the US aid programme
So, I and II are correct. (c) A complex mixture of DDT and other
126. Which one of the following is the largest mutual insecticides used as aerial sprays for protection
fund organisation in India? against malaria and other tropical diseases
(a) SBI Mutual Fund (d) Dioxin used as defoliants
(b) GIC Mutual Fund Ans. (d) : In the Vietnam war, the US used a mixture
(c) Industrial Bank Mutual Fund of weed killer chemicals called 2, 4, 5, 7 and 2, 4-D
(d) Unit Trust of India (which was contaminated with 2, 3, 7, 8 tetra
Ans. (d) : In 1963, the Unit Trust of India (UTI) was chlorodibenzodioxin) as a chemical weapon under the
established by an act of parliament. It was set up by the name ‘Agent Orange’. Agent orange is named because
Reserve Bank of India and is functional under the of orange colour of chemical.
regulatory and administrative control of it. 130. ‘Yellow cake’, an item of smuggling across borders
In 1964, UTI launched its first mutual fund scheme. Its is:
headquarter is in Mumbai and have 41 regional offices (a) A crude form of heroin
throughout the country. (b) A crude form of cocaine
127. The only snake that builds a nest is: (c) Uranium oxide
(a) Chain viper (b) King Cobra (d) Unrefined gold
(c) Krait (d) Saw-scaled viper Ans. (c) : Uranium oxide in Canadian local language is
Ans. (b): The female snake about 13 feet only build termed as ‘Yellow Cake’.
nest to lay its eggs before the monsoon. The king cobra Yellow cake uranium is a type of uranium concentrate
is the only snake in the world that builds a nest. powder obtained from leach solution.
128. “By weaving threads of physics, chemistry and 131. Consider the series given below:
biology into a rich tapestry, this remarkable 4/12/95, 1/1/96, 29/1/96, 26/2/96,...
scientist provided a unifying molecular view of The next term of the series is:
IAS (Pre) GS 1995 Paper I 570 YCT
(a) 24/3/96 (b) 25/3/96
(c) 26/3/96 (d) 27/3/96
Ans. (b) : 4/12/95, 1/1/96, 29/1/96, 26/2/96, from
sequence it is clear that there is a gap of 28 days
between two dates, so next date in sequence will be
25/3/96.
132. In track meets both 100 yards and 100 metres Ans. (b) :
From the given Venn diagram its clear that the shade
are used as distances. By how many metres is portion ‘u’ represents teacher who can teach
100 metres longer than 100 yards? mathematics and physics but not chemistry.
(a) 0.856 m (b) 8.56 m 135. Seven men, A, B, C, D, E, F and G are standing in
(c) 0.0856 m (d) 1.0 m a queue in that order. Each one is wearing a cap of
a different colour like violet, indigo, blue, green,
Ans. (b) : 1 yards = 36 inches yellow, orange and red. D is able to see in front of
1 inch = 2.54 cm him green and blue but not violet. E can see violet
100 yards = 2.54 × 36 × 100 and yellow, but not red. G can see caps of all
colours other than orange. If E is wearing an
= 91.44 × 100 cm
indigo coloured cap, then the colour of the cap
= 91.44 m worn by F is:
Difference between 100 metre and 100 yards is (a) blue (b) violet
100 – 91.44 (c) red (d) orange
Ans. (c) : As per the question 7 person are standing in
= 8.56 m the queue.
133. Examine the following statements:
A  → Green
I watch TV only if I am bored.
I am never bored when I have my brother’s B  → Blue
company. C  → Yellow
Whenever I go to the theatre, I take my brother D  → Violet
along. E  → Indigo
Which one of the following conclusions is valid in
F  →
the context of the above statements?
(a) If I am bored, I watch TV G  → Orange
So cap worn by f will be red in colour.
(b) If I am bored, I seek my brother’s company
(c) If I am not with my brother, then I watch TV 136. A person travelled from one place to another at
(d) If I am not bored, I do not watch TV an average speed of 40 kilometres/hour and
back to the original place at an average speed of
Ans. (d) : As per the statements, the conclusion which 50 kilometres/hour. What is his average speed in
is valid in the context of statement is if I am not bored, I kilometres/hour during the entire round-trip?
do not watch T.V. This is clear from first line of (a) 45
statement which states that I watch TV only if I am (b) 20 5
bored. So, if he is not bored, he will not watch T.V. (c) 400/9
(d) Impossible to find out unless the distance
134. In the given diagram, circle A represents teachers between the two places is known
who can teach Physics, circle B represents teachers Ans. (c): Let the distance be x km
who can teach Chemistry and circle C represents distance
those who can teach Mathematics. Among the Time = speed
regions marked p, q, r ........ the one which
Total time taken for going and coming back
represents teachers who can teach Physics and x x
Mathematics but not Chemistry, is: = +
40 50
5x + 4 x 9 x
= =
200 200
Total distance be covered = 2x
Total distance
Total time
Average speed =
(a) v (b) u 2x 200× 2 400
= = = km/hr
(c) s (d) t 9 x/200 9 9

IAS (Pre) GS 1995 Paper I 571 YCT


137. The value of: 140. A thief running at 8 km/hr is chased by a
(a – m) (b – m) ... (y – m) (z – m) is policeman whose speed is 10 km/hr. If the thief
(a) m26 + am25 + abm24 +...+ a.b.c. ...z is 100 metres ahead of the policeman, then the
26 25 24
(b) m – am + abm +...– a.b.c. ...z time required for the policeman to catch the
(c) 0 thief will be:
(d) Indeterminate (a) 2 minutes (b) 6 minutes
(c) 10 minutes (d) 3 minutes
Ans. (c) : Value of (a – m) (d – m) ….. (y – m) (z – m)
Ans. (d) : Difference between speed of policeman and
= (a – m) × (b – m) ×………..(m – m) × (n – m) …. × Thief is = (10 – 8) km/hr.
(y – m) (z – m) = 2km/hr
=m–m=0 Time taken to cover 2 km is 1 hour
= 0 × {(a –m) (b – m) ….. (y – m) (z – m)} = 2000 meter in 60 minutes.
=0 So, to cover 100 meter
 60 100 
= ×  minutes
138. A rectangular sump of dimensions 6 m x 5 m x 4  2 1000 
m is to be built by using bricks to make the = 3 minutes
outer dimension 6.2 m x 5.2 x 4.2 m. 141. A student has 60% chance of passing in English
Approximately how many bricks of size 20 cm x and 54% chance of passing in both English and
10 cm x 5 cm are required to build the sump for Mathematics. What is the percentage probability
storing water? that he will fail in Mathematics?
(a) 1500 (b) 3000 (a) 12 (b) 36
(c) 15000 (d) 30000 (c) 4 (d) 10
Ans. (c) : Volume of rectangular jump = outer volume Ans. (d) : Ans. (d) : Chance of passing in English is
Inner volume 60% = 0.6
= (6.2 × 5.2 × 4.2) – (6 × 5 × 4) And chance for passing in both English and
3 Mathematics is = 54% = 0.54
= 15.408 m
3 Percentage of Probability to pass in mathematics is
Volume of 1 brick is = 20 × 10 × 5 = 1000 cm
0.54
= 0 : 001 m3 = = 0.9 = 90%
So, number of bricks required is 6
So percentage probability to fail in mathematics is
15.408
= = 15408 ≈15000 = 100 – 90 = 10%
0.001 142. A table has three drawers. It is known that one
139. Consider the figure given below: of the drawers contains two silver coins,
PQRS is a square of side 1 unit and Q, S are another contains two gold coins and the third
the centres of the two circles. The area of the one contains a silver coin and a gold coin. One
shaded portion is: of the drawers is opened at random and a coin
is drawn. It is found to be a silver coin. What is
the probability that the other coin in the
drawer is a gold coin?
(a) 0.25 (b) 1.00
(c) 0.50 (d) 0.60
Ans. (c) : If one of the drawers is opened and a coin
drawn is silver. The probability of getting gold coin in
1
π 1 other two drawer is 0.5 or 2 .
(a) (b)
2 2 143. In the Cartesian plane four points P, Q, R, S
π 1 π have co-ordinates (1, 1), (4, 2), (4, 4) and (1, 4).
(c) − (d) − 1 The area of the quadrilateral PQRS is:
4 2 2
Ans. (d) : The area of shaded portion is = (Area of
curve RSP – Area of triangle PSR) × 2
1 1
= π(1) 2 − × 1× 1
4 2
π 1 π
=  −  × 2 = −1
 4 2  2

IAS (Pre) GS 1995 Paper I 572 YCT


(a) 9 Aver Noise
(b) 7.5 Tot % of age noise pollution
(c) 4.5 Loc al traffic Heavy level in level, in
(d) Impossible to find unless the lengths of the ation volume Vehicles dB (A) dB (A)
diagonals are known 24.4
Ans. (d) Ans. (b) : I 377 0 73.50 84.00
12.5
II 380 0 72.60 83.00
30.0
III 377 0 73.50 86.50
12.5
IV 225 0 72.98 80.90
length of RQ = 4 – 2 = 2 unit When the total traffic volume is the same, the
length of RS = 4 – 1 = 3 unit factor(s) which affect(s) the noise pollution
So, level is/are:
RS = OQ (a) % of heavy vehicles
OS = RQ (b) Noise pollution level and average noise level
Area of Rectangle OSRQ = 2 × 3 = 6 unit (c) Average noise level and % of heavy vehicles
(d) Indeterminable on the basis of details given
1
Area of Triangle QOP = (OQ) × OP Ans. (a) : Considering the table and details of traffic
2
volume per for location and when the volume of total
1
= × 3 × 1 = 1.5 unit traffic is the same then the percentage of heavy vehicle
2 affects the noise pollution level.
{OP = SP – OS} 146. The price fluctuations of 4 scrips in a stock
= (4 –1) unit 2 unit market in the four quarters of a year are shown
= 3 – 2 = 1 unit in the table below. Four different investors had
Thus total area = Area of Rectangle OS RQ + OQP the following portfolios of investment in the four
= 6 + 1.5 = 7.5 unit companies throughout the year:
144. Consider the diagram given below: Portfolios
Investor 1 : 10 of A, 20 of B, 30 of C and 40 of D
Investor 2 : 40 of A, 10 of B, 20 of C and 30 of D
Investor 3 : 30 of A, 40 of B, 10 of C and 20 of D
Investor 4 : 20 of A, 30 of B, 40 of C and 10 of D
Stock Market Performance
I II III IV
Quarter Quarter Quarter Quarter
T : Transport Scrip A Up 10% Down 15% Up 10% Down 10%
Ec : Education of children Scrip B Up 2% Up 1% Up 2% Up 2%
H : Housing
Scrip C Up 1% Up 1% Down 5% Down 1%
C : Clothing
Scrip D Up 20% Down 15% Up 30% Down 10%
F : Food
S : Savings In the light of the above which one of the
O : Others following statements is correct?
From the diagram shown it would be right to (a) Investor 2 has made the best investment
conclude that (b) Investor 1 has made the best investment
(a) the family spent more than half the income on (c) Investor 2 suffered a net loss during the year
food and clothing (d) Investor 3 suffered a net loss during the year
(b) the amount saved by the family was too little Ans. (b) : Analyzing the investor’s portfolios and
(c) the family had no health problems calculating with stock market performance investor has
(d) the family managed to meet all the essential
made the best investment.
expenses out of the income earned
147. X and Y are two variables whose values at any
Ans. (d) : From the diagram shown it would be right to time are related to each other as shown in Fig.
conclude that the family managed to meet all the (i). X is known to vary periodically with
essential expenses out of the income earned. reference to time as shown in Fig. (ii).
145. Consider the table given below providing
details of traffic volume per hour for locations:
IAS (Pre) GS 1995 Paper I 573 YCT
5% of all can play all the three instrument
5
= ×120 = 6 member
100
Who can play any two and only two of the above
instrument is 30.
40 person can play guitar alone
Let y = 30, z = 40
Which of the following curves depicts correctly
the dependence of Y on time? So, those who can play violin alone or flute alone.
= 120 – (x + y + z) = 120 – (6 + 30 + 40) = 120 – 76
= 44
149. Which of the following can be inferred from the
statement that ‘Either John is stupid or John is
lazy’?
I. John is lazy/therefore, John is not stupid.
II. John is not lazy/therefore, John is stupid.
III. John is not stupid/therefore, John is lazy.
IV. John is stupid/therefore, John is not lazy.
Choose the correct answer from the codes given
below:
Codes:
(a) I and II (b) II and III
(c) III and IV (d) I and IV
Ans. (b) : As per the statement, John is either lazy or
stupid. But he cannot be both at the same time.
So, only II and III are correct.
Ans. (c) : 150. A person earns Rs. 2000 per month over and
above his salary as additional charge allowance.
However, 30% of this additional income will be
deducted as additional income-tax at source. If
the person would deposit Rs. 1000 per month on
(i) x2 + y2 = a2 …..……(i) a long-term saving fetching 12% interest his tax
(ii) x = a sin ωt ……….(ii) liability on the additional allowance would
So y2 = a2 – x2 reduce to 10%. What is the effective interest for
= a2 – (a sin ωt)2 this person for money invested in the long-term
= a2 – a2 sin2 ωt savings scheme?
= a2 (1 – sin2 ωt) (a) 12% (b) 18%
y2 = a2 cos2 ωt (c) 19% (d) 20%
y = ± a cos ωt → which is shown as. Ans. (b) : The additional charge allowance is Rs.
2000/month
Additional Income Tax is 30% of this additional income
30
= × 2000 = 600 rupees
100
If person deposits Rs. 1000 per month on a long term
additional allowance would reduce to 10%
So tax deduction on 600 rupees is = 10%
148. Out of a total of 120 musicians in a club, 5% can 10
= × 600 = 60 rupees
play all the three instruments-guitar, violin and 100
flute. It so happens that the number of musicians The total gain will be 12% + 60 rupees
who can play any two and only two of the above
instruments is 30. The number of musicians who
12
So, 12% of 1000 + 60 rupees = ×1000 + 60rupees
can play the guitar alone is 40. What is the total 100
number of those who can play violin alone or So, total effective interest is 180 rupees on 1000 which
flute alone? is 18%.
(a) 45 (b) 44
(c) 38 (d) 30
Ans. (b) : Total musicians in a club is 120

IAS (Pre) GS 1995 Paper I 574 YCT


UNION PUBLIC SERVICE COMMISSION
Civil Services (Preliminary Exam) - 1994
GENERAL STUDIES : PAPER-I
Time: 2 hours Maximum Number: 200
4. “Live well, as long as you live. Live well even by
ANCIENT HISTORY borrowings, for, once cremated, there is no
‘return.” This rejection of afterlife is an
1. The earliest evidence of silver in India is found in
aphorism of the
the
(a) Kapalika sect
(a) Harappan culture
(b) Sunyavada of Nagarjuna
(b) Chalcolithic cultures of Western India
(c) Ajivikas
(c) Vedic texts
(d) Charvakas
(d) Silver punchmarked coins
Ans. (d) : Charvakas said, “Live well, as long as you
Ans. (a) : The earliest evidence of silver in India is
live. Live well even by borrowings, for once
found in the Harappan culture.
cremated, there is no return”.
Most of the things that have been founded by
Charvaka holds direct perception, empiricism, and
archaeologists are made of stone, shell and metal,
conditional inference as proper sources of knowledge,
including copper, bronze, gold & silver. Gold & silver
embraces philosophical skepticism and rejects
were used to make ornaments and vessels.
ritualism and supernaturalism.
2. Which one of the following usages was a post The Charvaka did not believe in karma, rebirth or
Vedic development ? after life. He believed that there was nothing wrong
(a) Dharma — Artha — Kama — Moksha with sensual pleasure.
(b) Brahmana—Kshatriya—Vaishya—Shudra
5. Vidhushaka, a common character in Sanskrit
(c) Brahmacharya—Grihastashrama— Vanaprastha
drama, is invariably a
— Sanyasa (a) Brahmana
(d) Indra — Surya — Rudra — Marut (b) Kshatriya
Ans. (c) : During the vedic period the society was (c) Vaisya
organised into four varnas known as varnadharma (d) Shudra
namely, Brahmin (the learned classes); kshatriya (the Ans. (a) : Vidhushaka, a common character in
warrior classes), vaishya (the business classes), and Sanskrit drama, is invariably a ‘Brahmana’.
Shudra (the service classes). Early Hindu philosophy In Sanskrit drama, the ‘Vidhushaka’ has been used by
prescribed the four stages of life, that is brahmacharya playwriter like ‘kalidas’, ‘Bhasa’, etc. It was often a
(the student), grihastha (the householder), vanaprastha
Brahmin varna and a friend of the king.
(the recluse), and sanyasa (the freeman). It was
6. In the Gandhara sculptures the preaching mudra
believed that the law of karma governed the cycle of
associated with the Buddha’s First Sermon at
life from birth to death to re-birth & so on.
Sarnath is
3. Which one of the following is the most (a) abhaya (b) dhyana
fundamental difference between Mahayana
(c) dharmachakra (d) bhumisparsa
Buddhism and Hinayana Buddhism ?
(a) Emphasis on ahimsa Ans. (c) : In the Gandhara sculptures the preaching
(b) Casteless society mudra associated with the Buddha’s first sermon at
(c) Worship of gods and goddesses sarnath is ‘Dharmachakra’.
(d) Worship of stupa
The statues of Buddha built in the Gandhara
Ans. (c) : In the Hinayana community, Mahatma sculptures are made of black-grey stone, lime and
Buddha was considered a great man, but in the
clay. It depicts Buddha in different postures. Buddha
Mahayana community, he was considered a God & he
was worshipped, along with the Buddha Worship of statues are in meditation, padmasana, dharmachakra
many bodhisattvas also started. pravartana, varada & abhaya etc.
IAS (Pre) GS 1994 Paper I 575 YCT
7. Toramana belonged to the ethnic horde of the Ans. (d) : The Ellora Caves chiseled into perfection
(a) Scythians (b) Hunas between the 4th and the 9th century belongs to the
(c) Yuechis (d) Sakas Buddhist, Hindu & Jain faiths. One of the most
Ans. (b) : Toramana was a Hunas. It attacked India fascinating archoeological sites in Maharashtra, Ellora
during the Guptas period. It was a race living in dates back to the Rashtrakuta dynasty, about 1,500
central Asia. The Yuechis were a genus of the Hunas. years ago, and is the epitome of Indian rock-cut
Scythians was a branch of Sakas. architecture. There are in all 34 caves of which 12 are
8. The name of the poet Kalidasa is mentioned in Buddhist, 17 Hindu and 5 Jain.
the
(a) Allahabad pillar inscription MEDIEVAL HISTORY
(b) Aihole inscription
(c) Alapadu grant 12. Who among the following is said to have
(d) Hanumakonda inscription witnessed the reigns of eight Delhi Sultans?
Ans. (b) : The name of the poet Kalidasa is mentioned (a) Ziauddin Barani
in the Aihole inscription. The inscription is found at (b) Shamsi Siraj Afif
Aihole in Karnataka state. The Aihole inscription was (c) MinhajusSiraj
written by the Ravikirti, court poet of Chalukya king, (d) Amir Khusrau
Pulakesin II who reigned from 610 to 642CE. Ans. (d) : Amir Khusrau witnessed the reigns of eight
9. The statue of Gomateswara at Sravanabelagola Delhi sultans. He is regarded as the “Father of
was built by Qawwali”. Amir Khusrau was an Indian musician,
(a) Chandragupta Maurya scholar and poet. He is said to have witnessed the
(b) Kharavela reigns of eight Delhi sultans from ‘Ghiyasuddin
(c) Amoghavarsha Balban to Sultan Muhammad bin Tughlaq’.
(d) Chamundaraya 13. The first Indian ruler to organize Haj pilgrimage at
Ans. (d) : Shravanabelagola is home to the 18 m high the expense of the state was
statue of Lord Gometeshwara; Considered to be one (a) Alauddin Khilji (b) Feroz Tughlaq
of the world’s tallest free-standing monolithic status. (c) Akbar (d) Aurangzeb
Constructed in 981 AD by Chavundaraya, a Western Ans. (b) : Firoz Shah Tughlaq was the first Indian
Ganga dynasty warrior. It is carved out of a single ruler to organize Haj pilgrimage at the expense of the
block of granite and looms on the top of Vindhyagiri state. He was a Muslim ruler of Tughlaq dynasty, who
Hill. reigned over the Delhi Sultanate from 1351 to 1388.
10. Which one of the following important trade He succeeded to the throne of Delhi Sultanate after
centres of ancient India was on the trade route the death of Muhammad bin Tughlaq. Firoz Shah
connecting Kalyana with Vengi ? Tughlaq is remembered in history abolishing about
(a) Tagara (b) Sripuram twenty four types of taxes on people. Even land tax
(c) Tripuri (d) Tamralipti was reduced. He only collected four taxes sanctioned
Ans. (a) : Tagara was the important trade centre of in Quran.
ancient India and it was on the trade route connecting 14. The Mansabdari System introduced by Akbar
kalyana with vengi. It is located on east coast of India was borrowed from the system followed in
and horses, coral and wine were procured and traded (a) Afghanistan (b) Turkey
with Greek or Roman traders. Unlike other places on (c) Mongolia (d) Persia
the Satavahana period trade routes, which can be Ans. (c) : Akbar introduced Mansabdari system of
physically identified with the places existing today, administration in 1571 with the help of Shahbaz
“Tagara” remained illusive. Khan. However, the origin of Mansabdari system can
11. The caves and rockcut temples at Ellora are be traced back to Mongols (Changez Khan). In India,
(a) Buddhist it was initially implemented by Babur and Humayun
(b) Buddhist and Jain but it was Akbar who reformed and institutionalised
(c) Hindu and Jain the mansabdari system as the basis of civil and
(d) Hindu, Buddhist and Jain military administration.

IAS (Pre) GS 1994 Paper I 576 YCT


15. Who among the following ladies wrote a Ans. (a) : The Nizam of Hyderabad was the first to
historical account during the Mughal period ? accept a well-framed subsidiary alliance in 1798.
(a) Gulbadan Begum (b) Nurjahan Begum After the Third Anglo-Maratha War (1817-19),
(c) Jahanara Begum (d) Zebunnissa Begum Maratha ruler Baji Rao II also accepted a subsidiary
Ans. (a) : Gulbadan Begum wrote a historical account alliance.
during the Mughal period. Gulbadan Begum born as a 18. The capital of the kingdom of Maharaja Ranjit
Mughal princess and the daughter of the founder of
Singh was
the Mughal Empire Babur. She is mostly known for
(a) Amritsar (b) Patiala
her book on biography of Humayun, Humayun-Nama
(c) Lahore (d) Kapurthala
who was the half-brother and the great Emperor. It is
Ans. (c) : The capital of the kingdom of Maharaja
said that she wrote Humayun-Nama on the Emperor
and her nephew Akbar’s request. She has mentioned Ranjit Singh was Lahore. In July, 1799 he seized
some of the rare information regarding the conflicts Lahore, the capital of the Punjab (now the capital of
between her half brother humayun and Kamran Mirza. Punjab province, Pakistan). The Afghan king, Zaman
16. Consider the map given below : Shah, confirmed Ranjit Singh as governor of the city,
but in 1801 Ranjit Singh proclaimed himself Maharaja
of the Punjab.
19. The first to start a joint stock company to trade
with India were the
(a) Portuguese (b) Dutch
(c) French (d) Danish
Ans. (b) : Dutch East India Company was the first to
start a joint stock company to trade with India. It was
trading company founded in the Dutch Republic in
1602 to protect that state’s trade in the Indian Ocean
The shaded area in the map shows the kingdom and to assist in the Dutch war of Independence from
of the Spain. The company prospered through most of the
(a) Satavahanas 17th century as the instrument of the powerful Dutch
(b) Chalukyas of Vatapi commercial empire in the East Indies (Preset-day
(c) Rashtrakutas Indonesia). It was dissolved in 1799.
(d) Marathas 20. Who among the following was the leader of a
Ans. (d) : The shaded area in the map shows the number of anti-British revolts in Sambalpur ?
kingdom of the Marathas, Maratha confederacy, (a) Utirat Singh
alliance formed in the 18th century after Mughal (b) Surendra Sai
pressure forced the collapse of Shivaji’s kingdom of (c) Kattabomman
Maharashtra in western India. After the Mughal (d) Sayyid Ahmad Barelvi
Emperor Aurangzeb’s death (1707), Maratha power
Ans. (b) : Surendra Sai was the leader of a number of
revived under Shivaji’s grandson Shahu Ji. He
Anti British revolts in Sambalpur. Surendra Sai was
decided to expand northward with armies under the
born on 23rd January, 1809 in Khinda village near
Peshwas control.
Lapanga on the way to Jharsuguda about 40 km to the
north of Sambalpur, Odisha. He ascended the throne
MODERN HISTORY sambalpur in the year 1827. Popularly known as a
17. The first Indian ruler who joined the Subsidiary warrior, Surendra Sai led a revolution against the
Alliance was British Rulers, assembling thousands of protesters.
(a) the Nawab of Oudh 21. Which one of the following upheavals took place
(b) the Nizam of Hyderabad in Bengal immediately after the Revolt of 1857?
(c) Peshwa Baji Rao II (a) Sanyasi Rebellion (b) Santhal Rebellion
(d) the King of Travancore (c) Indigo Disturbances(d) Pabna Disturbances

IAS (Pre) GS 1994 Paper I 577 YCT


Ans. (c) : Indigo Disturbances were upheavals after (a) All of them were leading members of the
the Revolt of 1857. Indigo Disturbances was a revolt International Communist Movement
by the farmers against British planters who had forced (b) M. Barkatulla was the Prime Minister and the
them to grow under terms that were very exploitative. rest were Ministers in the Provisional Government
It took place in Bengal in 1859-60. They attacked the of Free India established by a group of
policemen who intervened. Pabna uprisings was a revolutionaries at Kabul
resistance movement by peasants against lords of
(c) All of them were among the prominent
Bengal in Yusufshahi pargana in 1873-76.
revolutionaries outside India operating in
Sanyasi uprising lasted from 1770 to 1820s. The
Santhal Revolt took place in 1855-56. different countries during the Freedom
Movement
22. The significance of the Bengal Regulation of 1793
lies in the fact that (d) All of them were accused in the case relating
(a) it provided for the establishment of the Supreme to the throwing of a bomb on Lord Hardinge
Court Ans. (c) : Madam Bhikaji Cama (Germany), M.
(b) it restricted the application of English law to Barkatulla (Afaghanistan), V.V.S. Iyer and M.N. Roy
Englishmen only were the prominent revolutionaries and during the
(c) it accommodated the personal laws of Hindus freedom movement, all of them were working in
and Muslims various countries outside India.
(d) it provided for the appointment of the Indian
Law Commission 25. Which of the following international events
Ans. (c) : The system as codified in Bengal influenced the course of the national movement
Regulation of 1793, provided that the East India in India before the advent of Mahatma Gandhi ?
Company’s service personnel be divided into three I. Italian-Abyssinian War, 1898
branches: II. Boxer Movement in China
Revenue, judicial, and commercial. III. Revolutionary Movement in Ireland
The judiciary was recorganized; there were district IV. Victory of Japan in the Russo Japanese War
judges with magisterial powers responsible to Choose the correct answer from the code given
provincial courts in civil cases and to courts of circuit below.
in criminal cases. The law administered was Hindu Codes:
and Muslims personal law and a modified Muslim
(a) I, II and III only (b) I, II and IV only
criminal code. The higher ranks of services were
(c) III and IV only (d) I, II, III and IV
restricted to Europeans, thus depriving Indians of any
responsible office. Ans. (d) : Italian-Abyssinian War, 1898, Boxer
23. Those who joined Mahatma Gandhi during the Movement in China, Revolutionary Movement in
Champaran struggle included Ireland and victory of Japan in the Russo-Japanese
(a) Vallabhbhai Patel and Vinobha Bhave war were influenced by the course of the national
(b) Jawaharlal Nehru and Rajendra Prasad movement in India before the advent of Mahatma
(c) Rajendra Prasad and Anugraha Narayan Gandhi. All of them are correct. The Indian
Sinha revolutionary movement was most affected by the
(d) Mahadev Desai and Maniben Patel defeat of Russia by a small country like Japan.
Ans. (c) : Champaran (West Champaran) is the name
26. Who among the following was the leader of a
of a district which is situated in Bihar, and to provide
number of anti-British revolts in Sambalpur?
justice to the farmers of champaran distict, Mahatma
(a) Utirot Singh
Gandhi started the Champaran Satyagraha
(b) Surendra Sai
movement.
(c) Kattabomman
He had few other people with him like Braj Kishore
(d) Sayyid Ahmad Barelvi
Prasad, Rajendra Prasad, Narayan Sinha and
Ramnavmi Prasad. Ans. (b) : By virtue of the Doctrine of Lapse, Lord
This was Gandhi’s first disobedience movement and Dalhousie annexed Sambalpur in 1849, as Narayan
because of this movement, the British government Singh had no male successor to succeed him. During
saw the power of Gandhi. the uprising of 1857 the sepoys set Surendra Sai and
During the Champaran movement, Sant Raut called his brother Udyant Sai free. The resistance to British
Gandhi as “Bapu”, which means “Father”.
continued in Sambalpur under the leadership of
24. What is common among Madam Bhikaji Cama,
Surendra Sai.
M. Barkatulla, V.V.S. Iyer and M.N. Roy ?
IAS (Pre) GS 1994 Paper I 578 YCT
INDIAN GEOGRAPHY Ans. (c) : In ancient Indian historical geography the
name ‘Ratnakara’ denoted the Indian ocean.
27. Consider the map of India given below : Ratnakara is the sea between India and Sri Lanka,
which has been famous for beautiful gems especially
pearls since ancient times. In ‘Raghuvansh’, the great
poet Kalidas has used the word ‘Ratnakara’ for the sea
situated between Indo-Srilanka.
30. The average elevation of Tibetan Plateau above
sea level is
(a) 2 km (b) 3 km
(c) 4 km (d) 5 km
Ans. (d) : The Tibetan plateau is also called the
plateau of Tibet or the inland plateau of Asia. It is the
largest and highest plateau in the world with an
average elevation of around 4,500 to 5,000 meters.
The map shows The Tibetan plateau is known as “the roof of the
(a) an isohyet world”.
(b) allIndia water divide
31. The only anthropoid ape to be found in India is
(c) a combination of national highways
(d) a combination of major railway routes the
Ans. (b) : The map shows all-India water divide. The (a) hanuman monkey
major water divides of India are as follow: (b) liontailed macaque of Western Ghats
Western Ghats: (c) slow loris of Assam
The Western Ghats is a mountain range that runs (d) whitebrowed gibbon of Assam
parallel to the western coast of the Indian peninsula, Ans. (d) : The only anthropoid ape to be found in
located entirely in India. It forms a major water divide India is the whitebrowed gibbon of Assam.
in the peninsula. The western hoolock gibbon has a much wider range,
Aravali Range: as it found in all states of the north-east, restricted
The Aravali Hills lie on the western and north-western between the south of the Brahmaputra river and east
margins of the peninsular plateau. of the Dibang river. The eastern hoolock gibbon
In this way, Aravali acts as water divide of India. inhabits specific pockets of Arunachal Pradesh and
28. Which one of the following types of erosion is Assam in India.
responsible for the formation of Chambal 32. The Himalayas are formed of parallel fold ranges,
Ravines ? of which the oldest range is
(a) Splash (b) Sheet (a) the Siwalik Range
(c) Rill (d) Gully (b) the Lesser Himalayas
Ans. (d) : Badland topography is a major feature of (c) the Great Himalayan Range
the Chambal valley and it is characterized by an (d) the Dhaula Dhar Range
undulating flood plain, gullies and ravines. Ravines Ans. (c) : The oldest range of Himalayas are the Great
are a type of fluvial erosional feature and are formed
Himalayan Range. The Himalayan Ranges can be
as a result of constant vertical erosion by streams and
rivers flowing over semiarid and arid regions. grouped into four parallel longitudinal Mountain belts
The region through which the Chambal river flows of varying width, each having distinct physiographic
does not receive enough rainfall to create ravines that features and its own geological history. They are
are 60 – 80 m deep. designated, from south to north, as the outer, or sub-
29. In ancient Indian historical geography the name Himalayas (also called the Siwalik Range); the lesser,
‘Ratnakara’ denoted or lower, Himalayas; the Great Himalayan Range
(a) the Arabian Sea (Great Himalayas); and the Tethys, or Tibetan,
(b) the Bay of Bengal Himalayas. Farther north lie the Trans-Himalayas in
(c) the Indian Ocean Tibet proper. From west to east the Himalayas are
(d) the confluence of the Ganga, the Yamuna divided broadly into three mountainous regions:
and the mythical Saraswati at Prayaga western, central, and eastern.

IAS (Pre) GS 1994 Paper I 579 YCT


33. The hallmark of watershed development in the (a) Both A and R are true, and R is the correct
semiarid regions of India is the explanation of A
(a) establishment of sheds on large scale, so that (b) Both A and R are true, but R is not a correct
the rain water does not evaporate explanation of A
(b) undertaking of earthworks, soil conservation (c) A is true, but R is false
measures and treeplantation, so as to (d) A is false, but R is true
conserve soil moisture and recharge
Ans. (a) : The monsoonal rainfall decreases as one
underground water
goes towards the west and north-west in the Ganga
(c) drilling deep tubewells so as to tap water at the
plain due to the moisture bearing Monsoonal winds go
rockstrata level
higher up as one moves up in the Ganga plain.
(d) setting up a system of tanks by banking
water from seasonal rivers 36. If safe storage is to be ensured, the moisture
content of foodgrains at the time of harvesting
Ans. (b) : The agriculture in low rainfall areas of
eastern Rajasthan is characterized by high risks from should not be higher than
drought degraded natural resources and pervasive (a) 14% (b) 16%
poverty, food insecurity and mal-nutrition. In this (c) 18% (d) 20%
region, water is the main limiting factor for upgrading Ans. (a) : If safe storage is to be ensured, the moisture
rainfed agriculture. For such areas integrated content of food grains at the time of harvesting should
watershed management is recognized as a potential not be higher than 14%. One of the most critical
approach for agriculture growth and rehabilitation of physiological factors in successful grains storage is
fragile and degraded lands. the moisture content of the crop. High moisture
34. Consider the map of India given below : content leads to storage problems because it
encourages fungal and insect problems, respiration
and germination. However, moisture content in the
growing crop is naturally high and only starts to
decrease as the crop reaches maturity and the grains
are drying.
37. The highest milk producing breed of goat in
India is
The map, read with the index provided, refers to
(a) Barbari (b) Jamunapari
the distribution of
(c) Black Bengal (d) Beetal
(a) winter rains (b) pressure
Ans. (b) : The highest milk producing breed of goat in
(c) annual rainy days (d) mean temperature
Indian is Jamunapari. This goat breed is native to
Ans. (d) : The map, read with the index provided,
India subcontinent and the goat breed name is derived
refers to the distribution of mean temperature. India is
a vast country and can be categorized into six main from the river “Jamuna Par” in Uttar Pradesh.
subcategories that have been described by the koppen Jamunapari goats are exported to Indonesia and they
climate classification. The annual temperature of the have great success over there. Basically this goat
country can be classified according to the season that breed is reared for both milk and meat.
comprise of the summer season, winter season and the 38. Which of the following fish are bred exclusively
rainy season. in the cold regions of India ?
35. Assertion (A): The monsoonal rainfall decreases I. Catla.
as one goes towards the west and northwest in the II. Tinca.
Ganga plain. III. Bhekti.
Reason (R): The moisture bearing monsoonal IV. Trout.
winds go higher up as one moves up in the Ganga Choose the correct answer from the codes given
plain. below.
In the context of the above two statements which Codes:
one of the (a) I and lV (b) I and IV
following is correct ? (c) II and IV (d) III and IV
IAS (Pre) GS 1994 Paper I 580 YCT
Ans. (c) : The exotic cold water fishes introduced in transport which received the maximum impetus
India include trouts, mirror carps, crucian carps and in most of the post-independence period.
tenches (Tinca). The fist attempt to transplant trout in Which of the following are the reasons for this ?
India was made at Nilgiris in 1907. In the Himalayas, I. Railways are cheap to operate but the associated
trout has been transplanted in Kashmir and Himachal capital costs are very high.
Pradesh, in Garhwal Himalayas, Arunachal Pradesh, II. Given the geographic spread of the human
Nagaland, Meghalaya and in certain areas of Nepal. settlements, it is unrealistic that the railways
Tinca also called the doctorfish. It was first introduced alone can serve the needs of transport.
into the Ooty lake as early as 1874. III. The per unit (tonne/km) cost of transport is
39. Which one of the following soil types of India is cheaper in road compared to rail transport.
rendered infertile by the presence of excess iron ? IV. Given the indivisible nature of the Railways, it
(a) Desert sand (b) Alluvial is not always convenient for the population to
(c) Podzolic (d) Lateritic avail it with the same ease as private cars, buses
Ans. (d) : Lateritic soil is rendered infertile by the or two wheelers.
presence of excess iron. Lateritic soil are rich in iron Select the correct answer from the codes given
and aluminum, low in silica, chemically acidic, and below
exhibit a soil profile different from that of other soils. Codes:
Lateritic soils are one of important soils and are (a) I, II and III (b) I, II and IV
widespread in tropical areas and subtropical climates. (c) I, III and IV (d) II, III and IV
40. The ideal climatic conditions for the cultivation Ans. (b) : Even though the Railways are the most
of rice are pervasive mode of transport in India. It is the road
(a) rainfall above 100 cm, temperature above transport which received the maximum impetus in
25°C most of the post independence period because of
(b) cool and moist climate for the entire crop Railways are cheap to operate but the associated
period capital costs are very high & the geographic spread of
(c) rainfall below 100 cm, temperature below the human settlements. It is unrealistic that the
25°C railways alone can serve the needs of transport.
(d) warm and dry climate during the entire crop The indivisible nature of the Railways. It is not always
period convenient for the population to avail of it with the
same ease as private cars, buses or two-wheelers.
Ans. (a) : The ideal climatic conditions for the
cultivation of rice are rainfall above 100 cm, 43. Which one of the following is the most important
reason for the establishment of the Vikram
temperature above 250C. Rice crop needs a hot and
Sarabhai Space Centre at Thumba?
humid climate. It is best suited to regions which have (a) The sea is near to the place in the East, West
high humidity, prolonged sunshine and an assured and South directions
supply of water. (b) It is a place near to a city in India, which is
In India rice is grown under widely varying conditions nearest to the Geographic Equator
of altitude and climate. Rice cultivation in India (c) It is situated on the Geomagnetic Equator
0 (d) Outer space near Thumba exhibits queer
extends from 8 to 35 N latitude and from sea level to
atmospheric phenomena
as high as 3000 meters.
Ans. (c) : Vikram Sarabhai Space Centre (VSSC),
41. The density of cattle population per 100 hectares Thiruvananthpuram, is the lead centre of ISRO
of gross cropped area is the highest in responsible for the design and development of launch
(a) Bihar (b) Haryana vehicle technology.
(c) Madhya Pradesh (d) Uttar Pradesh The major programmes at VSSC include Polar
Ans. (b) : The density of cattle population per 100 Satellite Launch Vehicle (PSLV), Geosynchronous
hectares of gross cropped area is the highest in Satellite Launch Vehicle (GSLV) and Rohini
Haryana. Sounding Rockets as well as the development of
Cattle population in Haryana is 11.85%. GSLV Mk-III. The most important reason to
42. Even though the Railways are the most pervasive establishment of the VSSC at Thumba, is that it is
mode of transport in India, it is the road situated on the Geomagnetic Equator.

IAS (Pre) GS 1994 Paper I 581 YCT


44. In recent years, there has been some concern 47. Consider the map given below :
over the threat posed by the Mathura Oil
Refinery and the thermal power plants to the Taj
Mahal in Agra. The scientific basis of any
possible damage to the Taj is mainly
(a) stratospheric ozone and the
chlorofluorocarbons which destroy it
(b) acid precipitation and tropospheric ozone
(c) increasing levels of atmospheric CO2 which
produce the greenhouse effect
(d) ultraviolet radiation (240260 nanometers
wavelength) and the fumes from the heavily
polluted Yamuna river
Ans. (b) : The Taj Mahal, a mausoleum constructed in
white Marble by the mughal emperor Shahjahan in the
The numbers marked 1, 2, 3 and 4 respectively
beloved memory of his wife, has been accredited as
indicate the islands of
being one of the seven wonders of the world, by
(a) the Bahamas, Azores, Falkland and Cape Verde
millions of people worldwide.
(b) the Bahamas, Azores, Cape Verde and
The most damaging of all forms of pollution on the
Falkland
marble of Taj have been air and water pollution.
(c) Azores, Cape Verde, the Bahamas and
Effluents are released from over 1700 factories that
Falkland
exist in and around Agra, in the form of gases such as
(d) Cape Verde, the Bahamas, Falkland and
CO, CO2, SO2, chlorofluoro carbons, nitrogen oxides
Azores
particulate matter etc. and in recent years, there has
been some concern over the threat posed by the Ans. (c) : From the Map, the numbers marked 1, 2, 3
Mathura Oil Refinary and the thermal power plants to & 4 respectively indicate the islands of Azores, Cape
the Taj Mahal in Agra. The scientific basis of any Verde, the Bahamas and Falkland.
possible damage to the Taj is mainly by acid 48. The greatest diversity of animal and plant species
precipitation and tropospheric ozone. occurs in
(a) temperate deciduous forests
WORLD GEOGRAPHY (b) tropical moist forests
(c) heavily polluted rivers
45. The time taken by the sun to revolve around the (d) deserts and savannas
centre of our galaxy is Ans. (b) : Tropical and subtropical moist broadleaf
(a) 25 million years (b) 100 million years forests generally found in large, discontinuous patches
(c) 250 million years (d) 500 million years centered on the equatorial belt and between the
Ans. (c) : Our Milky Way galaxy is a big place. Even Tropics of cancer and Capricorn.
at this blazing speed, it takes the sun approximately These trees number in the thousands and contribute to
225-250 million years to complete one journey around the highest levels of species diversity in any terrestrial
the galaxy’s center. This amount of time-the time it major habitat type. In general, biodiversity is highest
takes us to orbit the center of the galaxy is sometimes in the forest canopy.
called a cosmic year. These forests are home to more species than any other
46. The length of its day and the tilt of its axis are terrestrial ecosystem: Half of the world’s species may
almost identical to those of the earth’. This is true live in these forests.
of 49. The acceleration due to gravity of a catastrophic
(a) Uranus (b) Neptune earthquake will be
(c) Saturn (d) Mars (a) > 550 cm/sec2
Ans. (d) : Mars has two poles like Earth, it is inclined (b) > 750 cm/sec2
at an angle of 20 degrees to its axis, due to which the (c) > 950 cm/sec2
seasons change here like Earth. (d) > 980 cm/sec2

IAS (Pre) GS 1994 Paper I 582 YCT


Ans. (d) : The acceleration due to gravity of 53. Which one of the following is in the correct
catastrophic earthquake will be > 980 cm/sec2. sequential order in which these appeared under
50. When it is noon along 82° 30' E longitude along simulated primitive earth condition ?
what degree of longitude it will be at 6.30 am ? (a) Methane, Hydrogen cyanide, nitriles,
(a) along 165° E aminoacids
(b) Hydrogen cyanide, methane, nitriles,
(b) along 67°30' W
aminoacids
(c) 0° E or W
(c) Aminoacids, nitriles. Hydrogen cyanide,
(d) along 82° 30' W
methane
Ans. (c) : When it is noon along 820 30’ E longitude (d) Nitriles, aminoacids, methane, Hydrogen
i.e. at the time 12:00 pm = 820 30’ E cyanide
Degree of longitude 6:30 am = ?
Ans. (a) : In the primitive land conditions simulated
12:00 – 6:30 = 5:30 h
in the earth, complex organic compounds gradually
∵1 longitude is equal to 4 minute emerged from simple organic compounds. In this
5 × 60 + 30 330
∴5:30 h = = = 82.5 = 82 .30 '
0 sequence, first methane gas, then hydrogen cyanide,
4 4 nitrile and amino acids were produced. It is known
From solving we prove that at the time of 6:30 am the that with the help of amino acids, the first organism
longitude = 00 mid line originated in water.
51. The first bird is believed to have appeared on the 54. The shortest air route from Perth to London is
surface of the earth (a) Perth, Bombay, Rome, London
(a) 300 million years ago (b) Perth, Ankara, Paris, London
(b) 200 million years ago (c) Perth, Aden, Paris, London
(d) Perth, Mombasa, Rome, London
(c) 150 million years ago
Ans. (b) : The distance between London and Perth-
(d) 100 million years ago 8989 miles (or- 14466 km).
Ans. (c) : Birds are one of the most recognizable and The shortest air route Perth-Ankara, Paris - London.
diverse groups of modern vertebrates. Over the past 55. Given below is a map of America: The shaded
two decades, a wealth of new fossil discoveries and (marked 1 and 2), dotted (marked 3) and the
phylogenetic and macro evolutionary studies had plain areas (marked 4) indicate respectively, the
transformed our understanding of how birds regions where the languages spoken are
originated and became so successful. Birds evolved
from theropoda dinosaurs during the Jurassic (around
165-150 million years ago) and their classic small,
lightweight, feathered and winged body plan was
pieced together gradually over tens of millions of
years of evolution rather than in one burst of
innovation.
52. An aeroplane takes off from 30° North Latitude,
50° East Longitude, and lands at the opposite end
of the earth. Where does it land ?
(a) 30° North Lat., 50° West Long
(b) 30° South Lat., 50° West Long
(c) 50° North Lat., 30° West Long
(d) 30° South Lat., 130° West Long
Ans. (a) : When a aeroplane takes off from 300 North
Latitude, 500 East Longitude, and lands at the opposite
end of the earth then its landing position is 300 North
Latitude & 500 West Longitude because to go from (a) Spanish, French, Portuguese and others
one hemisphere to the other the airplane has to pass (b) French, Spanish, Portuguese and others
through the poles. (c) French, Portuguese, Spanish and English
(d) Portuguese, French, Spanish and others
IAS (Pre) GS 1994 Paper I 583 YCT
Ans. (c) : From the map of America (South America 58. Freedom of the Press in India
& North America). The shaded area marked (1) is the (a) is specifically provided in Article 19 (1)(a) of
Eastern coast of Canada where French is spoken. the Constitution
The shaded area marked (2) is the area of Brazil, (b) is implied in the wider freedom of expression
where Portuguese is spoken.
guaranteed by Article 19(1)(a) of the
The shade marked (3) is the area of Argentina &
Mexico where Spanish is spoken and the shaded Constitution
marked (4) is the area of Canada & USA where (c) is guaranteed under the provisions of Article
English is spoken. 361 A of the Constitution
56. Consider the map given below : (d) emanates from the operation of the Rule of
Law In the country
Ans. (b) : Freedom of the Press in India is implied in
the wider freedom of expression guaranteed by Article
19(1)(a) of the constitution. Article 19(1)(a) says that
all citizens shall have the right to freedom of speech
and expression.
59. Who among the following are appointed by the
President of India ?
I. The Chairman, Finance Commission .
II. The Deputy Chairman, Planning
Commission
The map shows the sea voyage route followed by III. The Chief Minister of a Union Territory.
(a) Thomas Cook (b) Ferdinand Magellan Choose the correct answer from the codes given
(c) John Cabot (d) Vasco da Gama below.
Ans. (b) : In search of fame and fortune, Portuguese Codes:
explorer Ferdinand Magellan (C 1480-1521) set out (a) I only (b) I and II only
from Spain in 1519 with a fleet of five ships to (c) I and III only (d) II and III only
discover a western sea route to the Spice Islands. En Ans. (c) : From the question the Chairman, Finance
route he discovered what is now known as the strait of Commission and the Chief Minister of a Union
Magellan and became the first European to cross the Territory are appointed by the President of India. The
Pacific Ocean. Finance Commission is a constitutionally mandated
The voyage was long and dangerous, and only one body that is at the centre of fiscal federalism. It was
ship returned home three years later. set up under Article 280 of the constitution.
By the Central Government Act Section 45 in the
INDIAN CONSTITUTION AND POLITY Government of Union Territories Act, 1963 the Chief
Minister shall be appointed by the president and the
57. The basic structure theory of the Constitution of other ministers shall be appointed by the President on
India implies that the advice of the Chief Ministers.
(a) certain features of the Constitution are so 60. Assertion (A): In India, the political parties
essential to it that they cannot be abrogated which formed the governments represented the
(b) fundamental rights cannot be abridged or majority of seats secured in the elections to the
taken away House of the People at the Centre and the
(c) the Constitution cannot be amended except in Legislative Assemblies in the States but not the
accordance with the procedure prescribed in majority of votes.
Art, 368 Reason (R): The elections based on the majority
(d) the Preamble of the Constitution cannot be vote system decide the result on the basis of
amended for it is not a part of the Constitution relative majority of votes secured.
and at the same time represents its real spirit In the context of the above statements, which one
Ans. (a) : The basic structure theory of the of the following is correct?
(a) Both A and R are true, and R is the correct
constitution of India implies that the certain features
explanation of A
of constitution are so essential to it that they cannot be (b) Both A and R are true, but R is not a correct
abrogated. explanation of A

IAS (Pre) GS 1994 Paper I 584 YCT


(c) A is true, but R is false 63. In which respect have the Centre State relations
(d) A is false, but R is true been specifically termed as “municipal relations”?
Ans. (c) : In India, the political parties which formed (a) Centre’s control of the State in the legislative
the governments represented the majority of seats sphere
secured in the elections to the House of the people at (b) Centre’s control of the State in financial
the Centre and the Legislative Assemblies in the states matters
but not the majority of votes. (c) Centre’s control of the State in the
61. Which one of the following statements is correct ? administrative sector
(a) All the members of the Rajya Sabha are (d) Centre’s control of the State in the planning
elected by State Legislative Assemblies process
(b) As the Vice-President is the exofficio Ans. (d) : Manitoba Municipal Relations works with
Chairman of the Rajya Sabha, only a municipalities, planning districts, and non-
member of the Rajya Sabha can contest for the
governmental organizations to build and maintain
office of the Vice-President
strong, healthy and well managed communities across
(c) A point of difference between the Lok Sabha
our province.
and the Rajya Sabha is that while a candidate to
As part of this work, Municipal Relations delivers
the Lok Sabha can contest from
training, ongoing advice, technical analysis and
any State in India, a candidate to the Rajya
funding related to: land management, community
Sabha should ordinarily be are resident of the
revitalization, infrastructure & building the capacity
State from where he is contesting
(d) The Constitution of India explicitly prohibits of local governments to provide services.
the appointment of a nominated member of 64. Which of the following statements regarding the
Rajya Sabha to the post of a minister advisory jurisdiction of the Supreme Court are
Ans. (c) : A point of difference between the Lok correct ?
Sabha and the Rajya Sabha is that while a candidate to I. It is binding on the Supreme Court to give
the Lok Sabha can contest form any state in India, a its opinion on any matter referred to it by
candidate to the Rajya Sabha should ordinarily be a the President.
resident of the state from where he is contesting and II. The full bench of the Supreme Court hears
the Rajya Sabha elections are different from the Lok any reference made to it under its power of
Sabha. Unlike the Lok Sabha or the Lower House of advisory jurisdiction.
the Parliament, member of Rajya Sabha or the Upper III. The opinion given by the Supreme Court on
House of Parliament are not directly elected by the a reference under advisory jurisdiction
public. The members of the Rajya Sabha are elected is not binding on the
by elected representatives of states and Union government.
Territories. IV. Not more than one reference at a time can
62. Which one of the following determines that the be made to the Supreme Court under its
Indian Constitution is Federal? power of advisory jurisdiction.
(a) A written and rigid Constitution Select the answer from the codes given
(b) An independent Judiciary below:
(c) Vesting of residuary powers with the Centre
Codes:
(d) Distribution of powers between the Centre
(a) I and II (b) I and III
and the States
Ans. (d) : The federal feature of Indian constitution (c) II and III (d) II and IV
states that there must be Distribution of powers Ans. (c) : Art 143 of the Indian Constitution confers
between central and state government. Article 246 of upon the Supreme Court advisory jurisdiction. The
constitution gives power to both union and state President may seek the opinion of the Supreme Court
government to make laws in the respective matters on any question of law or fact of public importance on
provided in the union list, state list and concurrent list. which he thinks it expedient to obtain such an opinion.

IAS (Pre) GS 1994 Paper I 585 YCT


65. Which of the following electoral systems have 67. Which one of the following statements regarding
been adopted for various elections in India? ‘Exit Poll’ is correct ?
I. System of direct elections on the basis of (a) ‘Exit Poll’ is a term used to denote a
adult suffrage. post-election survey of voters regarding the
II. System of proportional representation by candidate in whose favour they had
means of the single transferable vote. exercised their franchise
III. List system of proportional representation. (b) ‘Exit Poll’ and ‘Opinion Poll’ are one and the
IV. Cumulative system of indirect elections. same
Select the correct answer from the codes given (c) ‘Exit Poll’ is a device through which results of
below. voting can be most exactly predicted
Codes: (d) ‘Exit Poll’ is an administrative device made
(a) I and II (b) I and III recently by the Chief Election Commissioner to
(c) I, II and III (d) II, III and Lv prevent impersonation
Ans. (a) : The Election Commission of India is an Ans. (a) : An election exit poll is a survey based on
autonomous constitutional authority responsible for interviews with voters as they leave (or exit) their
administering Union and State election processes in balloting location. To estimate the outcome of an
India. The body administers elections to the Lok election in a particular constituency, a sample of its
Sabha, Rajya Sabha, State Legislative Assemblies in smallest voting units is drawn and at least one
India, and the offices of the president and vice interviewer is assigned to each sampled location on
president in the country. System of direct elections on Election Day. On a pre-determined and systematic
the basis of adult suffrage & system of proportional basis, the interviewer approaches people who have
representation by means of the single transferable vote already voted in order to obtain an interview.
have been adopted for various election in India. 68. Which of the following is/are among the functions
66. Given below are voting percentage of a political of the Election Commission of India ?
party secured in three successive elections to the I. Conduct of election for the posts of the
Lok Sabha: Speaker and the Deputy Speaker, Lok Sabha
Years : 1984 1989 1991 and the Deputy Chairman, Rajya Sabha.
Percent Votes : 7.4 11.4 22.4 II. Conduct of election to the Corporations and
The party which secured these percentages of votes Municipalities.
was III. Deciding on all doubts and disputes arising out
(a) Congress (I) of the elections.
(b) Bahujan Samaj Party Select the correct answer from the codes
(c) Bharatiya Janata Party given below.
(d) Communist Party of India (Marxist) Codes :
Ans. (c) : Bharatiaya Janta Party contested its first (a) I and II (b) I and III
election in 1984. It won only two Lok Sabha seats. (c) II and III (d) None of these
Party president LK Advani made Hindutva as the core Ans. (d) : In the above question, the members do the
ideology of the party and used the rhetoric of “pseudo work of option(1) themselves. Option (2) belongs to
secularism” and “Muslim appeasement” to great effect the State Election Commission while option (3) is
in winning popular support among the Hindus. related to the court. Hence, option (d) is the correct
The hardline Hindutva politics, which was inclined answer.
with its ideological parent RSS’s ideology, paid rich
electoral dividends in the next general elections in ECONOMY
1989 when the BJP won 85 Lok Sabha seats.
69. The principal reason why national, economic
Following Advanis ‘Ram Rath Yatra’ in 1990, the
planning is still being pursued inspite of
party grew in strength and in the 1991 general
embracing a market economy since 1991 is that
elections, it increased its strength to 120. (a) it is a Constitutional requirement
The party’s vote share went up to 20.1 percent in 1991 (b) the vast quantity of capital already deployed in
from 11.4 percent in 1989 and 7.4 percent in 1984. the public sector needs to be nurtured
IAS (Pre) GS 1994 Paper I 586 YCT
(c) Five Year Plans can continue to provide a 72. Convertibility of the rupee implies
long term perspective to the economy in (a) being able to convert rupee notes into gold
market friendly fashions (b) allowing the value of the rupee to be fixed by
(d) the market economy is mainly confined to market forces
industry and commerce and central planning (c) freely permitting the conversion of rupee to
in agriculture is necessary other major currencies and vice versa
(d) developing an international market for
Ans. (d) : The national economic planning is still currencies in India
being pursued inspite of embracing a market economy Ans. (c) : Convertibility is the ease with which a
since 1991 because the market economy is mainly
country’s currency can be converted into gold or
confined to industry and commerce and central
another currency through global exchanges. India’s
planning in agriculture is necessary.
rupee is a partially convertible currency. Rupees can
70. Fiscal deficit in the Union Budget means
be exchanged at market rates in certain cases, but
(a) the sum of budgetary deficit and net increase
approval is required for larger amounts.
in internal and external borrowings
(b) the difference between current expenditure and 73. Which one of the following governmental steps
has proved relatively effective in controlling the
current revenue
double digit rate of inflation in the Indian
(c) the sum of monetized deficit and budgetary economy during recent years ?
deficit (a) Enhanced rate of production of all consumer
(d) net increase in Union Government’s goods
borrowing from the Reserve Bank of India (b) Streamlined public distribution system
Ans. (a) : The difference between total revenue and (c) Pursuing an export oriented strategy
total expenditure of the government is fiscal deficit. It (d) Containing budgetary deficits and unproductive
is the negative balance that arises whenever a expenditure
government spend more money than it receives in the Ans. (d) : Government use fiscal policy such as
form of taxes and other revenues. government spending and levied taxes to stimulate
71. Which of the following are correctly matched? economic change. Expansionary policy is
Establishment of Manufacturing Industries Place characterised by increased government spending or
I. Setting up of the first cotton mill 1854 lower taxes to boost productivity.
Bombay Contractionary policy is characterized by decreased
II. Manufacture of the first machine made government spending or increased taxes to combat
paper 1870 near Calcutta rising inflation.
III. Setting up of the first cement factory 1904 Expansionary policy leads to higher budget deficits,
Madras and contractionary policy reduces deficits.
Choose the correct answer from the codes given
74. Devaluation of a currency means
below :
(a) reduction in the value of a currency visavis
Codes:
major internationally traded currencies
(a) I and II only (b) I and III only
(c) II and III only (d) I, II and III (b) permitting the currency to seek its worth in the
Ans. (b) : The first cotton mill in India was international market
established in 1818 at fort Gloster near Kolkata but (c) fixing the value of the currency in conjunction
with the movement in the value of a basket of
was a commercial failure. The second cotton mill in
predetermined currencies
India was established by KGN Daber in 1854 and was
(d) fixing the value of a currency in multilateral
named Bombay Spinning and Weaving company. consultation with the IMF, the World Bank
This mill is said to mark the true foundation of the and major trading partners
modern cotton industry in India. Ans. (a) : Under a fixed exchange rate system,
The first paper mill was set up as early as 1832 at devaluation and revaluation are official changes in the
Serampore (Rishra) in Hooghly district of West
value of a country’s currency relative to other
Bengal. But in terms of production capacity, the
industry remains very small. currencies.
First cement plant was set up in Porbandar, Gujarat in Under a floating exchange rate system market forces
1904 whereas the production of cement was started in generate changes in the value of the currency, known
1904 in Madras (Now Chennai). as currency depreciation or appreciation.

IAS (Pre) GS 1994 Paper I 587 YCT


75. Consider the following chart : Ans. (b) : To know whether the rich are getting richer
Interest Rates (in percentage) and the poor getting poorer, it is necessary to compare
the distribution of income of different sets of income
recipients different periods of time.
77. The difference between a bank and a Non
Banking Financial Institution (NBFI) is that
(a) a bank interacts directly with customers
while an NBFI interacts with banks and
governments
(b) a bank indulges in a number of activities
relating to finance with a range of customers,
while an NBFI is mainly concerned with the
term loan needs of large enterprises
(c) a bank deals with both internal and
international customers while an NBFI is
mainly concerned with the finances of
foreign companies
(d) a bank’s main interest is to help in business
The chart above shows the movement of nominal transactions and savings/ investment
interest rates and real interest rates (defined as activities while an NBFI’s main interest is in
nominal interest rate minus inflation). the stabilisation of the currency
Which one of the following is the correct
Ans. (b) : The basic difference between banks &
interpretation ?
NBFCs is that NBFC cannot issue cheques and
(a) Nominal interest rates decreased at a
demand draft like banks. Banks take part in country’s
constant rate in the period Feb. 1992 —
payment mechanism whereas Non-Banking Financial
Sept. 1993
Comparies are not involved in such transaction.
(b) Inflation has constantly decreased from Feb.
1992 — Sept. 1993 78. Which of the following taxes is/are levied by the
(c) Although the nominal rates of interest have Union and collected and appropriated by the
constantly declined from Feb. 1992—Sept. States ?
1993, the real rates have been steadily (a) Stamp Duties
increasing throughout the period (b) Passenger and Goods Tax
(d) Nominal rate of interest and real rate of (c) Estate Duty
interest are likely to become equal in 1994 (d) Taxes on Newspapers
Ans. (b) : The rates obtained by deducting inflation Ans. (a) : Stamp duty is a legal tax payable in full and
from the designated interest rate represent the real acts as an evidence for any sale or purchase of a
interest rates. It is clear from the chart that the real property. The levy of stamp duty is a state subject and
rates are increasing gradually. While the real interest thus the rates of stamp duty vary from state to state.
rate is 5.3 in Februarty, 1992, it rises to 8.8 in The centre levies stamps duty on specified
September, 1993 while the nominal interest rate has instruments and also fixes the rates for these
shown a gradual declining trends. So its is clear that instruments.
the inflation has constantly decreased from February, 79. Study the graphs given below :
1992 to September, 1993.
76. To know whether the rich are getting richer and
the poor getting poorer, it is necessary to
compare
(a) the wholesale price index over different
periods of time for different regions
(b) the distribution of income of an identical set of
income recipients in different periods of
time.
(c) The availability of foodgrains among two
sets of people, one rich and the other poor,
over different periods of time.
(d) The distribution of income of different sets of
income recipients at a point of time.
IAS (Pre) GS 1994 Paper I 588 YCT
The first graph depicts a hypothetical movement Ans. (b) : The export for all given eight countries has
of Bombay Stock. Exchange Sensitive index increased between 1980 and 1990, but the reason
which is an indicator of the average price behind this increment is not mentioned anywhere in
movement of a representative set of stocks. The
second graph depicts the averaged price of all question. Hence, it would be wrong to say that this is
automobile manufacturing companies. Which due to global inflation. Therefore, statement given in
one of the following is a valid inference ? option (b) is not true.
(a) The market in unstable because prices are
continuously fluctuating PHYSICS
(b) Automobile shares are more stable than
81. The colour of a star is an indication of its
shares in general
(a) distance from the sun
(c) There was a major political change in
(b) luminosity
June/July
(c) distance from the earth
(d) Automobile shares have been steadily
(d) temperature
improving in price rather than being affected
Ans. (d) : The colour of a star is an indication of its
by major market movements
temperature, and it can also suggest the star’s age.
Ans. (a) : From the graph we know that the first graph
Astronomers have long used stars colour to identify
depicts a hypothetical movement of Bombay Stock
their temperature, and they created a classification
Exchange sensitive index which is an indicator of the
average price movement of a representative set of system called spectral classes to do so, which places
stocks. stars into a category based on their temperature.
And the second graph depicts the averaged price of all 82. The basic reason for the extraordinary sparkle of
automobile manufacturing companies, So Automobile a suitably cut diamond is that
shares have been steadily improving in price rather (a) it has a very high transparency
than being affected by major market movements. (b) it has a very high refractive index
80. The table below compares apparel exports of (c) it is very hard
various countries between the years 1980 and (d) it has welldefined cleavage planes
1990. Ans. (b) : The refractive index of diamond (µ = 2.42)
Apparel Exports of Various Countries is very high. Due to which its angle of incidence is
(in billions) greater than the critical angle. This results in total
Country internal reflection of the light entering it, which is the
Hong Kong 4.98 15.41 fundamental factor in the brilliance of the diamond.
Italy 4.58 11.84 83. A corked bottle full of water when frozen will
China 1.63 9.67 break because
S.Korea 2.95 7.88
(a) the bottle contracts on freezing
Thailand 0.27 3.33
(b) the volume of water decreases on freezing
India 0.59 2.50
(c) the volume of water increases on freezing
Indonesia 0.10 1.65
Malaysia 0.15 1.32 (d) glass is a bad conductor of heat
Total 15.25 53.60 Ans. (c) : A corked bottle full of water when frozen
Which one of the following statements is not true ? will break because the volume of water increases on
(Restricting consideration to countries listed in the
freezing.
table only)
(a) Thailand has overtaken India in apparel 84. Which one of the following combinations of
export performance between 1980 and 1990 aperture and shutter speed of a camera will allow
(b) Total apparel export from the countries the maximum exposure ?
listed has increased (in dollar terms)
between 1980 and 1990 but part of this is (a) f22, 1/60 (b) f16, 1/125
likely to be due to global inflation (c) f8, 1/250 (d) f5.6, 1/1000
(c) India’s share of the apparel export market has Ans. (a) : The exposure of the camera depends on the
increased
area and time of the aperture of the camera, so the
(d) Maximum improvement in apparel export
performance has been achieved by Indonesia exposure will be directly proportional to d2×t.

IAS (Pre) GS 1994 Paper I 589 YCT


85. The speed of light will be minimum while passing (a) 1/2 (b) 1/4
through (c) 1/8 (d) 1/16
(a) glass (b) vacuum Ans. (d) : Let Ai be initial concentration
(c) water (d) air t1/2 = 5 years
Ans. (a) : Out of glass, vacuum, water and air, the Ai
∴After 5 years, concentration remaining is
velocity of light in glass will be the least. The velocity 2
of light in vacuum is 3 lakh km/sec & in glass 2 lakh For first order reaction, we know
km/se. 0.693
k = constant=
86. The colour of an opaque object is due to the t1/2
colour it 0.693
(a) absorbs (b) refracts ⇒k=
5
(c) reflects (d) scatters Now, let A20 be the concentration after 20 years.
Ans. (c) : The colour of an opaque object is due to the
2.303 A 
light waves that it reflects. The rest of the wavelengths Then, k = log10  i 
t  At 
get reflected from the surface of the object. These
reflected wavelengths of light reach our eyes and we 0.693 2.303  A 
⇒ = log10  i 
perceive them. 5 20  A20 
87. The pitch of the voice of women is in general
 A  0.693×20
(a) marginally lower than that of men ⇒ log10  i  =
(b) higher than that of men  A 20  5×2.303
(c) much lower than that of men  A 
(d) the same as that of men ⇒  i  = 16 (After solving)
 A 20 
Ans. (b) : Generally, the pitch of women’s voice is
Ai 1
higher than that of men, due to which the voice of ⇒ A 20 = so, fraction remaining =
A16 16
women is thin and sweet.
88. In an electronic watch, the component 91. Monazite is an ore of
corresponding to pendulum of a pendulum clock (a) zirconium (b) thorium
is a (c) titanium (d) iron
(a) transistor (b) crystal oscillator Ans. (b) : Monazite is an ore of thorium. In a typical
(c) diode (d) balance wheel acid digestion process, monazite is cracked using
Ans. (b) : The crystalline oscillator is the same sulfuric acid at 2300C for 4 h. The process is followed
component (instrument) in both the electronic clock by a selective precipitation process using ammonium
and the pendulum clock, due to this device, the hydroxide (NH4OH), sodium hydroxide (NaOH), and
electronic and pendulum clocks indicates the correct hydrogen peroxide (H2O2) to separate thorium from
time. rare earth elements and uranium.
92. Galvanised iron sheets are protected from rusting
CHEMISTRY due to the presence of a layer of
(a) lead (b) chromium
89. One mole of hydrogen gas burns in excess of (c) zinc (d) tin
oxygen to give 290 kJ of heat. What is the amount Ans. (c) : Galvanised iron sheets are protected from
of heat produced when 4g of hydrogen gas is
burnt under the same conditions ? rusting due to the presence of a layer of zinc.
(a) 145 kJ (b) 290 kJ Galvanizing is a process of coating iron or steel with
(c) 580 kJ (d) 1160 kJ zinc in order to provide greater protection against
Ans. (c) : One mole of hydrogen gas = 2 gm corrosion for the iron.
∵when 4 gm of hydrogen gas burnt the
The zinc coating on galvanized iron and steel
energy = 290×2 = 580 kj develops a natural carbonate on its surface by
90. The halflife of a radio active element is 5 years. exposure to the atmosphere and by the action of rain
The fraction of the radioactive substance that water.
remains after 20 years is
IAS (Pre) GS 1994 Paper I 590 YCT
93. Water is a good solvent of ionic salts because BIOLOGY
(a) it has a high boiling point
(b) it has a high dipole moment 96. The heart of human embryo starts beating
(c) it has a high specific heat
(a) in the first week of its development
(d) it has no colour
(b) in the third week of its development
Ans. (b) : Water is a good solvent of ionic salts
(c) in the fourth week of its development
because it has a high dipole moment. Water molecules (d) in the sixth week of its development
have a polar arrangement of the oxygen and hydrogen
Ans. (c) : The human heart is one of the first organs to
atoms – one side (hydrogen) has a positive electrical form and function during embryogenesis. By the end
charge and the other side (oxygen) had a negative of gestational week 3, passive oxygen diffusion
charge. This allows the water molecule to become becomes insufficient to support metabolism of the
attracted to many other different types of molecules. developing embryo, and thus the fetal heart becomes
Water can become so heavily attracted to a different vital for oxygen and nutrient distribution. The
molecule, like salt NaCl, that it can disrupt the initiation of the first heart beat via the primitive heart
attractive forces that hold the sodium and chloride in tube begins at gestational day 22, followed by active
the salt molecule together and, thus, dissolve it. fetal blood circulation by the end of week 4.
94. If a gas is compressed to half of its original 97. If father has blood group A and mother has
volume at 27°C, to what temperature should it be blood group O, then which one of the following
blood group may be found in their son ?
heated to make it occupy its original volume ?
(a) B (b) AB
(a) 327°C (b) 600°C
(c) O (d) B, AB or O
(c) 54°C (d) 300°C
Ans. (c) : Every person has a blood type (O, A, B, or
Ans. (a) : By the Charles’s Gas law, the volume
AB) and an Rh factor, either positive or negative. The
occupied by a fixed amount of gas is directly blood type and the Rh factor simply mean that a
proportional to its absolute temperature, if the person’s blood has certain specific features. The blood
pressure remains constant. type is found as proteins on red blood cells and in
PV body fluids. The Rh factor is a protein that is found on
i.e. =k the covering of the red blood cells. If the Rh factor
T
protein is on the cells, the person is Rh-positive. If
PV P V
∴ 1 1= 2 2 there is not Rh factor protein, the person is Rh-
T1 T2 negative.
Where, P1 is initial pressure, v1 is initial volume & ABO blood explained in following chart-
T1 is initial temperature. Father’s Blood type
P2 = last pressure, V2 = last volume & T2 = Last A
A B O
temperature. B
From the question p1, p2 = constant, v1 = 2v, v2 = v A, Chi
A,
T2 =270C or 273 + 27 = 3000k, T1 = ? B, ld’s
A B, A
From the law, A Blo
or or or
B, od
v1 v 2 2v v 2v×300 O A O
= or = or T2 = = 600k or typ
T1 T2 T2 300 v B
Moth O e
Initial temperature = 600 – 273 = 3270C er’s A, mu
A,
95. The main constituents of pearl are Blood B, st
B B, B
(a) calcium carbonate and magnesium carbonate type A Be
or or or
(b) aragonite and conchiolin B
O A O
(c) ammonium sulphate and sodium carbonate or
B
(d) calcium oxide and ammonium chloride O
Ans. (a) : Pearls are formed by calcium carbonate and A, A, A, A
magnesium carbonate. B, B, B, or

IAS (Pre) GS 1994 Paper I 591 YCT


or or or B II. It shows greater capacity, to resist diseases.
A A A III. It gets more vitamins as proteins.
B B B IV. Its growth in height is abnormal.
A B A Select the correct answer from the codes given
or or or O below.
O O B Codes:
(a) I, II and III (b) I, II and IV
98. What is the correct sequence of the following in a
(c) I, III and IV (d) II, III and IV
heart attack ?
I. Narrowing of the inner orifice of the vessel Ans. (a) : Breastfed infants is less obese, it shows
II. ‘Plaque’ from fibrous tissue and high greater capacity, to resist diseases & it gets more
cholesterol vitamins and proteins when compared to a bottle-fed
III. Inadequate supply of blood and oxygen infants. Breastfed infants have less difficulty with
IV. Clots of blood carried into the coronary digestion than do bottle-fed infants-so that breastfed
arteries. babies have fewer bouts of diarrhea or constipation.
Choose the answer from the codes given below : 101.In the balanced dietaries for a day of an adult
Codes: working man, weights of carbohydrate food and
(a) I, II, III, IV (b) II, IV, I, III total protein food (both superior and inferior)
(c) II, III, I, IV (d) IV, II, I, III should be in order of
(a) 600 gm and 420 gm respectively
Ans. (b) : Most heart attacks are the result of coronary
(b) 600 gm and 600 gm respectively
artery disease, also known as atherosclerosis or
(c) 420 gm and 600 gm respectively
“hardening of the arteries”, a condition that clogs
(d) 420 gm and 420 gm respectively
coronary arteries with fatty, calcified plaques over
time. The typical trigger for a heart attack is often a Ans. (a) : In the balanced dietaries for a day of an
blood clot that blocks the flow of blood through a adult working man, weights of carbohydrate food and
coronary artery. total protein food should be in order of 600 gm and
While the step-by-step process leading to a heart 420 gm respectively. Proteins are often called the
attack is not fully understood, major risk factors for body’s building blocks. They are used to build and
coronary artery disease are well-known. You can repair tissues. They help you fight infection.
control some of them including high blood pressure, Carbohydrates are the body’s main source of energy.
high cholesterol obesity, smoking and a sedentary The fruit, vegetables, dairy and grain food groups all
lifestyle. contain carbohydrates.
Fat give you energy, and they help the body absorb
99. Even though an animal is fed with carbohydrate
certain vitamins. Essential fatty acids help the body
rich diet, its blood sugar concentration tends to
function, but they aren’t made by your body-you have
remain constant. This is on account of the fact
to consume them.
that in the case of an animal
(a) hormones of pituitary glands control 102.Which one of the following animals stores water
metabolic process in the intestine ?
(b) hormones of pancreas cause such a condition (a) Moloch (b) Camel
(c) blood sugar is readily absorbed by liver (c) Zebra (d) Uromastix
(d) glucose undergoes autolysis Ans. (b) : Camels are able to survive for long periods
Ans. (b) : Even though an animal is fed with of time without either food or water as they store fat
carbohydrate-rich diet, its blood sugar concentration in their hump which can be used to give the camel
tends to remain constant. This is on account of the fact energy when resources are scarce. Camels are an
that in the case of an animal hormones of pancreas animal with multiple stomachs, having three different
cause such a condition. stomachs that specialize in absorbing as many
100. Which of the following are characteristics shown nutrients as possible from their harsh environment.
by a breastfed baby when compared to a bottle 103.Which one of the following is caused by the
fed baby ? expression of a recessive gene present on sex
I. It is less obese. chromosome ?
IAS (Pre) GS 1994 Paper I 592 YCT
(a) Rheumatism 107.Phenylketonuria is an example of an in born
(b) Nervous shock error of metabolism. This “error” refers to
(c) Muscular dystrophy (a) hormonal overproduction
(d) Cerebral hemorrhage (b) atrophy of endocrine glands
Ans. (c) : X-linked recessive diseases most often (c) inherited lack of an enzyme
occur in males. Males have only one X chromosome. (d) non disjunction
A single recessive gene on that X chromosome will Ans. (c) : Phenylketonuria is an example of an born
cause the disease. The Y chromosome is the other half error of metabolism. This “error” refers to inherited
of the XY gene pair in the male. However, the Y lack of an enzyme.
chromosome dosen’t contain most of the genes of the Mutations in the PAH gene cause phenylketonuria.
X-chromosome. Because of that, it doesn’t protect the The PAH gene provides instructions for making an
male. Diseases such as hemophilia and Duchenne enzyme called phenylalanine hydroxylase. This
Muscular dystrophy occur from a recessive gene on enzyme converts the amino acid phenylalanine to
the X-chromosome. other important compounds in the body.
104.Which one of the following vitamins is considered 108.Assertion (A): A lock of Einstein’s hair, if scientists
to be a hormone ? could locate it and extract its DNA, could help in
(a) A (b) B producing another Einstein, by cloning.
(c) C (d) D Reason (R): The DNA extracted from the cell of an
Ans. (d) : The body makes vitamin D in a chemical embryo at an early stage of development, can be
reaction that occurs when sunlight hits skin. This transferred to individual eggs which in turn can be
reaction produces cholecalciferol, and the liver implanted into the uterus of a surrogate mother to
converts it to calcidiol. The kidneys then convert the give birth to an identical off spring.
substance to calcitriol, which is the active form of the In the context of the above two statements, which
hormone in the body. one of the following is correct ?
Vitamin D helps the body absorb calcium so that (a) Both A and R are true, and R is the correct
blood calcium levels are at the ideal point. This helps explanation of A
(b) Both A and R are true, but R is not a correct
enable the mineralization of bone that is required for
explanation of A
strong, healthy bones. Yet this is just one function of (c) A is true, but R is false
the hormone. (d) A is false, but R is true
105.Rennin and lactase, the enzymes required to Ans. (a) : A lock of Einstein’s hair, if scientists could
digest milk, disappear in the human body by the locate it and extract its DNA, could help in producing
age of another Einstein, by cloning. Because the DNA
extracted from the cell of an embryo at an early stage
(a) two (b) three
of development, can be transferred to individual eggs
(c) five (d) eight which in turn can be implanted into the uterus of a
Ans. (d) : By the age of eight years, renin and lactase surrogate mother to give birth to an identical off
disappear in the human body. Rennin is a proteolytic spring.
enzyme found in the gut of many animals and
synthesized by the chief cells located in the mucosal ENVIRONMENT AND ECOLOGY
epithelium of the stomach. Its function is to coagulate
the proteins of milk and aid in its digestion. 109.Farm land tends to erode most rapidly when
Lactase is an enzyme that is required by the body for planted with
cleaving lactose into glucose and galactose. Lactose is (a) sorghum (b) potato
a sugar that is found in milk or its products. This (c) wheat (d) clover
enzyme is only found in the small intestine of Ans. (a) : Farm land tends to erode most rapidly when
planted with sorghum. The transition to agriculture
mammals.
from natural vegetation often can not hold on to the
106. Haemoglobin is dissolved in the plasma of soil and many of these plants, such as coffee, cotton,
(a) frog (b) fish palm oil, soyabean and wheat, can actually increase
(c) man (d) earthworm soil erosion beyond the soil’s ability to maintain it
Ans. (d) : Just like other animals, earthworms need to self. The effects of soil erosion go beyond the loss of
transport blood around the body to survive. fertile land. It has led to increased pollution and
Earthworm blood-unlike that of most invertebrates sedimentation in streams and rivers, clogging these
contains haemoglobin, a protein in red blood cells to water ways and causing declines in fish and other
carry oxygen around their bodies. species.

IAS (Pre) GS 1994 Paper I 593 YCT


110.Match List I with List II and select the correct 112. Which one of the following crops enriches the
answer from the codes given below the lists : nitrogen content in soil ?
List I List II (a) Potato (b) Sorghum
(Crops) (Crop Pests) (c) Sunflower (d) Pea
I. Rice (A) Aphid Ans. (d) :
II. Wheat (B) Gundhi bug • Pea crops enrich the soil with nitrogen.
III. Sugarcane (C) Greasy cutworm • Pea and beans are nitrogen-fixing crops and hence
IV. Gram (D) Top shoot borer moth they are called Legumes.
(E) Bollworm • They fix nitrogen in the soil with the help of
Codes: nitrogen-fixing bacteria present in their roots called
(a) I-B II-C III-D IV-E Rhizobium.
(b) I-C II-A III-B IV-D
•Nitrogen is essential for plant for its photosynthesis
(c) I-B II-A III-D IV-C
as it is a main component in chlorophyll and for plant
(d) I-E II-D III-A IV-C
rapid growth.
Ans. (c) : The damage caused by agricultural pests is
113. If water pollution continues at its present rate, it
one that is devastating. On a global scale, the losses
will eventually
caused by pests in arable crops accounts for about
(a) stop water cycle
10% - 15% of the total yield value. (b) prevent precipitation
Aphids are 1 – 3 mm, soft bodies insects that can be (c) make oxygen molecules unavailable to
green, grey, or black. It’s found in wheat. Gundhi bug water plants
found in rice, in the case of heavy infestation, the (d) make nitrate unavailable to water plants
whole year ahead may become devoid of mature Ans. (c) : If water pollution continues at its present
grains. rate, it will eventually make oxygen molecules
Buggy odour in rice field during milky stage. Greasy unavailable to water plants.
cutworm’s young larva feed on the epidermis of the Plants require specific nutrients, which they get from
leaves. the soil, water and air. Polluted water might kill some
Older larva come out at night and feed young plants of those nutrients, further denying the plant the ability
by cutting their stems. to get them.
111. Which of the following is/are used as
biofertilizers ? CURRENT AFFAIRS
I. Azolla
II. Bluegreen algae 114. Which of the following are the purposes to which
the National Renewal Fund has been set up ?
III. Alfalfa. I. To restructure sick small scale industries.
Select the correct answer using the codes given II. To help workers likely to be displaced due to
below. retrenchment in the process of industrial
Codes: restructuring.
(a) II alone (b) I and II III. To modernise existing industrial units.
(c) I and III (d) I, II and III IV. To help refugees from Bangladesh, Sri
Ans. (b) : ‘Biofertilizer’ is a substance which contains Lanka etc.
living microorganism which, when applied to seed, Choose the correct answer from the code given
below :
plant surfaces, or soil, colonizes the rhizosphere or the
Codes:
interior of the plant and promotes growth by (a) I and II (b) I and III
increasing the supply or availability of primary (c) I and IV (d) II and IV
nutrients to the host plant. Ans. (a) : The National Renewal Fund (NRF) was set
Biofertilizers add nutrients through the natural process up in February, 1992 by a Government resolution
of fixing atmospheric nitrogen, solubilizing which provided for its operation for a limited period
phosphorus and stimulating plant growth through the of time up to a maximum of ten years. Assistance
from NRF has been provided for the Voluntary
synthesis of growth promoting substances. There are
Retirement Scheme (VRS) and the scheme of
may types of bioferlizers- Azotobacter, clostridium counseling retraining etc. of workers rationalised from
Anabaena, Nostoc, Rhizobium, Frankia, etc. the organised sector.
IAS (Pre) GS 1994 Paper I 594 YCT
115. Which radioactive pollutant has recently drawn MISCELLANEOUS
the attention of the public, due to its occurrence
in the building material ?
(a) Plutonium (b) Thorium 119. Which of the following statements about the
(c) Radon (d) Radium Dunkel draft is/are correct ?
4+
Ans. (b) : Thorium (Th ) is a radioactive ion, which I. It is mandatory for the Government of India to
is used as nuclear fuel and discharged to the accept all its proposals in all the sectors.
environment as wastewaters produced from the II. In the field of agriculture, the main proposal is
nuclear fuels. to cut agricultural subsidies.
Thorium has recently drawn the attention of the public III. In the field of textiles, it reiterates the
due to its occurrence in the building material. operation of the multi fibre agreement.
Thorium powders or sponge are melted in electric arc IV. The proposals have already been accepted by
furnaces or in vacuum induction furnaces. the Parliament.
116.Indian expertise was recently used in the Select the correct answer from the codes given
archaeological restoration of the monuments at below.
(a) Jerusalem (b) Angkor Wat
Codes :
(c) Rome (d) Yucatan
(a) I only (b) I and II only
Ans. (b) : Indian expertise was recently used in the
archaeological restoration of the monument of Angkor (c) I, II and IV only (d) III and IV only
Wat. Ans. (c) : The “Dunkel Draft” in December 1991 was
Angkor, in Cambodia’s northern province of Siem an historic turning point in the negotiations. Mr
Reap, is one of the most important archaeological site Dunkel’s deep understanding of the technical issues
of Southeast Asia. It extends over approximately 400
combined with his shrewd diplomacy transformed
square km and consists of scores of temples, hydraulic
structure as well as communication routes. hundreds of thousands of pages of diverse, often
conflicting, proposal into a manageable single
117. “The Uniting for Peace Resolution” adopted by
the U.N. has document of some 500 pages, distilling the essence of
(a) enhanced the powers of the General Assembly the future WTO.
in a comprehensive way “Dunkel Draft” is mandatory for the government of
(b) reduced the powers of the General Assembly India to accept all its proposal in all the sectors. In the
(c) made no difference to the powers of the field of agriculture, the main proposal of “Dunkel
General Assembly Draft” is to cut agricultural subsidies.
(d) empowered the General Assembly to act in 120. Examine the following statements:
specific situations in matters of peace and I. Either A and B are of the same age or A is
security older then B.
Ans. (d) : General Assembly resolution 377 (v) is II. Either C and Dare of the same age or D is
known as the United for peace resolution. Adopted in older than C.
1950, the resolution resolves that the security council, III. B is older than C.
because of lack of unanimity of the permanent Which one of the following conclusions can be
members, fails to exercise its primary responsibility to drawn from the
act as required to maintain international peace and above statements?
security, the General Assembly shall consider the (a) A is older than B
matter immediately with the view to making (b) B and D are of the same age
recommendations to members in order to restore (c) D is older than C
international peace and security. (d) A is older than C
118. According to the newly adopted Constitution of Ans. (d) : We know that-
Russia, the Prime Minister is appointed by 1. A ≥ B
(a) the State Duma, the Lower Chamber of the 2. D ≥ C
Federal Assembly 3. B > C
(b) the President, with the consent of the State From 1 & 3
Duma A is greater than B & B is greater than C
(c) the President, with the consent of the ∴A is also greater than C i.e. A > C
Federal Council
121. E.T.A. is a terrorist organization in
(d) the Federal Council
(a) Cyprus
Ans. (b) : According to the newly-adopted (b) Sri Lanka
constitution of Russia, the Prime Minister is appointed (c) Spain
by the President, with the consort of the state Duma. (d) South Africa
IAS (Pre) GS 1994 Paper I 595 YCT
Ans. (c) : Created in 1959 at the height of Francisco Ans. (b) : The veto power is probably the UN
Franco’s dictatorship, which repressed Basque culture charter’s most significant distinction between
and language, ETA is accused of killing more than permanent and non-permanent members. Article 27(3)
850 people in its fight for an independent Basque of the charter establishes that all substantive decisions
homeland in northern Spain and southwest France. of the council must be made with “the concurring
122. Nirmal Hriday, Sanjiwini and Saheli are votes of the permanent Members.
(a) organisations engaged in social work Permanent members use the veto to defend their
(b) herbal medicines produced in India for national interests, to uphold a tenet of their foreign
hypertension, heart disturbances and policy or, in some cases, to promote a single issue of
neurological problems particular importance to a state.
(c) names of some savings schemes mooted by Since 16 February 1946, when the USSR cast the first
nationalised banks veto on a draft resolution regarding the withdrawal of
(d) institutions set up by Jayaprakash Narayan for foreign troops form Lebanon and Syria. The veto has
the rehabilitation of erstwhile dacoits been recorded 293 times.
Ans. (a) : Nirmal Hriday, Sanjivini and Saheli are 125.Who among the following became the Prime
agaristations engaged in social work, which were Minister of India without being earlier the Chief
started by Mother Teressa. These were social Minister of a State ?
organization working for the various problems of I. Morarji Desai
women and their rights. II. Charan Singh
123.The latest regional economic block to be formed III. V. P. Singh
is IV. Chandrashekhar
(a) ASEAN (b) COMECON V. P.V. Narasimha Rao.
(c) APEC (d) NAFTA Select the correct answer from the codes given
Ans. (d) : The North American Free Trade Agreement below.
(NAFTA), which was enacted in 1994 and created a Codes :
free trade zone for Mexico, Canada and the United (a) I, II and IV (b) II, II and V
States, is the most important feature in the U.S.- (c) II only (d) IV only
Mexico bilateral commercial relationship.
Ans. (d) : Shri Chandra Shekhar was born on 1st July,
NAFTA provides coverage to services except for aviation
1927, in a farmer’s family in village Ibrahimpatti in
transport, maritime, and basic telecommunications.
District Ballia, Uttar Pradesh. He was president of the
124.Which one of the following statements regarding
Janta Party 1977 to 1988. In 1988 Shekhar’s Janta
the “veto” power in the Security Council is
Party merged with several other opposition parties to
correct according to the United Nations Charter?
form the Janta Dal Party under the leadership of V.P.
(a) The decisions of the Security Council on all
Singh. He replaced V.P. Singh as India’s Prime
non-procedural matters must be made by an
Minister on 10th November, 1990, as head of a weak
affirmative vote of nine members, including the
minority government.
concurring, votes of the permanent members of
the Council. 126.Which one of the following is a language of
(b) Every permanent member of the Security Baluchistan but linguistically Dravidian ?
Council can prevent any decision from being (a) Brahui (b) Kui
(c) Parji (d) Pengo
accepted, by vetoing it.
(c) The term veto was used in Article 27 of the Ans. (a) : Brahui is a northern Draviadian language
United Nations Charter to enable any spoken Primarily by the Brahvi people in the central
permanent member of the Security Council to part of Balochistan Province, in Pakistan and
prevent any resolution from being passed by inscattered parts of Iran, Afghanistan and
the majority. Turkmenistan and by expatriate Brahui communities
(d) Any member of the Security Council can in Iraq, Qatar and U.A.E. It is isolated from the neared
prevent any resolution from being passed by Dravidian-speaking neighbor population of South
voting against it. India by a distance of more than 1,500 km.

IAS (Pre) GS 1994 Paper I 596 YCT


127.Assertion (A): The USA reemerged as India’s Ans. (a) : Eighth Five Year Plant 1992 – 1997
single largest import source in the early nineties. Objectives:
Reason (R): With swift political developments in Creation of employment, primary health facilities,
the erstwhile Soviet Union, India gradually began to check population growth, Drinking water &
rely on the USA for its defence requirements. In the vaccination in all villages and over all human
context of the above two statements, which one of development.
the following is correct?
Growth and diversification of agricultural activities.
(a) Both A and R are true, and R is the correct
Annual target Growth rate: 5.6%
explanation of A
Achieved growth = 6.7&
(b) Both A and R are true, but R is not a correct
Target percapita income growth: 3.8%
explanation of A
Achieved Growth = 4.4%
(c) A is true, but R is false
The Eighth plan started in April, 1992. One of the
(d) A is false, but R is true
Major highlight was modernization of the industries.
Ans. (c) : The USA re-emerged as India’s single
The plan was launched with twin objectives of
largest import source in the early nineties. But with
alleviation of poverty and unemployment.
swift political developments in the erstwhile Soviet
Union, India did not depend on America for its 131. ‘Self sufficiency’ in food. In the true sense
defence needs. freedom from hunger, has not been achieved in
128. The oldest monarchy in the world is that of India in spite of a more than threefold rise in
(a) U.K. (b) Nepal. food grains production over 1950-1990.
(c) Saudi Arabia (d) Japan Which of the following are the reasons for it ?
Ans. (d) : According to legend, the Imperial House of I. The green revolution has been restricted to
Japan was founded in 660 BCE by Japan’s first small pockets of the country.
Emperor, Jimmu, making it the oldest continuous II. The cost of food is too high compared to the
hereditary monarchy in the world. Although Japan’s earnings of the poor.
monarch has mythological origins, the country III. Too much emphasis is laid on wheat and
recognizes February, 11, 660 BCE as the official date paddy compared to the coarse grains.
of its founding. IV. The gains of the green revolution have
129. Which one of the following regions of Asia is largely accrued to the cash crop rather than
experiencing the highest annual growth rate of food crops.
population ? Select the correct answer from the codes given
(a) South Asia (b) South East Asia below.
(c) Central Asia (d) West Asia Codes :
Ans. (c) : The annual rate of population of the Central (a) I, II and III (b) I, II and IV
Asia region has been more than 2.5% during 2000-05 (c) I, III and IV (d) II, III and IV
and 0.3% during 2005-10. So. Central Asia regions is Ans. (a) : There are multiple understandings of food
experiencing the highest annual growth rate of self sufficiency that can apply at different levels of
population. analysis. According to FAO, “The concept of food
130. A major shift in the 8th FiveYear Plan from its self-sufficiency is generally taken to mean the extent
preceding ones is to which a country can satisfy its food needs from its
(a) the significant reduction in pubic sector own domestic production” (FAO, 1999). This most
outlays basic definition can apply at the level of individuals,
(b) the concentration of pubic investment in countries, or regions.
infrastructural sectors The green revolution has been restricted to small
(c) major investment in agriculture with a view to
pockets of the country in 1950-1990, the cost of food
promote exports
is too high compared to the earnings of the poor & too
(d) major investment in sectors in which
much emphasis is laid on wheat and paddy compared
industrial sickness has been a chronic
to the coarse grains are also a reasons for hunger.
problem

IAS (Pre) GS 1994 Paper I 597 YCT


132. Consider the following figures marked A and B : A test-tube baby is the product of a successful human
reproduction that result from methods beyond sexual
intercourse between a man and a woman and instead
utilizes medical intervention that manipulates both the
egg and sperm cells for successful fertilization.
135. Consider the diagram given below :

The manufacturing cost and projected sales for a


product are shown in figures A and B
respectively. What is the minimum number of
pieces that should be manufactured to avoid a
loss ?
(a) 2000 (b) 2500
(c) 3000 (d) 3500
Ans. (a) : From the graph ‘B’, we know that there is Five hundred candidates appeared in an
cost on the construction of two thousand pieces = 7 examination comprising tests in English, Hindi
lakh Rs. and Mathematics. The diagram gives the number
And two thousand pieces selling price = 2000 × 350
of candidates who failed in different tests. What
= 7 lakh Rs.
In this way, there will be no loss on the construction is the percentage of candidates who failed in at
of at least Rs. 2000 pieces. least two subjects ?
133. A man jumped at a speed of 5 metres per second (a) 7.8 (b) 0.078
from a stationary boat and the boat moved off (c) 6.8 (d) 1.0
with the speed of 0.5 metre per second. How Ans. (a) : Failed candidates in English & Hindi =
many times is the mass of the boat greater than
10%
that of the man ?
(a) 5.5 times (b) 4.5 times Failed candidates in Math & English = 12%
(c) 2.5 times (d) 10 times Failed condition in Math & Hindi = 12%
Ans. (d) : Let the mass of the person be m and that of Failed candidates percentage in three subjects = 5
the boat be n, the momentum of the boat and the So, failed candidates in two or more = 12 + 12 + 10 + 5
person before jumping = 0 = 39
Momentum of both after the person jumps = m×5 – Failed candidates in minimum two subjects
n×0.5
39
From the principle of conservation of momentum, = × 100 = 7.8%
m×5 – n×95 = 0 500
or n×0.5 = m×5 136. Sound waves travel at 300 m/s. Sound produced
n 5 at a point is heard by a person after 5 seconds
⇒ = = 10 times
m 0.5 while the same sound is heard by another person
134. In the case of a Test tube baby’ after 6 seconds. What could be the maximum and
(a) fertilization takes place inside the test tube minimum distances between the two persons?
(b) development of the baby taken place inside the
(a) 1.8 km, 0.15 km (b) 2.2 km, 0.20 km
test tube
(c) fertilization takes place outside the mother’s (c) 2.8 km, 0.25 km (d) 3.3 km 0.3 km
body Ans. (d) : When both listeners in one side then the
(d) unfertilized egg develops inside the test tube distance between them = 6×300 – 5×300 = 1800 –
Ans. (a) : The term “test-tube baby”, prior to the 1500
development of in vitro fertilization technologies in = 300 m or 0.3 km
the twentieth century, was used to refer to babies born When both are in different side then,
as a result of artificial insemination, William, Distance = 6×300 + 5×300 = 3300 m = 3.3 km
Pancoast, a physician from Philadelphia, performed 137. The surface area of a cube is 216 sq.m. What is
the first artificial insemination that led to a successful its volume ?
birth in 1884, marking the birth of the first test-tube (a) 100 cu m (b) 216 cu m
baby. (c) 480 cu m (d) 512 cu m

IAS (Pre) GS 1994 Paper I 598 YCT


Ans. (b) : The surface area of a cube = 6a2 = 216 141. A person at a point D on a straight road AC has
216 four options to go to B which lies on a
⇒ a2 = = 36 perpendicular to AC through D.
6
⇒ a =6
Volume of a cube = a3
= 63
= 216 m3
138. If Rs. 1000 is invested at 12% interest and
interest is compounded half yearly, what will be
the total amount at the end of one year ?
(a) Rs. 1120.00 (b) Rs. 1123.60 Which one of the following is the shortest route to
(c) Rs. 1126.20 (d) Rs. 1134.40 B?
Ans. (b) : (a) D to A and A to B
(b) D to E and E to B
Principal (p) = 1000 Rs. (c) D to C and C to B
Rate (r) = 12% yearly or 6% half yearly (d) D to F and F to B
Time (t) = 1 year Ans. (d) : From the figure, in the right angled triangle
 6 
2 DAE
So, amount (A) = 1000  1 +  DE
 100  sin 450 =
53×53 AD
= 1000 × = 1123.60 Rs. So, DE = AD × sin 450
50×50 1
139. Two persons start walking at a steady pace of 3 Or DE = 2 ×
km/hour from a road 1 intersection along two 2
roads that make an angle of 60° with each other. 2× 2
What will be the (shortest) distance separating =
2
them at the end of 20 minutes ?
(a) 3 km (b) 2 km = 2
(c) 1.5 km (d) 1 km Same as in ∆DBF-
DF
Ans. (d) : sin 300 =
BD
B 1 2
So, DF = BD × sin 300 = BD × = = 1 km
2 2
So, if a person move from point D to E and point F to
B then road will be too short. So right answer is (d)
O A 142. There are three tables containing two drawers
each. It is known that one of the table contains a
1 silver coin in each of its drawers, another table
After 20 minute the distance = × 3 = 1 contains a gold coin in each drawer while the
3 third table contains a silver coin in one drawer
From ∆AOB, and a gold coin in the other. One of the drawers
∠AOB = 60 0 of a table is opened and found to contain a silver
So, AOB is a Isosceles triangle in which distance is 1 km. coin. What is the probability that the other
drawer of that table contains a gold coin ?
140.Which one of the following figures will generate a (a) 1 (b) 0.5
cone when rotated about one of its straight edges (c) 0.25 (d) 0.75
? Ans. (b) : There are three tables containing two
(a) An equilateral triangle drawers each
(b) A sector of a circle i.e. S S G G SG
(c) A segment of a circle
(d) A rightangled triangle here S = silver coin
G = Gold coin
Ans. (d) : When a right angled triangle is rotated on We know that one of the table contains a silver coin in
one of its straight sides, the shape of a cone will be each of its drawers, another table contains a gold coin
formed. This can be clearly understood by the in each drawer while the third table contains a silver
following picture. coin in one drawer and a gold coin in the other. One
of the drawers of a table is opened and found to
contain a silver coin.
So, the probability of other drawer to contain gold
1
coin = = 0.5
2

IAS (Pre) GS 1994 Paper I 599 YCT


143. In the sequence 462, 420, 380, X, 306. X stands
for
(a) 352 (b) 342
(c) 332 (d) 322
Ans. (b) : There is a difference of 2 respectively in the
consecutive terms,
∵462 – 420 = 42 Which one of the holes in the fountain will throw
420 – 380 = 40 the water farthest ?
380 – 342 = 38 (a) 4 (b) 3
342 – 306 = 36 (c) 2 (d) 1
∴x = 380 – 38 = 342 Ans. Ans. (c) : According to the application of Bernoulli’s
theorem, if a hole is made in the middle of a tank full
144.Ram and Shyam work on a job together for four of water, then the stream of water will fall the farthest.
days and complete 60% of it. Ram takes leave But there is no hole in the middle of the fountain, so
then and Shyam works for eight more days to the pressure of the water column at hole 2 will be
complete the job. How long would Ram take to higher than at hole 3, hence stream will be thrown up
complete the entire job alone? to the maximum distance from the hole 2.
(a) 6 days (b) 8 days 148. If B J P is D M K then R S S
(c) 10 days (d) 11 days
would be
Ans. (c) : Ram and Shyam work in 4 days = 60% or
(a) C P I
3/5 (parts)
3 (b) T V N
∴Ram and Shyam’s work in 1 day =
20 (c) S J P
3 2 (d) T D P
∴Remaining work after 4 days = 1 − =
5 5 Ans. (b) : From question-
2 Given, So,
∴Shyam work in 8 days = +2 +2
5 B D R T
5
∴Shyam done total work = 8 × = 20 days J
+3
M S
+3
V
2
1 -5 -5
∴Shyam’s work in 1 days = part P K S N
20 Ans- TVN
3 1 1 149. The letters L, M, N, O, P, Q, R, S and T in their
Hence, Ram’s work in 1 day = − =
20 20 10 order are substituted by nine integers 1 to 9 but
Hence, Ram will do that work = 10 days not in that order, 4 is assigned to P. The
145.The square root of the cube of this number is the difference between P and T is 5. The difference
cube of its square root. It is not 1 and it is less between N and T is 3. What is the integer
than 6. What is it ? assigned to N?
(a) 2 (b) 3 (a) 7 (b) 5
(c) 4 (d) 5 (c) 4 (d) 6
Ans. (c) : By option, given number = 4 (let) Ans. (d) : From question
L M N O P Q R ST
( )
3
then, (43 ) = 4 6 4 9
∵ Difference between P & T = 5
⇒ 64 = ( 2 )
3
∴T = P + 5 = 4 + 5 = 9
⇒ 8 = 8 proved hence the number will be 4. ∵Difference between N & T = 3
∴N = T – 3 = 9 – 3 = 6 Ans.
146. Ten per cent of twenty plus twenty per cent of 150. Which one of the following four logical diagrams
ten equals represents correctly the relationship between
(a) 10 per cent of 20 (b) 20 per cent of 200 musicians, instrumentalists, violinists ?
(c) 1 per cent of 200 (d) 2 per cent of 200
Ans. (d) : From the question,
10 20
20 × + 10 × =2+2=4
100 100
= 2% of 200 Ans. (a) : Option (a) represents correctly the
147. Consider the figure of a fountain with four holes relationship between musicians, instrumentalists,
given below : violinists in logical diagrams.
IAS (Pre) GS 1994 Paper I 600 YCT
UNION PUBLIC SERVICE COMMISSION
Civil Services (Preliminary Exam) - 1993
GENERAL STUDIES : PAPER-I
Time: 2 hours Maximum Number: 200
4. The last in the succession of Jaina Tirthankaras
ANCIENT HISTORY was:
1. The term nishka, which meant an ornament in (a) Parsvanatha (b) Rishabha
the Vedic period, was used in later times to (c) Mahavira (d) Manisubrata
denote a/an: Ans. (c) : The 24th and the last Tirthankara of Jainism
(a) Weapon was Mahavir Swami. In Jainism, a Tirthankara is a
(b) Agricultural implement saviour and spiritual teacher of the dharma. The first
(c) Script tirthankara was Rishabhanath, who is credited for
(d) Coin formulating and organising humans to live in a society
Ans. (d) : During the Vedic Period, the term ‘nishka’ harmoniously.
was used for an ornament. But in later time it was
Mahavira, also known as Vardhamana was born in the
referred to denote gold coins of the Mauryans. Nishka
early part of the 6th century BCE into a Royal family in
is also referred as the world’s oldest currency coins. The
unit value of goods was a gold bar called ‘nishka’ Kundagram, Vaishali district, Bihar.
weighing three hundred and twenty ratis. 5. Ashokan inscriptions were first deciphered by:
2. The Buddhist sect Mahayana formally came into (a) Buhler (b) Robert Sewell
existence during the reign of: (c) James Prinsep (d) Codplngton
(a) Ajatashatru (b) Ashoka Ans. (c) : James Prinsep was a European scholar who
(c) Dharmapala (d) Kanishka deciphered the edicts of the Ashoka.
Ans. (d) : During the reign of Kanishka, the fourth Ashoka inscription were written in Magadhi, Prakrit
Buddhist council was held at Kundalvana in Kashmir in
using the Brahmi script and Kharosthi script. The
1st century to settle the dispute among the various sects
inscription and edicts of Ashoka refer to a collection of
of Buddhist. The council was headed by Vasumitra and
Ashvaghosha. After the end of council Buddhism was 33 inscriptions of the pillars of Ashoka, as well as
divided into two major sects named as Hinayana and boulders and cave walls. These were made by Emperor
Mahayana. Ashoka during his reign from 272 to 231 BC.
Mahayana sects considered Buddha as God. This sect 6. Chanakya, was also known as:
was in favour of idol worship. Sanskrit language was (a) Bhattasvamin (b) Rajasekhara
used for preaching and teaching. (c) Vishnugupta (d) Vishakhadatta
Hinayana sects considered themselves as original or old
form of Buddhism followers. They considered Buddha Ans. (c) : Chanakya who is also known as Kautilya or
as a pious man and were against of idol worship. They Vishnugupta was an ancient Indian teacher a great
use Pali language for Preaching. philosopher and a great economist. He helped
3. In Jainism ‘perfect knowledge’ is referred to as: Chandragupta Maurya to establish Mauryan Empire.
(a) Jina (b) Ratna Chanakya wrote two books, Arthashastra and Chanakya
(c) Kaivalya (d) Nirvana Niti. Arthashastra dealt with monetary and fiscal
Ans. (c) : The 24th Tirthanakara and real founder of policies, welfare, international relation and war
Jainism, Mahavir Swami get enlightenment at the age of strategies in detail.
42 under sal tree on the banks of the river Rijupalika.
As such he is considered the pioneer of the field of
He did rigorous penance for 12 years in the jungle in the
political science and economics in India.
search of Kaivalya or enlightenment. In Jainism
Kaivalya is also referred to as perfect knowledge. 7. The earliest rock-cut caves in western India are
The three jewels or Triratna of Jainism include Right those at:
Faith, Right Knowledge and Right Conduct. (a) Nasik, Ellora and Ajanta
Jainism does not condemn the Varna System and is (b) Junnar, Kalyan and Pitalkhora
considered as a religion of self-half. There are no gods (c) Ajanta, Bhaja and Kondane
or spiritual beings that will help human beings. (d) Bhala, Pitalkhora and Kondane
IAS (Pre) GS 1993 Paper I 601 YCT
Ans. (c) : The earliest rock-cut caves in Western India Ans. (d) : A tenth century inscription on a temple wall
are those at Ajanta, Bhaja and Kondane. of the brahman village of Uttaramerur gives the details
Ajanta caves are 30 rock-cut Buddhist cave monuments of village administration under Cholas. The Chola
in Aurangabad district of Maharashtra, India. Village system was very well enveloped and systematic.
The cave also present painting depicting the past lives The assemblies of the villages of chola inscription
and rebirth of the Buddha, pictorial tales from describe assemblies as the “Ur” and “Sabha”.
Aryasura’s Jatakmala and rock cut sculptures of
Buddhist deities. Ur refers to village and Nadu refers to district in Chola
empire. The Sabha was an important unit of
8. Among the four works mentioned below which
one is encyclopedic in nature? administration during the rule of the Chola empire. It
(a) Amarakosha (b) Siddhanta Shiromani was an assembly of members which consisted of rich
(c) Brihat-Samhita (d) Ashtangahrdaya Brahmin landowners. The function of the Sabha was to
collect revenues, enforce justice and to manage the
Ans. (c) : Brihat Samhita of Varahamihira is an
encylopedia of a astrological and other subjects of lands that were being gifted to the Brahmins. The
human interest including astronomy, planetary village assemblies looked after the maintenance of
movement, eclipses, rainfall, clouds, manufacture of peace, tanks, roads, public ponds revenue collection,
perfume, matrimony and domestic relations. judiciary, education and temples. The village assemblies
9. Who among the following is NOT associated with were in charge of the payment of taxes due from the
medicine in India? villages to the treasury.
(a) Dhanvantari (b) Bhaskaracharya 12 . Which one of the following is considered to be the
(c) Charaka (d) Susruta world’s greatest iconographical creations made
Ans. (b) : Bhaskaracharya was born in 1114 CE in the by the sthapatis of south India, particularly
Sahyadhri Mountain range, near the town of Patan in during the Chola period?
Maharashtra. He was an Indian mathematician and (a) Mahishasura mardini (b) Nataraja
astronomer. His notable work is Siddhanta Shiromani. (c) Rama (d) Somaskanda
Dhanvantri, Charaka and Susruta were associated with
Ans. (b) : Nataraja is considered to be the world’s
Medicine in India.
greatest iconographical creations made by the sthapatis
Charaka is usually known as the father of Indian
of south India, particularly during the Chola period.
medicine. He wrote Charak Samhita. Susruta wrote
Sushruta Samhita. The Chola Empire was founded by Vijayalaya. He
10. Consider the following passage: took over the Tanjore kingdom in the 8th century and
“In the course of a career on the road spanning led to the rise of the mighty Cholas by defeating the
almost thirty years, he crossed the breadth of the Pallavas. Tanjore was hence made the first capital of the
Eastern hemisphere, visited territories equivalent eminent Chola Empire.
to about 44 modern countries, and put behind
him a total distance of approximately 73,000 13. Which one of the following pairs of kings of
miles.” The world’s greatest traveller of pre- ancient and medieval periods of Indian history
modern times to whom the above passage refers and the works authored by them is correctly
is: matched?
(a) Megasthenes (b) Fa-hien (a) Krishnadevaraya : Samarangana sutradhara
(c) Marco Polo (d) Ibn Battuta (b) Mahendravarman : Mattavilasa prahasana
Ans. (d) : Ibn Battuta (also called Islamic Marco Polo) (c) Mahendravarman : Mattavilasaprahasana
was a Moroccan scholar and explorer who travelled (d) Somesvara : Amuktamalyada
extensively in Africa and Eurasia. He travelled more Ans. (b) : The Pallavas reached their zenith during the
than any other explorer in Pre-modern history totaling reign of Mahendravarman I (CE 600–630), a
around 73,000 miles in his 30 years travelling journey. contemporary of Harsha and Pulakeshin II. Among the
He came to India in reign of Muhammad Bin Tughluq.
sources of the period, Xuanzang’s account serves as a
He wrote a famous book named Rihlah.
link, as he traveled through the domains of all three
11. A lot of details, regarding the village
kings. Mahendravarman was a writer and artist of some
administration under the Cholas is provided by
the Inscriptions at distinction. The play associated with him, Mattavilasa
(a) Thanjavur (b) Uraiyur prahasana, treats in a farcical manner the idiosyncrasies
(c) Kanchipuram (d) Uttaramerur of Buddhist and Shaiva ascetics.

IAS (Pre) GS 1993 Paper I 602 YCT


MEDIEVAL HISTORY 17. In Mughal paintings one notices the adoption of
principle of foreshortening whereby near and
14. Who among the following introduced the famous
distant people and things could be placed in
Persian Festival of NAWROJ in India?
perspective. This was due to the influence of the
(a) Balban (b) Iltutmish
(a) British (b) Dutch
(c) Firoz Tughlaq (d) Alauddin Khilji
(c) Portuguese (d) Danish
Ans. (a) : The festival of Navroz or Jamshed-i-
Navroz/Jamshed-i-Nouroz is named after the Persian Ans. (c) : During Jahangir's reign (1605-1627), Mughal
king, Jamshed, who is credited for creating the Persian Paintings reached its zenith. He took inspiration from
or the Shahenshahi calendar. As per the legend, his life events and pushed for paintings on those
Jamshed saved the world from an apocalypse that came scenarios rather than illustrations on fiction.
in the form of a winter and destined to kill everyone. Mughal painting emerged from the Persian miniature
In India Balban introduced the famous Persian festival painting tradition, with additional Hindu, Buddhist, and
of Navroj in order to impress nobles and the rich Jain influences; it usually took the form of book
people. In India, the Parsi community, predominant in illustrations or single sheets preserved in albums.
Mumbai and Gujarat who follow Zoroastrianism, The adoption of principle foreshortening whereby near
celebrate Navroz or the Parsi New Year. and distant people and things could be placed in
15. Assertion (A): Babur wrote his memoirs perspective. This was due to the influence of the
Baburnama in Turki. Portuguese.
Reason (R): Turki was the official language of the
Mughal Court. MODERN HISTORY
In the context of the above two statements which
one of the following is correct? 18. The Ryotwari Settlement was introduced by the
(a) Both A and R are true and R is the correct British in the:
explanation of A (a) Bengal Presidency
(b) Both A and R are true but R is not a correct (b) Madras Presidency
explanation of A (c) Bombay Presidency
(c) A is true but R is false (d) Madras and Bombay Presidencies
(d) A is false but R is true Ans. (d) : Ryotwari System was introduced by Thomas
Ans. (c) : Babur wrote his autobiography named Munro in 1820. In Ryotwari System the ownership
Baburnama. Baburnama was written in Turkish rights were handed over to the peasants. British
language and hence, assertion A is correct. Abdur Government collected taxes directly from the peasants.
Rahim khanakhana translated Baburnama into Persian The revenue rates of the Ryotwari System were 50%
language. where the lands were dry and 60% in irrigated land.
The court language of Mughal was Persian not Turkish, It was introduced in madras presidency first then later
so reason mentioned here is incorrect. introduced in regions of Bombay, parts of Assam and
Hence, option (c) is correct , A is true and R is false. Coorg provinces of British India.
16. Which one of the following is a monument The Permanent Settlement was introduced first
constructed by Sher Shah? in Bengal and Bihar and later in the south district of
(a) Qila-i-Kuhna Mosque at Delhi Madras and Varanasi.
(b) Atala Masjid at Jaunpur 19. Uplift of the backward classes was the main
(c) Bara Sona Masjid at Gaur programme of the:
(d) Quwwat-ul-Islam Mosque at Delhi (a) Prarthana Samaj (b) Satyashodhak Samaj
Ans. (a) : Qila-i-Kuhna mosque is part of the Purana (c) Arya Samaj (d) Ramakrishna Mission
Qila fort complex located near the Junna River, north of Ans. (b) : Satyashodhak Samaj was founded with a
Humayun's Tomb. Qila-i-Kuhna mosque was built by purpose to give education to the lower castes.
Sher Shah Suri in 1541 C.E. an Afghan who sent Scheduled caste, scheduled tribes and made them aware
Emperor Humayun into exile between 1538-1555. He
of the exploiting tradition of the society. It was founded
also completed building the Purana Qila.
by Jyotiba Phule in 1873 in Pune. Like every other
Atala Masjid at Jaunpur was built by Ibrahim Shah
society this society also imposed a ban on the admission
Sharqui in1408.
of the high-class people, aristocrats, bureaucracy,
Bara Sona Masjid at Gaur was built by Nusrat Shah, a and Brahmins. This society only admits in it the members of
Bengal province ruler. Shudra Samaj or the people of the lower caste. Upper caste
Quwwat-ul-Islam Mosque at Delhi was built by Qutb- members can only become members of the society by
ud-din-Aibak in 1206-1210. analyzing its actions and habit.
IAS (Pre) GS 1993 Paper I 603 YCT
Prarthana Samaj was founded by Atmaram Pandurang 23. Which of the following were the main objectives
in 1867. of the Khilafat movement?
Arya Samaj was founded by Swami Dayanand Saraswati I. To rouse anti-British feelings among the
in 1875 in Bombay. Muslims of India.
Ramkrishna Mission was founded by Swami Vivekanand II. To reform the Muslim society.
in 1896 at Belur (Calcutta). III. To demand separate electorates and preserve
20. The foundation of modern educational system in the Khilafat.
India was laid by: IV. To save the Ottoman Empire and preserve
(a) The Charter Act of 1813 the Khilafat.
(b) Macaulay’s Minutes of 1835 Choose the correct answer from the codes given
below:
(c) The Hunter Commission of 1882
Codes:
(d) Wood’s Dispatch of 1854
(a) I and II (b) II and III
Ans. (b) : In 1835, the education policy of Lord (c) III and IV (d) I and IV
Macaulay was published titled “Minute on Indian
Education” in which he advocated educating Indians Ans. (d) : The Khilafat movement (1919-1924) was an
through English and also enrich Indian Languages so agitation by Indian Muslims allied with Indian
nationalism in the years following World War I. Its
that they become the vehicles of European scientific,
purpose was to pressure the British government to
historical and literary expression.
preserve the authority of the Ottoman Sultan as Caliph
Macaulay wanted the government to spend money only of Islam following the breakup of the Ottoman Empire
on imparting western education and not on oriental at the end of the war. Gandhiji saw an opportunity for a
education. He advocated the shutting down of all Hindu-Muslim mass movement and hence joined his
colleges where only eastern philosophy and subjects non-cooperation with the khilafat issue. It was started in
were taught. the year 1919. The non-cooperation movement started
Woods Dispatch of 1854 suggested that primary schools to spread in different parts of the country. In September
must adopt vernacular languages, high schools must 1921, the Ali brothers (Muhammad Ali and Shaukat
adopt Anglo-vernacular language and at college-level Ali) were arrested who had started the movement.
English should be the medium of education. Moreover, Gandhiji suspended the Non-Cooperation
21. Among the four political parties listed below, Movement after the ChauriChaura incident in 1922.
which one was the last to be formed? 24. “Indian cotton merchant, banker, Congressman
(a) The Conservative Party in Britain and a close associate of Mahatma Gandhi”. This
(b) The Democratic Party in U.S.A. description fits:
(c) The Republican Party in U.S.A. (a) G.D. Birla (b) M.R. Jayakar
(d) The Indian National Congress (c) Jamanalal Bajaj (d) V.S. Srinivasa Sastri
Ans. (d) : The Conservative Party in Britain was Ans. (c) : Jamnalal Bajaj was famous cotton merchant,
established in 1834. banker ,Congressman and a close associate of Mahatma
The Democratic Party in U.S.A was established in Gandhi. Jamanalal Bajaj was interested in initiatives
1828. such as the removal of untouchability, promotion
The Republican Party in U.S.A was established in 1854. of Hindi, and Khadi and village Industries. With the
The Indian National Congress was established in 1885. intent of eradicating untouchability, he fought for the
Thus it is very clear that the Indian National Congress non-admission of Harijans into Hindu temples in his
Party was established at the last. home town of Wardha. He began a campaign by eating
a meal with Harijans and opening public wells to them.
22. The first political organization established in
He opened several wells in his fields and gardens.
India in 1838 was known as:
In honour of his social initiatives the Jamanalal Bajaj
(a) British India Society
Award has been instituted by the Bajaj Foundation.
(b) Bengal British India Society
25. The title given by British Government to
(c) Settlers Association
Mahatma Gandhi which, he surrendered during
(d) Zamindari Association the Non-cooperation Movement, was:
Ans. (d) : The Zamindari Association, which was later (a) Hind Kesari (b) Kaisar-e-Hind
renamed Landholders' Society, was established on 12th (c) Rai Bahadur (d) Rt. Honourable
November 1837 by Dwarkanath Tagore, Prasanna Ans. (b) : Mohandas Karamchand Gandhi, who was
Kumar Tagore, Radhakanta Deb, Ramkamal Sen and awarded the Kaiser-i-Hind in 1915 by The Lord
Bhabani Charan Mitra. Hardinge of Penshurst for his contribution to ambulance
It has been described as “the first organisation of services in South Africa during the time of First World
Bengal with distinct political object.” War.

IAS (Pre) GS 1993 Paper I 604 YCT


In 1920, as a sign of protest, Gandhi returned the Ans. (b) : Champaran Movement (1917)
Kaiser-e-Hind medal which was awarded to him by the The Champaran Movement in 1917 was the first
British Empire. One of the reasons for this act was the Satyagraha movement inspired by Gandhi and a major
massacre of unarmed and non-violent protesters revolt in the Indian Independence Movement. Farmers
at Jallianwala Bagh in Amritsar, India, on 13th April
of Bihar were protesting against the farming of indigo
1919.
with barely any payment for it. The farmers approached
26. Which one of the following statements does
to Mahatma Gandhi when he returned to India from
correctly defines the term ‘drain theory’ as
propounded by Dadabhai Naoroji? South Africa in 1915. They wanted Mahatma Gandhi to
(a) That the resources of the country were being use the same methods that he had used in South Africa
utilised in the interest of Britain. to organize mass uprisings by people to protest against
(b) That a part of India’s national wealth or total injustice.
annual product was being exported to Britain
Ahmedabad Mill Strike (Feb -March, 1918)
for which India got no material returns.
(c) That the British Industrialists were being given In Ahmedabad there were many textile mills. Prices had
an opportunity to invest in India under the gone up and the mill workers were demanding higher
protection of the imperial power. wages. The mill owners did not agree. Gandhi
(d) That the British goods were being imported to sympathized with the workers and took up their cause.
the country making it poorer day by day.
He launched a struggle and resorted to peaceful
Ans. (b) : Dadabhai Naoroji was the first man to say
that internal factors were not the reasons of poverty in resistance. The workers proudly followed Gandhi and
India but poverty was caused by the colonial rule that pledged their full support to him. They paraded the
was draining the wealth and prosperity of India. In streets with large banners, and said they would not go
1867, Dadabhai Naoroji put forward the ‘drain of back to work until a settlement had been reached.
wealth’ theory in which he stated that the Britain was
completely draining India. He mentioned this theory in Kheda Movement (March - June, 1918)
his book Poverty and Un-British Rule in India, The Kheda Movement of 1918 was a major revolt in the
published in 1901. Further in his book, he stated the loss Indian independence movement. The movement was
of 200-300 million pounds of revenue to Britain. started in the Kheda district of Gujarat by the Mahatma
Dadabhai Naoroji considered it as a major evil of
British in India. Gandhi during the period of the British Raj. People of
Dadabhai Naoroji gave six factors that caused external Kheda were unable to pay the high taxes levied by the
drain. These are: British due to crop failure and a plague epidemic.
1. External rule and administration in India. Therefore, Mahatma Gandhi organised this movement
2. Funds and labour needed for economic
development was brought in by immigrants but to support the peasants of Kheda district.
India did not draw immigrants. Non-cooperation Movement (1920)
3. All the civil administration and army expenses of The Non-cooperation movement was launched on 1st
Britain were paid by India. August, 1920 by Mahatma Gandhi. Following the
4. India was bearing the burden of territory building
both inside and outside India. Rowlatt Act of 17th March, 1919, and the Jallian Wala
5. India was further exploited by opening the country Bagh massacre of 13th April, 1919, Indian National
to free trade. Congress withdrew its support for British reforms.
6. Major earners in India during British rule were Mahatma Gandhi launches the Non-cooperation
foreigners. The money they earned was never movement with the aim of self-governance and
invested in India to buy anything. Moreover they
left India with that money. obtaining full independence. His main motive was to
The drain of wealth was the portion of India’s wealth establish non-cooperation, non-violence and made this
and economy that was not available to Indians for non-violent movement as his weapons against
consumption, it was exported to Britain. Britishers.
27. What is the correct chronological sequence of the
28. Mahatma Gandhi was referred to as the ‘Father
following stages in the political life of Mahatma
of the nation’ first by:
Gandhi?
(a) Jawaharlal Nehru
I. Champaran
II. Ahmedabad Mill Strike (b) Vallabhbhai Patel
III. Kheda (c) C. Rajagopalachari
IV. Non-cooperation Movement. (d) Subhash Chandra Bose
Choose the answer from the codes given below: Ans. (d) : The title of Father of the Nation was given to
Codes: the Mahatma by Netaji Subhash Chandra Bose, who in
(a) II, IV, III, I (b) I, II, III, IV his address on Singapore Radio on 6th July, 1944 had
(c) IV, III, II, I (d) III, IV, II, I addressed Mahatma Gandhi as Father of the Nation.

IAS (Pre) GS 1993 Paper I 605 YCT


GEOGRAPHY 32. Consider the map given below

29. Which among the following pairs of places had


most marked difference in total rainfall per
annum even though located approximately along
the same latitude?
(a) Bangalore and Madras
(b) Bombay and Visakhapatnam
(c) Ajmer and Shillong
(d) Nagpur and Calcutta
Ans. (c) : Ajmer and Shillong have most marked
difference in total rainfall per annum even though
located approximately along the same latitude. There is The mean annual rainfall in the shaded parts of
not much rainfall in Ajmer all year long. The average India varies from
annual temperature is 24.7° C in Ajmer. The average (a) 100 to 150 cm (b) 150 to 200 cm
annual rainfall is 557 mm. The average annual (c) 200 to 250 cm (d) 250 to 300 cm
temperature in Shillong is 17.1° C. Precipitation here Ans. (a) : From given Map it is clear that the range of
averages 3385 mm. rainfall of shaded region is around 100-150 cm.
33. Consider the Climograph given below:
30. In Mizoram, the settlement pattern is mostly of
‘linear type’ along the ridges, because
(a) The valleys are cooler than the ridges
(b) Accessibility is easier on the ridge tops
(c) The ridges are cooler than the valleys
(d) The valleys are densely forested
Ans. (a) : A valley is an elongated depression in the
landscape that is formed by the action of water (V-
shaped) or carved out by glaciers (U-shaped). A ridge is
a landform feature characterized by a continuous
elevational crest with sloping sides.
In Mizoram, the settlement pattern is mostly of ‘linear
type’ along the ridges, because the valleys are cooler
than the ridge.
31. According to the ancient Indian geographical
concept, ‘Bharatvarsha’ was a part of: The above climograph relates to
(a) Northwestern region of India
(a) Pushkaradweepa (b) Jambudweepa
(b) Southern region of India
(c) Kraunchadweepa (d) Kushdweepa (c) Mid-central region of India
Ans. (b) : The geographical area and ancient name (d) Northeastern region of India
of Greater India in Ancient Indian History. The Ans. (d) : The above mentioned climography refers to
term Jambudweepa is used by Ashoka perhaps to represent North -Eastern region of India.
his realm in 3rd century BC, same terminology is then 34. A rainy day, as defined by the Indian
repeated in subsequent inscriptions for instance in Meteorological department, is a day when the
Mysorean inscription from the tenth century AD which rainfall at a point received is:
also describes the region, presumably Ancient India, (a) 0.5 mm to 1 mm in 24 hours
as Jambudweepa . (b) 1.1 mm to 1.5 mm in 24 hours
Its name is said to derive from a Jambu tree (another (c) 1.6 mm to 2 mm in 24 hours
name for the Indian Blackberry). The river so formed is (d) Above 2.5 mm in 24 hours
called Jambunadi (Jambu river) and flows through Ans.(d) : According to Indian Meteorological Department
Jambudweepa , whose inhabitants drink its water. (IMD), a rainy day has been defined as a day with
rainfall of 2.5 mm or more rainfall. IMD further defines
Insular continent Jambudweepa is said to comprise nine
that rainfall for a station is called heavy if it is greater
varshas (zones) and eight significant parvatas than 650 mm and very heavy if it is greater than 1300
(mountains). mm.

IAS (Pre) GS 1993 Paper I 606 YCT


35. Soils of Western Rajasthan have a high content of: 39. Which one of the following is the best strategy for
(a) Aluminum (b) Calcium environment friendly sustainable development in
(c) Nitrogen (d) Phosphorus Indian agriculture?
(a) Expansion of cultivable land, increased use of
Ans. (b) : Soil of Western Rajasthan have a high
superphosphate, urea and effective biocides.
content of Calcium. These soils have rich content of
(b) Wider popularization of high yielding crop
iron- oxide and devoid of calcium salts because calcium varieties, better and more frequent irrigation and
salts are soluble in water and are easily washed away. increased frequency of aerial sprays of
36. The Neyveli thermal power plant is fed by inorganic fertilizers and pesticides.
(a) Gondwana coal (b) Tertiary coal (c) Mixed cropping, organic manures, nitrogen
(c) Quarternary coal (d) Cambrian coal fixing plants and pest resistant crop varieties.
(d) Improved farm implements and machinery, use
Ans. (b) : Liginite Coal is mainly found in Neyveli of potent insecticides to minimize post harvest
Lignite fields of Cuddalore district and it is used in grain losses and mono culture cropping
Neyveli thermal power plant. Liginite coal are termed practices.
as tertiary Coal .Around 90% of total lignite reserve is Ans. (c) : Mixed cropping, organic manures, nitrogen
found in Tamil Nadu. fixing plants and pest resistant crop varieties are the
37. At the present state of our knowledge and best strategy for environment friendly sustainable
resources position, India will remain self development in Indian agriculture.
sufficient for the next three decades in: 40. The earnings of India from diamond export is
quite high. Which one of the following factors has
(a) Tin (b) Coking Coal
contributed to it?
(c) Copper (d) Petroleum
(a) Pre independence stockpiling of diamonds in
Ans. (b) : While India, with meagre reserves and the country which are now exported.
production of coking coal, does not figure in the world (b) Large production of industrial diamonds in the
rankings. India’s Coking Coal reserves have been country.
stagnant over the years. The vulnerability of the steel (c) Expertise available for cutting and polishing of
sector springs from the low availability and poor quality imported diamonds which are then exported.
of the material in the domestic market. India’s total coal (d) As in the past, India produces huge quantity of
reserves have shown an increase of about 7 billion gem diamonds which are exported.
tonnes during 2014-16, but there has hardly been any Ans. (c) : The earnings of India from diamond export is
addition to coking coal reserves and there is no increase quite high because of expertise available for cutting and
in the prime coking coal category. The prime coking polishing of imported diamonds which are then
coal reserves stand at 5.313 billion tonnes and proved exported.
prime coking coal reserves are 4.614 billions. Jharia 41. Which one of the following provides the
Coal fields in Jharkhand, which hold the major share of correct sequence of occurrence of four
towns/cities as one proceeds from west to east?
quality coking coal reserves continue to witness raging
(a) Rajkot, Baroda, Kharagpur, Bilaspur
fires despite the best efforts being put in over the
(b) Bikaner, Aligarh, Darbhanga, Nowgong
decade. (c) Indore, Rourkela, Agartala, Jamshedpur
38. Which one of the following shows the descending (d) Nasik, Aurangabad, Berhampur, Nanded
order of the four metropolitan cities in the Indian Ans. (b) : The correct sequence of occurrence of four
subcontinent in so far as their population towns/cities as one proceeds from west to east are :
strength is concerned? Bikaner 
→ Aligarh 
→ Darbhanga 
→ Nowgong
(Rajasthan) (Uttar Pradesh) (Bihar) (Assam)
(a) Bombay, Calcutta, Dhaka, New Delhi
(b) Bombay, Calcutta, New Delhi, Karachi 42. Consider the map given below:
(c) Bombay, Calcutta, Karachi, New Delhi
(d) Calcutta, Bombay, Dhaka, Karachi Rail Root
Ans. (b) : At the time when question was asked , the Karachi Delhi
correct order of population strength in descending order
are : Bhuj
Bombay > Calcutta > New Delhi > Karachi.
But at present (Census 2011) the population strength in
correct order are:
Bombay > New Delhi > Calcutta >Karachi.

IAS (Pre) GS 1993 Paper I 607 YCT


Two tourists, one travelling from Delhi to WORLD GEOGRAPHY
Karachi and the other to Bhuj, wanted company.
The railway Junction up to which they can travel 45. Even though Pluto is usually the farthest planet,
its path is highly elliptical, crossing inside the
together as shown in the:
path of some other planet. As a result for a
(a) Phulera period of twenty years from 1979, the farthest
(b) Jodhpur planet is not Pluto but:
(c) Luni (a) Jupiter (b) Saturn
(d) Balotra (c) Uranus (d) Neptune
Ans. (d) : As per the location mentioned in the map it is Ans. (d) : Neptune is the eighth and farthest-known
cleared that the location is Balotra. Solar planet from the Sun. In the Solar System, it is the
fourth-largest planet by diameter, the third-most-
43. Which one of the following is the best strategy for massive planet, and the densest giant planet. It is 17
environment friendly sustainable development in times the mass of Earth, slightly more massive than its
Indian agriculture? near-twin Uranus. Neptune is denser and physically
(a) Expansion of cultivable land, increased use of smaller than Uranus because its greater mass causes
superphosphate, urea and effective biocides. more gravitational compression of its atmosphere.
(b) Wider popularization of high yielding crop 46. When there is depletion of ozone in the
varieties, better and more frequent irrigation stratosphere, the wavelength of radiation striking
the earth’s surface will be:
and increased frequency of aerial sprays of
(a) 10^-7 M (b) 10^-10 M
inorganic fertilizers and pesticides.
(c) 10^-2 M (d) 100 M
(c) Mixed cropping, organic manures, nitrogen Ans. (a) : UV light has a shorter wavelength than
fixing plants and pest resistant crop varieties. visible light. Ultraviolet radiation lies between visible
(d) Improved farm implements and machinery, light and X-rays along the electromagnetic spectrum.
use of potent insecticides to minimise UV “light” spans a range of wavelengths between about
postharvest grain losses and mono culture 10 and 400 nanometers.
cropping practices. 1nm =10^ - 9
400 nm = 4 *10 ^ - 7 M
Ans. (c) : For sustainable development in Indian
Agriculture- 47. The solar eclipse achieves totality only in the
limited geographical regions because:
•Mixed cropping (Crop Rotation)
(a) The earth is not a smooth flat surface, but has
• Organic Manures (Organic farming) elevations and depressions
•N. Fixing Plant (Source -CEEW. Gov.) (b) The size of the shadow of the moon on the earth
India has huge reserve of natural gas. Natural gas is small compared to the cross section of the
reserves have been found in Bombay High, Gujarat, earth
Assam, Tamil Nadu and Tripura. Despite the (c) The trajectories of the earth around the sun and
the moon around the earth are not perfect circles
availability of natural gas in abundance, it is not being
(d) Sun rays can reach most of the peripheral
fully utilized. Natural gas is used in the production of
regions of the shadow of the moon due to a
chemical fertilizers especially urea. IFFCO fertilizer at atmospheric refraction
Amla (Bareilly) and Kondala (Gujarat) are based on Ans. (b) : The solar eclipse achieves totality only in the
natural gas. limited geographical regions because the size of the
44. Next to Hindi, the language spoken by the largest shadow of the moon on the earth is small compared to
number of people in the Indian subcontinent is: the cross section of the earth.
(a) Urdu (b) Telugu 48. Which one of the following is NOT an astronomical
(c) Bengali (d) Tamil object?
(a) Pulsar (b) Brittle Star
Ans. (b) : Bengali is the most spoken language in the
(c) Black hole (d) Quasar
Indian subcontinent after India. The Bengali language is
Ans. (b) : Brittle star, also called serpent star, any of the
spoken by about 215 million people in the Indian State
2,100 living species of marine invertebrates constituting
of West Bengal and Bangladesh.
the subclass Ophiuro idea (phylum Echinodermata).
After Hindi and Bengali, Urdu and Punjabi are the most Their long, thin arms usually five and often forked and
spoken language. spiny are distinctly set off from the small disk-shaped
In India Telugu is 2nd most spoken language after Hindi. body.

IAS (Pre) GS 1993 Paper I 608 YCT


49. Which of the following places have their standard
times, that are the same as GMT?
I. Accra II. Dublin
III. Madrid IV. Lisbon
Choose the correct answer from the codes given
below:
(a) I, II and III (b) I, III and IV
(c) I, II and IV (d) II, III and IV
Ans. (c) : Madrid uses GMT +1.00, whereas Accra,
Dublin, Lisbon shares same GMT of London.
Greenwich Mean Time is the mean solar time at the
Royal Observatory in Greenwich, London, counted Ans. (a) : Pygmy people are ethnic groups whose
average height is unusually short. The term pygmyism
from midnight. At different times in the past, it has been
is used to describe the phenotype of endemic short
calculated in different ways, including being calculated
from noon, as a consequence, it cannot be used to stature (as opposed to proportionate dwarfism occurring
in isolated cases in a population) for populations in
specify a particular time unless a context is given.
which adult men are on average less than 150 cm (4 ft
50. The production of cultured pearls is an important 11 in) tall.
cottage industry of: Pygmy house made with sticks and leaves in northern
(a) Belgium (b) West Indies Republic of the Congo.
(c) New Zealand (d) Japan 53. Which one of the diagrams given below represent
Ans. (d) : Japan has been the world’s major marine most closely the path of geostationary satellite in
pearl producer for over a century, and has developed space?
advanced technology in pearl oyster culture and pearl
production.
51. Consider the map given below. The countries
marked 1, 2, 3, and 4 in the map respectively
represent: Ans. (d) : A geostationary satellite is in an orbit that can
only be achieved at an altitude very close to 35,786 km
(22,236 miles) and which keeps the satellite fixed over
one longitude at the equator. The satellite appears
motionless at a fixed position in the sky to ground
observers. Thus it is very clear from option (D) that
geostationary satellite in space have path as shown in
diagram.
54. The solar eclipse achieves totality only in the
limited geographical regions because:
(a) The earth is not a smooth flat surface, but has
elevations and depressions.
(b) The size of the shadow of the moon on the earth
(a) Libya, Somalia, Nigeria & Namibia is small compared to the cross section of the
earth the trajectories of the earth around the sun
(b) Egypt, Somalia, Congo and Namibia
and the moon around the earth are not perfect
(c) Sudan, Ethiopia, Angola and Botswana circles.
(d) Algeria, Kenya, Ghana and Zambia (c) Evaporation of water from atmospheric
Ans. (a) : From given map of Africa , the location is precipitation occurs only in winter.
clear and they are as follows: (d) Sun rays can reach most of the peripheral
regions of the shadow of the moon due to a
• Libya – Tripoli (Capital)
atmospheric refraction.
• Somalia –Mogadishu (Capital) Ans. (b) : Size of the moon is roughly one fourth of the
• Nigeria –Abuja (Capital) size of earth, thus the moon casts a smaller shadow
• Namibia-Windhoek (Capital) which falls upon a limited portion of the earth. Thus
52. Among the four house types shown below, which solar eclipse achieves totality only in a limited portion
one represents the house type of the Pygmies? on earth.

IAS (Pre) GS 1993 Paper I 609 YCT


55. The Lifeline Express’ is: III. It was a multiparty body.
(a) The world’s first hospital on rails operating in IV. It worked through several committees.
India. Select the correct answer from the codes given
(b) An instrument used for watching the Below:
Codes:
fluctuations in the blood pressure of a patient
under general anesthesia. (a) I and II (b) II and III
(c) III and IV (d) I, II, III and IV
(c) A de-addiction programme to save drug
Ans. (c) : The Constitution of India was drafted by the
addicts.
Constituent Assembly, and it was implemented under
(d) A popular health magazine founded by the Cabinet Mission Plan on 16 May 1946. The
Ramnath Goenka. members of the Constituent Assembly were elected by
Ans. (a) : The only mobile medical train of its kind the provincial assemblies by a single, transferable-vote
started by Indian Railways, ‘Life Line Express’ is the system of proportional representation.
first such hospital in the world which is operated on theThe constituent assembly was based on limited adult
railway tracks of India to serve the sick people of the franchise and Constituent assembly was indirectly
country. This medical train is being operated for the elected and nominated body.
areas without the provision of good medical and health Thus statement 1 and 2 both are incorrect and statement
services. 3 and 4 are correct.
56. The only landlocked country in Southeast 59. The Swaran Singh Committee considered the
Asia is: question of:
(a) Laos (b) Thailand (a) More autonomy to Punjab on the model of
Jammu & Kashmir.
(c) Malaysia (d) Kampuchea
(b) The suitability of the Presidential form of
Ans. (a) : ‘Laos’ is the only Southeast-Asian country, government for India.
which is landlocked from all sides, that is, the borders (c) The precedence of the Directive Principles over
of this country do not touch the water or sea area. The Fundamental Rights.
countries around Laos are Vietnam, Thailand, (d) Administrative reforms.
Myanmar, Kampuchea (Cambodia) and China. The
Ans. (c) : In 1976, the Government of India set up the
capital of Laos is Vientiane.
Sardar Swaran Singh Committee to make
INDIAN CONSTITUTION AND POLITY recommendations about fundamental duties, the need
and necessity of which was felt during the operation of
57. Match List I with List II and select the correct the internal emergency. The committee recommended
answer by using the codes given below the lists: the inclusion of a separate chapter on fundamental
List I List II duties in the Constitution. It stressed that the citizens
should become conscious that in addition to the
(Features of the (Borrowed from)
enjoyment of rights, they also have certain duties to
Indian Constitution) perform as well. The Fundamental Duties were added in
I. Fundamental Rights (A) U.K. Part IV–A of Indian Constitution by 42nd Amendment
II. Parliamentary system (B) U.S.A. Act 1976. It also considered the question of the
of Government precedence of the Directive principle over Fundamental
III. Emergency provisions (C) Ireland Right.
IV. Directive Principles (D) German Reich 60. The only instance when the President of India
of State Policy exercised his power of veto related to:
(E) Canada (a) The Hindu Code Bill
Codes: (b) The PEPSU Appropriation Bill
(a) IB, IID, IIIE, IVA (b) IB, IID, IIIC, IVA (c) The Indian Post Office (Amendments) Bill
(c) IB,IIA,IIID, IVC (d) IB,IIC, IIIE, IVA (d) The Dowry Prohibition Bill
Ans. (c) : Fundamental Rights – USA Ans. (c) : Giani Zail Singh, the President of India from
Parliamentary system of government – U.K 1982 till 1987, exercised a pocket veto to prevent the
Emergency provision – German Indian Post Office (Amendment) Bill 1986 from
Directive Principles of State Policy – Ireland becoming law.
58. Which of the following statements regarding the But in case of the PEPSU Appropriation bill, absolute
Constituent Assembly are true? veto was used.
I. It was not based on adult franchise. A pocket veto allows a president to exercise veto power
II. It resulted from direct elections. over a bill by taking no action.

IAS (Pre) GS 1993 Paper I 610 YCT


Article 111 of the Indian constitution states that the Ans. (d) : Under Article 94 (b) it is clearly mentioned
President shall declare his assent to a bill passed by both that the speaker intends to resign, he can address his
houses of Parliament or withhold his assent, provided letter of resignation to Deputy Speaker.
that may he return the bill to Parliament for Whereas office of Speaker does not falls under pleasure
reconsideration. of president, so option a is incorrect.
61. The Chief Minister of a State in India is NOT And also he hold his office till new Loksabha’s first
eligible to vote in the Presidential election if:
assembly, so option c is also incorrect.
(a) He himself is a candidate.
He need to be member of the House at the time of
(b) He is yet to prove his majority on the floor of
election so option b is also incorrect.
the Lower House of the State legislature.
(c) He is a member of the Upper House of the State 64. Which one of the following statements is correct?
legislature. (a) Neither the Finance Commission nor the
(d) He is a caretaker Chief Minister. Planning Commission is a constitutional body.
Ans. (c) : According to article 54 of Indian (b) The scope of the Finance Commission is limited
Constitution, the member of Upper House of the state to a review of the revenue segment of the
legislature are not entitled to take part in presidential budget while the Planning Commission takes an
election. Only elected members of both Houses of overall review embracing both capital and
Parliament and of the State Legislative Assemblies are revenue requirements of the States.
members of the Electoral College for Presidential (c) No one can be a member of both the Finance
Election. Therefore, nominated members cannot vote in Commission and the Planning Commission at
this election.
the same time.
62. The abolition of the I.A.S. and the I.P.S. has been (d) There is no overlapping of work and
recommended by the:
responsibility of the Finance Commission and
(a) Dhebar Commission
those of the Planning Commission.
(b) Kalekar Commission
(c) Kher Commission Ans. (d) : There is no overlapping of work and
(d) Rajamannar Commission responsibility of the Finance Commission and those of
the Planning Commission as Finance Commission
Ans. (d) : In September 1969, the Government of Tamil
Nadu appointed a three member committee under the recommends on tax distribution between states and
chairmanship of Dr. P.V. Rajamannar to consider the Centre. Whereas Planning Commission suggest about
entire question of Centre-state relations. overall development and growth of the country.
After two years, in 1971, the committee submitted its 65. Which one of the following is INCORRECT in
report. respect of Parliamentary Control over the
Its major recommendation were as follows: Budget?
1. An Inter-State Council should be constituted (a) Parliament has no say in the preparation of the
immediately. budget.
2. Certain subjects of the Union List and the (b) Parliament has the power to increase
Concurrent List should be transferred to the State expenditure charged on the Consolidated Fund.
List.
(c) Parliament has no power to impose a tax
3. The residuary powers should be allocated to the without the President’s recommendation.
states.
(d) Parliament has no power to increase a tax
4. All-India services (IAS, IPS and IFS) should be without the President’s recommendation.
abolished.
63. Which one of the following statements regarding Ans. (a) : It is prepared by the ministry of finance in
the office of the Speaker is correct? consultation with NITI Aayog and other concerned
ministries. The Budget division of the department of
(a) He holds office during the pleasure of the
economic affairs (DEA) in the finance ministry is the
President.
nodal body responsible for producing the Budget.
(b) He need not be a member of the House at the Parliament has no say in the preparation of the budget.
time of his election but has to become a member
of the House within six months from the date of 66. Which one of the following comes under the
his election. jurisdiction of both the High Courts and the
(c) He loses his office if the House is dissolved Supreme Court?
before the end of its normal tenure. (a) Disputes between the Centre and the States.
(d) If he intends to resign, the letter of his (b) Disputes between the States interest.
resignation is to be addressed to the Deputy (c) Protection of the Fundamental Rights.
Speaker. (d) Protection against the violation of the Constitution.
IAS (Pre) GS 1993 Paper I 611 YCT
Ans. (c) : Under Article 32, Supreme court of India has (c) Dr. Zakir Hussain and K.R. Narayanan
jurisdiction over protection of Fundamental Right and (d) B.D. Jatti and K.R. Narayanan
whereas, under article 226 High Court of India has Ans. (b) : From 12th May, 1952 to 19th May, 1957, Dr.
jurisdiction over protection of Fundamental Rights, thus S. Radhakrishnan was Ambassador to the Soviet Union
option C is correct answer. before becoming the vice president from (1949 to
67. Which one of the following is a feature common 1952). It is noteworthy that Dr. Radhakrishnan also
to both the Indian Federation and the American served as the president of India from May 13, 1962 to
Federation? May 13, 1967. Shri V.V. Giri was the vice president of
(a) A single citizenship. India from 6 May, 1967 to 29 August, 1969. V.V. Giri
(b) Three lists in the Constitution. was the ambassador to Sri Lanka before becoming the
(c) Dual judiciary. vice president. It is noteworthy that V.V. Giri became
(d) A federal Supreme Court to interpret the the first non-congress (non-official) president of India
constitution. (1967-74).
Ans. (d) : The common feature of both the Indian
Federation and the American Federation is a federal ECONOMY
Supreme court to interpret the Constitution .India has
adopted the concept of Supreme Court from American 71. The New Population Bomb refers to:
Constitution. (a) An increase in the population of the aged in the
68. The Anti Defection Law was enacted as early as Third World.
1979 in: (b) Rapidly growing urban population in the Third
(a) Kerala (b) Jammu and Kashmir World.
(c) West Bengal (d) Tamil Nadu (c) Large scale distress migration in the Third
Ans. (b) : The 10th Schedule of the Indian Constitution World.
(which talks about the anti-defection law) is designed to (d) Deluge of Soviet emigrants.
prevent political defections prompted by the lure of Ans. (b) : The cause of the population explosion
office or material benefits or other like considerations. includes many factors and reasons. These includes:
The Anti-defection law was passed by Parliament in Increase in the birth rate- Due to lack of control on
1985 under 52nd Constitutional Amendment Act and delivery and unawareness of people the birth rate is
reinforced in 2002.The anti-defection law was enacted increasing rapidly. Furthermore, the birth rate has
to ensure that a party member does not violate the increased many folds in comparison to the death rate. A
mandate of the party and in case he does so, he will lose predicted explosive growth in the human population
his membership of the House. The law applies to both which had been expected to reach 12 billion (since
Parliament and state assemblies. The Anti Defection decreased to 9 billion) by 2050.The rapidly growing
Law was enacted as early as 1979 in Jammu Kashmir. urban population in the Third World is referred to
69. Which one of the following is in the Concurrent population bomb.
List in the Constitution of India?
72. Which one of the following sets of economists
(a) Population control and family planning strongly favoured market economy?
(b) Public health and sanitation (a) Adam Smith, Keynes, Hicks
(c) Capitation taxes (b) Adam Smith, Marx, Strumlin
(d) Treasure trove (c) Adam Smith, Hayek, Friedman
Ans. (a) : Under 7th Schedule of Indian Constitution (d) Adam Smith, Ricardo, J.K. Galbraith
union list, state list and Concurrent list are mentioned. Ans. (d) : Adam Smith, Ricardo and J.K Galbraith
Under concurrent list law can be made by Centre as strongly favoured a market economy. A market
well as State. There are 52 items mentioned under economy is an economy in which supply and demand
Concurrent list these contain education, forest, trade drive economic decisions, such as the production of
unions, marriage, population control and family goods and services, investments, pricing, and
planning. Thus option (a) is correct. distribution. A market economy promotes free
70. Among the four pairs given below which one competition among market participants.
consists of a correct combination of dignitaries 73. Which arrangement of the following show the
who became Vice Presidents after having held correct sequence of demographic transition as
diplomatic posts like Ambassadors and High typically associated with economic development?
Commissioners? I. High birth rate with high death
(a) Dr. S. Radhakrishnan and G.S. Pathak II. Low birth rate with low death
(b) Dr. S. Radhakrishnan and V.V. Giri III. High birth rate with low death
IAS (Pre) GS 1993 Paper I 612 YCT
Choose the answer from the codes given below: Ans. (d) : The main cause of slow rate of growth of per
Codes: capita income are:
(a) I, II, III (b) I, III, II • High Growth Rate of Population
(c) III, I, II (d) II, I, III • Excessive Dependence on Agriculture
Ans. (b) : The Demographic Transition Model (DTM) • Occupational Structure
is based on historical population trends of two
• Low Level of Technology and its Poor Adoption
demographic characteristics – birth rate and death rate –
to suggest that a country's total population growth rate • Poor Industrial Development
cycles through stages as that country develops • Poor Development of infrastructural Facilities
economically. • Poor Rate of Saving and Investment
The correct sequence of demographic transition as • High capital output ratio
typically associated with economic development 1-3-2. Thus option (d), only statement I and II is correct.
74. The growth rate of which one of the following
77. In India the Public Sector is most dominant in :
sectors has very low employment elasticity?
(a) Steel production
(a) Manufacturing (b) Construction
(b) Organised term lending financial institutions
(c) Financial services (d) Mixed farming
(c) Transport
Ans. (a) : The growth rate of low employment elasticity
is mentioned as below: (d) Commercial banking
SECTOR EMPLOYMENT ELASTICITY Ans. (b) : In India the Public Sector is most dominant in
Construction 0.60 organised term lending financial institutions as they
Financial Services 0.53 provides loans to public for fixed amount of time. They
Mixed Farming 0.40 provides loans mainly to agriculture sector and Service
sector. In Steel production more public participation is
Manufacturing 0.25
required.
75. The basic difference between imperative and
indicative planning is that: 78. Which one of the following modes of privatisation
is the most comprehensive and complete?
(a) In the case of the imperative planning the
market mechanism is entirely replaced by a (a) Introduction of private capital in public sector.
command hierarchy, while in the case of (b) Contracting out management of public
indicative planning, it is looked upon as a way enterprises to the private sector.
to improve the functioning of the market system. (c) Transferring ownership and management to the
(b) In the case of indicative planning there is no workers.
need to nationalize any industry. (d) Transferring ownership and management to the
(c) In the case of imperative planning all economic private sector.
activities belong to public sector, while in the
Ans. (d) : Privatisation is complete and comprehensive
other type they belong to the private sector.
when ownership and management are transferred
(d) It is easier to achieve targets in imperative type
completely to private sector. The decision making
of planning.
completely lie in the hand of private sector without
Ans. (a) : The basic difference between imperative and much interference of government. Privatisation comes
indicative planning is that in the case of the imperative
under Capitalism model of economy.
planning the market mechanism is entirely replaced by a
command hierarchy, while in the case of indicative 79. When partial convertibility obtains the exchange
planning, it is looked upon as a way to improve the rate is:
functioning of the market system. (a) A weighted average
(b) A simple average
76. Which of the following are the main causes of
(c) Fully floating
slow rate of growth of per capita income In
(d) Fully administered
India?
I. High capital output ratio Ans. (d) : When partial exchangeability is implemented
II. High rate of growth of population by a country in its currency for mutual transaction with
the currencies of other countries, then the exchange rate
III. High rate of capital formation
of that country is completely administered by the
IV. High level of fiscal deficits
government.
Select the correct answer from the codes given Full convertibility of exchange rate of currency refers
below: to a system under which the currency of a country is
Codes: freely convertible into major foreign currencies and
(a) I, II, III and lV (b) II, III and IV major foreign currencies are freely convertible into
(c) I and IV (d) I and II local currency.
IAS (Pre) GS 1993 Paper I 613 YCT
80. A zero rate of Inflation obtains necessarily in a 84. Most Favoured Nations (MFN) clause under
year where the annual rate of inflation: GATT implies
(a) Most favour to some countries.
(a) in every week of the year is zero. (b) Most favour to all countries.
(b) is falling in every week of the year. (c) No favour to any country.
(c) is both falling and rising in a year. (d) No favour to some countries.
(d) is constant in every week of the year. Ans. (b) : Most Favoured Nation status is given to an
international trade partner to ensure non-discriminatory
Ans. (a) : A zero rate of Inflation obtains necessarily in trade between all partner countries of the WTO. A
a year where the annual rate of inflation in every week country which provides MFN status to another country
of the year is zero. A zero rate of Inflation obtains has to provide concessions, privileges, and immunity in
necessarily in a year where the annual rate of inflation is trade agreements.
both falling and rising in a year. Unexpected inflation 85. Consider the charts given below:
tends to hurt those whose money received in terms of
wages and interest payments does not rise with
inflation.
81. Which one of the following pairs is correctly
matched?
(a) Rationing : Fiscal control
(b) Cash Reserve Ratio : Selective Credit Control
(c) Licensing : Comprehensive Control
(d) Import Quota : Physical control
Ans. (d) : An import quota is a type of trade
restriction that sets a physical limit on the quantity of a
good that can be imported into a country in a given
period of time Quotas, like other trade restrictions, are
typically used to benefit the producers of a good in that
economy.
Thus option (d) is correctly matched.
82. Euro dollars are:
(a) A currency issued by European Monetary Union.
(b) Special currency issued by federal government
The given charts depict the price variation in a
of U.S.A. to be used only in Europe. stock market during a year. Movement of
(c) U.S. dollars circulating In Europe. industry wise average price is plotted for the
(d) European currencies exchanged for the U.S. sectors of iron and steel and petro products. Price
dollar in U.S. movements of four companies A and B (iron and
steel), C and D (petro products) are also plotted.
Ans. (a) : Eurodollar, a United States dollar that has From the curves, a significant feature that can be
been deposited outside the United States, especially deducted is:
in Europe. Foreign banks holding Eurodollars are (a) Company C made large profit around June.
obligated to pay in U.S. dollars when the deposits are (b) All share prices will increase steadily in the next
year.
withdrawn. Dollars form the largest component of all (c) Company B seems to be performing better than
currencies in which such deposits are held and which the market average.
are generally known as Eurocurrency. (d) Petro products are performing better than iron
83. Variable reserve rates and Open Market and steel.
Operations are instruments of: Ans. (c) : From the given charts, the price variation in a
stock market during a year is depicted and Company B
(a) Fiscal Policy (b) Monetary Policy seems to be performing better than the average market.
(c) Budgetary Policy (d) Trade Policy 86. As on today, which one of the following countries
Ans. (b) : Variable reserve rates and Open Market has the largest external debt?
Operations are instruments of Monetary Policy. (a) India (b) Brazil
(c) USA (d) Mexico
Open Market Operations is when the RBI involves itself
Ans. (b) : Brazil, a South American country ranks first
directly and buys or sells short-term securities in the among the countries taking loans from foreign countries
open market. This is a direct and effective way to and other international financial institutions. As per
increase or decrease the supply of money in the market. World Development Report 1992 released by World
It also has a direct effect on the ongoing rate of interest Bank. Brazil’s external debt was highest and it was
in the market. 116,773 million dollar.
IAS (Pre) GS 1993 Paper I 614 YCT
PHYSICS (a) Multiple reflection and interference
(b) Multiple refraction and dispersion
87. Consider the figures of a metal ball and a metal (c) Diffraction and dispersion
ring given below:
(d) Polarization and interference
Ans. (a) : The colours seen in a soap bubble arise
from interference of light reflecting off the front and
back surfaces of the thin soap film. The
brightest colors are those that interfere constructively.
These rays will interfere in a way that depends on the
thickness of the film and the indices of refraction of the
various media.
The metal ball can just pass through the hole of a
metal ring formed out of a strip. When the ball is
heated it gets stuck. But when the metal ring is CHEMISTRY
heated.
(a) The ball can still pass through it because the 91. Which one of the following is used for dating
ring diameter expands on heating. archaeological specimens in wood, bones and
(b) The ball gets stuck because the diameter of the shells?
hole decreases on expansion. (a) Uranium 238 (b) Argon isotope
(c) The ball will still pass through because the hole (c) Carbon 14 (d) Strontium 90
diameter does not change.
(d) The ball will pass through because there is no Ans. (c) : The Carbon isotope also is used as a tracer in
change in the ring. following the course of particular carbon atoms through
Ans. (a) : The metal ball can just pass through the hole chemical or biological transformations. In carbon-14
of a metal ring formed out of a strip. When the ball is dating, measurements of the amount of carbon-14
heated it gets stuck. But when the metal ring is heated present in an archaeological specimen, such as a tree,
the ball can still pass through it because the ring
are used to estimate the specimen's age.
diameter expands on heating. This happens by virtue of
a property of metal as it elongate and expand on heating 92. Acid precipitation is now regarded as a serious
and contract while cooling. problem in some European and Asian countries.
88. The working of the quartz crystal in the watch is Its major cause or source is:
based on the: (a) Discharge of acidic effluents onto neutral or
(a) Photoelectric Effect (b) Johnson Effect slightly alkaline land where the acidic
(c) Piezoelectric Effect (d) Edison Effect components precipitate.
Ans. (c) : Quartz crystal clock based on piezo-electric (b) Emissions of sulphur oxides and nitrogen
effect, it is an oscillating EMF develops in one pair of oxides from thermal power plants and burning
faces of the crystal when a variable pressure is applied of fossil fuels; these oxides dissolve in
on another pair of faces of the crystal perpendicular to atmospheric water vapour and fall back on earth
the first one. as acid rain.
(Source - 12th NCERT) (c) Natural carbon dioxide released during
89. At what temperature are the readings of a respiration of living organisms dissolves in
Centigrade and Fahrenheit thermometer the water, forming carbonic acid which is the chief
same? contributor to acidity in rain water.
(a) -40 (b) 212
(c) 40 (d) 100 (d) Chlorofluorocarbons readily react with various
chemicals near the earth’s surface, producing
Ans. (a) : This is the most common temperature scale in acidic intermediates which cause acid
the world and the simplest to understand. Interestingly, precipitation.
centigrade is a name for Celsius. The freezing point of
Ans. (b) : Acid precipitation is now regarded as a
water for Celsius is 0°C, whereas the boiling point of
serious problem in some European and Asian countries.
water is 100°C.
Its major cause or source is emissions of sulphur oxides
The freezing point of water for Fahrenheit is 32°F,
and nitrogen oxides from thermal power plants and
whereas the boiling point of water is 212°F. The
burning of fossil fuels; these oxides dissolve in
abbreviate of Fahrenheit is °F.
atmospheric water vapour and fall back on earth as acid
The Conversion from Celsius to Fahrenheit and vice
versa can be done as rain.
°C = °F – 32 x (5/9) Acid rain results when sulfur dioxide (SO2) and
°F = °C / (5/9) + 32 nitrogen oxides (NO) are emitted into the atmosphere
At, -40 temperature the readings of a Centigrade and and transported by wind and air currents. The SO2 and
Fahrenheit thermometer will be the same. NO react with water, oxygen and other chemicals to
90. The formation of brilliant colours in soap film is form sulfuric and nitric acids. These then mix with
a consequence of the phenomenon of: water and other materials before falling to the ground.
IAS (Pre) GS 1993 Paper I 615 YCT
93. Which one of the following colloidal systems is BIOLOGY
represented by fog?
(a) Liquid in gas (b) Gas in liquid 98. The theory of ‘jumping genes’ was propounded
(c) Solid in gas (d) Liquid in liquid by:
Ans. (a) : A colloidal system is a type of mixture in (a) Gregor Johann Mendel
which one part is dispersed constantly throughout (b) Thomas Hunt Morgan
another. Colloidal systems are usually formed when one
(c) Barbara McClintock
part is dispersed through another, but does not combine
to form a solution. (d) Watson and Crick
Fog is an example of an aerosol particle thus cloud or Ans. (c) : Transposable elements (TEs), also known as
fog is a colloidal system in which the dispersed phase is "jumping genes," are DNA sequences that move from
liquid and dispersion medium is gas. Therefore the one location on the genome to another. These elements
correct answer to the question cloud or fog is a colloidal were first identified more than 50 years ago by
system in which the dispersed phase and dispersed geneticist Barbara McClintock of Cold Spring Harbor
medium are liquid and gas. Laboratory in New York.
94. The Government of India has totally banned the 99. Camel uses its hump for:
export of ‘Burnt Coal’ (Jhama), because it (a) Storing water
contains recoverable amount of a metal/element
used in transistors. Which one is the element? (b) Storing fat
(a) Phosphorus (b) Germanium (c) Balancing the body during walking in desert sand
(c) Silicon (d) Tungsten (d) Temperature regulation
Ans. (c) : Depending on the coal that was burned, the Ans. (b) : A camel's hump is used for storing fat. The
chemical composition found in coal ash can vary. camel uses it as nourishment when food is scarce. If a
However, most of coal ash contains aluminium oxide camel uses the fat inside the hump, the hump will
(Al2O3), calcium oxide (CaO) and silicon dioxide (SiO2) become limp and droop down.
which is very harmful for humans. 100. Poison glands of snakes are homologous to:
Silicon is a relatively non reactive element but can react
at high temperature (molten state). Silicon acts like a (a) Electric organs of fishes
metal and non metal which enables it to be used in (b) Stings of rays
electronics and can be used in extremely precise (c) Sebaceous glands of mammals
applications like semiconductors. (d) Salivary glands of vertebrates
95. Assertions (A): A mixture of salt and ice gives Ans. (d) : Poison glands of snakes are homologous to
temperature below 0°C. salivary glands of vertebrates.
Reason (R): The salt raises the freezing point of ice.
In the context of the above two statements, which 101. Which one of the following seeds can benefit a
one of the following is correct? patient of diabetes mellitus by normalizing his
(a) Both A and R are true and R is the correct blood sugar level?
explanation of A. (a) Coriander seeds (b) Mustard seeds
(b) Both A and R are true but R is NOT the correct (c) Cumin seeds (d) Fenugreek seeds
explanation of A.
(c) A is true but R is false. Ans. (d) : Diabetes is a disease that occurs when your
(d) A is false but R is true. blood glucose, also called blood sugar, is too high.
Ans. (c) : A mixture of salt and ice gives temperature Blood glucose is your main source of energy and comes
below 0°C and thus assertion A is true, whereas the salt from the food you eat. Insulin, a hormone made by the
mixed with ice decreases freezing point of ice and thus pancreas, helps glucose from food get into your cells to
reason R is false. be used for energy.
96. Vinegar is an aqueous solution of: Fenugreek seeds can benefit a patient of diabetes
(a) oxalic acid (b) citric acid mellitus by normalizing the blood sugar level.
(c) acetic acid (d) hydrochloric acid
102. Hemophilia is a hereditary disease carried by:
Ans. (c) : Vinegar is an aqueous solution of acetic acid
and trace compounds that may include flavorings. (a) Females and expressed by females
Vinegar typically contains 5–8% acetic acid by volume. (b) Females and expressed by males
Usually, the acetic acid is produced by a double (c) Males and expressed by females
fermentation; converting simple sugars to ethanol using (d) Males and expressed by males
yeast and ethanol to acetic acid by acetic acid bacteria.
Ans. (b) : Hemophilia is caused by a mutation or
97. A fertile soil, suitable for growing common crops, change, in one of the genes, that provides instructions
is likely to have a pH value of:
(a) Three (b) Four for making the clotting factor proteins needed to form a
(c) Six to seven (d) Nine to ten blood clot. This change or mutation can prevent the
Ans. (c) : It has been determined that most plant clotting protein from working properly or to be missing
nutrients are optimally available to plants within this 6.5 altogether. These genes are located on the X
to 7.5 pH range, plus this range of pH is generally very chromosome. So, it is expressed by males and carried
compatible to plant root growth. by females.
IAS (Pre) GS 1993 Paper I 616 YCT
103. If a mouse of over eight times its normal size has 107. The blood pressure values of four persons are
been produced by introducing human growth given below:
hormone gene, the technique involved is called: 1. Mrs. X : 90/60 2. Mr. X : 160/120
(a) Hybridisation (b) Genetic engineering 3. Mr. Y : 120/80 4. Mrs. Y : 140/100
(c) Mutation Breeding (d) Hormonal feeding Who among the following has normal blood
Ans. (b) : If a mouse of over eight times its normal size pressure?
has been produced by introducing human growth (a) Mrs. X (b) Mr. X
hormone gene, the technique involved is Genetic (c) Mrs. Y (d) Mr. Y
engineering. Ans. (d) : Blood pressure is the force of your blood
Genetic engineering is the process of using recombinant pushing against the walls of your arteries. Each time
your heart beats, it pumps blood into the arteries. Your
DNA (rDNA) technology to alter the genetic makeup of
blood pressure is highest when your heart beats,
an organism. Genetic engineering involves the direct
pumping the blood. This is called systolic pressure.
manipulation of one or more genes. Most often, a gene
When your heart is at rest, between beats, your blood
from another species is added to an organism's genome pressure falls. This is called diastolic pressure.
to give it a desired phenotype. The normal blood pressure is in range of 120/80.Thus
104. Match List I with List II and select the correct Mr. Y has normal blood pressure.
answer using the codes given below the lists: 108. Sucrose content in sugarcane decreases:
List I List II (a) If high rainfall occurs during the period of
(Characters tics) (Animal) growth of the plant.
I. Wingless insect (A) Kiwi (b) If frost occurs during the period of ripening.
II. Flightless bird (B) Silver fish (c) If there is fluctuation in temperature during the
III. Limbless reptile (C) Turtle period of growth of the plant.
(d) If there is high temperature during the time of
IV. Limbless animal (D) Snake
ripening.
(E) Fish
Ans. (a) : Sugarcane ripening is the process of sucrose
Codes: accumulation in stalks, which is heavily influenced by
(a) IA, IIC, IIIB, IVE (b) IB, IIA, IIID, IVE several factors, mainly by climatic conditions such as
(c) IB, IIA, IIIC, IVD (d) IC, IIA, IIID, IVB air temperature and water deficits.
Ans. (b) : The correct match for list 1 and list 2 is So, Sucrose content in sugarcane decreases If high
Wingless insect – Silver fish rainfall occurs during the period of growth of the plant.
Flightless bird – Kiwi 109. Sugarcane, sugarbeet, sweetpea, chickpea,
Limbless reptile – Snake pigeonpea and French bean belong to:
(a) Two plant families
Limbless animal – Fish
(b) Three plant families
105. Three communicable diseases prevalent in (c) Four plant families
developing countries caused by unsafe drinking (d) Five plant families
water and bad sanitation are: Ans. (b) : Sugarcane, sugarbeet, sweetpea, chickpea,
(a) acutediarrhoea, cancer and gout pigeonpea and French bean belong to three plant family.
(b) Malaria, acute diarrhea and schistosomiasis 110. Biofilms are:
(c) onchocerciasis, leukaemia and arthritis (a) colour films used by biologists to photograph
(d) Rheumatism, malaria and AIDS living organisms.
Ans. (b) : Three communicable diseases prevalent in (b) photomicrographs of microorganisms from
developing Countries caused by unsafe drinking water various habitats.
and bad sanitation are malaria, acute diarrhea and (c) accumulations of microorganisms at surfaces
schistosomiasis. such as those of a rock, a tooth or an oil droplet.
106. Scratching eases itching because: (d) photographic films made through fermentation
(a) It removes the outer dust in the skin. processes of certain industrial microorganisms.
(b) It kills the germs. Ans. (c) : Bio films are accumulations of
(c) It stimulates certain nerves which direct the brain to microorganisms at surfaces such as those of a rock, a
increase the production of antihistaminic chemicals. tooth or an oil droplet.
A bio film forms when certain microorganisms (for
(d) It suppresses the production of enzymes which
example, some types of bacteria) adhere to the surface
cause itching. of some object in a moist environment and begin to
Ans. (c) : Scratching eases itching because it stimulates reproduce. The microorganisms form an attachment to
certain nerves which direct the brain to increase the the surface of the object by secreting a slimy, glue-like
production of antihistaminic chemicals. substance.
IAS (Pre) GS 1993 Paper I 617 YCT
111. Which one of the following approaches comes 114. Which one of the following is a man made”
under the category of biotechnology ? cereal NOT found in nature?
(a) Use of living organisms or substances obtained (a) Dwarf wheat (b) Hybrid maize
from them in industrial processes. (c) Triticale (d) Soya bean
(b) Modernizing the commercial industrial to Ans. (c) : ‘Triticale’ was prepared by mixing wheat and
produce products for use in biological research. rye. Man-made artificial grain. It is not available in
(c) Use of modern technology to investigate nature. ‘Triticale’ is the first artificial grain prepared by
biological disorders scientists.
(d) Use of industrial technology to increase the 115. In many developing countries, three major
biosphere causes of death and disease are:
Ans. (a) : Use of living organisms or substances (a) Lack of safe drinking water, increasing use of
obtained from them in industrial processes. pesticides and thinning of the ozone layer.
(b) Contaminated food, global warming and
ENVIRONMENT AND ECOLOGY
industrial chlorofluorocarbons.
112. Which of the following strongly threatens (c) Polluted air, greenhouse effect and soil
biodiversity? erosion.
(a) Fragile ecosystems such as mangroves and
wetlands. (d) Dirty water, contaminated food and polluted
(b) Inaccessible habitats in Himalayas. air.
(c) Destruction of natural habitats and vegetation
Ans. (d) : In many developing countries of the world
and shifting cultivation.
(d) Creation of biosphere reserves. which are also called ‘third world countries’. The three
Ans. (c) : Biodiversity is the biological variety and main causes of death and disease of people in these
variability of life on Earth. Biodiversity is a measure of countries are dirty drinking water and contaminated
variation at the genetic, species, and ecosystem level. food and polluted air.
Terrestrial biodiversity is usually greater near the
equator, which is the result of the warm climate and CURRENT AFFAIRS
high primary productivity.
Major threats to biodiversity are commonly recognized 116. The book The Man who knew Infinity by
in the programmes of work of the Convention: invasive Robert Kanigel, published in 1991, and
alien species, climate change, nutrient loading and
extensively reviewed in newspapers in India is
pollution, habitat change, and over exploitation.
Destruction of natural habitats and vegetation and the biography of:
shifting cultivation strongly threatened the biodiversity. (a) Vikram Sarabhai
113. Which one of the following does provide the (b) Homi Bhabha
best estimate of world’s biological diversity? (c) Sir C.V. Raman
(a) Of about ten million species probably alive
today, some 20 species are lost everyday, (d) Srinivasa Ramanujan
most of them unknown because no more than Ans. (d) : The life character of Indian scientist and
half a million have yet been actually mathematician ‘Srinivasa Ramanujan’ has been
identified by scientists.
(b) Of about thirty million living species, some mentioned in the book ‘The Man who knew Infinity’
50 are lost everyday, actually identified. written by Robert Kanigel in 1991.
(c) Of about forty five million living species, 117. Which one of the following was part of the
some 100 are lost every day, most of them erstwhile Soviet Union over which Kurdish
unknown because no more than 1.5 million population is spread?
have been actually identified.
(d) Of about seventy five million living species, (a) Armenia (b) Azerbaijan
some 500 are lost every day, most of them (c) Georgia (d) Turkmenistan
unknown because no more than 3 million Ans. (a) : Armenia was part of the erstwhile soviet
have been actually identified.
union over which Kurdish population is spreaded.
Ans. (c) : The term biodiversity (from “biological
diversity”) refers to the variety of life on Earth at all its 118. Which one of the following monuments figured
levels, from genes to ecosystems, and can encompass in the news recently in connection with a fall of
the evolutionary, ecological, and cultural processes that some of its parts and its consequent
sustain life. archaeological conservation?
The best estimate of world’s biological diversity of (a) Jagannath Temple, Puri
about forty five million living species, some 100 are lost (b) Sun Temple, Konark
everyday, most of them unknown because no more than (c) Khandariya Mahadeva Temple, Khajuraho
1.5 million have been actually identified.
(d) Chaunsat Yogini Temple, Bheraghat
IAS (Pre) GS 1993 Paper I 618 YCT
Ans. (a) : Jagannath Temple, Puri was recently in news 122. Which one of the following is correctly
in connection with a fall of some of its parts and its matched?
consequent archaeological conservation. (a) Dr. Kurien : Space Science
119. Given below is a diary of disasters involving (b) Dr. Malcom Adiseshiah : Cooperative Movement
hazardous chemicals. Which one of the (c) Dr. Abdul Kalam : Development Economics
following is correctly matched?
(a) 1959, Minamata/Nigiata, Japan: Leakage at (d) Dr. Pramod Karan Sethi : Jaipur Foot
pesticide plant. Ans. (d) : Dr. Pramod Karan Sethi a famous Indian
(b) 1976, Sevesor Italy: Mercury discharged into doctor who is excelled in making artificial limbs like
water ways. arms and legs for the disabled.
(c) 1984, Bhopal, India: Leakage of dioxin. He hails from Jaipur, Rajasthan and thus the name of
(d) 1987, Kotka, Finland: Mono chlorobenzene artificial limbs is Jaipuri feet.
split in harbor. The lightweight artificial Jaipuri feets and hands have
Ans. (d) : Out of the four catastrophic disasters made it possible for all the disabled to walk and perform
presented as a result of the emission of hazardous other activities.
chemicals, the disaster given in option (d) is matched
with the related hazardous chemical. In 1987, a major MISCELLANEOUS
casualty was caused by an overflow of a chemical
called monochloro benzene in the port of Kotka, 123. When the Couple Protection Rate is going up:
Finland.
(a) Birth rate must necessarily fall.
120. Match List I with List II and select the correct (b) Birth rate will fall only if couples are of younger
answer by using the codes given below the lists:
List I List II age.
I. Gro Harlem (A) Prime Minister of (c) Death rate must necessarily fall.
Brundtland Slovakia (d) Infant mortality rate must be falling.
II. Stephen Hawking (B) Prime Minister of Ans. (b) : When the Couple Protection Rate is going up
Russia then birth rate will fall only if couples are of younger
III. Vladimir Meciar (C) Environmentalist age.
and Prime Minister of Norway
IV. Viktor Chernomyrdin (D) Cosmologist 124. The Employment Guarantee Scheme, a rural
Codes: work programme, was first started in:
(a) ID, IIC, IIIB, IVA (a) West Bengal (b) Punjab
(b) IC, IID, IIIB, IVA (c) Kerala (d) Maharashtra
(c) ID, IIC, IIIA, IVB Ans. (d) : The Employment Guarantee Scheme in
(d) IC, IID, IIIA, IVB Maharashtra, which came into existence in 1972-73, is
Ans. (d) : The correct match between List-I and List-II similar to the Rural Work Programme, the subsequent
is : Pilot Intensive Rural Employment Programme and the
List-I List-II Crash Scheme for Rural Employment .Later it was
Gro Harlem Brundtland – Environment a list
merged into the Mahatma Gandhi National Rural
and Prime Minister
of Norway Employment Guarantee Act (MGNREGA), 2005,
Stephen Hawking – Cosmologist which provides at least one hundred days of guaranteed
wage employment in every financial year to every
Vladimir Meciar – Prime Minister of
household whose adult members volunteer to do
Slovakia
unskilled manual work.
Viktor Chernomyrdin – Prime Minister of
Russia 125. Which one of the following types of borrowings
from the IMF has the softest servicing conditions?
121. Which one of the following is the underground
army of the African National Congress? (a) Second tranche loan (b) SAF
(a) Umkhonto (b) Kwazulu (c) ESAF (d) Oil Facility
(c) Xhosa (d) Ulundi Ans. (c) : The Enhanced Structural Adjustment
Ans. (a) : The African National Congress (ANC) is the Facility (ESAF) was a program of financial assistance
major political party in South Africa. This party’s is aim given to poor countries from December 1987 through
was to give social and political rights to the blacks and 1999 through the International Monetary Fund. It
also opposes atrocities done against black. replaced the Structural Adjustment Facility (SAF) and
‘UMKHONTO’ we Sizwe we is the secret army of the was itself replaced by the Poverty Reduction and
African National Congress. It means ‘The spear of the Growth Facility (PRGF). ESAF has the softest servicing
nation’. conditions among the given options.

IAS (Pre) GS 1993 Paper I 619 YCT


126. Which one of the following approaches comes Ans. (d) : East Timor is the eastern part of the island of
under the category of biotechnology? Timor, located in the Malaya peninsula of Indonesia.
(a) Use of living organisms or substances obtained Initially East Timor was under control of the
from them in industrial processes. Portuguese. But in 1976 after the civil war, Indonesia
(b) Modernizing the commercial industries to took control of it and it became an apple of discord
produce products for use in biological research. between Indonesia and Australia. Now East Timor is an
(c) Use of modern technology to investigate
independent country.
biological disorders.
(d) Use of industrial technology to increase the 130. Match List I with List II and select the correct
biosphere. answer using the codes given below the lists:
Ans. (a) : Biotechnology, the use of biology to solve List I List II
problems and make useful products. The most I. Grand Slam A. Chess
prominent area of biotechnology is the production of II. Grand Prix B. Polo
therapeutic proteins and other drugs through genetic III. Grand Master C. Bridge
engineering .Use of living organisms or substances D. International race
obtained from them in industrial processes comes under Codes:
the category of biotechnology. (a) IA, IIB, IIIC (b) IB, IID, IIIC
127. Assertion (A) : Hong Kong is to revert to China (c) IC, IID, IIIA (d) ID, IIB, IIIA
from British Control in a few years.
Ans. (c) : The correct match between List-I and List-II
Reason (R) : The people of Hong Kong have
opted for it in a referendum. is :
In the context of the above two statements, List-I List-II
which one of the following is true? Grand slam – Bridge
(a) Both A and R are true and R is the correct Grand Prix – International Race
explanation of A Grand Master – Chess
(b) Both A and R are true but R is NOT the
131. The founder of Boys Scouts and Girl Guides
correct explanation of A
Movement in India was:
(c) A is true but R is false
(a) Charles Andrews
(d) A is false but R is true
(b) Robert Montgomery
Ans. (c) : Hong Kong, a group of more than 200 small
islands located in the Southeast of China. It was under (c) Richard Temple
British control since 1843 AD. But as per agreement (d) Baden Powel
between British and China in 1984, Ans. (d) : Baden Powell is the father of the Boy Scout
On July 1, 1997, Britain handed over Hong Kong to the (Walcher) and civil guide movement in India. Scout
Suzerainty of China. Thus transfer of Hong Kong to Guide Andolan is an international social and self-help
China was not because of will expressed in the organization being run by youth and students.
referendum by the resident there but because of
agreement. So Assertion (A) is true and Reason (R) is 132. Four persons Alok, Bhupesh, Chandu and
false. Dinesh have a total of Rs. 100/among
themselves. Alok and Bhupesh between them
128. Which one of the following is correctly matched? have as much money as Chandu and Dinesh
Countries Purpose for which the U.N. Peace between them but Alok has more money than
Keeping Force is maintained: Bhupesh, and Chandu has only half the money
(a) Lebanon for supervising the 1992 accord. that Dinesh has Alok has in fact Rs. 5/more
(b) El Salvador to deliver humanitarian aid. than Dinesh. Who has the most money?
(c) Mozambique to supervise a referendum. (a) Alok (b) Bhupesh
(d) Cyprus to maintain peace between the two (c) Chandu (d) Dinesh
dominant ethnic groups in the country. Ans. (a) : Let Alok, Bhupesh, Chandu and Dinesh have
Ans. (d) : Peace keeping forces deployed by the United A, B, C, D money respectively.
Nations have been working since 1964 for the purpose
A + B + C + D = 100 ……..(i)
of maintaining peace between the two major ethnic
groups of Cyprus, Greek and Turkish speaking Muslim. A+B=C+D …….(ii)
129. The dispute regarding East Timor is between: But A > B
(a) Japan and Russia D
C=
(b) China and Philippines 2
(c) Indonesia and Malaysia A=D+5
(d) Australia and Indonesia From eqn. (i) and (ii)
IAS (Pre) GS 1993 Paper I 620 YCT
2C + 2D = 100 134.Consider the Vena diagram given below:
News Paper I News Paper II
C + D = 50 …….(iii)
D 3 4
And also C = 12
2
2
D 8 5
+ D = 50
2
6
3D = 100
News Paper III
100
D= The numbers In the Venn diagram are persons
3 reading the newspapers indicated. The diagram is
100 100 50 drawn after surveying 50 persons. In a population
C= = = of 10,000, how many can be expected to read at least
3× 2 6 3
two newspapers?
A=D+5 Choose the correct answer from the codes given
100 115 below:
= +5 =
3 3 (a) 5000 (b) 6250
Now (c) 6000 (d) 5400
B = 100 – A – C – D Ans. (d) :
115 50 100
= 100 − − −
3 3 3
35
B=
3
It is very clear that Alok has the most money.
50 persons reading atleast two newspapers.
133. Given below is the performance of two learners
= 12 + 2 + 8 + 5 = 27
A and B:
27
1 person =
50
27
So for 10,000 person = ×10,000 = 5400
50
135. In an examination every candidates took either
physics or Mathematics or both. 65.8% of the
candidates took physics and those who took
Mathematics were 59.2% . The total candidates
were 2000. How many candidates took both
mathematics and physics?
(a) 750 (b) 700
(c) 550 (d) 500
Ans. (d) : Total numbers of candidates are = 2000
% of candidates who take physics = 68. 5%
Which one of the following can be deduced from
68.5
the graph? No. of candidate = × 2000
100
(a) The learning task was difficult Who took physics = 1316
(b) B learned faster than A % of person who took mathematics out of 2000 are
(c) B learned faster than A throughout the task 59.2
= × 2000
(d) A learned faster than B throughout the task 100
Total number of candidate, who took physics and
Ans. (c) : It is very clear from the graph that B commits mathematics are = 1316 + 1184 = 2500
less error as compared to A throughout the task. So B ∴ The number of candidates who took both
learned faster than A throughout the task. mathematics and physics = 2500 – 2000 = 500

IAS (Pre) GS 1993 Paper I 621 YCT


136. A point to selected at random inside a rectangle 138. Four equal circles are described at the four
and perpendiculars are drawn on each side corners of a square so that each touches two of
from the point. The sum of these perpendicular the others as shown in the figure:
is 24 cm. If the length of rectangle is 3 times the
width, then what will be the perimeter of the
rectangle?
(a) 36 cm (b) 42 cm
(c) 24 cm (d) 48 cm
Ans. (d) :
The area enclosed by the circumferences of the
circles (shaded area in the figure) is 13(5/7) sq.
cm. Find the radius of the circles. (Assume = p =
22/7)
(a) 4 cm (b) 2.5 cm
(c) 7.5 cm (d) 6 cm
Rectangle ABCD has width = x Ans. (a) :
Length = 3x
But, also
From point O, perpendicular are drawn on each side and
there sum in 24 cm.
Now, Radius of circle is ‘r’ cm
5
x x 3x 3x Area of shaded region = 13 cm 2
+ + + = 24 7
2 2 2 2 As per given figure
x + 3x = 24 Area of shaded region = Area of square 4 × quarter of circle
x = 6 cm area
Length = 6 × 3 = 18 cm πr 2
= (2r) 2 − 4 × = 4r 2 − πr 2
Width = 6 4
Perimeter of Rectangle = 2 (l + b) 96  22 
= r 2 4 − 
= 2 [18 + 6] 7  7
= 2 [24] 2 96 7
r = ×
Perimeter = 48cm 7 6
r 2 = 16
137. If the product of two numbers is 320 and their r=4
ratio is 1 : 5, what is the difference between the
squares of these two numbers ? 139. A rectangular sheet of aluminum foil measuring 5
cm by 10 cm is rolled into a cylinder of height 10
(a) 1024 (b) 1365 cm. If the cost of painting the cylinder is Rs. 2/per
(c) 1536 (d) 1635 sq cm, then the cost of painting the surface of the
Ans. (c) : Let the two number be x and y cylinder will be:
(a) Rs. 400 (b) Rs. 200
x × y = 320 ………(i) (c) Rs. 150 (d) Rs. 100
and also
Ans. (d) :
x 1
=
y 5
5x = y ………(ii)
Putting equn. (ii) to equn. (i) Aluminium sheet is rolled into cylinder of height 10 cm.
So
x × 5x = 320
5x2 = 320
x2 = 64
y=8×5
The difference between square of these number is
y2 – x2
= (40)2 – (8)2 Surface Area of cylinder = 5 × 10 cm2 = 50 cm2
= 1600 – 64 Cost of painting is 2 Rs./cm2, so 50 cm2 × 2 rs/cm2
= 1536 = Rs. 100
IAS (Pre) GS 1993 Paper I 622 YCT
140. In a single throw of a pair of dice, what is the 143. Consider the following pie charts?
probability of getting a sum of 8?
(a) 8/12 (b) 1/36
(c) 4/36 (d) 5/36
Ans. (d) : In a single throw of a pair of dice, the
probability of getting a sum of 8 is –
Total number of outcome = 6 × 6 = 36
Total possible outcome = (2, 6), (4, 4), (3, 5), (5, 3), (6, 2)
=5
So the probability of getting a sum of 8 is The pie charts given here depict the area under
5 cultivation of different cash crops, for two years,
=
36 1985 and 1990. Which one of the following
statements is FALSE?
141. A lottery system has ticket numbers with eight
digits. There are ten first prizes of Rs. 1.00 lakh (a) In 1985, the area under jute cultivation was
each, one hundred second prizes of Rs. 10,000 less than that under rice cultivation.
each and one thousand third prizes of Rs. 1,000 (b) More land was used for growing vegetables in
each. 1990 than in 1985.
At the draw of lots, an eight digit number is (c) In 1990, the area under rice and wheat
generated by choosing for each digit from left to cultivation taken together was less than half
right a random number in the range of 0 to 9. the total cultivation area.
Any one who has a ticket with the first two (d) The percentage area under Jute cultivation
digits (from left to right) matching, the chosen was less in 1985 than in 1990.
number gets a first prize, and so on. Ans. (d) : From Pie graph its clear that, option (d) is
One individual purchases a large number of incorrect as the percentage area under jute cultivation
tickets at a discounted price of Re. 0.75 each. was more in 1985 as compared to 1990.
For how much money should he purchase
144. The following graph shows the annual sales of a
tickets to achieve a 10% chance of winning a
company’s products in millions of rupees.
third prize?
Study the trend and find out which of the
(a) Rs. 10,000 (b) Rs. 75,000 following is/are correct?
(c) Rs. 1,00,000 (d) Rs. 7,500
Ans. (d) : To win a lottery of third prize of 1000
Rs. (in ten l akhs)

Rupees with a change of 10% at a discounted price of


rupees 0.75 each. He should purchase a lottery of atleast
0.75 × 1000 × 100 = 7500 Rupees
142. The average salary of skilled employees in a Years
firm is Rs. 520 and that of the unskilled
employees is Rs. 420. If the average salary of 1983 84 85 86 87 88
both the groups of employees is Rs. 500, then I. A deliberate attempt has been made to
the percentage of skilled employees in the firm distort the figures in one year.
is: II. Some one is interested in misleading the
(a) 50 (b) 20 management.
III. The kink in the graph is unexplained.
(c) 80 (d) 70
Choose the correct answer from the codes given
Ans. (c) : Let the skilled employee number is = x below:
Number of unskilled employee = y (a) I only (b) II only
Now, (c) III only (d) I and III
520 × x + 420 xy = 500 x + 500 y Ans. (c) : The kink in the graph is unexplained and
20x = 80y exact explanation is not given to study deflection in the
x = 4y ……….(i) graph.
Now the percentage of skilled employee is 145. The following table shows the percentage
x 4y 4y change in the consumption of electricity by
= ×100 = × = 100 ×100
x+y 4y + y 5y five towns P, Q, R, S and T from 1989 to 1991.
Per cent change from 1989 to 1990 From 1990
= 80%
to 1991:
IAS (Pre) GS 1993 Paper I 623 YCT
Percentage change 148. In a group of six women there are four dancers,
Town From 1989 to From 1990 to four vocal musicians, one actress and three
1990 1991 violinists. Girija and Vanaja are among the
P +8 18 violinists while Jalaja and Shailaja do not know
Q 15 +11
R +6 +9 how to play on the violin. Shailaja and Tanuja
S -7 -5 are among the dancers.
T +13 6 Jalaja, Vanaja, Shailaja and Tanuja are all
What was the percentage by which the total vocal musicians and two of them are also
consumption of electricity by the five cities violinists. If Pooja is an actress who among the
changed between 1989 and 1991? following is both a dancer and a violinists?
(a) 4 (a) Jalaja (b) Shailaja
(b) 4.45
(c) Tanuja (d) Pooja
(c) 4
(d) Data given are insufficient to answer Ans. (c) : According to question in a group of six
women there are four dancers, four vocal musicians one
Ans. (d) : The total consumption of electricity by the
five cities changed between 1989 and 1991 is not clear actress and three violinists.
from the given table as data given are insufficient to But from statement it is clear that Girija and Vanaja are
answer. among the violinists and Jalaja and Shailaja do not
146. The monthly incomes of Peter and Paul are in known how to play violin.
the ratio of 4:3. Their expenses are in the ratio Shailaja and Tanuja are dancer, so its very clear that
of 3 : 2. If each saves Rs. 600 at the end of the Tanuja is the person who is both a dancer and a violinst.
month, their monthly incomes respectively are
149. If you are the ninth person in the queue
(in rupees)
starting from either end, than the number of
(a) 3200 and 2400 (b) 2800 and 2100
persons in the queue is:
(c) 2400 and 1800 (d) 2000 and 1500
(a) 10 (b) 15
Ans. (c) : Let Peter and Paul income be 4x and 3x and
they both save 600 rupees at the end of the month and (c) 17 (d) 19
their expenses are in ratio 3 : 2. Ans. (c) : If the person is sitting at the 9th place from
4 x − 600 3 both the end so it is at middle position.
⇒ =
3x − 600 2 So, the number of person in queue is–
⇒ 8x – 1200 = 9x – 1800 8 + 1 + 8 = 17
x = 600 150. A father is now three times as old as his son.
Peter income is Rs. 2400 and Paul income is Rs. 1800. Five years back, he was four times as old as
147. If ABRA CADABRA is 1, 2, 9, 1 3, 1, 4, 1, 2, 9, his son. The age of the son is:
1, HOCUS POCUS will be: (a) 12 years (b) 15 years
(a) 5, 6, 3, 7, 8, 16, 15, 5, 21, 19 (c) 18 years (d) 20 years
(b) 8, 16, 5, 21, 19, 16, 15, 5, 21, 19 Ans. (b) : Let the present age of father is x years.
(c) 8, 16, 5, 21, 19, 15, 16, 5, 21, 19
x
(d) 5, 6, 3, 7, 8, 6, 5, 3, 8, 7 and son’s age is years
3
Ans. (a) : According to question
According to question
A B R A  → C A D A B R A
x 
1 2 9 1 
→ 3 1 4 1 2 9 1 ( x − 5) = 4  − 5 
 3 
[R is 9th from the last]
Similary x − 5 x − 15
=
H O C U S 
→ P O C U S 4 3
3x – 15 = 4x – 60
Option (b) and (c) are wrong as given number are
wrong. x = 45 years
where as x
U S  → 7,8 Present age of son is =
3
But in option (d) it is mentioned as 7, 8 and 8, 7 so (d)
45
is also wrong. = = 15 years
So, only option (a) is correct. 3

IAS (Pre) GS 1993 Paper I 624 YCT


UNION PUBLIC SERVICE COMMISSION
Civil Services (Preliminary Exam) - 2011
CSAT : PAPER-II
(Chapterwise Analysis with Explanation)
Time : 2 hours Maximum Number : 200
3. The government should engage in maximum
COMPREHENSION interference in market processes.
4. There is a need to change the size of the
Directions for the following 8 (eight) items : government.
Read each of the following two passages and Which of the statements given above are correct?
answer the items that follow. Your answer to these (a) 1 and 2 only (b) 2 and 3 only
items should be based on the passage only. (c) 1 and 4 only (d) 1, 2, 3 and 4
Passage-1 Ans. (a) According to the passage, the aim must be to
For achieving inclusive growth there is a critical stay with the objective of inclusive growth, which was
need to rethink the role of the State. The early debate laid down by the founding fathers of Nation. Thus,
statement-1 is correct which says "The objective of
among economists about the size of the Government can
inclusive growth was laid down by the founding father
be misleading. The need of the hour is to have an of Nation". "Need of the hour is to have enabling
enabling Government. India is too large and complex a government", is clearly mentioned in the passage.
nation for the State to be able to deliver all that is Thus, statement-2 is correct. Statement 3 and
needed. Asking the Government to produce all the statement-4 is not correct as per the passage. As in
essential goods, create all the necessary jobs, and keep a passage, we need a government that, when it comes to
the market set, effective, incentive-compatible rules
crup on the prices of all goods is to lead to a large and remains on the sidelines with minimal
cumbersome bureaucracy and widespread corruption. interference.
The aim must be to stay with the objective of
2. According to the passage, the strategy of inclusive
inclusive growth that was laid down by the founding growth can be effected by focusing on
fathers of the nation and also to take a more modern view (a) Meeting all the needs of every citizen in the
of what the State can realistically deliver. country
This is what leads to the idea of an enabling State, (b) Increasing the regulations over the
that is a Government that does not try directly deliver to manufacturing sector
the citizens everything that they need. Instead, it (1) (c) Controlling the distribution of manufactured
creates an enabling ethos for the market so that goods
(d) Delivery of the basic services to the deprived
individual enterprise can flourish and citizens can, for the
sections of the society.
most part, provide for the needs of one another, and (2)
Ans. (d) According to the passage, the strategy of
steps in to help those who do not manage to do well for
inclusive growth can be effected by focusing on
themselves, for there will always be individuals, no
delivery of the basic services to the deprived section of
matter what the system, who need support and help. society. Government steps into help those who do not
Hence we need a Government that, when it comes to the manage to do well for themselves, for there will
market, sets effective, incentive-compatible rules and always be individual, no matter what the system, who
remains on the sidelines with minimal interference, and need support and help.
at the same time, plays and important role in directly 3. What constitutes an enabling Government ?
helping the poor by ensuring that they get basic 1. A large bureaucracy
education and health services and receive adequate 2 Implementation of welfare programmes through
nutrition and food. representatives
1. According to the passage- 3. Creating an ethos that shelps individual enterprise
1. The objective of inclusive growth was laid down 4. Providing resources to those who are
by the founding fathers of the nation underprivileged
2. Need of the hour is to have a enabling 5. Offering direct help to the poor regarding basic
government. services
IAS (Pre) GS IInd Paper (CSAT), 2011 1 YCT
Select the correct answer from the codes given Passage-2
below : The concept of ‘’creative society‘’ refers to a phase
(a) 1, 2 and 3 only of development of a society in which a large number of
(b) 4 and 5 only potential contradictions become articulate and active.
(c) 3, 4 and 5 only This is most evident when oppressed social groups get
(d) 1, 2, 3, 4 and 5 politically mobilize and demand their rights. The upsurge
Ans. (c) The constitutes of enabling government are of the peasants and tribals, the movements for regional
ethos enterprise. Government create an enabling ethos autonomy and self-determination, the environmental
for the market so that individual interprise can flourish movements, and the women’s movements in the
and citizens can, for the most part provide for the need developing countries are signs of emergence of creative
of one another and to provide resources to who are society in contemporary times. The forms of social
under privileged and offering direct help to poor movements and their intensity may vary from country to
regarding basic services like health and education. country and place to place within a country. But the very
Thus statement 3, 4, 5 are correct. As India is too large presence of movements for social transformation in
and complex, a large bureaucracy and implementation various spheres of a society indicates the emergence of a
of welfare program through representative will lead to creative society in a country.
huge corruption and thus statement 1, 2 are wrong. 6. What does the author imply by ‘’creative
4. Why is the state unable to deliver "all that is society’’?
needed’’? 1. A society where diverse art forms and literary
writings seek incentive.
1. It does not have sufficient bureaucracy.
2. A society where social inequalities are accepted
2. It does not promote inclusive growth.
as the norm
Select the correct answer from the codes given
3. A society where a large number of contradictions
below
are recognized.
(1) 1 only
4. A society where the exploited and the oppressed
(b) 2 only groups grow conscious of their human rights and
(c) Both 1 and 2 upliftment
(d) Neither 1 nor 2 Select the correct answer using the codes given
Ans. (d) India is too large and complex a nation for the below :
(a) 1, 2 and 3 (b) 4 only
state to be able to delivers all that is needed and thus it
(c) 3 and 4 (d) 2 and 4
is not an easy to delivers what is needed and thus
statement 1, 2 are not the reason for state unable to Ans. (c) According to the passage, the concept of
'creative society' refers to a phase of development of a
delivers all that is needed.
society in which large number of potential
5. What is the essential massage being conveyed by contradiction became articulate and active. This is
the author of the passage? most evident when oppressed social group get
(a) The objectives of inclusive growth laid down by politically mobilize and demand their rights. So,
the founding fathers of the nation should be statement 3 and 4 clearly explains creative society.
remembered 7. What according to the passage are the
(b) The government needs to make available more manifestations of social movements ?
schools and health services 1. Aggressiveness and being incendiary
(c) The government needs to establish markets and 2. Instigation by external forces
3. Quest for social equality and individual freedom
industries to meet the needs of the poor strata of 4. Urge for granting privileges and self-respect to
the society dispsaraged sections of the society
(d) There is a need to rethink the role of the state in Select the correct answer using the codes given
achieving inclusive growth below :
(a) 1 and 3 only (b) 2 and 4 only
Ans. (d) The essential message being conveyed by the (c) 3 and 4 only (d) 1, 2, 3 and 4
author of the passage is, there is need to rethink the role Ans. (c) According to the passage, the manifestations
of the state in achieving inclusive growth. of social movements are quest for social equality and
IAS (Pre) GS IInd Paper (CSAT), 2011 2 YCT
individual freedom and urge for granting privilege and Expanding human population has an adverse
self respect to disparaged sections of the society. The effect on :
upsurge of the peasants and tribals, the movements for 1. Spiritual fulfillment
regional autonomy and self determination, the 2. Aesthetic enjoyment
environmental movements and the women's movement
3. Potable fresh water
in the developing countries are sign of emergence of
creative society. 4. Production of food and fiber
5. Biodiversity
8. With reference to the passage, consider the Which of the statements given above are correct?
following statements. (a) 1, 2 and 3 only (b) 2, 4 and 5 only
1. To be a creative society, it is essential to have a
(c) 3 and 5 only (d) 1, 2, 3, 4 and 5 only
variety of social movements
2. To be a creative society, it is imperative to have Ans. (c) The rapidly expanding human population has
potential contradictions and conflicts greatly modified the earth's ecosystem, As effort to
Which of the statements given above is/are increase production of food and fiber have decreased
correct? the availability of some ecosystem to provide clean
(a) 1 only water, regulate flooding and support Biodiversity.
(b) 2 only Thus, statement 3 and 5 is correct.
(c) Both 1 and 2
(d) Neither 1 nor 2 10. The passage mentions that ‘‘Some people have
Ans. (c) The upsurge of the peasants and tribals, the actually been harmed by these changes’’. What
movements for regional autonomy and self does it imply?
determination, the environmental movements and the 1. The rapid expansion of population has adversely
women movements in the developing countries are affected some people.
signs of emergence of creative society in contemporary 2. Sufficient efforts have not been made to increase
times. The concept of "creative society" refers to a the production of food and fibre.
phase of development of a society in which a large
3. In the short term some people may be harmed,
number of potential contradiction became articulate
and active. Thus both the statements are correct. but in the long term everyone will benefit from
modifications in the Earth’s ecosystems.
Directions for the following 6 (six) items : Which of the statements given above is/are
Read each of the following two passages and
correct?
answer the items that follow. Your answers to these
items should be based on the passages only. (a) 1 only
(b) 2 only
Passage-1
(c) 1 and 3
Ecosystems provide people with a variety of goods (d) None of the statements given above
and services; food, clean water, clean air, flood control,
soil stabilization, pollination, climate regulation, spiritual Ans. (a) Passage mentions that some people have
fulfillment and aesthetic enjoyment, to name just a few. actually been harmed by these change which means,
Most of these benefits either are irreplaceable or the the rapid expanding human population has greatly
technology necessary to replace them is prohibitively modified the earth ecosystem to meet their increased
expensive. For example, potable fresh water can be requirement of some goods and services, particularly
provided by desalinating sea-water, but only at great food, fresh water, timber and fuel. Thus rapid
cost. expansion of population has adversely affected some
The rapidly expanding human population has greatly
people.
modified the Earth’s ecosystems to meet their increased
requirements of some of the goods and services, 11. With reference to the passage, consider the
particularly food, fresh water, timber, fiber and fuel. following statements:
These modifications have contributed substantially to 1. It is imperative to modify the Earth’s ecosystems
human well being and economic development. The for the well being of mankind.
benefits have not been equally distributed. Some people 2. Technology can never replace all the goods and
have actually been harmed by these changes. Moreover, services provided by ecosystems.
short–term increases in some ecosystem goods and Which of the statements given above is/are correct?
services have come at the cost of the long-term (a) 1 only (b) 2 only
degradation of others. For example, efforts to increase (c) Both 1 and 2 (d) Neither 1 nor 2
the production of food and fiber have decreased the
ability of some ecosystems to provide clean water, Ans. (d) There is nowhere mentioned in passage that,
regulate flooding and support biodiversity. to modify the earth ecosystems for the well being of
mankind is really imperative and also technology can
9. With reference to the passage, consider the
never replace all the goods and services provided by
following statements.
ecosystem and thus both the statements are wrong.
IAS (Pre) GS IInd Paper (CSAT), 2011 3 YCT
Passage-2 14. The passage contains a statement ‘’lets himself be
A moral act must be our own act; must spring from swept along like a log of wood by a current’’
our own will. If we act mechanically, there is no moral Among the following statements, which is/are
content in our act. Such action would be moral, if we nearest in meaning to this ?
think it proper to act like a machine and do so. For, in
doing so, we use our discrimination, We should bear in 1. A person does not use his owns reason.
mind the distinction between acting mechanically and 2. He is susceptible to influence/pressure.
acting intentionally. It may be moral of a king to pardon 3. He cannot withstand difficulties/challenges.
a culprit. But the messenger carrying out the order of 4. He is like a log of wood
pardon plays only a mechanical part in the king’s moral
act. But if the messenger were to carry out the king’s Select the correct answer using the codes given
order considering it to be his duty, his action would be a below :
moral one. How can a man understand morality who (a) 1 only (b) 1 and 2
does not use his own intelligence and power of thought, (c) 2 and 3 (d) 1 and 4
but lets himself be swept along like log of wood by a
current ? Sometimes a man defies convention and acts on Ans. (b) According to the passage, 'let himself be
his own with a view to absolute good. swept along like a log of wood by a current' means a
12. Which of the following statements best describe/ person who does not use his own reason he is
describes the thought of the writer? susceptible to influence/pressure and sometime man
1. A moral act calls for using our discretion. defies convention and acts on his own with view to
2. Man should react to a situation immediately.
3. Man must do his duty. absolute good.
4. Man should be able to defies convention in order Directions for the following 4 (four) items :
to be moral.
Read the following passage and answer the items
Select the correct answer from the codes given
that follow. Your answers to these items should be
below : based son the passage only.
(a) 1 only (b) 1 and 3 A country under foreign domination seeks escape
(c) 2 and 3 (d) 1 and 4 from the present in dreams of a vanished age, and finds
Ans. (d)
consolation in visions of past greatness. That is a foolish
A moral act must be our own act and must spring from and dangerous pastime in which many of us indulge. An
our own will. Thus, statement 1 is correct and
equally questionable practice for us in India is to imagine
according to the statement 4 man should be able to
that we are still spiritually great though we have come
defies convention in order to be moral is true. Because
down in the world in other respects. Spiritual or any
man does not understand morality who does not use
other greatness cannot be founded on lack of freedom
his own intelligence and power of thought but left
himself be swept along like log of wood by a current. and opportunity, or on starvation and misery. Manys
western writers have encouraged that notion that Indians
13. Which of the following statements is the nearest
are other-worldly. I suppose the poor and unfortunate in
definition of moral action, according to the
every country become to some extent other-worldly,
writer?
unless they become revolutionaries, for this world is
(a) It is a mechanical action based on official
evidently not meant for them. So also subject peoples.
orders from superiors
(b) It is an action based on our sense of discretion. As a man grows to maturity he is not entirely
(c) It is a clever action based on the clarity of engrossed in, or satisfied with the external objective
purpose. world. He seeks also some inner meaning, some
(d) It is a religious action based on understanding. psychological and physical satisfactions. So also with
Ans. (b) According to the writer the nearest definition peoples and civilizations as they mature and grow adult.
of moral action is based on our sense of discretion. Every civilizations and every people exhibit these
Such action will be moral if we think it proper to act parallel streams of an external life and an internal life.
like a machine and do so. For, in doing so, we use our Where they meet or keep close to each other, there is an
discrimination. We should bear in mind the distinction equilibrium and stability. When they diverge conflict
between acting mechanically and acting intentionally. arises and the crises that torture the mind and spirit.

IAS (Pre) GS IInd Paper (CSAT), 2011 4 YCT


15. The passage mentions that ‘‘this world is (a) by the impact of foreign domination.
evidently not meant for them’’. It refers to people (b) by the desire to escape from foreign domination
who : and find consolation in visions of past
1. Seek freedom from foreign domination. greatness.
2. Live in starvation and misery. (c) due to lack of equilibrium between an external
3. Become revolutionaries. life and an internal life
Which of the statements given above is/are correct? (d) due to one’s inability to be either revolutionary
(a) 1 and 2 (b) 2 only or other-worldly
(c) 2 and 3 (d) 3 only Ans. (c) Every civilization and every people exit these
Ans. (b) According to the passage, this world is parallel streams of an external life and internal life.
evidently not mean for them who lives in starvation Where they meet or keep close to each other, there is
and misery as spiritual or any other greatness cannot an equilibrium and safety and when they diverge
be founded on lack of freedom and opportunity or on conflict arises and the crisis that torture the mind and
starvation and misery. sprit. Thus, statement C is correct.
Directions for the following 4 (four) items :
16. Consider the following assumptions :
Read the following passage and answer the items
1. A country under foreign domination cannot that follow. Your answers to these items should be
indulge in spiritual pursuit. based on the passage only.
2. Poverty is an impediment in the spiritual pursuit.
Passage
3. Subject peoples may become other-worldly.
A species that exerts an influence out of proportion
With reference to the passage, which of the above
to its abundance in an ecosystems is called a keystone
assumptions is/are valid? species. The keystone species may influence both the
(a) 1 and 2 (b) 2 only species richness of communities and the flow of energy
(c) 2 and 3 (d) 3 only and materials through ecosystems. The Sea star Pisaster
Ans. (c) According to the passage, a country under ochraceus, which lives in rocky intertidal ecosystems on
foreign domination seeks escape from the present in the Pacific coast of North America, is also an example of
dreams of vanished age and finds consolation in visionsa keystone species. Its preferred prey is the mussel
of past greatness. Thus, statement-1 is wrong. There isMytilus californianus. In the absence of sea stars, these
also mentioned in paragraph that spiritual or any othermussels crowd out other competitors in a broad belt of
greatness cannot be found on lack of freedom or the intertidal zone. By consuming mussels, sea star
creates bare spaces that are taken over by a variety of
opportunity or on starvation and misery, Thus, statement
2 is true. According to some western writers have other species.
encouraged notion that Indian's are other worldly. Thus, A study at the University of Washington
statement 3 is true. demonstrated the influence of Pisaster on species
richness by removing sea stars from selected parts of the
17. The passage thematically centres on :
intertidal zone repeatedly over a period of five years.
(a) the state of mind of oppressed people.
Two major changes occurred in the areas from which sea
(b) starvation and misery.
stars were removed. First, the lower edge of the mussel
(c) the growth of civilization.
bed extended farther down into the intertidal zone,
(d) body, mind and spirit of people in general.
showing that sea stars are able to eliminate mussels
Ans. (a) The theme of passage is centrally based on completely where they are covered with water most of
the state of mind of oppressed people. The poor and the time. Second, and more dramatically, 28 species of
unfortunate in every country became to some extent animals and algae disappeared from the sea star removal
other worldly, unless they became revolutionaries, for zone. Eventually only Mytilus, the dominant competitor,
this world is evidently not means for them. So also occupied the entire substratum.
subject peoples. Through its effect on competitive relationships,
18. According to the passage, the torture of the mind predation by Pisaster largely determines which species
and spirit is caused live in these rocky intertidal ecosystems.
IAS (Pre) GS IInd Paper (CSAT), 2011 5 YCT
19. What is the crux of the passage ? 3. If the keystone species is completely removed
(a) Sea star has a preferred prey. from an ecosystem, it will lead to the collapse of
(b) A preferred prey determines the survival of a the ecosystems
keystone species. With reference to the passage, which of the above
(c) Keystone species ensures species diversity. assumptions is/are valid?
(a) 1 only (b) 2 and 3 only
(d) Sea star is the only keystone species on the
(c) 1 and 3 only (d) 1, 2 and 3
Pacific coast of North America.
Ans. (c) The keystone species may influences both the
Ans. (c) The crux of the passage is keystone species species richness of community and flow of energy and
ensure species diversity as keystone species may material through ecosystem which means if keystone
influence both the species richness of community and species are completely removed or disturbed from an
flow of energy and material through the ecosystem and ecosystem then it may lead to disturbance or collapse
its preferred prey is mytilus californianus. of the ecosystem. So, statement 3 is valid as per the
20. With reference to the passage, consider the passage. The sea star which is an example of keystone
following statements : species, its preferred prey is the mussel which shows
1. Mussels are generally the dominant species in that food chain/food web in an ecosystem are
intertidal ecosystems. influenced by keystone species.
2. The survival of sea stars is generally determined Direction for the following 5 (five) items :
by the abundance of mussels. Read the following passage and answer the items
Which of the statements given above is/are correct? that follow. Your answer to these items should be
(a) 1 only based on the passage only.
(b) 2 only Passage
(c) Both 1 and 2 Now India’s children have a right to receive at least
(d) Neither 1 nor 2 eight years of education, the gnawing question is whether
it will remain on paper or become a reality. One hardly
Ans. (d) Statement 1 & 2 both are wrong as mussels
needs a reminder that this right is different from the
are not the dominant species in the intertidal ecosystem
others enshrined in the Constitution, that the beneficiary-
and also the survival of sea stars in not generally
a six year old child cannot demand it, nor can she or he
determined by the abundance of mussels as mussels
fight a legal battle when the right is denied or violated. In
are only a preferred prey of sea stars.
all cases, it is the adult society which must act on behalf
21. Which of the following is/are implied by the or the child. In another peculiarity, where a child’s right
passage? to education is denied, no compensation offered later can
1. Mussels are always hard competitors for sea be adequate or relevant. This is so because childhood
stars. does not last. If a legal battle fought on behalf of a child
2. Sea stars sof the Pacific coast have reached the is eventually won, it may be of little use to the boy or girl
climax of their evolution. because the opportunity missed at school during
3. Sea stars constitute an important component in childhood cannot serve the same purpose later in life.
the energy flow in intertidal ecosystem. This may be painfully true for girls because our society
Which of the statements given above is/are permits them only a short childhood, If at all. The Right
correct? to Education (RTE) has become law at a point in India’s
(a) 1 and 2 (b) 2 only history when the ghastly practice of female infanticide
has resurfaced in the form of feticide. This is
(c) 1 and 3 (d) 3 only
‘’symptomatic of a deeper turmoil’ in society which is
Ans. (d) According to the passage, keystone species compounding the traditional obstacles to the education of
may influence both the species richness of community girls. Tenacious prejudice against the intellectual
and flow of energy and material through ecosystem. potential of girls runs across our cultural diversity and
The sea stars Pisaster ochraceus is an example of the system of education has not been able to address it.
keystone species. 23. With reference to the passage, consider the
22. Consider the following assumptions : following statements :
1. The food chains/food web in an ecosystem are 1. When children are denied education, adult
influenced by keystone species. society does not act on behalf of them.
2. The presence of keystone species is a specific 2. Right to Education as a law cannot be enforced
characteristic of aquatic ecosystems. in the country.
IAS (Pre) GS IInd Paper (CSAT), 2011 6 YCT
Which of the statements given above is/are (a) 1 only
correct (b) 2 only
(a) 1 only (c) Both 1 and 2
(b) 2 only (c) Both 1 and 2
(d) Neither 1 nor 2 (d) Neither 1 nor 2
Ans. (d) According to the passage children have a Ans. (d) According to the passage right to education is
right to receive at least eight years of education. This fundamental right not the legal right, So, statement 1 is
right is differently from the others enshrined in the incorrect. There is nowhere mentioned goal of
constitution that the beneficiary a six year old child universal education, the education system in the
cannot demand it nor can she or he fight a legal battle country must be made identical to that of developed
when the right is denied or violated. In all cases it is countries. So, statement 2 also not hold valid. So,
the adult society which must act on behalf or the child. option (d) is correct.
Thus, statement 1 is incorrect. The right to education
has become law at a point in India's history when the 26. Which one of the following statements conveys
ghastly practice of female infanticide has resurged in the key message of the passage?
the form of feticide. Thus, statement 2 is also incorrect. (a) India has declared that education is compulsory
So, option (d) is correct. for its children
(b) Adult society is not keen on implementing the
24. According to the passage, what could be the
traditional obstacles to the education of girls? Right to Education
1. Inability of parents to fight a legal battle when (c) The Right to Education, particularly of a girl
the Right to Education is denied to their children child, needs to be safeguarded
2. The traditional way of thinking about girls’ role (d) The system of education should address the
in society. issue of Right to Education
3. The prejudice against the intellectual potential of Ans. (c) According to the passage, this may be
girls. painfully true for girls to get the right to education
4. Improper system of education. rights as our society permits them only a short
Select the correct answer from the codes given childhood so the right to education, particularly of girls
below child needs to be safeguarded.
(a) 1 and 2 only
(b) 2, 3 and 4 only 27. Which one of the following statements conveys
(c) 1, 3 and 4 only the inference of the passage?
(d) 1, 2, 3 and 4 (a) The society has a tenacious prejudice against the
Ans. (b) According to the passage the traditional intellectual potential of girls.
obstacle to the education of girl are mainly because of (b)Adults cannot be relied upon to fight on behalf
traditional way of thinking about girls role in society. of children for their Right to Education.
This 'symptomatic' of a deeper turmoil in society
(c) The legal fight to get education for children is
which is compounding the traditional obstacle to
education of girl. Tenacious prejudice against the often protracted and prohibitive.
intellectual potential of girls run across our cultural (d) There is no sufficient substitute for education
diversity and the system of education has not been able received in childhood
to address it. So, code 2, 3, 4 are correct. Ans. (d) According to the passage, in another
25. On the basis of the passage, consider the peculiarity, where a child's right to education is denied,
following statements : no compensation offered later can be adequate or
1. Right of Education is a legal right and not a relevant. This is so because childhood does not last and
fundamental right. opportunity missed at school during childhood cannot
2. For realizing the goal of universal education, the
serve the same purpose dates in life. So there is no
education system in the country must be made
substitute sufficient in respect to education received in
identical to that of developed countries.
childhood. So, statement (d) conveys the inference of
Which of the statements given above is/are
correct? the passage.

IAS (Pre) GS IInd Paper (CSAT), 2011 7 YCT


INTERPERSONAL & (a) ask for an extension of deadline

COMMUNICATION SKILL (b) Inform Headquarters of your inability to finish


on time
Directions for the following 8 (eight) items :
(c) suggest alternate person to Headquarters who
Given below are eight items. Each item describes
may do the needful
a situation and is followed by four possible responses.
Indicate the response you find most appropriate. (d) stay away till you recover
Choose only one response for each item. The
Ans. (a or c) According to the situation, we need to
responses will be evaluated based on the level of
ask for an extension of deadline or mostly we can
appropriateness for the given situation.
suggest alternate person to headquarters who may do
Please attempt all the items. There is no penalty
the needful.
for wrong answers for these eight items.
28. You have been asked to give an explanation for 31. You are an officer-in-charge for providing basic
not attending an important official meeting. Your medical facilities to the survivors of an
immediate boss who has not informed you about earthquake affected area. Despite your best
meeting is now putting pressure on you not to possible effort, people put allegations against you
place an allegation against him/her. You would for making money out of the funds given for
(a) send a written reply explaining the fact relief. You would
(b) seek an appointment with the top boss to explain (a) let an enquiry be set up to look into the matter
the situation
(c) admit your fault to save the situation (b) ask your senior to appoint some other person in
(d) put the responsibility on the coordinator of the your place
meeting for not informing (c) not pay attention to allegations
Ans. (a or b) According to the situation, we can seek (d) stops undertaking any initiative till the matter is
an appointment with the top boss to explain the resolved
situation or we can also send a written reply
Ans. (a or c) According to the situation, we should not
thoroughly explaining the fact.
pay attention to allegation and keep working in right
29. A local thug (bad element) has started illegal
direction and also to prove allegation wrong we can let
construction on your vacant plot. He has refused
an enquiry be set up to look into the matter.
your request to vacate and threatened you of dire
consequences in case you do not sell the property 32. You have been made responsible to hire boats at
at a cheap price to him. You would a short notice to be used for an area under flood.
(a) sell the property at a cheap price to him On seeing the price mentioned by the boat
(b) go to the police for necessary action owners you found that the lowest price was
(c) ask from help fro your neighbors approximately three times more than the
(d) negotiate with the goon to get a higher price
approved rate of the government. You would .
Ans. (b or d) According to the situation, the major
(a) reject the proposal and call for a fresh price
response comes in mind that we need to go to police
station and ask for necessary action or sometime we (b) accept the lowest price
can ask goon to sit and talk and to negotiate to get (c) refer the matter to the government and wait
higher price. So, option (b) or (d). (d) threaten the boat owners about a possible
cancellation of the license
30. You have to accomplish a very important task for
your Headquarters within the next two days. Ans. (b or d) According to the situation, we can accept
Suddenly you meet with an accident. Your office the lowest price as mentioned or threaten the boat
insists that you complete the task. You would - owners about a possible cancellation of the license.

IAS (Pre) GS IInd Paper (CSAT), 2011 8 YCT


LOGICAL & ANALYTICAL ABILITY The blood relation graph from question no. 35 to
question no. 37 (Solution)
33. Consider the following argument:
‘‘In order to be a teacher one must graduate
from collage. All poets are poor. Some
Mathematicians are poets. No college graduate is
poor.’’
Which one of the following is not a valid
conclusion regarding the above argument?
(a) Some Mathematicians are not teachers.
(b) Some teachers are not Mathematicians.
(c) Teachers are not poor.
(d) Poets are not teachers
Ans. (b)
35. How is D related to E ?
(a) Husband (b) Son
(c) Father (d) Wife
Ans. (a) "D" is husband of "E"
∵ In order to be a teacher one must graduate from
college. 36. Who are the females in the group?
Conclusion "Some teachers are not mathematicians" is (a) C and E (b) C and D
not valid. (c) E and A (d) D and E
34. The houses of A and B face each other on a road Ans. (c) The females in the group ⇒ E and A
going north-south, A’s being on the western side.
37. Whose wife is the teacher?
A comes out of his house, turns left, travels 5 km,
turn right, travels 5 km to the front of D’s house. (a) C (b) D
B does exactly the same and reaches the front of (c) A (d) B
C’s house. In this context, which one of he Ans. (d) B's wife is teachers.
following statements is correct?
(a) C and D live on the same street . DECISION MAKING ABILITY
(b) C’s house faces south.
38. You are the officer-in-charge of a village
(c) The house of C and D are less than 20 km apart.
(d) None of the above. administering distribution of vaccine in an
Ans. (c) On making the map according to the question, isolated epidemic hit village, and you are left with
only one vaccine. There is a requirement of that
vaccine from the Gram Pradhan and also a poor
villager. You are being pressurized by the Gram
Pradhan to issue the vaccine to him. You would
(a) initiate the procedure to expedite the next
supply without issueing the vaccine to either
Hence, (b) arrange vaccine for the poor villager from the
By map, The house of C and are less than distributor of another area.
20km. (c) ask both to approach a doctor and get an input
about the urgency.
Read the following passage and answer the 3 (d) arrange vaccine for the Gram Pradhan from the
(three) items that follow : distributor of another area.
A, B, C, D and E are members of the same
Ans. (b or d ) According to the situation, we need to
family. There are two fathers, two sons, two wives,
three males and two females. The teacher was the act like, firstly arrange vaccine for the poor villager
wife of a lawyer who was the son of a doctor. E is not from the distributor of another area or arrange vaccine
a male, neither also a wife of a professional. C is the for the Gram Pradhan from the distributor of another
youngest person in the family and D is the eldest. B is area.
a male.
IAS (Pre) GS IInd Paper (CSAT), 2011 9 YCT
39. You have taken up a project to create night- Direction for the following 2 (two) items
shelters for homeless people during the winter Each of the following two items consists of four
season. Within a week of establishing the shelters, statements. Of these four statements, two cannot both
be true, but both can be false. Study the statements
you have received complaints from the residents carefully and identify the two that satisfy the above
of the area about the increase in theft cases with a condition Select the correct answer using the codes
demand to remove the shelters. You would given below each set of statements :
(a) Ask them to lodge a written complaint in the 42. Examine the following statements :
police station 1. All animal are carnivorous
(b)Assure residents of an enquiry into the mater 2. Some animals are not carnivorous.
(c) Ask residents to consider the humanitarian effort 3. Animals are not carnivorous.
made 4. Some animals are carnivorous.
(d)Continue with the project and ignore their Code:
complaint (a) 1 and 3 (b) 1 and 2
Ans. (b or c) According to the situation, we need to (c) 2 and 3 (d) 3 and 4
assure residents of an enquiry into the matter or also
Ans. (a) ∵ The given condition is, of these statements,
ask residents to consider the humanitarian effort made.
two cannot both true, but both can be false.
40. You, as an administrative authority, have been ∴ Statement 1 (All animals are carnivorous) and
approached, by the daughter-in-law of an statement 3 (Animals are not carnivorous) are followed
influential person regarding harassment by her the condition.
in-laws on account of insufficient dowry. Her 43. Examine the following statements :
parents are not able to approach you because of 1. All trains are run by diesel engine
social pressures. You would- 2. Some trains are run by diesel engine
3. No train is run by diesel engine.
(a) call the in-laws for an explanation 4. Some trains are not run by diesel engine.
(b)counsel the lady to adjust, given such a Code:
circumstance (a) 1 and 2 (b) 2 and 3
(c) take action after her parents approach you (c) 1 and 3 (d) 1 and 4
(d)ask her to lodge a complaint with the police
Ans. (c) ∵ The given condition is, of these statements,
Ans. (a or d) According to the situation, we need to two cannot both true, but both can be false.
ask explanation from in-laws family or ask her to ∴ Statement (1) (All trains are run by diesel engine)
lodge a complaint with the police. and statement (3) (No train is run by diesel engine)
44. Consider the following figures :
GENERAL MENTAL APTITUDE
41. Consider the following three statements :
1. Only students can participate in the race
2. Some participants in the race are girls
3. All girl participants in the race are invited for What is the missing number?
coaching (a) 7 (b) 8
Which one of the following conclusions can be (c) 9 (d) 10
drawn from the above statements? Ans. (c) Just as,
(a) All participants in the race are invited for 2×3=6
coaching 6 × 4 = 24
(b) All students are invited for coaching 24 × 3 + 2 × 4 = 80
(c) All participants in the race are students Similarly,
(d) None of the statements (a), (b) and (c) given
3×3=9
above is correct
9 × 4 = 36
Ans. (c) "All participants in the race are students" is 36 × 3 + 3 × 4 = 120
correct. ∴ Missing Number = 3 × 3 = 9
nd
IAS (Pre) GS II Paper (CSAT), 2011 10 YCT
45. Study the following Figure ∴ No. of triangle formed by one-digit = 12
A person goes from A to B always moving to the ∵ Triangle formed by three-digit = (5, 10, 11) (9, 13,
right or downwards along the lines. How many
140
different routes can he adopt ?
∴ No. of triangle form by three digit = 2
∵ Triangle formed by four digit = (6, 7, 8, 12) (7, 11,
12, 13) (1, 2, 3, 4) (2, 5, 6, 7) (4, 7, 8, 9)
∴ No. of triangle by four digit = 5
triangle formed by others digit ⇒
Select the correct answer from the codes given (1, 2, 3, 4, 5, 6, 7, 8, 9)
below : (2, 5, 6, 7, 10, 11, 12, 13)
(a) 4 (b) 5
(c) 6 (d) 7 (4, 7, 8, 9, 11, 12, 13, 14)
(1, 2, 3, 4, 5, 6, 7, 8, 9, 10, 11, 12, 13, 14)
Ans. (c)
∴ No. of triangular formed other digit = 4
Hence,
Total no. of triangle in grid
= 12 + 2 + 5 + 4
= 23
47. In a queue, Mr. X is fourteenth from the front
and Mr. Y is seventeenth from the end, while Mr.
Z is exactly in between Mr. X and Mr. Y . If Mr.
X is ahead of Mr. Y and there are 48 persons in
the queue, how many persons are there between
Mr. X and Mr. Z ?
(a) 6 (b) 7
Hence, A person can adopt six different routes from A (c) 8 (d) 9
to B.
Ans. (c)
46. Consider the following figure and answer the
item that follows :

The no. of persons between Mr. X and Mr. Y


⇒ 48 – (14 + 17)
= 48 – 31
= 17
∴ There are 8 persons between Mr. X and Mr. Z.
What is the total number of triangles in the above
grid ?
(a) 27 (b) 26
BASIC NUMERICAL ABILITY
(c) 23 (d) 22
48. Consider the four age pyramids given below
Ans. (c)
namely A, B, C and D representing four different
countries

∵Triangle formed by one-digit = 1, 2, 3, 4, 5, 6, 7, 8,


z
9, 11, 12, 13

IAS (Pre) GS IInd Paper (CSAT), 2011 11 YCT


Which one of them indicates the declining (a) Town /Menj A
population? (b) Town /Menj B
(a) A (b) B (c) Same in Town A and Town B
(c) C (d) D (d) No inference can be drawn
Ans. (d) No inference can be drawn.
Ans. (c) Option 'C' indicates the declining population
∵ Given the disease categories as percentage of the
49. The following figure has four curves namely A, total number of patient but not given that number of
B,C and D. Study the figure and answer the item patients.
that follows. 51. What can we say about persons with more than
one disease from these graphs ?
(a) There are likely to be persons with more than
one disease in Town A
(b) There are likely to be persons with more than
one disease in Town B
(c) There are likely to be persons with more than
one disease in both Towns A and B
(d) No Inference can be drawn.
Which curve indicates the exponential growth? Ans. (b) ∵ Town A = (18 + 13 + 22 +34 + 13)%
(a) A (b) B ⇒ 100%
(c) C (d) D and town B = (24 + 19 + 30 + 28 + 20)% = 121%
Ans. (c) Curve C indicates the exponential growth. Hence,
There are likely to be person with more than
Direction for the following 2 (two) items : one disease in town B.
The following pie charts show the break-up of
52. Consider the following Velocity-Time Graph. It
disease categories recorded in the patients from two
shows two trains starting simultaneously on
towns. Town A and Town B. Pie charts plot the parallel tracks.
disease categories as percentage of the total number
of patients.
Based on these, answer the two items that follow
the charts.
Distribution of diseases in Town-A

With reference to the above graph, which one of


the following statements is not correct?
(a) Train B has an initial acceleration greater than
that of Train A.
(b) Train B is faster than Train A at all times
(c) Both trains have the same velocity at time T0
Distribution of diseases in Town-B (d) Both trains travel the same distance in time t0
units
Ans. (d) The velocity of an object is the rate of charge
of its position with respect to a frame of reference and
is a function of time.
∆d
ν=
∆t
Acceleration is the rate of velocity.
ν − ν0
a=
t1 − t 2
∆ν
a=
50. Which of the two town has a higher number of ∆t
persons with Diabetes ?
IAS (Pre) GS IInd Paper (CSAT), 2011 12 YCT
Hence, 54. Which region/regions of the curve correspond/
Option A, B and C is correct and option D corresponds to incubation phase of the infection?
"Both trains travel the same distance in time to units" (a) A only
(b) B only
is not correct.
(c) B and C
53. Consider the following distance-time graphs. The (d) No part of the curve indicates the incubation
graph shows three athletes A,B and C running phase
side by side for a 30 km race. Ans. (a) Region 'A' of the curve corresponds to
incubation phase of the infection.
55. Which region of the curve indicates that the
person began showing the symptoms of infection?
(a) A (b) B
(c) C (d) D
Ans. (b) Region 'B' of the curve indicates that the
person began showing the symptoms of infection.
56. Which region of the curve indicates that the
treatments yielded effective relief ?
With reference to the above graph, consider the (a) C
following statements : (b) D
1. The race was won by A . (c) E
2. B was ahead of A up to 25 km mark . (d) The curve does indicate the treatment
3. C ran very slowly from the beginning. Ans. (c) Region 'E' of the curve indicates that the
Which of the statements given above is /are treatment yielded effective relief.
correct? 57. There are four routes to travel from city A to city
(a) 1 only B and six routes from city B to city C. How many
(b) 1 and 2 only routes are possible to travel from the city A to
(c) 2 and 3 only city C ?
(d) 1, 2 and 3 (a) 24 (b) 12
Ans. (b) With reference to the given graph, 1 and 2 (c) 10 (d) 8
statements are correct. Ans. (a) Possible routes to travel from the city A to
city C = 4 × 6
Directions for the following 3 (three) items ⇒ 24
Read the passage given below, study the graph
58. A contract on construction job specifies a penalty
that follows and answer the three items given below
the figure. for delay in completion of the work beyond a
During a party, a person was exposed to certain date is as follows : Rs. 200 for the first
contaminated water. A few days later, he developed day etc. Rs. 250 for the second day Rs. 300 for the
fever and loose motions. He suffered from some days third day etc. the penalty for each succeeding day
before going to a doctor for treatment. On starting being Rs.50 more than that of the preceding day.
the treatment, he soon became better and recovered How much penalty should the contractor pay if
completely a few days later. The following graph he delays the work by 10 days ?
shows different phases of the person’s disease
condition as regions A, B, C, D and E of the curve. (a) ` 4950 (b) ` 4250
(c) ` 3600 (d) ` 650
Ans. (b) According to question, we find the arithmetic
progression,
200, 250, 300 ................ (till 10 days)
∵ a = 200
d = 50
n = 10

IAS (Pre) GS IInd Paper (CSAT), 2011 13 YCT


n Ans. (a)
Sn =  2a + ( n − 1) d 
2
10
= [2 × 200 + (10 – 1) 50]
2
= 5 [400 + 450]
= 5 (850) = 4250 `

59. Consider the figure given below and answer the


item that follows :

The minimum number of different colours required to


paint the figure = 3
61. Consider the following figure and answer the
item that follows :
In the figure shown, OP1 and OP2 are two plane
mirrors kept perpendicular to each other. S is the
direction of a beam of light falling on the mirror
OP1 the direction of the reflected beam of ligh
from the mirror OP2 wills
(a) Perpendicular to the direction S.
(b) At 450 to the direction S.
A square is divided into four rectangles as shown
(c) Opposite and parallel to the direction S.
above. The lengths of the side of rectangles are
(d) At 600 to the directions S.
natural numbers. The areas of two rectangles are
Ans. (c) The direction of the reflected beam of light indicated in the figure.
from the mirror OP2 wills opposite and parallel to (a) 10
direction S. (b) 11
(c) 15
60. Consider the following figure and answer the
(d) Cannot be determined as the given data are
item that follows :
insufficient.
Ans. (b)

The length of the square = 11


What is the minimum number of different
colours required to paint the figure given above 62. Three persons start walking together and their
such that no two adjacent regions have the same steps measure 40 cm, 42 cm and 45 cm
colour? respectively. What is the minimum distance ech
(a) 3 (b) 4 should walk so that each can cover the same
distance in complete steps?
(c) 5 (d) 6
IAS (Pre) GS IInd Paper (CSAT), 2011 14 YCT
(a) 25m 20 cm = 40 cm
(b) 50 m 40 cm 320
and covered distance by train in one hour =
(c) 75 m 60 cm 5
(d) 100 m 80 cm = 64 cm
Ans. (a) ∴ Required Ratio = 40 : 64
On taking L.C.M. of 40, 42 and 45:- =5:8
(a) L.C.M. = 2 × 2 × 2 × 3 × 3 × 5 × 7
65. There are 100 students in a particular class. 60%
= 2520cm
students play cricket, 30% student play football
2520
Required minimum distance = m and 10% students play both the games. What is
100
the number of students who play neither cricket
= 25.20m nor football?
⇒ 25m 20 cm (a) 25
63. A person has only Rs. 1 and Rs. 2 coins with her. (b) 20
If the total number of coins that she has is 50 and (c) 18
the amount of money with her is Rs. 75, then the (d) 15
number of Rs. 1 and Rs. 2 coins are, respectively. Ans. (b)
(a) 15 and 35 By Venn diagram:-
(b) 35 and 15
(c) 30 and 20
(d) 25 and 25
Ans. (d)
Let the number of 1 ` coins = x
∴ the number of 2 ` coins = (50 – x)
then,
x × 1 + (50 – x) 2 = 75
Where,
x + 100 –2x = 75
C√ ⇒ play cricket
100 – x = 75
F√ ⇒ play football
x = 25
C× ⇒ don't play cricket
∴ y = 50 – x
F× ⇒ don't play football
= 50 – 25
Let the number of students who play neither cricket
Hence,
nor football x
The number of Rs. 1 and Rs. 2 coins are 25
and 25 respectively. ∴ x + 50 + 10 20 = 100
x + 80 = 100
64. If a bus travels 160 km in 4 hours and a train
x = 100 – 80
travels 320 km in 5 hours at uniform speeds, then
x = 20
what is the ratio of the distances traveled by them
in one hour? 66. A village having a population of 4000 requires
(a) 8 : 5 150 litres of water per head per day. It has a tank
(b) 5 : 8 measuring 20m×15m×6m. The water of this tank
(c) 4 : 5 will last for.
(d) 1 : 2 (a) 2 days
Ans. (b) (b) 3 days
160 (c) 4 days
Covered distance by Bus in one hour =
4 (d) 5 days

IAS (Pre) GS IInd Paper (CSAT), 2011 15 YCT


Ans. (b) ∵ total number of married person = 30
∵ Per head per day requires water = 150 liters and total number of married males = 70 × 2/7
∴ The total population of 4000 requires ⇒ 20
water = 150 × 4000 ∴ The total number of married females = 30 – 20
= 600000 liters ⇒ 10
∵ Volume of Tank = 20m × 15m × 6m Then
Total number of unmarried females
= 1800 m3
= 30 – 10
∵ 1m3 = 1000 liters ⇒ 20
∴ Capacity of Tank = 1800 × 1000 20
∴ Required fraction =
= 1800000 liters 30
1800000 2
Required days = = 3 days. ⇒
600000 3
or
67. A student on her first 3 rests received and
average scor of N points. If she exceeds her
previous average score by 20 points on her fourth
test, then what is the average score for the first 4
tests ?
(a) N + 20
(b) N + 10
(c) N + 4
(∵Total number of married = 30
(d) N + 5
∴ Female married = 30 – 20 = 10)
Ans. (d) 20
Required fraction =
∵ the average of 30
2
first three test = N =
3
∴ The total score of first + three test = 3 N
Read the following passage and answer the 3
According to question, (three) items that follow :
The total score of first (3N + (N + 20)) In a survey regarding a proposed measure to be
Four test introduced, 2878 persons took part of which 1652
were males. 1226 persons voted against the proposal
⇒ 4N + 20
of which 796 were males. 1425 persons voted for the
4N + 20 proposal. 196 females were undecided.
∴ Required Average =
4 Flow chart from question 69 to question 71.
= (N + 5)
Flow chart:-
68. In a group of persons, 70% of the persons are
male and 30% of the persons are married. If two
sevenths of the males are married, what fraction
of the females is single ?
(a) 2/7 (b) 1/3
(c) 3/7 (d) 2/3
Ans. (d)
Let the total number of person = 100
∴ Males = 70
females = 30

IAS (Pre) GS IInd Paper (CSAT), 2011 16 YCT


69. How many females voted for the proposal? 73. He soon fell asleep because
(a) 430 (b) 600 (a) he was exhausted
(c) 624 (d) 640 (b) he was all alone

Ans. (b) 600 females voted for the proposal. (c) he had not slept for days
(d) he was very frightened
70. How many males wre undecided?
(a) 31 (b) 227 Ans. (a) He soon fell a sleep because he was
exhausted. He walked several miles without eat or
(c) 426 (d) 581
drink, except some dry bread and some water which he
Ans. (a) 31 males were undecided. got from cottagers and farmers.

71. How many females were not in favour of 74. With reference to the passage, consider the
proposal following statements :
(a) 430 1. He was walking through the countryside.
(b) 496 2. The cottagers and farmers gave him enough
food so that he could sleep at night without
(c) 586 feeling hungry.
(d) 1226 Which of the statements given above is/are
correct?
Ans. (a) 430 females were not in favour of proposal.
(a) 1 only
ENGLISH LANGUAGE/GRAMMAR (b) 2 only
& COMPREHENSION (c) both 1 and 2
(d) Neither 1 nor 2
Passage-I
Ans. (a) With reference to the passage, the statement
He walked several miles that day but could not get first is correct. He was walking through the country
anything to eat or drink except some dry bread and side. The cottagers and farmers did not give him
some water, which he got from cottagers and farmers. enough food. He slept under a pile of dry grass
As night fell, he slept under a haystack lying in a because he was exhausted.
meadow. He felt frightened at first, for the wind blew
awfully over the empty fields. He felt cold and hungry, Passage-II
and was feeling more lonely than he had ever felt I opened the bag and packed the boots in; and
before. He, however, soon fell asleep, being much tired then, just as I was going to close it, a horrible idea
with his long walk. When he got up next day, he was occurred to me. Had I packed my toothbrush? I don't
feeling terribly hungry so he purchased a loaf of bread know how it is, but I never do know whether I've
with a few coins that he had. packed my toothbrush.
72. When the night fell, he slept My toothbrush is a thing that haunts me when
(a) in the open field I'm travelling, and makes my life a misery. I dream
that I haven't packed it, and wake up in a cold
(b) under a pile of dry grass
perspiration, and get out of bed and hunt for it. And,
(c) in a farmer's cottage in the morning, I pack it before I have used it, and it
(d) under a tree is always the last thing I turn out of the bag; and then
Ans. (b) When the nights fell, he slept under a pile of I repack and forget it, and have to rush upstairs for it
dry grass that day he walked several miles. So as night at the last moment and carry it to the railway station,
fell he slept under a hay stack lying in a meadow. wrapped up in my pocket-handkerchief.

IAS (Pre) GS IInd Paper (CSAT), 2011 17 YCT


75. When he was going to close the bag, the idea that Females are protective of their cubs but tend to
occurred to him was ignore family rivalry over food. In 21 years of
(a) unpleasant photographing polar bears, I've only once seen the
smallest of triplets survive till autumn.
(b) sad
(c) fantastic 78. Female polar bears give birth during :

(d) amusing (a) spring

Ans. (a) When he was going to close the bag, the idea (b) summer
that occurred to him was unpleasant. He understood (c) autumn
that he had not packed his toothbrush. So, he was
(d) winter
unpleasant.
Ans. (d) Female polar bears give birth during winter.
76. What makes his life miserable whenever he
undertakes travelling? Polar bear mothers emerge from dens with three month
old cubs.
(a) Going to railway station
(b) Forgetting the toothbrush 79. Mother bear

(c) Packing his bag (a) takes sides over cubs

(d) Bad dreams (b) lets the cubs fend for themselves

Ans. (b) For getting the toothbrush makes his life (c)feeds only their favourites
miserable whenever he undertakes traveling. (d) sees that all cubs get an equal share
77. His toothbrush is finally
Ans. (b) Mother bear lets the cubs fend themselves.
(a) in his bag The mother bear has fastened for as long as eight
(b) in his bed months but that does not stop the young from

(c) in his handkerchief demanding full acces to her remaining reserves.

(d) lost 80. With reference to the passage, the following


assumptions have been made :
Ans. (c) His toothbrush is finally in his handkerchief.
At the last moment he carry his toothbrush to the 1. Polar bears fast as long as eight months due to
railway station, wrapped up in his pocket – non-availability of prey.
handkerchief.
2. Polar bears always give birth to triplets.
Passage-III Which of the assumptions given above is/are
In spring, polar bear mothers emerge from dens valid?
(a) 1 only
with three month old cubs. The mother bear has
(b) 2 only
fasted for as long as eight months but that does not
(c) Both 1 and 2
stop the young from demanding full access to her
(d) Neither 1 nor 2
remaining reserves. If there are triplets, the most
persistent stands to gain an extra meal and it may Ans. (d) With Reference to the passage following
have the meal at the expense of others. The smallest assumptions neither 1 nor 2 are correct. Polar bears do
of the litter forfeits many meals to stronger siblings. not always give birth to triplets.

IAS (Pre) GS IInd Paper (CSAT), 2011 18 YCT


UNION PUBLIC SERVICE COMMISSION
Civil Services (Preliminary Exam) - 2012
CSAT : PAPER-II
(Chapterwise Analysis with Explanation)
Time : 2 hours Maximum Number : 200
economic well being and voice and as a protection
COMPREHENSION against free market individualism. So agricultural
Directions for the following 6 (six) items : collectives such as a group based farming, can provide
Read the following two passages and answer the empowerment increased agricultural productivity and
items that follow each passage. Your answers to these
safeguard against exploitative markets. Thus answer
items should be based on the passages only.
(b) is correct.
Passage-1
The poor especially in market economies, need the 2. What does the author imply by ‘‘gender
strength the collectivities offer for creating more impact’’?
economic, social and political space for themselves, for (a) Women are doubtful participants in cooperatives.
enhancing their socio-economic well-being and voice, (b) Family cooperatives may not include women.
and as a protection against free market individualism. It (c) Women benefiting from group farming .
has been argued that a group approach to farming,
(d) Woman’s role in transition economies is highly
especially in the form of bottom up agricultural
restrictive
production collectivities, offers substantial scope for
poverty alleviation and empowering the poor as well as Ans. (c) According to the passage, the author imply by
enhancing agricultural productivities. To realise this gender impact is that the gender impact of the family
potential, however, the groups would need to be co-operative in the transition economies are uncertain,
voluntary in nature, small in size, participative in the Indian example of women only groups farming
decision making and equitable in work sharing and offer considerable potential for benefiting women. So,
benefit distribution. There are many notable examples of
option (c) is correct.
such collectivities to be found sin varied contexts, such
as in the transition economies All of them bear witness to 3. Considers the following assumptions :
the possibility of successful cooperation under given 1. It is imperative for transition economies to have
conditions. And although the gender impact of the family agricultural collectivities.
cooperatives in the transition economies are uncertain,
2. Agricultural productivity can be increased by
the Indian examples of women–only groups farming
group approach to farming.
offer considerable potential for benefiting women.
1. Agricultural collectivities such as group based With reference to the above passage, which of these
farming. Can provide the rural poor assumptions is/are valid ?
1. empowerment. (a) 1 only
2. Increased agricultural productivity. (b) 2 only
3. safeguard against exploitative markets. (c) Both 1 and 2
4. surplus production of agricultural commodities. (d) Neither 1 nor 2
Select the correct answer using the codes given Ans. (b) With reference from the passage, we can
below :
(a) 1, 2, 3 and 4 conclude that agricultural productivity can be
(b) 1, 2 and 3 only increased by group approach to the farming is correct
(c) 2 and 4 only as it is mentioned in passage that, "It has been argued
(d) 1, 3 and 4 only that a group approaching to farming, especially in the
Ans. (b) According to the passage, the poor especially form of bottom up agriculture production collectivities,
in market economies, need the strength the collectives offers substantial scope for poverty alleviation and
offer for creating more economic, social and political empowering the poor as well as enhancing agricultural
space for themselves, to enhancing their socio- productivity''. Thus, only assumption (2) is correct.

IAS (Pre) GS IInd Paper (CSAT), 2012 19 YCT


Passage-2 Ans. (b) According to the passage, it is very clear that
In a typical Western liberal context, deepening of greater democratization in India has not necessarily led
democracy invariably leads to consolidation of ‘liberal to irrelevance of community idea as a governing force
values’. In the Indian context, democratization is in Indian Politics.
translated into greater involvement of people not as 6. What is the ‘silent revolution’’ that has occurred
‘individuals’ which is a staple to liberal discourse, but as in the Indian democratic process ?
communities or groups. Individuals are getting, involved (a) Irrelevance of caste and class hierarchies in
in the public sphere not as ‘atomizes’ individuals but as political processes.
members of primordial communities drawn on religious (b) Loosening of social strictures in voting behaviour
or caste identity. Community - Identity seems to be the and patterns.
governing force. It is not therefore surprising that the so- (c) Social change through transfer of power from
called peripheral groups continue to maintain their upper caste elites to subaltern groups.
identities with reference to the social groups (caste, (d) All the statements (a), (b) and (c) given above
religion or sect) to which they belong while getting are correct in this context.
involved in the political processes despite the fact that Ans. (c) According to the passage the 'Silent
their political goals remain more or less identical. By Revolution' that has occurred in the Indian democratic
helping to articulate the political voice of the process is through a meaningful transfer of power from
the upper caste elites to various subaltern group within
marginalized, democracy in India has led to loosening of
the democratic framework of public governance, this
social strictures’ and empowered the peripherals to be
happened mainly because of so-called peripheral
confident of their ability to improve to improve the socio
groups continue to maintain their identities with
economic conditions in which they are placed. This is a reference to the social group to which they belong
significant political process that had led to a silent while getting involved in political process.
revolution through a meaningful transfer of power from
the upper caste elites to various subaltern groups within Directions for the following 5 (five) items :
the democratic framework of public governance. Examine the information given in the following
4. According to the passage, what does ‘’deepening paragraph and answer the items there follow :
of democracy’’ mean in the Western context ? Guest lectures on five subjects viz., Economics,
(a) Consolidation of group and class identities. History, Statistics, English and Mathematics have to be
(b) Democratization translated as greater arranged in a week from Monday to Friday. Only one
involvement of people. lecture can be arranged on each day. Economics cannot
(c) Democratization as greater involvement of be scheduled on Tuesday. Guest faculty for History is
‘atomized’ individuals in the public sphere available only on Tuesday. Mathematics lecture has to be
(d) None of the statements (a), (b) and (c) given scheduled immediately after the day of Economics
above is correct in this context. lecture. English lecture has to be scheduled immediately
Ans. (c) According to the passage "deepening of before the day of Economics lecture.
democracy" invariably leads to consolidation of liberal 7. Which lecture is scheduled on Monday ?
values. In the Indian context democratization is (a) History
translated into greater involvement of people as a (b) Economics
communities or groups whereas individuals are getting (c) Mathematics
involved in the public sphere not as atomize individual
(d) Statistics
according to western liberal context. So, option (c) is
correct. Ans. (d) According to the passage mathematics lecture
needs to be followed after economics and economic
5. Greater democratization in India has not lecture need to be followed after English
necessarily led to English - Economics - Maths
(a) the dilution of caste and communal identities in So,
the public sphere. Monday - Statistics
(b) Irrelevance of Community identity as a Tuesday - History
governing force in Indian politics.
Wednesday - English
(c) marginalization of elite groups in society
(d) relative unimportance of hereditary identities Thursday - Economics
over class identities. Friday - Maths
IAS (Pre) GS IInd Paper (CSAT), 2012 20 YCT
8. Which lecture is scheduled between Statistics and Ans. (a) According to the passage, education without
English? doubt has an important functional instrumental and
(a) Economics utilitarian dimension. But in deepest sense education is
(b) History not instrumentalist. That is to say, it is not to be
(c) Mathematics
justified outside to itself because it leads the
(d) No Lecture
acquisition of formal skill or the certain desired
Ans. (b)
psychological social attributes.
9. Which lecture is the last one in the week ?
(a) History (b) English 13. According to the passage, education must be
(c) Math (d) Economic respected in itself because
Ans. (c) (a) it helps to acquire qualifications for employment
10. Which lecture is scheduled on Wednesday ? (b) it helps in upward mobility and acquiring social
(a) Statistics (b) Economics status
(c) English (d) History (c) it is an inner process of moral and intellectual
Ans. (c)
development.
11. Which lecture is scheduled before the
(d) All the (a),(b) and (c) given above are correct in
Mathematics lecture
(a) Economics (b) History this context.
(c) Statistics (d) English Ans. (c) If we observe in the passage, it is clearly
Ans. (a) mentioned that education is process of expansion and
Direction for the following 15 (fifteen) items : conversion, not in the sense of converting or turning
Read the following three passages and answer students into doctors and engineer's and providing
the items that follow each passage. Your answer to employment but also it helps us in widening and
these items should be based on the passage only. turning out of the mind, the creation, sustenance and
Passage–1 development of self critical awareness and
Education, without a doubt, has an important independence of thought. It is also inner process of
functional Instrumental and utilitarian dimension. This is moral - intellectual development.
revealed when one asks question such as what is the
14 Education is a process in which
purpose of education ? ‘The answers, too often, are’ to
acquire qualifications for employment/ upward (a) students are converted into trained professionals.
mobility’, 'wider/higher (in terms of income) (b) opportunities for higher income are generated.
opportunities’, and ‘to meet the need s for trained human (c) individuals develop self-critical awareness and
power in diverse fields for national development’. But in independence of thought.
its deepest sense education is not instrumentalist. That is (d) qualifications for upward mobility are acquired.
to say, it is not to be justified outside to itself because it Ans. (c) According to the passage education is a
leads to the acquisition of formal skills or the certain process of expansion and conversion not in the sense
desired psychological social attributes. It must be of converting or turning out of the mind the creation,
respected in itself. Education is thus not a commodity to
sustenance and development of self critical awareness
be acquired or possessed and then used, but a process of
of thought.
inestimable importance it individuals and society,
although it can and does have enormous use value. Passage-2
Education then, is a process of expansion and
Chemical pesticides lose their role in sustainable
conversion, not in the sense of converting or turning
students into doctors or engineers, but the widening and agriculture if the pests evolve resistance. The evolution
turning out of the mind- the creation, sustenance and of pesticide resistance is simply natural selection in
development of self-critical awareness and independence action. It is almost certain to occur when vast numbers of
of thought. It is an inner process of moral –intellectual a genetically variable population are killed. One or a few
development. individuals may be unusually resistant (perhaps because
12. What do you understand by the ‘instrumentalist’ they possess an enzyme that can detoxify the pesticide).
view of education ? If the pesticide is applied repeatedly, each successive
(a) Education is functional and utilitarian in its generation of the pest will contain a larger proportion of
purposes. resistant individuals. Pests typically have a high intrinsic
(b) Education is meant to fulfils human needs.
rate of reproduction and so a few individuals in one
(c) The purpose of education is to train the human
intellect. generation may give rise to hundreds or thousands in the
(d) Education is meant to achieve moral development next, and resistance spreads very rapidly in a population.
IAS (Pre) GS IInd Paper (CSAT), 2012 21 YCT
This problem was often ignored in the past even 16. With reference to the passage consider the
though the first case of DDT (dichlorodiphynyltrich following statements :
Ioroethane) resistance was reported early as 1946. There 1. Use of chemical pesticides has become
is exponential increase in the numbers of invertebrates imperative in all the poor countries of the world.
that have evolved resistance and in the number of
2. Chemical pesticides should not have any role in
pesticides against which resistance have evolved.
sustainable agriculture.
Resistance has been recorded in every family of
arthropod pests (including dipterans such as mosquitoes 3. One pest can develop resistance to many
and house files, as well as beetles, moths, wasps, fleas, pesticides.
lice and mites) as well as well as in weeds and plant Which of the statements given above is/are correct?
pathogens. Take the Alabama leafworm, a moth pest of (a) 1 and 2 only
cotton, as an example . It has developed resistance in one (b) 3 only
or more regions of the world to aldrin, DDT dieldrin, (c) 1 and 3 only
endrin lindane and toxaphene. (d) 1, 2 and 3
If chemical pesticides brought nothing but problems- Ans. (b) With reference to the passage, if pesticides
if their use was intrinsically and acutely unsustainable- are used repeatedly, each successive generation of the
then they would already have fallen out of widespread pest will contain a large proportion of resistance
use. This has not happened. Instead, their rate of individuals. It is as so because few individual posses
production has increased rapidly. The ratio of cost of enzymes that can detoxify the pesticide.
benefit for the individual agricultural producer has 17. Though the problems associated with the use of
remained in favour of pesticide use. In the USA, chemical pesticides is known for a long time.
insecticides have been estimated to benefit the
Their widespread use has not waned. Why ?
agricultural products to the tune of around $5 for every
(a) Alternatives to chemical pesticides do not exist at
$1 spent.
all.
Moreover, in many poorer countries, the prospect of (b) New pesticides are not invented at all.
imminent mass starvation, or of an epidemic diseases are (c) Pesticides are biodegradable.
so frightening that the social and health costs of using (d) None of the statements (a), (b) and (c) given
pesticides have to be ignored. In general the use of above is correct.
pesticides is justified by objective measures such as
Ans. (d) With reference to the passage, moreover, in
‘lives saved’, ‘economic efficiency of food production’
and ‘total food produced’. In these very fundamental many poorer countries, the prospect of imminent mass
senses, their use may be subscribed as sustainable. In saturation or of an epidemic disease are so frightening
practices, sustainability depends on continually that the social and health cost of using pesticides have
developing new pesticides that keep at least one step to be ignored. In general the use of pesticide is
ahead of the pests-pesticides that are less persistent, justified by objective measure such as life saved. This
biodegradable and more accurately targeted at the pests. sense suggest that alternative to chemical pesticides
15. ‘’The evolution of pesticide resistance is natural are existing and new variety of pesticides are
selection in action.’’ What does it actually imply? experiment and they are targeting biodegradable pest
(a) It is very natural for many organisms to have which are not available now. So, option a, b, c are
pesticide resistance. wrong and thus, option (d) is correct.
(b) Pesticide resistance among organisms is a 18. How do pesticides act as agents for the selection
universal phenomenon. of resistant individuals in any pest population ?
(c) Some individuals in any given population show 1. It is possible that in a pest population the
resistance after the application of pesticides. individuals will behave differently due to their
(d) None of the statements (a), (b) and (c) given genetic makeup.
above is correct. 2. Pests do possess the ability to detoxify the
Ans. (c) According to the passage, it is very clear that pesticides.
chemical pesticide lose their role in sustainable 3. Evolution of pesticide resistance is equally
agriculture if the pests evolve resistance. The distributed in pest population.
evaluation of pesticide resistance is simply naturally Which of the statements given above is/are
selection in action. It is almost certain to occur when correct ?
vast numbers of a genetically, variable population are (a) 1 only
killed. One or a few individuals may be unusually (b) 1 and 2 only
(c) 3 only
resistance.
(d) 1, 2 and 3
IAS (Pre) GS IInd Paper (CSAT), 2012 22 YCT
Ans. (b) From the passage it is very clear that some of Passage-3
the pest population have special enzyme that can Today’s developing economies use much less
detoxify the pesticide and increases the resistant energy per capita than developed countries such as the
power. This individual pest behave differently due to United States did at similar incomes, showing the
potential for lower-carbon growth. Adaptation and
their different genetic makeup. Thus, option (1) and (2)
mitigation need to be integrated into a climate-smart
is correct, so option (b) is correct. development strategy that increases resilience, reduces
19. Why is the use of chemical pesticides generally the threat of further global warming, and improves
development outcomes. Adaptation and mitigation
justified by giving the examples of poor and measures can advance development, and prosperity can
developing countries ? raise incomes and foster better institutions. A healthier
1. Developed countries can afford to do away with population living in better-built houses and with access
use of pesticides by adapting to organize to bank loans and social security is better equipped to
deal with a changing climate and its consequences.
farming, but it is imperative for poor and
Advancing robust, resilient development policies that
developing countries to use chemical pesticides. promote adaptation is needed today because changes in
2. In poor and developing countries, the pesticides the climate, already begun, will increase even in the short
addresses the problem of epidemic diseases of term.
crops and eases the food problem. The spread of economic prosperity has always been
3. The social and health costs of pesticide use are intertwined with adaptation to changing ecological
generally ignored in poor and developing conditions. But as growth has altered the environment
and as environmental change has accelerated, sustaining
countries.
growth and adaptability demands greater capacity to
Which of the statements given above is/are correct? understand out environment generate new adaptive
(a) 1 only technologies and practices and diffuse them widely. As
(b) 1 and 2 only economic historian have explained, much of
(c) 2 only humankind’s creative potential has been directed at
adapting to the changing world. But adaptation cannot
(d) 1, 2 and 3 cope with all the impacts related to climate change’s
Ans. (c) These use of chemical pesticide generally especially as larger changes unfold in the long terms.
justified by giving the example of poor and developing Countries cannot grow out of harm’s way fast
countries, if we use pesticides in poor and developing enough to match the changing climates. And some
countries it addresses the problem of epidemic disease growth strategies. Whether driven by the government or
the market, can also add to vulnerability-
of crops and eases the food problem.
Particularly if the overexploit natural resources.
20. What does the passage imply? Under the Soviet development plan, irrigated cotton
(a) Alternative options to chemical pesticides should cultivation expanded in water-stressed Central Asia and
be promoted. led to the near disappearance of the Aral Sea, threatening
(b) Too much use of chemicals is not good for the the livelihoods of fishermen, herders and farmers. And
clearing mangroves-the natural coastal buffers against
ecosystem.
storm surges- to make way for intensive farming or
(c) There is no scope for the improvement of housing development, increases the physical
pesticides and making their use sustainable. vulnerability of coastal settlements, whether in Guinea or
(d) Both the statements (a) and (b) above are correct. in Louisiana.
Ans. (d) Given passage implies that too much uses of 21. Which of the following conditions of growth can
chemical is not good for the ecosystem and it may lead add to vulnerability ?
to ecological imbalance and thus there should be 1. When the growth occurs due to excessive
alternative options to chemical pesticides should be exploitation of mineral resources and forests.
promoted thus statement (a) and (b) both are correct. 2. When the growth brings about a change in
Whereas statement (c) says, there is no scope for the humankind’s creative potential
improvement of pesticides and making their use 3. When the growth is envisaged only for providing
houses and social security's to the people.
sustainable, which is not correct. So, option (d) is
4. When the growth occurs due to emphasis on
correct.
farming only.
IAS (Pre) GS IInd Paper (CSAT), 2012 23 YCT
Select the correct answer using the code given Ans. (b) According to the passage, adaptation and
below. mitigation measures can advance development and
(a) 1 only prosperity can raise income and foster better institution
(b) 2, 3 and 4 only but whereas adaptation and mitigation need to be
(c) 1 and 4 only integrated into a climate smart development strategy
for sustainable growth so the necessary conditions for
(d) 1, 2, 3 and 4 only
sustainable economic growth are
Ans. (a) According to the passage, countries cannot popularising/spreading of adaptive technology widely
grow out of harms way fast enough to match the and also investing on research in adaptation and
changing climates. And some growth strategies, mitigation technologies. Thus, statement (2) and (3)
whether driven by the government or the market can are correct.
also add to vulnerability, particularly if they 24. Which of the following inferences can be made
overexploit natural resource like minerals and forest, from the passage?
thus option (a) only 1 is correct and other statements 1. Rainfed crops should not be cultivated in irrigated
does not supports the conditions. areas.
2. Farming under water-deficient areas should not be
22. What does low-carbon growth imply in the a part of development strategy.
present context? Select the correct answer using the codes given
1. More emphasis on the use of renewable sources below :
of energy. (a) 1 only
2. Less emphasis on manufacturing sector and more (b) 2 only
(c) Both 1 and 2
emphasis on agriculture sector.
(d) Neither 1 nor 2
3. Switching over from monsoculture practices to
Ans. (d) If we go through passage these is nowhere
mixed farming. mentioned about Rainfed crops irrigation and farming
4. Less demand for goods and service under water deficient areas. So, we can't conclude
Select the correct answer using the codes given about any of the given statement. Thus, option (d) is
below. correct.
(a) 1 only 25. Consider the following assumptions :
(b) 2, 3 and 4 1. Sustainable economic growth demands the use of
creative potential of man.
(c) 1 and 4 only
2. Intensive agriculture can lead to ecological
(d) None of the above implies low carbon growth backlash.
Ans. (a) The first paragraph of the passage talks about 3. Spread of economic prosperities can adversely
how developing countries are using less energy per affect the ecology and environment.
capital than developed countries and similarly they With reference to the passage, which of the above
assumptions is/ are valid?
develop same income and this shows potential for
(a) 1 only
lower carbon growth and gave more emphasis on the (b) 2 and 3 only
use of renewable energy resource. (c) 1 and 3 only
(d) 1, 2 and 3
23. Which of the following conditions is/are
Ans. (d) According to passage the spread of economic
necessary for sustainable economic growth ?
prosperity has always been intertwined with adaptation
1. Spreading of economic prosperity more. to changing ecological conditions. But as growth has
2. Popularising/spreading of adaptive technologies altered the environment and as environmental change
widely has accelerated sustaining growth and adaptability
3. Investing on research in adaptation and mitigation
demands greater capacity to understand out
technologies.
environmental generate new adaptive technologies and
Select the correct answer using the codes given
practices and diffuse them widely. Thus, statement (1)
below and (3) are correct and also clearing mangroves the
(a) 1 only natural coastal buffers against storm surges to make
(b) 2 and 3 only way for intensive farming increases the vulnerability
(c) 1 and 3 only and dead to ecological back clash. Thus, statement (2)
(d) 1, 2 and 3 is also correct.
IAS (Pre) GS IInd Paper (CSAT), 2012 24 YCT
26. Which one of the following statements species. The effects of introductions have been to convert
constitutes the central theme of this passage ? a hugely diverse range of local community compositions
(a) Countries with greater economic prosperity are into something much more homogeneous.
better equipped to deal with the consequences It would be wrong, however, to conclude that
of climate change. introducing species to a region will inevitably cause a
(b) Adaptation and mitigation should be integrated
decline in species richness there. For example, there are
with development strategies.
numerous species of plants, invertebrates and vertebrates
(c) Rapid economic growth should not be pursued
by both developed and developing economies. found in continental Europe but absent from the British
(d) Some countries resort to over exploitation of Isles (many because they have so far failed to recolonize
natural resources for the sake of rapid after the last glaciations). Their introduction would be
development. likely to segment British biodiversity. The significant
Ans. (b) The central theme of the passage is detrimental effect noted above arises where aggressive
"Adaptation and mitigation should be integrated with species provide a noval challenge to endemic biotas. ill –
development strategies", as it is clearly stated in equipped to deal with them.
passage that adaptation and mitigation need to be
27. With reference to the passage, which of the
integrated into a climate smart development strategy
following statements is correct?
that increases resilience, reduces the threat of global
(a) Introduction of exotic species into new
warming and improve development outcomes. geographical areas always leads to reduced
Directions for the following 11 (eleven) items : biodiversity.
Read the following three passages and answer the (b) Exotic species introduced by man into new
items that follow each passage. Your answers to areas have always greatly altered the native
these items should be based on the passages only. ecosystems.
Passage-1 (c) Man is only reason to convert a hugely diverse
Invasions of exotic species into new geographic range of local community compositions into
areas sometimes occur naturally and without human more homogeneous ones.
agency. However, human actions have increased this (d) None of the statements (a), (b) and (c) is correct
in this context.
trickle to a flood. Human-caused introductions may
occur either accidentally as a consequence of human Ans. (d) According to the passage, invasion of exotic
transport, or intentionally but illegally to serve some species into new geographic areas, sometimes occur
private purpose or legitimately to procure some hoped- naturally and without human agency. Thus statement 1
is wrong. Exotic species introduced by man into new
for public benefit by bringing a pest under control.
areas have always greatly altered the native ecosystem
Producing new agricultural products or providing noval
is not correct as per the passage. Thus, statement 2 is
recreational opportunities. Many introduced species are wrong. Man is not only reason to convert a hugely
assimilated into communities without much obvious diverse range of local community compositions into
effect. However, some have been responsible for more homogeneous one. Thus, all the statements are
dramatic changes to native species and natural wrong and option (d) is correct.
communities. For example, the accidental introduction of
28. Why does man introduce exotic species into new
the brown tree snake Bioga irregularies into Guam, an
geographical areas ?
island in the Pacific, has through nest predation reduced
1. To breed exotic species with local varieties.
10 endemic forest bird species to the point of extinction.
2. To increase agricultural productivity.
One of the major reasons for the world’s great 3. For beautification and landscaping.
biodiversity is the occurrence of centers of endemism so Which of the above statements is /are correct ?
that similar habitats in different parts of the world are (a) 1 only
occupied by different groups of species that happen to (b) 2 and 3 only
have evolved there. If every species naturally had access
(c) 1 and 3 only
to everywhere on the globe, we might expect a relatively
small number of successful species to become dominant (d) 1, 2 and 3
in each biome. The extents to which this homogenization Ans. (d) According to the passage man introduce
can happen naturally is restricted by the limited powers exotic species into new geographical areas to breed
of dispersal of most species in the face of the physical exotic species with local varieties and to increase
barriers that exist to dispersal. By virtue of the transport
agricultural productivity and for beautification and
opportunities offered by humans, these barriers have
been breached by an ever-increasing number of exotic landscaping. All the statements are correct.

IAS (Pre) GS IInd Paper (CSAT), 2012 25 YCT


29. How is homogenization prevented under natural development ?’’-often came, at least to start with, from
conditions? East Asian countries, and their voice grew in influence as
(a) Evolution of groups of species specific to local several of these countries were immensely successful-
habitats. through the 1970s and 1980s and even later- in
(b) Presence of oceans and mountain ranges. promoting economic growth without pursuing
(c) Strong adaptation of groups of species to local democracy.
physical and climatic conditions.
To deal with these issues we have to pay particular
(d) All the statements (a),(b) and (c) given above are
correct in this context. attention to both the content of what can be called
development and to the interpretation of democracy (in
Ans. (d) Evolution of groups of species specific to
particular to the respective roles of voting and of public
local habitats and presence of oceans and mountain
reasoning). The assessment of development cannot be
ranges and strong adaptation of groups of species to
local physical and climatic conditions are divorced from the lives that people can lead and the real
freedom that they enjoy Development can scarcely be
homogenization prevented under natural conditions.
Thus, all the statements a, b, c are correct. seen merely in terms of enhancement of inanimate
objects of convenience, such as a rise in the GNP (or in
30. How have the human beings influenced the personal incomes), or industrialization- important as they
biodiversity ?
may be as means to the real ends. Their value must
1. By smuggling live organisms.
depend on What they do to the lives and freedom of the
2. By building highway.
people involved, which must be central to the idea of
3. By making ecosystems sensitive so that new
species are not allowed. development.
4. By ensuring that new species do not have major If development is understood in a broader way. With
impact. a focus on human lives, then it becomes immediately
Which of the statements given above are correct? clear that the relation between development and
(a) 1 and 2 (b) 2 and 3 democracy has to be seen partly interims of their
(c) 1 and 3 (d) 2 and 4 constitutive connection, rather than only through their
Ans. (a) The human beings influenced the biodiversity external links. Even though the question has often been
by smuggling live organism and by building highway. asked whether political freedom is ‘‘conducive to
Thus, only options (1) and (2) are correct. development’’ we must not miss the crucial recognition
31. What can be the impact of invasion of exotic that political liberties and democratic rights are among
species on an ecosystem ? the ‘’constituent components’’ of development. Their
relevance for development does not have to be
1. Erosion of endemic species
established indirectly through their contribution to the
2. Change in the species compositions of the
growth of GNP.
community of the ecosystem.
Select the correct answer using the code given below : 32. According to the passage, why is a serious tension
(a) 1 only perceived between democracy and development
(b) 2 only by the detractors of democracy ?
(c) Both 1 and 2 (a) Democracy and development are distinct and
(d) Neither 1 nor 2 separate goals.
Ans. (c) According to the passage, many introduced (b) Economic growth can be promoted successfully
species are assimilated into communities without much without pursuing a democratic system of
obvious effect. However, some have been responsible governance.
for dramatic changes to native species and natural (c) Non-democratic regimes deliver economic faster
communities and thus both the statements are correct. and far more successfully than democratic ones.
(d) All the statements (a), (b) and (c) given above are
Passage-2
correct in this context.
Most champions of democracy have been rather Ans. (b) According to the passage, the serious tension
reticent in suggesting that democracy would itself perceived between democracy and development by the
promote and enhancement of social welfare-they have detractors of democracy because the economic growth
tended to see them as good but distinctly separate and can be promoted successfully without pursuing a
largely independent goals. The detractors of democracy, democratic system of governance.
on the other hand, seemed to have been quite willing to
express their diagnosis of what they see as serious 33. According to the passage, what should be the
tensions between democracy and development. The ultimate assessment/aim /view of development ?
theorists of the practical split-‘’Make up your mind do (a) Rise in the per capita income and industrial
you want democracy, or instead, do you want growth rates.

IAS (Pre) GS IInd Paper (CSAT), 2012 26 YCT


(b) Improvement in the Human Development Index 35. With reference to the passage, consider the
and GNP. following statements :
(c) Rise in the savings and consumption trends. 1. It is desirable that the impact of Foreign Direct
(d) Extent of real freedom that citizens enjoy. Investment should be pro- competitive.
Ans. (d) The assessment of development cannot be 2. The entry of foreign investors invariably leads to
divorced from the lives that people can lead and the the inflated prices in domestic markets.
real freedom that enjoy. Development can scarcely be Which of the statements given above is/are
seen merely in terms of enhancement of inanimate correct?
objects of convenience. (a) 1 only (b) 2 only
34. What does a ‘’constitutive’’ connection between (c) Both 1 and 2 (d) Neither 1 nor 2
democracy and development imply ? Ans. (a) The need of competition law becomes more
(a) The relation between them has to be seen evident when foreign direct investment is liberalized.
through external links. The impact of FDI is not always procompetitive. Thus
(b) Political and civil rights only can lead to statement 1 is correct. The foreign investor may
economic development. substantially lessen competition and gain dominant
(c) Political liberties and democratic rights are position in the relevant market thus charging higher
essential elements of development. price and thus statement 2 is wrong.
(d) None of the statements (a), (b) and (c) given
36. According to the passage, how does a foreign
above is correct in this context.
investor dominate the relevant domestic market ?
Ans. (c) According to the passage, even though the
1. Multinational companies get accustomed to
question has often been asked whether political
domestic laws.
freedom is "conducive to development". We must not
2. Foreign companies establish joint ventures with
miss the crucial recognition that political liberties and
domestic companies.
democratic rights are among the "Constituent
3. Affiliates in a particular market / sector lose their
components of development".
independence as their parent companies overseas
Passage-3 merge.
4. Foreign companies lower the cost of their
The need of competition Law becomes more evident products as compared to that of products of
when foreign direct investment (FDI) is liberalized. The domestic companies.
impact of FDI is not always procompetitive. Very often Which of the statements given above are correct?
FDI takes the form of foreign corporation acquiring a (a) 1 and 2 only (b) 2 and 3 only
domestic enterprise or establishing a joint ventures with (c) 1, 2 and 3 only (d) 1,2, 3 and 4
one. By making such an acquisition the foreign investor Ans. (b) Very often FDI takes the form of foreign
may substantially lessen competition and gain a corporation acquiring domestic enterprises or
dominant position in the relevant market, thus charging establishing a joint ventures with ones. Subsequently,
higher prices. Another scenario is where the affiliates of the parent companies overseas merge. With the
two separate multinational companies (MNCs) have been affiliates no longer remaining independent,
established in competition with one another in a competition in the host country may be virtually
particular developing economy, following the eliminated thus only statement (2) and (3) are correct.
liberalization of FDI. Subsequently, the parent 37. What is the inference from this passage ?
companies overseas merge. With the affiliates no longer (a) Foreign investors and multinational companies
remaining independent, competition in the host country always dominate the domestic market.
may be virtually eliminated and the prices of the (b) It is not in the best interests of the domestic
products may be artificially inflated. Most of these economy to allow mergers of companies
adverse consequences of mergers and acquisitions by (c) With competition law, it is easy to ensure a level
MNCs can be avoided is an effective completion law is playing field between domestic and foreign
in place. Also, an economy that has implemented and firms.
(d) For countries with open economies, Foreign
effective competition law is in a better position to attract
Direct Investment is essential for growth.
FDI than one that has not. This is not just because most
MNCs are expected to be accustomed to the operation of Ans. (c) The inference from this passage is with
such a law in their home countries and know how to deal competition law, it is easy to ensure a level playing
field between domestic and foreign to firms. The need
with such concerns but also that MNCs expect
of competition law becomes more evident when
competition authorities to ensure a level playing field
foreign direct investment is liberalized.
between domestic and foreign firms.
nd
IAS (Pre) GS II Paper (CSAT), 2012 27 YCT
INTERPERSONAL SKILL & (a) All expensive things are regarded as luxury
(b) All essential things for learning are not luxury.
COMMUNICATION SKILLS (c) Television is essential for learning.
38. Examine the following statements : (d) Television is not luxury item.
1. Only those who have a pair of binoculars can Ans. (d) According to the statement it is very much
become the members sof the birdwatcher’s club. clear that T.V. is not luxury item.
2. Some members of the bird watcher’s club have
Cameras. LOGICAL & ANALYTICAL
3. Those members who have cameras can take part
in photos-contests. ABILITY
Which of the following conclusion can be drawn 41. Consider the following statements :
from the above statements? 1. All machines consume energy.
(a) All those who have a pair of binocsulars are 2. Electricity provides energy.
members of the bir watcher’s club.
3. Electricity operated machines are cheap to
(b) All members of the birdwatcher’s club have a
maintain.
pair of binoculars.
(c) All those who take part in photo-contests are 4. Electrically operated machines do not causes
members of the birdwatcher’s club. pollution.
(d) No conclusion can be drawn. Which one of the following inferences can be
drawn from the above statements?
Ans. (b)
(a) All machines are run by electric energy.
(b) There is no form of energy other than electricity.
(c) Most machines are operated on electric energy
(d) Electrically operate machines are preferable to
use
Ans. (d) On the basis of statement, It can be inferred
that electrically operate machines are preferable to use
because electrically operated machines are cheap to
maintain and do not causes pollution.
42. Examine the following statements :
39. During the last summer vacation, Ankit went to a 1. None but the rich can afford air-travel.
summer camp where he took part in hiking, 2. Some of those who travel by air become sick
swimming and boating. This summer, he is 3. Some of those who become sick require treatment.
looking forward to a music camp where he hopes
Which one of the following conclusions can be
to sing, dance and learn to play the guitar.
Based on the above information, four conclusions, drawn from the above statements?
as given below, have been made. Which one of these (a) All the rich persons travel by air.
logically follows from the information given above? (b) Those who travel by air become sick.
(a) Ankit, parents want him to play the guitar. (c) All the rich persons become sick.
(b) Ankit, prefers music to outdoor activities. (d) All those who travel by air are rich.
(c) Ankit, goes to some type of camp every summer. Ans. (d)
(d) Ankit likes to sing and dance
Ans. (d) According to the information, there is
nowhere mentioned about Ankit parent and also no
outdoor activities thus option (a) and (b) are wrong.
According to the information, last summer and this
summer he went to camp but it is not sure that usually
he goes to camp every summer. Thus option (c) is
wrong. But, this summer he is looking forward to a Hence,
music camp where he hopes to sing and dance and All those who travel by air are rich.
learn to play guitar. Thus, option (d) is correct. 43. In five flats, one above the other, live five
40. Consider the following statement : professionals. The professor has to go up to meet
"Thought quite expensive, television is not a his IAS officer friend. The doctor is equally
luxury item, as one can learn many things friendly to all, and has to go up as frequently as
through television." go down. The engineer has to go up to meet his
Which one of the following is valid inference MLA friend above whose flat lives the professor’s
from the above statement? friend.
IAS (Pre) GS IInd Paper (CSAT), 2012 28 YCT
From the ground floor to the top floor, in what 46. Examine the following statements :
order do the five professionals live? 1. None but students are members of the club.
(a) Engineer, Professor, Doctor, IAS officer, MLA 2. Some members of the club are married persons.
(b) Professor, Engineer, Doctor, IAS officer, MLA 3. All married persons are invited for dance.
(c) IAS officer, Engineer, Doctor, Professor, MLA Which sone of the following conclusions can be
(d) Professor, Engineer, Doctor, MLA, IAS officer. drawn from the above statements?
Ans. (d) (a) All students are invited for dance.
According to the question, The order of residence of 5 (b) All married students of the club are invited for
professionals. dance.
IAS (c) All members of the club are married persons.
(d) None of the above conclusions can be drawn.
MLA
Ans. (b)
Doctor
Professor/Engineer
Professor/Engineer

44. Examine the following statements :


1. I watch TV only if I am bored.
2. I am never bored when I have my brother’s 47. Six squares are colored, front ant back, red (R),
company.
blue (B), yellow (Y), green (G), white (W) and
3. Whenever I go to the theater I take my brother’s
orange (O) and are hinged together as shown in
with me.
Which one of the following conclusions is valid in the figure given. If they are folded to form a
the context of the above statements cube, what would be the face opposite the white
(a) If I am bored, I watch TV. face?
(b) If I am bored, I seek my brother, company.
(c) If I am not with my brother, then I watch TV.
(d) If I am not bored, I do not watch TV.
Ans. (d) On examining the statements,
It can be concluded that, "If I am not bored, I do not
watch T.V." (a) R (b) G (c) B (d) O
45. Only six roads A, B, C, P, Q and R connect a Ans. (c)
military camp to the rest of the country. Only one On folding the squares in the form of cube:-
out of A, P and R is open at any one time. If B is Opposite Opposite
B ←→ W G ←→ O
closed, so is Q. Only one of A and B is open Opposite
during storms. P is closed during floods. R ←→ Y
In this context, which one of the following Hence,
The blue face would be the face opposite the
statements is correct?
white face.
(a) Under normal conditions only three roads are
48.
open.
(b) During storms at least one road is open.
(c) During floods only three roads are open.
(d) During calamities all roads are closed.
Ans. (b)

In the figure, circle P represents hardworking


people, circle Q represents intelligent people,
circle R represents truthful people, and circle S
represents honest people. Which region
(If B is closed, so is Q) represents the people who are intelligent, honest
Hence, During storms at least one road is open. and truthful but not hardworking?
IAS (Pre) GS IInd Paper (CSAT), 2012 29 YCT
(a) 6 (b) 7 Ans. (b)
(c) 8 (d) 11
Ans. (a)

51. Examine the following statements :


1. Either A and B are of the same age or A is older
than B.
2. Either C and D are of the same age or D is older
than C.
3. B is older than C.
Hence, The region 6 represents the people who are Which one of the following conclusions can be
intelligent, honest and truthful but not hardworking. drawn from the above statements?
49. Three views of a cube following a particular (a) A is older then B
motion are given below ? (b) B and D are of the same age
(c) D is older than C
(d) A is older than C
Ans. (d)
A≥B
D≥C
B>C
What is the letter opposite to A? then A ≥ B > C
(a) H (b) P (c) B (d) M D≥C
Ans. (a) Hence, A > C
In the first two cube, "It can be concluded that A is older than C"
Taking K as origin and rotating the cubes clockwise:- 52. Ten new TV shows started in Januarys-5 sitcoms,
3 drama and 2 news magazines. By April, only
seven of the new shows were still on, five of them
being sitcoms Based on the above information,
four conclusions, as given below, have been made.
Which one of these logically follows from the
information given above?
K, B, A (a) Only one news magazine show is still on.
↓ ↓ ↓ (b) Only one of the drama shows is still on.
K, M, H (c) At least one discontinued show was a drama.
Opposite (d) Viewers prefer sitcoms over drama.
Then, B ←→ M
Opposite Ans. (c)
A ←→ H "T.V. shows" starting in January=
Opposite 5 sitcoms + 3 drama + 2 News magazines
K ←→ P
Hence, ∵ By April, only seven of shows were still on five of
H is the letter opposite to A. them being sitcoms based,
50. Consider the following statements : Then,
1. All artists are whimsical. Probability-
2. Some artists are drug addicts. 5 sitcoms + 2 drama = 7
3. Frustrated people are prone to become drug 5 sitcoms + 2 news Magazines = 7
addicts. 5 sitcoms + 1 drama + 1 new Magazines = 7
Hence,
From the above three statements it may be
At least one discontinued shows was a drama.
concluded that
(a) Artists are frustrated. 53. Mr. Kumar drives to work at an average speed of
(b) Some drug addicts are whimsical. 48 km per hour. The time taken to cover the first
(c) All frustrated people are drug addicts 60% of the distance is 10 minutes more than the
time taken to cover the remaining distance. How
(d) Whimsical people are generally frustrated.
far is his office?
IAS (Pre) GS IInd Paper (CSAT), 2012 30 YCT
(a) 30 km (b) 40 km Ans. (d)
(c) 45 km (d) 48 km Rama > Rani
Ans. (b) Ratna > Rani
Let the time taken by Mr. Kumar to cover 40% of Ratna > Rama
distance is x minutes. Ratna > Padma > Rama
∴ Time taken to cover 60% of the distance = (x + 10) Then the order,
Ratna > Padma > Rama > Rani
60% of D 40% of D
According to question, = Hence, Ratna scored the highest.
x + 10 x
6x = 4(x + 10) DEISION MAKING & PROBLEM
2x = 40
x = 20 57. Consider the following statement :
∵ t1 = x = 20 min The Third World War, if it ever starts, will end
t2 = (x + 10) = 20 + 10 = 30 min very quickly with the possible end of civilization.
30 20 It is sonlsy the misuse of nuclear power which
Total Distance = D1 + D2 = 48 × + 48 × will trigger it.
60 60
Based on the above statement, which one of the
= 24 + 16 = 40 Km. following inferences is correct ?
54. Gita is prettier than Sita but not as pretty as (a) Nuclear power will be used in the Third World
Rita. Then. War
(a) Sita is not as pretty as Gita (b) There will be no civilization left after the Third
(b) Sita is prettier than Rita. World War.
(c) Rita is not as pretty as Gita (c) The growth of nuclear power will destroys
(d) Gita is prettier than Rita civilization in the long run.
(d) The Third World War will not take place.
Ans. (a)
Ans. (a)
According to questions,
Directions for the following 7 (Seven) items :
Gita > Sita, Rita > Sita
Given below are seven items. Each item describes
∴ Rita > Gita > Sita. a situation and is followed by four possible responses.
Hence, Sita is not as pretty as Gita. Indicae the response your find most appropriate.
55. Given that, Choose only one response for each item. The
1. A is the brother of B. responses will be evaluated based on the level of
2. C is the father of A. appropriateness for the given situation.
3. D is the brother of E. Please attempt all the items. There is no penalty,
4. E is the daughter of B. for wrong answers for these seven items.
Then, the uncle of D is 58. You have differences of opinion regarding the
(a) A (b) B final report prepared by your subordinate that is
(c) C (d) E to be submitted, urgently. The subordinate is
Ans. (a) justifying the information given in the report.
You would…..
(a) Convince the subordinate that he is wrong.
(b) Tell him to reconsider the results
(c) Revise the report on your own.
(d) Tell him not to justify the mistake.
Ans. (a or c) With reference to the final report
prepared by his subordinate, which is to be submitted
(Where + Male, – Female) very soon differs from your opinion. If your
G → Generation subordinate is justifying the information given in the
The uncle of D is A. report. Then in that situation, you will convince that
56. Examine the following statements : your subordinate is wrong or you amend the report
1. Rama scored more than Rani. yourself.
2. Rani scored less than Ratna.
59. You are competing with your batch-mate for a
3. Ratna scored more than Rama. prestigious award to be decide based on an oral
4. Padma scored more than Rama but less than presentation. Ten minutes are allowed for each
Ratna. presentation. You have been asked by the
Who scored the hightest ? committee to finish on time. Your friend
(a) Rama (b) Padma however, is allowed more than the stipulated time
(c) Rani (d) Ratna period. You would….
IAS (Pre) GS IInd Paper (CSAT), 2012 31 YCT
(a) Lodge a complaint to the chairperson against the (a) Recommend that the supply of water be free of
discrimination. charge in all respects.
(b) Not listen to any justification from the (b) Recommend that the users pay a onetime fixed
committee. sum for the installation of taps and the usage of
(c) Ask for withdrawal of your name. water be free.
(d) Protest and leave the place.
(c) Recommend that a fixed monthly charge be
Ans. (a or d) In questioned situation, for this levied only on the non-BPL families and for BPL
discrimination you file a complain with chairperson or families water should be free.
opposing you will leave that place.
(d) Recommend that the users pay a charge based on
60. You are handling a time-bound project. During the consumption of water with differentiated
the project review meeting you find that the charges for non-BPL and BPL families.
project is likely to go delayed due to lack of Ans. (c or d) In the given situation, you would
cooperation of the team members. You would ….. recommend that a monthly fee be levied for families
(a) Warns the team members for their non above the poverty line and water supply should be free
cooperation
for the families below the poverty line or you will
(b) Look into reasons for non-cooperation.
(c) Ask for the replacement of team members. recommend that the users pay a fee based on the
(d) Ask for extension of time citing reasons. consumption of water, in which differentiated charges
should be fixed for the families above and below the
Ans. (a or b) In the situation in question, you will
poverty line.
warn the members of your group for their non-co-
operation or scrutinize the reason for non - 64. As a citizen you have some work with a
cooperation. government department. The official calls you
again and again, and without directly asking you,
61. You are the chairperson of a state sports
sends out feelers for a bribe. You want to get
committee. You have received a complaint and
your work done. You would.
later it was found that an athlete in the junior age
(a) Give a bribe
category who has won a medal has crossed-the
(b) Behave as if you have not understood the feelers
age criteria by 5 days. You would…..
and persist with your application.
(a) Ask the screening committee for a clarification.
(c) Go to the higher officer for helps verbally
(b) Ask the athlete to return the medal.
complaining about feelers.
(c) Ask the athlete to get an affidavit from the court
(d) Send in a formal complaint.
declaring his/her age.
(d) Ask the members of the sports committee for Ans. (b or c ) In the situation in question, you will
behave as if you do not understand his gestures. and
views. stick to your application or you will go to higher
Ans. (a or d) In the questioned situation, you will ask authority for assistance with verbal complaint
the screening committee for clarification or ask the regarding bribe gestures.
members of the sports committee for their opinion. 65. Read the passage given below and the two
statements that follow (given on the basis of the
62. You are handling a priority and have been
passage)
meeting all the deadlines and are therefore Four men are waiting at Delhi airport for a
planning your leave during the project. Your Mumbai flight. Two are doctors and the other
immediate boss does not grant your leave citing two are businessmen. Two speak Gujarati and
the urgency of the project. You would. two speak Tamil. No two of the same profession
(a) Proceed on level without waiting for the sanction. speak the same language. Two are Muslims and
(b) Pretend to be sick and take leave. two are Christians. No two of the same religion
(c) Approach higher authority to reconsider the are of the same profession, nor do they speak the
leave application same language. The Tamil-speaking doctor is a
Christian.
(d) Tell the boss that it is not justified. 1. The Christian-businessman speak Gujarati.
Ans. (c or d ) In the situation in question, you will talk 2. The Gujarati-speaking doctor is a Muslim.
to the higher authority to reconsider the leave Which of the above statements is/are correct
application, or tell the adjacent officer that this is not conclusion/ conclusions ?
fair. (a) 1 only
(b) 2 only
63. You are involved in setting up a water supply (c) Both 1 and 2
project in a remote area. Full recovery of cost is (d) Neither 1 nor 2
impossible in any case. The income levels in the Ans. (c) It is clear from reading the passage that
area are low and 25% of the population is below Christian businessman speaks Gujarati. The doctor
poverty line (BPL). When a decision has to be who speaks Gujarati is a Muslim.
taken of pricing you would. Hence, both the conclusions are valid.
IAS (Pre) GS IInd Paper (CSAT), 2012 32 YCT
GENERAL MENTAL ABILITY 68. Three persons A, B and C wore shirts of black,
66. Four political parties W,X,Y and Z decided to set blue and orange colours (not necessarily in that
up a joint candidate for the coming order) and pants of green, yellow and orange
parliamentary elections. The formula agreed by colours (not necessarily in that order). No person
them was the acceptance of the candidates by wore shirt and pants of the same colour.
them was the acceptance of a candidate by most
of the parties. Four aspiring candidates, A, B,C, Further, it is given that
and D approached the parties for their tickets. 1. A did not wear shirt of black colour
A, B, C and D approached the parties for their 2. B did not wear shirt of blue colour
tickets.
3. C did not wear shirt of orange colour
A was acceptable to W but not to Z.
B was acceptable to Y but not to X. 4. A did not wear pants of green colour
C was acceptable to W and Y. 5. B wore pants of orange colour
D was acceptable to W and X. What were the colours of the pants and shirt
When candidate B was preferred by W and Z, worn by C, respectively?
candidate C was preferred by X and Z, and (a) Orange and black
candidates A was acceptable to X but not to Y, (b) Green and blue
Who got the ticket ? (c) Yellow and blue
(a) A (b) B (c) C (d) D (d) Yellow and black
Ans. (c) Ans. (b)
Political Parties Persons Pants Shirts
W X Y Z A Yellow Orange
Candidates

A × ×
B Orange Black
B ×
C C Green Blue
D — — Hence,
According to the statement, All political parties accept C wore Pant of green colour and shirt of Blue
C candidate. colour.
Hence, C got the ticket.
67. Consider the following statements : BASIC NUMERICAL ABILITY
1. All X-brand cars parked here are white 69. Two glasses of equal volume are respectively half
2. Some of them have radial tyres. and three-fourths filled with milk. They are then
3. All X-brand cars manufactured after 1986 have filled to the brim by adding water. Their contents
radial tyres are then poured into another vessel. What will be
4. All cars are not X-brand the reatio of milk to water in this vessel.
Which one of the following conclusions can be (a) 1 : 3 (b) 2 : 3 (c) 3 : 2 (d) 5 : 3
drawn from the above statements ? Ans. (d) Let the quantity of equal volume in both the
(a) Only white cars are parked here. vessel be x.
(b) Some white X-brand cars with radial tyres are
parked here.
(c) Cars other than X-brand cannot have radial tyres.
(d) Most of the X-brand cars are manufactured
before 1986.
Ans. (b)

1 x 3 3x
Milk = x × = Milk = x × =
2 2 4 4
x x 3x 3x x
Water = x – = Water = x– = =
2 2 4 4 4
The ratio of milk and water in both vessels.
x + 3x 2x +3x
5x 5
Hence, = 2x x4 = 4 = =
+ 2x + x 3x 3
Some white X-brand cars with radial tyres are 2 4 4
parked here. The required ratio = 5 : 3

IAS (Pre) GS IInd Paper (CSAT), 2012 33 YCT


70. The elements of the problem figures given below Which one of the figures shown below occupies
are changing with a certain rule as we observe the blank space (?) in the matrix given above ?
them from left to right.

(a) (b)

According to this rule, which of the following


would be the next figure if the changes were (c) (d)
continued with the same rule ?
Ans.(d)Basic Numerical Ability

ENGLISH LANGUAGE COMPREHENSION

Passage─1
For fourteen and a half months I lived in my little
cell or room in the Dehradun jail, and I began to feel as if
I was almost a part of it. I was familiar with every bit of
Ans-(d) it, I knew every mark and dent on the whitewashed walls
and on the uneven floor and the ceilling with its moth-
71. Consider the following information regarding the
eaten rafters. In the little yard outside I greeted little tufts
performance of a class of 1000 students in four
of grass and odd bits of stone as old friends. I was not
different tests :
alone in my cell, for several colonies of wasp and hornets
Tests I II III IV lived there, and many lizards found a home behind the
Average marks/ 60 60 70 80 rafters, emerging in the evenings in search of prey.
Range of marks/ 30 45 20 0 73. Which of the following explains best the sentence
to to to to in the passage "I was almost a part of it"?
90 75 100 100 (a) I was not alone in the cell.
If a student scores 74 marks in each of the four (b) I was familiar with every bit of the cell.
tests, in which one of the following tests is her (c) I greeted little tufts of grass like old friends.
performance the best comparatively ? (d) I felt quite at home in the cell.
(a) Test I (b) Test II Ans. (b) According to the passage, for fourteen and a
(c) Test III (d) Test IV half months, I lived in my little cell or room in the
Ans. (b) On observing all the tests, Dehradun Jail, and I began to feel as if I was almost a
The maximum limit of the second test is 75. Which is part of it.
close to the marks obtained by a student at 74. 74. The passage attempts to describe
∴ In comparison, the second test is the best. (a) the general conditions of the country's jails
(b) the prisoner's capacity to notice the minute
72. details of his surroundings.
(c) the prisoner's conscious efforts to overcome the
loneliness.
(d) the prisoner's ability to live happily with other
creatures.
Ans. (c) According to the passage, if attempts to
describe about the prisoner conscious effort to
overcome the loneliness.

IAS (Pre) GS IInd Paper (CSAT), 2012 34 YCT


75. The author of the passage seems to suggest that 78. When they crawled into the tent
(a) it is possible to adjust oneself to uncongenial (a) they took off their gloves because it was not very
surroundings. cold.
(b) the conditions in Indian prisons are not bad. (b) they could not take off their gloves because it
(c) it is not difficult to spend one's time in a prison. was very cold.
(d) there is a need to improve the conditions in our (c) they took off their gloves though it was very
jails. cold.
Ans. (a) The author of the passage seems to suggest (d) they did not take off their gloves though it was not
that it is possible to adjust oneself to uncongenial cold.
surroundings. Ans. (a) When they crawled into the tent, they took off
their gloves because it was not very cold at night, thus
Passage─2
most climbers take of their boots but the writer prefer
We started pitching the highest camp that has ever to keep them on.
been made. Everything took five times as long as it
would have taken in a place where there was enough air
Passage─3
to breathe; but at last we got the tent up, and when we A local man, staying on the top floor of an old
crawled in, it was not too bad. There was only a light wooden house, was awakened at midnight by a fire.
wind, and inside it was not too cold for us to take off our Losing his way in a smoke-filled passage, he missed the
gloves. At night most climbers take off their boots; but I stairway and went into another room. He picked up a
prefer to keep them on. Hillary, on the other hand, took bundle to protect his face from the fire and immediately
his off and laid them next to his sleeping bag. fell through the floor below where he managed to escape
76. What does the expression ``pitching the highest through a clear doorway. The ``bundle" proved to be the
camp" imply? baby of the Mayor's wife. The ``hero" was congratulated
(a) They reached the summit of the highest mountain by all.
in the world. 79. The man went into another room because
(b) Those who climbed that far earlier did not pitch (a) he did not know where exactly the stairway was.
any camp. (b) the passage was full of smoke.
(c) So far nobody has ever climbed that high. (c) he was extremely nervous.
(d) They were too many climbers and needed to (d) he stumbled on a bundle.
pitch a big camp. Ans. (b) The man went into another room because the
Ans. (c) The writer started pitching to highest camp passage was full of smoke. That's why he missed the
that has ever been made the expression, "Pitching the stair way and went into another room.
highest camp", imply those who climbed that for
80. The man was called a hero because he
earlier did not pitch any camp. (a) expressed his willingness to risk his life to save
others.
77. They took a long time to finish the work because
(b) managed to escape from the fire
(a) they were very tired. (c) showed great courage in fighting the fire.
(b) there was not enough air to breathe (d) saved a life.
(c) it was very cold Ans. (d) The man was called a hero because he saved a
(d) it was very dark. life. He picked up a bundle to protect his face from the
Ans. (b) They took a long time to finish the work fire and immediately fell through the floor below
because at height there was not enough air to breath. where he managed to escape through a clear doorway.
Everything took five minutes as long as it would have The "bundle'' proved to be the baby of the Mayor's
taken in place where there was enough air to breath. wife.

IAS (Pre) GS IInd Paper (CSAT), 2012 35 YCT


UNION PUBLIC SERVICE COMMISSION
Civil Services (Preliminary Exam) - 2013
CSAT : PAPER-II
(Chapterwise Analysis with Explanation)
Time : 2 hours Maximum Number : 200
fragmented into smaller bits, the edges of these bits
COMPREHENSION come into contact with human activities resulting in
Directions for the following 2 (two) items : degradation of entire forest and continuity of forested
Read the following passage and answer the two landscape and corridor thus gets disrupted affecting
items that follow. Your answers to these items should several extinction prone species of wildlife.
be based on the passage only. Directions for the following 7 (seven) items :
Passage Read the following two passage and answer the
Ecological research over the last quarter of the items that follow each passage. Your answers to these
century has established the deleterious effects of habitat items should be based on the passages only.
fragmentation due to mining, highways and such other Passage -1
intrusions on forests. When a large block of forests gets The law in many parts of the world increasingly
fragmented into smaller bits, the edges of all these bits restricts the discharge of agricultural slurry into
come into contact with human activities resulting in the watercourses. The simplest and often the most
economically sound practice returns and material to the
degradation of the entire forests. Continuity of forested land as semisolid manure or as sprayed slurry. This
landscapes and corridors gets disrupted affecting several dilutes its concentration in the environment to what
extinction prone species of wildlife. Habitat might have occurred in a more primitive and sustainable
fragmentation is therefore considered as the most serious type of agriculture and converts pollutant into fertilizer.
threat to biodiversity conservation. Ad hoc grants of Soil microorganisms decompose the organic components
forest lands to mining companies coupled with rampant of sewage and slurry and most of the mineral nutrients
become available to be absorbed again by the vegetation.
illegal mining is aggravating this threat. The excess input of nutrients, both nitrogen and
1. What is the central focus of this passage phosphorus-based, from agricultural runoff (and human
(a) Illegal mining in forests sewage) has caused many ‘healthy’ oligotrophic lakes
(b) Extinction of wildlife (low nutrient concentrations. low plant productivity with
(c) Conservation of nature abundant water weeds and clear water) to change to
(d) Disruption of habitat eutrophic condition where high nutrient inputs lead to
high phytoplankton productivity (Sometimes dominated
Ans. (d) The central focus of this passage is disruption by bloom-forming toxic species). This makes the water
of habitat. The deleterious effects of habitat turbid, eliminates large plant and, in the worst situations,
fragmentation due to mining, highway and such other leads to anoxia and fish kills; so called cultural
intrusion forest. Continuity of forested landscapes and eutrophication. Thus, important ecosystem services are
lost, including the provisioning service of wild-caught
corridors gets disrupted affecting several extinction fish and the cultural services associated with recreation .
prone species of wildlife. Thus in present we can see the The process of cultural eutrophication of lakes has
disruption of habitat. been understood for some time. But only recently did
2. What is the purpose of maintaining the scientists notice huge’ dead zones’ in the oceans near
continuity of forested landscapes and corridors ? river outlets, particularly those draining large catchments
1. Preservation of biodiversity. areas such as the Mississippi in North America and the
2. Management of mineral resources. Yangtze in China. The nutrient enriched water flows
3. Grant of forest lands for human activities through streams, rivers and lakes, and eventually to the
estuary and ocean where the ecological impact may be
Select the correct answer using the codes given
huge, killing virtually all invertebrates and fish in areas
below : up to 70,000 km2 in extent. More than 150 sea areas
(a) 1 only (b) 1 and 2 worldwide are now regularly starved of oxygen as a
(c) 2 and 3 (d) 1, 2 and 3 result of decomposition of algal blooms, fuelled
Ans. (a) : The purpose of maintaining the continuity of particularly by nitrogen from agricultural runoff of
forested landscapes and corridors is preservation of fertilizers and sewage from large cities. Oceanic dead
biodiversity. The deleterious effects of habitat zones are typically associated with industrialized nations
and usually lie off countries that subsidize their
fragmentation due to mining, highway and such other agriculture, encouraging farmers to increase productivity
intrusion of forest when a larger block of forests gets and use more fertilizer.
IAS (Pre) GS IInd Paper (CSAT), 2013 36 YCT
3. According to the passage, why should the Ans. (b) : According to the passage, the indiscriminate
discharge of agricultural slurry into watercourses use of fertilizer can cause addition of pollutants to the
be restricted ? soil and water, nutrients enrichments of water bodies
1. Losing nutrients in this way is not a good practice and thus creation of algal blooms. The excess input of
economically nutrients both nitrogen and phosphorus-based from
2. Watercourses do not contain the microorganisms agricultural runoff has caused many healthy lakes to
that can decompose organic components of high phytoplankton. This makes the water turbid
agricultural slurry. eliminates large plants and in worst situation leads to
3. The discharge may lead to the eutrophication of anoxia and fish kill and thus disturbs aquatic ecosystem.
water bodies. 6.
What is /are the characteristics of a water body
Select the correct answer from the codes given with cultural eutrophication ?
below : 1. Loss of ecosystem service
(a) 1 only 2. Loss of flora and fauna
(b) 2 and 3 only 3. Loss of mineral nutrients
(c) 1 and 3 only Select the correct answer from the codes given
(d) 1,2 and 3 below :
Ans. (c) : According to the passage the discharge (a) 1 only
(b) 1 and 2 only
agricultural slurry into watercourses should be restricted
(c) 2 and 3
because the discharge may lead to the eutrophication of
(d) 1, 2 and 3
water bodies and losing nutrients in this way is not a
good practice economically and thus only statement (1) Ans. (b) : In cultural eutrophication, important
ecosystem service are lost, including the provisioning
and (3) are corrected.
service of wild-caught fish and the cultural services
4. The passage refers to the conversion of associated with recreation and thus it results in loss of
‘’pollutant to fertilizer’’. What is pollutant and flora and fauna and loss of ecosystem service and thus,
what is fertilizer in this context ? only statement (1) and (2) are correct.
(a) Decomposed organic component of slurry is 7. What is the central theme of this passage ?
pollutant and microorganisms in soil constituent (a) Appropriate legislation is essential to protect the
fertilizer. environment.
(b) Discharged agricultural slurry is pollutant and (b) Modern agriculture is responsible for the
decomposed slurry in soil is fertilizer. destruction of environment.
(c) Sprayed slurry is pollutant and watercourse is (c) Improper waste disposal from agriculture can
fertilizer. destroy the aquatic ecosystems.
(d) None of the above expressions is correct in this (d) Use of chemical fertilizers is undesirable in
context. agriculture.
Ans. (b) : According to the passage, "Discharged Ans. (c) : The central theme of the passage is to
agricultural slurry is pollutant and decomposed slurry in maintain cleanliness of water and to balance aquatic
soil is fertilizer. Soil microorganisms decompose the ecosystem by using proper fertilizers and thus improper
organic components of sewage and slurry and most of waste disposal from agriculture can destroy the aquatic
mineral nutrients become available to be absorbed again ecosystems.
by the vegetation." Passage-2
5. According to the passage, what are the effects of The miseries of the world cannot be cured by
indiscriminate use of fertilizers ? physical help only. Until man’s nature changes, his
1. Addition of pollutants to the soil and water physical needs will always arise, and miseries will
2. Destruction of decomposer microorganisms in always be felt, and no amount of physical help will
soil. remove them completely. The only solution of the
3. Nutrient enrichment of water bodies. problem is to make making pure. Ignorance is the mother
4. Creation of algal blooms. of evil and of all the misery we see. Let men have light,
Select the correct answer from the codes given let them be pure and spiritually strong and educated;
below-
them alone will misery cease in the world . We may
(a) 1, 2 and 3 only
(b) 1, 3 and 4 convert every house in the country into a charitable
(c) 2 and 4 asylum, but human misery will continue until man’s
(d) 1, 2, 3 and 4 character changes.

IAS (Pre) GS IInd Paper (CSAT), 2013 37 YCT


8. According to the passage, which of the following have to take an interest in the history of people-
statements is most likely to be true as the reason participation and pubic reasoning in different parts of the
for man’s miseries ? world. We have to look beyond thinking of democracy
(a) The poor economic and social conditions only in terms of European and American evolution . We
prevailing in society would fail to understand the pervasive demands for
(b) The refusal on the part of man to change his
participatory living, on which Aristotle spoke with far-
character.
(c) The absence of physical and material help from reaching insight, if we take democracy to be a kind of a
his society specialized cultural product of the West.
(d) Ever increasing physical needs due to changing It cannot, of course be doubted that the institutional
social structure. structure of the contemporary practice of democracy is
Ans. (b) : According to the passage, the miseries of the largely the product of European and American
world cannot be cured by physical help only untils man experience over the last few countries. This is extremely
nature changes, his physical needs will always arise and important to recognize since these developments in
miseries will always be felt. Thus option (b) is correct. institutional formats were immensely innovative and
9. With reference to the passage, the following ultimately effective. There can be little doubt that there is
assumptions have been made : a major ‘Western’ achievement here.
1. The author gives primary importance to physical 10. Which of the following is closest to the view of
and material help in eradicating human misery. democracy as mentioned in the above passage ?
2. Charitable homes, hospitals, etc. can remove (a) The subject of democracy is a muddle due to a
human misery to a great extent.
desire to portray it as a Western concept, ‘alien’
Which of the assumptions is/are valid ?
(a) 1 only to non-Western countries.
(b) 2 only (b) The language of imposition of democracy is
(c) Both 1 and 2 inappropriate. There is, however, a need to
(d) Neither 1 nor 2 consider this concept in the backdrop of culture
Ans. (d) : According to the passage, the author gives of ‘own ways’ of non-Western society.
primary importance to man's character while eradicating (c) While democracy is not essentially a Western
human misery thus, statement (1) is not correct. Only idea belonging exclusively to the West, the
Humans character can remove human misery to greater institutional structure of current democratic
extent. Charitable homes, hospitals etc can not remove practices has been their contribution.
human misery. Thus both the assumptions made are (d) None of the statements (a), (b) and (c) given
wrong. above is correct
Directions for the following 8 (eight) items :
Ans. (c) : According to the passage, while democracy is
Read the following four passages and answer the
items that follow each passage. Your answers to these not essentially a western idea belonging exclusively to
items should be based on the passages only. the West. The institutional structure of current
Passage-1 democratic practices has been their contribution. It
The subject of democracy has become severely cannot, of course be doubted that the institutional
muddled because of the way the rhetoric surrounding it structure of the contemporary practice of democracy is
has been used in recent years. There is, increasingly, an largely the product of European and American
oddly confused dichotomy between those who want to experience over the last few countries.
‘impose’ democracy on countries in the non-Western
world (in these countries’ ‘own interest’, of course) and 11. With reference to the passage, the following
those who are opposed to such ‘imposition’ (because of assumption have been made :
the respect for the countries’, own ways’). But the entire 1. Many of the non-Western countries are unable to
language of ‘imposition’, used by both sides, is have democracy because they take democracy to
extraordinarily inappropriate since it makes in implicit be a specialized cultural product of the West.
assumption that democracy belongs exclusively to the 2. Western countries are always trying, to impose
West, taking it to be quintessentially’ Western’ idea democracy on non-Western countries.
which has originated and flourished only in the West.
Which of the above is/are valid assumption/
But the thesis and the pessimism it generates about
assumptions ?
the possibility of democratic practice in the world would
(a) 1 only
be extremely hard to justify. There were several
(b) 2 only
experiments in local democracy in ancient India. Indeed,
(c) Both 1 and 2
in understanding the roots of democracy in the world, we
(d) Neither 1 nor 2
IAS (Pre) GS IInd Paper (CSAT), 2013 38 YCT
Ans. (d) : According to the passage, western countries 13. According to the passage, which of the following
are not always trying to impose democracy on non- is/are the major benefit/benefits of good
western countries implicit assumption that democracy' corporate governance ?
belongs to exclusively to the west, taking it to be 1. Good corporate governance leads to increase in
quintessentially western idea which has originated and share price of the company.
flourished only in the west. 2. A company with good corporate governance
always increases its business turnover rapidly.
Passage -2 3. Good corporate governance is the main criterion
Corporate governance is based on principles such as for foreign institutional investors when they
conducting the business with all integrity and fairness, decide to buy a company.
being transparent with regard to all transactions, making Select the correct answer using the codes given
below :
all the necessary disclosures and decisions, complying
(a) 1 only
with all the laws of the land, accountability and
(b) 2 and 3 only
responsibility towards the stakeholders and commitment (c) 1 and 3 only
to conducting business in an ethical manner. Another (d) 1, 2 and 3
point which is highlighted on corporate governance is the
Ans. (a) : According to the passage corporate
need for those in control to be able to distinguish
governance is known to be one of the criteria that
between what are personal and corporate funds while foreign institutional investor are increasingly depending
managing a company. on when deciding on which companies to invest in It
influence on share price of the company. Thus only
Fundamentally, there is a level of confidence that is
associated with a company that is known to have good statement (1) is correct.
corporate governance. The presence of an active group of Passage -3
independent directors on the board contributes a great
deal towards ensuring confidence in the market. Malnutrition most commonly occurs between the ages
Corporate governance is known to be one of the criteria of six months and two years . This happens despite
that foreign institutional investors are increasingly the child’s food requirements being less than that of
depending on when deciding on which companies to an older child . Malnutrition is often attributed to
invest in. It is also known to have a positive influence onpoverty, but it has been found that even in households
the share price of the company. Having a clean image on where adults eat adequate quantities of food. more
the corporate governance front could also make it easier than 50 percent of children-under –five do not
for companies to source capital at more reasonable costs. consume enough food. The child’s dependence on
Unfortunately, corporate governance often becomes the someone else to feed him/her is primarily responsible
centre of discussion only after the exposure of a large for the malnutrition. Very often the mother is
scam. working and the responsibility of feeding the young
child is left to an older sibling. It is therefore crucial
12. According to the passage, which of the following
to increase awareness regarding the child’s food
should be the practice/practices in good needs and how to satisfy them.
corporate governance ?
1. Companies should always comply with labour 14. According to the passage, malnutrition in
and tax laws of the land. children can be reduced.
2. Every company in the country should have a (a) if the children have regular intake of food.
government representative as one of the (b) after they cross the age of five.
(c) if the food needs of younger children are known.
independent directors on the board to ensure
(d) if the responsibility of feeding younger children
transparency.
is given to adults.
3. The manager of a company should never invest
Ans. (c) : According to the passage malnutrition in
his personal funds in the company.
children can be reduced if the food needs of younger
Select the correct answer answer using the codes children are known
given below :
(a) only 1 (b) 2 and 3 only 15. According to the author, poverty is not the main
(c) 1 and 3 only (d) 1, 2 and 3 cause of malnutrition, but the fact that
1. taking care of younger one is not a priority for
Ans. (a) : Corporate governance is based on principles
working mothers.
such as conducting the business with all integrity and 2. awareness of nutritional needs is not propagated
fairness, being transparent with regard to all transaction by the public Health authorities.
making all the necessary disclosures and decisions, Select the correct answer using the codes given
complying with all the laws of land. below :
IAS (Pre) GS IInd Paper (CSAT), 2013 39 YCT
(a) 1 only 17. Which of the following observations emerges
(b) 2 only from the above passage ?
(c) Both 1 and 2 (a) One can identify a single policy that can reduce
(d) Neither 1 nor 2 risk without any side-effect.
Ans. (b) : According to the author poverty is not the (b) No single risk-specific policy is sufficient to
main cause of malnutrition but the fact that is awareness reduce agricultural risk.
of nutritional needs is not propagated by the public (c) Policies which affect risk indirectly can eliminate
health authorities.
it.
Passage – 4 (d) Government’s policy intervention can mitigate
agricultural risk completely
A number of empirical studies find that the farmers
are risk-averse, though only moderately in many cases. Ans. (b) : The observation can be made from above
There is also evidence to show that farmers’ risk passage is that there is no single risk specific policy is
aversion results in cropping patterns and input use sufficient to reduce agricultural risk and is without side-
designed to reduce risk rather than to maximize income. effect.
Farmers adopt a number of strategies to manage and Directions for the following 4 (four) items :
cope with agricultural risks. These include practices like Read the following passage and answer the four
crop and field diversification, non-farm employment, items that follow. Your answers to these items should
storage of stocks and strategic migration of family be based on the passage only.
members. There are also institutions ranging from share
tenancy to kinship, extended family and informal credit Passage
agencies. One major obstacle to risk sharing by farmers Financial markets in India have acquired greater
is that the same type of risks can affect the large number depth and liquidity over the years. Steady reforms since
of farmers in the region. Empirical studies show that the 1991 have led to growing linkages and integration of the
traditional methods are not adequate. Hence there is a Indian economy and its financial system with the global
need for policy interventions, especially measures that economy. Weak global economic prospects and
cut across geographical regions. continuing uncertainties in the international financial
Policies may aim at tackling agricultural risk markets therefore, have had their impact on the emerging
directly or Indirectly. Examples or risk-specific policies market economics. Sovereign risk concerns, particularly
are crop insurance, price stabilization and the in the Euro area, affected financial markets for the
development of varieties resistant to pests and diseases. greater part of the year, with the contagion of Greece’s
Policies which affect risk indirectly are irrigation, sovereign debt problem spreading to India and other
subsidized credit and access to information. No single economies by way of higher –than –normal levels of
risk-specific policy is sufficient to reduce risk and is volatility
without side-effects, whereas policies not specific to risk
influence the general situation and affect risk only The funding constraints in international financial
indirectly . Crop insurances, as a policy measure to markets could impact both the availability and cost of
tackle agricultural risk directly, deserves careful foreign funding for banks and corporate. Since the Indian
consideration in the Indian context and in many other financial system is bank dominated, banks’ ability to
developing countries-because the majority of farmers withstand stress is critical to overall financial stability.
depend on rain-red agriculture and in many areas yield Indian banks, however, remain robust, notwithstanding a
variability is the predominant cause of their income
decline in capital to risk-weighted assets ratio and a rise
instability.
in non-performing asset levels in the recent past. Capital
16. The need for policy intervention to mitigate risks adequacy levels remain above the regulatory
in agriculture is because requirements. The financial market infrastructure
(a) farmers are extremely risk-averse.
(b) farmers do no know how to mitigate risks. continues to function without any major disruption. With
(c) the methods adopted by farmers and existing risk further globalization, consolidation, deregulation, and
sharing institutions are not adequate. diversification of the financial system, the banking
(d) majority of farmers depend on rain–fed business may become more complex and riskier. Issues
agriculture . like risk and liquidity management and enhancing skill
Ans. (c) : According to the passage, the need for policy therefore assume greater significance.
intervention to mitigate risk in agriculture because of
the method adopted by farmers and is risk sharing 18. According to the passage, the financial markets
institution's are not adequate. No single risk specific in the emerging market economies including
policy is sufficient to reduce risk and is without side- India had the adverse impact in recent years due
effect. to
IAS (Pre) GS IInd Paper (CSAT), 2013 40 YCT
1. weak global economic prospects. Ans. (a) : According to the passage, with further
2. uncertainties in the international financial markets. globalization, consolidation, deregulation and
3. sovereign risk concerns in the Euro area. diversification of the financial system, the banking
4. bad monsoons and the resultant crop loss. business may became more complex and riskier. Issues
Select the correct answer using the codes given like risk and liquidity management and enhancing skill
therefore assume greater significance. Thus option (a) is
below . correct.
(a) 1 and 2 only (b) 1, 2 and 3
Directions for the following 2 (two) items :
(c) 2 and 3 only (d) 2, 3 and 4
Read the following passage and answer the two
Ans. (b) : According to the passage, the financial items that follow. Your answers to these items should
markets in the emerging market economies including be based on the passage only.
India had the adverse impact in recent years due to Passage
weak global economic prospects and continuing Crude mineral oil comes out of the earth as a thick
uncertainties in the international financial market and brown or black liquid with a strong smell. It is a complex
also sovereign risk concern in the Euro Area have had mixture of many different substances, each with its own
their negative impact on the emerging market individual qualities. Most of the are combinations of
economics. hydrogen and carbon in varying proportions. Such
hydrocarbons are also found in other forms such as
19. The Indian financial markets are affected by bitumen, asphalt and natural gas. Mineral oil originates
global changes mainly due to the from the carcasses of tiny animals and from plants that
(a) increased inflow of remittances from abroad. live in the sea. Over millions of years, these dead
(b) enormous increase in the foreign exchange creatures from large deposits under the sea-bed; and
reserves. ocean currents cover them with a blanket of sand and silt.
(c) growing global linkages and integration of the As this mineral hardens, it becomes sedimentary rock
and effectively shuts out the oxygen, so preventing the
Indian financial markets. complete decomposition of the marine deposits
(d) contagion of Greece’s sovereign dept problem. underneath. The layers of sedimentary rock become
Ans. (c) : The Indian financial markets are affected by thicker and heavier. Their pressure produces heat, which
global changes mainly due to the growing global transfers the tiny carcasses into crude oil in a process that
linkages and integration of the Indian financial markets. is still going on today.

20. According to the passage, in the Indian financial 22. Mineral oil deposits under the sea do not get
system, banks’ ability to withstand stress is completely decomposed because they
critical to ensure overall financial stability (a) are constantly washed by the ocean currents.
(b) become rock and prevent oxygen from entering
because Indian financial system is
them.
(a) controlled by the Government of India. (c) contain a mixture of hydrogen and carbon.
(b) less integrated with banks. (d) are carcasses of organisms lying in saline
(c) controlled by Reserve Bank of India. conditions.
(d) dominated by banks. Ans. (b) : According to the passage mineral oil deposits
Ans. (d) : According to the passage, in the Indian under the sea do not completely decomposed because as
financial systems banks ability to withstand stress is mineral hardens they became sedimentary rocks and
effectively shuts out the oxygen, so preventing the
critical to ensure overall financial stability because
complete decomposition of the marine deposits
Indian financial system is bank dominated system. underneath.
21. Risk and liquidity management assumes more 23. Sedimentary rock leads to the formation of oil
importance in the Indian banking system in deposits because
future due to (a) There are not saline conditions below it.
1. further globalization. (b) It allows some dissolved oxygen to enter the
2. more consolidation and deregulation of the dead organic matter below it.
financial system. (c) Weight of overlying sediment layers causes the
3. further diversification of the financial system. production of heat.
4. more financial inclusion in the economy. (d) It contains the substances that catalyze the
Select the correct answer using the codes given chemical reactions required to change dead
below : organism into oil.
(a) 1, 2 and 3 Ans. (c) : Sedimentary rock leads to the formation of
oil deposits because the layers of sedimentary rock
(b) 2, 3 and 4
became thicker and heavier. Their pressure produces
(c) 1 and 2 only heat, which transfers the tiny carcasses into crude oil
(d) 3 and 4 only in a process that is till going on today.
IAS (Pre) GS IInd Paper (CSAT), 2013 41 YCT
REASONING AND ANALYTICAL Ans. (b) The route, when a person visits each of the
places starting from P and gets back to P-
EDIBILITY
24. Consider the following statements : Hence, He will definitely go twice to the place R.
(i) A primary group is relatively smaller in size 27. Which one of the following pairs of cities is
(ii) Intimacy is an essential characteristic of a connected by any of the routes directly without
primary group. going to any other city ?
(iii) A family may be an example of a primary (a) P and T (b) T and S
group. (c) Q and S (Q and R) (d) None of these
In the light of the above statements, which one of Ans. (d) Option (A) : From P to T
the following is true ? Option (B): From T to S
(a) All families are primary groups.
(b) All primary groups are families Option (C): From Q to S/from Q to R
(c) A group of smaller size is always a primary
group. None of these pairs of cities in connected by any of the
(d) Members of a primary group known each other routes directly without going to any other city.
intimately. 28. Between which two cities among the pairs of
Ans. (d) As intimacy is an essential characteristic of a cities given below are there maximum travel
primary group. As members of a primary group knows options available?
each other intimately. (a) Q and S (b) P and R
(c) P and T (d) Q and R
Direction for the following 4 (four) items :
Ans. (a)
Read the following statements and answer the
four items that follow :
Five cities P, Q, R, S and T are connected by
different modes of transport as follows :
P and Q connected by boat as well as rail. Maximum travel options are available between Q and S
S and R are connected by bus and boat. cities.
Q and T are connected by air only. 29. The music director of a film wants to select four
P and R are connected by boat only. persons to work on different aspects of the
composition of a place of music. Seven persons
T and R are connected by rail and bus.
are available for this work; they are Rohit,
Boat/
P ← Rail →Q Tanya, Shobha, Kaushal, Kunal, Mukesh and
Jaswant.
Bus/Boat → R
S ← Rohit and Tanya will not work together
Air → T Kunal and Shobha will not work together
Q ← Mukesh and Kunal want to work together
Boat Which of the following is the most acceptable group
P ← →R
of people that can be selected by the music director?
Rail/Bus R
T ←→ (a) Rohit, Shobha, Kunal and Kaushal
(b) Tanya, Kaushal, Shobha and Rohit
25. Which mode of transport would help one to reach (c) Tanya, Mukesh, Kunal and Jaswant
R starting from Q, but without changing the (d) Shobha, Tanya, Rohit and Mukesh
mode of transport.
(a) Boat (b) Rail Ans. (c) The most acceptable group of people = Tanya,
(c) Bus (d) Air Mukesh, Kunal and Jaswant.
Ans. (a) To reach R starting from Q without changing Directions for the following 3 (three) items :
the mode of transport- Examine carefully the following statements and
answer the three items that follow :
Hence, the mode of transport = Boat. Out of four friends A,B,C and D .
26. If, a person visits each of the places starting from A and B play football and cricket.
and P and gets back to P, which of the following B and C play cricket and hockey
places must he visit twice ?
(a) Q (b) R A and D play basketball and football
(c) S (d) T C and D play hockey and basketball
IAS (Pre) GS IInd Paper (CSAT), 2013 42 YCT
Solution for Question (31 to 33) Ans. (a or c) : According to the situation, we will call
Name Football Cricket Hocke Basketball personal secretary over to our room and explain him
y personally that these are some sets of rule made for
allotment and it has been made public and to avoid
A √ √ × √ favoritism and biased nature this allotment can not be
B √ √ √ × done. Thus option (a) or (c) are correct.
C √ √ √ 34. While traveling in a Delhi-registered commercial
taxi from Delhi to an adjacent city (another
D √ × √ √ State), your taxi driver informs you that as he has
30. Who does not play hockey ? no permit for running the taxi in that city, he will
(a) D (b) C stop at its Transport Office and pay the
(c) B (d) A prescribed fee of Rs. forty for a day. While
Ans. (d) 'A' does not play hockey. paying the fee at the counter you find that the
transport clerk is taking an extra fifty rupees for
31. Who plays football, basketball and hockey ? which no receipt is being given. You are in a
(a) D (b) C hurry for you meeting. In such circumstances,
(c) B (d) A what would you do?
(a) Go up to the counter and ask the clerk to give
Ans. (a) 'D' plays football, basketball and hockey
back the money which he has illegally taken.
32. Which game do B, C and D play ? (b) Do not interfer at all as this is a matter between
(a) Basketball the taxi driver and the tax authorities.
(b) hockey (c) Take note of the incident and subsequently
(c) Cricket report the matter to the concerned authorities.
(d) Football (d) Treat it as a normal affair and simply forget
Ans. (b) B, C and D play Hockey, in common. about it.
Ans. (a or c) : According to the given situation, will
DECISION MAKING ABILITY take note of the incident and subsequently report the
matter to the concerned authorities and will try that in
Directions for the following 6 (six) items : future this type of incident should not happen again and
Given below are six items. Each item describes a other suitable alternative will be to go up to the counter
situation and is followed by four possible responses. and ask the clerk to give back the money which he has
Indicate the response you find most appropriate. illegally taken.
Choose only one response for each item. The
35. A person lives in a far off village which is almost
responses will be evaluated based on the level of
appropriateness for the given situation. two hours by bus. The villager’s neighbour is a
Please attempt all the items. There is no penalty very powerful landlord who is trying to occupy
for wrong answer for the six items. the poor villager’s land by force. You are the
33. You are the head of your office. There are certain District Magistrate and busy in a meeting called
houses reserved for the allotment to the office by a local Minister. The villager has come all the
staff and you have been given the discretion to do way, by bus and on foot, to see you and given an
so. A set of rules for the allotment of the houses application seeking protection from the powerful
has been laid down by you and has been made landlord. The villager keeps on waiting outside
public. Your personal secretary, who is very close
the meeting hall for an hour, You came out of the
to you, comes to you and pleads that as his father
meeting and are rushing to another meeting. The
is seriously ill, he should be given priority in
allotment of a house. The office secretariat that villager follows you to submit his application.
examined the request as per the rules runs down What would you do ?
the request and recommends the procedure to be (a) Tell him to wait for another two hours till you
followed according to the rules . You do not want come back from your next meeting .
to annoy your personal secretary. In such (b) Tell him that the matter is actually to be dealt by
circumstances, what would you do ? a Junior Officer and that he should give the
(a) Call him over to your room and personally
application to him.
explain why the allotment cannot be done.
(b) Allot the house to him to win his loyalty. (c) Call one of your senior subordinate officers and
(c) Agree with the office note to show that you are ask him to solve the villager’s problem.
not biased and that you do not indulge in (d) Quickly take the application from him, ask him
favoritism. a few relevant questions regarding his problem
(d) Keep the file with you and not pass any orders. and then proceed to the meeting.
IAS (Pre) GS IInd Paper (CSAT), 2013 43 YCT
Ans. (c or d) : According to the given situation, the Ans. (a & c) : According to the situation, the best
suitable answer will be calling one of our senior response will be by logically explaining to the residents
subordinate officer's and assign him the problem and the needs for family planning to improve the health and
ask him to solve the villager's problem as soon as living standard and also the benefits and happiness that
possible and other suitable response will be quickly take will come because of small family and also sometime to
the application from him, ask him few relevant attract people start offering incentive for adopting
questions regarding problem and then proceed to the family planning devices. Thus, option (a) and (c) are
meeting. Thus option (c) or (d) can be correct. correct.
36. There is a shortage of sugar in your District 38. You are the teacher in a University and are
where you are the District Magistrate. The setting a question paper on a particular subject.
Government has ordered that only a maximum One of your colleague, whose son is preparing for
amount of 30 kg sugar is to be released for the examination on the subject, comes to you and
wedding celebrations. A son of your close friend informs you that it is his son’s last chance to pass
is getting married and your friend requests you that examination and whether you could help
to release at least 50 kg sugar for his son’s him by indication what questions are going to be
wedding. He expresses annoyance when you tell in the examination. In the past, your colleague
him about the Government’s restrictions on this had helped you in another matter. Your colleague
matter. He feels that since you are the District informs you that his son will suffer from
Magistrate you can release any amount. You do depression if the fails in this examination. In such
not want to spoil your friendship with him. In circumstances, what would you do?
such circumstances, how would you deal with the (a) In view of the help given you, extend your help
situation? to him.
(a) Release the extra amount of sugar which your (b) Regret that you cannot be of any help to him.
friend has requested for. (c) Explain to your colleague that this would be
(b) Refuse your friend the extra amount and strictly violating the trust of the University authorities
follow the rules . and you are not in a position to help him.
(d) Report the conduct of your colleague to the
(c) Show your friend the copy of the Government
higher authorities.
instructions and then persuade him to accept the
lower amount as prescribed in the rules. Ans. (b & c) : According to the situation, we need to
explain colleague that by indicating what the questions
(d) Advise him to directly apply to the allotting
are going to be in examination will be violating the trust
authority and inform him that you do not of the university authority and we are not in position to
interfere in this matter. help you and also explain that it will demoralise
Ans. (b & c) : According to the situation the suitable talented students. Thus, option (b) & (c) can be correct.
response will be refusing our friend the extra amount
and strictly follow the rules and tell him about the GENERAL MENTAL APTITUDE
government instruction that maximum of 30kg can be 39. Consider the following figures 1, 2,3 and 4 :
released and then persuade him to accept the lower
amount as it prescribed in the rules. Thus, option (b)
and (c) are correct.
37. You are in-charge of implementing The Family
Planning programmed in an area where there is a
strong opposition to the present policy. You want
In the figures from 1 to 4 above, two symbols are
to convince the residents of the need for keeping
shown to change their position in a regular
small families. What would be the best way the
direction. Following the same sequence, which
communicating this message?
one of the following will appear at the fifth stage?
(a) By logically explaining to the residents the need
for family planning to improve the health and
living standards.
(a) (b)
(b) By encouraging late marriages and proper
spacing of children.
(c) By offering incentives for adopting family
planning devices.
(d) By asking people who have been sterilized or
are using contraceptives to directly talk to the (c) (d)
residents.
IAS (Pre) GS IInd Paper (CSAT), 2013 44 YCT
Ans. (b) In the Given figure shown-
The triangle is moving down.
The semi-circle is moving in antilock wise direction
and only covering the square areas.
So, the figure to fifth stage will be option 2, as shown
above.
Hence, option 2, is the correct answer.
Directions for the following 2 (two) items –In
each item, there are two sets of figures, first four
figures named Problem figures and next four figures
named Answer figures indicated as (a), (b), (c) and
(d). The problem figures follow a particular sequence, (a) diagram I (b) diagram II
In accordance with the same, which one of the four (c) diagram III (d) diagram IV
answer figures should appear as the fifth figure? Ans. (d) Time taken to finish a work is inversely
40. Problem figures proportional to the number of persons working at it.
∴ More persons then less is the time taken to finish a
job. Hence, diagram IV represents the relation between
X and Y.

Answer figures

Ans. (c) In each step arrow [↑] symbol is inverting


180º and cross circle [⊕] symbol in each step touches
one vertex of triangle in clock wise direction. 43. Consider the following matrix ?
Hence, option (c) is the correct answer. 3 370 7
2 224 6
41. Problem figures
1 730 X
What is the number at ‘X’ in the above matrix ?
(a) 5 (b) 8
(c) 9 (d) 11
Ans. (c) Just as, 33 + 73 = 27+ 343 = 370,
23 + 63 = 8 + 216 = 224
Answer figures Similarly 13 + X3 = 730 ⇒ X3 = 730 – 1 ⇒ X3 = 729
⇒ x = (729)1/3 = (93)1/3 = 9
44. Examine the following three figures in which the
number follow a specific pattern :

The missing number (?) in the third figure above is


nd rd (a) 7 (b) 16
Ans. (a) From figure (1) to (2), 2 and 3 design (c) 21 (d) 28
makes a pair and comes forward while 1st design gets 14
Ans. (b) Just as 1 : 12 × = 12 × 7 = 84
180º inverted. Same from figure (2) and (3) and figure 2
(3) to (4). 18
2:9× = 9 × 9 = 81
Hence, option (b) is correct answer. 2
?
42. Consider the following diagrams : Similarly 11 × = 88
2
x men, working at constant speed, do a certain
88 × 2
job in y days. Which one the these diagrams ∴?= = 8 × 2 = 16
11
shows the relation between x and y?
IAS (Pre) GS IInd Paper (CSAT), 2013 45 YCT
45. Consider the following figures : Given that,
Condition (I)-
Male Females
A W
B X
C Y
Z
Condition (II)-

Which one of the following figures would


logically come in the 7th position indicated above Condition (III)- At least 2 males will be in the team.
by a question mark ?
46. If B is selected and Y is rejected, the team will
consist of which one of the following groups ?
(a) A, B, C and W
(b) A, B, C and Z
(c) A, B, C and X
(d) A, W, Y and Z
Ans. (c) If B is selected and y is rejected, then the team
will be:-
A, B, C X or A, B, X, Z
Ans. (d) At first sight 47. If all the three males are selected, then how many
combinations of four member teams are possible?
∵Odd numbered figures have circle and even (a) 1 (b) 2
numbered figures have squares. (c) 3 (d) 4
∴ Seven numbered figure will have a circle. Ans. (b) If all the three males are selected, Then the
At second sight, possible combinations:-
Similarly the sign of the arrow of the first figure (A, B, C, X), (A, B, C, Y)
th
= The sign of arrow of the 4 figure Hence, the number of possible combinations is 2.
th 48. If Y is selected and B is rejected, the team will
= The sign of arrow of 7 figure
consist of which one the following groups ?
Just as, The sign of the arrow of the second figure
(a) A, C, W and Y
= The sign of the arrow of 5th figure (b) A, C, X and Y
The sign of the arrow of the IIIrd figure (c) A, C, Y and Z
= The sign of the arrow of 6th figure (d) A, W, Y and Z
Ans. (b) If y is selected and B is rejected then the team
Direction for the following 3 (three) items :
will be:-
Read the following passage and answer the three
(A, C, X, Y)
items that follow :
A tennis coach is trying to put together a team of 49. Five people A,B,C,D and E are seated about a
round table. Every chair is spaced equidistant
four players for the forthcoming tournament. For this 7
from adjacent chairs.
players are available :
(i) C is seated next to A.
males A, B and C; and females W, X, Y and Z. All
players have equal capability and at least 2 males will be (ii) A is seated two seats from D.
there in the team . For a team of four, all players must be (iii) B is not seated next to A.
able to play with each other. But, B cannot play with W, Which of the following must be true ?
C cannot play with Z and W cannot play with Y. (I) D is seated next to B.
Solution for (46 to 48):- (II) E is seated next to A
nd
IAS (Pre) GS II Paper (CSAT), 2013 46 YCT
Select the correct answer using the codes given Each one of them comes from a different city,
below : Further it is given that
(a) 1 only 1. B and C do not belong to Q.
(b) II only 2. B and E do not belong to P and R.
(c) Both I and II 3. A and C do not belong to R, S and T.
(d) Neither I nor II 4. D and E do not belong to Q and T .
Which one of the following statements is not
Ans. (c) From the given conditions two cases are correct?
possible as shown below : (a) C belongs to P
(b) D belongs to R
(c) A belongs to Q
(d) B belongs to S
Ans. (d)

Hence, In any case, both the conditions are true.


50. Geeta is older than her cousin Meena. Meena’s
brother Bipin is older than Geeta. When Meena
and Bipin visit Geeta, they like to play chess.
Meena wins the game more often than Geeta . According to the chart, 'B' belongs to 'T'
Based on the above information, four
Hence,
conclusions, as given below, have been made.
Which one these logically follow from the The statement "B belongs to S" is not correct.
information given above? 53. Seven men, A, B, C, D, E, F and G are standing in
(a) While playing chess with Geeta and Meena, a queue in that order. Each one is wearing a cap
Bipin often loses. of a different colour like violet, indigo, blue,
(b) Geeta is the oldest among the three. green, yellow orange and red. D is able to see in
(c) Geeta hates to lose the game . front of him green and blue, but not violet. E can
(d) Meena is the youngest of the three . see violet and yellow, but not red. G can see caps
Ans. (d) According to age, of all colors other than orange. If E is wearing an
indigo coloured cap, then the colour of the cap
Bipin > Geeta > Meena
worn by F is
Hence, Meena is the youngest of the three. (a) blue (b) Violet
51. There are five hobby clubs in a college viz., (c) Red (d) Orange
photography, yachting, chess, electronics and Ans. (c) According to order:-
gardening. The gardening group meets every A → Yellow
second day, the electronics group meets every B → Blue
third day, the chess group meets every fourth C → Green
day, the yachting group meets every fifth day and D → Violet
the photography group meets every sixth day.
E → Indigo
How many times do all the five groups meet on
the same day within 180 days ? F → Red
(a) 3 (b) 5 G → Orange
(c) 10 (d) 18 Hence,
Ans. (a) The L.C.M. of 2, 3, 4, 5 and 6 The colour of the cap worn by F is red.

= 60 54. There are some balls of red, green and yellow


colour lying on a table. There are as many red
Hence, ball as there are yellow balls. there are twice as
180 many yellow as there are green ones. The number
The required number of times = of red balls.
60
(a) is equal to the sum of yellow and green balls.
=3
(b) is double to the number of green balls.
52. A, B, C, D and E belong to five different cities P, (c) is equal to yellow balls minus green balls.
Q, R, S and T (not necessarily in that order). (d) cannot be ascertained.
IAS (Pre) GS IInd Paper (CSAT), 2013 47 YCT
Ans. (b) Let the number of green balls = x (a) Rs. 30 (b) Rs. 40
(c) Rs. 60 (d) Rs 80
∴The number of yellow balls = 2x
Ans. (b) Let the least value of the prize = x `
∵ The number of red balls = The number of yellow According to question, x + (x + 20) + (x + 40) + (x +
balls. 60) + (x + 80) + (x + 100) + (x + 120) = 700
7x + 420 = 700 ⇒ 7x = 700 – 420
∴ The number of red balls = 2x
⇒ 7x = 280
Hence, 280
∴x= = 40
The number of red balls is double to the number 7
of green balls. 59. Out of 120 applications for a post, 70 are mal and
80 have a driver’s license. What is the ratio
55. In a class of 45 students, a boy is ranked 20th. between the minimum to maximum number of
When two boys joined, his rank was dropped by males shaving driver’s license ?
one. What is his new rank from the end ? (a) 1 to 2 (b) 2 to 3
(a) 25th (b) 26th (c) 3 to 7 (d) 5 to 7
(c) 27th (d) 28th Ans. (c) According to question n (A ∪ B) = n(A) +
Ans. (c) Required Rank = (45 + 2) – (20 +1) + 1 n(B) – n (A∩B)
⇒ 120 = 70 + 80 – n(A ∩ B)
= 47 – 21 + 1 = 48 – 21 = 27th
⇒ n(A ∩ B) = 150 – 120 = 30
BASIC NUMERICAL ABILITY ∴ Required Ratio = 30 : 70 = 3 : 7

56. In a rare coin collection, there is one gold coin for 60. In a garrison, there was food for 1000 soldiers for
one month. After 10 days, 1000 more soldiers
every three non-gold coins. 10 more gold coins joined the garrison. How long would the soldiers
are added to the collection and the ratio of gold be able to carry on with the remaining food ?
coins to non-gold coins would be 1 : 2. Based on (a) 25 days (b) 20 days
the information, the total number of coins in the (c) 15 days (d) 10 days
collection now becomes. Ans. (d) According to question, M1D1 = M2D2
(a) 90 (b) 80 1000 × (30 – 10) = (1000 + 1000) × x
(c) 60 (d) 50 20,000
1000 × 20 = 2000 × x ∴x= = 10
Ans. (a) Let the number of gold coins = x 2000
and the number of non-gold coins = 3x 61. The tank-full petrol in Arun’s motor-cycle lasts
According to question, for 10 days. If he starts using 25% more every
x + 10 1 day, how many days will the tank-full petrol last?
= (a) 5 (b) 6
3x 2
(c) 7 (d) 8
2x + 20 = 3x
3x + 2x = 20 Ans. (d) Let the capacity of tank = 1000 lit
x = 20 According to question,
The total number of coins in the collection row ∵Petrol used in 10 day = 1000 lit
becomes = (x + 10) + 3x 1000
= (20 + 10) + 3 × 20 ∴Petrol used in one day =
10
= 30 + 60 = 90 = 100 lit
57. A gardener has 1000 plants. He wants to plant When Arun starts using 25% more every day,
them in such a way that he number of rows and then petrol used in one day = 125 lit
the number of columns remains the same. What Hence,
is the minimum number of plants that he needs 1000
The required number of days =
more for the this purpose ? 125
(a) 14 (b) 24 = 8 days.
(c) 32 (d) 34 62. A person can walk a certain distance and drive
Ans. (b) The number of required plants = (32)2 – 1000 back in six hours. He can also walk both ways in
1024 – 1000 = 24. 10 hours. How much time will he take to drive
58. A sum of Rs. 700 has to be used to give seven cash both way ?
prizes to the students of a school for their overall (a) Two hours
academic performance. If each Prize is Rs. 20 less (b) Two and a half hours
than its preceding prize, what is the least value of (c) Five and a half hours
the prize ? (d) Four hours
IAS (Pre) GS IInd Paper (CSAT), 2013 48 YCT
Ans. (a) Let the time taken to walk = x 325
The average speed of Car B = = 13km / hr
and the time taken to drive = y 25
According to question, 344
The average speed of car C =
x + y = 6 ______(i) 24
and x + x = 10 ⇒ 14.33 km.
2x = 10 302
x=5 The average speed of Car D =
22
Putting the value of x in eqn (i) = 13.72 km
∵x + y = 6 Hence Car A maintained the maximum average speed.
5+y=6 64. A cube has six number marked 1, 2, 3,4 , 5 and 6
y=1 on its faces. Three views of the cube are shown
time taken by the person to drive both way below
⇒ y + y = 2y
= 2 × 1 {∵y = 1}
= 2 hr
63. Four cars are hired at the rate of Rs. 6 per km. What possible numbers can exist on the two faces
and the cost of diesel at Rs. 40 a litre. In this marked (A) and (B), respectively on the cube ?
context, consider the details given in the following
table :
Car Mileage Hours Total
(Km/L) Payment
(Rs.) (a) 2 and 3 (b) 6 and 1
A 8 20 2120 (c) 1 and 4 (d) 3 and1
B 10 25 1950 Ans. (a) Considering three views of the cube,
opposite
C 9 24 2064 2  →6
D 11 22 1812 opposite
1  →5
Which car maintained the maximum average opposite
3  →4
speed ?
(a) Car A (b) Car B ∵ The opposite of 5 is 1.
(c) Car C (d) Car D ∴Neighbour of 5 will not be 1
Hence, 2 and 3 numbers can exist on the two faces
Ans. (a) ∵ four cars are hired at the rate of Rs. 6 per
marked (A) and (B), respectively on the cube.
km.
Directions for the following 5 (five) items –
2120
Distance covered by Car A = = 353.33 km. Study the two figures given below and answer the
6
five items that follow :
1950
Distance covered by Car B = = 325 km.
6
2064
Distance covered by car C = = 344 km.
6
1812
Distance covered by Car D = = 302 km.
6
totaldistance
∵ Average speed =
total time
353.33
The average speed of Car A = = 17.66 km/hr
20

IAS (Pre) GS IInd Paper (CSAT), 2013 49 YCT


Figure 1 : Number of Professors in selected 68. If the number of female Physical professors in
disciplines in a University by sex the age group 25-34 equals 25% of all the
Physical professors in the age group, then what is
the number of male Physics professors in the age
group 25-34 ?
(a) 9 (b) 6 (c) 3 (d) 2
Ans. (a) The number of all physical professors =
32 + 8
= 40
65. How many Physics professors belong to the age
The number of all physical professors which age group
group 35-44 ?
30
(a) 18 (b) 16 (25 – 34) = 40 ×
100
(c) 14 (d) 12
= 12
Ans. (b) The Number of Physics teacher The number of female physical professors which age
40 40
= (32 + 8) × = 40 × = 16 25
100 100 group (25 – 34) = 12 ×
100
66. Which one of the following disciplines has the =3
highest ratio of males to females ? Then, the number of male physical professors which
(a) Physics (b) Mathematics age group (25 – 34) = 12 – 3
(c) Chemistry (d) Economics =9
Ans. (a) The ratio of males to females, 69. If the Psychology professors in the University
In physics, constitute 2% of all the professors in the
M = F = 32 : 8 University, then what is the number of professors
4 in the University ?
=4:1⇒ =4 (a) 400 (b) 500
1
(c) 600 (d) 700
In mathematics,
Ans. (b) Let the number of professors in the university
M = F = 28 : 8
=x
7
=7:2⇒ = 3.5 According to question,
2
x × 2% = (6 + 4)
In chemistry,
2
M : F = 16 : 22 x× = 10
100
8
= 8 : 11 ⇒ = 0.72 x = 500
11
In economics, 70. Four friends, A, B, C and D distribute some
money among themselves in such a manner that
M : F = 24 : 8
A gets one less than B, C gets 5 more than D, D
3
=3:1⇒ =3 gets 3 more than B. Who gets the smallest
1
amount ?
67. What percentage of all Psychology professors are (a) A (b) B
females? (c) C (d) D
(a) 40% (b) 50% Ans. (a) Let A=x
(c) 60% (d) 70%
B = (x + 1)
6
Ans. (c) Required % = × 100% C = (x + 4) + 5 ⇒ (x + 9)
(6 + 4)

6 D = (x + 1) + 3
= ×100% = 60%
10 Hence, A gets the smallest amount.

IAS (Pre) GS IInd Paper (CSAT), 2013 50 YCT


71. A train travels at a certain average speed for a English Passage─1
distance of 63 km and then travels a distance of
Seven-year-old Jim came home from the park without
72 km at an average speed of 6 Km/hr more then
his new bicycle. "An old man and a little boy borrowed it,"
its original speed. If it takes 3 hours to complete he explained. "they are going to bring it back at four
the total journey, what is the original speed of the O'clock." His parents were upset that he had given his
train in km/hr ? expensive new bicycle, but were secretly proud of his
(a) 24 (b) 33 kindness and faith. Came four O'clock, no bicycle. The
(c) 42 (d) 66 parents were anxious. But at 4 : 30, the door bell rang, and
there stood a happy man and a boy, with the bicycle and a
Ans. (c) Let the original speed of the train = x km/hr
box of chocolates. Jim suddenly disappeared into his
63 72
According to the questions t1 + t2 = + ⇒ 3 = bedroom, and then came running out. "All right," he said.
x x+6
after examining the bicycle. "You can have your watch
63( x + 6) + 72 x back!"
x ( x + 6)
73. When Jim came home without his bicycle, his
x(x+6) = 21(x + 6) + 24x ⇒ x2 + 6x = 21x + 126 + 24x parents -
⇒ x2 + 6x – 45x – 126 = 0 ⇒ x2 – 39x – 126 = 0 (a) were angry with him
(b) were worried
⇒ x2 – 42x + 3x – 126 = 0 ⇒ x(x – 42) + 3(x – 42) = 0
(c) did not feel concerned
(x – 42) (x + 3) = 0 (d) were eager to meet the old man and the little boy
∴ x = 42 and x = - 3 not possible Ans. (b) : According to the passage, when Jim came
Hence, the original speed of the train 42 km/hr home without his bicycle which was given by his
parents, they became more worried because that was
72. A thief running at 8km/hr. is chased by a expensive new bicycle.
policeman whose speed is 10km/hr. If the thief is
74. Jim returned the watch to the old man and the
100m ahead of the policeman, then the time little boy because
required for the policeman to catch the thief will be (a) they gave him chocolates
(a) 2 min (b) 3 min (b) his father was proud of him
(c) 4 min (d) 6 min (c) he was satisfied with the condition of his bicycle
100 1000 (d) they were late only by 30 minutes
Ans. (b) Required time = ×
(10 − 8 ) 60 Ans. (c) : According to the passage, at 4: 30 the door
100 × 60 bell rang and these stood a happy man and a boy with
=
2 × 1000 the bicycle. After examine the bicycle he said "All
= 3 min right" he was satisfied with the condition of his bicycle.

ENGLISH LANGUAGE/GRAMMAR English Passage─2


It was already late when we set out for the next
& COMPREHENSION town, which according to the map was about fifteen
Direction for the following 8 (eight), items – The kilometres away on the other side of the hills. There we
following eight items (items 73 to 80) are based on felt that we would find a bed for the night. Darkness fell
three passages in English to test the comprehension of soon after we left the village, but luckily we met no one
as we drove swiftly we met no one as we drove swiftly
English language and therefore these items do not
along the narrow winding road that led to the hills. As
have Hindi version.
we climbed higher, it became colder and rain began to
Read each passage the answer the items that fall, making it difficult at times to see the road. I asked
follows. John, my companion, to drive more slowly. After we had
IAS (Pre) GS IInd Paper (CSAT), 2013 51 YCT
travelled for about twenty kilometres, there was still no English Passage─3
sign of the town which was marked on the map. We were
beginning to get worried. Then without warning, the car A stout old lady was walking with her basket down
stopped and we found we had run out of petrol. the middle of a street in Petrograd to the great confusion of
75. The author asked John to drive more slowly the traffic and no small peril to herself. It was pointed out
because - to her that the pavement was the place for foot-passengers,
but she replied, "I'm going to walk where I like. We've got
(a) the road led to the hills
liberty now." It did not occur to the dear lady that if liberty
(b) John was an inexperienced driver
entitled the foot-passenger to walk down the middle of the
(c) the road was not clearly visible road it also entitled the taxi-driver to drive on the
(d) they were in wilderness pavement, and that the end of such liberty would be
universal chaos. Everything would be getting in everybody
Ans. (c) : According to the passage, "we drove swiftly
else's way and nobody would get anywhere. Individual
along the narrow winding road that led to the hills. It
liberty would have become social anarchy.
was colder and rain began to fall, making it difficult at
time of see the roads." 78. It was pointed out to the lady that she should
walk on the pavement because she was -
76. The travellers set out for the town although it
(a) a pedestrian
was getting dark because -
(b) carrying a basket
(a) they were in a hurry
(c) stout
(b) the next town was a short distance away and was
a hill resort (d)an old lady

(c) they were in wilderness Ans. (a) : It was pointed out to the lady that she should
walk on the pavement because she was a pedestrian
(d) the next town was a short distance away and 79. The lady refused to move from the middle of the
promised a good rest for the night street because -
(a) she was not afraid of being killed
Ans. (d) : The Travellers set out for the town although
(b) she felt that she is entitled to do whatever she
it was getting dark because the next town was a short
liked
distance away and promised a good rest for the night.
(c) she did not like walking on the pavement
77. The travellers were worried after twenty
(d) she was confused
kilometres because -
Ans. (b) : The lady refused to move from the middle of
(a) it was a lonely countryside
the street because she felt that she is entitled to do
(b) they probably feared of having lost their way whatever she liked.

(c) the rain begin to fall 80. The old lady failed to realise that -

(d) it was getting colder as they drove (a) she was not really free
(b) her liberty was not unlimited
Ans. (b) : According to the passage, "when after we had (c) she was an old person
travelled for about twenty kilometers, there was still no (d) roads are made for motor vehicles only
sign of the town which was marked on map." Thus, they Ans. (b) : According to the passage, "The old lady
probably feared of having lost their way. failed to realize that her liberty was not unlimited."

IAS (Pre) GS IInd Paper (CSAT), 2013 52 YCT


UNION PUBLIC SERVICE COMMISSION
Civil Services (Preliminary Exam) - 2014
CSAT : PAPER-II
(Chapterwise Analysis with Explanation)
Time : 2 hours Maximum Number : 200
of the bottom quintile do not get ignored. It that were
COMPREHENSION done, then those people would in all likelihood drop
down into the bottom quintile and so would
Direction for the following 5 (five) items.
Read the following two passages and answer the automatically become a direct target of our policies.
items that follow-each passage. Your answers to these Hence the criterion being suggested here is a statistical
items should be based on the passages only. summing up of the idea of inclusive growth. Which, in
Passage-1 turn, leads to two corollaries: to wish that India must
In recent times, India has grown fast not compared strive to achieve high growth and that we must work to
to its-own past but also in comparison with other nations. ensure that the weakest segments benefit from the
But there cannot be any room for complacency because it growth.
is possible for the Indian economy to develop even faster 1. The author’s central focus is on
and also to spread the benefits of this growth more (a) applauding India’s economic growth not only
widely than has been done thus far, Before going into against its own past performance, but against
details of the kinds of micro-structural changes that we other nations.
need to conceptualize and then proceed to implement, it (b) emphasizing the need for economic growth
is worthwhile elaborating on the idea of inclusive growth which is the sole determinant of a country's
that constitutes the defining concept behind this prosperity.
Government’s various economic policies and decisions. (c) emphasizing inclusive growth where gains of
A nation interested in inclusive growth views the same growth are shared widely by the population.
growth differently depending on whether the gains of the (d) emphasizing high growth.
growth are heaped primarily on a small segment or Ans. (c) The author's central focus is on emphasizing
shared widely by the population. The latter is cause for the need for economic growth which is the sole
celebration but not the former. In other words, growth determinant of a country's prosperity as a simple way
must not be treated as an end in itself but as an of giving this idea of inclusive growth a sharper form
instrument for spreading prosperity to all. India’s own is to measure a nation's progress in terms of the
past experience and the experience of other nations progress of its poorest segment for instance the bottom
suggests that growth is necessary for eradicating poverty 20 percent of the population. One could measure the
but it is not a sufficient condition. In other words, per capita income of the bottom quintile of the
policies for promoting growth need to be complemented population and also calculate the growth rate of
with policies to ensure the more and more people join’ in income and evaluate our economic success in terms of
the growth process and further, that there are these measures that pertain to the poorest segment.
mechanisms in place to redistribute some of the gains to
2. The author supports policies which will help
those who are unable to partake in the market process (a) develop economic growth.
and, hence, get left behind (b) better distribution of incomes irrespective of
A simple way of giving this idea of inclusive growth rate of growth
a sharper form is to measure a nation’s progress in terms (c) develop economic growth and redistribute
of the progress of its poorest segment, for instance the economic gains to those getting left behind.
bottom 20 per cent of the population. One could measure (d) put an emphasis on the development of the
the per capita income of the bottom quintile of the poorest segments of society
population and also calculate the growth rate of income; Ans. (c) According to the passage, the author supports
and evaluate our economic success in terms of these policies for promoting growth need to be
complemented with policies to ensure the more and
measures that pertain’ to the poorest segment. This
more people join in growth process and further that
approach is attractive because it does not ignore growth there are mechanisms in place to redistribute some of
like some of the older heterodox criteria did. It simply the gains to those who are unable to partake in the
looks at the growth of income of the poorest sections of market process and hence, get left behind.
the population. It also ensures that those who are outside Thus option (c) is correct.
IAS (Pre) GS IInd Paper (CSAT), 2014 53 YCT
3. Consider the following statements : 1. a transparent rule-based petrol pricing exists.
According to the author, India’s economy has 2. there is no government interference in the oil
grown but there is no room for complacency as producing market.
1. growth eradicates poverty. 3. subsidies are given by the government.
2. growth has resulted in properity for all. 4. regulations of anti-trust are removed.
Which of the statements given above is /are Which of the statements give above are correct?
correct (a) 1 and 2 (b) 2 and 3
(a) 1 only (c) 3 and 4 (d) 2 and 4
(b) 2 only
Ans.(a) According to the passage, private oil
(c) Both 1 and 2
companies re-enter the oil producing market if a
(d) Neither 1 nor 2
transparent role based petrol pricing exist and by there
Ans. (d) According to the passage, it is possible for the should be rule that has to be worked out to make sure
Indian economy to develop even faster and also to that oil marketing companies can in general cover the
spread benefits of this growth more widely than has cost and once the rule is announced there should be no
been done thus for, before going into details of the interference by the government in the oil producing
kinds of micro structural changes that we need to market.
conceptualize and then proceed to implement thus
neither of statement are correct. So option (d) is Directions for the following (six) items :
correct. Read the following two passage and answer the
items that follow each passage. Your answers to these
Passage-2 items should be based on the passages only.
It is easy for the government to control State owned
companies through nods and winks. So what really needs Passage -1
to be done as first step is to put petrol pricing on a Climate change poses potentially devastating effects
on India’s agriculture. While the overall parameters of
transparent formula-if the price of crude is x and the
exchange rate y, then every month or fortnight, the climate change are increasingly accepted –a10C average
government announces a maximum price of petrol, temperature increase over the next 30 years, sea level
which anybody can work out from the x any y. The rulerise of less than 10 cm in the same period, and regional
monsoon variations and corresponding, droughts-the
has to be worked out to make sure that the oil-marketing
impacts in India are likely to be quite site and crop
companies can, in general, cover their costs . This will
mean that one company can innovate and cut costs. it specific. Some crops may respond favourably to the
will make greater profits. Hence, firms will be more changing conditions, others may not. This emphasizes
prone to innovate and be efficient under this system.the need to promote agricultural research and create
Once the rule is announced, there should be no maximum flexibility in the system to permit adaptations.
interference by the government. If this is done for a The key ingredient for ‘‘drought proofing’’ is the
while, private companies will re-enter this market. And
managed recharge of aquifers. To ensure continued
once a sufficient number of them are in the fray, we can
yields to important staple crops (e.g. wheat), It may also
remove the rule-based pricing and leave it truly to the
be necessary to shift the locations were these crops are
market (subject to, of course, the usual relations of anti-
grown, in response to temperature changes as well as to
trust and other competition laws.) water availability. The letter will be a key factor in
4. Consider the following statements : making long term investment decisions.
According to the passage, an oil company can For example, water runoff from the Himalayas
is predicted to increase over the next 30 years as glaciers
make greater profits, if a transparent formula for
petrol pricing is announced every fortnight or melt, but then decline substantially there after. It will be
month, by critical to provide incentives to plan for these large-scale
1. promoting its sales. shifts in agro-ecological conditions.
2. undertaking innovation. India needs to make long term investment in
3. cutting costs. research and development in agriculture. India is likely
4. selling its equity shares at higher prices . to experience changed weather patterns in future.
6. Consider the following statements :
Which of the statements given above is/are correct?
(a) only 1 (b) 2 and 3 Climate change may force the shifting of
(c) 3 and 4 (d) 1, 2 and 4 locations of the existing crops due to
1. melting of glaciers.
Ans. (b) According to the passage one company can 2. water availability and temperature suitgability at
innovate and cut costs it will make greater profits. other locations.
Hence, firms will be more prone to innovate and be 3. poor productivity of crops.
efficient under this system. Thus option (b) is correct. 4. wider adaptability of crop plants.
5. Consider the following statements : Which of the statements given above are correct?
According to the passage, private oil companies (a) 1, 2 and 3 (b) 2 and 3 only
re-enter the oil producing market if (c) 1 and 4 only (d) 1,2,3 and 4
IAS (Pre) GS IInd Paper (CSAT), 2014 54 YCT
Ans. (b) In the passage, it has been argued that some effort , however, would require lifestyles to change
crops may respond favorably to climate change while appropriately also. Mitigation of green house gas
others may have poor productivity because of climate emission is not a mere technological fix. And clearly
change. requires changes in lifestyles and transformation of a
A plan can be made to shift the locations of existing country’s economic structure, whereby effective
crops based on changes made in water availability and reduction in emissions is brought about, such as through
the consumption of much lower quantities of animal
temperature stability at different locations.
protein, The Food and Agriculture Organization (FARO)
This will ensure high productivity and adoptability of a
has determined that the emissions from the livestock
agricultural system to climate change. sector amount to 18 percent of the total. The reduction of
Hence, the correct answer is 2 and 3 only. emissions from this Source is entirely in the hands of
7. According to the passage, why is it important to human beings, who have never questioned the impacts
promote agricultural research in India ? that their dietary habits of consuming more and more
(a) To predict variations in monsoon patterns and animal protein are bringing about. Mitigation overall has
to manage water resources . huge co benefits, such as lower air pollution and health
(b) To make long term investment decisions for benefits, higher energy security and greater employment
economic growth . 8. According to the passage, which of the following
(c) To facilitate wider adaptability of crops would help in the mitigation of greenhouse gases
(d) To predict drought conditions and recharge 1. Reducing the consumption of meat
aquifers. 2. Rapid economic liberalization
Ans. (c) According to the passage some crops may 3. Reducing the consumption
respond favorably to the changing conditions other 4. Modern management practices of livestock
may not. This emphasizes the need to promote Select the correct answer using the code given
agricultural research and create maximum flexibility in below
the system to permit adaptations and thus India needs (a) 1, 2 and 3 (b) 2,3 and 4
to make long term investment in research and (c) 1 and 3 only (d) 2 and 4
development in agriculture. Ans. (c) According to the passage, mitigation would
Passage-2 require a major shift in the way we produce and
It is essential that we mitigate the emissions of consume energy. Mitigation of green house gas is not a
greenhouse gases and thus avoid some of the worst merely a technological fix. And clearly requires
impacts of climate change that would take place in changes in lifestyle. Whereas effective reduction in
coming years and decades. Mitigation would require a emissions is brought about, such as through the
major shift in the way we produce and consume energy. consumption of much cover quantities of protein. So it
A shift away from overwhelming dependence on fossil is very clear that reducing the consumption of meat
fuels is now long overdue, but unfortunately, and reducing the consumption. Thus, option (c) is
technological development has been slow arid correct.
inadequate largely because government policies have not 9. Why do we continue to depend on the fossil fuels
promoted investment in research an development, heavily ?
myopically as a result of relatively low prices of oil . It is 1. Inadequate technological development
now, therefore, imperative for a country like India 2. Inadequate funds for research and development
treading the opportunity of harnessing renewable energy 3. Inadequate availability of alternative sources of
on a large scale as a national imperative. This country is energy
extremely well endowed with solar, wind and biomass Select the correct answer using the code given
courses of energy. Where we have lagged, unfortunately, below :
is in our ability to develop and to create technological (a) only 1 (b) 2 and 3
solutions for harnessing these resources.
(c) 1 and 3 only (d) 1, 2 and 3
One particular trajectory for carry in out stringent
mitigation of greenhouse gas emissions assessed by the Ans. (d) According to the passage, it has been stated
Inter-governmental Panel on Climate Change (IPCC) that in spite of large climate change patterns
elderly shows the need for ensuring that global emissions happening, a desirable shift away from fossil fuel-
of greenhouse gases peak no later than 2015 and reduce based sources of energy is not happening is India.
rapidly thereafter. The cost associated with such a It can be attributed to low-level of technological
trajectory is truly modest’’ and would amount, in the advancement to develop non-fossil fuel-based sources
estimation of IPCC, to not more than 3 percent of the of energy, this is because of inadequate funds for
global GDP in 2030. In other words, the level of research and development provided by the government
prosperity that the world would have reached without to develop technology for cleaner sources of energy.
mitigation would at worst be postponed by a few month These lead to inadequate availability of alternative
or a year at the most. This is clearly not a very high price sources of energy in the country.
to pay for protecting hundreds of millions of people from Hence, the correct option is 1,2 and 3.
the worst risks associated with climate change. Any such
IAS (Pre) GS IInd Paper (CSAT), 2014 55 YCT
10. According to the passage, how does the representative ecosystems/habitats assumes a great
mitigation of greenhouse gases help us ? significance while drawing conservation plans for the
1. Reduces expenditure on public health region.
2. Reduces dependence on livestock Towards achieving the above, we will have to shift
3. Reduces energy requirements towards contemporary conservation approaches, which
4. Reduces rate of global climate change include a paradigm of landscape level interconnectivity
Select the correct answer using the code given between protected area systems. The concept advocates a
below shift from the species-habitat focus to an inclusive focus
(a) 1, 2 and 3 on expanding the biogeography range so that natural
(b) 1, 3 and 4
adjustments to climate change can proceed without being
(c) 2, 3 and 4
(d) 1 and 4 only restrictive.
12. Consider the following statements:
Ans. (b) According to the passage, it has been
highlighted that mitigation of climate change has the According to the passage, the Adverse impact of
climate change on the ecosystem can be a
following co-benefits.
1. Permanent disappearance of some of its flora
• Improved public health, leading to reduced and fauna.
expenditure on health. 2. Permanent disappearance of ecosystem itself.
• Reduced rate and impact of climate change. Which of the statements given above is/are
• Energy security, a dimension of which is demand- correct?
side management i.e. reducing energy requirements. (a) 1 only (b) 2 only
Hence, the correct answer is 1, 3 and 4. (c) Both 1 and 2 (d) Neither 1 nor 2
11. What is the essential message of the passage. Ans. (a) According to the passage, it is very possible
(a) We continue to depend on fossil fuels heavily that climate change may adverse impact the Himalayan
(b) Mitigation of the greenhouse gases is imperative ecosystem through increased temperature altered
(c) We must invest in research and development/ precipitation patterns, episode of drought and biotic
(d) People must change their lifestyle influence thus it will impact on flora and fauna and
Ans. (b) According to the passage, the essential sometime permanent disappearance of flora and fauna
will take place. Whereas permanent disappearance of
message is mitigation of the greenhouse gas is
ecosystem is not possible thus only statement (1) is
imperative. correct and statement (2) is wrong. So option (a) is
Directions for the following 8 (Eight) items : correct.
Read the following two passages and answer the 13. Which one of the following statements best
items that follow each passage. Your answer to these implies the need to shift toward contemporary
items should be based on the passages only. conservation approach?
Passage-1 (a) Exploitation of natural resources causes a
The Himalayan ecosystem is highly vulnerable to stress on the Himalayan ecosystem.
damage, both due to geological reasons and on account (b) Climate change alters precipitation patterns.
of the stress caused by increased pressure of population, Causes episodes of drought and biotic
exploitation of natural resources and other related interference.
challenges. These aspects may be exacerbated due to the (c) The rich biodiversity, including endemic
impact of climate change. It is possible that climate diversity, makes the Himalayan region a
change may adversely impact the Himalayan ecosystem biodiversity hotspot.
through increased temperature altered precipitation (d) The Himalayan biogeographic region should be
patterns, episodes of drought and biotic influences. This enabled to adapt to climate change smoothly
would not only impact the very sustenance of the Ans. (d) According to the passage, the Himalayan
indigenous communities in uplands but also the life of biogeographic region should be enabled to adapt to
downstream dwellers across the country and beyond, climate change smoothly is in very much need to shift
Therefore, there is an urgent need for giving special contemporary conservation approach.
attention to sustain The Himalayan ecosystem , This 14. What is the most important message conveyed by
would require conscious efforts for conserving all the the passage?
representative systems. (a) Endemism is a characteristic feature of
Further, it needs to be emphasized that the endemics Himalayan region.
with restricted distribution, and most often with (b) Conservation efforts should emphasize on
specialized habitat requirements are among the most biogeographic ranges rather than on some
vulnerable elements. In this respect the Himalayan species or habitats.
biodiversity hot spot, with rich endemic diversity, is (c) Climate change has adverse impact on the
vulnerable to climate change. The threats include Himalayan ecosystem.
possible loss of genetic resources and species, habitats (d) Without Himalayan ecosystem, the life of the
and concomitantly a decrease in ecosystems services. communities of uplands and downstream will
Therefore, conservation of endemic elements in have no sustenance
IAS (Pre) GS IInd Paper (CSAT), 2014 56 YCT
Ans. (b) According to the passage the most important 16. According to the passage, under the
message being conveyed is that the conservation globalization, government interventions are
efforts should emphasize on biogeography ranges viewed as processes leading to
rather than on some species or habitats. (a) distortions and inefficiency in the economy.
(b) optimum use of resources.
15. With reference to the passage the following (c) more profitability to industries.
assumptions have been made (d) free play of market forces with regard to the
1. To maintain natural ecosystems, exploitation of
industries.
natural resources should be completely avoided.
2. Not only anthropogenic but also natural reasons Ans. (a) According to the passage, the basic
can adversely affect ecosystems. philosophy underlying globalization emphasizes
3. Loss of endemic diversity leads to the extinction absolute freedom to market to determine prices and
of ecosystems. production and distribution patterns, and view
Which of the above assumption is/are correct? government intervention as process that create
(a) 1 and 2 (b) only 2 distortion and brings inefficiency in the economy.
(c) 2 and 3 (d) 3 only 17. According to the passage, the basic philosophy of
Ans. (b) In the passage, it has been argued that the globalization is to
Himalayan system is vulnerable to damage. (a) Give absolute freedom to producers to
• Assumption 1 says that exploitation of natural determine prices and production.
resources should be completely avoided to maintain (b) Give freedom to producers to evolve
natural ecosystems. This is not a practical strategy. So distribution patterns.
it is incorrect. (c) Give absolute freedom to markets to determine
• Assumption 3 is incorrect because the loss of
prices, production and employment.
endemic diversity leads to a change in the composition
of flora and fauna of an ecosystem rather than (d) Give freedom to producers to import and
complete extinction of the ecosystem. export.
Therefore, the correct assumption is 2 only. Ans. (c) According to the passage, the basic
Passage-2 philosophy of globalization is to give absolute freedom
It is often forgotten that globalization, is not only to markets to determine prices, production and
about policies on international economic relationships employment.
and transactions, but has equally to do with domestic 18. According to the passage, which of the following
policies of a nation. Policy changes necessitated by is/are necessary for ensuring globalization ?
meeting the internationally set conditions (by WTO etc.) 1. Privatization of public enterprises
of free trade and investment flows obviously affect
2. Expansionary policy of public expenditure
domestic producers and investors. But the basic
3. Free play of market forces to determine wages
philosophy underlying globalization emphasizes absolute
freedom to markets to determine prices and production and employment
and distribution patterns, and view government inter- 4. Privatization of social services like education
ventions as processes that create distortions and bring in and health.
inefficiency. Thus public enterprises have to be Select the correct answer using the code given
privatized through disinvestments and sales; sectors and below
activities hitherto reserved for the public sector have to (a) 1 only
be opened to the private sector. This logic extends to the (b) 2 and 3 only
social services like education and health. Any restrictions (c) 1, 3 and 4
on the adjustments in workforce by way of retrenchment (d) 2, 3 and 4
of workers should also be removed and exit should be
made easier by removing any restrictions on closures. Ans. (c) According to the passage, basic philosophy
Employment and wages should be governed by free play underlying globalization emphasizes absolute freedom
of market forces, any measure to regulate them can to markets to determines prices and production and
discourage investment and also create inefficiency in distribution patterns, and also privatization of social
production Above all , in line with the overall philosophy services like education and health, thus option (c) is
of reduction in the role of the state, fiscal reforms should correct.
be undertaken to have generally low levels of taxation
19. According to the passage, in the process of
and government expenditure should be kept to the
globalization the State should have/
minimum to abide by the principle of fiscal prudence.
(a) expanding role
All these are policy actions on the domestic front and are
not directly related to the core items of the globalization (b) reducing role
agenda, namely free international flow of goods and (c) statutory role
finance. (d) none of the above roles
IAS (Pre) GS IInd Paper (CSAT), 2014 57 YCT
Ans. (b) According to the passages, in the process of Ans. (c) According to the passage, it is very clear that
globalization the State shall have reducing role and their arguments an ideological bias with the
fiscal reform should be undertaken to have generally assumption that an unregulated market is fair and
low level of taxation. competent, thus option (c) is correct.
Directions for the following 7 (seven) items : 22. "The exercise of private greed will be in the
Read the following two passages and answer the larger public interest" from the passage .
1. refers to the false ideology of capitalism.
items that follow each passage. Your answers to these
2. underlies the righteous claims of the free
items should be based on the passages only.
market.
Passage-1 3. shows the benevolent face of capitalism.
Many nations now place their faith in capitalism and 4. ignores resultant gross inequity.
governments choose it as the strategy to create wealth for Which of the statements given above is/are
their people. The spectacular econ comic growth seen in correct?
Brazil, China and India after the liberalization of their (a) 1 only (b) 2 and 3
economies is proof of its enormous potential and success. (c) 1 and 4 (d) 4 only
However, the global banking crisis and the economic Ans. (c) According to the passage, the ideology of
recession have left many bewildered. The debates tend to capitalism is that free market will ensure a just
distribution of goods and services in the society,
focus on free market operations and forces, their according to one's capability and capacity.
efficiency and their ability for self correction. Issues of However, this ignores the inequality, exploitation and
justice, Integrity and honesty and rarely elaborated to exclusion of the poor from the growth and
highlight the failure of the global banking system. The development process.
apologists of the system continue to justify the success of Therefore, the ideology of capitalism is false. Also, the
capitalist ideology ignores resultant gross inequity.
capitalism and argue that the recent crisis was a blip.
Hence, the correct answer is 1 and 4.
Their arguments betray an Ideological bias with the
assumptions that an unregulated market is fair and Passage-2
competent, and the exercise of private greed will be in Net profits are only 2.2% of their total assets for
the larger pubic interest. central public undertakings, lower than for the private
Few recognize the bidirectional relationship between corporate sector. While the public sector or the State-led
capitalism and greed; that each reinforces the other. entrepreneurship played an important role in triggering
Surely, a more honest conceptualization of the conflicts India’s industrialization, our evolving development
of interest among the rich and powerful players who needs, comparatively less-than-satisfactory performance
have benefited from the system, their biases the ideology of the public sector enterprises, the maturing of our
is needed; the focus on the wealth creation should also private sector, a much larger social base now available
highlight the resultant gross inequity. for expanding entrepreneurship and the growing
20. The apologists of the ‘Free Market System’’, institutional capabilities to enforce competition policies
according to the passage, believe in would suggest that the time has come to review the role
(a) Market without control by government of public sector.
authorities What should the portfolio composition of the
(b) Market without protection by the government. government be ? It should not remain static all times.
(c) Ability of market of self correct. The airline industry work well as a purely private.
(d) Market for free goods and services. Affair. At the opposite end, rural, roads, whose sparse
Ans. (c) According to the apologists of the " Free traffic makes tolling unviable, have to be on the balance
Market System", the economic growth seen in Brazil, –sheet of the State. If the government did not own rural
China and India after liberalisation justifies that roads, they would not exist. Similarly, public health
unregulated market is fair and competent. They believe capital in our towns and cities will need to come from
that a free market has a capacity for self-correction the public sector. Equally, preservation and
because of which such market, where private interest improvement of forest cover will have to be a new
dominate, will be in the largest public interest. priority for the public sector assets.
Therefore, option (c) will correct. Take the example of steel. With near-zero tariffs,
21. With reference to ‘’ideological bias’’, the passage India is a globally competitive market for the metal.
implies that Indian firms export steel into the global market which
(a) free market is fair but not competent. demonstrates there is no gap in technology. Indian
(b) free market is not fair but competent companies are buying up global steel companies, which
(c) free market is fair and competent. shows there is no gap in capital availability, Under these
(d) free market is neither fair nor biased. conditions, private ownership works best.
IAS (Pre) GS IInd Paper (CSAT), 2014 58 YCT
Private ownership is clearly desirable in regulated 25. The portfolio composition of the government
industries, ranging from, finance to infrastructure, where refers to
a government agency performs the function of regulation
and multiple competing firms are located in the private (a) Public sector assets quality.
sector. Here, the simple and clean solution-government
as the umpire and the private sector as the players is what (b) Investment in liquid assets
works best. In many of these industries, we have a legacy (c) Mix of government investment in different
of government ownership, where productivity tends to be
industrial sectors.
lower, fear of bankruptcy is absent, and the risk of asking
for money from the tax payer is ever present. (d) Buying Return on Investment yielding capital
There is also the conflict of interest between assets.
government as an owner and as the regulator. The Ans. (c) According to the passage, we can cleary
formulation and implementation of competition policy
will be more vigorous and fair if government companies understand that the portfolio of composition of the
are out of action. government refers to mix of government investment in
23. According to the passage, what is/ are the different industrial sector.
reason/reasons for saying that the time has come
to review the role of public sector ? 26. The author prefers government as the umpire
1. Now public sector has lost its relevance in the and private sector as players because
industrialization process. (a) Government prescribes norms for a fair play by
2. Public sector does not perform satisfactorily. the private sector
3. Entrepreneurship in private sector is expanding
4. Effective competition policies are available now. (b) Government is the ultimate in policy
Which of the statements given above is/are formulation
correct in the given context? (c) Government has not control over private sector
(a) 1 and 3 only players.
(b) 2 only
(d) None of the above statements is correct in this
(c) 2, 3 and 4
context.
(d) 1, 2, 3 and 4
Ans. (c) According to the passage, the public sector or Ans. (a) According to the passage, private ownership
the state led entrepreneurship played an important role is clearly desirable in regulated industries ranging
in triggering India's industrialization, our evolving from, finance to infrastructure, where a government
development needs, comparatively less than
agency performs the function of regulation and
satisfactory performance of the public sector
enterprises, the maturing of our private sector, a much multiple competing firms are located in the private
larger social base now available for expanding sector. Here, the simple and clean solution -
entrepreneurship and growing institutional capabilities government as the umpire and the private sector as the
to enforce competition policies. Thus statement 2, 3, 4 players is what works best. Thus option (a) is correct.
are correct so option (c) is correct.
24. According to the passage, rural roads should be LOGICAL & ANALYTICAL ABILITY
in the domain of public sector only. Why?
(a) Rural development work is the domain of 27. Examine the following figure :
government only.
(b) Private sector cannot have monetary gains in
this.
(c) Government takes money from tax payers and
hence it is the responsibility of government Which one of the following figures has the above
only. figure embedded in it ?
(d) Private sector need not have any social
responsibility. (a) (b)
Ans. (b) According to the passage, the rural roads
whose sparse traffic make tolling unviable, have to be
on the balance sheet of the state. If the government did (c) (d)
not own rural roads, they would not exist and also
private sector cannot have monetary gains in this thus
option (b) is correct. Ans. (c)

IAS (Pre) GS IInd Paper (CSAT), 2014 59 YCT


28. Consider the following matrix : Which of the following figures could fit in the
empty block and thus complete the matrix ?

(a) (b)

(c) (d)

Which one of the following figures fits into blank Ans. (a) In the given matrix- Coming from top to
part of the above matrix? bottom, A circle and a triangle are getting lesser.
31. ‘‘Liberty, therefore, is never real unless the
(a) (b) Government can be called to account when it
invades rights.’’
Which one of the following is the best
justification of the above statement ?
(c) (d) (a) In the realisation that the government can be
brought to book in a court of law
(b) In identifying-a man as a political unit in a way
Ans. (b) In the given matrix:- Coming from top to which distinguished him from other citizens
bottom, one arrow is constant while the other arrow is (c) In a decentralized society wherein the basic
rotating 90º each time in a clockwise direction needs of men can find satisfaction
29. Consider the table given below in which the (d) In the understanding that liberty and restraints
numbers bear certain relationship among are complementary.
themselves along the rows: Ans. (d) According to the question, liberty is only real
when anyone who violates it can be held accountable
29 13 18 even when it is the movement i.e. the most powerful
33 x 19 organisation of any country.
Hence, the (1st) option i.e. ' In the realisation that the
30 27 3 government can be brought to book in a court of Law,'
is the best justification of the given statement.
Which one of the following numbers is the * But, according to UPSC (2014) official answer key
missing number indicated above by X ? option (d) is the correct answer.
(a) 19 (b) 15 32. A question paper must have a question on one of
(c) 14 (d) 8 the eight poets : A, B,C, D, E,F,G or H. The first
four belong to the medieval period while the rest
Ans. (b/d) Just as,
are considered modern poets. Generally, modern
29 + 13 + 18 = 60
poets figure in the question paper in alternate
30 + 27 + 3 = 60
years. Generally those who like H like G also;
Similarly,
and those who like F like E also. The paper-stter
33 + X + 19 = 60
does not like to ask about F as he has written a
X +52 = 60
book on F, but he likes F. Last year, the paper
X=8
contained a question on A. On the basis of the
* According to UPSC (b/d).
information given, this year’s paper is most
30. Consider the following matrix with one empty likely to contain a question on
block in the lower extreme corner : (a) C (b) E
(c) F (d) H
Ans. (b) ∵ Poets of medieval period = A, B, C, D and
poets of modern period = E, F, G, H
∵ Generally, modern poets figure in the question
paper in alternate years and lost year, the paper
contained a question on A
That means,
This year, the paper will contain on one of the poets of
modern period i.e., E,F,G,H.

IAS (Pre) GS IInd Paper (CSAT), 2014 60 YCT


∵ The paper-setter does not like to ask about F on he Ans. (d) In the given statement, roads, carefulness of
has written a book on F but he likes f. driver and capacity of driver is not mentioned.
and who likes F likes E. Hence, it is not possible to draw conclusions.
Hence, This year's paper is most likely contain a 36. If political leadership fails to emerge, there is
question on E. likelihood of military taking over power in
33. In a group of six women there are four dancers, developing countries. Radical student groups or
four vocal musicians, one actress and three labour may try to raise revolution but they are
violinists. Girija and Vanaja are among the not likely to compete with the military. Millitary
violinists while Jalaja and Shailaja do not know intervention , rule, and withdrawal from politics
how to play on the violin. Shailaja and Tanuja is closely related to a society’s level of political
are among the dancers. Jalaja, Vanaja, Shailaja development.
and Tanuja are vocal musicians and two of them In the context of political development, the
are also violinists. If Pooja is an actress, who assumption in the above passage is that
among the following is certainly a dancer and (a) Political leadership is not an effective Instrument.
violinist ? (b) Military fills in political vacuum.
(a) Jalaja (b) Pooja (c) Military intervention is inevitable for development.
(c) Shailaja (d) Tanuja (d) None of the above
Ans. (d) Ans. (b) In the context of political development. If
S.N. Name Dancers Vocal Actrss Violinists political leadership fails to emerge, there is likelihood
Musicians of military taking over power that means military fills
1 Girija √ in political vacuum.
2 Vanaga √ √
37. Examine the following statements :
3 Jalaja √ × 1. George attends Music classes on Monday.
4 Shailaja √ √ × 2. He attends Mathematics classes on Wednesday.
5 Tanuja √ √ √ 3. His Literature classes are not on Friday
6 Pooza √ 4. He attends History classes on the day following
Hence, the day of his mathematics classes.
Tanuja is certainly a dancer and violinists in general way. 5. On Tuesday, he attends his Sports classes.
If he attends just one subject in a day and his
34. The number of deaths among the army personnel
Sunday is free, then he is also free on
is 8 in 1000. but among the civilian population it
(a) Monday (b) Thursday
is 20 per 1000. Which one of the following
(c) Saturday (d) Friday
inferences can be drawn from the statement ?
(a) It is better to joint the army. Ans. (d)
(b) The relationship is fortuitous Day Class
(c) Quality of Life Index is very high within the
armed forces. Monday Music
(d) The groups cannot be compared due to their Tuesday Sport
heterogeneity. Wednesday Mathematics
Ans. (d) Army personnel go through immense training Thursday History
everyday and they are generally fit and belong to a Friday Holiday
certain age group while the civilian population is more Saturday Literature
diverse in terms of age group. It contains old people, Sunday Holiday
sick people, women, children etc. Thus, it is not Hence, George is also free on Friday.
relevant to compare both groups. Hence, statement (d)
is the correct answer. Directions for the following 3 (three) items :
35. Given the statement: ‘’Buses are the cause of Read the passage given below and answer the
more accidents than cars, and trucks causes items that follow.
fewer accidents than buses’’, which of the A, B, C, D, E, F are members of a family. They are
following conclusions can we draw ? engineer, stenographer, doctor, draughtsman, lawyer and
There are more buses on the road then trucks. judge (not in order). A, the engineer is married to the
(a) There are more buses on the road than trucks. lady stenographer. The judge is married to the lawyer. F,
(b) Car drivers are more careful than bus drivers. the draughtsman is the son of B and brother of E.C, the
(c) Truck drivers are more skilled than either bus lawyer is the daughter-in-law of D.E is the unmarried
drivers. doctor. D is the grandmother of F. There are two married
(d) None of the above couples in the family.
IAS (Pre) GS IInd Paper (CSAT), 2014 61 YCT
Solution for (38 to 40) 42. There are seven persons up on a ladder, A,B,C ,
D, E, F and G (not in that order). A is further up
than E but is lower than C. B is in the middle. G
is between A and B .E is between B and F. If F is
between E and D, the person on the bottom step
of the ladder will be
(a) B (b) F
(c) D (d) E
Ans. (c) According to question,
•C
•A
Where, ⇔ Married couple •G
(+) Male •B
(–) Female •E
↔ Siblings •F
38. What is the profession of B . •D
(a) Judge The person on the bottom step of the ledder will be D.
(b) Lawyer 43. Consider that :
(c) Draughtsman 1. A is taller than B
(d) Can’t be determined 2. C is taller than A
Ans. (a) The profession of B is Judge. 3. D is taller than C
39. Which of the following is /are couple / couples ? 4. E is the tallest of all.
(a) AD only If they are made to sit in the above order of their
(b) BC only height, who will occupy the mid position ?
(c) Both AD and BC (a) A (b) B
(d) Both AC and BD (c) C (d) D
Ans. (c) A D and B C are couples. Ans. (c) According to question,
•E
40. What is the profession of D?
(a) Judge •D
(b) Stenographer •C
(c) Doctor •A
(d) Can’t be determined •B
Ans. (b) The profession of D is stenographer "C" will occupy the mid position.
44. Consider the following statements :
GENERAL MENTAL APTITUDE There are six villages A, B, C, D, E and F
41. A group of 630 children is seated in rows for a F is 1 km to the west of D .
B is 1 km to the east of E.
group photo session. Each row contains three less A is 2 km to the north of E.
children than the row in front of it. Which one of C is one km to the east of A.
the following number of rows is not possible ? D is 1 km to the south of A.
Which three villages are in a line ?
(a) 3 (b) 4 (c) 5 (d) 6 (a) A, C, B (b) A, D E
Ans. (d) Condition (I)- (c) C, B, F (d) E, B, D
213 + 210 + 207 = 630 Ans. (b)
Condition (II)-
162 + 159 + 156 + 153 = 630
Condition (III)-
132 + 129 + 126 + 123 + 120 = 630
Condition (IV)-
A, D, E village are in a line.
113 + 110 + 107 + 104 + 101 + 98 = 633
45. Four children are sitting in a row . A is occupying
Condition (V)-
the seat next to B but not next to C . If C is not
112 + 109 + 106 + 103 + 100 + 97 = 627
sitting next you D, who is / are occupying seat/
Hence, 6 rows are not possible.
seats adjacent to D?
IAS (Pre) GS IInd Paper (CSAT), 2014 62 YCT
(a) B Ans. (a) The right option is (a) 3 tries
(b) A
(c) B and A
(d) Impossible to tell
Ans. (b)
I. A B C D
or × ×
II. D C A B
× ×
∴ D A B C
A is occupying seat adjacent D.
46. Six books are labelled A,B,C,D, E and F and are 48. With reference to the figure given below number
placed side by side. Books B,C,E and F have of different routes from S without retracing from
green covers while others have yellow covers, U and or V, is .
Books A, B and D are new while the restare old
volumes. Books A, B and C are law reports while
the rest are medical extracts. Which two books
are old medical extracts and he green covers ?
(a) B and C (b) E and F
(c) C and E (d) C and F
Ans. (b)
Book Gree Yello Ne Ol Law Medi (a) 3 (b) 6 (c) 9 (d) 18
n w w d reports cal Ans. (d)
extrac
ts
A √ √ √
B √ √ √
C √ √ √
D √ √ √
E √ √ √
The number of different routes = 3 × 2 × 3 = 18
F √ √ √
49. Consider the following figures :
Hence, the required books E and F.
47. Six identical cards are placed on a table. Each
card has number ‘1’ marked on one side and
number ‘2’ marked on its other side. All the six
cards are placed in such a manner that the Change in positions of beads in the four figures
number ‘1’ is on the upper side. In one try, avove follows a sequences. Following the same
exactly four (neither more nor less) cards are sequence, Which of the figures below should
turned upside down. In how many least number appear as the fifth figure above ?
of tries can the cards be turned upside down such
that all the six cards show number ‘2’ on the
(a) (b)
upper side ?
(a) 3
(b) 5
(c) 7
(c) (d)
(d) This cannot be achieved

IAS (Pre) GS IInd Paper (CSAT), 2014 63 YCT


Ans. (b) The position of the beads in each figure are Ans. (a)
rotating by 180º and 90º respectively. Chander → x
50. ‘’Price is not the same thing as value, Suppose Dinesh → 2x
that on a day the price of everything viz ., coal , Bhupesh < Alok
bread, postage stamps, a day’s labour, the rent of Alok → 2x + 5
houses, etc. Were to double. Prices then would Alok has the maximum amount of money.
certainly rise, but-values of all things except one 53. In a row ‘A’ is in the 11th Position from the left
would not.’’ and ‘B’ is in the 10th position from the right. If
‘A’ and ‘B’ interchange, then ‘A’ becomes 18th
The writer wants to say that if prices of all things
from the left. How many persons are there in the
were doubled row other than ‘A’ and ‘B’ ?
(a) the values of al things would remain constant. (a) 27 (b) 26
(b) the values of the things sold would be doubled. (c) 25 (d) 24
(c) the values of things bought would be halved. Ans. (c) According to question total number of persons
(d) the value of money only would be halved. = 18 + 10 – 1 = 27
Ans. (d) Option (a) and (b) can be eliminated because Hence, 25 persons are there in the row other than 'A'
it can be understood from the statement that value and 'B'.
would not remain constant and value would not get 54. If the 3rd day of a month is Monday, which one of
doubled. Option (c) can be eliminated because value of the following will be the fifth day from 21st of this
things does not depend on its price. month ?
Option (d) is correct because when prices and doubled, (a) Monday (b) Tuesday
the value of money becomes half. One can buy half the (c) Wednesday (d) Friday
thing with same amount of money.
Ans. (c) ∵ The 3rd day of a month is Monday and the
Hence, option (d) is the correct answer.
date of the fifth day from 21st = 26th day of month
51. The letters L, M, N, O, P, Q, R, S and T in their
∴ 3 + 7 + 7 + 7 = 24th day of month
order the substituted by nine integers 1 to 9 but
not in that order 4 is assigned to P. The ∴ 24th of day is Monday
difference between P and T is 5. The difference then 26th of day will be Wednesday.
between N and T is 3.
BASIC NUMERICAL ABILITY
What is the integer assigned to N ?
(a) 7 (b) 5 55. Five persons fire bullets at the target at an
(c) 4 (d) 6 interval of 6,7, 8, 9 and 12 seconds respectively .
Ans. (d) The number of times they would fire the bullets
together at the target in an hour is/
(a) 6 (b) 7
(c) 8 (d) 9
Ans. (b) The L.C.M. of 6, 7, 8, 9 and 12 = 504
The integer assigned to N is 6. 60 × 60
for required number =
52. Four persons, Alok, Bhupesh, Chander and 504
Dinesh have a total of Rs. 100 among themselves. = 7.14 times
Alok and Bhupesh between them have as much Hence, the bullets together at the target in an hour = 7
money as Chander and Dinesh between them, but times.
Alok has more money than Bhupesh; and Note :- According to UPSC both option (b/c) are
Chander has only half the money that Dinesh correct.
has. Alok has in fact Rs. 5 more than Dinesh has. 56. Assume that
Who has the maximum amount of money ? 1. the hour and minute hands of a clock move
(a) Alok without jerking.
(b) Bhupesh 2. the clock hows a time between 8 ‘o’clock and
(c) Chander 9 ‘o’clock.
(d) Dinesh 3. the two hands of th clock are one above the other.
IAS (Pre) GS IInd Paper (CSAT), 2014 64 YCT
After how many minutes (neares interger) with third point is at a distance of 3 cm from the
the two hands be again lying one above the other. second point and so on. If the points on the ends
(a) 60 (b) 62 are not counted and the common points are
(c) 65 (d) 67 counted as one, what is the number of points ?
60 5 (a) 10 (b) 12
Ans. (c) Required time = × 60 min = 65 ≈ 65
55 11 (c) 14 (d) 16
min Ans. (b) The line with points as per the data would
57. There are 50 students admitted to a nursery look like this.
class. Some students can speak only English and End End
point point
some can speak only Hindi. 10 student can speak
both English and Hind. If the number of students I II III IV V V IV III II I
who can speak English is 21, then how many 1 2 3 4 5 6 5 4 3 2 1
students can speak Hindi, how many can speak • • • • • • • • • • • •
only Hindi and how many can speak only
1 2 3 4 5 5 4 3 2 1
English.
Number of points = 10
(a) 21, 11 and 29 respectively
* According to UPSC (b).
(b) 28,18 and 22 respectively
(c) 37, 27 and 13 respectively 60. If Sohan, while selling two goats at the same
(d) 39, 29 and 11 respectively price, makes a profit of 10% on one goat and
suffers a loss of 10% on the other
Ans. (d)
(a) he make no profit no loss
(b) he makes a profit of 1%
(c) he suffers a loss of 1%
(d) he suffers a loss of 2%
Ans. (c) Required profit/loss
 10 ×10 
= 10 − 10 − %
 100 
39 can speak Hindi. = – 1%
29 Can only speak Hindi ∵ Hence, loss of 1%
11 Can only speak English
61. Out of a total of 120 musicians in a club, 5% can
58. A gardener increased the area of his rectangular play all the three instruments, guitar violin and
garden by increasing its length by 40% and flute. It so happens that the number of musicians
decreasing its width by 20%. The area of the new who can play any two and only two of the above
garden. instruments is 30. The number of musicians who
(a) has increased by 20% can play the guitar alone is 40. What is the total
(b) has increased by 12 % number of those who can play violin alone of
(c) had increased by 8% flute alone?
(d) is exactly the same as the old area (a) 45 (b) 44
Ans. (b) (c) 38 (d) 30
The area of the new garden Ans. (b) The number of musicians who play three
 40 × 20  instruments
=  40 − 20 − %
 100  = 120 × 5%
⇒ + 20 – 8= + 12%= 10% Increase. 5
= 120 × =6
100
59. A straight line segment is 36 cm long. Points are =6
to be marked on the line from both the end The number of musicians who play two instruments
points. From each end the, first point is at a = 30
distance of 1 cm from the end, the second point is The number of musicians who play only guitar
at a distance of 2 cm from the first point and the = 40
IAS (Pre) GS IInd Paper (CSAT), 2014 65 YCT
Then, the total number of those who can play violin Ans. (b) According to the Graph in the question, we
done of flute alone have
= 120 – (6 + 30 + 40)
= 120 – (76) = 44 The average profit of B from the year 1997 to the year
2000 is :
Directions for the following 4 (four ) items : 1997 = 4000
The following graph shows the average profit of 1998 = 4000
two fruit-sellers A and B in thousands (R) per 1999 = 5000
year from the year 1995 to 2000. Consider the 2000 = 5400
graph and answer the 4 (four) items that the Hence, The Average profit of B from the year 1997 to
follow: 2000 is Non- decreasing.
66. The following table shows the marks obtained by
two students in different subjects :
Student MaximumStudent-BMaximum
A Marks Marks
English
60 100 80 150

Psychology
70 100 70 100

History
50 100 60 100
62. In which year is the average profit of A and B
same ? Sanskrit
30 50 15 25
(a) 1995 (b) 1996
(c) 1997 (d) 1998 The difference in the mean aggregate percentage
Ans. (b) The average profit of A and B same is year marks of the students is
1996. (a) 2.5% (b) 13.75%
(c) 1. 25% (d) Zero
63. What is the difference between the average profit
Ans. (d) The mean of A (in % )
of B and A in the year 1998 ?
 60 + 70 + 50 + 30 
(a) – Rs.100 (b) – Rs.1,000 =  × 100%
 100 + 100 + 100 + 50 
(c) + Rs.600 (d) – Rs. 300
210
= × 100%
Ans. (c) The difference between the average profit of 350
B and A in the year 1998 = 60%
= 4000 – 3400 The mean of B (in %)
 80 + 70 + 60 + 15 
= 600 =  × 100%
 150 + 100 + 100 + 25 
64. How much more average profit did A make in the 225
= × 100%
year 2000 than in the year 1999 ? 375
(a) Rs. 200 (b) Rs. 1,000 = 60%
(c) Rs. 1,500 (d) Rs. 2,000 The required difference
= 60% – 60%
Ans. (d) The trend of the average profit of B from the =0
year 1997 to the year 2000 is non-decreasing.
67. The following table gives population and total
65. What is the trend of the average profit of B from income of a city for four years :
the year 1997 to the year 2000? Year 1992 1993 1994 1995
(a) Non- Increasing Population in 20 21 22 23
(b) Non-decreasing lakhs
(c) Steady Income in croes 1010 1111 1225 1345
(d) Fluctuating (Rs.)

IAS (Pre) GS IInd Paper (CSAT), 2014 66 YCT


Which one of the following statements correct in 70. As per agreement with a bank, a businessman
respect of the above data ? had to refund a loan in some equal installments
(a) Population increase by 5% or more every year without interest. After paying 18 installments he
(b) Income increased by 10% or more every year. found that 60 percent of his loan was refunded.
(c) Per capita income was always above Rs. 5,000 How many installments were there in the
(d) Per capita income was highest in 1994. agreement.
(a) 22 (b) 24
Ans. (c) Per capital income in 1992 = 5050 (c) 30 (d) 33
Per capita income in 1993 = 5290
Ans. (c) Let the number of installments = x
Per capita income in 1994 = 5568 According to question
Per capita income in 1995 = 5847 6 % of x 18
Hence, Per capita income was always above Rs. 5000 60
x× = 18
100
68. A bell rings every 18 minutes. A second bell rings x = 30
every 24 minutes. A third bell rings every 32
minutes. If all the three bells ring at the same 71. A worker reaches his factory 3 minutes late. If his
time at 8 O’clock in the morning, at what other speed from his house to the factory is 5 km/ hr. If
time will they all ring together? he walks at a speed of 6 km/hr then he reaches
(a) 12 : 40 hrs the factory 7 minutes early. The distance of the
factory from his house is .
(b) 12 : 48 hrs
(a) 3 km (b) 4 km.
(c) 12 : 56 hrs (c) 5 km (d) 6 km.
(d) 13 : 04 hrs Ans. (c) Factory from his house
Ans. (b) The L.C.M. 18, 24 and 32 5× 6  3 7  30 10
= × + = × = 5 km
= 288 min 6 − 5  60 60  1 60
288 72. Location of B is north of A and location of C is
= hr
60 east of A . The distances AB and AC are 5 km.
= 4 : 48 min and 12 km respectively. The shortest distance (in
The time when they will all ring together km) between the location B and C is
= (8 + 4 : 48) (a) 60 (b) 13
= 12 : 48 hr (c) 17 (d) 7
69. A an B decide to travel from place X to place Y by Ans. (b) According to question,
bus. A has Rs. 10 with him and he finds that it is
80% of the bus fare for two persons. B finds that
he has Rs. 3 with him and hands it over to A. In
this context, which one of the following
statements is correct ?
(a) Now the money A has just enough to buy two
tickets.
(b) A still needs Rs. 2 for buying the tickets The distance between B and C
(c) After bying the two tickets A will be left with 50 = 5 2 + 12 2 = 169 = 13 km.
paise.
73. Two cars starts towards each other, from two
(d) The money A now has is still not sufficient to places A and B which are at a distance of 160 km.
buy two tickets. They start at the same time 08:10 AM. If the
Ans. (c) Let the required Amount = x ` speeds of the cars are 50 km. and 30 km per hour
According to question, respectively, they will meet each other at
80% of x = 10 (a) 10 : 10 AM (b) 10 : 30 AM
10 × 100 (c) 11.10 AM (d) 11 : 20 AM
x= = 10 Ans. (a)
80
x = 12.5
The total amount which A and B have
= 10 + 3 ∴ Relative speed = (50 + 30) km/hr
= 13 = 80 km/hr
Remaining Amount = 13 – 12.5 d 160
∵ t= t= , t = 2 hr.
= 0.50 ` = 50 paise ν 80

IAS (Pre) GS IInd Paper (CSAT), 2014 67 YCT


74. For a charity show, the total tickets sold were 420 (c) the dog looked at the procession with big,
Half of these tickets were sold at the rate of Rs.5 wondering eyes
each. one-third at the rate Rs. 3 each and the rest (d) the dog stood there with his eyes closed
for Rs. 2 each. What was the total amount Ans. (b) The expression "a stray dog watched the
received. procession philosophically means the dog stood aloof,
(a) Rs. 900 (b) Rs. 1,540 looking at the procession with seriousness. So option
(c) Rs. 1,610 (d) Rs. 2,000 (b) is correct.
Ans. (c) Total amount =
Passage-2
 1   1   1 1
 420 × × 5  +  420 × × 3  + 420 × 1 − −  × 2 Cynthia was a shy girl. She believed that she was
 2   3   2 3
plain and untalented. One day her teacher ordered the
 6 − 3 − 2 
= ( 210 × 5 ) + ( 420 ) + 420 ×  × 2 entire class to show up for audition for the school play.
 6  Cynthia nearly died of fright when she was told that she
1 would have to stand on stage in front of the entire class
= 1050 + 420 + 420 × × 2 = 1050 + 420 + 140 = 1610 `
6 and deliver dialogues. The mere thought of it made her
feel sick. But a remarkable transformation occurred
during the audition. A thin, shy girl, her knees quaking,
ENGLISH LANGUAGE/GRAMMAR her stomach churning in terror, began to stun everyone
& COMPREHENSION with her excellent performance. Her bored classmates
suddenly stopped their noisy chat to stare at her slender
Directions for the following 6 (six items) : The figure on the stage. At the end of her audition, the entire
following six items are based on two passages in room erupted in thunderous applause.
English to test the comprehension of English language 77. Cynthia was afraid to stand on stage because –
and therefore these items do not have Hindi version. (a) she felt her classmates may laugh at her
Read each passage and answer the item that follows. (b) her stomach was churning
Passage-1 (c) she lacked self-confidence
In front of us was walking a bareheaded old man in (d) she did not like school plays
tattered clothes. He was driving his beasts. They were all Ans. (c) According to the passage, Cynthia was afraid
laden with heavy loads of clay from the hills and looked to stand on stage because she lacked self-confidence.
tired. The man carried a long whip which perhaps he 78. Cynthia's classmates were chatting because –
himself had made. As he walked down the road he (a) it was their turn to act next
stopped now and then to eat the wild berries that grew on (b) they were bored of the performances
bushes along the uneven road. When he threw away the (c) Cynthia did not act well
seeds, the bold birds would fly to peck at them. (d) the teacher had no control over them
Sometimes a stray dog watched the procession Ans. (b) According to the passage, Cynthia's classmate
philosophically and then began to bark. When this were chatting because they were bored at the
happened, my two little sons would stand still holding performance
my hands firmly. A dog can sometimes be dangerous 79. Cynthia's knees were quaking because –
indeed. (a) she felt nervous and shy
75. The author's children held his hands firmly (b) the teacher scolded her
because – (c) she was very thin and weak
(a) they were scared of the barking dogs (d) she was afraid of her classmates
(b) they wanted him to pluck berries Ans. (a) Cynthia's knees were quaking because, she
(c) they saw the whip in the old man's hand felt nervous and shy.
(d) the road was uneven 80. The transformation that occurred during the
Ans. (a) According to the passage, the author's audition refers to –
children held his hands firmly as they were scared of (a) the nervousness of Cynthia
the barking dogs as sometime a stray dog watched the (b) the eruption of the entire room in thunderous
procession philosophically and then began to bark and applause
also dog can sometimes be dangerous indeed. (c) the surprise on the faces of her classmates
76. The expression "a stray dog watched the (d) the stunning performance of Cynthia
procession philosophically" means that – Ans. (d) According to the passage, a thin shy girl has
(a) the dog was restless and ferocious knees quaking her stomach churning in terror began to
(b) the dog stood aloof, looking at the procession stun everyone with her excellent performance and thus
with seriousness option (d) is correct.

IAS (Pre) GS IInd Paper (CSAT), 2014 68 YCT


UNION PUBLIC SERVICE COMMISSION
Civil Services (Preliminary Exam) - 2015
CSAT : PAPER-II
(Chapterwise Analysis with Explanation)
Time : 2 hours Maximum Number : 200

2. Which among the following is the most rational


COMPREHENSION and critical inference that can be made from the
Directions for the following 7 (seven) items : above passage?
Read the following six passages and answer the (a) Foreign portfolio investments are not good for
items that follow. Your answers to these items should emerging economies.
be based on the passages only. (b) Advanced economies undermine the global
Passage-1 financial stability.
Climate change is already making many people (c) India should desist from accepting foreign
hungry all over the world, by disrupting crop yields and portfolio investments in the future.
pushing up prices. And it is not just food but nutrients (d) Emerging economies are at a risk of shock from
that are becoming scarcer as the climate changes. It is the advanced economies.
poorest communities that will suffer the worst effects of Ans. (d) : The most rational and conclusive inference
climate change, including increased hunger and from the above passage is that emerging economies are
malnutrition as crop production and livelihoods are at risk of shock from advanced economies.
threatened. On the other hand, poverty is a driver of
climate change, as desperate communities resort to Passage-3
unsustainable use of resources to meet current needs.
Open defecation is disastrous when practised in very
1. Which among the following is the most logical
corollary to the above passage? densely populated areas, where it is impossible to keep
(a) Government should allocate more funds to away human faeces from crops, wells, food and
poverty alleviation programmes and increase children's hands. Groundwater is also contaminated by
food subsidies to the poor communities. open defecation. Many ingested germs and worms spread
(b) Poverty and climate impacts reinforce each
other and therefore we have to re-imagine our diseases. They prevent the body from absorbing calories
food systems. and nutrients. Nearly one-half of India's children remain
(c) All the countries of the world must unite in malnourished. Lakhs of them die from preventable
fighting poverty and malnutrition and treat conditions. Diarrhoea leaves Indians' bodies smaller on
poverty as a global problem.
(d) We must stop unsustainable agricultural average than those of people in some poorer countries
practices immediately and control food prices. where people eat fewer calories. Underweight mothers
Ans. (b) : The passage clearly states that the poorest produce stunted babies prone to sickness who may fail to
community only, with the increase of hunger and develop their full cognitive potential. The germs released
malnutrition, will have to suffer the worst effect of into environment harm rich and poor alike, even those
climate change. Poverty is a factor in climate change. who use latrines.
Hence the most logical subset of the passage is 3. Which among the following is the most critical
option (b). inference that can be made from the above
Passage-2 passage?
The Global Financial Stability Report finds that the (a) The Central and State governments in India do
share of portfolio investments from advanced economies not have enough resources to afford a latrine for
in the total debt and equity investments in emerging each household.
economies has doubled in the past decade to 12 percent. (b) Open defecation is the most important public
The phenomenon has implications for Indian policy
makers as foreign portfolio investments in the debt and health problem of India.
equity markets have been on the rise. The phenomenon is (c) Open defecation reduces the human capital of
also flagged as a threat that could compromise global India's workforce.
financial stability in a chain reaction, in the event of (d) Open defecation is a public health problem in
United States Federal Reserve's imminent reversal of its
"Quantitative Easing'' policy. all developing countries.
IAS (Pre) IInd Paper CSAT, 2015 69 YCT
Ans. (c) : The passage states that open defecation can Which of the statements given above is/are
be disastrous when practiced in highly populated areas correct?
where it is impossible to keep human excreta away (a) 1 only (b) 2 only
from crops, wells, food items and children's hand. (c) Both 1 and 2 (d) Neither 1 nor 2
Ground water also gets contaminated by open Ans. (b) : It is wrong to claim that Indian financial
defecation. Many germs and warms in the diet, spread institutions do not provide any financial instrument
disease and they do not allow the body to absorb related support. Option (b) will be the correct answer.
calories and nutrients. Because of this, almost half of This is confirmed by the quotation given at the end of
India's children remain malnourished. It is proved from
the passage.
the above facts that the most conclusive inference of
the passage would be option (c). 6. What is the crucial message conveyed in the
passage?
Passage-4 (a) Establish more banks
We generally talk about democracy but when it (b) Increase the Gross Domestic Product (GDP)
comes to any particular thing, we prefer a belonging to growth rate
our caste or community or religion. So long as we have (c) Increase the interest rate of bank deposits
this kind of temptation, our democracy will remain a (d) Promote financial inclusion
phoney kind of democracy. We must be in a position to
Ans. (d) : After reading the passage, it clearly shows
respect a man as a man and to extend opportunities for
that the important message is to encourage financial
development to those who deserve them and not to those
inclusion.
who happen to belong to our community or race. This
fact of favouritism has been responsible for much Passage-6
discontent and ill-will in our country. Governments may have to take steps which would
4. Which one of the following statements best sums otherwise be an infringement on the Fundamental Rights
up the above passage? of individuals, such as acquiring a person's land against
(a) Our country has a lot of diversity with its many his will, or refusing permission for putting up a building,
castes, communities and religions. but the larger public interest for which these are done
(b) True democracy could be established by must be authorized by the people (Parliament).
providing equal opportunities to all. Discretionary powers to the administration can be done
(c) So far none of us have actually understood the away with. It is becoming more and more difficult to
meaning of democracy. keep this power within limits as the government has
(d) It will never be possible for us to establish truly many number of tasks to perform. Where discretion has
democratic governance in our country. to be used, there must be rules and safeguards to prevent
Ans. (b) : The democracy can be established only by misuse of that power. Systems have to be devised which
giving equal opportunities to all. This statement gives minimise, if not prevent, the abuse of discretionary
the best summary of the passage. power. Government work must be conducted within a
Passage-5 framework of recognised rules and principles, and
decisions should be similar and predictable.
The existence/establishment of formal financial
institutions that offer safe, reliable, and alternative 7. Which among the following is the most logical
financial instruments is fundamental in mobilising assumption that can be made from the above
savings. To save, individuals need access to safe and passage?
reliable financial institution s, such as banks, and to (a) Government should always be given wide
appropriate financial instruments and reasonable discretionary power in all matters of
financial incentives. Such access is not always available administration.
to all people in developing countries like India and more (b) The supremacy of rules and safeguards should
so, in rural areas. Savings help poor households manage prevail as opposed to the influence of exclusive
volatility in cash flow, smoothen consumption, and build discretion of authority.
working capital. Poor households without access to a (c) Parliamentary democracy is possible only if the
formal savings mechanism encourage immediate Government has wider discretionary power.
spending temptations. (d) None of the above statements is a logical
5. With reference to the above passage, consider the assumption that can be made from this passage.
following statements : Ans. (b) : In the passage the author opposes the
1. Indian financial institutions do not offer any autocracy of powers by an authority. Therefore, the
financial instruments to rural households to
most logical assumption that can be made from the
mobilise their savings.
passage is the rather than the exclusive privilege of
2. Poor households tend to spend their
earnings/savings due to lack of access to authority to take effect, the rules and safeguards should
appropriate financial instruments. prevail.
IAS (Pre) IInd Paper CSAT, 2015 70 YCT
Directions for the following 7 (seven) items : Ans. (d) : The most credible explanation in the light of
Read the following four passages and answer the the arguments given in the above passage is that the
items that follow. Your answers to these items should broad idea of economic justice demands that the right
be based on the passages only. of every individual to acquire property should be
Passage-7 harmonized with the right to acquire property of
others.
India has suffered from persistent high inflation.
Increase in administered prices, demand and supply Passage-9
imbalances, imported inflation aggravated by rupee The conflict between man and State is as old as State
depreciation, and speculation─have combined to keep history. Although attempts have been made for centuries
high inflation going. If there is an element common to all to bring about a proper adjustment between the
of them, it is that many of them are the outcomes of competing claims of State and the individual, the
economic reforms. India's vulnerability to the effects of solution seems to be still far off. This is primarily
changes in international prices has increased with trade because of the dynamic nature of human society where
liberalisation. The effort to reduce subsidies has resulted old values and ideas constantly yield place to new ones.
in a continuous increase in the prices of commodities that It is obvious that if individuals are allowed to have
are administered. absolute freedom of speech and action, the result would
8. What is the most logical, rational and crucial be chaos, ruin and anarchy.
message that is implied in the above passage? 10. The author's viewpoint can be best summed up in
(a) Under the present circumstances, India should which of the following statements?
completely avoid all trade liberalisation policies (a) The conflict between the claims of State and
and all subsidies. individual remains unresolved.
(b) Due to its peculiar socio-economic situation, (b) Anarchy and chaos are the obvious results of
India is not yet ready for trade liberalisation democratic traditions.
process. (c) Old values, ideas and traditions persist despite
(c) There is no solution in sight for the problems of the dynamic nature of human society.
continuing poverty and inflation in India in the (d) Constitutional guarantee of freedom of speech is
near future. not in the interest of society.
(d) Economic reforms can often create a high Ans. (a) : It is clearly said in the passage that, "The
inflation economy. struggle between man and the state is as old as the
Ans. (d) : It is clearly mentioned in the passage that history of the state". Although there have been
India has been suffering from persistent high inflation attempts over the centuries to reconcile the competing
due to liberalization of trade and increase in economic claims of the state and the individual, the solution still
reforms. Hence, option (d) is the correct answer. seems to be faraway on the basis, option (d) presents
the best summary of the authors point of view.
Passage-8
Passage-10
No Right is absolute, exclusive or inviolable. The
Climate change is a complex policy issue with major
Right of personal property, similarly, has to be perceived
implications in terms of finance. All actions to address
in the larger context of its assumed legitimacy. The Right
climate change ultimately involve costs. Funding is vital
of personal property should unite the principle of liberty
for countries like India to design and implement
with that of equality, and both with the principle of
adaptation and mitigation plans and projects. Lack of
cooperation.
funding is a large impediment to implementing
9. In the light of the argument in the above passage, adaptation plans. The scale and magnitude of the
which one of the following statements is the most financial support required by developing countries to
convincing explanation? enhance their domestic mitigation and adaptation actions
(a) The Right of personal property is a Natural Right are a matter of intense debate in the multilateral
duly supported by statutes and scriptures. negotiations under the United Nations Framework
(b) Personal property is a theft and an instrument of Convention on Climate Change (UNFCCC). The
exploitation. The Right of personal property is Convention squarely puts the responsibility for provision
therefore violative of economic justice. of financial support on the developed countries, taking
(c) The Right of personal property is violative of into account their contribution to the stock of greenhouse
distributive justice and negates the principle of gases (GHGs) in the atmosphere. Given the magnitude of
cooperation. the task and the funds required, domestic finances are
(d) The comprehensive idea of economic justice likely to fall short of the current and projected needs of
demands that the Right of each person to the developing countries. Global funding through the
acquisition of property has to be reconciled with multilateral mechanism of the Convention will enhance
that of others. their domestic capacity to finance the mitigation efforts.
IAS (Pre) IInd Paper CSAT, 2015 71 YCT
11. According to the passage, which of the following (c) Lack of political will on the part of all the
is/are a matter of intense debate in the countries to implement adaptation plans
multilateral negotiations under UNFCCC (d) Governance problems of developing countries as
regarding the role of developing countries in a result of climate change
climate change?
Ans. (a) : This passage essentially discusses the
1. The scale and size of required financial support.
conflict between developed and developing countries
2. The crop loss due to climate change in the
developing countries. about providing aid for mitigation.
3. To enhance the mitigation and adaptation actions Directions for the following 8 (eight) items :
in the developing countries. Read the following seven passages and answer the
Select the correct answer using the code given items that follow. Your answers to these items should
below : be based on the passages only.
(a) 1 only (b) 2 and 3 only
(c) 1 and 3 only (d) 1, 2 and 3 Passage-11
Ans. (c) : From reading the passage, it is known that, The richer States have a responsibility to cut down
the scale and magnitude of financial assistance carbon emissions and promote clean energy investments.
required by developing countries for the promotion of These are the States that got electricity, grew faster and
their domestic mitigation and adaptation actions in now have high per capita income, making them capable
climate change is a matter of intense or serious
of sharing India's burden of becoming eco-friendly.
discussion. The role of developing countries should
enhance the mitigation and adoption actions under Delhi, for example, can help by generating its own clean
UNFCCC. Hence, option (c) will be the correct electricity using solar rooftop panels or even help poor
answer. States finance their clean energy projects. It is no secret
that State Electricity Boards, which control 95% of the
12. In this passage, the Convention puts the distribution network, are neck-deep in losses. These
responsibility for the Provision of financial
losses further discourage State utilities from adopting
support on the developed countries because of
renewable energy as it is more expensive than fossil
1. their higher level of per capita incomes.
2. their large quantum of GDP. fuels.
3. their large contribution to the stock of GHGs in 15. Which among the following is the most logical
the atmosphere. and rational assumption that can be made from
Select the correct answer using the code given the above passage?
below : (a) The richer States must lead in the production and
(a) 1 only (b) 1 and 2 only adoption of renewable energy.
(c) 3 only (d) 1, 2 and 3 (b) The poor States always have to depend on rich
Ans. (c) : The passage clearly states that, The States for electricity.
UNFCCC convention places an equal responsibility on (c) The State Electricity Boards can improve their
the developed countries to provide financial assistance, finances by undertaking clean energy projects.
based on their contribution to the accumulation of (d) The high economic disparity between the rich
greenhouse gases in the atmosphere. Hence option (c) and poor States is the major cause of high carbon
would be the correct answer.
emissions in India.
13. With regards to developing countries, it can be Ans. (a) : It is stated at the very beginning of the
inferred from the passage that climate change is
passage that relatively prosperous states should take the
likely to have implications on their
lead in producing and adopting renewable energy.
1. domestic finances.
2. capacity for multilateral trade. Passage-12
Select the correct answer using the code given Set against a rural backdrop, `Stench of kerosene' is
below : the story of a couple, Guleri and Manak, who have been
(a) 1 only (b) 2 only
happily married for several years but do not have a child.
(c) Both 1 and 2 (d) Neither 1 nor 2
Manak's mother is desperate to have a grandchild to
Ans. (a) : With respect to developing countries, it can
carry on the family name. Hence, she gets Manak
be inferred from the passage that climate change is
likely to impact their domestic finances. remarried in Guleri's absence. Manak, who acts as a
reluctant but passive spectator, is meanwhile, informed
14. Which one of the following is essentially by a friend that Guleri, on hearing about her husband's
discussed in this passage? second marriage, poured kerosene on her clothes and set
(a) Conflict between developed and developing fire to them. Manak is heartbroken and begins to live as
countries regarding support for mitigation
if he were a dead man. When his second wife delivers a
(b) Occurrence of climate change due to excessive
exploitation of natural resources by the son, Manak stares at the child for a long time and blurts
developed countries out, ``Take him away! He stinks of kerosene."
IAS (Pre) IInd Paper CSAT, 2015 72 YCT
16. This is a sensitive issue-based story which tries to (a) The task of providing urban services is a
sensitise the readers about complex issue which requires the organisational
(a) Male chauvinism and infidelity expansion of municipal bodies all over the
(b) Love and betrayal country.
(c) Lack of legal safeguards for women (b) Our cities can provide better quality of life if our
(d) Influence of patriarchal mindset local government bodies have adequate staff
Ans. (d) : According to the passage, Manak's mother is with required skills and competencies.
yearning to have a grandson to continue the family (c) Lack of skilled staff is due to the absence of
name, which shows the ill effects of patriarchal institutions which offer the requisite skills in city
attitude. management.
(d) Our country is not taking advantage of the
Passage-13 demographic dividend to manage the problems
The ultimate aim of government is not to rule or associated with rapid urbanization.
control by fear, nor to demand obedience, but Ans. (b) : In the passage, the author is showing
conversely, to free every man from fear, that he may live concern about the small number of employees working
in all possible security. In other words, to strengthen his in municipal corporation and their skill and efficiency.
natural right to exist and work without injury to himself Hence option (b) is the most logical and rational
or others. The object of government is not to change men presumption, because the author is demonstrating that
from rational beings into beasts or puppets. It should excellent and efficient employee practices, can
enable them to develop their minds and bodies in improve the quality of urban life.
security, and to employ their reason unshackled.
17. Which among the following is the most logical
Passage-15
and rational inference that can be made from the Flamingos in large flocks in the wild are social
above passage? and extremely loyal. They perform group mating
(a) The true aim of government is to secure the dances. Parents are very fond of their chicks,
citizens their social and political freedom. gathering them into creches for protection while both
(b) The primary concern of government is to provide males and females fly off to search for food.
absolute social security to all its citizens. 19. Which among the following is the most logical
(c) The best government is the one that allows the corollary to the above passage?
citizens to enjoy absolute liberty in all matters of (a) Mass nesting in all species of birds is essential to
life. ensure complete survival of their offspring.
(d) The best government is the one that provides (b) Only birds have the capacity to develop social
absolute physical security to the people of the behaviour and thus can do mass nesting to raise
country. their chicks in safety.
Ans. (a) : The passage refers to the major work of the (c) Social behaviour in some species of birds
government, so it would not be appropriate to consider increases the odds of survival in an unsafe world.
the work of the government as the basis or principle of (d) All species of birds set up creches for their
the best government. On this basis option (c) and (d) chicks to teach them social behaviour and
are proved wrong. The passage talks about social and loyalty.
political freedom. Hence option (a) is correct answer. Ans. (c) : Flamingos are social & loyal and when the
Passage-14 male and female fly away in search of food, their
chicks are taken care by other member of the group.
Our municipal corporations are understaffed. The So, the most logical regression would be option (c) as
issue of skills and competencies of the staff poses an it shows the importance of group and social behavior.
even greater challenge. Urban services delivery and
infrastructure are complex to plan and execute. They Passage-16
require a high degree of specialization and Vast numbers of Indian citizens without bank
professionalism. The current framework within which accounts live in rural areas, are financially and
municipal employees, including senior management, are functionally illiterate, and have little experience with
recruited does not adequately factor in the technical and technology. A research study was conducted in a
managerial competencies required. Cadre and particular area in which electronic wage payments in
recruitment rules only specify the bare minimum in Mahatma Gandhi National Rural Employment Guarantee
academic qualifications. There is no mention of Scheme (MGNREGS) are meant to go directly to the
managerial or technical competencies, or of relevant poor. It was observed that recipients often assume that
work experience. This is the case with most municipal the village leader needs to mediate the process, as was
corporations. They also suffer from weak organisation the case under the previous paper-based system. Among
design and structure. households under this research study area who claimed
18. Which among the following is the most logical to have at least one bank account, over a third reported
and rational assumption that can be made from still receiving MGNREGS wages in cash directly from a
the above passage? village leader.
IAS (Pre) IInd Paper CSAT, 2015 73 YCT
20. What is the most logical, rational and crucial (a) 1 only (b) 2 only
message that is implied in the above passage? (c) Both 1 and 2 (d) Neither 1 nor 2
(a) MGNREGS should be extended only to those Ans. (b) : In reality, environmental degradation is a
who have a bank account.
(b) The paper-based system of payments is more natural result of economic development. Hence, option
efficient than electronic payment in the present (b) will be the correct answer. Although the passage is
scenario. explaining the importance of economic development,
(c) The goal of electronic wage payments was not to but at the same time it also considers it problematic.
eliminate mediation by village leaders. Therefore, we cannot conclude that opinion to make
(d) It is essential to provide financial literacy to economic development an important factor in reducing
the rural poor. economic inequality.
Ans. (d) : Concern has been expressed in the passage
that instead of electronic payment under MNREGA, Directions for the following 8 (eight) items :
labourers consider the mediation of the village leader Read the following six passages and answer the
necessary to get wages in this process. This fact items that follow. Your answers to these items should
emphasizes the increasing spread of financial be based on the passages only.
technology and functionality. Hence, option (d) is the
correct answer. Passage-18
Human history abounds in claims and theories
Passage-17 confining the right of governing to a few select citizens.
Individuals, groups and leaders who promote human Exclusion of the many is justified on the ground that
development operate under strong institutional, structural human beings may be rightfully segregated for the good
and political constraints that affect policy options. But of society and viability of the political process.
experience suggests broad principles for shaping an
appropriate agenda for human development. One 23. Which one of the following statement is least
important finding from several decades of human essential as a part of the argument in the above
development experience is that focusing exclusively on passage?
economic growth is problematic. While we have good (a) Man seeks control over external things affecting
knowledge about how to advance health and education, him.
the causes of growth are much less certain and growth is (b) In society, there are `super' and `sub' human
often elusive. Further, an unbalanced emphasis on
growth is often associated with negative environmental beings.
consequences and adverse distributional effects. The (c) Exceptions to universal citizen participation are
experience of China, with its impressive growth record, conducive to systemic efficacy.
reflects these broader concerns and underlines the (d) Governing implies recognition of disparities in
importance of balanced approaches that emphasize individual capacities.
investments in the non-income aspects of human Ans. (a) : Minimum requirement means - almost no
development. need. The author thinks that it would be justified to
21. With reference to the above passage, consider the give preference to a few over the rights of many. From
following statements : this it is clear that options (b), (c) & (d) are related to
1. In developing countries, a strong institutional
framework is the only requirement for human the above argument but not (a). Hence, the correct
development and policy options. answer will be (a).
2. Human development and economic growth are
not always positively inter-related.
Passage-19
3. Focusing only on human development should be By 2050, the Earth's population will likely have
the goal of economic growth. swelled from seven to nine billion people. To fill all
Which of the above statements is/are correct? those stomaches─while accounting for shifting
(a) 1 only (b) 2 and 3 only consumption patterns, climate change, and a finite
(c) 2 only (d) 1, 2 and 3 amount of arable land and potable water─some experts
Ans. (c) : To say that the only requirement for human say food production will have to double. How can we
resource development is a strong institutional make the numbers add up? Experts say higher yielding
framework would be a different point. This is in sync
with option (3). The example of China has a crop varieties and more efficient farming methods will
resemblance to option (2). Hence, option (c) is the be crucial. So will waste reduction. Experts urge cities to
correct answer. reclaim nutrients and water from waste streams and
preserve farmland. Poor countries, they say, can improve
22. With reference to the above passage, the
following assumptions have been made : crop storage and packaging and rich nations could cut
back on resource-intensive foods like meat.
1. Higher economic growth is essential to ensure
reduction in economic disparity. 24. Which one of the following statements best sums
2. Environmental degradation is sometimes a up the above passage?
consequence of economic growth. (a) The population of the world is growing very fast.
Which of the above is/are valid assumption/ (b) Food security is a perennial problem only in
assumptions? developing countries.
IAS (Pre) IInd Paper CSAT, 2015 74 YCT
(c) The world does not have enough resources to Ans. (c) : In the passage the author is telling that steps
meet the impending food scarcity. have not been taken to spread basic health practices
(d) Food security is increasingly a collective like breast feeding and immunization to tackle the
challenge. problem of child mortality.
Ans. (d) : In the passage, the author is telling that food Passage-22
security is an ever increasing collective challenge
through the changing patterns of consumption, arable Foods travel more than the people who eat them.
land and timely improvement and development of Grocery stores and supermarkets are loaded with
agriculture. preserved and processed foods. This, however, often
leads to environmental threats, such as pollution
Passage-20 generated by long distance food transportation and
wastage of food during processing and transportation,
Many people in India feel that if we cut our defence destruction of rain forests, reduced nutritional content,
expenditure on weapon-building, we can create a climate increased demand for preservation and packaging. Food
of peace with our neighbours, subsequently reducing the insecurity also increases as the produce comes from
conflict or creating a no-war situation. People who regions that are not feeding their own population
proclaim such ideas are either the victims of war or the properly.
propagators of false argument. 27. With reference to the above passage, which of the
25. With reference to the above passage, which of the following statements is/are true?
following is the most valid assumption? 1. Consuming regionally grown food and not
(a) Building of weapons systems by us has depending on long travelled food is a part of eco-
instigated our neighbours to wage wars against friendly behaviour.
us. 2. Food processing industry puts a burden on our
(b) The greater spending on weapon-building by us natural resources.
would lessen the possibility of armed conflict Select the correct answer using the code given
with our neighbours. below :
(c) It is necessary to have state of the art weapons (a) 1 only (b) 2 only
systems for national security. (c) Both 1 and 2 (d) Neither 1 nor 2
(d) Many people in India believe that we are wasting Ans. (c) : Pollution caused by transportation of food
our resources on weapon-building. items over long distances and wastages of food during
processing and transportation, destruction of rain-
Ans. (b) : The passage rejects the arguments on forests, reduction in nutrient content etc. By looking at
defense expenditure, which means that there should be the statement, it is known that option (c) is correct
a belief and trust that a high expenditure on arms answer.
manufacturing can act as a major deterrent in averting
a targeted war with our neighbours. Passage-23
I must say that, beyond occasionally exposing me to
Passage-21 laughter, my constitutional shyness has been of no
India accounts for nearly a fifth of the world's child disadvantage whatever. In fact I can see that, on the
deaths. In terms of numbers, it is the highest in the contrary, it has been all to my advantage. My hesitancy
world─nearly 16 lakhs every year. Of these, more than in speech, which was once an annoyance, is now a
half die in the first month of life. Officials believe that pleasure. Its greatest benefit has been that it has taught
the reason for this is the absence of steps to propagate me the economy of words. I have naturally formed the
basic health practices relating to breast feeding and habit of restraining my thoughts. And I can now give
immunisation. Also the large reproductive population of myself the certificate that a thoughtless word hardly ever
2.6 crore remains bereft of care during the critical phases escapes my tongue or pen. I do not recollect ever having
had to regret anything in my speech or writing. I have
of pregnancy and post-delivery. Added to this is the
thus been spared many a mishap and waste of time.
prevalence of child marriages, anaemia among young Experience has taught me that silence is part of the
women and lack of focus on adolescent sanitation, all of spiritual discipline of a votary of truth. Proneness to
which impact child death rates. exaggerate, to suppress or modify the truth, wittingly or
26. Which is the critical inference that can be made unwittingly, is a natural weakness of man, and silence is
from the above passage? necessary in order to surmount it. A man of few words
(a) A lot of Indians are illiterate and hence do not will rarely be thoughtless in his speech; he will measure
recognize the value of basic health practices. every word. We find so many people impatient to talk.
(b) India has a very huge population and the There is no chairman of a meeting who is not pestered
government alone cannot manage public health with notes for permission to speak. And whenever the
services. permission is given the speaker generally exceeds the
time-limit, asks for more time, and keeps on talking
(c) Universalization and integration of maternal
without permission. All this talking can hardly be said to
health and child health services can effectively be of any benefit to the world. It is so much waste of
address the problem. time. My shyness has been in reality my shield and
(d) The nutrition of women in child bearing age does buckler. It has allowed me to grow. It has helped me in
not affect child mortality rate. my discernment of truth.
IAS (Pre) IInd Paper CSAT, 2015 75 YCT
28. The author says that a thoughtless word hardly 32. A student has to opt for 2 subjects out of 5
ever escapes his tongue or pen. Which one of the subjects for a course, namely, Commerce,
following is not a valid reason for this? Economics, Statistics, Mathematics I and
(a) He has no intention to waste his time. Mathematics II. Mathematics II can be offered
(b) He believes in the economy of words. only if Mathematics I is also opted. The number
(c) He believes in restraining his thoughts. of different combinations of two subjects which
(d) He has hesitancy in his speech. can be opted is
(a) 5 (b) 6
Ans. (a) : The writer says that hardly a word comes (c) 7 (d) 8
out of his tongue or pen without thinking. Because he
Ans. (c) :
pays a lot of attention to his word selection or balance In two types of situations,
in speech or control of thoughts. Hence, option (b), (c) 1. Taking mathematics II.
& (d) are valid reasons and option (a) is invalid. 2. Not taking mathematics II.
(i) One can take mathematics I and mathematics II
29. The most appropriate reason for the author to be only in one way.
spared many a mishap is that (ii) 2 out of the remaining 3 can be selected in the way
3
(a) he hardly utters or writes a thoughtless word. C2 = 6.
(b) he is a man of immense patience. Hence, the total number of ways = 1 + 6 = 7
Hence option (c) will be the correct answer.
(c) he believes that he is a spiritual person.
(d) he is a votary of truth. 33. A person ordered 5 pairs of black socks and some
pairs of brown socks. The price of a black pair
Ans. (a) : The author categorically states in the was thrice that of a brown pair. While preparing
passage that I can now testify to myself that hardly any the bill, the bill clerk interchanged the number of
words come out of my tongue or pen without thought. black and brown pairs by mistake which
In this way I am saved from many dangers and also increased the bill by 100%. What was the
number of pairs of brown socks in the original
saved a lot of time. Hence, on this basis option (a) is order?
the correct answer. (a) 10 (b) 15
30. For the author, silence is necessary in order to (c) 20 (d) 25
surmount Ans. (d) :
Let the number of pairs of brown socks be X.
(a) constitutional shyness. Let the cost of a pair of brown socks be Rs. 1.
(b) hesitancy in speech. Hence, the value of the pair of black socks = Rs. 3
(c) suppression of thoughts. Initial Bill should be Rs. (5 × 3 + x × 1)
(d) tendency to overstate. Bill after exchange = (x × 3 + 5 × 1)
Now, bill has increased by 100%
Ans. (d) : In the passage, the author says that the Hence, it is double of the initial bill.
natural weakness of man is that he often exaggerates, Therefore,
knowingly or unknowingly. Silence is necessary to 3x + 5 = 2(15 + x)
overcome this. Hence, option (d) is the correct answer. x = 30 – 5
x = 25
LOGICAL & ANALYTICAL ABILITY So, option (d) is the correct answer.
31. A selection is to be made for one post of Principal 34. The number of persons who read magazine X
and two posts of Vice-Principal. Amongst the six only is thrice the number of persons who read
candidates called for the interview, only two are magazine Y. The number of persons who read
eligible for the post of Principal while they all are magazine Y only is thrice the number of persons
eligible for the post of Vice-Principal. The who read magazine X. Then, which of the
number of possible combinations of selectees is following conclusions can be drawn?
(a) 4 (b) 12 1. The number of persons who read both the
(c) 18 (d) None of the above magazines is twice the number of persons who
read only magazine X.
Ans. (d) : First of all, out of two eligible characters, 2. The total number of persons who read either one
the principal can be selected in two ways. Thereafter magazine or both the magazines is twice the
the Vice-Principal can be selected from the remaining number of persons who read both the magazines.
5 candidates in the 5C2 mode. Hence, total number of Select the correct answer using the code given
variants, below :
= 2 × 5C2 (a) 1 only (b) 2 only
= 2 × 10 = 20 (c) Both 1 and 2 (d) Neither 1 nor 2
Hence option (d) is the correct answer. Ans. (*) : This question was dropped.
IAS (Pre) IInd Paper CSAT, 2015 76 YCT
35. Consider the following statements followed by 38. Two men, Anil and David, and two women,
two conclusions : Shabnam and Rekha are in a sales group. Only
Statements : Some men are great. two speak Tamil. The other two speak Marathi.
Some men are wise. Only one man and one woman can drive a car.
Conclusion I : Men are either great or wise. Shabnam speaks Marathi. Anil speaks Tamil.
Conclusion II : Some men are neither great nor wise. Both Rekha and David can drive.
Which one of the following is correct? Which of the following statements is true?
(a) Only conclusion I is valid (a) Both the Tamil speakers can drive a car.
(b) Only conclusion II is valid (b) Both the Marathi speakers can drive a car.
(c) Both the conclusions are valid (c) Both of those who can drive a car speak Marathi.
(d) Neither of the conclusions is valid (d) One of those who can drive a car speaks Tamil.
Ans. (d) : Ans. (d) :
Wise men great Let ∆ represent male and O represents female.
Tamil ∆ ∆ Driver
Men
Anil David
Marathi O O Driver
Shabnam Rekha
So, it is clear that either David or Rekha is a Tamil
I. Men are either great or wise - It is possible but not Speaker.
definite, hence false. 39. A society consists of only two types of people -
II. Some men are neither great nor wise - It is possible fighters and cowards. Two cowards are always
but not definite, hence false. friends. A fighter and a coward are always
Hence, neither conclusion I nor conclusion II is valid. enemies. Fighters are indifferent to one another.
36. Consider the following statements : If A and B are enemies, C and D are friends, E
1. Some claim to have seen UFOs (Unidentified and F are indifferent to each other, A and E are
Flying Objects). not enemies, while B and F are enemies.
2. Life on other heavenly bodies is considered to be Which of the following statements is correct?
a possibility. (a) B, C and F are cowards.
3. Voyage to space is now an established fact. (b) A, E and F are fighters.
From the above statements, it may be concluded (c) B and E are in the same category.
that (d) A and F are in different categories.
(a) UFOs are heavenly bodies Ans. (b) :
(b) UFOs are sent from other heavenly bodies Fighters Cowards
(c) Some living species in other heavenly bodies are A B
more intelligent than man F C
(d) Nothing definite can be said about the UFOs E D
Ans. (d) : According to question, there is no direct Hence, option (b) is the correct answer.
connection between UFO celestial bodies and space 40. Candidates in a competitive examination
travels. Hence it is clear that option (d) is the correct consisted of 60% men and 40% women. 70%
answer. men and 75% women cleared the qualifying test
37. In a society it is customary for friends of the and entered the final test where 80% men and
same sex to hug and for friends of opposite sex to 70% women were successful.
shake hands when they meet. A group of friends Which of the following statements is correct?
met in a party and there were 24 handshakes. (a) Success rate is higher for women.
Which one among the following numbers (b) Overall success rate is below 50%
indicates the possible number of hugs? (c) More men cleared the examination than women.
(a) 39 (b) 30 (d) Both (a) and (b) above are correct.
(c) 21 (d) 20 Ans. (c) : Let initial population be 100.
Ans. (c) : Let the number of men and women be m & f Men Women
m × f = 24 ↓ 60% ↓ 40%
(1) 24 × 1
(2) 12 × 2 60 40
(3) 4 × 6
Number of hugs in the first position ↓ 70% ↓ 7 5%
= 24C2 = 273
Number of hugs in the second position 42 30
12 2
= C2 + C2 = 67 ↓ 80% ↓ 70%
Number of hugs in the third position
4 6
= C2+ C2 = 6+15 = 21 33 21
Only one of the three values in given so the answer It clear that more men cleared the examination than
will be 21. women.
IAS (Pre) IInd Paper CSAT, 2015 77 YCT
41. In a box of marbles, there are three less white group can speak two languages only and one
marbles than the red ones and five more white person can speak all the three languages, then
marbles than the green ones. If there are a total how many persons are there in the group?
of 10 white marbles, how many marbles are there (a) 21 (b) 22
in the box? (c) 23 (d) 24
(a) 26 (b) 28 Ans. (c) :
(c) 32 (d) 36
Ans. (b) : Number of white marbles = 10
Number of red marbles = 10 + 3 = 13
Number of green marbles = 10 – 5 = 5
Total number of marbles in the box = 10+13+5=25
42. In a town, 45% population read magazine A,
55% read magazine B, 40% read magazine C,
30% read magazines A and B, 15% read
magazines B and C, 25% read magazines A and
C; and 10% read all the three magazines. What
percentage do not read any magazine? ∴ The correct answer is option (c).
(a) 10% (b) 15% 45. In a parking area, the total number of wheels of
(c) 20% (d) 25% all the cars (four-wheelers) and scooters/
Ans. (c) : motorbikes (two-wheelers) is 100 more than twice
the number of parked vehicles. The number of
cars parked is
(a) 35 (b) 45
(c) 50 (d) 55
Ans. (c) :
Let, number of cars = a
Number of two – wheelers = b
4a + 2b = 2(a + b) + 100
2a = 100
a = 50
∴ Correct answer is option (c).
Number of population who read the magazine
= (45 + 55 + 40) – (30 + 25 + 15) + 10 46. The mangroves can shed tons of leaves per acre
= 140 – 70 + 10 = 80% every year; fungi and bacteria break down this
Number of population who do not read magazine leaf litter and consume it, they then are
= 100 – 80 = 20% consumed by tiny worms and crustaceans, which
in turn feed small fish, which feed larger fish and
43. Out of 130 students appearing in an examination, birds and crocodiles.
62 failed in English, 52 failed in Mathematics,
whereas 24 failed in both English and Which among the following is the most logical
Mathematics. The number of students who inference of the above statement?
passed finally is (a) Coastal areas cannot have food chains without
(a) 40 (b) 50 mangroves.
(c) 55 (d) 60 (b) Mangroves are an essential component of all
Ans. (a) : marine ecosystems.
(c) Mangroves have a crucial role in some of the
coastal food chains.
(d) The composition of marine flora and fauna is
largely determined by mangroves.
Ans. (c) : It is clearly known from reading the passage
that mangrove forests play a crucial role in certain
coastal food chains. Hence, option (C) is the correct
Total students = 130 answer.
Total students failed = (62 + 52) – 24
= 114 – 24 = 90 47. "By liberty I mean the eager maintenance of that
Number of students passed finally atmosphere in which men have the opportunity
= 130 – 90 to be their best selves."
= 40 Which one of the following expresses the view
44. In a group of persons travelling in a bus, 6 implied in the above statement?
persons can speak Tamil, 15 can speak Hindi and (a) Liberty is the absence of restraint on human
6 can speak Gujarati. In that group none can action.
speak any other language. If 2 persons in the (b) Liberty is what law permits people to perform.
IAS (Pre) IInd Paper CSAT, 2015 78 YCT
(c) Liberty is the ability to do what one desires. 51. All good athletes want to win and all athletes who
(d) Liberty is the maintenance of conditions for the want to win eat a well-balanced diet; therefore all
growth of human personality. athletes who do not eat a well-balanced diet are
Ans. (d) : The view implied in the statement in bad athletes.
question, liberty is to maintain the conditions The best conclusion from this statement is that
necessary for the growth of human personality. (a) no bad athlete wants to win.
Emphasizes perspective. (b) no athlete who does not eat a well-balanced diet
48. Twelve people form a club. By picking lots, one of is a good athlete.
them will host a dinner for all once in a month. (c) every athlete who eats a well-balanced diet is a
The number of dinners a particular member has good athlete.
to host in one year is (d) all athletes who want to win are good athletes.
(a) One (b) Zero Ans. (b) : The given statements can be represented as-
(c) Three (d) Cannot be predicted
Ans. (d) : How many times particular member will
have host the dinner in a year? It cannot be known
clearly as it is chosen at random so may be not
everyone gets a chance. Hence, option (d) is the
correct answer.
49. Consider the following statements :
It is stated that all good athletes want to win. It does
1. A man had a wife, two sons and two daughters in
not mean that no one wants to win. Hence, (b) is true.
his family.
There can be people other than good athletes, who eat
2. The daughters were invited to a feast and the
well balanced diet.
male members of the family went out to take part
in a picnic.
3. The man's father did not return from his work. GENERAL MENTAL APTITUDE
Which of the following statements is true? 52. The graph below depicts the earnings of A and B
(a) Only the man's wife was left at home. over the period 2000 to 2010 :
(b) It is likely that the man's wife was left at home.
(c) None was left at home.
(d) More than one person was left at home.
Ans. (b) : Since, the father has not returned from
work. The male members have gone for a picnic. The
daughters are out for the feast. We can say that there is
a strong possibility that only the wife of that person is
alone in the house. Hence, option (b) is the correct
answer.
50. Geeta : Naresh has become a better boxer since he
started meditation.
Radha : Impossible. A boxer's most important asset
is his aggressiveness. From the graph, which one of the following can
Radha's statement reflects her belief that be concluded?
(a) meditation tends to make a person less (a) On the average A earned more than B during this
aggressive. period.
(b) meditation has little or no effect on the person (b) On the average B earned more than A during this
who practises it. period.
(c) Naresh was a poor boxer earlier because he was (c) The earnings of A and B were equal during this
not aggressive enough. period.
(d) Naresh would not have taken to meditation as he (d) The earnings of A were less as compared to B
was a boxer. during this period.
Ans. (a) : Radha refuses to accept Geeta's logic Ans. (a) :
because she believes that one cannot become a good From given years 2000 to 2007 Earnings of A >
boxer by meditating, as the most important quality of Earnings of B,
any boxer is his aggression. Therefore, we can easily So, Average of A > Average of B
conclude that Radha believes that meditation reduces Hence, It can be conducted that the average of A
aggression. Hence, option (a) is the correct answer. earned more than B during this period.
IAS (Pre) IInd Paper CSAT, 2015 79 YCT
53. Each of the six different faces of a cube has been 56. The proportion of expenditure on various items
coated with a different colour i.e. V, I, B, G, Y by two families A and B are represented in the
and O. Following information is given : following Bar Charts :
1. Colours Y, O and B are on adjacent faces. 50% Food 10% Food
2. Colours I, G and Y are on adjacent faces.
30% Other 60% Other
3. Colours B, G and Y are on adjacent faces.
items items
4. Colours O, V and B are on adjacent faces.
20% Education 30% Education
Which is the colour of the face opposite to the
face coloured with O? Family A Family B
(a) B (b) V Total expenditure : Total expenditure :
(c) G (d) I ` 20,000 per month ` 1,00,000 per month
Ans. (c) : From 2 and 3, I and B are on opposite faces. From these charts, we can conclude that
From 1 and 4, Y and V are adjacent to O. (a) Family A spent more money on food than Family B.
(b) Family B spent more money on food than Family A.
(c) Family A and Family B spent the same amount
on food.
(d) The expenditure on food by Family A and
Family B cannot be compared.
Ans. (c) :
Hence, G is the colour of the face opposite to the face Expenditure on food by A = 50% of 20,000 = 10,000
coloured with O Expenditure on food by B = 10% of 1,00,000
54. If ABC × DEED = ABCABC; where A, B, C, D = 10,000
and E are different digits, what are the values of ∴The correct answer is option (c).
D and E? 57. Usha runs faster than Kamala, Priti runs slower
(a) D = 2, E = 0 (b) D = 0, E = 1 than Swati, Swati runs slower than Kamala. Who
(c) D = 1, E = 0 (d) D = 1, E = 2 is the slowest runner?
Ans. (c) : Whenever are multiply a three digit number, (a) Kamala (b) Priti
let us say xyz to 1001 then we get xyz xyz as a result. (c) Swati (d) Usha
Implies, DEED represents 1001.
Ans. (b) :
Hence, the values of D and E are 1 and 0
Usha > Kamala, Swati > Priti and Kamala > Swati
55. Year-wise variation of the price of a certain ∴ Therefore sequence will be-
commodity is shown in the following graph : Usha > Kamala > Swati > Priti
⇒ Correct answer is option (b)
58. Between 6 PM and 7 PM the minute hand of a
clock will be ahead of the hour hand by 3 minutes
at
(a) 6:15 PM (b) 6:18 PM
(c) 6:36 PM (d) 6:48 PM
Ans. (c) : Between 6 pm and 7 pm at 6 : 36 the minute
hand of a clock will be 3 minutes ahead of the hour
hand.
59. There are 5 tasks and 5 persons. Task-1 cannot
be assigned to either person-1 or person-2. Task-
2 must be assigned to either person-3 or person-4.
The price of the commodity in the year 1990 Every person is to be assigned one task. In how
(a) must have been ` 10/- many ways can the assignment be done?
(b) must have been ` 12/- (a) 6 (b) 12
(c) must have been anywhere between ` 10/- and ` 20/- (c) 24 (d) 144
(d) is higher than that in the year 1991 Ans. (c) : Case I : Let us assume that task 2 was
Ans. (c) : On looking at the graph it is clearly known assigned to person 3.
that in 1990 the price should be between Rs. 10 to Rs. Case II : Let us assume that task 2 was assigned to
20. Hence, option (c) is the correct answer. person 4.
IAS (Pre) IInd Paper CSAT, 2015 80 YCT
Task Case-1 Case-2 63. A cow costs more than 4 goats but less than 5
goats. If a goat costs between ` 600 and ` 800,
1 Two ways (4/5) Two ways (3/5) which of the following is a most valid conclusion?
2 3 4 (a) A cow costs more than ` 2,500
3 Three ways Three ways (b) A cow costs less than ` 3,600
(c) A cow costs between ` 2,600 and ` 3,800
4 Two ways Two ways (d) A cow costs between ` 2,400 and ` 4,000
5 One way One way Ans. (d) : Let 'g' be the cost of goats
In case I, the number of ways are = 2 × 3 × 2 × 1 = 12 4g < Cow < 5g
In case II, the number of ways are = 2 × 3 × 2 × 1 = 12 Now, given a goat price = ` 600
2400 < Cow < 3000 …………..(i)
Total number of ways = 12 + 12 = 24 Given, a goat price = `800
Hence, there are 24 ways to do the assignments. 3200 < Cow < 4000…………(ii)
Directions for the following 2 (two) items : Hence, it can be seen that from above equations that coast
of cow is between
Read the following passage and answer the 2 ` 2400 and ` 4000.
(two) items that follow :
A, B, C, D, E and F are cousins. No two cousins 64. If A runs less fast than B, and B runs as fast but
are of the same age, but all have birthdays on the not faster than C; then, as compared to A, C runs
same day of the same month. The youngest is 17 years (a) slower than A
old and the oldest E is 22 years old. F is somewhere (b) faster than A
between B and D is age. A is older than B. C is older (c) with same speed as A
than D.A. is one year older than C. (d) Given data is not sufficient to determine
60. Which one of the following is possible? Ans. (b) :
B is faster than A.
(a) D is 20 years old (b) F is 18 years old C has same speed as A or more than that.
(c) F is 19 years old (d) F is 20 years old ∴ The correct answer is option (b).
Ans. (b) :
E A C D F B 65. Each of A, B, C and D has ` 100. A pays ` 20 to B,
who pays ` 10 to C, who gets ` 30 from D. In this
22 21 20 19 18 17
context, which one of the following statements is
Means not correct?
E A C B F D (a) C is the richest.
22 21 20 19 18 17 (b) D is the poorest.
∴ The correct answer is option (b). (c) C has more than what A and D have together.
61. What is the number of logically possible orders of (d) B is richer than D.
all six cousins in terms of increasing age? Ans. (c) :
(a) 1 (b) 2 A B C D
(c) 3 (d) 4 100 100 100 100
Ans. (b) : Based on increasing age of all six cousins, it (i) 80 120 100 100
in logically possible to place them in 2 sequences as (ii) 80 110 110 100
shown in the above explanation. (iii) 80 110 140 70
62. In a plane, line X is perpendicular to line Y and ∴ The correct answer is option (c).
parallel to line Z; line U is perpendicular to both
66. Examine the following statements :
lines V and W; line X is perpendicular to line V.
1. Lady's finger is tastier than cabbage.
Which one of the following statements is correct?
(a) Z, U and W are parallel. 2. Cauliflower is tastier than lady's finger.
(b) X, V and Y are parallel. 3. Cabbage is not tastier than peas.
(c) Z, V and U are all perpendicular to W. The conclusion that can be drawn from these
(d) Y, V and W are parallel. statements is that
Ans. (d) : From the given statements, following graph (a) peas are as tasty as lady's finger.
can be drawn (b) peas are as tasty as cauliflower and lady's finger.
V Y W (c) cabbage is the least tasty of the four vegetables.
(d) cauliflower is tastier than cabbage.
X Ans. (d) :
Z 1. Lady finger > Cabbage
U 2. Cauliflower > Lady finger > Cabbage
3. Cabbage ≤ Peas
∴ It is clear that cauliflower is tastier than cabbage.
∴ The correct answer is option (d). The correct answer is option (d).
IAS (Pre) IInd Paper CSAT, 2015 81 YCT
67. Shahid and Rohit start from the same point in Ans. (a) :
opposite directions. After each 1 km, Shahid 32 – 3 = 6
always turns left and Rohit always turns right. 82 – 8 = 56
Which of the following statements is correct? 102 – 10 = 90
(a) After both have travelled 2 km, the distance 22 – 2 = 2
between them is 4 km. 52 – 5 = 20
(b) They meet after each has travelled 3 km. 12 – 1 = 0
(c) They meet for the first time after each has ∴ The correct answer is option (a).
travelled 4 km. 70. What is the missing number `X' of the series 7, X,
(d) They go on without ever meeting again. 21, 31, 43?
(a) 11 (b) 12
Ans. (b) :
(c) 13 (d) 14
Ans. (c) : The series will be-
+6→13 
7  +8→ 21  +10→ 31 
+12→ 43
∴ The correct answer is option (c)
71. Four cardboard pieces of specific shapes are
shown in the following figure :

∴It is clear from the figure that option (b) is the


correct answer.
68. Consider the figures given below :

Which one of the following figures given can be


To fit the question mark, the correct answer is formed by joining these pieces together?

(a)
(a)

(b) (b)

(c)
(c)

(d) (d)

Ans. (a) : The triangle is rotating clockwise and Ans. (d) : The following figure can be formed by
reverse, while the rest of the images in the figure - •, joining these pieces together is-
∎, O are rotating anti-clockwise.
69. Consider the following matrix :

3 8 10 2 ? 1 72. In a test, a candidate attempted only 8 questions


and secured 50% marks in each of the questions.
6 56 90 2 20 0 If he obtained a total of 40% in the test and all
questions in the test carried equal marks, how
What is the missing number at `?' in the matrix? many questions were there in the test?
(a) 5 (b) 0 (a) 8 (b) 10
(c) 7 (d) 3 (c) 15 (d) 16
IAS (Pre) IInd Paper CSAT, 2015 82 YCT
Ans. (b) : Let the number of question is test be X 76. The monthly incomes of Peter and Paul are in the
number of solved question = 4 ratio of 4 : 3. Their expenses are in the ratio of 3 :
2. If each saves ` 6,000 at the end of the month,
4
× 100 = 40 ⇒ × = 10 their monthly incomes respectively are (in `)
X (a) 24,000 and 18,000 (b) 28,000 and 21,000
∴ The correct answer is option (b). (c) 32,000 and 24,000 (d) 34,000 and 26,000
73. A father is nine times as old as his son and the Ans. (a) :
mother is eight times as old as the son. The sum Income = 4x : 3x
of the father's and the mother's age is 51 years. Expense = 3y : 2y
What is the age of the son? Saving = Income – Expense = 6000
(a) 7 years (b) 5 years 4x – 3y = 6000 and 3x – 2y = 6000
(c) 4 years (d) 3 years 4x – 3y = 3x – 2y
x = y = 6000
Ans. (d) : ∴ The monthly salary will be 24,000 and 18,000.
Let, the present age of father be F.
The present age of mother be M and son be S. 77. Two cities A and B are 360 km apart. A car goes
F = 9S from A to B with a speed of 40 km/hr and returns
M = 8S to A with a speed of 60 km/hr. What is the
9S + 8S = 51 average speed of the car?
17S = 51 (a) 45 km/hr (b) 48 km/hr
(c) 50 km/hr (d) 55 km/hr
S=3
∴ The correct answer is option (d). Ans. (b) :
Since, the distance covered is the same, the average
74. Four persons A, B, C and D consisting of two 2S1S 2 2 × 40 × 60
married couples are in a group. Both the women speed = = = 48 km/hr.
are shorter than their respective husbands. A is S1 + S 2 40 + 60
the tallest among the four. C is taller than B. D is 78. In a 500 metres race, B starts 45 metres ahead of
B's brother. In this context, which one of the A, but A wins the race while B is still 35 metres
following statements is not correct? behind. What is the ratio of the speeds of A to B
(a) All four have family ties. assuming that both start at the same time?
(b) B is the shortest among the four. (a) 25 : 21 (b) 25 : 20
(c) C is taller than D. (c) 5 : 3 (d) 5 : 7
(d) A is B's husband. Ans. (a) :
Ans. (c) : A = 500
Husband Wife B = 500 – 45 – 35 = 420
A B Since, total time taken by A and B are same.
D C ∴The ratio of speed of A and B = 500 : 420 = 25 : 21
• A is the tallest it mean A must be husband. 79. Two equal glasses of same type are respectively
• D is brother of B it means husband of the other will 1/3 and 1/4 full of milk. They are then filled up
be D. with water and the contents are mixed in a pot.
• Since, it is given that the height of the wife is less What is the ratio of milk and water in the pot?
than that of the husband. (a) 7 : 17 (b) 1 : 3
Hence, as the statement the height of wife C. Cannot (c) 9 : 21 (d) 11 : 23
be more than that of husband (D). Ans. (a) :
∴ The correct answer is option (C). Let the volume of each glass be 60 litres
Glass A Glass B
Milk = 20 litres Milk = 15 litres
GENERAL MATHEMATICS Water = 40 litres Water = 45 litres
NUMERAL ABILITY Required Ratio =
Milk 35 7
= = = 7 :17
75. Two pipes A and B can independently fill a tank Water 85 17
completely in 20 and 30 minutes respectively. If 80. An automobile owner reduced his monthly
both the pipes are opened simultaneously, how petrol consumption when the prices went up. The
much time will they take to fill the tank price-consumption relationship is as follows :
completely? Price (in ` per litre) 40 50 60 75
(a) 10 minutes (b) 12 minutes Monthly consumption (in litres) 60 48 40 32
(c) 15 minutes (d) 25 minutes If the price goes up to ` 80 per litre, his expected
consumption (in litres) will be
Ans. (b) : (a) 30 (b) 28
Let capacity of tank be 60 litres (LCM of 20 and 30). (c) 26 (d) 24
Pipe A can fill 3 litres in a minutes. Pipe B can fill 2
litres in one minutes. Both pipes can fill 5 litres in one Ans. (a) : The product of price (in Rs./litre) and the
monthly consumption (in litres) in constraint is equal
minute. to 2400.
60 Expected consumption when the price goes upto ` 80
Time required = = 12 minutes
5 2400
per litre = = 30 litres.
∴ The correct answer is option (c). 80
IAS (Pre) IInd Paper CSAT, 2015 83 YCT
UNION PUBLIC SERVICE COMMISSION
Civil Services (Preliminary Exam) - 2016
CSAT : PAPER-II
(Chapterwise Analysis with Explanation)
Time : 2 hours Maximum Number : 200

Directions for the following 6 (six) items : 1. According to the passage, which of the following
Read the following two passages and answer the items factor/factors led to the adverse consequences
that follow each passage. Your answers to these items for governance/public administration?
should be based on the passages only. 1. Inability of civil services to strike a balance
Passage-1 between internal and external accountabilities
Accountability, or the lack of it, in governance 2. Lack of sufficient professional training to the
generally, and civil services, in particular, is a major officers to all India Services .
factor underlying the deficiencies in governance and 3. Lack of Proper service benefits in civil services
public administration. Designing an effective frame work 4. Lack of Constitutional provisions to define the
for accountability has been a key element of the reform respective roles of professional civil services vis-a
agenda. A fundamental issue is wither civil services –vis political executive in this context
should be accountable to the political executive of the Select the correct answer using the code given
day or to society at large. In other words, how should below :
internal and external accountability be reconciled? (a) 1 only
Internal accountability is sought to be achieved by
(b) 2 and 3 only
internal performance monitoring, official supervision by
bodies like the Central Vigilance Commission and (c) 1 and 4 only
Comptroller and Auditor General and judicial review of (d) 2, 3 and 4
executive decisions. Articles 311 and 312 of the Indian Ans. (c) : According to the passage, the inability of the
Constitution provide job security and safeguards to the civil services to strike a balance between internal and
civil services, especially the All India Services. The external accountability and the absence of
framers of the Constitution had envisaged that provision constitutional provisions defining their respective roles
of these safeguards would result in a civil service that is in the professional civil services as compared to the
not totally subservient to the political executive but will political executive.
have the strength to function in larger public interest.
The need to balance internal and external accountability 2. With reference to the passage, the following
is thus built into the Constitution. The issue is where to assumptions have been made:
draw the line. Over the years, the emphasis seems to 1. Political executive is an obstacle to the
have tilted in favor of greater internal accountability of accountability of the civil services to the society
the civil services to the political leaders of the day who 2. In the present framework of Indian polity, the
in turn are expected to be externally accountable to the Political executive is no longer accountable to the
society at large through the election process. This system society
for seeking accountability to Society has not worked out, Which of these assumptions is /are valid ?
and has led to several adverse consequences for (a) 1 only
governance.
(b) 2 only
Some special measures can be considered for
improving accountability in civil services. Provisions of (c) Both 1 and 2
articles 311 and 312 should be reviewed and laws and (d) Neither 1 nor 2
regulations framed to ensure external accountability of Ans. (a) : Assumption 1 states that political executive
civil services. The proposed Civil Services Bill seeks to hinders the working of civil services and its role in
address some of these requirements. The respective roles public interest .
of professional civil Services Bill seeks to address some This is correct because the emphasis is on internal
of these requirements. The respective roles of accountability to the political leaders and the larger
professional civil services and the political executive public interest is undermined.
should he defined so that professional managerial
functions and management of civil services are 3. Which one of the following is the essential
depoliticized. For this purpose, effective statutory civil massage implied by this passage ?
service boards should be created at the centre and in the (a) Civil services are not accountable to the
states. Decentralization and devolution of authority to society they are serving
bring government and decision making closer to the (b) Educated and enlightened persons are not
people also helps to enhance accountability. taking up political leadership
IAS (Pre) GS IInd Paper (CSAT), 2016 84 YCT
(c) The framers of the Constitution did not 6. Which one of the following is the crux of this
envisage the problems being encountered by passage ?
the civil services (a) Our duties to one another derive from our
(d) There is a need and scope for reforms to religious traditions
improve the accountability of civil services (b) Having relationship to the divine principle is
Ans. (d) : The essential message underlying the a great virtue
passage is that there is a need and scope for reforms to (c) Balance between rights and duties is crucial
enhance the accountability of the civil services. Some to the delivery of justice in a society
such rules can be made in which the responsibility of (d) Religious concept of rights is primarily
the civil servants can be made and the work can be derived from our relationship to god
completed on time. Ans. (c) : The point of this passage is that the balance
4. According to the passage, which one of the between rights and duties is crucial for the delivery of
justice in the society. Complete justice is possible only
following is not a means of enhancing internal
when the ruling class performs its duties fully, which is
accountability of civil services ? in the public interest, this will fulfill the demand of
(a) Better job security and safeguards rights.
(b) Supervision by Central Vigilance Commission
(c) Judicial review of executive decisions 7. A ate grapes and pineapple; B ate grapes and
(d) Seeking accountability through enhanced oranges; C ate oranges, pineapple and apple; D
participation by people in decision making ate grapes, apple and pineapple. After taking
fruits, B and C fell sick . In the light of the
process
above facts, it can be said that the cause of
Ans. (d) : According to the passage, finding sickness was :
accountability by increased people's participation in (a) Apple (b) Pineapple
the decision-making process is not a means for (c) Grapes (d) Oranges
promoting internal accountability of the civil services.
But for better services, it is necessary to give Ans. (d) :
importance to the participation of the people, only then A - Grapes, Pineapples
these will be improvement in the bureaucracy and B - Grapes, Oranges
accountability will increase. C - Oranges, Pineapples, apples
D - Grapes, Apples, Pineapples
Passage -2
In general religious traditions stress our duty to god, ∵ After taking fruits B and C fell sick.
or to some universal ethical principle. Our duties to one ∴ The fruits which was eaten by B and C was oranges.
another derive from these. The religious concept of rights hence, it can be said that the cause of sickness was
is primarily derived from our relationship to this divinity oranges.
or principles and the implication it has on our other 8. Consider the following statements :
relationships. This correspondence between rights and 1. The rate of population growth is increasing in
duties is critical to any further understanding of justice the country
But for justice to be practiced; virtue, rights and duties 2. The death rqate is declining faster in the
cannot remain formal abstraction. They must be country compared to birth rate
grounded in a community (common unity) bound 3. The birth rate is declining faster in the
together by a sense of common union (communion).Even country compared to death rate
as a personal virtue, this solidarity is essential to the 4. Rural–urban migration is taking place
practice and understanding of justice. regularly in the country
5. With reference to the passage, the following Which one of the following conclusions may be
assumptions have been made : true the light of the above facts ?
1. Human relationships are derived from their (a) The rate of population growth is increasing
religious traditions due to rural –urban migration
2. Human beings can be duty bound only if they (b) The rate of population growth is increasing
believe in god due to decline in death rate only
3. Religious traditions are essential to practice and (c) The rate of population growth is increasing
understand justice due to increase in birth rate only
Which of these assumption (s) is /are valid? (d) The rate of population growth is increasing
(a) 1 only due to faster decline in death rate than in
(b) 2 and 3 birth rate
(c) 1 and 3 only Ans. (d) : In the light of the above facts, it is
(d) 1, 2 and 3 concluded that the population growth rate is increasing
Ans. (a) : The religious concept of rights is derived due to the rapid decline in the death rate. The reason
primarily from our relationship to this divinity or for the reduction in mortality is better health. For this
principle and its implications for our other reason life expectancy is increasing. In spite of the
relationships. decrease in the birth rate, the population is increasing.

IAS (Pre) GS IInd Paper (CSAT), 2016 85 YCT


9. A person X was driving in a place where all (i) C is seated next to A
roads ran either north-south or east-west, (ii) A is seated two seats from D
forming a grid. Roads are at a distance of 1 km (iii) B is not seated next to A
from each other in a parallel. He started at the On the basis of above information, which of
intersection of two roads, drove 3 km north, 3 the following must be true ?
km west and 4 km south. Which further route 1. D is seated next to B
could bring him back to his starting point, if 2. E is seated next to A
the some route is not repeated? 3. D and C are separated by two seats
(a) 3 km east, then 2 km south Select the correct answer using the code give
(b) 3 km east, then 1 km north below :
(c) 1 km north, then 2 km west (a) 1 only
(d) 3 km south, then 1 km north (b) 1 and 2 only
Ans. (b) : (c) 3 only
(d) Neither 1 nor 2 nor 3
Ans. (b) : B
D

C E

A
3 km east, then 1 km north.
Only 1 & 2 most be true.
10. Consider the following statement :
‘‘We shall go either for a picnic or for 13. There are five hobby clubs in a college
trekking’’ photography, yachting, chess, electronics and
gardening. The gardening, group meets every
Which of the following, if true, would falsify
this claim ? second day, the electronics groups meet every
third day, the chess group meets every fourth
(a) We go for a picnic but not for trekking
day, the yachting group meets every fifth day
(b) Activities such as picnic and trekking are
and the photography group meets every sixth
encouraged by the health authorities
day. How many times do all the five groups
(c) We go for trekking and not for picnic
meet on the same day within 180 day?
(d) We do not go either for picnic or for trekking
(a) 5 (b) 18
Ans. (d) : If "we do not go either for picnic or for (c) 10 (d) 3
trekking" is true, then we shall go either for a picnic or
for trekking would falsify. Ans. (d) :
The L.C.M. of (2, 3, 4, 5, 6) = 60
11. There were 50 faculty member comprising, 30
males and the rest females. No male faculty Required number of days 180
=
member knew music, but many of the female 60
faculty members did. The Head of the =3
institution invited six faculty members to a tea
party by draw of lots. At the party it was 14. There are some nectar-filled flowers on a tree
discovered that no members knew music. The and some bees are hovering on it. If one bee
conclusion is that : lands on each flower, one bee will be left out. If
(a) the party comprised male faculty members two bees land on each flower, one flower will be
only left out. The number of flowers and bees
(b) the party comprised only those female faculty respectively are :
members who could not give renderings in (a) 2 and 4 (b) 3 and 2
music (c) 3 and 4 (d) 4 and 3
(c) the party comprised both male and female
faculty members Ans. (c) :
(d) nothing can be said about the gender Let number of bees = x
composition of the party and number of flowers = y
Ans. (d) : Among the invited people, there may be y×1+1=x
men as well as women who do not have knowledge of y+1=x ....... (i)
music. and 2(y – 1) = x ........(ii)
Hence, nothing can be said about the gender
From eqn (i) & (ii)
composition of the party.
y + 1 = 2(y – 1)
12. Five people A, B, C, D and E are seated about a
round table. Every chair is spaced equidistant y + 1 = 2y – 2
from adjacent chairs. 3=y

IAS (Pre) GS IInd Paper (CSAT), 2016 86 YCT


y=3 19. Who is the following is definitely a man ?
Putting the value of y in eqn (i) (a) P
(b) S
y+1=x
(c) Q
3+1=x
(d) None of the above
x=y
Ans. (c) : Since Q is the brother of P. Therefore 'Q' is
the number of flowers and bees respectivd are 3 and 4. definitely a man.
Directions for the following 5 (five) items : 20. There is an order of 19000 quantity of a
Consider the following information and answer the particular product from a customer. The firm
five item that follows : produces 1000 quantity of that product per out
There are five persons in a group-P, Q, R, S and of which 5% are unfit for sale. In how many
T. The group has one doctor, one lawyer and one days will the order be completed ?
artist. P and S are unmarried students. T is a (a) 18 (b) 19
man married to one of the group members. Q is (c) 20 (d) 22
the brother of P and is neither doctor nor artist. Ans. (c) : The number of days when the order be
R is not doctor . 19,000
completed ⇒
For 15 to 19 1000 − 1000 × 5
100
Since Q is a brother of P, so Q must be a man . 19000 19000
Now , Q and T (a married man) are male = =
1000 − 50 950
• P and S are unmarried, So T must be married to R. = 20 days.
So, R is a female. Direction for the following 5 (five) items : Read
P and S are unmarried students. the following two passages and answer the items that
Q is neither a doctor nor an artist. follow each passage. Your answers to these items
R is not a doctor should be based on the passages only.
• T must be a doctor Passage-1
Biomass as fuel for power, heat, and transport has
Out of Q and R, one is a lawyer and another is an artist.
the highest mitigation potential of all renewable sources.
And Q is not an artist, so he must be a lawyer. It comes from agriculture and forest residues as well as
∴ R is an artist. from energy crops. The biggest challenge in using
The solution of question no. 15 to 19:- biomass residues is a long-term reliable supply delivered
Person Profession to the power plant at reasonable costs; the key problems
P Unmarried student are logistical constraints and the costs of fuel collection.
Energy crops, if not managed properly, compete with
Q Lawyer food production and may have undesirable impacts on
R Artist food prices. Biomass production is also sensitive to the
S Unmarried Student physical impacts of a changing climate.
T Doctor Projections of the future role of biomass are
Male ⇀ probably overestimated, given the limits to the
and T ↽ R Female sustainable biomass supply, unless breakthrough
Married to
technologies substantially increase productivity. Climate-
15. Who is the doctor ? energy models project that biomass use could increase
(a) T (b) P nearly four-fold to around 150-200 exajoules, almost a
(c) Q (d) R quarter of world primary energy in 2050. However the
maximum sustainable technical potential of biomass
Ans. (a) : 'T' is the doctor. resources (both residues and energy crops) without
16. Who is the artist ? disruption of food and forest resources ranges from 80-
(a) P (b) Q 170 exajoules a year by 2050 and only part of this is
realistically and economically feasible. In addition, some
(c) R (d) T climate models rely on biomass-based carbon capture
Ans. (c) : 'R' is the artist. and storage, an unproven technology to achieve negative
17. Who is the spouse of R emissions and to buy some time during the first half of
(a) P (b) T the century.
Some liquid bio fuels such as corn-based ethanol,
(c) Q (d) S mainly for transport, may aggravate rather than
Ans. (b) : 'T' is the husband of R. ameliorate carbon emissions on a life-cycle basis.
18. Who is the lawyer ? Second generation befouls, based on lingo-cellulose
feedstock –such as straw, bagasse, grass and wood-hold
(a) P (b) Q the promise of sustainable production that is high-
(c) R (d) S yielding and emit low levels of green house gases, but
Ans. (b) : Q is the lawyer. these are still in the R & D stage.

IAS (Pre) GS IInd Paper (CSAT), 2016 87 YCT


21. What is/are the present constraint/ constraints Ans. (c) : Considering on emerging technologies,
in using biomass as fuel for power generation ? biomass as a fuel for power generation can help in
1. Lack of sustainable supply of biomass achieving negative emissions. It is characterized by
2. Biomass production competes with food sustainable production in biofuels in terms of biomass
production use.
3. Bio-energy may not always be low carbon on 24. With reference to the passage, following
life-cycle basis assumptions have been made :
Select the correct answer using the code given 1. Some climate-energy models suggest that the use
below of biomass as a fuel for power generation helps in
(a) 1 and 2 only mitigating greenhouse gas emissions.
(b) 3 only 2. It is not possible to use biomass as a fuel for power
(c) 2 and 3 only generation without disrupting food and forest
resources
(d) 1, 2 and 3
Which of these assumptions is / are valid ?
Ans. (d) : Lack of a sustainable supply of biomass
(a) 1 only
production competitive with food production and
(b) 2 only
bioenergy may not always be low carbon, depending on
the life cycle. The above facts are the existing (c) Both 1 and 2
constraints in using biomass as fuel for power (d) Neither 1 nor 2
generation. Ans. (a) : Some climate energy models suggest that the
22. Which of the following can lead to food use of biomass as a fuel for power generation is helpful
security problem ? in reducing greenhouse gas emissions. This written
assumption is made in the context of this passage.
1. Using agricultural and forest residues as
feedstock for power generation Passage–2
2. Using biomass for carbon capture and storage We are witnessing a dangerous dwindling of
biodiversity in our food supply. The green revolution is a
3. Promoting the cultivation of energy crops
mixed blessing. Over time farmers have come to rely
Select the correct answer using the code given heavily on broadly adapted, high yield crops to the
below? exclusion of varieties adapted to the local conditions.
(a) 1 and 2 only Monocropping vast fields with the same genetically
(b) 3 only uniform seeds helps boots yield and meet immediate
(c) 2 and 3 only hunger needs. Yet high-yield varieties are also
(d) 1, 2 and 3 genetically weaker crops that require expensive chemical
Ans. (b) : The problem of food security may arise due fertilizers and toxic pesticides. In our focus on increasing
to the promotion of the cultivation of energy crops. The the amount of food we produce today, we have
main food items will not be affected due to high accidentally put over selves at risk for food shortages in
production of coarse grains and Jatropha in the food future
items, otherwise there will be problem if the main food 25. Which among the following is the most logical
items are used as energy crops. and critical inference that can be made from
the above passage ?
23. In the context of using biomass, which of the
(a) In our agricultural practices, we have become
following is/are the characteristic/characteristics
heavily dependent on expensive chemical
of the sustainable production of biofuel.
fertilizers and toxic pesticides only due to
1. Biomass as a fuel for power generation could green revolution .
meet all the primary energy requirements of
(b) Monocroping vast fields with high-yield
the world by 2050.
varieties is possible due to green revolution
2. Biomass as a fuel for power generation does (c) Monocroping with high-yield varieties is the
not necessarily disrupt food and forest only way to ensure food security to millions
resources
(d) Green revolution can pose a threat to
3. Biomass as a fuel for power generation could biodiversity in food supply and food security
help in achieving negative emissions, given in the long run
certain nascent technologies.
Ans. (d) : The most logical and critical inference that
Select the correct answer using the code given comes from the above passage is that the green
below : revolution can present a threat to biodiversity in the
(a) 1 and 2 only food security in the long run. Because the root of the
(b) 3 only green revolution is to use chemical fertilizers in large
(c) 2 and 3 only quantities which are harmful for biodiversity and also
(d) 1, 2 and 3 not beneficial for the long term of the land.
IAS (Pre) GS IInd Paper (CSAT), 2016 88 YCT
26. A class starts at 11 : 00 am and lasts till 2 : 27 Ans. (d) : Sugar in per ml water ⇒
pm. Four periods of equal duration are held 30 1
during this interval. After every period, a rest A= ⇒
180 6
of 5 minutes is given to the students. The exact 40 1
duration of each period is: B= ⇒
280 7
(a) 48 minutes (b) 50 minutes
20 1
(c) 51 minutes (d) 53 minutes C= =
100 5
Ans. (a) : Let the time of each period = x min Hence,
∵ Number of period = 4 1 1 1
> >
∴ Number of interval = 3 5 6 7
According to question, C > A > B
4 × x + 3 × 5 = (2:27p.m-11 a.m.) ↓ ↓ ↓
4x + 15 = 3 hr 27 min Sweetest Sweeter Sweet
4x + 15 = 207 min Hence, the solution in vessel B is less sweet than that in
C.
4x = 207 – 15
29. In aid of charity, every student in a class
4x = 192
contributes as many rupees as the number of
x = 48 min. students in that class. With the additional
Each time A will go with his friends and last he will go contribution of Rs. 2 by one student only the
alone. total collection is Rs. 443. Then how many
students are there in the class?
27. Four friends A, B, C and D need to cross a (a) 12 (b) 21
bridge. A maximum of two persons can cross it (c) 43 (d) 45
at a time. It is night and they just have one
Ans. (b) : The number of student in the class
lamp. Persons that cross the bridge must carry
the lamp to find the way. A pair must walk = 443 − 2
together at a speed of slower person. After = 441
crossing the bridge, the person having faster = 21
speed in the pair will return with the lamp each 30. Anita‘s mathematics test had 70 problems
time to accompany another person in the carrying equal marks i.e., 10 arithmetic, 30
group. Finally, the lamp has to be returned at algebra and 30 geometry. Although she
the original place and the person who returns answered 70% of the arithmetic, 40% of the
the lamp has to cross the bridge, again without algebra and 60% of the geometry problems
lamp. To cross the bridge, the time taken by correctly, she did not pass the test because she
got less then 60% marks. The number of more
them is as follows : A : 1 minute , B : 2 minutes questions she would have to answer correctly
C : 7 minutes and D : 10 minutes. What is the to earn a 60% Passing marks is .
total minimum time required by all the friends (a) 1 (b) 5
to cross the bridge? (c) 7 (d) 9
(a) 23 minute (b) 22 minute Ans. (b) :
(c) 21 minute (d) 20 minute Number of questions correct by Anita
Ans. (a) : 70 40 60
⇒ 10 × + 30 × + 30 ×
The total time taken by A, B, C and D ⇒ 100 100 100
D (10 + 1) min + C (7 + 1) min + B (2 + 1) min + A (1) = 7 + 12 + 18
min = 37
⇒ (11 + 8 + 3 + 1) min The number of correct questions required to pass the
60
= 23 min exam = 70 × = 42
100
28. 30g of sugar was mixed in 180 ml water in a
∴The numberof more questions to earn passing mark
vessel A, 40 g of sugar Was mixed in 280 ml of
42 – 37 ⇒ 5
waterin vessel B and 20 g of sugar was mixed
in 100 ml of water in vessel C. The solution in 31. In the class, there are 18 very tall boys. If these
constitute three –fourths of the boys and the
vessel B is
total number of boys is two –thirds of the total
(a) sweeter than that in C number of students in the class, what is the
(b) sweeter than that in A number of girls in the class ?
(c) as sweet as that in C (a) 6 (b) 12
(d) less sweet than that in C (c) 18 (d) 21
IAS (Pre) GS IInd Paper (CSAT), 2016 89 YCT
Ans. (b) : Let the number of boys in the class Direction for the following 3 (three) items:
and the number of girls in the class Consider the given formation and answer the three
According to question, items that follow.
Six boxes A,B, C, D, E and F have been painted
3x 18 × 4
= 18 ⇒ x = = 24 with six different colours viz. violet, Indigo, blue, green,
4 3 yellow and orange and arranged from left to right (not
Again, necessarily either kept or painted with the colours in the
2 same order). Each box contains a ball of any one of the
24 = (24+y) × following six games: cricket, hockey, tennis, golf,
3
football and volleyball (not necessarily in the same
2(24+y) = 24×3 order). The golf ball is in violet box and is not in the box
24 × 3 D. The box A which contains tennis ball is orange in
24 + y =
2 colour and is at the extreme right. The hockey ball is
24 + y = 36 neither in box D nor in box E. The box C having cricket
y = 36 – 24 = 12 ball is painted blue nor in the box painted yellow. The
The number of girls = 12 box C is fifth from right and next to box B. The box B
contains volleyball. The box containing the hockey ball
32. Consider the following statements : is between the boxes containing golf ball and volley ball.
1. Either A and B are of the same age or A is Solution for 34 to 36
older than B
2. Either C and D are the same age or D is older
than C
3. B is older than C
Which of the following conclusions can be
drawn from the above statements ?
(a) A is older than B
(b) B and D are of the same age
(c) D is older than C
(d) A is older than C
Ans. (d) :
A≥B ......(i) 34. Which one of the following boxes contains the
D≥C ......(ii) golf ball ?
B>C ......(iii) (a) F
from eq (i), (ii) & (iii) (b) E
Hence A is older than C. (c) D
(d) None of the above
33. The monthly average salary paid to all the
employees of a company was Rs. 5000. The Ans. (b) : E box contains the golf ball.
monthly average salary paid to male and 35. Which of the following statements is/are
female employees was Rs. 5200 and Rs. 4200 correct ?
respectively. The percentage of males employed (a) D is painted yellow
in the company is (b) F is painted indigo
(a) 75% (b) 80% (c) B is painted blue
(c) 85% (d) 90% (d) All of the above
Ans. (b) : Ans. (b) : 'F is painted indigo" is correct.
Let
36. The football is in the box of which colour ?
Male = x
(a) Yellow
Female = y
According to question (b) Indigo
x × 5200 + y × 4200 = (x+y) 5000 (c) Cannot be determined as data are inadequate
5200x + 4200 y = 5000x + 5000y (d) Blue
200 x = 800 y Ans. (c) : The football is in either yellow or blue
x=4y colour.
The percentage of male employed of company Hence, colour can not be determined or data are
inadequate.
x
= × 100 37. Two number X and Y are respectively 20% an
x+ y
28% less than a third number Z. By what
{∵ x = y} percentage is the number Y less han the
number X ?
4y 4y (a) 12% (b) 10%
= ×100 = × 100 = 80%
4y + y 5y (c) 9% (d) 8%
IAS (Pre) GS IInd Paper (CSAT), 2016 90 YCT
Ans. (b) : Le the third number Z = 100 (a) 10 (b) 18
∴ x = 100 - 20 = 80 (c) 26 (d) 32
y = 100 - 28 = 72 Ans. (d) :
Required % the maximum number candidates =
(80 − 72) ⇒ [5C1 + 5C2 + 5C3 + 5C4 + 5C5 + 1]
= ×100
80  5 5 5 5 5 
= 10% ⇒  + ]+ + + +1
 1 5 -1 2 5 - 2 3 5-3 4 5-4 5 5-5 
38. A daily train is to be introduced between
 5 5 5 5 5 
station A and station B starting from each end ⇒  + + + + +1
at 6 am and the journey is to be completed in 14 2 3 3 2 41 5 0 
42 hours.
5× 4 5× 4 × 3 5× 4 × 3 5× 4
What is the number of trains needed in order ⇒ + + + +1+1
to maintain the Shuttle Service? 4 1× 2 × 3 1× 2 3 4
(a) 2 (b) 3 ⇒ 5+5×2 +5×2 +5×1+1+1
(c) 4 (d) 7 = 5+10+10+5+1+1
Ans. (c) : = 32
Station (A) (6AM)(1) Station (B) (6AM) Directions for the following 8 (eight) items:
Monday 42 (2) Monday Read the following eight passages and answer the
items that follow each passage. Your answers to these
Tuesday (3) (4) Tuesday items should be based on the passages only.
00 : 00 00 : 00 Passage -1
Wednesday (2) (1) Wednesday By killing transparency and competition, crony
capitalism is harmful to free enterprise, opportunity and
Thursday(2) (3) Thursday economic growth. Crony capitalism, where rich and the
00:00 00:00 influential are alleged to have received land and natural
Friday (1) (2) Friday resources and various licenses in return for payoffs to
venal politicians, is now a major issue to be tackled. One
Saturday (3) (4) Saturday of the greatest dangers to growth of developing
00:00 00:00 economies like India is the middle-income where crony
Sunday (2) (1) Sunday capitalism creates oligarchies that slow down the growth
41. Which among the following is the most logical
Monday (4) (3) Monday corollary to the above passage ?
00:00 (a) Launching more welfare schemes and
Tuesday (1) (2) Tuesday allocating more finances for the current
schemes are urgently needed
(b) Efforts should be made to push up economic
growth by other means and provide licences
to the poor
(c) Greater transparency in the functioning of the
The number of trains needed in order to maintain the government and promoting the financial
shuttle service is 4. inclusion are needed at present
39. A piece of tin is in the form of a rectangle (d) We should concentrate more on developing
having length 12 cm and width 8 cm. This is manufacturing sector than service sector
used to construct a closed cube. The side of the Ans. (c) : The most logical corollary to the above
cube is ?
passage is that greater transparency in the functioning of
(a) 2 cm (b) 3 cm the government and promoting the financial inclusion
(c) 4 cm (d) 6 cm are needed at present. Transparancy will reduce
Ans. (c) : corruption and those who take undue advantage in
Let the side of cube = a collusion with the government will be curbed.
then, Passage 2
6a2 = 12 × 8 Climate adaptation may be rendered ineffective if
a2 = 2 × 8 = 16 policies are not designed in the context of other
a2 = 16 ⇒ a = 4 Cm. development concerns. For instance, a comprehensive
40. In a question paper there are five questions to strategy that seeks to improve food security in the
be attempted and answer to each questions has context of climate change may include a set of
two choices-True (T) or False (F). It is given coordinated measures related to agricultural extension,
that no two candidates have given the answers crop diversification, integrated water and pest
to the five questions in an identical sequence. management and agricultural information services. Some
For this to happen the maximum number of of these measures may have to do with climate changes
candidates is: and others with economic development.
IAS (Pre) GS IInd Paper (CSAT), 2016 91 YCT
42. What is the most logical and rational inference (c) Need to integrate banking and non-banking
that can be made from the above passage? institutions
(a) It is difficult to pursue climate adaptation in (d) Need to promote financial inclusion
the developing countries Ans. (d) : The message that is essentially implied in the
(b) Improving food security is a far more above passage is that there is a need to promote
complex. issue than climate adaptation financial inclusion. Financial inclusion will increase the
(c) Every developmental activity is directly or liquidity of money and if money reaches to the people
indirectly linked to climate adaptation easily then people will automatically reach the bank.
(d) Climate adaptation should be examined in Passage -5
tandem with other economic development Safe and sustainable sanitation in slums has
options. immeasurable benefits to women and girls in terms of
Ans. (d) : The most logical and rational inference than their health, safety, privacy and dignity. However,
can be made from the above passage is that climate women do not feature in most of the schemes and
adaptation should be examined in relation with other policies on urban sanitation. The fact that even now the
economic development options. manual scavenging exists only goes to show that not
enough had been done to promote pour-flush toilets and
Passage -3 discontinue the use of dry latrines. A more sustained and
Understanding of the role of biodiversity in the rigorous campaign needs to be launched towards the
hydrological cycle enables better policy-making. The right to sanitation on very large scale. This should
term biodiversity refers to the variety of plants, animals, primarily focus on the abolition of manual scavenging.
microorganisms, and the ecosystems in which they 45. With reference to the above passage, consider
occur. Water and biodiversity are interdependent. In the following statements :
reality, the hydrological cycle decides how biodiversity 1. Urban sanitation problems can be fully
functions. In turn, vegetation and soil drive the solved by the abolition of manual scavenging
movement of water. Every glass of water we drink has, only
at least in part, passage through fish, trees, bacteria, soil 2. There is a need to promote greater awareness
and other organisms. Passing through these ecosystems, on safe sanitation practices in urban areas
it is cleansed and made fit for consumption .The supply Which of the statements given above is /are
of water is a critical service that the environment correct ?
provides. (a) 1 only
43. Which among the following is the most critical (b) 2 only
inference that can be made from the above (c) Both 1 and 2
passage? (d) Neither 1 nor 2
(a) Biodiversity sustains the ability of nature of
Ans. (b) : There is a need to encourage more awareness
recycle water of safe clearing practices is urban areas. The above
(b) We cannot get potable water without the statement is true in that context of the passage.
existence of living organisms
(c) Plants, animals and microorganisms
Passage – 6
continuously interact among themselves To Understand the nature and quantity of
Government proper for man, it is necessary to attend to
(d) Living organisms could not have come into
his character. As nature created him for social life, she
existence without hydrological cycle. fitted him for the station she intended, In all cases she
Ans. (a) : The most critical inference that can be made made his natural wants greater than his individual owers.
from the above passage is that biodiversity sustains the No one man is capable, without the aid to society , of
ability of nature of recycling water and water is purified spplying his own wants; and those wants, acting upon
through biodiversity. There is a difference between every individual, impel the whole of them into society.
artificial purified water and naturally purified water. 46. Which among the following is the most logical
Passage -4 and rational inference that can be made from
In the last decade, the banking sector has been the above passage?
restructured with a high degree of automation and (a) Nature had created a great diversity in human
products that mainly serve middle-class and upper society
middle-class society. Today there is need for a new (b) Any given human society is always short of
agenda for the banking and non-banking financial its wants
services that does not exclude the common man. (c) Social life is a specific characteristic of man
44. Which one of the following is the massage that (d) Diverse natural wants forced man towards
is essentially implied in the above passage? social system.
(a) Need for more automation and more products Ans. (d) : The most logical and rational inference that
of bank can be drawn from the above passage is that various
(b) Need for a radical restructuring of our entire natural needs have compelled human beings towards the
public finance system social system.

IAS (Pre) GS IInd Paper (CSAT), 2016 92 YCT


Passage-7 Ans. (b) :
The nature of the legal imperatives in any given
state corresponds to the effective demands that state
encounters. And that these, in their turn, depend, in a
general way, upon the manner in which economic power
is distributed in the society which the state controls.
47. The statement refers to :
(a) the antithesis of Politics and Economics
(b) the interrelationship of Politics and Economics
Distance from point B to starting point 'O'
(c) the predominance of Economics over
Politics By Pythagoras theorem-
(d) The predominance of Politics over Economics = 52 + 122
Ans. (b) : The passage refers to the interrelationships = 25 + 144
between politics and economy. According to the
demands of the majority of the people, laws are made in = 169 = 13 Km.
which economic interests are embedded. 50. An agricultural field is in the from of a
Passage -8 rectangle having length X1 meters and breadth
X2 meters (X1 and X2 are variable) . If X1+X2 =
About 15 percent of global greenhouse gas
40 meters then the area of the agricultural field
emissions come from agricultural practices. This
will not exceed which one of the following
includes nitrous oxide from fertilizers; methane from
livestock, rice production, and manure storage; and values ?
carbon dioxide (CO2) from burning biomass, but this (a) 400 sq m (b) 300 sq m
excludes CO2 emissions from soil management practices, (c) 200 sq m (d) 80 sq m
savannah burning and deforestation Forestry land use, Ans. (a) :
and land-use change account for another 17 percent of Given, length = X1 m and breadth = X2 m Also, X1 +
greenhouse gas emissions each year, three quarters of X2 = 40, where X1 and X2 are variables we know that,
which come from tropical deforestation. The remainder of all the rectangles, a square has the largest area.
is largely from draining and burning tropical peat land. For the given rectangle to be a square X1 = X2
About the same amount of carbon is stored in the world’s So, X + X = 40 ⇒ X = X = 20 m
1 2 1 2
peat lads as is stored in the Amazon rainforest.
Maximum area of the given rectangle
48. Which among the following is the most logical = X X = (20 × 20) = 400 m2
1 2
and rational inference that can be made from
the above passage? 51. The sum of the ages of 5 members comprising a
(a) Organic farming should immediately replace family, 3 years ago was 80 years. The average
mechanised and chemical dependant age of the family today is the same as it was 3
agricultural practices all over the world year age, because of an addition of a baby
during the intervening period. How old is the
(b) It is imperative for us to modify our land use
practices in order to mitigate climate change. baby ?
(a) 6 months
(c) There are no technological solutions to the
problem of greenhouse gas emissions (b) 1 years
(d) Tropical areas are the chief sites of carbon (c) 2 year
sequestration. (d) 2 year and 6 month
Ans. (b) : The most logical and rational inference that Ans. (b) : The average age of family 3 years ago was
can be drawn from the above passage is that, it is 80
= = 16 years
imperative for us to modify our land use practices in 5
order to mitigate climate change. The sum of the ages of 5 members today will be = 80 +
49. A person climbs a hill in a straight path from (3 × 5) = 80 + 15 = 95 years
point ‘O’ on the ground in the direction of Now, the average age of family including new body will
north east and reaches a point ‘A’ after 95 + N
traveling a distance of 5 km. Then, from the be = 6
, which is same as 3 years ago, i.e., 16
point ‘A’ he moves to point ‘B’ in the direction years.
of north-west. Let the distance AB be 12 km.
N is the age of baby now
Now, how far is the person away from the
95 + N
starting point ‘O’? ∴ = 16 ⇒ 95 + N = 16 × 6
(a) 7 km (b) 13 km 6
(c) 17 km (d) 11 km N = 96 – 95 ⇒ N = 1 years

IAS (Pre) GS IInd Paper (CSAT), 2016 93 YCT


52. The total emoluments of two persons are the 54. What is C’s surname ?
same, but one gets allowances to the extent of (a) Ribeiro
65% of his basic pay and the other gets
allowances to the extent of 80% of his basic (b) Kumar
pay.The ratio of the basic pay of the former to (c) Singh
the basic pay of the latter is ? (d) Cannot be determined
(a) 16 : 13 (b) 5 : 4
(c) 7 : 5 (d) 12 : 11 Ans. (a) : C's surname is Riberio.
Ans. (d) : 55. What is the colour of tie ?
Let the basic pay of the first person be 'x' and that of (a) Black
second person be 'y'. (b) Blue
According to the question,
Emolument of both the persons are same. (c) White
So, x + 0.65 x = y + 0.8y (d) Cannot be determined
x 1.8 Ans. (b) : The colour of tie is blue.
⇒ x (1 + 0.65 ) = y (1 + 0.8 ) ⇒ =
y 1.65 56. Who wore the sweater ?
x 12 (a) A
⇒ = or 12 : 11
y 11 (b) B
53. A person is standing on the first step from the (c) C
bottom of a ladder . If he has to climb 4 more (d) Cannot be determined
steps to reach exactly the middle step, how
many steps does the ladder have ? Ans. (a) : A wore the sweater.
(a) 8 (b) 9 57. A B is a vertical trunk of a huge tree with A
(c) 10 (d) 11 being the point where the base of the trunk
Ans. (b) :
touches the ground. Due to a cyclone, the trunk
A person is standing on the first step from the bottom of
a ladder. has been broken at C which is at a height of 12
Now, he has to climb 4 more steps to reach exactly the meters, broken part is partially attached to the
middle step. vertical portion of the trunk at C. If the end of
So, it is clear that the ladder have 9 steps. the broken part B touches the ground at D
which is at a distance of 5 meters from A, then
the original height of the trunk is:
(a) 20 m
(b) 25 m
(c) 30 m
(d) 35 m
Ans. (b) :
Direction for the following 3 (three) items :
Consider the given information and answer the three
items that follow.
When three friends A, B and C met, it was found
that each of them wore an outer garment of a
different colour. In random order, the garments are:
jacket, sweater and tie; and the colours are : bule ,
white and black . Their surnames in random order
are: Ribeiro, Kumar, Singh we know that : The distance between broken part of tree on surface and
1. neither B nor Ribeiro wore a white sweater trunk of the tree = 5cm
2. C wore a tie
Hypotenuses2 = Perpendicular2 + base2
3. Singh’s garment was not white
4. Kumar does not wear a jacket In, ∆ CAD,
5. Ribeiro does not like to wear the black colour CD2 = AC2 +AD2
6. Each of the friends wore only one outer garment
of only one colour CD = AC2 + AD 2
Solution for 54 to 56- = 122 + 52 = 144 + 25
Person A B C
Surnames Kumar Singh Ribeiro CD = 169 = 13meter
Garments Sweater Jacket Tie AB = CA + CD
Colour White Black Blue = (12 + 13) meter = 25 meter
IAS (Pre) GS IInd Paper (CSAT), 2016 94 YCT
58. A person walks 12 km due north, then 15 km Ans. (c) : Let : R → Ram, S → Shyam
due east, after that 19 km due west and then 15 4 4 60
km due south. How far is he from the starting ∵ + =
R S 100
point?
(a) 5 km (b) 9 km  1 1  60
4 + =
(c) 37km (d) 61 km  R S  100
Ans. (a) : 1 1 3
4 + =
R S  5
1 1 3
+ = ........(i)
R S 20
According to the question,
4 12
+ =1
R S
4 4 8
+ + =1
R S S
1 1 8
4 +  + =1
Distance between starting point A and end point C is- R S  S
2 2 2 3 8
(AC) = (AB) + (BC) 4 × + =1
(AC)2 = 42 + 32 20 S
(AC)2 = 25 ⇒ AC = 5 8 12 8 8
=1 − ⇒ =
∴Distance between starting point and end point in S 20 S 20
5Km. 1 1
59. A cube has all its faces painted with different = .......(ii)
S 20
Colurs. It is cut into smaller cubes of equal Putting values from equation (ii) to (i)-
sizes such that the side of the small cube is one-
1 1 3 1 3 1
fourth the big cube. The number of small cubes + = ⇒ = −
with only one of the sides painted is : R 20 20 R 20 20
(a) 32 (b) 24 1 2 1
= ⇒
(c) 16 (d) 8 R 20 10
Ans. (b) : Therefore, time taken by Ram to complete the entire job
Each face of the cube will look like the figure below: alone = 10 days.
61. A military code writes SYSTEM AS SYSMET
and NEARER as AENRER. Using the same
code, FRACTION can be written as :
(a) CARFTION (b) FRACNOIT
(c) NOITCARF (d) CARFNOIT
Ans. (d) :

On each face, the smaller cubes which do not lie on the


edges of bigger cube will have only one side painted.
As shown in the figure above, there are four cubes and
which has only on face painted. accordingly, there will
be (6) such faces of bigger cube so total 24 small cubes
have one painted face.
Hence 24 is correct answer.
60. Ram and Shyam work on a job together for Same way, FRACTION can be written as-
four days and complete 60% of it. Ram takes
leave then and Shyam works for eight more
days to complete the job. How long would Ram
take to complete the entire job alone ?
(a) 6 days (b) 8 days
(c) 10 days (d) 11 days ∴ Correct answer in (d) CARFNOIT

IAS (Pre) GS IInd Paper (CSAT), 2016 95 YCT


62. If R and S are different integers both divisible (b) Diets based on grains, nuts, fruits and
by 5, then which of the following is not vegetables are best suited for health in
necessarily true ? developing countries
(a) R – S, is divisible by 5 (c) Human beings change their food habits from
(b) R + S, is divisible by 10 time to time irrespective of the health
concerns
(c) R × S, is divisible by 25
(d) From a global perspective, we still do not
(d) R2 + S2, is divisible by 5 know which type of diet is best for us
Ans. (b) : Ans. (a) : The conclusive message that emerges from
According to question- the above passage in that the increasing demand for
Let, R = 10 and S = 5 animal based food sources places a greater burden on
R − S 10 − 5 our natural resources. A diet high in meat and dairy
then, (a) = products will have an impact on human food
5 5 production.
= 1 Whole Number Passage 2
R + S 10 + 5 15 All humans digest mother’s milk as infant, but until
(b) = =
10 10 10 cattle began being domesticated 10,000 years ago,
= 1.5 not a Whole Number Children once weaned no logger needed to digest milk.
R × S 10 × 5 50 As a result, they stopped making the enzyme lactase,
(c) = = which breaks down the sugar lactose into simple sugars.
25 25 25
After humans began herding cattle, it became
= 2 Whole Number tremendously advantageous to digest milk, and lactose
R 2 + S 2 (10) 2 + (5)2 100 + 25 125 tolerance evolved independently among cattle herders in
(d) = = =
S 5 5 5 Europe, the middle East and Africa. Groups not
= 25 Whole Number dependant on cattle, such as the Chinese and Thai,
R+S Remain lactose intolerant.
Therefore, is not necessarily true. The correct 65. Which among the following is the most logical
10
assumption that can be made from the above
answer is option (b).
passage ?
63. How many numbers are there between 100 and (a) About 10,000 years ago, the domestication of
300 which either begin with or end with 2 ?
animals took place in some parts of the world
(a) 110
(b) A permanent change in the food habits of a
(b) 111
community can bring about a genetic change
(c) 112 in its members
(d) None of the above (c) Lactose tolerant people only are capable of
Ans. (a) : getting simple sugars in their bodies
Number between 100 and 200 which either begin with (d) People who are not lactose tolerant cannot
or end with 2 are 102, 112, 122, 132, 142, 152, 162, digest and dairy product
172, 182, 192 = 10 Ans. (b) : The most logical assumption that can be
between 200 to 300 are 100 made from the above passage is that a permanent
∴Total = 100 + 10 = 110. change in the eating habits in a community can bring
Directions for the following 8 (eight) items : Read about genetic changes in the members of the
the following five passages and answer the items that community.
follows each passage. Your answers to these items Passage 3
should be based on the passages only . “The conceptual difficulties in National Income
Passage-1 comparisons between underdeveloped and industrialized
As we look to 2050, when we will need to feed two countries are particularly serious because a part of the
billion more people, the question of which diet is best national output in various underdeveloped countries is
has taken on new urgency. The foods we choose to eat produced without passing through the commercial
in the coming decades will have dramatic ramifications
for the planet. Simply put, a diet that revolves around channels.’’
meat and dairy a way of eating that is on the rise 66. In the above statement, the author implies that :
throughout the developing world, will take a greater toll (a) the entire national output produced ad
on the world’s resources than one that revolves around consumed in industrialized countries passes
unrefined grains, nuts, fruits and vegetables. through commercial channels
64. What is the critical message conveyed by the (b) the existence of a non-commercialized sector
above passage? in different underdeveloped countries renders
(a) Our increasing1 demand for foods sourced the national income comparisons over
from animals puts a greater burden on our
countries difficult
natural resources
IAS (Pre) GS IInd Paper (CSAT), 2016 96 YCT
(c) no part of national output should be produced the presence of empty-pocket hungry mouths,
and consumed without passing through overwhelmingly confined to the underdeveloped
commercial channels economies. In as much as’ a two square meal’ is of
(d) a part of the national output being produced elemental significance to basic human existence, the
and consumed without passing through issue of worldwide supply of food has been gaining
commercial channels is a sign of significance, in recent times, both because the quantum
underdevelopment and the composition of demand has undergoing big
Ans. (b) : In the passage, the author argues that in changes, and because, in recent years, the capabilities of
underdeveloped countries, a fraction of national output individual countries to generate uninterrupted chain of
does not pass through commercial channels. food supplies have come under strain. Food production
It means that it is not recognized explicitly in national marketing and prices, especially price-affordability by
income accounting or may ever go unrecognized. the poor in the developing world, have become global
issues that need global thinking and global solutions.
Passage 4 68. According to the above passage, which of the
An increase in human-made carbon dioxide in the following are the fundamental solutions for the
atmosphere could initiate a chain reaction between plant world food security problem ?
and microorganisms that would unsettle one of the
largest carbon reservoirs on the planet soil. In a study, it 1. Setting up more agro-based industries
was found that the soil, which contains twice the amount 2. Improving the price affordability by the poor
of carbon present in a plants and Earth’s atmosphere 3. Regulating the conditions of marketing
combined, could become increasingly volatile as people 4. Providing food subsidy to one and all
add more carbon dioxide to the atmosphere. This is Select the correct answer usig the code given
largely because of increased plant growth. Although a below :
greenhouse gas and a pollutant, carbon dioxide also (a) 1 and 2 only (b) 2 and 3 only
supports plant growth. As trees and other vegetation (c) 1, 3 and 4 only (d) 1, 2, 3 and 4
flourish in a carbon dioxide –rich future, their roots
could stimulate microbial activity in soil carbon and its Ans. (b) : According to the above passage, there are
release into the atmosphere as carbon dioxide. two fundamental solutions for food security in the
67. Which among the following is the most logical world-first to improve the price bearing capacity of the
corollary to the above passage ? poor and secondly to provide regulation of marketing
(a) Carbon dioxide is essential for the survival of conditions.
microorganisms and plants 69. According to the above passage, the biggest
(b) Humans are solely responsible for the release challenge to world agriculture is :
of carbon dioxide into the atmosphere (a) to find sufficient land for agriculture and to
(c) Microorganisms and soil carbon are mainly expand food processing industries
responsible for the increased plant growth (b) to eradicate hunger in uinderdeveloped
(d) Increasing green cover could trigger the countries
release of carbon trapped in soil (c) to achieve a balance between the production
Ans. (d) : The most logical corollary to the above of food and non-food items
passage in that increasing green cover could trigger the (d) to achieve a balance between demand for and
release of carbon trapped in soil. On the one hand, while supply of food
carbon dioxide is beneficial for plants, it is not Ans. (d) : According to the above passage, the biggest
beneficial for animals. At present, the amount of carbon challenge before the world agriculture is to achieve a
dioxide in the earth is 0.03%. balance between the demand and supply of food.
Passage 5 70. According to the above passage, which of the
Historically, the biggest Challenge to world following helps/help in reducing hunger and
agriculture has been to achieve a balance between starvation in the developing economies ?
demand for and supply of food. At the level of individual 1. Balancing demand and supply of food
countries, the demand-supply balance can be a critical 2. Increasing imports of food
issue for a closed economy, especially if it is a populous
economy and its domestic agriculture is not growing 3. Increasing purchasing power of the poor
sufficiently enough to ensure food supplies, on an 4. Changing the food consumption patterns and
enduring basis; it is not so much and not always, of a practices
constraint for an open, and growing economy, which has Select the correct answer using the code given
adequate exchange surpluses to buy food abroad. For the below :
world as a whole, supply-demand balance is always an (a) 1 only (b) 2, 3 and 4 only
inescapable prerequisite for warding off hunger a (c) 1 and 3 only (d) 1, 2, 3 and 4
starvation. However, global availability of adequate Ans. (c) : According to the above passage, establishing
supply does not necessarily mean that food would a balance between the demand supply of food and
automatically move from countries of surplus to of increasing purchasing power of the poor helps in
deficit if the latter lack in purchasing power. The uneven reducing hunger and starvation in the developing
distribution of hunger, starvation, under or mal-
nourishment, etc., at the world –level, thus owes itself to economies.

IAS (Pre) GS IInd Paper (CSAT), 2016 97 YCT


71. The Issue of worldwide supply of food has ahead of Kamal, the number of boys in rank
gained importance mainly because of : after him is :
1. overgrowth of population world wide (a) 13 (b) 12
2. sharp declined in the area of food production (c) 7 (d) 3
3. limitation in the capabilities for sustained Ans. (b) : Let the number of boys in the class = x
supply of food. ∴ The number of Girls 2x
Select the correct answer using the code given
According to question
below :
x + 2x = 60
(a) 1 and 2 only
(b) 3 only 3x = 60
(c) 2 and 3 only x = 20
(d) 1, 2 and 3 ∴ The number of boys (x) = 20
Ans. (b) : The issue of worldwide food supply has and the number of Girls (2x) = 40
gained importance due to the limitation in capacities for The number of boys before Kamal = 16 – 9 = 7
sustainable food supply. The number of boys after Kamal = 20 – (7 + 1)
72. Four –digit numbers are to be formed using the = 20 – 8 = 12
digits 1,2,3 and 4; and none of these four digits 75. A and B walk around a circular park. They
are repeated in any mammer, Futher. start at 8 a.m. from the same point in the
1. 2 and 3 and are not to immediately follow opposite directions. A and B walk at a speed of
each other 2 rounds per hour and 3 rounds per hour
2. 1 is not to be immediately followed by 3 respectively.
3. 4 is not to appear at he last place How many times shall they cross each other
4. 1 is not to appear at the first palce after 8.00 a.m. and before 9.30 a.m.?
How many different numbers can be formed (a) 7 (b) 6
(a) 6
(c) 5 (d) 8
(b) 8
Ans. (a) :
(c) 9
(d) None of the above 60
Time taken by A to complete a round = = 30 min.
Ans. (a) : 6 different numbers can be formed i.e., (2143, 2
2431, 3142, 3412, 3421, 4312) 60
and time taken by B to complete a round = = 20
73. A cylindrical overhead tank of radius 2 m and 3
height 7 m is to be filled from an underground min.
tank of size 5.5 m × 4m × 6cm. How much 1 1
portion of the underground tanks is still filled ∴ Relative speed of A and B = +
30 20
with water after filling the overhead tank
completely ? 2+3 5 1
= = =
(a) 1/3 (b) 1/2 60 60 12
(c) 1/4 (d) 1/6 Then time taken by A and B to complete the correct
Ans. (a) : round = 12 min .
The volume of cylindrical overhead tank = πr2h for the number of crossing each other,

V1 =
22
× 2× 2× = 7 ⇒
( 9 : 30 − 8 : 00 )
7 12 min
V1 = 88 m3 90 min
The volume of underground tank = 5.5× 4 × 6 = = 7.5 ≈ 7times
3 12 min
= V2 = 132.0 m
The required portion of underground water 76. W can do 25% of a work-in 30 days. X can do
132 − 88 44 1 1/4 of the work in 10 days, Y can do 40% of
= = ⇒ the work in 40 days and Z can do 1/3 of the
132 132 3
after filling the over head tank. work in 13 days . Who will complete the work
first ?
74. In a class of 60 students, where the number of
girls is twice that of boys. Kamal, a boy, ranked (a) W (b) X
seventeenth from the top. If there are 9 girls (c) Y (d) Z

IAS (Pre) GS IInd Paper (CSAT), 2016 98 YCT


Ans. (d) : Taken time by 'w' for complete the work bucket, how much total distance he has to run
30 if the bucket is 5 meters from the first apple
⇒ ×100 and all other apples are placed 3 meters apart?
25 (a) 40 m (b) 50 m
⇒ 120 days (c) 75 m (d) 150 m
taken time by 'x' for complete the work Ans. (d) : The distance covered by competitor
4 = 2(5+8+11+14+17+20) = 2 × 75 = 150 m.
= 10 × = 40 days
1 79. A round archery target of diameter 1 m is
taken time by 'y' for complete the work marked with four scoring regions from the
40 centre outwards as red, blue, yellow and white.
= × 100 = 100 days
40 The radius of the red band is 0.2m. The width
taken time by 'z' for complete the work of all the remaining bands is equal. If archers
3 throw arrows towards the target, what is the
= 13 × probability, that the arrows fall in the red
1
= 39 days region of the archery target ?
(a) 0.40 (b) 0.20
1
∵ Efficiency ∝ (c) 0.16 (d) 0.04
time
Hence, Z will complete the work first. Ans. (c) :
77. The average monthly income of a person in a
certain family of 5 is Rs. 10,000 . What will be
the average monthly income of a person in the
same family if the income of one person
increased by Rs. 1,20,000 per year ?
(a) Rs 12,000
(b) Rs 16,000
(c) Rs 20,000 Diameter of archery target = 1m
(d) Rs 34,000 Radius of red Band = .2m
Ans. (a) : Area of circle = πR 2
∵ Average monthly income of family of 5 members 22 88
= 10,000 ` = π.(.2)2 = 0.04π = × 0.04 = m2
7 700
∴ Total monthly income = 10,000 × 5 22 22 5 5
= 50,000 ` Total area = × .5 × .5 or × ×
7 7 10 10
∵ The income of one person increased by 120,000 per
22 11 2
year. = = m
∴ Monthly increased income of that person 7 × 4 14
Area of red 88
1, 20, 000 Required probability = =
= = ` 10,000 TotalArea 700
12
Total income per month = 50,000 + 10,000 11
= 60,000 14
88 14
∴Required average =
60,000 = × = 0.16
5 700 11
= 12,000 80. A person allows 10% discount for cash
OR payment from the marked price of a toy and
still he makes a 10% gain. What is the cost
 1, 20000  1
Required Average = 10,000 +   × price of the toy which is marked Rs.770 ?
 12  5 (a) Rs. 610 (b) Rs. 620
1 (c) Rs. 630 (d) Rs. 640
= 10,000 + (10,000)
5 Ans. (c) : Marked Price of Toy = Rs 770
= 10,000 + 2000 Discount % = 10%
= 12000 Formula :
78. In a race, a competitor has to collect 6 apples Marked Price:cost price=100+Profits%:100 – Discount
which are kept in a straight line on a track and Marked Price : Cost Price = 110 : 90 = 11:9
a bucket is placed at the beginning of he track Marked Price =770
which is a starting point. The condition is that 11x = 770
the competitor can pick only one apple at a x=7
time, run back with it and drop it in the Cost Price of Toy = 9x = 9×70 = 630
bucket. If he has to drop all the apples in the
IAS (Pre) GS IInd Paper (CSAT), 2016 99 YCT
UNION PUBLIC SERVICE COMMISSION
Civil Services (Preliminary Exam) - 2017
CSAT : PAPER-II
(Chapterwise Analysis with Explanation)
Time : 2 hours Maximum Number : 200
(b) we should reject suggestions of friends and
COMPREHENSION remain unchanged
(c) we should always remain broad-minded
Directions for the following 8 (eight) items : (d) we should be resolute and achievement-oriented
Read the Following eight passages and answer the Ans. (a) : The exact keynote that emerges from the
items that follow the passages. Your answers to these passage, we can see that the author is talking about to
items should be based on the passages only. consult wisely and to resolve firmly with perseverance
Passage-1 in the absence of flexibility.
Passage-3
What climate change will undeniably do is cause of During the summer in the Arctic Ocean, sea ice ha,
amplify events that hasten the reduction of resources.
been melting earlier and faster, and the winter freeze has
Competition over these diminising resources would
been coming later. In the last three decades, the extent of
ensue in the form of political or even violent conflict.
Resource based conflicts have rarely been overt and are summer ice has declined by about 30 per cent. The
thus difficult to isolate. Instead they take on veneers that lengthening period of summer melt threatens to
appear more politically palatable. Conflicts over undermine the whole Arctic food web, atop which stand
resources like water are often cloaked in the guise of polar bears.
identity or ideology. 3. Which among the following is the most crucial
1. What does the above passage imply? message conveyed by the above passage?
(a) Resource-based conflicts are always politically (a) Climate change has caused Arctic summer to be
motivated. short but temperature to be high.
(b) There are no political solutions to resolve (b) Polar bears can be shifted to South Pole to
environmental and resource based conflicts. ensure their survival.
(c) Environmental issues contrilaite to resource (c) Without the presence of polar bears, the food
stresses and politic ril conflict. chains in Arctic region will disappear.
(d) Political conflict based on identity or ideology (d) Climate change poses a threat to the survival of
cannot be resolved. polar bears.
Ans. (c) : Indiscriminate industrialization and Ans. (d) : The problem of global warming has
urbanization have led to excessive exploitation of intensified climate change. This has put a threat to
natural resources. The problem of climate change has biodiversity, Due to global warming, the glaciers of
increased this complexity and has created a the arctic are melting, due to which there has been a
competition for supremacy over natural resources. The serious threat to the flora and fauna of the Arctic.
political leadership continues to generate political Polar bear have been affected the most by the crisis.
struggle for the hegemony of natural resources by Polar bears are listed as endangered in the IUCN Red
promoting these issues. Data Book.
Passage-2 Passage-4
Why do people prefer open defecation and not want
The man who is perpetually hesitating which of the toilets or, if they have them, only use them sometimes?
two things he will do first, will do neither. The man who Recent research has shown two critical elements: ideas
resolves, but suffers his resolution to be changed by the of purity and pollutions, and not wanting pits or septic
first counter Suggestion of a friend—who fluctuates tanks to fill they have to be emptied. These are the issue
from opinion to opinion and veers from plan to plan-can that nobody wants to talk about, but if we want to
never accomplish anything. He will at best be stationary eradicate the practice of open defection, they have to be
and probably retrograde in all. It is only the man who
confronted and dealt properly.
first consults wisely, then resolves firmly and then
executes his purpose with inflexible perseverance, 4. Which among the following is the most crucial
undismayed by those petty difficulties which daunt a message conveyed by the above passage?
weaker spirit—that can advance to eminence in any line. (a) The ideas of purity and pollutions are so deep-
2. The keynote that seems to be emerging from the rooted that they cannot be removed from the
passage is that minds of the people.
(a) we should first consult wisely and then resolve (b) People have to perceive toilet use and pit-
firmly emptying as clean and not polluting.

IAS (Pre) GS IInd Paper (CSAT), 2017 100 YCT


(c) People cannot change their old habits. (c) India is poised to achieve the double-digit
(d) People have neither civic sense nor sense of growth very soon.
privacy. (d) India is capable of supplying the skilled young
Ans. (b) : Cleanliness campaign is being conducted people to other countries.
with great vigor in India. Yet the trend of open Ans. (b) : There is a need for labor reforms in India to
defecation continues. An attempt has been made to make optimum use of India's vast workforce
understand the root cause of thin trend through productively.
research related to open defecation. People consider Passage-7
open defecation to be more environmentally cleaner The very first lesson that should be taught to us
than toilets and the other is suffering from the fear of when are old enough to understand it, is that complete
filling the safety tank quickly. Unless these two freedom from the obligation to work is unnatural, and
ideological problems are not addressed by public ought to be illegal, as we can escape our share of the
awareness, it cannot be overcome. burden of work only by throwing it on someone else’s
shoulders. Nature ordains that the human race shall
Passage-5 perish of famine if it stops working. We cannot escape
from this tyranny. The question we have to settle is how
In the last two decades, the world's gross domestic much leisure we can afford to allow ourselves.
product (GDP) has increased 50 percent, whereas 7. The main idea of the passage is that
inclusive wealth has increased by a mere 6 percent. In (a) it is essential for human beings to work
recent decades, GDP-driven economic performance, has (b) there should be a balance between work and
only harmed inclusive wealth like human capital; and leisure
natural capital like forests, land and water. While the (c) working is a tyranny which we to face
world's human capital which stands at 57 percent of total (d) human's understanding of the nature of work is
inclusive wealth grew by only 8 percent, the natural essential
which is 23 percent of total inclusive wealth declined by Ans. (a) : The given passage talks about our
30 per cent worldwide in the last two decades. obligations to do our work and it is essential for
5. Which of the following is the most crucial human being to work as without work how can they
inference from the above passage? get food. This statement gives the main idea which is
(a) More emphasis should be laid on the the crux of the whole passage.
development of natural capital.
(b) The growth driven by GDP only is neither Passage-8
desirable nor sustainable. There is no harm in cultivating habits so long as they
(c) The economic performance of the countries of are not injurious. Indeed, most of us are little more than
the world is not satisfactory. bundle of habits. Take away our habits and the residuum
(d) The world needs more human capital under the would hardly be worth bothering about. We could not get
present circumstances. on without them. They simplify the mechanism of life.
Ans. (b) : The most conclusive inference of the given They enable us to do a multitude of things automatically,
passage the growth driven by GDP only is neither which, if we had to give fresh and original thought to
desirable nor sustainable, as GDP driven economic them each time, would make existence an impossible
performance in recent decades has led to inclusive confusion.
wealth such as human capital and natural resources 8. The author suggests that habits
such as forests, land and water have been damaged. (a) tend to make our lives difficult
(b) add precision to our lives
Passage-6 (c) make it easier for us to live
(d) tend to mechanize our lives
By 2020, when the global economy is expected to
run short of 56 million young people, India, with its Ans. (c) : According to the author, habits make the life
youth surplus of 47 million, could fill the gap. It is in this process easy, due to which many things start
happening automatically. So, habits make life easier.
context that labour reforms are often cited as the way to
unlock double-digit growth in India. In 2014, India's Directions for the following 7 (seven) items:
labour force was estimated to be about 40 per cent of the Read the following seven passages and answer the
population, but 93 per cent of this force was in items that follow the passages. Your answer to these
unorganized sector. Over the last decade, the compound items should be based on the passages only.
annual growth rate (CAGR) of employment has slowed Passage-1
to 0.5 per cent, with about 14 million jobs created during We have hard work ahead. There is no resting for
last year when the labour force increased by about 15 any of us till we redeem our pledge in fullfill we make
million. all the people of India what destiny intends them to be.
6. Which of the following is most rational inference We are citizens of a great country, on the verge of bold
from the above passage? advance, and we have to live up to that high standard. All
(a) India must control its population growth so as to of us, to whatever religion we may belong, are equally
reduce its unemployment rate. the children of India with, equal rights, privileges and
(b) Labour reforms are required in India to make obligations. We cannot encourage communalism or
optimum use of its vast labour force narrow mindedness, for no nation can be great whose
productively. people are narrow in thought or action.
IAS (Pre) GS IInd Paper (CSAT), 2017 101 YCT
9. The challenge the author of the above passage Passage-4
throws to the public is to achieve A successful democracy depends upon widespread
(a) a high standard of living, progress and interest and participation in politics, in which voting is
privileges an essential part. To deliberately refrain from taking such
(b) equal privileges, fulfillment of destiny and an interest, and from voting, is a kind of implied anarchy,
political tolerance it is to refuse one's political responsibility while enjoying
(c) spirit of adventure and economic parity the benefits of a free political society.
(d) hard work, brotherhood out national unity 12. This passage relates to
Ans. (d) : The passage is talking about hardwork, (a) duty to vote
brotherhood and national unity in India. This option is (b) right to vote
the most appropriate answer. (c) freedom to vote
(d) right to participate in politics
Passage-2 Ans. (a) : The responsibility of voting has been
highlighted in this passage. India is a democratic
"The individual, according to Rousseau, puts his
country. Here the leaders who lead the country and the
person and all his power in common under the supreme state are elected on the basis of voting. With regard to
direction of the General Will and in our corporate sending representatives of their choice to the
capacity we receive each member as an indivisible part parliament and state legislatures, the democratic
of the whole." representation system is of relevance to both the
10. In the light of the above passage, the nature of electorate and the political system. Therefore, voters
General Will is best described as should be aware of their responsibility towards the
(a) the sum total of the private wills of the country.
individuals
(b) what is articulated by the elected representatives Passage-5
of the individuals In a free country, the man who reaches the position
(c) the collective good as distinct from private wills of leader is usually one of outstanding, character and
of the individuals ability. Moreover, it is usually possible to foresee that he
(d) the material interests of the community will reach such a position, since early in life one can see
Ans. (c) : The given passage is a philosophical one in his qualities of character. But this is not always true in
which collective good as distinct from private wills of the case of a dictator; often he reaches his position of
the individuals, is correct Answer. power through chance, very often through the unhappy
state of his country.
Passage-3 13. The passage seems to suggest that
(a) a leader foresees his future position
In a democratic State, where a high degree of (b) a leader is chosen only by a free country
Political maturity of the people obtains, the conflict (c) a leader must see that his country is free from
between the will of the sovereign law-making body and despair
the organized will of the people seldom occurs. (d) despair in a country sometimes leads to
11. What does the above passage imply? dictatorship
(a) In a democracy, force is the main phenomenon Ans. (d) : A person reaching a constitutional position
in the actual exercise of sovereignty. in an independent country is generally of excellent
(b) In a mature democracy, force to a great extent is character and ability. But instead of doing good to the
the main phenomenon in the actual exercise of country, a dictator does harm to the country. Such a
sovereignty. person reaches his position of power by chance, which
(c) In a mature democracy, use of force is irrelevant becomes the main reason for the sad condition of the
in the actual exercise of sovereignty. country.
(d) In a mature democracy, force is narrowed down
to a marginal phenomenon in the actual exercise Passage-6
of sovereignty. The greatest blessing that technological progress has
Ans. (d) : The most important condition of any in store for mankind is not, of course, an accumulation of
democratic system is that there should be 'sovereignty' material possessions. The amount of these that can be
in the hands of the people. The success as failure of effectively enjoyed by one individual in one lifetime is
democracy depends on how much people's not great. But there is not the same narrow limit to the
participation is in it. Political consciousness among possibilities of the enjoyment of leisure. The gift of
the people is essential for the success of democracy. leisure may be abused by people who have had no
experience of making use of it. Yet the creative use of
For a mature democracy, the spread of education
leisure by a minority in societies has been the mainspring
among the citizens is very important.
of all human progress beyond the primitive level.
IAS (Pre) GS IInd Paper (CSAT), 2017 102 YCT
14. With reference to the above passage, the 16. Which one of the following is the best explanation
following assumptions have been made : of the above passage?
1. People always see the leisure time as a gift and (a) Political development is not a unilinear process
use it for acquiring more material possessions. for it involves both growth and decay.
2. Use of leisure by some people to produce new (b) Traditional societies succeed in resisting
and original things has been the chief source of positive aspects of political development.
human progress. (c) It is impossible for traditional societies to break
Which of these assumptions is/are valid? away from lingering loyalties.
(a) 1 only (d) Sustenance of traditional loyalties is conducive
(b) 2 only to political development.
(c) Both 1 and 2 Ans. (a) : The pace of political development is not
(d) Neither 1 nor 2 uniform. It keeps on changing according to the time of
Ans. (b) : As per the passage, assumption 2 talk about the country. That is why it has been said that political
use of leisure by some people to produce new original development is not a one-way process as it involves
things has been the chief source of human progress. both growth and decline.
As it is mentioned in the passage that the gift of
leisure may be abused by people who had no Passage-2
experience of making use of it. Thus, option (b) is
correct. There has been a significant trend worldwide
towards regionalism in government, resulting in a
Passage-7 widespread transfer of powers downwards towards
regions and communities since 1990s. This process,
There is more than a modicum of truth in the which involves the creation of new political entities and
assertion that "a working knowledge of ancient history is
bodies at a sub-national level and an increase in their
necessary to the intelligent interpretation of current
content and powers, is known as devolution. Devolution
events". But the sage who uttered these words of wisdom
has been characterized as being made up of three
might well have added something on the benefits of
studying particularly the famous battles of history for the factors—political legitimacy, decentralization of
lessons they contain for those of us who lead or aspire to authority and decentralization of resources. Political
leadership. Such a study will reveal certain qualities and legitimacy here means a mass demand from below for
attributes which enabled the winners to win—and certain the decentralization process, which is able to create a
deficiencies which caused the losers to lose and the political force for it to take place. In many cases,
student will see that the same pattern recurs consistently, decentralization is initiated by the upper tier of
again and again, throughout the centuries. government without sufficient political mobilization for
15. With reference to the above passage, the it at the grassroots level, and in such cases the
following assumptions have been made : decentralization process often does not fulfill its
1. A study of the famous battles it history would objectives.
help us understand the modern warfare. 17. Which among the following is the most logical,
2. Studying the history is essential fin anyone who rational and critical inference that can be made
aspires to be a leader. from the above passage?
Which of these assumptions is/are valid? (a) Emergence of powerful mass leaders is essential
(a) 1 only to create sub-national political entities and thus
(b) 2 only ensure successful devolution and decentralization.
(c) Both 1 and 2 (b) The upper tier of government should impose
(d) Neither 1 nor 2 devolution and decentralization on the regional
Ans. (d) : The study of ancient history will not helps communities by law or otherwise.
in understanding the present war situation as the mode (c) Devolution, to be successful, requires a
of warfare has changed in the present. Earlier war was democracy in which there is free expression of
with arrows, now with bombs and modern weapons the will of the people at lower level and their
and missilies. active participation at the grassroots level.
Read the following seven passages and answer (d) For devolution to take place, a strong feeling of
the items that follow the passages. Your answers to regionalism in the masses is essential.
these items should be based on the passages only. Ans. (c) : A significant trend of territorialism in
Passage-1 government across the world has led to a downward
Disruption of traditional institutions, identifications transfer of power across regions and communities for
and loyalties is likely to lead to ambivalent situations. It decades in the 1990s. Under this process, new political
is possible that some people may renew their authorities and bodies are formed at the international
identification with traditional groups whereas others level and their potential power increases, which is
align themselves with new groups and symbols emergent called anti-development. On the same side, the
from processes of political development. In addition, success of development requires a democracy in
political development tends to foster group awareness of which the will of the people at the grassroots level has
a variety of class, tribe, region, clan, language, religion, free expression and their active participation at the
occupation and others. gross roots level.
IAS (Pre) GS IInd Paper (CSAT), 2017 103 YCT
Passage-3 Ans. (a) : The given passage states that IMF views on
the fast-growing economies of Asia, which are
We live in digital times. The digital is not just gradually moving towards the middle-income trap,
something we use strategically and specifically to do a that is the point where the average income of these
few tasks. Our very perception of who we are, how we economies will become stagnant.
connect to the world around us, and the ways in which
we define our domains of life, labour and language are Passage-5
hugely structured by the digital technologies. The digital An innovative India will be inclusive as well as
is everywhere and; like air, invisible. We live within technologically advanced, improving the lives of all
digital systems, we live with intimate gadgets, we Indians. Innovation and R&D can mitigate increases in
interact through digital media, and the very presence and social inequality and relieve the pressures created by
imagination of the digital has dramatically restructured rapid urbanization. The growing divergence in
productivity between agriculture and knowledge-
our lives. The digital, far from being a tool, is a condition
and context that defines the shapes and boundaries of our intensive manufacturing and services threatens to
understanding of the self, the society, and the structure ofincrease income inequality. By encouraging India's R&D
governance. labs and universities to focus on the needs of poor people
and by improving the ability of informal firms to absorb
18. Which among the following is the most logical knowledge, an innovation and research agenda can
and essential message conveyed by the above counter this effect. Inclusive innovation can lower the
passage? coste of goods and services and create income - earning
(a) All problems of governance can be solved by opportunities for the poor people.
using digital technologies. 20. Which among the following is the most logical
(b) Speaking of digital technologies is speaking of and rational assumption that can be made from
our life and living. the above passage?
(c) Our creativity and imagination cannot be (a) Innovation and R&D is the only way to reduce
expressed without digital media. rural to urban migration.
(d) Use of digital systems is imperative for the (b) Every rapidly growing country needs to
existence of mankind in future. minimize the divergence between productivity
in agriculture and other sectors.
Ans. (b) : The present era is the digital era. Today the (c) Inclusive innovation and R&D can help create
digital system exists everywhere and is invisible like an egalitarian society.
air. We live within digital system, transact digitally (d) Rapid urbanization takes place only when a
and interact through digital mediums and the presence country's economic growth is rapid.
and imagination of digital has effectively restructured Ans. (c) : According to the facts mentioned in the
our lives. So, talking about digital technologies is passage, innovation and R & D can reduce the
talking about lifestyle. increasing social inequality and relieve the pressure
Passage-4 arising from rapid urbanization. It has the most logical
and rational assumption that inclusive innovation and
The IMF has pointed out that the fast growing R & D can help build an egalitarian society.
economies of Asia face the risk of falling into 'middle- Passage-6
income trap'. It means that average incomes in these Climate change is likely to expose a large number of
countries, which till now have been growing rapidly, will people to increasing environmental risks forcing them to
stop growing beyond a point—a point that is well short migrate. The international community is yet to recognize
of incomes in the developed West. The IMF identifies a this new category of migrants. There is no consensus on
number of causes of middle-income trap—none of which the definition and status of climate refugees owing to the
is surprising—from infrastructure to weak institutions, to distinct meaning the term refugees carry under
international laws. There are still gaps in understanding
leas than favourable macroeconomic conditions. But the how climate change will work as the root cause of
broad, overall cause, says IMF, is a collapse in the migration. Even if there is recognition of climate
growth of productivity. refugees, who is going to provide protection? More
19. Which among the following is the most logical, emphasis has been given to international migration due
rational and critical inference that can be made to climate change. But there is a need to recognize the
from the above passage? migration of such people within the countries also so that
(a) Once a country reaches middle-income stage, it their problems can be addressed properly.
runs the risk of falling productivity which leads 21. Which of the following is the most rational
to stagnant incomes. inference from the above passage?
(b) Falling into middle-income trap is a general (a) The world will not be able to cope with large
characteristic of fast growing economies. scale migration of climate refugees.
(b) We must find the ways and means to stop
(c) There is no hope at all for emerging Asian
further climate change.
economies to sustain the growth momentum. (c) Climate change will be the most important
(d) As regards growth of productivity, the reason for the migration of people in the future.
performance of Asian economies is not (d) Relation between climate change and migration
satisfactory. is not yet properly understood.
IAS (Pre) GS IInd Paper (CSAT), 2017 104 YCT
Ans. (d) : According to the section, the international (a) 1 only
community is yet to identify the migration category of (b) 1 and 2 only
migrants. Climate change has also led to migration (c) 1 and 3 only
within the country, which is being identified. (d) 1, 2 and 3
Therefore, the most logical inference is that the Ans. (a) : Of the given statements about biopesticides,
relationship between climate charge and migration is only statement (1) is correct that they are not
not yet well understood. dangerous to human health.
Directions for the following 7 (seven) items:
Passage-7 Read the following seven passages and answer
Many farmers use synthetic pesticides to kill the items that follow the passages. Your answers to
infesting insects. The consumption of pesticides in some these items should be based on the Passages only.
of the developed countries is touching 3000 Passage-1
grams/hectare. Unfortunately, there are reports that these An air quality index (AQI) is a way to combine
compounds possess inherent toxicities that endanger the measurements of multiple air pollutants into a single
health of the farm operators, consumers and the number or rating. This index is ideally kept constantly
environment. Synthetic pesticides are generally updated and available in different places. The AQI is
persistent in environment. Entering in food chain they most useful when lots of pollution data are being
destroy the microbial diversity and cause ecological gathered and when pollution levels are normally, but not
imbalance. Their indiscriminate use has resulted in always, low. In such cases, if pollution levels spike for a
development of resistance among insects to insecticides, few days, the public can quickly take preventive action
upsetting of balance in nature and resurgence of treated (like staying indoors) in response to an air quality
populations. Natural pest control using the botanical warning. Unfortunately, that is not urban India. Pollution
pesticides is safer to the user and the environment levels in many large Indian cities are so high that they
because they break down into harmless compounds remain well above any health or regulatory standard for
within hours or days in the presence of sunlight. Plants large part of the year. If our index stays in the
with pesticidal properties have been in nature for Red/Dangerous' region day after day, there is not much
millions of years without any ill or adverse effects on the any one can do, other than getting used to ignoring it.
ecosystem. They are easily decomposed by many 24. Which among the following is the most logical
microbes common in most soil. They help in the and rational inference that can be made from the
maintenance of biological diversity, of predators and the above passage?
reduction of environmental contamination and human (a) Our governments are not responsible enough to
health hazards. Botanical pesticides formulated from keep our cities pollution free.
plants are biodegradable and their use in crop protection (b) There is absolutely no need for air quality
is a practical sustainable alternative. indices in our country.
22. On the basis of the above passage, the following (c) Air quality index is not helpful to the residents
assumptions have been made: of many of our large cities.
1. Synthetic pesticides should never be used in (d) In every city, public awareness about pollution
modem agriculture. problems should increase.
2. One of the aims of sustainable agriculture is to Ans. (c) : According to the passage, "The situation in
ensure minimal ecological imbalance. Urban India is not like this. Pollution level in many
3. Botanical pesticides are more effective as large Indian cities are so high that they remain above
compared to synthetic pesticides. health standards as regulatory standards for most days
Which of the assumptions given above is/are of the year." The most air quality indices are not
correct? helpful for many residents of our big cities.
(a) 1 and 2 only
(b) 2 only Passage-2
(c) 1 and 3 only Productive jobs are vital for growth and a good lob
(d) 1, 2 and 3 is the best form of inclusion. More than half of our
Ans. (b) : One of the most important assumptions population depends on agriculture, but the experience of
made on the basis of the above passage is that one of other countries suggests that the number of people
the objectives of sustainable agriculture is to ensure dependent on agriculture will have to shrink if per capita
minimum ecological imbalance. Hence, option (b) is incomes in agriculture are to go up substantially. While
the only correct answer. industry is creating jobs, too many such jobs are low-
23. Which of the following statements is/are correct productivity non-contractual jobs in the unorganized
regarding biopesticides? sector, offering low incomes, little protection, and no
1. They are not hazardous to human health. benefits. Service jobs are relatively of high productivity,
2. They are persistent in environment. but employment growth in services has been slow in
3. They are essential to maintain the biodiversity recent years.
of any ecosystem. 25. Which among the following is the most logical
Select the correct answer using the code given and rational inference that can be made from the
below. above passage?
IAS (Pre) GS IInd Paper (CSAT), 2017 105 YCT
(a) We must create conditions for the faster growth Passage-4
of highly productive service jobs to ensure The medium term challenge for Indian
employment growth and inclusion. manufacturing is to move from lower to higher tech
(b) We must shift the farm workers to the highly sectors, from lower to higher value-added sectors, and
productive manufacturing and service sectors to from lower to higher productivity sectors. Medium tech
ensure the economic growth and inclusion. industries are primarily capital intensive and resource
(c) We must create conditions for the faster growth processing; and high tech industries are mainly capital
and technology intensive. In order to push the share of
of productive jobs outside of agriculture even
manufacturing in overall GDP to the projected 25 per
while improving the productivity of agriculture. cent, Indian manufacturing needs to capture the global
(d) We must emphasize the cultivation of high- market in sectors showing a rising trend in demand.
yielding hybrid varieties and genetically These sectors are largely high technology and capital
modified crops to increase the per capita income intensive.
in agriculture. 27. Which among the following is the most logical
and rational inference that can be made from the
Ans. (c) : Option (c) is the best answer that can be above passage?
inferred logically this is what we should do now as a (a) India's GDP displays high value-added and high
nation to absorb agriculture workforce and improve productivity levels in medium tech and resource
agriculture itself. processing industries.
(b) Promotion of capital and technology intensive
Passage-3 manufacturing is not possible in India.
(c) India should push up the public investments and
A Landscape-scale approach to land use can encourage the private investments in research
encourage greater biodiversity outside cdected areas. and development, technology upgradation and
During hurricane 'Mitch' in 1998, farms using eco skill development.
agricultural practices suffered 58 percent, 70 percent and (d) India has already gained a great share in global
99 percent less damage in Honduras, Nicaragua and markets in sectors showing a rising trend in
demand.
Guatemala, respectively, than farms using conventional
Ans. (c) : The given passage is on 'Manufacturing in
techniques. In Costa, vegetative windbreaks and India'. We need to find the most logical and rational
fencerows boosted farmers' income from pasture and inference- that the idea of Indian manufacturing needs
coffee while also increasing bird diversity. Bee to go from low to high in the areas of productivity,
pollination is more effective when agricultural fields are technology and skill development.
closer to natural or seminatural habitat, a finding that Passage-5
matters because 87 percent of the world's 107 leading Over the last decade, Indian agriculture has become
crops depend on animal pollinators. In Costa Rica, more robust with record production of food grains and
Nicaragua and Colombia silvopastoral systems t tint oilseeds. Increased procurement, consequently, has
added huge of food grains in the granaries. India is one
integrate trees with pastureland are improving the of the world's top producers of rice, wheat, milk, fruits
sustainability of cattle production, and diversifying and and vegetables. India is still home the quarter of all
increasing farmers' income. undernourished people in the world. On an average,
26. Which among the following is the most logical almost half of the total expenditure of nearly half of the
and rational inference that can be made from the households is on food.
above passage? 28. Which among the following is the most logical
corollary to the above passage?
(a) Agricultural practices that enhance biodiversity (a) Increasing the efficiency of farm to-fork value
can often increase farm output and reduce the chain is necessary to reduce the poverty and
vulnerability to disasters. malnutrition.
(b) All the countries of the world should be (b) Increasing the agricultural productivity will
encouraged to replace ecoagriculture with automatically eliminate the poverty and
conventional agriculture. malnutrition in India.
(c) India's agricultural productivity is already great
(c) Ecoagriculture should be permitted in protected
and it is not necessary to increase it further.
areas without destroying the biodiversity there. (d) Allocation of more funds for social welfare and
(d) The yield of food crops will be very high if poverty alleviation programmes will ultimately
ecoagricultural practices are adopted to cultivate eliminate the poverty and malnutrition in India.
them. Ans. (a) : India is an agricultural country. Most of the
Ans. (a) : The most logical and rational inference that people here are dependent on agriculture. But today
can be drawn on the basis of the section of the farmer of the country is compelled to commit
suicide. He is suffering from poverty and malnutrition.
Biodiversity is as follows - Agricultural practices that Therefore, it is necessary to increase the efficiency of
increases biodiversity often increases farm production the value chain from farm to plate to prevent poverty,
and reduce the vulnerability to disasters. malnutrition and suicide.
IAS (Pre) GS IInd Paper (CSAT), 2017 106 YCT
Passage-6 (c) No supporters of 'party X' supported Z's
campaign strategy.
The States are like pearls and the Centre is the
(d) No supporters of 'party X' knew Z.
thread which turns them into a necklace; if the read
Ans. (b) :
snaps, the pearls are scattered.
29. Which one of the following views corroborates
the above statement?
(a) A strong Centre and strong States make the
federation strong. and
(b) A strong Centre is a binding force for national
integrity.
(c) A strong Centre is a hindrance to State autonomy.
by flow chart,
(d) State autonomy is a prerequisite for a federation.
"There is at least one supporter of party y who knew
Ans. (b) : Our country India is a country with many some supporters of party 'x' as a friend". Must be true.
states. The living style, dialect, costume of each state 32. With reference to the above information,
is different. These states are controlled by the consider the following statement.
governments there, but the central government 1. Some supporters of 'party X' knew Z
controls the country that has been formed by 2. Some supporters of 'party X', who opposed Z's
combining all these states. therefore, a powerful center campaign strategy, knew Z.
is a binding force for national integrity. 3. No supporters of 'party X' supported Z's
Passage-7 campaign strategy
Which of the statements given above, is/are not
Really I think that the poorest he that is in England correct?
has a life to live, as the greatest he, and therefore truly, I (a) 1 only
think it is clear that every inan that is to live under a (b) 2 and 3 only
government ought first by his own consent to put himself (c) 3 only
under the government, and I do think that the poorest (d) 1, 2 and 3
man in England is not at all bound in a strict sense to that Ans. (c) : Statement 1 is correct as it is given that
government that he has not had a voice to put himself there are some supporters of party X who knew Z and
under. supported his campaign strategy
30. The above statement argues for Statement 2 : nothing is mentioned about those
(a) distribution of wealth equally to all supporters of party X who opposed Z
Statement 3 - is not correct as it goes against the
(b) rule according to the consent of the governed
information given in the question. Hence option C
(c) rule of the poor
correct Ans.
(d) expropriation of the rich
33. Four tests—Physics, Chemistry, Mathematics
Ans. (b) : The statement of the given passage is and Biology are to be conducted on four
presenting an argument in support of government consecutive days, not necessarily in the same
according to the consent of the governed', because in order. The Physics test is held before the test
this passage the person has been asked to submit which is conducted after Biology. Chemistry is
himself under the government with his consent. conducted exactly after two tests are held. Which
is the last test held?
LOGICAL AND ANALYTICAL ABILITY (a) Physics (b) Biology
(c) Mathematics (d) Chemistry
Directions for the following 2 (two) items:
Ans. (c) : According to question,
Consider the given information and answer the
Physics, Biology, Chemistry, Mathematics.
two items that follow.
Hence,
No supporters of 'party X', who knew Z and
The last test is held mathematics.
supported his campaign strategy, agreed for the alliance
34. The sum of income of A and B is more than that
with 'party Y’; but some of them had friends in 'party Y'.
of C and D taken together. The sum of income of
31. With reference to the above information, which A and C is the same as that of B and D taken
one among the following statements must be true? together. Moreover, A earns half as much as the
(a) Some supporters of 'party Y' did not agree for sum of the income of B and D. Whose income is
the alliance with the `party X'. the highest?
(b) There is at least one supporter of `party Y' who (a) A (b) B
knew some supporters of 'party X' as a friend. (c) C (d) D

IAS (Pre) GS IInd Paper (CSAT), 2017 107 YCT


Ans. (b) : A+B > C+D –––– (i)
A+C = B+D –––– (ii)
(B+D) = 2A –––– (iii)
from eq (ii) & (iii)
A+C = 2A
C=A Hence,
Putting the value of 'c' in eq (i), Q is the father of P.
A+B > A+D 38. In a group of six women, there are four tennis
B>D players, four postgraduates in Sociology, one
'B's income is the highest. postgraduate in Commerce and three bank
35. Consider the following : employees. Vimala and Kamla are the bank
Statement : Good voice is a natural gift but one has employees while Amala and Komala are
to keep practising to improve and well in the field of unemployed. Komala and Nirmala are among the
music. tennis players. Amala, Kamla, Komala and
Conclusions: Nirmala are postgraduates in Sociology of whom
I. Natural gifts need nurturing and care. two are bank employees. If Shyamala is a
II. Even though one's voice is not good, one can postgraduate in Commerce, who among the
keep practising. following is both a tennis player and a bank
Which one of the following is correct, in respect employee?
(a) Amala (b) Komala
of the above statement and conclusions?
(c) Nirmala (d) Shyamala
(a) Only conclusion I follows from the statement.
(b) Only conclusion II follows from the statement. Ans. (c) :
(c) Either conclusion I or conclusion II follows Women Work Player Study
from the statement. Vimla Bank – –
(d) Neither couclusion I nor conclusion II follows Kamla Bank – Sociology
from the statement. Amla Unemployed – Sociology
Ans. (a) : Conclusion 1- Natural gifts need nurturing Komala Unemployed Tennis Sociology
and care this is the correct because it is mentioned in Nirmala Bank Tennis Sociology
the statement that one needs to keep practicing to Shyamala – – Commerce
improve and excel. Hence,
36. "Rights are certain advantageous conditions of Nirmala is both a tennis player and a bank
social well being indispensable to the true employee.
development of the citizen." 39. P = (40% of A) + (65% of B) and
In the light of this statement, which one of the Q = (50% of A) + (50% of B)
following is the correct understanding of rights? Where A is greater than B.
(a) Rights aim at individual good only. In this context, which of the following statements
(b) Rights aim at social good only. is correct?
(c) Rights aim at both individual and social good. (a) P is greater than Q.
(d) Rights aim at individual good devoid of social (b) Q is greater than P.
well-being. (c) P is equal to Q.
Ans. (c) : In respect of statement "Rights are certain (d) None of the above can be concluded with
advantageous condition of social well being certainty.
indispensable to the development of the citizen," Ans. (d) :
Rights aim at both individual and social good. ∵A > B
37. Consider the following: Condition (I)
A+ B means A is the son of B. ∴Let A = 100 and B = 80
A - B means A is the wife of B. then + = (40% of A) + (65% of B)
What does the expression P + R - Q mean?  40   65 
P= × 100  +  × 80 
(a) Q is the son of P.  100   100 
(b) Q is the wife of P.
 65 
(c) Q is the father of P. P = ( 40 ) +  × 4 
(d) None of the above  5 
P = 40 + 52
Ans. (c) : ∵ A + B means A is the son of B.
P = 92
A – B means A is the wife of B
and
then (P + R – Q) Q = (50% of A) + (5% of B)
P + R → P is the son of R
 50   50 
R – Q → P is the wife of Q Q= ×100  +  × 80 
by sum up,  100   100 

IAS (Pre) GS IInd Paper (CSAT), 2017 108 YCT


Q= (50 + 40) Ans. (b) :
Q = 90 Boys ⇒
Hence,
from condition (I), P = 92
Q = 90
P>Q
Again,
∴ D + E = 15 + 11 ⇒ 26
Condition (II)
by option (b)
∵A>B B + C + f = 7 + 12 + 7
A = 100, B = 50 = 26
then P = (40% of A) + (65% of B) 42. In a test, Randhir obtained more marks than the
 40   65  total marks obtained by Kunal and Debu. The
P= × 100  +  × 50  total marks obtained by Kunal and Shankar are
 100   100  more than those of Randhir. Sonal obtained more
P = 40 + 32.5 marks than Shankar. Neha obtained more marks
P = 72.5 than Randhir. Who amongst them obtained
highest marks?
and Q = (50% of A + 50% of B) (a) Randhir
 50 50  (b) Neha
Q= × 100 + × 50  (c) Sonal
 100 100 
(d) Data are inadequate
= 50 + 25
Ans. (d) :
Q = 75 Randhir > Kunal + Debu………(i)
Kunal + Shankar > Randhir……..(i)
Hence, from condition (II)
from eqn (i) & (ii)
P = 72.5 Randhir + Kunal + Shankar > Kunal + Debu +
Q = 75 Randhir
∵ Q>P Shankar > Debu − ( iii )
Thus, and Sonal > Shankar − ( iv )
None of above can be concluded with certainty. from eqn (iii) & (iv)
40. Consider the given statement and the two Sonal > Shankar > Debu − ( v )
conclusions that follow :
Statement : Morning walk is good for health. and Neha > Randhir − ( vi )
Conclusions : So the data are inadequate for who obtained highest
1. All healthy people go for morning walk. marks.
2. Morning walk is essential for maintaining good 43. Examine the following statements :
health. 1. All colours are pleasant
2. Some colours are pleasant
What is/are the valid conclusion/ conclusions?
3. No colour is pleasant
(a) 1 only 4. Some colours are not pleasant.
(b) 2 only Given that statement 4 is true, what can be
(c) Both 1 and 2 definitely concluded?
(d) Neither 1 nor 2 (a) 1 and 2 are true.
Ans. (d) : Since the given statement "Morning walk is (b) 3 is true.
good for health" does not means that all healthy (c) 2 is false.
people go for morning walk and morning walk is (d) 1 is false.
essential for maintaining good health. Ans. (d) : If statement 4, "Some colours are not
Hence, Neither 1 nor 2 conclusion is valid. pleasant" is true, then statement 1, "All colours are
41. Six boys A, B, C, D, E and F play a game of pleasant" is definitely false.
cards. Each has a pack of 10 cards. F borrows 2 44. Consider the following relationships among
cards from A and gives away 5 to C who in turn members of a family of six persons A, B, C, D, E
gives 3 to B while B gives 6 to D who passes 1 to and F :
E. Then the number of cards possessed by D and 1. The number of males equals that of females.
E is equal to the number of cards possessed by 2. A and E are sons of F.
(a) A, B and C 3. D is the mother of two, one boy and one girl.
(b) B, C and F 4. B is the son of A.
(c) A, B and F 5. There is only one married couple in the family
(d) A, C and F at present.
nd
IAS (Pre) GS II Paper (CSAT), 2017 109 YCT
Which one of the following inferences can be Directions for the following 3 (three) items:
drawn from the above? Consider the given information and answer the three
(a) A, B and C are all females. items that follow.
(b) A is the husband of D. A, B, C, D, E, F and G are Lecturers from different
(c) E and F are children of D. cities—Hyderabad, Delhi, Shillong, Kanpur, Chennai,
(d) D is the daughter of F. Mumbai and Srinagar (not necessarily in the same order)
who participated in a conference. Each one of them is
Ans. (b) : specialized in a different subject, viz., Economics,
Commerce, History, Sociology, Geography,
Mathematics and Statistics (not necessarily in the same
order). Further
1. Lecturer from Kanpur is specialized in Geography
2. Lecturer D is from Shillong
3. Lecturer C from Delhi is specialized in Sociology
4. Lecturer B is specialized in neither History nor
Mathematics
5. Lecturer A who is specialized in Economics does
+ = Male not belong to Hyderabad
– = Female 6. Lecturer F who is specialized in Commerce belongs
∵ There s only one married couple in the family at to Srinagar
7. Lecturer G who is specialized in Statistics belongs to
present and B, A and E are males and C, D and F are Chennai
females. 47. Who is specialized in Geography?
Hence "A is the husband of D" (a) B
45. Consider the following : (b) D
A, B, C, D, E, F, G and H are standing in a row (c) E
facing North. (d) Cannot be determined as data are inadequate
B is not neighbour of G. Ans. (a) :
F is to the immediate right of G and neighbour of E. A → Mumbai → Economics
G is not at the extreme end. B → Kanpur → Geography
A is sixth to the left of E. C → Delhi → Sociology
H is sixth to the right of C. D → Shillong → History
Which one of the following is correct in respect of E → Hyderabad → Mathematics
the above? F → Srinagar → Commerce
(a) C is to the immediate left of A. G → Chennai → Statistics
(b) D is immediate neighbour of B and F. Hence,
(c) G is to the immediate right of D. B is specialized in Geography.
(d) A and E are at the extreme ends. 48. To which city does the Lecturer specialized in
Ans. (c) : Economics belong?
(a) Hyderabad
(b) Mumbai
(c) Neither Hyderabad nor Mumbai
(d) Cannot be determined as data are inadequate
Ans. (b) :
Hence, "G is to the immediate right of D" The lecturer specialized in economics belongs to
46. In a certain code, '256' means 'red colour chalk', Mumbai.
'589' means 'green colour flower' and '254' 49. Who of the following belongs to Hyderabad?
means 'white colour chalk'. The digit in the code (a) B
that indicates ‘white’ is (b) E
(a) 2 (b) 4 (c) 5 (d) 8 (c) Neither B nor E
(d) Cannot be determined as data are inadequate
Ans. (b) :
Ans. (b) : "E" belongs to Hyderabad.
50. In a school, there are five teachers A, B, C, D and
E. A and B teach Hindi and English. C and B
teach English and Geography. D and A teach
Mathematics and Hindi. E and B teach History
and French. Who teaches maximum numb, of
subjects?
(a) A (b) B
Hence 4 → White (c) D (d) E

IAS (Pre) GS IInd Paper (CSAT), 2017 110 YCT


Ans. (b) : 55. If there is a policy that 1/3rd of population of a
Teacher Subject community has migrated every year from one
A Hindi English Mathematics – place, to some other place, what is the leftover
B Hindi English Geography History French population of that community after the sixth
C – English Geography – – year, if there is no further growth in the
D Hindi – Mathematics – –
population during this period?
(a) 16/243rd part of the population
E – – – History French (b) 32/243rd part of the population
Directions for the following 3 (three) items : (c) 32/729th part of the population
Consider the given information and answer the three (d) 64/729th part of the population
items that follow. 6
Eight railway stations A, B, C, D, E, F, G and H are  1
Ans. (d) : The leftover population = = 1 − 
connected either by two-way passages or one-way  3
passages. One-way passages are from C to A, E to G, B 6
to F, D to H, G to C, E to C and H to G. Two-way 2 64
=  = th part
passages are between A and E, G and B, F and D, and E 3 729
and D. 56. 15 students failed in a class of 52. After removing
51. While travelling from C to H, which one of the the names of failed students, a merit order list
following stations must be passed through? has been prepared in which the position of
(a) G (b) E (c) B (d) F
Ramesh is 22nd from the top. What is his position
Ans. (b) : from the bottom?
(a) 18th (b) 17th
th
(c) 16 (d) 15th
Ans. (c) :
∵ Total number of students = 52
While travelling from C to H, station 'E' must be and number of failed student = 15
passed through.
∴ Number of passed student = 52 – 15
Route ⇒ C →A ⇔ E ⇔ D →H
= 37
52. In how many different ways can a train navel Then, Ramesh's position from the bottom
from F to A without passing through any station = (37 – 22 + 1)
more than once? = (15 + 1)
(a) 1 (b) 2 (c) 3 (d) 4 = 16th
Ans. (d) : Route from F to A:- 57. There are three pillars X, Y and Z of different
(i) F⇔D→H→G→C→A heights. Three spiders A, B and C start to climb
(ii) F⇔D⇔E⇔A on these pillars simultaneously. In one chance, A
(iii) F⇔D⇔E→G→C→A climbs on X by 6 cm but slips down 1 cm. B
(iv) F⇔D⇔E→C→A climbs on Y by 7 cm but slips down 3 cm. C
Hence, 4 different ways. climbs on Z by 6.5 cm but slips down 2 cm. If
53. If the route between G and C is closed, which one each of them requires 40 chances to reach the top
of the following stations need not be passed of the pillars, what is the height of the shortest
through while travelling from H to C? pillar?
(a) E (b) D (c) A (d) B (a) 161 cm (b) 163 cm
Ans. (c) : If the route between G and C is closed. (c) 182 cm (d) 210 cm
Then, Ans. (b) :
The route si:- The height climbed by spider 'A' in 40 Attempts
H→G⇔B→F⇔ D⇔E→C ⇒ 39 (6 – 1) + 6
Hence, there is no need to pass through station "A"
= 39 × 5 + 6
= 195 + 6
GENERAL MENTAL ABILITY = 201 cm
The height climbed by spider 'B' in 40 attempts
54. If second and fourth Saturdays and ,all the
Sundays are taken as only holidays for an office, ⇒ 39 (7 – 3) + 7
what would be the minimum number of possible ⇒ 39 × 4 + 7
working days of any month of any year? = 156 +7
(a) 23 (b) 22 (c) 21 (d) 20 ⇒ 163 cm
Ans. (b) : The height climbed by spider 'C' in 40 attempts
Let taking month of February of non-leap year = 39 (5.5 – 2) + 6.5
Then, the total number of minimum holidays = 39 × 4.5 + 6.5
= 4 (Sunday) + 2 (Saturday) = 175.5 + 6.5
=6 = 182.0 cm
∴ Minimum working days = 28 – 6 Hence,
= 22 The height of shortest pillar is 163 cm.

IAS (Pre) GS IInd Paper (CSAT), 2017 111 YCT


58. A watch loses 2 minutes in every 24 while another 61. A bag contains 20 balls. 8 balls are green, 7 are
watch gains 2 minutes, in 24 hours. At a white and 5 are red. What is Hie minimum
particular instant, the two watches showed an number of balls that must be picked up from the
identical time. Which of the following statements bag blindfolded (without replacing any of it) to be
assured of picking at least one ball of each colour?
is correct if 24-hour clock is (a) 17 (b) 16 (c) 13 (d) 11
(a) The two watches show the identical time again Ans. (b) :
on completion of 30 days. To be assured of picking at least one ball of each
(b) The two watches show the identical time again colour = 8 + 7 + 1
on completion of 90 days. = 16 balls.
(c) The two watches show the identical time again 62. If 2 boys and 2 girls are to be arranged in a row
on completion of 120 days. so that the girls are not next to each other, how
(d) None of the above statements correct. many possible arrangements are there?
Ans. (d) : (a) 3 (b) 6 (c) 12 (d) 24
∵ A watch loses 2 minutes in every 24 hours while Ans. (c) :
another watch gains 2 minutes in 24 hours. 4! 4×3×2×1
All possible arrangements = = = 12
∴ The difference in 24 hours ⇒ 4 min 2! 2×1
The difference in 1 day = 4 min 63. The outer surface of a 4 cm x 4 cm x 4 cm cube is
by option A:- [∵ 1 day = 24 hours] painted completely in red. It is sliced parallel to
the faces to yield sixty four 1 cm x 1 cm x 1 cm
The difference in 30 days = 4 × 30
small cubes. How many small cubes do not have
= 120 min painted faces?
= 2 hrs (a) 8 (b) 16 (c) 24 (d) 36
by option B:-
The difference in 90 days = 4 × 90 Ans. (a) :
= 360 min ∵ The length of larger cube = 4 cm.
= 6 hrs and the length of smaller cube = 1 cm.
by option C:-
The difference in 120 days = 4 × 120
= 480 min
= 8 hrs
Hence, None of the given statement correct option 'd'
is correct.
59. A clock strikes once at 1 o'clock, twice at 2
o'clock and thrice at 3 o'clock, and so on. If it
takes 12 seconds to strike at 5 o'clock, what is the
time taken by it to strike at 10 o'clock? ∵ The number of cubes which is sliced from one side
(a) 20 seconds (b) 24 seconds = 4 (four cut)
(c) 28 seconds (d) 30 seconds ∴ The number of colourless (not painted) cube
Ans. (b) : = (x – 2)3 = (4 – 2)3 = (2)3 = 8
∵ A clock strikes one at 1 o'clock, twice at 2 o'clock
and thrice at 3 o'clock and so on. General Mathematics/Numerical Ability
∴ The clock strikes 5 times at 5 o'clock.
then, taken time to strike 5 times = 12 sec. 64. Gopal bought a cell phone and sold it to Ram at
12 10% profit. Then Ram wanted to sell it back to
∴ taken time to strike 10 timer = × 10sec Gopal at 10% loss. What will be Gopal's position
5 if he agreed?
= 24 sec (a) Neither loss nor gain (b) Loss 1%
∵ The clock strikes 10 times at 10 o'clock. (c) Gain 1% (d) Gain 0.5%
Hence, The time taken by it to strike at 10 o'clock is Ans. (c) :
24 second. Suppose that Cell phone price = 100 Rs.
60. There are 4 horizontal and 4 vertical lines,
parallel and equidistant to one another on a
board. What is the maximum number of
rectangles and squares that can be formed?
(a) 16 (b) 24 (c) 36 (d) 42
Ans. (c) :
The number of squares = 12 + 22 + 32 = 14 Now Ram sold Cell phone to Gopal
The number of rectangle = (13+23+33) – (12+22+32) 110 × 90
= 36 –14 = 22 = 99
The maximum number of rectangles and squares 100
= 14+22 = 36 Gopal's Profit = 100 – 99 =1%

IAS (Pre) GS IInd Paper (CSAT), 2017 112 YCT


65. Suppose the average weight of 9 persons is 50 kg. then, Series⇒ 35, 37, 39 …………….13th
The average weight of the first 5 persons is 45 kg, n
whereas the average weight of the last 5 persons Sn  2a + ( n − 1) d 
2
is 55 kg. Then the weight of the 5th person will be
13
(a) 45 kg (b) 47.5 kg S13 =  2 × 35 + (13 − 1) 2 
(c) 50 kg (d) 52.5 kg 2
13
Ans. (c) : S13 = [ 70 + 24]
Total weight of 9 persons = 50 × 9 = 450 kg Sn= 2
Total weight of the first 5 person = 45 × 5 = 225 kg 13
and total weight of the last 5 person = 55 × 5 = 275 kg ( 70 + 24 )
Re quired Average = 2
= (225 + 275) – 450 13
= 500 – 450 = 50 kg
1
66. In a city, 12% of households earn less than Rs. = [94]
2
30,000 per year, 6% households earn more than
Rs. 2,00,000 per year, 22% households earn more = 47
than Rs. 1,00,000 per year and 990 households 68. There is a milk sample with 50% water in it. If
earn between Rs. 30,000 and Rs. 1,00,000 per 1/3rd of this milk is added to equal amount of
year. How many households earn between Rs. pure milk, then water in the new mixture will fall
1,00,000 and Rs. 2,00,000 per year? down to
(a) 250 (b) 240
(a) 25% (b) 30%
(c) 230 (d) 225
(c) 35% (d) 40%
Ans. (b) :
6% > 20,0000…………..(i) Ans. (a) :
22% > 100000…………(ii) Let total amount of mixture = x
12% < 30,000…………..(iii) 1 1
from eqn (i) & (ii) rd of mixture = x
3 3
20,0000 > (20 – 6)% >10,0000
20,0000 > 16% > 10,000
The percentage between 30,000 and 10,0000
households=(100-(16+12+6)]% ⇒ Water Milk
→(100–34)% 1 1 1 1
=66% ×x ×x
3 2 3 2
∴ 66%=990 households
x x
990 ⇒
∴100%= × 100 households ⇒ 1500 6 6
66
The number of households earn between 100000 to x
∵ pure milk added to mixture,
16 3
20000 ⇒ 1500× 100 x x 2x
Total amount of mixture = + =
= 240 3 3 3
67. There are thirteen 2-digit consecutive odd Hence, The percentage of water in new
numbers. If 39 is the mean of the first five such x
numbers, then what is the mean of all the thirteen
numbers? mixture = 6 ×100%
2x
(a) 47 (b) 49 3
(c) 51 (d) 45
x 3
Ans. (a) : ⇒ × ×100% = 25%
6 2x
Let the first number =x
69. A freight train left Delhi for Mumbai at an
(a)
average speed of 40 km/hr. Two hours later, an
x + ( x + 2) + ( x + 4) + ( x + 6) + ( x + 8)
= 39 express train left Delhi for Mumbai, following the
5
freight train on a parallel track at an average
5x + 20
then, = 39 speed of 60 km/hr. How far from Delhi would the
5
express train meet the freight train?
x + 4 = 39
(a) 480 km (b) 260 km
x = 35 (c) 240 km (d) 120 km
IAS (Pre) GS IInd Paper (CSAT), 2017 113 YCT
Ans. (c) : 72. If for a sample data
Mean < Median < Mode
Then, the distribution is
(a) symmetric
(b) skewed to the right
(c) neither symmetric nor skewed
(d) skewed to the left
Ans. (d) :
∵ distance =speed × time
∴ d= 40 ×t ---------- (i)
d= 60 × (t – 2) ------ (ii) If for sample data
From eg (i) & (ii) mean < median <mode
40t = 60 (t –2) then the distribution skewed to the left
40t =60 60t –120 and if for a sample date
t = 6hr mean > median > mode
∴Required distance ⇒ d =40 ×t then the distribution is skewed to the right
d =40 ×6 73. The age of Mr. X last year was the square of a
d =240/cm number and it would be the cube of a number
70. Certain 3-digit numbers following characteristics : next year. What is the least number of years he
1. All the three digits are different. must wait for his age to become the cube of a
2. The number is divisible by 7. number again?
3. The number on reversing the digits is also (a) 43 (b) 38 (c) 25 (d) 16
divisible by 7. Ans. (b) :
How many such 3-digit numbers are there? According to question,
(a) 2 (b) 4 (c) 6 (d) 8 A3–B2 =2
Ans. (b) The Numbers are : If, A = 3, B = 5
168 (3)3 – (5)2 =2
861 27 – 25 = 2
259 2=2
952 L.H.S. =R.H.S.
So that total Numbers (4). Hence,
71. How many numbers are there between 99 and Present age = 26
1000 such that the digit 8 occupies the units ∵ (3)3 = 27
place?
and (4)3 = 64
(a) 64 (b) 80 (c) 90 (d) 104 Required year for waiting = 64 – 26
Ans. (c) : First number = a ⇒108 =38 years
and last number =l ⇒998
74. P works thrice as fast as Q, whereas P and Q
∴ difference d ⇒ 10 together can work four times as fast as R. If P, Q
∵ In = a + (n –1) d and R together work on a job, in what ratio
should they share the earnings?
l = a + (n – 1) d (a) 3 : 1 : 1 (b) 3 : 2 : 4
998 =108 + (n – 1)10
(c) 4 : 3 : 4 (d) 3 : 1 : 4
998 –108 = (n – 1)10
Ans. (a) :
990
− n −1 P : Q = 3 : 1 ______ (i)
10
(a) and (P + Q) : R = 4 : 1____ (ii)
89 – n – 1
n = 89 + 1 From eq (i) & (ii),
n = 90 ∴P:Q:R=3:1:1

IAS (Pre) GS IInd Paper (CSAT), 2017 114 YCT


75. The average rainfall in a city for the first four 78. Two walls and a ceiling of a room meet at right
days was recorded to be 0.40 inch. The rainfall on angles at a point P. A fly is in the air 1 m from
the last two days was in the ratio of 4 : 3. The one wall, 8 m from the other wall and 9 m from
the point P. How many meters is the fly from the
average of six days was 0.50 inch. What was the
ceiling?
rainfall on the fifth day? (a) 4 (b) 6 (c) 12 (d) 15
(a) 0.60 inch (b) 0.70 inch Ans. (a) :
(c) 0.80 inch (d) 0.90 inch
Ans. (c) :
The total rainfall for the first
(c) Four days = 4 × 0.40 = 1.60
the total rainfall for 6 days = 6 × 0.50
= 3.00
∴ The total rainfall for 2 days = 3.00 – 1.60
=1.40
According to question,
4x + 3x =1.40
7x =1.40
x = 0.20 ∵ d = l 2 + b2 + h 2
∴ The rainfall of 5th day = 4 × 0.20 l = 1, b=8, h= ?
(1) + (8 )
2 2
= 0.80 inch 9= + h2
76. A 2-digit number is reversed. The larger of the
two numbers is divided by It smaller one. What is 9= 1 + 64 + h 2
the largest possible remainder? 9 = 65 + h 2
(a) 9 (b) 27 (c) 36 (d) 45 81=65 +h2
Ans. (d) : h2 =81–65
If two digit number = 49 h2 =16
then two digit reversed number = 94 h=4
when the larger number is divided by small number, 79. There are certain 2-digit numbers. The difference
94 between the number and the one obtained on
∴ ⇒ Remainder = 45 reversing it is always 27. How many such
49
maximum 2-digit numbers are there?
77. The monthly incomes of X and Y are in the ratio (a) 3 (b) 4
of 4 : 3 and their monthly expenses are in the (c) 5 (d) None of the above
ratio of 3: 2. However, each saves Rs. 6,000 per Ans. (d) :
month. What is their total monthly income? Total 2-digit numbers satisfying the conditions given
(a) Rs. 28,000 (b) Rs. 42,000 in the questions :
(c) Rs. 56,000 (d) Rs. 84,000 14, 25, 36, 47, 58, 69
According to question,
Ans. (b) : 41–14 = 27 52–25=27
Let the monthly income of x and y are 4k, 3k 63–36=27 74–47 =27
According to question, 85–50=27 96–69 =27
4k − 6000 3 Hence, total number of required two-digit number=6
= [∵ income – saving = Expenditure]
3k − 6000 2 80. What is the total number of digits printed, if a
2(4k – 6000) =3(3k – 6000) book containing 150 pages is to numbered from 1
to 150?
8k – 12000 = 9k –18000
(a) 262 (b) 342 (c) 360 (d) 450
6000 = k Ans. (b) :
∵Total income x and y = (4k + 3k) Page No. Number of digit
From 1 to 9 → 1×9=9
= 7k From 10 to 99 → 2 × 90 = 180
= 7 × 6000 From 100 to 150 → 3 × 51 = 153
= 42000 Total Numbers of digit printed = 342

IAS (Pre) GS IInd Paper (CSAT), 2017 115 YCT


UNION PUBLIC SERVICE COMMISSION
Civil Services (Preliminary Exam) - 2018
CSAT : PAPER-II
(Chapterwise Analysis with Explanation)
Time : 2 hours Maximum Number : 200
illnesses is unknown. Improvements in international
COMPREHENSION monitoring have led to greater public awareness, yet the
rapid globalization of food production increases
Directions for the following 4 (four) items :
consumers' vulnerability by making food harder to
Read the following four passages and answer the items
regulate and trace. "We have the world on our plates",
that follow. Your answers to these items should be based
says an official of WHO.
on the passages only.
2. Which of the following is the most logical
Passage–1 corollary to the above passage?
Global population was around 1.6 billion in 1990–today (a) With more options for food come more risks.
it is around 7.2 billion and growing. Recent estimates on
(b) Food processing is the source of all food
population growth predict a global population of 9.6
borne illnesses.
billion in 2050 and 10.9 billion in 2100. Unlike Europe
and North America, where only three to four per cent of (c) We should depend on locally produced food
population is engaged in agriculture, around 47 per cent only.
of India's population is dependent upon agriculture. Even (d) Globalization of food production should be
if India continues to do well in the service sector and the curtailed.
manufacturing sector picks up, it is expected that around Ans. (a) As the most logical and corollary from the
2030 when India overtakes China as the world's most above passage, there are more risks due to more choices
populous country, nearly 42 per cent of India's of food as there has been an increase in public
population will still be predominantly dependent on awareness due to the improvement in the international
agriculture. production, food is controlled and traced. Being more
1. Which of the following is the most logical and difficult, the vulnerability of the consumer has increased
rational inference that can be made from the and the outbreak of diseases has also increased.
above passage?
Passage–3
(a) Prosperity of agriculture sector is of critical
importance to India. I am a scientist, privileged to be somebody who tries to
understand nature using the tools of science. But it is also
(b) Indian economy greatly depends on its
agriculture. clear that there are some really important questions that
science cannot really answer, such as : Why is there
(c) India should take strict measures to control its
rapid population growth. something instead of nothing? Why are we here? In those
domains, I have found that faith provides a better path to
(d) India's farming communities should switch
over to other occupations to improve their answers. I find it oddly anachronistic that in today's
economic conditions. culture there seems to be a widespread presumption that
scientific and spiritual views are incompatible.
Ans. (a) The most logical and rational inference that
3. Which of the following is the most logical and
can be made from the above passage, the prosperity of
the agricultural sector is of critical importance to India rational inference that can be made from the
because agriculture is the backbone of the Indian above passage?
economy because about 47% of India's population is (a) It is the faith and not science that can finally
dependent on agriculture. Unlike India, not only in the solve all the problems of mankind.
major continents of the world like Europe and North (b) Science and faith can be mutually
America, only 3-4 percent of the population is engaged complementary if their proper domains are
in agriculture. understood.
Passage–2 (c) There are some very fundamental questions
Many pathogens that cause food borne illnesses are which cannot be answered by either science
unknown. Food contamination can occur at any stage of faith.
from farm to plate. Since most cases of food poisoning (d) In today's culture, scientific views are given
go unreported, the true extent of global food borne more importance than spiritual views.
IAS (Pre) GS IInd Paper (CSAT), 2018 116 YCT
Ans. (b) The last line of the passage, "I find it oddly children as passive receivers of knowledge, and to move
anachronistic that in today's culture there seems to be a beyond the convention of using textbooks as the basis of
widespread presumption that scientific and spiritual examinations. The teaching-learning process must
views are incompatible", provides the inference that become stress-free; and a massive programme for
science and faith can be mutually complementary. The curricular reform should be initiated to provide for a
correct option is (b). child-friendly learning system, that is more relevant and
Passage–4 empowering. Teacher accountability systems and
Though I have discarded much of past tradition and processes must ensure that children are learning, and that
custom, and am anxious that India should rid herself of their right to learn in a child-friendly environment is not
all shackles that bind and contain her and divide her violated. Testing and assessment systems must be
people, and suppress vast numbers of them, and prevent reexamined and redesigned to ensure that these do not
the free numbers of them, and prevent the free force children to struggle between school and tuition
development of the body and the spirit; though I seek all centres, and bypass childhood.
this, yet I do not wish to cut myself off from that past 5. According to the passage, which of the
completely. I am proud of that great inheritance that has following is/are of paramount importance
been and is, ours and I am conscious that I too, like all of under the Right to Education?
us, am a link in that unbroken chain which goes back to 1. Sending of children to school by all parents
the dawn of history in the immemorial past of India. 2. Provision of adequate physical infrastructure
4. The author wants India to rid herself of certain in schools
past bonds because 3. Curricular reforms for developing child-
(a) he is not able to see the relevance of the past friendly learning system
(b) there is not much to be proud of Select the correct answer using the code given
(c) he is not interested in the history of India below.
(d) they obstruct her physical and spiritual (a) 1 only (b) 1 and 2 only
growth (c) 3 only (d) None of the above
Ans. (d) It can be inferred from the first line of the
Ans. (c) According to the passage, of paramount
passage especially from the past "India should rid
importance under the right to education, curriculum
herself……..development of the body and the spirit".
reforms are extremely important to develop learner-
Directions for the following 4 (four) items : friendly learning systems. Through the passage, it has
Read the following passage and answer the four items been discussed by the author that providing school
that follow. Your answers to these items should be facilities is a necessary pre-requisite, but to ensure that
based on the passage only. all children go to school and participate in the process
Passage of learning. Hence, option (a), (b) and (d) are false only
It is no longer enough for us to talk about providing option (c) is true.
available for universal access to education. Making 6. With reference to the above passage, the
available schooling facilities is an essential prerequisite, following assumptions have been made :
but is insufficient to ensure that all children attend school 1. The Right to Education guarantees
and participate in the learning process. The school may
teachers' accountability for the learning
be there, but children may not attend or they may drop
process of children.
out after a few months. Through school and social
mapping, we must address the entire gamut of social, 2. The Right to Education guarantees 100%
economic, cultural and indeed linguistic and pedagogic enrolment of children in the schools.
issues, factors that prevent children from weaker sections 3. The Right to Education intends to take full
and disadvantaged groups, as also girls, from regularly advantage of demographic dividend.
attending and complementing elementary education. The Which of the above assumptions is/are valid?
focus must be on the poorest and most vulnerable since (a) 1 only (b) 2 and 3 only
these groups are the most disempowered and at the (c) 3 only (d) 1, 2 and 3
greatest risk of violation or denial of their right to Ans. (a) Statement (1) is true in the context of the above
education. passage, hence option (a) is correct, right to education
The right to education goes beyond free and compulsory guarantees the responsibility of the teacher for the
education to include quality education for all. Quality is learning process of the children, that is, the teacher's
an integral part of the right to education. If the education responsibility systems and procedures must ensure that
process lacks quality, children are being denied their children are receiving education and their right to
right. The Right of Children to Free and Compulsory receive education a learner friendly environment is not
Education Act lays down that the curriculum should being violated, therefore, under the right of children to
provide for learning through activities, exploration and free and compulsory education act, there should be a
discovery. This places an obligation and discovery. This provision for education through curricular, investigation
places an obligation on us to change our perception of and discovery.
IAS (Pre) GS IInd Paper (CSAT), 2018 117 YCT
7. According to the passage, which one of the monoculture of rice/wheat across the Indian plains. A
following is critical in bringing quality in major consequence of deforestation is that it relates to
education? adverse alterations in the hydrology and related soil and
(a) Ensuring regular attendance of children as nutrient losses. The consequences of deforestation
well as teachers in school invariably arise out of site degradation through erosive
(b) Giving pecuniary benefits to teachers to losses. Tropical Asia, Africa and South America have the
motivate them highest levels of erosion. The already high rates for the
(c) Understanding the socio-cultural background tropics are increasing at an alarming rate (e.g., through
of children the major river systems–Ganga and Brahmaputra, in the
(d) Inculcating learning through activities and Indian context), due to deforestation and ill-suited land
discovery management practices subsequent to forest clearing. In
the mountain context, the declining moisture retention of
Ans. (d) According to the passage, inculcating learning the mountain soils, drying up of the underground springs
through activity and discovery is crucial in bringing
and smaller rivers in the Himalayan region region could
quality to education. In order to bring quality education,
be attributed to drastic changes in the forest cover. An
the right of children to free and compulsory education indirect consequence is drastic alteration in the upland-
Act lays down that the curriculum should provide for lowland interaction, mediated through water. The current
learning through activity, exploration and discovery.
concern the tea planter of Assam has is about the damage
Hence option (d) is correct. to tea plantations due to frequent inundation along the
8. What is the essential massage in this passage? flood-plains of Brahmaputra, and the damage to tea
(a) The Right to Education now is a Fundamental plantation and the consequent loss in tea productivity is
Right. due to rising level of the river bottom because of siltation
(b) The Right to Education enables the children and the changing course of site desertification are soil
of poor and weaker sections of the society to degradation, alteration in available water and its quality,
attend schools. and the consequent decline in food, fodder and fuel-
(c) The Right to Free and Compulsory Education wood yields essential for the economic well-being of
should include quality education for all. rural communities.
(d) The Government as well as parents should 9. According to the passage, which of the
ensure that all children attend schools. following are the consequences of decline in
Ans. (c) The essential message of the passage is that the forest cover?
right to free and compulsory education should include 1. Loss of topsoil
quality education for all because quality education is the 2. Loss of smaller rivers
last part of the right. If the education process lacks 3. Adverse effect on agricultural production
quality, then children are being deprived of their right 4. Declining of groundwater
and the and the compulsory education Act mandates Select the correct answer using the code given
that the attitude of viewing children as passive below.
recipients of learning should be changed and going
(a) 1, 2 and 3 only (b) 2, 3 and 4 only
beyond the practice of using textbooks as the basis for
examinations, hence the inclusion of quality education (c) 1 and 4 only (d) 1, 2, 3 and 4
is absolutely essential. Ans. (d) All the given options are the consequences of
Direction for the following 7 (seven) items : decline is forest cover; these can be inferred from the
Read the following four passages and answer the two following lines of the passage- "leading to total loss of
items that follow. Your answers to these items should productivity", "accelerated erosion as in the mountain
be based on the passages only. regions of the country", "adverse alternations in the
hydrology and related soil and nutrient losses" and
Passage–1
"drying up of the underground springs and smaller
'Desertification' is a term used to explain a process of rivers in the Himalayan region………"
decline in the biological productivity of an ecosystem,
leading to total loss of productivity. While this 10. Which of the following is/are the correct
phenomenon is often linked to the arid, semi-arid and inference/inferences that can be made from the
sub-humid ecosystems, even in the humid tropics, the passage?
impact could be most dramatic. Impoverishment of 1. Deforestation can cause changes in the
human-impacted terrestrial ecosystems may exhibit itself course of rivers.
in a variety of ways : accelerated erosion as in the 2. Salinization of land takes place due to
mountain regions of the country, salinization of land as human activities only.
in the semi-arid and arid 'green revolution' western Uttar 3. Intense monoculture practice in plains is a
Pradesh, and site quality decline–a common phenomenon major reason for desertification in
due to general decline in tree cover and monotonous Tropical Asia, Africa and South America.
IAS (Pre) GS IInd Paper (CSAT), 2018 118 YCT
Select the correct answer using the code given Select the correct answer using the code given
below. below.
(a) 1 only (a) 1, 2 and 3 only (b) 1, 2 and 4 only
(b) 1 and 2 only
(c) 3 and 4 only (d) 1, 2, 3 and 4
(c) 2 and 3 only
(d) None of the above is a correct inference Ans. (b) Statement (1), (2) and (4) are correct in the
Ans. (a) According to the passage, the correct context of the above passage in that conservation of
conclusion is that due to deforestation, the course of natural water resources, conservation of a wider gene
rivers can be changed. The drying up of inland springs pool and migration corridors would be helpful in
and relatively small rivers in the Himalayan region can combating climate change as well as linkages between
be attributed to drastic charges in forest cover. An natural areas such as migration corridors. The need will
indirect consequence is drastic charge in the highland be to facilitate the mobility of species so as to cope with
lowland interactions through water, which is the reason climate change.
for the diversion of rivers.
13. With reference to the above passage, the
11. With reference to 'desertification', as described
in the passage, the following assumptions have following assumptions have been made :
been made : 1. Diversification of livelihoods acts as a
1. Desertification is a phenomenon in tropical coping strategy for climate change.
areas only. 2. Adoption of monocropping practice leads
2. Deforestation invariably leads to floods to the extinction of plant varieties and their
and desertification. wild relatives.
Which of the above assumptions is/are valid? Which of the above assumptions is/are valid?
(a) 1 only (b) 2 only
(c) Both 1 and 2 (d) Neither 1 nor 2 (a) 1 only (b) 2 only
Ans. (b) Assumption (1) is invalid with reference to the (c) Both 1 and 2 (d) Neither 1 nor 2
line, 'While this phenomenon…….dramatic "which Ans. (c) The assumption which can be made as per the
indicates that the phenomenon is not limited to the passage are diversification of livelihood helps to
tropical areas. combat climate change strategy and adoption of
Assumption (2) is valid as the passage mentions monocropping practices leads to soil erosion and reduce
"desertification and inundation" as the effects of micro organism habitats. Thus both the assumption are
deforestation. valid.
Passage – 2
A diversity of natural assets will be needed to cope with Passage – 3
climate change and ensure productive agriculture, Today the top environmental challenge is a combination
forestry, and fisheries. For example, crop varieties are of people and their aspirations. If the aspirations are
needed that perform well under drought, heat, and more like the frugal ones we had after the Second World
enhanced CO2. But the private-sector and farmer-led War, a lot more is possible than if we view the planet as
process of choosing crops favours homogeneity adapted a giant shopping mall. We need to get beyond the
to past or current conditions, not varieties capable of fascination with glitter and understand that the planet
producing consistently high yields in warmer, wetter, or works as a biological system.
drier conditions. Accelerated breeding programmes are
14. Which of the following is the most crucial and
needed to conserve a wider pool of genetic resources of
existing crops, breeds, and their wild relatives. Relatively logical inference that can be made from the
intact ecosystems, such as forested catchments, above passage?
mangroves, wetlands, can buffer the impacts of climate (a) The Earth can meet only the basic needs of
change. Under a changing climate, these ecosystems are humans for food, clothing and shelter.
themselves at risk, and management approaches will (b) The only way to meet environmental
need to be more proactive and adaptive. Connections challenge is to limit human population.
between natural areas, such as migration corridors, may (c) Reducing our consumerism is very much in
be needed to facilitate species movements to keep up
our own interest.
with the change in climate.
(d) Knowledge of biological systems can only
12. With reference to the above passage, which of
the following would assist us in coping with the help us save this planet.
climate change? Ans. (c) The most conclusive and logical conclusion
1. Conservation of natural water sources from the above passage is "it is in our own interest to
2. Conservation of wider gene pool reduce consumerism, so option (c) is correct. According
3. Existing crop management practices to the author, today the consumerist is making improper
4. Migration corridors use of organic elements to fulfill his selfish interests.
IAS (Pre) GS IInd Paper (CSAT), 2018 119 YCT
Passage – 4 Passage-2
Some people believe that leadership is a quality which Cooking with biomass and coal in India is now
you have at birth or not at all. This theory is false, for the recognised to cause major health problems, with women
art of leadership can be acquired and can indeed be and children in poor populations facing the greatest risk.
taught. This discovery is made in time of war and the There are more than 10 lakh premature deaths each year
results achieved can surprise even the instructors. Faced from household air pollution due polluting cooking fuels
with the alternatives of going left or right, every soldier with another 1.5 lakh due to their contribution to general
soon grasps that a prompt decision either way is better outdoor air pollution in the country. Although the
than an endless discussion. A firm choice of direction fraction of the Indian population using clean cooking
has an even chance of being right while to do nothing fuels, such as LPG, natural gas and electricity, is slowly
will be almost certainly wrong. rising, the number using polluting solid fuels as their
15. The author of the passage holds the view that primary cooking fuel has remained static for nearly 30
(a) leadership can be taught through war years at about 70 crore.
experience only 17. Which of the following is the most crucial and
(b) leadership can be acquired as well as taught logical inference that can be made from the
(c) the results of training show that more people passage?
acquire leadership than are expected (a) Rural people are giving up the use of
(d) despite rigorous instruction, very few leaders polluting solid fuels due to their increasing
are produced awareness of health hazards.
Ans. (b) The author of the above passage is of the (b) Subsidizing the use of clean cooking fuels
opinion that leadership can be acquired as well as will solve the problem of India's indoor air
taught. In the above passage, the author has given the pollution.
example of soldiers taking quick decisions, in which (c) India should increase its import of natural gas
according to the author, if they make a firm choice of and produce more electricity.
any direction, the there can be a chance of being right in (d) Access to cooking gas can reduce premature
it even when doing nothing is almost certainly wrong. deaths in poor households.
Directions for the following 8 (eight) items :
Ans. (d) Option (d) is the most conclusive and logical
Read the following eight passages and answer the
conclusion that can be drawn from the above passage,
items that follow. Your answers to these items should
according to which premature deaths can be reduced in
be based on the passages only.
poor households by providing them access to cooking
Passage - 1 gas, because every year more than 10 lakh
All actions to address climate change ultimately involve asymptomatic cases are reported in these households.
costs. Funding is vital in order for countries like India to Deaths occur due to domestic air pollution, which is
design and implement adaptation and mitigation plans caused by polluting fuels used in cooking, and upto 1.5
and projects. The problem is more severe for developing lakh deaths occur due to outdoor air pollution in the
countries like India, which would be one of the hardest country due to such fuels.
hit by climate change, given its need to finance
development. Most countries do indeed treat climate Passage- 3
change as real threat and are striving to address it in a Scientific knowledge has its dangers, but so has every
more comprehensive and integrated manner with the great thing. Over and beyond the dangers with which if
limited resources at their disposal. threatens the present, it opens up as nothing else can, the
16. With reference to the above passage, the vision of a possible happy world; a world without
following assumptions have been made : poverty, without war, with little illness. Science,
1. Climate change is not a challenge for whatever unpleasant consequences it may have by the
developed countries. way, is in its very nature a liberator.
2. Climate change is a complex policy issue 18. Which one of the following is the most
and also a development issue for many important implication of the passage?
countries. (a) A happy world is a dream of science.
3. Ways and means of finance must be found (b) Science only can build a happy world, but it is
to enable developing countries to enhance also the only major threat.
their adaptive capacity. (c) A happy world is not possible without
Which of the above assumptions is/are valid? science.
(a) 1 and 2 only (b) 3 only (d) A happy world is not at all possible with or
(c) 2 and 3 only (d) 1, 2 and 3 without science.
Ans. (c) According to the author in the passage. climate Ans. (a) The most important implication of the above
change is a complex policy issue, also development passage is option (a), science has the potential to uplift
issue for many countries developing countries should be the state of living of mankind so that a happy world is a
enable to enhance their climate adoptive capacity. dream of science.

IAS (Pre) GS IInd Paper (CSAT), 2018 120 YCT


Passage - 4 Passage - 6
The Arctic's vast reserves of fossil fuel, fish and minerals
India's educational system is modelled on the mass
are now accessible for a longer period in a year. But education system that developed in the 19th century in
unlike Antarctica, which is protected from exploitation Europe and later spread around the world. The goal of
by the Antarctic Treaty framed during the Cold war and the system is to condition children as 'good' citizens and
is not subject to territorial claims by any country, there is
productive workers. This suited the industrial age that
no legal regime protecting the Arctic from
needed the constant supply of a compliant workforce
industrialization, especially at a time when the world
craves for more and more resources. The distinct with a narrow set of capabilities. Our educational
possibility of ice-free summer has prompted countries institutes resemble factories with bells, uniforms and
with Arctic coastline to scramble for great chunks of the batch-processing of learners, designed to get learners to
melting ocean. conform. But, from an economic point of view, the
19. Which one of the following is the most environment today is very different. It is a complex,
important implication of the passage? volatile and globally interconnected world.
(a) India can have territorial claims in the Arctic
21. With reference to the above passage, the
territory and free acess to its resources. following assumptions have been made :
(b) Melting of summer ice in the Arctic leads to 1. India continues to be a developing country
changes in the geopolitics. essentially due to its faulty education
(c) The Arctic region will solve the world's future system.
problem of resource crunch. 2. Today's learners need to acquire new-age
(d) The Arctic region has more resources than
skill-sets.
Antarctica.
3. A good number of Indians go to some
Ans. (b) The most important implication of the above
developed countries for education because
passage is option (b) according to which the melting of
the educational systems there are a perfect
summer ice in the Arctic causes changes in geopolitics.
In the passage the author explains that there is no legal reflection of the societies in which they
system to protect the Arctic from industrialization, function.
especially at a time when the world is clamoring for Which of the above assumptions is/are valid?
more and more resources. In the passage, the author (a) 1 and 3 only (b) 2 only
makes it clear that the apparent prospect of an ice-free (c) 2 and 3 only (d) 1, 2 and 3
summer has prompted countries along the Arctic coast Ans. (b) In the passage, the need of the current world is
to scavenge for standing stretches of melting ocean. people with entrepreneurial mindset new age skills and
Passage-5 innovative spirit. Hence the assumption in the passage
Being a member of the WTO, India is bound by the that today's learners need to acquire new age skill-sets.
agreements that have been signed and ratified by its
members, including itself. According to Article 6 of the
Passage-7
Agriculture Agreement, providing minimum support The practice of dieting has become an epidemic;
prices for agricultural products is considered distorting everyone is looking out for a way to attain that perfect
and is subject to limits. The subsidy arising from body. We are all different with respect to our ethnicity,
minimal supports cannot exceed 10 percent of the value genetics, family history, gender, age, physical and mental
of agricultural production for developing countries. PDS and spiritual health status, lifestyles and preferences.
in India entails minimum support prices and public Thereby we also differ in what foods we tolerate or are
stockholding of food grains. It is possible that, in some sensitive to. So we really cannot reduce so many
years, the subsidy to producers will exceed 10 percent of complexities into one diet or diet book. This explains the
the value of agricultural production.
failure of diets across the world in curbing obesity.
20. What is the crucial message conveyed by the Unless the reasons for weight gain are well understood
above passage?
(a) India should revise its PDS. and addressed and unless habits are changed
(b) India should not be a member of WTO. permanently, no diet is likely to succeed.
(c) For India, food security collides with trade. 22. What is the most, logical and rational inference
(d) India provides food security to its poor. that can be made from the above passage?
Ans. (c) In the passage food security Programme of (a) Obesity has become an epidemic all over the
India requires the Indian government to support farmers world.
through MSP and distribute the procured food grains to (b) A lot of people are obsessed with attaining a
the public through PDS. This support price should be perfect body.
rational this support might cross the maximum limits (c) Obesity is essentially an incurable disease.
therefore the food security Programme of India collides (d) There is no perfect diet or one solution for
with WTO agreement, is correct Answer. obesity.
IAS (Pre) GS IInd Paper (CSAT), 2018 121 YCT
Ans. (d) The most logical and rational inference from 24. What is the most rational and crucial message
the above passage is option (d). The above passage given by the passage?
explains the failure of diets to control obesity (a) Mass production of meat through industrial
worldwide. Unless the cause of weight gain are well farming is cheap and is suitable for providing
understood and addressed, and unless habits are protein nutrition to poor countries.
permanently changed, no diet is likely to be successful. (b) Meat - producing industry violates the laws
Therefore, there is no complete diet or a solution for against cruelty to animals.
obesity. (c) Mass production of meat through industrial
farming is undesirable and should be stopped
Passage-8 immediately.
Monoculture carries great risks. A single disease of pest (d) Environmental cost of meat production is
can wipe out swathes of the world's food production, an unsustainable when it is produced through
alarming prospect given that its growing and wealthier industrial farming.
population will eat 70% more by 2050. The risks are
Ans. (d) The most rotational and crucial suspicion
magnified by the changing climate. As the planet warms
raised by the above passage in that the environmental
and monsoon rains intensify, farmlands in Asia will
cost of meat production is unsustainable when it is
flood. North America will suffer more intense droughts,
produced from industrial animal husbandry. As
and crop diseases will spread to new latitudes. livestock rearing around the world continues to be in
23. Which of the following is the most logical, increasingly cruel and cramped conditions, animals
rational and crucial message given by the spend their short lives amidst the use of artificial light,
(a) Preserving crop genetic diversity is an antibiotics and growth harmones, until the day of
insurance against the effects of climate slaughter. Meat production leads to excessive
change consumption of water. 15000 liters of water is required
(b) Despite great risks, monoculture is the only for every kilogram of meat. Whereas, 3400 liters for 1
way to ensure food security in the world. kg of paddy which shows the environmental cost of
(c) More and more genetically modified crops meat production.
only can save the world from impending
Passage-2
shortages of food.
A male tiger was removed from Pench Tiger Reserve and
(d) Asia and North America will be worst
was relocated in Panna National Park. Later, this tiger
sufferers from climate change and the
trekked toward his home 250 miles away. The trek of
consequent shortage of food.
this solitary tiger highlights a crisis. Many wildlife
Ans. (a) The most logical, rational and crucial message reserves exist as islands of fragile habitat in a vast sea of
of the passage in that preserving crop genetic diversity humanity, yet tigers can range over a hundred miles,
is insurance against the effects of climate change seeking prey, mates and territory. Nearly one third of
because of the great risks involved in doing a crop India's tigers live outside tiger reserves, a situation that is
farming. A single disease or pest can completely dangerous for both human and animal. Prey and tigers
destroy the lines of food production around the world. can only disperse if there are recognized corridors of
Directions for the following 3 (three) items : land between protected areas to allow unmolested
Read the following two passages and answer the items passage.
that follow. Your answers to these items should be 25. Which of the following is the most rational and
based on the passages only. crucial message given by the passage?
Passage-1 (a) The conflict between man and wildlife cannot
The quest for cheap and plentiful meat has resulted in be resolved, no matter what efforts we make.
factory farms where more and more animals are (b) Safe wildlife corridors between protected
squeezed into smaller lots in cruel and shocking areas is an essential aspect of conservation
conditions. Such practices have resulted in many of the efforts.
world's health pandemics such as the avian flu. (c) India needs to declare more protected areas
Worldwide, livestock are increasingly raised in cruel, and set up more tiger reserves.
cramped conditions, where animals spend their short live (d) India's National Parks and Tiger Reserves
under artificial light, pumped full of antibiotics and need to be professionally managed.
growth hormones, until the day they are slaughtered. Ans. (b) The most rational and crucial message
Meat production is water-intensive. 15000 litres of water conveyed by the above passage is that protected wildlife
is needed for every kilogram of meat compared with conservation and said that there should be valid
3400 litres for rice, 3300 litres for eggs and 255 litres for corridors of land between the sanctuaries so that they
a kilogram of potatoes. can come without tampering.

IAS (Pre) GS IInd Paper (CSAT), 2018 122 YCT


26. With reference to the above passage, the 29. Consider the following graph
following assumptions have been made :
1. The strategy of conservation of wildlife by
relocating them from one protected area to
another is not often successful.
2. India does not have suitable legislation to
save the tigers, and its conservation efforts
have failed which forced the tigers to live
outside protected areas.
Which of the above assumptions is/are valid?
(a) 1 only (b) 2 only
(c) Both 1 and 2 (d) Neither 1 nor 2 Which one of the following statements is not
Ans. (a) The concept given in the passage throws light correct with reference to the graph given
regarding the sanctuary, in which the male tiger was above?
removed from the pench tiger reserve and kept in the (a) On 1st June, the actual progress of work was
Panna National Park, this tiger traveled a long distance less than expected.
of 250 miles towards its home. This journey of the tiger (b) The actual rate of progress of work was the
alone sheds light on this crisis. Therefore, from this greatest during the month of August.
statement, the strategy of conserving wildlife, which has (c) The work was actually completed before the
changed from one protected areas to another protected expected time.
areas, is often not successful, so option (a) is correct. (d) During the period from 1st April to 1st
September, at no time was the actual progress
GENERAL MENTAL ABILITY more than the expected progress.
Ans. (d) Among the given options, option (d) is not
Direction for the following 2 (two) items :
correct. In the period from 1st April to 1st September, the
Read the following information and answer the two
actual progress at about 15th august was more than the
items that follow.
expected progress.
The plan of an office block for six officers A, B, C, D, E
and F is as follows : Both B and C occupy offices to the 30. Consider the following graphs. The curve in
right of the corridor (as one enters the office block) and the graphs indicate different age groups in the
A occupies on the left of the corridor. E and F occupy populations of two countries A and B over a
offices on opposite sides of the corridor but their offices period of few decades :
do not face each other. The offices of C and D face each Country A
other. E does not have a corner office. F's office is
further down the corridor than A's, but on the same side.
27. If E sits in his office and faces the corridor,
whose office is to his left?
(a) A (b) B
(c) C (d) D
Ans. (c)

Country B

From the figure,


If E sits in his office and faces the corridor, to his left
will be c's office.
28. Who is/are F's immediate neighbor
(a) A only (b) A and D
(c) C only (d) B and C
Ans. (a) From the figure,
F's immediate neighbor is only A.

IAS (Pre) GS IInd Paper (CSAT), 2018 123 YCT


With reference to the above graphs, which of Ans. (a)
the following are the most logical and rational
inferences that can be made?
1. Over the last two and a half decades, the
dependency ratio for country B has
decreased.
2. By the end of next two and a half decades,
the dependency ratio of country A will be
much less than that country B.
3. In the next two decades the work. force
relative to its total population will increase
in country B as compared to country A.
Select the correct answer using the code given
below. Lakshmi's brother is the best player.
(a) 1 and 2 only (b) 2 and 3 only 32. The graph given below indicates the changes in
(c) 1 and 3 only (d) 1, 2 and 3 key policy rates made by the Central Bank
Ans. (c) From the given graphs Age groups of < 15 and several times in a year :
more than 64 + = Dependent population and age group Which one of the following can be the most
of 15 to 64 years = work force population. likely reason for the Central Bank for such an
Dependency ratio% of population action?
< 15yr + % of population > ( 64 years )
= ×100
% of working age population (15 − 64 )
If we look at statement (2), i.e., by 2050 dependency
ratio of country A < country B.
Country A dependency ratio is 2050.
<15 yr population A = 0.2 million
64 yr population A = 0.3 millions
15 to 64 yrs population A = 0.9 millions
0.2to3 5
∴ Dependency ratio A = = ×100
0.9 9 (a) Encouraging foreign investment
= 55.5% (b) Increasing the liquidity
Country B dependency ratio in 2050. (c) Encouraging both public and private savings
< 15 yr population B = 0.2 (d) Anti - inflationary stance
< + 64 yr population B = 0.2 Ans. (d) The picture shows that when the repo rate is
15 to 64 yr population B = 11 rising, the central bank is trying to correct inflation by
reducing liquidity. Hence, option (d) is correct.
∴ Dependency ratio
Directions for the following 2 (two) items :
0.2 + 0.2
B= ×100 = 36.36% The following table gives the GDP growth rate and
1.1 Tele-density data of different States of a country in a
∴ Option (c) is the correct answer. particular year. Study the table and answer the two
31. Lakshmi, her brother, her daughter and her items follow.
son are badminton players. A game of doubles States Per GDP growth Tele-
is about to begins : capita rate (%) density
(i) Lakshmi's brother is directly across the net income
from her daughter. State 1 704 9.52 70.27
(ii) Her son is diagonally across the net from State 2 419 5.31 35.88
the worst player's sibling. State 3 254 10.83 50.07
(iii) The best player and the worst player are State 4 545 9.78 5.94
on the same side of the net. State 5 891 10.8 76.12
Who is the best player? State 6 1077 11.69 77.5
(a) Her brother (b) Her daughter State 7 900 8.88 104.86
(c) Her son (d) Lakshmi State 8 395 5.92 6
IAS (Pre) GS IInd Paper (CSAT), 2018 124 YCT
State 9 720 7.76 82.25
State 10 893 9.55 96.7
State 11 363 4.7 57.7
State 12 966 7.85 63.8
State 13 495 9.37 52.3
State 14 864 5.46 97.9
State 15 497 7.48 62.3
State 16 777 7.03 93.8
State 17 335 5.8 49.9
State 18 599 7.49 47.84
33. With reference to the above table, which of the
following is are the most logical and rational With reference to the above graph, which of
inference/inferences that can be made? the following is/are the most logical and
1. Higher per capita income is generally rational inference/inferences that can be made?
associated with higher Tele-density. 1. During the given period, the revenue from
2. Higher GDP growth rate always ensures Direct Taxes as percentage of gross tax
higher per capita income. revenue has increased while that of
3. Higher GDP growth rate does not Indirect Taxes decreased.
necessarily ensure higher Tele-density. 2. The trend in the revenue from excise Duty
Select the correct answer using the code given demonstrates that the growth of
below : manufacturing sector has been negative
(a) 1 only during the given period.
(b) 2 and 3 Select the correct answer using the code given
(c) 1 and 3 below.
(d) 3 only (a) 1 only (b) 2 only
Ans. (c) Statement 2 is wrong because of seeing state (c) Both 1 and 2 (d) Neither 1 nor 2
no 15 & 16 this is not clear that higher GDP growth rate Ans. (a) It is clear from the above picture that revenue
always ensures higher per capita income. from direct taxes as a percentage of overall tax revenue
According to statement -1 higher per capita income is has increased while revenue from indirect taxes has
generally associated with higher tele-density so that decreased in the given time period.
statement -1 is correct.
According to statement -3 higher GDP growth rate dose INTERPERSONAL
not necessarily ensure higher tele-density so statement - SKILLS/COMMUNICATION
3 is correct .
So option - (c) is correct. 36. Consider the following three-dimensional figure :
34. With reference to the above table, the following
assumptions have been made:
1. Nowadays, prosperity of an already high
performing state cannot be sustained
without making further large investments
in its telecom infrastructure.
2. Nowadays, a very high Tele-density is the
most essential condition for promoting the
business and economic growth in a State.
Which of the above assumptions is/are valid? How many triangles does the above figure
(a) 1 only have?
(b) 2 only (a) 18 (b) 20
(c) Both 1 and 2 (c) 22 (d) 24
(d) Neither 1 nor 2 Ans. (b)
Ans. (d) Both presumption, statement -1 and statement-
2 are not conclusion of above table. So that neither
statement - 1 nor statement- 2 is correct. option (d) is
correct answer. 4
35. The following graph indicates the composition
of our tax revenue for a period of two decades :
IAS (Pre) GS IInd Paper (CSAT), 2018 125 YCT
∵ Given figure is three-dimensional figure Similarly,

∴ The more triangles are on front, the more triangles


will be on the back.
Hence, The total number of triangles ⇒ 10 + 10⇒ 20
Directions for the following 3 (three) items :
Rotated positions of a single solid are shown below.
The various faces of the solid are marked with
Hence,
different symbols like dots. cross and line. Answer the
three items that follow the given figures. CCIDD
41. If RAMON is written as 12345 and DINESH as
675849, then HAMAM will be written as
(a) 92233 (b) 92323
(c) 93322 (d) 93232
Ans. (b)
37. What is the symbol on the face opposite to that Just as,
containing a single dot? R = 1, A = 2, M = 3, O = 4, N = 5, D = 6
(a) Four dots I = 7, N = 5, E = 8, S = 4, H = 9
(b) Three dots Similarly,
(c) Two dots HAMAM = 92323
(d) Cross 42. If Pen<Pencil, Pencil<Book and Book>Cap,
Ans. (b) According to figure No (i), (ii), (iii) & (iv) then which one of the following is always true?
(a) Pen>Cap (b) Pen<Book
(c) Pencil = Cap (d) Pencil>Cap
Ans. (b) According to question,
Pen < Pencil < Book > Cap
∴ Pen < Book
43. 19 boys turn out for playing hockey. Of these,
Option (b) is correct. 11 are wearing hockey shirts and 14 are
38. What is the symbol on the face opposite to that wearing hockey pants. There are no boys
containing two dots? without shirts and/or pants. What is the
(a) Single dot number of boys wearing full uniform?
(b) Three dots (a) 3 (b) 5
(c) Four dots (c) 6 (d) 8
(d) Line Ans. (c) The number of boys wearing full uniform
Ans. (c) ⇒ The number of boys wearing only pants + The
39. What is the symbol on the face opposite to that number of boys wearing only shirt – The total of boys
containing the cross? = (14 + 11) – 19 = 25 – 19 = 6
(a) Single dot (b) Two dots Direction for the following 6 (six) items :
(c) Line (d) Four dots Read the information given below and answer the six
Ans. (c) items that follow.
40. If LSJXVC is the code for MUMBAI, the code A, B, C and D are students. They are studying in four
for DELHI is different cities, viz., P,Q,R and S (not necessarily in
(a) CCIDD (b) CDKGH that order). They are studying in Science college, Arts
(c) CCJFG (d) CCIFE college, Commerce college and Engineering college
Ans. (a) (not necessarily in that order), Which are situated in
Just as, four different States, viz, Gujarat, Rajasthan, Assam
and Kerala (not necessarily in that order). Further, it
is given that-
(i) D is studying in Assam
(ii) Arts college is located in city S which is in
Rajasthan
(iii) A is studying in Commerce college
(iv) B is studying in city Q
(v) Science college is located in Kerala

IAS (Pre) GS IInd Paper (CSAT), 2018 126 YCT


44. A is studying in There are six different colours available to
(a) Rajasthan (b) Gujarat choose from and each of the three wooden
(c) City Q (d) Kerala blocks is to be painted such that no two of them
Ans. (b) has the same colour. In how many different
ways can the winner's stand be painted?
Student City Subject State (a) 120 (b) 81
A P/R Commerce Gujarat (c) 66 (d) 36
B Q Science Kerala Ans. (a)
C S Arts Rajasthan ∵ Number of colours = 6
D P/R Engineering Assam and number of wooden block = 3
A is studing in Gujarat 6!
Required ways =
45. Science college is located in 3!
(a) City Q (b) City S 6 × 5 × 4 × 3× 2 ×!
= = 120
(c) City R (d) City P 3!
Ans. (a) Science college is located in city Q. Directions for the following 2 (two) items :
46. C is studying in Consider the following graph in which the birth rate
(a) Science college (b) Rajasthan and death rate of a country are given, and answer the
(c) Gujarat (d) city Q two items that follow.
Ans. (b) C is studying in Rajasthan.
47. Which one of the following statements is
correct?
(a) D is not studying in city S.
(b) A is studying in Science college
(c) A is studying in Kerala
(d) Engineering college is located in Gujarat.
Ans. (a) The correct statements:- D is not studying in
city S.
48. Which one of the following statements is
correct regarding Engineering college? 51. Looking at the graph, it can be inferred that
from 1990 to 2010
(a) C is studying there
(a) population growth rate has increased
(b) B is studying there
(b) population growth rate has decreased
(c) It is located in Gujarat
(c) growth rate of population has remained stable
(d) D is studying there
(d) population growth rate shows no trend
Ans. (d) The correct statement:- D is studying in
Ans. (d)
engineering college.
Year Birth rate Death Rate Population
49. Which one of the following statements is growth rate
correct? 1990 22.5 13 9.5
(a) Engineering college is located in Assam. 2000 18 7 11
(b) City Q is situated in Assam. 2010 17.5 7 10.5
(c) C is studying in Kerala According to the above table conclusion is that, growth
(d) B is studying in Gujarat. rate dose not show any trend. Option (d) is correct
Ans. (a) The correct statement, answer.
(a) Engineering college is located in Assam. 52. With reference to the above graph, consider
the following statements considering 1970 as
50. For a sports meet, a winners' stand comprising
base year :
three wooden blocks is in the following form :
1. Population has stabilized after 35 years.
2. Population growth rate has stabilized after
35 years.
3. Death rate has fallen by 10% in the first 10
years.
4. Birthrate has stabilized after 35 years.
IAS (Pre) GS IInd Paper (CSAT), 2018 127 YCT
Which of the above are the most logical and 54. Consider the figures given below :
rational statements that can be made from the
above graph
(a) 1 and 2 only (b) 1, 2 and 3
(c) 3 and 4 (d) 2 and 4
Ans. (d) Statement (2) and (4) (population growth has
become stable after 35 years and birth rate has become
stable after 35 years) according to option (d) among the
given alternatives is the most logical and rational
statement that can be made from the graph.
53. Average hourly earnings per year (E) of the
workers in a firm are represented in figures A
and B as follows :
To fit the question mark, the correct answer is

Ans. (a) On the basis of above figures, the correct


answer in the place of question mark is appropriate
option (a).
55. Consider the following figures :

In the figures (I) to (VI) above, some parts are


shown to change their positions in regular
directions. Following the same sequence, which
of the figures given below will appear at (VII)
stage?
From the figures, it is observed that the
(a) values of E are different
(b) ranges (i.e., the difference between the
maximum and the minimum) of E are
different
(c) Slopes of the graphs are same
(d) rates of increase of E are different
Ans. (c) Option a is wrong because value of E, are same
in both graphs.
Option b is wrong because range of E are same in both
graphs. Ans. (b) In the given figures,
Option c is correct because slopes of graphs are same, Triangle round the antic lock wise and square round the
Option d is wrong because the rates of increase of E are clock wise.
same in both graphs. Hence, option (b) will appear at (VII) stage.

IAS (Pre) GS IInd Paper (CSAT), 2018 128 YCT


DECISION MAKING
56. Consider the following sum:
• + 1• + 2 • + • 3 + • 1 = 21 •
In the above sum, • stands for
(a) 4 (b) 5
(c) 6 (d) 8
Ans. (d) Suppose • equal to x then
• + 1• + 2 • + • 3 + • 1 = 21 • What is the ratio between the distances covered
x + (10 + x ) + ( 20 + x ) + (10x + 3) + (10x + 1) = 210 + x by vehicles A and B in the time interval OL?
23x+34= 210 +x (a) 1 : 2 (b) 2 : 3
22x =176 (c) 3 : 4 (d) 1 : 1
x=8 Ans. (c)
57. Consider the following pattern of numbers:
8 10 15 13
6 5 7 4
4 6 8 8
6 11 16 ?
What is the number at ? in the above pattern?
(a) 17 (b) 19 ∵ Velocity
(c) 21 (d) 23 d
Ans. (a) v=
t
8 10 15 13
d = vt
6 5 7 4
The distance covered by vehicle A = Area of ∆OPL
4 6 8 8 1
dA = × OL × PL
6 11 16 ? 2
1
= × OL × ( PD + DL )
∵ 8+4–6=6 2
10 + 6 – 5 = 11 1 1 
= × OL ×  LD + DL 
15 + 8 – 7 = 16 2 2 
13 + 8 – 4 = 17
1 3
58. How many diagonals can be drawn by joining = × OL × LD
2 2
the vertices of an octagon?
(a) 20 (b) 24  1 
 ∵ PD = LD 
(c) 28 (d) 64  2 
Ans. (a) 3
n ( n − 3) d A ⇒ OL × L D
The number of diagonals 4
2
The distance covered by B = Area of figure OCDL
8 (8 − 3)
= = 4 × 5 = 20 dB = OL × LD
2 then,
59. The figure drawn below gives the velocity
3
graphs of two vehicles A and B. The straight × OL × LD
dA 4
line OKP represents the velocity of vehicle A at =
any instant, whereas the horizontal straight dB OL × LD
line CKD represents the velocity of vehicle B at dA 3
any instant. In the figure, D is the point where =
perpendicular from P meets the horizontal line dB 2
1 dA : dB = 3 : 4
CKD such that PD = LD :
2
IAS (Pre) GS IInd Paper (CSAT), 2018 129 YCT
60. A train 200 metres long is moving at the rate of ∵ Total cast = 217 × 256
40 kmph. In how many seconds will it cross a
man standing near the railway line? = 55552
(a) 12 (b) 15 ≈ 55550 (approximate)
(c) 16 (d) 18 63. What was the approximate ratio of sheet steel
and scrap steel imports in the first three
Ans. (d)
months of the year?
distance
Speed = (a) 1 : 1 (b) 1.2 : 1
time (c) 1.4 : 1 (d) 1.6 : 1
5 200 Ans. (b) Import of sheet steel in the first three months =
40 × =
18 t 40 + 37 + 36 = 113
200 200 Import of scrap steel in the first three months
=
18 t ⇒ 32 + 34 + 32
t = 18sec. ⇒ 98
113
Directions for the following 3 (three) items : Required Ratio =
98
The Following three items are based on the graph given
below which shows imports of three different types of 1.15 1.2
= = (approximate)
steel over a period of six months of a year. Study the 1 1
graph and answer the three items that follow. ⇒ 1.2 : 1
64. If X is between –3 and –1, and Y is between –1
and 1, then X2 – Y2 is in between which of the
following?
(a) –9 and 1 (b) –9 and –1
(c) 0 and 8 (d) 0 and 9
Ans. (d)
∵ Range of x = –3 to – 1
Range of y = –1 to + 1
Max value of x2 = (–3)2 = 9
Min value of x2 = (–1)2 = 1
Max value of y2 = 12 =1
Min value of y2 =(0)2 =0
then max value of (x2 –y2) = 9 – 0
min value of (x2 –y2) = 1–1 = 0
The figures in the brackets indicate the average cost
per ton over six months period. Range of ( x 2 – y 2 ) is – 0 to 9
61. By how much (measured in thousands of tons)
did the import of sheet steel exceed the import 65. X and Y are natural numbers other than 1, and
of coil steel in the first three months of the year? Y is greater than X. Which of the following
(a) 11 (b) 15 represents the largest number?
(c) 19 (d) 23 (a) XY (b) X / Y
(c) Y / X (d) (X + Y) / XY
Ans. (c) Inport of sheet steel in the first three months of
the year = 40 + 37 + 36 = 113 Ans. (a) ∵ x and y are natural numbers other than 1 and
Import of coil sheet in the first three months of the year y is greater than x
= 30 + 31 + 33 = 94
∵ x = 2 and y = 3
Repuired difference = 113 – 94
⇒ 19 by option,
62. What was the approximate total value (in $) of (a) xy = 2 × 3 ⇒ 6
x 2
sheet steel imported over the six months period? (b) =
(a) 45,555 (b) 50,555 y 3
(c) 55,550 (d) 65,750 y 3
(c) =
Ans. (c) x 2
Import of steel sheet ones the six momths period = x + y 3+ 2 5
(d) = =
40 + 37 + 36 + 36 + 34 + 34 = 217 tons xy 2×3 6
∵ average cast of steel sheet = 256 $/ton Hence option (a) "x y" represents the largest number.

IAS (Pre) GS IInd Paper (CSAT), 2018 130 YCT


66. A number consists of three digits of which the Ans. (b)
middle one is zero and their sum is 4. If the Total sell = ax + (a + 2) (x + 2)
number formed by interchanging the first and + (a – 2) (x –2)
last digits is greater than the number itself by = ax + ax + 2x + 2a + y
198, then the difference between the first and + ax – 2x – 2a + y
last digits is ⇒ 3ax + 8
(a) 1 (b) 2
70. A bag contains 15 red balls and 20 black balls.
(c) 3 (d) 4 Each ball is numbered either 1 or 2 or 3. 20%
Ans. (b) Let unit digit of the number = x of the red balls are numbered 1 and 40% of
and hundred digit of the number = y them are numbered 3. Similarly, among the
according to question, black balls, 45% are numbered 2 and 30% are
∵ the number = 100y + x numbered 3. A boy picks a ball at random. He
wins if the ball is red and numbered 3 or if it is
and number after interchanging =100x + y black and numbered 1 or 2. What are the
then (100x + y) – (100y + x) =198 chances of his winning?
100x + y – 100y –x = 198 1 4
99x – 99y = 198 (a) (b)
2 7
x−y=2 5 12
(c) (d)
67. A solid cube of 3 cm side, painted on all its 9 13
faces, is cut up into small cubes of 1 cm side. Ans. (b)
How many of the small cubes will have exactly ∵ The number of red balls = 15
two painted faces?
(a) 12 (b) 8 The number of red balls which are numbered 1
(c) 6 (d) 4 20
⇒ 15 × ⇒3
Ans. (a) The number of two painted 100
Face = 12 (x – 2) The number of red balls which numbered 3
= 12 (3 – 2) 40
= 15 × =6
= 12 × 1 100
= 12 The number of red balls which are numbered 2
68. While writing all the numbers from 700 to =15 – (6 + 3) = 6
1000, how many numbers occur in which the ∵ The number of black balls = 20
digit at hundred's place is greater than the 45
digit at ten's place, and the digit at ten's place The number of which are numbered to = 20× =9
100
is greater than the digit at unit's place? The number of black balls which are numbered 3
(a) 61 (b) 64 30
(c) 85 (d) 91 = 20 × =6
100
Ans. (c) the number of black balls which are numbered 1 = 20 –
For required numbers, (9 + 6) = 5
From 700 to 800, The probability of winning
1 + 2 + 3 + 4 + 5 + 6 = 21 ⇒ The number of Red balls which are numbered 3
From 800 to 900, + the number of black balls which are
1 + 2 + 3 + 4 + 5 + 6 + 7 = 28 + numbered 1 + the number of black ball which are
From 900 to 1000, numbered 2
1 + 2 + 3 + 4 + 5 + 6 + 7 + 8 = 36 ⇒ total number of black and Red balls
∵ Total numbers = 21 + 28 + 36 = 85 6+5+9
=
69. A bookseller sold 'a' number of Geography 15 + 20
textbooks at the rate of ` x per book, a+2 = 20 4
=
number of History textbooks at the rate of ` 35 7
(x+2) per book and 'a–2' number of 71. A student has to get 40% marks to pass in an
Mathematics textbooks at the rate of ` (x-2) examination. Suppose he gets 30 marks and
per book. What is his total sale in ` ? fails by 30 marks, then what are the maximum
(a) 3x + 3a (b) 3ax + 8 marks in the examination?
(a) 100 (b) 120
(c) 9ax (d) x 3a 3 (c) 150 (d) 300
IAS (Pre) GS IInd Paper (CSAT), 2018 131 YCT
Ans. (c) According to question,
Let maximum number = m P=L
SP – CP = CP – SP
according to questions,
40 – x = x – 20
40 2x = 60
m× = 30 + 30
100 x = 30`
40
m× = 60 74. There are 24 equally spaced points lying on the
100 circumference of a circle. What is the
2 maximum number of equilateral triangles that
m× = 60 can be drawn by taking sets of three points as
5
the vertices?
m = 150 (a) 4 (b) 6
72. Two persons, A and B are running on a (c) 8 (d) 12
circular track. At the start, B is ahead of A and Ans. (c)
their positions make an angle of 300 at the 24
The maximum number of triangles = =8
centre of the circle. When A reaches the point 3
diametrically opposite to his starting point, he 75. Consider the sequence given below :
meets B. What is the ratio of speeds of A and B, 4/12/95, 1/1/96, 29/1/96, 26/2/96,.......
if they are running with uniform speeds? What is the next term of the series?
(a) 24/3/96 (b) 25/3/96
(a) 6 : 5 (b) 4 : 3
(c) 26/3/96 (d) 27/3/96
(c) 6 : 1 (d) 4 : 2
Ans. (b)
Ans. (a) +28days +28days
4 /12 / 95  → 01/ 01/1996  →
+28days
29 / 01/ 96  → 26 / 02 / 96
+28days
 → 25 / 03 / 96
76. Twelve equal squares are placed to fit in a
rectangle of diagonal 5 cm. There are three
rows containing four squares each. No gaps are
left between adjacent squares. What is the area
of each square?
d 5 7
∵ V= (a) sq cm (b) sq cm
t 7 5
180 25
dA= × 2πr (c) 1 sq cm (d) sq cm
360 12

d B=
(180 − 30 ) × 2πr Ans. (c)
360
d d
∵ VA : VB = A : B
tA tB
180
× 2πr
(180 − 30 ) × 2πr
VA : VB = 360 : 360
t t
VA : VB :180 : 150
VA : VB = 6 : 5 [∵taken time is equal]

73. A shopkeeper sells an articale at ` 40 and gets


x% profit. However, when he sells it at ` 20, he
faces same percentage of loss. What is the
original cost of the article?
(a) `10 (b) `20
(c) `30 (d) `40
Ans. (c) ∵ P =SP – CP The area of each square=
and L= CP – SP =1 × 1= 1 cm2

IAS (Pre) GS IInd Paper (CSAT), 2018 132 YCT


77. Consider the following figures A and B : 30
Remaining adults = 18–12= 6 adults⇒ 6 × children
18
 30 
∴1adult = children  = 10 children
 18 
79. A person bought a refrigerator worth Rs.
22,800 with 12.5% interest compounded yearly.
At the end of first year he paid Rs. 8,650 and at
the end of second year Rs. 9,125. How much
will he have to pay at the end of third year to
clear the debt?
(a) Rs. 9,990 (b) Rs.10,000
(c) Rs.10, 590 (d) Rs.11, 250
Ans. (d) After 1 year amount
 12.5 ×1   125 
= 22800 1 +  = 22800 1 + 
 100   1000 
 5 
= 22800 1 + 
 40 
9
= 22800   = 2850 × 9 = 25650
8
∵ At the end of first year he paid = 8560`
Remaining Amount = 25650 – 8650
= 17000`
 12.5 
After two years, amount = ⇒ 17000 1 + 
The manufacturing cost and projected sales for  100 
a product are shown in the above figures A and
9
B respectively. What is the minimum number = 17000   = 2125 × 9 = 19125
of pieces that should be manufactured to avoid 8
a loss? ∵ At end of second year he paid = 9125`
(a) 2000 (b) 2500 ∴ Now remaining amount
(c) 3000 (d) 3500
⇒19125 – 9125 ⇒ 10,000
Ans. (a) According to figure A and B, ∴ After year, given amount
1000 CP = 600000 Rs
12.5 × 1 
1000 SP = 400 × 1000 · 10000 1 + 9
 = 10000  
= 400000 Rs.  100  8
∵ 1000 L = 60, 0000 – 400000 = 1250 × 9 ⇒ 11250
80. If x-y = 8, Then which of the following must be
= 20,0000
true?
∵ 2000 CP = 70,0000 1. Both x and y must be positive for any value of
SP = 350 ×2000 = 700000 x and y.
The cost price is equal to the sell price on 2000 pieces 2. If x is positive, y must be negative for any
Hence, minimum 2000 pieces should be manufactured value of x and y.
to avoid a loss 3. If x is negative, y must be positive for any
78. A lift has the capacity of 18 adults or 30 value of x and y.
children. How many children can board the lift Select the correct answer using the code given
with 12 Adults? below.
(a) 6 (b) 10 (a) 1 only (b) 2 only
(c) 12 (d) 15 (c) Both 1 and 2 (d) Neither 1 nor 2 nor 3
Ans. (b) ∵ 18 adults = 30 children Ans. (d) ∵ Given x – y = 8
30 Consider the statements,
∴ 1 adult = children
18 None of them are true.

IAS (Pre) GS IInd Paper (CSAT), 2018 133 YCT


UNION PUBLIC SERVICE COMMISSION
Civil Services (Preliminary Exam) - 2019
CSAT : PAPER-II
(Chapterwise Analysis with Explanation)
Time : 2 hours Maximum Number : 200

COMPREHENSION 1. Removal of economic discrimination leads to


removal of social discrimination.
Directions for the following 8 (eight) items : 2. Democratic polity is the best way to repair
Read the following seven passages and answer the items historical wrongs.
that follow each passage. Your answers to these items Which of the above assumptions is/are valid?
should be based on the passages only. (a) 1only
(b) 2only
Passage- I
(c) Both 1 and 2
Political theorists no doubt have to take history of
(d) Neither 1 nor 2
injustice, for example, untouchability, seriously. The
Ans : (d) The passage clearly says that historical
concept of historical injustice takes note of a variety of
injustice spans across many categories like economic
historical wrongs that continue into the present in some deprivation, social discrimination and lack of
form or the other and tend to resist repair. Two reasons recognition. So, assumption (1) is not necessarily right.
might account for resistance to repair. One not only are Assumption (2) is not explicitly stated anywhere here,
the roots of injustice buried deep in history, injustice both are invalid.
itself constitutes economic structures of exploitation, Passage- 2
ideologies of discrimination and modes of representation. Education plays a great transformatory role in life,
Two, the category of historical injustice generally particularly so in this rapidly changing and globalizing
extends across a number of wrongs such as economic world. Universities are the custodians of the intellectual
deprivation, social discrimination and lack of capital and promoters of culture and specialized
recognition. This category is complex, not only because knowledge. Culture is an activity of thought, and
of the overlap between a number of wrongs, but because receptiveness to beauty and human feelings. A merely
well informed man is only a bore on God's earth. What
one or the other wrong generally discrimination, tends to
we should aim at is producing men who possess both
acquire partial autonomy from others. This is borne out culture and expert knowledge, Their expert knowledge
by the history of repair in India. will give them a firm ground to start from and their
1. What is the main idea that we can infer from culture will lead them as deep as philosophy and as high
the passage as art. Together it will impart meaning to human
(a) Untouchability in India has not been taken existence.
seriously by political theorists. 3. On the basis of the above passage, the following
assumptions have been made :
(b) Historical injustice is inevitable in any
1. A society without well educated people
society and is always beyond repair.
cannot be transformed into a modern society
(c) Social discrimination and deprivation have 2. Without acquiring culture, a person's
their roots in bad economies. education is not complete.
(d) It is difficult, if not impossible, to repair Which of the above assumptions is/are valid?
every manifestation of historical injustice. (a) 1 only
Ans : (d) Obtion (b) is clearly ruled out due to (b) 2 only
"……beyond repair". The passage howhere mentions (c) Both 1 and 2
that option (c) is not mentioned in this fashion. Option (d) Neither 1 nor 2
(a) sound correct, but is not our choice because the first
Ans : (b) According to above passage, the assumption
political theorists have taken it seriously. Option (d) is
(1) is not valid, because it is not mentioned in the
the best. passage. Assumption (2) is certainly valid, as it, is
2. On the basis of the above passage, the following explicitly mentioned in the passage. Hence, option (b) is
assumptions have been made: valid.
IAS (Pre) GS IInd Paper (CSAT), 2019 134 YCT
Passage - 3 Ans : (b) On the basis of the above passage, the
Soil, in which nearly all our food grows, is a living assumption (1), "economic power is the only reason for
resource that takes years to form. Yet it can vanish in the existence of inequality in a society," cannot be the
minutes. Each year 75 billion tonnes of fertile soil is lost only reason, hence the assumption (1) is not valid.
to erosion. That is alarming----and not just for food Assumption (2) is also the opposite when it is the forces
producers. Soil can trap huge quantities of carbon that create and drive inequality. Hence, Assumption (2)
dioxide in the form of organic carbon and prevent it from is also wrong. Assumption (3) is valid because
escaping into the atmosphere. economic power can be analyzed through its effects
4. On the basis of the above passage, the following rather than directly. Hence, Assumption (3) is valid.
assumptions have been made: Hence option (b) is valid.
1. Large scale soil erosion is a major reason Passage - 5
for widespread food insecurity in the Climate change may actually benefit some plants by
world. lengthening growing seasons and increasing carbon
2. Soil erosion is mainly anthropogenic. dioxide. Yet other effects of a warmer world, such as
3. Sustainable management of soils helps in more pests, droughts, and flooding, will be less benign.
combating climate change. How will the world adapt? Researchers project that by
Which of the above assumptions is/are valid ? 2050, suitable croplands for four commodities--maize,
(a) 1 and 2 only potatoes, rich and wheat--will shift, in some cases
(b) 3 only pushing farmers to plant new crops. Some farmlands
(c) 2 and 3 only may benefit from warming, but others won't . Climate
(d) 1, 2 and 3 alone does not dictate yields; political shifts, global
demand, and agricultural practices will influence how
Ans : (b) Assumption (1) is wrong as the link between
farms fare in the future .
soil erosion and food inrecurity is not established in the
passage. The word alarming" does not mean that food 6. Which one of the following is the most logical
insecurity has already arrived. Assumption (2) is and rational inference that can be made from
incorrect. It is not mentioned anywhere in the passage the above passage
that man is responsible for soil erosion. Assumption (3) (a) Farmers who modernize their methods and
in surely correct as borne out by the last line. diversify their fields will be in an
advantageous position in future
Passage - 4 (b) Climate change will adversely affect the crop
Inequality is visible, even statistically measurable in diversity
many instances, but the economic power that drives it is (c) Shifting major crops to new croplands will
invisible and not measurable. Like the force of gravity, lead to a great increase in the total area under
power is the organising principle of inequality, be it of cultivation and thus an increase in overall
income, or wealth, gender, race, religion and region. Its agricultural production.
effects are seen in a pervasive manner in all spheres, but (d) Climate change is the most important factor
the ways in which economic power pulls and tilts visible affecting the agricultural economy in the
economic variables remain invisibly obscure. future.
5. On the basis of the above passage, the following Ands :(a) From the passage option (a) is correct
assumptions have been made : inference. Farmers can be in an advantageous position
1. Economic power is the only reason for the by adopting new methods and diversifying their field.
existence of inequality in a society.
2. Inequality of different kinds, income, Passage - 6
wealth, etc. reinforces power.
3. Economic power can be analysed more A bat's wings may look like sheets of skin. But
underneath, a bat has the same five fingers as an
through its effects than by direct empirical
orangutan or the human, as well as a wrist connected to
methods.
the some cluster of wrist bones connected to the same
Which of the above assumption is /are valid ?
long bones of the arm. What can be more curious than
(a) 1 and 2 only that the hand of a man, formed for grasping, that of the
(b) 3 only mole for digging, the led of the horse, the paddle of the
(c) 1 and 3 only porpoise, and the wing of the bat, should all be
(d) 1, 2 and 3 constructed on the same pattern?
IAS (Pre) GS IInd Paper (CSAT), 2019 135 YCT
7. Which one of the following is the most logical, (a) 1 and 2 (b) 3 and 4
scientific and rational inference that can be (c) 1 and 3 (d) 2 and 4
made from the above passage ? Ans : (c) Assumption (1) is clearly valid, Assumption
(a) Different species having similar structure of (2) is too far-fetched and cannot be deduced.
hands is an example of biodiversity Assumption (3) is valid as clearly stated. Assumption
(b) Limbs being used by different species for (4) again is too far fetched as the amounts involved are
different kinds of work is an example of too high.
biodiversity Directions for the following 8 (eight) items :
(c) Man and the aforementioned animals having Read the following six passage and answer the items
that follow each passage. Your answer to these items
similar structure of limbs is an example of
should be based on the passages only.
coincidence in evolution.
Passage - 1
(d) Man and the aforementioned animals have a
Low-end IoT (Internet of Things) devices are cheap
shared evolutionary history. commodity items : addressing security would add to the
Ans (d) : Option (a) is not mentioned specifically. cost. This class of items is proliferating with new
Option (b) again talks about "biodiversity" but limbs applications; many home appliances, thermostats,
being used for different purpose is not biodiversity. security and monitoring devices and personal
Option (c) sound correct, but is not, as "Coincidence in convenience devices are part of the IoT. So are fitness
evolution" is not explicitly mentioned. Option (d) is best trackers, certain medical implants and computer-like
as "a shared evolutionary history" makes it a "very devices in automobiles. The IoT is expected to expand
curious situation". exponentially-but new security challenges are daunting.
9. Which one of the following statements is the
Passage - 7 most logical and rational inference that can be
made from the above passage ?
Around 56 million years ago, the Atlantic Ocean had not (a) Development of enabling technologies in
fully opened and animals, perhaps including our primate India can be a big boost to its manufacturing
ancestors, could walk from Asia to North America sector.
through Europe and across Greenland. Earth was warmer (b) India is not yet fully ready to adopt IoT in
that it is today, but as the Paleocene epoch gave way to view of the imminent security challenges.
Eocene, it was about to get much warmer still-- rapidly (c) Life becomes more comfortable with the
and radically. The cause was a massive geologically development of cheap low-end IoT devices.
sudden release of carbon. During this period called (d) As we go digital, we must recognise the huge
Paleocene-Eocene Thermal Maximum or PETM, the threat to Internet security from some IoT
carbon injected into the atmosphere was roughly the devices.
amount that would be injected today if human burned all Ans : (d) Option (a) is irrelevant as per the above
the Earth's reserves of coal, oil and natural gas. The passage. Option (b) is also irrelevant according to this
PETM lasted for about 1,50,000 years, until the excess passage. Because in the passage there was no mention
carbon was reabsorbed. It brought on drought, floods of the matter in the context of India. Option (c) is not
relevant in the context of the above paragraph. Option
insect plagues and a few extinctions. Life on Earth
(d) is the most logical and prudent conclusion as it is
survived--indeed, it prospered----but it was drastically warning about future dangers while interpreting the
different. subjective passage.
8. Based on the above passage, the following
assumption have been made : Passage - 2
1. Global warming has a bearing on the With the digital phenomenon restructuring most social
planet's biological evolution. sectors, it is little surprise that global trade negotiations
2. Separation of land masses causes the are now eyeing the digital area in an attempt to pre-
release of huge quantities of carbon into emptively colonise it. Big data is freely collected or
the atmosphere. mined from developing countries, and converted into
digital intelligence in developed countries. This
3. Increased warming of Earth's atmosphere
intelligence begging to control different sectors and
can change the composition of its flora and extract monopoly rents. A large foreign company
fauna. providing cab service, for instance, is not a network of
4. The present man-made global warming will cars and drivers ; it is digital intelligence about
finally lead to conditions similar to those commuting, public transport, roads, traffic, city events,
which happened 56 million years ago. personal behavioural characteristics of commuters and
Which of the assumptions given above are valid drivers and so on.
IAS (Pre) GS IInd Paper (CSAT), 2019 136 YCT
10. Which on of the following is the most logical 12. What is the most logical and rational corollary
and rational corollary to the above passage ? to the above passage?
(a) Globalization is not in the interests of India (a) Supporting small farmers is an important part
as it undermines its socio-economic structures. of any agenda regarding environmentally
(b) India should be careful to protect its digital sustainable development.
sovereignty in global trade talks. (b) Poor countries have little role to play in the
(c) India should charge monopoly rents from mitigation of global warming.
multinational companies in exchanges for
(c) Due to a large number of farmer households,
Big Data.
India will not have food security problem in
(d) The loss of Big Data from India is
the foreseeable future.
proportional to the degree/value of its foreign
trade. (d) Only small-holder farmers in India can
ensure food security.
Ans (b) : Option (b) is the most rational and prudent
approach as per the above passage as it deals with how Ans : (a) The most logical and rational corollary in the
ownership of digital data in now driving business and above passage is option (a) because the poor may have
commercial gains in the context of India, and having contributed to climate change, but they may have
Indian data in the hands of foreign firms not a good played a role in mitigating it. Option (b) is
idea. Thus, option (b) is rational corollary to the above misinterpreted. Option (c) is a larger exploration and
passage. may be wrong and (d) is not mentioned anywhere.
11. Which of the following is most definitively 13. The above passage implies that
implied by the above passage ?
1. There is a potential problem of food
(a) Big Data is the key resource in the digital
insecurity in India.
space
2. India will have to strengthen its disaster
(b) Big economies create Big Data .
management capabilities.
(c) Access to Big Data is the prerogative of
developed countries. Which of the above assumptions is/are valid?
(d) Access to and possession of Big Data is a (a) 1 only
characteristic of developed countries. (b) 2 only
Ans : (a) According to the above passage, option (a) is (c) Both 1 and 2
the most rational and prudent subject as big data is the (d) Neither 1 nor 2
main resource in a digital space. Option (b) and (d) are
Ans : (c) The above passage implies that, there is
not mentioned in the above passage. Option (c) is wrong
potential problem of food security in India and also as
as access to big data is not only a privilege of developed
countries both developing countries also have their millions of people in India depend on farming for food.
right. So, India will have to strengthen its disaster
management capabilities. Thus, both the assumptions
Passage-3 are valid.
The rural poor across the world, including India, have
contributed little to human-induced climate change, yet Passage-4
they are on the frontline in coping with its effects. A changing climate, and the eventual efforts of
Farmers can no longer rely on historical averages for governments (however reluctant) to deal with it, could
rainfall and temperature, and the more frequent and
have a big impact on investors' returns. Companies that
extreme weather events, such as droughts and floods, can
produce or use large amounts of fossil fuels will face
spell disaster. And there are new threats, such as sea
higher taxes and regulatory burdens. Some energy
level rise and the impact of melting glaciers on water
supply. How significant are small farms? As many as producers may find it impossible to exploit their known
two billion people worldwide depend on them for their reserves, and be left with ''stranded assets''-deposits of
food and livelihood. Small-holder farmers in India oil and coal that have to be left in the ground. Other
produce 41 percent of the country's food grains, and industries could be affected by the economic damage
other food items that contribute to local and national caused by more extreme weather-storms, floods, heat
food security. waves and droughts.

IAS (Pre) GS IInd Paper (CSAT), 2019 137 YCT


14. On the basis of the above passage, the following Passage- 6
assumptions have been made :
A majority of the TB infected in India are poor and lack
1. Governments and companies need to be
sufficient nutrition, suitable housing and have little
adequately prepared to face the climate
change. understanding of prevention. TB than devastates
2. Extreme weather events will reduce the families, makes the poor poorer, particularly affects
economic growth of governments and women and children, and leads to ostracisation and loss
companies in future. of employment. The truth is that even if TB does not kill
them, hunger and poverty will. Another truth is that
3. Ignoring climate change is a huge risk for
deep-seated stigma, lack of adequate support from
investors.
providers and family, couple with torturous side-effects
Which of the above assumptions is/are valid?
demotivate patients to continue treatment-with disastrous
(a) 1 and 2 only
health consequences.
(b) 3 only
16. Which one of the following is the most logical,
(c) 1 and 3 only rational and crucial message conveyed by the
(d) 1, 2 and 3 above passage ?
Ans : (c) Based on the above passage, Assumption (1) (a) TB is not a curable disease in Indian
as a warning for future is true. Assumption (2) is not circumstances.
true and Assumption (3) is definitely true. Hence, (b) Curing TB requires more than diagnosis and
option (c) is valid. medical treatment.
Passage- 5 (c) government's surveillance mechanism is
deficients; and poor people have no access to
Access to schooling for those coming of school age is treatment.
close to universal, but access to quality exhibits a sharp (d) India will be free from diseases like TB only
gradient with socio-economic status. Quotas for the when its poverty alleviation programmes are
weaker sections in private schools is a provision
effectively and successfully implemented.
introduced by the Right of Children to Free and
Compulsory Education Act, 2009 The quotas have Ans : (b) Option (a) is wrong. It is nowhere mentioned
imposed a debate on issues of social integration and thus, with an air of fatality. Option (c) is wrong as poor
equity in education that private actors had escaped by do have access to treatment through it seems to be not
and large. The idea of egalitarian education system with working out well. The phrase "……..have no access" is
equality of opportunity as its primary goal appears to be wrong. Option (d) goes tangentially into a different
outside the space that private school principals inhabit. direction. So, correct answer is option (b).
Therefore, the imposition of the quotas has led to
resistance, sometimes justified. Directions for the following 7 (seven) items :
15. With reference to the above passage, the Read the following six passages and answer the items
following assumptions have been made : that follow each passage. Your answers to these items
1. Making equality of opportunity a reality is should be based on the passages only.
the fundamental goal of the Indian Passage-1
education system. What stands in the way of the widespread and careful
2. The present Indian school system is unable adoption of 'Genetic Modification (GM)' technology is
to provide egalitarian education.
an 'Intellectual Property Rights' regime that seeks to
3.Abolition of private schools and
establishment of more government schools create private monopolies for such technologies. If GM
is the only way to ensure egalitarian technology is largely corporate driven, it seeks to
education. maximize profits and that too in the short run. That is
Which of the above assumptions is/are valid why corporations make major investments for herbicide-
(a) 1 and 2 only tolerant and pest-resistant crops. Such properties have
(b) 2 only
only a short window, as soon enough, pests and weeds
(c) 2 and 3 only
(d) 3 only will evolve to overcome such resistance. This suits the
Ans : (b) According to the above passage, valid corporations. The National Farmers Commission pointed
assumption is (2) The present Indian School system is out that priority must be given in genetic modification to
unable to provide egalitarian education. Hence option the incorporation of genes that can help impart resistance
(b) will be correct answer. to drought, salinity and other stresses.
IAS (Pre) GS IInd Paper (CSAT), 2019 138 YCT
17. Which one of the following is the most logical, 19. Which one of the following is the most logical
rational and crucial message conveyed by the and rational inference that can be made from
above passages? the above passage
(a) Public research institutions should take the (a) Invasive species should be used to
lead in GM technology and priorities the rehabilitate desert areas and wastelands of a
technology agenda. country.
(b) Developing countries should raise this issue (b) Laws against the introduction of foreign
in WTO and ensure the abolition of plants are unnecessary.
Intellectual Property Rights (c) Sometimes, the campaigns against foreign
(c) Private corporations should not be allowed to plants are pointless.
do agribusiness in India, particularly the seed (d) Foreign plants should be used to increase the
business. biodiversity of a country.
(d) Present India circumstances do not favour the Ans : (c) According to above passage the most logical
cultivation of genetically modified crops. & rational inference that sometimes, the campaigns
Ans : (a) Option (b) is not relevant to this passage. against foreign plants are pointless.
Option (c) is and extreme, and is not stated thus, in the Passage - 3
passage. Option (d) is wrong, as the public research Diarrhoeal deaths among Indian children are mostly due
institution will invest in technology that can help impart to food and water contamination. Use of contaminated
resistance to drought, salinity and other stresses rather groundwater and unsafe chemicals in agriculture, poor
than only technologies that work in short run like pest hygiene in storage and handling of food items to food
resistance. cooked and distributed in unhygienic surroundings ;
18. On the basis of the above passage, the following there are myriad factors that need regulation and
monitoring. People need to have awareness of
assumptions have been made :
adulteration and ways of complaining to the relevant
1. The issue of effects of natural calamities on
authorities. Surveillance of food-borne diseases involves
agriculture is not given due consideration
a number of government agencies and entails a good
by GM technology companies.
training of inspection 'staff. Considering the proportion
2. In the long run, GM technology will not be of the urban population that depends on street food for its
able to solve agricultural problems arising daily meals, investing in training and education of street
due to global warming. vendors is of great significance.
Which of the above assumptions is/are valid ? 20. On the basis of the above passage, the following
(a) 1 only assumptions have been made :
(b) 2 only 1. Food safety is a complex issue that calls for a
(c) Both 1 and 2 multipronged solution.
(d) Neither 1 nor 2 2. Great investments need to be made in
Ans : (a) Assumption (1) is correct, as that is clearly developing the manpower for surveillance
mentioned in the passage. Assumption (2) is too broad, and training.
as it assumes that GM technology will "never" be able 3. India needs to make sufficient legislation for
to do it, which may be wrong. Hence, option (a) is best. governing food processing industry.
Which of the above assumptions is/are valid
Passage-2
(a) 1 and 2 only
Most invasive species are neither terribly successful nor
(b) 3 only
very harmful. Britain's invasive plants are no widespread,
(c) 1 and 3 only
not spreading especially quickly, and often less of
(d) 1, 2 and 3
nuisance than vigorous natives such as bracken. The
Ans : (a) Assumptions (1) and (2) on food safety is a
arrival of new species almost always increases biological
complex issue that requires multiple solutions and
diversity in a region; in many cases a flood of requires huge investment in augmenting manpower for
newcomers drives no native species to extinction. One monitoring and training, both are valid as per the above
reason is that invaders tend to colonise disturbed habitats paragraph whereas assumption (3) is not mentioned in
like polluted lakes and post- industrial wasteland, where the above paragraph. hence it is not true. Hence, the
little else lives. They are nature's opportunists. most rational, prudent and decisive option is (a).

IAS (Pre) GS IInd Paper (CSAT), 2019 139 YCT


Passage - 4 (a) 1 and 2 only
The interests of working and poor people have (b) 3 and 4 only
historically been neglected in the planning of our cities. (c) 1, 2 and 3 only
Our cities are increasingly intolerant, unsafe and (d) 1, 2, 3 and 4
unlivable places for large numbers of citizens and yet we
Ans : (c) According to the given passage statement (1)
continue to plan via the old ways-the static Development
is correct because modern economic growth is based on
Plan---that drawn exclusively from technical expertise,
technological progress.
distanced from people's live experiences and needs, and
actively excluding large number of people, places, Statement 2 is correct because modern economy and
activities and practices that are an integral part of the economic growth that is being projected does not reflect
city. the numbers from unorganized sector.
21. The passage seems to argue Statement 3 is also correct. Hence option (c) is correct
(a) against the monopoly of builders and the answer.
interests of elite groups Passage - 6
(b) against the need for global and smart cities India has banking correspondents, who help bring people
(c) in favour of planning cities mainly for in the hinterland into the banking fold. For them to
working class and poor people succeed, banks cannot crimp on costs. They also cannot
(d) in favour of participation of peoples' groups afford to ignore investing in financial education and
in city planning. literacy. Banking correspondents are way too small to be
Ans : (d) Option (a) and (b) are clearly wrong. Option viewed as a systemic risk. Yet India's banking regulator
(c) is not wrong, as it is in line with the argument that has restricted the to serving only one bank, perhaps to
"The interests of the working and the poor have prevent arbitrage. Efforts at banking outreach may
historically been neglected in the planning of our succeed only if there are better incentives at work for
cities". But option (d) is the most logical and rational such last-mile workers and also those providers who
corollary as it also includes option (c). It we start
ensure not just basic bank accounts but also products
involving groups of people in city planning,
such as accident and life insurance and micro pension
automatically we will have the interests of the working
schemes.
class and poor people taken care of. In fact, much more
than that is possible. 23. Which one of the following is the most logical,
rational and crucial inference that can be
Passage - 5
derived from the above passage ?
A vast majority of Indians are poor, with barely 10
(a) Efforts to bring people in India's hinterland
percent employed in the organised sector. We are being
into the banking system are not successful.
convinced that vigorous economic growth is generating
(b) For meaningful financial inclusion, India's
substantial employment. But this is not so. When our
banking system needs more number of
economy was growing at 3 percent per year, employment
in the organised sector was growing at 2 percent per banking correspondents and other such last-
year. As the economy began to grow at 7-8 percent per mile workers.
year, the rate of growth of employment in the organised (c) Meaningful financial inclusion in India
sector actually declined to 1 percent per year. requires that banking correspondents have
22. The above passage seems to imply that diverse skills.
1. most of modern economic growth based on (d) Better banking outreach would be impossible
technological progress unless each banking correspondent is allowed
2. much of modern Indian economy does not to serve a number of banks.
nurture sufficient symbiotic relationship
with labour-intensive natural resource- Ans : (c) Option (c) is correct because the passage
based livelihoods. mentions that if the banking correspondents ensured
3. service sector in India is not very labour- diverse service like bank accounts, life insurance
intensive. pension scheme etc. Then there are more changes of
4. literate rural population is not willing to success at banking outreach. Thus meaningful financial
enter organised sector
inclusion in India requires that banking correspondents
Which of the statements given above are
have diverse skills.
correct ?
IAS (Pre) GS IInd Paper (CSAT), 2019 140 YCT
Directions for the following 7 (seven) items : (a) 1 only
Read the following five passages and answer the items (b) 2 only
that follow each passage. Your answers to these items (c) Both 1and 2
should be based on the passages only. (d) Neither 1 nor 2
Passage - 1 Ans : (d) In the given passage, presently in global
India's economic footprint, given its population, still economic order regulations are also effectively
remains small compared to the US, the European Union incorporated to manage local markets. So option (1) is
or China . It has much to learn from other economies, yet invalid.
must implement solutions that fit its unique Statement II Social and environmental concerns are
circumstances. India especially needs an effective long- generally ignored the governments, is not explained in
term regulatory system based on collaboration rather the passage. So this statement is also invalid.
than the current top-down approach. Regulation seek Hence, option (d) is correct answer.
desirable outcomes yet are repeatedly used as political Passage-2
tools to push one agenda or another. Often, regulations In a study, scientists compared the microbiomes of
fail to consider impacts on jobs and economic growth- or poorly nourished and well nourished infants and young
less restrictive alternatives. Regulations may be used to children. Gut microbes were isolated from faecal
protect local markets at the expense of more widely samples of malnourished and healthy children. The
shared prosperity in the future. Additionally, regulations microbiome was "immature" and less diverse in
inevitably result in numerous unintended consequences. malnourished children compared to the better developed
In today's hyper competitive global economy, regulations "mature" microbiome found in healthy children of the
need to be viewed as ''weapons'' that seek cost Justified same age. According to some studies, the chemical
social and environmental benefits while improving the composition of mother's milk has shown the presence of
economic well-being of most citizens. a modified sugar (sialylated oligosaccharides). This is
24. Which one of the following is the most logical, not utilized by the baby for its own nutrition. However,
rational and crucial inference that can be the bacteria constituting the infant's microbiome thrive
derived from the above passage? on this sugar which serves as their food. Malnourished
(a) A better regulatory system will help India mothers have low levels of this sugar in their milk.
achieve the size of economy appropriate to its Consequently, the microbiomes of their infants fail to
population. mature. That in turn, leads to malnourished babies.
(b) In a competitive global economy, India must 26. Which one of the following is the most logical,
use regulations strategically. rational and crucial inference that can be
(c) Regulations in India do not favour its derived from the above passage?
integration with today's hyper competitive (a) If malnourished condition in children is
global economy. caused by gut bacteria, it cannot be treated.
(d) Job creation and economic growth should be (b) The guts of malnourished babies should be
dominant considerations in developing inoculated with mature microbiomes.
India's regulatory system. (c) Babies of malnourished mothers should be
Ans : (b) In the above given passage, India must need fed with dairy milk fortified with sialylated
regulations strategically in global economic competition oligosaccharides instead of mother's milk.
to set up in world economic order. Hence option (b) is (d) Research on benign effects of gut bacteria on
nutrition has policy implications
correct answer.
Ans : (d) The correct and logical inference is option
25. On the basis of the above passage, the following (d) that research on benign effects of gut bacteria on the
assumptions have been made : nutrition has policy implications.
In today's global economy, 27. On the basis of the above passage, the following
1. regulations are not effectively used to assumptions have been made :
protect local markets. 1. Processed probiotic foods are a solution to
2. Social and environmental concerns are treat the children suffering from
generally ignored by the governments malnutrition due to immature gut bacteria
across the world while implementing the composition.
regulations. 2. The babies of malnourished mothers
Which of the above assumption is/are generally tend to be malnourished.
valid? Which of the above assumptions is/are valid?
IAS (Pre) GS IInd Paper (CSAT), 2019 141 YCT
(a) 1 only 1. Global warming is a frequently occurring
(b) 2 only natural phenomenon.
(c) Both 1 and 2 2. The impending global warming will not
(d) Neither 1 nor 2 adversely affect small mammals.
Ans : (b) Assumption (1) is not correct as it is not 3. Humans are responsible for the loss of the
mentioned in the above passage. Assumption (2) is true Earth's natural resilience.
as in the prose "As a result, the micro biomes of their Which of the above assumptions is/are valid?
babies fail to mature. "Hence, option (b) is correct.
(a) 1 and 2 only
Passage-3 (b) 3 only
Temperatures have risen nearly five times as rapidly on (c) 2 and 3 only
the Western Antarctic Peninsula than the global average (d) 1, 2 and 3
over the past five decades. Researchers have now found Ans : (b) According to the passage, assumption (1)
that melting glaciers are causing a loss of species cannot be taken to be true simply because global
diversity among benthos in the coastal waters off the warming cannot be the only cause. Hence, options (a)
Antarctic Peninsula, impacting an entire seafloor
and (d) cannot be true. Assumption (2) is clearly wrong,
ecosystem. They believe increased levels of suspended
sediment in water to be the cause of the dwindling as it is meant and referred to the opposite. Hence, option
biodiversity in the coastal region. (c) can be rejected. Assumption (3) is clearly mentioned
28. On the basis of the above passage, the following in the passage that man is responsible for the decrease
assumptions have been made : in the natural repulsion of the earth. So option (b) is
1. Regions of glaciers warm faster than other correct.
regions due to global warming.
Passage-5
2. Global warming can lead to seafloor
sedimentation in some areas. Food varieties extinction is happening all over the word
3. Melting glaciers can reduce marine and it is happening fast. For example, of the 7, 000 apple
biodiversity in some areas. varieties that were grown during the nineteenth century,
Which of the above assumptions is/are valid? fewer than a hundred remain. In the Philippines,
(a) 1 and 2 only thousands of varieties of rich once thrived; now only up
(b) 3 only to a hundred are grown there. In China, 90 percent of the
(c) 2 and 3 only wheat varieties cultivated just a century ago have
(d) 1, 2 and 3 disappeared. Farmers in the past painstakingly bred and
Ans : (c) Based on the above passage, assumption 2 and developed crops well suited to the peculiarities of their
3 is true. Because global warming can lead to seaflour local climate and environment. In the recent past, our
sedimentation. Melting glaciers can reduce marine heavy dependence on a few high yielding varieties and
biodiversity is correct statement. Hence option (c) is technology-driven production and distribution of food is
correct answer. causing the dwindling of diversity in food crops. If some
mutating crop disease or future climate change decimates
Passage-4 the few crop plants we have come to depend on to feed
A research team examined a long-term owl roost. Owls our growing population, we might desperately need some
prey on small mammals and the excreted remains of of those varieties we have let go extinct.
those meals that accumulated over the time, provide us 30. On the basis of the above passage, the following
an insight into the composition and structure of small assumptions have been made :
mammals over the past millennia. The research 1. Humans have been the main reason for the
suggested that when the Earth went through a period of
large scale extinction of plant species.
rapid warming about 13,000 years ago, the small
mammal community was stable and resilient. But, from 2. Consumption of food mainly from locally
the last quarter of the nineteenth century, human-made cultivated crops ensures crop diversity.
changes to the environment had caused an enormous 3. The present style of production and
drop in biomass and energy flow. This dramatic decline distribution of food will finally lead to the
in energy flow means modern ecosystems are not problem of food scarcity in the near future.
adapting as easily as they did in the past. 4. Our food security may depend on our
29. On the basis of the above passage, the following ability to preserve the locally cultivated
assumptions have been made : varieties of crops.
IAS (Pre) GS IInd Paper (CSAT), 2019 142 YCT
Which of the above assumptions are valid? 33. What is X in the sequence
(a) 1 and 3 (b) 2 and 4 4, 196, 16, 144, 36, 100, 64, X ?
(a) 48 (b) 64
(c) 2 and 3 (d) 1 and 4
(c) 125 (d) 256
Ans : (b) If you check assumption (3), it is not Ans : (b)
necessarily valid always, but only when a crisis
(mutating crop disease as future climate change)
happens. Hence, eliminate option (a) and (c).
Assumption (1) talks about. "extinction of plant
species" but the passage is about "food varieties".
Assumption (2) is surely correct. So, (2) and (4) are
right and answer is option (b).
∴ x = (8)2
LOGICAL & ANALYTICAL ABILITY x = 64
34.Consider the following sequence that follows
31. Consider the following Statements and some arrangement :
Conclusions : c-accaa-aa-bc-b
Statements The letters that appear in the gaps are
c-accaa-aa-bc-b
1. Some rats are cats (a) abba (b) cbbb
2. Some rats are dogs (c) bbbb (d) cccc
3. No dog is a cow. Ans : (b) c c acc/aa b aa/ b bc b b
Conclusions: ∴The letters that appear in the gaps are cbbb.
I. No cow is a cat 35. Which year has the same calendar as that of
2009 ?
II. No dog is a rat (a) 2018 (b) 2017
III. Some cats are rats. (c) 2016 (d) 2015
Which of the above conclusions is/ are drawn Ans : (d)
from the statements?
(a) I, II and III
(b) Only I and II
(c) Only III
(d) Only II and III
Ans : (c)

Hence, the year of 2015 has the same calendar at that of


2009.
36. Mr 'X' has three children. The birthday of the
first child falls on the 5th Monday of April, that
of the second one falls on the 5th Thursday of
November. On which day is the birthday of his
The conclusion "Some cats are rats" is drawn from the third child, which falls on 20th December ?
statements. (a) Monday (b) Thursday
32. What is X in the sequence (c) Saturday (d) Sunday
Ans : (b)
132, 129, 124, 117, 106, 93, X
(a) 74 (b) 75
(c) 76 (d) 77
Ans : (c)

∴ x = 93 – 17
x = 76

IAS (Pre) GS IInd Paper (CSAT), 2019 143 YCT


If monday falls on 29th April:- Ans : (d)
Then, The remaining day of

215 5 → Remainder
⇒ ⇒3
7 7
Friday falls on 30th November and Thursday
th (i) E, wife of F → true
falls 29 November.
Hence, Thursday is the birthday of his third (ii) E, Grand mother of A → true
child, which falls on 20th December. (iii) E, Aunt of C → true
∴ All three statements are true.
40. If $ means 'divided by'; @ means 'multiplied
by; # means 'minus', then the value of
10#5@1$6 is,
(a) 0 (b) 1
37. In the sequences 1,5,7,3,5,7,4,3,5,7, how many (c) 2 (d) 9
such 5s are there which are not immediately Ans : (d) 10 # 5 @ 1 $ 5
preceded by 3 but are immediately followed by 7? ⇒ 10 − 5 × 1 ÷ 5
(a) 1 (b) 2
1
(c) 3 (d) None of these ⇒ 10 − 5 ×
5
Ans : (a) Given sequences- 1,5,7,3,5,7,4,3,5,7
= 10 – 1
The number of required 5s =1
=9
38. If every alternative letter of the English
41. A solid cube is painted yellow, blue and black
alphabet from B onwards (including B) is
such that opposite faces are of same colour.
written in lower case (small letters) and the
The cube is then cut into 36 cubes of two
remaining letters are capitalized, then how is
different sizes such that 32 cubes are small and
the first month of the second half of the year
the other four cubes are big. None of the faces
written?
of the bigger cubes is painted blue. How many
(a) JuLY (b) jULy
cubes have only on face painted ?
(c) jUly (d) jUlY
(a) 4 (b) 6
Ans : (d) According to question, the English alphabet- (c) 8 (d) 10
b C d E f G h I j……………Yz.
Ans : (c)
Hence, Small letter will be on even places and capital
letters will be on odd places.
Then, The first month of the second half of the year is
written as:- JULY

39. A joint family consists of seven members A, B,


C,D,E,F and G with three females. G is a
widow and sister-in-law of D's father F. B and
D are siblings and A is daughter of B, C is
cousin B. Who is E?
The number of cubes which have only on face printed is
1. Wife of F 8
2. Grandmother of A Note: 4 from + side cube + 4 back side cube = 8 cube.
3. Aunt of C
42. P, Q and R are three towns. The distance
Select the correct answer using the code given
between P and Q is 60 km, whereas the
below- distance between P and R is 80 Km. Q is in the
(a) 1 and 2 only (b) 2 and 3 only West of P and R is in the South of P. What is
(c) 1 and 3 only (d) 1, 2 and 3 the distance between Q and R ?
IAS (Pre) GS IInd Paper (CSAT), 2019 144 YCT
(a) 140 km (b) 130 km GENERAL MENTAL ABILITY
(c) 110 km (d) 100 km
Ans : (d) Direction for the following 3 (three) items:
Read the following information and answer the three
items that follow:
Six students A,B,C,D,E and F appeared in several tests.
Either C or F scores the highest. Whenever C scores the
highest, then E scores the least. Whenever F scores the
highest, B scores the least.
In all the tests they got different marks; D scores higher
than A, but they are close competitors; A scores higher
In ∆ PQR,
than B; C scores higher than A.
QR2 = QP2 + PR2
44. If F stands second in the ranking, then the
QR2 = (60)2 + (80)2
position of B is-
QR2 = 3600 + 6400 (a) Third (b) Fourth
QR2 = 10, 000 (c) Fifth (d) Sixth
QR = 100Km Ans : (c) According to question,
43. 'A' started from his house and walked 20m Conditions will be as follows:-
towards East, where his friend 'B' joined him. efmLeefleÙeeB efvecve nesieer-
They together walked 10m in the same Condition-I Condition-II
direction. Then 'A' turned left while 'B' turned 1→ C 1→ F
right and travelled 2m and 8m respectively. 2 2
Again 'B' turned left to travel 4m followed by 3 3
5m to his right to reach his office. 'A' turned 4 4
right and travelled 12m to reach his office. 5 5
What is the shortest distance between the two 6→E 6→B
offices? Condition III:-
(a) 15 m (b) 17 m
(c) 19 m (d) 20 m
Ans : (b)
If F stands second in the ranking then the position of B-
∴ 1→ C
2→ F
3→D
4→A
5→B
6→E
Hence, The position of B is fifth.
45. If B scroes the least, the rank of C will be-
(a) Second
(b) Third
(c) Fourth
(d) Second or third
From figure, Ans : (d) If B scores the least, for the position of C,
PN = (12 – 4) from conditions IInd & IIIrd
= 8m. Ranking Condition-I Condition-
MN = (2 + 8 + 5) II
= 15m 1. _________ F_________ F
By pythagorus theorem, In ∆ PNM 2. _________ C_________ __
PM2 = MN2 + PN2 3. _________ D_________ C
2 2 2 4. _________ A _________ D
PM = (15) + (8)
5. _________ __ _________ A
PM2 = 225 + 64 6. _________ B _________ B
PP2 = 289 Hence,
PM2 = 17 The position of c will be either IInd or IIIrd

IAS (Pre) GS IInd Paper (CSAT), 2019 145 YCT


46. If E is ranked third, then which one of the and 2R > S ⇒ R ⊁ S
following is correct? either R > S or R = S
(a) E gets more marks than C Hence ,
(b) C gets more marks than E weight of Sohan is least
(c) A is ranked fourth 49. A five-storeyed building with floors from I to V
(d) D is ranked fifth is painted using four different colours and only
Ans : (b) one colour is used to paint a floor.
If E is ranked third, from condition IInd & IIIrd Consider the following statements :
1→F 1. The middle three floors are painted in
2→C different colours.
3→E 2. The second (II) and the fourth (IV) floors
4→D are painted in different colours.
5→A 3. The first (I) and the fifth (V) floors are
6→B painted red.
Hence, C gets more marks than E To ensure that any two consecutive floors have
different colour
Directions for the following 2 (two) items-
(a) Only statement 2 is sufficient
Read the following statements S1 and S2 and answer
(b) Only statement 3 is sufficient
the two items that follow:
(c) Statement 1 is not sufficient, but statement 1
S1. Twice the weight of Sohan is less than the
along with statement 2 is sufficient
weight of Mohan or that of Rohan.
(d) Statement 3 is not sufficient, but statement 3
S2. Twice the weight of Rohan is greater than the
along with statement 2 is sufficient
weight of Mohan or that of Sohan.
Ans : (b)
47. Which one of the following statements is
correct?
(a) Weight of Mohan is greatest
(b) Weight of Sohan is greatest
(c) Weight of Rohan is greatest
(d) 'Whose weight is greatest' cannot be
determined
Ans : (d) From statements S (I)
2S <M ⇒ S<M
2S <R ⇒ S<R I
From State S (2)
2R > M ⇒ R⊁M Any two consectuve floors for different colours
either R>M or R=M Only statement 3 is sufficient
and If the first and fifth floors are painted red, then the
2R > S ⇒ R⊁ S middle three floors will be of different colours. Because
either R > S or R = S the total numbers of colours is 4.
Hence,
50. In a conference, out of a total 100 participants,
Whose weight is greatest can not be
70 are Indians. If 60 of the total participants
determined.
are vegetarian, then which of the following
48. Which one of the following statements is correct?
(a) Weight of Mohan is least statements is/are correct?
(b) Weight of Sohan is least 1. At least 30 Indian participants are
(c) Weight of Rohan is least vegetarian.
(d) 'Whose weight is least' cannot be determined 2. At least 10 Indian participants are non-
Ans : (b) From statement S(I) vegetarian
2S < M ⇒ S < M Select the correct answer using the codes given
below:
2S < R ⇒ S < R
(a) 1 only
From statement S(2)
(b) 2 only
2R > M ⇒ R ⊁ M
(c) Both 1 and 2
either R > M or R = M
(d) Neither 1 nor 2
IAS (Pre) GS IInd Paper (CSAT), 2019 146 YCT
Ans : (c)
Total participants= 100

Hence, 40% members went for shopping as well as


sight seeing
therefore, conclusion (I) is correct
52. Consider two statements S1 and S2 followed by
a question :
S1 : p and q both are prime numbers.
Again, S2 : p + q is an odd integer
Total participants= 100 Question : is pq an odd integer ?
Which one of the following is correct ?
(a) S1 alone is sufficient to answer the question
(b) S2 alone is sufficient to answer the question
(c) Both S1 and S2 taken together are not
sufficient to answer the question
(d) Both S1 and S2 are necessary to answer the
1100 question
Ans : (b)
Hence, The answer of the question "is PQ an add integer",
P + Q = odd integer
∵ 2 + 3 = 5 (odd integer)
then 2 × 3 = 6 (even integer)
Hence p.q is not an odd integer
53. In a school, 60% students play cricket. A
Hence, statements I and II are true.
student who does not play cricket, plays
51. All members of a club went to Mumbai and football. Every football player has got a two-
stayed in a hotel. On the first day, 80% went wheeler. Which or the following conclusions
for shopping and 50% went for sightseeing, cannot be drawn for the above data ?
whereas 10% took rest in the hotel. Which of 1. 60% of the students do not have two-
the following conclusion (s) can be drawn from wheelers.
the above data ? 2. No cricketer has a two-wheeler.
1. 40% members went for shopping as well 3. Cricket players do not play football.
as sightseeing. Select the correct answer using the code given
2. 20% members went for only shopping. below :
Select the correct answer using the code given (a) 1 and 2 only
below (b) 2 and 3 only
(a) 1 only (c) 1 and 3 only
(b) 2 only (d) 1, 2 and 3 only
(c) Both 1 and 2 Ans : (a) then
(d) Neither 1 nor 2
Ans : (a)

∵Every football player has got a two wheeler,


then,
then

IAS (Pre) GS IInd Paper (CSAT), 2019 147 YCT


It does not mean that cricket players has not got 3 + (7 – x) + x + (8 – x) = 15
a two wheeler and can not say, 60% of students do not 18 – x = 15
have two wheelers. It is clear that cricket players do not x =3
play football. Then, the number of people who can read exactly one
Hence Conclusions I and II can not be drawn for thelanguage
given data. = (7 – x) + 8 – x
54. A wall clock moves 10 minutes fast in every 24 = (7 – 3) + (8 – 3)
hours. The clock was set right to show the =4+5
correct time at 8 : 00 a.m. on Monday. When =9
the clock shows the time 6:00 p.m. on
General Mathematics/Numerical Ability
Wednesday, what is the correct time ?
(a) 5: 36 p.m 56. The number of times the digit 5 will appear
(b) 5: 30 p.m while writing the integers from 1 to 1000 is
(c) 5: 24 p.m (a) 269 (b) 271
(d) 5: 18 p.m (c) 300 (d) 302
Ans : (c) writing the integers from 1 to 1000 the
Ans : (a) According to question,
number of 5 which place in unit digit = 100
from, at 8 : 00 AM on Monday to at 6 : 00 PM
The number of 5 which place in tenth digit =100
the number of 5 which place in hundred digit =100
∴ Total number of 5 = 300
57. A and B are two heavy steel blocks. If B is
placed on the top of A, the weight increases by
60% . How much weight will reduce with
respect to the total weight of A and B, if B is
removed from the top of A ?
∵ Every 24 hr clock moves fast 10 minutes (a) 60% (b) 45.5%
∴total 58 hr clock moves fast (c) 40 % (d) 37.5%
10 Ans : (d) Let the weight of A = 100kg
= × 58min
24 ∴A + B = 160
∴B = 160 –100
= 24.16 (Fast) ≃ 24.16 min B = 60
∴ the correct time is as 6 pm on wednesday= 6: 00– ∵B is removed from A,
0.24 =5 : 36 PM ∴ Required decrease % =
55. In a group of 15 people, 7 can read French, 8 60
= ×100%
can read English while 3 of them can read 160
neither of these two languages. The number of 3 3
= × 100% = × 25%
people who can read exactly one language is ? 8 2
(a) 10 (b) 9 75 1
= % ⇒ 37 %
(c) 5 (d) 4 2 2
Ans : (b) 58. The number of parallelograms that can be
formed from a set of four parallel lines
intersecting another set of four parallel lines, is
(a) 18 (b) 24
(c) 32 (d) 36
Ans : (d) P Q R S

IAS (Pre) GS IInd Paper (CSAT), 2019 148 YCT


(a) Raju still does not have enough amount to
buy the handset.
(b) Raju has exactly the same amount as required
to buy the handset.
(c) Raju has enough amount to buy the handset
and he will have ` 500 with him after buying
the handset.
(d) Raju has enough amount to buy the handset
and he will have ` 1000 with him after
buying the handset
The number of parallelogram when are smaller Ans : (a) ∵ Raju has 9000 Rs
parallegram is taken = 9 Required amount to buy the hand set
the number of parallelogram were two smaller 100
parallelogram is taken = 12 ⇒ 9000 ×
75
The number of parallelogram when three smaller
4
parallelogram is taken = 6 =9000 ×
the number of parallelogram when four smaller 3
parallelogram is taken = 4 = 3000 ×4
The number of parallelogram when six smaller = 12000
parallelogram is taken = 4 ∵ Raju borrowed 2000 Rs. from a friend.
the number of parallelogram when nine smaller
parallelogram is taken = 1 ∴ Raj has total amount (9000 + 2000)
the number of total parallelogram = 11000
⇒ 9 + 12 + 6 + 4 + 4 + 1 Hence,
⇒ 36 Raju still does not have the required amount to buy
59. In a school every student is assigned a unique mobile.
identification number. A student is a football 62. In 2002, Meenu's age was one-third of the age
player if and only if the identification number of Meera, whereas in 2010, Meenu's age was
is divisible by 4, whereas a student is a half the age of Meera. What is Meenu's year of
cricketer if and only if the identification
birth ?
number is divisible by 6. If every number from
(a) 1992 (b) 1994
1 to 100 is assigned to a student, then how
many of them play cricket as well as football? (c) 1996 (d) 1998
(a) 4 (b) 8 Ans : (b) In 2002,
(c) 10 (d) 12 Let age of Meera = 3x
Ans : (b) L.C.M. of (4, 6) = 12 ∴ Age of Meenu = x
then, In 2010,
the number of student who play cricket as well Age of Meera = (3x + 8)
100 Age of Meenu = (x + 8)
as football = ⇒ 8.33
12 According to questions,
≈ 8 (taken integer) (3x + 8) = 2(x + 8)
60. When a runner was crossing the 12 km mark 3x + 8 = 2x + 16
she was informed that she had completed only
80% of the race. How many kilometres was the x =8
runner supposed to run in this event ? Hence, the age of Meenu in 2002 = x = 8 years
(a) 14 (b) 15 then Meenu's year of birth = 2002 – 8 ⇒ 1994
(c) 16 (d) 16.5 63. Rakesh and Rajesh together bought 10 balls
Ans : (b) ∵ 80% = 12 km and 10 rackets. Rakesh spent ì 1300 and Rajesh
12 12
∴ 100% = × 100 = × 5 = 15 km spent ì 1500. If each racket costs three times a
80 4 ball does then what is the price of a racket ?
61. Raju has ì 9000 with him and he wants to buy (a) ì 70 (b) ì 90
a mobile handset; but he finds that he has only (c) ì 210 (d) ì 240
75% of the amount required to buy the Ans : (c) Let the price of ball = x
handset. Therefore, be borrows ì 2000 from a ∴ the price of rocket= 3x
friend. Then 10 balls + 10 Racekts = 1300 + 1500

IAS (Pre) GS IInd Paper (CSAT), 2019 149 YCT


10 ×x + 10 + 3 x = 2800 y= (1000-40)m z = (1000-64)m
40x =2800 = 960 m z = 936 m
Hence, x:y:z= 1000 : 960:936
x = 70Rs for y and z
then the price of rocket= 3x =3 × 70 = 210 Rs. y 960
=
64. The ratio of a two-digit natural number to a z 936
number formed by reversing its digits is 4 : 7 1000 960
=
The number of such pair is z 936
(a) 5 (b) 4 936 × 1000
(c) 3 (d) 2 z=
960
Ans : (b) According to questions, z = 975 m
Requred numbers ⇒ (12, 21) ⇒ (24, 42) Hence, y will give the start of 25 meters to z.
⇒ (36, 63) ⇒ (48, 84) 68. If x is grater then or equal to 25 and y is less
Hence, than or equal to 40, than which one of the
the numbers of required pairs = 4 following is always correct ?
(a) x is greater than y
65. In an examination, A has scored 20 marks
(b) (y–x) is greater than 15
more than B. If B has scoured 5% less marks (c) (y–x) is less than or equal to 15
than A. how much has B scored? (d) (x+y) is greater than or equal to 65
(a) 360 (b) 380
(c) 400 (d) 420 Ans : (c) ∵ x ≥ 25.............................................(i)
Ans: (b) Let B obtained x marks, 40 ≥ y.............................................(ii)
From egn (i) & (ii),
then As obtained makrs= (x + 20) (x+40) ≥ (y + 25)
According to question, (40–25) ≥ (y–x)
5% =
20
× 100% 15 ≥ (y–x)
( x + 20 ) Hence, (y – x) is less than or equal to 15
5 (x + 20) = 2000 69. Ena was born 4 years after her parents'
5x +100 = 2000 marriage. Her mother is three years younger
5x = 1900 than her father and 24 years older than Ena,
x = 380 who is 13 years old. At what age did Ena's
Hence, father get married ?
B obtained 380 marks (a) 22 years (b) 23 years
(c) 24 years (d) 25 years
66. Seeta and Geeta go for a swim after a gap of
every 2 days and every 3 days respectively. If Ans : (b)
on 1st January both of them went of a swim Father Mother Ena
together, when will they go together next? When Era 37 + 3 13 + 24 13
(a) 7th January (b) 8th January was 13 = 40 =37
th
(c) 12 January (d) 13th January years old
Ans: (d) time of 40–13 =27 37–13 =24 0
birth of
∵ Seeta go for a swim after a gap of every 2 days : that
Era
means Seeta go for a swim after 3 days. and Geeta go 4 years 27.4 ⇒ 23 24 – 4 Ena is not
for a swim after a gap of every 3 days : that means before ⇒ 20 born yet
Geeta go for a a swim after 4 days. Ena's birth
∴The L.C.M. of (3, 4) = 12 Hence, Era's father got married at the age of 23
∵Seeta and Geeta went of a swim together 1st of 70. Rakesh had money to buy 8 mobile handsets of
January a specific company. But the retailer offered
∴They go together next= (1 + 12)th January very good discount on that particular handset.
=13th January Rakesh could buy 10 mobile handsets with the
67. X, Y and Z are three contestants in a race of amount he had. What was the discount the
1000 m. Assume that all run with different retailer offered ?
uniform speeds. X gives Y a start of 40m and X (a) 15 % (b) 20 %
gives Z a start of 64m. If Y and Z were to (c) 25 % (d) 30 %
compete in a race of 1000m. how many metres
Free Items
start will Y give to Z ? Ans : (b) ∵ discount % = ×100%
(a) 20 (b) 25 Totalitems
(c) 30 (d) 35
Ans : (b) x = 1000m and x = 1000 m

IAS (Pre) GS IInd Paper (CSAT), 2019 150 YCT


2 96+m =150
= × 100% = 20% m = 150-96
10
71. The average marks of 100 students are given to m = 54kg
be 40. It was found later that marks of one 74. An 8-digit number 4252746B leaves remainder
student were 53 which were misread as 83. 0 when divided by 3. How many values of B are
The corrected mean marks are possible?
(a) 39 (b) 39.7 (a) 2 (b) 3
(c) 40 (d) 40.3 (c) 4 (d) 6
100 × 40 + 53 − 83 Ans : (c) An 8-digit number 4252746 B leaves
Ans : (b) The correct mean = remaineder 0,
100
when divided by 3
4000 − 30 3970
= = = 39.70 ∵ Divisibility of 3 : if the sum of digits of any number
100 100
divisible by 3, the number will completely divided by 3.
72. If the numerator and denominator of proper
then, 4 2 5 2 7 4 6 B
fraction are increased by the same positive
⇒4+2+5+2+7+4+6+B
quantity which is greater than zero , the = 30 + B
resulting fraction is ?
∴ Possible value of B,
(a) always less than the original fraction 30 + 0 = 30 divisible by 3
(b) always greater than the original fraction 30 + 3 = 33 divisible by 3
(c) always equal to the original fraction 36 + 6 = 36 divisile by 3
(d) such that noting can be claimed definitely 30 + 9 = 39 divisible by 3
Ans : (b) If the numerator and denominator of proper Hence,
fraction are increased by the same positive quantity The possible values of 13 are 4
which is greater than zero. The resulting fraction is 75. Sunita cuts a sheet of paper into three pieces.
always greater than the original fraction. Length of first piece is equal to the average of
For example, the three single digit odd prime numbers.
1 Length of the second piece is equal to that of
= 0.5
2 the first plus one-third the length of the third.
1+1 2 The third piece is as long as the other two
= ⇒ 0.66 pieces together. The length of the original sheet
2 +1 3
of paper is-
1 2 (a) 13 units (b) 15 units
Hence <
2 3 (c) 16 units (d) 30 units
73. A family has two children along with their Ans : (d) Let
parents .The average of the weights of the The length of three pieces of papes is x, y and z
respectively
children and their mother is 50 Kg. The
then,
average of the weights of the children and their
3+5+ 7  1
father is 52kg. If the weight of the father is x= x +z× = y
60kg then what is the weight of the mother ? 3  3
(a) 48 kg (b) 50 kg 15 z
x= x + = y ........(ii)
(c) 52 kg (d) 54 kg 3 3
2c + m x= 5........(i) z = x+y..........(iii)
Ans : (d) ∵ = 50
3 From eg.n (ii) & (iii)
⇒ 2c + m = 150...................(i) 2
x+ =y (∵ z = x + y)
2c + F 3
and ⇒ = 52 ( x + y) = y
3 x+
⇒ 2c+F= 156 3
⇒ 2c+60=156 (∵ F= 60 kg) 3x + x + y
=y
2c = 156-60 3
2c = 96 4x + y = 3y
c = 48 4x = 2y
Putting the value of C in equation (i) y = 2x
2c+m = 150
2×48+m =150 y=2×5 from eqn (i) {∵ x = 5}
IAS (Pre) GS IInd Paper (CSAT), 2019 151 YCT

y = 10 ( 6 − 1)C( 6− 4 )

∵z=x+y ⇒ 5c2
n
z = 5 + 10 5 ∵ n cr =
⇒ r (n − r)
z = 15 23
∴ The length of the original sheet of paper 5 5× 4× 3
= x + y + z = 5 + 10 + 15 = 30 units = = = 10
2× 3 2 × 1× 3
76. Number 136 is added to 5B7 and the sum
obtained is 7A3, where A and B are integers. It 79. Suppose you have sufficient amount of rupee
is given that 7A3 is exactly divisible by 3. currency in three denominations : ` 1, ` 10
The only possible value of B is and ` 50 . In how many different ways can you
(a) 2 (b) 5 pay a bill of ` 107 ?
(c) 7 (d) 8 (a) 16 (b) 17
Ans : (d) (c) 18 (d) 19
1 3 6 Ans : (c) Condition (I), when 50 rupee notes are not
used :
+ 5 B 7 (i) 0 ten rupee note + 107 ones = 107
7 2 3 (ii) ten rupee note + 97 one = 107
(iii) 2 ten rupee note + 87 ones = 107
∵ Divisibility of 3 : The sum of digits must be divisible : :
by 3 : :
then, 7 A 3 ⇒ 7 + A + 3 (iv) 10 tens rupee note + 7 ones = 107
⇒ 10 + A Possible ways =11
∴ A=2 Condition : (ii)
∴ 1 3 6 When one fifty rupee note is used :
(i) o ten rupee note + 57 ones + 1 fifty = 107
+ 5 B 7 (ii) 1 ten rupee note + 47 ones + 1 fifty = 107
7 2 3 (iii) 5 tens + 7 ones + 1 fifty = 107
Possible ways =6
Hence, B = 8 Conditions (III)
77. A printer numbers the pages of a book starting When two fifty rupee notes are used :
with 1 used 3089 digits in all. How many pages ⇒ (i) o ten + 7 ones + 2 fifty = 107
does the book have ? Possible ways =1
(a) 1040 (b) 1048 ∴ Total possible ways = 11 + 6 + 1 = 18
(c) 1049 (d) 1050 80. Each face of a cube can be painted in black or
Ans : (c) Number of digits from 1 to 9 = 9 × 1 =9 white colours. In how many different ways can
Number of digits from 10 to 99 = 90 × 2 ⇒ 180 the cube be painted?
Number of digits from 100 to 999 = 900 × 3 = 2700 (a) 9 (b) 10
Hence, (c) 11 (d) 12
Total number of digits from 1 to 999 =9 +180 +2700 Ans : (b) White→ (W)
= 2889 Black→(B)
For using 3089 digits, Possible Ways-
remaining digits= 3089 – 2889 = 200 (i) WW, WW, WW
Now all the pages after 999 will use 4 digits (ii) WW, WW, WB
200 (iii)WW, WB, WB
then number of pages with 4 digits = = 50 pages
4 (iv) WW, WW, BB
Hence, (v) WW, WB, BB
Total pages= 9 + 90 + 900 + 50 = 1049 (vi) WB, WB, WB
78. How many triplets (x,y,z) satisfy the equation (vii) WW, BB, BB
x+y+z=6, where x, y and z are natural
(viii) WB, WB, BB
numbers?
(a) 4 (b) 5 (ix) WB, BB, BB
(c) 9 (d) 10 (x) BB, BB,BB
→ Possible ways = 10
Ans : (d) From,
(permutation & combination)

IAS (Pre) GS IInd Paper (CSAT), 2019 152 YCT


UNION PUBLIC SERVICE COMMISSION
Civil Services (Preliminary Exam) - 2020
CSAT : PAPER-II
(Chapterwise Analysis with Explanation)
Time : 2 hours Maximum Number : 200

COMPREHENSION Ans. (b) : Statement 1 is a correct assumption since, the


passage clearly highlights the negative consequence of
Directions for the following 7 items: foreign investment. Statement 2 is wrong since, it
Read the following five passages and answer the items implies that the state has the ability to interfere and
that follow. Your answers to these items should be reduce volatility in foreign private investment. This is
based on the passages only. contrary to economic logic and the passage also makes
Passage-1 no such reference to capability of the state to reduce
such volubility.
Private investment in general is volatile. Foreign private
Statement 4 seems to be correct as the passage begins
investment is more volatile because the available
by highlighting that private investment is volatile.
investment avenues are significantly greater (i.e., the
Similarly, statement 5 sounds correct as it suggest
entire world). Therefore, the responsibility of providing
primary importance of public investment in same
employment cannot be left to Foreign Direct Investment
sectors. Hence, option (b) is correct.
(FDI). The current FDI inflows are volatile over time and
across sectors and regions, which is a necessary Passage-2
consequence of their search for the highest returns. The Many opportunities to harness the highly skewed,
adverse consequences are unstable employment and an seasonal and spatial distribution of monsoon flows,
accentuation of income and regional inequalities. A which occur in a four-month period from June to
probable positive consequence of foreign investment is September annually, have been lost. Since these few
the inflow of new technology and its subsequent months account for most of the rainfall and consequent
diffusion. However, the technology diffusion is not at all fresh water availability, the need for holding rainwater in
certain because the existing state of physical and human reservoirs, for subsequently releasing it for use over the
capital in India may prove inadequate for the diffusion. year, is a necessarily nobody can afford to overlook.
1. With reference to the above passage, the Climate change will continue to affect weather
following assumptions have been made : conditions and create water shortages and excesses.
1. Relying on foreign investment in the long run is While millions suffer from droughts and floods, waters
not an economically sound policy. in the country's many rivers flow unutilized and are
2. Policies must be undertaken to reduce volatility discharged into the sea every year.
in foreign private investment. 2. With reference to the above passage, which of
3. Policies must be undertaken to strengthen the following could be the most rational and
domestic private investment. practical implications for India?
4. Public investment should be given priority over 1. Inter-linking of rivers should be undertaken.
private investment. 2. A network of dams and canals should be built
across the country for proper distribution of
5. Substantial public investment in education and
water.
health should be undertaken.
3. Farmers should be provided easy loans for
Which of the above assumptions is/are valid?
digging borewells.
(a) 1, 2 and 4 4. Usage of water for agriculture should be
(b) 1, 3 and 5 regulated by law.
(c) 2, 4 and 5 5. Distribution of river water among regions
(d) 3 only should be regulated by the Union Government.
IAS (Pre) GS IInd Paper (CSAT), 2020 153 YCT
Select the correct answer using the codes given Ans. (c) : Statement (1) is false because according to it
below: lower tax rates will always increase investment in
higher education. Although there is a relationship
(a) 1 and 2
between the two, it is not always true. Statement (2) is
(b) 2, 4 and 5 false because it is economic freedom that results from
(c) 1, 3 and 4 lower tax rates. Which ensures investment in higher
(d) 2, 3 and 5 education. Statement (3) is true because more economic
freedom makes way for investment in education, which
Ans. (a) : Statement 1 is correct since this is an attempt is an important component of human capital formation,
to optimise and efficiently utilise the skewed hence the most appropriate option is (c).
distribution of water in the country's rivers. Statement 2 Passage-4
is correct because it implies the need to ensure proper Our urban bodies cannot possibly ensure sustainable
distribution of water, the need for which is brought out delivery of water in our cities unless financing
in the last last sentence of the passage that says millions mechanisms are put in place. Water delivery requires
suffer from droughts and floods. Statement 3 is contrary heavy investment in collecting it from a natural source,
treating it to make it potable, and laying a distribution
to logic since digging borewells would have an adverse network of pipes for delivery to the users. It also requires
impact on the distribution of water furthermore, the investments in sewage treatment plants so that the sewers
passage does not mention the inability of farmers to can carry the waste water to these plants to ensure that no
secure credit to dig borewells. Hence, it cannot be the untreated sewage is discharged back into natural water
bodies. If our cities were rich enough to meet the entire
correct implication from the same.
cost, water could be delivered free. They are not.
Statement 5 is incorrect since the passage does not bring 4. What is the most logical and crucial message
out inability of states or federal relations as reasons for conveyed by the passage?
the skewed distribution of water. By elimination, option (a) Urban local bodies must recover costs
(a) is the correct answer. through user charges
(b) Urban local bodies are not efficient enough to
Passage-3 meet the water requirements of our cities
(c) Water shortage in our cities is a perennial
People will invest in education whenever they are problem that cannot be solved
granted the economic freedom to fully enjoy its benefits. (d) In view of the water crisis in our cities, there
Again, this is for the obvious reason that the return on is an urgent need to limit the population of
education increases as the level of economic freedom cities by adopting an upper limit of
rises. When people, thanks to lower tax rates, are population size
allowed to retain most of the higher income that they Ans. (a) : Option (a) is correct since the passage clearly
implies that a funding mechanism should be put in place
gain from each incremental level of education, it makes
for maintaining the Urban local bodies. One such
eminent sense to invest in education. On the other hand, mechanism could be leveling consumer charges.
when the government decides to tax the higher income of
5. With reference to the above passage, the
educated individuals at even higher rates, it makes very following assumptions have been made:
little sense to invest in educating oneself further. The 1. Rich cities only can ensure sustainable delivery
same incentives apply to parents who decide on whether of water.
to invest in their children's education. 2. Sustainable delivery of water in cities means
3. With reference to the above passage, the much more than supplying water to
households.
following assumptions have been made:
Which of the above assumptions is/are valid?
1. Lower tax rates in a country invariably
(a) 1 only
translate into greater investments in higher (b) 2 only
education. (c) Both 1 and 2
2. Investment in the education of children ensure (d) Neither 1 nor 2
their economic freedom. Ans. (b) : The passage clearly indicates that only
3. Economic freedom has a positive impact on prosperous cities can ensure a steady and adequate
building up human capital. supply of water is not sufficient. However, the basic
assumption of the passage is that the long-term
Which of the above assumptions is/are valid?
sustainable supply of water cities is not limited to the
(a) 1 only (b) 2 only supply of water to households alone. Hence only
(c) 3 only (d) 1, 2 and 3 assumption (2) is valid.

IAS (Pre) GS IInd Paper (CSAT), 2020 154 YCT


Passage-5 Ans. (d) : The best transmission of the critical message
In India, agriculture still engages about half of its of the passage is that the development of inclusive and
sustainable agriculture can play an important role in
workforce, and about 85 percent of its farms are small
and marginal. Compared to China and Vietnam, which poverty alleviation in the near future. Hence, option (d)
have experienced fast structural and rural transformation, is the correct answer.
India's story is of slow transformation. As a result, Directions for the following 6 (six) items :
poverty reduction in India was at a much slower pace Read the following five passages and answer the
during 1988-2014, compared to China and Vietnam. items that follow. Your answers to these items should
India's poverty reduction was slow during 1988-2005, be based on the passage only.
but during 2005-2012, it accelerated dramatically–almost Passage–1
three times faster than during the earlier period. What did In India, over the last decade or so, labour has been
India do during this period? Research reveals that the departing agriculture, but is only going to
relative price scenario changed significantly (by more construction and unregistered manufacturing which
than 50%) in favour of agriculture in the wake of rising are not marketly better jobs. Services, where labour
global prices. This boosted private investments in tends to be most productive, are not generating the
agriculture by more than 50%. As a result, agri-GDP additional jobs the country needs. India will need 24
growth touched 4.1% during 2007-2012 as against 2.4% million or so jobs over the next decade. The new
during 2002-2007. The net surplus of agri-trade touched sector, e-commerce, can at best close only half the
$25 billion in 2013-14; real farm wages rose by 7% per jobs gap. Only those sectors that drive domestic
annum. All this led to unprecedented fall in poverty. demand such as health and education can
6. With reference to the above passage, the comfortably fill the other half.
following assumptions have been made : 8. Which one of the following is best implied in
1. Structural and rural transformation is the passage?
impossible when farms are mainly small and (a) Strong measure need to be taken to reduce the
marginal. rural to urban migration of labour.
2. A good price incentive can trigger investments (b) The working condition in construction and
in agriculture. unregistered manufacturing needs to be
3. India needs to build value chains for high value improved.
agri-products like livestock and horticulture. (c) Service sector has been reducing the problem
4. Higher global prices of agricultural of unemployment.
commodities are essential for India's poverty (d) Increased social sector spending is imperative
reduction. for large-scale job creation.
Which of the above assumptions are valid? Ans. (d) : The most important implication of the given
(a) 1 and 3 (b) 2 and 4 passage is that there is a need to increase the
(c) 2 and 3 (d) 3 and 4 expenditure of the social sector to stop the migration of
Ans. (c) : In the above passage the logical and practical workers from rural to urban areas and to generate
implication is that the remunerative price of the crops employment on a local scale. One such scheme is the
attracts more investment in incentives agricultural Atmanirbhar Bharat campaign, the ambitions plan of the
products. India needs to build value chains for high government of India during the recent Covid-19 global
value agriproducts like livestock and horticulture. disaster.
Hence, option (c) is correct answer.
Passage-2
7. Which one of the following statements best
reflects the critical message of the passage? In India, the current focus on the right to privacy is
(a) India should create large-scale off-farm rural based on some new realities of the digital age. A right
employment to reduce poverty in the near is a substantive righty only if it works in all situations,
future. and for everyone. A right to free expression for an
individual about her exploitation, for instance, is
(b) India should create a large number of farmer
meaningless without actual availability of security
producer companies.
that guarantees that private force cannot be used to
(c) Private investment in agriculture should be thwart this right. The role of the State, therefore, is
given priority over public investment. not just to abstain from preventing rightful free
(d) Inclusive agricultural growth is key to reduce expression, but also to actively ensure that private
poverty in the near future. parties are not able to block it.
IAS (Pre) GS IInd Paper (CSAT), 2020 155 YCT
9. On the basis of the above passage, the following Passage–4
assumptions have been made :
Asset allocation is the most important investment
1. State should have some institutions to decision we will ever make, and sadly, most of us do
ensure its appropriate role in a digital not give that decision the importance it deserves. We
society. are adamant about seeking predictability with our
2. State should ensure that private parties do future. We tend to think of investing in risky assets as
extremely volatile and value croding. we also dislike
not violate the citizens’ rights to privacy. fluctuating returns and the loss of control of
3. Digital economy is not compatible with the investment. We think our money is best left idle,
idea of not violating the citizen’s privacy. unproductive but safe. There is no asset that is risk-
Which of the above assumptions is/are valid? free. We could lose our jobs, our homes can lose
value, our banks can go bankrupt, our bonds can
(a) 1 and 2 (b) 3 only
default, the government can collapse and companies
(c) 1 and 3 (d) 2 only we chose fondly may cease to exist. But we cannot live
Ans. (a) : The presumption made by the passage shows life assuming that all these extreme events are waiting
that, to ensure its role and importance in the to happen, and all at the same time. All these extreme
forms of risks we know will not manifest at the same
technological (digital) Society, the state has to establish time.
same public institutions, at the same time the state must 11. Which one of the following statements best
also ensure that any, even private parties could not implies the suggestion given by the author of
infringe on the right to privacy of citizens. Hence, both the passage?
the assumptions (1) & (2) are valid. (a) Distribute your wealth across different kinds
of assets so that your risks would be
Passage-3 minimized.
(b) Risk-taking behavior should be a necessary
One of the biggest ironies around water is that it
component of your personality if you want to
comes from rivers and other wetlands. Yet it is seem generate wealth.
as divorced from them. White water is used as a (c) While making investments, find a trustworthy
resource, public policy does not always grasp that it a asset management organization which would
is a part of the natural ecosystem. Efforts at manage your wealth for you.
engineering water systems are thus efforts at (d) You should know that investing your money
augmenting water supply rather than strengthening is a risky business.
the capacities of ecological systems Ans. (a) : In the given passage, the author has given
utmost importance on the fact that it would be best to
10. Which one of the following is the most logical keep money and property by investing in different types
and rational inference that can be made from of assets, because by doing so the risk and loss will be
the above passage? minimized. Hence, option (a) indicates the best advice
(a) Rivers and other wetlands should be protected of the author.
under Ramsar Convention. Passage–5
(b) Engineering water systems should be Although most of the Genetically Modified (GM)
modernized and further augmented. crops cultivated now are genetically engineered for a
(c) Wetlands need to be reinforced as more than single trait, in future, crops genetically engineered for
just open sources of water. more than one trait will be the norm. thus,
(d) Water supply should not be free of cost so as biotechnology’s role in agriculture and the regulation
to prevent its misuse or overuse. of the same cannot be understood solely in the context
of the current generation of GM crops. Instead, there
Ans. (c) : Option (a) is incorrect as the statement is a need to take a comprehensive look, taking into
relating to Ramsar convention is not implicit in the account various aspects, including socio-economic
passage. Option (b) is incorrect as the option given in impacts, so that the potential of the technology can be
the passage suggest the contrary. Option (c) is correct harnessed while minimizing negative impacts. Given
because the passage states that "It is ironic that water the importance of biotechnology in developing
comes from rivers and wetlands, yet water is considered varieties that can help in climate change mitigation
and adaptation, not using biotechnology as a part of
separate from them." Therefore, they should be
the climate change action plan cannot be an option.
reinforced more, although option (d) is correct from its Domestic regulation of biotechnology cannot be
statement but it is not mentioned in the passage. Hence viewed in isolation of trade policy and obligations
option (c) is the most appropriate. under various international treaties and conventions.

IAS (Pre) GS IInd Paper (CSAT), 2020 156 YCT


12. With reference to the above passage, the 14. Which one of the following is the most logical
following assumptions have been made: and rational inference that can be made from
1. Biotechnology regulation is an evolving the above passage?
process.
(a) Indian banking system is not able to help the
2. Participation of people is needed in policy
decisions regarding biotechnology country in its economic growth.
regulation. (b) Economic reforms that started in 1991 have
3. Biotechnology regulation should take into not helped in improving the economy to
account socio-economic aspects in decision- expected levels.
making. (c) India lacks the institutional mechanism to
4. Wider involvement of political executive in deal with the failure of banks.
biotechnology regulation improves its
effectiveness in dealing with the country’s (d) Encouraging the foreign investments in our
trade policies and international obligations. industrial sector is a good alternative to this
Which of the above assumptions are valid? sector’s dependence on banks for credit.
(a) 1, 2 and 4 only (b) 1 and 3 only Ans. (c) : The most logical and rational conclusion can
(c) 2, 3 and 4 only (d) 1, 2 3 and 4 be drawn from the above passage that the economic
Ans. (b) : It can be easily inferred from the given reforms started in 1991 through libertization,
passage that biotechnology regulation is a developing or
privatization, and globalization have not achieved the
emerging biological process, due to which the
government must also take into account its socio- expected success in improving the condition of the
economic aspects before taking a decision on economy of the nation and the universal central
biotechnology regulation. Will have to keep in regulation institute and the lack of process have also
therefore, assumptions (1) & (3) seems to be valid. made the success of the banks doubtful.
13. Which one of the following statements best
implies the crux of the passage? Passage–2
(a) Precautionary principles is not given India has tremendous potential for solar energy. We
importance in current debate on developing all realize that we have to stop burning fossil fuels to
GM crops
meet our energy needs. But certain renewable
(b) Biotechnology is not currently used in climate
change mitigation and adaptation resources are still going through their cost curves and
mechanisms. learning curves to get the required amount of output.
(c) Biotechnology’s role is not confined to the The Indian government has strongly committed to its
current priorities of developing GM crops. targets of reducing emissions by 33 percent by 2030,
(d) The negative impacts of biotechnology are and towards this is has initiated a strong push
not properly understood. towards a gas-based economy and has also invested
Ans. (c) : It is clear from the passage that the role of heavily in renewable energy. However, business
biotechnology is not limited to the current priorities of houses are very of investing too heavily in renewable
developing GM crops in addition, domestic regulation
energy at a time when the technology is not yet ready.
of biotechnology should not be seen in isolation from
trade and international treaties and agreements. Hence, 15. Which one of the following is the most logical
option (c) is the correct answer. and rational inference that can be made from
Directions for the following 6 (six) items : the above passage?
Read the following five passage and answer the (a) India’s commitment to reduce emissions by
items that follow. Your answers to these items should 33% is unlikely to be achieved.
be based on the passages only. (b) India should import gas rather than invest in
Passage–1 renewable resources.
Bank credit to the industrial sector has started (c) Getting renewable resources to market too
shrinking. Its decline has been a serious concern as soon may be costly.
credit growth is essential to revive investment. The
problem’s origins lie in the incomplete reforms of the (d) India should put in more efforts in the
last 25 years. An institutional change that should have exploration of natural gas.
followed the 1991 reforms should have been setting Ans. (c) : The most rational & logical conclusion that
up of a resolution corporation for banks. In a market can be drawn from the above passage is that it can be
economy with booms and busts, banks should be very costly to bring renewable resources to the market
allowed to be set up and to fail. Today, we cannot
early due to unavailable of more cost-effective and high
shut down banks because there is no proper system to
shut them down. Weak loss-making banks continue to level of technological up-gradation at present. Hence
need more capital. option (c) is the correct answer.

IAS (Pre) GS IInd Paper (CSAT), 2020 157 YCT


16. With reference to the above passage, the Passage-4
following assumptions have been made :
Many people understand the connection between
1. Governments often provide inefficient and
solid waste management and health in terms of the
costly subsidies for technologies that may
consequences of unattended heaps of dry garbage
not be ready in the near future.
which become home for flies and other vermin.
2. India’s commitment of reducing emissions However, there is another aspect that is not well-
by 33% by 2030 shall be on the basis of understood, that is, what happens when unscientific
gas-based economy. solid waste management combines with poor
Which of the above assumptions is/are valid? drainage and dumping of untreated sewage into
(a) 1 only drains which are meant to carrying storm water
(b) 2 only during rains. The result is choked drains which are
(c) Both 1 and 2 full of stagnant water breeding mosquitoes, resulting
(d) Neither 1 nor 2 in the spread of water-borne diseases.
Ans. (a) : In above passage, given statement 1 is valid 18. In the context of India, which one of the
because provided costly and insufficient subsidies for following statements best reflects the critical
technologies that may not be ready in the near future. message of the passage?
Hence, option (a) is correct answer. (a) In India, the drainage networks are not
separate for sewerage and storm water.
Passage-3 (b) Urban local bodies do not have enough
Genome editing is different from genome resources and legislative authority to deal
modification, Genome editing typically involves with the problems of waste management.
finding the part of plant genome that could be (c) Solid waste management should be integrated
changed to render it less vulnerable to disease, yields. with the maintenance of drainage and
Researchers use ‘molecular scissors’ to dissect the sewerage networks.
genome and repair it, which is a process that occurs (d) Bad management of solid waste and sewerage
naturally when plants are under attack from diseases systems by our municipalities is the reason
and can throw up new mutations that enable the for drinking water shortages in our cities.
plant to survive future attacks. This evolutionary Ans. (c) : The logical and rational message of the given
process can effectively be speeded up now that it is passage is that solid waste management should be
possible to examine plant genomes in detail in integrated with drainage and sewerage network
laboratories, and create mechanisms through which management (Sewage System), as solid waste
management non-functioning can result in various water
the relevant genes can be altered very precisely.
borne problems and diseases, face by Urban Citizens.
17. With reference to the above passage, the
following assumptions have been made : Passage-5
1. Genome editing does not required the In part III of the Constitution, which assures people
transfer of genes from one plant to certain fundamental rights, Article 25 proclaims that
another. “all persons are equally entitled to freedom of
2. Through genome editing, the chosen genes conscience and the right freely to profess, practice
can be altered precisely in a manner akin and propagate religion”. What people fail to notice is
to the natural process that helps plants to that this proclamation is prefixed with the words
adapt to the environmental factors. “subject to public order, morality, health and to the
(a) 1 only other provisions of this Part,” which set conditions
(b) 2 only precedent for the legal protection of religious
(c) Both 1 and 2 practices of any community. The closing words of this
(d) Neither 1 nor 2 prefatory rider in Article 25 virtually constitute a
subordination clause placing other fundamental
Ans. (c) : The valid assumption in the context of the rights mentioned in Part III over and above the right
passage is that genome editing does not require the to religious freedom. Among those other fundamental
transfer of genes from one plant to another. It is rights is the right to equality before law and equal
mentioned again in the passage that through genome protection of laws-assured at the outset and
editing, selected genes can be changed in a natural way, elaborated in later articles to mean, inter alia, that
which ultimately helps the plants to adopt to the State shall not deny equal protection of laws to
environmental factors. Hence both the assumptions (1) any person or group of persons on the basis of
& (2) are valid. religion alone.
IAS (Pre) GS IInd Paper (CSAT), 2020 158 YCT
19. What is the most logical inference from the Passage–2
above passage?
India is at once among the fastest growing global
(a) State shall not interfere with the religious economies and home to the largest number of
affairs of the citizens. malnourished children in the world. There are
(b) Religious freedom under the Constitutions is regions where malnutrition is not the exception but
open to State intervention. the norm. And across the country, malnutrition is the
(c) Religious freedom of the citizens is not cause of death for roughly half the 1.3 million
covered under fundamental rights. children who die before their fifth birthday each year.
(d) Religious practices of any community are Even those children who survive suffer permanently
immune to State laws. from the damage that has already been done to their
Ans. (b) : The most logical and rational conclusion bodies and minds from not getting enough of the right
foods and nutrients. Around 44 million children
from the given passage is that the state is free to
under 5 are stunted. That makes it harder for them to
interfere even in religious freedom under the
learn in school and subsequently earn a living as
constitution. That is, one of the main reasons for the
adults. Their lifetime earnings potential is almost a
burning of straw in the state.
quarter less than that of their healthy peers.
Directions for the following 6 (six) items: 21. With reference to the above passage, which of
Read the following five passage and answer the the following is/are the most rational and
items that follow. Your answers to these items should practical implication/implications?
be based on the passages only. 1. India’s Public Distribution System should
Passage–1 be monitored by the Union Government.
2. Girls should be encouraged to delay
Spanish ships in the late 16th century first brought the marriage and first pregnancy.
potato tuber from South America to Europe whereby 3. Mothers should be encouraged to
in the early 19th century, it had become a reliable breastfeed their children immediately after
backup to cereal crops, particularly in the cold, rain- birth.
soaked soils of Ireland. The Irish were soon almost 4. The supply of safe drinking water and
wholly dependent on the potato as their staple food. proper sanitation facilities to all should be
And they were planting primarily one prodigious ensured.
variety, the ‘Lumper’ potato, whose genetic frailty 5. Authorities should ensure the vaccination
would be cruelly exposed by the fungus as prescribed.
‘Phytophthora infestans’. In 1845, spores of the Select the correct answer using the code given
deadly fungus began spreading across the country, below.
(a) 1, 2, 3 and 4
destroying nearly all the Lumpers in its path. The
(b) 2, 3, 4 and 5
resulting famine killed or displaced millions.
(c) 1 only
20. Which one of the following statements best
(d) 3 and 5 only
reflects the critical message of the passage?
Ans. (b) : Statement (1) is incorrect because monitoring
(a) For introducing any foreign plant into a
of the public distribution system does not necessarily
country, the soil and climate conditions of
lead to good nutrition of children. Statement (2) is true
that country should be suitable.
as it is related to the health of mother & Child, delaying
(b) As a staple food of a country, tuber crops like
of marriage and first pregnancy by the girls will
potato cannot replace cereal crops.
improve the health of both. Statement (3) is true
(c) Some of the fungal infections of plants cannot because the mother has immunoglobin which is passed
be prevented or stopped from spreading on to the infant through breast milk. This increases the
across large areas. nutrition and life expectancy of the child. Statement (4)
(d) Relying on a homogeneous food source is not is true. According to UNICEF less than 50% of the
desirable. population has access to clean water. That is why safe
Ans. (d) : The most important message to be conveyed drinking water and sanitation should be ensured for all.
from the passage is that relying solely on a common Statement (5) is true because lock of proper nutrition
food item/source may prove disastrous in future from and association also leads to poor health of children,
the point of view of food security. Hence, option (d) is vaccination drive should be ensured by the authorities
the correct answer. for better survival rate of children.

IAS (Pre) GS IInd Paper (CSAT), 2020 159 YCT


Passage –3 Passage-4
The pulse variety ‘Pusa Arhar 16’ has the potential to In India, authorities always look to store the
be grown in the paddy-growing regions of Punjab, maximum amount of water in reservoirs during the
Haryana and Uttar Pradesh and eventually in all of monsoon season, which is then used for irrigation and
India. Its yield (about 2000 kg/hectare) will be generation of electricity during the summer months.
It is an internationally accepted practice that the
significantly greater than those of the existing
water level of a reservoir should be kept below a
varieties and because its size will be uniform, it will certain level before the onset of monsoon season. This
be amenable to mechanical harvesting, an attractive is so that when monsoon rains come, there is space to
feature for farmers in northern India who currently store the excess rainwater and also so that water can
use this technology for paddy. Most important, Arhar be released in a regulated manner. But the authorities
straw, unlike paddy straw, is green and can be store the maximum amount of water in reservoirs
ploughed back into the soil. In paddy straw, the even before the close of the monsoon, only to ensure
problem is the high silica content, which does not greater electricity generation and irrigation.
allow for easy decomposition. In the case of Arhar, 23. With reference to the above passage, the
following assumptions have been made :
the farmer, even after combine harvesting, just needs
1. High risks involved in holding maximum
to run a rotovator to cut the leftover straw into water in reservoirs are due to our over-
pieces, which can be ploughed back and will dependence on hydropower projects.
decompose very fast. All this is difficult with leftover 2. Storage capacity of dams should not be
paddy stalks that cannot be easily salvaged or fully used before or during monsoon
ploughed back. Farmers, therefore, choose the easiest season.
option of simply burning it. 3. Role of dams in flood control is
22. Which of the following are the most rational underestimated in India.
inferences that can be made from the passage? Which of the above assumption is/are valid?
(a) 1 and 2 only (b) 2 only
1. Farmers’ income will be higher with pulse
(c) 3 only (d) 1, 2 and 3
cultivation than with paddy cultivation.
Ans. (d) : Here all three assumptions are valid.
2. Pulse cultivation causes less pollution as
Statement (1) and (3) are at odds with the international
compared to paddy cultivation. effort to manage dams and water. Statement (2) is also
3. Pulse straw can be used to improve soil valid because the whole paragraph is focused on this
quality., topic.
4. In the context of northern India
Passage–5
agriculture, paddy straw has no usefulness.
5. Mechanized agriculture is the main cause Economic liberalization in India was shaped largely
by the economic problems of the government than by
for stubble burning. the economic priorities of the people or by the long-
Select the correct answer using the code given term development objectives. Thus, there were
below. limitations in conception and design which have been
(a) 2, 3 and 5 subsequently validated by experience. Jobless growth,
(b) 1, 4 and 5 persistent poverty and rising inequality have
mounted as problems since economic liberalization
(c) 2 and 3 only began. And all these years later, four quiet crises
(d) 1 and 4 only confront the economy; agriculture, infrastructure,
Ans. (c) : Statement (1) is false as no where in the industrialization and education as constraints on the
assertion there is a reference to the farmer's country’s future prospects. These problems must be
remuneration for both the crops. It speaks of higher resolved if economic growth has to be sustained and
productivity only in the case of pulses and not paddy. transformed into meaningful development.
Statement (2) is true because pulse cultivation causes 24. Which of the following is/are the most rational
and logical inference/inferences that can be
less pollution as compared to paddy cultivation as it
made from the passage?
does not emit methane. Statement (3) is also true, the
1. It is essential to rethink and redefine the
pulse straw increases the growth and production.
economic role of the State in the quest for
Statement (4) is false because manure of paddy development.
increases crop growth and production and reduces 2. India has not made effective
environmental pollution. Statement (5) is false because implementation of its policies in social
mechanized farming is not a major factor in stubble sectors nor made sufficient investments in
burning. them.
IAS (Pre) GS IInd Paper (CSAT), 2020 160 YCT
Select the correct answer using the code given (a) Only Conclusion-I
below. (b) Only Conclusion-II
(a) 1 only (c) Neither Conclusion-I nor Conclusion-II
(b) 2 only (d) Both Conclusion-I and Conclusion-II
(c) Both 1 and 2 Ans. (a) : By taking LCM of 2 & 3.
(d) Neither 1 nor 2 ∴ LCM = 2 × 3 = 6
Ans. (c) : The most rational and logical inference that So, the numbers which are divisible by 2 & 3 will also
can be drawn from the given passage is that redefining be divisible by 6.
and rethinking the economic role of the state running in 27. Consider the following arrangement that has
the blind pursuit of development has become desirable some missing letters.
not acceptable. Similarly, to say that India has not abab_b_bcd_dcdcded_d
effectively implemented its policies in the social sectors The missing letters which complete the
and has not invested enough in social progress is also arrangement are
not well established from the basic content of the (a) a, b, c, d (b) a, b, d, e
passage. Hence, option (c) is correct answer. (c) a, c, c, e (d) b, c, d, e
25. With reference to the above passage, the Ans. (c) : abababcbcdcdcdcdeded
following assumptions have been made :
So, [a, c, c, e] option (c) would be most appropriate.
1. India’s economy needs to be greatly
28. Let A3BC and DE2F be four-digit number
integrated with global economy so as to
where each letter represents a different digit
create large number of jobs and to sustain
greater than 3. If the sum of the numbers is
its growth momentum.
15902, then what is the difference between the
2. Economic liberalization would cause large
values of A and D?
economic growth which would reduce
(a) 1 (b) 2
poverty and create sufficient employment
in the long run. (c) 3 (d) 4
Why of the above assumptions is/are valid? A 3 B C
(a) 1 only Ans. (c) : + D E 2 F
(b) 2 only 15 9 0 2
(c) Both 1 and 2
As per question,
(d) Neither 1 nor 2
each letter is greater & different than 32.
Ans. (d) : In the context of the given passage, the
preconceived notion is that India's economy needs to be 6=A 3 7=B 8=C
integrated with the global economy, So that its growth Then, + 9=D 5=E 2 4=F
rate can be sustained, resulting in a reduction in poverty
15 9 0 2
and employment over a period of time. The increase
does not match with the complete concept of the Hence, difference between A & D = D – A
paragraph. Thus, both the assumptions (1) and (2) are =9–6=3
not valid. 29.
If in a particular year 12th January is a
Sunday, then which one of the following is
LOGICAL AND ANALITICAL ABILITY correct?
26. Two statements are given followed by two (a) 15th July is a Sunday if the year is a leap year.
conclusions: (b) 15th July is a Sunday if the year is not a leap
Statements year.
All numbers are divisible by 2. (c) 12th July is a Sunday if the year is a leap year.
All numbers are divisible by 3. (d) 12th July is not a Sunday if the year is a leap
Conclusion-I year.
All numbers are divisible by 6. Ans. (c) : To determine 12th July day. Assuming that
Conclusion-II the year is not a leap year.
All numbers are divisible by 4. = 19 + 28 + 31 + 30 + 31 + 30 + 12 = 181
Which of the above conclusions logically ↓ ↓ ↓ ↓ ↓ ↓ ↓ ↓
follow/follow from the two given statements? Jan Feb Mar Apr May Jun July Total days

IAS (Pre) GS IInd Paper (CSAT), 2020 161 YCT


∵ There is 7 days in a week, Conclusion I : All dogs are black (True in case 1 but
Total number of days remaining- false in case 2 and 3)
181 Conclusion II : Some dogs are not black (true in case 2
in remaining days ⇒ 6 (remaining days) and 3 but false in case -1)
7
If 12 January is Sunday the 12 July will be Saturday Hence Neither conclusion -1 nor conclusion II follows
th th

(Where there is no leap year). If there is a leap year the statement.


then, 12th July will be a Sunday. 32. Consider the following sequence of numbers:
51473985726315
GENERAL MENATAL ABILITY 863852243496
30. A man walks down the backside of his house How many odd numbers are followed by the
straight 25 meters, then turns to the right and odd number in the above sequence?
walks 50 meters again; then he turns towards (a) 5 (b) 6
left and again walks 25 meters. If his house (c) 7 (d) 8
faces to the East, what is his direction from the
starting point? Ans. (b) : Arrange in one line
(a) South-East (b) South-West
(c) North-East (d) North-West (5,1) (7,3) (3,9) (5,7) (3,1) (1,5)
Ans. (d) : Required numbers = 6
33. A is 16th from the left end in a row of boys and
V is 18th from the right end. G is 11th from A
towards the right and 3rd from V towards the
right end. How many boys are there in the
row?
(a) 40
(b) 41
Hence, he is in "North-West direction from his starting (c) 42
point. (d) Cannot be determined due to insufficient data
31. Two statements are given followed by two Ans. (b) :
conclusions:
Statements:
All cats are dogs.
All cats are black.
Conclusion–I
All dogs are black.
Conclusion-II
Some dogs are not black. Required Numbers = (16 + 18 + 11 – 4)
Which of the above conclusions logically = [45 – 4]
follow/follow from the two given statements? = 41
disregarding commonly known facts 34. Three statements S1, S2 and S3 are given below
(a) Only Conclusion-I followed by a question:
(b) Only Conclusion-II S1 : C is younger than D, but older than A and B.
(c) Neither Conclusion-I nor Conclusion-II S2 : D is the oldest.
(d) Both Conclusion-I and Conclusion-II S3 : A is older than B
Ans. (c) : Here three venn diagrams are possible: Question:
Who among A, B, C and D is the youngest?
Which one of the following is correct in respect
of the above statements and the question?
(a) S1 alone is sufficient to answer the question
(b) S1 and S2 together are sufficient to answer the
question
(c) S2 and S3 together are sufficient to answer the
question
(d) S1 and S3 together are sufficient to answer the
question

IAS (Pre) GS IInd Paper (CSAT), 2020 162 YCT


Ans. (d) : From S1 - (c) S1 and S2 together are sufficient to answer the
Question, but neither S1 alone nor S2 alone is
C < D sufficient to answer the Question.

C > A (d) S1 and S2 together are not sufficient to answer
C > B  the Question.
Ans. (d) : It's given that n is a unique natural number
From, S2 - D is eldest of all, between 10 & 20. According to S1 n is a prime number
S3 - A > B, So, our number n may be : 11, 13, 17 on 19 According
S1 & S3 to S2, on dividing n by 4 we get a remainder of 1.
So, n can be : 13, 17
B<A<C<D
Either of the two statement alone is not sufficient to
So, S1 & S3 together are sufficient to answer the answer the question. Even if we read both the
question. statements together, we do not get a unique value of n.
35. Two Statements S1 and S2 are given below with Instead, we end up with two possible values of n, i.e. 13
regard to four numbers P, Q, R and S followed & 17. Hence, we can say that even S1 & S2 together are
by a Question : not sufficient to answer the question. Hence, option (d)
is the answer.
S1 : R is greater than P as well as Q.
37. In the sum
S2 : S is not the largest one.
⊗ + 1 ⊗ +5 ⊗ + ⊗ ⊗ + ⊗1 = 1 ⊗ ⊗
Question:
for which digit does the symbol ⊗ stand?
Among four numbers P, Q, R and S, which one (a) 2 (b) 3
is the largest? (c) 4 (d) 5
Which one of the following is correct in respect Ans. (b) : Suppose that ⊗ = x
of the above Statements and the Question?
Then x + (10 + x ) + ( 50 + x ) + (10x + x ) + (10x + 1)
(a) S1 alone is sufficient to answer the Question.
= 100 +10x +x
(b) S2 alone is sufficient to answer the Question.
= 24x +61 = 100 +11x
(c) S1 and S2 together are sufficient to answer the
= 13x = 39
Question, but neither S1 alone nor S2 alone is
39
sufficient to answer the Question. x= =3
13
(d) S1 and S2 together are not sufficient to answer
⊗= 3
the Question.
38. Two Statements S1 and S2 are given below
Ans. (c) : From S1 followed by a Question :
R >P S1 : There are not more than two figures on
and  any page of a 51-page book.
R > Q
S2 : There is at least one figure on every page.
From S2 Question:
S is not the eldest
Are there more than 100 figures in that book?
∴ R will be the eldest. Which one of the following is correct in respect
Hence, S1 & S2 together are sufficient to answer the of the above Statements and the Question?
question. But neither S1 alone nor S2 alone is sufficient (a) Both S1 and S2 are sufficient to answer the
to answer the question. Question, but neither S1 alone nor S2 alone is
sufficient to answer the Question.
36. Two Statements S1 and S2 are given below (b) S1 alone the sufficient to answer the Question.
followed by a Question : (c) S1 and S2 together are not sufficient to answer
S1 : n is a prime number. the Question.
S2 : n leaves a remainder of 1 when divided by 4. (d) S2 alone is sufficient to answer the Question.
Question: Ans. (c) : As per statement 1, there are 2, 1 or 0 figures
on each page. There are 51 pages in the book.
If n is a unique natural number between 10 and
So, maximum possible images in the book
20, then what is n?
51 × 2 = 102
Which one of the following is correct in respect
& the minimum possible images in the book = 0
of the above Statements and the Question? So, statement 1 alone is not sufficient. As per statement
(a) S1 alone is sufficient to answer the Question. 2, there is 1, 2, 3, 4……….∞ images on each page. But
(b) S2 alone is sufficient to answer the Question. we do not know the number of pages in the book.

IAS (Pre) GS IInd Paper (CSAT), 2020 163 YCT


39. A family of two generations consisting of six 4v × 12
members P, Q, R, S, T and U has three males ⇒
25
and three females. There are two married 48v
couples and two unmarried siblings. U is P's =
25
daughter And Q is R's mother-in-law. T is an = 1.92 V km/hr
unmarried male and S is male. Which one of Hence, The average speed of the car for the entire
the following is correct? journey lies between v and 2v km/hr.
(a) R is U's husband. 41. Consider the following statements:
(b) R is S's wife 1. The minimum number of points of
(c) S is unmarried. intersection of a square and a circle is 2.
(d) None of the above 2. The maximum number of points of
Ans. (b) : intersection of a square and a circle is 8.
Which of the above statements is/are correct?
(a) 1 only
(b) 2 only
(c) Both 1 and 2
(d) Neither 1 nor 2
Ans. (b) : From statement (1):-

Hence, R is S's wife.


40. A car travels from a place X to place Y at an
average speed of v km/hr, from Y to X at an
average speed of 2v km/hr, again from X to Y
The minimum number of points of the intersection of a
at an average speed of 3 v km/hr and again
square and a circle is one.
from Y to X at an average speed of 4v km/hr.
then the average speed of the car for the entire Hence statement 1 is not correct.
journey. From statement (2):-
(a) is less than v km/hr/ v km
(b) lies between v and 2v km/hr
(c) lies between 2v and 3v km/hr
(d) lies between 3v and 4v km/hr
Ans. (b) : Let the distance between place X and place Y "The maximum number of points of intersection of a
= d km square and a circle is 8.
Total Distance Hence, Both 1 and 2 are correct.
∵ Average speed =
TotalTime 42. How many different 5-letter words (with or
According to question, without meaning) can be constructed using all
d+d+d+d the letters of the word ‘DELHI’ so that each
Average speed =
d d d d word has to start with D and end with I?
+ + +
v 2v 3v 4v (a) 24 (b) 18
4d (c) 12 (d) 6
d 1 1 1 1  Ans. (d) : The word "D E L H I"
+ + +
v 1 2 3 4  According to question,
4v each word has to start with 'D' and end with 'I'.
 12 + 6 + 4 + 3  ∴ D___I
 
 12  Then,
4 The number of different 5 - letter words (with

25 or without) meaning = 31
12 =3×2×1=6

IAS (Pre) GS IInd Paper (CSAT), 2020 164 YCT


General Mathematics /Numerical Ability 46. A person bought a car and sold it for `
3,00,000. If he incurred a loss of 20%, then how
43. How many integers are there between 1 and much did he spend a buy the car?
100 which have 4 as a digit but are not divisible (a) ` 3,00,000 (b) ` 3,65,000
by 4? (c) ` 3,70,000 (d) ` 3,75,000
(a) 5 (b) 11 Ans. (d) : ∵ SP = 300000 `
(c) 12 (d) 13
and loss % = 20%
Ans. (c) : Such integers, there between 1 and 100 which
have 4 as a digit but are not divisible by 4:- 100
∵ CP = SP ×
14, 34, 41, 42, 43, 45, 46, 47, 49, 54, 74, 94 (100 − L )
∴ The number of such integers = 12 100
44. Let x, y be the volumes m, n be the masses of CP = 30,0000 ×
80
two metallic cubes P and Q respectively. Each
5
side of Q is two times that of P and mass of Q is = 300000 ×
two times that of P. Let u = m/x and v = n/y. 4
Which one of the following is correct? = 75000 × 5
(a) u = 4v (b) u = 2v CP = 375000 `
(c) u = v (d) v = 4u 47. Two Statement S1 and S2 are given below with
Ans. (a) : ∵ Mass of P = m Mass of Q = n regard to two numbers followed by a
Question :
Volume of P = x Volume of Q = y
S1 : Their product is 21
m n
∴ Density of P = Density of Q = S2 : Their sum is 10
x y Question :
m n What are the two numbers?
and u = ν= Which one of the following is correct in respect
x y of the above Statements and the Question?
Let side of P = a (a) S1 alone is sufficient to answer the Question.
∴ Side of Q = 2a (b) S2 alone is sufficient to answer the Question.
Volume of P, (c) S1 and S2 together are sufficient to answer the
X = a3 Question, but neither S1 alone nor S2 alone is
and volume of Q, sufficient to answer the Question.
y = 8a3 (d) S1 and S2 together are not sufficient to answer
According to question, n = 2m the Question.
n = 2m Ans. (c) : Let the numbers a, b
m n From statement (S1):-
u= ν= a × b = 21 ______(i)
x y From statement (S2):-
m 2m a + b = 10______(ii)
u= 3 v= 3
a 8a ∵ ( a − b )2 = ( a + b )2 − 4ab
m 2m
( a − b ) = (10 ) − 4 × 21
2 2
u :ν = 3 : 3
a 8a
( a − b ) = 100 − 84
2
u:v=4:1
u = 4v
( a − b ) = 16
2

45. The average age of a teacher and three students a − b = 4 ______(iii)


is 20 years. If all the three students are of same
age and the difference between the age of the from eq (ii) & (iii)
teacher and each student is 20 years, then what a = 7 and b = 3
is the age of the teacher? Hence,
(a) 25 years (b) 30 years S1 and S2 together are sufficient to answer the
(c) 35 years (d) 45 years question, but neither S1 alone nor S2 alone is sufficient
Ans. (c) : Let the age of teacher = x years then the age to answer the question.
of each student = (x – 20) 48. If you have to straight sticks of length 7.5 feet
According to question, and 3.25 feet, what is the minimum length can
( x − 20 ) + ( x − 20 ) + ( x − 20 ) + x you measure?
= 20
4 (a) 0.05 foot (b) 0.25 foot
4x – 60 = 80 (c) 1 foot (d) 3.25 feet
4x = 140 Ans. (b) : For minimum length,
x = 35 taken H.C.F. 7.5 feet and 3.25 feet.
Hence the age of the teacher = 35 years. On equalizing the denominator-
IAS (Pre) GS IInd Paper (CSAT), 2020 165 YCT
750 Ans. (a) : Let the number of girls = g
⇒ 7.5 = and the number of boys = b
100
325 According to question,
⇒3.25 = 9× g + 8× b
100 Average =
( 750,325 ) H.C.F. (g + b)
H.C.F. =
(100,100 ) L.C.M. ( g + b )( 8.8 ) = 9g + 8b
25 8.8g + 8.8b = 9g + 8b
H.C.F. = 8.8b – 8b = 9g – 8.8g
100
H.C.F. = 0.25 foot 0.8b = 0.2g
Hence, the minimum length you can measure = 0.25 8b = 2g
foot. 4b = g
49. A simple mathematical operation in each b:g=1:4
number of the sequence 14, 18, 20, 24, 30, 32, Again,
.... results in a sequence with respect to prime Overall average marks in Hindi
numbers. Which one of the following is the next 8× g + 7× b
x=
number in the sequence? g+b
(a) 34 (b) 36
8 × 4 + 7 ×1
(c) 38 (d) 40 x=
Ans. (c) : 1+ 4
Sequence : − 14, 18,
32 +7
20, 24, 30, 32 38 x=
↑ +1 ↑ +1 ↑ +1 ↑ +1 ↑ +1 ↑ +1 ↑ +1 5
Prime number : − 13 39
17 19 23 29 31 37 x=
50. One page is torn from a booklet whose pages 5
are numbered in the usual manner starting x = 7.8
from the first page as 1. The sum of the
52. How many zeroes are there at the end of the
numbers o the remaining pages is 195. The torn
following product?
page contains which of the following numbers?
(a) 5, 6 (b) 7, 8 1 × 5 × 10 × 15 × 20 × 25 × 30
(c) 9, 10 (d) 11, 12 × 35 × 40 × 45 × 50 × 55 × 60
(a) 10 (b) 12
Ans. (b) : Let there are n pages in a book. (c) 14 (d) 15
According to question,
1 + 2 + 3 + 4 + 5………n pages= 195 + two pages Ans. (a) : ∵ Multiplying 2nd 5 gives zero.
n ( n + 1) ∴ Find the power of 2 and 5, In 2 and 5 whose power is
= 195 + the sum of two pages less, the same number of zeros will be obtain.
2
n ( n + 1) ⇒1×5×10×15×20×25×30×35×40×45×50×55×60
∵ The sum of first n natural numbers = Power of 5 = 514
2 and Power of 2 = 210
Put n = 20
∵The number of zeros = 10
Then
20 ( 20 + 1) Note:-
= 195 + the sum of two pages. Power of 5
2
10 × 21 = 195 + the sum of two pages 5 →1 
The sum of two pages = 210 – 195 10 → 1 
= 15
∵ The sum of the number of torn pages = 15 15 → 1  Power of 2

∴ The page number of torn pages = 7, 8 20 → 1  10 → 1 
51. Consider the following data: 25 → 2 20 → 2

Average Average
30 → 1  30 → 1  10
marks in marks in ⇒ 5 14
2
35 → 1  40 → 3 
English Hindi 
Girls 9 8 40 → 1  50 → 1 

Boys 8 7  60 → 2 
45 → 1 
Overall average 8.8 x
marks 50 → 2 

What is the value of x in the above table?
. .
55 → 1 
(a) 7 8 (b) 7 6
60 → 1 
(c) 7.4 (d) 7.2
IAS (Pre) GS IInd Paper (CSAT), 2020 166 YCT
53. Let XYZ be a three-digit number, where, According to question,
(X+Y+Z) is not a multiple of 3. Then (XYZ + 1 1
YZX + ZXY) is not divisible by x+y= +
40 24
(a) 3 (b) 9 3+5
(c) 37 (d) (X + Y + Z) ( x + y) =
Ans. (b) : Let a three-digit number = 100a + 10b + c 120
8 1
Which is not multiple of 3. ( x + y) = ⇒ Part
∴ xyz = 100a + 10b + c 120 15
yzx = 100 b + 10c + a Hence, The taken by '(x+y)' to do 100% work = 15 days
zxy = 100 c + 10a + b ∴Time taken by '(x + y)' to do 40% work
then, (xyz + yzx + 2 xy) = 111a + 111b + 111c 15
(xyz + yzx + zxy) = 111 (a + b + c) = × 40
100
(xyz + yzx + zxy) = 3 × 37 (a + b + c) 15
Hence, = ×2
(xyz + yzx + zxy) is not divisible by 9. 5
= 6 days.
54. Let p, q, r and s be natural numbers such that
P – 2016 = q + 2017 = r – 2018 = s + 2019 57. A man takes half time in rowing a certain
Which one of the following is the largest distance downstream than upstream. What is
natural number? the ratio of the speed in still water to the speed
(a) p (b) q of current?
(c) r (d) s (a) 1 : 2 (b) 2 : 1
(c) 1 : 3 (d) 3 : 1
Ans. (c) : Let,
P – 2016 = q + 2017 = r – 2018 = s + 2019 = K Ans. (d) : Let certain distance = d
∴ P – 2016 = K ⇒ P = K + 2016 The speed of man in still water = B
and speed of current = R
q + 2017 = K ⇒ q = K – 2017
According to question,
r – 2018 = K ⇒ r = K + 2018
d = (B – R) t ............(i)
s + 2019 = K ⇒ s = K – 2019
t
Hence, The largest natural number = r and d = (B + R) .........(ii)
55. How many five-digit prime numbers can be 2
n
obtained by using all the digits 1, 2, 3, 4 and 5 from eq (i) & (ii)
without repetition of digits? t
(a) Zero (b) One
(B − R ) t = (B + R )
2
(c) Nine (d) Ten 2B − 2R = B + R
Ans. (a) : Prime numbers : A prime is a number which 2B − B = R + 2R
can only be completely divided by 1 and itself. B = 3R ⇒ B : R = 3 :1
Given digits are 1, 2, 3, 4 and 5 and we are supposed to
make five digit prime numbers. We know that if the 58. How many pairs of natural numbers are there
sum of the digits of a number is divisible by 3, then that such that the difference of whose squares is 63?
number is also divisible by 3. (a) 3 (b) 4
Here, Sum of the given digits = 1 + 2 + 3 + 4 + 5 = 15 (c) 5 (d) 2
(which is divisible by 3) Ans. (a) : Let natural numbers are x, y
So, any number formed using these 5 digits then x 2 − y 2 = 63
Will always be divisible 3.
Hence, no five-digit prime number can be made using
( x + y )( x − y ) = 63
these five-digits (without repetition) for possible values
56. A person X can complete 20% of work in 8 ( x + y )( x − y ) = 63 ×1
days and another person Y can complete 25% or ( x + y )( x − y ) ⇒ 21× 3
of the same work in 6 days. If they work
together, in how many days will 40% of the or ( x + y )( x − y ) ⇒ 9 × 7
work be completed? when ( x + y )( x − y ) = 63 ×1
(a) 6 (b) 8
(c) 10 (d) 12 ∴ x + y = 63
x − y =1
Ans. (a) : ∵ The taken by 'x' to do 100% work
8 2x = 64
= × 100 x = 32and y = 31
20
= 40 days. when ( x + y )( x − y ) = 21× 3
6
Time taken by 'y' to do 100% work = ×100 x + y = 21
25 x−y=3
= 24 days
IAS (Pre) GS IInd Paper (CSAT), 2020 167 YCT
2x = 24 Ans. (c) : ∵ 1 litre = 1 kg
x = 12 ∴ y = 9 1000 cm3 = 1000 g
When ( x + y )( x − y ) = 9 × 7 {∵ 1 litre = 1000 cm3, 1 kg = 1000 g, 1 cm = 10 mm}
x+y=9 1 cm3 = 1g
(10 mm)3 = 1 g
x−y=7
1000 mm3 = 1 g
2x = 16 ∵ 1 g = 1000 mm3
x = 8, y = 1 ∴ 0.1 g = 1000 × 0.1 mm3
3 pairs of natural numbers are possible. 0.1g = 100 mm3
59. Which one of the following will have minimum
change in its value if 5 is added to both 62. A vessel full of water weights 40 kg. If it is one-
numerator and the denominator of the third filled, its weight becomes 20 kg. What is
fractions 2/3, 3/4, 4/5 and 5/6? the weight of the empty vessel?
(a) 2/3 (b) 3/4 (a) 10 kg (b) 15 kg
(c) 4/5 (d) 5/6 (c) 20 kg (d) 25 kg
Ans. (a) : Let the weight of vessel = x
2 2+5 7
Ans. (d) : ⇒ ⇒ and The weight of water= y
3 3+5 8 then x + y = 40 ..........(i)
7 2 21 − 16 5
⇒ − ⇒ ⇒ ⇒ 0.208 1
8 3 24 24 and x + y = 20 ..........(ii)
3
3 3+5 8 Subtracting eqn (ii) from eqn (ii)
= ⇒ ⇒
4 4+5 9 1
y − y = 40 − 20
8 3 32 − 27 5 3
⇒ − ⇒ ⇒ ⇒ 0.138
9 4 36 36 2y
= 20
4 4+5 9 3
= ⇒ ⇒
5 5 + 5 10 y = 30 kg
9 4 27 − 24 3 Putting the value of y in eqn (i)
⇒ − ⇒ = ⇒ 0.100
10 5 30 30 x + 30 = 40
5 5 + 5 10 x = 10 kg
= ⇒ ⇒
6 6+5 11 Hence, the weight of vessel = 10 kg
10 5 60 − 55 5 63. A frog tries to come out of a dried well 4.5 m
= − ⇒ ⇒ ⇒ 0.075
11 6 66 66 deep with slippery walls. Every time the frog
There will be minimum charge in the value obtained by jumps 30 cm, slides down 15 cm. what is the
doing the process in question in the fraction 5/6. number of jumps required for the frog to come
out of the well?
60. A digit n > 3 is divisible by 3 but not divisible
(a) 28 (b) 29
by 6. Which one of the following is divisible (c) 30 (d) 31
by4?
(a) 2n (b) 3n Ans. (b) : ∵ Each Jump of frog = 30 cm
(c) 2n + 4 (d) 3n + 1 and sides every time = 15 cm
Ans. (d) : By option, First situation,
taking option (d)- Complete process for obtained height
Required number = (3x + 1) = Jump – Slide
= 30 – 15
According to question,
= 15 cm
A digit n > 3 is divisible by 3 but not divisible by 6.
Second situation,
∴ n=9 The last complete process for obtained height
then, (3x + 1) = Only Jump
Put x = 9 Because, After reading the top of wall, the frog does not
= (3 × 9 + 1) slide down.
= 28, Which is divisible by 4 but not divisible by ∴ Covered height - in first situation = 450 – 30
6. = 420 cm
Covered height in second situation = 30 cm
61. If 1 litre of water weighs 1 kg, then how many
 420 30 
cubic millimeters of water will weight 0.1 gm? ∴ The total number of jumps =  + 
(a) 1 (b) 10  15 30 
(c) 100 (d) 1000 = 28 + 1 = 29
IAS (Pre) GS IInd Paper (CSAT), 2020 168 YCT
64. A bottle contains 20 litres of liquid A. 4 litres of 66. As a result of 25% hike in the price of rice per
liquid A is taken out of the it and replaced by kg, a person is able to purchase 6 kg less rice
same quantity of liquid B. Again 4 litres of the for ` 1,200. What was the original price of rice
mixture is taken out and replaced by same
per kg?
quantity of liquid B. What is the ratio of
quantity of liquid A to that of liquid B in the (a) ` 30 (b) ` 40
final mixture? (c) ` 50 (d) ` 60
(a) 4 : 1 (b) 5 : 1 a x
(c) 16 : 9 (d) 17 : 8 Ans. (b) : Original Price = ×
n (100 ± x )
Ans. (c) : Remaining quantity of liquid (A) =
n Where a = amount
 R
P 1 −  n = Less/More things
 P x = Increase or decrease percentage
where P = initial quantity
R → Replace quantity 1200 25
= ×
n → number of processes 6 125
2
 4  1
= 20  1 −  = 200 × = 40 `/kg
 20  5
2
 1 1
= 20  1 −  67. What is the greatest length x such that 3 m
 5 2
2 3
4 16 and 8 m are integral multiples of x?
= 20 ×   = 20 × 4
5 25
1 1
64 (a) 1 m (b) 1 m
Remaining quantity of liquid (A) = Litre 2 3
5
64 1 3
The, the quantity of B in mixture = 20 − (c) 1 m (d) 1 m
5 4 4
36 1 3
= litre Ans. (d) : On taking H.C.F. of 3 m, 8 m
5 2 4
64 36 1 7 3 35
Required Ratio = : ∵ 3 = m and8 = m
5 5 2 2 4 4
= 16 : 9 are fraction
65. The average score of a batsman after his 50th H.C.F.of Numerator
∵ H.C.F. of fraction =
innings was 46.4. After 60th innings, his average L.C.M.of Denominator
score increases by 2.6. what was his average
score in the last ten innings? A.C.F.of ( 7,35 )
=
(a) 122 (b) 91 L.C.M.of ( 2, 4 )
(c) 62 (d) 49
7
Ans. (c) : ∵ The average score of batsman after 50 =
innings - 46.4 4
∴The total score of batsman after 50 innings 3
=1 m
= 46.4 × 50 4
= 2320.0 68. Consider the following data :
∵After 60 innings average score = 46.4 + 2.6 Year Birthrate Death rate
= 49.0 1911-1921 48.1 35.5
The total score after 60 innings = 49 × 60 1921-1931 46.4 36.3
= 2940 1931-1941 45.2 31.2
2940 − 2320 1941-1951 39.9 27.4
Then the average score of 10 innings =
10 1951-1961 41.7 22.8
620 1961-1971 41.1 18.9
= = 62
10 1971-1981 37.1 14.8
or For which period was the natural growth rate
60 × 2.6 maximum?
The average score of 10 innings = 46.4 +
10 (a) 1911-1921 (b) 1941-1951
=46.4 + 15.6 = 62.0 (c) 1961-1971 (d) 1971-1981
IAS (Pre) GS IInd Paper (CSAT), 2020 169 YCT
Ans. (d) : 71. In adult population of a city, 40% men and
30% women are married. What is the
∵ Naturalgrowth rate = Birth rate − Death rate
percentage of married adult population if no
The growth rate of 1911 – 1921 = 48.1 – 35.5 man marries more than one woman and no
= 12.6 woman marries more than one man; and there
are no widows and widowers?
The growth rate of 1941 – 1951 = 39.9 – 27.4
= 12.5 1
(a) 33 % (b) 34%
7
The growth rate of 1961 – 1971 = 41.1 – 18 – 9
= 22.2 2
(c) 34 % (d) 35%
The growth rate of 1971 – 1981 = 37.1 – 14.8 7
= 22.3 Ans. (c) : Let total population = 100
Hence, The natural growth rate maximum was the and population of men = x
period 1971 – 1981. then population of women = (100 – x)

69. The recurring decimal representation


1.272727……is equivalent to
(a) 13/11
(b) 14/11
(c) 127/99
(d) 137/99
Ans. (b) : 1.272727.......... = 1.27
= 1 + 0.27
27
= 1+
99
According to question
3
= 1+ 40 30
11 x× = (100 − x ) ×
100 100
14 4x = 300 − 3x
=
11 7x = 300
70. What is the least four-digit number when
300
divided by 3, 4, 5 and 6 leaves as remainder 2 x=
in each case? 7
(a) 1012 40 300 40 120
then married men = x × = × ⇒
(b) 1022 100 7 100 7
(c) 1122 30
(d) 1222 married women = (100 − x ) ×
100
Ans. (b) : On taking L.C.M. 3, 4, 5 and 6.
 300  30
The L.C.M. of (3, 4, 5, 6) = 2 × 2 × 3 × 5 = 60 =  100 − ×
 7  100
∵ The least four digit number = 1000
400 30 120
= × ⇒
1000 Remainder 7 100 7
then  → 40
60 120 120 240
∴ The least four digit number which is completely Total married men and women = 7 + 7 ⇒ 7
divisible by 3, 4, 5 and 6 = 100 + (60 – 40)
240
= 1020 240
Married percentage = 7 = %
∵Leaves remainder 2 in each case, 100 7
Hence, The required number = 1020 + 2 2
= 34 %
= 1022 7
IAS (Pre) GS IInd Paper (CSAT), 2020 170 YCT
72. What is the remainder when 51×27×35×62×75 74. For what value of n, the sum of digits in the
is divided by 100? number (10n + 1) is 2?
(a) 50 (b) 25 (a) For n = 0 only
(c) 5 (d) 1 (b) For any whole number n
51× 27 × 35 × 62 × 75 (c) For any positive integer n only
Ans. (a) :
100 (d) For any real number n
Simplify by 25 (on dividing the numerator and
denominator by 25) Ans. (b) : (10n + 1)

51× 27 × 35 × 62 × 3 option (a) n = 0



4 100 + 1 ⇒ 1 + 1 ⇒ 2 (Sum of digits)
51
⇒ → Remainder = 3 option (b) ∵ n hetCe& mebKÙee nw,
4
n=0
27
⇒ → Remainder = 3
4 100 + 1 = 1 + 1 = 2 (Sum of digits)
35 10 + 1 = 10 + 1 = 2
1
(Sum of digits)
⇒ → Remainder = 3
4 10 + 1 ⇒ 100 + 1 ⇒ 2 (Sum of digits)
2

62 Option (c),
⇒ → Remainder = 2
4 When the sum of the digits on whole numbers
3 is 2 then It will be the same for positive integer numbers
⇒ → Remainder = 3
4 as well.
3× 3× 3× 2 × 3 Option (d),
then
4
162
⇒ → Remainder = 2
4
∴ Required remainder - last quotient × Simplify
= 2 × 25
= 50
73. A sum of ` 2,500 is distributed among X, Y and According to flow chart,
1 3 5 for real numbers,
Z in the ratio : : . What is the difference
2 4 6 The sum of (10n + 1) digits will not be 2.
between the maximum share and the minimum Hence,
share? The option (c) "for any positive integer n only"
(a) ` 300 (b) ` 350 is correct.
(c) ` 400 (d) ` 450
75. In a class, there are three groups A, B and C. If
1 3 5 one student from group A and two students
Ans. (c) : ∵ x : y : z = : :
2 4 6 from group B are shifted to group C, then what
6 9 10 happens to the average weight of the students
x: y:z = : : of the class?
12 12 12
x : y : z = 6 : 9 :10 (a) It increases
The difference between the maximum share and (b) It decreases
minimum share- (c) It remains the same.
(10 − 6 ) (d) No conclusion can be drawn due to
= 2500 × insufficient data.
6 + 9 + 10
Ans. (c) : The average weight of the students of the
4
= 2500 × class will remain the same as the number of students in
25 the class is the same, only the students have been
= 100 × 4 = 400 transferred in groups.

IAS (Pre) GS IInd Paper (CSAT), 2020 171 YCT


76. How many different sums can be formed with 78. The letters from A to Z are numbered from 1
the denominations ` 50, ` 100, ` 200, ` 500 and to 26 respectively. If GHI = 1578 and DEF =
` 2,000 taking at least three denominations at a 912, then what is ABC equal to
time? (a) 492 (b) 468
(a) 16 (b) 15 (c) 262 (d) 246
(c) 14 (d) 10
Ans. (d) : ∵ G H I = 7 8 9 (From Alphabetically order)
Ans. (a) : Required number = 5C3 + 5C4 + 5C5
then G H I = 789 × 2
5! 5! 5!
= + + = 1578
3!× ( 5 − 3) ! 4!× ( 5 − 4 ) ! 5!( 5 − 5 ) ! D E F = 4 5 6 (From Alphabetically order)
 n!  = 456 × 2
∵ n Cr = 
 r!( ) 
n − r ! = 912
Similarly
5! 5! 5!
= + + A B C = 1 2 3 (From Alphabetically order)
3!× 2! 4!× 1! 5!× 0!
= 123 × 2
5 × 4 × 3! 5 × 4! 5!
= + + [∵ 0! = 1 ] ABC = 246
3!× 2 × 1 4!× 1 5!
= 10 + 5 + 1 = 16 79. What is the missing term @ in the following?
77. A shop owner offers the following discount ACPQ : BESU :: MNGI : @
option on an article to a customer: (a) NPJL (b) NOJM
1. Successive discounts of 10% and 20%, and (c) NPIL (d) NPJM
then pay a service tax of 10%
Ans. (d) :Just as,
2. Successive discounts of 20% and 10%, and
then pay a service tax of 10% 1 3 16 17
3. Pay a service tax of 10% first, then A C P Q
successive discounts of 20% and 10% +1 ↓ +2 ↓ +3 ↓ +4 ↓
Which one of the following is correct? B E S U
(a) 1 only is the best option for the customer. 2 5 19 21
(b) 2 only is the best option for the customer. Similarly
(c) 3 only is the best option for the customer 13 14 7 9
(d) All the options are equally good for the M N G I
customer. +1 ↓ +2 ↓ +3 ↓ +4 ↓
Ans. (d) : Let lebelled price = ` 100 N P J M
from statement (I) 14 16 10 13
First discount = 10% = ` 90 80. What is the largest number among the
80 following?
Second discount = 20% = 90 × ⇒ ` 72 −6 −3
100 1 1
(a)   (b)  
90 2 4
Service Tax = 10% = 72 × ⇒ ` 64.8
100 −4 −2
1 1
From statement-(ii) (c)   (d)  
First discount = 20% ⇒ ` 80 3 6
−6
Second discount = 10% ⇒ 80 ×
90 1 1
= ` 72 Ans. (c) :   ⇒ −6 ⇒ 26 ⇒ 64
100  
2 2
90 −3
Service Tax = 10% = 72 × = ` 64.8 1 1
  ⇒ −3 ⇒ 4 = 64
3
100
4 4
From statement-(iii) −4
Service Tax = 10% = ` 90 1 1
  ⇒ −4 ⇒ 3 ⇒ 81
4

80 3 3
First discount = 20% ⇒ 90 × = ` 72 −2
100 1 1
  ⇒ −2 = 6 = 36
2
90  6  6
Second discount = 10% = 72 × = ` 64.8
100 −4
Hence, All the options are equally good for the Hence, The largest number =  1 
customers. 3
IAS (Pre) GS IInd Paper (CSAT), 2020 172 YCT
UNION PUBLIC SERVICE COMMISSION
Civil Services (Preliminary Exam) - 2021
CSAT : PAPER-II
(Chapterwise Analysis with Explanation)
Time : 2 hours Maximum Number : 200
Passage – 2
DECISION MAKINNG ABILITY Our cities are extremely vulnerable to climate
Directions for the following 2 (two) items: change because of large concentrations of populations
and poor infrastructure. Moreover, population densities
Read the following two passages and answer the items are increasing in them but we have not yet developed the
that follow. Your answers to these items should be based systems to address climate change impacts. Our cities
on the passage only. contribute to 65 per cent of the GDP, but there are not
Passage – 1 enough facilities to cater to the needs of the people. It is
The best universities like Harvard and MIT, despite important to address the issues of aire quality, transport,
etc., that are vital to identifying sustainable solutions. We
having the luxury of having some truly excellent teachers need to involve citizens in city planning and create an
on their payroll, are increasingly embracing the "flipped ecosystem that meets the needs of the people.
classroom" format, where students listen to video 2. Which among the following is the most logical
lectures at home, and spend class time applying their and rational inference that can be made from
knowledge, solving problems, discussing examples, etc. the passage given above?
Professors guide that discussion and fill in wherever (a) Our cities need to have well-defined
administrative set-up with sufficient
necessary, explaining those bits that seem to be eluding autonomy.
the students and throwing in advanced ideas that happen (b) Ever increasing population densities is a
to be topical. These universities have made their video hindrance in our efforts to achieve sustainable
lectures available free for anyone in the world. They are development.
also encouraging colleges and universities all over the (c) To maintain and develop our cities we need to
world to integrate these online courses into their own adopt sustainability related interventions.
pedagogy, picking the pieces that are appropriate for (d) Public-Private Partnership mode of
their needs and building a package around them. development is the viable long-term solution
for the infrastructure and sustainability
1. Which one of the following statements best problems of India.
reflects the central idea of the passage given Ans. (c) : The most logical and rational interference
above? statement is option (c), it reflects the need for a
(a) Efficacy of universities would be better in sustainability related intervention to develop the cities.
online mode of conducting classroom tuition Option (a) is not discussed in the passage. Option (b)
as compared to conventional method. cannot be true as the population is being addressed as a
challenge and not a hindrance to achieve sustainable
(b) Availability of higher education can be made development. Option (d) is incorrect as public private
easier and cheaper without diluting the partnership is out of the context.
content. Hence, option (c) is correct.
(c) We need not invest much in infrastructure Directions for the following 4 (four) items :
related to higher education and yet develop Read the following four passage and answer the items
better human and social capital. that follow. Your answers to these items should be based
on the passage only.
(d) Private sector institutions in higher education
Passage – 1
as well as coaching institutes can take India faces a challenging immediate future in energy
advantage of this opportunity and thrive well. and climate policy-making. The problems are multiple. :
Ans. (b) : From all the given statement, the statement sputtering fossil fuel production capabilities; limited
(b) reflects the central idea of the given passage, as the access to electricity and modern cooking fuel for the
availability of quality content worldwide, free of cost. poorest; rising fuel imports in an unstable global energy
context; continued electricity pricing and governance
That is what's happening here. challenges leading to its costly deficits or surplus supply;
Option (c) is incorrect, nothing has been mentioned and not least, growing environmental contestation around
about the investment in infrastructure, and option (d) land, water and air. But all is not bleak: growing energy
incorrect because 'coaching institute or private sectors efficiency programmes; integrated urbanisation and
not mentioned in the passage, and option (a) is also transport policy discussion; inroads to enhancing energy
access and security; and bold renewable energy
inappropriate. initiatives, even if not fully conceptualised, suggest the
Hence, option (b) is correct. promise of transformation.
IAS (Pre) GS IInd Paper (CSAT), 2021 173 YCT
3. Which one of the following statements best Passage – 3
reflects the critical message conveyed by the Policy makers and media have placed the blame for
passage given above? skyrocketing foods prices on a variety of factors,
(a) India's energy decision-making process is including high fuel prices, bad weather in key food
ever more complex and interconnected. producing countries, and the diversion of land to non-
food production. Increased emphasis, however, has been
(b) India's energy and climate policy is heavily placed on a surge in demand for food from the most
tuned to sustainable development goals. populous emerging economies. It seems highly probable
(c) India's energy and climate actions are not that mass consumption in these countries could be well
compatible with its broader social, economic poised to create food crisis.
and environmental goals. 5. With reference to the above passage, the
(d) India's energy decision-making process in following assumptions have been made :
straightforward supply-oriented and ignores 1. Oil producing countries are one of the reasons
the demand side. for high food prices.
Ans. (a) : According to the passage as it talks about the 2. If there is a food crisis in the world in the near
challenges and prospects of energy and climate policy. future, it will be in the emerging economies.
The problems are multiple & the mention of myriad Which of the above assumptions is/are valid?
factors like sputtering fossil fuel production capabilities, (a) 1 only (b) 2 only
unstable global market, etc. illustrates now the energy (c) Both 1 and 2 (d) Neither 1 nor 2
decision making process is complex adn interconnected. Ans. (d) : Statement 1 is incorrect because as the
Hence, option (a) is correct. passage only mentions that high fuel prices are leading
Option (b) is incorrect because it does not talk about the to increasing food prices, it no where explicitly
variety of functions that are involved in energy adn mentioned oil-producing countries being one of the
reasons for high fuel prices. And as the passage
climate policy maker. mentions that food crisis will be caused due to emerging
Passage – 2 economies, tendency of mass consumption. However,
There are reports that some of the antibiotics-sold in the passage only mentions emerging economies as a
the market are fed to poultry and other livestock as causative agent rather than resultant aspect. And it can
growth promoters. Overusing these substances can create not be derived the passage that the food crisis will occur
superbugs, pathogens that are resistant to multiple drugs in Emerging economies. The given option is more
and could be passed along humans. Mindful of that, suitable for inference of the passage rather than
assumption.
some farming companies have stopped using the drugs to
Hence, option (d) is correct.
make chickens gain weight faster. Since Denmark
banned antibiotic growth promoters in the 1990s, the
major pork exporter says it is producing more pigs ––
LOGICAL & ANLYTICAL ABILITY
and the animals get fewer diseases. Directions for the following 3 (three) items :
4. Which one of the following statements best Read the following two passage and answer the items
reflects the critical message conveyed by the that follow. Your answers to these items should be based
passage given above? on the passage only.
(a) People should avoid consuming the products Passage – 1
of animal forming. Medieval merchants risked the hazards of the Silk
(b) Foods of animal origin should be replaced Road to reach the markets of China; Portuguese caravels
with foods of plant origin. is the 15th century sailed beyond the bounds of the
(c) Using antibiotics on animals should be known world, searching less for knowledge than for gold
banned. and spices. Historically, the driver for opening frontiers
(d) Antibiotics should only be used to treat has always been the search for resources. Science and
diseases. curiosity are weaker drivers. The only way to open up
Ans. (d) : In the given passage mentions the ill effects space, whether the space of solar system or interstellar
of antibiotics as growth promoters and how some space is to create an economic engine and that engine is
farming companies and alxo, Denmark-a major port resources extraction.
exporter banned antibiotic growth promoters bu not its 6. Which one of the following statements best
use as a drug to treat diseases. sums up the passage given above?
Hence, option (d) is correct. (a) Wealth generation is the primary motive for
Option (a) and (b) are the out of the context. Option (c) any human endeavour.
ban on total antibiotics is extreme in nature, this does (b) Space, whether space in solar system or
not specify that antibiotics be banned on farming interstellar space, will govern our future
animals alone. economy.
IAS (Pre) GS IInd Paper (CSAT), 2021 174 YCT
(c) Human beings are motivated to explore new 8. With reference to the above passage, the
frontiers principally by economic following assumptions have made :
considerations. 1. Modern democracies rely on the market forces
(d) Wealth generation is based on the risk-taking to enable them to be welfare states.
behaviour of some men.
2. Markets ensure sufficient economic growth
Ans. (c) : According to the passage the correct answer necessary for democracies to be effective.
is statement (c), Human beings are motivated to explore
3. Government programmes are needed for those
new frontiers principally by economic considerations.
because the passage talks about the economic left behind in economic growth.
consideration as principal for exploration like Which of the above asumptions is/are valid?
exploration for gold, spice, and Chinese market. (a) 1 and 3 only (b) 3 only
Rest of all statements are incorrect. (c) 2 and 3 only (d) 1, 2 and 3
Passage – 2 Ans. (b) : Statement 3 is correct, Since markets cannot
"...... most people would agree that telling deliberate ensure social equity, as can be understood from the
lies is wrong, except perhaps in certain special situations various lines of the given passage. Government
where more harm will be done by telling the truth. Even programmers are needed to benefited from the effecient
the most truthful people probably tell a good many more distribution of resources left behind in economic
lies that might be regarded as semantic lies; their use of
growth.
words contains some measure of falsehood, more of less
deliberate." Passage – 2
7. The idea which the first part of the passage In our schools we teach our children all that is there
mentions is to know about physics, maths and history and what-have-
(a) agreement about telling lies. you. But do we teach them about the bitter caste divide
(b) disagreement about telling lies. that plagues the country about the spectre of famine that
(c) disagreement about telling the truth. stalks large parts of our land, about gender sensitivity,
(d) disagreement about the harm in telling the about the possibility of atheism as a choice, etc.? Equally
truth. important, do we teach them to ask questions, or do we
Ans. (a) : In the given passage, the question is asking teach them only to passively receive our wisdom? From
for ideas from the first part of the passage. But the first the cocooned world of school, suddenly, the adolescent
part is missing, and the given passage in the finds himself/herself in the unfettered world of
continuation of the first part. Hence we need to imply university. Here he/she is swept up in a turmoil of ideas,
the idea from the passage the idea of agreement in influences and ideologies. For someone who has been
telling lies while in the second part they disagree in discouraged from asking questions and forming an
truth in every situation. Hence this option is correct.
opinion, this transition can be painful.
Rest of all are incorrect.
9. Which one of the following statements best
Directions for the following 2 (two) items :
reflects the central idea of the passage given
Read the following two passage and answer the items
above?
that follow. Your answers to these items should be based
(a) School curriculum is not compatible with the
on the passages only.
expectations of children and parents.
Passage – 1
(b) Emphasis on academic achievements
Can a democracy avoid being a welfare state for
personality and skills.
long? Why cannot mass welfare be left entirely to the
(c) Preparing the children to be better citizens
markets? There is a built-in tension between markets and
democracy. Markets do not work on a one-person-one- should be the responsibility of the education
vote principle as democracies do. What one gets out of system.
the market place depends on one's endowments, skills, (d) To be a better citizen, the present world order
purchasing power and the forces of demand and supply. demands societal and life-coping skills in
Markets reward individual initiative and skill, and may addition to academic content.
also lift many from the bottom rungs of society, but some Ans. (d) : Clearly demands from a citizen to be
people never get the opportunity to develop slills that equipped with societal and life-coping skill as wall.
markets demand; they are simply too poor and too Apart from academic content a better citizen needs
handicapped; or skill formation takes too long. By various qualities and awareness about his ecosystem
creating jobs, markets may be able to help even unskilled
living with and dare to be free with mind and soul with
people, but capitalism has always witnessed bursts of
unemployment. certain questions need to be asked.

IAS (Pre) GS IInd Paper (CSAT), 2021 175 YCT


Passage – 2 increase the capital cushion of banks and induce them to
In the immediate future, we will see the increasing lend more and boost economic activity. But bas debt
commodification of many new technologies – artificial resolution and recapitalisation are only a part of the
intelligence and robotics, 3D manufacturing, custom solution as they, be themselves, can do very little to rein
made biological and pharmaceutical products, lethat in reckless lending that hat pushed the India banking
autonomous weapons and driverless cars. This will pose system to its current sorry state. Unless there are
systemic reforms that address the problem of
conundrums. The moral question of how a driverless car
unsustainable lending, future credit cycles will continue
will decide between hitting a jaywalker and swerving to stress the banking system.
and damaging the car has often been debated. The
11. Which one of the following statements best
answer is both simple – save the human life – and reflects the most logical, rational and practical
complex. At which angle should the car swerve – just suggestion implied by the passage given above?
enough to save the jaywalker or more than enough? If (a) Lending by the banks should be closely
the driverless car is in Dublin, who would take the monitored and regulated by the Centeal
decision? The lrish Government, or the car's original Government.
code writer in California, or a software programmer in (b) Interest rates should be kept low so as to
Hyderabad to whom maintenance is outsourced? If induce banks to lend more, promote credit
different national jurisdictions have different fine print growth and thereby boost economic activity.
on prioritising a human life, how will it affect insurance (c) Merger of many banks into a few large banks
and investment decisions, including transnational ones? alone is the long-term solution to make them
10. Which of the following statement best reflect viable and prevent their bad performance.
the rational, plausible and practical (d) Indian banking system requires structural
implications that can be derived from the reforms as a long-term solution for bad loans
passage given above? problem.
1. Too much globalization is not in the best Ans. (d) : In the given passage, the author clearly states
interests of any country. the need for the in the lines, "Unless there are systemic
2. Modern technologies are increasingly blurring reforms that address the problem of unsustainable
the economic borders. lending, future credit cycles will continue to stress the
3. Innovation and capital have impinged on the banking system". So option (d) is correct.
domain of the State. Option (b) and (c) are not correct, as they are not
4. Public policy of every country should focus on mentioned anywhere in the passage. Option (a) cannot
developing its won supply chains. be implied as it talks of monitoring by central
5. Geopolitics will have to reconcile to many government, where as that is not mentioned anywhere.
ambiguities and uncertainties. Passage – 4
Select the correct answer using the code given In India, the objective of macroeconomic policy in to
below : enhance the economic welfare of the people, and any one
(a) 1, 4 and 5 only (b) 1, 2, 3 and 4 only wing of such macro policy, monetary or fiscal, cannot
(c) 2, 3 and 5 only (d) 1, 2, 3, 4 and 5 indecently work without active support of another.
12. Which one on of the following statements best
Ans. (c) : In the given passage the lines, "If the driver
reflects the corollary to the passage given
less car is in Dublin, who would take the decision? The
above?
Irish Government, or the car's original code writer in
California, or a software programe in Hyderabad to (a) The central bank cannot work independently
whom maintenance is out sourced?" is valid for of the Government.
statement (2) and (3). (b) Government should regulate financial
Statement (5) can be implied from the given passage, as markets and institutions closely.
there is a need to frame certain laws to clear the (c) Market economy is not compatible with the
socialist policies of the Government.
ambiguities and uncertainties. Statement (1) is extreme
in nature and statement (4) cannot be implied nothing (d) Financial sector reforms are required for
enhancing the economic welfare of the
has been mentioned strengthening the supply chain of a
people.
country.
Ans. (a) : In the passage states that "anyone wing of
Hence, option (c) is correct.
such macro policy, monetary or fiscal, cannot
Passage – 3 independently work without active support of another".
The resolution of bankruptcy cases of Indian banks It is the best corollary that can be drawn from the
under the Insolvency and Bankruptcy Code should help
passage.
bring non-performing assets (NPA) situation under some
contro,. Despite the slow pace of resolutions by the Rest of all options are incorrect because. Regulating
National Company Law Tribunal, the Code can be bodies, social policies by the government and reforms
helpful in cleaning up bank books in future credit cycles. of the financial institutions are not discussed in the
The recapitalisation of public sector banks too can help passage.
IAS (Pre) GS IInd Paper (CSAT), 2021 176 YCT
13. Consider two Statements and a Question : Which one of the following is correct is respect
Statement-1 : Priya is 4 ranks below Seema and of the Statements and the Question?
is 31st from the bottom. (a) Statement-1 alone is sufficient to answer the
Statement-2 : Ena is 2 ranks above Seema and is
Question
37th from the bottom.
Question : What is Seema's rank from the top (b) Statement-2 alone is sufficient to answer the
in the class of 40 students? Question
Which one of the following is correct is respect (c) Both Statements-1 and Statements-2 are
of the Statements and the Question? required to answer the Question
(a) Statement-1 alone is not sufficient to answer (d) Neither Statements-I alone nor Statement-2
the Question
alone nor Statement-2 alone in suffucient to
(b) Statement-2 alone is not sufficient to answer
the Question answer the Question
(c) Either Statements-1 alone or Statement-2 Ans. (a) : ∵ The question is who is the heaviest
alone is sufficient to answer the Question A, B, C, D and E–
(d) Both Statements-1 and Statement-2 are From statement (I) : Each of A and D is heavier than
required to answer the Question :
each of B, E and F, but none of them is the heaviest.
Ans. (c) : Considering the statement-(1) and the
question Then
A>B & D>B
A>E D>E
A>F D>F
∴ C is the heaviest.
Hence,
Statement-1 alone is sufficient to answer the Question.

Directions for the following 4 (four) items :


Read the following four passage and answer the items
that follow. Your answers to these items should be based
on the passages only.
From statements (1) and the question, Passage – 1
Seem's rank from the top is 6th. With respect to what are called denominations of
From statement (2) and the question, religion, if everyone is left to be a judge of his own
religion, there is no such thing as religion that is wrong;
but if they are to be a judge of each other's religion, ther
is no such thing as a religion that is right, ant therefore
all the world is right or all the world is wrong in the
matter of religion.
15. What is the most logical assumption that can be
made from the passage given above?
(a) No man can live without adhering to some
religious denomination.
Seema's Rank from the top is 6th. (b) It is the duty of everyone to propagate one's
Hence, religious denomination.
Either statement-1 alone or statement-2 alone is (c) Religious denominations tend to ignore the
sufficient to answer the question. unity of man.
14. Consider two Statements and a Question: (d) Men do not understand their own religious
Statement-1 : Each of A and D is heavier than denomination.
each of B, E and F, but none of
Ans. (c) : From the given passage the most logical
them is the heaviest.
assumption is option (c), as when we consider our
Statement-2 : A is heavier than D, but is lighter
than C. religion to be the correct one and all others as false, we
Question : Who is the heaviest among A, B, are assumed that all men are not united by any shared
C, D and E? common values or principles.

IAS (Pre) GS IInd Paper (CSAT), 2021 177 YCT


Passage – 2 3. Equal standing of all citizens is an idea that
It is certain that sedition, wars, and contempt of cannot actually be realised even in a
breach of the laws are not so much to be imputed to the democracy.
wickedness of the subjects, as to the bad state of a 4. Right to equality should be incorporated into
dominion, For men are not born fit for citizenship, but
our values and day-to-day political vocabulary.
must be made so. Besides, men's natural passions are
everywhere the same; and if wickedness more prevails, Which of the above assumptions are valid?
and more offences are committed in one commonwealth (a) 1 and 2 only (b) 2 and 3 only
than in another, it is certain that the former has neither (c) 1 and 4 only (d) 3 and 4 only
enough pursued the end of unity, nor framed its laws Ans. (a) : From the given passage the assumption (1)
with sufficient forethought; and that, therefore, it has
states that Equality is most important in democracy
failed in making quite goods its right as a
commonwealth. which encourages people to participate in society with
16. Which among the following is the most logical confidence. From the assumption (2), when inequality
and rational inference that can be made from persist the violation of democratic norms bound to take
the passage given above? place. So we can assume that occurrence of inequality is
(a) Seditions, ware and breach of the laws are detrimental to the survival of democracy assumption (4)
inevitable in every domination. is not correct as there is no discussion on a 'Right to
(b) It is not the people, but the sovereign who is equality', it is spoken of as a default principle.
responsible for all the problems of any Assumption (3) is incorrect, as it says equality as an
domination. idea is impossible even in a democracy. This paragraph
(c) That dominion is the best which pursues the denotes negative sense.
aim of unity and has laws for good Hence, option (a) is correct.
citizenship.
Passage – 4
(d) it is impossible for men to establish a good
dominion. Aristocratic government ruins itself by limiting too
narrowly the circle within which power is confined;
Ans. (c) : According to the passage the appropriable
oligarchic government ruins itself by the incautious
answer is : That dominion is the best which pursues the
aim of unity and has laws for good citizenship; because scramble for immediate wealth. But even democracy
option (a) states that issues of sedition many not be ruins itself by excess of democracy. its basic principle is
inevitable in every dominion so option (a) is incorrect the equal right of all to hold office and determine public
and option (b) is also incorrect because the passage policy. This is, at first glance, a delightful arrangements;
talks about the dominion rather than a sovereign option it becomes disastrous because the people are not properly
(d) cannot be correct as the passage hints at the use of equipped by education to select the best rulers and the
laws with sufficient forethought, here the impossible wisest courses. The people have no understanding and
word is extreme. only repeat what their rulers are pleased to tell them.
Hence, option (c) is the most logical ad rational Such a democracy is tyranny or autocracy. –– Plato
interference that can be drawn.
18. Which one of the following statements best
Passage – 3 reflects the crux of the passage given above?
Inequality violates a basic democratic norm : the
(a) Human societies experiment with different
equal standing of citizens. equality is a relation that
obtains between persons in respect of some fundamental forms of governments.
characteristic that they share in common. Equality is, (b) Any form of government tends to deteriorate
morally speaking, a default principle. Therefore, persons by excess of its basic principle.
should not be discriminated on grounds such as race, (c) Education of all citizens ensures a perfect,
castes, gender, ethnicity, disability, or class. These functional and sustainable democracy.
features of human condition are morally irrelevant. The (d) Having a government is a necessary evil
idea that one should treat persons with respect not only because tyranny is inherent in any form of
because some of these persons possess some special government.
features of talent, for example skilled cricketers, gifted
musicians, or literary giants, but because persons are Ans. (b) : According to the passage two forms of
human beings, is by now part of commonsense morality. government-oligarchies and democracy have been
17. With reference to the above passage, the discussed and it has been shown how they can turn into
following assumptions have been made : tyrannies due to their excesses. So, option (b) is correct.
1. Equality is a prerequisite for people to Option (a) is not reflect the crux of the passage. Option
participate in the multiple transactions of (c) is not correct the reason being that the use of
society from a position of confidence. 'perfect' democracy, is hypothetical in nature. Option (d)
2. Occurrence of inequality is detrimental to the is incorrect, the use of the word 'tyranny' is extreme.
survival of democracy. Hence, option (b) is correct.
IAS (Pre) GS IInd Paper (CSAT), 2021 178 YCT
Passage – 4 Directions for the following 4 (four) items :
Read the following four passage and answer the items
A central message of modern developed economic is
the importance of income growth, be which is meant that follow. Your answers to these items should be based
growth in Gross Domestic Product (GDP). In theory, on the passages only.
rising GDP creates employment and investment Passage-1
opportunities. As incomes grow in a country where the Fig trees (genus Ficus) are considered sacred in
level of GDP was once lows, household, communities,India, East Asia and Africa and are common in
agricultural and urban landscaped where other large trees
and governments are increasingly able to set aside some
are absent. In natural forests, fig three provide food for
funds for the production of things that make for a good
wildlife when other resources are scarce and support a
life. Today GDP has assumed such a significant pace in
the development lexicon, that if someone mentions high density and diversity of freugivores (fruit-eating
"economic growth", we know they means growth in animals). If frugivorous birds and basts continue to visit
GDP. fig trees located in sites with high human disturbance,
19. With reference to the above passage, the sacred fig trees may promote frugivore abundance.
following assumptions have been made : Under favourable microclimate, plenty of seedlings of
other tree species would around fig trees.
1. Rising GDP is essential for a country to be a
developed country. 21. On the basis of the passage given above, the
2. Rising GDP guarantees a reasonable following assumptions have been made :
distribution of income to all households. 1. Fig trees can often be keystone species in
Which of the above assumptions is/are valid? natural forests.
(a) 1 only (b) 2 only 2. Fig trees can grow where other large woody
(c) Both 1 and 2 (d) Neither 1 nor 2 species cannot grow.
Ans. (d) : From the given passage, both statement 1 and 3. Sacred trees can have a role in biodiversity
2 are incorrect. conservation.
Let assume from statement 1. only rising GDP cannot 4. Fig trees have a role in the seed dispersal of
be said as essential criteri2a for a develop country but other species.
income redistribution also plays a critical role as
Which of the above assumptions is/are valid?
mentioned in the passage, from the statement 2 no
where mentions in the passage that rising GDP (a) 1 and 2 only
guarantees reasonable distribution to all households. (b) 3 only
20. A statement followed by Conclusion-I and (c) 2 and 4 only
Conclusion-II is given below. You have to take (d) 1, 3 and 4 only
the Statement to be true even if it seems to be
Ans. (d) : In the given passage Assumption I is valid as
at variance from the commonly known facts. the passage explains the importance of the fig trees
Read all Conclusions and then decide which of
species in natural forests (Ecosystem).
the given Conclusion (s) logically follows/follow
Assumption 3 is relevant from the passage, as the sacred
from the Statement, disregarding the grooves have protection from the humans, resulting in
commonly know facts. conservation of the same.
Statement : Some radios are mobiles, All
Assumption 4 is valid, as given in the passage, "plenty
mobiles are computers. Some
of seedlings of other trees species would grow around
computer are watches. fig tree".
Conclusion-I : Certainly some radios are
watches. Assumption 2 cannot be concluded from the given
Conclusion-II : Certainly some mobiles are passage, as the definition of large woody species is not
watches. specified.
Which one of the following is correct? Hence, option (d) is correct.
(a) Only Conclusion-I Passage – 2
(b) Only Conclusion-II At the heart of agroecology is the idea that
(c) Both Conclusion-I and Conclusion-II agroecosystems should mimic the biodiversity levels and
(d) Neither Conclusion-I nor Conclusion-II functioning of natural ecosystems. Such agricultural
Ans. (d) : mimics, like their natural models, can by productive,
pest-resistant, nutrient conserving, and resilient to shocks
and stresses. In ecosystems there is no 'waste', nutrients
are recycled indefinitely. Agroecology aims at closing
nutrient loops, i.e., returning all nutrients that come out
of the soil back to the soil such as through application of
farmyard manure. It also harnesses natural processes to
Hence, control pests and build soil fertility i.e., through
Neither conclusion-I nor Conclusion-II is intercropping. Agroecological practices include
correct. integrating threes with livestock and crops.
IAS (Pre) GS IInd Paper (CSAT), 2021 179 YCT
22. Consider the following : Passage – 4
1. Cover corps A social and physical environment riddled with
2. Fertigation poverty, inequities, unhygienic and insanitary conditions
3. Hydroponics generates the risk of infectious diseases. Hygiene has
different levels : personal, domestic and community
4. Mixed farming
hygiene. There is no doubt that personal cleanliness
5. Polyculture being down the rate of infections diseases. But the entry
6. Vertical farming of the market into this domain has created a false sense
Which of the above farming practices can be of security that gets conditioned and reinforced by the
compatible with agroecology, as implied by the onslaught of advertisements. Experience in Western
passage? Europe shows that along with personal hygiene, general
(a) 1, 4 and 5 only improvements in environmental conditions and
(b) 2, 3, 4 and 5 only components like clean water, sanitation and food security
have brought down infant/child death/infection rates
(c) 1, 2 3 and 6 only considerably. The obsession with hand hygiene also
(d) 4 and 6 only brings in the persisting influence of the market on
Ans. (a) : From the given passage, the forming personal health, overriding or marginalising the negative
practices of fertigation and Hydroponics are artificial impact on ecology and the emergence of resistant germs.
methods and are not compatible with agroecological 24. On the basis of the passage given above, the
systems mentioned in the passage. So, these two following assumptions have been made :
practices can be eliminated. 1. People who are obsessed with personal
The vertical farming akin to hydroponics is soilless hygiene tend to ignore the community hygiene.
farming and hence not compatible with agroecology. So 2. Emergence of multi-drug resistant germs can
it can also be eliminated. be prevented by personal cleanliness.
So the farming practices compatible with cover 3. Entry of the market in the domain of hygiene
cropping, mixed farming and Polyculture. increases the risk of infectious diseases.
Hence, option (a) is correct. 4. Scientific and micro-level interventions are not
Passage – 3 sufficient to being down the burden of
Computers increasingly deal not just with abstract infectious diseases.
date like credit card details and databases, but also with 5. It is community hygiene implemented through
the real world of physical objects and vulnerable human public health measures that is really effective
bodies. A modern car is a computer on wheels; an in the battle against infectious diseases.
aeroplance is a computer on wings. The arrival of the Which of the above assumptions are valid?
"Internet of Things" will see computers baked into (a) 1 and 2 only (b) 3 and 4 only
everything from road signs and MRI scanners to (c) 4 and 5 only (d) 1, 2 and 4
prosthetics and insulin pumps. There is little evidence Ans. (c) : From the given passage, the Experiences
that these gadgets will be any more trustworthy than their from western Europe have established the direct role of
desktop counterparts. Hackers have already proved that community hygiene through public measures effective
they can take remote control of internet connected cars in reducing infections diseases and death rates. Hence
and pacemakers. statement 5 stands a valid assumption. Scientific and
23. Which on the following statements best reflects Micro-level interventions referring to personal and
the most critical inference that can be made domestic hygiene not sufficient to be bring down the
from the passage given above? burden of diseases can be directly inferred from the
(a) Computers are not completely safe essence of the passage. Thus the statement a stands as a
(b) Companies producing the software do not valid assumption. Rest of all statements are incorrect.
take cyber security seriously Hence, option (c) is correct.
(c) Stringent data security laws are needed
(d) The present trend of communication GENERAL MENTAL APTITUDE
technologies will affect our lives in future
Ans. (a) : According to the last paragraph in the given 25. There are 6 persons arranged in a row.
passage "Hackers have already proved that they can Another person has to shake hands with 3 of
take remote control of internet connected car and pace them so that he should not shake hands with
Makers", it indicate clearly that computers are not two consecutive persons. In how many distinct
completely safe. Hence, option (a) is correct. possible combinations can the handshakes take
Option (b) is not mentioned anywhere in the passage. place?
Option (c) is a final goal we have to target, but that not (a) 3 (b) 4
the inference that can be drawn from the passage. (c) 5 (d) 6

IAS (Pre) GS IInd Paper (CSAT), 2021 180 YCT


Ans. (b) : Let the 6 person A, B, C, D, E, F Ans. (b) : Just as,
and another person X
Possible combinations ⇒ {XA, XC, XE}
{XA, XC, XF}
{XA, XD, XF}
{XB, XD, XF} Similarly
Hence, the no. of possible combinations = 4
26. An amount of money was distributed among A,
B and C in the ratio p : q : r.
Consider the following statements :
1. A gets the maximum share if p is greater than
(q + r).
2. C gets the minimum share if r is less than (p + Hence, "CHEMISTRY" is written as BIDNHTSSX
q). 29. At which one of the following times, do the
Which of the above statements is/are correct? hour hand and the minute hand of the clock
(a) 1 only make an angle of 180º with each other?
(a) At 7:00 hours
(b) 2 only
(b) Between 7:00 hours and 7:05 hours
(c) Both 1 and 2 (c) At 7:05 hours
(d) Neither 1 nor 2 (d) Between 7:05 hours and 7:10 hours
Ans. (a) : ∵ The amount of money was distributed Ans. (d) : The hour hand and the minute hand of the
clock make an angle of 180º with each other between
among A, B and C in the ratio = p : q : r. 7.05 hours and 7:10 hours.
from statement (I), 30. Jay and Vijay spent an equal amount of money
P>q to buy some pens and special pencils of the
P > (q + r ) ⇒ same quality from the same store. If Jay
P>r
bought 3 pens an d5 pencils, and Vijay bought
then, 2 pens an d7 pencils, then which one of the
A gets the maximum share. following is correct?
This statement is correct. (a) A pencil costs more than a pen
From statement (2), (b) The price of a pencil is equal to that of a pen
(c) The price of a pen is two times the price of a
pencil.
(d) The price of a pen is three times the price of a
then, To say "C gets the minimum share" is not pencil.
definitely true. Ans. (c) : According to question,
Hence, only (I) statement is correct 3 Pens + 5 Pencils = 2 Pens + 7 Pencils
27. In the series_b_a_ba_b_abab_aab; fill in the six 3 Pens – 2 Pens = 7 Pencils – 5 Pencils
blanks ( _ ) using one of the following given 1 Pen = 2 Pencils
four choices such that the series follows a Pen 2
=
specific order. Pencil 1
(a) b a b a b a (b) b a a b b a Pen : Pencil = 2 : 1
(c) b b a a b b (d) a b a b a b Hence, the price of a pen is two times the price of a
pencil.
Ans. (d) : _b_a_ba_b_abab_aab 31. Consider two statements and a Question:
a b b a a b/a b b a a b/a b b a a b Statement-I : The last day of the month is a
Hence, Option (d) "ababab" is correct. Wednesday.
28. In a code language 'MATHEMATICS' is Statement-2 : The third Saturday of the month
was the seventeenth day.
written as 'LBSIDNZUHDR'. How is
Question : What day is the fourteenth of the
'CHEMISTRY' written in that code language? given month?
(a) DIDLHRSSX (b) BIDNHTSSX Which of the following is correct in respect of
(c) BIDLHTSSX (d) DGFLIRUQZ the Statements and the Question?
IAS (Pre) GS IInd Paper (CSAT), 2021 181 YCT
(a) Statement-1 alone is sufficient to answer the Statement-I : Some greens are blues.
Question Statement-2 : Some blues are blacks.
(b) Statement-2 alone is sufficient to answer the Conclusion-1 : Some greens are blacks.
Question Conclusion-2 : No green is black.
(c) Both Statement-1 and Statements -2 are Conclusion-3 : All greens are blacks.
required to answer the Question Conclusion-4 : All blacks are greens.
(d) Neither Statements-1 alone nor Statements-2 Which one of the following is correct?
alone is sufficient to answer the Question
(a) Conclusion-1 and Conclusion-2 only
Ans. (b) : Consider statement (2) & Question
(b) Conclusion-2 and Conclusion-2 only
(c) Conclusion-3 and Conclusion-4 only
(d) Neither Conclusion 1 nor 2 nor 3 nor 4
Ans. (d) :
^ 'Date of 14th will be wednesday.
Hence,
Statement-2 alone is the sufficient to answer the
question.
32. Which day is 10th October, 2027? Neither conclusion 1 nor 2 nor 3 nor 4.
(a) Sunday (b) Monday
34. Consider the following Table :
(c) Tuesday (d) Saturday
Playe Runs Balls Runs Balls faced
Ans. (a) : ∵10th of October 2021, is Sunday, r scored in faced in scored in in the
{The paper held on sunday in 10th of October 2021} the First the First the Second
For day of 10th of October 2027, Innings Innings Second Innings
total odd days = (2027 – 2021) + leap year 2024 Innings
=6+1 A 61 99 14 76
=7 B 05 12 50 85
7 Remainder
Again odd days =  →0 C 15 75 20 50
7
D 13 55 12 50
Hence,
10th of October 2027 will be sunday. Who is the fastest run scorer in the Test
OR Match?
For day of 10th of October 2027 (a) A (b) B
2026 + Jan + Feb + Mar + Apr + May + June + July + (c) C (d) D
Aug + Sep + 10 October Ans. (b) :
2000 + 26 + 6* + Jan + Feb + Mar + Apr + May + June Scored total runs
+ July + Aug + Sep + 10 Oct ∵ Run scored per ball =
Total ball faced
26 Quotient
∵ No. of leap years =  → 6.5 → 6 ( leap year ) 61 + 14 75 3
4 Player (A) = = ⇒ = 0.4285
26 + 6 + 31 + 28 + 31 + 30 + 31 + 30 + 31 + 31 + 30 + 10 99 + 76 175 7
7 5 + 50 55
Player (B) = = ⇒ 0.5670
5+ 6+3+ 0+3+ 2 +3+ 2+3+3+ 2 +3 12 + 85 97
7 15 + 20 35 7
Player (C) = = ⇒ = 0.28
35 75 + 50 125 25
Odd days = → Remainder = 0
7 13 + 12 25 5
Player (D) = = ⇒ = 0.2380
Hence, 55 + 50 105 21
10th of October 2021 will be Sunday Player (B) is the fastest run scorer in the test match.
{The number of odd days = Any number is divided by 7 35. In the English alphabet, the first 4 letters are
then obtained remainder is odd days} written in opposite order; and the next 4 letters
33. Consider two Statements and four Conclusions are written in opposite order and so on; and at
given below. You have to take the Statements to the end Y and Z are interchanged. Which will
be true even if they seem to be at variance from be the forth letter to the rights of the 13th
the commonly known facts. Read all Conclusions
letter?
and then decide which of the given Conclusion (s)
logically follows/follow from the Statements, (a) N (b) T
disregarding the commonly known facts. (c) H (d) I
IAS (Pre) GS IInd Paper (CSAT), 2021 182 YCT
Ans. (b) : English alphabet Ans. (c) :
ABCD EFGH IJKL MNOP QRST UVWX YZ
7B 10A 3C ⇒ 7 + 3 = 10 & BAC
On changing according to question-
DCBA HGFE LKJI PONM TSRQ XWVU ZY 3C 9B 6A ⇒ 6 + 3 = 9 & CBA
↓ ↓ 10A 13C ? ⇒ 10 + ? = 13 & AC B
th th
13 4 ⇒ ?=3
T Hence, ? = 3B
Hence, the Letter is T.
39. You are given two identical sequences in two
36. Seven books P, Q, R, S, T, U and V are placed
side by side. R, Q and T have blue covers and rows :
other books have red covers. Only S and U are Sequence –I : 8 4 6 15 52.5 236.5
new books and the rest are old. P, R and S are Sequence-II : 5 A B C D E
law reports; the rest are Gazetteers. Books of
old Gazetteers with blue covers are What is the entry in the place of C for the
(a) Q and R (b) Q and U Sequence-II?
(c) Q and T (d) T and U (a) 2.5 (b) 5
Ans. (c) : (c) 9.375 (d) 32.8125
Book Colour Condition Relation Ans. (c) :
P Red Old Law Reports
Q Blue Old Gazetteers
R Blue Old Law Reports
S Red New Law Reports
T Blue Old Gazetteers
U Red New Gazetteers
V Red Old Gazetteers
Hence, Books of old Gazetters with blue covers are Q 5 × 0.5 = A
and T.
2.5 = A
37. Replace the incorrect term by the correct term
in the given sequence A × 1.5 = B
3, 2, 7, 4, 13, 10, 21, 18, 31, 28, 43, 40 2.5 × 1.5 = B
Where odd terms and even terms follow the
same pattern. 3.75 = B
(a) 0 (b) 1
B × 2.5 = C
(c) 3 (d) 6
3.75 × 2.5 = C
Ans. (a) : Sequence:- 3,2,7,4,13,10,21,18,31,28,43,40
9.375 = C
Odd terms:-
Hence, C = 9.375
40. A person X from a place A and another person
Even terms:- Y from a place B set out at the same time to
walk towards each other. The places are
separated by a distance of 15 km. X walks with
a uniform speed of 1.5 km/hr and Y walks with
a uniform speed of 1 km/hr in the first hour,
with a uniform speed of 1.25 km/hr in the
Hence, Replace the '2' by the '0' in the given seuqnce. second hour and with a uniform speed of 1.5
38. Following is a matrix of certain entries. The km/hr in the third hour and so on.
entries follow a certain trend row-wide. Choose Which of the following is/are correct?
the missing entry (?) accordingly. 1. They take 5 hours to meet.
7B 10A 3C 2. The meet midway between A and B.
3C 9B 6A Select the correct answer using the code given
10A 13C ? below.
(a) 9B (b) 3A (a) 1 only (b) 2 only
(c) 3B (d) 3C (c) Both 1 and 2 (d) Neither 1 nor 2
IAS (Pre) GS IInd Paper (CSAT), 2021 183 YCT
Ans. (c) : 45− + 45+ = 90
∵ ∠OAC = 90
and ∠OAB = 45+
∴∠CAB = 45−
Distance Distance Distance
covered covered covered by ∠CAB < 45
by person by person the both Hence, the woman from his starting point is an angle
'X' 'Y' person less than 45º North of East.
Distance 1.5 km 1 km 2.5 km 42. Images of consonants of the English alphabet
covered (Capitals) are observed in a mirror. What is
by Ist hour the number of images of these which do not
Distance 1.5 km 1.25 km 2.75 km look like their original shapes?
covered (a) 13 (b) 14
by IInd (c) 15 (d) 16
hour Ans. (c) :
Distance 1.5 km 1.50 km 3.00 km
covered
by IIIrd
hour
Distance 1.5 km 1.75 3.25 km
covered
by IVth
hour
Distance 1.5 km 2.00 km 3.50 km
covered
by Vth
hour
Total 7.5 km 7.5 km 15 km
distance
Both 1 and 2 are correct.
41. A woman runs 12 km towards her North, then
6 km towards her South and then 8 km
towards her East. In which direction is she
from her starting point?
(a) An angle less than 45º South of East
(b) An angle less than 45º North of East
(c) An angle more than 45º South of East
(d) An angle more than 45º North of East
Ans. (b) :

Right angle ∆ AOB ∵Total letters = 26


∠OBA + ∠OAB + ∠AOB = 180 Total Vowels =5
∠OBA + ∠OAB + 90 = 180 Total Cosonants = 25 – 5
⇒ ∠OBA + ∠OAB = 90º = 21
No. of consonants which are not charge.
∵ OB > AO = 6 {H, M, T, V, W, X}
∴∠OAB > ∠OBA Hence, the number of images which are charge
∵ ∠OBA + ∠OAB = 90 = 21 – 6 = 15
IAS (Pre) GS IInd Paper (CSAT), 2021 184 YCT
43. A statement followed by Conclusion-I and Ans. (c) :
Conclusion-II is given below. You have to take
the Statements to be true even if it seems to be
at variance from the commonly known facts.
Read all Conclusions and then decide which of
the given Conclusion (s) logically follows/fallow
from the Statement, disregarding the
commonly known facts.
Statement : Some cats are almirahs. Some
almirahs are chairs. All chairs
are tables. ∵ Other dry elements %
Conclusion-I : Certainly some almirahs are = 100 − ( 70 + 16 )  %
 
tables.
Conclusion-II : Some cats may not be chairs. = (100 − 86 ) %
Which one the following is correct? = 14%
(a) Only Conclusion-I then, proteins and other dry elements
(b) Only Conclusion-II ( P% ) = 16% + 14%
(c) Both Conclusion-I and Conclusion-II
(d) Neither Conclusion-I nor Conclusion-II P% = 30%
Ans. (c) : 30
∴ Required Angle = × 360º
100
= 108º
46. Joseph visits the club on every 5th day, Harsh
visits on every 24th day, while Sumit visits on
every 9th day. If all three of them met at the
club on a Sunday, then on which day will all
Both Conclusion-I and Conclusion-II are correct. three of them meet again?
44. The number 3798125P369 is divisible by 7. (a) Monday (b) Wednesday
What is the value of the digit P? (c) Thursday (d) Sunday
(a) 1 (b) 6 Ans. (b) : ∵ Joseph visits the club on every 5th day,
(c) 7 (d) 9 Harsh visits the club on every 24th day.
Ans. (b) : 3798125P369 and Sumit visits the club on every 9th day.
By using divisibility of 7 then,
Required number of day meeting again
⇒ (369 + 981) – (25 P + 37) = the LCM of 5, 24 and 9.
⇒ 1350 – 25 P – 37 ∵ LCM = 360 Days
⇒ 1313 – 25 P 360
by option, Put P = 6 ∵ Odd days = remainder of
7
⇒ 1313 – 256 ⇒ 1057 Odd days = 3
∵ 1057, is completely divisible by 7. Hence, the day of again meeting = Sunday + 3
∴ P=6 = Wednesday.
47. The difference between a 2-digit number and
BASIC NUMERICAL ABILITY the number obtained by interchanging the
45. A pie diagram shows the percentage positions of the digits is 54.
distribution of proteins, water and other dry 1. The sum of the two digits of the number can be
elements in the human body. Given that determined only if the product of the two digits
proteins in the human body. Given that is known.
proteins correspond to 16% and water 2. The difference between the two digits of the
corresponds to 70%. If both proteins and the number can be determined.
other dry elements correspond to p%, then Which of the above statements is/are correct?
what is the central angle of the sector (a) 1 only
representing p on the pie diagram? (b) 2 only
(a) 54º (b) 96º (c) Both 1 and 2
(c) 108º (d) 120º (d) Neither 1 nor 2
IAS (Pre) GS IInd Paper (CSAT), 2021 185 YCT
Ans. (b) : Let two digit number = 10y + x 49. If the price of an article is decreased by 20%
The number obtained by interchanging the positions and them the new price is increased by 25%,
= 10x + y then what is the net change is the price?
(a) 0%
According to question
(b) 5% increase
(10y + x ) − (10x + y ) = 54 (c) 5% decrease
9y − 9x = 54 (d) Cannot be determined due to insufficient data
y−x =6 Ans. (a) : Let the initial price = ` 100
(100 − 20) (100 + 25)
Considering the statements After changing price = 100 × ×
From statement (1) 100 100
"The sum of the two digits of the number can be 80 125
determined only if the product of the two digits is = 100 × ×
100 100
known."
= ` 100
∴ The statement is not correct due to "only if".
(100 − 100)
If both digits of the number is given, we conclude Net change % = × 100%
product of the two digits. 100
From statement (2) = 0%
"The difference between the two digits of the number  20 × 25 
can be determined." Price change % =  −20 + 25 − %
 100 
Hence, Only statement (2) is correct.
48. X said to Y, "At the time of your birth I was = [+5 – 5] % = 0%
twice as old as you are at present." If the 50. When a certain numbers is multiplied by 7, the
present age of X is 42 years, then consider the product entirely comprises ones only (1111....).
following statements. What is the smallest such number?
1. 8 years ago, the age of X was five times the (a) 15713 (b) 15723
age of Y. (c) 15783 (d) 15873
2. After 14 years, the age of X would be two Ans. (d) : By options:-
times the age of Y.
Which of the above statements is/are correct? Option (a) ⇒ 15713 × 7
(a) 1 only (b) 2 only = 109991
(c) Both 1 and 2 (d) Neither 1 nor 2 Option (b) ⇒ 15723 × 7
Ans. (b) : = 110061
Age/person X Y Option (c) ⇒ 15783 × 7
The age, at the time of 2a 0 = 110481
Birth of y Option (d) ⇒ 15873 × 7 = 111111
Present age 3a a Hence, option d is correct for the given condition
Let the present age of y = a ∴ smallest number = 15873
then the age of x at the time of birdh of y = 2a 51. A man completes 7/8 of a job in 21 days. How
and present age of x = 2a + a ⇒ 3a many more days will it take him to finish the
According to question job if quantum of work is further increased by
3a = 42 50%?
a = 14 (a) 24 (b) 21
Consider the statement (I) (c) 18 (d) 15
∵ Present age of x = 42 7
Ans. (d) : ∵ Time taken to do of a work = 21 days.
and present age of y = 14 8
8
∴ Time taken to complete the work = 21 ×
7
= 24 days
∵ 34 ≠ 30 and time taken to do 150% of a work
Statement (1) is not correct = 24 × 150%
Considering the statement (2) 150
= 24 × = 36 days
100
then, the number of more days = 36 – 21 ⇒ 15 days
∵ 56 = 56 52. Using 2, 2, 3, 3, 3 as digits, how many distinct
numbers greater than 30000 can be formed?
Statement (2) is correct (a) 3 (b) 6
Hence, statement (2) is only correct. (c) 9 (d) 12
IAS (Pre) GS IInd Paper (CSAT), 2021 186 YCT
Ans. (b) : Required numbers Ans. (b) :
32233
32323
32332
33223
33232
33322
Hence, the no. of required numbers = 6
53. Consider the following statements: ∵ any diagonal of chess board has 8 consecutive squares.
1. The sum of 5 consecutive integers can be 100.
2. The product of three consecutive natural
numbers can be equal to their sum.
Which of the above statements is/are correct?
(a) 1 only
(b) 2 only
(c) Both 1 and 2 thus, each diagonal less 3 ways.
(d) Neither 1 nor 2 Hence, the number of total ways is 6.
Ans. (c) : For, the sum of 5 consecutive integers can be 57. In an objective type test of 90 questions, 5
marks are allotted for every correct answer
100 :
and 2 marks are deducted for every wrong
18 + 19 + 20 + 21 + 22 answer. After attempting all the 90 questions, a
= 100 student got a total of 387 marks. What is the
For, the product of three consecutive natural numbers number of incorrect responses?
can be equal to their sum : (a) 9 (b) 13
1 ×2 ×3 = 1 + 2 + 3 (c) 27 (d) 43
6=6 Ans. (a) : Let the number of incorrect responses = x
Hence, Both 1 and 2 are correct. ∴ the number of correct answer = (90 – x)
54. A cubical vessel of side 1 m is filled completely According to question,
with water. How many milliliters of water is (90 – x) (+ 5) + (x) (–2) = 387
contained in it (neglect thickness of the vessel)? 450 − 5x − 2x = 387
(a) 1000 (b) 10000
450 − 7x = 387
(c) 100000 (d) 1000000
7x = 450 − 387
Ans. (d) : ∵ 1 m × 1 m × 1 m = 1000 liters 7x = 63
1 m3 = 1000 liters x =9
1 m3 = 1000 × 1000 milliliters Hence, the number of incorrect responses = 9
1 m3 = 1000000 milliliters 58. Consider the following addition problem :
55. There are three points P, Q and R on a straight 3P + 4P + PP + PP = RQ2; where P, Q and R
line such that PQ : QR = 3 : 5. If n is the are different digits.
number of possible values of PQ : PR, then What is the arithmetic mean of all such
what is n equal to ? possible sums?
(a) 1 (b) 2 (a) 102 (b) 120
(c) 3 (d) 4 (c) 202 (d) 220
Ans. (b) : ∵ PQ:QR=3:5 Ans. (c) : 3P + 4P + PP + PP = RQ2
PQ : PR = 3 : 8 For finding the unit digit 2 in given sum
When put P = 3
33 + 43 + 33 + 33 = 142
PQ : PR = 3 : 2 When put P = 8
38 + 48 + 88 + 88 = 262
The arithmetic mean of all such possible sums
The number of possible values n =2 142 + 262
56. On a chess board, in how many different ways =
2
can 6 consecutive squares be chosen on the 404
diagonals along a straight path? =
(a) 4 (b) 6 2
(c) 8 (d) 12 = 202

IAS (Pre) GS IInd Paper (CSAT), 2021 187 YCT


59. Consider the following multiplication problem: z
(PQ) × 3 = RQQ, where P, Q and R are (x + 40) (y – 40) =
2
different digits and R ≠ 0. Z = 2y – 80 ......... (i)
What is the value of (P + R) ÷ Q ? again
(a) 1 (x + 40) = y = (Z – 40)
(b) 2 ∴x + 40 = y ……. (i)
(c) 5 z – 40 = y ……. (iii)
(d) Cannot be determined due to insufficient data From eq. (i) and (ii)
Ans. (b) : It is possible only when z – 40 = y
2y – 80 – 40 = y {∵ z = 2y – 80}
y = 120
Put the value of y in the eq. (iii)
P, Q, R are different digits R ≠ 0
z – 40 = y
∴ P = 8, Q = 5 R = 2 z – 40 = 120
then (P + R) ÷ Q z = 160
(8 + 2) ÷ 5 ⇒10 ÷ 5 = 2 Put the value of y in eq. (ii)
60. P scored 40 marks more than Q in an x + 40 = y
examination. If Q scored 10% less marks than x + 40 = 120
P, then how much did Q score? x = 80
(a) 360 (b) 380
Hence, x + y + z ⇒ 80 + 120 + 160
(c) 400 (d) 420
= ` 360
Ans. (a) :
62. What is the value of 'X' in the sequence 2, 7, 22,
67, 202, X, 1822 ?
(a) 603 (b) 605
(c) 607 (d) 608
Ans. (c) :
Let scored marks by Q = x
then, scored marks by P = (x + 40)
∵ Q scored 10% less marks than P.

Required ³ =
40
× 100%
∴ X = 202 × 3 + 1 and 607 × 3 + 1
( x + 40 ) X = 606 + 1 1821+ 1
40 X = 607 ⇒ 1822
10% = × 100%
( x + 40 ) 63. Half of the villagers of a certain village have
their own houses. One fifth of the villagers
x + 40 = 400 cultivate paddy. One-third of the villagers are
x = 360 literate. Four-fifth of the villagers are under 25
Hence, years of age. Which one of the following
Q scored 360 marks. statements is certainly correct?
61. A person P asks one of his three friends X an to (a) All the villagers who have their own houses
how much money he had. X replied, "If Y gives are literate
me Rs. 40, then Y will have half of as much as (b) Some villagers under 25 years of age are
Z, but if Z gives me Rs. 40, then three of us will literate
have equal amount." What is the total amount (c) Only half of the villagers who cultivate paddy
of money that X, Y and Z have? are literate
(a) Rs. 420 (b) Rs. 360 (d) No villager under 25 years of age has his own
(c) Rs. 300 (d) Rs. 270 house
Ans. (b) : 1 1 1 4
Ans. (b) : ∵ Fractions ⇒ , , ,
2 5 3 5
and denominators of factions = 2, 5, 3, 5
∴ LCM of (2, 5, 3, 5)
LCM = 30
According to question, ∴ Let total villagers = 30

IAS (Pre) GS IInd Paper (CSAT), 2021 188 YCT


Ans. (c) : According to question,
Required number123, 231, 321
126, 261, 612
129, 291, 912
Similarly all numbers obtained
∵ the numbers are without repetition of digits and non
zero.
S = (666) + (995) + …….
Which is divisible 3 and 37.
Considering that, Hence, S is always divisible by 74 and S is always
Any person can include divisible by 9.
One programme or more than two programme on all 66. There are two Classes A and B having 25 and
programme. 30 students respectively. In Class-A the highest
∴ The sum of active person of all programme can not score is 21 and lowest score is 17. In Class-B
be 30. the highest score is 30 and lowest score is 22.
∵ Total villagers = 30 Four students are shifted from Class-A to
Class-B.
∴ The number of villagers whose age under 25 = 24 Consider the following statements :
and the number of villagers who are literate = 10 1. The average score of Class-B will definitely
Hence, decrease.
It is clearly said that, some villagers under 25 2. The average score of Class-A will definitely
years of age are literate. increase.
64. In a group of 120 persons, 80 are Indians and Which of the above statements is/are correct?
rest are foreigners. Further, 70 persons in the (a) 1 only (b) 2 only
group can speak English. The number of (c) Both 1 and 2 (d) Neither 1 nor 2
Indians who can speak English is Ans. (a) :
(a) 20 (b) 30
(c) 30 or less (d) 30 or more
Ans. (d) :

∵ 70 persons in the group can speak English. If a Student who obtained highest score is shifted from
If all 40 foreigners speak English, then 30 Indians will speak class A to class B, then his position in class B will be
English. lower than the minimum marks of class B.
If 39 foreigner speak English, then 31 Indians will speak English. Hence, the average score of class B will definitely
" " " " " " decrease.
" " " " " " 67. A student appeared in 6 papers. The maximum
" " " " " " marks are the same for each paper. His marks
If 1 foreigner speaks English, then 69 Indians will speak English. in these papers are in the proportion of 5 : 6 : 7
If No one foreigner speaks English, then 70 Indians will English : 8 : 9 : 10. Overall he scored 60%. In how
Hence, the number of Indians who can speak English is 30 or many number of papers did he score less than
more. 60% of the maximum marks?
65. Consider all 3-digit numbers (without (a) 2 (b) 3
repetition of digits) obtained using three non- (c) 4 (d) 5
zero digits which are multiples of 3. Let S be Ans. (b) : Let, obtained marks by the student in 6
their sum. papers
Which of the following is/are correct? = 5x, 6x, 7x, 8x, 9x, 10x,
1. S is always divisible by 74. Total marks = 5x + 6x + 7x + 8x + 9x + 10x ⇒ 45x
2. S is always divisible by 9. According to question
Select the correct answer using the code given ∵ 60% = 45x Marks
below 45x
∴ Maximum Marks (100%) = ×100 Marks
(a) 1 only (b) 2 only 60
(c) Both 1 and 2 (d) Neither 1 nor 2 = 75x Marks
IAS (Pre) GS IInd Paper (CSAT), 2021 189 YCT
75x 4
∴ Maximum marks of each paper = The height after fourth Bounce = 76.8 × ⇒ 61.44cm
6 5
75x 60 4
∴ 60% of marks each paper = × The height after fifth Bounce = 61.44 × ⇒ 49.152cm
6 100 5
⇒ 7.5 x ∵ The ball does not bounce further if the previsous
Hence, The number of papers which he scored less than height is less than 50 cm.
60% the maximum marks = 5x, 6x, 7x ∴ The number of times the ball hits the ground before
∴ Required numbers = 3 the ball stop bouncing = 5
68. A bank employee drives 10 km towards South 71. A biology class of high school predicted that a
from her house and turns to her left and drives local population of animals will double in size
another 20 km. She again turns left and drives every 12 years. The population at the beginning
40 km, then she turns to her right and drives of the year 2021 was estimated to be 50
for another 5 km. She again turns to her right animals. If P represents the population after n
and drives another 30 km to reach her bank years, then which one of the following
where she works. What is the shortest distance equations represents the model of the class for
between her bank and her house? the population?
(a) 20 km (b) 25 km (a) P = 12 + 50n (b) P = 50 + 12n
(c) 30 km (d) 35 km (c) P = 50 (2)12n (d) P = 50 (2) n/12
Ans. (b) : n
 R 
Ans. (d) : Amount = Principal 1 + 
 100 
12

2x = x 1 +
R 

 100 
12

2 = 1 +
R 

 100 
1
 R 
212 = 1 + 
 100 
1
 R 
The shortest distance between her bank and her house 1 + =2
12
= 25 km  100 
69. Integers are listed from 700 to 1000. In how Here,
many integers is the sum of the digits 10? n

P = 50 1 +
R 
(a) 6 (b) 7 
(c) 8 (d) 9  100 
1 n
Ans. (d) : P = 50 ( 2 )12 
703, 730, 712, 721
  n   R  12 
1

Rqeuired Numbers 802, 820, 811  P = 50 ( 2 )12 ∵ 1 + =2 


901, 910    100  
  72. In a class, 60% of students are from India and
Hence, Total numbers = 9
50% of the students are girls. If 30% of the
70. A boy plays with a ball and he drops it from a Indian students are girls, then what percentage
height of 1.5 m. Every time the ball hits the of foreign students are boys?
ground, it bounces back to attain a height 4/5th (a) 45% (b) 40%
of the previous height. The ball does not (c) 30% (d) 20%
bounce further if the previous height is less
than 50 cm. What is the number of times the Ans. (d) : Let total number of students = 100 x
ball hits the ground before the ball stops
bouncing?
(a) 4 (b) 5
(c) 6 (d) 7
Ans. (b) : ∵ Initial height = 1.5 m ⇒ 150 cm
4
The height after first Bounce = 150 × ⇒ 120cm
5
4 ⇒ 42x ⇒ 18x
The height after second Bounce = 120 × ⇒ 96cm (∵ 50% Girls of the students)
5
4 8x
The height after third Bounce = 96 × ⇒ 76.8cm Required % = × 100% = 20%
5 40x
IAS (Pre) GS IInd Paper (CSAT), 2021 190 YCT
73. From January 1, 2021, the price of petrol (in 75. Which one of the following statements best
Rupees per liter) on mth day of the year is 80 + reflects the most critical inference that can be
0.1m, where m = 1, 2, 3 ....., 100 and thereafter made from the passage given above?
(a) Low intensity light has more adverse effect
remains constant. On the other hand, the price on the plants as compared to high intensity
of diesel (in Rupees per litre) on nth day of 2021 light
is 69 + 0.15n for any n. On which date in the (b) Light pollution can have a permanent adverse
year 2021 are the prices of these two fuels impact on ecosystem
equal? (c) White light is better for the flowering of
(a) 21st May (b) 20th May plants as compared to the light of other
th
(c) 19 May (d) 18th May colours
(d) Proper intensity of light in an ecosystem is
Ans. (b) : ∵ Price of petrol = 80 + 0.1 m
important not only for plants but for animals
and m = 1, 2, 3 ……..100 and therefore too
remains constant. Ans. (d) : According to the given passage low-intensity
∴ Price of petrol after 100 days light inhibits the flowering of pants. Further, it high
= 80 + 0.1 × 100= 80 + 10 = ` 90 lights that the number of aphids was also significantly
suppressed under the light treatment due to the limited
∵ Price of Diesel = 69 + 0.15 m amount of food available. So, it can be inferred from the
then price of Diesel after 100 days passage that the proper intensity of light in a ecosystem
= 69 + 0.15 × 100 = 69 + 15 = ` 84 is important not only for plants (for flowering) but also
After 100 days, for animals like aphid. Hence, option (d) is correct and
time taken the price of these two fuels equal rest of three are incorrect.
Passage – 2
` ( 90 − 84 ) 6 ×100
= ⇒ [∵ ` 1 = 100 paise] = 40 days. Approximately 80 percent of all flowering plant
15 paise 15 species are pollinated by animals, including birds and
∴ total days = 100 + 40 = 140 days mammals, but the main pollinators are insects.
∴ Required date = 1st of January 2021 + 140 days Pollination is responsible for providing us with a wide
= 20th of May 2021. variety of food, as well as many plant-derived medicines.
74. If 32019 is divided by 10 then what is the At least one-third of the world's agricultural crops
remainder? depend upon pollination. Bees are the most dominant
(a) 1 (b) 3 taxa when it comes to pollination and they are crucial to
(c) 7 (d) 9 more than four hundred crops. Pollination is an essential
service that is the result of intricate relationships between
32019 plants and animals, and the reduction or loss of either
Ans. (c) :
10 affects the survival of both. Effective pollination requires
( )
2 1009 .31
91009.3 ( −1)1009 .3 ( −1× 3) −3 resources, such as refuges of pristine natural vegetation.
= 3 = = = = 76. On the basis of the passage given above, the
10 10 10 10 10
following assumptions have been made :
∴Remainder = 10 – 3 = 7 1. Sustainable production of India's cereal food
grains is impossible without the diversity of
ENGLISH LANGUAGE & pollinating animals
GRAMMAR COMPREHENSION 2. Monoculture of horticultural crops hampers the
survival of insects
Directions for the following 4 (four) items : 3. Pollinators become scarce in cultivated areas
Read the following four passages and answer the items devoid of natural vegetation
that follow. Your answers to these items should be based 4. Diversity in insects induces diversity of plants
on the passages only. Which of the above assumptions is/are valid?
Passage – 1 (a) 1 only (b) 2, 3 and 4 only
Researchers simulated street lighting on artificial (c) 1 and 2 only (d) 3 and 4 only
grassland plots containing pen-aphids, sap-sucking Ans. (d) : According to the above given lines of the
insects, at night. These were exposed to two different passage "Pollination is an essential service that is the
types of light––a white light similar to newer commercial result of an intricate relationship between plant and
LED lights and an amber light similar to sodium street animals, and the reduction or loss of either affects the
lamps. The low intensity amber light was shown to survival of both. Effective Pollination requires
inhibit, rather than induce, flowering in a wild plant of resources, such as refuges of pristine natural
the pea family which is a sources of food for the pea- vegetation", it can be assumed that the pollinators
aphids in grasslands. The number of aphids was also become scarce in cultivated areas devoid of natural
significantly suppressed under the light treatment due to vegetation and diversity in insects induced diversity of
the limited amount of foods available. plants. Hence, option (d) is correct.
IAS (Pre) GS IInd Paper (CSAT), 2021 191 YCT
Passage – 3 Ans. (c) : According to the passage the Bisphenal A
A study conducted on the impacts of climate change (BPA). May alter embryonic development in vivo from
over the Cauvery basin of Tamil Nadu using regional the line "mouse embryonic stem cells upon treatment
climate models showed an increasing trend for maximum with BPA, a compound known to cause heart diseases,
and minimum temperatures, and a decrease in the diabetes, and developmental abnormalities in humans".
number of rainy days. These climatic shifts will have an So it can be clearly assumed statement 1 is correct.
impact on the hydrological cycles in the region, lead to Statement 3 is correct because from the passage that
more run-off and less recharge, and effect the embryonic stem cells could serve as a model to evaluate
groundwater tables. Further, there has been and increase the physiological effect of environmental pollutants.
in the frequency of droughts in the State. This has driven Hence, option (c) is correct.
farmers to increase dependency on groundwater Direction for the following 4 (Four) item :
resources to secure their crops.
Read the Following four passages and answer the
77. Which one of the following statements best items that follow. Your answers to these items should be
reflects the crux of the passage given above? based on the passages only.
(a) Development of regional climate models Passage – 1
helps in choosing climate-smart agricultural Nothing can exist in a natural state which can be
practices called good or bad by common assent, since every man
(b) Heavy dependence on groundwater resources who is in a natural state consults only his own advantage,
can be reduced by adopting dry-land cropping and determines what is good or bad according to his own
systems fancy and insofar as he has regard for his own advantage
(c) Climate changes increases the criticality of alone, and holds himself responsible to no one save
water resources while simultaneously himself by any law; and therefore sin cannot be
threatening it conceived in a natural state, but only in a civil state,
(d) Climate changes cause the farmers to adopt which is decreed by common consent what is good or
unsustainable livelihoods and risky coping bad, and each one holds himself responsible to the state.
strategies 79. Which one of the following statement best
Ans. (c) : The main point of the passage from the given reflects the central idea of the passage given
lines, "These climatic shifts will have an impact on the above?
hydrological cycles in the region, lead to more run-off (a) The conceptions of what is right or wrong
and less recharge, and affect the ground water tables." exist due to the formation of a state.
Can be stated that the climate changes increase the
criticality of water resources while simultaneously (b) Unless a ruling authority decides as to what is
threatening it is the main crux of the passage. So, option right or wrong, no man would be morally
(c) is correct and rest of all three are incorrect. right.
Passage–4 (c) Man is inherently immoral and selfish in a
Researchers were able to use stem cells to gauge the natural state.
neurotoxic effects of the environmental pollutant Bisphenol (d) The idea of what is right or wrong is
A (BPA). They used a combination of biochemical and cell- necessary for the survival of human species.
based assays to examine the gene expression profile during Ans. (a) : The conceptions of what is Right or what is
the differentiation of mouse embryonic stem cells upon wrong exist due to the formation of a state, is justified
treatment with BPA, a compound known to cause heart from the lines, "sin cannot be conceived is a natural
diseases, diabetes, and developmental abnormalities in state, but only a civil state, which is decreed by
humans. They were able to detect and measure BPA common consent what is good or bad, and each one
toxicity towards the proper specification of primary germ holds himself responsible to the sate". So option (a) is
layers, such as endoderm and ectoderm, and the
right. Reset of all are incorrect.
establishment of neural progenitor cells.
78. On the basis of the passage given above, the 80. Which one of the following habits is found
following assumptions have been made: more often in good people ?
1. BPA may alter embryonic developments in (a) Mixing up the true and false
vivo. (b) Intentional mixing up of truth with the false
2. Biochemical and cell-based assays are useful (c) Falsification of facts
in finding out treatments for pollution-induced (d) Complete concealment of truth
diseases. Ans. (b) : According to the passage the most
3. Embryonic stem cells could serve as a model appropriate answer will be option (b). The second half
to evaluate the physiological effects of of the passage mentions that good people often tell lies
environmental pollutants. which contains some kind of truth. Hence there is an
Which of the above assumptions are valid? intentional mix up of truth and false. Another options
(a) 1 and 2 only (b) 2 and 3 only are inappropriate.
(c) 1 and 3 only (d) 1, 2 and 3 Hence, option (b) is correct.

IAS (Pre) GS IInd Paper (CSAT), 2021 192 YCT


UNION PUBLIC SERVICE COMMISSION
Civil Services (Preliminary Exam) - 2022
CSAT : PAPER-II
(Analysis with Explanation)
Time : 2 hours Maximum Number : 200
Directions for the following 3 (three) items 2. Consider the following statements :
Read the following two passages and answer Developed countries can support developing
the items that follow the passages. Your countries transition sustainable human
answers tothese items should be based on the development by
passages only. 1. Making clean energy sources available low
Passage -1 cost
The main threat to maintaining progress in 2. Providing loans for improving their public
human development comes from the transport at nominal interest rates
increasingly evident unsustainability of 3. Encouraging them to change their
production and consumption patterns. Current production and consumption patterns
production models rely heavily on fossil fuels. Which of the statement given above is/are
We now know that this is unsustainable correct
because the resources are finite. The close link (a) 1 only (b) 1 and 2 only
between economic growth and greenhouse gas (c) 2 and 3 only (d) 1, 2 and 3
emissions needs to served for human Ans. (a) : In the passage, the author has clearly
development to become truly sustainable. Some mentioned that developing countries have to face high
development countries have begun to alleviate prices and low availability of refined energy. Therefore,
the worst effects by expanding recycling and developed countries can lead developing countries
inverting in public transport and towards sustainable human development by producing
infrastructure. But most developing countries refined energy sources at low cost. Hence statement 1 is
are hampered by the high costs and low correct.
availability of clean energy sources. Developed Passage-2
countries need to support developing countries'
Unless the forces and tendencies which are
transition to sustainable human development.
responsible for destroying the country's
1. Unsustainability in production pattern is due to environment are checked in the near future
which of the following? and afforestation of denuded areas is taken up
1. Heavy dependence on fossil fuels on a massive scale, the harshness of the climatic
2. Limited availability of resources conditions and soil erosion by wing by wind
and water will increase to such an extent that
3. Expansion of recycling
agriculture, which is the mainstay of our
Select the correct answer using the code given people, will gradually become impossible. The
below: desert countries of the world and our own
(a) 1 and 2 only (b) 2 only desert areas in Rajasthan are a grim reminder
(c) 1 and 3 only (d) 1, 2 and 3 of the consequences of large-scale
Ans. (a) : As given in the passage, the present deforestation. Pockets of deserts-like landscape
production model is heavily dependent on fossil fuels. are now appearing in other parts of the country
Along with this, the matter of limited resources has also including the Sutlaj-Ganga Plains and the
been said in the passage. Both of these are said to be Deccan Plateau. Where only a few decades
unsustainable in the nature of production. Hence back there used to be lush green forests with
statement 1 and 2 are true. At the same time, the spread perennial streams and springs, there is only
of recycling is not related to the unsustainable nature brown earth, bare of vegetation without any
but is related to the sustainable nature. Hence statement water in the streams and springs except in the
3 is false. rainy season.

IAS (Pre) GS IInd Paper (CSAT), 2022 193 YCT


3. According to the passage given above (a) 100 m (b) 90 m
deforestation and denudation will ultimately (c) 60 m (d) 50 m
lead to which of the following: Ans. (a) : According to the question,
1. Depletion of soil resource
2. Shortage of land for the common man
3. Lack of water for cultivation
Select the correct answer using the code given
below:
(a) 1 and 2 only (b) 2 and 3 only
(c) 1 and 3 only (d) 1, 2 and 3 Distance between X and Y = 100 meter
Ans. (c) : According to the passage given above, 7. Which date of June, 2099 among the following
deforestation and denudation will ultimately lead to– is Sunday?
– Depletion of soil resource (a) 4 (b) 5
– Lack of water for cultivation. (c) 6 (d) 7
4. What is the value of X in the sequence 20, 10, Ans. (d) : Number of odd days in 2098
10, 15, 30, 75, X ? = 2000 + (24 leap year) + 74 (Normal year)
(a) 105 (b) 120 = 0 (odd days) + 48 (odd days) + 74 (odd days)
(c) 150 (d) 225 = 122 (odd days)
= 3 (odd days) -------(i)
Ans. (d) : The given series is as follows–
In June 2099
= January + February + March + April + May +
June
X = 225 = 3 + 0 + 3 + 2 + 3 + June
5. An identity Card has the number ABCDEFG = 4 (odd days) + June-------(ii)
not necessarily in that order, where each letter Since, June 2099 will be a Sunday
represents a distinct digit (1,2,4,5,7,8,9 only). = 7 + 7 = 14 = 0 (odd days)
The number is divisible by 9. After deleting the So, 7th June 2099 will be Sunday.
first digit form the right, the resulting number 8. A bill for ` 1840 is paid is the denominations of
is divisible by 6. After deleting two digits from ` 50, ` 20 and ` 10 notes, 50 notes in all are
the right of original number, the resulting used. Consider the following statements :
number is divisible by 5. After deleting three 1. 25 notes of ` 50 are used and the remaining
digits from the right of original number, the are in the denominations of ` 20 and ` 10
resulting number is divisible by 4. After 2. 35 notes of ` 20 are used and the remaining
deleting four digits from the right of original
are in the denominations of ` 50 and ` 10
number, the resulting number is divisible by 3.
3. 20 notes of ` 10 are used and the remaining
After deleting five digits from the right of
original number, the resulting number is are in the denominations of ` 50 and ` 20
divisible by 2. Which of the following is a (a) 1 and 2 only (b) 2 and 3 only
possible value for the sum of the middle three (c) 1 and 3 only (d) 1, 2 and 3
digits of the numbers? Ans. (d) : According to the Ist statement received
(a) 8 (b) 9 formula from `50 = 50 × 25 = `1250
(c) 11 (d) 12 Remaining money = 1840 – 1250 = `590
Ans. (a) : Arranged, according to the question If remaining 25 note of `20, then received money = 25
Number = 7412589 × 20 = 500
Required value = 1 + 2 + 5 = 8 So, `1840 is not possible to received the bill.
According to the 2nd statement–
6. Two friends X and Y start running and they
run together for 50 m in the same direction and Received money from `20 = 20 × 35 = `700
reach a point. X turns right and runs 60 m, Remaining money = 1840 – 700 = `1140
while Y turns left and runs 40 m. Then X turns If remaining 15 note of `50 then received money
left and runs 50 m and stops, while Y turns = 15 × 50 = `750
right and runs 50 m and then stops. How far So, also this is not possible to received the bill.
are the two friends from each other now? And according to the 3rd statement–
IAS (Pre) GS IInd Paper (CSAT), 2022 194 YCT
Received money from `10 = 10 × 20 = `200 guarantee of specific preparation and technical
Remaining money = 1840 – 200 = `1640 competence-we do not ask for the handsomest
If remaining 30 note of `50, then received money = 30 physician, or the most eloquent one : well then,
× 50 = `1500. when the whole state is ill should we not look
This condition is not possible. for the service and guidance of the wisest and
the best?"
Hence, statement I, II and III are not true.
11. Which one of the following statements best
9. Which number amongst 240, 321, 418 and 812 is reflects the message of the author of the
the smallest? passage ?
(a) 240 (b) 321 (a) We assume that in a democracy any politician
(c) 418 (d) 812 is qualified to administer a State
Ans. (b) : According to the question, (b) Politicians should be selected from those
418 = 236 trained in administration
812 = 236 (c) We need to devise a method of barring
Let x = 240 incompetence from public office
log10x = 40 log102 (d) As voters select their administrators, the
eligibility of politicians to administer a State
= 40×0.3010 = 12.04
cannot be questioned
Let y = 321
Ans. (b) : Through the various examples and the
log10y = 21 log103
question asked by the author at the end reveals the
= 21×0.477 = 10.017 authors message, which conveys the message that
So, the smallest number = 321 politicians should be chosen from amongst those
10. The digit 1 to 9 are arranged in three rows in trained in administration.
such a way that each row contains three digits, Passage -2
and the number formed in the second for in The poverty line is quite unsatisfactory when it
twice the number formed in the first row; and comes to grasping the extent of poverty in
the number formed in the third row in thrice Indian. It is not only because of its extremely
number formed in the first row. Repetitions of narrow definition of 'who is poor' and the
digits is not allowed. If only three of the four debatable methodology used to count the poor,
digits 2, 3, 7 and 9 are allowed to used in the but also because of a more fundamental
first row, how many such combination are assumption underlying it. It exclusively relies
possible to be arranged in the three rows ? in the notion of poverty as insufficient income
(a) 4 (b) 3 or insufficient purchasing power. One can
(c) 2 (d) 1 better categorize it by calling it income poverty.
It poverty is ultimately about deprivations
Ans. (c) : Numbers arranged in the first row cannot be
affecting human well-being, then income
started with 9 or 7 because if they are doubled or
poverty is only one aspect of it. Poverty of a
tripled, they will become four digits. life, in our view, lies not merely in the
Unique digit is to be taken each time while forming the impoverished state in which the person actually
number, there should be no repetition. lives, but also in the lack of real opportunity
So we can take 273 and 327 in the first row. given by social constraints as well as personal
So, total 2 combinations are possible. circumstances- to choose other types of living.
Directions for the following 3 (three) items: Even the relevance of low incomes, meagre
possessions, and other aspects of what are
Read the following two passages and answer
standardly seen as economic poverty relate
the items that follow the passages. Your
ultimately to their role in curtailing
answers tothese items should be based on the capabilities, i.e. their role in severely restricting
passages only. the choices people have to lead variable and
Passage -1 valued lives.
"In simple matters like shoe-making, we think 12. Why is the methodology adopted in India to
only a specially trained person will serve our count the 'poor' debatable?
purpose, but in politics, we presume that (a) There is some confusion regarding what
everyone who knows how to get votes knows should constitute the poverty line'
how to administer a State. When we are ill, we (b) There are wide diversities in the condition of
call for a trained physician, whose degree is a the rural and urban poor

IAS (Pre) GS IInd Paper (CSAT), 2022 195 YCT


(c) There is no uniform global standard form 16. If the order of the letters in the English
measuring income poverty alphabet is reverses and each letter represents
(d) It is based on the proposition of poverty as the letter represents the letter whose position is
meagre income of buying capacity occupies, then which one of the following
Ans. (d) : It is clear from the passage that the reason for represents 'LUCKNOW'?
the controversial methodology of calculation of 'poor' (a) OGXPMLD
adopted in India is based on presenting poverty as (b) OGXQMLE
insufficient income or insufficient purchasing power. (c) OFXPMLE
13. Why is income poverty only one measure of (d) OFXPMLD
counting the 'poor'?
Ans. (d) : Here opposite letter–
(a) It talks of only one kind of deprivation
ignoring all others LUCKNOW→OFXPMLD
(b) Other deprivations in a human life have 17. In a tournament of Chess having 150 entrants,
nothing to do with lack of purchasing power a player is eliminated whenever he loses a
(c) Incomes poverty is not a permanent match. It is given that no match results in a
condition, it changes from time to time tie/draw. How many matches are played in the
(d) Income poverty restricts human choices only entire tournament ?
at a point of time (a) 151 (b) 150
Ans. (a) : According to the passage, income poverty is (c) 149 (d) 148
only one measure of counting the poor because it talks Ans. (c) : Only one person won the chess tournament.
of only one kind of deprivation and ignoring all others. So total 149 games were played.
14. What does the author mean by 'poverty of a 18. How many 3-digit natural numbers (without
life'? repetition of digits) are there such that each
(a) All deprivations in a human life which stem digit is odd and the number is divisible by 5 ?
not only from lack of income but lack of real
(a) 8 (b) 12
opportunities
(c) 16 (d) 24
(b) Impoverished state of poor people in rural and
urban areas Ans. (b) : The number is divisible by 5. So 5 will be the
(c) Missed opportunities in diverse personal units place. 1, 3, 7, 9 in place of hundred is a possibility.
circumstances The probability that any three out of 1, 3, 7, 9 will
(d) Material as well as non-material deprivations received in tenth place.
in a human life which restrict human choices So, total number = 4 × 3 × 1 = 12
permanently 19. Consider the Question and two statements
Ans. (a) : By 'Poverty of a life' the author means that all given below:
deprivations in a human life, which stem not only from Question : Is x an integer?
lack of income but lack of real opportunities. Statement-1 :x/3 is not an integer
15. X and Y run a 3 km race along a circular Statement-2 : 3x is an integer.
course of length 300m. Their speeds are in the Which one of the following is correct in respect
ratio 3:2. If they start together in, the same of the Question and the Statement
direction, how many times would the first one
(a) Statement-1 alone is sufficient to answer the
pass the other (the start-off if not counted as
Question
passing)?
(a) 2 (b) 3 (b) Statement-2 alone is sufficient of answer the
(c) 4 (d) 5 Question
(c) Both Statement-1 and Statement-2 are
Ans. (b) : Number of rounds made by each
sufficient to answer the Question
3000
= = 10 (d) Both Statement-1 and Statement-2 are not
300 sufficient to answer the Question
In the same time x takes 3 rounds, y takes 2 rounds Ans. (d) : 1st Statement
When they both meet, then number of rounds made by
x
them = 3 : 2 = Fraction
6:4 3
9:8 x = 3 × (Fraction)
So, hence such a situation is possible three times in So x is a fraction number
total. 2nd statement

IAS (Pre) GS IInd Paper (CSAT), 2022 196 YCT


3x = Integer 21. Based in the above passage, the following
x = 1/3 assumptions have been made:
1 1. Development of agricultural technology is
3 × = Integer confined to developed countries
3
2. Agricultural technology is not adapted in
So x = 1/3 is not an integer.
developing countries
Hence, statement 1 and statement 2 both are
Which of the above assumption is/are valid?
insufficient.
(a) 1 only
20. The increase in the price of a certain item was (b) 2 only
25%. Then the price was decreased by 20% (c) Both 1 and 2
and then again increased by 10%. What is the
(d) Neither 1 nor 2
resultant increase in the price ?
Ans. (d) : It is nowhere mentioned in the passage that
(a) 5% (b) 10%
agricultural technology development is limited to
(c) 12.5% (d) 15% developed countries only. Nor is it said that agricultural
Ans. (b) : According to the question, let the initial price technology has not been adopted in developing
of the item. countries. Therefore, neither statement (1) nor statement
125 80 110 (2) is valid.
Final price = 100 × × ×
100 100 100 22. Based on the above passage, the following
= 110 assumptions have been made:
Resultant increase % = 110 – 100 = 10% 1. Poor countries need to bring about change
in their existing farming techniques
Directions for the following 3 (three) items:
2. Developed countries have better
Read the following two passages and answer
infrastructure and they waste less food
the items that follow the passages. Your
Which of the above assumption is/are valid?
answers tothese items should be based on the
(a) 1 only (b) 2 only
passages only.
(c) Both 1 and 2 (d) Neither 1 nor 2
Passage -1
Ans. (a) : Based on the above passage the following
In some places in the world, the productivity of assumptions have been made–
staples such as rice and wheat has reached a – Poor countries need to bring about change in their
plateau. Neither new strains nor fancy existing farming techniques.
agrochemicals are raising the yields, Nor is – It is not mentioned anywhere in the passage that
there much unfarmed land left that is suitable developed countries have better infrastructure and they
to be brought under the plough. If global waste less food. Therefore, only assumption 1 is valid.
temperature continues to rise, some places will
23. Based on the above passage, the following
become unsuitable for farming. Application of assumption have been made:
technology can help overcame these problems. 1. Growing enough food for future
Agricultural technology is changing fast. Much generations will be a challenge
of this change is brought about by affluent 2. Corporate farming is a viable option for
farmer in the West/Americans. Techniques food security in poor countries
developed in the West are being adapted in Which of the above assumptions is/are valid
some place to make tropical crops more (a) 1 only
productive. Technology is of little use of it is (b) 2 only
not adapted. In the developing world, that (c) Both 1 and 2
applies as much to existing farming techniques (d) Neither 1 nor 2
as it does to the latest advances in genetic Ans. (d) : Based on the above passage, both of the
modification. Extending to the smallholders assumptions i.e. neither 1 nor 2 is valid. Therefore,
and subsistence farmers of Africa and Asia the option (d) is the correct answer.
best of today's agricultural practices, in such 24. The letter A, B, C, D and E are arranged is
simple matters as how much fertilizers to apply such a way that there are exactly two letters
and when, would lead to a greatly increased between A and E. How many such
availability of food roads and storage facilities, arrangements are possible ?
at allow for the carriage of surpluses to (a) 12 (b) 18
markers and reduce wastage. (c) 24 (d) 36

IAS (Pre) GS IInd Paper (CSAT), 2022 197 YCT


Ans. (c) : According to the question, 28. A has some coins. He gives half of the coins and
A –– –– E –– =6 2 more a B. B gives half of the coins and 2 more
–– A –– –– E =6 to C. C gives half of the coins and 2 more to D.
E –– –– A –– =6 The number of coins d has now, is the smallest
–– E –– –– A =6 tow digit number. How many coins does A have
is the beginning?
= 24
(a) 76 (b) 68
So total possible arrangements will be 24.
(c) 60 (d) 52
25. Consider the Question and two statements
given below: Ans. (d) : Let 'A' have number of coins = 8x
Question : Is Z brother of X ? B have number of coins = 4x + 2
Statement-1 : X is a brother of Y and Y is C have number of coins = 2x + 1 + 2
brother of Z. 3
D have number of coins = x + + 2
Statement-2 : X, Y and Z are siblings. 2
Which one of the following is correct in respect 7
of the Question and the Statement =x+
2
(a) Statement-1 alone is sufficient to answer the
Question 7
∵ x + = 10
(b) Statement-2 alone is sufficient of answer the 2
Question 13
∵ A have number of coins = 8 × = 52
(c) Both Statement-1 and Statement-2 are 2
sufficient to answer the Question
29. In the series AABABCABCDABCDABCDE....,
(d) Both Statement-1 and Statement-2 are not which letter appears at the 100th place?
sufficient to answer the Question (a) G (b) H
Ans. (d) : From, statement 1 and 2 (c) I (d) J
Brother Brother
X ← Y ← Z Ans. (c) : A AB ABC ABCD ABCDE……….
Here, it is not clear that Z is men or women. 1 + 2 + 3 + 4 + 5 + 6 + 7 + 8 + 9 + 10 + 11+12 +13 = 91
So, both statement 1 and statement 2 are not sufficient A B C D E F G H I J K L M N
to answer the question. 92 93 94 95 96 97 98 99 100
26. On one side of a 1.01 km long road, 101 plants So, 'I' will come at 100th place.
are planted at equal distance from each other. 30. Three persons A, B and C are standing in a
What is the total distance between 5 queue not necessarily in the same order. There
consecutive plants ? are 4 persons between A and B and 7 persons
(a) 40 m (b) 40.4 m between B and C. If there are 11 persons ahead
(c) 50 m (d) 50.5 m of C and 13 behind A, what could be the
Ans. (b) : Distance between two plants minimum number of persons in the queue?
1.01×1000 (a) 22 (b) 28
= = 10.1 meter (c) 32 (d) 38
100
Distance between 5 consecutive plants Ans. (a) :
= 10.1×4 = 40.4 meter.
27. A, B and C are three places such that there are
three different roads from A to B, four Total minimum number of person = 10 + 12 = 22
different roads from B to C and three different Directions for the following 3 (three) items:
roads from A to C. In how many different ways Read the following two passages and answer
can one travel from A to C using these roads ? the items that follow the passages. Your
(a) 10 (b) 13 answers tothese items should be based on the
(c) 15 (d) 36 passages only.
Ans. (c) : According to the question, Passage–1
Natural selection cannot anticipate future
environments on the earth. Therefore, the set of existing
organisms can never be fully prepared for environmental
So the total number of ways to go from A to C = (3 × 4) catastrophes that await life. An outcome of this is the
+ 3 = 15 extinction of those species which cannot overcome
IAS (Pre) GS IInd Paper (CSAT), 2022 198 YCT
environmental adversity. This failure to survive, in 32.
The passage suggests that Darwinian theory of
modern terms can be attributed to the genomes which are evolution is not a theory at all because.
unable to withstand geological. vagaries or biological (a) It does not satisfy the creationist doctrine
mishaps (infections, diseases and so on). In biological (b) Extinction is a faction of environment and
evolution on the earth, extinction of species has been a biological assaults
major feature. The earth may presently have up to ten (c) There are no evidences to refute it
million species, yet more than 90% of species that have (d) Existence of organisms is attributed to a
ever lived on the earth are now extinct. Once again, the creator
creationist doctrines fail to satisfactorily address why a
Ans. (c) : The passage suggests that Darwinian theory
divine creator will firstly bother to create millions of
of evolution is not a theory at all because there are no
species and then allow them to perish. The Darwinian evidences to refute it.
explanation for extinct life is once again simple, elegant
33. With reference to the passage, the following
and at once convincing-organisms go extinct as a assumptions have been made :
function of environmental of biological assaults for 1. Only species that have the ability to
which their inheritance deems them ill-equipped. overcome environment catastrophes will
Therefore , the so-called Darwinian theory of evolution survive and perpetuate
is not a theory at all. Evolution happens-this is a fact. 2. More than 90% of the species in the earth
The mechanism of evolution (Darwin proposed are in the danger of getting extinct due to
natural selection) is amply supported by scientific data. drastic changes in the environment
Indeed, to date no single zoological, botanical, 3. Darwin's theory explains all the natural
geological, paleontological, genetic or physical evidence phenomena.
has refuted either of the central two main Darwinian Which of the above assumptions is/are valid?
ideas. If religion is not taken into consideration, (a) 1 only (b) 1 and 2 only
(c) 3 only (d) 1, 2 and 3
Darwinian laws are acceptable just like the laws
proposed by Copernicus, Galileo, Newton and Einstein- Ans. (a) : With the reference to the above passage only
one assumption is valid, which is only species that have
sets of natural laws that explain natural phenomena in the
the ability to overcome environment catastrophes and
universe. will survive and perpetuate.
31. According to the passage natural selection Passage–2
cannot anticipate future environment of the With steady economic growth, higher literacy
earth as and increasing skill levels, the number of India
1. Species not fully prepared to face the middle-class families has gone up
environment changes await them will face exponentially. Direct results of the affluence
extinction have been changes in dietary patterns and
energy consumption levels. People have moved
2. All the existing species would get extinct as to a higher protein-based diet like milk
their genomes will not withstands products, fish and meat, all of which need
biological mishaps significantly more water to produce than cereal
3. Inability of the genome to withstands based diets. Increasing use of electronic and
environment changes would result in electric machines/gadgets and motor vehicles
extinction needs more and more energy and generation of
energy needs water.
4. Extinction of species is a common feature
34. Which one of the following statements best
Select the correct answer using the code given reflects the crux of the passage?
below: (a) People should be persuaded to continue with
(a) 1, 2 and 3 (b) 2, 3 and 4 the mainly Indian traditional cereal-based
(c) 1, 3 and 4 (d) 1, 2 and 4 diets.
Ans. (c) : According to the passage, natural section (b) India needs to focus on developing
cannot anticipate future environment of the earth as– agricultural productivity and capacity for
more energy generation in the coming years.
1. Species not fully prepared to face the environment
(c) Modern technological developments result in
changes await them will face extinction.
the change of cultural and social behavior of
2. Inability of the genome to withstand environment the people.
changes would result in extinction. (d) Water management practices in India need to
3. Extinction of species is a common feature. change dramatically in the coming years.

IAS (Pre) GS IInd Paper (CSAT), 2022 199 YCT


Ans. (d) : The statement water management practices in 38. Two Statements following by four conclusions
India need to change dramatically in the coming years are given below. You have to take the
reflects the best curse of the passage. Statements to be true even if they seem to be at
variance from the commonly known facts.
35. How many seconds in total are there in x Read all the Conclusions and then decide which
weeks, x days, x hours, x minutes and x of the given Conclusions logically follows/follow
seconds? from the statements, disregarding the
(a) 11580x (b) 11581x commonly known facts
(c) 694860x (d) 694861x Statement-1: All pens are books.
Statement : No chair is a pen.
Ans. (d) : From question,
Conclusion-I: All chairs are books.
x week, x day, x hours, x minute and x second Conclusion-II: Some chairs are pens.
= x×7×24×60×60 + x×24×60×60 + x×60×60 + x×60 + Conclusion-III: All books are chairs.
x Conclusion-IV: No chair is a book.
= x×24×60×60×8 + 3661x Which one of the following is correct?
= 691200x + 3661x (a) Only Conclusion-I
= 694861x (b) Only Conclusion-II
36. P, Q,R, S, T and U are six members of a family. (c) Both Conclusion-III and conclusion-IV
R is the spouse of Q. U is the mother of T and S (d) None of the Conclusions follows
is the daughter of U.P's daughter is T and R's Ans. (d) : From question,
son Is P. There are two couples in the family.
Which one of the following is correct?
(a) Q is the grandfather of T
(b) Q is the grandmother of T
(c) R is the mother of P None of the conclusion follows.
(d) T is the granddaughter of Q 39. Three Statements following by three
Ans. (d) : According to the question, Conclusions are given below. You have to take
the Statements to be true even if they seem to
be at variance from the commonly known facts.
Read all the conclusions and then decide which
of the given Conclusions logically
follows/follow from the Statements,
'T' is the granddaughter of 'Q'. disregarding the commonly known facts:
37. Consider the Question and two Statements Statement-1: Some doctors are teachers.
given below in respect of three cities P, Q and Statement-2: All teachers are engineers.
R in a State: Statement-3: All engineers are scientists.
Question: How far is city P from city Q? Conclusion -I: Some scientists are doctors.
Statement-1: City Q is 18 km from city R. Conclusion-II: All engineers are doctors.
Statement-2: City P is 43 km from city R. Conclusion-III : Some engineers are doctors.
Which one of the following is correct in respect Which one of the following is correct?
of the Question and the Statements? (a) Only Conclusion-I
(b) Only Conclusion-II
(a) Statement-1 alone is sufficient to answer the
(c) Both Conclusion-I and Conclusion-III
Question
(d) Both Conclusion-I and Conclusion-II
(b) Statement-2 alone is sufficient to answer the
Question Ans. (c) :
(c) Both Statement-1 and Statements-2 are
sufficient to answer the Question
(d) Both Statement-1 and Statement-2 are not
sufficient to answer the Question
Ans. (d) : Both statement 1 and statement 2 are not
sufficient to answer the question. Both conclusion I and conclusion II follows.

IAS (Pre) GS IInd Paper (CSAT), 2022 200 YCT


40. Eight students A, B, C, D, E, F, G and H sit 42. Man does not seek self-improvement because
around a circular table, equidistant from each he
other, facing the center of the table, not (a) is not intellectually capable
necessarily in the same order. B and D sit (b) has no time to do so
neither adjacent to C nor opposite to C. A sits (c) is distracted by materialism
in between E and D, and F sits in between B (d) loves amusement and is mentally idle
and H. Which one of the following is definitely Ans. (b) : According to the passage, man does not seek
correct? self improvement because he finds it difficult to engage
himself in some serious activity in the little leisure he
(a) B sits in between A and G
gets after doing a lot of hard work. So he has no time
(b) C sits opposite to G for self improvement.
(c) E sits opposite to F Passage-2
(d) None of the above The demographic dividend, which has begun in
Ans. (d) : India and is expected to last another few
decades, is a great window of opportunity. The
demographic dividend is basically a swelling in
the working age population, which conversely
means that the relative ratio of very young and
very old will, for a while, be on the decline
From the experience of Irelnad and China, we
know that this can be a source of energy and an
engine of economic growth. The demographic
Hence, 'C' is sitting opposite to 'G'. dividend tends to raise a nation's savings rate
Directions for the following 3 (three) items: since in any nation, it is the working age
Read the following two passages and answer population that is the main saver. And since the
savings rate is an important driver of growth,
the items that follow the passages. Your
this should help elevate our growth rate.
answers to these items should be based on the However, the benefits of demographic dividend
passages only. depend on the quality of the working age
Passage-1 population. And this implies bringing back the
For two or three generations past, ever- importance of education, acquisition of skills
increasing numbers of individuals have been and human capital.
living as workers merely, no as human beings. 43. Which of the following would invariably
An excessive amount of labour is rule today in happen in a country, when the demographic
dividend has begun to operate?
every circle of society, with the result that
(a) The number of illiterate- people will decrease.
man's spiritual elements cannot thrive. He
(b) The ratio of very old and very young will
finds it very difficult to spend his little leisure decrease for a while.
in serious activities. He does not want to think; (c) Population growth rate will quickly stabilize.
or he cannot even if he wants to. He seeks not Select the correct answer using the code given
self-improvement, but entertainment which below.
would enable him to be mentally idle and to (a) 1 and 2 only (b) 2 only
forget his usual activities. Therefore, the so- (c) 1 and 3 only (d) 1, 2 and 3
called culture of our age is dependant more on Ans. (b) : The ratio of very old and very young will
cinema than on theater, more on newspapers, decrease for a while. This would invariably happen in a
magazines and crime stories than on serious country, when the demographic dividend begins to
literature. operate.
41. The passage is based on the idea that 44. With reference to the passage, which of the
(a) Man should not work hard following inferences can be drawn?
(b) The great evil of our age is overstrain 1. Demographic dividend is an essential
condition for a country to rapidly increase
(c) Man cannot think well
its economic growth rate.
(d) Man cannot care for his spiritual welfare 2. Promotion of higher education is an
Ans. (b) : The passage is based on the idea that the essential condition for a country for its
great evil of our age is over strain. rapid economic growth
IAS (Pre) GS IInd Paper (CSAT), 2022 201 YCT
Select the correct answer using the code given Ans. (d) : Let the distance of the village from the home
below. be x km.
(a) 1 only (b) 2 only And speed of the man is V km/hour. It takes a total of
(c) Both 1 and 2 (d) Neither 1 nor 2 45 minutes for a man to reach the village clock.
Ans. (d) : It is clear from the passage that we know According to the question,
from the experience of Ireland and China that the x 1.25x 65
demographic dividend can be source of energy and an + =
V 2V 60
engine of economic growth, but this is an essential
condition, which is not mentioned anywhere in the 3.25x/2V = 65/60
passage, so statement 1 is false. x 65
= hours
According to the passage, education is not only V 3.25 × 30
essential condition, which leads to economic growth. 65 × 60
Along with this, the importance of skill acquisition and t=
human capital is also essential. Therefore, statement (2) 3.25 × 30
is false. t = 40 minute
45. Five friends P, Q, X, Y and Z purchased some Hence, the clock of the village is 5 minute faster.
notebooks. The relevant information is given 47. A person X wants to distribute some pens
below: among six children A, B, C, D, E and F.
1. Z purchased 8 notebooks more than X did. Suppose A gets twice the number of pens
2. P and Q together purchased 21 notebooks. received by B, three times that of C, four times
3. Q purchased 5 notebooks less than P did. that of D, five times that of E and six times that
4. X and Y together purchased 28 notebooks. of F. What is the minimum number of pens X
5. P purchased 5 notebooks more than X did. should buy so that the number of pens each one
If each notebook is priced ` 40, then what is the gets is an even number?
total cost of all the notebooks? (a) 147 (b) 150
(a) ` 2,600 (b) ` 2,400 (c) 294 (d) 300
(c) ` 2,360 (d) ` 2, 320 Ans. (c) : According to the question,
Ans. (a) : According to the question, A = 2B = 3C = 4D = 5E = 6F
Z – X = 8 ---------(i) A B C D E F
P + Q = 21 -------- (ii) = = = = =
60 30 20 15 12 10
P – Q = 5 --------(iii) ∵ Total sum = (60 + 30 + 20 + 15 + 12 + 10) x = 147x
X + Y = 28 ---------(iv) If x = 1
P – X =5 ---------(v) Total number of pens = 147
From equation (ii) and (iii) But total number of D = 15 (Odd number)
2P = 26 So we take x = 2
P = 13, Q = 8 Total number of pen = 147 × 2 = 194
X = 8, Y = 20, Z = 16
48. Six persons A, B, C, D, E and F are sitting
Total cost price of notebooks equidistant from each other around a circular
= (13 + 8 + 8 + 20 + 16) × 40 table (facing the centre of the table).
= 65 × 40 Consider the Question and two Statements
= ` 2600 given below:
46. A man started from home at 14:30 hours and Question: Who is sitting on the immediate left
drove to village, arriving there when the village of A?
clock indicated 15:15 hours. After staying for Statement-1: B is sitting opposite to C and D is
25 minutes, he drove back by a different route sitting opposite to E.
of length 1.25 times the first route at a rate Statement-2: F is sitting on the immediate left
twice as fast reaching home at 16:00 hours As of B.
compared to the clock at home, the village Which one of the following is correct in respect
clock is of the Question and the Statements?
(a) 10 minutes slow (a) Statement-1 alone is sufficient to answer the
(b) 5 minutes slow Question
(c) 10 minutes fast (b) Statement-2 alone is sufficient to answer the
(d) 5 minutes fast Question

IAS (Pre) GS IInd Paper (CSAT), 2022 202 YCT


(c) Both Statement-1 and Statement-2 are (a) Statement-1 alone is sufficient to answer the
sufficient to answer the Question Question
(d) Both Statement-1 and Statement-2 are not (b) Statement-2 alone is sufficient to answer the
sufficient to answer the Question Question
Ans. (d) : According to the question, (c) Both Statement-1 and Statement-2 are
sufficient to answer the Question
(d) Both Statement-1 and Statement-2 are
not sufficient to answered the Question
Ans. (d) According to statement 1st, the possible
sequence of lecture period-
It is not clear from the above that who will be to the left
of A, either D or E.
Option (d) will be right.
49. Consider the Question and two Statements
given below:
Question: What is the age of Manisha?
Statement-1 : Manisha is 24 years younger
than her mother.
Statement-2 : 5 years later, the ages of Manisha It is clear from statement Ist, anyone of the C, F and A
and her mother will be the ration 3 : 5 can be lectured in the third period.
Which one of the following is correct in respect According to statement 2nd, the possible sequence of
of the Question and the Statement? lecture -
(a) Statement-1 alone is sufficient to answer the
Question
(b) Statement-2 alone is sufficient to answer the It is clear from statement 2- anyone of the A, C, D, E
Question and F can be lectured in the IIIrd period.
(c) Both Statement-1 and Statement-2 are According to statement 1st and 2nd, the possible
sufficient to answer the Question sequence of lecture period-
(d) Both Statement-1 and Statement-2 are not
sufficient to answered the Question
Ans. (c) : From statement-1
Let the Manisha's mother age = x year
Age of Manisha = (x – 24) year
From statement-2
x − 24 + 5 3 Statement Ist and 2nd both together not able to decide
= who will be lectured in the third period because any of
x +5 5
C, F and A can be lectured.
x − 19 3
= So, both statement-1 and Statement-2 are not sufficient
x +5 5 to answer the question.
5x − 95 = 3x + 15 Directions for the following 3 (three) items:
2x =110 Read the following two passages and answer
x =55 the items that follow the passages. Your
Both statement 1 and statement 2 are sufficient to answers to these items should be based on the
answer the question. passages only.
50. Six lectures A, B, C, D, E and F, each of one Passage-1
hour duration, are scheduled between 8:00 a.m. In an economic organization, allowing mankind
and 2:00 p.m. to benefit by the productivity of machines
Consider the Question and two Statement given should lead to a very good life of leisure, and
below: much leisure is apt to be tedious except to those
Question: Which lecture is in the third period? who have intelligent activates and interests. If a
Statement-1 : Lecture F is preceded by A and leisured population is to be happy, it must be
following by C.
an educated population, and must be educated
Statement-1: There is no lecture after lecture B.
with a view to enjoyment as well as to the direct
Which one of the following is correct in respect
usefulness of technical knowledge.
of the Question and the Statement?
IAS (Pre) GS IInd Paper (CSAT), 2022 203 YCT
51. Which of the following statements best reflects Which of the above statements is/are correct?
the underlying tone of the passage? (a) 1 only (b) 2 only
(a) Only an educated population can best make (c) Both 1 and 2 (d) Neither 1 nor 2
use of the benefits of economic progress. Ans. (a) : We know that, A, B and C are non-zero
(b) All economic development should be aimed number which can be of 1, 2, 3, 4, 5, 6, 7, 8, 9
at the creation of leisure. From statement-1–
(c) An increase in the educated population of a The total number formed by taking the numbers 1, 2
country leads to an increase in the happiness and 3 will be 6
of its people. ∵ 123 +132 + 213 + 231 + 312 + 321 = 1332
(d) Use of machines should be encouraged in From statement-2–
order to create a large leisured population. The total number formed by taking the number 7, 8 and
Ans. (a) : The statement, “Only an educated population 9 will be 6
can best make use of the benefits of economic = 789 + 798 + 879+ 897 + 978 + 987 = 5328
progress” best reflects the underlying tone of the Hence, only statement 1 will be correct.
passage. 55. There is a numeric lock which has a 3-digit
Passage-2 PIN. The PIN contains digits 1 to 7. There is no
If presents bring less thrill now that we are repetition of digits The digits in the PIN from
grown up, perhaps it is because we have too left to right are in decreasing order. Any two
digits in the PIN differ by at least 2. How many
much already; or perhaps it is because we have
maximum attempts does one need to find out
lost the fullness of the joy of receiving. the PIN with certainty?
Children's fears are poignant, their miseries
(a) 6 (b) 8
are acute, but they do not look too forward nor
(c) 10 (d) 13
too far backward. Their joys are clear and
complete, because they have not yet learnt Ans. (c) : According to the question,
always to add 'but' to every proposition. Possible pin code number
Perhaps we are too cautious, too anxious, too = n{753, 752, 751, 742, 741, 731, 642, 641, 631,531}
skeptical, perhaps less about them and entered = 10
with more single minded enjoyment into the 56. There are eight equidistant points on a circle.
happiness that come our way. How many right-angled triangles can be drawn
52. With reference to the passage, which one of the using these points as vertices and taking the
following statements is correct? diameter as one side of the triangle?
(a) It is not possible for adults to feel thrilled by (a) 24 (b) 16
presents. (c) 12 (d) 8
(b) There can be more than one reason why Ans. (a) :
adults feel less thrilled by presents.
(c) The author does not know why adults feel
less thrilled by presents.
(d) Adults have less capacity to feel the joy of
loving or being loved.
Ans. (b) : According to the passage, the statement
'there can be more than one reason why adults feel less
thrilled by presents’, best reflects the underlying tone of Total 6 right angle triangle can be formed on one
this passage. diameter.
53. The author of the passage is against So total right angle triangle = 4 × 6 = 24
(a) worrying too much about the past and future 57. 24 men and 12 women can do a piece of work
(b) being in the habit of thinking about presents in 30 days. In how many days can 12 men and
(c) not being thrilled by new things 24 women do the same piece of work?
(d) giving and receiving joy only partially (a) 30 days
Ans. (a) : The author of the passage is against worrying (b) More than 30 days
too much about the past and the future. (c) Less than 30 days or more than 30 days
(d) Data is inadequate to draw any conclusion
54. Let A, B and C represent distinct non zero
digits. Suppose x is the sum of all possible 3- Ans. (d) : Data is inadequate to draw any conclusion.
digit numbers formed by A, B and C without 58. What is the remainder when
repetition. 91×92×93×94×95×96×97×98×99 is divided by
Consider the following statements: 1261?
(1) The 4-digit least value of x is 1332 (a) 3 (b) 2
(2) The 3-digit greatest value of x is 888. (c) 1 (d) 0

IAS (Pre) GS IInd Paper (CSAT), 2022 204 YCT


Ans. (d) : From question, Directions for the following 3 (three) items:
91×92×93×94×95×96×97×98×99 Read the following two passages and answer
13 × 97 the items that follow the passages. Your
Remainder = 0 answers to these items should be based on the
59. Consider the following statements in respect of passages only.
a rectangular sheet of length 20 cm and Passage–1
breadth 8 cm: The majority of people who fail to accumulate
1. It is possible to cut the sheet exactly into 4 money sufficient for their needs, are generally,
square sheets.
easily influenced by the opinions of others.
2. It is possible to cut the sheet into 10
triangular sheets of equal area. They permit the newspapers and the gossiping
Which of the above statements is/are correct? neighbors to do their thinking for them.
(a) 1 only (b) 2 only Opinions are the cheapest commodities on the
(c) Both 1 and 2 (d) Neither 1 nor 2 earth. Everyone has a flock of opinions ready
Ans. (c) : From statement 1 them. If you are influenced by opinions when
you reach decisions, you will not succeed in any
undertaking.
61. Which one of the following is implied by the
passage?
(a) Most of the people do not accumulate money
It is possible to construct the four square.
for their needs.
Hence, statement 1 is correct.
From statement 2 (b) Most of the people never fail to accumulate
1 money for their needs.
10× × 8 × a = 20 × 8 (c) There are people who fail to accumulate
2
a=4 money for their needs.
(d) There is no need to accumulate money.
Ans. (c) : There are people who fail to a accumulate
money for their needs is implied by the passage.
62. What is the main idea of the passage?
Hence, both statement 1 and 2 are correct.
(a) People should not be influenced by the
60. When 70% of a number x is added to another opinions of others.
number y, the sum becomes 165% of the value
of y. When 60% of the number x is added to (b) People should accumulate as much money as
another number z, then the sum becomes 165% they can.
of the value of z. Which one of the following is (c) People should neither give nor accept the
correct? opinions.
(a) z < x < y (b) x < y < z
(d) People will succeed in any undertaking if they
(c) y < x < z (d) z < y < x
do not accept any opinion at all.
Ans. (a) : According to the question,
70 165 Ans. (a) : The main idea of this passage is that the
x× + y = y× people should not be influenced by the opinions of
100 100
others.
70 65
x× = y× Passage -2
100 100
" The social order is a sacred right which is the
x 13
= basis of all other rights. Nevertheless, this right
y 14
does not come from nature, and must therefore
60 65 be founded on conventions."
Similarly, x× + z = z×
100 100 63. With reference to the above passage, which of
60 65 x 13 the following statements is/are correct?
x× = z× ⇒ =
100 100 z 12 1. Conventions are the sources of rights of man.
x : y : z = 13 : 14 : 12 2. Rights of man can be exercised only when
∴z<x<y there is a social order.

IAS (Pre) GS IInd Paper (CSAT), 2022 205 YCT


Select the correct answer using the code given 67. Consider the following statements in respect of
below. two natural numbers p and q such that p is a
(a) 1 only prime number and q is a composite umber:
(b) 2 only 1. p × q can be an add number.
(c) Both 1 and 2 2. q/p can be a prime number.
(d) Neither 1 nor 2 3. p + q can be a prime number.
Which of the above statements are correct?
Ans. (c) : With reference to the above passage, both of
(a) 1 and 2 only (b) 2 and 3 only
the statements are correct. The statements are–
(c) 1 and 3 only (d) 1, 2 and 3
1. Conventions are the sources of rights of man.
Ans. (d) : Let, p = 3, q = 9
2. Rights of man can be exercised only when there is a
From statement 1
social order.
p× q = 27 (Odd number)
64. Two candidates X and Y contested an election. From statement 2
80% of voters cast their vote and there were no
21
invalid votes. There was no NOTA (None of the ∴ =7
above) option. X got 56% of the votes cast and 3
won by 1440 votes. What is the total number of So, statement 2 is correct.
voters in the voters list? If p = 3, q = 4
(a) 15000 (b) 12000 ∴p+q=7
(c) 9600 (d) 5000 Hence, statement 3 is correct.
Ans. (a) : 68. Consider the following statements:
Let the number of voters in the voter list = x 1. Between 3:16 p.m. and 3:17 p.m. both hour
hand and minute hand coincide.
Voting people = 80x/100
2. Between 4:58 p.m. and 4:59 p.m. both minute
Percentage difference of votes obtained by X and Y
hand and second hand coincide.
80x ( 56 − 44 ) Which of the above statements is/are correct?
1440 = ×
100 100 (a) 1 only
8x 12 (b) 2 only
1440 = × (c) Both 1 and 2
10 100
(d) Neither 1 nor 2
x = 15000
Ans. (c) : From statement 1
65. What is the smallest number greater than 1000
that when divided by any one of the numbers 6, 11M
θ= ∼ 30H
9, 12, 15, 18, leaves a remainder of 3? 2
(a) 1063 (b) 1037 11M
0= ∼30 × 3
(c) 1083 (d) 1183 2
Ans. (c) : LCM of 6, 9, 12, 15 18 = 180 11M
= 90
Required number = 180k + 3 2
K = 6 taking 180
M=
= 1080 + 3 = 1083 11
66. Let p be a two-digit number and q be the 4
M = 16
number consisting of same digits written in 11
reverse order. If p × q = 2430, then what is the Minute hand and hour hand will meet at 3: 16
difference between p and q? 4
(a) 45 O'clock.
11
(b) 27 Both statements 1 and 2 are correct.
(c) 18 69. There are two containers X and Y. X contains
(d) 9 100 ml of milk and Y contains 100 ml of water,
Ans. (d) : From question, 20 ml of milk from X is transferred to Y. After
p × q = 2430 mixing well, 20 ml of the mixture in Y is
p × q = 54 × 45 transferred back to X. If m denotes the
proportion of milk in X and n denotes the
∴ p = 54, q = 45 proportion of water in Y, then which on of the
p–q=9 following is correct?
IAS (Pre) GS IInd Paper (CSAT), 2022 206 YCT
(a) m=n 71. The author's assumption about scientific
(b) m>n research is that
(c) m<n (a) it is never very valuable
(d) Cannot be determined to due to insufficient (b) it is sometime very valuable
data
(c) it is never without some value
Ans. (a) : Quantity of milk in X = 100 ml
(d) it is always very valuable
Quantity of water in Y = 100 ml
After first process– Ans. (c) : The author's assumption about scientific
Total number of mixture in container = 120 ml research is that a scientific research is never without
In container ratio of water and milk = 1 : 5 some value.
After second process– 72. According to the author
Quantity of milk in container X = 80 + 3.33 = 83.33 ml (a) not many research results can be of value to
5 an intelligent investigator
Ratio of water in container Y = 100 × = 83.33 ml
6 (b) a research result is always valuable to an
Hence, m = n intelligent investigator
70. A pie chart gives the expenditure on five (c) any research result can be value to an
different items A, B, C, D and E in a household. intelligent investigator
If B, C, D and E correspond to 90°, 50° 45° and (d) a research result must always be of some
75° respectively, then what is the percentage of value to an intelligent investigator
expenditure on item A ?
Ans. (c) : According to the author, any research result
112 125
(a) (b) can be of value to an intelligent investigation.
9 6
155 250 Passage– 2
(c) (d) How best can the problems of floods and
9 9
droughts be addressed so that the losses are
Ans. (d) : Central angle for item 'A'.
= 360º – (90º + 50º 45º + 75º) minimal and the system becomes resilient? In
= 360º – 260º = 100º this context, one important point that needs to
Percentage of expenditure on item be noted is that India gets 'too much' water
(about 75% of annual precipitation) during 120
100 250
= × 100 = % days (June to September) and 'too little' for the
360 9
remaining 245 days. This skewed water
Directions for the following 3 (three) items availability has to be managed and regulated
Read the following two passages and answer for its consumption throughout the year.
the items that follow the passages. Your
answers to these items should be based on the 73. Which one of the following best reflects the
passages only. practical, rational and lasting solution?
Passage –1 (a) Constructing huge concrete storage tanks and
To encourage research is one of the functions of canals across the county
a university. Contemporary universities have (b) Changing the cropping patterns and farming
encouraged research, not only in those cases practices
where research is necessary, but on all sorts of
(c) Interlinking of river across the country
entirely unprofitable subjects as well. Scientific
research is probably never completely (d) Buffer stocking of water through dams and
valueless. However silly and insignificant it recharging aquifers
may seem, however mechanical and Ans. (d) : Buffer stocking of water through dams and
unintelligent the labours of the researchers, recharging aquifers best reflects the practical, rational
there is always a chance that the results may be
and lasting solution.
of value to the investigator of talent, who can
use the facts collected for him by uninspired 74. If 15 ×14 ×13 × ....× 3 × 2 ×1 = 3m × n where m
but industrious researchers as the basis of some and n are positive integers, then what is the
fruitful generalization. But where research is maximum value of m?
not original, but consists in the mere
rearrangement of existing materials, where its (a) 7
object is not scientific but literary or historical, (b) 6
then there is a risk of the whole business (c) 5
becoming merely futile. (d) 4
IAS (Pre) GS IInd Paper (CSAT), 2022 207 YCT
Ans. (b) : From question, 78. The sum of three consecutive integers is equal
15×14×13×12×11×10×9×8×7×6×5×4×3×2×1 = 3m × n to their product. How many such possibilities
36×5×14×13×4×11×10×8×7×2×5×4×2×1 = 3m × n are there?
∴m=6 (a) Only one
(b) Only two
75. What is the value of X in the sequence 2, 12, 36,
(c) Only three
80, 150, X?
(a) 248 (b) 252 (d) No such possibility is there
(c) 258 (d) 262 Ans. (c) : Let three consecutive integer x, (x + 1) and (x
+ 2)
Ans. (b) : Given series is as follows–
2, 12, 36, 80, 150, X ∴ x+(x + 1) + (x + 2) = x (x + 1) (x + 2)
2
2 = 1 +1 3 3x + 3 = x (x + 1) (x + 2)
2
12 = 2 + 2 3 3(x + 1) = x (x + 1) (x + 2)
2
36 = 3 + 3 3 0 = (x + 1) [x2 + 2x – 3]
80 = 42 + 43 If x+1=0
2
150 = 5 + 5 2 x =–1
2
2 3
X = 6 + 6 = 36 + 216 If x + 2x – 3 = 0
X = 252 (x + 3) (x – 1) = 0
76. One non-zero digit, one vowel and one ∴ x = – 3 or 1
consonant from English alphabet (in capital) Hence, there are three possibilities.
are to be used in forming passwords, such that 79. What is the number of numbers of the form 0
each password has to start with a vowel and XY, where X and Y are distinct non-zero
end with a consonant. How many such digits?
passwords can be generated? (a) 72
(a) 105 (b) 525 (b) 81
(c) 945 (d) 1050 (c) 90
Ans. (c) : Hence, password is one non-zero digit one (d) 100
vowel and one consonant formed English alphabet. Ans. (a) : x and y is non digit.
So, five vowels can be used in the first place, nine digit Numbers between 11 and 20 = 12, 13, 14, 15, 16, 17,
can be used in second place and 21 consonant can be 18, 19 = 8
used in the third place. Numbers between 20 and 30 = 21, 23, 24, 25, 26, 27,
Total number of password = 5×9×21= 945 28, 29 = 8
77. There are 9 cups placed on a table arranged in Thus, total numbers between 31 and 100 = 7 × 8 = 56
equal number of rows and columns out of Total number = 65+ 8 + 8 = 72
which 6 cups contain coffee and 3 cups contain
80. The average weight of A, B, C, is 40 kg, the
tea. In how many ways can they be arranged so
average weight of B, D, E is 42 kg and the
that each row should contain at least one cup of
weight of F is equal to that of B. What is the
coffee?
average weight of A, B, C, D, E and F?
(a) 18 (b) 27
(a) 40.5 kg
(c) 54 (d) 81
(b) 40.8 kg
Ans. (d) : Case-I–If one cup of coffee is placed in each
(c) 41 kg
line
(d) Cannot be determined as data is inadequate
CCT, CTC, TCC
Ans. (c) : According to the question,
Total ways = 3 × 3 × 3 = 27
A + B + C = 40 × 3 = 120
Case-II–If three cups of coffee are placed in one line
B + D + E = 42 × 3 = 126
and two cups of coffee in the other and one cup of
∴F=B
coffee in the third, then
CCC, CCT or CTC or TCC, CTT or TCT or TTC ∴A + B + C + D + E + F = A+B+C+D+E+B
Total ways = 3! × 3 × 3 = 54 = 120 + 126 = 246
From Case-I and Case-II 246
Average weight= = 41 kg
Total ways = 27 + 54 = 81 6

IAS (Pre) GS IInd Paper (CSAT), 2022 208 YCT


UNION PUBLIC SERVICE COMMISSION
Civil Services (Preliminary Exam) - 2023
CSAT : PAPER-II
(Analysis with Explanation)
Time : 2 hours Maximum Number : 200
Passage-1 And it is one that really should be addressed given
India's demographic outlook.
We often hear about conflicts among different states
n India over river waters. Of the 20 major river systems, 2. Which one of the following statements best
14 are already water-stressed; 75% of the population reflects what is implied by the passage?
lives in water-stressed regions, a third of whom live in (a) Education system must be strengthened n
water-scarce areas. Climate change, the demands of rural areas.
rising population and the need for agriculture to keep (b) Large scale and effective implementation of
pace, and increased rate of urbanization and skill development programme is the need of
industrialization will exacerbate water stress. According the hour.
to the constitution of the India, water s a state subject and (c) For economic development, health an
not that of the Union, except for regulation of inter-state nutrition of only skilled workers needs special
rivers. Key to ensuring balance between competing attention.
demands of various stakeholders is a basin-based (d) For rapid economic growth as envisaged by
approach to allocate water amongst constituent regions us, attention should be paid to health and
and states. Allocating fair share of water to them requires nutrition of the people.
assessments based on objective criteria, such as Ans. (d) : According to the passage, for rapid economic
specificities of the river basin, size of dependent growth as envisaged by us, attention should be paid to
population, existing water use and demand, efficiency of health and nutrition of the people.
use, projected future use, etc. while ensuring the
environmental needs of the river and aquifers. Passage-3
1. Which one of the following statements best In India, a majority of farmers are marginal and
reflects the most rational, practical and small, less educated and possess low adaptive
immediate actions required to ensure fair and capabilities to climate change, perhaps because of credit
equitable allocation of water to different and other constraints. So, one cannot expect autonomous
stakeholders? adaptation to climate change. Even if it was possible, it
(a) A national, pragmatic, legal and policy would not be sufficient to offset losses from climate
framework for water allocation should be change. To deal with this, adaptation to climate change is
made. paramount, alongside a fast mitigation response. Another
(b) all river systems of the country should be solution is to have a planned or policy-driven adaption,
linked and huge aquifers created. which would require the government to come up with
policy recommendations. Perception is a necessary pre-
(c) Water channels between regions of water requisite for adaptation. Whether farmers are adapting
surplus and regions of water deficit should be agricultural practices to climate change depends on
created. whether they perceive it or not. However, this is not
(d) To mitigate water crisis, water demand of always enough for adaptation. It is important how a
sectors such as agriculture and industry farmer perceives the risks associated with climate
should be reduced. change.
Ans. (A) : According to the passage, a national. 3. Which one of the following statements best
pragmatic, legal and policy framework for water reflects the most logical and rational message
allocation should be made for the most rational, conveyed by the author of the passage?
practical and immediate action to ensure fair and (a) Adaptation to climate change and mitigation
equitable allocation of water to different stakeholders. response are basically the responsibilities of
Passage-2 the government.
(b) Climate change causes a change in
More than half of Indian women and almost a government policies regarding land use
quarter of Indian men of working age suffer from patterns in the country.
anemia. According to studies, they are anywhere from 5- (c) Risk perceptions of farmers are important for
15% less productive than they could, be, as a result motivating them for taking adaptation
thereof. India also has the largest tuberculosis burden in decisions.
the would, costing 170 million workdays to the country (d) Since mitigation is not possible, governments
annually. But what is just as important as lost should come up with policies for quick
productivity now is lost potential in the future. It is response to climate change.
becoming increasingly clear that on many measures of
cognitive ability, malnourished India children perform Ans. (c) : According to the passage, risk perceptions of
two or three times worse than their adequately nourished farmers are important for motivating them for taking
peers. For an economy that will be more dependent on adaptation decisions; is the most logical and rational
highly skilled workers, this poses a significant challenge. message conveyed by the author.

IAS (Pre) GS IInd Paper (CSAT), 2023 209 YCT


4. Raj has ten pairs of red, nine pairs of white and Ans. (b) : Statement 1 - It is not possible that exactly
eight pairs of black shoes in a box. If he one letter goes into on incorrect envelope. If 3 letters go
randomly picks shoes one by one (without in correct envelope then 4th will also go in correct
replacement) from the box to get a red pair of envelope.
shoes to wear, what is the maximum number of
attempts he has to make? Statement 2 - Number of ways in which only two
(a) 27 (b) 36 letters can go into the correct envelopes out of 4 letter
(c) 44 (d) 45 4! 4×3
= 4C2 = = = 6 ways
Ans. (d) : According to the given, 2!× 2! 2
Pairs of red shoes = 10 7. What is the remainder when
Pairs of white shoes = 9 (a) 0 (b) 1
Pairs of black shoes = 8 (c) 2 (d) 4
Let Raj drowns 10 red shoes (all of left foot), 18 white
shoes and 16 black shoes. Ans. (a) : According to the question,
Total number of shoes drawn = 44 85 × 87 × 89 × 91× 95 × 96
But still red pair is not complete =
100
Therefore, one more shoe drawn from the box will
complete the red pair. −15 × −13 × −11× −9 × −5 × −4 386100
= =
44 + 1 = 45 100 100
5. In how many ways can a batsman score exactly Remainder = 0
25 runs by scoring single runs, fours and sixes 8. What is the unit digit in the expansion of
only, irrespective of the sequence of scoring (57242)9 × 7 × 5 × 3 × 1
shots? (a) 2 (b) 4
(a) 18 (b) 19
(c) 20 (d) 21 (c) 6 (d) 8
Ans. (b) : According to the question, 25 runs is to be Ans. (a) : According to the question,
made with single runs, fours and sixes only. Therefore, (57242)9×7×5×3×1
1s 4s 6s (57242)945
1 25 0 0 By dividing the power 945 by 4 we get Remainder = 1
Therefore, (57242)1 = 57242
2 21 1 0
Hence, unit digit = 2
3 17 2 0
9. If ABC and DEF are both 3-digit numbers such
4 13 3 0 that A, B, C, D, E and F are distinct non-zero
5 9 4 0 digits such that ABC + DEF = 1111, then what
6 5 5 0 is the value of A + B + C + D + E + F + ?
7 1 6 0 (a) 28 (b) 29
8 19 0 1 (c) 30 (d) 31
9 15 1 1 Ans. (d) : According to the question,
10 11 2 1 A B C
11 7 3 1
+ D E F
12 3 4 1
13 13 0 2 1 1 1 1
14 9 1 2 C + F must be equal to 11
15 5 2 2 B + E must be equal to 10 (because 1 is carryover)
16 1 3 2 A + D must be equal to 10
17 4 0 3 Therefore, A+D+B+E+C+F
18 1 1 3 =10 + 10 +11
= 31
19 1 0 4
10. D is a 3-digit number such that the ratio of the
Hence, 19 ways. number to the sum of its digits is least. What is
6. There are four letters and four envelopes and the difference between the digit at the
exactly one letter is to be put in exactly one hundred's place and the digit at the unit's place
envelope with the correct address. If the letters of D?
are randomly inserted into the envelopes, then (a) 0 (b) 7
consider the following statements:
1. It is possible that exactly one letter goes (c) 8 (d) 9
into an incorrect envelope. Ans. (c) : For the ratio of the number to the sum of its
2. There are only six ways in which only two digits to be least, the value of D = 199
letters can go into the correct envelopes. 199 9
Which of the statements given above is/are Then, ratio = = 10
correct? 19 19
(a) 1 only (b) 2 only Therefore, difference between the digit at hundred's
(c) Both 1 and 2 (d) Neither 1 nor 2 place and the digit at units place of D = 9–1 = 8

IAS (Pre) GS IInd Paper (CSAT), 2023 210 YCT


Passage-1 leading to monotonous diets that do not provide the daily
nutrients for them to develop fully. Part of the reason
The emissions humans put into the atmosphere now nutrition is under threat worldwide is that our food
will affect the climate in the middle of the century and systems are not properly responding to nutritional needs.
onwards. Technological change, meanwhile, could make Somewhere along that long road from farm to fork, there
a future transition away from fossil fuels cheap or it are serious detours taking place.
might not, leaving the world with a terrible choice
between sharply reducing emissions at huge cost or 13. Which one of the following statements best
suffering through the effects of unabated warming. reflects the crux of the passage?
Businesses that do not hedge against the threat of (a) The scheme of Universal basic Income should
uncertain outcomes fail. The world cannot afford such be implemented worldwide as a way of
recklessness on climate change. poverty alleviation.
11. Which one of the following statements best (b) We must place food-based nutrition at the
reflects the crucial message conveyed by the centre of our policy debate.
author of the passage? (c) Nutritional status of food should be improved
(a) Businesses that cause emissions may need to by creating appropriate genetically modified
close down or pay for pollution in future. crops.
(b) The only solution is technological (d) Using modern food processing technologies,
development related to the issues of climate we must fortify food items with required
change. nutrient elements.
(c) Waiting to deal with carbon emission until Ans. (b) : Statement that best reflects the curse of the
technology improves is not a wise strategy. passage is- "We must place food based nutrition at the
(d) Since future technological change is center of our policy debate".
uncertain, new industries should be based on 14. Three of the five positive integers p, q, r, s, t
renewable energy sources. are even and two of them are odd (not
Ans. (c) : According to the passage, statement that best necessarily in order). Consider the following:
reflects the crucial message conveyed by the author is - 1. p, q, r, s, t is definitely even.
"Waiting to deal with carbon emissions until technology 2. 2p + q + 2r – 2s + t is definitely odd.
improves is not a wise strategy". Which of the above statements is/are correct?
Passage-2 (a) 1 only (b) 2 only
(c) Both 1 and 2 (d) Neither 1 nor 2
Environmental problems cause health problems. Ans. (a) : Let p, q, r are even numbers and s, t are odd
Substantial changes in lifestyle can reduce environmental numbers
or health problems, but this idea appears almost
impossible to adopt with environmental problems, Statement 1 - (p + q + r) – (s + t)
individual efforts can be perceived as having a negligible We know, sum of three even numbers is a even number,
effect and therefore lead to inertia. With health, on the therefore, p + q + r = even
other hand, individual choices can make the difference And sum of two odd numbers is even therefore, s + t =
between life and death, literally. And yet, barring a few, even
there seems to be the same collective lethargy towards even – even = even
making their choices. i.e. (p + q + r) – (s + t) = even number
12. Which one of the following statements best Statement 1 is correct.
implies the most rational assumption that can Statement 2 - (2p + q + 2r – 2s + t) cannot be always
be made from the passage? odd.
(a) We are likely to spend more money on cure Hence, this is wrong statement.
than prevention. 15. Consider the following in respect of prime
(b) It is the job of the government to solve our number p and composite number c.
environmental and public health problems.
(c) Health can be protected even if environmental p+c
1. can be even.
problems go on unattended. p -c
(d) Loss of traditional lifestyle and the influence 2. p + c can be odd
of western values led to some unhealthy ways 3. pc can be odd
of living. Which of the statements given above are corret?
Ans. (c) : Statement that best implies most rational (a) 1 and 2 only (b) 2 and 3 only
assumption that can be made from the passage is- (c) 1 and 3 only (d) 1, 2 and 3
"Health can be protected even if environmental Ans. (d) : Let p = 11 and c = 9
problems go on unattended".
p+c
Passage-3 Statement 1 -
p−c
Many people are not eating the right food. For some,
it is simply a decision to stick with food they enjoy but 11 + 9 20
= = = 10 = even
which is not too health. This is leading to an increase in 11 − 9 2
non-communicable diseases. This in turn leads to major Statement 1 is correct
burden on our health-care systems that have the potential Statement 2 - = 2 × p + c
to derail the economic progress which is essential for the
poor to improve their lives. For others, it is about limited = 2 × 11 + 9 = 22 + 9
access to nutritious food or a lack of affordability, = 31 = odd
IAS (Pre) GS IInd Paper (CSAT), 2023 211 YCT
Statement 2 is correct positions and even digits occupy even
Statement 3 - pc positions?
= 11 × 9 (a) 12 (b) 18
= 99 = odd (c) 36 (d) 72
Statement 3 is correct. Ans. (c) : According to the question,
16. A 3-digit number ABC, on multiplication with In an eight digit number, even positions will be 2, 4, 6,
D gives 37DD where A, B, C and D are 8 and odd positions will be 1, 3, 5, 7.
different non-zero digits. What is the value of A There are 4 even digits and they will occupy even
+ B + C? positions, so they can be arranged is 4!.
(a) 18 But is these 4 digits, two digits are repeated, so now
(b) 16 4!
(c) 15 these 4 even digits are arranged in =6
2!× 2!
(d) Cannot be determined due to insufficient data
4!
Ans. (d) : According to the given, Similarly, 4 odd digits are arranged in =6
D (100 A + 10B + C) = 3000 + 700 + 10D + D 2!× 2!
= 3700 + 11D Therefore, the odd and even digits that can be arranged
Case (i) - Let D = 4 as per the given condition will be 6×6 = 36.
The number 3700 + 11D = 3744 20. A, B, C working independently can do a piece
of work in 8, 16 and 12 days respectively. A
3744 along works on Monday, B alone works on
100 A + 10B + C = = 936
2 Tuesday, C alone works on Wednesday; A
Therefore, A = 9, B = 3 and C = 6 alone, again works on Thursday and so on.
Case (ii) - Let D = 5 Consider the following statements:
The number (3700 + 11D) = 3755 1. The work will be finished on Thursday.
3755 2. The work will be finished in 10 days.
100A + 10B + C = = 751 Which of the above statements is/are correct?
5 (a) 1 only (b) 2 only
Therefore, A = 7, B = 5, C = 1 (c) Both 1 and 2 (d) Neither 1 nor 2
There is no unique value for (A + B + C)
Ans. (a) : A alone can do a piece of work in 8 days
Hence, A + B + C cannot be determined B alone can do a piece of work in 16 days
17. For any choices of values of X, Y and Z, the 6- C alone can do a piece of work in 12 days
digit number of the form XYZXYZ, is divisible LCM of 8, 16 and 12 = 48
by : Let total work is 48 units
(a) 7 and 11 only (b) 11 and 13 only Then,
(c) 7 and 13 only (d) 7, 11 and 13 Monday - 6 unit
Ans. (d) : According to the question, Tuesday - 3 unit
Any number of the form 'XYZXYZ' will be a multiple Wednesday - 4 unit
of 1001 Thursday - 6 unit
Then, irrespective of the values of X, Y and Z, this 6 Friday - 3 unit
digit number will be divided by 1001 Saturday - 4 unit
On factorizing 1001, we get, 1001 = 7 × 11 × 3 Sunday - 6 unit
Hence, 'XYZXYZ' is divisible by 7, 11 and 13. Monday - 3 unit
18. 125 identical cubes are arranged in the from of Tuesday - 4 unit
a cubical block. How many cubes are Wednesday - 6 unit
surrounded by other cubes from each side? Thursday - 3 unit
(a) 27 (b) 25 48 unit
(c) 21 (d) 18 The work will be finished on Thursday.
Ans. (a) : According to the question.
→ If 125 identical cubes are arranged in the form of a Passage-1
cubical, then, there will be 25 cubes in each of the 5 To tackle the problem of pollution in cities, policy
layers. makers think that drastic actions like temporary use of
→ Now, separating the upper layer and lower layer odd-even number scheme for vehicles, closing schools,
total number of cubes separated = 50 factories, construction activates, and banning the use of
→ Now, separating the outer 16 cubes from the certain type of vehicles are a way forward. Even then the
remaining three rows, number of removed cubes = air is not clean. Vehicles more than 15 years old
16×3 = 48 comprise one percent of the total; and taking them off the
road will not make any difference. Banning certain fuels
→ Total number of cubes which are not surrounded by and car types arbitrarily is not proper. Diesel engines
other cubes from each side = 50 + 48 = 98 produce more PM 2.5 and less CO2 than petrol or CNG
→ Therefore, number of cubes surrounded by other engines. On the other hand, both diesel and CNG engines
cubes from each side = 125 – 98 = 27 produce more NOx than petrol engines. No one has
19. How many distinct 8-digit numbers can be measured the amount of NOx that CNG engines are
formed by rearranging the digits of the number emitting. Arbitrary bans on vehicles that have passed
11223344 such that odd digits occupy odd mandated fitness tests and periodic pollution tests are
IAS (Pre) GS IInd Paper (CSAT), 2023 212 YCT
unfair. What is needed is the scientific and reliable (c) Rich biodiversity cannot be maintained in the
information about the source of pollutants on a forests without the presence of elephants.
continuing basis and the technologies that will work to (d) Elephants are capable of regenerating forests
reduce pollution from them. with species as per their requirement.
21. Which one of the following statements best Ans. (b) : The statement best reflects the most logical
reflects the most logical and rational and rational inference that can be drawn from the
implication conveyed by the passage? passage is -
(a) Arbitrary curbs on vehicles to reduce
pollution are difficult to implement. "Elephants are the keystone species and they benefit the
biodiversity".
(b) Knee-jerk reactions cannot solve the problem
of pollution but an evidence-based approach 24. If 7 ⊕ 9 ⊕ 10 = 8, 9 ⊕ 11 ⊕ 30 = 5, 11 ⊕ 17 ⊕ 21
will be more effective. = 13 what is the value of 23 ⊕ 4 ⊕ 15?
(c) A heavy penalty should be enforced on those (a) 6 (b) 8
driving without periodic pollution tests. (c) 13 (d) 15
(d) In the absence of laws to deal with the Ans. (a) : According to the given,
problems of pollution, the administration (i) 7 ⊕ 9 ⊕ 10
tends to make arbitrary decisions.
It can be written as,
Ans. (b) : According to the passage, knee-jerk reactions 7 + 9 + 10 = 26
cannot solve the problem of pollution but an evidence
based approach will be more effective. and now, 2 + 6 = 8
(ii) 9 ⊕ 11 ⊕ 30
Passage-2 It can be written as,
Good corporate governance structures encourage 9 + 11 + 30 = 50
companies to provide accountability and control. A Now, 5 + 0 = 5
fundamental reason why corporate governance has (iii) 11 ⊕ 17 ⊕ 21
moved not the economic and political agenda worldwide
has been the rapid growth in international capital It can be written as,
markets. Effective corporate governance enhances access 11 + 17 + 21 = 49
to external financing by firms, leading to greater Now, 4 + 9 = 13
investments, higher growth and employment, Investors Therefore, 23 ⊕ 4 ⊕ 15
look to place their funds where the standards of ⇒ 23 + 4 + 15 = 42
disclosure, of timely and accurate financial reporting, Now, 4 + 2 = 6
and of equal treatment to all stakeholders are met.
22. Which of the following statements best reflects 25. Let x be a positive integer such that 7x + 96 is
the logical inference from the passage given divisible by x. How many values of x are
above? possible?
(a) It is an important agenda of the countries (a) 10 (b) 11
around the world to ensure access to good (c) 12 (d) Infinitely many
external financing. Ans. (c) : According to the given,
(b) Good corporate markets ensure that the firms It 7x+96 is to be divisible by x, 96 should
maintain good corporate governance. be divisible by x
(c) International capital markets ensure that the 96 = 25 × 3
firms maintain good corporate governance. So number of factors of 96 = (5+1) (1+1)
(d) Good corporate governance paves the way for
robust supply chains. = 12
Hence, there are 12 possible values of x.
Ans. (b) : The statement best reflects the logical
inference from the passage is - "Good corporate 26. If p, q, r and s are distinct single digit positive
governance improves the credibility of the firm". numbers then what is the greatest value of (p +
q) (r + s)?
Passage-3 (a) 230 (b) 225
Elephants are landscape architects, creating (c) 224 (d) 221
clearings in the forest, preventing overgrowth of certain Ans. (b) : According to the given,
plant species and allowing space for the regeneration of Possible values of p, q, r and s can be 9, 8, 7 and 6 (not
others, which in turn provide sustenance to other in the same order)
herbivorous animals. Elephants eat plants, fruits and
seeds, propagating the seeds when they defecate in other Then, p + q + r + s = 30
places as they travel. Elephant dung provides Product of two numbers is highest, is when
nourishment to plants and animals and acts as a breeding both the numbers are equal
ground for insects. In times of drought, they access water 9 + 6 = 15
by digging holes which benefits other wildlife. and 8 + 7 = 15
23. Which one of the following statements best Therefore, 15 × 15 = 225
reflects the most logical and rational inference 225 is the greatest value of (p+q) (r+s).
that can be drawn from the passage?
27. A number N is formed by writing 9 for 99 times.
(a) The home range of elephants needs to be a What is the remainder if N is divided by 13?
vast area or rich biodiversity.
(b) Elephants are the keystone species and they (a) 11 (b) 9
benefit the biodiversity. (c) 7 (d) 1
IAS (Pre) GS IInd Paper (CSAT), 2023 213 YCT
9999....... Ans. (c) : According to the given,
Ans. (a) : According to question,
13
9
⇒ Q = 0, R = 9
13
99
⇒ Q = 7, R = 8 No three of the six points are collinear.
13
Then, total number of triangles = 6C3
999
⇒ Q = 76, R = 11 6!
13 = = 20
3! 3!
9999
⇒ Q = 769 , R = 2 Passage-1
13
In India, the segregation of municipal waste at
99999
⇒ Q = 7692, R = 3 source is rare. Recycling is mostly with the informal
13 sector. More than three-fourths of the municipal budget
999999 goes into collection and transportation, which leaves
⇒ Q = 76923, R = 0 very little for processing/resource recovery and disposal.
13 Where does waste-to-energy fit into all this? Ideally it
9999999 fits in the chain after segregation (between wet waste and
⇒ Q = 769230 , R = 9 the rest), collection, recycling, and before getting to the
13 landfill. Which technology is most appropriate in
Similarly, after every 6 divisions, the sequence, of the converting waste (that is biodegradable versus non-
remainder will be repeated biodegradable component) and its calorific value. The
Therefore, 16 × 16 = 96 biodegradable component of India's municipal solid
96 ⇒ R = 0 waste is a little over 50 per cent, and biomethanation
97 ⇒ R = 9 offers a major solution for processing this.
31. Based on the above passage, the following
98 ⇒ R = 8 assumptions have been made:
99 ⇒ R = 11 1. Collection, processing and segregation of
Hence, remainder will be 11. municipal waste should be with
28. Each digit of a 9-digit number is 1. It is government agencies.
multiplied by itself. What is the sum of the 2. Resource recovery and recycling require
digits of the resulting number? technological inputs that can be best
(a) 100 (b) 80 handled by private sector enterprises.
(c) 81 (d) 100 Which of the assumptions given above is/are
correct?
Ans. (c) : As we know, (a) 1 only (b) 2 only
(11)2 = 121 (c) Both 1 and 2 (d) Neither 1 nor 2
(111)2 = 12321 and so an Ans. (d) : Neither statement 1 nor 2 given above are
Therefore, (111111111)2 = 1234567898765321 correct.
Sum of digits = 81 32. Which one of the following statements best
29. What is the sum of all digits which appear in all reflects the crux of the passage?
the integers from 10 to 100? (a) Generation of energy from municipal solid
(a) 855 (b) 856 waste is inexpensive.
(c) 910 (d) 911 (b) Biomethanation is the most ideal way of
Ans. (b) : According to the given, generating energy from municipal solid waste.
Firstly, consider integers from 10 to 99 (c) Segregation of municipal solid waste is the
first step in ensuring the success of water-to-
Sum of all the digits at unit's place = (1+2+....+9) × 9 energy plants.
= 45 × 9 (d) The biodegradable component of India's
= 405 municipal solid waste is not adequate to
Sum of all the digits at ten's place = 10(1+2+....+9) provide energy from waste efficiently/
= 450 effectively.
Hundred's place = 1 Ans. (c) : Statement which best reflects the crux of
Therefore, sum of all the digits = 405 + 450 + 1 passage is -
= 856 "Segregation of municipal solid waste is the first step in
ensuring the success of waste to energy plants".
30. ABCD is a square. One point on each of AB
and CD; and two distinct points on each of BC Passage-2
and DA are chosen. How many distinct There is a claim that organic farming is inherently
triangles can be drawn using any three points safer and healthier. The reality is that because the
as vertices out of these six points? organic farming industry is still young and not well-
(a) 16 (b) 18 regulated in India, farmers and consumers, alike, are not
(c) 20 (d) 24 only confused about what products are best for them, but
IAS (Pre) GS IInd Paper (CSAT), 2023 214 YCT
sometimes use products in ways that could harm them as C. Government policies regarding food planning
well. For example, since organic fertilizers are difficult need to incorporate nutritional security.
to obtain on a large scale in India, farmers often use D. For the present monoculture agriculture
farmyard manure, which may contain toxic chemicals practices, farmers receive subsidies in various
and heavy metals. Certain plant sprays, such as Datura ways and government offers remunerative
flower and leaf spray, have an element called atropine. If prices for grains and therefore they do not
it is not applied in the right does, it can act on the tend to consider crop diversity.
nervous system of the consumer. Which of the above assumptions are valid?
Unfortunately, how much and when to use it are not (a) 1, 2 and 4 only (b) 2 and 3 only
well-researched or regulated issues. (c) 3 and 4 only (d) 1, 2, 3 and 4
33. Based on the above passage, the following Ans. (a) : Assumption 1, 2 and 4 are valid.
assumptions have been made:
36. A box contains 14 black balls, 20 blue balls, 26
1. Organic farming is inherently unsafe for green balls, 28 yellow balls, 38 red balls and 54
both farmers and consumers. white balls.
2. Farmers and consumers need to be Consider the following statements:
educated about eco-friendly food. 1. The smallest number n such that any n
Which of the assumptions given above is/are balls drawn from the box randomly must
correct? contain one full group of at least one colour
(a) 1 only (b) 2 only is 175.
(c) Both 1 and 2 (d) Neither 1 nor 2 2. The smallest number m such that any m
Ans. (d) : Neither statement 1 nor 2 is correct. balls draws from the box randomly must
34. Which one of the following statements best contain at least one ball of each colour is
reflects the most logical, rational and practical 167.
message conveyed by the author of the Which of the above statements is/are correct?
passage? (a) 1 only (b) 2 only
(a) In India, organic farming should not be (c) Both 1 and 2 (d) Neither 1 nor 2
promoted as a substitute for conventional Ans. (c) : Statement 1- Let the first 13 balls drawn
farming. from the box are black, next 19 balls are blue, next 25
(b) There are no safe organic alternatives to are green, next 27 are yellow, next 37 are red, next 53
chemical fertilizers. are white.
(c) In India, farmers need to be guided and Now, in this process total balls drawn = 174.
helped to make their organic farming If one more ball is drawn from the box, one colour
sustainable. group will be complete.
(d) The aim of organic farming should not be to Hence, Statement 1 is correct.
generate huge profits as there is still no global Statement 2- Let the first 54 balls are white, next 38 are
market for its products. red, next 28 are yellow, next 26 are green, next 20 are
Ans. (c) : Statement that best reflects the most logical, blue. In this process, total number of balls drawn = 166.
rational and practical message conveyed by the author In this group, no black ball is present. so the next ball
of the passage is - "In India, farmers need to be guided drawn will be black and total number of balls will
and helped to make their organic farming sustainable". become 167.
Hence, statement 2 is also correct.
Passage-3 37. If 'ZERO' is written as 'CHUR', then how is
Food consumption patterns have changed 'PLAYER' written?
substantially in India over the past few decades. This has (a) SOCAGT (b) SODBGT
resulted in the disappearance of many nutritious foods (c) SODBHT (d) SODBHU
such as millets. While food grain production has Ans. (d) : According to the given,
increased over five times since independence, it has not
sufficiently addressed the issue of malnutrition. For long,
the agriculture sector focused on increasing food
production particularly staples, which led to lower
production and consumption of indigenous traditional
crops/grains, fruits and other vegetables, impacting food
and nutrition security in the process. Further, intensive,
monoculture agriculture practices can perpetuate the
food and nutrition security problem by degrading the
quality of land, water and food derived through them.
35. Based on the above passage, the following
assumptions have been made:
A. To implement the Sustainable Development
Goals and to achieve zero-hunger goal,
monoculture agriculture practices are inevitable
even if they do not address malnutrition. 38. Consider the following statements:
B. Dependence on a few crops has negative 1. A is older than B
consequences for human health and the 2. C and D are of the same age
ecosystem. 3. E is the youngest

IAS (Pre) GS IInd Paper (CSAT), 2023 215 YCT


4. F is younger than D 2. Early spring and longer period of spring is
5. F is older than A not good for bird populations.
How many statements given above are required Which of the above assumptions is/are correct?
to determine the oldest person/persons? (a) 1 only (b) 2 only
(a) Only two (b) Only three (c) Both 1 and 2 (d) Neither 1 nor 2
(c) Only four (d) All five Ans. (a) : According to the passage, "Global Warming
Ans. (d) : According to the given, is causing spring to come early and for longer
C=D>F>A>B>E durations" is correct.
All the five statements are required to determine the
oldest person. Passage–2
39. Consider the following including the Question A global analysis of nitrogen use efficiency –a
and the Staements : measure of the amount of nitrogen a plant takes in to
There are 5 members A, B, C, D, E in a family grow versus what is left behind as pollution – says that
What is the relation of E to B? using to much fertilizers will lead to increased pollution
Staement-1 : A and B are a married couple of waterways and the air. Currently, the global average
Staement-2 : D is the father of C for nitrogen use efficiency is approximately 0.4, meaning
40 percent of the total nitrogen added to cropland goes
Staement-3 : E is D's son. into the harvested crop while 60 percent is lost to the
Staement-4 : A and C are sisters. environment, says a study. More than half of the world's
Which one of the following is correct in respect population is nourished by food grown with fertilizers
of the above Question and Statements? containing synthetic nitrogen, which is needed to
(a) Statement-1, Statement-2 and Statement-3 are produce high crop yields. Plants take the nitrogen they
sufficient to answer the Question. need to grow, and the excess is left in the ground, water
(b) Statement-1, Statement-3 and Statement-4 are and air. This results in significant emissions of nitrous
sufficient to answer the Question. oxide, a potent greenhouse and ozone depleting gas, and
(c) All four statements together are suffeicent to other forms of nitrogen pollution, including
answer the Question. eutrophication of lakes and rivers and contamination of
(d) All four statements are not sufficient to river water.
answer the Question. 42. Which one of the following statements best
Ans. (c) : According to the given, Passage-2 reflects the most logical, rational and crucial
message implied by the passage?
(a) An enhanced efficiency of use of nitrogen is
imperative for both food production and
environment.
(b) Production of synthetic nitrogen fertilizers
Therefore, E is brother-in-law of B. cannot be stopped as it well adversely affect
All four statements together are sufficient to answer the global food security.
question. (c) Alternatives to crops that require excess of
40. Choose the group which is different from the nitrogen should be identified and cultivated.
others : (d) Conventional agriculture using synthetic
(a) 17, 37, 47, 97 (b) 31, 41, 53, 67 fertilizers should be replaced with agroforestry,
(c) 71, 73, 79, 83 (d) 83, 89, 91, 97 agro ecosystems and organic farming.
Ans. (d) : Options (a), (b) and (c) contain only prime Ans. (a) : Statement that best reflects the most logical,
numbers, while option (d) contains a composite rational and crucial message implied by the passage is-
number 91. "An enhanced efficiency of use of nitrogen is
Direction for the following 3 (three) items : imperative for both food production and environment".
Read the following three passages and answer Passage–3
the items that follow the passages. Your
answers to these items should be based on the Along with sustainable lifestyles, climate justice is
passages only. regarded as a significant principle in environmental
parlance. Both the principles have bearings on political
Passage-1 and economic choices of the nation. So far, in our
Scientists studied the vernal window – transition climate change summits or compacts, both the principles
period from winter to the growing season. They found have eluded consensus among nations. Justice, in the
that warmer winters with less snow resulted in a longer judicial sense, is well defined. However, in the context of
lag time between spring events and a more protracted climate change, it has scientific as well as socio-political
vernal window. This change in the spring timetable has connotations. The crucial question in the next few years
ecological, social and economic consequences – for will be how resources, technologies and regulations are
agriculture, fisheries and tourism. As the ice melts used to support the victims of climate change. Justice in
earlier, the birds don't return, causing a delay, or climate is not confined to actions relating to mitigation,
lengthening in springtime ecological events. but includes the wider notion of support for adaptation to
41. With reference to the above passage, the climate change and compensation for loss and damage.
following assumptions have been made : 43. Which one of the following statements best
1. Global warming is causing spring to come reflects the most logical, rational and crucial
early and for longer durations. message conveyed by the passage?
IAS (Pre) GS IInd Paper (CSAT), 2023 216 YCT
(a) Climate justice should be ingrained in detail 1. The number of possible values of p is 5.
in the rules of all the new climate compacts/ 2. The number of possible values of q is 6.
agreements. Which of the above statements is/are correct?
(b) Environmental resources are unevenly (a) 1 only (b) 2 only
distributed and exploited across the globe. (c) Both 1 and 2 (d) Neither 1 nor 2
(c) There is an impending issue of dealing with a Ans. (c) : According to the given,
huge number of climate change victims/
climate refuges. p<q<r
(d) Climate change in all its connotations is and pp + qq + rr = tto
mostly due to developed countries and Since unit digit of the given sum is zero.
therefore their share of burden should be Therefore
more. Case (i) Let p = 1
Ans. (a) : Statement that best reflects the most logical, p q r
rational and crucial message conveyed by the passage (a) 1 2 7 → unit digit = 0
is- (b) 1 3 6 → unit digit = 0
"Climate justice should be ingrained in detail in the (c) 1 4 5 → unit digit = 0
rules of all the new climate compacts/agreements". Case (ii) Let p = 2
44. A principal P becomes Q in 1 year when Respective values of p, q and r is 2, 3 and 5.
compounded half-yearly with R% annual rate 2, 3, 5 → unit digit = 0
of interest. If the same principal P becomes Q Case (iii) Let p = 3
in 1 year when compounded annually with S% Respective values of p, q and r is 3, 8 and 9.
annual rate of interest, then which one of the
following is correct? 3, 8, 9 → unit digit = 0
(a) R = S (b) R > S Case (iv) Let p = 4
(c) R < S (d) R ≤ S Respective values of p, q and r is 4, 7 and 9.
4, 7, 9 → unit digit = 0
Ans. (c) : According to the given,
Case (v) Let p = 5
P becomes Q in 1 year when compounded half yearly
with R% annual rate of interest Respective values of p, q and r is 5,6,9
Also, P becomes Q in the same time period of 1 year 5,6,9 → unit digit = 0
with S% annual rate of interest compounded annually None of the other values of p will fulfill the condition.
Whenever, Principle and amount are same for the same So, number of possible values of p is 5 (1,2,3,4,5) and
time period then, the rate of interest compound half number of possible values of q is 6 (2, 3, 4, 6, 7, 8).
yearly is always less than rate of interest compounded Therefore both the statements 1 and 2 are correct.
annually 47. What is the sum of all 4-digit numbers less than
Hence, R < S 2000 formed by the digits 1, 2, 3 and 4, where
45. How many natural numbers are there which none of the digits is repeated?
give a remainder of 31 when 1186 is divided by (a) 7998 (b) 8028 (c) 8878 (d) 9238
these natural numbers? Ans. (a) : According to the given,
(a) 6 (b) 7 All four digit numbers which are less than 2000 and
(c) 8 (d) 9 formed by digits 1, 2, 3, 4 are -
Ans. (d) : According to the given, remainder should 1234
always be 31. 1243
Therefore, 1186–31 = 1155 1324
1155 will be exactly divisible by these numbers. 1342
1423
5 1155
+ 1432
11 231 7998
7 21 48. What is the number of selections of 10
consecutive things out of 12 things in a circle
3 taken in the clockwise direction?
1155 = 5 × 11 × 7 × 3 (a) 3 (b) 11 (c) 12 (d) 66
Since, remainder should always be 31. Ans. (c) : According to the given,
Therefore all the required numbers should be greater
than 31.
(i) 3 × 11 = 33 (ii) 5 × 7 = 35
(iii) 5 × 11 = 55 (iv) 7 × 11 = 77
(v) 15 × 7 = 105 (vi) 15 × 11 = 165
(vii) 15 × 7 × 11 = 1155 (viii) 35 × 11 = 385
(ix) 77 × 3 = 231
So, total number of natural numbers is 9.
46. Let pp, qq and rr b 2-digit numbers where p <
q < r. If pp + qq + rr = tt0, where tt0 is a 3-digit
number ending with zero, consider the
following statements :
IAS (Pre) GS IInd Paper (CSAT), 2023 217 YCT
So, number of selections of 10 consecutive things from Passage–1
12 things in clockwise direction = Sourcing food from non-agricultural lands
(i) 1 to 10 (ii) 2 to 11 (iii) 3 to 12 (uncultivated system such as forests, wetlands, pastures,
(iv) 4 to 1 (v) 5 to 2 (vi) 6 to 3 etc) in addition to agricultural lands enables a systemic
(vii) 7 to 4 (viii) 8 to 5 (ix) 9 to 6 approach to food consumption. It allows rural and tribal
(x) 10 to 7 (xi) 11 to 8 (xii) 12 to 9 communities to sustain themselves for the whole year
Hence, there are 12 possible number of selections. and steer clear of natural disasters and season-induced
49. If today is Sunday, then which day is it exactly shortfalls of agricultural food. Since the productivity of
on 1010th day? trees is often more resilient to adverse weather
(a) Wednesday (b) Thursday conditions than annual crops, forest foods often provide a
safety net during periods of food shortages caused by
(c) Friday (d) Saturday crop failure; forest foods also make important
Ans. (b) : According to the given, contributions during seasonal crop production gaps.
By dividing 1010 by 7 we will get the number of odd 51. Which one of the following statements best
days. reflects the most logical and rational message
1010 (10 )
2 5 conveyed by the author of the passage?
(100 )
5

= = (a) Food yielding trees should replace other trees


7 7 7 in rural and tribal areas and community
5 owned lands.
2 32
⇒ Remainder = = ⇒ Remainder = 4 (b) Food security cannot be ensured in India with
7 7 the present practice of conventional
Therefore, Remainder = odd days = 4 agriculture.
So, If today is Sunday, then 1010th day will be Thursday (c) Wastelands and degraded areas in India
50. There are three traffic signals. Each signal should be converted into agro forestry
changes colour from green to red and then systems to help the poor.
form red to green. The first signal takes 25 (d) Agroecosystems should be developed in
seconds, the second signal takes 39 seconds and addition to or along with conventional
the third signal takes 60 seconds to change the agriculture.
colour from green to red. The durations for Ans. (d) : Statement that best reflects the most logical
green and red colours are same. At 2 : 00 p.m, and rational message conveyed by the author of the
they together turn green. At what time will passage is-
they change to green next, simultaneously? Agroecosystems should be developed in addition to or
(a) 4:00 p.m. (b) 4:10 p.m. along with conventional agriculture.
(c) 4:20 p.m. (d) 4:30 p.m. Passage–2
Ans. (b) : According to the given, While awareness on use/misuse and abuse of
1st signal takes time to change colour = 25 sec antibiotics is common knowledge, as is the impact of
2st signal takes time to change colour = 39 sec dosing poultry with antibiotics, the environmental impact
3st signal takes time to change colour = 60 sec of antibiotics-manufacturing companies not treating their
L.C.M. of 25, 39 and 60 = waste has scarcely been discussed at any length or
seriousness thus far. Pollution from antibiotics factories
5 25, 39 , 60 is fueling the rise of drug-resistant infections. The
5 5, 39, 12 occurrence of drug-resistant bacteria surrounding the
pharma manufacturing plants is well known.
2 1, 39 , 12 52. Which one of the following statements best
2 1, 39 , 6 reflects the most logical and practical message
conveyed by the passage?
3 1, 39, 3 (a) It is necessary to put proper effluent treatment
13 1, 13, 1 protocols in place.
(b) It is necessary to promote environmental
1, 1, 1 awareness among people.
LCM = 5 × 5 × 2 × 2 × 3 × 13 = 3900 (c) Spread of drug-resistant bacteria cannot be
So, all the three will become red after 3900 sec. done away with, as it is inherent in modern
So, they will become green after another 3900 sec medical care.
Total time from green to green = 7800 sec (d) Pharma-manufacturing companies should be
7800 set up in remote rural areas, away from
= = 130 min = 2 hr 10 min crowded towns and cities.
60 Ans. (a) : Statement that best reflects the most logical
Therefore, again they will together become green at, and practical message conveyed by the passage is- "It is
2 pm + 2 hr 10 min necessary to put proper effluent treatment protocols in
= 4:10 PM place."
Directions for the followings 3 (three) items: Passage–3
Read the following three passages and answer
the items that follow the passages. Your answer Benefits of good quality school education accrue
to these items should be based on the passages only when students complete and leave school after
only. having acquired the gateway skills. Like one learns to

IAS (Pre) GS IInd Paper (CSAT), 2023 218 YCT


walk before running, similarly one picks up advanced 377 + 1
skills only after picking the basic foundational skills. The So, middle term = = 189 th term
advent of the knowledge economy poses new challenges, 2
and one of the severe consequences of having an Therefore, 2 + 3 + 4 + ...... x ≤ 189
uneducated workforce will be our inability to keep pace x (x + 1) ≤ 380
with the global economy. Without a strong learning x = 19 satisfies the condition.
foundation at the primary level, there can be no So, the 19th letter (starting from Z) would be 8th letter (2
improvement in higher education or skill development. – 19 = 8) starting form A, which is H.
53. Which one of the following statements best
reflects the crux of the passage? 56. Question : Is p greater than q?
(a) To become a global power, India needs to Statement-1: p × q is greater than zero.
invest in universal quality education. Statement-2: p2 is greater than q2.
(b) India is unable to become a global power Which one of the following is correct in respect
because it is not focussing or promoting of the above Question and the Statements?
knowledge economy. (a) The Question can be answered by using one
(c) Our education system should focus more on of the Statements alone, but cannot be
imparting skills during higher education. answered using the other Statement alone.
(d) Parents of many school children are illiterate (b) The Question can be answered by using either
and are unaware of the benefits of quality Statement alone.
education. (c) The Question can be answered by using both
Ans. (c) : Statement that best reflects the curse of the the Statements together, but cannot be
passage is- "Our education system should focus more answered using either Statement alone.
on imparting skills during higher education." (d) The Question cannot be answered even by
54. 40 children are standing in a circle and one of using both the Statements together.
them (say child-1) has a ring. The ring is passed Ans. (d) : According to the given,
clockwise. Child-1 passes on to child-2, child-2
passes on to child-4, child-4 passes on to child-7 Statement I:- p × q > 0
and so on. After how many such changes It means that p and q are non - zero and both have the
(including child-1) will the ring be in the hands same sign.
of child-1 again? Statement II:- p2 > q2
(a) 14 (b) 15 It means that both p and q can have positive or negative
(c) 16 (d) 17 values. For example values of p and q can be (2, 1) or (–
Ans. (b) : According to the given, 2, – 1).
The pattern is, So, the question can not be answered even by using
both the statements together.
57. Question : Is (p + q – r) greater than (p – q +
r), where p, q and r are integers?
Ring is following a circular path, so the ring will come Statement-1 : (p – q) is positive.
back to 1st child if the number in the pattern is (40x + Statement-2 : (p – r) is negative.
1), like 41st, 81st, 121st etc. Which one of the following is correct in respect
(i) 1–2 (ix) 37 – 46 of the above Question and the Statements?
(ii) 2 – 4 (x) 46 – 56 (a) The Question can be answered by using one
(iii) 4 – 7 (xi) 56 – 67 of the Statement alone, but cannot be
(iv) 7 – 11 (xii) 67 – 79 answered using the other Statement alone.
(v) 11 – 16 (xiii) 79 – 92 (b) The Question can be answered by using either
(vi) 16 – 22 (xiv) 92 – 106 Statement alone.
(vii) 22 – 29 (xv) 106 – 121 (c) The Question can be answered by using both
(viii) 29 – 37 the Statements together, but cannot be
Therefore, after 15 changes, the ring will be in the answered using either Statement alone.
hands of child one again. (d) The Question cannot be answered even by
55. What is the middle term of the sequence using both the Statements together.
Z, Z, Y, Y, Y, X, X, X, X, W, W, W, W, W, ..., A? Ans. (c) : If p + q – r > p – q + r
(a) H (b) I Then q > r
(c) J (d) M Statement 1:- (p – q) is positive
Ans. (a) : In the given sequence, the total number of Statement 2:- (p – r) is negative
terms = 2 + 3 + ....... + 27
From Statement 1:- p – q > 0
n =p>q
Sum = Sn = (2a + (n − 1)d)
2 From Statement 2:- p – r < 0
26 26 p<r
= (2 × 2 + (26 − 1)1) = ( 4 + 25) So, r>p>q
2 2
26 × 29 The question can be answered by using both the
= = 13 × 29 = 377 statements together but cannot be answered using either
2 statement alone.
IAS (Pre) GS IInd Paper (CSAT), 2023 219 YCT
58. In a party, 75 persons took tea, 60 persons took (c) The Question can be answered by using both
coffee and 15 persons took both tea and coffee. the Statements together, but cannot be
No one taking milk takes tea. Each person answered using either Statement alone.
takes at least one drink. (d) The Question cannot be answered even by
Question : How many persons attended the using both the Statements together.
party? Ans. (d) : According to the given,
Statement-1 : 50 persons took milk. Let the digits are x, y and z
Statement-2 : Number of persons who Statement 1:- x + y + z = x y z
attended the party is five times
the number of persons who took 100x + 10y + z
Statement 2:-
milk only. x+y+z
Which one of the following is correct in respect So, the number cannot be found even by using both the
of the above Question and the Statements? statements together.
(a) The Question can be answered by using one 60. For five children with ages a < b < c < d < e;
of the Statements alone, but cannot be any two successive ages differ by 2 years.
answered using the other Statement alone.
Question : What is the age of the youngest
(b) The Question can be answered by using either child?
Statement alone.
Statement-1 : What is the age of the youngest
(c) The Question can be answered by using both child?
the Statements together, but cannot be
answered using either Statement alone. Statement-2 : The average age of the children
is 8 years.
(d) The Question cannot be answered even by
using both the Statements together. Which one of the following is correct in respect
of the above Question and the Statement?
Ans. (c) : According to the given, (a) The Question can be answered by using one
of the Statements alone, but cannot be
answered using the other Statement alone.
(b) The Question can be answered by using either
Statement alone.
(c) The Question can be answered by using both
the Statements together, but cannot be
answered using either Statement alone.
60 + 15 + 45 – b + b + 50 – b = 5 (50 – b) (d) The Question cannot be answered even by
120 + 50 – b = 5 (50 – b) using both the Statements together.
120 + 50 – b = 250 – 5b Ans. (b) : According to the given, Sequence of the age
4b = 80 is a < b < c < d < e and two successive age differ by 2
80 years
b= = 20 Statement 1:- e = 3a
4
Therefore, Since, e=a+8
Therefore, a + 8 = 3a
2a = 8
a = 4 years
a+b+c+d+e
Statement - 2:- =8
5
a + (a + 2) + (a + 4) + (a + 6) + (a + 8)
Total number of person attended the party =8
= 60 + 15 + 25 + 20 + 30 5
= 150 5a + 20 = 40
The question can be answered by using both the 5a = 20
statements together but cannot be answered using either a = 4 years
statement alone. Hence, the question can be answered by using either
59. Consider a 3-digit number. statement alone.
Question: What is the number? Directions for the following 4 (four) items:
Statement-1: The sum of the digits of the Read the following four passages and answer
number is equal to the product the items that follow the passages. Your
of the digits. answers to these items should be based on the
Statement-2: The number is divisible by the passages only.
sum of the digits of the number. Passage–1
Which one of the following is correct in respect The paradox of choice is illustrated by the story of
of the above Question and the Statements? Buridan's ass. Jean Buridan, the 14th century philosopher,
(a) The Question can be answered by using one wrote about free will and the inability to choose due to
of the Statements alone, but cannot be numerous choices and uncertainties. In the story, a
answered using the other Statement alone. donkey stands between two equally appealing stacks of
(b) The Question can be answered by using either hay. Unable to decide which to eat, if starves to death.
Statement alone. Changes in technology and innovations such as smart
IAS (Pre) GS IInd Paper (CSAT), 2023 220 YCT
phones and tablets only exacerbate our glut of choices. 63. Based on the above passage, the following
Constant connectivity and overconsumption of real-time assumptions have been made :
data and social media can leave little room for self- 1. Patent protection given to patentees puts a
reflection and rest, making decisions more difficult. Life huge burden on public's purchasing power
is about choices. Many people are overwhelmed with in accessing patented medicines.
attractive life choices, yet find themselves unhappy and 2. Dependence on other countries for
anxious. pharmaceutical products is a huge burden
61. Which one of the following statements best for developing and poor countries.
reflects the most logical message implied by the 3. Providing medicines to the public at
above passage? affordable prices is a key goal during the
(a) Modern technology enfeebles societal public health policy design in many
structure and makes life difficult. countries.
(b) Modern life is full of uncertainties and 4. Governments need to find an appropriate
endless difficult choices. balance between the rights of patentees and
(c) We are influenced by the opinion of others the requirements of the patients.
and have no courage to follow our own Which of the above assumptions are valid?
convictions. (a) 1 and 2 (b) 1 and 4 (c) 3 and 4 (d) 2 and 3
(d) In our lives, having too few choices may not Ans. (b) : According to the given passage, assumption-
be a good thing, but having too many can be 1 and assumption 4 are valid.
equally as difficult.
Ans. (d) : Statement that best reflects the most logical Passage–4
message implied by the above passage is - "In our lives, India should ensure the growth of the digital
having too few choices may not be a good thing, but economy while keeping personal data of citizens secure
having too many can be equally as difficult." and protected. No one will innovate in a surveillance-
oriented environment or in a place where an individual's
Passage–2 personal information is compromised. The ultimate
Household finance in India is unique. We have a control of data must reside with the individuals who
tendency of invest heavily in physical

You might also like